Download as pdf or txt
Download as pdf or txt
You are on page 1of 1030

Sel

f-pr
epar
ati
onmodul
e

s
2nd
sse
la
_c
ob
bo
ah
sm
ur

edi
tion
yo
e/
t.m

NEW
://
tp
ht

SSCMat
h
5000MCQ
Al
lTCSquest
ionsaskedi
n
SSCexams
Chapt
er-
wisecover
age
Bi
li
ngual
Withdetail
edexpl
anat
ion
&shortt
ricks
SSCCGLTi
er1
,SSCCGLTi
er2,
SSCCPO,SSCCHSL,SSCMTS Bal
ji
tDhakaSi
r

ebook www.
ssccgl
pinnacl
e.com
Pi
nnacl
ePubl
icat
ions
s
sse
la
_c
ob
bo
ah
sm
ur
yo
e/

INDEX
t.m
://
tp
ht

SR. NO. TOPICS PAGE NO.

1. NUMBER SYSTEM 1-33

2. HCF - LCM 34-47

3. SIMPLIFICATION 48-83

4. AVERAGE 84-127

5. RATIO & PROPORTION 128-175

6. MIXTURE & ALLIGATION 176-182

7. PERCENTAGE 183-224

8. PROFIT & LOSS 225-278

9. DISCOUNT 279-309

10. SIMPLE INTEREST 310-330

11. COMPOUND INTEREST 331-366

12. WORK AND TIME 367-412

13. PIPE & CISTERN 413-428

14. SPEED AND DISTANCE 429-473

15. BOAT & STREAM 474-481

16. MENSURATION 482-576

17. GEOMETRY 577-693

18. ALGEBRA 694-756

19. TRIGONOMETRY 757-816

20. HEIGHT & DISTANCE 817-827

21. COORDINATE GEOMETRY 828-831

22. DATA INTERPRETATION 832-1018

23. MEAN, MEDIAN & MODE 1019-1027

www.ssccglpinnacle.com support@ssccglpinnacle.com Ph. 09729327755, 09817390373


Day 1-4 days Number System

s
sse
la
_c
ob
Basics of Number System / integers and negative numbers are
bo
ah called negative integers. By 9:- When sum of digit is
सं ा प ित की मूल बात
sm
ur

divisible by 9/अंकों का योग 9 से


yo

सभी धना क सं ाएं और


e/
t.m

भा है |
://

ऋणा क सं ाएं | धना क सं ा


tp

1) Face Value/ अंिकत मू :


ht

को धना क पूणाक कहा जाता है eg: 9216


It is nothing but the number itself
तथा ऋणा क सं ा को ऋणा क
about which it has been asked. By 10:- When last digit is 0/जब
पूणाक कहा जाता है |
यह कुछ भी नहीं है , ब वही अंितम अंक 0 है |
I =………………, -4, -3, -2, -1,
सं ा है िजसके बारे म पूछा गया है | eg: 452600
0, 1, 2, 3, 4…………….
Example: In the number 23576
Face value of 5 is 5 and face By 11:- When sum of odd and
4) Even Numbers/ सम सं ा:
value of 7 is 7. even place digits difference is 0
2, 4, 6, 8, 10….. [Divisible by 2
or divisible by 11/ जब सम और
completely]
2) Place Value/ थान मू : The िवषम
place value of a number depends थान के अंको के योग का अंतर 0 हो

e
on its position in the number. 5) Odd Numbers/ िवषम सं ा: 1, या 11 से भा हो |
Each position has a value 10 3, 5, 7, 9, 11….. [Not divisible by eg: 217382
times the places to its right. 2 completely]
िकसी सं ा का थान मू , सं ा
म उसकी थित पर िनभर करता है | Sum of odd place digits = 2+7+8

l
ेक थान का मू उसके दाएं Divisibility Test / िवभा ता के = 17
तरफ की जगहों का 10 गुना होता है |
ac िनयम
Sum of even place digits = 1+3+2
Example: In the number 23576 =6
Place value of 5 is 500 and place By 2:- When last digit is 0 or an 17 – 6 = 11, hence 217382 is
value of 3 is 3000. even number/जब अंितम अंक 0 या
divisible by 11.
एक सम सं ा है |
eg: 520, 588
Types of Numbers/ सं ाओं के Important Formulas / मह पूण
कार
By 3:- Sum of digits is divisible सू
nn
by 3/अंकों का योग 3 से भा है |
1) Natural Numbers / ाकृितक
eg: 1971, 1974 1. Sum of first n natural numbers
सं ा (N) : n(n+1)
= 2
All positive counting numbers. (0
is not included in it.) By 4:- When last two digits are पहली n ाकृितक सं ा का योग =
n(n+1)
सभी धना क गणना सं ाएं | (0 divisible by 4 or, they are 2

इसम शािमल नहीं है | ) zeros/जब अंितम दो अंक 4 से भा


Examples: 1, 2, 3, 4… etc. हो या, वे शू ह | 2. Sum of first n odd numbers =
eg: 1528, 1700 n2
Pi

पहली n िवषम सं ा का योग = n2


2) Whole Numbers / पूण सं ा
(W): All non- negative numbers By 5: When last digit is 0 or 5/
जब अंितम अंक 0 या 5 हो | 3. Sum of first n even numbers =
are all whole numbers.
eg: 1725, 1790 n(n + 1)
सभी धना क सं ाएं तथा 0 पूण
पहली n सम सं ा का योग =
सं ाएं ह|
By 6:- When the number is n(n + 1)
Examples: 0, 1, 2, 3, 4… etc.
divisible by 2 and 3 both/ जब
सं ा 2 और 3 दोनों से भा हो | 4. Sum of square of first n natural
3) Integer Numbers/ पूणाक सं ा eg: 36, 72 numbers = n(n+1)(2n+1)
6
(I): All negative numbers and
पहली n ाकृितक सं ाओं के वग
positive numbers. Positive By 8:- When last three digit is n(n+1)(2n+1)
का योग =
numbers are called positive divisible by 8/ जब अंितम तीन अंक 6

8 से िवभा हो |
eg: 2256

www.ssccglpinnacle.com support@ssccglpinnacle.com Ph. 09729327755, 09817390373 1


/
Day 1-4 days Number System

s
sse
la
_c
ob
5. Sum of cubes of first n natural there will be no zero in the above 54 × 58 × 512 × 516 × 518 × 540 ;
bo
ah n(n+1) 2 expression./ एक शू तब बनता है i.e. 4+8+12+16+18+40 = 98
sm
number = ( n )
ur
yo

जब 2 और 5 के संयोजन होते ह|
e/

पहली n ाकृितक सं ाओं के घन


t.m

चूंिक कोई '5' नहीं है इसिलए


://

Remainder Theorem /
tp

n(n+1) 2
ht

का योग = ( n ) उपरो ंजक म कोई शू नहीं शेषफल मेय


होगा|
6. (xm − am ) is divisible by Q. What will be the remainder
when 17×23 is divided by 12?
(x − a) for all values of m. Example:
17×23 को 12 से िवभािजत करते
(xm − m के सभी मू
am ) , ों के िलए 8 × 15 × 23 × 17 × 25 × 22 =
समय शेषफल ा होगा?
(x − a) से भा है | 23 × (31 × 51 )× 23 × 17 × 52 × 21 × 11 Ans. We can write:/ हम िलख
In this expression there are 4 twos सकते है :
7. (xm
− am )
is divisible by and 3 fives. From this 3 pairs of 17 × 23 = (12 + 5) × (12 + 11)
(x + a) for even values of m. 5×2 can be formed. Therefore, = 12×12 + 12×11 + 5×12 + 11×5
(xm − am ) , m के सम मू ों के िलए there will be 3 zeros in the final In the above expression we will
product. find that remainder will depend

e
(x + a) से भा है |
on the last term i.e. 11×5
इस ंजक म चार 2 तथा तीन 5 है |
उपरो ंजक म हम पाएं गे िक
8. (xm + am ) is divisible by इससे 3 जोड़े 5×2 का गठन िकया
शेष अंितम टम पर िनभर करे गा, जो
(x + a) for odd values of m. जा सकता है | इसिलए अंितम
है 11×5

l
गुणनफल म 3 शू होंगे।
(xm + am ) , m के िवषम मू ों के Now, rem ( 1112× 5 ) = 7.
िलए (x + a) से भा है |
ac So, 12×12+12×11+5×12+11×5 and 11×5
Q. Find the number of zeros in 12 12

the value of: remainder is same in both cases


9. Number of prime factors of which is 7.
ap , bq , cr , ds is p + q + r + s 22 × 54 × 46 × 108 × 610 × 1512 × 814
शेषफल दोनों ही थित म 7 ही है |
when a, b, c, d are all prime 16 18 20
× 20 × 10 × 25 ./
numbers.
Example: Remainder when
ap , bq , cr , ds के अभा गुणनखंडो 22 × 54 × 46 × 108 × 610 × 1512 × 814
1421×1423×1425 is divided by
की सं ा p + q + r + s होगी, जब
nn
× 2016 × 1018 × 2520 के गुणनफल 12?
a, b, c, d अभा है |
म शू की सं ा खोज: शेषफल जब 1421×1423×1425
Ans. को 12 से िवभािजत िकया जाता है ?
Number of Zeros in an
22 × 54 × 46 × 108 × 610 × 1512 × 814 rem ( 1421 × 1423 × 1425
) = rem
expression / िकसी ंजक म शू 12

की सं ा 12 )
( 5×7×9 = rem ( 35×9 12 ) = rem
× 2016 × 1018 × 2520 =
We shall understand this concept ( 12 )
11×9 = 3
22 × 54 × 212 × 28 × 58 × 210 × 310 × 312
with the help of an example. / हम
Pi

एक उदाहरण की सहायता से इस × 512 × 242 × 232 × 516 × 218 × 518 × 540 Negative Remainder / ऋणा क
िवचार को समझगे। शेषफल
Zeros are possible with a Taking negative remainder will
combination of 2 ×5 ./ शू 2 × 5 make our calculation easier. /
Let’s find the number of zeros in
के संयोजन के साथ संभव है | ऋणा क शेषफल हमारी गणना को
the following expression: /
िन िल खत ंजक म शू की Here number of 5’s are less so आसान बना दे गा|
number of zeros will be limited to
सं ा ढू ँ ढ़ते ह: Examples/ उदाहरण:
the number of 5’s.
3 1 i)
24 × 32 × 17 × 23 × 19 = (2 × 3 ) यहां 5 की सं ा कम है इसीिलए 0
की सं ा 5 की सं ा तक सीिमत 9 ) = r em ( 9 ) =− 2 × − 1
rem ( 7×8 −2×−1
× 25 × 17 × 23× 19 होगी। =2
Notice that a zero is made only In this expression number of ii)
fives are: / इस ंजक म 5 की rem ( 55×56
57 ) = r em ( 57 ) =− 2× − 1
−2×−1
when there is a combination of 2
सं ा है :
and 5. Since there is no ‘5’ here =2

www.ssccglpinnacle.com support@ssccglpinnacle.com Ph. 09729327755, 09817390373 2


/
Day 1-4 days Number System

s
sse
la
_c
ob
iii) So remainder will be,/ तो शेषफल (c) 2
bo
ah
rem ( 7×10
9 ) = r em ( 9 ) =− 2×1 होगा,
−2×1 (d) 6
sm
ur
yo

14 × 4 = 56, which will also be


e/

=− 2 or, 7
t.m
://

the last two digits of the Q4. What is the least value of x
tp
ht

expression. / जो ंजक के अंितम such that 517x324 is divisible by


Large Power Concepts
दो अंक भी होंगे| 12?
Look at the following examples: / X का वह सबसे छोटा मान ात कर
िन िल खत उदाहरण दे ख: तािक 517x324 12 से िवभािजत हो
Varieties Questions
i) rem ( 2812345
9 ) = rem जाए ?
SSC CGL 11 June 2019
( ) ( )=
12345
(27+1)
= rem 112345 Q1. If a nine-digit number
9 9
985x3678y is divisible by 72, (Morning)
112345 = 1 then the value of (4x - 3y) is : (a) 3
(b) 1
ii) rem ( 2612345
9 ) = rem यिद 9 अंकों की सं ा 985x3678y
(c) 0
72 से िवभािजत है , तो (4x-3y) का
( ) = rem
12345
(27−1)
9 मान ात कर | (d) 2

e
( −1
12345

9 )=−1 12345 SSC CGL 4 June 2019


(Morning) Q5. When an integer n is divided
=− 1 or, 8 (a) 5 by 8, the remainder is 3. What
(b) 4 will be the remainder if 6n-1 is

l
Application of Remainder (c) 6 divided by 8?
Theorem / शेषफल मेय के (d) 3 जब पूणाक n को 8 से िवभािजत
उपयोग
ac
Find the last two digits of the
expression / ंजक के अंितम दो
अंक ढू ं ढ:
Q2. If a 11 digit number
5y5884805x6 is divisible by 72,
where x = y, then the value of
√xy is :
िकया जाता है , तो शेषफल 3 आता है
| शेषफल ा होगा जब 6n-1 को 8
से िवभािजत िकया जाता है ?
SSC CGL 13 June 2019
(Evening)
22 × 31 × 44 × 27 × 37 × 43 (a)4
यिद 11 अंकों की एक सं ा
If we divide the above expression (b)1
5y5884805x6 है िजसम x = y और
by 100, we will get the last two (c)0
nn
यह 72 से िवभािजत है , तो √xy का
digits as remainder. (d)2
यिद हम उपरो ंजक को 100 से मान ात कर |
िवभािजत करते ह, तो हम अंितम दो SSC CGL 10 June 2019
Q6. If a nine-digit number
अंक शेष के प म ा करगेI (Morning)
43x1145y2 is divisible by 88,
(a) √7
⇒ rem ( 22×31×44×27×37×43
100 ) , then the value of (3x-2y), for the
(b) 3 smallest value of y, is:
dividing by 4 to make it simple /
(c) 7 यिद नौ अंकों की एक सं ा
सरल बनाने के िलए 4 से िवभािजत
कर: (d) 2 √7 43x1145y2 88 से िवभािजत है , तो
Pi

= rem ( 22×31×11×27×37×43 ) = rem y के सबसे छोटे मान के िलए (3x -


25
Q3. If a 10 digit number 2y) का मान ात कर |
( 132×22×216
25 ) = rem ( 7×22×16
25 ) 2094x843y2 is divisible by 88, SSC CHSL 1 July
= rem ( 25 ) = rem ( 25 ) = 14
4×16 14
then the value of ( 5x - 7y ) for 2019(Evening)
the largest possible value of x, is : (a) 22
Since we had divided by 4 यिद 10 अंकों की एक सं ा (b) 18
initially now to get the correct 2094x843y2, 88 से िवभािजत है , तो (c) 20
answer, we need to multiply the x का सबसे बड़ा संभव मान लेते ए ( (d) 9
remainder by 4. 5x - 7y ) का मान ात कर |
चूंिक हमने 4 से िवभािजत िकया था SSC CGL 6 June 2019 Q7. If the eight-digit number
इसिलए हम 4 से शेष को गुणा करना (Evening) 342x18y6 is divisible by 72, then
होगा| what is the value of √9x + y , for
(a) 3 the largest value of y ?
(b) 5

www.ssccglpinnacle.com support@ssccglpinnacle.com Ph. 09729327755, 09817390373 3


/
Day 1-4 days Number System

s
sse
la
_c
ob
यिद आठ अंको की एक सं ा है और यह 13 से भी पूणतः िवभािजत 1000 म कम से कम कौन सी सं ा
bo
ah
342x18y6 72 से िवभािजत है , तो y है | जोड़ी जानी चािहए तािक ा होने
sm
ur
yo

के सबसे बड़े मान के िलए √9x + y SSC CPO 12 March 2019 वाली सं ा 37 से पूणतः िवभािजत
e/
t.m

हो ?
://

का मान ा होगा ? (Evening)


tp
ht

SSC CHSL 2 July (a) 36 SSC CPO 16 March 2019


2019(Morning) (b) 16 (Afternoon)
(a) 2√7 (c) 9 (a)1
(d) 27 (b)25
(b) 4√7
(c)36
(c) 8
Q11. When the integer n is (d)13
(d) 6
divided by 7, the remainder is 3.
What is the remainder if 5n is Q15. In number 16008, The
Q8. The ten digit number
divided by 7? numeral 6 has a face value:
2x600000y8 is exactly divisible
जब पूणाक n को 7 से िवभािजत 16008 म अंक 6 का जातीय मान
by 24. If x =/ 0 and y =/ 0, then the
िकया जाता है , तो शेषफल 3 आता है (face value) है :
least value of (x + y) is | जब 5n को 7 से िवभािजत िकया

e
SSC CPO 15 March 2019
10 अंकों की सं ा 2x600000y8 जाएगा तो शेषफल ा आएगा ? (Evening)
24 से पूणतः िवभािजत है | यिद x =/ 0 SSC CPO 16 March 2019 (a)6000
तथा y =/ 0 है , तो (x + y) का ूनतम (Evening) (b)6
मान ात कर |

l
(a)3 (c)60
SSC CHSL 11 July (b)0 (d)600
2019(Morning)
(a) 5
(b) 8
(c) 9
(d) 2
ac (c)1
(d)2

Q12. The number 23474 is


exactly divisible by: / सं ा
Q16. What is the difference
between the largest and smallest
numbers of the four digits created
using numbers 2, 9, 6, 5? | (Each
23474 िवभा है : number can be used only once)
Q 9. On dividing a number by 38, SSC CPO 14 March 2019 अंक 2,9,6, और 5 का उपयोग करके
the quotient is 24 and the (Morning) बनी चार अंको की सबसे बड़ी और
nn
remainder is 13, then the number (a)2 and 3 only / केवल 2 और 2 सबसे छोटी सं ायों का अंतर ा है
is: (b)2 and 4 only / केवल 2 और 4 ? |( ेक अंक केवल एक ही बार
िकसी सं ा को 38 से भाग दे ने पर (c)2 and 11 only / केवल 2 और 11 यु हो सकता है )
भागफल 38 तथा शेषफल 13 आता (d)2 only / केवल 2 SSC CPO 14 March 2019
है | वह सं ा कौन सी है ? (Evening)
SSC CPO 16 March 2019 Q13.The least number that should (a) 6993
(Morning) be added to 10000 so that it is (b) 7056
(a) 925 exactly divisible by 327 is: (c) 6606
Pi

(b) 975 10000 म ूनतम कौन सी सं ा (d) 7083


(c) 904 जोड़ी जानी चािहए तािक यह 327 से
(d) 956 पूणतः िवभािजत हो जाए ? Q17. A gardener planted 1936
SSC CPO 15 March 2019 saplings in a garden such that
Q10. What is the sum of the (Morning) there were as many rows of
digits of the least number, which (a) 327 saplings as the columns. The
when divided by 12, 16 and 54, (b) 237 number of rows planted is:
leaves the same remainder 7 in (c) 137 एक माली ने िकसी उ ान म 1936
each case and is also completely (d) 190 पौधे इस कार लगाए िक पौधों की
divisible by 13? पं याँ तथा कतार बराबर थे |
उस सबसे छोटी सं ा के अंकों का Q14. Which least number should पं यों की सं ा ात कर |
योग ात कर िजसे 12, 16 और 24 से be added to 1000 so that the SSC CPO 16 March 2019
भाग दे ने पर हर बार 7 शेषफल आता number obtained is exactly (Afternoon)
divisible by 37? (a) 46

www.ssccglpinnacle.com support@ssccglpinnacle.com Ph. 09729327755, 09817390373 4


/
Day 1-4 days Number System

s
sse
la
_c
ob
(b) 44 यिद आठ अंकों की सं ा (c) 7
bo
ah
(c) 48 2074x4y2 88 से िवभा है , तो (d) 5
sm
ur
yo

(d)42 (4x+ 3y) का मान ात कर |


e/
t.m
://

SSC CGL 6 June 2019 Q7. If a 10 digit number


tp
ht

Q18. The sum of all possible (Morning) 67127y76x2 is divisible by 88,


three digit numbers formed by (a) 49 then the value of (7x - 2y) is :
digits 3, 0 and 7, using each digit (b) 36 यिद 10 अंकों की एक सं ा
only once is: (c) 42 67127y76x2 88 से िवभािजत है ,
अंक 3, 0 और 7 म से ेक अंक (d) 45 तो (7x - 2y) का मान ात कर |
का केवल एक ही बार योग करके SSC CGL 7 June 2019
बनने वाली सभी संभािवत तीन अंकों Q4. If a 9-digit number (Evening)
की सं ाओं का योग ात कर | 32x4115y2 is divisible by 88, (a) 10
SSC CPO 14 March 2019 then the value of (4x - y) from the (b) 7
(Morning) smallest possible value of x is : (c) 3
(a)2010 यिद 9 अंकों की सं ा 32x4115y2, (d) 5
88 से िवभा है , x के सबसे छोटे

e
(b)1990
(c)2220 संभव मान से (4x - y) का मान Q8. If the six digit number
(d)2110 ात कर | 15x1y2 is divisible by 44, then (x
SSC CGL 6 June 2019 + y) is equal to :
यिद छः अंकों की एक सं ा

l
Practice Questions (Afternoon)
(a) 31 15x1y2 44 से िवभािजत है , तो (x +
ac
Q1. If the 8-digit number
789x531y is divisible by 72, then
the value of (5x - 3y) is :
यिद आठ अंकों की सं ा
789x531y 72 से िवभा है , तो (5x
(b) 20
(c) -1
(d) 11

Q5. If a 10 digit number


y) का मान िकसके बराबर होगा ?
SSC CGL 10 June 2019
(Afternoon)
(a) 8
(b) 7
- 3y) का मान ात कर | 1330x558y2 is divisible by 88, (c) 6
SSC CGL 4 June 2019 then the value of (x + y) is : (d) 9
(Afternoon) यिद 10 अंकों की एक सं ा
nn
(a) 0 1330x558y2 88 से िवभािजत है , तो Q9. If the six digit number
(b) -1 (x + y) का मान ा होगा ? 6x2904 is divisible by 88, then
(c) 2 SSC CGL 7 June 2019 the value of x is :
(d) 1 (Morning) यिद छः अंकों की एक सं ा
(a) 7 6x2904 88 से िवभािजत है , तो x का
Q2. If the 8-digit number (b) 9 मान ात कर |
179x091y is divisible by 88. The (c) 6 SSC CGL 10 June 2019
value of (5x - 8y) is: (d) 8 (Evening)
Pi

यिद 8 अंकों की सं ा 179x091y, (a) 5


88 से िवभा है , तो (5x - 8y) का Q6. If a 10-digit number (b) 6
मान ात कर | 897359y7x2 is divisible by 72, (c) 7
SSC CGL 4 June 2019 then what is the value of ( 3x - y (d) 8
(Evening) ), for the possible greatest value
(a) 4 of y? Q10. If a six digit number 4x573y
(b) 7 यिद 10 अंकों की एक सं ा is divisible by 72, then the value
(c) 9 897359y7x2 72 से िवभािजत है , तो of (x + y) is:
(d)5 y के सबसे बड़े संभव मान को लेते यिद छः अंकों की एक सं ा
ए ( 3x - y ) का मान ात कर | 4x573y 72 से िवभािजत है , तो (x +
Q3. If the 8-digit number SSC CGL 7 June 2019 y) का मान ा होगा ?
2074x4y2 is divisible by 88, then (Afternoon) SSC CGL 11 June 2019
the value of (4x+3y) is: (a) 3 (Afternoon)
(b) 8 (a) 9

www.ssccglpinnacle.com support@ssccglpinnacle.com Ph. 09729327755, 09817390373 5


/
Day 1-4 days Number System

s
sse
la
_c
ob
(b) 4 then what is the value of (9x -
bo
ah
(c) 8 (a) 5 2y), for the least value of x?
sm
ur
yo

(d) 6 (b) 4 यिद 11 अंकों की एक सं ा


e/
t.m

5y5888406x6 72 से िवभािजत है ,
://

(c) 7
tp
ht

Q11. For what value of x is the (d) 8 तो x के सबसे छोटे मान के िलए (9x
seven digit number 46393x8 - 2y) का मान ा होगा ?
divisible by 11? Q15. What is the value of x so SSC CHSL 3 July
x का कौन सा मान रखने पर सात that the seven digit number 2019(Morning)
अंकों की सं ा 46393x8, 11 से 55350x2 is divisible by 72? (a) 5
िवभािजत हो जायेगी ? x का मान ा होना चािहए तािक (b) 3
SSC CGL 11 June 2019 सात अंकों की सं ा 55350x2 72 (c) 4
(Evening) से िवभािजत हो जाए ? (d) 7
(a) 5 SSC CGL 13 June 2019
(b) 3 (Morning) Q19. If a 10-digit number
(c) 2 (a)1 46789x531y is divisible by 72,

e
(d) 7 (b)8 then the value of (2x + 5y), for
(c)7 the largest value of x is:
Q12. What is the value of x so (d)3 यिद 10 अंकों की एक सं ा
that the seven digit number 46789x531y 72 से िवभािजत है , तो
x का सबसे बड़ा मान लेते ए (2x +

l
91876x2 is divisible by 72? Q16. What is the value of x so
X का वह मान ात कर िजससे सात that the seven digit number 5y) का मान ात कर |
ac
अंकों की सं ा 91876x2, 72 से
िवभािजत हो जाए |
SSC CGL 12 June 2019
(Morning)
(a) 2
8439x53 is divisible by 99?
x का मान ा होना चािहए तािक
सात अंकों की सं ा 8439x53, 99
से िवभािजत हो जाए ?
SSC CGL 13 June 2019
SSC CHSL
2019(Evening)
(a) 28
(b) 16
(c) 10
3 July

(b) 7 (Afternoon) (d) 38


(c) 5 (a) 9
(d) 3 (b) 4 Q20. If a 10-digit number
nn
(c) 3 75y97405x2 is divisible by 72,
Q13. What is the value of x such (d) 6 then the value of (2x-y), for the
that the seven digit number greatest value of x, is:
6913x08 is divisible by 88? Q17. If the nine-digit number यिद 10 अंकों की एक सं ा
x का मान ात कर िजससे सात 8175x45y2 is divisible by 72, 75y97405x2, 72 से िवभािजत है , तो
अंकों की सं ा 6913x08, 88 से then the value of √4x + y , for (2x-y) का मान ात कर |
िवभािजत हो जाए | the largest value of y, is: SSC CHSL 4 July
SSC CGL 12 June 2019 यिद नौ अंकों की सं ा 8175x45y2 2019(Morning)
Pi

(Afternoon) 72 से िवभािजत है , तो y का सबसे (a) 24


(a) 4 बड़ा मान लेते ए का मान (b) 21
(b) 2
√4x + y (c) 12
ात कर |
(c) 8 (d) 18
SSC CHSL 2 July
(d) 6
2019(Afternoon)
Q21. If a 10-digit number
(a) 8
Q14. What is the value of x so 1220x558y2 is divisible by 88,
(b) 4
that the seven digit number then the value of (x + y) is:
(c) 5
5656x52 is divisible by 72? यिद 10 अंकों की एक सं ा
(d) 6
x का मान ा होना चािहए तािक 1220x558y2, 88 से िवभािजत है , तो
सात अंकों की सं ा 5656x52 72 (x+y) का मान ा होगा ?
Q18. If an eleven-digit number
से िवभािजत हो जाए ? SSC CHSL 4 July
5y5888406x6 is divisible by 72,
SSC CGL 12 June 2019 2019(Afternoon)
(Evening) (a) 9

www.ssccglpinnacle.com support@ssccglpinnacle.com Ph. 09729327755, 09817390373 6


/
Day 1-4 days Number System

s
sse
la
_c
ob
(b) 7 (b) 66 SSC CHSL 9 July
bo
ah
(c) 15 (c) 30 2019(Evening)
sm
ur
yo

(d) 11 (d) 35 (a) 13


e/
t.m
://

(b) 10
tp
ht

Q22. If a 10-digit number Q26. If the seven digit number (c) 11


6220x558y2 is divisible by 88, 54x29y6 (x > y) is divisible by (d) 14
then the value of (5x+5y) is: 72, what is the value of (2x +
यिद 10 अंकों की एक सं ा 3y)? Q30. The 10-digit number
6220x558y2, 88 से िवभािजत है , तो यिद सात अंकों की सं ा 54x29y6 79x00001y6 is exactly divisible
(5x+5y) का मान ा होगा ? (x > y) 72 से िवभािजत है , तो (2x + by 88. What is the value of (x +
SSC CHSL 5 July 3y) का मान ा होगा ? y)?
2019(Morning) SSC CHSL 8 July 10 अंकों की सं ा 79x00001y6
(a) 20 2019(Evening) 88 से पूणतः िवभािजत है | (x + y)
(b) 55 (a) 32 का मान ात कर |
(c) 25 (b) 13 SSC CHSL 11 July 2019

e
(d) 45 (c) 38 (Afternoon)
(d) 23 (a) 5
Q23. If a 10-digit number (b) 9
7220x558y2 is divisible by 88, Q27. If the seven digit number (c) 6

l
then the value of (5x + 5y) is: 64x29y6 (x > y) is divisible by (d) 7
यिद 10 अंकों की एक सं ा 72, what is the value of (2x - 3y)

SSC
ac
7220x558y2 88 से िवभािजत है , तो
(5x + 5y) का मान ात कर |
CHSL
2019(Afternoon)
(a) 10
5 July
?
यिद सात अंकों की सं ा 64x29y6
(x > y) 72 से िवभािजत है , तो (2x -
3y) का मान ा होगा ?
SSC CHSL 9 July
Q31. The eight digit number
5x32465y is divisible by 88.
What is the value of (2x + 3y)?
आठ अंकों की सं ा 5x32465y
88 से िवभािजत है | (2x + 3y) का
(b) 25 2019(Morning) मान ा होगा ?
(c) 15 (a) 13 SSC CHSL 11 July 2019
(d) 35 (b) 3 (Evening)
nn
(c) 9 (a) 18
Q24. If a 10-digit number (d) 7 (b) 20
1230x558y2 is divisible by 88, (c) 16
then the value of (5x + 5y) is: Q28. If the seven digit number (d) 24
यिद 10 अंकों की एक सं ा 64x29y6 (x > y) is divisible by
1230x558y2 , 88 से िवभािजत है , तो 72, what is the value of (2x - y)? Q32. The number 45789 is
(5x + 5y) का मान ात कर| यिद सात अंकों की सं ा 64x29y6 divisible by which of the single
SSC CHSL 5 July (x > y) 72 से िवभािजत है , तो (2x - digit number:
Pi

2019(Evening) y) का मान ा होगा ? सं ा 45789 िकस एक अंक वाली


(a) 20 SSC CHSL 9 July सं ा से िवभािजत है ?
(b) 40 2019(Afternoon) SSC CPO 16 March 2019
(c) 30 (a) 3 (Morning)
(d) 50 (b) 13
Q25. If an 8-digit number (c) 7
30x558y2 is divisible by 88, then (d) 9
the value of (6x + 6y) is: (a) Only by 3/ केवल 3
यिद 8 अंकों की सं ा 30x558y2 Q29. The seven digit number (b) Only by 9/ केवल 9
88 से िवभािजत है , तो (6x + 6y) का 78x1y68 is divisible by 88. The (c) Only by 3 and 7/ केवल 3 और
मान ात कर | value of (x + y) is: 7
SSC CHSL 8 July सात अंकों की सं ा 78x1y68 88 (d) Only by 3 and 9 / केवल 3 और
2019(Morning) से िवभािजत है | (x+y) का मान ात 9
(a) 42 कर |

www.ssccglpinnacle.com support@ssccglpinnacle.com Ph. 09729327755, 09817390373 7


/
Day 1-4 days Number System

s
sse
la
_c
ob
Q33. If the seven digit number (c) 5 (a) Only 2,3,6 and 7 / केवल 2,3,6
bo
ah
74x29y6 is divisible by 72, then (d) 14 और
sm
ur
yo

what will be the value of (2x + 7 से


e/
t.m

(b) Only 2 and 3 / केवल 2 और 3


://

3y)? Q37. The number 66249 is


tp
ht

यिद सात अंकों की सं ा 74x29y6, divisible by which of the single से


72 से िवभािजत है , तो (2x + 3y) का digit numbers : (c) Only 2,3 and 4 / केवल 2,3 और
मान ा होगा ? सं ा 66249 िकस एक अंक वाली 4 से
SSC CPO 12 March 2019 सं ा से िवभा है ? (d) Only 2,3 and 7 / केवल 2,3 और
(Evening) 7 से
(a) 20 SSC CPO 15 March 2019
(b) 21 (Morning) Q41. If a = 3 , b = 4 and c = 5 ,
b 4 c 5 d 6
(c) 19 (a)Only by 3 and 9 then the sum of the numerator
(d) 16 (b)Only by 3 and 7 and the denominator (which are
(c)Only by 9 coprimes) of ( da )10 is:
Q34. If the seven digit number (d)Only by 3
यिद a
b = 4
3 , b
c = 4
5 और c
d = 5
6 है ,

e
3x6349y is divisible by 88, then
what will be the value of (2x + Q38. 210102 can be divided तो ( da )10 के अंश एवं हर ( जो
3y)? exactly by: सह-अभा ह ) का जोड़ ा होगा
यिद सात अंकों की सं ा 3x6349y 210102 को पूणतः िकसके ारा ?
88 से िवभािजत है , तो (2x + 3y) का िवभािजत िकया जा सकता है : SSC MTS 2 August 2019

l
मान ा होगा ? SSC CPO 16 March 2019 (Morning)

(Evening)
(a) 32
(b) 30
(c) 28
ac
SSC CPO 13 March 2019 (Afternoon)
(a)7
(b)3
(c)4
(d)8
(a) 1025
(b) 4097
(c) 2049
(d) 513

(d) 35 Q42. A fraction is such that the


Q 39. Number 30744, is divisible numerator is five less than the
Q35. If the six digit number by which one digit number? denominator. Also four times the
nn
4x4y96 is divisible by 88, then सं ां 30744 ,एक अंक वाली िकस numerator is one more than the
what will be the value of (x+2y)? सं ां से िवभा है ? denominator. The fraction is:
यिद छः अंकों की सं ा 4x4y96, 88 SSC CPO 14 March 2019 एक िभ इस कार है िक अंश , हर
से िवभा है , तो (x+2y) का मान (Evening) से 5 कम है | साथ ही, अंश का चार
ा होगा ? (a) All the other numbers except गुना हर से एक अिधक है | यह िभ
SSC CPO 12 March 2019 5 and 7. / 5 और 7 को छोड़कर है :
(Morning) अ सभी सं ाएँ | SSC MTS 9 August 2019
(a) 13 (b) Only 2,3 and 6 / केवल 2, 3 (Afternoon)
Pi

(b) 10 और 6 से (a) 74
(c) 12 (c) Only 2,3,6 and 9 / केवल 2, 3, (b) 3
8
(d) 11 6 और 9 से (c) 7
12
(d) All the other numbers except (d) 2
7
Q36. If the seven digit number 5 / / 5 को छोड़कर अ सभी
56x34y4 is divisible by 72, then सं ाएँ |
Q43. Which number should be
what is the least value of (x + y)?
subtracted from the numerator
यिद सात अंकों की सं ा 56x34y4 Q40. Number 106974 is divisible
and denominator of the fraction
72 से िवभािजत है , तो (x+y) का by which one digit number ? 4
9 so that the fraction could be
ूनतम मान ात कर | सं ा 106974 एक अंक वाली िकस 1
made equal to ?
SSC CPO 13 March 2019 सं ा से िवभा है ? 6

(Morning) SSC CPO 15 March 2019 िभ 4


9 के अंश तथा हर ेक म
(a) 8 (Evening) कौन सी सं ा घटाई जानी चािहए
(b) 12

www.ssccglpinnacle.com support@ssccglpinnacle.com Ph. 09729327755, 09817390373 8


/
Day 1-4 days Number System

s
sse
la
_c
ob
िजससे िभ को 1 के बराबर बनाया SSC MTS 22 August 2019 (a) 49
bo
6
ah (Afternoon) (b) 51
sm
जा सके?
ur
yo

SSC MTS 20 August 2019 (a) 123 (c) 59


e/
t.m
://

(Evening) (b) 137 (d) 61


tp
ht

(a) 3 (c) 147


(b) 7 (d) 113 Q 50. The square root of which of
(c) 2 the following is a rational
(d) 5 Q47. x is the greatest number by number?
which, when 2460, 2633 and िन म से िकसका वग मूल एक
Q44. Three times a number is 24 2806 are divided, the remainder प रमेय सं ा है ?
more than the one-third of this in each case is the same. What is SSC CPO 12 March 2019
number. This number is : the sum of digits of x? (Morning)
िकसी सं ा का तीन गुना उसकी x वह सबसे बड़ी सं ा है िजससे (a) 1250.49
एक-ितहाई से 24 अिधक है | सं ा 2460, 2633 तथा 2806 को िवभािजत (b) 6250.49
ा है : करने पर, हर मामले म शेषफल (c) 1354.24
समान आता है | x के अंकों का योग

e
SSC MTS 21 August 2019 (d) 5768.28
(Afternoon) ा है ?
(a) 9 SSC MTS 22 August 2019 Q 51. The square root of which of
(b) 15 (Evening) the following is a rational number
? िन म से िकसका वगमूल एक

l
(c) 12 (a) 11
(d) 8 (b) 1 प रमेय सं ा है ?
ac
Q45. Let x be the greatest number
which when divides 6475, 4984
and 4132, the remainder in each
case is the same. What is the sum
(c) 13
(d) 9

Q 48. Given n is an integer, what


is the remainder when (6n + 3)2 is
SSC CPO 12 March 2019
(Evening)
(a) 5823.82
(b) 22504.9
(c) 2460.14
of digits of x? divided by 9? (d) 1489.96
मान लीिजये िक x सबसे बड़ी सं ा िदया गया n एक पूणाक है | जब
है जो 6475, 4984 और 4132 को (6n + 3)2 को 9 से भाग िदया जाता Q 52. What is the sum of digits of
nn
िवभािजत करने पर हर मामले म है , तब शेषफल ा आएगा ? the least number, which when
समान शेषफल छोड़ती है | x के SSC CGL 8 July 2019 divided by 15, 18 and 42 leaves
अंकों का जोड़ ा है ? (Afternoon) the same remainder 8 in each case
SSC MTS 22 August 2019 (a) 3 and is also divisible by 13?
(Morning) (b) 2 उस ूनतम सं ा के अंकों का योग
(a) 4 (c) 1 ा होगा, जो 15, 18 और 42 से
(b) 7 (d) 0 िवभािजत होने पर ेक थित म
(c) 5 एक ही शेष 8 रहता है और 13 से
Pi

(d) 6 Q 49. The students of a class िवभा भी है ?


donated Rs. 3,481 towards relief SSC CPO 13 March 2019
Q46. When 6892, 7105 and 7531 fund. Each student donated an (Evening)
are divided by the greatest amount equal to the number of (a) 25
number x, then the remainder in students in the class. The number (b) 24
each case is y. What is the value of students in the class are: (c) 22
of (x-y) ? क ा के छा ों ने राहत िनिध म 3481 (d) 26
जब 6892, 7105 और 7531 को पये का दान िदया | ेक छा ने
सबसे बड़ी सं ा x से िवभािजत क ा म छा ों की सं ा के बराबर Q 53. The square root which of
िकया जाता है , तो ेक मामले म रािश का योगदान िदया | इस क ा म the following is a rational
शेषफल y आता है | (x-y) का मान छा ों की सं ा है | number?
ात कर | SSC CPO 16 March 2019 िन म से िकसका वगमूल एक
(Morning) प रमेय सं ा है ?

www.ssccglpinnacle.com support@ssccglpinnacle.com Ph. 09729327755, 09817390373 9


/
Day 1-4 days Number System

s
sse
la
_c
ob
SSC CPO 13 March 2019 SSC CHSL 11 July 2019 शेषफल समान आता है | x के अंकों
bo
ah
(Evening) (Morning) का जोड़ है :
sm
ur
yo

(a) 5535.36 (a) 72 SSC CGL Tier II- 11


e/
t.m
://

(b) 3152.88 (b) 7 September 2019


tp
ht

3
(c) 72905.2 (c) 3 (a) 14
7
(d) 67508.5 5 (b) 5
(d) 7
(c) 9
Q54. To what power -3 should be (d) 6
SSC CGL TIER II
raised to get -2187?
-2187 ा करने के िलए -3 को Q4. One of the factors of (
Q1. If a nine-digit number
िकस घात तक बढ़ाना होगा ? 82k + 52k ), where k is an odd
389x6378y is divisible by 72,
SSC CPO 14 March 2019 number, is :
(Morning) then the value of √6x + 7y will
( 82k + 52k ) का एक गुणक ात
(a) 5 be :
कर, जहाँ k एक िवषम सं ा है |
(b) 7 यिद नौ अंकों की एक सं ा
SSC CGL Tier II- 11
389x6378y , 72 से िवभािजत है , तो

e
(c) -7 September 2019
(d) -5 √6x + 7y का मान होगा : (a) 86
SSC CGL Tier II- 11 (b) 88
Q55.The cube root of 3375 is September 2019 (c) 84

l
equal to: (a) 6 (d) 89
3375 का घन मूल ात कर | (b) √13
(Morning)
(a) 35
(b) 25
(c) 55
ac
SSC CPO 15 March 2019
(c) √46
(d) 8

Q2. When 12,16,18,20 and 25


divide the least number x, the
Q5. Let x = (633)24 − (277)38 +
+ (266)54 . what is the unit digit
of x ?
मान लीिजये िक
38 54
x = (633)24 −
(d) 15 − (277) + (266) है | x का
remainder in each case is 4 but x
इकाई अंक ा है ?
is divisible by 7. What is the digit
Q56.To get a perfect square, what SSC CGL Tier II- 11
nn
at the thousands’ place in x ?
should the minimum number be September 2019
जब 12, 16, 18, 20 और 25 ूनतम
added to 8212. (a) 7
सं ा x को िवभािजत करते ह, तो
एक पूण वग (perfect square) ा (b) 6
हर मामले म शेषफल 4 आता है
करने के िलए, म 8212 कौन सी (c) 4
लेिकन x, 7 से िवभािजत है | x के
सबसे छोटी सं ा जोड़ी जानी (d) 8
हजारव थान पर कौन सा अंक है ?
चािहए:
SSC CGL Tier II- 11
SSC CPO 15 March 2019 Q6. The sum of the digits of a
September 2019
(Evening) two-digit number is 71 of the
Pi

(a) 5
(a) 123 number. The units digit is 4 less
(b) 8
(b) 69 than the tens digit. If the number
(c) 4
(c) 112 obtained on reversing its digit is
(d) 3
(d)54 divided by 7, the remainder will
be :
Q3. When 7897, 8110 and 8536
Q57. The ratio of square of a दो अंकों की एक सं ा के अंकों का
are divided by the greatest
number and the reciprocal of its जोड़ सं ा का 71 है | इकाई अंक
243 . What is the
number x, then the remainder in
cube is 16807
each case is the same. The sum of दहाईं के अंक से 4 कम है | यिद
number? the digits of x is : इसके अंकों को पलटने से बनी सं ा
िकसी सं ा के वग तथा इसके घन जब 7897, 8110 और 8536 को को 7 से भाग िदया जाए, तो शेषफल
के पार रक ( reciprocal ) का सबसे बड़ी सं ा x से िवभािजत होगा :
अनुपात 16807
243 है | यह सं ा कौन िकया जाता है , तो ेक मामले म SSC CGL Tier II- 11
सी है ? September 2019

www.ssccglpinnacle.com support@ssccglpinnacle.com Ph. 09729327755, 09817390373 10


/
Day 1-4 days Number System

s
sse
la
_c
ob
(a) 4 िवभािजत करने पर ेक थित म (c) -2
bo
ah
(b) 5 शेषफल 10 आता है और x, 31 का (d) 2
sm
ur
yo

(c) 1 एक गुणज है | x म ूनतम कौन सी


e/
t.m

सं ा जोड़ी जानी चािहए तािक यह


://

(d) 6 Q13. In finding the HCF of two


tp
ht

पूण वग बन जाए ? numbers by division method, the


Q7. If the 11-digit number SSC CGL Tier II- 12 last divisor is 17 and the quotients
5678x43267y is divisible by 72, September 2019 are 1, 11 and 2, respectively.
then the value of √5x + 8y is : (a) 39 What is the sum of the two
यिद 11-अंकों की सं ा (b) 37 numbers ?
5678x43267y , 72 से िवभािजत है , (c) 43 िवभाजन िविध से दो सं ाओं का
तो का मान ा होगा ? (d) 36 HCF िनकालने के दौरान अंितम
√5x + 8y भाजक 17 है तथा शेषफल मशः 1,
SSC CGL Tier II- 12
Q11. When a two-digit number is 11 तथा 2 ह | दोनों सं ाओं का योग
September 2019
multiplied by the sum of its ा है ?
(a) 6
digits, the product is 424. When SSC CGL Tier II- 13
(b) 4

e
the number obtained by September 2019
(c) 7
interchanging its digits is (a) 833
(d) 8
multiplied by the sum of the (b) 867
digits, the result is 280. The sum (c) 816
Q8. The number of factors of

l
of the digits of the given number (d) 901
3600 is : / 3600 के गुणकों की
is :
सं ा है : ac दो अंकों की एक सं ा को जब Q14. If a 10-digit number
SSC CGL Tier II- 12
इसके अंकों के योग से गुना िकया 5432y1749x is divisible by 72,
September 2019
जाता है , तो गुणनफल 424 आता है | then what is the value of (5x-4y)
(a) 45
जब इसके अंकों को पलटने से बनी ?
(b) 44
सं ा को अंकों के जोड़ से गुना यिद 10 अंकों की एक सं ा
(c) 43
िकया जाता है , तो प रणाम 280 5432y1749x, 72 से िवभािजत है , तो
(d) 42
आता है | दी गयी सं ा के अंकों का (5x-4y) का मान ा है ?
योग है - SSC CGL Tier II- 13
nn
Q9. If x = (164)169 + (333)337 -
SSC CGL Tier II- 12 September 2019
(727)726 , then what is the unit September 2019 (a) 14
digit of x ? (a) 6 (b) 15
यिद x = (164)169 + (333)337 - (b) 9 (c) 10
(727)726 है , तो x का इकाई अंक (c) 8 (d) 9
ा है ? (d) 7
SSC CGL Tier II- 12 Q15. What is the remainder when
September 2019 Q12. If x is the remainder when ( 12797 + 9797 ) is divided by 32 ?
Pi

(a) 5 361284 is divided by 5 and y is the जब ( 12797 + 9797 ) को 32 से भाग


(b) 7 remainder when 496 is divided by िदया जाता है , तो शेषफल ा
(c) 8 6, then what is the value of (2x-y) आएगा ?
(d) 9 ? SSC CGL Tier II- 13
यिद x, 361284 को 5 से िवभािजत September 2019
Q 10. Let x be the least number करने पर आने वाला शेषफल है तथा (a) 4
which when divided by 15,18,20 y, 496 को 6 से िवभािजत करने पर (b) 2
and 27, the remainder in each (c) 7
आने वाला शेषफल है , तो (2x-y) का
case is 10 and x is a multiple of (d) 0
मान ा है ?
31. What least number should be
SSC CGL Tier II- 13
added to x to make it a perfect Q16. Two positive numbers differ
September 2019
square ? by 2001. When the larger number
(a) -4
मान लीिजये िक x वह सबसे छोटी is divided by the smaller number,
(b) 4
सं ा है िजसे 15, 18, 20 और 27 से

www.ssccglpinnacle.com support@ssccglpinnacle.com Ph. 09729327755, 09817390373 11


/
Day 1-4 days Number System

s
sse
la
_c
ob
the quotient is 9 and the SSC CGL 3 March 2020 Q7. If the nine-digit number
bo
ah
remainder is 41. The sum of the (Evening) 708x6y8z9 is divisible by 99,
sm
ur
yo

digits of the larger number is : then what is the value of x+y+z?


e/
t.m

दो धना क सं ाओं म 2001 का यिद नौ अंकों की सं ा 708x6y8z9


://

(a) 3
tp
ht

अंतर है | जब बड़ी सं ा को छोटी (b) 7 , 99 से िवभा है , तो x+y+z का


सं ा से भाग िदया जाता है , तो (c) 1 मान ा है ?
भागफल 9 आता है तथा शेषफल 41 (d) 8 SSC CGL 5 March 2020
आता है | बड़ी सं ा के अंकों का (Morning)
योग है : Q4. When 732 is divided by a (a) 9
SSC CGL Tier II- 13 positive integer x, the remainder (b) 16
September 2019 is 12. How many values of x are (c) 5
(a) 15 there? / (d) 27
(b) 11 जब 732 को एक धना क पूणाक x
(c) 10 से भाग िदया जाता है , तो शेषफल 12 Q8. When a positive integer is
(d) 14 आता है | x के िकतने मान ह ? divided by d, the remainder is 15.

e
SSC CGL 4 March 2020 When ten times of the same
SSC CGL TIER I (Morning) number is divided by d, the
(a) 19 remainder is 6. The least possible
Q1. If the number 1005x4 is (b) 20 value of d is:
जब एक धना क पूणाक को d से

l
completely divisible by 8, then (c) 18
the smallest integer in place of x (d) 16 भाग िदया जाता है , तो शेषफल 15
will be:
ac
यिद सं ा 1005x4 , 8 से पूणतः
िवभािजत है , तो x के थान पर सबसे
छोटा पूणाक ा आएगा ?
SSC CGL 3 March 2020
Q5. If the 6-digit numbers
x35624 and 1257y4 are divisible
by 11 and 72, respectively, then
what is the value of (5x-2y)?
आता है | जब इसी सं ा के 10 गुना
को d से भाग िदया जाता है , तो
शेषफल 6 आता है | d का ूनतम
संभव मान हो सकता है :
SSC CGL 5 March 2020
(Morning) यिद 6 अंकों की सं ा x35624 तथा (Afternoon)
(a) 2 1257y4 मशः 11 और 72 से (a) 9
(b) 4 िवभा है , तो (5x-2y) का मान ा (b) 12
nn
(c) 1 होगा ? (c) 16
(d) 0 SSC CGL 4 March 2020 (d) 18
(Afternoon)
Q2. When 200 is divided by a (a) 14 Q9. The greatest number which
positive integer x, the remainder (b) 12 should be replace ‘*’ in the
is 8. How many values of x are (c) 10 number 146*48 to make it
divisible by 8 is:
there? (d) 13
146*48 को 8 से िवभा बनाने के
जब 200 को िकसी धना क पूणाक
Pi

िलए * के थान पर सबसे बड़ी िकस


x से भाग िदया जाता है , तो शेषफल 8 Q6. How many numbers are there सं ा को रखा जाना चािहए ?
आता है | x के िकतने मान ह ? from 200 to 800 which are neither SSC CGL 5 March 2020
SSC CGL 3 March 2020 divisible by 5 nor by 7? (Evening)
(Afternoon) 200 से 800 तक ऐसी िकतनी सं ाएँ (a) 9
(a) 7 ह जो ना तो 5 से ना ही 7 से िवभा (b) 2
(b) 5 ह? (c) 8
(c) 8 SSC CGL 4 March 2020 (d) 0
(d) 6 (Evening)
Q10. If the number 687x29 is
Q3. What should replace * in the (a) 407
divisible by 9, then the value of
number 94*2357, so that the (b) 410
2x is:
number is divisible by 11? (c) 413
यिद सं ा 687x29 , 9 से िवभा
सं ा 94*2357 म * के थान पर (d) 411
है , तो 2x का मान ा होगा ?
कौन सी सं ा आनी चािहए तािक
यह सं ा 11 से िवभािजत हो जाए |

www.ssccglpinnacle.com support@ssccglpinnacle.com Ph. 09729327755, 09817390373 12


/
Day 1-4 days Number System

s
sse
la
_c
ob
SSC CGL 6 March 2020 Q14. What is the remainder when (c) 963391
bo
ah
(Morning) we divide 570+770 by 74? (d) 107611
sm
ur
yo

(a) 8 जब हम 570+770 म 74 से भाग दे ते ह,


e/
t.m

तो शेषफल ा आएगा ?
://

(b) 3 Q2. When ( 7777 + 77) is divided


tp
ht

(c) 2 SSC CGL 7 March 2020 by 78, the remainder is:


(d) 4 (Evening) जब ( 7777 + 77) को 78 से िवभािजत
(a) 7 िकया जाता है , तो शेषफल िकतना
Q11. The largest number which (b) 1 आएगा?
could replace * in the 2365*4 to (c) 0 CHSL 2019 12-10-2020
make the number divisible by 4 is (d) 5 (Afternoon shift)
: (a) 75
2365*4 को 4 से िवभा बनाने के Q15. If 5 divided the integer n, (b) 77
िलए * के थान पर सबसे बड़ी कौन the remainder is 2. What will be (c) 76
सी सं ा आ सकती है ? the remainder if 7n is divided by (d) 74
SSC CGL 6 March 2020 5?
यिद 5 से पूणाक n म भाग िदया जाए,

e
(Afternoon) Q3. When an integer n is divided
तो शेषफल 2 आता है | यिद 7n म 5 by 5, the remainder is 3. What is
(a) 8 से भाग िदया जाए, तो शेषफल the remainder if 8n is divided by
(b) 9 िकतना आएगा ? 5?

l
(c) 2 SSC CGL 9 March 2020 जब िकसी पूणाक n को 5 से भाग
(d) 0 (Afternoon) िदया जाता है , तो शेषफल 3 आता है ।
ac
Q12. If the given number 925x85
is divisible by 11, then the
smallest value of x is:
यिद दी गयी सं ा 925x85 , 11 से
(a) 1
(b) 4
(c) 3
(d) 2
जब 8n को 5 से भाग िदया जाएगा, तो
शेषफल िकतना आएगा?
CHSL 2019
(Evening shift)
(a) 1
12-10-2020

िवभा है , तो x का सबसे छोटा मान (b) 4


िकतना होगा ? Q16. The greatest number which (c) 2
SSC CGL 7 March 2020 may replace * in the number
nn
(d) 3
(Morning) 1190*6 to make the number
(a) 4 divisible by 9 is: Q4. Which of the following
(b) 2 सं ा 1190*6 म * के थान पर numbers is divisible by 6?
(c) 1 सबसे बड़ी कौन सी सं ा आ सकती िन म से कौन सा सं ा 6 से
(d) 3 है , िजससे यह 9 से िवभा हो िवभाजय है
जाएगी ? CHSL 2019 13-10-2020
Q13. If 7 divided a positive SSC CGL 9 March 2020 (Morning Shift)
integer n, the remainder is 2. (Evening) (a)23,408
Pi

Which of the following numbers (a) 1 (b)43,923


gives a remainder of 0 when (b) 9 (c)1,00,246
divided by 7? (c) 3 (d)3,49,722
यिद िकसी धना क पूणाक n को 7 (d) 0
से भाग दे ने पर शेषफल 2 आता है , 2 1
Q5. The value of(a 3 + 2a 2 + 3a 3 +
1

तो िन िल खत म से िकस सं ा को Q1. Which of the following 1 1 1 1 1


2a 6 +1)(a 3 -2a 6 + 1) - a 2 (a 2 -2),
7 से भाग दे ने पर शेषफल 0 आएगा ? options is completely divisible by when a = 7, is:
SSC CGL 7 March 2020 11 2 1 1 1 1 1
(a 3 + 2a 2 + 3a 3 + 2a 6 +1)(a 3 -2a 6 +
(Afternoon) िन िल खत म से कौन सा िवक 11 1) - a 2 (a 2 -2)का मान ात कीिजए,
1 1

(a) n-5 से पूणतः िवभािजत है ? जब a = 7 है :


(b) n+5 CHSL 2019 12-10-2020 CHSL 2019 13-10-2020
(c) n+2 (Morning Shift) (Morning Shift)
(d) n+1 (a) 809781 (a)7
(b) 116571

www.ssccglpinnacle.com support@ssccglpinnacle.com Ph. 09729327755, 09817390373 13


/
Day 1-4 days Number System

s
se
s
la
_c
ob
(b)0 (c) 12 िन िल खत म से कौन सी सं ा 3 से
bo
ah
(c)1 (d) 11 िवभािजत है ?
sm
ur
yo

(d) √7 CHSL 2019 15-10-2020


e/
t.m
://

Q10. The largest five-digit (Evening shift)


tp
ht

Q6. If the number 59a44b is number that is exactly divisible (a) 8703572
divisible by 36, then the by 81 is: (b) 8703541
maximum value of a + b is: पाँ च अंकों की सबसे बड़ी सं ा, जो (c) 8703593
यिद सं ा 59a44b, 36 से िवभा 81 से पूणतः िवभािजत है : (d) 8765001
है , तो a+b का अिधकतम मान CHSL 2019 14-10-2020
िकतना होगा ? (Evening shift) Q15. If ‘a’ is a natural number,
CHSL 2019 13-10-2020
(a) 99989 then (7a2 + 7a) is always divisible
(Afternoon shift)
(b) 99991 by:
(a) 16
(c) 99954 अगर ‘a’ एक ाकृितक सं ा है ,तो
(b) 12
(d) 99876 (7a2 + 7a) हमेसा िवभािजत होगी:
(c) 14
(d) 10 CHSL 2019 16-10-2020

e
Q11.The factors of the expression (Morning Shift)
Q7. Find the factors of the 2x2 - 5x - 12 are: (a) 7 and 14 both
expression 3x2- 5x - 8. ंजक 2x2 - 5x - 12 के (b) 7 only
ंजक 3x2- 5x - 8 के गुणनख गुणनख ा है ात करे : (c) 14 only

l
का पता लगाएं CHSL 2019 15-10-2020 (d) 21 only
CHSL 2019 ac 13-10-2020 (Morning Shift)
(Evening shift) (a)(x - 4) and (2x - 3) Q16. (ax + by) is a factor of:
(a) (x - 1) and (3x - 8) (b)(x + 4) and (2x + 3) (ax + by) िकसका एक गुणनख
(b) (x + 1) and (3x - 8) (c)(x - 4) and (2x + 3) है :
(c) (x + 1) and (3x + 8) (d)(x + 4) and (2x - 3) CHSL 2019 16-10-2020
(d) (x - 1) and (3x + 8) (Afternoon shift)
Q12.If 7129p465 is divisible by (a) a2x2 + 2ab - b2y2
Q8. Which are the two nearest 9, then the value of p is: (b) a2x2 + 2abxy + b2y2
यिद 7129p465 , 9 से िवभा है , तो (c) a2x3 + 2abx + b2y2x
nn
numbers to 19,596, divisible by
9? p का मान है : (d) a2x2 + 2ab + b2y2
19,596 से कौन सी दो िनकटतम CHSL 2019 15-10-2020
सं ाएँ ह, जो 9 से िवभा ? (Morning Shift) Q17. 2x-3y is a factor of:
CHSL 2019 14-10-2020 (a)3 2x-3y िकसका एक गुणनख है :
(Morning Shift) (b)0 CHSL 2019 16-10-2020
(a) 19,564; 19,620 (c)4 (Afternoon shift)
(b) 19,611; 19,575 (d)2 (a) 4x2 + 2x - 3y + 9y2 - 12xy
(c) 19,509; 19,611 (b) 8x3 + 27y3
Pi

(d) 19,593; 19,602 Q13. What is the least 5-digit (c) 4x2 + 9y2 + 12xy
number that is divisible 91? (d) 4x2 + 2x - 3y + 36y2 + 12xy
Q9. If the 8-digit number कम से कम 5-अंकीय सं ा ा है
1a765b12 is to be divisible by 72, जो 91 से िवभा है ? Q18. Given that 220+1 is
the least value of (2a + 3b) is: CHSL 2019 15-10-2020 completely divisible by a whole
यिद 8-अंकीय सं ा 1a765b12 को (Afternoon shift) number. Which of the following
72 से िवभा करना है , तो (2a + (a) 10283 is completely divisible by the
3b) का कम से कम मान ा है ात (b) 10101 same number?
करे : (c) 10010 यह िदया गया है , 220+1 एक पूरी
CHSL 2019 14-10-2020 (d) 10192 सं ा से िवभािजत है । िन िल खत म
(Afternoon shift) से कौन उसी सं ा से पूरी तरह से
(a) 10 Q14. Which of the following िवभा है ?
(b) 9 numbers is divided by 3? CHSL 2019 16-10-2020
(Afternoon shift)

www.ssccglpinnacle.com support@ssccglpinnacle.com Ph. 09729327755, 09817390373 14


/
Day 1-4 days Number System

s
sse
la
_c
ob
(a) 215+1 अंक पूणाक ह, तो M और N के मान divided by 14, then the remainder
bo
ah
(b) 5 × 230 ह will be:
sm
ur
yo

(c) 290+1 CHSL 2019 20-10-2020 जब एक सं ा को 14 से िवभािजत


e/
t.m

िकया जाता है , तो शेष 9 बचता है |


://

(d) 260+1 (Morning Shift)


tp
ht

(a) M = 5, N = 6 यिद उसी सं ा का वग 14 से


Q19. If a positive integer 'n' is (b) M = 2, N = 5 िवभािजत िकया जाता है , तो शेष
divisible by 3, 5, and 7, then what (c) M = 5, N = 2 होगा |
is the next larger integer divisible (d) M = 5, N = 4 CHSL 2019 21-10-2020
by all these numbers? (Afternoon shift)
यिद एक सकारा क पूणाक ‘n’ 3, Q.23. If the 8-digit number (a) 11
5, और 7 से िवभा है , तो सभी 43A5325B is divisible by 8 and 9, (b) 9
सं ाओं के ारा अगले कौन सा then the sum of A and B is equal (c) 10
बड़ा पूणाक िवभा होगा ? to: (d) 8
CHSL 2019 16-10-2020 यिद 8-अंकीय सं ा 43A5325B, 8
(Evening shift) और 9 से िवभा है , तो A और B Q.27. 218 − 1 is divisible by:
का योग ात करे |

e
(a) n + 21 218 − 1 िकसके ारा िवभा है
(b) n + 35 CHSL 2019 20-10-2020 CHSL 2019 21-10-2020
(c) n + 105 (Afternoon shift) (Evening shift)
(d) n + 110 (a) 12 (a) 11

l
(b) 18 (b) 17
Q20. How many numbers (c) 14 (c) 13
ac
between 800 and 2000 are
divisible by 13?
800 और 2000 के बीच की िकतनी
सं ाएँ 13 से िवभा ह?
CHSL 2019 19-10-2020
(d) 15

Q.24. 225 + 226 + 227 is divisible


by / िकसके ारा िवभा है
CHSL 2019 19-10-2020
(d) 7

Q.28. If a number is divided by


899, the remainder is 63. If the
same number is divided by 29,
(Morning Shift) (Evening shift) the remainder will be:
(a) 90 (a) 6 यिद एक सं ा को 899 से िवभािजत
(b) 92 (b) 7 िकया जाता है , तो शेष 63 बचता है ।
nn
(c) 91 (c) 5 यिद उसी सं ा को 29 से िवभािजत
(d) 93 (d) 9 िकया जाता है , तो शेष सं ा ा
बचेगा
Q21. What should be the value of Q.25. If 8- digit number CHSL 2019 26-10-2020
N to make 396258N divisible by 4432A43B is divisible by 9 and 5, (Morning Shift)
8? then the sum of A and B is equal (a) 10
8 से 396258N को िवभा बनाने के to: (b) 2
िलए N का मान ा होना चािहए? यिद 8- अंक सं ा 4432A43B, 9 (c) 4
Pi

CHSL 2019 19-10-2020 और 5 से िवभा है , तो A और B (d) 5


(Evening shift) का योग ात करे |
(a) 2 CHSL 2019 21-10-2020 Q.29. If the number 62783xy is
(b) 8 (Morning Shift) divisible by both 8 and 5, then the
(c) 4 (a) 12 smallest possible value of x and y
(d) 6 (b) 5 is:
(c) 7 यिद सं ा 62783xy 8 और 5 दोनों
Q22. If 4M37094267N is (d) 8 से िवभा है , तो x और y का सबसे
divisible by both 8 and 11, where छोटा संभािवत मान है
M and N are single digit integers, Q.26. When a number is divided CHSL 2019 21-10-2020
then the values of M and N are: by 14, the remainder is 9. If the (Afternoon shift)
यिद 4M37094267N 8 और 11 दोनों square of the same number is (a) x = 2, y = 2
से िवभा है , जहाँ M और N एकल (b) x = 6, y = 0

www.ssccglpinnacle.com support@ssccglpinnacle.com Ph. 09729327755, 09817390373 15


/
Day 1-4 days Number System

s
sse
la
_c
ob
(c) x = 2, y = 0 सं ा को 5 से जोड़ा जाता है और 1 + 3 + 5 + 7 + ……………. (2n -
bo
ah
(d) x = 2, y = 5 िफर 3 से िवभािजत िकया जाता है , तो 1) का मान है :
sm
ur
yo

शेष ा होगा? CHSL 2019 18-03-2020


e/
t.m
://

Q.30. If x3 + 2x2 − ax − b is CHSL 2019 17-03-2020 (Evening Shift)


tp
ht

exactly divisible by (x2 − 1) , then (Afternoon shift) (a) (2n - 1) × (2n - 1)


the values of a and b are: (a) 3 (b) 2n
यिद x3 + 2x2 − ax − b , (x2 − 1) से (b) 1 (c) n × n
संपूण िवभा है , तो a और b के मान (c) 2 n(n + 1)
(d) 2
ात कर | (d) 0
CHSL 2019 21-10-2020
Q:38. Which of the following
(Afternoon shift) Q34. Which of the following
options is divisible by 3?
(a) a = -1, and b = 2 numbers is divisible by 9?
िन िल खत म से कौन सा िवक 3
(b) a = 1 and b = -2 िन िल खत म से कौन सी सं ा 9
से िवभा है ?
(c) a = 1 and b = 2 से िवभा है ?
CHSL 2019 18-03-2020
(d) a = 2 and b = 2 CHSL 2019 17-03-2020
(Evening Shift)

e
(Evening Shift)
(a) 2362735
Q.31. The divisor is 24 times the (a) 897342
(b) 6342589
quotient and 8 times the (b) 594327
(c) 3745932
remainder. If the quotient is 18, (c) 346217
(d) 4539763

l
then the dividend is: (d) 734895
भाजक भागफल का 24 गुना और
ac Q:39. Which of the following
शेष 8 गुना है । यिद भागफल 18 है , Q35. Which of the following
numbers is divisible by both 7
तो लाभां श है : numbers is divisible by 2, 5 and
and 11?
CHSL 2019 26-10-2020 10?
िन िल खत म से कौन सी सं ा 7
(Evening Shift) िन िल खत म से कौन सी सं ा 2, 5
और 11 दोनों से िवभा है ?
(a) 7830 और 10 से िवभा है ?
CHSL 2019 19-03-2020
(b) 7630 CHSL 2019 18-03-2020
(Morning Shift)
(c) 7840 (Morning Shift)
(a) 16,324
(d) 7450 (a) 7,20,345
nn
(b) 12,335
(b) 1,25,372
(c) 16,257
Q.32. Which of the following (c) 19,400
(d) 16,425
numbers will completely divide (d) 149
781 + 782 + 783 ? Q.40. Which number is divisible
िन िल खत म से कौन सी सं ा Q36. If a positive integer n is
by both 9 and 11?
781 + 782 + 783 को पूरी तरह से divided by 7, the remainder is 2.
कौन सी सं ा 9 और 11 दोनों से
िवभािजत करे गी? Which of the numbers in the
िवभा है ?
options yields a remainder of 0
Pi

CHSL 2019 17-03-2020 CHSL 2019 19-03-2020


(Morning Shift) when it is divided by 7?
(Evening Shift)
(a) 399 यिद एक धना क पूणाक n को 7 से
(a) 10,089
(b) 389 िवभािजत िकया जाता है , तो शेष 2
(b) 10,098
(c) 387 है । िवक ों म से कौन सी सं ा 7 से
(c) 10,108
(d) 397 िवभािजत होने पर शेष 0 दे ता है ?
(d) 10,087
CHSL 2019 18-03-2020
Q.33. If a number is divided by 3, (Afternoon shift)
SSC CGL 2019 TIER II
the remainder will be 2. If the (a) n + 3
Q41. Two positive numbers differ
number is added by 5 and then (b) n + 1
by 1280. When the greater
divided by 3, then what will be (c) n + 2
number is divided by the smaller
the remainder? (d) n + 5
number, the quotient is 7 and the
यिद एक सं ा को 3 से िवभािजत remainder is 50.The greater
िकया जाता है , तो शेष 2 होगा। यिद Q37. The value of 1 + 3 + 5 + 7 +
number is:
……………. (2n - 1) is:

www.ssccglpinnacle.com support@ssccglpinnacle.com Ph. 09729327755, 09817390373 16


/
Day 1-4 days Number System

s
sse
la
_c
ob
दो सकारा क सं ाएँ 1280 से िभ (a) 8 (c) 9
bo
ah
होती ह। जब बड़ी सं ा को छोटी (b) 9 (d) 8
sm
ur
yo

सं ा से िवभािजत िकया जाता है , तो (c) 5


e/
t.m

भागफल 7 होता है और शेष 50 होता


://

(d) 10 Q48. Find the sum of


tp
ht

है । अिधक सं ा का मान ात 6+8+10+12+14…………..+40.


कीिजए | Q45. Let ab, a =/ b, is a 2-digit 6+8+10+12+14…………..+40 का
CGL 2019 Tier-II (15-11-2020) prime number such that ba is also योग ात करे |
(a) 1458 a prime number. The sum of all CGL 2019 Tier-II (18-11-2020 )
(b) 1485 such number is: (a) 400
(c) 1585 ab,( a =/ b) एक 2-अंकीय अभा (b) 424
(d) 1558 सं ा है जैसे िक ba भी एक अभा (c) 1600
सं ा है । ऐसी सभी सं ाओं का (d) 414
Q42. When positive numbers x, y योग है
and z are divided by 31, the CGL 2019 Tier-II (16-11-2020)
reminders are 17, 24 and 27 (a) 374 Q49. Find the number of prime

e
respectively. When (4x - 2y + 3z) (b) 418 factors in the product (30)5 ×
is divided by 31, the reminder (c) 407 (24)5 .
will be: (d) 396 (30)5 × (24)5 उ ाद म अभा
जब सकारा क सं ा x, y और z
गुणनख की सं ा ात कीिजए
को 31 से िवभािजत िकया जाता है , तो

l
Q46. Let x be the least number CGL 2019 Tier-II (18-11-2020 )
शेषफल मशः 17, 24 और 27 होते which subtracted from 10424
ac (a) 45
ह। जब (4x - 2y 3z) को 31 से gives a perfect square number. (b) 35
िवभािजत िकया जाता है , तो शेषफल What is the least number by (c) 10
ा होगा | which x should be multiplied to (d) 30
CGL 2019 Tier-II (15-11-2020) get a perfect square?
(a) 9 मान ल िक x सबसे कम सं ा है जो SSC CPO 2019
(b) 8 10424 से घटाया गया है , एक पूण वग Q50. If 14331433 × 1422 × 1425
(c) 16 सं ा दे ता है । एक पूण वग ा
is divided by 12, then what is the
(d) 19 करने के िलए िकस सं ा को x से
nn
remainder?
गुणा िकया जाना चािहए|
यिद 14331433 × 1422 × 1425 को
Q43: If the 5-digit number 535ab CGL 2019 Tier-II (16-11-2020)
12 से िवभािजत िकया जाता है , तो
is divisible by 3, 7 and 11, then (a) 3
शेषफल ात करे |
what is the value of ( (b) 6
CPO 2019
a 2 − b 2 + ab )? (c) 5
23-11-2020(Morning Shift)
यिद 5-अंकीय सं ा 535ab 3, 7 (d) 2
(a) 3
और 11 से िवभा है , तो (
(b) 9
a − b + ab ) का मान ा है ?
2 2 Q47. When positive number a, b
Pi

(c) 8
CGL 2019 Tier-II (15-11-2020) and c are divided by 13, the
(d) 6
(a) 95 remainder are 9, 7 and 10,
(b) 83 respectively. What will be the
Q51. If a nine-digit number
(c) 89 remainder when (a + 2b + 5c) is
785x3678y is divisible by 72,
(d) 77 divided by 13?
then the value of ( x - y ) is:
जब सकारा क सं ा A, B और C
यिद एक नौ अंकों की सं ा
Q44. If the five digit number को 13 से िवभािजत िकया जाता है , तो
785x3678y, 72 से िवभा है , तो (x
235xy is divisible by 3, 7 and 11 शेष मश 9, 7 और 10 होते ह। जब
- y) का मान ात करे |
then what is the value of (3x-4y)? (a + 2b + 5c) को 13 से िवभािजत
CPO 2019
यिद पाँ च-सं ा वाली सं ा 235xy िकया जाता है तो शेषफल ा होगा?
23-11-2020(Morning Shift)
3, 7 और 11 से िवभा है तो मान CGL 2019 Tier-II (16-11-2020)
(a) 0
ा है (3x-4y) ? (a) 10
(b) -2
CGL 2019 Tier-II (16-11-2020) (b) 5
(c) -1

www.ssccglpinnacle.com support@ssccglpinnacle.com Ph. 09729327755, 09817390373 17


/
Day 1-4 days Number System

s
sse
la
_c
ob
(d) 2 CPO 2019 24-11-2020(Evening Q59. If a nine digit number
bo
ah Shift) 785x3678y is divisible by 72,
sm
ur
yo

Q52. The remainder when 75 × 73 (a) 71 then the value of (x + y) is:


e/
t.m

यिद एक नौ अंकों की सं ा
://

× 78 × 76 is divided by 34 is: (b) 53


tp
ht

शेष 75 × 73 × 78 × 76 को 34 से (c) 30 785x3678y 72 से िवभा है , तो (x


िवभािजत िकया गया है : (d) 48 y) का मान ा है ात करे |
CPO 2019 23-11-2020(Evening CPO 2019 25-11-2020(Morning
Shift) Q56. What is the least number of Shift)
(a) 18 soldiers that can be drawn up in (a) 10
(b) 12 troops of 10, 12, 15, 18 and 20 (b) 20
(c) 22 soldiers, and also in form of a (c) 5
(d) 15 solid square? (d) 12
सैिनकों की वह काम से काम सं ा
Q53. If six-digit number 5x2y6z ा है िजससे 10, 12, 15, 18 और Q60. How many numbers
is divisible by 7, 11 and 13, then 20 सैिनकों की टु किड़या बनाई जा between 300 and 700 are divisible
सकती है | और एक वग भी बनाया जा

e
the value of (x - y + 3z) is: by 5, 6 and 8?
यिद छह अंकों की सं ा 5x2y6z 7, सकता है | 300 और 700 के बीच िकतनी सं ा
11 और 13 से िवभा है , तो (x - y CPO 2019 24-11-2020(Evening 5, 6 और 8 से िवभा है |
+3z) का मान ात करे | Shift) CPO 2019 25-11-2020(Evening

l
CPO 2019 (a) 180 Shift)
24-11-2020(Morning Shift) (b) 625 (a) 20
(a) 4
(b) 0
(c) 7
(d) 9
ac (c) 900
(d) 400

Q57. How many numbers


between 400 and 700 are divisible
(b) 2
(c) 5
(d) 3

Q61. When a number is


Q54. The remainder when 72 × by 5, 6 and 7? successively divided by 3, 4 and 7
73 × 78 × 76 is divided by 35 is : 400 और 700 के बीच िकतनी सं ा , the remainder obtained is 2,3
को 35 से 5, 6 और 7 से िवभा है and 5, respectively. What will be
nn
72 × 73 × 78 × 76
िवभािजत िकया गया है तो शेष ात CPO 2019 24-11-2020(Evening the remainder when 42 divides
करे | Shift) the same number?
CPO 2019 24-11-2020(Morning (a) 2 जब कोई सं ा िमक प से 3, 4
Shift) (b) 5 और 7 से िवभािजत होती है , तो ा
(a) 8 (c) 10 शेष मशः 2,3 और 5 होती है । जब
(b) 15 (d) 20 42 एक ही सं ा को िवभािजत करते
(c) 22 ह तो शेष ा होगा |
Q58. The remainder when CPO 2019 25-11-2020(Evening
Pi

(d) 12
14331433 × 1422 × 1425 is Shift)
Q55. When a number is divided by 10 is : (a) 31
successively divided by 3, 4 and 14331433 × 1422 × 1425 को 10 (b) 41
7, the remainder obtained are 2, 3 से िवभािजत िकया गया है तो शेष (c) 30
and 5 respectively. What will be ात करे | (d) 29
the remainder when 84 divides CPO 2019 25-11-2020(Morning
the same number? Shift)
जब कोई सं ा िमक प से 3, 4 (a) 0
और 7 से िवभािजत होती है , तो ा (b) 9
शेष मशः 2, 3 और 5 होते ह। जब (c) 3
उसी सं ा को 84 से िवभािजत करते (d) 8
ह तो शेष ा होगा ?

www.ssccglpinnacle.com support@ssccglpinnacle.com Ph. 09729327755, 09817390373 18


/
Day 1-4 days Number System

s
sse
la
_c
ob
SOLUTION: So, (2+9+x+4+y)-(0+4+8+3+2) = Remainder when 65 is divided by
bo
ah 0 or 11 8=1
sm
ur
yo

Variety Questions ⇒ x+y-2 = 0 or 11


e/
t.m
://

For x+y-2 = 0 Sol 6. (a) Since, the number


tp
ht

Sol 1. (b) x+y = 2 43x1145y2 is divisible by 88, it


Since, 985x3678y is divisible by Possible values of x,y = must be divisible by 11 and 8.
72 it must be divisible by 9 and 8 (0,2),(1,1),(2,0) For the number to be divisible by
(coprime factors of 72) and y For x+y-2 = 11 11,
must be an even number. So the x+y = 13 [(4+x+1+5+2)-(3+1+4+y) = 0 or
sum of digits of this number must Possible values of x,y = (4,9), 11
be divisible by 9 and last three (5,8), (6,7), (7,6), (8,5), (9,4) So, 4+x-y = 0 or 11
digits by 8. Last three digits of the number 4+x-y =0 is possible for
9+8+5+x+3+6+7+8+y= 46 + x + are divisible by 8 only for y = 1,5, x=0,1,2,3,4,5 and y=5,6,7,8,9
y 9 4+x-y = 11 is possible for x =
x+y must be 8 as after 46 nearest For y=1,9 (5x-7y) gives negative 7,8,9 and y = 0,1,2

e
multiple of 9 is 54. results which are not given in the Also a number is divisible by 8 if
Pairs for 8 = (0,8)(1,7), (2,6), option. its last three digits are divisible
(3,5), (4,4), (5,3)(6,2)(7,1)(8,0) So the desired values of x and y by 8.
Only pair which satisfies these are 8 and 5 respectively. We will check for minimum value

l
conditions is (4,4). So the (5x-7y) ⇒ 5(8)-7(5) = 5 ans of y
required value is 4(4)-3(4) = 4 For y=0 , the last three digits of

Sol 2. (c)
ac
Since 5y5884805x6 is divisible
by 72, it must be divisible by 9
and 8 (coprime factors of 72). So
Sol 4. (d)
Since, 517x324 is divisible by 12
it must be divisible by 3 and
4(coprime factors of 12).
For a number to be divisible by 3
the number are not divisible by 8.
For y = 1, the last three digits are
512 which is divisible by 8.
So the desired values of x and y
are 8 and 1 respectively.
the sum of digits of this number sum of its digits must be divisible (3x - 2y) = 3(8)-2(1) = 22 ans
must be divisible by 9 and last by 3.
three digits by 8. So, 5+1+7+x+3+2+4 ⇒ 22+x Sol 7. (d) Since, the number
nn
5+y+5+8+8+4+8+0+5+x+6 ⇒ must be divisible by 3 342x18y6 is divisible by 72, it
49+x+y, Possible values of x = 2,5,8 must be divisible by 9 and 8.
Possible values of x+y = 5, 14 Cleary the smallest value of x = 2 We know that the sum of the
For x+y = 5 digits of a number must be
Possible values of x,y = (1,4), Sol 5. (b) Let the quotient is x. divisible by 9 if the number is
(2,3), (3,2),(4,1) So, n = 8x+3 divisible by 9.
For x+y = 14 6n-1 ⇒ 6(8x+3)-1 Clearly, 3+4+2+x+1+8+y+6 ⇒
Possible values of x,y = (5,9), = 48x+17 24+x+y must be multiple of 9.
Pi

(6,8),(7,7), (8,6),(9,5) 48 is multiple of 8 so 48 will be Possible values of x+y = 3, 12


Among these values last three exactly divisible by 8. But when …….[45 ≻ (24+x+y) ≻ 24, as
digits of the number are divisible we divide 17 by 8 the remainder values of x and y can’t be more
by 8 only when x=3 or 7 is 1. Ans than 9]
But for x=3, y = 2…...(x =/ y ) Also a number is divisible by 8 if
Clearly the desired values of x Alternate its last three digits are divisible
and y are 7 and 7 respectively. by 8.
√xy = √7 × 7 = 7 choose the smallest value of n for We will check for maximum
which remainder is 3 when the value of y
Sol 3. (b) number is divided by 8. For x+y = 12
2094x843y2 is divisible by 88, it Let n = 11 ( x,y ) = (3,9),(4,8),(5,7), (6,6),
must be divisible by 11 and 6n-1 = 6(11)-1 = 65 (7,5), (8,4),(9,3)
8(coprime factors of 88).

www.ssccglpinnacle.com support@ssccglpinnacle.com Ph. 09729327755, 09817390373 19


/
Day 1-4 days Number System

s
sse
la
_c
ob
For y=9, the last three digits of Let the number be (432k + 7)
bo
ah
the number are 896 which is ATQ: For (432k+7) to be exactly
sm
ur
yo

divisible by 8. divisible by 13. {429k+(3k+7)}


e/
t.m
://

So the values of x and y are 3 and should also be divisible by 13.


tp
ht

9 respectively. Putting the value of k=1,2,3,... in


√9x + y = √9(3) + 9 = 6 (3k+7), k=2 satisfies the equation.
Therefore, Least possible number
Sol 8. (a) = 871
Since 2x600000y8 is exactly Sum of digits = 8+7+1 = 16 Sol 18.(d) Possible numbers are:
divisible by 24, it must be 307, 370, 703 and 730.
divisible by 3 and 8. Sol 11. (c) Let n be the any Therefore, Their Sum = 2110
We know that the sum of the number that gives remainder 3
digits of a number must be when divided by 7. Practice Questions
divisible by 3 if the number is For example 10.
divisible by 3. 5n = 50 Sol 1. (b)
For 8-digit number 789x531y to

e
Clearly, 2+x+6+y+8 ⇒ (16+x+y Required remainder = 50
7 =1
)must be the multiple of 3. be divisible by 72. It must be
Possible values of x+y = Sol 12. (c) The number 23474 is divisible by 8 and 9.
2,5,8,11,14 exactly divisible by both 2 and For 789x531y to be divisible by
8. Value of y = 2

l
[(16+x+y) ≻ 16] 11.
Again, for 789x5312 to be
Numbers more than 9 get
ac Sol 13. (c) divisible by 9. Value of x = 1
automatically eliminated as not
Therefore, 5x - 3y = -1
given in the options.
So x+y must be 2,5 or 8.
Sol 2. (a) 4
Also a number is divisible by 8 if
179x091y is divisible by 88, it
its last three digits are divisible
must be divisible by 11 and 8.
by 8.
This number to be divisible by
For x+y = 2
Required number = 327-190 = 11 ⇒ (7+x+9+y) - (1+9+0+1) =
nn
x=y=1 ...(x and y is not equal to
137 0 or 11
zero)
5+x+y = 0 or 11
Last three digits = 018 clearly it is
Sol 14. (c) x+y = -5 or 6
not divisible by 8.
(x+y can’t be negative)
Possible values of x,y for x+y =
For x+y=5
6
x,y = (1,4), (2,3), (3,2) and (4,1)
(0,6), (1,5), (2,4), (3,3), (4,2),
(5,1), (6,0)
Pi

For y=3,2,1 last three digits of the


Last three digits of the number
number are not divisible by 8 so
will be divisible by 8 only in
these three values get eliminated. Least number to be needed = 37-1
case y = 2
For y=4 last three digits are 048 = 36
Clearly x=4 and y=2 are the
which is divisible by 8. Hence
desired values
x=1 and y=4 are the desired Sol 15. (b) Face value of 6 in
5x-8y = 5(4)-8(2) = 4
values. 16008 is 6. As it is the value
itself.
Sol 3. (d) 45
Sol 9. (a) Required number =
Since the number is divisible by
Divisor x Quotient + Remainder Sol 16. (d) Difference between
88 it must be divisible by 11 and
= 38 x 24 + 13 = 925 the largest and the smallest
8.
9652-2569 = 7083
This number to be divisible by
Sol 10. (b) LCM of (12, 16 and
11 ⇒
54) = 432 Sol 17. (b) 44
2+7+x+y - (0+4+4+2) = 0 or 11

www.ssccglpinnacle.com support@ssccglpinnacle.com Ph. 09729327755, 09817390373 20


/
Day 1-4 days Number System

s
sse
la
_c
ob
Case 1: For x=0 , the last three digits of Case 2:
bo
ah
For the difference = 0 the number are not divisible by 8. For the sum = 13
sm
ur
yo

x+y = 1 For x = 1, the last three digits are So, x and y must be
e/
t.m
://

Clearly y is either 0 or 1. 552 which is divisible by 8. (4,9),(5,8),(6,7),(7,6),(8,5) and


tp
ht

The number to be divisible by 8 So the desired values of x and y (9,4)


also last three digits must be are 1 and 5 respectively. The number to be divisible by 8
divisible by 8. 4x-y = 4(1)-5 = -1 also last three digits must be
For y =1 divisible by 8 this is only
Last three digits are 412 and this Sol 5. (b) possible when x = 5 and 9
is not divisible by 8. Since the number 1330x558y2 is When x =5, y =8
For Y = 0 divisible by 88, it must be 3x-y = 3(5)-8 = 7
Last three digits are 402 and this divisible by 11 and 8. When x=9, y = 4
is also not divisible by 8 For the number to be divisible 3x-y = 3(9)-4 = 23
by 11
Case 2: (1+3+x+5+y)-(3+0+5+8+2) = 0 Clearly option c is the right

e
For the difference = 11 or 11 answer.
x+y = 12 x+y-9 = 0 or 11
For Sol 7. (b)
So, x and y must be x+y-9 = 0 Since the number is divisible by

l
(3,9),(4,8),(5,7),(6,6),(7,5),(8,4) x+y=9 88 it must be divisible by 11 and
and (9,3) For 8.
ac
The number to be divisible by 8
also last three digits must be
divisible by 8 this is only
possible when y = 7 and 3
When y =7, x =5
x+y-9 = 11
x+y=20

Sol 6. (c) 7
Since the number is divisible by
This number to be divisible by
11 ⇒ (6+1+7+7+x)
(7+2+y+6+2) = 0 or 11
4+x-y = 0 or 11
-

4x+3y = 4(5)+3(7) = 41 72 it must be divisible by 9 and Case 1:


When y=3, x=9 8. For 4+x-y = 0
4x+3y = 4(9)+3(3) = 45 This number to be divisible by 9 (x,y) is (0,4), (1,5), (2,6), (3,7),
nn
Clearly option d is the right ⇒ (8+9+7+3+5+9+y+7+x+2) (4,8), (5,9)
answer. must be divisible by 9. The number to be divisible by 8
For this x+y = 4 or x+y = 13 also last three digits must be
Sol 4. (c) -1 divisible by 8.
Since the number 32x4115y2 is Case 1: For x =0,1,2,4,5
divisible by 88, it must be For the sum = 4 Last three digits are not divisible
divisible by 11 and 8. x,y ⇒ (0,4) , (1,3), (2,2), (3,1), by 8.
For the number to be divisible (4,0) For x = 3
Pi

by 11 The number to be divisible by 8 Last three digits are 632 and this
(3+x+1+5+2)-(2+4+1+y) = 0 or also last three digits must be is divisible by 8
11 divisible by 8. Clearly, y = 7 …...(4+x-y)
So, 4+x-y = 0 or 11 For x =4,3,2,0
4+x-y =0 is possible for Last three digits are not divisible Case 2 :
x=0,1,2,3,4,5 and y=4,5,6,7,8,9 by 8. For 4+x-y = 11
4+x-y = 11 is possible for x = For x = 1 (x,y) is (9,2), (8,1), (7,0)
7,8,9 and y = 0,1,2 Last three digits are divisible by For x =9,7 the last three digits of
Also a number is divisible by 8 if 8 the number are not divisible by
its last three digits are divisible Clearly, y = 3 8.
by 8. 3x-y = 3(1)-3 = 0 (which is not For x=8 last three digits are
We will check for minimum value in the given options) divisible by 8 and y = 1
of x For x=8 and y=1
(7x-2y) = 7(8)-2(1) = 54

www.ssccglpinnacle.com support@ssccglpinnacle.com Ph. 09729327755, 09817390373 21


/
Day 1-4 days Number System

s
sse
la
_c
ob
For x=3 and y=7 be divisible by 9 and 8. For a 5+5+3+5+0+x+2 ⇒ 20+x must be
bo
ah
(7x-2y) = 7(3)-2(7) = 7 number to be divisible by 9, the
sm
divisible by 9
ur
yo

sum of its digits must be divisible Cleary x = 7


e/
t.m
://

Sol 8. (b) by 9. ……………….(27 is multiple of


tp
ht

Number 15x1y2 is divisible by 9+1+8+7+6+x+2 ⇒ 33+x must be 9 and next multiple is 36 and for
44, clearly it will also be divisible divisible by 9 that x =16 which is not possible)
by 11 and 4 . Cleary x = 3 072 are the last three digits of the
A number to be divisible by 11 ……………….(36 is multiple of number which is divisible by 8 so
(1+x+y)-(5+1+2) = 0 or 11 9 and next multiple is 45 and for x =7 is the correct answer.
For the difference = 0 that x =12 which is not possible)
x+y = 7 632 are the last three digits of the Sol 16. (b) 4
For the difference = 11 number which is divisible by 8 so 8439x53 is divisible by 99,
x+y = 18 x =3 is the correct answer. clearly it will also be divisible by
But 18 is not given in the options 11 and 9.
so option b is the right answer. Sol 13. (c) For a number to be divisible by

e
Number 6913x08 is divisible by 9, sum of its digits must be
Sol 9. (b) 6 88, clearly it will also be divisible divisible by 9.
6x2904 is divisible by 88, It will by 11 and 8. 8+4+3+9+x+5+3 ⇒ 32+x must be
also be divisible by 11 and 8. For the number to be divisible by divisible by 9

l
For a number to be divisible by 11 Cleary x = 4
11 (6+1+x+8)-(9+3+0) = 0 or 11 ……………….(36 is multiple of

x = -5 or 6

Sol 10. (c) 8


ac
(x+9+4)-(6+2+0) = 0 or 11

4x573y is divisible by 72, clearly


x= -3 or 8 (but x cannot be
negative)
So, x=8
808 are the last three digits of the
number which is divisible by 8 so
9 and next multiple is 45 and for
that x =13 which is not possible)

Also, (8+3+4+3)-(4+9+5) = 0, so
the number is divisible by 11 also
it will also be divisible by 9 and x =8 is the correct answer. clearly x=4 will be the right
8. answer.
For a number to be divisible by 9, Sol 14. (c)
nn
sum of its digits must be divisible 5656x52 is divisible by 72, Sol 17. (c) 8175x45y2 is divisible
by 9 clearly it will also be divisible by by 72, clearly it will also be
(4+x+5+7+3+y) ⇒ 19+x+y must 9 and 8. For a number to be divisible by 9 and 8. For a
be divisible by 9. divisible by 9, sum of its digits number to be divisible by 9, sum
Possible values of x+y = 8 or 17 must be divisible by 9. of its digits must be divisible by
…..(x and y can’t be greater than 5+6+5+6+x+5+2 ⇒ 29+x must be 9.
9) divisible by 9 8+1+7+5+x+4+5+y+2 ⇒ 32+x+y
17 is not given in the options Cleary x = 7 must be divisible by 9
Pi

clearly correct option is c . ……………….(36 is multiple of So x+y = 4 or 13


9 and next multiple is 45 and for ..(as x and y can’t
Sol 11. (b) 3 that x =16 which is not possible) be greater than 9)
number 46393x8 divisible by 11, 752 are the last three digits of the For x+y = 4
For the number to be divisible by number which is divisible by 8 so (x,y) = (0,4), (1,3), (2,2), (3,1),
11 x =7 is the correct answer. (4,0)
(4+3+3+8)-(6+9+x) = 0 or 11 For x+y = 13
For the difference being 0 Sol 15. (c) 55350x2 is divisible (x,y) = (4,9), (5,8), (6,7), (7,6),
x=3 by 72, clearly it will also be (8,5), (9,4)
For the difference being 11 divisible by 9 and 8. For a
x = 14 which is not possible. number to be divisible by 9, sum We will check for the largest
of its digits must be divisible by values of y which is 9.
Sol 12. (d) Number 91876x2 is 9.
divisible by 72, clearly it will also

www.ssccglpinnacle.com support@ssccglpinnacle.com Ph. 09729327755, 09817390373 22


/
Day 1-4 days Number System

s
sse
la
_c
ob
Last three digits of the number We will check for the maximum 6220x558y2 is divisible by 88,
bo
ah
are 592 which is divisible by 8. value of x which is 9. clearly it will also be divisible by
sm
ur
yo

Clearly x=4 and y=9 are the Last three digits of the number is 11 and 8. For a number to be
e/
t.m
://

desired numbers. 312 which is divisible by 8 divisible by 11


tp
ht

√4x + y = √4(4) + 9 = 5 clearly x=9 and y=2 are the (6+2+x+5+y)-(2+0+5+8+2) = 0


desired values. or 11
Sol 18. (b) 5y5888406x6 is 2x+5y = 2(9)+5(2) = 28 x+y-4 = 0 or 11
divisible by 72, clearly it will also x+y = 4 or 15
be divisible by 9 and 8. For a Sol 20. (c) 12 Multiplying both sides by 5
number to be divisible by 9, sum 75y97405x2 is divisible by 72, 5x+5y = 20 or 75
of its digits must be divisible by clearly it will also be divisible by 75 is not given in the options so
9. 9 and 8. For a number to be 20 is the correct answer.
5+y+5+8+8+8+4+0+6+x+6 ⇒ divisible by 9, sum of its digits
50+x+y must be divisible by 9 must be divisible by 9. Sol 23. (c)
So x+y = 4 or 13 7+5+y+9+7+4+0+5+x+2 ⇒ 7220x558y2 is divisible by 88,

e
..(as x and y can’t 39+x+y must be divisible by 9 clearly it will also be divisible by
be greater than 9) So x+y = 6 or 15 11 and 8. For a number to be
For x+y = 4 ..(as x and y can’t divisible by 11
(x,y) = (0,4), (1,3), (2,2), (3,1), be greater than 9) (7+2+x+5+y)-(2+0+5+8+2) = 0

l
(4,0) For x+y = 6 or 11
For x+y = 13 Possible values of x and y = (0,6), x+y-3 = 0 or 11

(8,5), (9,4)
ac
(x,y) = (4,9), (5,8), (6,7), (7,6),

We will check for the smallest


value of x which is 0
Last three digits of the number
(1,5), (2,4), (3,3), (4,2), (5,1),
(6,0)
For x+y = 15
Possible values of x and y = (6,9),
(7,8), (8,7), (9,6)
x+y = 3 or 14
Multiplying both sides by 5
5x+5y = 15 or 70
70 is not given in the options so
15 is the correct answer.
are 606 which is not divisible by We will check for the maximum
8. value of x which is 9. Sol 24. (b)
Last three digits of the number is 1230x558y2 is divisible by 88,
nn
Now we will check for the second
smallest value of x which is 1 592 which is divisible by 8 clearly it will also be divisible by
Last three digits of the number clearly x=9 and y=6 are the 11 and 8. For a number to be
are 616 which is divisible by 8. desired values. divisible by 11
Clearly x=1 and y=3 are the 2x-y = 2(9)-6 = 12 (1+3+x+5+y)-(2+0+5+8+2) = 0
desired values. or 11
9x-2y ⇒ 9(1)-2(3) = 3 Sol 21. (a) x+y-8 = 0 or 11
1220x558y2 is divisible by 88, x+y = 8 or 19
clearly it will also be divisible by (but x+y can’t be greater than 18)
Pi

Sol 19.(a) 28
46789x531y is divisible by 11 and 8. For a number to be Multiplying both sides by 5
72,clearly it will also be divisible divisible by 11 5x+5y = 40
by 9 and 8. For a number to be (1+2+x+5+y)-(2+0+5+8+2) = 0
divisible by 9, sum of its digits or 11 Sol 25. (a)
must be divisible by 9. x+y-9 = 0 or 11 30x558y2 is divisible by 88,
4+6+7+8+9+x+5+3+1+y ⇒ So, clearly it will also be divisible by
43+x+y must be divisible by 9 x+y = 9 or 20 11 and 8. For a number to be
So x+y = 11 (but x and y can’t be more than 9 divisible by 11
..(as x and y can’t be so their sum can’t be more than (3+x+5+y)-(0+5+8+2) = 0 or 11
greater than 9) 18) x+y-7 = 0 or 11
Possible values of x,y = (2,9), Clearly 9 is the correct answer. x+y = 7 or 18
(3,8), (4,7), (5,6), (6,5),(7,4),(8,3) Multiplying both sides by 6
and (9,2) Sol 22. (a) 6x+6y = 42 or 108

www.ssccglpinnacle.com support@ssccglpinnacle.com Ph. 09729327755, 09817390373 23


/
Day 1-4 days Number System

s
sse
la
_c
ob
108 is not given in the options Clearly x+y = 9 is the desired
bo
ah
clearly 42 is the right answer. Sol 28. (d) 64x29y6 is divisible answer.
sm
ur
yo

by 72, clearly it will also be


e/
t.m
://

Sol 26.(d) 54x29y6 is divisible by divisible by 9 and 8. For a Sol 31.(d) 5x32465y is divisible
tp
ht

72, clearly it will also be number to be divisible by 9, sum by 88, clearly it will also be
divisible by 9 and 8. For a of its digits must be divisible by divisible by 11 and 8.
number to be divisible by 9, sum 9. For a number to be divisible by
of its digits must be divisible by 6+4+x+2+9+y+6 ⇒ 27+x+y must 11
9. be divisible by 9 (x+2+6+y)-(5+3+4+5) = 0 or 11
5+4+x+2+9+y+6 ⇒ 26+x+y must Possible values of x+y = 0 or 9 or x+y -9 = 0 or 11
be divisible by 9 18 (27,36 and 45 are x+y = 9 or 20 (but x+y
Possible values of x+y = 1 or 10 multiples of 9) can’t be 20 as x and y can’t
…..(27 and 36 are multiples of 9) x+y =/ 0, ..(x ≻ y ) exceed 9)
For x+y =1, possible values of x For x+y =9, possible values of x Clearly x+y = 9
and y are 1 and 0 respectively. and y are Possible values of (x,y) =

e
..(x ≻ y ) (9,0),(8,1),(7,2),(6,3),(5,4) (9,0),(8,1),(7,2),(6,3),(5,4),(4,5),(
For x+y =7, possible values of x …(x ≻ y ) 3,6),(2,7),(1,8),(0,9)
and y are (9,1), (8,2), (7,3), (6,4) x+y =/ 18, ….(x+y =18 possible Last three digits of the number
…(x ≻ y ) only if x=y=9 but as per are divisible by 8 only if y=6,

l
Only for y=3 last three digits of instructions x must be greater clearly x=3 and y=6 are the
the number are divisible by 8. than y) desired values.
ac
Clearly x=7 and y=3 are the
desired values.
2x+3y = 2(7)+3(3) = 23

Sol 27.(b) 64x29y6 is divisible by


Only for y=3 last three digits of
the number are divisible by 8.
Clearly x=6 and y=3 are the
desired values.
(2x - y) = 2(6)-3 = 9
2x+3y = 2(3)+3(6) = 24

Sol 32. (a) only by 3

Sol 33. (c) 74x29y6 to be


72, clearly it will also be Sol 29. (c) 78x1y68 is divisible divisible by 72. It should also be
divisible by 9 and 8. For a by 88, clearly it will also be divisible by 8 and 9.
number to be divisible by 9, sum So, the value of 74x29y6 to be
nn
divisible by 11 and 8.
of its digits must be divisible by For a number to be divisible by divisible by 8. It should have y =
9. 11 3
6+4+x+2+9+y+6 ⇒ 27+x+y must (7+x+y+8)-(8+1+6) = 0 or 11 For 74x29y6 to be divisible by 9,
be divisible by 9 x+y = 0 or 11 sum of digits should be 9. So, x =
Possible values of x+y = 0 or 9 or For x+y = 0, x=y=0 5
18 (27,36 and 45 are But for y=0 last three digits of the Therefore, 2x+3y = 2(5)+3(3) =
multiples of 9) number are not divisible by 8. 10+9 = 19
x+y =/ 0, ..(x ≻ y ) Clearly x+y = 11 is the desired
Pi

For x+y =9, possible values of x answer. Sol 34. (a) Given number:
and y are 3x6349y
(9,0),(8,1),(7,2),(6,3),(5,4) Sol 30.(b) 79x00001y6 is On factorizing 88 we get 8 and 11
…(x ≻ y ) divisible by 88, clearly it will also as the factors.
x+y =/ 18, ….(x+y =18 possible be divisible by 11 and 8. For 3x6349y to be divisible by 8.
only if x=y=9 but as per For a number to be divisible by Value of y = 6
instructions x must be greater 11 For 3x6349y to be divisible by
than y) (7+x+0+0+y)-(9+0+0+1+6) = 0 11, Value of x = 7
Only for y=3 last three digits of or 11 Therefore, 2x+3y = 2(7) + 3 (6) =
the number are divisible by 8. x+y -9 = 0 or 11 14 + 18 = 32
Clearly x=6 and y=3 are the x+y = 9 or 20 (but x+y
desired values. can’t be 20 as x and y can’t Sol 35. (a) On factorizing 88 we
(2x - 3y) = 2(6)-3(3) = 3 exceed 9) get 8x11.

www.ssccglpinnacle.com support@ssccglpinnacle.com Ph. 09729327755, 09817390373 24


/
Day 1-4 days Number System

s
sse
la
_c
ob
The number 4x4y96, to be According to the question 7531 7105
The remainder = = =
bo
213 213
ah
divisible by 8, y96 should also be 4(d-5) = d+1
sm
6892
=y= 76
ur

213
yo

divisible by 8. 4d-20=d+1
e/

⇒ (x − y ) = 213 − 76 = 137
t.m
://

Therefore, possible value of y = ⇒ d= 7


tp
ht

2, 4, 6, 8 Desired fraction = 7−75 = 72


Sol 47.(a)
For 4x4296 to be divisible by 11,
If a number 'a' and a number 'b'
x=9 Sol 43. (a) are divisible by a number 'n' then,
Therefore, x+2y = 9+4 = 13 Let the desired number = k a+b and a-b is also divisible by n.
According to the question Here the numbers are 2460, 2633
Sol 36. (c) 72 = 9 x 8 4−k = 1
9−k 6 and 2806 and let the remainder =
For 56x34y4 to be divisible by 8. 24-6k=9-k y
y=2 ⇒k=3 the required number then
For 56x34y4 to be divisible by 9.
becomes H.C.F of 2460-y, 2633-y
x=3
Sol 44. (a) and 2806-y or the HCF of
Therefore, x+y = 3+2 = 5
Let the number = 3k (2633-y)-(2460-y) and

e
According to the question (2806-y)-(2633-y)
Sol 37. (a) Only by 3 and 9
9k-k = 24 (2633-y)-(2460-y) = 173
A number is divisible by 3 and 9 ⇒ k=3 (2806-y)-(2633-y) = 173
if sum of digits is divisible by 3
Desired number = 3k = 3x3 = 9 So, HCF of 173 and 173 =x= 173

l
and 9.
Desired sum = 1+7+3 = 11
66249 = 6+6+2+4+9 = 27
ac Sol 45. (d)
Clearly only option A satisfies the
x will be the HCF of the Sol 48. (d)
given condition.
differences of the three numbers. Given, (6n + 3)2
6475-4984 = 1491 On expanding the given
Sol 38. (b)
4984-4132 = 852 polynomial, we get
Sum of digits of 210102 =
6475-4132 = 2343 2
36n + 36n + 9
2+1+0+1+0+2 = 6
1491 = 3 x 7 x 71 Since, 36 and 9 are divisible by 9.
6 is divisible by 3. Clearly this
852 = 2 x 2 x 3 x 71 Therefore, On dividing (6n + 3)2
number is divisible by 3.
nn
2343 = 3 x 11 x 71 by 9. We get 0 as remainder.
So, HCF of 2343, 1491 and 852 = Alternate :
Sol 39. (d) Number 30744 is
3 x 71 = 213 Put any integer value of n for
divisible by 2, 3, 6 and 7. Except
5. example 1.
Sol 46.(b) 2
(6n + 3)2 ⇒ { 6(1) + 3} = 81
If a number 'a' and a number 'b'
Sol 40. (a) Number 106974 is Since 81 is divisible by 9 the
are divisible by a number 'n' then,
divisible by 2, 3, 6 and 7. remainder will be 0.
a+b and a-b is also divisible by n.
Pi

Here the numbers are 7531, 7105


Sol 41. (a) Sol 49. (c)
and 6892
a:b:c:d Let the number of students = s, so
the required number then
3:4 amount donated by each student =
becomes H.C.F of 7531-y, 6892-y
4:5 s
and 7105-y or the HCF of
5:6 According to the question
(7531-y)-(7105-y) and
⇒a:b:c:d= 3: 4:5:6 s × s = 3481
(7105-y)-(6892-y)
⇒ da = 63 = 21 ⇒ s = √3481 = 59
(7531-y)-(7105-y) = 426
10 10
( da ) = ( 21 ) = 1
1024 (7105-y)-(6892-y) = 213
Required sum = 1024 + 1 = 1025 213 = 3 x 71 Sol 50. (c) Since,
426 = 2 x 3 x 71 √1354.24 = 36.8
Sol 42. (d) So, HCF of 426 and 213 =x= 3 x
Let the denominator = d 71 = 213 Sol 51. (d) Going through the
⇒ numerator = d-5 options:

www.ssccglpinnacle.com support@ssccglpinnacle.com Ph. 09729327755, 09817390373 25


/
Day 1-4 days Number System

s
se
s
la
_c
ob
Option (d) √1489.96 = 38.6 must be an even number. So sum Put any odd value of k. For
bo
ah of digits of this number must be example k=1
sm
Hence required rational number is
ur
yo

divisible by 9 and last three digits ( 82k + 52k ) ⇒ ( 82(1) + 52(1) ) = 89


e/

1489.96
t.m
://

by 8. 3+8+9+x+6+3+7+8+y = Clearly 89 will be the factor.


tp
ht

Sol 52. (d) LCM of (15, 18 and 44+x+y


42) = 630 x+y must be 1 or 10 as after 45 Sol 5.(d)
Let the number be (630k+8). nearest multiple of 9 is 45 and 54. Given, x =
Minimum value of k for which For x+y =1 x must be 1 as y can’t 24 38
(633) − (277) + (266) 54

be odd number. And 780 is not


(630k+8) is divisible by 13, is Unit digit of (633)24 = 34 = 1
equal to 3. divisible by 8 so it will get
Unit digit of (277)38 = 72 = 9
Hence the number (630k+8) = neglected.
Pairs for 10 = (1,9)(2,8), (3,7), Unit digit of (266)54 = 62 = 6
{630(3)+8} = 1898.
(4,6), (5,5), (6,4)(7,3)(8,2)(9,1) Unit digit of x = 1-9+6 = -2
Sum of digits = 1+8+9+8 = 26
Only pair which satisfies these But unit digit can’t be negative
conditions is (6,4). So the so, required unit digit = 10 + (-2)
Sol 53. (a)
=8

e
Going through the options: required value is √6x + 7y =
Option (a) √5535.36 = 74.4 √6(6) + 7(4) = 8 Sol 6.(d)
Hence required answer rational Let the number = 10x+y
number is 5535.36 . Sol 2. (b)
According to the question

l
12 = 2x2x3
(x+y) = 71 (10x+y) ………..(1)
Sol 54. (b) let the required power
ac 16 = 2x2x2x2
And
=n 18 = 2x3x3
y = x-4
According to the question, 20=2x2x5
Put this value in equation (1)
(− 3)n = − 2187 25 = 5x5
(x+x-4) = 71 (10x+x-4)
= (− 3)7 LCM of 12,16,18,20 and 25 =
2x2x2x2x3x3x5x5 = 3600 ⇒ 14x-28 = 11x-4
Therefore, Required power will ⇒ x = 8 and y = 8-4 = 4
⇒ x must be = 3600k + 4
be 7.
Where 3600k+4 is multiple of 7 the number obtained on reversing
the digit = 10y+x = 10(4)+8 = 48
nn
Sol 55. (d) The condition gets satisfied when
k=5 Required remainder = 487 =6
√3375 = √3 × 3 × 3 × 5 × 5 × 5
3 3

Required number = 3600(5)+4 =


= 15
18004 Sol 7. (a) Since, 5678x43267y is
divisible by 72 it must be
Sol 56. (b) closest square to 8212
⇒ digit at the thousands’ place in x = 8 divisible by 9 and 8 (coprime
is 8281.
factors of 72) and y must be an
Therefore, 69 is to be added to
Sol 3.(d) even number. So sum of digits of
8212 to make it a perfect square.
Pi

⇒ let the number be n which this number must be divisible by


divides 7897, 8110 and 8536 9 and last three digits by 8.
Sol 57. (c)
leaving a reminder r 5+6+7+8+x+4+3+2+6+7+y =
Let the number = x
⇒ the required number then 48+x+y
x2 = 243
x3
1 16807
becomes H.C.F of (7897-r), x+y must be 6 or 15 as after 48
⇒ x5 = 35 (8110-r) and (8536-r) nearest multiples of 9 are 54 and
75
3 ⇒ it could also be the H.C.F of 63.
⇒x= 7
(8536 - r) - (8110 - r) and Pairs for 6 = (1,5)(2,4), (3,3),
(8110-r)-(7897-r) (4,2), (5,1), (6,0)
SSC CGL TIER II
i.e. 426 and 213 y can’t be an odd number. Only
⇒ H.C.F of 426 and 213 = 213 pair that satisfies all the condition
Sol 1. (d)
the required sum = 2+1+3 = 6 is (4,2)
Since, 389x6378y is divisible by
Pairs for 15 = (6,9), (7,8), (8,7),
72 it must be divisible by 9 and 8
Sol 4.(d) (9,6)
(coprime factors of 72) and y

www.ssccglpinnacle.com support@ssccglpinnacle.com Ph. 09729327755, 09817390373 26


/
Day 1-4 days Number System

s
sse
la
_c
ob
None of the pairs of 15 satisfies ⇒ x = 5 divisible by 9 and last three digits
bo
y 3
ah
the given conditions. by 8. 5+4+3+2+y+1+7+4+9+x =
sm
Put this value in any of the
ur
yo

So the required value is 35+x+y


e/

equations
t.m
://

√5x + 8y = √5(4) + 8(2) = 6 (10x+y)(x+y)=[{10(5)+3}(5+3)] x+y must be 1 or 10 as after 35


tp
ht

= 424 nearest multiples of 9 are 36 and


Sol 8. (a) Or 45.
3600 = 24 × 32 × 52 (10y+x)(x+y)=[{10(3)+5}(5+3)] Pairs for 1 = (1,0)
⇒ N umber of f actors of 3600 = = 280 x can’t be an odd number.
(4+1) (2+1) (2+1) = 45 Clearly 5 and 3 are the desired Condition not satisfied.
values and the sum of the digits = Pairs for 10 = (1,9), (2,8), (3,7),
Sol 9. (c) 5+3 = 8 (4,6), (5,5), (6,4), (7,3), (8,2),
Given, (9,1)
Sol 12. (c) Only (6,4) satisfies the given
x = (164)169 + (333)337 - (727)726
15321 conditions.
Unit digit of (164) 169 1
= 4 =4 361284 = (34 )
⇒ (5x − 4y) = 5(6) − 4(4) = 14
Unit digit of (333)337 = 31 = 3 Now, 34 = 81. 81 divided by 5

e
Unit digit of (727)726 = 72 = 9 gives remainder 1.
Sol 15. (d)
Unit digit of x = 4+3-9 = -2 ⇒ (1)15321 = 1
( 12797 + 9797 ) ⇒ (
But unit digit can’t be negative So, remainder when 361284 is 97 97
128 − 1) + (96 + 1)
divided by 5= x = 1

l
so, required unit digit = 10 + (-2)
Remainder from ( 128 − 1)97 =
=8 When 4 raised to any power is
ac divided by 6 it will give the (− 1)97 =− 1
Sol 10. (a) remainder 4. Remainder from ( 96 + 1)97 =
97
15 = 3x5 ⇒y=4 (1) = 1
18 = 2x3x3 ⇒ (2x − y ) = 2(1) − 4 = − 2 Final remainder = -1+1 = 0
20 = 2x2x5
27 = 3x3x3 Sol 13. (c) Sol 16. (d)
LCM of 15,18,20 and 27 = Since the last divisor is 17 and the Let the numbers are x and y.
2x2x3x3x3x5 = 540 quotient is 2, we have the According to the question
nn
⇒ x must be = 540k + 10 dividend = 17x2 = 34.(Because x-y = 2001 ……(1)
Where 540k+10 is multiple of 31 the last divisor will not leave any And
The condition gets satisfied when remainder) 9y+41 = x ………(2)
k=4 Now, 34 will become the divisor Put the value of x in eq (1)
Required number = 540 (4)+10 = and 11 will become the quotient 9y+41-y=2001
2170 and 17 will be the remainder. 8y = 1960
Nearest square to 2170 = 2209 Therefore the dividend will be 34 y=245
Required number = 2209-2170 = x 11 + 17 = 391 x = 9(245)+41 = 2246
Pi

39 Now, 391 will become the divisor Required sum = 2+2+4+6 = 14


and 1 will be the quotient and 34
Sol 11. (c) will be the remainder. Therefore SSC CGL TIER I
Let the number = 10x+y the dividend will be 391 x 1 + 34
According to the question = 425 Sol 1. (d) For any number to be
(10x+y)(x+y)=424 …(1) So, the numbers are 391 and 425 divisible by 8, its last 3 digits
And Required sum = 391+425 = 816 must be divisible by 8
(10y+x)(x+y)=280 ……..(2) By putting x=0 in 1005x4 , we
Divide equation (1) by (2) Sol 14. (a) see that 504 will be divisible by
(10x+y)= 424
⇒ (10y+x)= Since, 5432y1749x is divisible by 8.
280
(10x+y)= 53 72 it must be divisible by 9 and 8
⇒ (10y+x)= 35 (coprime factors of 72) and x Sol 2. (c) When 200 is divided by
⇒ 350x+35y = 530y + 53x must be an even number. So sum x remainder is 8. So, the number
⇒ 297x = 495y of digits of this number must be exactly divisible by x is 192.

www.ssccglpinnacle.com support@ssccglpinnacle.com Ph. 09729327755, 09817390373 27


/
Day 1-4 days Number System

s
sse
la
_c
ob
Multiples of 192 = 2, 3, 4, 6, 8, Sol 9. (c) For 146*48 to be
bo
ah
12, 16, 24, 32, 48, 64, 96, 192 Sol 6. (d) From 200 to 800, divisible by 8, *48 must be
sm
ur

The remainder is always less than divisible by 8.


yo

numbers which are divisible by


e/
t.m

the divisor , so :x > 8 Check options: * = 2,8


://

(i) 5 : 800 200


5 - 5 = 120
tp
ht

⇒ Desired values are 12, 16, 24, 800 - 200 =


satisfies the condition.
32, 48, 64, 96, 192. (ii) 7 : 7 7 114 - 28 = 86 But 8>2. option c is the
(iii) 35 : 35 - 200
800
35 = 22 - 5 = 17 correct answer.
Sol 3. (a) For a number to be Total number = 120+86 - 17 =
divisible by 11, the difference of 189 Sol 10. (a) For 687x29 to be
sum of alternative numbers is Numbers from 200 to 800 which divisible by 9, the sum of digits
divisible by 11. are neither divisible by 5 nor by of 687x29 must be divisible by 9.
Hence, (7+3+*+9)-(5+2+4) 7= (800-200)- 189 = 411 Thus, x = 4 and 2x = 4
=(19+*)-(11)
=8+* Sol 7. (b) It is given that Sol 11. (a) For a number to be
divisible by 4, the last two digits
* must be 3 for 94*2357 to be 708x6y8z9 is divisible by 99.
must be divisible by 4. Among
divisible by 11. Thus, 708x6y8z9 is divisible by the given options 8 is maximum

e
both 11 and 9 number to replace * .
Sol 4. (b) When 732 is divided by For divisibility by 9, sum of digits
x remainder is 12. So, the number are divisible by 9 Sol 12. (a) For a number to be
exactly divisible by x is 720. (7+0+8+x+6+y+8+z+9 = divisible by 11, the difference of

l
The remainder is always less than 38+x+y+z. We get 2 as remainder the sum of digits at odd and even
the divisor , so :x > 12
ac when 38 ÷ 9. Thus, 2+x+y+z must places must be divisible by 11.
Now, the factors of 720 which are be divisible by 9) (9+5+8)-(2+x+5) = 15-x
more than 12 are possible values Possible values of (z+y+x) = For x = 4, the number is divisible
of x, i.e. (15,16,18,20,24,30,36, 7,16,25, etc. by 11.
40,45, 48,60, 72, 80, 90, 120, For divisibility by 11, the
144, 180, 240, 360, 720). difference of sum of digits at odd Sol 13. (b) n = 7Q+2
and even place is divisible by 11 For remainder 0, add 5 both sides,
Sol 5. (a) In such questions (i.e. in 708x6y8z9 : (9 + 8 + 6 + 8 we get: n+5 = 7Q+7
nn
directly check divisibility by 11, 9 + 7 ) - (z + y + x + 0) = 38 - ( z +
and 8. y + x) is divisible by 11 ) 570 +770 2535 +4935
Sol 14. (c) 74 = 74
For a number to be divisible by Possible values of (z+y+x) 35
⇒ 25 + 49 = 35
11, the difference of the sum of =38,5,16 etc.
digits at odd or even places must (25+49)(x) = (74)(x)
be divisible by 11. Remainder = 0
In such questions, we must
For a number to be divisible by 9, directly verify options.
the sum of numbers must be
divisible by 9. Sol 15. (b) n = 5Q+2
Pi

Sol 8. (c) Let N be the number


For divisibility by 8, the last 3 7n = 35Q+14
which gives Q as quotient and 15
numbers must be divisible by 8. 7n = 5×
as remainder when divided by d.
Accordingly, For x35624 7Q+10+4
Thus, d > 15
divisible by 11 Remainder = 4
N = d × Q+ 15
(x+5+2)-(3+6+4) = 0 or 11 10N = 10(d × Q)+ 144 + 6
⇒ x=6 Sol 16. (a) For a number to be
clearly d is a multiple of 144
And For 1257y4 divisible by 72, divisible by 9, the sum of its
which are: 2, 3, 4, 6, 8, 9, 12, 16,
1+2+5+7+y+4 must be divisible digits must be divisible by 9.
18, 24 and so on.
by 9 and the only possible value In 1190*6, 1+1+9+0+*+6 = 17+*
The least possible value of d is
of y is 9, here. By replacing * by 1, 1190*6 is
16.(d > 15)
also 784 is divisible by 8 so divisible by 9.
desired value of y = 8
Then, 5x-2y = 30-16 = 14

www.ssccglpinnacle.com support@ssccglpinnacle.com Ph. 09729327755, 09817390373 28


/
Day 1-4 days Number System

s
sse
la
_c
ob
2 1 1 1 1 1
SSC CHSL 2019 (a 3 + 2a 2 + 3a 3 + 2a 6 +1)(a 3 -2a 6 +
bo
ah
Sol:1. (c)
1 1
1) -a 2 (a 2 -2) Sol:10. (c)
sm
ur
yo

1
For divisibility of 11, The Let a 6 = p then, The largest five-digit number =
e/
t.m
://

difference of alternate digits sum (p4 + 2p3 + 3p2 + 2p + 1)(p2 - 2p + 99999


tp
ht

should be equal to 0 or 11 1) - p3(p3 - 2) When 99999 is divisible by 81


(8+9+8)-(0+7+1) = 12 p6 - 2p5 + p4 + 2p5 - 4p4 + 2p3 + then remainder = 46
(1+6+7)-(1+5+1) = 7 3p4 - 6p3 + 3p2 + 2p3 - 4p2 + 2p Required number = 99999-46 =
(9+3+9)-(6+3+1) = 11 +p2 -2p + 1 - p6 + 2p3 99954
(1+7+1)-(0+6+1) = 2 =1
so, option C is the answer. Sol:11. (c)
ALTERNATE SOLUTION 2x2 - 5x - 12
Sol:2. (c) As the equation is independent of 2x2 - 8x+3x - 12
( 7777 + 77) /78 a, so you can put a = 0 for the 2x(x-4)+3(x-4)
= (− 1)77 + (-1) solution (2x+3)(x-4)
= -2 Which gives 1 as answer

e
So, remainder will 78-2 = 76 Sol:12. (d)
Sol:6.(c) We know,
Sol:3.(b) 36 will be divisible by 4 and 9. For divisibility of 9, apply digit
4 divisibility of 4 last two digits
When an integer n is divided by sum
should be divisible by 4.

l
5, the remainder is 3 and let 7+1+2+9+p+4+6+5 = 9 or its
For maximum value b=8 and a=6
quotient is x, then
ac a+b=8+6=14 multiple
n = 5x +3 34+p = 9 or its multiple
Now, 8n = 40x + 24, when it is p=2
divided by 5 , we get remainder 4. Sol:7.(b)
(By remainder theorem) 3x2 - 5x - 8 Sol:13. (c)
Alternate solution 3x 2 - 8x+3x - 8=0 The least number which is
Consider the least value of integer completely divisible by 91 is
x(3x-8)+1(3x-8)=0
n, which is divided by 5, the 10010
(x+1)(3x-8)
nn
remainder is 3, that is 8.
When 8n (8 × 8) is divided by 5, Sol:14. (d)
Sol:8. (d)
we get remainder 4. For divisibility of 3, the sum of
For divisibility of 9, we check
digits of the number should be
digit sum
Sol:4.(d) divisible by 3.
sum of digits of 19,596 =
When the number is divisible by so, only the sum of option D is
1+9+5+9+6 = 30 it means sum of
2 and 3 both, then numbers are divisible by 3
digits should be 27 or 36
divisible by 6. it means numbers are =
Sol:15.(a)
Pi

By 2:- When the last digit is 0 or (19,596-3) or (19,596+6)


an even number. 7a2 + 7a = 7a(a +1) is clearly
so, numbers are = 19,593 and
By 3:- Sum of digits is divisible divisible by 7
19,602
by a(a +1), are two consecutive digit
Now, and out of them one is divisible
Sol:9. (d)
(a)23,408 = not divisible by 3. by 2
for divisibility of 8, check last 3
(b)43,923 = not divisible by 2. So, the number is divisible by 7
digits
(c)1,00,246 = not divisible by 3. b12 = so least value of b = 1 and 14 both
8
(d)3,49,722 = divisible by both 2 ALTERNATE SOLUTION
for divisibility of 9, check digits
and 3, so it is divisible by 6. Put the value of a = 1
sum
7a2 + 7a = 7(1)2 + 7(1) = 14
1+a+7+6+5+1+1+2 = 9 or its
Sol:5.(c) So, number is divisible by 7 and
multiple
14 both
a=4
(2a + 3b) = 2(4)+3(1) = 11

www.ssccglpinnacle.com support@ssccglpinnacle.com Ph. 09729327755, 09817390373 29


/
Day 1-4 days Number System

s
sse
la
_c
ob
Sol:16. (b) 25B =B=6 Sol:29. (c)
bo
8
ah
(ax + by), on squaring it, Divisibility by 5 condition:- Unit
sm
For divisibility of 9, check sum of
ur
yo

2 2 2 2 2 digit of a number must be either 0


(ax + by) = a x + 2abxy + b y digits
e/
t.m
://

So, (ax + by) is a factor of a2x2 + 4+3+A+5+3+2+5+6=9 or its or 5


tp
ht

2abxy + b2y2 multiple Divisibility by 8 condition:- Last


28+A=9 or its multiple 3 digits of a number must be
Sol:17. (a) A=8 divisible by 8.
Go through option (a) A+B=8+6=14 For a number to be divisible by
4x2 + 2x - 3y + 9y2 - 12xy both 8 and 5, above condition
2 Sol:24. (b) must satisfy.
(2x − 3y) +(2x-3y)
225 + 226 + 227 Therefore, in 62783xy, y = 0 or 5
So, (2x-3y) is a factor of 4x2 + 2x
225 (1 + 21 + 22 ) Also , 3xy must be divisible by 8.
- 3y + 9y2 - 12xy
Case I:- x = 1 and y = 0, 310 is
225 × 7
not divisible by 8.
Sol:18. (d) So, number is divisible by 7
Case II:- x = 2 and y = 0, It is
220+1 = 0

e
divisible by both 8 and 5.
220 = -1 Sol:25. (c)
After cubing both sides For divisibility of 5, last digit
Sol:30. (c)
260 = -1 should be 5 or 0
Given:- x3 + 2x2 − ax − b is
260+1 = 0 and for divisibility of 9, check

l
divisible by x2 − 1 .
So, answer is 260+1 digit sum
Therefore, x2 = 1 satisfies the
ac 4+4+3+2+A+4+3+B =20 + A + B
equation and x3 + 2x2 − ax − b =
Sol:19. (c) 20+A + B = 27 (nearest multiple
LCM of 3,5 and 7 = 105 greater than 20) 0 at x2 = 1
so, next larger integer divisible by A+B=7 Or x = +1 or -1
all these numbers = n+105 At x = 1, x3 + 2x2 − ax − b = 0
Sol:26. (a) ⇒ 1 + 2 − a − b = 0 ⇒ 3 -a - b =
Sol:20. (b) Let the number is N and x is the 0 ⇒ a + b = 3 … (i)
Numbers which are divisible by quotient when N is divided by 14. At x = -1, x3 + 2x2 − ax − b = 0
nn
13 between 800 and 1300 = 806; N = 14x + 9 ⇒ −1+2+a−b =0 ⇒ 1+a-
819…….1989 When square of (14x + 9) is b = 0 ⇒ a - b = -1 … (ii)
Required number = 198913−806 + 1 divided by 14 From (i) and (ii):- a = 1 and b = 2
2
= 92 (14x+9) 81
14 = 14 (by remainder
Sol:31. (a)
theorem)
Sol:21. (c) Divisor = 24 × Quotient = 8 ×
We get remainder 11
for divisibility of 8, we check last Remainder
3 digits Given:- Quotient = 18, Divisor =
Sol:27. (d)
Pi

58N , so value of N = 4 24 × 18 = 432


8 218 − 1 = 86 − 1
Remainder = 3 × 18 = 54
If n is even number then ,
Sol:22. (c) Dividend = Divisor × Quotient +
(an − bn ) is divisible by (a+b)
for divisibility of 8, we check last Remainder = 432 × 18 + 54 =
and (a-b)
3 digit 7830
So, 86 − 1 is divisible by 7.
67N = N = 2
8
There is only one option where Sol:32. (a)
Sol:28. (d)
value of N = 2 781 + 782 + 783 ⇒
a number is divided by 899, the
so, option (c) is correct. remainder is 63 781 × (1 + 7 + 72 ) ⇒
the same number is divided by 781 × (1 + 7 + 49) ⇒ 781 × (57)
Sol:23.(c) 63 = 5 (by
29, the remainder = 29 ⇒ 780 × (7 × 57) = 780 × (399)
For divisibility of 8, check 3 remainder theorem) Option (a) follows.
digits.

www.ssccglpinnacle.com support@ssccglpinnacle.com Ph. 09729327755, 09817390373 30


/
Day 1-4 days Number System

s
sse
la
_c
ob
Sol:33. (b) 1 + 3 + 5 + 7 + ……………. (2n - So, answer (b) is correct.
bo
ah
when a number is divided by 3 1) = n2 = n × n
sm
ur
yo

and gives remainder 2 it means SSC CGL 2019


e/
t.m
://

minimum number will be 5 Sol:38. (c) Sol:41.(b)


tp
ht

when 5 more is added to it then For a number to be divisible by 3; Let larger number = a
the new number = 10 sum of its digits must be divisible Smaller number = b
when 10 is divided by 3 then by 3. a - b =1280
remainder will be 1 Option (a):-sum of digits of (1)
2362735 = 28 a = 7b + 50
Sol:34. (d) Option (b):-sum of digits of a - 7b = 50
For divisibility of 9, check digit 6342589 = 37 (2)
sum Option (c):-sum of digits of Multiplying eq 1 by 7
(i) 897342 = 8+9+7+3+4+2 = 33 3745932 = 33 7a - 7b = 8960
(ii) 594327 = 5+9+4+3+2+7 = 30 Option (d):-sum of digits of (3)
(iii) 346217 = 3+4+6+2+1+7 = 23 4539763 = 37 Subtracting eq 2 from eq 3

e
(iv) 734895 = 7+3+4+8+9+5 = 36 6a = 8910
Only option (d) is divisible by 9 Sol:39. (a) a = 1485
Condition (i) for a number to be
divisible by 11:- Sol:42.(b)

l
Sol 35. (c) Difference between the sum of (4x - 2y 3z) =
for divisibility for 2, last digit digits at odd and even place must 4 × 17 − 24 × 2 + 27 × 3
ac
should be divisible by 2
for divisibility for 5, last digit
should be 0 and 5
for divisibility for 10, last digit
should be 0
be divisible by 11.
Condition (ii) for a number to be
divisible by 7:-
Subtract twice the last digit from
the remaining digits, the result
(4x - 2y 3z) = 101
When 101 is divided by 31 we get
reminder = 8

Option (a) and (d) are not must be divisible by 7. Repeat the HCF of 72 and 48 = 24
divisible by 2 and option (b) are process until you get the smallest When 1062 is divided by 24 we
not divisible by 5 digit. get reminder as = 6
nn
so, option C is answer. Check options: 24 - 6 = 18
Option (a) :-
Sol:36. (d) 16324 :- (1+3+4)-(6+2) = 0….
n = 7 × Quotient + 2 Condition (i) follows Sol:43.(a)
For remainder to become 0, we Condition (ii) :- 1632 - 8 = 1624 3 × 7 × 11 = 231
must add such a number to 2 so 162-8 = 154; 154 is divisible by Let a and b = 9 and 9
that it becomes divisible by 7. 7. When 53599 is divided by 231
n + 5 = 7 × Quotient + 2 + 5 = 7 Thus, condition (ii) follows. reminder = 7
Pi

× Quotient + 7 Means 53599 - 7 = 53592 is that


Now, n + 5 is divisible by 7. Sol:40. (b) number
For divisibility of 9, check digit a=9
Sol:37. (c) sum b=2
We know, The sum of odd For divisibility of 11, difference a 2 − b 2 + ab = 81 - 4 + 18 = 95
numbers = (no. of terms)2 of alternate digits sum should be
In 1 + 3 + 5 + 7 + ……………. equal to 0 or 11 Sol:44.(d)
(2n - 1); let the no. of terms = x (a) 10089 = (1+0+9)-(0+8) = 2 LCM of 3,7,11 is = 231
2n - 1 = 1 + (x -1)2 (not divisible by 11) Let the number be 23599
2n - 2 = (x - 1)2 (b) 10098 = (1+0+8)-(0+9) = 0 (it When 23599 is divided by 231
n-1=x-1 is divisible by 11) now, check we get remainder as 37
x=n digits sum So the number is = 23599 - 37 =
Therefore, no. of terms, x = n 1+0+0+9+8 = 18 (it is also 23562
divisible by 9)

www.ssccglpinnacle.com support@ssccglpinnacle.com Ph. 09729327755, 09817390373 31


/
Day 1-4 days Number System

s
sse
la
_c
ob
x=6 1432×1431×1001×1425 Smallest number which leaves
bo
12
ah
y =2 remainder 5 when divided by 7 is
sm
(−7)×(−7)×(−6)×(−3)
ur

12
yo

(3x-4y) = 18 - 8 = 10 5
e/

49×18 leaves remainder


t.m

12 as
://

Smallest number which give


tp
ht

=6
Sol:45.(b) quietent as 5 and remainder as 3
HINT: ab and ba both can be is 4 × 5 + 3 = 23
Sol:51.(d)
prime only and only when both Smallest number which give
Co-prime factors of 72 are 8 and
are odd number quietent as 23 and remainder as 2
9
Such numbers are is 3 × 23+ 2 = 71
Divisibility rule of 9 says that
13,31,17,71,37,73,79,97 71 when divided by 84 gives
sum of numbers should be
Sum = 13 + 31 + 17 + 71 + 37 + remainder as 71
divisible by 9
73 + 79 + 97 = 418
Divisibility of 8 says last 3 digits
Sol:56.(c)
should be divisible by 8
LCM of 10, 12, 15, 18 and 20 is
In 785X3678Y
Sol:46.(c) 900
Y should be 4 to be divisible by

e
As 10404 is a perfect square
8.
So x = 20 Sol:57.(a)
X should be 6 to make digit sum
Prime factorization of 20 = 2 × 2 LCM of 5, 6 and 7 is 150
divisible by 9
×5 Least number above 400 which is
So X-Y will be 6-4=2.

l
So, when 20 is multiplied by 5 it a multiple of 150 is 450
become a perfect square
ac After 450 is 600
Sol:52.(b)
So , there are 2 numbers
75 × 73 × 78 × 76 when
Sol:47.(d)
individual is divided by 34 we get
a=9 Sol:58.(a)
,
b=7 Divisibility by 10 totally depend
7 × 5 × 10 × 8
c= 10 upon last digit
2800 when divided by 34 gives
(a + 2b + 5c) =(9 + 14 + 50) = 73 Product of last digit = 3 × 2 × 5
remainder as 12
When 73 is divided by 13 =30
remainder is 8 So, last digit is 0
nn
Sol:53.(c)
Product of 7, 11 and 13 = 1001
Sol:48.(d) Sol:59.(d)
If a number is divisible by 7, 11
Number of terms in the series = Co-prime factors of 72 are 8 and
and 13 then it is divisible by 1001
18 9
If 5x2y6z is divisible by 1001
Sum = (number of terms/2) (first Divisibility rule of 9 says that
then its starting three digits
term + last term) sum of numbers should be
should repeat
Sum = 182 (6 + 40) = 414 divisible by 9
Which means that the number is
Divisibility of 8 says last 3 digits
Pi

562562
should be divisible by 8
So, x = 6 , y = 5 and z = 2
Sol:49.(b) In 785X3678Y
So, x - y + 3z = 6 - 5 + 6 = 7
(30)5 × (24)5 Y should be 4 to be divisible by
5 5 8.
(2 × 3 × 5) × (23 × 3) Sol:54.(b)
20 10 5
X should be 6 to make digit sum
2 ×3 ×5 72 × 73 × 78 × 76 when
divisible by 9
Number of prime factors = individual is divided by 35 we get
So X+Y will be 6+4=10.
(20+10+5) ,
Number of prime factors = 35 2×3×8×6
Sol:60.(d)
288 when divided by 35 gives LCM of 5, 6 and 8 is 120
SSC CPO 2019 remainder as 8 Least number above 300 which is
Sol:50.(d) a multiple of 120 is 360
1432×14331433×1425 Sol:55.(a)
12 After 360 is 480 and 600

www.ssccglpinnacle.com support@ssccglpinnacle.com Ph. 09729327755, 09817390373 32


/
Day 1-4 days Number System

s
sse
la
_c
ob
So , there are 3 numbers
bo
ah
sm
ur
yo

Sol:61.(d)
e/
t.m
://

Smallest number which leaves


tp
ht

remainder 5 when divided by 7 is


5
Smallest number which give
quietent as 5 and remainder as 3
is 4 × 5 + 3 = 23
Smallest number which give
quietent as 23 and remainder as 2
is 3 × 23+ 2 = 71
71 when divided by 42 gives
remainder as 29

l e
ac
nn
Pi

www.ssccglpinnacle.com support@ssccglpinnacle.com Ph. 09729327755, 09817390373 33


/
Days 5-7 : HCF- LCM

s
sse
la
_c
ob
What are HCF and LCM? / Example: For numbers 8 and 12,
bo
ah LCM = 24 and HCF = 4
मह म समापवतक और लघु म
sm
ur
yo

समापव ा है ? Now, LCM × HCF = 24 × 4 =


e/
t.m

96 also, 8 × 12 = 96
://
tp
ht

1. LCM (Least common multiple)


2) HCF, of some numbers, is
of two or more given numbers is always a factor of LCM of the
the least number which is exactly 6 = 2×3 , 12 = 2×2×3 , 18 =
numbers.
divisible by each of them. 2×3×3
कुछ सं ाओं का एचसीएफ, हमेशा
दो या अिधक िदए गए नंबरों की HCF = 2×3 = 6 सं ाओं के एलसीएम का एक
एलसीएम (लघु म समापव ) कम Try finding HCF and LCM of 3, गुणक है ।
से कम सं ा है जो उनम से ेक 6, 9, 12 yourself./ 3, 6, 9, 12 के
के ारा िब ु ल िवभािजत है । एचसीएफ और एलसीएम यं 3) LCM of Fraction/िभ =
LCM of numerator
खोजने का यास कर। HCF of denominator

2. HCF (Highest common factor)


of two or more number is the HCF can also be found by 4) HCF of Fraction/िभ =

e
HCF of numerator
greatest number which divides Division method. It is useful LCM of denominator
each of them exactly. HCF is also when the numbers are bigger.
known as ‘Highest common एचसीएफ िडवीजन िविध ारा भी Example: LCM and HCF of
Divisor’ (HCD) and Greatest पाया जा सकता है । यह उपयोगी है

l
1 2 3
2 , 3 and 4
common Measure’ (GCM). जब सं ा बड़ी हो।
ac
दो या दो से अिधक सं ा के
एचसीएफ (उ तम सामा कारक)
सबसे बड़ी सं ा है जो उनम से

है |
ेक को पूरी तरह िवभािजत करता
HCF by Division Method/
िवभाजन िविध ारा एचसीएफ
Example/ उदाहरण: HCF of 24,
48, 72, and 100.
LCM =
LCM of 1,2, 3
HCF of 2,3,4

HCF =
= 6
1
LCM of numerator
HCF of denominator

HCF of numerator
LCM of denominator
=

=
HCF of 1,2, 3 1
To start the division method LCM of 2,3,4 = 12

Finding LCM and HCF/ select the smallest two numbers./


िवभाजन िविध को शु करने के िलए
5) Co- Prime numbers / सह
nn
एलसीएम और एचसीएफ खोजना
सबसे छोटी सं ा दो चुन।
Example/ उदाहरण: LCM of 6, अभा सं ा: If the HCF of two
numbers is 1 then they are called
12, 8
co-prime numbers./ यिद दो नंबरों
का एचसीएफ 1 है तो उ सह- धान
सं ा कहा जाता है ।
HCF of 24, 48, 72, and 100 = 4.
6) LCM = P roduct ; where LCM
HCF
Example/ उदाहरण: HCF of 1785, and HCF are of two numbers
Pi

1995, 3381. N 1 and N 2 ./ जहां एलसीएम और


एचसीएफ दो नंबरों N 1 और N 2 के
LCM = 2×2×3×2 = 24
ह|
HCF of 6, 12, 18 If we find two co-prime factors,
F 1 and F 2 , of the Product as
Firstly find out the factors of 6,
obtained above then:
12, 18 and the multiply the
common factors. अगर हम Product के दो सह
HCF of 1785, 1995 and 3381 = अभा गुणक, F 1 और F 2 ,
सबसे पहले 6, 12, 18 के गुणक का 21 खोजते है , तब:
पता लगाएं और आम गुणक को गुणा
HCF × F 1 = N 1 and HCF
कर|
Important Concepts: × F2 = N2
1) LCM × HCF = First number
× Second number Varieties Questions

www.ssccglpinnacle.com support@ssccglpinnacle.com Ph. 09729327755, 09817390373


34
/
Days 5-7 : HCF- LCM

s
sse
la
_c
ob
respectively. If one of the SSC CPO 16 March 2019
bo
ah
Q1. The product of HCF and numbers is 210, then the other (Evening)
sm
ur
yo

LCM of two numbers is 3321. If number is: (a)112


e/
t.m

दो सं ाओं का मह म समापवतक
://

one of the numbers is 369. The (b)116


tp
ht

HCF of the number is और लघु म समापव मशः 6 (c)108


यिद दो सं ाओं के HCF और और 5040 है | यिद इनम से एक (d)122
LCM का गुणनफल 3321 है ।यिद सं ा 210 है , तो दू सरी सं ा ात
इनमे से एक सं ा 369 है ।तो HCF कर | Q7. The product of two numbers
ात कीिजये? SSC CPO 14 March 2019 is 6760 and their HCF is 13. How
SSC CPO 16 March 2019 (Morning) many such pair of numbers can
(Morning) (a)256 be formed?
(a) 21 (b)144 दो सं ाओं का गुणनफल 6760 है
(b) 9 (c)30 तथा उनका मह म समापवतक 13
(c) 3 (d)630 है | सं ाओं के ऐसे िकतने यु ों का
(d) 27 िनमाण िकया जा सकता है ?

e
Q5. Which of the following SSC CPO 16 March 2019
Q2.Two numbers are in the ratio statements is true ? (Evening)
4:7, If the HCF is 26, then the िन िल खत म से कौन सा कथन स (a)2
sum of the numbers will be: है ? (b)3
दो सं ाएं 4 : 7 के अनुपात म ह |

l
SSC CPO 14 March 2019 (c)1
यिद उनका मह म समापवतक 26 (Morning) (d)4

होगा ?
ac
है , तो इन सं ाओं का योग

SSC CPO 12 March 2019


(Evening)
(a) 312
ा (a) LCM of two natural numbers
is divisible by their HCF. / दो
ाकृितक सं ाओं का LCM उनके
HCF ारा िवभा है ।
(b) HCF + LCM of two numbers
Q8. The product of two numbers
is 45360; if the HCF of the
numbers is 36, then their LCM is
:
(b) 364 = Product of the two numbers. / दो सं ाओं का गुणनफल 45360 है |
(c) 338 दो सं ाओं का HCF + LCM = दो यिद इन सं ाओं का मह म
(d) 286 सं ाओं का गुणनफल | समापवतक 36 है , तो उनका लघु म
nn
(c) Two prime numbers are समापव ात कर |
Q3. What is the sum of the digits co-prime numbers if their LCM is SSC CPO 15 March 2019
of the least number, which when 1. / यिद दो अभा सं ाएँ सह (Morning)
divided by 12, 16 and 54, leaves अभा ह तो उनके LCM 1 ह। (a) 252
the same remainder 7 in each case (d) HCF of two numbers is the (b) 630
and is also completely divisible smallest common divisor of both (c) 126
by 13? numbers. / दो सं ाओं का HCF (d) 1260
उस सबसे छोटी सं ा के अंकों का दोनों सं ाओं का सबसे छोटा
Pi

योग ात कर िजसे 12, 16 और 24 से भाजक है । Q9.The greatest number of four


भाग दे ने पर हर बार 7 शेषफल आता digits which is exactly divisible
है और यह 13 से भी पूणतः िवभािजत Q6. The LCM of two numbers is by 24, 36 and 54 is:
है | 168 and their HCF is 12. If the चार अंकों की सबसे बड़ी सं ा जो
SSC CPO 12 March 2019 difference between the numbers is 24, 36 तथा 54 से पूणतः िवभािजत है
(Evening) 60, what is the sum of the :
(a) 36 numbers? SSC CPO 15 March 2019
(b) 16 दो सं ाओं का लघु म समापव (Morning)
(c) 9 168 है तथा उनका मह म (a) 9990
(d) 27 समापवतक 12 है | यिद सं ाओं के (b) 9924
बीच 60 का अंतर है , तो इन सं ाओं (c) 9936
Q4. The HCF and LCM of two का योग ात कर | (d) 9960
numbers is 6 and 5040

www.ssccglpinnacle.com support@ssccglpinnacle.com Ph. 09729327755, 09817390373


35
/
Days 5-7 : HCF- LCM

s
sse
la
_c
ob
Q10. An oil merchant has 3 (d) 420 Q5. Two numbers are in the ratio
bo
ah
varieties of oil of volumes 432, 5 : 11. If their HCF is 24, then the
sm
ur
yo

594 and 702 respectively. The Q2. What is the sum of digits of sum of two these numbers is:
e/
t.m

दो सं ाएं 5 : 11 के अनुपात म ह |
://

number of cans of equal size that the least number, which when
tp
ht

would be required to fill the oil divided by 15, 18 and 42 leaves यिद उनका मह म समापवतक 24
separately is: the same remainder 8 in each case है , तो इन दो सं ाओं का योग ा
िकसी तेल ापारी के पास तीन and is also divisible by 13? होगा ?
िक के तेल की 432, 594 एवं 702 उस ूनतम सं ा के अंकों का योग SSC CPO 13 March 2019
की मा ा म ह | तेल को अलग-अलग ा होगा, जो 15, 18 और 42 से (Morning)
भरने के िलए आव क बराबर िवभािजत होने पर ेक थित म (a) 384
आकार के पा ों की सं ा ात कर | एक ही शेष 8 रहता है और 13 से (b) 408
SSC CPO 16 March 2019 िवभा भी है ? (c) 120
(Afternoon) SSC CPO 13 March 2019 (d) 264
(a)13, 15, 17 (Evening)
(b)8, 11, 13 (a) 25 Q6. What is the sum of the digits

e
(c)8, 13, 15 (b) 24 of the least number, which when
(d)6, 9, 11 (c) 22 divided by 15, 25 and 27 leaves
(d) 26 the same remainder 9 in each case
Q11. The length, breadth and and is also completely divisible

l
height of a box is 506 cm, 345 Q3. What is the sum of digits of by 11?
cm and 230 cm respectively, give the least number, which when उस सबसे छोटी सं ा के अंकों का
ac
the length of the longest scale, so
that the three dimensions of the
box can be measured
एक िड े की ल ाई चौड़ाई और
ऊंचाई मश : 506 cm, 345 cm
divided by 15, 18 and 24 leaves
the remainder 8 in each case and
is also divisible by 13?
उस सबसे छोटी सं ा के अंकों का
योग ात कर िजसे 15, 18 तथा 24 से
योग ात कर िजसे 15, 25 और 27 से
भाग दे ने पर हर बार शेषफल 9 आता
है तथा यह 11 से भी पूणतया
िवभािजत है |
SSC CPO 13 March 2019
और 230 cm है | सबसे ल े पैमाने भाग दे ने पर हर बार शेषफल 8 आता (Morning)
की ल ाई बताये िजससे बॉ के है तथा यह 13 से भी िवभािजत है | (a) 20
तीनो आयामों को मापा जा सकता है SSC CPO 12 March 2019 (b) 17
nn
SSC CPO 14 March 2019 (Morning) (c) 18
(Evening) (a) 17 (d) 19
(a) 23 cm (b) 16
(b) 15 cm (c) 15 Q7. The largest number of four
(c) 30 cm (d) 18 digits that is exactly divisible by
(d) 46 cm 15, 21 and 30 is:
Q4. Two numbers are in the ratio चार अंकों की सबसे बड़ी सं ा ात
Practice Questions 4:5. If their HCF is 16, then the कर जो 15, 21 एवं 30 से पूणतः
Pi

sum of these two numbers is: िवभािजत है |


Q1. Two numbers are in ratio 6 : दो सं ाएं 4 : 5 के अनुपात म ह | SSC CPO 16 March 2019
11, If there HCF is 28, then the यिद उनका मह म समापवतक 16 (Afternoon)
sum of these two numbers is: है , तो इन दो सं ाओं का योग ा (a)9840
दो सं ाएं 6 : 11 के अनुपात म ह | होगा ? (b)9910
यिद उनका मह म समापवतक 28 SSC CPO 12 March 2019 (c)9830
है , तो इन दो सं ाओं का योग ा (Morning) (d)9870
है ? (a) 144
SSC CPO 13 March 2019 (b) 124 Q8. Four bells ring together at a
(Evening) (c) 160 certain time. After this, they rang
(a) 476 (d) 150 at intervals of 6, 8, 10 and 12
(b) 448 seconds, respectively. After how
(c) 392

www.ssccglpinnacle.com support@ssccglpinnacle.com Ph. 09729327755, 09817390373


36
/
Days 5-7 : HCF- LCM

s
sse
la
_c
ob
many minutes will they be rang (a) 250 Q15. What is the largest number
bo
81
ah
together for the first time? that divides 460, 491 and 553 and
sm
(b) 70
ur

23
yo

चार घंिटयाँ एक िनि त समय पर एक leaves remainder 26 in each case


e/

(c) 271
t.m

80
साथ बजती ह | इसके बाद वे मश :
://

?
tp

(d) 260
ht

6, 8, 10 और 12 सेकंड के अंतराल 71
वह सबसे बड़ी सं ा कौन सी है जो
पर बजती ह | िकतने िमनटों के बाद 460, 491 तथा 553 को िवभािजत
Q12. What is the largest two digit
वे िफर से पहली बार एक साथ बजेगी करती है तो ेक मामले म 26 शेष
number which when divided by 6
? रहता है ?
and 5 leaves remainder 1 in each
SSC CPO 15 March 2019 SSC MTS 6 August 2019
case?
(Evening) (Afternoon)
दो अंकों की सबसे बड़ी सं ा ात
(a) 1 Minute / िमनट (a) 27
कर िजसे 6 और 5 से िवभािजत करने
(b) 1 21 Minute / िमनट (b) 35
पर हर थित म शेष फल 1 आता है |
(c) 2 41 Minute / िमनट (c) 33
SSC MTS 5 August 2019
(d) 2 Minute / िमनट (d) 31
(Morning)
(a) 61

e
Q9. The greatest number of 5 Q16. What is the least number of
(b) 93
digits that is exactly divisible by four digits which is exactly
(c) 91
each of 8,12,15 and 20 is:/5 अंकों divisible by 2, 4, 6 and 8?
(d) 97
की सबसे बड़ी सं ा कौन सी है जो चार अंकों की वह सबसे छोटी सं ा
कौन सी है जो 2, 4, 6 और 8 से

l
8, 12, 15 और 20 म से ेक से Q13. A is the smallest three-digit
पूणत : िवभा है : पूणतः िवभािजत है ?

(a)99960
(b)99940
(c)99980
ac
SSC CPO 15 March 2019
(Evening)
number which when divided by
3,4 and 5 gives remainder 1,2 and
3 respectively. What is the sum of
the digits of A ?
A वह तीन अंको की सबसे छोटी
SSC MTS
(Evening)
(a) 1016
(b) 1024
(c) 1008
6 August 2019

सं ा है , जो 3, 4 और 5 से िवभािजत
(d)99950 (d) 1006
करने पर मसः 1, 2 तथा 3 शेषफल
दे ती है | A के अंको का योग िकतना
Q17. What is the Least Common
nn
Q10. What is the HCF of 23 × 34 है ?
Multiple of all the even numbers
and 25 × 32 ? SSC MTS 5 August 2019
between 5 and 13?
23 × 34 तथा 25 × 32 का मह म (Afternoon)
5 और 13 के बीच की सबसे सम
समापवतक ा है ? (a) 11
सं ाओं का लघु म समापव ा
SSC MTS 2 August 2019 (b) 10
होगा ?
(Morning) (c) 6
SSC MTS 7 August 2019
(a) 25 × 33 (d) 8
(Morning)
(b) 23 × 34 (a) 120
Pi

(c) 23 × 32 Q14. What is the Highest


7 ,
(b) 90
(d) 25 × 34 Common Factor ( H.C.F ) of 16
(c) 180
21 and 49 ?
32 8 (d) 60
Q11. A = HCF of 3
4 and 9
16 and 7
16 , 21
32 तथा 49
8 का मह म
B = LCM of 16
5 and 4
25 , then the समापवतक (H. C. F.) िकतना है ? Q18. If the Least Common
value of A+B will be : SSC MTS 6 August 2019 Multiple of 56, 57 and 58 is K,
A 43 तथा 16 9 का मह म (Morning) then what will be the Least
7
(a) 64 Common Multiple of 56, 57, 58
समापवतक (HCF) है , B 16
5 तथा
(b) 147 and 59?
4 का लघुतम समापवतक (LCM) 32
25
(c) 147 यिद 56, 57 और 58 का लघु म
है , A + B का मान िकतना है ? 8
समापव K है , तो 56, 57, 58 और
(d) 7
SSC MTS 2 August 2019 32 59 का लघु म समापव ा होगा ?
(Afternoon)

www.ssccglpinnacle.com support@ssccglpinnacle.com Ph. 09729327755, 09817390373


37
/
Days 5-7 : HCF- LCM

s
sse
la
_c
ob
SSC MTS 7 August 2019 x and y , then the value of (72 × floor of a room 15m 17 cm long
bo
ah
(Afternoon) and 9 m 43 cm broad?
sm
84) @ 144 is:
ur
yo

(a) 177 K यिद x × y , x तथा y का मह म 15 मी 17 सेमी लंबे तथा 9 मी 43


e/
t.m

सेमी चौड़े कमरे की फश पर िबछाने


://

(b) 59 K समापवतक है और x @ y , x तथा y


tp
ht

(c) 56 K का लघु म समापव है , तो (72 × के िलए आव क वगाकार टाइलों


(d) 57 K 84) @ 144 का मान होगा : की ूनतम सं ा ात कर |
SSC MTS 9 August 2019 SSC MTS 13 August 2019
Q19. If Least Common Multiple (Afternoon) (Evening)
of 23 and 24 is A and Highest (a) 144 (a) 851
Common Factor of 23 and 24 is (b) 504 (b) 841
B, then what is the value of A+B? (c) 210 (c) 840
यिद 23 और 24 का लघु म (d) 420 (d) 830
समापव A है और 23 तथा 24 का
मह म समापवतक B है , तो A+B Q23. The Highest Common Q26. The Highest Common
का मान ात कर | Factor and Lowest Common Factor and Lowest Common

e
SSC MTS 7 August 2019 Multiple of two numbers p and q Factor of two numbers are 20 and
(Evening) are A and B respectively. IF A+B 120. If one number is 50% more
(a) 451 = p+q, then the value of A3 + B 3 than the other number, then what
(b) 551 is : is the smaller number ?
दो सं ाओं का मह म समापवतक

l
(c) 553 दो सं ाओं p तथा q का मह म
(d) 452 समापवतक (HCF) एवं लघु म और लघुतम समापवतक मसः 20

210?
ac
Q20. What is the Highest
Common Factor of 42, 168 and

42, 168 तथा 210 का मह म


समापवतक (LCM) मसः A तथा
B है , यिद A+B=p+q है , तो
A3 + B 3 का मान है :
SSC MTS
(Evening)
9 August 2019
और 120 है | यिद एक सं ा दू सरी
सं ा से 50 % अिधक है , तो छोटी
सं ा कौन सी है |
SSC MTS 14 August 2019
(Morning)
समापवतक ा होगा ? (a) 2
(a) p3
SSC MTS 8 August 2019 (b) 60
(b) q 3
(Afternoon) (c) 40
nn
(c) p3 + q 3
(a) 14 (d) 80
(d) p3 − q 3
(b) 21
(c) 42 Q27. If the LCM of two numbers
Q24. If LCM of two numbers is
(d) 7 390 and 420 is 5460, then the
231, HCF of these two numbers is
HCF of two numbers is:
11 and the first number is 77, then
Q21. The product of two numbers यिद दो सं ाओं 390 तथा 420 का
find the second number.
is 6845, if the HCF of the number लघु म समापव 5460 है , तो इन
यिद दो सं ाओं का लघुतम
is 37, then the greater number is: दोनों सं ाओं का मह म
Pi

समापवतक 231 है , दोनों सं ाओं


दो सं ाओं का गुणनफल 6845 है | समापवतक ा होगा ?
का मह म समापवतक 11 है तथा
यिद सं ाओं का H.C.F 37 है , तो SSC MTS 14 August 2019
पहली सं ा 77 है , तो दू सरी सं ा
बड़ी सं ा ात कर | (Evening)
है :
SSC MTS 9 August 2019 (a) 35
SSC MTS 13 August 2019
(Morning) (b) 45
(Morning)
(a) 111 (c) 30
(a) 47
(b) 37 (d) 42
(b) 37
(c) 148
(c) 57
(d) 185 Q28. A temple has five bells
(d) 33
which ring at intervals of 12, 15,
Q22. If x × y denotes HCF of x 16, 20 and 25 minutes
Q25. What is the least number of
and y and x @ y denotes LCM of respectively. If they ring together
square tiles required to pave the

www.ssccglpinnacle.com support@ssccglpinnacle.com Ph. 09729327755, 09817390373


38
/
Days 5-7 : HCF- LCM

s
sse
la
_c
ob
at midnight, then at what time िवभािजत करने पर शेष-फल मशः
bo
ah
next will they ring together? Q32. The H.C.F and L.C.M of 3 और 4 आता है |
sm
ur
yo

.एक मंिदर म 5 घंिटयाँ ह जो मशः two numbers are 37 and 444 SSC MTS 8 August 2019
e/
t.m

12, 15, 16, 20 और 25 िमनट के


://

respectively. If the first number is (Morning)


tp
ht

अंतराल पर बजती ह | यिद वे आधी 111, then the other number will (a) 81
रात म एक साथ बजती ह, तो अगली be : (b) 94
बार वे िकस समय एक साथ बजगी ? दो सं ाओं का मह म और लघुतम (c) 83
SSC MTS 16 August 2019 समापवतक मसः 37 और 444 है | (d) 84
(Morning) यिद पहली सं ा 111 है तो दू सरी
(a) 8:00 PM सं ा है : Q36. If A is the smallest three
(b) 7:00 PM SSC MTS 21 August 2019 digit number divisible by both 6
(c) 7:30 PM (Afternoon) and 7 and B is the largest four
(d) 8:30 PM (a) 333 digit number divisible by both 6
(b) 74 and 7, then what is the value of
Q29. What is the HCF of 20, 250 (c) 148 B-A?
यिद A , 6 और 7 दोनों से िवभािजत

e
and 120 ? (d) 111
20, 250 तथा 120 का मह म होने वाली तीन अंकों की सबसे छोटी
समापवतक (HCF) िकतना है ? Q 33. What is the smallest सं ा है तथा B , 6 और 7 दोनों से
SSC MTS 19 August 2019 number which when increased by िवभािजत होने वाली चार अंकों की
सबसे बड़ी सं ा है , तो B-A का

l
(Afternoon) 5 is divisible by 12, 18 and 30?
(a) 15 वह सबसे छोटी सं ा कौन सी है मान ा होगा ?
(b) 10
(c) 25
(d) 20
ac
Q30. What is the LCM of the
िजसम 5 की वृ करने पर वह 12,
18 और 30 से िवभािजत हो जाती है ?
SSC MTS
(Evening)
(a) 175
2 August 2019
SSC MTS
(Evening)
(a) 9912
(b) 9870
(c) 9996
8 August 2019

HCF of 32 , 43 and LCM of 65 , 87 (b) 115 (d) 9954


? (c) 235
2 , 3 के म. स. तथा 5 , 7 के ल. (d) 195 Q37. The largest three digit
nn
3 4 6 8
स. का ल. स. ा है ? number that is exactly divisible
SSC MTS 20 August 2019 Q34. What is the largest 4-digit by 6, 7 and 8 is:
(Evening) number that is divisible by 32, 40, 6,7 और 8 से िवभा तीन अंकों की
(a) 21 36 and 48 ? सबसे बड़ी सं ा है :
35 4 अंकों की सबसे बड़ी सं ा कौन SSC MTS 13 August 2019
(b)
1
2
सी है , जो 32, 40, 36 तथा 48 से (Afternoon)
(c) 12 िवभा है ? (a) 999
(d) 35
12 SSC MTS 5 August 2019 (b) 168
Pi

(Evening) (c) 358


Q31. If two numbers are in the (a) 9220 (d) 840
ratio of 7 : 11, and their HCF is (b) 8820
13 then find their LCM. (c) 8640 Q38. What is the highest number
यिद दो सं ाओं का अनुपात 7:11 है (d) 9120 which when divides the numbers
और उनका म. स. 13 है , तो उसका 1026, 2052 and 4102, leaves
ल. स. है : Q 35. What is the largest two remainders 2, 4 and 6
SSC MTS 21 August 2019 digit number which when divided respectively.
(Morning) by 6 and 7 gives remainder 3 and वह सबसे बड़ी सं ा कौन सी है जो
(a) 101 4 respectively? सं ाओं 1026, 2052 और 4102 को
(b) 1001 दो अंकों की वह सबसे बड़ी सं ा िवभािजत करने पर शेषफल मशः
(c) 143 कौन सी है िजसे 6 और 7 से 2, 4 और 6 छोड़ती है ?
(d) 234

www.ssccglpinnacle.com support@ssccglpinnacle.com Ph. 09729327755, 09817390373


39
/
Days 5-7 : HCF- LCM

s
sse
la
_c
ob
SSC MTS 14 August 2019 को 31 से िवभािजत िकया जाएगा तो वह सबसे बड़ी सं ा कौन सी है जो
bo
ah
(Afternoon) ा शेष िकतना होगा? 192, 1056 तथा 1584 को पूणतः
sm
ur
yo

(a) 512 SSC MTS 20 August 2019 िवभािजत कर सकती है ?


e/
t.m
://

(b) 1024 (Afternoon) SSC MTS 19 August 2019


tp
ht

(c) 128 (a) 1 (Morning)


(d) 256 (b) 5 (a) 48
(c) 3 (b) 56
Q39. Find the sum of digits of the (d) 0 (c) 44
largest 6-digit number that is (d) 36
divisible by 3, 4, 5 and 6. Q42. Let x be the largest 4-digit
3, 4, 5 और 6 ारा िवभािजत होने number which when divided by 7, Q45. Let x and y be the 3-digit
वाली सबसे बड़ी 6-अंकीय सं ा के 8 and 11 leaves remainders 4, 5 numbers such that their difference
अंकों का योग िकतना है ? and 8 respectively. When x is is 729 and HCF is 81. Find the
SSC MTS 16 August 2019 divided by (7+8+11), then the value of (x+y) ? / मान लीिजए एक
(Afternoon) remainder will be : x और y, 3-अंको की ऐसी सं ाएँ है
मान िलिजएं की x, 4 अंकों की सबसे िक उनका अंतर 729 और मह म

e
(a) 45
(b) 39 बड़ी सं ा है , िजसे 7, 8 और 11 से समापवतक 81 है | (x+y) का मान
(c) 48 िवभािजत करने पर मशः 4, 5 और ात कीिजए ?
(d) 42 8 शेष बचता है | जब x को (7+8+11) SSC MTS 20 August 2019
से िवभािजत िकया जाता है , तब शेष

l
(Morning)
Q40. Let x be the smallest होगा: (a) 1053
ac
number greater than 600 which
gives the remainders 2, 3 and 4,
when divided by 5, 6 and 7
respectively. The sum of digits of
x is:
SSC MTS 21 August 2019
(Evening)
(a) 23
(b) 25
(c) 21
(b) 891
(c) 1539
(d) 1377

Q 46. A, B and C are started


मान लीिजये िक x, 600 से बड़ी (d) 19 walking from a point, Their step
सबसे छोटी सं ा है जो 5, 6 और 7 measure 42 cm, 56 cm, and 64 cm
से िवभािजत होने पर शेषफल मशः Q43. The difference between the respectively. What is the
nn
2, 3 और 4 छोड़ती है | x के अंकों का two numbers is 15 and their HCF minimum distance they should
जोड़ है - and LCM are 3 and 108 walk so that each takes exact
SSC MTS 19 August 2019 respectively. Find the difference number of steps?
(Evening) between their inverses? A ,B और C एक िबंदु से चलना शु
दो सं ाओं के बीच का अंतर 15 है करते ह, उनका कदम मशः 42
(a) 14 और उनका मह म समापवतक (एच सेमी, 56 सेमी और 64 सेमी के है ।
(b) 15 सी एफ) और लघुतम समापवतक ूनतम दू री ा है जो उ चलना
(c) 13 (एल सी एम) मश : 3 और 108 है | चािहए तािक वे ेक सटीक सं ा
Pi

(d) 16 उनके ु म के बीच अंतर ात म कदम उठाए?


कीिजए | SSC CPO 16 March 2019
Q41. When the smallest number x SSC MTS 16 August 2019 (Morning)
is divided by 5,6,8,9 and 12, it (Evening) (a) 14.58 m
gives remainder 1 in each case. 5
(a) 54 (b) 15.60 m
But x is divisible by 13. What (b) 5 (c) 13.44 m
108
will be the remainder when x will (c) 5 (d) 14.06 m
81
be divided by 31 ? 5
(d)
जब 5, 6, 8, 9 और 12 से सबसे छोटी 112
Q47. A and B begin to move
सं ा x को िवभािजत िकया जाता है , simultaneously from one point.
Q44. What is the greatest number
तो ेक मामले म 1 शेष ा होता The measurements of their cards
which can exactly divide 192,
है , िक ु x, 13 से िवभा है | जब x are 72cm and 84cm respectively.
1056 and 1584 ?
The minimum distance they

www.ssccglpinnacle.com support@ssccglpinnacle.com Ph. 09729327755, 09817390373


40
/
Days 5-7 : HCF- LCM

s
sse
la
_c
ob
travel so that the numbers of each (d) 348 मन ले िक x सबसे बड़ी सं ा है जो
bo
ah
of them are the same number : 955, 1027, 1075 को िवभािजत
sm
ur
yo

A और B एक िबंदु से एक साथ SSC CGL TIER I करती है ,तो ेक मामले म शेष


e/
t.m

चलना शु करते है | उनके कदमो समान है । िन िल खत म से कौन x


://
tp
ht

की माप मश : 72cm और 84cm Q1. What is the smallest integer का गुणनखंड नहीं है ?
कम है | वे ूनतम िकतनी दू री चले that is divisible by 3,7 and 18?/ 3, CGL 2019 Tier-II 16-11-2020
तािक ेक के कदमो की सं ां 7 और 18 से िवभािजत सबसे छोटा (a) 6
एक सामान सं ा हो : पूणाक कौन सा है ? (b) 16
SSC CPO 14 March 2019 SSC CGL 6 March 2020 (c) 4
(Evening) (Evening) (d) 8
(a) 6.3 m
(b) 2.7 m (a) 72 Q3. Find the least number which
(c) 3.54 m (b) 63 when divided by 12,18,24and 30
(d) 5.04 m (c) 252 leaves 4 as remainder in each
(d) 126 case, but when divided by 7

e
SSC CGL TIER II leaves no remainder.
Q2. What is the smallest integer वह सबसे कम सं ा ात कर िजसे
Q1. The HCF of two numbers is that is a multiple of 5,8 and 15? 12,18,24 और 30 से िवभािजत करने
21 and their LCM is 221 times वह सबसे छोटा पूणाक कौन सा है जो पर शेष 4 आता है , लेिकन 7 से
5, 8 तथा 15 का एक गुणज िवभािजत करने पर कोई शेष नहीं

l
the HCF. If one of the numbers
lies between 200 and 300, then है ? बचता है |

number is :
ac
the sum of the digits of the other

दो सं ाओं का HCF 21 है तथा


उनका LCM, HCF से 221 गुना है |
यिद उनम से एक सं ा 200 से 300
SSC CGL 9 March 2020
(Morning)
(a) 40
(b) 60
(c) 600
CGL 2019 Tier-II (18-11-2020 )
(a) 634
(b) 366
(c) 364
(d) 384
के बीच आती है , तो दू सरी सं ा ा (d) 120
है ? SSC CPO 2019
SSC CGL Tier II- 12 SSC CGL 2019 Tier-II Q4. What is the least number
nn
September 2019 Q1. When 1062, 1134 and 1182 which when divided by 15, 18
(a) 14 are divided by the greatest and 36 leaves the same remainder
(b) 17 number x, the reminder in case is 9 in each case and is divisible by
(c) 18 y. What is the value of (x-y)? 11?
(d) 15 जब 1062, 1134 और 1182 को सबसे कम सं ा ा है जो 15, 18
सबसे बड़ी सं ा x से िवभािजत और 36 से िवभािजत होने पर ेक
Q2. The LCM of two numbers x िकया जाता है , तो हर बार शेषफल y मामले म 9 शेष बचे ह और 11 से
and y is 204 times their HCF. If होता है । (x-y) का मान ा ात िवभा ह |
Pi

their HCF is 12 and the difference कीिजए ? CPO 2019


between the numbers is 60, then CGL 2019 Tier-II (15-11-2020) 23-11-2020(Morning Shift)
x+y = ? (a) 19 (a) 1269
दो सं ाओं x और y का LCM (b) 17 (b) 1089
उनके HCF से 204 गुना है | यिद (c) 16 (c) 1080
उनका HCF 12 है तथा सं ाओं के (d) 18 (d) 1071
बीच का अंतर 60 है , तो x+y का मान
है : Q2. Let x be the greatest number Q5. The least number which is
SSC CGL Tier II- 13 which when divides 955, 1027, exactly divisible by 5, 6, 8, 10
September 2019 1075, the remainder in each case and 12 is :
(a) 660 is the same. Which of the सबसे कम सं ा जो 5, 6, 8, 10 और
(b) 426 following is NOT a factor of x? 12 से पूण िवभा है :
(c) 852

www.ssccglpinnacle.com support@ssccglpinnacle.com Ph. 09729327755, 09817390373


41
/
Days 5-7 : HCF- LCM

s
sse
la
_c
ob
CPO 2019 23-11-2020(Evening Q9.The HCF of two numbers is As LCM of two numbers is
bo
ah
Shift) 29, and the other two factors of divisible by their HCF.
sm
ur
yo

(a) 240 their LCM are 15 and 13. The


e/
t.m
://

(b) 180 smaller of the two number is : Sol 6. (c)


tp
ht

(c) 150 दो सं ाओं का HCF 29 है , और Let one number =x so other


(d) 120 उनके LCM के अ दो गुणनखंड number = x+60
15 और 13. दो सं ाओं म से छोटी We know that
Q6. The least number which is सं ा ात कर | 168 × 12 = x (x+60)
exactly divisible by 4, 5, 8, 10 CPO 2019 25-11-2020(Evening 2016 = x2 + 60x
and 12 is: Shift) x2 + 60x - 2016 = 0
सबसे कम सं ा जो 4, 5, 8, 10 और (a) 406 x2 + 84x-24x - 2016 = 0
12 से पूण िवभा है (b) 377 (x+84)(x-24) = 0
CPO 2019 (c) 435 x=24
24-11-2020(Morning Shift) (d) 464 2nd number = 24+60 = 84
(a) 240 SOLUTION: Required sum = 84+24 = 108

e
(b) 180 Sol 7. (a)
(c) 120 Varieties Questions Let the numbers are 13x and 13y.
(d) 150 Sol 1. (b) According to question
Let the other number is n. 13x × 13y = 6760

l
Q7. The HCF of two numbers is We know that xy = 40
29, and the other two factors of Product of H.C.F. and L.C.M = Required number of pairs are
ac
their LCM are 15 and 13. The
larger of two number is :
दो सं ाओं का HCF 29 है , और
उनके LCM के अ दो गुणनखंड
15 और 13 है | दो सं ाओं म से बड़ी
Product of two numbers.
⇒ 3321 = 369 × n
n=9
369 = 3 × 3 × 41
9 = 3×3
1,40 and 5,8

Sol 8. (d)
45360 = 36 × LCM
LCM = 1260
सं ा ात करे | Clearly the HCF = 3 × 3 = 9
CPO 2019 24-11-2020(Evening Sol 9.(c)
Shift)
nn
Sol 2. (d) Ratio = 4:7 24 = 2 × 2 × 2 × 3
(a) 406 HCF = 26 36 = 2 × 2 × 3 × 3
(b) 435 Therefore, Sum = 26x(4+7) = 54 = 2 × 3 × 3 × 3
(c) 377 26x11 = 286 LCM of these numbers = 2 × 2
(d) 464
× 2 × 3 × 3 × 3 = 216
Sol 3. (b) LCM of (12, 16 and 54)
Q8. What is the sum of the digits = 432
of the least number which when Let the number be (432k + 7)
divided by 15, 18 and 36 leaves ATQ: For (432k+7) to be exactly
Pi

the same reminder 9 in each case divisible by 13. {429k+(3k+7)}


and is divisible by 11? should also be divisible by 13.
सबसे कम सं ा के अंकों का योग Putting the value of k=1,2,3,... in Required number = 9999-63 =
ा है जो 15, 18 और 36 से (3k+7), k=2 satisfies the equation. 9936
िवभािजत करने पर ेक बार म 9 Therefore, Least possible number
ही शेष दे ता है और 11 से िवभािजत है = 871 Sol 10. (b)
CPO 2019 Sum of digits = 8+7+1 = 16 432 = 2 × 2 × 2 × 2 × 3 × 3 × 3
25-11-2020(Morning Shift) 594 = 2 × 3 × 3 × 3 × 11
(a) 18 Sol 4. (b) Other number = 6×5040
702 = 2 × 3 × 3 × 3 × 13
210
(b) 16 = 144 HCF = 2 × 3 × 3 × 3 = 54
(c) 17
Cans for first variety of oil = 432
54
(d) 15
Sol 5. (a) From the given =8
statements only option (a) is true.

www.ssccglpinnacle.com support@ssccglpinnacle.com Ph. 09729327755, 09817390373


42
/
Days 5-7 : HCF- LCM

s
sse
la
_c
ob
Cans for 2nd variety of oil = 594 Sum of digits = 4+0+5+9 = 18 HCF of 3 and 9 = 3 and LCM of
bo
54
ah 4 and 16 = 16
sm
= 11
ur
yo

Cans for 3rd variety of oil = 702 = Sol 7. (d) Largest number of 4 9 =A= 3
HCF of 43 and 16
e/
t.m

54 16
://

digits = 9999 LCM of 16 4


5 and = LCM of 16
tp

13
ht

25
15 = 3 × 5 and 4 divided by HCF of 5 and 25
Sol 11. (a) Longest scale = HCF 21 = 3 × 7 LCM of 16 and 4 = 16
of (506, 345, 230) = 23cm 30 = 2 × 3 × 5 HCF of 5 and 25 = 5
LCM = 2 × 3 × 5 × 7 = 210 LCM of 16 4 16
5 and 25 = B = 5
Practice Questions ⇒ A+B = 3 + 16 15+256
16 5 = 80 =
271
80
Sol 1. (a) Sum of the numbers =
28x(6+11) = 28x17 = 476
Sol 12. (c)
Largest number of two digits = 99
Sol 2. (d) LCM of (15, 18 and 42) Required four digit number = HCF of 6 and 5 = 30
= 630 9999-129 = 9870

e
Let the number be (630k+8).
For (630k+8) to be divisible by Sol 8. (d) LCM of (6, 8, 10, 12) =
13, (6k+8) should be divisible by 120
13. Therefore, Required time = 120 s ⇒ Largest two digit number

l
Therefore, for k=3, (6k+8) is = 2 min divisible by 30 = 99-9
divisible by 13.
ac Required number = 99-9+1 = 91
Hence the number is 1898. Sol 9. (a) LCM of (8, 12, 15 &
Sum of digits = 1+8+9+8 = 26 20) = 120 Sol 13. (b)
Largest number of 5 digits = 3-1 = 2, 4-2 = 2 and 5-3 = 2
Sol 3. (a) LCM of (15, 18 and 24) 99999 Smallest number of 3 digit = 100
= 360 HCF of 3,4 and 5 = 60
Required number = 360k+8
For it to be divisible by 13.
nn
(9k+8) should also be divisible by
13.
Therefore, for k=2, (9k+8) is Required number = 100 + (60-40)
divisible by 13. - 2 = 118
Therefore, required number = 728 Sum of the digits = 1+1+8 = 10
Sum of digits = 7+2+8 = 17 Clearly, On dividing 99999 by
120 we get remainder 39. Sol 14. (d)
Sol4. (a) Sum of the numbers = Therefore, Desired number = HCF of 16 7 , 21 and 49 = HCF
32 8
Pi

16x(4+5) = 16x9 =144 99999-39 = 99960 of 7,21 and 49 divided by LCM


of 16,32 and 8.
Sol5. (a) Sum of number = Sol 10. (c) HCF of 7,21 and 49 = 7
24x(5+11) = 24x16 = 384 Lowest power of 2 is 3 and LCM of 16,32 and 8 = 32
lowest power of 3 is 2 so, the Required HCF = 32 7
Sol6. (c) LCM of (15, 25 and 27) highest common factor 23 × 34
= 675
and 25 × 32 is 23 × 32 . Sol 15.(d)
Required number = 675k+9
Let the required number = k
For (675k+9) to be divisible by
Sol 11. (c) According to the question
11. (4k+9) should also be
HCF of 43 and 9
16 = HCF of 3 kx + 26 = 460
divisible by 11.
and 9 divided by LCM of 4 and ⇒ kx = 434
At k=6, (4k+9) is divisible by 11.
16 ky + 26 = 491
Therefore, Required number =
⇒ ky = 465
4059

www.ssccglpinnacle.com support@ssccglpinnacle.com Ph. 09729327755, 09817390373


43
/
Days 5-7 : HCF- LCM

s
sse
la
_c
ob
kz + 26 = 553 Let the numbers are 37x and 37y
bo
ah
⇒ kz = 527 According to the question
sm
ur
yo

So, k will be the HCF of 434, 465 6845 = 37x x 37y


e/
t.m
://

and 527 ⇒x×y = 5


tp
ht

434 = 2x7x31 Only possible factors of x and y


465 = 3x5x31 are 1 and 5.
527 = 17x31 ⇒ Greatest number = 37 x 5 =
⇒ k = 31 185

Sol 16. (c) Sol 22.(a)


Least number of 4 digits = 1000 72 = 2x2x2x3x3
HCF of 2,4,6 and 8 = 24 84 = 2x2x3x7
HCF of 72 and 84 = 2x2x3 = 12 HCF of 1517 and 943 = 41
Now, Required number of tiles =
1517×943 = 851
12 = 2x2x3 41×41

e
144 = 2x2x2x2x3x3
LCM of 12 and 144 Sol 26.(c)
Required number = 1000 + =2x2x2x2x3x3 = 144 50% = 21
(24-16) = 1008 Let the smaller number = 2x and

l
Sol 23. (c) the bigger number = 3x
Sol 17. (a) ac We know that According to the question
all the even numbers between 5 A3 + B 3 = (A + B )(A2 + B 2 − AB) 20 x 120 = 2x × 3x
and 13 are 6,8,10 and 12. = ⇒ x = 20
6 = 3x2 (A + B ){(A + B )2 − 2AB − AB} So, the smaller number = 2 x 20 =
8 = 2x2x2 = 40
10 = 2x5 (A + B ){(A + B )2 − 3AB}
12 = 2x2x3 ……..(1) Sol 27. (c)
Required LCM = 2x2x2x3x5 = We know that , Let HCF of the numbers = h
nn
120 HCF × LCM = N umber1 × According to the question
N umber2 5460 x h = 390 x 420
Sol 18. (b) ⇒A×B = p × q ⇒ h = 30
LCM of 56, 57 and 58 = K Put the values of AB and (A+B)
59 is a prime number and have in equation (1) Sol 28. (a)
nothing common to 56,57 and 58. A3 + B 3 = (p + q ){(p + q )2 − 3pq} 12 = 2x2x3
⇒ Required LCM = 59 x k = 59k = (p+q)( 15 = 3x5
p + q 2 + 2pq − 3pq )
2 16 = 2x2x2x2
Pi

Sol 19. (c) = (p+q)( p2 + q 2 − pq ) 20 = 2x2x5


LCM of 23 and 24 =A = 23 x 24 25 = 5x5
= p3 + q 3
= 552 LCM of the numbers =
HCF of 23 and 24 = B = 1 2x2x2x2x3x5x5 = 1200 minutes
Sol 24. (d)
...(23 is a prime number) = 20 hours
Let the required number = k
⇒ A+B = 552+1 = 553 Required time = 12:00 AM + 20
According to the question
hours = 8:00 PM
231 x 11 = 77 x k
Sol 20. (c) ⇒ k = 23177× 11 = 33
42 = 2x3x7 Sol 29.(b)
168 = 2x2x2x3x7 20 = 2x2x5
Sol 25.(a)
210 = 2x3x5x7 250 = 2x5x5x5
Length of the room = 1517 cm
HCF of 42, 168 and 210 = 2x3x7 120 = 2x2x2x3x5
Breadth of the room = 943 cm
= 42 Required HCF = 2x5 = 10
Sol 21.(d)

www.ssccglpinnacle.com support@ssccglpinnacle.com Ph. 09729327755, 09817390373


44
/
Days 5-7 : HCF- LCM

s
sse
la
_c
ob
Sol 30. (b)
bo
ah
HCF of 32 , 3 = HCF of 2 and 3
sm
4
ur
yo

divided by LCM of 3 and 4.


e/
t.m
://

HCF of 2 and 3 = 1
tp
ht

Required number = 9999-1359 = The largest three digit number


LCM of 3 and 4 = 12
8640 divisible by 6,7 and 8 = 999-159
⇒ HCF of 32 , 43 = 12
1

5 7 = 840
LCM of , = LCM of 5 and 7
6 8 Sol 35. (a)
divided by HCF of 6 and 8. 6-3 = 3 and 7-4 = 3 Sol 38. (b)
LCM of 5 and 7 = 35 Largest number of two digit = 99 Let the required number = k
HCF of 6 and 8 = 2 LCM of 6 and 7 = 6x7 = 42 According to the question
⇒ LCM of 65 , 87 = 35
2 kx + 2 = 1026
Now, ⇒ kx = 1024
LCM of 1 , 35 = LCM of 1 and
12 2 ky + 4 = 2052
35 divided by HCF of 12 and 2. ⇒ ky = 2048
LCM of 1 and 35 = 35

e
Required number = 99-15-3 = 81 kz + 6 = 4102
HCF of 12 and 2 = 2 ⇒ kz = 4096
⇒ LCM of 12 1 , 35 = 35
2 2 Sol 36. (b) So, k will be the HCF of 1024,
LCM of 6 and 7 = 7 x 6 = 42 2048 and 4096
Sol 31.(b)

l
Smallest number of three digits = 1024 = 210
Ratio of two numbers = 7:11 100 2048 = 211
HCF = 13

1001

Sol 32. (c)


ac
Required LCM = 7 x 11 x 13 =

Smallest number of three digits


4096 = 212
⇒ k = 210 =1024

Sol 39. (d)


Biggest number of 6 digits =
Let the second number = y divisible by 6 and 7 = A = 100 +
(42-16) = 126 999999
According to the question
LCM of 6 and 7 = 7 x 6 = 42 LCM of ,3,4,5,6 = 60
37 x 444 = 111 x y
nn
⇒ y = 148 Largest number of four digits =
9999
Sol 33. (a)
12 = 2x2x3
18 = 2x3x3
30 = 2x3x5
LCM of 12, 18 and 30
=2x2x3x3x5 = 180
Pi

⇒ Required number = 180-5 =


175 Largest number of four digits Required number = 999999-39 =
divisible by 6 and 7 = A = 9999-3 999960
Sol 34. (c) = 9996 Desired sum = 9+9+9+9+6+0 =
Largest number of four digits = Required difference = B-A = 42
9999 9996-126 = 9870
32 = 2x2x2x2x2 Sol 40. (b)
40 = 2x2x2x5 Sol 37.(d) 5-2 = 3, 6-3 = 3 and 7-4=3
36 = 2x2x3x3 Largest three digit number = 999 LCM of 5, 6 and 7 = 210
48 = 2x2x2x2x3 6 = 2x3 Smallest number greater than 600
LCM of 32, 40, 36 and 48 = 7 = 1x7 which is divisible by 5,6 and 7 =
2x2x2x2x2x3x3x5 = 1440 8 = 2x2x2 210 x 3 = 630
LCM of 6, 7 and 8 = 2x2x2x3x7 Desired number = 630-3 = 627
= 168

www.ssccglpinnacle.com support@ssccglpinnacle.com Ph. 09729327755, 09817390373


45
/
Days 5-7 : HCF- LCM

s
sse
la
_c
ob
Desired sum = 6+2+7 = 15 Required HCF = 2x2x2x3 = 48 So, the numbers will be 13x21 =
bo
ah 273 and 17x21 = 357
sm
ur
yo

Sol 41. (b) Sol 45. (a) Required sum = 3+5+7 = 15


e/
t.m
://

LCM of 5,6,8,9,12 = 360 According to the question


tp
ht

⇒ x = 360k+1 x-y = 729 Sol 2. (d)


360k+1 will be divisible by 31 for HCF= 81 HCF = 12
the minimum value of k = 10 Let the numbers be 81a and 81b. LCM = 12 x 204
Desired number = 3601 Now, 81a - 81b = 729 According to the question
When 3601 is divided by 31 the ⇒a-b=9 x-y = 60
remainder will be 5. Since, 81a < 1000 And
⇒ a < 12.34 x × y = 12 x (12 x 204)
Sol 42.(b) a could be 10, 11 and 12 ⇒ (60 + y ) × y = 12 × (12 × 204)
7-4 = 3, 8-5 = 3 and 11-8 = 3 So, b could be 1, 2 and 3 ⇒ 60y + y 2 − 144 × 204 = 0
Largest number of 4 digit = 9999 ⇒ y 2 + 204y − 144y − 144 × 204 = 0
LCM of 7,8 and 11 = 616 If 81b > 100 ⇒ (y + 204)(y − 144) = 0

e
⇒ b > 1.23 ⇒ y = 144 and x = 144 + 60 = 204
Hence, b could be 2 and 3. Required sum = x+y = 204+144 =
and a could be 11 and 12. 348
Possible pairs of x and y = (891,

l
162) or (972, 243) SSC CGL TIER I
ac Since, HCF of (972, 243) = 243
Therefore, the required pair is Sol 1. (d) LCM (3,7,18) = 18 × 7
Desired number = x =
(891, 162) = 126
9999-143-3 = 9853
Hence, x + y = 891+162 = 1053
Divisor = 7+8+11 = 26
Sol 2. (d) LCM(5,8,15) = 15 × 8
Desired remainder = 9853
26 = 25 Sol 46. (c) = 120
42 = 2x3x7
Sol 43. (b) 56 = 2x2x2x7 SSC CGL 2019 TIER-II
Let the numbers are 3k and 3m 64 = 2x2x2x2x2
nn
Sol:1.(d)
According to the question Required distance = LCM of Difference 1134-1062=72
3k-3m = 15 42,56 and 64 = 2x2x2x2x2x3x7 =
⇒ k − m = 5 or k = 5+m 1344 cm or 13.44 meters 1182-1134= 48
And
HCF of 72 and 48 = 24
3 x 108 = 3k x 3m Sol 47. (d) When 1062 is divided by 24 we
⇒ k .m = 36 72=2x2x2x3x3 get reminder as = 6
⇒ m(5 + m) = 36 84=2x2x3x7 24 - 6 = 18
⇒ 5m + m2 − 36 = 0 Required distance = LCM of (72
Pi

⇒ m2 + 9m − 4m − 36 = 0 and 84) = 2x2x2x3x3x7 = 5.04 m Sol:2.(b)


⇒m= 4 Difference between 955 and 1027
…….(m =/− 9 ) SSC CGL TIER II = 72
So, k=5+4 = 9 Difference between 1027 and
The two numbers will be 3(4) = Sol 1. (d) 1075 = 48
12 and 3(9) = 27 HCF = 21 LCM of 48 and 72 = 24
1
Desired difference = 12 1 =
− 27 LCM = 21 x 221 When 955 is divided by 24
5 Let the numbers are 21a and 21b remainder is = 19
108
⇒ 21a x 21b = 21 x (21 x 221) Multiples of 24 = 1, 2, 3, 4, 6, 8,
⇒ a × b = 221 12, 24
Sol 44. (a)
192 = 2x2x2x2x2x2x3 Only possible pairs of a,b are 13 So, 16 is the answer
1056 = 2x2x2x2x2x3x11 and 17.
1584 = 2x2x2x2x3x3x11 Sol:3.(c)

www.ssccglpinnacle.com support@ssccglpinnacle.com Ph. 09729327755, 09817390373


46
/
Days 5-7 : HCF- LCM

s
sse
la
_c
ob
LCM of 12, 18, 24 and 30 = 360 We have to find the number
bo
ah
To get reminder as 4 divisible by 11 so dividing
sm
ur
yo

Number = 360x + 4 (180k+9) by 11 we get


e/
t.m
://

Now by value putting Remainder 4k+9 which should be


tp
ht

Starting from 1 divisible by 11


Number = 364 (divisible by 7) By using trial and error we found
So, 364 is the answer out that by putting K=6 we get 33
which is divisible by 11
SSC CPO 2019 So the number will be 180✕6+9
Sol:4.(b) 1089.
Least number divisible by 15 18 Sum of digit = 18
and 36 leaving remainder 9 will
be of the form Sol:9.(b)
LCM (15,18,36)✕K+9 (where K Let the number be 29x and 29y
can be 0,1,2,3……) If, the smaller factor is 15 then

e
LCM will be 180 so the number the number will be = 29 × 13=
will be of the form 180k+9 377
We have to find the number
divisible by 11 so dividing

l
(180k+9) by 11 we get
Remainder 4k+9 which should be
ac
divisible by 11
By using trial and error we found
out that by putting K=6 we get 33
which is divisible by 11
So the number will be 180✕6+9
1089.

Sol:5.(d)
nn
LCM of 5, 6, 8, 10 and 12 is 120

Sol:6.(d)
LCM of 4, 5, 8, 10 and 12 is
120

Sol:7.(b)
Let the number be 29x and 29y
Pi

If, the larger factor is 15 then the


number will be = 29 × 15= 435

Sol:8.(a)
Least number divisible by 15 18
and 36 leaving remainder 9 will
be of the form
LCM (15,18,36)K+9 (where K
can be 0,1,2,3……)
LCM will be 180 so the number
will be of the form 180k+9

www.ssccglpinnacle.com support@ssccglpinnacle.com Ph. 09729327755, 09817390373


47
/
Days 8-12 : Simpli ication

s
sse
la
_c
ob
SIMPLIFICATION / Example: √225 = 15 ,
bo
ah
सरलीकरण
sm
√625 = 25
ur
yo
e/
t.m
://

SQUARE ROOT / वगमूल 13) If any number ends with 6, its


tp
ht

square root will end with 6, given


KEY POINTS / मुख िबंदु : that the number is a perfect
square.
2
1) (a + b) = a2 + b2 + 2ab
√2304 = 48 अगर कोई सं ा 6 के साथ समा
2
होती है , तो इसका वगमूल 6 के साथ
2) (a − b) = a2 + b2 − 2ab 7) If √? = x, then the required समा होगा, यह दे खते ए िक
2 2
3) a − b = (a + b)(a − b) number will be = x2 / यिद √? = x सं ा एक पूण वग है |
, तो आव क सं ा = x2 होगी|
4) Square root of a number is a Example: √256 = 16 ,
value that, when multiplied by √676 = 26
itself, gives the given number. 8) √a4 × b4 × c4 = a2 b2 c2
िकसी सं ा का वगमूल एक ऐसा 14) If any number ends with 9, its
मान होता है , जो अपने आप से गुणा square root will end with 3 or 7,

e
m
√an × b m n
करने पर, दी गयी सं ा दान 9) =a ×b
2 2

given that the number is a perfect


करता है | square.
Example / उदाहरण:
36 has two square roots 6 and -6;
10) √x
√y
=
√ x
y अगर कोई सं ा 9 के साथ समा
होती है , तो इसका वगमूल 3 या 7 के

l
2 2
6 = 36 and − 6 = 36, hence we साथ समा होगा, यह दे खते ए िक
11) √x × √y = √xy
write √36 = ±6
ac सं ा एक पूण वग है |
36 के दो वगमूल है , 6 और -6; Example: √49 = 7 , √169 = 13
10) If any number ends with 1, its
62 = 36 और − 62 = 36, इसिलये square root will end with 1 or, 9,
हम िलखते ह √36 = ±6 given that the number is a perfect 15) The square of any number
square. always ends with 0, 1, 4, 5, 6 or, 9
5) Square root by Factorization अगर कोई सं ा 1 के साथ समा but will never end with 2, 3, 7 or,
method/ फै ोराइज़ेशन िविध ारा होती है , तो इसका वगमूल 1 या 9 के 8.
वगमूल: साथ समा होगा, यह दे खते ए िक िकसी भी सं ा का वग हमेशा 0, 1,
सं ा एक पूण वग है | 4, 5, 6 या 9 के साथ समा होता है ,
nn
Find square root of 1089./1089
का वगमूल खोज| लेिकन 2, 3, 7 या 8 के साथ कभी
Example: √81 = 9 , √121 = 11 ख नहीं होगा।
First find the factors of 1089./
सबसे पहले 1089 के गुणां कों का पता 11) If any number ends with 4, its
लगाएं 16) If in a given number, the total
square root will end with 2 or, 8, number of digits are n and if n is
given that the number is a perfect even, the square root of that
square. number will have 2n digits and if
अगर कोई सं ा 4 के साथ समा n is odd, then the number of
होती है , तो इसका वगमूल 2 या 8 के
Pi

2 .अगर िकसी िदए


digits will be n+1
साथ समा होगा, यह दे खते ए िक
गए सं ा म, अंकों की कुल सं ा n
सं ा एक पूण वग है |
है और यिद n सम है , तो उस सं ा
Example: √324 = 18 , के वगमूल म 2n अंक होंगे और यिद
√124 = 12 n िवषम है , तो अंकों की सं ा n+12

√1089 = 3×11 = 33 होगी|


12) If any number ends with 5, its
square root will end with 5, given
6) Square root by Division
method/ भाग िविध ारा वगमूल:
that the number is a perfect
square.
17)
√ √x√x…n = x
x
2n −1
2n

Find square root of 2304. अगर कोई सं ा 5 के साथ समा


Example/उदाहरण:
होती है , तो इसका वगमूल 5 के साथ
समा होगा, यह दे खते ए िक
सं ा एक पूण वग है |

www.ssccglpinnacle.comsupport@ssccglpinnacle.com Ph. 09729327755, 09817390373


48
Days 8-12 : Simpli ication

s
sse
la
_c
ob
Find the value of the larger number. ∴ Value will 1) If a number is multiplied two
bo
ah

√√√ √
be 4. times with itself, then the result of
sm
ur

1 के अंतर के साथ 12 के गुणां क ह


yo

2√2 this multiplication is called the


e/

2 2 2
t.m

4, 3. यहां , 4 बड़ी सं ा है . अतः


://

cube of that number.


tp
ht

मू 4 होगा|
यिद एक सं ा म दो बार अपने आप
Here, n = 5, ∴ से गुणा िकया जाता है , तो इस गुणन
Conventional method/ पार रक

√√√ √
के प रणाम को उस सं ा के घन
िविध:
2 2 2 2√2 कहा जाता है ।
Let
Example/उदाहरण: 2×2×2=

25 −1


31
= 2 25 = 2 32 12 + 12 + √12 + …∞ = x 2
3

Cube root of 23 = √23 = 23× 3


3 1
⇒ √12 + x = x
18)
√√√ x x x…∞ = x ; ⇒
2
12 + x = x2
x − x − 12 = 0


=2

x2 − 4x + 3x − 12 = 0 2) Algebraic method to calculate


Example/उदाहरण: cube: / घन की गणना करने के िलए
⇒ (x − 4) (x + 3) = 0 ⇒

e
√√√ 7 7 7…∞ = 7 x = 4 and x =− 3
∴x=4
बीजीय िविध:
3
(a + b) = a3 + 3ab (a + b) + b3
3
(a − b) = a3 − 3ab (a − b) − b3
Example/ उदाहरण:

l
19) To find the value of Find the value of
Indices and Surds/ घातांक और
√ x+ ac√ x + √x + …∞
the factors of x, such that the
, find
√ 30 − √30 − √30 − …∞ करणी

difference between the factors is Key Points:/ मुख िबंदु:


The factors of 30 with a
1, then the larger factor will be
the result.
difference of 1 are 5, 6. Here, 5 is 1) pm × pn = pm+n
the smaller number. ∴ Value will


n
2 ) (pm ) = pmn
x+ √ x + √x + …∞ के मू be 5.
1 के अंतर के साथ 30 के गुणां क ह 3)
pm
= pm−n
का पता लगाने के िलए, x के गुणां कों
5, 6. यहां , 5 छोटी सं ा है . अतः pn
nn
का पता लगाएं , ऐसा िक गुणां कों के
मू 5 होगा| 4) ( qp ) =
n pn
बीच का अंतर 1 है , तो बड़ा गुणां क pn
प रणाम होगा|
21) If any number is in 1 5) p0 = 1
√a ± √b
20) To find the value of form, then we multiply by its 6) p−n = 1
pn
rationalization factor √a ∓ √b in
√ x− √x − √x − …∞
the factors of x, such that the
, find both numerator and denominator.
यिद कोई सं ा 1 प म है ,
1
7) p m = m√p
1
√a ± √b 8) m√p = p m
difference between the factors is तब हम उसे उसके प रमेयकरण
Pi

1, then the smaller factor will be 9) m√pq = m√p × √


m
q
गुणां क √a ∓ √b से गुणा करते है |
the result.

√ √
p m
m √p
10) =
x− √ x − √x − …∞ के मू Example / उदाहरण: Find the
value of 1 ;
q

m
m
√q
का पता लगाने के िलए, x के गुणां कों √9−√8 11) (m√p) = p
का पता लगाएं , ऐसा िक गुणां कों के
= 1 × √ √ = √2 √ 2
1 9+ 8 9+ 8
बीच का अंतर 1 है , तो छोटा गुणां क √ −√
9 8 √ −√
9 8 √ √
9+ 8 √9 −√8
प रणाम होगा| Simplification / सरलीकरण
3+2√2
= 1 = 3 + 2√2
Example/उदाहरण: Find the value KEY POINTS:/ मुख िबंदु:
Cube and Cube Root / घन और
of
√ 12 + √12 + √12 + …∞
The factors of 12 with a
घनमूल
1). Conversion of complex
arithmetic expression into simple
one is called simplification.
difference of 1 are 4, 3. Here, 4 is KEY POINTS:/ मुख िबंदु:

www.ssccglpinnacle.comsupport@ssccglpinnacle.com Ph. 09729327755, 09817390373


49
Days 8-12 : Simpli ication

s
se
s
la
_c
ob
जिटल अंकगिणतीय ंजक का (d) 1.0505
bo
ah
सरल प म पां तरण सरलीकरण Varieties Questions
sm
ur
yo

कहलाता है | Q5. The value of 5 ÷ 5 of 5 × 2


e/
t.m
://

2). VBODMAS Rule: Vinculum Q1. The value of 2 × 3 ÷ 2 of 3 × 2 + 2 ÷ 2 of 2 × 5 - (5-2) ÷ 6 × 2


tp
ht

> Brackets > Of > division > ÷ (4 + 4 × 4 ÷ 4 of 4-4 ÷ 4 × 4 ) is: is:


multiply > addition > subtraction 2 × 3 ÷ 2 of 3 × 2 ÷ (4 + 4 × 4 ÷ 4 5 ÷ 5 of 5 × 2 + 2 ÷ 2 of 2 × 5
● First solve vinculum/ of 4-4 ÷ 4 × 4 ) का मान ात कर | - (5-2) ÷ 6 × 2 का मान ात कर |
रे खाको क i.e. bar. SSC CGL 4 June 2019 SSC CGL 4 June 2019
(Morning) (Afternoon)
Eg: (7 − 5 − 4) =? (a) 8 (a) 59
First solve 5 – 4 i.e. 1 (b) 1 (b) 19
10
then 7 -1 = 6. (c) 4 (c) 19
● For brackets/को क (d) 2 (d) 23
open first small 2

brackets (….), then Q2. The value of


Q6. The value of 9 of (
{…} and then […].

e
15
2 87 ÷ (3 65 ÷ 72 of 2 31 ) 2
● For modulus e.g. |-3| ÷ of 23 )
2
× [(2 76 of 4 51 ÷ 2
3 ) × 95 ] is : / 3 3
we write magnitude ÷ ( 43 × 43 ÷ 43 of 34 ) of
only not sign i.e. 3 in 2 87 ÷ (3 65 ÷ 72 of 2 31 )
( 45 ÷ 25 × 52 of 54 ) is :
this case.मापां क, जैसे × [(2 76 of 4 51 ÷ 2
) × 95 ] का मान

l
3 9
|-3| के िलए, हम केवल of (
ात कर | 15
ac अंक िलखगे, ना िक 2 ÷ 2 of 3 ) ÷ ( 3 × 3 ÷ 3 of
3 3 2 4 4 4
4
3) of
SSC CGL 4 June 2019
उसका िच | इस ( 45 ÷ 25 × 52 of 54 ) का मान ात कर
थित म केवल 3 िलखा (Evening)
(a) 41 |
जाएगा |
(b) 4 SSC CGL 6 June 2019
1 (Afternoon)
Important Formulas/ मह पूण (c) 23
सू : (a) 20
9
(d) 5
(b) 4
25
2
1). (a + b) = a2 + 2ab + b2 Q3. The value of (5+3 ÷ 5 × 5) / (3 (c) 18
nn
125
2 2 40
2). (a − b) = − 2ab + b a2 ÷ 3 of 6) of (4 × 4 ÷ 4 of 4+4 ÷ 4 × (d) 9
2 4) is (5+3 ÷ 5 × 5) / (3 ÷ 3 of 6) of
3). (a2 − b ) = (a + b)(a − b)
4). (4 × 4 ÷ 4 of 4+4 ÷ 4 × 4) का मान Q7. The value of 16 ÷ 4 of 4 ⨯ [
3
(a + b) = a3 + b3 + 3ab(a + b) ात कर | 3 ÷ 4 of { 4 ⨯ 3 ÷ ( 3 + 3 )}] ÷ (2
5). SSC CGL 6 June 2019 ÷ 4 of 8) is :
3 16 ÷ 4 of 4 ⨯ [ 3 ÷ 4 of { 4 ⨯ 3 ÷
(a − b) = a3 − b3 − 3ab(a − b) (Morning)
6). (a) 8 51 ( 3 + 3 )}] ÷ (2 ÷ 4 of 8) का मान
ात कर |
Pi

3 2
(a3 + b ) = (a + b)(a2 − ab + b ) (b) 7 31
7). (c) 9 53 SSC CGL 6 June 2019
3 2
(a3 − b ) = (a − b)(a2 + ab + b ) (Evening)
(d) 6 32
3 3 3 (a) 6
8). a + b + c − 3abc
(b) 9
( )
= (a + b + c) a2 + b2 + c2 − ab − bc − ca Q4. The simplified value of
1.0025+6.25×10−6
(c) 48
= 1 (a + b + c)[(a − b)2 + (b − c)2 + (c − a)2 ] 0.0025+0.95 is:
2 (d) 16
1.0025+6.25×10−6
Here, if (a + b + c) = 0, 0.0025+0.95 सरलीकृत मान है
3
then a3 + b + c3 − 3abc = 0 ⇒ SSC CHSL 9 June 2019 Q8. The value of
3 8 of (5 1 ÷ 2 1 of 4) ÷ (8
a3 + b + c3 = 3abc (Evening) 9 4 3
2 ÷ 2 of 54 ) of (8 × 2 ÷ 54 ) is
9). (a + b + c) 3 3

(
= a2 + b2 + c2 + 2(ab + bc + ca) ) (a) 1.0025
(b) 1.0525
(c) 1.0005

www.ssccglpinnacle.comsupport@ssccglpinnacle.com Ph. 09729327755, 09817390373


50
Days 8-12 : Simpli ication

s
se
s
la
_c
ob
8 of (5 41 ÷ 2 31 of 4) ÷ (8 5.75×5.75×5.75+3.25×3.25×3.25 िकसके Q3. The value of 3 × 2 32 ÷ 5 of
bo
9 57.5×57.5+32.5×32.5−57.5×32.5 4 9
ah
sm
÷ 2 of 4
5) of (8 × 2 ÷ 4
5) का मान बराबर है ? 1 1 2
+ 23 ×3 5 ÷ 2of 2 31 is :
ur

3 3 5 6 7
yo

ात कर | SSC CPO 12 March 2019


e/

3 × 2 32 ÷ 5 of 1 51 + 23
2 ×3 5 ÷
t.m

4 9 6
://

(Evening)
tp

SSC CGL 7 June 2019 2 of 2 31 का मान ात कर |


ht

7
(Morning) (a) 0.009
SSC CGL 10 June 2019
(a) 1 81 (b) 0.0009
(Morning)
(b) 4 (c) 0.9
15 (a) 1 65
1 (d) 0.09
(c) 200 (b) 1 32
(d) 1
100 Q13. Find the value of (c) 3 21
(d) 4 65
Q9. The value of 4.5 - ( 3.2 ÷ 0.8 √4 + √144
⨯ 5 ) + 3 ⨯ 4 ÷ 6 is
4.5 - ( 3.2 ÷ 0.8 ⨯ 5 ) + 3 ⨯ 4 ÷ 6
√4 + √144 का मान ा है ? Q4. The value of 3.8 - (4.2 ÷ 0.7
⨯ 3) + 5 ⨯ 2 ÷ 0.5 is
SSC CPO 14 March 2019
का मान ात कर | 3.8 - (4.2 ÷ 0.7 ⨯ 3) + 5 ⨯ 2 ÷ 0.5
(Evening)
SSC CGL 10 June 2019

e
(a) 14 का मान ात कर |
(Afternoon) SSC CGL 10 June 2019
(b) 12.17
(a) -13.5 (Evening)
(c) 4
(b) 4.2 (a) 5.8
(d) 3.74
(c) -8.5

l
(b) 18.4
(d) 5.7 (c) 21.8
ac Practice Questions
(d) 15.6
Q10. What is the simplified value
Q1. The value of
of Q5. The value of : 2.8 + (5.2 ÷
7 21 × (3 51 ÷ 4 21 of 5 31 ) + [11 − ( 85 + 3
5 ÷ 10 of 10 × 4 + 4 ÷ 4 of 4 × 10 − (10 1.3 ⨯ 2) - 6 ⨯ 3 ÷ 8 + 2
− 1 41 )] ÷ 5 43 - 5 ÷ 5 ⨯ 5 of 5 ÷ 25
− 4) ÷ 16 × 4 2.8 + (5.2 ÷ 1.3 ⨯ 2) - 6 ⨯ 3 ÷ 8 +
5 ÷ 10 of 10 × 4 + 4 ÷ 4 of 4 × 10 − (10 is:
2 का मान ात कर |
7 21 × (3 51 ÷ 4 21 of 5 31 ) + [11 − ( 85 + 3
− 4) ÷ 16 × 4 का सरलीकृत मान है - SSC CGL 11 June 2019
SSC CHSL 3 July − 1 41 )] ÷ 5 43 - 5 ÷ 5 ⨯ 5 of 5 ÷ 25
nn
(Morning)
2019(Evening) का मान ात कर | (a) 6.45
(a) 1.2 SSC CGL 7 June 2019 (b) 4.55
(b) 2.5 (Afternoon) (c) 8.45
(c) 21 (a) 21 (d) 10.55
(d) 58.5 (b) 1
10
(c) 3 Q6. The value of : 7.2 + (8.4 ÷
10
Q11. The simplified value of (d) 1 21 0.12 ⨯ 0.2) - 5 ⨯ 3 ÷ 0.05 + 3
(3 15 − 35 ) ÷ 8
7.2 + (8.4 ÷ 0.12 ⨯ 0.2) - 5 ⨯ 3 ÷
Pi

1 6 1 1
5
1 7 ÷{ 7 −( 7 ÷ 5 )}
is:
Q2. The value of 6 - 6 ÷ 6 ⨯ 6 + 0.05 + 3 का मान ात कर |
(3 15 − 35 ) ÷ 85
का सरलीकृत मान है SSC CGL 11 June 2019
1 7 ÷{ 67 −( 17 ÷ 15 )}
1 11 of 2 ) ÷
(6 ÷ 6 of 6) ⨯ 6 - (3 32 ÷ 30 3
SSC CHSL 5 July (Afternoon)
5 is :
2019(Afternoon) (a) -75.8
6 - 6 ÷ 6 ⨯ 6 + (6 ÷ 6 of 6) ⨯ 6 -
13
(a) 64 11 of 2 ) ÷ 5 का मान (b) -275.8
(3 32 ÷ 30 3 ात
13 (c) 21.2
(b) 16 कर |
13
(d) -175.8
(c) 8 SSC CGL 7 June 2019
(d) 13
7
(Evening) Q7. The value of : 5.8 + (7.4 ÷
(a) 0 3.7 ✕ 5) - 6 ✕ 2 ÷ 2.5
Q12. 5.75×5.75×5.75+3.25×3.25×3.25 is (b) 2 5.8 + (7.4 ÷ 3.7 ✕ 5) - 6 ✕ 2 ÷
57.5×57.5+32.5×32.5−57.5×32.5
(c) -1 2.5 का मान ात कर |
equal to:
(d) -2

www.ssccglpinnacle.comsupport@ssccglpinnacle.com Ph. 09729327755, 09817390373


51
Days 8-12 : Simpli ication

s
sse
la
_c
ob
SSC CGL 11 June 2019 (d)7 Q16. The value of 3 × 2 ÷ 3 of 12
bo
ah
(Evening)
sm
- 3 ÷ 2 × (2-3) × 2 + 3 ÷ 2 of 3 is
ur
yo

(a) 12 Q12. The value of 15.2 + 5.8 ÷ 3 × 2 ÷ 3 of 12 - 3 ÷ 2 × (2-3) × 2 +


e/
t.m
://

(b) 11 2.9 × 2 - 3.5 × 2 ÷ 0.5 is equal to : 3 ÷ 2 of 3 का मान ात कर |


tp
ht

(c) 10 15.2 + 5.8 ÷ 2.9 × 2 - 3.5 × 2 ÷ 0.5 SSC CHSL 2 July


(d) 9 का मान ात कर | 2019(Afternoon)
SSC CGL 13 June 2019 (a) 2 31
Q8. The value of: 3.8 + (8.2 ÷ (Afternoon) (b) -2 31
4.1 × 2) - 4 × 3 ÷ 1.2 (a)4.8 (c) -3 32
3.8 + (8.2 ÷ 4.1 × 2) - 4 × 3 ÷ (b)3.2
(d) 3 32
1.2 का मान ात कर | (c)5.2
SSC CGL 12 June 2019 (d)5.4
(Morning) Q17. The simplified value of
(a) 2.2 Q13. 9 3 × 2 ÷ 3 of 2 × 3
(b) -1.2 3 ÷ [2 1 ÷ {4 1 1 3 ÷ (5 + 5 × 5 ÷ 5 of 5 − 5 ÷ 10
4 6 3 − (2 2 + 4 )}] is
of 51 is:

e
(c) 1.2 equal to:
(d) -2.2 9 43 ÷ [2 61 ÷ {4 31 − (2 21 + 43 )}] का 3 × 2 ÷ 3 of 2 × 3 ÷
मान ात कर | (5 + 5 × 5 ÷ 5 of
Q9. The value of : 7.5 + (5.4 ÷ SSC CGL 13 June 2019 5 − 5 ÷ 10 of 51 ) का सरलीकृत

l
4.5 ⨯ 2) - 8 ⨯ 4 ÷ 3.2 (Evening) मान है -
7.5 + (5.4 ÷ 4.5 ⨯ 2) - 8 ⨯ 4 ÷ 3.2
ac (a) 15 SSC CHSL 3 July
4
का मान ात कर | 2019(Morning)
(b)3
SSC CGL 12 June 2019 (a) 76
(c) 39
8
(Afternoon) (b) 17
(d)4 5
(a) 0.1 2
(c) 3
(b) -0.1 3 3 3 30
Q14. The value of ÷ of × (d)
(c) -0.2 4 4 4 59
4 + 25 ÷ 2 of 5 2 2 5
(d) 0.2 3 5 4 -( 3 + 3 of 6 ) is:
3 ÷ 3 of 3 4 5 2 5 2 Q18. The simplified value of
4 × 3 + 2 ÷ 5 of 4 -( 3
nn
4 4
Q10. The value of : 108 ÷ 36 ⨯ 4 + 32 of 5 ) का मान ात कर |
7 of 3 ) ÷ 4
( 57 ÷ 10 4
7 1
9 − ( 16 ÷ 10 2
6
+ 2.5 ⨯ 4 ÷ 0.5 - 10 SSC CHSL 1 July × 7 51 ) × 5
12 is: /
108 ÷ 36 ⨯ 4 + 2.5 ⨯ 4 ÷ 0.5 - 10 2019(Evening) ( 57 ÷ 7
10 of 43 ) ÷ 4
7 ÷ 10 1
− ( 16
9 2
का मान ात कर | (a) 14 × 7 51 ) × 5 का सरलीकृत मान है -
3 12
SSC CGL 12 June 2019 (b) 41 SSC CHSL 3 July
9
(Evening) 22 2019(Afternoon)
(c) 3
(a) 18 (a) 47
(d) 50 8
(b) 16
Pi

9
(b) 39
(c) 22 4
(c) 49
(d) 20 Q15. The value of 8
3÷{5−5÷(6−7)×8+9} (d) 41
4+4×4÷4 of 4 is 4
Q11. 21.6 ÷ 3.6 3÷{5−5÷(6−7)×8+9}
का मान ात कर |
4+4×4÷4 of 4
× 2 + 0.25 × 16 ÷ 4 Q19. The simplified value of
SSC CHSL 2 July 3 × 6 ÷ 4 of 6 − 6 ÷ 2 × (4 − 6) + 4
− 6 is equal to:
2019(Morning)
21.6 ÷ 3.6 × 2 + 0.25 × 16 ÷ 4 − 6 1 − 2 × 3 ÷ 6 of 31 is
(a) 45
का मान ात कर | 1
3 × 6 ÷ 4 of 6 − 6 ÷ 2 × (4 − 6) + 4
(b)
SSC CGL 13 June 2019 18
1
− 2 × 3 ÷ 6 of 31 का सरलीकृत मान
(Morning) (c) 90 है
(c) 1
(a)6 3 SSC CHSL 4 July
(b)5 2019(Morning)
(c)8 (a) 1 43

www.ssccglpinnacle.comsupport@ssccglpinnacle.com Ph. 09729327755, 09817390373


52
Days 8-12 : Simpli ication

s
se
s
la
_c
ob
(b) 7 43 (3 15 + 35 )÷ 85 SSC CHSL 9 July
bo
Q24. The value of is:
ah 1 8 ÷{ 58 +( 18 ÷ 13 )}
1
2019(Evening)
sm
(c) 13 43
ur

(3 15 + 35 )÷ 85
yo

का सरलीकृत मान है (a) 0


e/

(d) 8 31 1 18 ÷{ 58 +( 18 ÷ 13 )}
t.m
://

(b) 41
tp

SSC CHSL 8 July


ht

(c) 1
Q20. The simplified value of 15 2019(Afternoon) 2

of 8 - 6 + [(27 - 3) ÷ 6 − 4] is: 19
(a) 16 (d) 1
15 of 8 - 6 + [(27 - 3) ÷ 6 − 4] (b) 19
7
का सरलीकृत मान है 19 Q29. The simplified value of
(c) 9
SSC CHSL 4 July 19
[1 51 of { 73 − (1 15
4 13 ) × 5 }] ÷ ( 6 ÷ 5)
− 15 7 7
(d) 64
2019(Afternoon) is: /
(a) 114 [1 51 of { 73 − (1 15
4
− 13
15 ) × 5
7 }] ÷ ( 76 ÷ 5)
Q25. The simplified value of
(b) 120 46− 34 of 32−6 का सरलीकृत मान है
(c) 124 37− 34 of (34−6)
is: SSC CHSL 10 July
(d) 116 46− 34 of 32−6 2019(Morning)
37− 34 of (34−6)
का सरलीकृत मान है 2
(a) 15

e
Q21. The simplified value of 15 SSC CHSL 8 July (b) 1
5
of 8 + 6 + [(27 - 3) ÷ 6 + 4] is: 2019(Evening)
(c) 1
15 of 8 + 6 + [(27 - 3) ÷ 6 + 4] (a) 2 4
19 (d) 15
का सरलीकृत मान है (b) 16

l
(c) 19
SSC CHSL 5 July 64 Q30. The simplified value of
2019(Morning)
(a) 128
(b) 134
(c) 130
(d) 136
ac (d) 1

Q26. The simplified value of


46− 34 of 32−6
11+ 34 of (34−6)
46− 34 of 32−6
is:
1

is:
1
2
8

of
1

8
5
का सरलीकृत मान है
SSC CHSL
5

5 + 3 × 1 7 ÷ 2 )}
÷ {2 51 − ( 16
3

10
7

8
2
2 of 5 ÷ {2 5 − ( 16 + 5 × 1 8 ÷ 3 )}

July
11+ 34 of (34−6)
का सरलीकृत मान है
Q22. The simplified value of 2019(Afternoon)
(3 15 + 35 )÷ 85
SSC CHSL 9 July (a) 1
is: 2019(Morning)
nn
1 17 ÷{ 67 −( 17 ÷ 15 )} (b) 52
(3 15 + 35 )÷ 85 (a) 71 1
का सरलीकृत मान है (c) 5
1 7 ÷{ 67 −( 17 ÷ 15 )}
1
(b) 1
(d) 4
SSC CHSL 5 July (c) 41
2019(Evening) (d) 1
2 Q31. The simplified value of
(a) 19
7 0.01404 is:
19 2 2
(b) 8 Q27. The simplified value of
24 +6 −144

19
0.01404 का सरलीकृत मान है
(c) 16 46 + 3
of 32 − 6 242 +62 −144
4
is: SSC CHSL 10 July
Pi

3
19 11 + 4 of (34 − 6)
(d) 64
46 + 34 of 32 − 6 2019(Evening)
11 + 34 of (34 − 6)
का सरलीकृत मान है
Q23. The simplified value of SSC CHSL 9 July (a) 3 × 10−5
(3 15 + 35 )÷ 85 5
is: / 2019(Afternoon) (b) 6 × 10−
1 17 ÷{ 57 +( 17 ÷ 13 )} 4
(3 15 + 35 )÷ 85 (a) 1 (c) 2.4 × 10−
का सरलीकृत मान है (b) 41 4
1 17 ÷{ 57 +( 17 ÷ 13 )} (d) 3 × 10−
SSC CHSL 8 July (c) 2
2019(Morning) (d) 21 Q32. The simplified value of {1
(a) 19
8
1
4 of (2
19
(b) 16 Q28. The simplified value of 2 31 1
3
5 }+
÷ 1 52 ) − 1 12 1
9 ÷ 2 31 + 2
7 + 1
6
19
(c) 64 of ( 53 ÷ 92 ) − (4 52 + 20
19 ÷ 1 ) is:
2 is: /
(d) 19
7 2 3 of ( 5 ÷ 9 ) − (4 5 + 20 ÷ 21 ) का
1 3 2 2 19

सरलीकृत मान है

www.ssccglpinnacle.comsupport@ssccglpinnacle.com Ph. 09729327755, 09817390373


53
Days 8-12 : Simpli ication

s
se
s
la
_c
ob
{1 41 of 5 }
(2 31 ÷ 1 52 ) − 1 12 (24 ÷ 6 − 2 ) + (3 × 2 + 4 ) िकसके (c) 100
bo
ah बराबर है ? (d) 1,000
sm
+ 1 ÷ 2 31 + 2 + 1 का सरलीकृत
ur

9 7 6
yo

मान है SSC CPO 16 March 2019


e/
t.m
://

(Morning) Q41. The value of 6 51 -[4


tp

SSC CHSL 11 July


ht

2019(Morning) (a) 24 1
2
3
− { 65 − ( 53 + 10 7 )} is
− 15
(a) 37 (b) 16 6 51 -[4 21 − { 65 − ( 53 + 10
3
− 157 )} का
(c) 20
(b) 3
2 मान ा है ?
(d) 12
(c) 7 SSC CPO 13 March 2019
6
(d) 1 14−6×2 is equal to ?
(Evening)
Q37. 15÷3+3 (a) 2.5
14−6×2 का मान ा होगा?
Q33. The value of 15÷3+3 (b) 1.8
18.43×18.43−6.57×6.57 SSC CPO 16 March 2019 (c) 2.1
11.86 is:
(Morning) (d) 2.8
18.43×18.43−6.57×6.57 का सरलीकृत
11.86 (a) 6 52
मान है | 1 17 + [3 1 5 4 1
(b) Q42. 30 5 − { 6 − (3 5 ÷ 9 2 )}]

e
4
SSC CHSL 11 July 2019 4
(c) is equal to:
(Morning) 5
17 1 5 4 1
(a) 23.62 (d) 2 30 + [3 5 − { 6 − (3 5 ÷ 9 2 )}]

(b) 25 िकसके बराबर है ?


Q38. 5 + [ 5 2 32 − {3 43 (3 54 ÷ ]
9 21 )} SSC CPO 13 March 2019

l
(c) 26 6
(d) 24.12 is equal to: (Evening)

is:
2
3
3

÷
14

{ 73 −
ac
Q34. The simplified value of
2 2

of

1

14
5
1
3 ÷ { 7 of 5 × 1 3 − (3 2 − 2 6 )}

× 1 32 (3 21 2 61 )}
5 65 + [2 32 − {3 43 (3 54 ÷ 9 21 )}]
िकसके बराबर है ?
SSC CPO 12 March 2019
(Evening)
(a) 44
7
(a) 53
(b)
(c)
(d)
1
5
11
3
10
3

का सरलीकृत मान है (b) 7 Q43. 675×675×675+325×325×325


67.5×67.5+32.5×32.5−67.5×32.5 is
SSC CHSL 11 July 2019 (c) 43
6 equal to:
nn
(Afternoon) 22
(d) 3
675×675×675+325×325×325
67.5×67.5+32.5×32.5−67.5×32.5 िकसके
(a) 31 बराबर है ?
(b) 1 Q39. The value of SSC CPO 12 March 2019
(c) 2 3 51 -[2 21 − { 65 − ( 52 + 10
3 4 )}] is:
− 15 (Morning)
(d) 32 3 51 − [2 21 − { 65 − ( 52 + 103
− 154
)}] (a) 100
का मान है : (b) 10,000
Q35. The simplified value of SSC CPO 12 March 2019 (c) 1,000
20-[2.8 (Evening) (d) 1,00,000
Pi

× 5 ÷ 0.7 − 3 ÷ 0.9 × 1.5 + 2] is (a) 56


equal to: (b) 9 Q44. 9 43 ÷ [2 61 +{4 31 - (2 21 + 43
10
20-[2.8 11 )}] is equal to:
(c)
× 5 ÷ 0.7 − 3 ÷ 0.9 × 1.5 + 2] का 10
13 9 43 ÷ [2 61 +{4 31 - (2 21 + 43 )}]
(d)
सरलीकृत मान है 5
िकसके बराबर है ?
SSC CHSL 11 July SSC CPO 12 March 2019
Q40. 63.5×63.5×63.5+36.5×36.5×36.5 is
2019(Evening) 6.35×6.35+3.65×3.65−6.35×3.65 (Morning)
(a) 3 equal to (a) 3
(b) 3.4
63.5×63.5×63.5+36.5×36.5×36.5
6.35×6.35+3.65×3.65−6.35×3.65 िकसके (b) 15
4
(c) 3.8 बराबर है ? (c) 4
(d) 3.6 SSC CPO 13 March 2019 (d) 17
4
(Evening)
Q36. (24 ÷ 6 − 2 ) + (3 × 2 + 4 ) is (a) 10,000
equal to- (b) 1,00,000

www.ssccglpinnacle.comsupport@ssccglpinnacle.com Ph. 09729327755, 09817390373


54
Days 8-12 : Simpli ication

s
sse
la
_c
ob
Q45. 4 54 ÷ 3 of 7 + 4 × 3 - 51 3 × 3-[6-{12+15 ÷ (7-2)}] का मान (b)0.36
bo
7 5 10
ah िकसके बराबर है ? (c)0.33
sm
is equal to:
ur
yo

4 54 ÷ 73 of 7 + 4 × 3 - 51 का मान SSC CPO 14 March 2019 (d)0.45


e/
t.m

5 10
://

िकसके बराबर है ? (Morning)


tp
ht

SSC CPO 12 March 2019 (a)-15 Q54. (-4) × (− 8) ÷ (− 2) + 3 × 5 is


(Morning) (b)18 equal to:
(a) 57 (c)0 (-4) × (− 8) ÷ (− 2) + 3 × 5 का मान
(b) 8 (d)15 िकसके बराबर है ?
5
34 SSC CPO 15 March 2019
(c) 25 1 5 −5
Q50. The value of 3 ÷ 6 × 8 is (Morning)
(d) 41
25 equal to: (a)-1
1 5
3 ÷ 6 ×
−5
8 का मान िकसके (b)1
Q46. 6.75×6.75×6.75−4.25×4.25×4.25 is
67.5×67.5+42.5×42.5+67.5×42.5 बराबर है ? (c) 31
equal to : SSC CPO 14 March 2019 (d)-31
6.75×6.75×6.75−4.25×4.25×4.25
67.5×67.5+42.5×42.5+67.5×42.5 िकसके (Morning)

e
बराबर है ? (a)1 Q55. 43 + 25 [ 41 × ( 58 − 34 ) is equal
SSC CPO 13 March 2019 (b) 41 to: 43 + 25 [ 41 × ( 58 − 34 ) का मान ात
(Morning) (c) −41 कर |
(a) 2.5 SSC CPO 15 March 2019
(d)0

l
(b) 0.25 (Morning)
(c) 0.0025 ac 13
(a) 24
Q51. 3 × 7+4-6 ÷ 3-7+45 ÷ 5 × 4 +
(d) 0.025 (b) 43
49 is equal to
3 × 7 + 4 - 6 ÷ 3 - 7 + 45 ÷ 5 × 4 + (c) 41
Q47. 5 51 -[3 21 -{ 65 -( 53 + 10
1 4
− 15
49 िकसके बराबर है ? 11
(d) 12
)}]is equal to:
4 )}] का
SSC CPO 16 March 2019
5 51 -[3 21 -{ 65 -( 53 + 10
1
− 15 (Evening) Q56. 15- {5+24 ÷ (3 × 9-15)} is
मान िकसके बराबर है ? (a)99 equal to:
SSC CPO 13 March 2019 (b)101 15- { 5+24 ÷ (3 × 9-15)} का मान
nn
(Morning) (c)103
21 ात कर |
(a) 10 (d)67
7
SSC CPO 16 March 2019
(b) 5 (Afternoon)
(c) 7 Q52. (8+4-2) × (17-12) × 10 -
3 (a) -2
(d 8 89 is equal to: (b) 11 31
3
(8+4-2) × (17-12) × 10 - 89 का
(c) 6 41
Q48. The value of मान िकसके बराबर है ?
(d) 8
3 65 +[3 32 -{ 15 4 1 SSC CPO 16 March 2019
4 (5 5 ÷ 14 2 )}] is
Pi

(Evening)
equal to: Q57. (-4) × (1020 ÷ 85 × 3 -
(a)413
4 (5 5 ÷ 14 2 )}] का
3 65 +[3 32 -{ 15 4 1
22) is equal to:
(b)411
मान िकसके बराबर है ? (-4) × (1020 ÷ 85 × 3 - 22)
(c)4120
SSC CPO 13 March 2019
(d)4150 िकसके बराबर है ?
(Morning) SSC CPO 16 March 2019
(a) 37
6 0.72×0.72×0.72−0.39×0.39×0.39 (Afternoon)
Q53. 0.72×0.72+0.72×0.39+0.39×0.39 is
(b) 35 (a)-402
6 equal to:
(c) 6 (b)-56
0.72×0.72×0.72−0.39×0.39×0.39
0.72×0.72+0.72×0.39+0.39×0.39 का मान
(d) 19 (c)912
3 िकसके बराबर है ? (d)72
SSC CPO 16 March 2019
Q49. The value of 3 × (Evening) Q58.Find the value of 3 × 4[7-{
3-[6-{12+15 ÷ (7-2)}] is equal to: (a)0.39
5
2 × (13+2)}]
5

www.ssccglpinnacle.comsupport@ssccglpinnacle.com Ph. 09729327755, 09817390373


55
Days 8-12 : Simpli ication

s
se
s
la
_c
ob
3 × 4[7-{ 52 × (13+2)}] का मान (b) 17.5 3 ÷ 9 +2− 4 + 1 of 12 × 25 ÷ 5

bo
5 7 21 3 2 5 18 9
ah (c) 12.5
sm
ा है ? ?
ur
yo

SSC CPO 14 March 2019 (d) 24.5 SSC MTS 2 August 2019
e/
t.m
://

(Evening) (Evening)
tp
ht

(a) 1 Q63. What is the value of / मान (a) 2


(b) 51 ात कर : (b) 173
72÷9+3−6−(2×3)+5 of 3−(1+5×2−2)
(c) 2 52 8÷4+2−(6×8÷2)+(7×4−2×2) ? (c) 14
3
(d) 0 SSC MTS 2 August 2019 (d) 4
(Morning)
Q59. Find the value of 7-{4 × 3 (a) 11
4 Q68. What is the value of :
-(-10) × 8 ÷ (− 4) } (b) 5
4
90 × 3 ÷ 9 + 4 ÷ 2 × 3 of 4 × 8 ÷ (18 × 2
7-{4 × 3 -(-10) × 8 ÷ (− 4) } का (c) 0 ?
मान ा है ? (d) 154
90 × 3 ÷ 9 + 4 ÷ 2 × 3 of 4 × 8 ÷ (18 × 2
SSC CPO 14 March 2019 − 4) का मान ात कर |
(Evening) Q64. What is the value of / मान SSC MTS 5 August 2019

e
(a) -1 ात कर: (Morning)
(b) 0 7 ÷ 2 − [3 of 7 ÷ 4 ÷ {(2 ÷ 5) × (25 ÷ 8) (a) 48
(c) 53 ÷ (5 ÷ 2)}] (b) 40
(d) 15 (c) 36

l
SSC MTS 2 August 2019
(Afternoon) (d) 42

equal to:

ा होगा ?
ac
Q60. 10-{17-12 ÷ (5+9 × 2-17)} is

10-{17-12 ÷ (5+9 × 2-17)} का मान

SSC CPO 15 March 2019


(a) -8
(b) -7
(c) -1
(d) -9
Q69. If A= 40 ÷ 8 + 5 × 2 − 4 + 5
of 3 and
24 ÷ 4(4 + 2) + 19 of 2 ,
what is the value of A-B?
B=
then

(Evening) Q 65. Find the value / मान ात यिद A =


(a)-5 कर : 40 ÷ 8 + 5 × 2 − 4 + 5 of 3 तथा B
(b)5 3 of ( 1 ÷ 1 ) + (2 2) × 3 + 2
−5 = 24 ÷ 4(4 + 2) + 19 of 2 है , तो
nn
4 3 2 2 3
(c)7 A-B का मान ा होगा ?
SSC MTS 2 August 2019
(d)-7 SSC MTS 5 August 2019
(Afternoon)
(a) 107 (Morning)
30
Q61. 13 ÷ {4 of 2 - 3 + 4 × (6-4)} 101 (a) -11
(b) 6
is equal to: (b) 11
(c) 109
13 ÷ {4 of 2 - 3 + 4 × (6-4)} का 17 (c) 13
(d) 103
मान ा होगा ? 25 (d) -13
SSC CPO 15 March 2019
Pi

(Evening) Q 66. What is the value of / मान Q70. Find the value / मान ात कर
2 9 1 5
1
(a) -2 13 3 of 4 + 2 ÷ 4
ात कर : 1 1
1− 3 + 4 ×(1+ 13 )
? :
(b)0 36 ÷ 8 × 4 + 2 ÷ 4 − 1 + 5 of 3 ÷ (4 × 2
SSC MTS 2 August 2019
(c)1.3 (Evening) − 3) − 3
(d)1 7
(a) 10 SSC MTS 5 August 2019
9 (Afternoon)
(b) 19
Q 62.Find is the value /मान ात (a) 18
19
(c) 10
कर : (b) 16
7
(d) 11
32 ÷ 4 of 2 × 3 + [5 of 6 − {7 of 8(10 + 6 of (c) 35
2
5
6 ÷ 5 − 1) ÷ 80}] − 7 × 3 ÷ 2 (d) 31
2
SSC MTS 2 August 2019 Q67. Find the value / मान ात कर
(Morning) : Q71. If A= 7 × 3 ÷ (2 + 4) + 4 − 2 ,
(a) 7.5 B= 3 ÷ 6 × 4 + 2 − 2 of 3

www.ssccglpinnacle.comsupport@ssccglpinnacle.com Ph. 09729327755, 09817390373


56
Days 8-12 : Simpli ication

s
se
s
la
_c
ob
and C= 6 ÷ 2 + 4 × 3 − 2 . Then SSC MTS 6 August 2019 3 ÷ ( 21 + 1 2 4 ÷ ( 31 − 11
16 ) + of 81 )
bo
4 3 9
ah (Morning)
sm
the value of (A+B-C) is ? + 1 × 2 का मान ा होगा ?
ur

4 3
yo

यिद A = 7 × 3 ÷ (2 + 4) + 4 − 2 , (a) 139


e/

SSC MTS 6 August 2019


t.m

3
://

B= 3 ÷ 6 × 4 + 2 − 2 of 3 (b) 156
tp

(Evening)
ht

5
और C= 6 ÷ 2 + 4 × 3 − 2 है , तो (c) 121 (a) 3
7
(A+B+C ) का मान ा होगा ? (d) 56 (b) 1
3
SSC MTS 5 August 2019 (c) 2
(Afternoon) (d) 4
Q76. What is the value of:
(a) −316
(3 × 4 of 12 ÷ 2) ÷ 9 × 4 + 4 ÷ 8 + 3 × 2
(b) 19
2 (3 × 4 of 12 ÷ 2) ÷ 9 × 4 + 4 ÷ 8 + 3 × 2 Q80. Find the value/ मान ात कर
(c) −19 :
2 का मान ा होगा ? 2 2 2
16 (0.7) ÷0.14+(0.6) ÷0.18+(0.5) ÷0.05
(d) 3 SSC MTS 6 August 2019 { 4(2.5 of 4−13×0.25×3) } ?
(Afternoon) SSC MTS 7 August 2019
Q72. What is the value of : (a) 89
3 (Morning)
3 of 24÷8×3+4÷2−4×5 37 (a) 25
/ (b)

e
36÷12×4÷2+5×(6−4) 2 2
94 (b) 19
3 of 24÷8×3+4÷2−4×5
का मान ात कर| (c) 3 2
36÷12×4÷2+5×(6−4)
77 (c) 23
SSC MTS 5 August 2019 (d) 2 2
(d) 21
(Evening) 2

l
8
(a) 15 Q77. If A= 8 ÷ 4 × (3 − 1) + 6 × 3
(b) 9 ac ÷ 2 of 3 and B= Q81.Find the value/ मान ात कर :
16
4 ÷ 8 × 2 + 7 × 3 , then the value (1− 14 )+( 12 of 12 )÷ 25
(c) 3 2 1 3 8 ?
10 5 ÷ 4 + 2 (2− 5 )
3 of A+B ?
(d) SSC MTS 7 August 2019
4 यिद A =
(Morning)
8 ÷ 4 × (3 − 1) + 6 × 3 ÷ 2 of 3
Q73. Find the value/ मान ात कर (a) 32
3 9 4 2 27
और B= 4 ÷ 8 × 2 + 7 × 3 है , तो
÷ + × of 16 5
: 1
4 32 3 3
8 4 1 1 A+B का मान ा होगा ? (b) 8
2
×( 3 −2)÷ 9 +( 3 + 6 )
(c) 4
SSC MTS 5 August 2019 SSC MTS 6 August 2019 5
nn
(d) 5
(Evening) (Afternoon) 11

(a) 13 (a) 29
2
(b) 10 (b) 31 Q82. Find the value/ मान ात कर
3
25
(c) 33 :
(c) 2 (d) 35 3 of 5 1 ÷ 128 + 2 × 7
(1 + 43 ) × 21 3 49 3 11
(d) 31
2 121 15
× 49÷ ( 14 − 72 ) ?
Q78. What is the value of
SSC MTS 7 August 2019
Q74.Find the value/ मान ात कर (6 of 4 ÷
(Afternoon)
Pi

: 239÷26+22÷11×2+4×3
of 5−3(7+10÷2−3×3) 16 × 48) ÷ 8 × 4 + 2 × 3 ÷ 6 + 5(6 − 2) 69
(a) 25
SSC MTS 6 August 2019 ? 62
(b) 29
(Morning) (6 of 4 ÷ 16 × 48) ÷ 8 × 4 + 2 × 3 ÷ 6 + 57
(c)
61
(a) 2 5(6 − 2) का मान ा होगा ? 41
(d) 59
(b) 49
2
SSC MTS 6 August 2019 32

(c) 39 (Evening)
2
(a) 63 Q83. If A= 2 ÷ 3 × 4 , B=
(d) 35
2
(b) 79 3 of 4 + (7 − 2) and C= 4 + 5 − 6 ,
(c) 67 then what is the value of A+B+C?
Q75. What is the value of:
(d) 57 यिद A = 2 ÷ 3 × 4 , B=
(24 + 16 × 5 − 8 of 4) ÷ 84 × 48 ÷ 24 × 6 + 4
Q79. What is the value of 3 of 4 + (7 − 2) और C= 4 + 5 − 6
+3 3 ÷ ( 1 + 1 ) + 2 of 4 ÷ ( 1 11
3 − 81 ) है , तो A+B+C का मान ा होगा ?
(24 + 16 × 5 − 8 of 4) ÷ 84 × 48 ÷ 24 × 6 + 44 1 2 2 16 3 9
+ 4 × 3?/ SSC MTS 7 August 2019
+ 3 का मान ा होगा ? (Afternoon)

www.ssccglpinnacle.comsupport@ssccglpinnacle.com Ph. 09729327755, 09817390373


57
Days 8-12 : Simpli ication

s
sse
la
_c
ob
(a) 85 Q88. Find the value / मान ात कर (c) 9999
bo
3
ah 12 of 3÷6+12×2−(2×4−5) (d) 9801
sm
(b) 79 : ?
ur

3 12÷3×4+(2×4−5)
yo
e/

(c) 59
SSC MTS 8 August 2019
t.m

3
Q93. What is the value of/ मान
://
tp

(d) 68 (Afternoon)
ht

3
27 ात कर : 0.56×0.36+0.42×0.32 ?
(a) 22 0.8×0.21
SSC MTS 9 August 2019
Q84. What is the value of/ मान (b) 23
17
(49−13)×18÷9+4×12÷6+5 (Morning)
ात कर : ? (c) 27
19
98÷14+7×4 of 6÷8+4 (a) 1
(d) 21
SSC MTS 7 August 2019 9 (b) 23
(Evening) (c) 3
45
(a) 23 Q89. Find the value / मान ात कर (d) 2
(b) 37 :
18
(c) 85 5 of 5 of 5 ÷ 5 + 5 − 6 ÷ 3 × 4 + 2 + (3 ÷ Q94. Find the value of/
32
53 6 × 2) ? िन िल खत का मान ात कर :
(d) 17
SSC MTS 8 August 2019 (3576 + 4286 + 6593) ÷ (201 + 105 + 107)

e
(Afternoon) is:
Q85. What is the value of/ मान
2÷3×(1+3)+5−6 (a) 21 SSC MTS 9 August 2019
ा होगा : 2 of 3÷5×4+3−2 ? (b) 25 (Afternoon)
SSC MTS 7 August 2019 (c) 28 (a) 35

l
(Evening) (d) 19 (b) 31
36
(a) 89
ac (c) 22
(b) 31
73
Q90. Find the value / मान ात कर (d) 18
(c) 25 :
87 2 2
(d) 27 (9 ÷ 30) × 2.4 + 0.3 of 12 × (1 − 0.3) + Q95. The value of x, 45 × x =
92 2
9 × (0.3) ? 25% of 900 is:
Q86.Find the value / मान ात कर SSC MTS 8 August 2019 x का मान ात कर, जब 45 × x =
: (Evening) 25% of 900
2 of 16 ÷ 48 × 12 + 4 ÷ 8 × 16 + (7 − 2) (a) 3.43 SSC MTS 9 August 2019
nn
× 25 ÷ 15 ? (b) 3.69 (Afternoon)
SSC MTS 8 August 2019 (c) 2.79 (a) 16.2
(Morning) (d) 2.17 (b) 4
(a) 73 (c) 500
3
59 Q91. Find the value / मान ात कर (d) 5
(b) 3
49
:
(c) 3 2 of 3 ÷ 3 × 2 + {4 × 3 − (5 × 2 + 3)} 3
Q96. (x5 ÷ x4 ) ÷ x2 =?
(d) 56
3 ? SSC MTS 9 August 2019
Pi

SSC MTS 8 August 2019 (Evening)


Q87. Find the value / मान ात कर (Evening) (a) x2
: (a) 3 (b) x3
( 21 ÷ 21 × 21 + 21 − 21 + 21 × 21 ÷ 21 ) of (b) -24 (c) x−1
( 21 + 21 ) ? (c) 6
(d) x
SSC MTS 8 August 2019 (d) -21
(Morning) Q97. The value of x in the given
(a) 23 Q92. The value of
95 × 99 95 is: /
equation:
1 99 99 −
(b) 2 िदए गए समीकरण म x का मान ात
95 × 99 95 का मान है :
99 99
(c) 1 − कर |
SSC MTS 9 August 2019 2
(d) 25 23 + √x = 625
(Morning) SSC MTS 13 August 2019
(a) 9897 (Morning)
(b) 9993

www.ssccglpinnacle.comsupport@ssccglpinnacle.com Ph. 09729327755, 09817390373


58
Days 8-12 : Simpli ication

s
sse
la
_c
ob
(a) 9576 (15 + 3 × 1.1) ÷ 0.0003
bo
ah
(b) 9124 Q107. What is the value of the
sm
SSC MTS 13 August 2019
ur
yo

(c) 9216 (Evening) square root of:


e/
t.m

िन िल खत के वग मूल का मान ा
://

(d) 9028 (a) 61000


tp
ht

(b) 122000 होगा ?


Q98. What is the value of: (c) 16100 [{(100 of 0.9 × 0.8 − 7 × 1.2 ÷ 0.2 + 5 × 4
13
(2 61 + 1 18 − 61 ) × 16 ÷ 4 (d) 30500 − 3 × 2)} ÷ 10 + 1.85] ?
(2 61 + 1 1813
− 61 ) × 16 ÷ 4 का मान SSC MTS 16 August 2019
ा है ? Q103. If (Morning)
SSC MTS 13 August 2019 7 × 7 of 3 ÷ 3 − 14 × x = 7 , then (a) 12.25
(Morning) x is equal to? (b) 2.5
(a) 42 यिद 7 × 7 of 3 ÷ 3 − 14 × x = 7 है , (c) 6.25
(b) 41 72 1 तो x का मान िकसके बराबर होगा ? (d) 3.5
(c) 134 SSC MTS 14 August 2019
9
(Morning) Q108. Find the value/ मान ात
(d) 63

e
(a) 1 कर :
(b) 2 (1 x 2+3 x 4+5 x 6+7 x 8 - 9 x 10)
Q99. The value of [12 × 5 −
(c) 3 ÷ 2 of 5 ?
{200 − (501 + 247 − 386)}] ÷ 2
(d) 4 SSC MTS 16 August 2019
is:

l
(Morning)
[12 × 5 − {200 − (501 + 247 − 386)}] ÷ 2
ac Q104. What is the value of (a) 1
का मान है :
(1x2+2x3-3x4+4x5-5x6+6x7)? (b) 3
SSC MTS 13 August 2019
(1x2+2x3-3x4+4x5-5x6+6x7) का (c) 2
(Afternoon)
मान ा होगा ? (d) 4
(a) 162
SSC MTS 14 August 2019
(b) 161
(Afternoon) Q109. If (28 ÷ 4 × 7) + (44 ÷ 4 ×
(c) 111
7) - (12 × x ) = 18, then the value
(d) 82
(a) 28 of x is:
(b) 25 यिद (28 ÷ 4 × 7) + (44 ÷ 4 × 7) -
nn
Q100. The value of 5 31 × 2 71 × 9 52
(c) 20 (12 × x ) = 18 है , तो x का मान होगा
× 4 83 × 2 47
6 is:
(d) 24 :
6 का मान
5 31 × 2 71 × 9 52 × 4 83 × 2 47 SSC MTS 16 August 2019
है : Q105. The value of : 8 of 3 ÷ 6 + (Afternoon)
SSC MTS 13 August 2019 (10 + 2) × 3 − 96 ÷ 3 is: (a) 3
(Afternoon) 8 of 3 ÷ 6 + (10 + 2) × 3 − 96 ÷ 3 (b) 12
(a) 1 का मान ात कर | (c) 9
(b) 1000
Pi

SSC MTS 14 August 2019 (d) 6


(c) 100 (Afternoon)
(d) 10 (a) 7 Q110. Find the value / मान ात
(b) 9 कर:
Q101.
√ 25.60
72.90 +
√ 0.10
8.10 =? (c) 10
(d) 8
2 × 2 + 4 × 4 + 2 of 3 × 6 − 7 × (5 + 4
÷ 2)
SSC MTS 13 August 2019
(Evening) SSC MTS 16 August 2019
2
27 Q106. 12 + 16 of 3 − 20 ÷ 4 = ? (Afternoon)
(a) 30
27 SSC MTS 14 August 2019 (a) 5
(b) 20 (Evening) (b) 7
(c) 19
27 (a) 240 (c) 4
(d) 27
28 (b) 180 (d) 6
(c) 156
Q102. Compute/ गणना कर : (d) 187

www.ssccglpinnacle.comsupport@ssccglpinnacle.com Ph. 09729327755, 09817390373


59
Days 8-12 : Simpli ication

s
se
s
la
_c
ob
Q111.Find the value / मान ात (a) 33 (d) 26.25
bo
175
ah
कर:
sm
(b) 49
ur

115
yo

(0.4 of 50 × 6 ÷ 8) ÷ (12 × 10 ÷ 16) + 5 Q120.Find the value / मान ात


e/

(c) 29
t.m

175
कर
://

2 :
× 0.2 − 0.01 × 10
tp

(d) 47
ht

115 1 of 1 2 ÷ {5 1 5 3 7 1 3
SSC MTS 16 August 2019 7 5 2 − ( 32 + 5 × 1 8 ÷ 1 3 of 16 )}
(Evening) SSC MTS 20 August 2019
Q116. What is the value of :
(a) 1 (Morning)
3 ÷ 3 of 3 + 2 ÷ 4 + (4 × 2 − 2) ÷ 12 + 4 32
(b) 4 (a) 135
/
(c) 2 (b) 27
3 ÷ 3 of 3 + 2 ÷ 4 + (4 × 2 − 2) ÷ 12 + 4 32
(d) 3 (c) 27
का मान ा होगा ? 160
(d) 6
SSC MTS 19 August 2019 27
Q112. Find the value / मान ात
(Afternoon)
कर:
(a) 12
5 Q121. Find the value / मान ात
(2 of 14 ÷ 7 × 3) + (44 ÷ 11 × 8) − (12
(b) 16 कर :
× 9 ÷ 3) 3
1 × 3 ÷ 1 1 of 2 1 3 14 2
5 − [ 6 ÷ { 7 of 5 × 1 3 −

e
(c) 14 4 4 4
SSC MTS 16 August 2019 3
(d) 17 (3 21 − 2 61 )}]
(Evening) 6
(a) 9 SSC MTS 20 August 2019
(b) 6 Q117. The value of (8 ÷ 2 of 4 (Afternoon)
3 5)

l
(c) 8 2 (a) 83
÷ (8 × 3 ÷ 54 ) of (8 ÷ 2
3 × 54 ) is:
(b) 1
(d) 7 ac (8 ÷ 2 of 54 ) ÷ (8 × 2 ÷ 54 ) of (8 ÷ 2 × 54 ) 8
3 3 3 2
(c)
का मान है : 3
Q113. Find the value / मान ात (d) 1
SSC MTS 19 August 2019 3
कर:
(Evening)
(a) 21 53 Q122. Find the value / मान ात
2 − 2 ÷ 2 × 2 + 2(2 of 2 − 2 − 2 ÷ 2) 13 कर :
(b) 7 16
? 4 × 2 ÷ 4 of (4 + 4 ÷ 4 of 4) − (4 ÷ 4 of
SSC MTS 19 August 2019 (c) 1
15
(d) 64 2 × 4)
nn
(Morning)
SSC MTS 20 August 2019
(a) 4
(Afternoon)
(b) 0 Q118. The value of 165 − [135 −
(a) -1
(c) 2 {84 ÷ 4 of 3 − (16 − 18 ÷ 3)}] is:
(b) 6 21
(d) 1 165 − [135 − {84 ÷ 4 of 3 − (16 − 18 ÷
(c) −26
3)}] का मान है : 17

Q114. Find the value / मान ात SSC MTS 19 August 2019 (d) 4 41
कर: (Evening)
Q123. Find the value / मान ात
Pi

61 + 9 ÷ 1 of 4 (1 + 1 ) + 1 ×
3 43 − 122 4
2 2 3 3 2 3 (a) 81
? (b) 36 31 कर :
SSC MTS 19 August 2019 (c) 27 26 − [(2 of 6 ÷ 3) − 93 − {17 − (14 − 2)}]
(Morning) (d) 83 SSC MTS 20 August 2019
(a) 155 (Evening)
12
Q119. Find the value / मान ात (a) 100
(b) 3
कर : (b) 120
(c) 163
11
(d) 9 27 × 6 ÷ 24 + 6 ÷ 2 of 3 + 30 ÷ 24 × 18 − (c) 110
9 ÷ 54 of 3 × 216 (d) 90

Q115. Find the value / मान ात SSC MTS 20 August 2019


Q124. In the given equation, find
(1− 34 )+ 12 of 106
(Morning)
कर: 2 4
÷ +(1 − 1
) of 25 ? the value of x?
3 10 5 16 (a) -77.75
SSC MTS 19 August 2019 िदए गए समीकरण म, x का मान
(b) 18.25
(Afternoon) ात कर |
(c) -69.75

www.ssccglpinnacle.comsupport@ssccglpinnacle.com Ph. 09729327755, 09817390373


60
Days 8-12 : Simpli ication

s
se
s
la
_c
ob
2 2
(5) + (6) + (30) = (x)
2 2
Q129. Simplify / सरलीकृत कर :
bo
ah 6 81 ÷ (5 41 ÷ 73 of 21 ) − 8 × 32 ÷ 54 of 1 32 Q133. The value of
sm
SSC MTS 21 August 2019
ur
yo

(Morning) SSC MTS 21 August 2019 72 ÷ 6 of 12 + 4


e/
t.m
://

(a) 53 (Evening) × (5 − 3) of 2 ÷ 4 − 2 is:


tp
ht

(b) 37 (a) 154


72 ÷ 6 of 12 + 4 × (5 − 3) of 2 ÷ 4 − 2
(c) 41 (b) 3 का मान है :
(d) 31 (c) -3 SSC MTS 22 August 2019
(d) − 15 (Afternoon)
4
Q125. Find the value / मान ात (a) 5
3 2
कर : 56 + (4) − 3 × (3) Q130. The value of (b) 4
SSC MTS 21 August 2019 3 ÷ 21 of 3 × 7 + (c) 0
(Morning) 24 × 6 ÷ 18 − 3 ÷ 2 + 3 − 2 × 3 ÷ 6 (d) 3
(a) 93 is:
(b) 79 Q134. The value of
3 ÷ 21 of 3 × 7 + 24 × 6 ÷ 18 − 3 ÷ 2 + 3 5+2 of 3÷3 of 2×3
a
is b , where a and
(c) 76
− 2 × 3 ÷ 6 का मान है : 9+72÷3−2×(3−2)−3

e
(d) 83 SSC MTS 22 August 2019 b are prime numbers. The value
(Morning) of (b-a) is:
Q126. Find the value / मान ात 5+2 of 3÷3 of 2×3
का मान ba है ,
9+72÷3−2×(3−2)−3
कर :
√ 1
3 16 + 1
− 3
(a) 6 31 जहाँ a और b अभा सं ाएँ ह |

l
2 4
SSC MTS 21 August 2019 (b) 9 21 (b-a) का मान है :
(Afternoon)
(a) 1 43
(b) 1
(c) 1 21
(d) 1 41
ac (c)

Q131.
8 65
(d) 12 21

The
3 51 ÷ 4 21 of 5 31
value of
SSC MTS 22 August 2019
(Evening)
(a) 7
(b) 3
(c) 5
(d) 4
− 2 31 of { 73 − (1 15
4 13 ) × 1 1 } is:
− 30 5
Q127. Approximate value of 3 51 ÷ 4 21 of 5 31 − 2 31 of { 73 − (1 15
4 13
− 30 ) Q135. The value of
nn
(4488 ÷ 11.01 − 7.98) ÷ 15.99 is: × 1 5 } का मान है :
1 1 3 1
(5 4 ÷ 7 of 2 ) ×
(4488 ÷ 11.01 − 7.98) ÷ 15.99 का
SSC MTS 22 August 2019 (5 41 × 3 ÷ 21 ) ÷ (5 41 ÷ 3 × 21 ) is:
लगभग मान है : 7 7
(Morning) (5 41 ÷ 3 of 21 ) × (5 41 × 3 ÷ 21 ) ÷ (5 41 ÷ 3
SSC MTS 21 August 2019 7 7 7
(a) 1 31
(Afternoon) × 21 ) का मान है :
(b) 1 61
(a) 2.5 SSC MTS 22 August 2019
2
(c) 1 15
(b) 26 (Evening)
7
(d) 1 15
(c) 25 (a) 89
Pi

(d) 2.6 (b) 1


18
Q132. The value of (c) 18
Q128. Find the value / मान ात 1 3 ÷ { 7 of 14
2 3
5 (d) 98
कर : × 1 32 − (3 21 − 2 61 )} + 21 ÷ 23 of 21
1
7 ÷ 14 of 2 − 7 × 7 ÷ 49 + 3 of (14 ÷ 7 is: Q136. If x = 1 1 +
+ 13.14 1
12.13 14.15
+ 7) + 7 − 14 ÷ 2 1 32 ÷ { 73 of 14 2 1 1 1 3
5 × 1 3 − (3 2 − 2 6 )} + ÷ 1 , y =
…..+ 23.24 1 + 1
37.38 +
2 2
36.37
SSC MTS 21 August 2019 of 1 का मान है : 1 1 x
2
38.39 …..+ 71.72 then y is equal to
(Evening)
SSC MTS 22 August 2019
(a) 3 :
(Afternoon)
(b) 2 41 यिद x = 1
12.13
1 +
+ 13.14 1
14.15 …..+
(a) 3 61 1 1 1 1
(c) − 1 41 23.24 , y = 36.37 + 37.38 + 38.39
(b) 2 21
(d) 4 41
1
…..+ 71.72 है , तो xy का मान
(c) 1
(d) 1 32 िकसके बराबर है ?

www.ssccglpinnacle.comsupport@ssccglpinnacle.com Ph. 09729327755, 09817390373


61
Days 8-12 : Simpli ication

s
sse
la
_c
ob
SSC CHSL 10 July 2019 (2 76 of4 51 ÷ 32 ) × 1 91 ÷ ( 43 × 2 32 (d) -0.05 and -0.04
bo
ah
(Evening)
sm
of 1 ÷ 1
4) का मान है :
ur

2
yo

(a) 31 Q7. The value of


e/

SSC CGL Tier II - 11


t.m
://

1 (0.545)(0.081)(0.51)(5.2)
(b) is:
tp

September 2019
ht

24 3 3 3
(0.324) +(0.221) −(0.545)
(c) 1 (a) 5 (0.545)(0.081)(0.51)(5.2)
72 3 3 3 का मान है :
(b) 8 (0.324) +(0.221) −(0.545)
(d) 3
(c) 81 SSC CGL Tier II - 12
−5 (d) 1 September 2019
Q137. If (1.25)(1 − 6.4 × 10 ) = 5
(a) -1
1.2496 + a , then a is equal to :
(b) 1
यिद Q4. The value of
−5 3 3 (c) 3
(1.25)(1 − 6.4 × 10 ) = 1.2496 (253) + (247) k
25.3×25.3−624.91+24.7×24.7 is 50 × 10 (d) -3
+ a है , तो a का मान िकसके बराबर , where the value of k is :
है ? 3
(253) + (247)
3
Q8. The expression
25.3×25.3−624.91+24.7×24.7 का मान 50
SSC CHSL 11 July 2019
(Afternoon)
k
× 10 है , जहाँ k का मान है : √10 + 2(√6 − √15 − √10) is

e
(a) 0.0016 SSC CGL Tier II - 11 equal to :
(b) 0.00016 September 2019 ंजक
(a) 3
(c) 0.0032
(d) 0.00032 (b) 4 √10 + 2(√6 − √15 − √10)

l
(c) 2 िकसके बराबर है ?
ac (d) -3 SSC CGL Tier II - 12
SSC CGL TIER II
September 2019

Q1. The value of Q5. If ( √2 + √5 − √3 ) × k = -12, (a) √3 + √2 - √5


7 + 8 × 8 ÷ 8 of 8 + 8 ÷ 8 × 4 of 4
is: then what will be the value of k ? (b) √3 - √2 - √5
4 ÷ 4 of 4 + 4 × 4 ÷ 4 − 4 ÷ 4 of 2
7 + 8 × 8 ÷ 8 of 8 + 8 ÷ 8 × 4 of 4 यिद ( √2 + √5 − √3 ) × k = -12 है , (c) √3 - √2 + √5
4 ÷ 4 of 4 + 4 × 4 ÷ 4 − 4 ÷ 4 of 2 का मान
तो k का मान ा होगा ? (d) √2 - √3 - √5
है :
SSC CGL Tier II - 11
SSC CGL Tier II - 11
September 2019
nn
September 2019 Q9. The value of 0.5 6 -0.7 23
(a) ( √2 + √5 + √3 ) +0.3 9 × 0.7 is :
(a) 7.8
(b) 4.6 (b) ( √2 + √5 + √3 )( 2 − √10 ) 0.5 6 -0.7 23 +0.3 9 × 0.7 का मान
(c) 8.7 (c) ( √2 + √5 − √3 )( 2 + √5 ) है :
(d) 6.4 (c) ( √2 + √5 + √3 )( 2 − √5 ) SSC CGL Tier II - 12
September 2019
Q2. The value of 22. 4 + 11.5 67 - Q6. The value of (a) 0.1 54
33.5 9 is : ( 1 31 ÷ 2 76 of 5 53 ) ÷ (b) 0. 154
Pi

22. 4 + 11.5 67 - 33.5 9 का मान है : (6 52 ÷ 4 21 of 5 31 ) (c) 0. 158


SSC CGL Tier II - 11 × ( 43 × 2 32 ÷ 95 of 1 51 ) = 1 + k , (d) 0.1 58
September 2019 where k lies between:
( Q10. The value of 9 × 6 ÷ 24 +
(a) 0. 32 1 31 ÷ 2 76 of 5 53 ) ÷ (6 52 ÷ 4 21 of 5 31 ) 8 ÷ 2 of 5 − 30 ÷ 4 of 4 + 27 × 5 ÷ 9
(b) 0. 412 × ( 43 × 2 32 ÷ 5
9 of 1 51 ) का मान is :
(c) 0.3 1 1 + k है , जहाँ k का मान िकसके 9 × 6 ÷ 24 + 8 ÷ 2 of 5 − 30 ÷ 4 of 4 +
(d) 0.4 12 बीच है : 27 × 5 ÷ 9 का मान है :
SSC CGL Tier II - 12 SSC CGL Tier II - 12
Q3. The Value of (2 76 of 4 51 ÷ 32 ) September 2019 September 2019
(a) -0.07 and -0.06 (a) 647
× 1 91 ÷ ( 43 × 2 32 of 1
2 ÷ 41 ) is: 40
(b) -0.08 and -0.07 (b) 243
8
(c) -0.06 and -0.05

www.ssccglpinnacle.comsupport@ssccglpinnacle.com Ph. 09729327755, 09817390373


62
Days 8-12 : Simpli ication

s
sse
la
_c
ob
(c) 493 8
(b) 4 75 15
(b) − 19
bo
8
ah
sm
(d) 259 (c) 4 32 15
(c) 19
ur

8
yo
e/

32
(d) 4 75 (d) 5
t.m

3
://
tp

Q11. The value of


ht

√ √ Q15. A student was asked to find Q2. The value of ( 18 ÷ 2 of 1 )×


28 + 10√3 − 7 − 4√3 is 4
the value of ( 32 ÷ 3 × 85 ) ÷ ( 32 ÷ 43 of 3 ) is:
closest to: 4 1 1 1 4
4 4
9 9 ÷ 11 3 of 6 + (1 3 × 1 5 ( 18 ÷ 2 of 1 )×( 2 ÷ 3 × )÷( 5

√28 + 10√3 − √7 − 4√3 का ÷ 53 ) × 2 61 of 32 ÷ 34 of 32 . His answer 2


3
3
÷ of
4
3
4
4 3
) का मान है :
4 8

मान िकसके िनकटतम है ? was 19 41 . What is the difference SSC CGL 3 March 2020
SSC CGL Tier II - 13
between his answer and the (Afternoon)
September 2019
correct answer ? (a) 10 32
(a) 7.2
िकसी छा से 9 94 ÷ 11 31 of 61 (b) 8 85
(b) 6.1
+ (1 31 × 1 54 ÷ 53 ) × 2 61 of 32 ÷ 34 of 32 (c) 16 87
(c) 6.5
(d) 5.8 का मान ात करने को कहा गया | 7
(d) 2 64

e
उसका उ र 19 41 था | उसके उ र
Q12. The value of एवं सही उ र म ा अंतर है ? Q3. The value of - 25 + 3 ÷6× 1
2 2
0.47 + 0.503 − 0.39 × 0.8 is: SSC CGL Tier II - 13
is equal to:
September 2019
0.47 + 0.503 − 0.39 × 0.8 का मान - 25 + 23 ÷ 6 × 1 का मान िकसके

l
2
(a) 7 43
है : बराबर है ?
ac (b) 6 32
SSC CGL Tier II - 13 SSC CGL 3 March 2020
September 2019 (c) 7 21 (Evening)
(a) 0.6 15 (d) 6 31 1
(a) - 12
(b) 0. 615 (b) - 89
Q16. The value of (c) - 31
(c) 0.62 5 4 4 2
(4.6) +(5.4) +(24.84)
(d) 0.6 25 (4.6)2 +(5.4)2 +24.84
is: (d) - 19
8
4 4 2
(4.6) +(5.4) +(24.84)
का मान है :
nn
(4.6)2 +(5.4)2 +24.84
Q13. The value of Q4. The value of
SSC CGL Tier II - 13


36÷42 of 6×7+ 24×6÷18+3÷(2−6)−(4+3×2)÷8
2√10 √5−2 3 21÷3 of 7
− − is: September 2019
√ √2−√7
5+ √5+2 √7−2 is:
(a) 24.42


2√10 √5−2 36÷42 of 6×7+ 24×6÷18+3÷(2−6)−(4+3×2)÷8
− −
3
का (b) 24.24 21÷3 of 7
√5+√2−√7 √5+2 √7−2
मान है :
(c) 25.42 का मान है :
(d) 25.48 SSC CGL 3 March 2020
SSC CGL Tier II - 13
September 2019 (Evening)
Pi

SSC CGL TIER I (a) 7


(a) 2 + √2 (b) 71
(b) 2 √5 Q1. If ‘+’ means ‘-’, ‘-’ means (c) 7 21
(c) √2 ‘+’, ‘ × ’ means ‘ ÷ ’ and ‘ ÷ ’
(d) 8 21
(d) √7 means ‘ × ’ then the value of
42−12 ×3+8 ÷ 2+15 is:
8×2−4+9÷3 Q5. The value of
Q14. Find the value / मान ात कर यिद ‘+’ का अथ ‘-’, ‘-’ का अथ ‘+’, 7−[4+3(2−2×2+5)−8]÷5
is:
: ‘ × ’ का अथ ‘ ÷ ’ तथा ‘ ÷ ’ का अथ ‘ 2÷2 of (4+4÷4 of 4)
7−[4+3(2−2×2+5)−8]÷5
24 × 2 ÷ 12 + 12 ÷ 6 of 2 ÷ (15 ÷ 8 × 4) × ’ है , तो 42−12 ×3+8 ÷ 2+15 का मान
2÷2 of (4+4÷4 of 4) का मान है :
8×2−4+9÷3
of (28 ÷ 7 of 5) is: होगा : SSC CGL 4 March 2020
SSC CGL Tier II - 13 SSC CGL 3 March 2020 (Morning)
September 2019 (Morning)
(a) 4 61 (a) − 35 (a) 26
(b) 25 21

www.ssccglpinnacle.comsupport@ssccglpinnacle.com Ph. 09729327755, 09817390373


63
Days 8-12 : Simpli ication

s
se
s
la
_c
ob
(c) 8 21 (d) 4 81 Q14. The value of
[54−(5÷2)×8]+13

bo
48−4÷3×8−2
ah
sm
(d) 24 is:
ur
yo

Q10. The value of -1+


e/

[54−(5÷2)×8]+13
का मान ा होगा ?
t.m

48−4÷3×8−2
://

Q6. The value of 1 1


4 ÷ 2 × 2 + 5 is:
tp
ht

5 12 ÷3 23 of 14 +(5 19 −7 78 ÷9 20
9 9
)× 11
SSC CGL 6 March 2020
1 is: -1+ 41 ÷ 1
2 × 2 + 5 का मान ा (Evening)
5÷5of 10 −10×10÷20
1
5 2 ÷3 3 of 4 +(5 19 −7 78 ÷9 20
2 1 9 9
)× 11 होगा ? 141
(a) 106
1
5÷5of 10 −10×10÷20
का मान है : SSC CGL 5 March 2020 141
(b) 127
SSC CGL 4 March 2020 (Evening) 89
(c) 106
(Afternoon) (a) 2 89
(d)
(a) 1 54 (b) 5 127

9
(b) 1 10 (c) 17
4
Q15. The value of 1512-1492 is:
(c) 3 54 (d) - 27
1512-1492 का मान है :
(d) 9 21 SSC CGL 7 March 2020
Q11. If ‘+’ means ‘-’, ‘-’ means
(Morning)
‘+’, ‘ × ’ means ‘ ÷ ’ and ‘ ÷ ’

e
Q7. The value of (a) 300
8÷[(8−3)÷{(4÷4 of 8)+4−4×4÷8}−2] means ‘ × ’ , then the value of
8×8÷4−8÷8 of 2 − 7 is: (b) 400
[(30×5)+(84×6)]÷5
8÷[(8−3)÷{(4÷4 of 8)+4−4×4÷8}−2] [ 23 ÷18]−[4÷2]
is: (c) 22
8×8÷4−8÷8 of 2 − 7 का
यिद + का अथ ‘-’ है , ‘-’ का अथ ‘+’ (d) 600
मान है :

l
है , ‘x’ का अथ ‘ ÷ ’ है तथा ‘ ÷ ’ का
SSC CGL 4 March 2020 [(30×5)+(84×6)]÷5 Q16. The value of 3-(9-3 × 8 ÷ 2)
(Evening)
ac अथ ‘x’ है , तो [ 23 ÷18]−[4÷2]
का
is:
(a) 17
8 मान ा होगा ? 3-(9-3 × 8 ÷ 2) का मान ा होगा ?
(b) 8
3
SSC CGL 6 March 2020 SSC CGL 7 March 2020
(c) 16 (Morning) (Morning)
170
2 (a) 1
(d) 17
(b) -2 (a) -21
(c) 2 (b) 21
Q8. The value of 2
(d) -1
nn
3 23 ÷ 30
11
of 23 − 14 of 2 12 ÷ 35 ×4 45 (c) 0
2 1 3 3 1 1 is:
5 of 7 2 ÷ 4 − 4 ×1 2 ÷2 4 (d) 6
3 3 ÷ 30 of 3 − 4 of 2 2 ÷ 5 ×4 45
2 11 2 1 1 3
Q12. Solve the following/ हल
का मान है :
कीिजए :
2 1 3 3 1 1
5 of 7 2 ÷ 4 − 4 ×1 2 ÷2 4
Q17. The value of 1 81 ÷ (4
SSC CGL 5 March 2020 4 1
3 ÷ 6 ×2−1= ? 1 3 1 2 1 4 2
(Morning) 4 ÷ 5 of 8 2 )- 5 × 1 3 ÷ 5 of 1 3 +
SSC CGL 6 March 2020 11
(a) 2 76 (Afternoon) 20 is:
(b) 3 74 (a) 8 1 81 ÷ (4 41 ÷ 53 of 8 21 )- 52 × 1 31 ÷
5 of 1 3 + 20 का मान होगा :
4 2 11
(c) 2 92 (b) -2
Pi

(d) 10 (c) 3 SSC CGL 7 March 2020


21
(d) 15 (Afternoon)
Q9. The value of (a) 3 21
3 7 1 3 1 1 1 Q13. Solve the following/ हल (b) 1 21
5 × 1 8 ÷ 1 3 of 16 − (3 5 ÷ 4 2 of 5 3 )
कीिजए : (c) 1 41
× 2 21 + 21 + 81 ÷ 41 is:
113 × 87 = ?
3
5 × 1 87 ÷ 1 31 of 3
16 − (3 51 ÷ 4 21 of 5 31 ) (d) 3 81
SSC CGL 6 March 2020
× 2 21 + 1
2 + 1 1
8 ÷ 4 का मान है : (Afternoon)
SSC CGL 5 March 2020 Q18. Solve the following
(a) 10000
(Afternoon) expression/ िन िल खत ंजक को
(b) 10169
(a) 4 31 हल कीिजए :
(c) 10026
5.6-{2+0.6 of (2.1-2.6 × 1.12)}
(b) 5 65 (d) 9831
SSC CGL 7 March 2020
(c) 5 61
(Evening)

www.ssccglpinnacle.comsupport@ssccglpinnacle.com Ph. 09729327755, 09817390373


64
Days 8-12 : Simpli ication

s
sse
la
_c
ob
(a) 4.0871 Q1. What is the value of the (d)4 × 30 - 5 ÷ 15 + 14 = 21
bo
ah
(b) 4.0872 following?
sm
ur
yo

(c) 7.7112 िन िल खत का मान ा है ? Q5. Evaluate/ हल कीिजए:


e/
t.m

45 - 5 of (6.3 ÷ 9) +7 × 0.5
://

(d) 7.7113 -15 + 90


tp
ht

÷ [89 − {9 × 8 + (33 − 3 × 7)}] CHSL 13-10-2020 (Afternoon


Q19. The value of 1800 ÷ 20 × CHSL 12-10-2020 (morning Shift)
{(12-6)+(24-12)} is: shift) (a) 40
(b) 45
1800 ÷ 20 × {(12-6)+(24-12)} का (a) 3
(c) 50
मान ा होगा ? (b) 2
(d) 42
SSC CGL 9 March 2020 (c) 4
(Morning) (d) 5 Q6.The value of (72 + 34) ÷ 2 +
(a) 2720 [{75 ÷ 15) + 6} × 2] is:
(b) 1720 Q2. Find the value of 2.1 + 2.25 (72 + 34) ÷ 2 + [{75 ÷ 15) + 6} ×
(c) 840 ÷ [63 − {7.5 × 8 + (13 − 2.5 × 5)}]. 2] का मान ा होगा ?
(d) 1620 2.1 + 2.25 CHSL 13-10-2020 (Evening

e
÷ [63 − {7.5 × 8 + (13 − 2.5 × 5)}] Shift)
Q20. Solve the following का मान ात कीिजए। (a)74
expression/ िन िल खत ंजक को CHSL 12-10-2020 (Afternoon (b)75
हल कीिजए shift) (c)86

l
11+11 × 11-11 ÷ 11 (a) 2.8 (d)78
SSC CGL 9 March 2020
ac (b) 2.9
(Afternoon) (c) 3.0 Q7. If '+' means ' ÷ ′ ; ' × ′ means
(a) 121 (d) 3.1 '+'; ' ÷ ′ means '-' and '-' means '
(b) 22 × ′ . Then what will be the value
(c) 11
of the following expression? / यिद
(d) 131 Q3. The value of
‘+’ का अथ ‘ ÷ ’, ‘ ÷ ’ का अथ ‘-’, ‘
[0.9-{2.3-3.2-(7.1 - 5.4 - 3.5)}] is:
× ’ का अथ ‘+’ और ‘-’ का अथ ‘ ×
Q21. The value of 515 × 485 is: [0.9-{2.3-3.2-(7.1 - 5.4 - 3.5)}]
’ है ; तो िन ल खत ंजक का ा
515 × 485 का मान है : का मान है :
nn
िन ष होगा ?
SSC CGL 9 March 2020 CHSL 12-10-2020 (Evening
CHSL 14-10-2020 (Morning
(Evening) shift)
shift)
(a) 249775 (a) 0
18 + 3 - 5 × 6 ÷ 4
(b) 250225 (b) 1.8
(a) 30
(c) 20825 (c) 2.6
(b) 32
(d) 200825 (d) 0.18
(c) 15.5
(d) 13.5
Q4.If the following interchanges
Pi

Q22. The value of (26-13 × 2) ÷


2+1 is: are made in signs and numbers,
Q8. The value of 4 + [3{35 + (42
(26-13 × 2) ÷ 2+1 का मान है : which equation would be correct?
+ 10 ÷ 2 × 3 − 40)} + 7] is :
SSC CGL 9 March 2020 / िच ों और अंकों को बदलने पर
4 + [3{35 + (42 + 10
(Evening) िन िल खत म से कौन सा समीकरण
÷ 2 × 3 − 40)} + 7] का मान है :
(a) 0 सबसे उिचत है ?
Interchanges: CHSL 14-10-2020 (Morning
(b) 1
Signs: ÷ and - shift)
(c) 26
3 (a) 157
(d) 14 Numbers: 15 and 5
(b) 167
CHSL 13-10-2020 (Morning
(c) 185
Shift)
(d) 163
(a)4 × 30 - 15 ÷ 5 + 12 = 21
(b)12 × 30 - 15 ÷ 5 + 4 = 37
SSC CHSL 2019 (c)4 × 30 - 15 ÷ 5 + 12 = 27

www.ssccglpinnacle.comsupport@ssccglpinnacle.com Ph. 09729327755, 09817390373


65
Days 8-12 : Simpli ication

s
se
s
la
_c
ob
Q9. The value of 5 4 4 ÷ [2 1 - 1 {1 1 -( 1 - 1

bo
5 5 2 4 4 5
ah Q13. Find the value of 225 - [42 -
sm
√3 + 7√2 − √6 − √2+√233+√6 is: )}]
ur
yo

{25 - (18 - 18 + 13)}]. CHSL 19-10-2020 (Morning


e/
t.m

5 √3 + 7√2 − √6 − 23 का
://

√2+√3+√6 225 - [42 - {25 - (18 - 18 + 13)}] shift)


tp
ht

मान ा होगा ? का मान ात कीिजये ? (a) 1


CHSL 14-10-2020 (Afternoon CHSL 15-10-2020 (Evening (b) 3
shift) shift) (c) 2
(a) 0 (a) 222 (d) 4
(b) 16 (b) 221
(c) 12 (c) 223 Q.18. Find the value of 309
(d) 10 (d) 244 ÷ [( 23 ) of (25 + 35) − 12 43 ].
309 ÷ [( 23 ) of (25 + 35) − 12 43 ]
Q10. The value of [5 Q14. The value of 72-3(2+24 ÷ 4 का मान ात कीिजए।
4 13 2 4 2
9 ÷ ( 11
4 − 6
)] ÷ 3
[7 11 of 8 54 ÷ 1 75 − 3] × 3-2 × 2) + 8 is: CHSL 19-10-2020 (Afternoon
is: 72-3(2+24 ÷ 4 × 3-2 × 2) + 8 का shift)

e
[5 मान है : (a) 8
4
÷ ( 11 13 2
)] 3
÷ [7 11 of 8 54 ÷ 1 75 − 34 ]
2 CHSL 16-10-2020 (Morning (b) 16
9 4 − 6
shift) (c) 12
का मान है :
(a) 72 (d) 4

l
CHSL 14-10-2020 (Afternoon
(b) 32
shift)
1
ac (c) 36
(a) 81
(d) 24 Q.19. The value of 1
(b) 1
3 1 3 11
4 − [3 8 ÷ {6 − (2 4 − 12 )}] is:
61
(c) 1
71 Q15. The value of 10-[121 ÷ (11 1 43 − [3 81 ÷ {6 − 11 )}] का
(2 43 − 12
(d) 1
91 × 11)-(-4)-{3-(8-1)}] is: मान ा होगा
10-[121 ÷ (11 × CHSL 19-10-2020 (Evening
11)-(-4)-{3-(8-1)}] का मान है : shift)
Q11. Find the value of x in CHSL 16-10-2020 (Afternoon (a) 1
nn
√15625 - √x = 4
3
shift) (b) 2
(a) -1 (c) 3
√15625 - √x = 4 म x का मान
3

(b) 1 (d) 0
ात कीिजए।
(c) 0
CHSL 14-10-2020 (Evening
(d) 19 Q20. Evaluate: [7 + 7
shift)
(a) 625 × (7 + 7 ÷ 7 )] + 7 ÷ 7.
Q16. What is the value of [7 + 7 × (7 + 7 ÷ 7 )] + 7 ÷ 7
(b) 343
(9 + 3 - 16 सरल कीिजये
(c) 441
Pi

÷ 4 + 10) + {(3 + 5 × 2 ÷ 10)} × (18 − 4 ofCHSL


5)? 20-10-2020 (Morning
(d) 81
(9 + 3 - 16 shift)
÷ 4 + 10) + {(3 + 5 × 2 ÷ 10)} × (18 − 4 of(a)5)?
10
Q12.The value of 27 + [3(50 - 20) CHSL 16-10-2020 (Evening (b) 5
+ 168 ÷ 4 + 2 - 11 × 2] is: shift) (c) 63
27 + [3(50 - 20) + 168 ÷ 4 + 2 - (a) 15 (d) 64
(b) 10
11 × 2] का मान है :
(c) 5 Q.21. The value of 54 ÷ 3 41 of
CHSL 15-10-2020 (Morning
(d) 8 8
1 1
5− 8 1 5
shift) 13 − 1 + 1 × 5 5 + 6 is:
5 8
(a)245 1 1
5− 8
Q17. Simplify the following: 4
5 ÷ 3 41 of 8
13 − 1 1 × 5 51 + 65 का
(b)139 5+ 8
4 54 ÷ [2 51 - 21 {1 14 -( 14 - 1
5 मान ात करे
(c)149
(d)239 )}]
िन िल खत को सरल कीिजए:

www.ssccglpinnacle.comsupport@ssccglpinnacle.com Ph. 09729327755, 09817390373


66
Days 8-12 : Simpli ication

s
sse
la
_c
ob
CHSL 20-10-2020 (afternoon CHSL 26-10-2020 (Morning CHSL 18-03-2020 (Morning
bo
ah
shift) shift) shift)
sm
ur
yo

2
(a) 15 (a) 3/7 (a) 36
e/
t.m
://

(b) 7 (b) 5/12 (b) 16


tp
ht

30
(c) 1 (c) 3/8 (c) 9
15
1 (d) 5/8 (d) 4
(d) 30

Q.26. The value of Q30. If [{(


Q.22.. What is the value of −3 2 −6
[(3+5-4)+(17-3 × 4)]+[4 ÷ 2-16 ÷ 2 3 (2x+3) ] 4 = [{( 32 ) 3 }(3x + 7) ] 5
− 77 + 800 ÷ [83 − {8 × 9 + (18 − 3 × 5)}] 4+3] is: 3) }

?
[(3+5-4)+(17-3 × 4)]+[4 ÷ 2-16 ÷
then the value of √2 − 42x is:
22। यिद [{(
4+3] का मान है −3 2 −6
− 77 + 800 ÷ [83 − {8 × 9 + (18 − 3 × 5)}] CHSL 26-10-2020 (afternoon 2 3 (2x+3) ] 4 = [{( 32 ) 3 }(3x + 7) ] 5
3) }
का मान ात करे तो √2 − 42x का मान:
shift)
CHSL 20-10-2020 (Evening
(a) 16 CHSL 18-03-2020 (Morning
shift)

e
(b) 10 shift)
(a) 24
(c) 12 (a) 5
(b) 23
(d) 14 (b) 6
(c) 26
(c) 3
(d) 25

l
Q.27. The value of (d) 4
ac 3 31 − [ 49 + { 45 − 131 × (5 1
2 − 3 )}] is:
Q.23. The value of
Q31. If
3 31 − [ 49 + { 45 − 13 2 − 3 )}] का
1 × (5 1
1 1 1
5÷ 5× 5 1
1 − 4 5 ÷ 105 is:
1 1
5 ÷ 5 of 5 मू है :
4 = a + b√2 + c√3 − d√6,
1 1 1
1+√2+√3
5÷ 5× 5 CHSL 26-10-2020 (Evening
1 − 4 5 ÷ 105 का मान ात
1
1 1
5 ÷ 5 of 5
where a, b, c. d are natural
shift) numbers, then the value of a + b
करे
(a) 10 + c + d is:
CHSL 21-10-2020 (Morning
(b) 0 15। यिद
shift)
(c) 5
(a) 0 4 = a + b√2 + c√3 − d√6
nn
(d) 1 1+√2+√3
(b) 2 जहां a, b, c ,d ाकृितक सं ाएँ ह,
(c) 10 तो a+b+ c+ d का मान है :
Q.28. What will come at place of
(d) 5 (132÷12×x−3×3) CHSL 18-03-2020 (Morning
x,(x<10) for 52 6×4+x 2
= 1?
− shift)
(132÷12×x−3×3)
Q.24. The value of X (x <10) के िलए 52 −6×4+x2
= (a) 0
75 53 ÷ [15 ÷ 3 of 5 + 7 ÷ 1
14 − {78 ÷ 3 31 }]
1 के थान पर ा आएगा? (b) 2
is: CHSL 17-03-2020 (Morning (c) 4
1
75 53 ÷ [15 ÷ 3 of 5 + 7 ÷
− { 78 ÷ 3 1 }]
shift) (d) 1
Pi

14 3
का मान ात करे (a) 2
CHSL 21-10-2020 (Evening (b) 4
shift) (c) 3 Q.32. The value of
(a) 1 (d) 1 8 − [8 − (5 + 8) − {8 −
(b) 2 (8 − 5 + 8)} +10] is:
(c) 5 Q29. If x is the square of the 8- [8- 8- (5+8) - {8- (8-5+8)}+10]
(d) 0 number when ( 52 of 6 41 ÷ 73 )of 1 72 का मान है :
is divided by 11 41 , then the value CHSL 19-03-2020 (Morning
Q.25. The value of of 81x is: shift)
1 ÷ 5 of (5 1 + 2 1 ) + 2 ÷ 3 1 is:
2 36 9 10 5 5 5 10। यिद x सं ा का वग है जब (a) 5
1 ÷ 5 of (5 1 + 2 1 ) + 2 ÷ 3 1 का
2 36 (b) 20
9 10 5 5 5 ( 5 of 6 41 ÷ 73 ) 1 72 को 11 41 से
2

मान ात करे िवभािजत िकया जाता है , तो 81x का (c) 0


मान होता है : (d) 10

www.ssccglpinnacle.comsupport@ssccglpinnacle.com Ph. 09729327755, 09817390373


67
Days 8-12 : Simpli ication

s
se
s
la
_c
ob
(d) 98 (a) 47

bo
6
ah
Q.33. Simplify the expression
sm
(b) 41
ur

9
yo

25-[16- {14- [18 - 8 + 3)}] Q37. The value of


e/

(c) 35
t.m

9
://

ंजक को सरल कीिजए 25-[16- [ 74 of 2 54 × 1 32 − (3 21 − 2 61 )] ÷ (3 51 ÷ 4 21 of 5 31 )


tp

(d) 29
ht

6
{14- [18 - 8 + 3)}] is:
CHSL 19-03-2020 (afternoon
shift) [ 74 of 2 54 × 1 32 − (3 21 − 2 61 )] ÷ (3 51 ÷ 4 21 of 5 31 ) 0.0203×2.92
Q41. The value of 0.7×0.0365×2.9
(a) 14 का मान ात कीिजए ? (12.12) 2 −(8.12) 2
(b) 15 CGL MAINS Tier-II ÷ is:
(0.25) 2 +(0.25) (19.99)
(c) 17 (15-10-2020) 0.0203×2.92 (12.12) 2 −(8.12) 2
0.7×0.0365×2.9 ÷ (0.25) 2 +(0.25) (19.99)
(d) 16 (a) 10
(b) 7 21 का मान ात कीिजए
Q.34. The value of 7+[44 ÷ 4 + {9 (c) 15 CGL 2019 Tier-II (15-11-2020 )
× 2 − 14 ÷ 7 } + 5 × 2] is: (d) 1 3 1 (a) 0.05
7+[44 ÷ 4 + {9 × 2 − 14 ÷ 7 } + 5 (b) 0.5

e
× 2] का मू है : (c) 0.1
Q38. Let x = ( √ ÷√
1875 1200
) (d) 0.01
CHSL 19-03-2020 (Evening √3888 √768
× √
175
shift) . Then √x is equal to:
√1792 Q42. the value of
(a) 55

l
माना x = ( √ 1875
÷ √ 1200
)× √175
(b) 33 √3888 √768 √1792 0.57 − 0.432 + 0.35 is :
(c) 67 ac . तो √x का मान ात कीिजए 0.57 − 0.432 + 0.35 का मान
(d) 44 CGL MAINS Tier-II ा होगा :
(15-10-2020) CGL 2019 Tier-II (16-11-2020 )
SSC CGL-2019 TIER-II (a) 12 7
(a) 0.494
Q35. The value of (b) 94 (b) 0.498
(2.4 × 0.6 × 3 × 0.16) × [0.27 × (0.83 ÷ 0.16)] 5
(c) 12 (c) 0.4 98
is:
(d) 95 (d) 0.494
(2.4 × 0.6 × 3 × 0.16) × [0.27 × (0.83 ÷ 0.16)]
nn
का मान ात कीिजये Q39. The expression Q43.. If
CGL MAINS Tier-II 15(√10+√5)
(15-10-2020) √10+√20+√40−√5−√80
is equal to:
√11 − 3√8 = a + b√2, then
(a) 0.814 15(√10+√5)
का मान ात what is the value of (2a + 3b) ?
√10+√20+√40−√5−√80
(b) 0.11
(c) 1.1
कीिजए यिद √11 − 3√8 = a + b√2, तो
CGL MAINS Tier-II (2a + 3b) का मान ा होगा
(d) 1.36 (15-10-2020) CGL 2019 Tier-II (16-11-2020 )
Pi

(a) 5 + 2√2 (a) 5


Q36. If 1
+
3+2√2 3−2√2 (b) 7
4−√8 3−2√2
− 3+2√2 (b) 5 − 2√5
(c) 9
= a + b√2 , then what is the (c) 5(3 + 2√2) (d) 3
value of (3a+4b) ? (d) 10(3 + 2√5)
यिद 1 + √ − − √
3+2 2 3 2 2 Q44.. The value of
4−√8 3−2√2 3+2√2
Q40. The value of 3 51 ÷ 4 21 of 5 31 + 81 ÷ 1
2 of 1
4 − 1
4
= a + b√2 , तो (3a+4b) का मान 3 ÷ 18 of 3 × 6 + 21 × 6 ÷ 18 − 3 ( 21 ÷ 1
8 × 41 ) is:
ात कीिजए ? ÷ 2 + 3 − 3 ÷ 9 of 3 × 9 is: 3 51 ÷ 4 21 of 5 31 + 1
8 ÷ 1
2 of 1

4
1
4
CGL MAINS Tier-II 3 ÷ 18 of 3 × 6 + 21 × 6 ÷ 18 − 3
(15-10-2020) ( 21 ÷ 1
8 × 41 ) का मान ा होगा
÷ 2 + 3 − 3 ÷ 9 of 3 × 9 का मान
(a) 99 21 CGL 2019 Tier-II (16-11-2020 )
ात कीिजए 53
(b) 97 (a) 60
CGL MAINS Tier-II 13
(c) 98 21 (b)
(15-10-2020) 15

www.ssccglpinnacle.comsupport@ssccglpinnacle.com Ph. 09729327755, 09817390373


68
Days 8-12 : Simpli ication

s
se
s
la
_c
ob
(c) 7 (c) 4
bo
8
ah (d) 1 Q53. The value of
sm
(d) 3
ur

4
yo

(5 41 ÷ 73 of 21 ) ÷ (5 91 − 7 87 ÷ 9 20
9 )× 11

e/
t.m

21
://

8+2√15 (5 ÷ 2of 21 ) is:


tp

Q45.. The value of Q.49. The value of 5 - -


ht

4
4 ÷ 12 of [3 ÷ 4 of {(4 − 2) × 6 ÷ 2}] − 2 × 6 ÷ 8 +is3equal to:
1 (5 41 ÷ 73 of 21 ) ÷ (5 91 − 7 87 ÷ 9 20
9 )× 11
21 −
8+2√15
is: 8+2√15 (5 ÷ 2of 21 ) का मान ात करे
4 ÷ 12 of [3 ÷ 4 of {(4 − 2) × 6 ÷ 2}] − 2 ×56-÷ 8 4+ 3 - 8+2√15 का मान ात
1
CPO 23-11-2020 (Evening
का मान ा होगा ? करे shift)
CGL 2019 Tier-II (16-11-2020 ) CGL 2019 Tier-II (18-11-2020 ) 35
(a) 24
1
(a) 4 6 (a) 32 (b) 0
(b) 7 61 (b) 1 15
(c) 28
(c) 2 31 (c) 21 (d) -2
1 (d) 1
(d) 3 3 4
Q54. The value of
3 31 ÷ 2 21 of 1 53 + ( 83 + 1 × 1 43 ) is:

e
7+3√5 7−3√5
Q46. The value of − 7
3+√5 3−√5 Q50. Evaluate the following / मान 3 31 ÷ 2 21 of 1 53 + ( 83 + 1
7 × 1 43 ) का
lies between: ात करे
7+3√5
मान ात करे
− 3 √5 का मान िकसके
7−3 5
5-[96 ÷ 4 of 3 -(16-55 ÷ 5 )] CPO 23-11-2020 (Evening
3+√5 −√

l
बीच म होगा CGL 2019 Tier-II (18-11-2020 ) shift)
CGL 2019 Tier-II (16-11-2020 ) (a) 0
(a) 2 and 2.5
(b) 3 and 3.5
(c) 1.5 and 2
(d) 2.5 and 3
ac (b) 3
(c) 2
(d) 4

SSC CPO-2019
(a)
(b)
(c)
(d)
55
24
35
24
5
24
25
24

8+2√3 Q51.. The value of


Q47.. If = a √3 − b , then 8 − 3 ÷ 6 of 2 + (4 ÷ 4 of 41 ) ÷ 8 + Q55. The value of
3√3+5
the value of a + b is equal to: (4 × 8 ÷ 41 ) × 1 is: 2 31 ÷ 2 21 of 1 53 + ( 83 + 1 × 1 43 ) is:
nn
7
8
यिद
8+2√3
= a √3 − b , तब a+b 8 − 3 ÷ 6 of 2 + (4 ÷ 4 of 41 ) ÷ 8 + 2 31 ÷ 2 21 of 1 53 + ( 83 + 1
7 × 1 43 ) का
3√3+5
(4 × 8 ÷ 41 ) × 1 मान ात करे
का मान ा है ? 8
CPO 24-11-2020 (Morning
CGL 2019 Tier-II (18-11-2020 ) का मान ात करे |
shift)
(a) 18 CPO 23-11-2020 (Morning 35
(a) 24
(b) 15 shift)
(b) 25
(c) 16 (a) - 47 24
(c) 5
(d) 24 (b) 7
4
24
Pi

(d) 29
(c) - 97
4
24
Q.48.If x=
(d) 97


4
Q56. The value of
− √3 +
√ 3+8

where x>0, then the value of x is


√ 7 + 4√3
Q52. The value of
40− 34 of 32
37− 43 of (34 − 6 )
(5 ÷ 2of 21 ) ÷ (5 41 ÷ 73 of 21 ) ÷
9 ) × 11 is:
(5 91 − 7 87 ÷ 9 20 21
equal to: is: (5 ÷ 2of 21 ) ÷ (5 41 ÷ 73 of 21 ) ÷
40− 34 of 32
यिद x= 37− 43 of (34 − 6 )
का मान ात करे 9 ) × 11 का मान
(5 91 − 7 87 ÷ 9 20 21


ात करे |
− √3 +

तो x का मान ात करे : जहा x>0
3+8 √7 + 4√3 CPO 23-11-2020 (Morning
shift)
(a) 1
CPO 24-11-2020 (Morning
shift)
CGL 2019 Tier-II (18-11-2020 ) (b) 0 (a) -2
(a) 2 (c) - 21 (b) 8
(c) 2435
(b) 3 (d) 1
2

www.ssccglpinnacle.comsupport@ssccglpinnacle.com Ph. 09729327755, 09817390373


69
Days 8-12 : Simpli ication

s
sse
la
_c
ob
(d) 15 (c) 7

bo
28 4
ah
sm
(d) - 47
ur
yo

Q57. The value of


e/
t.m
://

2 3 4 4 1 8 3 3 1
3 ÷ 10 of 9 − 5 × 1 9 ÷ − + ÷
tp

Q61. The value of


ht

15 4 4 2
is: 2 3 4 4 1 8 3 1
3 ÷ 10 of 9 − 5 × 1 9 ÷ 15 + 4 ÷ 2
2 3 4
3 ÷ − 54 × 1 91 ÷ 15
10 of 9
8
− 43 + 3
4 ÷ 1
2 is:
का मान ात करे 2
3 ÷ 3 4
− 54 × 1 91 ÷ 15
10 of 9
8 + 3 ÷
4
1
2
CPO 24-11-2020 (Evening का मान ात करे
shift) CPO 25-11-2020 (Evening
49
(a) 12 shift)
(b) 25 49
(a) 12
6
(c) 17 (b) 17
9 9
(d) 14 (c) 29
3 6
(d) 14
3

e
Q58. The value of Q62. The value of
− 7 ÷ [5 + 1 ÷ 2 − {4 + 7 ÷ [5 + 1 ÷ 2 − {4 + (4 of 2 ÷ 4) +
(4of 2 ÷ 4) + (4 ÷ 4of 2)}] is: (5 ÷ 5 of 2)}] is:

l
− 7 ÷ [5 + 1 ÷ 2 − {4 + 7 ÷ [5 + 1 ÷ 2 − {4 + (4 of 2 ÷ 4) +
(4of 2 ÷ 4) + (4 ÷ 4of 2)}] का मान
ac (5 ÷ 5 of 2)}] का मान ात करे |
ात करे | CPO 25-11-2020 (Evening
CPO 24-11-2020 (Evening shift)
shift) (a) 7
(a) 7 (b) 27
(b) 27 (c) - 27
(c) - 27 (d) -7
(d) -7
nn
40+ 34 of 32
Q59. The value of 37+ 34 of (34−6)
is
3
40+ of 32
4
37+ 34 of (34−6)
का मान ात करे |
CPO 25-11-2020 (Morning
shift)
3
(a) 1 29
3
(b) 2 29
Pi

3
(c) -1 29
9
(d) 1 29

Q60. The value of


1 − 3 ÷ 6 of 2 + (4 ÷ 4 of 41 ) ÷ 8 +
(4 × 8 ÷ 41 ) × 1
8 is:
1 − 3 ÷ 6 of 2 + (4 ÷ 4 of 41 ) ÷ 8 +
(4 × 8 ÷ 41 ) × 1
8 का मान ात करे
CPO 25-11-2020 (Morning
shift)
(a) − 69
4
(b) 69
4

www.ssccglpinnacle.comsupport@ssccglpinnacle.com Ph. 09729327755, 09817390373


70
Days 8-12 : Simpli ication

s
sse
la
_c
ob
SOLUTION: ⇒1× 3 ÷ 1 =6 15 × ( 16 19 23
5 ÷ 24)+[11- 8 ] ÷

bo
8 16 2 4
ah
sm
-1 × 25 ÷ 25
ur
yo

Variety Questions Sol 8. (c) ⇒ 15 2 69 23


e/

2 × 15 + ÷ -1
t.m

8 4
://

8 1 1
9 of (5 4 ÷ 2 3 of 4) ÷ (8
tp

⇒ 1+ -1 = 3 1 21
ht

2
Sol 1. (d) ÷ 2 of 4 2 4
3 5) of (8 × 3 ÷ 5)
2 × 3 ÷ 2 of 3 × 2 ÷ (4 + 4 × 4 ÷ 4 8 21 28 8
⇒ 9 of ( 4 ÷ 3 ) ÷ (8 ÷ 15 ) of Sol 2. (d)
of 4-4 ÷ 4 × 4 )
(8 × ) 5
6
6 - 6 ÷ 6 ⨯ 6 + (6 ÷ 6 of 6) ⨯ 6 -
⇒ 2 × 3 ÷ 6 × 2 ÷ (4+4 × 4 ÷ 16-4
8 9 20 11 of 2 ) ÷ 5
(3 32 ÷ 30
9 of 16 ÷ 15 of
⇒ 3
÷4×4) 3
1 ⇒ 6-6+(6 ÷ 36) × 6 -( 11
3 ÷
11 )÷5
2 ÷ 100
⇒ 2 × 21 × 2 × 1 = 2 ⇒ 45
1 ⇒ 1 × 6 - 15 ÷ 5
⇒ 200 6
⇒ 1-3 = -2
Sol 2. (d)
2 87 ÷ (3 65 ÷ 72 of 2 31 ) × [(2 76 of 4 51 Sol 9. (a)
4.5 - ( 3.2 ÷ 0.8 ⨯ 5 ) + 3 ⨯ 4 ÷ 6 Sol 3. (c)
÷ 32 ) × 95 ] 3 2 5
⇒ 4.5-(4 × 5)+2 4 ×2 3 ÷ 9 of 1 51 + 23
2 ×3 5 ÷
6
23 ÷ ( 23 ÷ 2
3) × [10] = 5

e
8 6 2 of 2 31
⇒ 6.5-20 7
= -13.5 ⇒ 3 × 8 ÷ 2 2 ×
+ 23 23 ÷ 2
Sol 3. (c) 4 3 3 6 3
⇒ 3 × 4+ 21
(5+3 ÷ 5 × 5) / (3 ÷ 3 of 6) of (4 × 4
Sol 10.(a) ⇒ 3 21
4 ÷ 4 of 4+4 ÷ 4 × 4)

l
(5+3÷5×5)
5 ÷ 10 of 10 × 4 + 4 ÷ 4 of 4 × 10 −
⇒ (3÷3 of 6) of (4×4÷4 of 4+4÷ 4×4
ac (10 − 4) ÷ 16 × 4
8
Sol 4. (a)
⇒ ⇒ 5 ÷ 100 × 4+4 ÷ 16 × 10- 83 × 4
1
6 of 5 3.8 - (4.2 ÷ 0.7 ⨯ 3) + 5 ⨯ 2 ÷ 0.5
48 ⇒ 1 × 4+ 25 - 23
⇒ 5 = 9 53 20 ⇒ 3.8 - (6 × 3)+5 × 4
⇒ 0.2+1 = 1.2 ⇒ 3.8-18+20
Sol 4. (b) = 5.8
1.0025+6.25×10−6 ⇒ ( 1.00250625 Sol 11. (a)
0.0025+0.95 0.9525 ) =
(3 15 − 35 ) ÷ 85 ( 13 ) ÷ 85
1.0525 ⇒ 5 Sol 5. (d)
1 7 ÷{ 67 −( 17 ÷ 15 )}
1 8 6
÷{ 7 −( 57 )}
7
2.8 + (5.2 ÷ 1.3 ⨯ 2) - 6 ⨯ 3 ÷ 8 +
nn
13
⇒ 13 13
8 6
8
5 = 8×8 = 64 2
Sol 5. (b) 7 ÷{ 7 −( 7 )}

5 ÷ 5of 5 × 2 + 2 ÷ 2of 2 × 5 − (5 − 2) ⇒ 2.8+(4 × 2)-2.25+2


÷ 6 × 2 = 5 ÷ 25 × 2 + 2 ÷ 4 × 5 − 3 ÷ Sol 12. (d) Given fraction is ⇒ 2.8 + 8 - 2.25 + 2
6×2
5.75×5.75×5.75+3.25×3.25×3.25 = 10.55
57.5×57.5+32.5×32.5−57.5×32.5
5.75×5.75×5.75+3.25×3.25×3.25
= 51 × 2 + 21 × 5 − 21 × 2 = 52 + 25 − 1 ⇒ (5.75×5.75+3.25×3.25
−5.75×3.25) × 100
4+25 19 Sol 6. (b)
= 10 −1= 10 ⇒
(5.75 + 3.25) (5.75×5.75 + 3.25×3.25 − 5.75×3.25)
7.2 + (8.4 ÷ 0.12 ⨯ 0.2) - 5 ⨯ 3 ÷
(5.75×5.75 + 3.25×3.25−5.75×3.25) × 100 0.05 + 3
Pi

Sol 6. (d) ⇒ 100 9 = 0.09 ⇒ 7.2 + (70 × 0.2)-5 × 60+3


9 2 2 of 23 ) ÷ ( 43 × 3 3
15 of ( 3 ÷ 3 4 ÷ 4 ⇒ 7.2 + 14- 300 + 3
of 34 ) of ( 45 ÷ 5
2 × 2
5 of 54 ) Sol 13. (c) = -275.8
9 2 9 4
15 of 3 ÷ of 25

⇒ 2 9
5 ÷ 100
16
√4 + √144 = √4 + 12 = √16 = 4 Sol 7. (b)
⇒ 2
5 × 9
100 = 40
9
5.8 + (7.4 ÷ 3.7 ✕ 5) - 6 ✕ 2 ÷
Practice Questions
2.5
Sol 7. (a) ⇒ 5.8 + (2 × 5) - 6 × 0.8
Sol 1. (d)
16 ÷ 4 of 4 ⨯ [ 3 ÷ 4 of { 4 ⨯ 3 ÷ ⇒ 5.8 + 10 - 4.8
7 21 × (3 51 ÷ 4 21 of 5 31 ) + [11-(
( 3 + 3 )}] ÷ (2 ÷ 4 of 8) = 11
5 + 3 − 1 41 )] ÷ 5 43 - 5 ÷ 5 ⨯ 5 of
⇒ 16 ÷ 16 × [3 ÷ 4 of {4 × 3 ÷ 6}] 8
5 ÷ 25 Sol 8. (d)
÷ (2 ÷ 32)
⇒ 1 × [3 ÷ 4 of2] ÷ 161

www.ssccglpinnacle.comsupport@ssccglpinnacle.com Ph. 09729327755, 09817390373


71
Days 8-12 : Simpli ication

s
se
s
la
_c
ob
3.8 + (8.2 ÷ 4.1 × 2) - 4 × 3 ÷ ⇒ 3 = 1 (3 15 + 35 )÷ 85 ( 19 ) ÷ 85

bo
54×5 90 ⇒ 5

ah 1 7 ÷{ 57 +( 17 ÷ 13 )}
1 8
sm ÷{ 7 +( 37 )}
5
1.2 7
ur

19
yo

⇒ 19
⇒ 3.8 + (2 × 2) - 4 × 2.5 Sol 16.(d)
8
=
e/

8 8 8
t.m

7÷ 7
://

⇒ 3.8 + 4 - 10 3 × 2 ÷ 3 of 12 - 3 ÷ 2 × (2-3) × 2 +
tp
ht

= -2.2 3 ÷ 2 of 3 Sol 24. (c)


⇒ 3 × 2 ÷ 36-3 ÷ 2 × (-1) × 2+3 ÷ 6 (3 15 + 35 )÷ 85 ( 19 ) ÷ 85
⇒ 5
1
1 8 ÷{ 58 +( 18 ÷ 13 )} 9
÷{ 8 +( 38 )}
5
Sol 9. (b) ⇒ 61 +3+ 21 8
19
7.5 + (5.4 ÷ 4.5 ⨯ 2) - 8 ⨯ 4 ÷ ⇒ 19
⇒ 22 = 3 32 9
8
= 9
6 8 ÷1
3.2
⇒ 7.5 + (1.2 × 2)- 8 × 1.25
Sol 17.(a) Sol 25. (d)
⇒ 7.5 + 2.4 - 10 = -0.1 46− 34 of 32−6
3 × 2 ÷ 3 of ⇒ 4637−2421−6 = 1
1 37− 34 of (34−6) −
2 × 3 ÷ (5 + 5 × 5 ÷ 5 of 5 − 5 ÷ 10 of 5 )
Sol 10. (c)
⇒ 3 × 2 ÷ 6 × 3 ÷ (5+5 × 5 ÷ 25-5 ÷
108 ÷ 36 ⨯ 4 + 2.5 ⨯ 4 ÷ 0.5 - 10 Sol 26. (d)
⇒ 3 × 4 + 2.5 × 8 - 10 2) 46− 34 of 32−6
⇒ 3 × 31 × 3 ÷ (5+1- 25 ) ⇒ 4611+21
−24−6 = 1

e
11+ 34 of (34−6) 2
⇒ 12 + 10 = 22
⇒3× 2 = 6
7 7

Sol 11. (d) Sol 27. (c)


3
46 + of 32 −6
21.6 ÷ 3.6 × 2 + 0.25 × 16 ÷ 4 − 6 Sol 18. (a) 3
4
⇒ 46+24−6
=2
11 + of (34 − 6) 11+21

l
⇒ 6 × 2 + 0.25 × 4 - 6
7 of 3 ) ÷
( 57 ÷ 10 4
4
9
7 ÷ 10 1 × 7 1 )
− ( 16 2 5
4

× 5
⇒ 12 + 1 - 6 = 7
ac 12 Sol 28. (a)
⇒ ( 57 ÷ 21 4 1
40 ) ÷ 9 -( 24 × 36
5 )× 5
12 2 31 of ( 53 ÷ 92 ) − (4 52 + 19 ÷ 21 )
20
Sol 12. (c) 3 × 5
⇒ 38 ÷ 94 - 10 7 27 22 19 )
3 of ( 10 )-( 5 + 10
12 ⇒
15.2 + 5.8 ÷ 2.9 × 2 - 3.5 × 2 ÷ 0.5 ⇒ 6- 81 = 47 63 63
10 - 10 = 0
8 ⇒
⇒ 15.2 + 2 × 2 - 3.5 × 4
⇒ 19.2-14 = 5.2 Sol 19. (b) Sol 29. (c)
3 × 6 ÷ 4 of 6 − 6 ÷ 2 × (4 − 6) + 4 − 2 [ 1 51 of { 73 − (1 15
4 13 ) × 5 }] ÷ ( 6
− 15
Sol 13. (c) 7 7
× 3 ÷ 6 of 31
nn
9 43 ÷ [2 61 ÷ {4 31 − (2 21 + 43 )}] ÷ 5)
⇒ 3 × 6 ÷ 24-3 × (-2)+4-2 × 3
2 ⇒ [ 56 of { 73 - 156 × 5 ]÷ 6
⇒ 39 13 13 13
4 ÷[ 6 ÷{ 3 - 4 ] 3
7 35
⇒ 4 +6+4-3 ⇒ [ 56 of { 73 - 72 ] ÷ 6
⇒ 39 ÷[ 13 ÷ 13 ] 35
4 6 12 ⇒7 3 6 6
4 ⇒ ÷ =1
⇒ 394 ÷2 35 35

= 39
8 Sol 20. (a) Sol 30. (d)
15 of 8 - 6 + [(27 - 3) ÷ 6 − 4] 1 8 1 5 3 7 2
2 of 5 ÷ {2 5 − ( 16 + 5 × 1 8 ÷ 3 )}
CHSL ⇒ 120-6+[4-4] ⇒ 54 ÷ { 11 5 3 45
5 -( 16 + 5 × 16 )
Pi

= 114 4
Sol 14. (d) ⇒ 5 ÷ { 11
5 -2}
3 3 3 4 + 25 ÷ 2 5 =4
4 ÷ 4 of 4 × 3 5 of 4 - Sol 21. (b)
( 32 + 32 of 5
6 ) 15 of 8 + 6 + [(27 - 3) ÷ 6 + 4]
Sol 31. (a)
9 × 4
⇒ 43 ÷ 16 3 + 25 ÷ 1
2 -( 32 + 95 ) ⇒ 120+6+[4+4] 0.01404 0.01404 0.01404
⇒ 612−144
⇒ 612−144
⇒ 16 +5- 11 = 134 242 +62 −144
9 9 −5
50 ⇒ 0.01404 = 3 × 10
= 9 468
Sol 22. (d)
(3 15 + 35 )÷ 85 ( 19 ) ÷ 85 Sol 32. (c)
Sol 15. (c) ⇒ 5
1
1 7 ÷{ 67 −( 17 ÷ 15 )} 8 6
÷{ 7 −( 57 )}
3÷{5−5÷(6−7)×8+9} 19
7 {1 41 of (2 31 ÷ 1 52 ) − 1 12
5 }

⇒ 19 19
4+4×4÷4 of 4 8 6
8
5 = 8×8 = 64 + 1 ÷ 2 31 + 2 + 1
7 ÷{ 7 −( 7 )} 9 7 6
⇒ 3÷{54+4×4÷16
−5÷(−1)×8+9
⇒ { 45 of ( 35 )- 17 1
12 }+ 21 + 72 + 61
⇒ 3÷{5+5×8+9}
4+1 Sol 23.(a) ⇒ 25 - 17 + 21 = 14 = 7
12 12 12 6

www.ssccglpinnacle.comsupport@ssccglpinnacle.com Ph. 09729327755, 09817390373


72
Days 8-12 : Simpli ication

s
sse
la
_c
ob
= 21 = 2.1
bo
10
ah
Sol 33. (b) Sol 51. (b) 3 × 7+4-6 ÷ 3-7+45 ÷ 5
sm
ur
yo

18.43×18.43−6.57×6.57 ⇒
Sol 42. (d) × 4 + 49 = 21+4-2-7+36+49
e/
t.m

11.86
://

(18.43+6.57)(18.43−6.57) 17 1 5 4 1 17 = 101
30 + [3 5 − { 6 − (3 5 ÷ 9 2 )}] =
tp
ht

11.86 30
(25)(11.86)
⇒ 11.86 = 25 + [ 16 5 19 19
5 − { 6 − ( 5 ÷ 2 )}]
17 16 5 Sol 52. (b)
= 30 + [ 5 − { 6 − 52 }] = 17
30 +
(8+4-2) × (17-12) × 10 - 89 ⇒
Sol 34. (b) [ 16 13
5 − 30 ]
2 3 14 2 1 1 17 + 83 = 100 10
10 × 5 × 10-89 = 411
3 ÷ { 7 of 5 × 1 3 − (3 2 − 2 6 )} = 30 30 30 = 3
⇒ 32 ÷ { 56 × 35 − ( 27 − 13
6 )} Sol 53. (c)
⇒ 2 ÷ { 2 − 68 } =1 Sol 43. (d) We know that,
3
675×675×675+325×325×325 3 2
67.5×67.5+32.5×32.5−67.5×32.5 a3 − b = (a-b)( a2 + b + ab )
(675+325)(675×675+325×325−675×325)
Sol 35. (a) = 67.5×67.5+32.5×32.5−67.5×32.5 0.72×0.72×0.72−0.39×0.39×0.39 ⇒
0.72×0.72+0.72×0.39+0.39×0.39
20-[2.8 = (0.72−0.39)(0.722 +0.392 +0.72×0.39)
=
× 5 ÷ 0.7 − 3 ÷ 0.9 × 1.5 + 2] (675+325)(675×675+325×325−675×325)×100 (0.722 +0.392 +0.72×0.39)

e
(675×675+325×325−675×325)
⇒ 20-[20-5+2] 0.33
=1,00,000
=3
Sol 54. (a)
Sol 44. (a) 9 43 ÷ [2 61 +{4 31 - (2 21
CPO (-4) × (− 8) ÷ (− 2) + 3 × 5 ⇒ (-4)
+ 43 )}] = 39 ÷ [ 13 +{ 13 - 13 }]

l
4 6 3 4 × (4) + 3 × 5 = -1
= 39 ÷[ 13 + 13 ] = 39 ÷ 39 = 3
Sol 36.(d) ac 4 6 12 4 12
(24 ÷ 6 − 2 )+(3 × 2 + 4 ) Sol 55. (d)
Sol 45. (d) 3 5 1 8 4 ⇒
⇒ (4-2) + (6+4) 4 + 2[4 × (5 − 3)
= 12 4 54 ÷ 73 of 7 + 4 × 3 - 51 3 5 1 4
5 10 4 + 2 [ 4 × ( 15 )
= 24
5 ÷3+
6
25 −
1
5 ⇒ 3 + 61 = 11
4 12
Sol 37. (b) = + 8
5
1 41
25 = 25
14−6×2 = 14−12 = 1
15÷3+3 5+3 4 Sol 56. (d)
Sol 46. (d) 15- { 5+24 ÷ (3 × 9-15)} ⇒ 15- {
nn
Sol 38. (b) 6.75×6.75×6.75−4.25×4.25×4.25
67.5×67.5+42.5×42.5+67.5×42.5 5+24 ÷ 12} = 8
5 65 + [2 32 − {3 43 (3 54 ÷ 9 21 )}] 1
(675−425)× 1000000 250 1
35
= 1 = 10000 = 40
⇒ 6 + [ 38 − { 15 19
4 ( 5 ÷ 19 )}]
2
100
Sol 57. (b)
⇒ 35 + 38 − 23 = 0.025 (-4) × (1020 ÷ 85 × 3-22) ⇒ (-4)
6
⇒ 35 + 67 = 7 × (14) = -56
6
Sol 47. (a)
26 7 5 3 1 4
5 − [ 2 − { 6 − ( 5 + 10 − 15 )}] = Sol 58. (c)
Sol 39. (c) 26 7 5 13
5 − [ 2 − { 6 − 30 }]
3 2
3 51 [2 21 − { 65 − ( 52 + 10 ] 5 × 4[7 − { 5 × (13 + 2)}]
3 4
− 15 )}
Pi

= 26 7
5 − [2 −
12 26
30 ] = 5 − 31 21
= 10 = 3 × 4[7 − { 52 × 15}] =
⇒ 16 5 5 13
5 [ 2 − { 6 − 30 }]
10 5
16 3 × 4[7 − 6] = 3 × 4 = 2 52
⇒ 5 − [ 25 − 52 ] = 11
10 5 5
Sol 48. (c)
3 65 +[3 32 -{ 15 4 1
4 (5 5 ÷ 14 2 )}] Sol 59. (d)
Sol 40. (a)
63.5×63.5×63.5+36.5×36.5×36.5 = = 23 11
6 +[ 3 −
15
4 × 52 ] = 23
6 + 22−9
6 7 − {4 × 3 − (− 10) × 8 ÷ (− 4)}
6.35×6.35+3.65×3.65−6.35×3.65
(63.5+36.5)(63.5×63.5+36.5×36.5−63.5×36.5) = 36
6 =6 = 7 − {12 + 10 × (− 2)}
6.35×6.35+3.65×3.65−6.35×3.65
= 7 − {12 − 20} = 7 + 8 = 15
=10,000
Sol 49. (b) 3 × 3-[6-{12+15 ÷
(7-2)}] = 9-[6-{12+3}] = 9-[-9] = Sol 60. (a) 10-{17-12 ÷ (5+9 ×
Sol 41. (c)
9+9 = 18 2-17)} = 10-{17-12 ÷ (6)} =
3
6 51 − [4 21 − { 65 − ( 53 + 10 − 7
15 )}]
1 1 5 13
10-{15} = -5
= 6 5 − [4 2 − { 6 − 30 }] Sol 50. (c)
= 6 51 − [ 29 − 2
5] = 31
− 41 = 62−41 1 5 −5 1 6 −5 = - 41
5 10 10 3 ÷ 6 × 8 = 3 × 5 × 8

www.ssccglpinnacle.comsupport@ssccglpinnacle.com Ph. 09729327755, 09817390373


73
Days 8-12 : Simpli ication

s
sse
la
_c
ob
Sol 61. (d) 13 ÷ {4 of 2 - 3 + 4 × Sol 69. (d) Sol 76. (d)
bo
ah A= 40 ÷ 8 + 5 × 2 − 4 + 5 of 3 (3 × 4 of 12 ÷ 2) ÷ 9 × 4 + 4 ÷ 8 + 3 × 2
sm
(6-4)} = 13 ÷ {8 - 3 + 8} = 1
ur
yo

= 5+10-4+15 = 26 ⇒ (3 × 48 ÷ 2 ) ÷ 9 × 4 + 21 + 6
e/
t.m
://

Sol 62. (d) B= 24 ÷ 4(4 + 2) + 19 of 2 ⇒ 72 ÷ 9 × 4 + 21 + 6 = 77


tp
ht

2
32 ÷ 4 of 2 × 3 + [5 of 6 − {7 of 8(10 = 24 ÷ 24 + 38 = 39
+ 6 of 65 ÷ 5 − 1) ÷ 80}] − 7 × 3 ÷ 2 ⇒ A-B = 26-39 = -13 Sol 77.(a)
⇒ 32 ÷ 8 × 3 + [30 − {56(10 + 5 A= 8 ÷ 4 × (3 − 1) + 6 × 3 ÷ 2 of 3
÷ 5 − 1) ÷ 80}] − 7 × 3 ÷ 2 Sol 70. (c) = 2 × (2)+6 × 3 ÷ 6 = 7
⇒ 4 × 3 +[30-{56(10) ÷ 80}] - 7 36 ÷ 8 × 4 + 2 ÷ 4 − 1 + 5 of 3 ÷ B= 4 ÷ 8 × 2 + 7 × 3
× 23 (4 × 2 − 3) − 3 = 21 × 2 + 21 = 22
⇒ 12 + 23 - 21 ⇒ 4.5 × 4 + 21 -1+15 ÷ (5)-3
2 = 24.5 ⇒ A+B = 7+22 = 29
⇒ 18- 21 = 35
2

Sol 63. (b) Sol 78.(d)


72÷9+3−6−(2×3)+5 of 3−(1+5×2−2) Sol 71. (c) (6 of 4 ÷ 16 × 48) ÷ 8 × 4 + 2 × 3
8÷4+2−(6×8÷2)+(7×4−2×2)
A= 7 × 3 ÷ (2 + 4) + 4 − 2 ÷ 6 + 5(6 − 2)

e
8+3−6−6+15−(1+10−2)
⇒ 2+2 (6×4)+(28 4)
− − = 21 ÷ 6+2 = 112 ⇒ (24 ÷ 16 × 48 ) ÷ 8 × 4 +2 × 21 +
⇒ 11+− 1215− 9 5
=4
4 B= 3 ÷ 6 × 4 + 2 − 2 of 3 20
= 21 × 4 + 2 − 32 = 10
3 ⇒ 72 ÷ 8 × 4 +1+ 20
Sol 64. (b)

l
and C= 6 ÷ 2 + 4 × 3 − 2 ⇒ 9 × 4 +21 = 57
7 ÷ 2 − [3 of 7 ÷ 4 ÷ {(2 ÷ 5) × (25 ÷ 8) = 3+12-3 = 12
÷ (5 ÷ 2)}]
⇒ 3.5-[21 ÷ 4
ac
÷ {0.4 × 3.125 ÷ 2.5 ]
⇒ 3.5-[5.25 ÷ {0.4 × 1.25} ]
⇒ 3.5 - [5.25 ÷ 0.5] = -7
(A+B-C) = 11

Sol 72.(b)
10
2 + 3 -12 = −

3 of 24÷8×3+4÷2−4×5
36÷12×4÷2+5×(6−4)
⇒ 72÷8×3+2 −20
19
2 Sol 79. (a)
3

+


4
1

3
4
4
×
1
4 ÷ ( 2 + 16 ) +
1 2
3
9 )+
÷ ( 16
+ 23 + 61 = 3
8
27
2
3 of 4
9

16 ) +
÷ ( 81
÷ ( 31 −

1
6
11
81 )

3×2+10 3

Sol 65. (a) ⇒ 27+2 16


−20 = 9
16
3 1 1 2 3 2 Sol 80. (d)
4 of ( 3 ÷ 2 ) + (2 − 5 ) × +
nn
2 3 2 2 2
(0.7) ÷0.14+(0.6) ÷0.18+(0.5) ÷0.05
⇒ 3 of ( 32 ) + ( 58 ) × 3 + 2 Sol 73.(b) 4(2.5 of 4−13×0.25×3)
4 2 3 3 9 4 2
÷ + × of 16 27
1 + 12 2 107 4 32 3 3
⇒ 0.49÷0.14+0.36÷0.18+0.25÷0.05
5 + 3 =
⇒ 2 30
1 8 4 1 1
×( 3 −2)÷ 9 +( 3 + 6 ) 4(10−9.75)
2


8 4 9
3+ 3× 8 ⇒ 3.5+2+5
4×0.25 = 10.5 = 21 2
1 2 4 1
×( 3 )÷ 9 + 2
Sol 66. (c) 2
8 3
2 9 1 5 3+ 2 10
3 of 4 + 2 ÷ 4 ⇒ 1 3 1
× + = 3 Sol 81.(b)
1− 13 + 14 ×(1+ 13 ) 2 2 2

3 1 5
(1− 14 )+( 12 of 12 )÷ 25
2+ 2÷ 4 2 1 3 8
⇒ 2 1 4 5 ÷ 4 + 2 (2− 5 )
+ ×
3 4 3 Sol 74. (d) ( 34 )+( 14 )÷ 25
Pi

3 2
2+ 5 19 39÷26+22÷11×2+4×3 ⇒
⇒ 1 = 10 2 of 5−3(7+10÷2−3×3)
2 1 3 2
5÷ 4+ 2( 5)
3 ( 34 )+( 58 )
⇒ 102 +2×2+12 ⇒ 8 3
+ = 85
−3(7+5−9) 5 5
Sol 67. (c) 3
2 +4+12 35
3 9 4 1 12 25 5 ⇒ =
7 ÷ 21 + 2 − 3 + 2 of 5 × 18 ÷ 9 10−9 2
Sol 82. (d)
⇒ 1+ 32 + 56 × 5
2 3 of 5 1 ÷
(1 + 43 ) × 21 128 + 2 × 7
Sol 75.(c) 3 49 3 11
⇒ 1+ +3 = 2 14
121 15
3 3
(24 + 16 × 5 − 8 of 4) ÷ 84 × 48 ÷ × 49 ÷ ( 14 − 72 )
24 × 6 + 4 + 3 ⇒
Sol 68. (c) 7 × 16 ÷ 128 + 2 × 7 × 121 ÷ 11
⇒ 4 21 49 3 11 49 14
90 × 3 ÷ 9 + 4 ÷ 2 × 3 of 4 × 8 7 7 2 7 22
4 × 24 + 3 × 11 ×
(24 + 80 − 32) ÷ 84 × 2 × 6 + 4 + 3 ⇒ 7
÷ (18 × 2 − 4) 49 4 59
96 + 3 =
⇒ 72 ÷ 84 × 12+7 ⇒
⇒ 90 × 31 + 2 × 12 × 8 ÷ (36-4) 32

1
⇒ 76 × 12 +7 = 121
7
⇒ 30 + 24 × 4 = 36
Sol 83. (d)

www.ssccglpinnacle.comsupport@ssccglpinnacle.com Ph. 09729327755, 09817390373


74
Days 8-12 : Simpli ication

s
sse
la
_c
ob
2
A= 2 ÷ 3 × 4 + 9 × (0.3) 5 31 × 2 71 × 9 52 × 4 83 × 2 47
6

bo
ah
= 38
sm
⇒ 0.09 × 2.4 +3.6 × 0.49+0.81 ⇒ 16 × 15 × 47 × 35 × 100 = 1000
ur

3 7 5 8 47
yo
e/

B= 3 of 4 + (7 − 2) ⇒ 0.216 + 1.764 + 0.81 = 2.79


t.m
://

= 12 + 5 = 17
tp

Sol 101.(c)
ht

and C= 4 + 5 − 6 Sol 91. (a)


=3 2 of 3 ÷ 3 × 2 + {4 × 3 − (5 × 2 + 3)} √ 25.60
72.90 +
√ 0.10
8.10

⇒ A+B+C = 38 +17+3 = 68
3
⇒ 6 ÷ 3 × 2 + {12 − (10 + 3)}
⇒ 2 × 2 +12-13 = 3

√(
16
1.6 2 +
2.7 )
1 19
√( ) 1 2
9

27 + 9 = 27

Sol 84. (c)
(49−13)×18÷9+4×12÷6+5 Sol 92. (d)
98÷14+7×4 of 6÷8+4 Sol 102. (a)
95 × 99 95
99 99
⇒ 36×2+4×2+5 − (15 + 3 × 1.1) ÷ 0.0003
7+7×24÷8+4 95
72+8+5 = 85 ⇒ 99 × 99 + 99 × 99 -95 ⇒ (15+3.3) ÷ 0.0003
⇒ 7+21+4 32
⇒ 9801 + 95 - 95 = 9801 ⇒ 18.3
3 x 10000 = 61000

Sol 85.(c)
Sol 93. (d) Sol 103. (c)

e
2÷3×(1+3)+5−6
2 of 3÷5×4+3−2 0.56×0.36+0.42×0.32
2 0.8×0.21 7 × 7 of 3 ÷ 3 − 14 × x = 7
3 ×4+5−6
⇒ 6÷5×4+3−2 ⇒ 0.2016+0.1344 = 2 ⇒ 7 × 21 ÷ 3 − 14 × x = 7
0.168
5
⇒ 3
= 25 ⇒ 49-7 = 14x
24 87
5 +1

l
Sol 94. (a) ⇒ x =3
(3576 + 4286 + 6593) ÷ (201 + 105
Sol 86.(a)


32 ÷ 48 × 12 +
ac
2 of 16 ÷ 48 × 12 + 4 ÷ 8 × 16
+ (7 − 2) × 25 ÷ 15

1
2 × 16 + (7 − 2) × 25
+ 107)
⇒ 14455 ÷ 413 = 35

Sol 95. (d)


45 × x = 25% of 900
Sol 104. (a)
(1x2+2x3-3x4+4x5-5x6+6x7)
⇒ 2 + 6 - 12 + 20 - 30 + 42 = 28

Sol 105. (d)


÷ 15 ⇒ 45x = 100 25 x 900 8 of 3 ÷ 6 + (10 + 2) × 3 − 96 ÷ 3
⇒ 32 × 12 + 8 + 5 × 5
3 = 73
3 225 ⇒ 24 ÷ 6 + 12 × 3 − 32
⇒x= 45 =5
nn
⇒8
Sol 87. (c)
Sol 96. (d)
( 21 ÷ 21 × 21 + 1
2 − 1
2 + 1
2 × 1
2 ÷ 21 ) 3 Sol 106. (d)
(x5 ÷ x4 ) ÷ x2
of ( 21 + 1
2)
2
12 + 16 of 3 − 20 ÷ 4
⇒ (x)3 ÷ x2 = x
⇒ (1 × 1 1
2 + 2 × 1) of (1) = 1 ⇒ 144 + 48 - 5 = 187
Sol 97.(c)
Sol 88. (c) 2 Sol 107. (b)
12 of 3÷6+12×2−(2×4−5)
23 + √x = 625
[{(100 of 0.9 × 0.8 − 7 × 1.2 ÷ 0.2
12÷3×4+(2×4−5) ⇒ √x = 625 - 529
Pi

+ 5 × 4 − 3 × 2)} ÷ 10 + 1.85]
⇒ 36÷6+24 −(8−5)
4×4+(8−5)
⇒ x = 962 = 9216
⇒ [{(90 × 0.8 − 7 × 6 + 20 − 6)}
⇒ 6+24 −3
16+3 = 19
27
÷ 10 + 1.85]
Sol 98. (c)
⇒ [72-42+20-6] ÷ 10+1.85
Sol 89. (b)
13
(2 61 + 1 18 − 61 ) × 16 ÷ 4 ⇒ 44 ÷ 10 + 1.85 = 6.25
5 of 5 of 5 ÷ 5 + 5 − 6 ÷ 3 × 4 + 2 ⇒ ( 13 31 1
6 + 18 − 6 ) × 4
+ (3 ÷ 6 × 2) ⇒ 39+31−3 ×4= 134
Required square root = √6.25 =
18 9 2.5
⇒ 125
÷ 5 + 5 − 2 × 4 + 2 + ( 21 × 2) Sol 99. (c) Sol 108. (a)
⇒ 25+5-8+3 = 25 [12 × 5 − {200 − (501 + 247 − 386)}] ÷ 2 (1 x 2+3 x 4+5 x 6+7 x 8 - 9 x 10)
⇒ [60-{200-362}] ÷ 2 ÷ 2 of 5
Sol 90. (c) ⇒ [60+162] ÷ 2= 111 ⇒ (2 + 12 + 30 + 56 - 90 ) ÷ 10
2 2
(9 ÷ 30) × 2.4 + 0.3 of 12 × (1 − 0.3) Sol 100. (b) ⇒ 10 ÷ 10 = 1

www.ssccglpinnacle.comsupport@ssccglpinnacle.com Ph. 09729327755, 09817390373


75
Days 8-12 : Simpli ication

s
s se
la
_c
ob
Sol 122.(c)
bo
ah
Sol 109. (c) Sol 116.(b) 4 × 2 ÷ 4 of (4 + 4 ÷ 4 of 4) −
sm
ur
yo

(28 ÷ 4 × 7) + (44 ÷ 4 × 7) - (12 3 ÷ 3 of 3 + 2 ÷ 4 + (4 × 2 − 2) ÷ 12 + 4 (4 ÷ 4 of 2 × 4)


e/
t.m
://

× x ) = 18 ⇒ 3 ÷ 9 + 21 + (8 − 2) ÷ 12 + 4 ⇒
tp
ht

⇒ (7 × 7) + (11 × 7) - (12 × x ) ⇒ 1 + 21 + 21 +4 = 16 4 × 2 ÷ 4 of (4 + 4 ÷ 16) − (4 ÷ 8 × 4)


3 3
= 18 ⇒ 4 × 2 ÷ 4 of (4 + 41 ) − ( 21 × 4)
⇒ 49 + 77 - 12x = 18 Sol 117. (d) ⇒ 4 × 2 ÷ 4 of ( 17
4 )−2
⇒ 126-18 = 12x (8 ÷ 32 of 54 ) ÷ (8 × 2 ÷ 4 2 4
5) of (8 ÷ × 5) ⇒4× 2 -2= − 26
3 3 17 17
⇒x=9 ⇒ (8 ÷ 8 × 65 ) of (12 × 54 )
15 ) ÷ (8
⇒ 15 ÷ 20 of 48 Sol 123. (b)
3 5
Sol 110. (b) 15 26 − [(2 of 6 ÷ 3) − 93 − {17 − (14
⇒ 15 ÷ 64 = 64
2 × 2 + 4 × 4 + 2 of 3 × 6 − 7
− 2)}]
× (5 + 4 ÷ 2) ⇒
Sol 118. (c)
⇒ 4+16+ 6 × 6 -7 × (5 + 2)
165 − [135 − {84 ÷ 4 of 3 − 26 − [(12 ÷ 3) − 93 − {17 − 12}]
⇒ 20 + 36 -49 = 7
⇒ 26 - [4-93-5] = 120

e
(16 − 18 ÷ 3)}]

Sol 111. (c)
165 − [135 − {84 ÷ 12 − (16 − 6)}] Sol 124.(d)
(0.4 of 50 × 6 ÷ 8) ÷ (12 × 10 ÷ 16) 2 2 2 2
2 ⇒ 165 − [135 − {7 − 10}] (5) + (6) + (30) = (x)
+ 5 × 0.2 − 0.01 × 10

l
⇒ 165-138 = 27 ⇒ 25+36+900 = x2
⇒ (20 × 6 ÷ 8) ÷ (12 × 10 +1
16 ) ⇒ x = √961 = 31
−1

Sol 112. (c)


ac
⇒ (15) ÷ 7.5 = 2

(2 of 14 ÷ 7 × 3) + (44 ÷ 11 × 8)
Sol 119.(b)
27 × 6 ÷ 24 + 6 ÷ 2 of 3 + 30 ÷ 24 × 18
− 9 ÷ 54 of 3 × 216
⇒ 27 × 41 + 6 ÷ 6 + 45 × 18 − 9
÷ 162 × 216
Sol 125. (a)
3
56 + (4) − 3 × (3)
⇒ 56 + 64 - 27 = 93
2

− (12 × 9 ÷ 3)
⇒ 27 × 41 + 1 + 90
4 − 1
18 × 216 Sol 126.(c)
⇒ (28 ÷ 7 × 3) + (4 × 8) − (12
× 3)
⇒ 27 +1+ 90 -12
4 4
√ 3 1
+ 1
− 3
nn
16 2 4
⇒ 117 -11 = 18.25 7 1
⇒ (4 × 3) + (32) − (36) = 8 4 ⇒ 4 + 2 − 43 = 1 21

Sol 113.(c) Sol 120.(a) Sol 127.(c)


1 2 1 5 3 × 1 87 ÷ 1 31
2 − 2 ÷ 2 × 2 + 2(2 of 2 − 2 − 2 ÷ 2) 7 of 1 5 ÷ {5 2 − ( 32 + 5 (4488 ÷ 11.01 − 7.98) ÷ 15.99
3 )}
of 16
⇒ 2 − 1 × 2 + 2(4 − 2 − 1) ⇒ (407.63-7.98) ÷ 15.99
⇒ 2 - 2 + 2 (1) = 2 1 ÷ { 11 5 3 7 1
2 − ( 32 + 5 × 1 8 ÷ 4 )}
⇒ 5 ⇒ 399.64 ÷ 15.99 = 24.99 ≈ 25
1 ÷ { 11 5 3 15 Alternate :
2 − ( 32 + 5 × 2 )}
⇒ 5
Pi

Sol 114. (a) 1 ÷ { 11 5 9 Since none of the option is very


2 − ( 32 + 2 )}
⇒ 5
61 + 9 ÷ 1 of 4 (1 + 1 ) +
3 43 − 122 1 × 4
1 ÷ { 11 149 much close to one another we can
2 − 32 }
2 2 3 3 2 3 ⇒ 5
⇒ 15 − 1 + 9 ÷ 2(4) + 2
1 27 32 take 11.01 = 11 7.98 = 8 and
4 2 2 3 3 3 ⇒ 5 ÷ 32 = 135
⇒ 15
− 1 + 27 × 4 + 2 15.99 =16 to ease the calculation.
4 2 4 3 3
13 2 = 155 (4488 ÷ 11.01 − 7.98) ÷ 15.99 ⇒
⇒ 4 +9+ 3 12 Sol 121. (b)
1 3 1 2
(4488 ÷ 11 − 8) ÷ 16
4 × 4 ÷ 1 4 of 5 − [ 61 ÷ { 73 of 14
5 × 1 32
⇒ (408-8) ÷ 16 = 25
Sol 115.(a) − (3 21 − 2 61 )}]
(1− 34 )+ 12 of 10
6

2 4 1
÷ +(1− 5 ) of 16 25 ⇒ 41 × 43 ÷ 21 − [ 61 ÷ { 56 × 5
3 Sol 128. (b)
3 10


3
( 14 )+
10 − 68 }] 7 ÷ 14 of 2 − 7 × 7 ÷ 49 + 1
3 of
5 4 25
+ of 16
3 5
11
⇒ 41 × 3 1 8
2 − [ 6 ÷ {2 − 6 }] (14 ÷ 7 + 7) + 7 − 14 ÷ 2
⇒ 20
5 5
+ ⇒ 83 − [ 61 ÷ 64 }] = 81 ⇒
3 4
11×20 33 7 ÷ 28 − 7 × 1 + 1 of (2 + 7) + 7 − 7
⇒ 20×35 = 175 7 3

www.ssccglpinnacle.comsupport@ssccglpinnacle.com Ph. 09729327755, 09817390373


76
Days 8-12 : Simpli ication

s
sse
la
_c
ob
1 -1+3 = 2 41 ⇒ ( 49 36 ⇒ (12 ÷ 32 ) × 10 ÷ ( 43 × 16 )
2 ) × ( 49 ) = 18

bo
4 9 3
ah
sm
⇒ 18 × 10 ÷4 =5
ur

9
yo

Sol 129. (d) Sol 136. (d)


e/
t.m
://

6 81 ÷ (5 41 ÷ 73 of 21 ) − 8 × 32 ÷ 54 of 1 32 1 + 1 +
x = 12.13 1 1
…..+ 23.24
tp

Sol 4. (a)
ht

13.14 14.15
3 3
⇒ 49 1 3 2
8 ÷ (5 4 ÷ 14 ) − 8 × 3 ÷ 3
4 = (253) + (247)
1 1 1 1 1 1 1 1 25.3×25.3 −624.91+24.7×24.7
⇒ 49 ÷ ( 147 )
− 8× 1
12 − 13 + 13 − 14 + ... + 23 − 24 = 12 − 24 ⇒ (253+247)(253×253−62491+247×247
8 6 2 1
1 ×(253×253−62491+247×247)
1 15 =
4 −4= − 4
⇒ 24
100

1 1 + 1 1 ⇒ 500 x 100 = 50 x 1000


y= 36.37 + 37.38 38.39 …..+ 71.72
According to the question
Sol 130. (c) = k
3 ÷ 21 of 3 × 7 + 24 × 6 ÷ 18 − 3 ÷ 2 1 1 + 1 1 + ... + 1 1 = 1 1 50 x 1000 = 50 × 10
36 − 37 37 − 38 71 − 72 36 − 72
⇒k=3
+3−2×3÷6 = 1
72
⇒ x
1

1 3 1 Therefore, y = 24
1 =3 Sol 5. (b)
3 ÷ 63 × 7 + 2 4 × 3− +3−2×2 2 72

1 3 Alternate : ( √2 + √5 − √3 ) × k = -12
21 × 7 + 8 − 2 +3−1

1 + 1 + 1 1 ⇒k= −12
x = 12.13 …..+

e
1 3 53 = 8 65
3 + 10 − 2 = 6
⇒ 13.14 14.15 23.24 (√2+√5−√3)
= 13−1 12 ( 12
1 1 )= 1
− 24 x √ √ √
24 −12 ( 2+ 5+ 3)
=
1 1 1 1 (√2+√5−√3) (√2+√5+√3)
Sol 131. (d) y = 36.37 + 37.38 + 38.39 …..+ 71.72
−12(√2+√5+√3)
3 51 ÷ 4 21 of 5 31 − 2 31 of { 73 − (1 15
4 13 = 37−1 36 ( 36
1 1 )= 1 ⇒ ⇒
− 30 ) − 72 2 2
72 (√2+√5) −(√3)

l
1
× 15}1 x
Therefore, = 24
1 =3 −12(√2+√5+√3)
y
16 ac 7 72 4+2√10
⇒ 5 ÷ 24 - 3 of { 73 − 25
30 × 6
5 }
⇒ 2 + 34 = 7
1 15 ⇒ −6(√2+√5+√3) × √10−2
15 Sol 137. (d) Given, 2+√10 √10−2
5
(1.25)(1 − 6.4 × 10− ) = 1.2496 + a ⇒ −6(√2+√5+√3)(√10−2) = ( 6
Sol 132. (a) ⇒ (1.25)(0.999936) = 1.2496 + a
1 32 ÷ { 73 of 14 2 1 1
5 × 1 3 − (3 2 − 2 6 )} ⇒ 1.24992 = 1.2496 + a
√2 + √5 + √3 )( 2 − √10 )
+ 21 ÷ 23 of 21 ⇒ a = 0.00032
5 6 5
Sol 6. (a)
⇒ 3 ÷{ 5 × 3 − ( 68 )} + 1
2 ÷ 3
4 (
nn
5 2 2
3 ÷ 3 + 3
⇒ SSC CGL TIER II 1 31 ÷ 2 76 of 5 53 ) ÷ (6 52 ÷ 4 21 of 5 31 )
5 2 1
2 + 3 =3 6
⇒ × ( 43 × 2 32 ÷ 5 of 1 51 ) = 1 + k
9
Sol 1. (d)
7 + 8 × 8 ÷ 8 of 8 + 8 ÷ 8 × 4 of 4 ⇒ ( 34 ÷ 16) ÷ ( 32
5 ÷ 24)
Sol 133. (d) 4 ÷ 4 of 4 + 4 × 4 ÷ 4 − 4 ÷ 4 of 2
× ( 43 × 2 32 ÷ 2
3 ) =1+k
72 ÷ 6 of 12 + 4 × (5 − 3) of 2 ÷ 4 − 2 ⇒ 74 +÷ 816× +8 4÷ ×64+ 8 ÷ 8 × 16
4÷4−4÷8 1 ) ÷ ( 4 ) × (3) = 1 + k
⇒ ( 12 15
⇒ 72 ÷ 72 + 4 × 2 of 2 ÷ 4 − 2 ⇒ 71 ++ 41+ 161
4 −2 ⇒ 5 ×3 = 1 + k
⇒1 + 4× 4 ÷ 4- 2 = 3 24×4
16
⇒ 15 = 6.4 ⇒ 0.9375 = 1 + k
Pi

Sol 134. (c) ⇒ k = -0.0625


5+2 of 3÷3 of 2×3 Sol 2. (d) Clearly k lies between -0.07 and
9+72÷3−2×(3−2)−3
5+6÷6×3 = 8 2
22. 4 + 11.5 67 - 33.5 9 ⇒ -0.06
⇒ 9+24 −2×1−3 28 = 7 22.444444…. ∞ + 11.5676767…
a
⇒ b=7 2
∞ - 33.599999…….. ∞ Sol 7.(a)
So, (b-a) = 7-2 = 5 = 0.412121212…. ∞ (0.545)(0.081)(0.51)(5.2)

3 3 3
(0.324) +(0.221) −(0.545)
= 0.4 12 (0.545)(0.081)×4×3×(0.17)(1.3)
Sol 135. (c) 3
(0.324) +(0.221) −(0.545)
3 3

(5 41 ÷ 73 of 21 ) × (5 41 × 3
7 ÷ 21 ) ÷ (5 41 ⇒ 3×(0.545)(0.324)(0.221)
Sol 3. (a) 3
(0.324) +(0.221) −(0.545)
3 3
÷ 3 × 21 )
7 (2 76 of 4 51 ÷ 32 ) Now, since

× 1 91 ÷ ( 43 × 2 32 of 1
2 ÷ 41 ) (0.324+0.221-0.545 = 0)
( 21
4 ÷
3
14 ) × ( 21 6 21
4 × 7) ÷ ( 4 ÷
3 × 21 )
7 ⇒ ( 20
7 of
21
5 ÷ 32 ) ⇒ (0.324)3 + (0.221)3 − (0.545)3
⇒ ( 49 9 49 1
2 ) × (2) ÷ ( 4 × 2) 10
× 9 ÷ ( 43 × 4
3 ÷ 41 ) = -{ 3 × (0.545)(0.324)(0.221) }

www.ssccglpinnacle.comsupport@ssccglpinnacle.com Ph. 09729327755, 09817390373


77
Days 8-12 : Simpli ication

s
sse
la
_c
ob
13 23
ur
ah
bo
……….(Algebraic property)
sm ⇒ √(5 + √3) − √(2 − √3) 2 2 ⇒ 5+4 × 8
Desired difference =
= 2
77
4 - 23 3
2 =7 4
⇒ ( 5 + √3 )-( 2 − √3 )
yo

So,
e/
t.m

⇒ 3+ 2√3 = 6.46 ≈ 6.5


://

3×(0.545)(0.324)(0.221)

tp

Sol 16.(d)
ht

3 3 3
(0.324) +(0.221) −(0.545)
4 4 2
3×(0.545)(0.324)(0.221) (4.6) +(5.4) +(24.84)
= -1 ⇒
−{3×(0.545)(0.324)(0.221)} Sol 12.(d) (4.6)2 +(5.4)2 +24.84
2 2 2 2 2 2
0.47 + 0.503 − 0.39 × 0.8 {(4.6) +(5.4) } −2{(4.6) (5.4) }+(24.84)
Sol 8. (a) (4.6)2 +(5.4)2 +24.84
⇒ 4790−4 + 503−5 39−3
990 − 90 × 9
8
2 2 2 2


{(4.6) +(5.4) } −(24.84)
10 + 2(√6 − √15 − √10) ⇒ 43 498 32 ⇒ ⇒
90 + 990 − 90
⇒ (4.6)2 +(5.4)2 +24.84
473+498−352 = 619 2 2 2 2

√⇒10 + 2(√2.√3 − √5.√3 − √2.√5) ⇒ = [{(4.6) +(5.4) }−(24.84)][{(4.6) +(5.4) }+(24.84)]


990 990
(4.6)2 +(5.4)2 +24.84
0.6252525... ∞ = 0.6 25 2 2
⇒ (4.6) + (5.4) − 24.84
Alternate :
√{(√2) + (√5) + (√3) } +
2 2 2
0.47 + 0.503 − 0.39 × 0.8
⇒ (4.6 + 5.4)2 − 2(4.6)(5.4) − 24.84
⇒ 100 − 74.52 = 25.48
⇒ 0.477 + 0.503 − 39−3 8
90 × 9

√2(√2.√3 − √5.√3) − √2.√5

e
⇒ 0.477 + 0.503 − 32
90 SSC CGL TIER I
⇒ √(√2 + √3 − √5)
2
⇒ 0.477 + 0.503 − 0.35
Sol 1. (b) Replace symbols in
= (√2 + √3 − √5) ⇒ 0.477 + 0.503 − 0.355 = 0.6 25
equation 42−12 ×3+8 ÷ 2+15 and

l
8×2−4+9÷3
apply BODMAS rule
Sol 9.(a) ac Sol 13. (c)
0.5 6 -0.7 23 +0.3 9 × 0.7
⇒ 5690−5 − 723
51 716
−7

28
39−3
990 + 90 × 9
7

2√10
√ √2−√7
5+

√ √5−2

3
√5+2 √7−2
Hence;
42−12 ×3+8 ÷ 2+15
8×2−4+9÷3 becomes
90 − 990 + 90
⇒ 42+12 ÷3−8 × 2−15 = − 15


8÷2+4−9×3 19
2√10
⇒ 561−716+308 = 153
990 990 = × √5+√2+√7 − √5−2 × √5−2
√5+√2−√7 √5+√2+√7 √5+2 √5−2
0.1545454... ∞ = 0.1 54 Sol 2. (c) Apply BODMAS rule
×√
3 7+2

Alternate : √7−2 √7+2 in the given equation:
⇒ ( 18 ÷ 2 of 41 ) × ( 32 ÷ 43 × 85 ) ÷ (
nn
0.5 6 -0.7 23 +0.3 9 × 0.7 ⇒ 0.5 6


2 2 ÷ 43 of 3 )
-0.7 23 + 39−3
× 7 (2√10)(√5+√2+√7) (√5−2) 3(√7+2) 3 4
90 9 2 2 − 2 − 2
18
28 ⇒ (√5+√2) −(√7) (√5) −2 2
(√7) −22
⇒( ) × ( 32 × 4
3 × 85 ) ÷ ( 3× 23 × 3 )
⇒ 0.5 6 -0.7 23 + 90 0.5 6 -0.7 2× 14 4 4
⇒ 32 )
⇒ (36) × ( 95 ) ÷ ( 27
23 +0.3 1
(2√10)(√5+√2+√7)
⇒ 0.5 66 -0.7 23 +0.3 11 =0.1 54 7+2√10−7
− (√5 − 2) − (√7 + 2) 27 )
⇒ (36) × ( 95 ) × ( 32
⇒ 27×5 ⇒ 135 ⇒ 16 87
⇒ √5 + √2 + √7 8 8
Sol 10. (a) − √5 + 2 − √7 − 2) = √2
Sol 3. (d) On applying BODMAS
Pi

9 × 6 ÷ 24 + 8 ÷ 2 of 5 − 30 ÷ 4 of 4
+ 27 × 5 ÷ 9 rule in the given equation:
Sol 14. (a)
⇒ 9 × 41 +8 ÷ 10-30 ÷ 16 +15 We get , - 25 + 23 ÷ 6 × 21 = - 19
24 × 2 ÷ 12 + 12 ÷ 6 of 2 ÷ (15 ÷ 8 × 4) 8
9 4 15
4 + 5 - 8 +15
⇒ of (28 ÷ 7 of 5)
⇒ 90+32−75 + 15 ⇒ Sol 4. (a) Apply BODMAS rule
40
in the given equation
⇒ 47 +15 = 647
40 40
24 × 61 + 12 ÷ 12 ÷ ( 158 × 4) of (28 ÷ 35)

⇒ 4+ 12 ÷ 12 ÷ ( 152 ) of 54 36÷42 of 6×7+ 24×6÷18+3÷(2−6)−(4+3×2)÷8
Sol 11. (c) ⇒ 4+1 ÷ 6 = 4 61 21÷3 of 7

√28 + 10√3 − √7 − 4√3 Sol 15. (a)
36÷252×7+ 24×6÷18+3÷(2−6)−(4+3×2)÷8
21÷21


⇒ 25 + 3 + 2 × 5 × √3 9 94 ÷ 11 31 of 61 ⇒
36÷252×7+ 24×6÷18+3÷(2−6)−(4+3×2)÷8
− √4 + 3 − 2 × 2 × √3
+ (1 31 × 1 54 ÷ 53 ) × 2 61 of 32 ÷ 34 of 32 21÷21
⇒ 85 ÷ 34 + ( 34 × 9 ÷ 53 ) × 13 ÷ 8
9 18 5 9 9

www.ssccglpinnacle.comsupport@ssccglpinnacle.com Ph. 09729327755, 09817390373


78
Days 8-12 : Simpli ication

s
se
s
la
_c
ob
1+ 8− 34 − 10 -15 + 90 ÷ [89 − { 9 × 8 + 12}
bo
⇒ 8
=7
ah 21÷21
Sol 12. (d) 4 ÷ 1 ×2−1=
sm
3 6 -15 + 90 ÷ 5
ur
yo

4
3 × 6 × 2 − 1 = 15 3
e/
t.m

Sol 5. (b) Apply BODMAS rule


://
tp

in the given equation.


ht

7−[4+3(2−2×2+5)−8]÷5
Sol 13. (d) As last digit of 113 × Sol:2.(c)
2÷2 of (4+4÷4 of 4) 87 is 1, only option (d) satisfies. Use BODMAS : 2.1 + 2.25
⇒ 7−[4+3(3)−178]÷5 ÷ [63 − {7.5 × 8 + (13 − 2.5 × 5)}].
2÷2 of ( 4 ) [54−(5÷2)×8]+13 141
7−1 Sol 14. (a) 48−4÷3×8−2 = 106 = 2.1 + 2.25 ÷ 2.5
⇒ 4
17 (using BODMAS theorem) = 3.0
We get 25 21 as answer
Sol 15. (d) 1512-1492 = Sol:3.(a)
Sol 6. (b)On applying BODMAS (151-149)(151+149) = (2)(300) = [0.9-{2.3-3.2-(7.1 - 5.4 - 3.5)}]
rule, 600 =[0.9-{2.3-3.2+1.8}]
5 12 ÷3 23 of 14 +(5 19 −7 78 ÷9 20
9 9
)× 11
1
5÷5of 10 −10×10÷20
=[0.9-0.9]
Sol 16. (d) 3-(9-3 × 8 ÷ 2) = 6 =0
11
5 12 ÷ 12 77
+( 18 9
)× 11

e
5÷ 12 −10×10÷20 (using BODMAS theorem)
⇒ 6+ 72 9 Sol:4.(a)
10 −5 = 1 10
Sol 17. (b) Apply BODMAS 4 × 30 - 15 ÷ 5 + 12 = 21
1 81 ÷ (4 41 ÷ 53 of 8 21 )- 52 × 1 31 ÷ ⇒ 4 × 30 ÷ 5 − 15 + 12 = 21
Sol 7. (b) On applying BODMAS

l
4 of 1 32 + 20
11
5 ⇒ 4 × 6 − 15 + 12 = 21
rule 9
ac
8÷[(8−3)÷{(4÷4 of 8)+4−4×4÷8}−2] ⇒ 8 ÷ ( 17
4 ÷
51
10 )- 52 × 4
3 ÷ 4
3 + ⇒ 24 − 15 + 12 = 21
8×8÷4−8÷8 of 2 − 7 11 ⇒ 9 + 12 = 21
8÷[(5)÷{( 18 )+4−2}−2] 20
⇒ ⇒ 21 = 21
8×2− 12 − 7
8÷[5÷ 17 ⇒ 9 ÷ ( 65 )- 52 + 20
11 =1 1
8 −2]

8×2− 12 − 7
= 38 8 2
Sol:5.(b)
45-5 of (0.7)+3.5
Sol 18. (b) Apply BODMAS
Sol 8. (a) Apply BODMAS rule 45-3.5+3.5=45
theorem
3 23 ÷ 30
11
of 23 − 14 of 2 12 ÷ 35 ×4 45
2 1 3 3 1 1 5.6-{2+0.6 of (2.1-2.6 × 1.12 )}
nn
5 of 7 2 ÷ 4 − 4 ×1 2 ÷2 4 Sol:6.(b)
11 11 5 3 24
3 ÷ 45 − 8 ÷ 5 × 5
= 4.0872 106 ÷ 2+[5+6 × 2]

3÷ 34 − 34 × 32 ÷ 94
15−5 53+22=75

4− 12
=2 76 Sol 19. (d) Apply BODMAS rule
1800 ÷ 20 × {(12-6)+(24-12)} =
Sol:7. (b)
Sol 9. (c) Apply BODMAS rule 1620. 18 + 3 - 5 × 6 ÷ 4
3 7 1 3 1 1 1
5 × 1 8 ÷ 1 3 of 16 − (3 5 ÷ 4 2 of 5 3 ) ⇒ 18 ÷ 3 × 5 + 6 - 4
× 2 21 + 1 + 1 ÷ 1 Sol 20. (d) Apply BODMAS rule
2 8 4 ⇒6 × 5+6-4
or Digital sum rule in: 11+11 ×
Pi

⇒ ⇒ 30 + 6 - 4
3 15 1 11-11 ÷ 11 = 131
5 × 8 ÷ 4− (3 51 ÷ 24) × 25 + 1
2 + 1
2
⇒ 36 - 4
⇒ 9
− 2 × 25 + 21 + 21 =5 61 ⇒ 32
2 15 Sol 21. (a) 515 × 485 =
(500+15)(500-15) = 250000-225
Sol 10. (b) On Applying Sol:8.(b)
= 249775
BODMAS 4 + [3{35 + (42 + 10
-1+ 41 ÷ 21 × 2 + 5 = 5 ÷ 2 × 3 − 40)} + 7]
Sol 22. (b) Apply BODMAS rule
4 + [3{35 + (42+15-40)}+7]
(26-13 × 2) ÷ 2+1 = 1
Sol 11. (b) 4 + [3{35 + 17 }+ 7]
As per given condition 4 + [156+7]
[(30×5)+(84×6)]÷5 SSC CHSL 2019
⇒ [(30÷5)−(84÷6)]×5 167
2
[ 3 ÷18]−[4÷2] 2
[ 3 ×18]+[4×2] Sol:1. (a)
Now, Apply BODMAS rule -15 + 90
Sol:9. (c)
⇒ (6−12+8
14)×5
= -2 ÷ [89 − {9 × 8 + (33 − 3 × 7)}]

www.ssccglpinnacle.comsupport@ssccglpinnacle.com Ph. 09729327755, 09817390373


79
Days 8-12 : Simpli ication

s
se
s
la
_c
ob
1 1 1
(9 + 3 - 16 ÷ 4 + 10) + 5÷ 5× 5
5 √3 + 7√2 − √6 − 23
− 4 51 ÷ 105
bo
ah √2+√3+√6 1 1 1
5 ÷ 5 of 5
sm
{(3 + 5 × 2 ÷ 10)} × (18 − 4 of 5)
ur

1 1
(put value of √3 = 1.73 and √2 = − =0
yo

⇒ (9+3-4+10)+{(3+1)} × (-2) 25 25
e/
t.m

1.41 and √6 = 2.45


://

18-8
tp
ht

5(1.73)+7(1.41)-2.45- Sol:24. (a)


= 10
23 75 53 ÷ [15 ÷ 3 of 5 + 7 ÷ 1
14 − {78 ÷ 3 31 }]
1.41+1.73+2.45
378 ÷ (99 − 117 )
= 8.65+9.87-2.45-4.11 Sol:17. (b) 5 5
378 ÷ 378 =1
= 11.96 or 12 ⇒ 24 11 1 5 1
5 ÷ [ 5 − 2 { 4 − 20 }] 5 5
⇒ 24 11 1
5 ÷ [ 5 − 2 × 5]
6

Sol:10. (a) ⇒ 24 ÷ 8 Sol:25. (d)


5 5
[5 1 ÷ 5 of (5 1 + 2 1 ) + 2 ÷ 3 1
2 36
2 =3 9 10 5 5 5
4 13 2
9 ÷ ( 11
4 − 6
)] 3
÷ [7 11 of 8 54 ÷ 1 75 − 34 ] 73 73 1
36 ÷ 18 + 8
1 1
16 ÷ (36 × 36) = 1/81 Sol:18.(d) 2 + 8 = 5/8
309 ÷ [( 23 ) of (25 + 35) − 12 43 ].
Sol:11. (c) 309 ÷ [90- 51 Sol 26. (b)
4 ]

e
√15625 - √x = 4
3
309 ÷ 309 [(3+5-4)+(17-3 × 4)]+[4 ÷ 2-16 ÷
4
25 - √x = 4 =4 4+3]
21 = √x ⇒ [(3+5-4)+(17-12)]+[2-4+3]
x = 441 ⇒ [(8-4)+(5)]+[5-4]

l
Sol:19. (a)
1 43 − [3 81 ÷ {6 − (2 43 − 12
11 )}] ⇒ [(4)+(5)]+[1]
Sol:12.(c)

11 × 2]
ac
27 + [3(50 - 20) + 168 ÷ 4 + 2 -

27+ [90+42+2-22]
27+112
⇒ 47 − [ 25



7
4
7
4
7
4
7
− [ 25
− [ 25
− [ 25
8 ×
3
50
8 ÷ { 12 ]
11

22

12
11
8 ÷ {6 − ( 4 − 12 )}]

8 ÷ {6 − ( 12 )}]

50 ]
⇒ 10

Sol 27. (b)


3 31 − [ 49 + { 45 − 13
1 × (5

⇒ 103 − [ 49 + { 45 − 13
1
2 − 3 )}]
1 × ( 15 − 2 )}]
6
= 139 ⇒ − 10
4 4 ⇒ 3 − [ 49 + { 45 − 13
1 ×( 13
6 )}]
=1 10
⇒ 3 − [ 49 + { 45 − 61 }]
Sol:13. (b)
nn
10
Sol:20. (d)
⇒ 3 − [ 49 + { 1512− 2 }]
225 - [42 - {25 - (18 -
10
18 + 13)}]. [7 + 7 × (7 + 7 ÷ 7 )] + 7 ÷ 7. ⇒ 3 − [ 49 + { 12
13 }]

10
225 - [42 - {25+13} [7+7 × 8]+1 ⇒ 3 − [ 2712+ 13 ]

10 40
221 64 ⇒ 3 −[ 12 ]
10 10
⇒ 3 −[ 3 ]
Sol:14. (b) Sol:21.(d) ⇒ 0
1 1
72-3(2+24 ÷ 4 × 3-2 × 2) + 8 4 5− 8
5 ÷ 3 41 of 8
13 − 1 1 × 5 51 + 5
6
5+ 8
72-3(2+6 × 3-4)+8
Pi

= 54 ÷ 2 − 18 3 × 26 + 5 Sol 28. (d)


5 6 (132 ÷ 12 × x − 3 × 3 )
72-3(2+18-4)+8 =1
= 52 - 56 + 65 52 − 6 × 4 + x2
72-48+8
= 12−36−125 ⇒ (11 ×x−9)
25 − 24 + x2
=1
= 32 30
1 (11 × x − 9 )
= 30
⇒ 1 + x2 =1
Sol:15. (b) ⇒ 11 × x − 9 = 1 + x2
10-[121 ÷ (11 × Sol:22. (b) ⇒ x2 - 11x + 10 = 0
11)-(-4)-{3-(8-1)}] − 77 + 8 00 ÷ [83 − {8 × 9 + (18 − 3 × 5 )}] ⇒ x2 - x - 10x + 10 = 0
10-[121 ÷ 121+4+4] − 77 + 8 00 ÷ [83 − 7 5] ⇒ x(x-1) - 10(x - 1) = 0
10-[1+4+4] − 77 + 800 ÷ 8 ⇒ (x - 1)(x - 10) = 0
10-9 = 23 ⇒ x = 1, 10
=1 x = 1 satisfies.
Sol:23. (a)
Sol:16. (b) Sol 29. (a)

www.ssccglpinnacle.comsupport@ssccglpinnacle.com Ph. 09729327755, 09817390373


80
Days 8-12 : Simpli ication

s
s se
la
_c
ob
x =[ ( 52 of 6 41 ÷ 73 )of 1 72 is (2√3 + 2 − 2√2 ) (√3 − 1 ) => 10 = 1 91 = 1.1
bo
⇒ 9
ah 3−1
sm
divided by 11 41 ]2
ur

(2√3 + 2 − 2√2 ) (√3 − 1 )



yo
e/

⇒ x = [( 52 of 6 41 ÷ 73 )of 1 72 ÷ 11 41 2 Sol:36.(c)
t.m

⇒ ( √3 + 1 - √2 )( √3 - 1)
://

3+2√2
tp

]2 On rationalising
ht

3−2√2
⇒ x = [( 52 × 25
4 ÷ 3
7) × 9
7 ÷ 45
4
⇒3+ √3 - √6 - √3 - 1 + √2 3+2√2 3+2√2
⇒ 2 + √2 - √6 = × = 9 + 8 + 12√2
]2 3−2√2 3+2√2
⇒ x = [( 52 × 25
4 × 7
3 )× 9
7 × 4
45 a + b√2 + c√3 − d√6 On rationalising
3−2√2
3+2√2
]2 a = 2; b = 1; c = 0; d = 1
3−2√2 3−2√2
⇒x=[ 2 ]2 a+b+c+d=2+1+0+1=4 × = 9 + 8 − 12√2
3 3+2√2 3−2√2
⇒x= 4 On rationalising 1
9 4−√8
81x = 81 × 4 = 36 Sol 32. (c)
9 1 4+√8 4+√8
8 − [8 − (5 + 8) − {8 − × = 8
4−√8 4+√8
(8 − 5 + 8)} +10]
Sol 30. (a)
⇒ 8 − [8 − (13) − {8 − (11)} +10] 4+√8
[{( 8 + 17 + 12√2 − 17 + 12√2
⇒ 8 − [8 − 13 − {8 − 11} +10]

e
−3 2 −6
2 3 (2x+3) [{( 32 ) (3x + 7) 4+√8
3) } ] = } ] + 24√2
4 3 5
⇒ 8 − [8 − 13 − {− 3 } +10] 8
⇒ [{(
3 2 6
⇒ 8 − [8 − 13 + 3 +10] 1
2 + √42 + 24√2
2 3 (2x+3) [{( 32 ) 3 }(3x + 7) ]
3) } ] = 4 5
⇒ 8 − [21 − 13 ] 1 97√2
2 + 4

l
⇒ [{( ⇒ 8 − [8 ] 1
3 (2x + 3) 2 6 (3x + 7) On comparing a = 2 , b= 97
4
2 3 [{( 32 ) 3 } ⇒0
3) } ] = ]
4 5

⇒ [{(
2 3
3) }
⇒ [{(
2
3)
4
4

3(6x + 9)
ac
6x + 9
] = [{( 32 ) 3 }

} ] = [{( 32 )
2 18x + 42
5

2(18x + 42)
3×5
]

}]
Sol 33. (d)
25 - [16- {14- [18 - 8 + 3] }]
⇒ 25 - [16- {14- [18 - 11] }]
⇒ 25 - [16- {14- 7}]
3a+4b= 3 ×

Sol:37.(a)
1
2 +4 × 97
4 = 98 21

[ 74 of 2 54 × 1 32 − (3 21 − 2 61 )] ÷
(3 51 ÷ 4 21 of 5 31 )
⇒ [{( ⇒ 25- [16- 7] ⇒ [ 74 of 14
5 ×
5 7
3 − (2 −
13 )] ÷ ( 16 ÷ 9 of 16 )
6 5 2 3
2
18x + 27
} ] = [{( 32 )
36x + 84
⇒ 25- [9] ⇒ [ 74 × 14 5
5 × 3 − (2 −
7 13 )] ÷ ( 16 ÷ 24)
3) }]
4 15 6 5
nn
⇒ 18x + 27 = 36x + 84 ⇒ 16 ⇒ [ 74 × 14 5
5 × 3 − (2 −
7 13 )] ÷ ( 16 × 1 )
6 5 24
4 15
⇒ 15 (18x + 27) = 4 (36x + 84) ⇒ [ 74 × 14 5
5 × 3 − (2 −
7 13 )] ÷ ( 2 )
6 15
Sol:34. (d) [ 74 × 14 5 7 13 )] × ( 15 )
5 × 3 − (2 −
⇒ 270x + 405 = 144x + 336 ⇒ 6 2
7+[44 ÷ 4 + {9 × 2 − 14 ÷ 7 } + 5 [ 74 × 14 5 8 ( 15
5 × 3 − 6] × )
⇒ 270x - 144x = 336 - 405 ⇒ 2
× 2]
⇒ 126x = -69 ⇒ [ 58 × 35 − 68 ] × ( 15
2 )
69
⇒ x = - 126 7+[11+{16}+10]
⇒ [ 38 − 68 ] × ( 15
2 )
44
√2 − 42x = √2 − 42(− 126
69 ⇒ 8 15
[ 6 ] × ( 2 ) = 10
) =
Pi

SSC CGL-2019 TIER-II


√2 + 693 = √2 + 23 = √25 = 5 Sol:35.(c) Sol.38.(c)
=> 2.4 = 2 94 = 22
9 √1875 = 25√3
Sol 31. (c)
4 = a + b√2 + c√3 − d√6
=> 0.6 = 2
3 √3888 = 36√3
1+√2+√3 => 0.16 = 1
6
√1200 = 20√3
⇒ 4 × √3 + 1 − √2 => 0.27 = 3 √768 = 16√3
1+√2+√3 √3 + 1 − √2 11
⇒ => 0.83 = 5
6
√175 = 5√7
4√3 + 4 − 4√2
(2.4 × 0.6 × 3 × 0.16) × √1792 = 16√7
√3 + √6 + 3 + 1 + √2 + √3 − √2 − 2 − √6 25√3 16√3
× √ =
5 7 25
[0.27 × (0.83 ÷ 0.16)] ×
⇒ 4√3 + 4 − 4√2 36√3 20√3 16√7 144
2 √3 + 2
=> ( 22 2
9 × 3 ×3× 6 )
1
√x = 5
12
2√3 + 2 − 2√2
⇒ × √3 − 1 3 × 5 × 6]
× [ 11
√3 + 1 √3 − 1 6 1
22 15
=> ( 27 ) × ( 11 )

www.ssccglpinnacle.comsupport@ssccglpinnacle.com Ph. 09729327755, 09817390373


81
Days 8-12 : Simpli ication

s
sse
la
_c
ob
Sol:39.(c) 1 1 1 × 41 )
4(2 ÷ √4
bo
8
ah 15(√10+√5)
sm
⇒ 3 51 ÷ 4 21 of 5 31 + 81 ÷ 21 of 1
− 1
=2
ur

4 4
√10+√20+√40−√5−√80
yo
e/

⇒ 3 51 ÷ 24 + 81 ÷ 81 − 41
t.m

15√5(√2+1)
://

=
tp

√ √2+4+2√2−1−4) ⇒ 3 51 ÷ 24 + 1 − 41
ht

5(
Sol:49.(b)
= √
15( 2+1) 2 3
⇒ 15 + 4
(3√2−3)
5- √ -
8+2 15 1
⇒ 53 4 on
5(√2+1) 60 8+2√15
= ( on rationalising
(√2−1) rationalisation
= 5(3 + 2√2) Sol:45.(a) 5 - 4√ - 1 ×
8+2 15 8 − 2√15
8+2√15 8 − 2√15
4 ÷ 12 of [3 ÷ 4 of {(4 − 2) × 6 ÷ 2}]
8+2√15 8 − 2√15
−2×6÷8+3 5- 4 - 4
5- 4 = 1
Sol:40.(a) = 4 ÷ 12 of [3 ÷ 4 of {(2) × 3}] − 4
3 ÷ 18 of 3 × 6 + 21 × 6 ÷ 18 − 1.5 + 3
3 ÷ 2 + 3 − 3 ÷ 9 of 3 × 9 = 4 ÷ 12 of [3 ÷ 4 of 6] − 1.5 + 3 Sol:50.(c)
⇒ 3 ÷ 54 × 6 + 21 × 6 ÷ 18 − = 4 ÷ 12 of [ 81 ] − 1.5 + 3 5-[96 ÷ 4 of 3 -(16-55 ÷ 5 )]

e
3 ÷ 2 + 3 − 3 ÷ 27 × 9 = 4 ÷ 1.5 − 1.5 + 3 = 5-[96 ÷ 12 -(5)]
⇒ 31 + 7 − 23 + 3 − 1 8 25 = 5-[8 -(5)]
3 + 1.5 = 6
⇒ 31
− 5 = 47 = 5-3 = 2
3 2 6
Sol:46.(a)

l
Sol:41.(a) 7+3√5 SSC CPO-2019
× −√ − − √ × √
3 5 7 3 5 3+ 5
2 2 3+√5 3−√5 3−√5 3+√5 Sol:51.(d)

=
÷
=
0.0203×2.92
0.7×0.0365×2.9
ac ÷

7×365×29×10×10000×10
(12.12.+8.12)(12.12−8.12)
0.25×0.25+ 0.25×19.99
203×292 ÷
7×365×29
(12.12) −(8.12)
(0.25) 2 +(0.25) (19.99)
203×292 ×10000×100

(20.24)(4)
0.25(0.25+ 19.99)
=
21−7√5+9√5−15

= √4 + √4 =
2 5 2 5

= √5 = 2.2360
4 -

Option a is correct
21+7√5−9√5−15

√5
4
8-3 ÷ 6 of 2+(4 ÷ 4 of
×8÷ 1
4
Using BODMAS
)× 1
8

= 8-3 ÷ 12 + (4 ÷ 1 ) ÷ 8 + (4 × 32)
1
4 ) ÷ 8 + (4

4 20.24×0.25 × 81
= 5 × 20.24×4 = 0.05
= 8- 41 + 21 +128 × 1
8
Sol:47.(a)
Rearranging it we get
nn
Sol:42.(c) 8+2√3
on rationalising
3√3+5
Let x = 0.57
8+2√3
× 3√3−5 24+ 41
Then 100x = 57.57 3√3+5 3√3−5 96+1
2(4 +√3)× 3√3−5 4
99x = 57 97
2
19
x = 33 4
(4 + √3) × (3√3 − 5)
Similarly y = 214

16
495
12√3 − 20 + 9 − 5√3 Sol:52.(a)
And z = 45 40− 34 of 32
7√3 − 11
19 - 214 16 247 37− 43 of (34 − 6 )
495 + 45 = = 0.4 98
Pi

33 495 a = 7 and b = 11
Using Bodmas we get
a + b =18 40−24
Sol:43.(d) 37− 43 of (28)

√11 − 3√8 = a + b√2,


16
37−21
Sol:48.(a) 16

√9 + 2 − 6√2 = a + b√2,
16

√ √
=1
3 − √2 = a + b√2, − √3 + 3+8 √ 7 + 4√3
a=3 Sol:53.(d)
b = -1
2a + 3b = 6 - 3 = 3 √ − √3 +
√ 3+8 √ (2 + √3)
2 (5 41 ÷ 73 of 21 ) ÷ (5 91 − 7 87 ÷ 9 20
(5 ÷ 2of 21 )
9 )× 11
21 −

Sol:44.(a) √ √19 + 8√3


− √3 + = ( 21
4 ÷
(5 ÷ 1)
3
14 ) ÷ ( 46
9 −
63
8 ÷ 189 )
20 × 11
21 −

3 51 ÷ 4 21 of 5 31 +
√− √3 + √(4 + √3)
1 ÷ 1 of 1
8 2 4 − 2
= ( 21
4 ÷
3
14 ) ÷ ( 46
9 −
63
8 ÷ 189 )
20 × 11
21 −

www.ssccglpinnacle.comsupport@ssccglpinnacle.com Ph. 09729327755, 09817390373


82
Days 8-12 : Simpli ication

s
se
s
la
_c
ob
(5 ÷ 1)
bo
ah
= 49 46 5 11 Sol:59.(a)
sm
2 ÷ ( 9 − 6 ) × 21 − 5
ur
yo

40+ 34 of 32
= 49 77 11
e/

2 ÷ ( 18 ) × 21 − 5
t.m

37+ 34 of (34−6)
://
tp

= 63 × 11
−5
ht

11 21 = 40+ 24
37+ 34 of (28)
=3 - 5 = -2 64
=37+21
64
= 58
Sol:54.(b)
32
= 29
3 31 ÷ 2 21 of 1 53 + ( 83 + 1 3
7 × 14)
10 3
=1 29
= 3 ÷ 25 of 8
5 + ( 83 + 1 7
7 × 4)
= 10
3 ÷4+ ( 83 + 41 )
10
= 12 + 5 Sol:60.(b)
8
35 1 − 3 ÷ 6 of 2 + (4 ÷ 4 of 41 ) ÷ 8 +
= 24
(4 × 8 ÷ 41 ) × 1
8

Sol:55.(d ) Using Bodmas

e
= 1 − 3 ÷ 12 + (4) ÷ 8 + (4 × 32) × 1
2 31 ÷ 2 21 of 1 53 + ( 83 + 1
7 × 1 43 ) 8

= 7 5 8 3 1 × 47 ) = 1 − 41 + 21 + (4 × 32) × 1
3 ÷ 2 of 5 + ( 8 + 7 8

= 7 3 1 = 1+ 41 +16
3 ÷ 4 + (8 + 4)
= 7 5 = 17+ 41
12 + 8

l
29 = 68+1
= 24 4

Sol:56.(b)
ac
(5 ÷ 2of 21 ) + (5 41 ÷ 73 of 21 ) ÷ (5 91 −
9 ) × 11
7 87 ÷ 9 20
= (5 ÷ 1) + ( 21
21

4 ÷
3
14 ) ÷ ( 46
9 −
63
8 ÷ 189 )
20
=

2
69
4

Sol:61.(c)
2 3

2
4

4
4

1
1

8
8

3
3
3 ÷ 10 of 9 − 5 × 1 9 ÷ 15 + 4 ÷
1
3 ÷ 15 − 5 × 1 9 ÷ 15 + 4 ÷ 2
1
2

× 11 5- 54 × 150
72 + 3
2
21
= 5 + ( 49 46 5 11 5- 35 + 3
2 ) ÷ ( 9 − 6) × 21 2
nn
= 5 + 49 77 11 = 29
2 ÷ 18 × 21 6

=5+ 49 × 18 × 11
2 77 21
Sol:62.(d)
=5 + 3 = 8
7 ÷ [5 + 1 ÷ 2 − {4 + (4 of 2 ÷ 4) +
(5 ÷ 5 of 2)}]
Sol:57.(a)
2 3 4 4 1 8 3 3 1 = 7 ÷ [5 + 1 ÷ 2 − {4 + (8 ÷ 4) + (5 ÷ 10)}]
3 ÷ 10 of 9 − 5 × 1 9 ÷ 15 − 4 + 4 ÷ 2
= 7 ÷ [5 + 0.5 − {4 + 2 + 0.5}]
Using BODMAS
2 12 4 10 8 3 3 1 = 7 ÷ [5 + 0.5 − 6.5]
Pi

3 ÷ 90 − 5 × 9 ÷ 15 − 4 + 4 ÷ 2
= 7 ÷ [− 1]
= 5- 54 × 150
72 − 43 + 23
= -7
= 5- 35 + 43
= 60−20+9
12
= 49
12

Sol:58.(a)
− 7 ÷ [5 + 1 ÷ 2 − {4 + (4of 2 ÷ 4) +
(4 ÷ 4of 2)}]
Using bodmas we get
− 7 ÷ [5 + 0.5 − {4 + (2) + (0.5)}]
− 7 ÷ [5.5 − {6.5)}]
7

www.ssccglpinnacle.comsupport@ssccglpinnacle.com Ph. 09729327755, 09817390373


83
Days 13-16 Average

s
sse
la
_c
ob
bo
ah
Key Points:/ मुख िबंदु: यिद सभी नंबरों को 'a' से िवभािजत last five numbers is 40 and that of
sm
ur
yo

1) An average or arithmetic mean िकया जाता है तो उनके औसत को 'a' the first four numbers is 44. The
e/
t.m

से िवभािजत िकया जाना चािहए।


://

of given data is the sum of the 6th number is 6 less than the fifth
tp
ht

given observations divided by the and 5 less than the 7th number.
number of observations. The average of the 5th and the 7th
6) Average of first n natural
िदए गए आं कड़ों का औसत या numbers is:
अंकगिणत मा , आं कड़ों के योग numbers / थम n ाकृितक
बारह सं ाओं का औसत 42 है |
को आं कड़ों की सं ा ारा ( n+1
सं ाओं का औसत = 2 ) अंितम पां च सं ाओं का औसत 40
िवभािजत करके ा िकया जाता है |
तथा पहली चार सं ाओं का औसत
7) Average of first n even
44 है | छठी सं ा पाँ चवीं से 6 कम
For example : If we have to find numbers / थम n सम सं ाओं का है तथा 7वीं सं ा से 5 कम है | 5वीं
out the average of 10, 15, 25 and
30, then required average will be औसत = (n + 1) और 7वीं सं ा का औसत है :
equal to: SSC CGL - 4 June 2019
8) Average of first n odd
उदाहरण : अगर हम 10, 15, 25 (Morning)
numbers/ थम n िवषम सं ाओं

e
और 30 की औसत पता लगाना है तो (a) 44
आव क औसत होगा: का औसत = n (b) 44.5
10+15+25+30 = 80 = 20. (c) 43
4 4
9) Average of consecutive (d) 43.5
Therefore, we can say, Average सं ाओं का

l
numbers/लगातार
(A) = Sum of the given observations (S)
Q2. The average weight of a
N umber of Observations (N )
ac औसत = F irst number+Last number
2
इसिलए, हम कह सकते ह, औसत = certain number of students in a
दि◌ए गए आ◌ं कड◌़ ◌ो◌ं क◌ा य◌ोग 10) Average of 1 to n odd class is 68.5 kg. If 4 new students
आ◌ं कड◌़ ◌ो◌ं क◌ी स◌ं ख◌्य◌ा
numbers/1 से n िवषम सं ाओं का having weights 72.2 kg, 70.8kg,
70.3kg and 66.7 kg join the class,
2) If all the numbers increase by औसत = Last odd number+1
2
then the average weight of all the
‘a’ then the average of the
11) Average of 1 to n even students increases by 300 g. The
numbers will also increase by ‘a’.
numbers/1 से n सम सं ाओं का number of students in the class,
यिद सभी सं ाएं 'a' से बढ़ती ह तो initially is:
nn
सं ाओं का औसत भी 'a' से बढ़े गा। औसत = Last even number+2
2 िकसी क ा म छा ों की एक िनि त
12) Average of squares of first n सं ा का औसत वज़न 68.5 िकलो
3) If all the numbers decrease by ाम है | यिद 4 नए छा क ा म आ
natural numbers/ थम n ाकृितक
‘a’ then the average of the जाते ह, िजनका वज़न मशः 72.2
numbers will also decrease by ‘a’. सं ाओं के वग का औसत = िकलो ाम, 70.8 िकलो ाम, 70.3
अगर सभी नंबर 'a' से घटते ह तो (n+1)(2n+1) िकलो ाम तथा 66.7 िकलो ाम है ,
6
सं ाओं का औसत भी 'a' से कम तो सभी छा ों का औसत वज़न 300
13) Average of cubes of first n ाम से बढ़ जाता है | आरं भ म क ा
होगा।
Pi

natural numbers/ थम n ाकृितक म छा ों की सं ा िकतनी थी ?


सं ाओं के घन का औसत = SSC CGL - 4 June 2019
4) If all the numbers are
(Afternoon)
multiplied by ‘a’ then their n(n+1)
2

4 (a) 21
average must also be multiplied
(b) 16
by ‘a’. 14) Average of n multiples of any
(c) 11
यिद सभी नंबरों को 'a' से गुणा िकया number/िकसी भी सं ा के n (d) 26
जाता है तो उनके औसत को भी 'a' से number×(n+1)
गुणां क का औसत =
गुणा िकया जाना चािहए। 2
Q3. Three numbers are such that
if the average of any two of them
5) If all the numbers are divided Variety Questions is added to the third number, the
by ‘a’ then their average must sums obtained are 168, 174 and
also be divided by ‘a’. Q1. The average of twelve 180 respectively. What is the
numbers is 42. The average of the

www.ssccglpinnacle.com support@ssccglpinnacle.com Ph. 09729327755, 09817390373 84


/
Days 13-16 Average

s
sse
la
_c
ob
average of the original three greatest number among the given (d) 49.5
bo
ah
numbers? numbers is:
sm
ur
yo

तीन सं ाएँ इस कार ह िक यिद चार अलग-अलग धना क सं ाओं Q8. The average of 27 numbers is
e/
t.m

इनम से िकसी भी दो सं ा के को आरोही म म िलखा गया है |


://

zero. Out of them, how many


tp
ht

औसत को तीसरी सं ा म जोड़ा सभी चार सं ाओं की औसत का may be greater than zero, at the
जाए, तो ा होने वाले योगफल एक-ितहाई इनम से सबसे बड़ी most?
मशः 168, 174 और 180 होते ह | सं ा से 19 कम है | यिद पहली तीन 27 सं ाओं का औसत 0 है | इनम
इन तीन ारं िभक सं ाओं का सं ाओं का औसत 12 है , तो इनम से, अिधक से अिधक िकतनी सं ाएँ
औसत ात कर | से सबसे बड़ी सं ा ात कर | शू से अिधक सकती ह ?
SSC CGL - 4 June 2019 SSC CGL - 7 June 2019 SSC CGL - 13 June 2019
(Evening) (Evening) (Evening)
(a) 86 (a) 25 (a) 0
(b) 87 (b) 22 (b) 15
(c) 89 (c) 24 (c) 26
(d) 84 (d) 21 (d) 20

e
Q4. The total number of students Q6. The average marks of 40 Q9. 10 years ago, the average age
in section A and B of a class is students was found to be 68. If of a family of five members was
110. The number of students in the marks of two students were 38 years. Now, two new members

l
section A is 10 more than that of incorrectly entered as 48 and 64 join, whose age difference is 8
section B. The average score of instead of 84 and 46 respectively, years. If the present average age
ac
the students in B, in a test, is 20%
more than that of students in A. If
the average score of all the
students in the class is 72, then
what is the average score of the
then what is the correct average?
40 छा ों के औसत अंक 68 पाए गए |
यिद दो छा ों के अंक भूलवश 84 एवं
46 के बजाय मशः 48 और 64 के
प म शािमल िकये गए ह, तो सही
of the family is the same as it was
10 years ago, what is the age (in
years) of the new younger
member?
10 वष पहले, पां च सद ों के एक
students in A? औसत ात कर | प रवार की औसत उ 38 वष थी |
िकसी क ा के खंड A और खंड B SSC CGL - 10 June 2019 अब दो नए सद शािमल हो गए ह,
के छा ों की कुल सं ा 110 है | खंड (Afternoon) िजनकी उ म 8 वष का अंतर है |
nn
A म छा ों की सं ा खंड B के छा ों (a) 68.25 यिद प रवार की वतमान औसत उ
की सं ा से 10 अिधक है | िकसी (b) 68.15 उतनी ही है िजतनी यह 10 साल
परी ा म B के छा ों का औसत (c) 68.45 पहले थी, तो नए छोटे सद की उ
ा ां क A के छा ों के औसत (d) 68.35 ात कर |
ा ां क से 20% अिधक है | यिद SSC CHSL - 4 July 2019
सभी छा ों का औसत ा ां क 72 है , Q7. In a class of 40 students, 45% (Afternoon)
तो A के छा ों का औसत ा ां क are girls and the remaining are (a) 15
ात कर | boys. If the average of the girls (b) 9
Pi

SSC CGL - 7 June 2019 marks is 54 and that of the boys is (c) 10
(Afternoon) 46, what is the average of the (d) 17
(a) 66 whole class?
(b) 68 40 छा ों की एक क ा म, 45% Q10. The average of a number
(c) 63 लड़िकयाँ ह तथा शेष लड़के ह | यिद and its reciprocal is 4. The
(d) 70 लड़िकयों के अंकों का औसत 54 average of its cube and its
और लड़कों के अंकों का औसत 46 reciprocal is equal to: िकसी सं ा
Q5. Four different positive है , तो पूरी क ा का औसत अंक ात तथा उसके पार रक ( reciprocal
numbers are written in ascending कर | ) का औसत 4 है | इसके घन और
order. One-third of the average of SSC CGL - 11 June 2019 और उसके पार रक का औसत
all the four numbers is 19 less (Afternoon) िकसके बराबर होगा ?
than the greatest of these (a) 49.8 SSC CHSL - 9 July 2019
numbers. If the average of the (b) 49.7 (Evening)
first three numbers is 12, the (c) 49.6 (a) 256

www.ssccglpinnacle.com support@ssccglpinnacle.com Ph. 09729327755, 09817390373 85


/
Days 13-16 Average

s
sse
la
_c
ob
(b) 142 िकतनी सं ाएँ ऋणा क हो सकती Q18. What is the average of the
bo
ah
(c) 288 ह? first 8 multiples of 6 among the
sm
ur
yo

(d) 244 SSC CHSL - 11 July 2019 natural numbers?


e/
t.m

ाकृितक सं ाओं म 6 के पहले 8


://

(Morning)
tp
ht

Q11. The average of the first (a) 100 गुणजों का औसत ा है ?


1234 _______ numbers is equal (b) 88 SSC MTS - 7 August 2019
to 1234. (c) 544 (Evening)
Fill in the blank. (d) 1087 (a) 24
पहली 1234 ______ सं ाओं का (b) 26
औसत 1234 के बराबर होता है | Q15. The average age of four (c) 27
र थान की पूित कर | brothers is 15 years. If their father (d) 28
SSC CHSL - 10 July 2019 is included, the average is
(Morning) increased by 5 years. The age of Q19. The average of all the prime
(a) odd/ िवषम the father (in years) is: and composite numbers upto 100
(b) even/ सम चार भाइयों की औसत आयु 15 वष है is:
(c) prime/ अभा | यिद उनके िपता को शािमल कर 100 तक की सभी अभा और

e
(d) natural/ ाकृितक िलया जाए, तो औसत 5 वष बढ़ जाता िवभा सं ाओं का औसत है :
है | िपता की उ ( वष म ) है : SSC MTS - 9 August 2019
Q12. Fill in the blank. SSC CHSL - 5 July 2019 (Morning)
र थान की पूित कर |

l
(Afternoon) (a) 51
The average of the first 101 (a) 35 (b) 50
ac
______ numbers is equal to 102.
पहली 101 ______ सं ाओं का
औसत 102 के बराबर होगा |
SSC CHSL - 10 July 2019
(Afternoon)
(b) 40
(c) 38
(d) 36

Q16. The average of the squares


(c) 50.5
(d) 49.5

Q20. The average weight of 12


articles is 18 kg. Addition of
(a) natural/ ाकृितक of numbers 1 to 5 is: another new article reduces the
(b) odd/ िवषम 1 से 5 तक की सं ाओं के वग का average weight by 500g. What is
(c) even/ सम औसत ात कर | the weight of the new article?
nn
(d) perfect square/ पूण वग SSC CPO - 16 March 2019 12 व ुओं का औसत वज़न 18
(Afternoon) िकलो ाम है | एक अ नयी व ु
Q13. The difference between the (a)11 को शािमल करने से औसत वज़न
average of first ten prime (b)5 500 ाम कम हो जाता है | नयी व ु
numbers and the first ten prime (c)8 का वज़न ात कर |
numbers of two digits is: (d)9 SSC MTS - 9 August 2019
पहली दस अभा सं ाओं के (Morning)
औसत और पहली दस दो अंकों की Q17. The average of a and b is (a) 11.5 kg
Pi

अभा सं ाओं के औसत म अंतर 36. The average of b and c is 42. (b) 15.0 kg
होगा : What is the difference between c (c) 11.0 kg
SSC CHSL - 10 July 2019 and a ? (d) 10.1 kg
(Evening) a तथा b का औसत 36 है | b तथा c
(a) 14.5 का औसत 42 है | c तथा a के बीच Q21. Given four different
(b) 16.5 िकतना अंतर् है ? numbers, the average of first
(c) 12.5 SSC MTS - 6 August 2019 three numbers is four times the
(d) 13.5 (Afternoon) fourth number and the average of
(a) 18 all the four numbers is 52. What
Q14. The average of 1088 real (b) 12 is the average of the first three
numbers is zero. At most how (c) 16 number?
many of them can be negative? (d) 14 चार अलग-अलग सं ाएँ दी गयी ह,
1088 वा िवक सं ाओं का औसत पहली तीन सं ाओं का औसत चौथी
शू है | उनम अिधक से अिधक सं ा से चार गुना है तथा सभी चार

www.ssccglpinnacle.com support@ssccglpinnacle.com Ph. 09729327755, 09817390373 86


/
Days 13-16 Average

s
sse
la
_c
ob
सं ाओं का औसत 52 है | पहली थम 15 पूण सं ाओं का औसत Q2. The number of students in a
bo
ah
तीन सं ाओं का औसत ा है ? ात कर | class is 75, out of which 33 31 %
sm
ur
yo

SSC CHSL - 3 July 2019 SSC MTS - 8 August 2019 are boys and the rest are girls. The
e/
t.m
://

(Afternoon) (Morning) average score in mathematics of


tp
ht

(a) 39 (a) 8 the boys is 66 32 % more than that


(b) 65 (b) 7 of the girls. If the average score of
(c) 70 (c) 9 all the students is 66, then the
(d) 64 (d) 10 average score of the girls is:
एक क ा म 75 छा ह, िजनम से 33
Q22. The average of all prime Q26. Out of four numbers the
3 % लड़के ह और शे ष लड़िकयाँ ह |
1
numbers 21 to 50 is (round off to average of the first three is 16 and
लड़कों का गिणत म औसत अंक
one decimal number) that of the last three is 15. If the
लड़िकयों के गिणत म औसत अंक से
21 से 50 तक की सभी अभा last number is 21 then the first
66 32 % अिधक है | यिद सभी छा ों
सं ाओं का औसत ात कर | (एक number is: चार सं ाओं म से
का औसत अंक 66 है , तो लड़िकयों
दशमलव सं ा तक पूणाक कर ) पहली तीन सं ाओं का औसत 16 है
का औसत अंक ात कर |
और अंितम तीन सं ाओं का औसत

e
SSC CPO - 16 March 2019
SSC CGL Tier 2 - 11 September
(Morning) 15 है | यिद अंितम सं ा 21 है , तो
2019
(a) 35.9 पहली सं ा है :
(a) 52
(b) 34.8 SSC MTS - 13 August 2019
(b) 55

l
(c) 33.7 (Evening)
(c) 54
(d) 32.9 (a) 28
ac
Q23. What is the average of 59,
63, 68, 77, 74 and 73, when each
number is divided by 23 ?
59, 63, 68, 77, 74 तथा 73 का
(b) 22
(c) 21
(d) 24

SSC CGL TIER II


(d) 58

Q3.The average of 33 numbers is


74. The average of the first 17
numbers is 72.8 and that of the
last 17 numbers is 77.2. If the
औसत िकतना है , जब ेक सं ा
17th number is excluded, then
को 23 से िवभािजत िकया जाता है ? Q1.The average of thirteen
what will be the average of the
SSC MTS - 6 August 2019 numbers is 47. The average of the
nn
remaining numbers (correct to
(Afternoon) first three numbers is 39 and that
63 one decimal place)?
(a) 23 of the next seven numbers is 49.
33 सं ाओं का औसत 74 है | पहली
(b) 67 The 11th number is two times the
23 17 सं ाओं का औसत 72.8 है तथा
(c) 3 12th number and 12th number is
अंितम 17 सं ाओं का औसत 77.2
(d) 46 3 less than the 13th number. What
है | यिद 17वीं सं ा हटा दी जाए, तो
is the average of 11th and 13th
शेष सं ाओं का औसत ा होगा ?
Q24. What is the average of the numbers?
( एक दशमलव थान तक )
तेरह सं ाओं का औसत 47 है |
Pi

first 15 odd numbers among the SSC CGL Tier 2 - 12


natural numbers? पहली तीन सं ाओं का औसत 39 है
September 2019
ाकृितक सं ाओं म पहली 15 तथा अगली सात सं ाओं का औसत
(a)72.9
िवषम सं ाओं का औसत ा है ? 49 है | 11वीं सं ा 12वीं सं ा से
(b) 73.4
SSC MTS - 8 August 2019 दोगुनी है तथा 12वीं सं ा 13वीं
(c) 71.6
(Evening) सं ा से 3 कम है | 11वीं और 13वीं
(d) 70.8
(a) 18 सं ाओं का औसत ात कर |
(b) 15 SSC CGL Tier 2 - 11
Q4. The average weight of a
(c) 16 September 2019
certain number of students in a
(d) 17 (a)54.5
group is 72 kg. If 10 students
(b) 57
having an average weight of 78
Q25. What is the average of the (c) 56
kg leave and 4 students having an
first 15 whole numbers? (d) 55.5
average weight of 80 kg join the
group, the average weight of the

www.ssccglpinnacle.com support@ssccglpinnacle.com Ph. 09729327755, 09817390373 87


/
Days 13-16 Average

s
sse
la
_c
ob
students in the group decreases by औसत एक से बढ़ जाता है | x का SSC CGL - 6 June 2019
bo
ah
0.7 kg. The number of students मान है : (Afternoon)
sm
ur
yo

initially in the group is : SSC CGL Tier 2 - 13 (a) 43.5


e/
t.m

िनि त सं ा वाले एक समूह के


://

September 2019 (b) 43


tp
ht

छा ों का औसत वज़न 72 िकलो ाम (a) 40 (c) 44.5


है | यिद 78 िकलो ाम औसत वज़न (b) 41.5 (d) 44
वाले 10 छा चले जाते ह और 80 (c) 42
िकलो ाम औसत वज़न वाले 4 छा (d) 38.5 Q3. The average of eleven
आ जाते ह, तो समूह म छा ों का numbers is 54. The average of the
औसत वज़न 0.7 िकलो ाम से कम Practice Questions first four numbers is 48 and that
हो जाता है | आरं भ म समूह म छा ों of the next four numbers is 25%
की सं ा थी : Q1. The average of thirteen more than the average of the first
SSC CGL Tier 2 - 12 numbers is 80. The average of the four. The ninth number is 8
September 2019 first five numbers is 74.5 and that greater than the 11th number and
(a) 56 of the next five numbers is 82.5. the tenth number is 4 greater than

e
(b) 46 The 11th number is 6 more than the 11th number. What is the
(c) 44 the 12th number and the 12th average of the 9th and the 10th
(d) 54 number is 6 less than the 13th numbers?
number. What is the average of 11 सं ाओं का औसत 54 है | पहली
चार सं ाओं का औसत 48 है तथा

l
Q5.The average age of 120 the 11th and the 13th numbers?
students in a group is 13.56 years, तेरह सं ाओं का औसत 80 है | अगली चार सं ाओं का औसत
ac
35% of the number of students
are girls and the rest are boys. If
the ratio of the average age of
boys and girls is 6:5, then what is
the average age (in years) of the
पहली पां च सं ाओं का औसत 74.5
और अगली पां च सं ाओं का औसत
82.5 है | 11वीं सं ा 12वीं सं ा से
6 अिधक है तथा 12वीं सं ा 13वीं
सं ा से 6 कम है | 11वीं तथा 13वीं
पहली चार सं ाओं के औसत से
25% अिधक है | 9वीं सं ा 11वीं
सं ा से 8 अिधक है तथा दसवीं
सं ा 11वीं सं ा से 4 अिधक है |
9वीं और 10वीं सं ाओं का औसत
girls? सं ाओं का औसत ात कर | ात कर |
एक समूह के 120 छा ों की औसत SSC CGL - 6 June 2019 SSC CGL - 6 June 2019
उ 13.56 वष है | छा ों म 35% (Morning) (Evening)
nn
लड़िकयाँ ह तथा शेष लड़के ह | यिद (a) 87 (a) 54
लड़कों और लड़िकयों की औसत (b) 86 (b) 52.6
उ का अनुपात 6 : 5 है , तो (c) 86.5 (c) 56
लड़िकयों की औसत उ ( वष म ) (d) 87.5 (d) 54.4
िकतनी है ?
SSC CGL Tier 2 - 13 Q2. The average of twelve Q4. Three numbers are such that
September 2019 numbers is 46. The average of the if the average of any two of them
(a) 12 first four numbers is 43 and that is added to the third number, the
Pi

(b) 11.6 of the last five numbers is 49.4. sums obtained are 164, 158 and
(c) 10 The 5th and the 6th numbers are 132 respectively. What is the
(d) 14.4 respectively 4 and 6 more than average of the original three
the 7th number. What is the numbers?
Q6. The average of 18 numbers is average of the 5th and the 7th तीन सं ाएँ इस कार ह िक यिद
37.5. If six numbers of average x numbers? इनम से िकसी भी दो सं ा के
are added to them, then the बारह सं ाओं का औसत 46 है | औसत को तीसरी सं ा म जोड़ा
average of all the numbers पहली चार सं ाओं का औसत 43 जाए, तो योगफल मशः 164, 158
increases by one. The value of x तथा अंितम पां च सं ाओं का औसत और 132 ा होते ह | इन तीन
is : 49.4 है | 5वीं और छठी सं ा सातवीं आरं िभक सं ाओं का औसत ात
18 सं ाओं का औसत 37.5 है | यिद सं ा से मशः 4 और 6 अिधक है | कर |
औसत x वाली छः सं ाओं को उनम 5वीं और 7 वीं सं ा का औसत ात SSC CGL - 7 June 2019
जोड़ा जाए, तो इन सभी सं ाओं का कर | (Morning)

www.ssccglpinnacle.com support@ssccglpinnacle.com Ph. 09729327755, 09817390373 88


/
Days 13-16 Average

s
se
s
la
_c
ob
(a) 75 32 Q7. The average marks of 45 (b) 58.8kg
bo
ah students was found to be 66. If (c) 59kg
sm
(b) 74
ur
yo

(c) 76 the marks of two students were (d) 58.6kg


e/
t.m
://

(d) 75 31 incorrectly entered as 28 and 64


tp
ht

instead of 82 and 46 respectively, Q10. In a class of 60 students,


Q5. The average of twelve then what is the correct average? 40% are girls. The average
numbers is 55.5. The average of 45 छा ों के औसत अंक 66 पाए गए | weight of the whole class is 59.2
the first four numbers is 53.4 and यिद दो छा ों के अंक 82 और 46 के kg and the average weight of the
that of the next four numbers is बजाय भूलवश मशः 28 और 64 girls is 55 kg. What is the average
54.6. The 10th number is greater िलखे गए थे, तो सही औसत ा होगा weight of the boys?
than the 9th number by 3 but ? 60 छा ों की एक क ा म, 40%
lesser than the 11th and 12th SSC CGL - 11 June 2019 लड़िकयाँ ह | पूरी क ा का औसत
numbers by 2 and 3, respectively. (Morning) वज़न 59.2 िकलो ाम है और
What is the average of the 10th (a) 67.2 लड़िकयों का औसत वज़न 55 िकलो
and the 12th numbers? (b) 66.8 ाम है | लड़कों का औसत वज़न ात
कर |

e
बारह सं ाओं का औसत 55.5 है | (c) 66.4
पहली चार सं ाओं का औसत 53.4 (d) 66.6 SSC CGL - 12 June 2019
है तथा अगली चार सं ाओं का (Afternoon)
औसत 54.6 है | 10वीं सं ा 9वीं Q8. In a class of 50 students, 46% (a) 63 kg

l
सं ा से 3 अिधक है लेिकन 11वीं are girls and the remaining are (b) 60 kg
और 12वीं सं ाओं से मशः 2 और boys. The average of the boys (c) 61 kg
ac
3 कम है | 10वीं और 12वीं सं ाओं
का औसत ात कर |
SSC CGL - 10 June 2019
(Morning)
(a) 59.5
marks is 58 and that of the girls is
62. What are the average marks
of the whole class?
50 छा ों की एक क ा म, 46%
लड़िकयाँ ह तथा शेष लड़के ह |
(d) 62 kg

Q11. In a class of 40 students,


60% are girls. The average of the
girls’ marks is 72 and that of the
(b) 58 लड़कों के अंकों का औसत 58 है boys is 54. What are the average
(c) 57.5 तथा लड़िकयों के अंकों का औसत marks of the whole class?
62 है | पूरी क ा का औसत अंक 40 छा ों की एक क ा म, 60%
nn
(d) 56
ात कर | लड़िकयाँ ह | लड़िकयों के अंकों का
Q6. The average marks of 50 SSC CGL - 11 June 2019 औसत 72 है तथा लड़कों के अंकों
students in a class was found to (Evening) का औसत 54 है | पूरी क ा के
be 64. If the marks of two (a) 59.84 औसत अंक िकतने ह ?
students were incorrectly entered (b) 60.65 SSC CGL - 12 June 2019
as 38 and 42 instead of 83 and 24 (c) 60.38 (Evening)
respectively, then what is the (d) 60.12 (a) 65.4
(b) 65
Pi

correct average?
िकसी क ा म 50 छा ों के औसत Q9. In a class of 60 students, 40% (c) 64.8
अंक 64 पाए गए | यिद दो छा ों के are girls. The average weight of (d) 65.2
अंक भूलवश 83 एवं 24 के बजाय the boys is 62 kg and that of the
मशः 38 और 42 िलखे गए थे, तो girls is 55 kg. What is the average Q12. In a class of 50 students,
सही औसत ात कर | weight of the whole class? 60% are boys. The average of
SSC CGL - 10 June 2019 60 छा ों की एक क ा म, 40% marks of the boys is 62, and that
(Evening) लड़िकयाँ ह | लड़कों का औसत of the girl is 68. What is the
(a) 64.54 वज़न 62 िकलो ाम तथा लड़िकयों average marks of the whole class?
(b) 62.32 का औसत वज़न 55 िकलो ाम है | 50 छा ों की एक क ा म, 60%
(c) 61.24 पूरी क ा का औसत वज़न िकतना है लड़के ह | लड़कों के औसत अंक 62
(d) 61.86 ? तथा लड़िकयों के औसत अंक 68 ह |
SSC CGL - 12 June 2019 पूरी क ा के औसत अंक ात कर |
(Morning) SSC CGL - 13 June 2019
(a) 59.2kg (Morning)

www.ssccglpinnacle.com support@ssccglpinnacle.com Ph. 09729327755, 09817390373 89


/
Days 13-16 Average

s
sse
la
_c
ob
(a) 64.8 students is 53, then what is the in A and B taken together is 58
bo
ah
(b) 64.4 average score of the students kg, then what is the average
sm
ur
yo

(c) 65.2 from village B? weight of the students in B?


e/
t.m

एक क ा म 90 छा ह, िजनम से क ा A और क ा B के कुल छा ों
://

(d) 64.6
tp
ht

70% छा गाँ व A से ह तथा अ की सं ा 96 है | A के छा ों की


Q13. In a class of 50 students, छा गाँ व B से ह | गाँ व B के छा ों सं ा B के छा ों से 40% अिधक है |
40% are girls. The average marks का औसत अंक गाँ व A के छा ों के B क ा के छा ों का औसत वज़न A
of the whole class are 64.4 and औसत अंक से 20% अिधक है | यिद के छा ों के औसत वज़न से 50%
the average of the boys’ marks is सभी छा ों का औसत अंक 53 है , तो अिधक है | यिद A और B दोनों के
62. What are the average marks गाँ व B के छा ों का औसत अंक ात छा ों का कुल औसत वज़न 58 िकलो
of the girls? कर | ाम है , तो B के छा ों का औसत
50 छा ों की एक क ा म, 40% SSC CHSL - 2 July 2019 वज़न ात कर |
लड़िकयाँ ह | पूरी क ा के औसत (Morning) SSC CHSL - 2 July 2019
अंक 64.4 ह तथा लड़कों के औसत (a) 54 (Evening)
अंक 62 ह | लड़िकयों के औसत अंक (b) 60 (a) 72 kg
ात कर |

e
(c) 64 (b) 60 kg
SSC CGL - 13 June 2019 (d) 50 (c) 48 kg
(Afternoon) (d) 66 kg
(a) 67 Q16. In a class of 80 students,

l
(b) 66.8 60% participate in games and the Q18. The average weight of the
(c) 66.4 rest do not. The average weight of students in a group was 75.4 kg.
(d) 68
ac
Q14. There are three numbers. If
the average of any two of them is
added to the third number, the
the former group is 5% more than
that of the latter. If the average
weight of all the students is 51 21
kg, then what is the average
weight (in kg) of the former
Later on, four students having
weights, 72.9 kg, 73.8 kg, 79.5 kg
and 87.4 kg joined the group. As
a result, the average weight of all
the students in the group
sums obtained are 177, 163 and group? increased by 0.24 kg. What was
138. What is the average of the 80 छा ों की एक क ा म, 60% खेलों the number of students in the
largest and the smallest of the म भाग लेते ह तथा शेष खेलों म भाग group, initially?
nn
given numbers? नहीं लेते ह | पहले समूह का औसत एक समूह के छा ों का औसत वज़न
तीन सं ाएं ह | यिद इनम से िकसी वज़न दू सरे समूह के औसत वज़न से 75.4 िकलो ाम था | बाद म, चार
भी दो सं ा के औसत को तीसरी 5% अिधक है | यिद सभी छा ों का छा इस समूह म शािमल हो गए
सं ा म जोड़ा जाता है , तो इस औसत वज़न 51 21 िकलो ाम है , तो िजनके वज़न मशः 72.9 िकलो
कार योगफल के प म मशः पहले समूह का औसत वज़न ात ाम, 73.8 िकलो ाम, 79.5 िकलो
177, 163 और 138 ा होते ह | कर | ाम और 87.4 िकलो ाम थे |
इनम से सबसे बड़ी और सबसे छोटी SSC CHSL - 2 July 2019 प रणाम प, इस समूह के सभी
सं ाओं का औसत ात कर | छा ों का औसत वज़न 0.24 िकलो
Pi

(Afternoon)
SSC CHSL - 1 July 2019 (a) 57.6 ाम से बढ़ गया | आरं भ म इस समूह
(Evening) (b) 54.5 म िकतने छा थे ?
(a) 76 (c) 60 SSC CHSL - 3 July 2019
(b) 79 (d) 52.5 (Morning)
(c) 81 (a) 46
(d) 67 Q17. The total number of (b) 36
students in class A and B is 96. (c) 50
Q15. There are 90 students in a The number of students in A is (d) 48
class, out of which 70% are from 40% more than that in B. The
village A and others are from average weight (in kg) of the Q19. In a class of 80 students, the
village B. The average score of students in B is 50% more than ratio of the urban to the rural is 5
students from village B in a test is that of the students in A. If the : 3. In a test, the average score of
20% more than that from village average weight of all the students the rural students is 40% more
A. If the average score of all the than that of the urban students. If

www.ssccglpinnacle.com support@ssccglpinnacle.com Ph. 09729327755, 09817390373 90


/
Days 13-16 Average

s
sse
la
_c
ob
the average score of all the इनकी उ ों म आठ वष का अनुगामी (c) 40
bo
ah
students is 69, then what is the अंतराल है | यिद प रवार की वतमान (d) 38
sm
ur
yo

average score of the rural औसत उ उतनी ही है िजतनी यह 9


e/
t.m

वष पहले थी, तो नए छोटे सद की


://

students? Q24. The average age of fifteen


tp
ht

80 छा ों की एक क ा म, शहरी और उ ( वष म ) ात कर | persons is 32 years. If two more


ामीण का अनुपात 5 : 3 है | िकसी SSC CHSL - 4 July 2019 persons are added, then the
परी ा म, ामीण छा ों के औसत (Afternoon) average is increased by 3 years.
अंक शहरी छा ों के औसत अंक से (a) 15 The new persons have age
40% अिधक थे | यिद सभी छा ों के (b) 17 difference of 7 years. The age (in
औसत अंक 69 ह, तो ामीण छा ों (c) 10 years) of the younger among the
के औसत अंक ात कर | (d) 9 new persons is:
SSC CHSL - 3 July 2019 पं ह यों की औसत आयु 32
(Evening) Q22. 9 years ago, the average age वष है | यिद दो अित र यों
(a) 80 of a family of five members was को शािमल कर िलया जाए, तो यह
(b) 76 33 years. Now, three new औसत 3 वष बढ़ जाता है | नए
यों म 7 वष का उ अंतराल है

e
(c) 92 members join whose ages are in
(d) 84 ascending order with consecutive | नए यों म छोटे की उ
gaps of 8 years. If the present ( वष म ) ात कर |
Q20. The average of n number is average age of the family is the SSC CHSL - 8 July 2019

l
36. If each of 75% of the numbers same as it was 9 years ago, what (Morning)
is increased by 6 and each of the is the age (in years) of the eldest (a) 54

numbers is:
ac
remaining numbers is decreased
by 9, then the new average of the

n सं ाओं का औसत 36 है | यिद


इन सं ाओं के 75% म से ेक
new member?
9 वष पहले, पां च सद ों के एक
प रवार की औसत उ 33 वष थी |
अब, तीन नए सद शािमल हो गए
ह िजनकी उ आरोही म म है तथा
(b) 61
(c) 50
(d) 58

Q25.The average age of fifteen


को 6 से बढ़ा िदया जाए और ेक उ ों के बीच 8 वष का अनुगामी person is 32 years. If two more
शेष सं ा को 9 से कम कर िदया अंतराल है | यिद प रवार की वतमान persons are added then the
जाए, तो इन सं ाओं का नया औसत औसत आयु उतनी ही है िजतनी यह average is increased by 3 years.
nn
ा होगा ? 9 वष पहले थी, तो सबसे बड़े नए The new persons have an age
SSC CHSL - 4 July 2019 सद की उ ( वष म ) ात कर | difference of 7 years between
(Morning) SSC CHSL - 5 July 2019 them. The age (in years) of the
(a) 37.125 (Morning) elder among the new persons is:
(b) 33.75 (a) 29 पं ह यों की औसत आयु 32
(c) 38.25 (b) 26 वष है | यिद दो अित र यों
(d) 36.25 (c) 35 को शािमल कर िलया जाए, तो औसत
(d) 17 3 वष से बढ़ जाता है | नए यों
Pi

Q21. 9 years ago, the average age की उ ों म 7 वष का अंतर है | नए


of a family of five members was Q23. The average age of four यों म से बड़े की उ (
33 years. Now, three new brothers is 14 years. If their father वष म ) है :
members join whose ages are in is also included, the average is SSC CHSL - 8 July 2019
ascending order with consecutive increased by 4 years. The age of (Afternoon)
gaps of 8 years. If the present the father (in years) is: (a) 58
average age of the family is the चार भाइयों की औसत आयु 14 वष है (b) 61
same as it was 9 years ago, what | यिद उनके िपता को भी शािमल कर (c) 50
is the age (in years) of the िलया जाए, तो यह औसत 4 वष बढ़ (d) 54
youngest new member? जाता है | िपता की उ ( वष म ) है :
9 वष पहले, पां च सद ों के एक SSC CHSL - 5 July 2019 Q26. The average age of fifteen
प रवार की औसत उ 33 वष थी | (Evening) persons is 32 years. If two more
अब, तीन नए सद शािमल हो गए (a) 36 persons are added then the
ह िजनकी उ आरोही म म है एवं (b) 34 average is increased by 3 years.

www.ssccglpinnacle.com support@ssccglpinnacle.com Ph. 09729327755, 09817390373 91


/
Days 13-16 Average

s
sse
la
_c
ob
The new persons have an age पं ह यों की औसत उ 32 (a) 39
bo
ah
difference of 9 years between वष है | यिद दो नए यों को (b) 36
sm
ur
yo

them. The age (in years) of the शािमल िकया जाये, तो औसत उ 3 (c) 26
e/
t.m

वष से बढ़ जाती है | नए यों की
://

elder among the new persons is: (d) 42


tp
ht

15 यों की औसत उ 32 वष उ म 11 वष का अंतराल है | नए


है | यिद दो नए यों को शािमल यों म से छोटे की उ ( Q31. The average height of 12
कर िलया जाए, तो औसत 3 वष से वष म ) ात कर | students of a class is 132.5 cm. If
बढ़ जाता है | नए यों की उ म SSC CHSL - 9 July 2019 one more student joins, the
9 वष का अंतर है | नए यों म (Afternoon) average height becomes 131.2
से बड़े की उ ( वष म ) ात (a) 63 cm, the height of the new student
कर | (b) 52 is:
SSC CHSL - 8 July 2019 (c) 50 एक क ा के 12 छा ों की औसत
(Evening) (d) 58 ऊँचाई 132.5 सेमी है । यिद एक और
(a) 62 छा आ जाता है , तो औसत ऊंचाई
(b) 50 Q29. The average of a series of 131.2 सेमी हो जाती है , नए छा की
ऊंचाई िकतनी है ?

e
(c) 53 21 numbers is equal to 43. The
(d) 58 average of the first eleven of SSC CPO - 16 March 2019
them is 33. The average of the (Morning)
Q27. The average age of fifteen last eleven numbers is 53. The (a) 122.3 cm

l
persons is 32 years. If two more eleventh number of the series is: (b) 115.6 cm
persons are added then the 21 सं ाओं की एक ृंखला का (c) 128.5 cm
ac
average is increased by 3 years.
The new persons have an age
difference of 9 years between
them. The age (in years) of the
younger among the new persons
औसत 43 के बराबर है | इनम से
पहली ारह सं ाओं का औसत 33
है | अंितम ारह सं ाओं का
औसत 53 है | इस ृंखला की 11वीं
सं ा है :
(d) 112.7 cm

Q32. The average of 18 numbers


is 52. The average of first 8
numbers is 62 and the average of
is: SSC CHSL - 11 July 2019 the next 7 numbers is 45. If the
पं ह यों की औसत उ 32 (Afternoon) 16th number is 6 less than the
वष है | यिद दो नए यों को (a) 43 17th number and the 17th number
nn
शािमल िकया जाये, तो औसत उ 3 (b) 47 is one more than the 18th number
वष से बढ़ जाती है | नए यों की (c) 33 then what is the average of 16th
उ म 9 वष का अंतराल है | नए (d) 46 and 18th number?
यों म से छोटे की उ ( 18 सं ाओं का औसत 52 है | पहली
वष म ) ात कर | Q30. The average of 24 numbers 8 सं ाओं का औसत 62 तथा
SSC CHSL - 9 July 2019 is 65. The average of first 11 अगली 7 सं ाओं का औसत 45 है |
(Morning) numbers is 67 and the average of यिद 16वीं सं ा 17वीं सं ा से 6
(a) 62 last 10 numbers is 70. If the 12th कम है तथा 17वीं सं ा 18वीं सं ा
Pi

(b) 50 number is 13 less than the 13th से एक अिधक है , तो 16वीं और 18वीं


(c) 58 number and the 14th number is सं ा का औसत ात कर |
(d) 53 one more than the 13th number, SSC CPO - 12 March 2019
then the average of 12th and 14th (Evening)
Q28. The average age of fifteen number is: (a) 39
persons is 32 years. If two more 24 सं ाओं का औसत 65 है | पहली (b) 39.5
persons are added then the 11 सं ाओं का औसत 67 है तथा (c) 40.5
average is increased by 3 years. अंितम 10 सं ाओं का औसत 70 है (d) 40
The new persons have an age | यिद 12वीं सं ा 13वीं सं ा से 13
difference of 11 years between कम है तथा 14वीं सं ा 13वीं सं ा Q33. In a class of 45 students,
them. The age (in years) of the से एक अिधक है , तो 12वीं और 14वीं 40% are girls and the remaining
younger among the new persons सं ा का औसत ात कर | are boys. The average marks of
is: SSC CHSL - 11 July 2019 the girls is 64 and that of the boys
(Evening)

www.ssccglpinnacle.com support@ssccglpinnacle.com Ph. 09729327755, 09817390373 92


/
Days 13-16 Average

s
sse
la
_c
ob
is 60. What is the average marks SSC CPO - 13 March 2019 boys are 63 and that of the girls
bo
ah
of the whole class? (Evening) are 70. What is the average marks
sm
ur
yo

45 छा ों की एक क ा म 40 ितशत (a) 66 of the whole class?


e/
t.m

लड़िकयाँ ह और शेष लड़के ह | 70 छा ों की एक क ा म 40%


://

(b) 64.5
tp
ht

लड़िकयों का औसत अंक 64 तथा (c) 65 लड़िकयाँ और शेष लड़के ह | लड़कों


लड़कों का 60 है | पूरी क ा का (d) 65.5 का औसत अंक 63 तथा लड़िकयों
औसत अंक ात कर | का औसत अंक 70 है | पूरी क ा का
SSC CPO - 12 March 2019 Q36. The average of 16 numbers औसत अंक ात कर | SSC CPO -
(Evening) is 48. The average of the first 7 13 March 2019 (Morning)
(a) 62.4 numbers is 45 and the average of (a) 65.4
(b) 61.8 the next 6 numbers is 52. If the (b) 65.8
(c) 61.6 14th number is 11 less than the (c) 65.2
(d) 62.9 15th number and is 5 more than (d) 64.8
the 16th number, then the average
Q34. In a class of 45 students, of the 15th and 16th number is: Q39. The average of 22 numbers
16 सं ाओं का औसत 48 है | पहली

e
40% are boys and rest are girls. is 52. The average of the first 8
The average weight of the girls is 7 सं ाओं का औसत 45 तथा numbers is 48 and the average of
55 kg and that of boys is 65kg. अगली 6 सं ाओं का औसत 52 है | next 11 numbers is 54. The 20th
What is the average weight (in यिद 14वीं सं ा 15वीं सं ा से 11 number is 7 less than the 21st
कम तथा 16वीं सं ा से 5 अिधक है ,

l
kg) of the whole class? number and 21st number is 4
45 छा ों की एक क ा म, 40% तो 15वीं एवं 16वीं सं ा का औसत more than 22nd number. What is
ac
लड़के ह और शेष लड़िकयाँ ह |
लड़िकयों का औसत वज़न 55 िकलो
ाम है तथा लड़कों का औसत वज़न
65 िकलो ाम है | पूरी क ा का
औसत वज़न (िकलो ाम म ) ा है
ा होगा ?
SSC CPO - 12 March 2019
(Morning)
(a) 47.5
(b) 48.5
the average of the 20th and 22nd
numbers?
22 सं ाओं का औसत 52 है | पहली
8 सं ाओं का औसत 48 तथा
अगली 11 सं ाओं का औसत 54 है
? (c) 49
| 20वीं सं ा 21वीं सं ा से 7 कम है
SSC CPO - 13 March 2019 (d) 48
तथा 21वीं सं ा 22वीं सं ा से 4
(Evening)
nn
अिधक है | 20वीं सं ा और 22वीं
(a)58 kg Q37. In a class of 50 students,
सं ा का औसत ात कर |
(b)60 kg 40% are girls. The average
SSC CPO - 13 March 2019
(c)61 kg weight of the boys is 62 kg and
(Morning)
(d)59 kg that of the girls is 58 kg. What is
(a) 52
the average weight (in kg) of the
(b) 52.5
Q35. The average of 20 numbers whole class?
(c) 53
is 65. The average of the first 9 50 छा ों की एक क ा म 40%
(d) 53.5
numbers is 68 and the average of लड़िकयाँ ह | लड़कों का औसत
Pi

next 8 numbers is 62. If the 18th वज़न 62 िकलो ाम तथा लड़िकयों


Q40. The average age of a cricket
number is 3 more than 19th का औसत वज़न 58 िकलो ाम है |
team of eleven players is 27
number and 9 less than 20th पूरी क ा का औसत वज़न ( िकलो
years. If two more players are
number, then what is the average ाम म ) ा है ?
included in the team the average
of 19th and 20th number? SSC CPO - 12 March 2019
becomes 26 years, then the
(Morning)
20 सं ाओं का औसत 65 है | पहली average age( in years) of the two
(a) 60.4
9 सं ाओं का औसत 68 तथा included players is:
(b) 60.2
अगली 8 सं ाओं का औसत 62 है | 11 खलािड़यों की एक ि केट टीम
(c) 60.8
यिद 18वीं सं ा 19वीं सं ा से 3 की औसत उ 27 वष है | यिद इस
(d) 60.6
अिधक तथा 20वीं सं ा से 9 कम है , टीम म दो और खलाड़ी शािमल कर
तो 19वीं और 20वीं सं ा का औसत िलए जाते ह, तो औसत आयु 26 वष
Q38. In a class of 70 students,
ात कर | हो जाती है | शािमल िकये गए दो
40% are girls and remaining are
boys. The average marks of the

www.ssccglpinnacle.com support@ssccglpinnacle.com Ph. 09729327755, 09817390373 93


/
Days 13-16 Average

s
sse
la
_c
ob
खलािड़यों की औसत उ ( वष म ) Q44. A scored 73, 76, 20 and 7
bo
ah
ात कर | runs in four out of five innings. Q47. The average of 13 numbers
sm
ur
yo

SSC CPO - 14 March 2019 What should be his score in the is 42. If a 14th number is
e/
t.m
://

(Morning) fifth innings, if he has to make an included, then the average


tp
ht

(a)24.5 average of 55 runs in five becomes 44. What is the 14th


(b)20.5 innings? number?
(c)26 A ने पां च म से चार पा रयों म 73, 76, 13 सं ाओं का औसत 42 है | यिद
(d)27 20 और 6 रन बनाए | पां चवी पारी म 14वीं सं ा को शािमल कर िलया
उसका ोर ा होना चािहए, यिद जाए, तो यह औसत 44 हो जाता है |
Q41. The average of all prime वह पां च पा रयों म 55 रनों का औसत 14वीं सं ा कौन सी है ?
numbers between 10 and 25 is: रखना चाहता है ? SSC MTS - 2 August 2019
10 से 25 के बीच की सभी अभा SSC CPO - 15 March 2019 (Morning)
सं ाओं का औसत ात कर | (Morning) (a) 70
SSC CPO - 14 March 2019 (a) 99 (b) 62
(Morning) (b) 11 (c) 66

e
(a)18.67 (c) 55 (d) 68
(b)16.6 (d) 42
(c)15.3 Q48. What is the average of first
(d)14.7 Q45.A bought 600 gm, 750 gm, seven prime numbers (correct to

l
1.1 kg, 2.3 kg and 800 gm packs two decimal places)?
Q42. The average weight of 16 of dal from a shop. What is the पहली सात अभा सं ाओं का
ac
boys in a class is 60.25 kg and
that of the remaining 10 boys is
45.75 kg. The average weight of
all boys in the class is:
िकसी क ा म 16 लड़कों का औसत
average weight of the packs?
A ने िकसी दु कान से 600 ाम, 750
ाम, 1.1 िकलो ाम, 2.3 िकलो ाम
तथा 800 ाम की दाल की पैकेट
खरीदी | इन पैकेटों का औसत वज़न
औसत ात कर | ( दो दशमलव
थान तक )
SSC MTS - 2 August 2019
(Evening)
(a) 7.14
वज़न 60.25 िकलो ाम है तथा शेष ा है ? SSC CPO - 15 March (b) 8.76
10 लड़कों का औसत वज़न 45.75 2019 (Morning) (c) 7.64
िकलो ाम है | इस क ा म सभी (a) 11.1 kg (d) 8.29
nn
लड़कों का औसत वज़न ा होगा ? (b) 111 gm
SSC CPO - 16 March 2019 (c) 1.11 gm Q49. What is the average of first
(Evening) (d) 1.11 kg six natural numbers, which are
(a)56.27 multiples of 3?
(b)55.37 Q46. The average weight of six पहली छः ाकृितक सं ाओं का
(c)54.67 children is 32.8 kg. If two more औसत ात कर, जो 3 की गुणज ह |
(d)53.76 children with 26.5 kg and 28.3 kg SSC MTS - 2 August 2019
weight are added to the group, (Evening)
Pi

Q43. The average of 26 numbers then what will be the average (a) 10.5
is zero. Of them, how many may weight in kilograms? (b) 11
be greater than zero, at the most? छह ब ों का औसत वजन 32.8 (c) 12
26 सं ाओं का औसत शू है | िकलो ाम है | यिद 26.5 िकलो ाम (d) 9.5
अिधक से अिधक िकतनी शू से और 28.3 िकलो ाम वजन वाले दो
बड़ी हो सकती ह ? और ब ों को समूह म जोड़ा जाता है , Q50. The average age of 12 boys
SSC CPO - 16 March 2019 तो िकलो ाम म तथा औसत वजन is 15 years and the average age of
(Evening) ा होगा ? 18 girls is 12 years. What is the
(a)25 SSC CPO - 15 March 2019 combined average age of the boys
(b)20 (Evening) and girls, taken together?
(c)0 (a)31.45 12 लड़कों की औसत उ 15 वष है
(d)15 (b)30.3 तथा 18 लड़िकयों की औसत उ 12
(c)28.9 वष है | लड़कों एवं लड़िकयों की
(d)29.2 संयु औसत उ ात कर |

www.ssccglpinnacle.com support@ssccglpinnacle.com Ph. 09729327755, 09817390373 94


/
Days 13-16 Average

s
sse
la
_c
ob
SSC MTS - 5 August 2019 समूह के 25 व छा ारा ा अंक SSC MTS - 6 August 2019
bo
ah
(Morning) ह: (Morning)
sm
ur
yo

(a) 15.4 SSC MTS - 5 August 2019 (a) 34 kg


e/
t.m
://

(b) 13.2 (Evening) (b) 37 kg


tp
ht

(c) 16.6 (a) 82 (c) 35 kg


(d) 14.8 (b) 99 (d) 36 kg
(c) 76
Q51. What is the average of all (d) 89 Q57. Average age of 9 men is 45
the natural numbers from 49 to years. If age of one woman is
125? Q54. The average age of mother included, then the average age
49 से 125 तक की सभी ाकृितक and father is 28 years. The becomes 44 years. What is the
सं ाओं का औसत ा है ? average age of mother, father and age of the woman?
SSC MTS - 5 August 2019 their child is 20 years. What is the 9 पु षों की औसत आयु 45 वष है |
(Morning) age of the child ? यिद एक मिहला की उ को शािमल
(a) 85 माता तथा िपता की औसत आयु 28 िकया जाए, तो औसत आयु 44 वष हो
वष है | माता, िपता तथा उनके ब े जाती है | मिहला की उ िकतनी है ?

e
(b) 87
(c) 88 की औसत आयु 20 वष है | ब े की SSC MTS - 6 August 2019
(d) 86 आयु िकतनी है ? (Evening)
SSC MTS - 5 August 2019 (a) 44 years

l
Q52. The average age of a group (Evening) (b) 30 years
of 36 students is 23 years. 4 (a) 4 years / वष (c) 40 years
ac
students leave the group whose
average age is 22 years. What is
the average age of new group in
years ? ( Correct to one decimal
place ).
(b) 3 years / वष
(c) 6 years / वष
(d) 5 years / वष

Q55. The average of 4 numbers is


(d) 35 years

Q58. Average of 50 numbers was


calculated as 20 when three
numbers, 26, 36 and 64 were
36 छा ों के समूह की औसत आयु 9. If one more number 14 is wrongly read as 31, 46 and 59
23 वष है | 22 वष की औसत आयु included, then what will be the respectively. What is the correct
वाले 4 छा समूह छोड़ दे ते ह | नए new average ? 4 सं ाओं का average?
nn
समूह की औसत आयु वष म (एक औसत 9 है | यिद एक और सं ा 14 50 सं ाओं का औसत 20 पाया गया
दशमलव थान तक) िकतनी है ? को भी शािमल िकया जाए, तो नया जब तीन सं ाओं 26, 36 और 64
SSC MTS - 5 August 2019 औसत िकतना है ? को मशः 31, 46 और 59 पढ़ िलया
(Afternoon) SSC MTS - 5 August 2019 गया था | सही औसत ात कर |
(a) 24.4 (Evening) SSC MTS - 6 August 2019
(b) 24.5 (a) 9.5 (Evening)
(c) 23.8 (b) 10.5 (a) 26.2
(d) 23.1 (c) 9 (b) 19.8
Pi

(d) 10 (c) 20.2


Q53. The average marks of a (d) 24.4
group of 15 students in maths is Q56. The average weight of 14
87. Then, 25 other students join children is 22 kg. If the weight of Q59. Average of 12 numbers is
and the total average marks the teacher is added, then the 48. If each number is increased
becomes 79.5. The average of 24 average weight becomes 23 kg. by 11, then what will be the new
students of the other group is 74. What is the weight of the teacher average?
The marks obtained by the 25th ? 12 सं ाओं का औसत 48 है | यिद
student of the other group is : 14 ब ों का औसत वजन 22 kg है | ेक सं ा को 11 से बढ़ा िदया
15 छा ों के एक समूह का गिणत म यिद अ ापक का वजन भी िमला जाए, तो नया औसत िकतना होगा ?
औसत अंक 87 है | िफर अ 25 िलया जाए तो औसत वजन 23 kg हो SSC MTS - 7 August 2019
छा शािमल होते ह और कुल औसत जाता है | अ ापक का वजन िकतना (Morning)
अंक 79.5 हो जाता है | दू सरे समूह के है ? (a) 13
24 छा ों का औसत 74 है | दू सरे (b) 58

www.ssccglpinnacle.com support@ssccglpinnacle.com Ph. 09729327755, 09817390373 95


/
Days 13-16 Average

s
sse
la
_c
ob
(c) 64 छः अवलोकनों का औसत 15 है | तो अंितम दो सं ाओं का औसत
bo
ah
(d) 59 इनम से पहले तीन का औसत अंितम ात कर | SSC MTS - 8 August
sm
ur
yo

तीन के औसत से दोगुना है | अंितम 2019 (Evening)


e/
t.m

तीन अवलोकनों का योग ात कर |


://

Q60. What is the average of all (a) 22


tp
ht

the multiples of 6 from 20 to 80? SSC MTS - 7 August 2019 (b) 18


20 से 80 तक 6 के सभी गुणजों का (Evening) (c) 25
औसत ात कर | (a) 35 (d) 20
SSC MTS - 7 August 2019 (b) 20
(Morning) (c) 25 Q67. Set A contains seven
(a) 51 (d) 30 numbers and the average of these
(b) 50 numbers is 41. Set B contains five
(c) 48 Q64. The average weight of 18 numbers and the average of these
(d) 52 boys in a group is 35 kg. If four numbers is 44. The overall
new boys of weights 20kg, 22kg, average of both the sets is:
Q61. The average of 36, 28, 43, 26kg and 28kg are added to the समूह A म सात सं ाएं ह तथा इन
सं ाओं का औसत 41 है | समूह B

e
56, 74, 65, 12 and x is 45. What is group, then what is the average
the value of x? weight of the newly formed म पां च सं ाएं ह तथा इन सं ाओं
36, 28, 43, 56, 74, 65, 12 और x group? का औसत 44 है | इन दोनों समूहों का
का औसत 45 है | x का मान ा है ? एक समूह के 18 लड़कों का औसत कुल औसत है :
वज़न 35 िकलो ाम है | यिद चार नए

l
SSC MTS - 7 August 2019 SSC MTS - 9 August 2019
(Afternoon) लड़के समूह म शािमल हो जाते ह (Afternoon)
(a) 48
(b) 42
(c) 44
(d) 46
ac िजनके वज़न मशः 20 िकलो ाम,
22 िकलो ाम, 26 िकलो ाम और
28 िकलो ाम ह, तो नए बने समूह
का औसत वज़न ात कर |
SSC MTS - 8 August 2019
(a) 42.75
(b) 42.25
(c) 41.75
(d) 42.5

Q62. The average age of a group (Morning) Q68. Average of 11 numbers is


of 20 men is 30 years. A 50 year (a) 34 years 32. If the average of first six
old man leaves the group, while a (b) 32 years numbers is 26 and the average of
nn
woman joins the group. The (c) 35 years last six numbers is 36 then what
average age decreases by 1 year. (d) 33 years is the value of the sixth number?
What is the age of the woman? 11 सं ाओं का औसत 32 है | यिद
एक समूह के 20 पु षों की औसत Q65. What is the average of the पहली छः सं ाओं का औसत 26 है
आयु 30 वष है | 50 वष का एक first 15 even numbers starting और अंितम छः सं ाओं का औसत
समूह छोड़ दे ता है , जबिक from 2? 36 है , तो छठी सं ा का मान ात
समूह म एक मिहला शािमल हो जाती 2 से शु करते ए पहली 15 सम कर |
है | औसत आयु 1 वष कम हो जाती सं ाओं का औसत ात कर | SSC MTS - 9 August 2019
Pi

है | मिहला की उ िकतनी है ? SSC MTS - 8 August 2019 (Afternoon)


SSC MTS - 7 August 2019 (Afternoon) (a) 20
(Afternoon) (a) 16 (b) 26
(a) 40 years (b) 15 (c) 32
(b) 30 years (c) 17 (d) 30
(c) 35 years (d) 14
(d) 38 Q69.The average marks of two
Q66. The average of four classes M and N are 25 and 40
Q63. The average of six numbers is 20. If the average of respectively and the overall
observations is 15. The average the first two numbers is 15, then average is 30. The ratio of the
of first three among them is twice what is the average of the last two students of classes M and N is :
the average of the last three. What numbers? दो क ाओं M तथा N के औसत अंक
is the sum of the last three चार सं ाओं का औसत 20 है | यिद मश: 25 तथा 40 ह, और ा
observations? पहली दो सं ाओं का औसत 15 है ,

www.ssccglpinnacle.com support@ssccglpinnacle.com Ph. 09729327755, 09817390373 96


/
Days 13-16 Average

s
sse
la
_c
ob
सम औसत 30 है | क ा M और N correct average of the marks
bo
ah
के छा ों का अनुपात है : obtained by him is: Q75. The average of 50 numbers
sm
ur
yo

SSC MTS - 9 August 2019 एक छा ारा 9 िवषयों म ा is 75. If the average of first set of
e/
t.m

औसत अंक 98 है | िफर से जां च


://

(Evening) 25 numbers is 65, then what is the


tp
ht

(a) 2 : 1 करने पर यह पाया गया िक उसके average of the second set of 25


(b) 5 : 6 ारा एक िवषय म ा िकये गए numbers?
(c) 1 : 2 अंक 68 के बजाय 86 िलखे गए थे | 50 सं ाओं का औसत 75 है | यिद
(d) 5 : 3 उसके ारा ा अंकों का सही 25 सं ाओं के पहले समूह का
औसत है : औसत 65 है , तो 25 सं ाओं के
Q70. Five years ago, the average SSC MTS - 13 August 2019 दू सरे समूह का औसत ात कर |
age of 4 girls was 7 years. After (Evening) SSC MTS - 14 August 2019
including a new girl, the present (a) 94 (Afternoon)
average age becomes 13 years. (b) 95 (a) 105
The present age of the new girl is (c) 96 (b) 95
: (d) 97 (c) 85
पाँ च वष पहले 4 लड़िकयों की औसत

e
(d) 75
आयु 7 वष थी | एक नई लड़की के Q73. A party was held for 50
शािमल होने पर वतमान औसत आयु people on their own expenses. Q76. The average weight of 38
13 वष हो जाती है | नई लड़की की Out of them, 48 people paid Rs. students is 42 kg. It was found
वतमान आयु है :

l
950 each and the other two that the figure of 46 kg was
SSC MTS - 13 August 2019 persons paid Rs. 1200 more than misread as 26 kg in one of the
(Morning)
ac
(a) 14 years / 14 वष
(b) 19 years / 19 वष
(c) 16 years / 16 वष
(d) 17 years / 17 वष
the average expenses of the
group. The total amount spent
was :
50 लोगों के िलए एक पाट का
आयोजन उ ीं के खच पर िकया गया
readings. What is the correct
average? (correct to one decimal
places)
38 छा ों का औसत वज़न 42 िकलो
ाम है | यह पाया गया िक 46 िकलो
| उनम से 48 लोगों ने, ेक ने, ाम को भूल-वश 26 िकलो ाम पढ़
Q71. The average of 10 numbers 950 का भुगतान िकया, जबिक अ िलया गया था | सही औसत ा है ? (
is P and the average of 4 of these दो ने समूह के औसत य से एक दशमलव थान तक )
nn
numbers is Q. If the average of 1200 अिधक का भुगतान िकया | SSC MTS - 14 August 2019
remaining numbers is R then खच िकया गया कुल य था : SSC (Evening)
which of the following is correct: MTS - 14 August 2019 (a) 42.5 kg
10 सं ाओं का औसत P है तथा (Morning) (b) 45.5 kg
इनम से 4 सं ाओं का औसत Q है | (a) Rs 75,000 / 75,000 (c) 39 kg
यिद शेष सं ाओं का औसत R है , (b) Rs 40,000 / 40,000 (d) 44 kg
तो िन म से कौन सा िवक सही है (c) Rs 25,000 / 25,000
? (d) Rs 50,000 / 50,000 Q77. The average of 21 data is 36
Pi

SSC MTS - 13 August 2019 out of which the first 12 data are
(Afternoon) Q74. The average of 25 number is having an average of 15. The
(a) 3P = 2Q + 4R zero. At most, how many of these average of the rest 9 data is:
(b) 5P = 3Q + 2R numbers can be greater than zero? 21 आं कड़ों का औसत 36 है िजसम
(c) 5P = 2Q + 3R 25 सं ाओं का औसत शू है | से पहले 12 आं कड़ों का औसत 15 है
(d) 4P = 2Q + 3R इनम से अिधक से अिधक िकतनी | शेष 9 आं कड़ों का औसत है :
सं ाएँ शू से अिधक हो सकती ह SSC MTS - 14 August 2019
Q72. The average marks obtained ? (Evening)
by a student in 9 subjects is 98. SSC MTS - 14 August 2019 (a) 87
On subsequent verification it was (Afternoon) (b) 65
found that the marks obtained by (a) 25 (c) 64
him in a subject was wrongly (b) 13 (d) 50
copied as 86 instead of 68. The (c) 12
(d) 24

www.ssccglpinnacle.com support@ssccglpinnacle.com Ph. 09729327755, 09817390373 97


/
Days 13-16 Average

s
sse
la
_c
ob
Q78. A cricket team has scored (d) Rs 5.75 / 5.75 age of the teacher? 12 छा ों की
bo
ah
156 runs in 30 overs. They need औसत उ 14 वष है | यिद िश क
sm
ur
yo

to score 275 in 50 overs. What is Q81. An art exhibition was held की उ को भी शािमल कर िलया
e/
t.m

जाए, तो औसत 15 वष हो जाता है |


://

the average runs per over they in the month of november. The
tp
ht

need to score in the next 20 numbers of visitors on weekdays िश क की उ िकतनी है ?


overs? and on weekends were 450 and SSC MTS - 19 August 2019
एक ि केट टीम ने 30 ओवर म 156 750 respectively. If the first day (Morning)
रन बनाए ह | उ 50 ओवर म 275 of the month was Monday, then (a) 23 years
रन बनाने की आव कता है | अगले what was the average number of (b) 25 years
20 ओवर म उ ित ओवर िकस visitors in the month ? (c) 27 years
औसत से रन बनाने होंगे ? नवंबर माह म एक कला दशनी (d) 28 years
SSC MTS - 16 August 2019 आयोिजत की गई | स ाह के
(Morning) कायिदवसों (वीकडे ज) और Q84. Average of 10 numbers is
(a) 5.75 स ाहां त (वीक-एं ड) पर आने वाले 14.8. If two numbers 5 and 23 are
(b) 5.95 आगंतुकों की औसत सं ा मश: replaced by 13 and 26
450 और 750 थी | यिद माह का

e
(c) 5.85 respectively, then what is the new
(d) 5.9 पहला िदन सोमवार था, तो माह म average?
आगंतुकों की औसत सं ा िकतनी 10 सं ाओं का औसत 14.8 है | यिद
Q79. The average weight of a थी ? दो सं ाओं 5 और 23 को मशः
13 और 26 से बदल िदया जाता है , तो

l
group of eight people increased SSC MTS - 16 August 2019
by 2.5 kg when a person (Evening) नया औसत ा होगा ?
ac
weighing 80 kg joined in place of
one of the members of the group.
The weight of the member
replaced was:
आठ लोगों के एक समूह का औसत
(a) 520
(b) 530
(c) 510
(d) 540
SSC MTS - 19 August 2019
(Morning)
(a) 15.9
(b) 13.8
(c) 16.3
वज़न 2.5 िकलो ाम से बढ़ गया जब Q82. The owner of a bike buys (d) 14.5
80 िकलो ाम का एक समूह petrol at the rate of Rs. 64, Rs. 80
के िकसी एक सद के थान पर and Rs. 320 per litre for three Q85. The average weight of 13
nn
शािमल हो गया | इस समूह से बाहर consecutive years. If he spends students and their teacher is 24.5
गए का वज़न ात कर | Rs. 32000 on petrol every year, kg. If the weight of the teacher is
SSC MTS - 16 August 2019 then what is the average price of 31 kg, then find the average
(Morning) petrol per litre ? weight of the students.
(a) 60 kg एक बाइक का मािलक लगातार 3 13 िव ािथयों तथा उनके अ ापक
(b) 77.5 kg वष तक मश: 64, 80 और का औसत वजन 24.5 kg है | यिद
(c) 70 kg 320 ित लीटर के िहसाब से पेटोल अ ापक का वजन 31 kg है , तो 13
(d) 62.5 kg खरीदता है | यिद वह पेटोल खरीदने िवधािथयों का औसत वजन िकतना है
Pi

म हर वष 32000 खच करता है , तो ?
Q80. The cost of 6 pencils is Rs. पेटोल का ित लीटर औसत मू SSC MTS - 19 August 2019
30 and 12 pens is Rs. 120. Find िकतना है ? (Afternoon)
the average cost of 50 pencils and SSC MTS - 16 August 2019 (a) 23.5 kg
50 pens. (Evening) (b) 23 kg
6 पिसल का मू 30 और 12 (a) Rs 120 / 120 (c) 24 kg
कलमों का मू 120 है | 50 पिसल (b) Rs 84 / 84 (d) 25 kg
और 50 कलमों का औसत मू (c) Rs 108 / 108
िकतना है ? (d) Rs 96 / 96 Q86. The average of a and b is
SSC MTS - 16 August 2019 10, average of b and c is 12 and
(Afternoon) Q83. Average age of 12 students average of c and a is 15. Find the
(a) Rs 6.75 / 6.75 is 14 years. If the age of teacher is average of a, b and c.
(b) Rs 7.5 / 7.5 also included, the average a तथा b का औसत 10 है , b तथा c
(c) Rs 5 / 5 becomes 15 years. What is the का औसत 12 है और c तथा a का

www.ssccglpinnacle.com support@ssccglpinnacle.com Ph. 09729327755, 09817390373 98


/
Days 13-16 Average

s
sse
la
_c
ob
औसत 15 है | a, b और c का औसत (d) 85.5 अंितम 11 सं ाओं का औसत 42 है
bo
ah
िकतना है ? | यिद 11वीं सं ा को हटा िदया जाए,
sm
ur
yo

SSC MTS - 19 August 2019 Q89. The ratio of boys to girls in तो शेष सं ाओं का औसत ा
e/
t.m

होगा ?
://

(Afternoon) a class is 2 : 3 and the average


tp
ht

(a) 37
3 score of all the students in the SSC MTS - 20 August 2019
(b) 74 class in maths is 54. The average (Afternoon)
3
(c) 74 score of boys is 50% more than (a) 43
(d) 37 that of girls. What is the average (b) 42
score of girls ? (c) 42.9
Q87. In an examination, the एक क ा म लड़के और लड़िकयों (d) 43.5
average score of students in a की सं ा का अनुपात 2 : 3 है और
class who passed is 69 and that of क ा म सभी छा ों के गिणत म Q92. There are 90 students in a
who failed is 61. If the average औसत ा ां क 54 है | लड़कों का class out of which 60% are girls
score of all the students, who औसत ा ां क लड़िकयों के औसत and the rest are boys. The average
appeared in the examination is ा ां क से 50% अिधक है | score of boys in english is 30%
लड़िकयों का औसत ा ां क िकतना

e
66, then the percentage of more than that of girls. If the
students who passed is: एक परी ा है ? average score of all the students
म, िकसी क ा म पास करने वाले SSC MTS - 20 August 2019 in english is 56, then find the
छा ों का औसत अंक 69 है तथा फेल (Morning) average score of only girls in

l
करने वाले छा ों का औसत अंक 61 (a) 50 english.
है | यिद परी ा म शािमल होने वाले (b) 42 एक क ा म 90 छा ह, िजसम से
ac
सभी छा ों का औसत अंक 66 है , तो
पास करने वाले छा ों का ितशत
ात करे | SSC MTS - 19 August
2019 (Evening)
(a) 37.5
(c) 45
(d) 40

Q90. Four different numbers are


given here. The average of the
60% लड़िकयां ह और बाकी लड़के
ह | लड़कों का अं ेजी म औसत
ा ां क ( ोर ), लड़िकयों की
तुलना म 30% अिधक है | यिद
अं ेजी म सभी छा ों का औसत
(b) 60 first three numbers is four times ा ां क 56 है , तो अं ेजी म केवल
(c) 62.5 to the fourth number and the लड़िकयों का औसत ा ां क िकतना
average of all four numbers is है ?
nn
(d) 56
87.75. What is the average of the SSC MTS - 20 August 2019
Q88. The average of 20 numbers first three numbers ? यहाँ चार (Afternoon)
is 80. The average of first 10 अलग-अलग सं ाएँ दी गई ह | (a) 62
numbers is 76.5 and that of next 7 पहली तीन सं ाओं का औसत चौथी (b) 48
numbers is 82. The 18th number सं ा का चार गुणा है और सभी चार (c) 65
is 3 more than the 19th number सं ाओं का औसत 87.75 है | पहली (d) 50
but 3 less than the 20th number. तीन सं ाओं का औसत िकतना है ?
SSC MTS - 20 August 2019 Q93. The average marks of
Pi

What is the average of 18th and


19th numbers? (Morning) Madhu in four subjects is 64 and
20 सं ाओं का औसत 80 है | पहली (a) 90 she got 69 marks in the fifth
10 सं ाओं का औसत 76.5 है तथा (b) 108 subject. What is the new average
अगली 7 सं ाओं का औसत 82 है | (c) 100 of marks obtained by Madhu in 5
18वीं सं ा 19वीं सं ा से 3 अिधक (d) 96 subjects ?
है लेिकन 20वीं सं ा से 3 कम है | मधु ारा चार िवषयों म ा औसत
18वीं तथा 19वीं सं ा का औसत Q91. The average of 21 numbers अंक 64 है और पां चव िवषय म उसने
ात कर | is 44. The average of first 11 69 अंक हािसल िकए | मधु ारा पां च
SSC MTS - 19 August 2019 numbers is 48 and that of last 11 िवषयों म ा अंकों का नया औसत
(Evening) numbers is 42. If 11th number is ा है ?
(a) 85 excluded, what is the average of SSC MTS - 20 August 2019
(b) 88.5 the remaining numbers? (Evening)
(c) 86 21 सं ाओं का औसत 44 है | पहली (a) 66
11 सं ाओं का औसत 48 तथा (b) 67

www.ssccglpinnacle.com support@ssccglpinnacle.com Ph. 09729327755, 09817390373 99


/
Days 13-16 Average

s
sse
la
_c
ob
(c) 65 SSC MTS - 21 August 2019 एक क ा म, लड़िकयों की सं ा
bo
ah
(d) 64 (Morning) लड़कों की सं ा से 60% अिधक है |
sm
ur
yo

(a) 22.50 C लड़िकयों की तुलना म लड़कों का


e/
t.m

औसत वजन 2.6 kg अिधक है | यिद


://

Q94. The average of 20 (b) 21.50 C


tp
ht

observations is 27. If each (c) 24.50 C सभी लड़कों और लड़िकयों का


observation is multiplied by 3, (d) 23.50 C औसत वजन 50 kg है , तो लड़िकयों
then find the new average. का औसत वजन (kg म) िकतना है ?
20 मानों का औसत 27 है | यिद SSC MTS - 21 August 2019
Q97. The average score of 30
ेक मान को 3 से गुणा िकया जाता (Evening)
students is 65, 36 students is 35
है | तो नया औसत है : (a) 48.8
and 34 students is 45. The
SSC MTS - 20 August 2019 (b) 49.2
average score of all the students
(Evening) (c) 49
is :
(a) 91 (d) 48
30 छा ों का औसत अंक 65 है , 36
(b) 61
छा ों का 35 और 34 छा ों का 45 है
(c) 81 Q100. The average of 15 numbers
तो सभी छा ों का औसत अंक है :

e
(d) 71 is 45. The average of the first six
SSC MTS - 21 August 2019
numbers is 42 and that of last six
(Afternoon)
Q95. A person bought 15 articles numbers is 43. The seventh
(a) 45.3
@ Rs. 70 per article, 13 articles number is two times the eighth
(b) 46.4

l
@ Rs. 60 per article and 12 number but 5 more than the 9th
(c) 45.7
articles @ Rs. 65 per article. The number. The average of the
ac (d) 47.4
average cost ( in Rs. ) of per seventh and the ninth number is :
article is : 15 सं ाओं का औसत 45 है | थम
Q98. The average number of
िकसी ने 70 ित व ु की छह सं ाओं का औसत 42 है और
pages of 9 books is 400. If the
दर से 15 व ुएं, 60 ित व ु की अंितम छह सं ाओं का औसत 43 है
average number of pages in the
दर से 13 व ुएं और 65 ित व ु | 7 वीं सं ा, 8 वीं सं ा की दोगुनी
first 5 books is 430 and that of in
की दर से 12 व ुएं खरीदीं | ित है , लेिकन 9 वीं सं ा से 5 अिधक है
the last five books is 380, then
व ु औसत मू ( म) है : | तो 7 वीं और 9 वीं सं ाओं का
find the number of pages in the
SSC MTS - 21 August 2019 औसत िकतना है ?
nn
fifth book.
(Morning) SSC MTS - 21 August 2019
9 पु कों के पृ ों की औसत सं ा
(a) 65.25 (Evening)
400 है | यिद थम पां च पु कों के
(b) 63.25 (a) 65.5
पृ ों की औसत सं ा 430 और
(c) 65.00 (b) 64
अंितम पां च पु कों की 380 है , तो
(d) 65.75 (c) 55.5
पां चवीं पु क के पृ ों की सं ा
(d) 65
िकतनी है ?
Q96. The average temperature of
SSC MTS - 21 August 2019
the first three days of the week is Q101. The average weight of 60
Pi

(Afternoon)
230 C and the average (a) 430
students in a class of 56.4 kg. If
temperature of the next three days 12 students having average
(b) 440
is 240 C and the average weight of 59.5 kg leave the class
(c) 420
temperature of the whole week is and 15 students having average
(d) 450
23.50 C . Find the temperature of weight 54kg join the class, then
the last days of this week. what is the average weight ( in
Q99. In a class, the number of
स ाह के पहले तीन िदनों का औसत kg) of the students in the class
girls is 60% more than that of
तापमान 230 C है और अगले तीन (correct to one decimal place)?
boys. The average weight of the
िकसी क ा म 60 छा ों का औसत
िदनों का औसत तापमान 240 C है boys is 2.6 kg more than that of
वज़न 56.4 िकलो ाम है | यिद 59.5
और पूरे स ाह का औसत तापमान girls. If the average weight of all
िकलो ाम औसत वज़न वाले 12 छा
23.50 C है | स ाह के अंितम िदन the boys and girls is 50 kg, then
क ा से चले जाते ह और 54 िकलो
का तापमान है : find the average weight ( in Kg. )
ाम औसत वज़न वाले 15 छा क ा
of girls.
म आ जाते ह | क ा म छा ों का

www.ssccglpinnacle.com support@ssccglpinnacle.com Ph. 09729327755, 09817390373 100


/
Days 13-16 Average

s
sse
la
_c
ob
औसत वज़न ( िकलो ाम म ) ात number, the sums obtained are एक क ा म 96 छा ह, िजनम से
bo
ah
कर | ( एक दशमलव थान तक ) 68, 74 and 98. What is the लड़िकयों की सं ा लड़कों की
sm
ur
yo

SSC MTS - 22 August 2019 average of the smallest and the सं ा से 40% अिधक है | लड़कों का
e/
t.m

गिणत म औसत ा ां क लड़िकयों


://

(Morning) greatest of the given numbers?


tp
ht

(a) 56.1 तीन धना क सं ाएँ ह | यिद इनम के औसत ा ां क से 40% अिधक है
(b) 55.2 से िकसी भी दो सं ा का औसत | यिद गिणत म सभी छा ों का औसत
(c) 58.0 तीसरी सं ा म जोड़ा जाए, तो ा ां क 63 है , तो गिणत म लड़िकयों
(d) 54.9 योगफल के प म मशः 68, 74 का औसत ा ां क ात कर |
और 98 ा होता है | इनम से सबसे SSC MTS - 22 August 2019
Q102. The average of a few छोटी और सबसे बड़ी सं ा का (Evening)
numbers is 48. If 75% of the औसत ात कर | (a) 51
numbers are increased by 4 each SSC MTS - 22 August 2019 (b) 54
and the rest are decreased by 6 (Afternoon) (c) 55
each, then what is the average of (a) 46 (d) 57
the numbers, so obtained? कुछ (b) 48
सं ाओं का औसत 48 है | यिद

e
(c) 47 Q107. The average temperature
75% सं ाओं को 4 से बढ़ा िदया (d) 52 of one week was 30∘ c . If the
जाता है और शेष सं ाओं को 6 से average temperature of the first
कम कर िदया जाता है , तो इस कार Q105. The average of 12 numbers four days of the week was 31∘ c ,
इन सं ाओं का ा होने वाला

l
is 18.5. The average of first six what would be the average
औसत होगा : numbers is 16.8 and that of the temperature of the remaining

(Morning)
(a) 50.4
(b) 49
(c) 49.5
ac
SSC MTS - 22 August 2019 last seven numbers is 17.4. If the
6th number is excluded, then
what is the average (correct to
one decimal place) of remaining
11 numbers?
days of the week?
एक स ाह का औसत तापमान
30∘ c था | यिद स ाह के पहले चार
िदनों का औसत तापमान 31∘ c था ,
तो स ाह के शेष िदनों का औसत
(d) 50 12 सं ाओं का औसत 18.5 है | तापमान ा होगा ?
पहली छः सं ाओं का औसत 16.8 SSC CPO 14 March 2019
Q103. The number of students in है तथा अंितम सात सं ाओं का
nn
(Evening)
classes A and B are 60 and 70, औसत 17.4 है | यिद छठी सं ा को (a) 29.33∘ c
respectively. The average score in हटा िदया जाए, तो शेष 11 सं ाओं (b) 29∘ c
mathematics of students in B is का औसत ( एक दशमलव थान तक (c) 28.5∘ c
57 and that of all students in A ) ा होगा ? (d) 28.67∘ c
and B is 63. What is the average SSC MTS - 22 August 2019
score of students in A? (Evening)
Q108. The daily average rainfall
क ा A और B म छा ों की सं ा (a) 17.9
on 5 days of the week is 30 mm.
मशः 60 और 70 है | B म गिणत म (b) 18.9
Pi

If the rainfall on 6th and 7th day


छा ों का औसत अंक 57 तथा A और (c) 18.4
are 42 mm and 2tg 5 mm
B के सभी छा ों का गिणत म औसत (d) 20.1
respectively, then what is the
अंक 63 है | A के छा ों का औसत
average daily rainfall for the 7
अंक ात कर | Q106. There are 96 students in a
days ?
SSC MTS - 22 August 2019 class, out of which the number of
स ाह के 5 िदनों की दै िनक औसत
(Afternoon) girls is 40% more than that of the
बा रश 30 िममी है | यिद छठे और
(a) 69 boys. The average score in
सातव िदन मशः 42 िममी तथा 25
(b) 70 mathematics of the boys is 40%
िममी बा रश ई, तो 7 िदनों के िलए
(c) 68 more than the average score of
औसत दै िनक बा रश ात कर |
(d) 71 girls. If the average score in
SSC MTS 2 August 2019
mathematics of all the students is
(Morning)
Q104. There are three positive 63, then what is the average score
(a) 31
numbers. If the average of any of the girls in mathematics?
(b) 29.5
two of them is added to the third

www.ssccglpinnacle.com support@ssccglpinnacle.com Ph. 09729327755, 09817390373 101


/
Days 13-16 Average

s
sse
la
_c
ob
(c) 33 is the average cost of the mixture (a) 12
bo
ah
(d) 28.5 per kg? (b) 11
sm
ur
yo

50 पये ित िकलो ाम कीमत वाले (c) 14


e/
t.m

30 िकलो चावल को 60 पये ित


://

Q109. The average weight of 8 (d) 13


tp
ht

gold coins is 20 g per coin. The िकलो ाम वाले 20 िकलो चावल म


average weight of 12 coins of िमलाया जाता है | ित िकलो ाम Q115. A person was asked to
silver is 35 g per coin. What is the िम ण की औसत कीमत िकतनी है ? guess his own weight. He
average weight per coin for 2 SSC MTS 8 August 2019 estimated his weight to be over
coins? (Afternoon) 70 kg but less than 80 kg. His
8 सोने के िस ों का औसत भार 20 (a) Rs. 54 friend estimated his weight to be
g ित िस ा है | चाँ दी के 12 (b) Rs. 56 more than 74 kg but less than 84
िस ो का औसत भार 35 g ित (c) Rs. 52 kg. His mother said that his
िस ा है | 2 िस ो के िलए ित (d) Rs. 55 weight is less than 78 kg.
िस ा औसत भार िकतना है ? Assuming that the weights are a
SSC MTS 2 August 2019 Q113.A family has two whole number and that the three

e
(Afternoon) grandparents, two parents and had correctly guessed, then what
(a) 25 g four children. Two years ago, the is the average of the possible
(b) 29 g average age of grandparents was weights?
(c) 21 g 72 years. A year ago, the average िकसी को यं के वजन के
बारे म अनुमान लगाने के िलए कहा

l
(d) 31 g age of parents was 36 years.
Currently, the average age of गया | उसने अनुमान लगाया िक
ac
Q110.The average of a, b and c is
9. The average of b and c is 10.
What is the value of a?
a, b तथा c का औसत 9 है | b तथा c
का औसत 10 है | a का मान िकतना
children is 12 years. What is the
current average age of the family?
िकसी प रवार म दो दादा-दादी, दो
माता-िपता और चार ब े है | दो साल
पहले, दादा-दादी की औसत आयु 72
उसका वजन 70 kg से अिधक
लेिकन 80 kg से कम है | उसके
दो ने उसके वजन का अनुमान 74
kg से अिधक लेिकन से 84 kg कम
लगाया | उसकी माता ने कहा िक
है ? वष थी | एक साल पहले, माता-िपता उसका वजन 78 kg से कम है | यह
SSC MTS 5 August 2019 की औसत आयु 36 वष थी | वतमान मानते ए िक वजन एक पूणाक
(Afternoon) म, ब ों की औसत आयु 12 वष है | सं ा है और तीनों ने सही अनुमान
nn
(a) 6 प रवार की वतमान औसत आयु लगाया था, तो संभािवत वजनो का
(b) 7 िकतनी है ? औसत िकतना है ?
(c) 8 SSC MTS 16 August 2019 SSC MTS 14 August 2019
(d) 9 (Afternoon) (Morning)
(a) 32 years (a) 76.5 kg
Q111.The average of a and b is (b) 33.75 years (b) 76 kg
36. The average of b and c is 42. (c) 32.75 years (c) 77 kg
The difference between c and a is (d) 33 years (d) 75 kg
Pi

:
a तथा b का औसत 36 है | b तथा c Q114. Age of A is 6 years more SSC CGL TIER I
का औसत 42 है | c और a के बीच का than three times the age of B.
अंतर है ? After three years, A’s age will be Q1. Out of 6 numbers, the sum of
SSC MTS 6 August 2019 8 years more than twice the age the first 5 numbers is 7 times the
(Afternoon) of B. The average of present age 6th number. If their average is
(a) 18 of A and B (in years) is 136, then the 6th number is:
(b) 12 A की उ B की उ के ितगुने से 6 6 सं ाओं म से, पहली 5 सं ाओं
(c) 16 वष अिधक है | तीन वष बाद, A की का जोड़ छठी सं ा से 7 गुना है |
(d) 14 उ B की उ के दोगुने से 8 वष यिद उनका औसत 136 है , तो छठी
अिधक होगी | A और B की वतमान सं ा कौन सी है ?
Q112. 30 kg of rice costing Rs. औसत आयु ( वष म ) है : SSC CGL 3 March 2020
50 per kg is mixed with 20 kg of SSC MTS 13 August 2019 (Morning)
rice costing Rs. 60 per kg. What (Afternoon) (a) 84

www.ssccglpinnacle.com support@ssccglpinnacle.com Ph. 09729327755, 09817390373 102


/
Days 13-16 Average

s
sse
la
_c
ob
(b) 96 से 9 अिधक है | 10वीं तथा 12वीं SSC CGL 4 March 2020
bo
ah
(c) 116 सं ाओं का औसत िकतना है ? (Evening)
sm
ur
yo

(d) 102 SSC CGL 4 March 2020 (a) 11


e/
t.m
://

(Morning) (b) 12
tp
ht

Q2. The average of the first four (a) 46.5 (c) 15


numbers is three times the fifth (b) 47.8 (d) 13
number. If the average of all the (c) 46
five numbers is 85.8, then the (d) 47 Q7. The average of 24 numbers is
fifth number is: 56. The average of the first 10
पहली चार सं ाओं का औसत Q5. The average weight of some numbers is 71.7 and that of the
पाँ चवीं सं ा से तीन गुना है | यिद students in a class was 58.4kg. next 11 numbers is 42. The next
सभी पाँ च सं ाओं का औसत 85.8 When 5 students having the three numbers (i.e 22nd, 23rd and
है , तो पाँ चवीं सं ा होगी : average weight 62.8 kg joined the 24th) are in the ratio 21 : 31 : 12
5 .

SSC CGL 3 March 2020 class, the average weight of all What is the average of the 22nd
(Afternoon) the students in the class increased and 24th numbers?

e
(a) 33 by 0.55 kg. The number of 24 सं ाओं का औसत 56 है | पहली
(b) 34 students initially in the class 10 सं ाओं का औसत 71.7 है तथा
(c) 39 were: अगली 11 सं ाओं का औसत 42 है
(d) 29 एक क ा म कुछ छा ों का औसत | अगली तीन सं ाएँ ( अथात 22वीं,
वज़न 58.4 िकलो ाम था | जब 62.8

l
23वीं तथा 24वीं ) 21 : 31 : 12
5 के
Q3. The average of five िकलो ाम औसत वज़न वाले 5 छा अनुपात म ह | 22वीं तथा 24वीं
ac
consecutive even numbers is M.
If the next five even numbers are
also included, the average of ten
numbers will be:
पाँ च लगातार सम सं ाओं का
इस क ा म आते ह, तो क ा के सभी
छा ों का औसत वज़न 0.55 िकलो
ाम बढ़ जाता है | आरं भ म क ा म
छा ों की सं ा िकतनी थी ?
SSC CGL 4 March 2020
सं ाओं का औसत िकतना है ?
SSC CGL 5 March 2020
(Morning)
(a) 49.5
(b) 58
औसत M है | यिद अगली पाँ च सम (Afternoon) (c) 55
सं ाओं को भी शािमल कर िलया (a) 30 (d) 60.5
जाए, तो दस सं ाओं का औसत (b) 35
nn
िकतना होगा ? (c) 25 Q8. The average of some
SSC CGL 3 March 2020 (d) 40 numbers is 54.6. If 75% of the
(Evening) numbers are increased by 5.6
(a) M + 10 Q6. The average age of a number each and the rest are decreased by
(b) 11 of persons in a group was 8.4 each, then what is the average
(c) M + 5 calculated as 35 years, which was of the numbers so obtained?
(d) 10 2.5 years more than the correct कुछ सं ाओं का औसत 54.6 है |
average as there was an error in यिद इन सं ाओं के 75% म से
Pi

Q4. The average of twelve recording the age of two persons ेक को 5.6 से बढ़ा िदया जाए तथा
numbers is 45.5. The average of as 38.5 years and 40 years instead शेष म से ेक को 8.4 से कम कर
the first four numbers is 41.5 and of 29 years and 22 years िदया जाए, तो इस कार ा होने
that of the next five numbers is respectively. The number of वाली सं ाओं का औसत ा होगा
48. The 10th number is 4 more persons in the group was: ?
than the 11th number and 9 more एक समूह म कुछ लोगों की औसत SSC CGL 5 March 2020
than the 12th number. What is the उ की गणना 35 वष की गयी, जो (Afternoon)
average of the 10th and 12th वा िवक औसत से 2.5 वष अिधक (a) 56.7
numbers? / थी, ोंिक दो यों की उ 29 (b) 55.6
बारह सं ाओं का औसत 45.5 है | वष तथा 22 वष के थान पर भूलवश (c) 55.8
पहली चार सं ाओं का औसत 41.5 38.5 वष तथा 40 वष दज कर ली (d) 56.3
है तथा अगली पाँ च सं ाओं का गयी थी | इस समूह म लोगों की
औसत 48 है | 10वीं सं ा, 11वीं सं ा िकतनी थी ? Q9. The average height of 5 boys
सं ा से 4 अिधक तथा 12वीं सं ा is 175 cm. A sixth boy joined the

www.ssccglpinnacle.com support@ssccglpinnacle.com Ph. 09729327755, 09817390373 103


/
Days 13-16 Average

s
sse
la
_c
ob
group and the average height of (c) ₹ 106 अंक A के छा ों के औसत अंक से
bo
ah
all the boys in the group now (d) ₹ 194 20% अिधक है | A के छा ों का
sm
ur

increased by one centimeter. The


yo

औसत अंक िकतना है ?


e/
t.m

height of the sixth boy is:


://

Q12. The average of 60 student’s SSC CGL 7 March 2020


tp

5 लड़कों की औसत लंबाई 175 सेमी


ht

results is 38. If the average of the (Evening)


है | एक छठा लड़का समूह म शािमल
first 22 students is 36, and that of (a) 44.5
हो जाता है तथा सभी लड़कों की
औसत लंबाई एक सेमी से बढ़ जाती last 32 students is 32, then the (b) 44
है | छठे लड़के की लंबाई िकतनी average result of remaining (c) 45.5
होगी ? students is: (d) 45
SSC CGL 5 March 2020 60 छा ों के प रणामों का औसत 38
(Evening) है | यिद थम 22 छा ों का औसत 36 Q15. The average age of A,B and
(a) 175 cm तथा अंितम 32 छा ों का औसत 32 C is 20 years, and that of B and C
(b) 179 cm है , तो शेष छा ों का औसत प रणाम is 25 years. What is the age of A?
(c) 180 cm
िकतना है ? A, B तथा C की औसत आयु 20 वष
(d) 181 cm
SSC CGL 6 March 2020 है तथा B और C की औसत आयु 25
वष है | A की उ िकतनी है ?

e
(Evening)
Q10. The average marks of 30
(a) 52.12 SSC CGL 9 March 2020
boys is 88, and when the top
(b) 77.33 (Morning)
scores were excluded, the average
(c) 65.30 (a) 25 years
marks reduced to 87.5. If the top

l
(d) 81.90 (b) 10 years
two scores differ by 2, then the
(c) 15 years
highest marks is /
ac Q13. The average of five (d) 20 years
30 लड़कों का औसत अंक 88 है तथा
consecutive odd numbers is m. If
जब शीष अंक को हटा िदया जाता है ,
the next three odd numbers are Q16. The average of 4 items is 30
तो औसत अंक कम होकर 87.5 हो
also included, then what is the and the 1st term is 31 of the sum
जाता है | यिद शीष दो अंकों म 2 का
increase in the average? of the remaining terms. What is
अंतर है , तो शीष अंक िकतना है ?
पाँ च लगातार िवषम सं ाओं का the first term?
SSC CGL 6 March 2020
औसत m है | यिद अगली तीन िवषम 4 सं ाओं का औसत 30 है तथा
(Morning)
सं ाओं को भी शािमल कर िलया पहली सं ा शेष सं ाओं का ⅓
nn
(a) 94
जाए, तो औसत म िकतनी वृ होगी भाग है | पहली सं ा कौन सी है ?
(b) 96
? SSC CGL 9 March 2020
(c) 90
SSC CGL 7 March 2020 (Afternoon)
(d) 92
(Morning) (a) 30
(a) 3 (b) 40
Q11. 24 students collected money
(b) 17 (c) 20
for donation. The average
(c) 0 (d) 60
contribution was ₹ 50. Later on
(d) 8
Pi

their teacher also contributed


some money. Now the Q17. In a class, the average score
Q14. The average score in of thirty students on a test is 69.
contribution is ₹ 56. The teacher’s
Mathematics of 90 students of Later on it was found that the
contribution is:
sections A and B together is 49. score of one student was wrongly
24 छा ों ने दान दे ने के िलए धन
The number of students in A was read as 88 instead of 58. The
एकि त िकया | औसत योगदान 50
25% more than that of B, and the actual average score is:
पये था | बाद म, उनके िश क ने
average score of the students in B एक क ा म, िकसी परी ा म तीस
भी कुछ रािश का योगदान िदया | अब
was 20% higher than that of the छा ों का औसत अंक 69 है | बाद म,
योगदान 56 पये हो गया है | िश क
students in A. What is the average यह पाया गया िक एक छा का अंक
का योगदान िकतना है ?
score of the students in A? गलती से 58 के बजाय 88 दज कर
SSC CGL 6 March 2020
खंड A तथा खंड B के 90 छा ों का िलया गया था | वा िवक औसत अंक
(Afternoon)
गिणत म औसत अंक 49 है | A के िकतना है ?
(a) ₹ 56
छा ों की सं ा B के छा ों से 25%
(b) ₹ 200
अिधक है तथा B के छा ों का औसत

www.ssccglpinnacle.com support@ssccglpinnacle.com Ph. 09729327755, 09817390373 104


/
Days 13-16 Average

s
sse
la
_c
ob
SSC CGL 9 March 2020
bo
ah
(Evening)
sm
ur
yo

(a) 58
e/
t.m
://

(b) 88
tp
ht

(c) 69
(d) 68

l e
ac
nn
Pi

www.ssccglpinnacle.com support@ssccglpinnacle.com Ph. 09729327755, 09817390373 105


/
Days 13-16 Average

s
sse
la
_c
ob
Variety Questions {( a+b b+c a+c
2 )+c}+{ ( 2 )+a}+{( 2 Increase in total marks =
bo
ah 84+46-48-64 = 18
sm
)+b}= 168+174+180
ur
yo

Sol 1. (b) 18 = 0.45


Increase in average = 40
⇒ a+b+c =261
e/
t.m
://

Let the sixth number is k , 5th Desired average = 261 Desired average = 68+0.45 =
3 = 87
tp
ht

number is k+6 and 7th number is 68.45


k+5.
According to the question Sol 4. (a) Sol 7. (c)
12 x 42 = (4 x Let the number of students in Ratio Girls to Boys = 45 % :
44)+(k+k+6+k+5)+(5 x 40) class B = k 55%
504 = 176+3k+11+200 ⇒ k+k+10 = 110 = 9 : 11
⇒ k = 39 Let the number of girls = 9 unit
⇒ k = 50
Desired average = 45+44
2 =44.50 And number of boys = 11 unit
20% = 51
Alternate : Total marks of the class = (54 x
Let the average score of the
Average of the last 5 numbers is 9) + (11 x 46) = 1984
students of class A = 5k and the 992 = 49.6
less than the total average by 2 Desired average = 9+11
average score of the students of

e
and the average of the first four Alternate :
class B = 6k
numbers is more than total Ratio Girls to Boys = 45 % :
According to the question
average by 2. This will be 55%
60(5k) + 50(6k) = 110 x 72
managed by other numbers. ⇒ 300k + 300k = 7920 = 9 : 11

l
According to the question
⇒ k = 13.2
(42 x 3) = (4 x 2) + k + (k+6) +
ac the average score of the students
(k+5) + {5 x (-2)}
of class A = 5 x 13.2 = 66
117 = 3k
⇒ k = 39
Sol 5. (c)
Desired average = 45+442 =44.50 Let the average of all the four
numbers = A and the last number
Sol 2. (b) =k
Let the initial number of students According to the question
nn
=k A =k - 19
3
According to the question Let the ratio of y : z = k : m
⇒ A = 3k -57 k
68.5k + 72.2 + 70.8 + 70.3 + 66.7 Then y = D × k+m
3k - A = 57 m
= 68.8 (k+4) z = D× k+m
……..(1)
68.5k + 280 = 68.8k +275.2 Where D is the difference of the
And Sum of first three numbers =
⇒ k = 16 number of boys and girls
12 x 3 = 36
Alternate : D = 54-46 = 8
⇒ 36+k
4 =A
Weights of 4 new students is 3.7, k : m = 9 : 11
4A - k = 36 …..(2)
Pi

2.3, 1.8 kg more and 1.8 kg less 9 = 3.6


y = 8 × 9+11
Multiply equation (1) by 4 and
than the average weight of Z = 8× 11 = 4.4
add in equation (2) 9+11
students while the average weight
⇒ 12k-4A+4A-k = 228+36 We know that 54-z = y+46 = x
of the class increases by 0.3 kg.
⇒ k = 24 x = 54 - 4.4 = 3.6 +46 = 49.6
All this will be managed by the
Here, x is nothing but the average
new students’ weight.
Sol 6. (c) marks of the class.
According to the question
(0.3)(k+4) = 3.7+2.3+1.8-1.8 Total marks of the students = 68 x
40 = 2720 Sol 8. (c)
0.3k + 1.2 = 6
Actual total marks of the students Average of 27 numbers is zero,
⇒ k = 16
= 2720-48-64+84+46 = 2738 clearly the sum of 27 numbers is
Desired average = 2738 equal to zero.
Sol 3. (b) 40 = 68.45
So there can be total 26 number
Let the numbers are a, b and c. Alternate :
more than zero and the 27th
According to the question
number will be equal to the sum

www.ssccglpinnacle.com support@ssccglpinnacle.com Ph. 09729327755, 09817390373 106


/
Days 13-16 Average

s
se
s
la
_c
ob
of 26 numbers but with the
bo
ah
negative sign.{(i.e 27th number = Sol 15. (b) Sol 20. (a)
sm
ur
yo

-(sum of 26 numbers)} Sum of the ages of four brothers Total weight of 12 articles = 12 x
e/
t.m
://

= 15 x 4 = 60 18 = 216
tp
ht

Sol 9. (b) Let the age of father = k Let the weight of new articles = k
Let the age of the younger According to the question According to the question
member = k. 60+k = 20 216+k = 18-0.500
5 13
According to the question ⇒ k = 40 216 + k = 17.5 x 13
(38×5)+(10×5)+(k+k+8)
7 = 38 ⇒ k = 11.5
⇒ 2k+8 = 38(7-5)-50 Alternate :
⇒k=9 Average age of four brothers = 15 Alternate :
Increase in total average = 5 Average weight of the articles =
Sol 10. (d) Increase in total age = 5 x 5 = 25 18
Let the number = k This age increase is balanced by Reduction in average weight =
⇒ Its reciprocal = 1k father so age of father must be = 0.5 kg

e
According to the question 15+25 = 40 Total reduction = 13 x 0.5 = 6.5
k+ 1k kg
2 =4 Sol 16.(a) This decrease in weight is
⇒ k+ 1k = 8 Average of the squares of ‘n’ balanced by the third article =

l
⇒ k3 + 1 = 83 − 3(8) natural numbers =
(n+1)(2n+1) 18-6.5 = 11.5
k3 6
...(Algebraic identity)
ac
= 488 (5+1){2(5)+1} Sol 21.(d)
= 6 = 11
Desired average = 488 Let the average of first three
2 = 244 Alternate :
1+4+9+16+25
numbers = 4A
Required average = 5 ⇒ the 4th number = A
Sol 11 (a)
= 11 Sum of first three numbers = 4A
Average of n odd numbers is
always n. x 3 = 12A
Sol 17. (b) According to the question
Sum of a and b (a+b) = 2 x 36 =
nn
12A+A = 52
Sol 12. (c) 4
72
Average of n even numbers is ⇒ A = 16
Sum of b and c (b+c) = 2 x 42 =
always (n+1). Average of first three numbers =
84
4 x 16 = 64
Desired difference = 84-72 = 12
Sol 13. (d)
First 10 prime numbers = Sol 22. (a)
Sol 18. (c)
2,3,5,7,11,13,17,19,23,29 All the prime numbers between
Sum of first 8 multiples of six = 6
First 10 2 digit prime numbers = 21 to 50 = 23, 29, 31, 37, 41, 43
(1+2+3+4+5+6+7+8)= 216
Pi

11,13,17,19,23,29,31,37,41,43 and 47
Desired average = 2168 = 27
Desired average = ⇒ Desired average =
(31+37+41+43)−(2+3+5+7) 23+29+31+37+41+43+47 = 35.85 ≈ 35.9
10 = 13.5 7
Sol 19. (a)
Sum of numbers from 1 to 100 =
Sol 14. (d) 100(100+1) Sol 23. (c)
= 5050
Average of 1088 numbers is zero, 2 Average of the given numbers =
Now, excluding the number 1 59+63+68+77+74+73 = 69
clearly the sum of 1088 numbers 6
each and every number beginning
is equal to zero. So there can be When each number is divided by
from 2 to 100 are prime or either 69 = 3
total 1087 number less than zero 23, average = 23
composite.
and the 1088th number will be
Sum of all the prime and
equal to the sum of 1087 numbers Sol 24. (b)
composite numbers upto 100 =
but with the positive sign.{(i.e Average of ‘n’ odd numbers is
5050-1 = 5049
1088th number = (sum of 1087 always ‘n’.
numbers)} Desired average = 5049
99 = 51

www.ssccglpinnacle.com support@ssccglpinnacle.com Ph. 09729327755, 09817390373 107


/
Days 13-16 Average

s
sse
la
_c
ob
Sol 25. (b) Let the average score of girls = 3k Weight of the students who left is
bo
ah
First 15 whole numbers are ⇒ the average score of boys = more than the average weight by
sm
ur
yo

0,1,2,3,4 ….. 14 5k 6kg and weight of the students


e/
t.m
://

Sum of the numbers from 1-14 = According to the question who joined is more than the
tp
ht

n
2 [2a + (n-1)d] 3 x 66 = 1 x 5k + 2 x 3k average weight by 8 kg. Total
⇒ 14 [2(1)+(14-1)1] = 105 ⇒ k = 18 change in total weight = 4 x 8 -
2
Desired average = 105 =7 So, average of the marks of the 10 x 6 = -28kg.
15
girls = 3 x 18 = 54 Here, the negative sign shows
that the weight is decreasing.
Sol 26. (d)
Sol 3. (a) According to the question
Sum of first three = 3 x 16 = 48
Sum of the numbers = 33 x 74 = 28 = (k-6) x 0.7
Sum of last three = 3 x 15 = 45
2442 40 = k - 6
⇒ First term - Last term= 48-45
Sum of first 17 numbers = 17 x ⇒ k = 46
=3
⇒ First term = 3 + 21 = 24 72.8 = 1237.6
Sum of last 17 numbers = 17 x Sol 5. (a)

e
77.2 = 1312.4 Percentage of girls = 35
SSC CGL TIER II
⇒ 17th number = ⇒ percentage of boys = 65
1312.4+1237.6-2442 = 108 ⇒ Boys : Girls = 13 : 7
Sol 1. (b)
Desired average = 244232−108 = 72.9 Let the average age of boys = 6k
Let the 12th number is k , 11th

l
and the average age of girls = 5k
number is 2k and 13th number is
ac Alternate : According to the question
k+3.
Average of first 17 numbers is ⇒ 13(6k) + 7(5k) = (13+7) x
According to the question
13 x 47 = (3 x 39)+(7 x less than total numbers = 1.2 13.56
Sum of first 17 numbers is less ⇒ k = 12
49)+(k+2k+k+3) 5

611 = 463 + 4k than sum of total numbers = 17 x Average age of girls (5k) = 5 x
1.2 = 20.4 12 = 12
⇒ k =37 5
2(37) + 37 + 3 Average of last 17 numbers is
Desired average = 2 =
more than total numbers = 3.2 Sol 6. (b)
57
nn
Sum of last 17 numbers is more Sum of the numbers = 18 x 37.5
than sum of total numbers = 17 x = 675
Alternate :
3.2 = 54.4 According to the question
Average of the first 3 numbers is
final increase = 54.4-20.4 = 34 675 + 6x = 38.5
less than the total average by 8 18+6
This increase will be adjusted by ⇒ 675 + 6x = 924
and the average of the next seven
rest 32 numbers
numbers is more than the total 34 =1.0625
⇒ x = 41.5
Average increase = 32
average by 2 . This will be Alternate :
⇒ Average of rest 32 numbers =
managed by other numbers. Total increase = (18+6) x 1 = 24
Pi

74-1.0625 = 72.9375 This increase will be managed by


According to the question
(47 x 3) = {3 x (-8)} + (k + 2k + added six new numbers.
Sol 4. (b) ⇒ average increase per number =
k+3) + {7 x (2)}
Let the initial number of students 24 = 4
141 = 4k-7 6
⇒ k = 37 =k
⇒ x = 37.5+4 = 41.5
2(37) + 37 + 3 According to the question
Desired average = 2 = 72k−10(78)+4(80)
= 71.3
57 k−10+4 Practice Questions
72k-460 = 71.3k - 427.8
0.7k = 32.2 Sol 1. (a) Let the 12th number be
Sol 2. (c)
⇒ k = 46 k , 11th number is k+6 and 13th
33 31 % = 1
3 and 66 32 % = 2
3
Let the total students = 3 unit number is k+6.
Alternate : According to the question
Number of boys = 1 unit
Let the initial number of students 13 x 80 = (5 x 74.5)+(5 x
Number of girls = 2 unit
=k 82.5)+(k+k+6+k+6)

www.ssccglpinnacle.com support@ssccglpinnacle.com Ph. 09729327755, 09817390373 108


/
Days 13-16 Average

s
sse
la
_c
ob
1040 = 797 + 3k ⇒ Average of next four numbers average by 0.9 . This will be
bo
ah
⇒ k = 81 5 managed by other numbers.
sm
= 48 x = 60
ur

4
yo

Desired average = 81+6+81+6 = 87 According to the question


e/

According to the question


t.m

2
://

11 x 54 = (4 x 48)+(4 x (55.5 x 4) = {4 x (-2.1)} + {4 x


tp
ht

Alternate : 60)+(k+k+4+k+8) (-0.9)} + k + (k+3) + (k+5)+


Average of first 5 numbers is less 594 = 432 + 3k + 12 (k+6)
than the total average by 5.5 and ⇒ k = 50 222 = 4k+2
average of next five number is Desired average = 50+4+50+8 = 56 ⇒ k = 55
2
more than the total average by 2.5 Desired average = 55+3+55+6
2 =
. This will be managed by other Alternate : 59.5
numbers. Average of the first 4 numbers is
According to the question less than the total average by 4 Sol 6. (a)
(80 x 3) = {5 x (-5.5)} + k + and the average of the next four Total marks of the students = 64 x
(k+6) + (k+6) + {5 x (2.5)} numbers is more than the total 50 = 3200
240 = 3k-3 average by 8 . This will be Actual total marks of the students

e
⇒ k = 81 managed by other numbers. = 3200-38-42+83+24 = 3227
Desired average = 81+6+81+6
2 = 87 According to the question Desired average = 3227
50 = 64.54
(54 x 3) = {4 x (-6)} + {4 x (6)} + Alternate :
Sol 2. (b) k + (k+4) + (k+8) Increase in total marks =

l
Let the 7th number is k , 5th 162 = 3k+12 83+24-38-40 = 27
number is k+4 and 6th number is ⇒ k = 50 27 = 0.54
Increase in average = 50
k+6.
ac
According to the question
12 x 46 = (4 x 43)+(5 x
49.4)+(k+k+4+k+6)
552 = 419 + 3k + 10
Desired average = 50+4+50+8

Sol 4. (a)
2 = 56 Desired average = 64+0.54 =
64.54

Sol 7. (b)
Total marks of the students = 66 x
Let the numbers are a, b and c.
⇒ k = 41 According to the question 45 = 2970
Desired average = 41+4+41 = 43 {( a+b b+c a+c Actual total marks of the students
2 2 )+c}+{ ( 2 )+a}+{( 2
nn
)+b}= 164+158+132 = 2970-28-64+82+46 = 3006
Alternate : ⇒ a+b+c =227 Desired average = 3006
45 = 66.8
Average of the first 4 numbers is Desired average = 227 2
3 = 75 3
less than the total average by 3 Alternate :
and the average of the last five Sol 5. (a)
numbers is more than the total Let the 9th number is k, 10th Increase in total marks =
average by 3.4 . This will be number is k+3, 11th number is 82+46+-28-64 =
managed by other numbers. k+5 and 12th number is k+6 .
36 = 0.8
Increase in average = 45
Pi

According to the question According to the question Desired average = 66+0.8 = 66.80
(46 x 3) = {4 x (-3)} + {5 x (3.4)} 12 x 55.5 = (4 x 53.4)+(4 x
+ k + (k+4) + (k+6) 54.6)+(k+k+3+k+5+k+6) Sol 8. (a)
138 = 3k+15 666 = 432 + 4k + 14 Number of girls = 50 x 46 = 23
100
⇒ k = 41 ⇒ k = 55 ⇒ number of boys =50-23 = 27
Desired average = 41+41+4
2 = 43 Desired average = 55+3+55+6 =
2 Total marks of the class = (58 x
59.5 27) + (62 x 23) = 2992
Sol 3. (c) Alternate : Desired average = 2992
50 = 59.84
Let the 11th number is k, 10th Average of the first 4 numbers is
number is k+4 and the ninth less than the total average by 2.1 Alternate :
number is k+8 and the average of the next four
Average of first four numbers = numbers is less than the total Ratio Girls to Boys = 46 % :
48
54%

www.ssccglpinnacle.com support@ssccglpinnacle.com Ph. 09729327755, 09817390373 109


/
Days 13-16 Average

s
sse
la
_c
ob
= 23 : 27 So,
bo
ah 3 unit = 4.2
sm
ur
yo

1 unit = 1.4
e/
t.m
://

⇒ y = 2 unit = 2.8
tp
ht

Let the ratio of y : z = k : m ⇒ k = 59.2+y


Then y = D × k+m k
= 59.2+2.8 = 62
z = D× m
k+m
Where D is the difference of the Sol 11. (c)
number of boys and girls Ratio Girls to Boys = 60 % :
D = 62-55 = 7 40%
Let the ratio of y : z = k : m k:m=2:3 =3:2
Then y = D × k+m k
2 = 2.8
y = 7 × 3+2 Let the number of girls = 3 unit
z = D× m And number of boys = 2 unit
k+m Z = 7× 3 = 4.2
3+2
Where D is the difference of the Total marks of the class = (3 x
We know that 62-z = y+58 = x
number of boys and girls 72) + (2 x 54) = 324
x = 62 - 2.8 = 55 + 4.2 = 59.2

e
D = 62-58 = 4 Desired average = 3245 = 64.8
Here, x is nothing but the average
k : m = 23 : 27 marks of the class.
23 = 1.84
y = 4 × 23+27 Alternate :
Z = 4× 27 = 2.16
23+27 Sol 10. (d)

l
We know that 62-z = y+58 = x Ratio Girls to Boys = 40 % : Ratio Girls to Boys = 60 % :
ac
x = 62 - 2.16 = 58 + 1.84 = 59.84 60% 40%
Here, x is nothing but the average =2:3 =3:2
marks of the class. Let the number of girls = 2 unit
And number of boys = 3 unit
Sol 9. (a) Total marks of the class = (3 x k)
Ratio Girls to Boys = 40 % : + (2 x 55) = 5 x 59.2
60% ⇒ 3k + 110 = 296
=2:3 ⇒ k = 62
nn
Let the number of girls = 2 unit
And number of boys = 3 unit Alternate :
Total marks of the class = (3 x
62) + (2 x 55) = 296 Ratio Girls to Boys = 40 % :
Desired average = 2965 = 59.2 60%
Let the ratio of y : z = k : m
Alternate : =2:3 k
Then y = D × k+m
z = D× m
Ratio Girls to Boys = 40 % : k+m
Pi

60% Where D is the difference of the


=2:3 number of boys and girls
D = 72-54 = 18
k:m=2:3
3 = 10.8
y = 18 × 3+2
Z = 18 × 2 = 7.2
3+2
We know that 72-z = y+54 = x
x = 72 - 7.2 = 54 + 10.8 = 64.8
y:z = 24:26 Here, x is nothing but the average
=2:3 marks of the class.
Now,
x = 59.2 Sol 12. (b)
….(given) Ratio Girls to Boys = 40 % :
⇒ z = 59.2 - 55 = 4.2 60%

www.ssccglpinnacle.com support@ssccglpinnacle.com Ph. 09729327755, 09817390373 110


/
Days 13-16 Average

s
sse
la
_c
ob
=2:3 Alternate :
bo
ah
Let the number of girls = 2 unit = 7:3
sm
ur
yo

And number of boys = 3 unit Ratio Girls to Boys = 40 % : Let the Number of students from
e/
t.m
://

Total marks of the class = (3 x 60% village A = 7 unit


tp
ht

62) + (2 x 68) = 322 =2:3 ⇒ Number of students from


322 =64.4
Desired average = 3+2 village B = 3 unit
Alternate : 20% = 51
Marks scored by the students
Ratio Girls to Boys = 40 % : from village A = 5k
60% ⇒ Marks scored by the students
=2:3 from village B = 6k
According to the question
10 x 53 = (7 x 5k) + (3 x 6k)
⇒ k = 10
the average score of the students

e
from village B = 6 x 10 = 60
y:z = 20:30
=2:3 Sol 16. (d)
Now, Ratio of number of students who

l
x = 64.4 take part in games to the students
….(given) who do not participate = 60:40
ac
Let the ratio of y : z = k : m
Then y = D × k+m k

z = D× m
k+m
Where D is the difference of the
⇒ y = 64.4 - 62 = 2.4
So,
2 unit = 2.4
1 unit = 1.2
⇒ z = 3 unit = 3.6
=3:2
Let Number of students who take
part in games = 3 unit
⇒ Number of students who do
not take part in games = 2 unit
⇒ k = 64.4+3.6 5% = 20 1
number of boys and girls
D = 68-62 = 6 = 64.4+3.6 = 68 Average weight of students who
nn
k:m=2:3 take part in games = 20k
2 = 2.4
y = 6 × 3+2 Sol 14.(a) ⇒ Average weight of students
3 Let the numbers are a, b and c.
Z = 6× 3+2 = 3.6 who do not take part in games =
According to the question
We know that 68-z = y+62 = x b+c a+c
21k
{( a+b
2 )+c}+{ ( 2 )+a}+{( 2
x = 68 -3.6 = 64.4 = 62 + 2.4 = According to the question
)+b}= 177+163+138 5 x 51 21 = (3 x 21k) + (2 x 20k)
64.4
⇒ a+b+c =239
Here, x is nothing but the average ⇒ 257.5 = 103k
Now, {( a+b
2 )+c} = 177
marks of the class. ⇒ k = 2.5
Pi

⇒ a+b+2c = 354 the average weight (in kg) of the


Sol 13. (d) Similarly former group = 21 x 2.5 = 52.5
Ratio Girls to Boys = 40 % : c+b+2a = 326
60% And Sol 17. (a)
=2:3 c+a+2b = 276 40% = 52
Let the number of girls = 2 unit Biggest number =(a+b+2c) -
Let the number of students in
And number of boys = 3 unit (a+b+c) = 2(177)-239 = 115
class B = 5 unit
Let the average marks of the girls Smallest number = (c+a+2b) - ⇒ the number of students in
=k (a+b+c) = 2(138)-239 = 37
Class A = 7 unit
5 x 64.4 = (2 x k) + (3 x 62) = Desired average = 115+37
2 = 76
According to the question
3552 (7+5) unit = 96
322 = 2k + 186 Sol 15. (b) 1 unit = 8
k = 68 Ratio of students from Village A 7 unit = 56
to Village B = 70:30 5 unit = 40

www.ssccglpinnacle.com support@ssccglpinnacle.com Ph. 09729327755, 09817390373 111


/
Days 13-16 Average

s
sse
la
_c
ob
Number of students from village 8 x 69 = (5 x 5k ) + (3 x 7k) 56+k = 18
bo
5
ah
A = 56 ⇒ k = 12
sm
⇒ k = 34
ur
yo

⇒ Number of students from ⇒ the average score of rural


e/

Alternate :
t.m
://

village B = 40 students = 7 x 12 = 84 Increase in total age after the


tp
ht

50% = 21 addition of father’s age = 5 x 4 =


Let Weight of the students in Sol 20. (c) 20
Class A = 2k 75% = 43 This is managed by the father so
⇒ Weight of the students in Class Let n = 4 age of father = 14 + 20 = 34
B = 3k Total increase / decrease in the
According to the question sum of numbers = (3 x 6) - (1 x 9) Sol 24. (a)
96 x 58 = (56 x 2k) + (40 x 3k) =9 Total age of 15 persons = 15 x 32
⇒ k = 24 Note : The difference is positive = 480
⇒ Weight of the students in Class means total sum is increasing. Let the age of younger person = k
B = 3k = 3 x 24 = 72 Increase in average = 49 = 2.25 According to the question
480+k+k+7 = 35
Desired average = 36 + 2.25 = 17

e
Sol 18. (a) 38.25 487+2k = 595
Let the initial number of students ⇒ k = 54
=k Sol 21. (c)
According to the question 9 years ago total age of the family Sol 25. (b)

l
75.4k + 72.9 + 73.8 + 79.5 + 87.4 (5 members) = 33 x 5 = 165 Total age of 15 persons = 15 x 32
= 75.64 (k+4)ac Increase in the total age = 5 x 9 = = 480
75.4k + 313.6 = 75.64k +302.56 45 Let the age of younger person = k
⇒ k = 46 Current total age of the family (8 According to the question
members) = 33 x 8 = 264 480+k+k+7 = 35
17
Alternate : Let the age of younger person = k 487+2k = 595
Weights of 4 new students is 2.5 According to the question ⇒ k = 54
kg less, 1.6 kg less, 4.1 kg more k + k+8 + k+16 = (264-165-45) So age of elder person = 54+7 =
and 12 kg more than the average 3k + 24 = 54 61
nn
weight of students while average ⇒ k = 10
weight of the class increases by Sol 26. (a)
0.24 kg. All this will be managed Sol 22. (b)
by the new students’ weight. 9 years ago total age of the family Total age of 15 persons = 15 x 32
According to the question (5 members) = 33 x 5 = 165 = 480
(0.24)(k+4) = -2.5-1.6+4.1+12 Increase in the total age = 5 x 9 = Let the age of younger person = k
0.24k + 0.96 = 12 45 According to the question
⇒ k = 46 Current total age of the family (8 480+k+k+9 = 35
17
Pi

members) = 33 x 8 = 264 489+2k = 595


Sol 19. (d) Let the age of younger person = k ⇒ k = 53
Ratio of urban students to rural According to the question So age of elder person = 53+9 =
students = 5:3 k + k+8 + k+16 = (264-165-45) 62
Let the Number of urban students 3k + 24 = 54
= 5 unit ⇒ k = 10
Sol 27. (d)
⇒ Number of rural students = 3 So, age of the oldest member = Total age of 15 persons = 15 x 32
unit 10+16 = 26 = 480
40% = 52 Let the age of younger person = k
Let the average score of urban Sol 23. (b) According to the question
students = 5k Total age of four brothers = 4 x 480+k+k+9 = 35
17
⇒ the average score of rural 14 = 56
489+2k = 595
students = 7k Let the age of father = k
⇒ k = 53
According to the question According to the question

www.ssccglpinnacle.com support@ssccglpinnacle.com Ph. 09729327755, 09817390373 112


/
Days 13-16 Average

s
sse
la
_c
ob
Sol 28. (b) Alternate : ⇒ x = 38
bo
ah
Total age of 15 persons = 15 x 32 Average of first 11 number is 38+38+5
sm
Desired average = = 40.5
ur

2
yo

= 480 more than total average by 2 and


e/
t.m
://

Let the age of younger person = k average of last 10 number is more Sol 33. (c)
tp
ht

According to the question than total average by 5. Ratio of Girls to Boys = 40 % :


480+k+k+11 = 35 Total increase in sum of the
17 60%
491+2k = 595 numbers = 11 x 2 + 10 x 5 = 72 =2:3
⇒ k = 52 This will be managed by 12th, Let the number of girls = 2 unit
13th and 14th number And number of boys = 3 unit
Sol 29. (a) ⇒ k-13 + k + k+1 = (65 x 3)-72 Total marks of the class = (3 x
Sum of the numbers = 21 x 43 = ⇒ k = 45 60) + (2 x 64) = 308
903 Desired average = 45−13+45+1
2 = 39 308 =61.6
Desired average = 3+2
Sum of first 11 numbers = 11 x
33 = 363 Sol 31. (b) Alternate :
Sum of last 11 numbers = 11 x 53 Total height of the students = 12 x

e
= 583 132.5 = 1590 Ratio Girls to Boys = 40 % :
⇒ 11th number = 583+363-903 = Let the height of 13th student = k 60%
43 According to the question
1590+k = = 131.2
13 =2:3

l
Alternate : 1590 + k = 1705.6
Average of first 11 numbers is
ac ⇒ k = 115.6
less than total numbers = 10
Sum of first 11 numbers is less Alternate :
than sum of total numbers = 11 x Decrease in total height of
10 = 110 students due to addition of new
Average of last11 numbers is student = (132.5-131.2) x 13 =
more than total numbers = 10 16.9
Sum of last 11 numbers is more ⇒ Height of the 13th student =
nn
than sum of total numbers = 11 x 132.5-16.9 = 115.6
10 = 110 Let the ratio of y : z = k : m
final increase / decrease = Sol 32. (c) Then y = D × k+m k

110-110 = 0 Let 16th, 17th and 18th number z = D× m


k+m
⇒ 11th number = 43+0 = 43 be x, x+6 and x+5 respectively. Where D is the difference of the
According to the question number of boys and girls
Sol 30. (a) 18 x 52 = (8 x 62) + (7 x 45) + x D = 64-60 = 4
Let the 13th number = k + (x+6) + (x+5) k:m=2:3
Pi

⇒ 12th number = k-13 and 14th 125 = 3x + 11 2 = 1.6


y = 4 × 3+2
number = k+1 ⇒ x = 38 3
Z = 4× 3+2 = 2.4
Total sum of numbers = 24 x 65 = Desired average = 38+38+5 = 40.5
2 We know that 64-z = y+60 = x
1560 Alternate : x = 64-2.4 = 61.6 = 60 + 1.6 =
Sum of first 11 numbers = 11 x Average of first 8 numbers is 61.6
67 = 737 more than the total average by 10 Here, x is nothing but the average
Sum of last 10 numbers = 10 x 70 and average of next 7 numbers is marks of the class.
= 700 less than total average by 7 . This
According to the question will be managed by other Sol 34. (d)
k-13 + k + k+1 = 1560 - 1437 numbers. Ratio of Girls to Boys = 40 % :
3k - 12 = 123 According to the question 60%
k = 45 (52 x 3) = {8 x (10)} + {7 x (-7)} =2:3
Desired average = 45−13+45+1
2 = 39 + x + (x+6) + (x+5) Let the number of girls = 2 unit
156 = 3x + 42 And number of boys = 3 unit

www.ssccglpinnacle.com support@ssccglpinnacle.com Ph. 09729327755, 09817390373 113


/
Days 13-16 Average

s
sse
la
_c
ob
Total marks of the class = (3 x (65 x 3) = {9 x (3)} + {8 x (-3)} +
bo
ah
65) + (2 x 55) = 305 x + (x+3) + (x+12)
sm
ur
yo

305 =61
Desired average = 3+2 195 = 3x + 18
e/
t.m
://

Alternate : ⇒ x =59 Let the ratio of y : z = k : m


tp
ht

k
Ratio Girls to Boys = 60 % : Desired average = 59+59+12
2 = 65 Then y = D × k+m
40% z = D× m
k+m
=3:2 Sol 36. (d) Where D is the difference of the
Let the 14th, 15th and 16th number of boys and girls
numbers be x, (x+11) and (x-5) D = 62-58 = 4
respectively. k:m=3:2
According to the question 3 = 2.4
y = 4 × 3+2
16 x 48 = (7 x 45) + (6 x 52) + x z = 4× 2 = 1.6
3+2
+ (x+11) + (x-5)
We know that 62-z = y+58 = x
768 = 627+3x + 6
x = 62-1.6 = 58+2.4 = 60.4
⇒ x = 45
Here, x is nothing but the average

e
Desired average = 45+11+45
2
−5 = 48
marks of the class.

Alternate : Sol 38. (b)


Let the ratio of y : z = k : m Average of first 7 numbers is less 40% = 52

l
Then y = D × k+m k
than the total average by 3 and Boys:Girls = 3:2
z = D× m
ac k+m average of next 6 numbers is Therefore, Average weight =
Where D is the difference of the more than total average by 4. This 63×3+70×2 = 329 = 65.8
3+2 5
number of boys and girls will be managed by other
D = 65-55 = 10 numbers.
Alternate :
k:m=3:2 According to the question
3 =6
z = 10 × 3+2 (48 x 3) = {7 x (-3)} + {6 x (4)} +
40% = 2
2 x + (x+11) + (x-5) 5
y = 10 × 3+2 =4
144 = 3x + 9 Boys:Girls = 3:2
We know that 65-z = y+55 = x
nn
⇒ x = 45
x = 65-6 = 55+4 = 59
Here, x is nothing but the average Desired average = 45+11+45
2
−5 = 48

marks of the class.


Sol 37. (a)
Sol 35. (c) 40% = 52
Let the 18th, 19th and 20th Boys:Girls = 3:2
numbers be (x+3), x and (x+12). Therefore, Average weight =
According to the question 62×3+58×2 = 302 = 60.4
3+2 5
Pi

20 x 65 = (9 x 68) + (8 x 62) + x
+ (x+3) + (x+12) Alternate :
192 = 3x + 15
⇒ x = 59 40% = 2
Let the ratio of y : z = k : m
5
Desired average = 59+59+12
2 = 65 Boys:Girls = 3:2 Then y = D × k+m k

z = D× m
k+m
Alternate :
Where D is the difference of the
Average of first 9 numbers is
number of boys and girls
more than the total average by 3
D = 70-63 = 7
and average of next 8 numbers is
k:m=2:3
less than total average by 3 . This 2 = 2.8
y = 7 × 3+2
will be managed by other 3
z = 7× 3+2 = 4.2
numbers.
According to the question We know that 70-z = y+63 = x

www.ssccglpinnacle.com support@ssccglpinnacle.com Ph. 09729327755, 09817390373 114


/
Days 13-16 Average

s
sse
la
_c
ob
x = 70 - 4.2 = 63+2.8 = 65.8 Desired average = 11+13+17+19+23 Sol 44. (a)
bo
5
ah
Here, x is nothing but the average Runs to be scored = (55 × 5
sm
= 16.6
ur
yo

marks of the class. )-73-76-20-7 = 99


e/
t.m
://

Sol 42. (c) Alternate :


tp
ht

Sol 39. (d) Required average =


Let the 20th, 21st and 22nd (16×60.25) + (10×45.75)
= 54.67
16+10
numbers be x, (x+7) and (x+3)
respectively.
Alternate :
According to the question
Ratio of Boys of group 1 to Boys
22 x 52 = (8 x 48) + (11 x 54) + x Score in 5th innings =
of group 2 = 16:10
+ (x+7) + (x+3) 55+48+35-18-21 = 99
=8:5
1144 = 978+3x + 10
⇒ x = 52 Sol 45. (d)
Desired average = 52 + 52 +3 = Required average =
2
600+750+1100+2300+800 = 1110 gram
53.5 5

e
= 1.11 kg
Alternate :
Average of first 8 numbers is less Sol 46. (a) New average =
32.8×6+26.5+28.3 = 31.45 kg
than the total average by 4 and 8

l
average of next 11 numbers is
more than total average by 2. This
ac Sol 47. (a)
will be managed by other Total increase in sum of numbers
numbers. Let the ratio of y : z = k : m = 2 x 14 = 28
According to the question Then y = D × k+m k ⇒ 14th number = 42+28 = 70
(52 x 3) = {8 x (-4)} + {11 x (2)} m
z = D× k+m
+ x + (x+7) + (x+3) Sol 48. (d)
Where D is the difference of the
156 = 3x First seven prime numbers are
⇒ x = 52 number of boys and girls
2,3,5,7,11,13 and 17
D = 60.25-45.75 = 14.50
nn
Desired average = 52 + 52
2
+3 = Desired average =
k:m=8:5 2+3+5+7+11+13+17 = 8.29
53.5 8 = 8.9 7
y = 14.50 × 8+5
z = 14.50 × 5 = 5.58
Sol 40. (b) Total age of 11 players 8+5 Sol 49. (a)
= 27 × 11 = 297 We know that 60.25-z = y+45.75 Sum of first 6 multiples of 3 = 3
Total age of 13 players = 26 × 13 =x (1+2+3+4+5+6) = 63
x = 60.25- 5.58 = 45.75 + 8.9 = Desired average = 63
= 338 6 = 10.5
54.67
Total age of two new students =
Here, x is nothing but the average
Pi

338-297 = 41 Sol 50. (b) Total age = (12 x 15)


marks of the class.
Desired average = 41 2 = 20.5 + (18 x 12) = 396
Alternate : Desired average = 39630 = 13.2
Sol 43. (a)
Reduction in the total age of 13 Alternate :
Average of 26 numbers is zero,
players = 13 x 1 = 13 Boys : Girls = 12: 18
clearly sum of 26 numbers is
This reduction is due the age of =2:3
equal to zero. (2×15) + (3×12)
new added players, so total age of Desired average = =
So there can be total 25 number 2+3
two new players = (2 x 27) -13 = 13.2
more than zero and the 26th
41
number will be equal to the sum
Desired average = = 41 2 = 20.5 Sol 51. (b)
of 25 numbers but with the
negative sign.{(i.e 26th number = Sum of first 125 natural numbers
Sol 41. (b) -(sum of 25 numbers)} =
125(125+1)
= 7875
2
Prime numbers between 10 to 25
= 11, 13, 17, 19 and 23

www.ssccglpinnacle.com support@ssccglpinnacle.com Ph. 09729327755, 09817390373 115


/
Days 13-16 Average

s
sse
la
_c
ob
Sum of first 48 natural numbers = 308+k = 23
bo
14+1
ah
48(48+1) Sol 60. (a)
sm
= 1176 ⇒ k = 37
ur

2
yo

Multiples of 6 from 20 to 80 is
e/

Desired average = 7875−1176 = 87 Alternate :


t.m

125−48
://

Average increase per person = 24,30,36,42,48,54,60,66,72,78.


tp
ht

23-22 = 1 Sum of these multiples = 6 x


Sol 52. (d)
Total increase = 15 x 1 = 15 (4+5+6+7+8+9+10+11+12+13) =
Total age = 36 x 23 = 828
This increase is due to addition of 510
Total age of the students who left
teacher, so weight of the teacher Desired average = 510
10 = 51
= 22 x 4 = 88
Desired average = 82832−88 = 23.1 = 22+15 = 37 kg Alternate :
The numbers form an A.P. series
Sol 57. (d) with the common difference = 6
Alternate :
Sum of the age of the men = 45 x ⇒ Average of the numbers =
9 = 405 f irst term+last term 24+78
Sol 53. (b) 2 = 2 = 51
Let the age of the woman = k
Let the average marks of 25
According to the question Sol 61. (d)
students = k

e
405+k = 44
According to the question 9+1 According to the question
(15 × 87) + 25k ⇒ k = 35 36+28+43+56+74+65+12+x = 45
15+25 =79.5 8

⇒ 25k = 1875 ⇒ x = 46
Alternate :

l
Total marks of 24 students = 24 x
74 = 1776 Sol 62. (b)
ac Average decrease per person = Total age of the group = 20 x 30 =
Marks of 25th student =
45-44 = 1 600
1875-1776 = 99
Total decrease = 10 x 1 = 10 Let the age of the woman = k
This decrease is due to addition According to the question
Sol 54. (a)
of the woman, so age of the 600−50+k = 29
Total age of mother and father = 2 20
woman = 35 kg ⇒ 550+k = 580
x 28 = 56
Total age of mother, father and ⇒ k = 30
Sol 58. (b)
son = 3 x 20 = 60
nn
Sum of 50 numbers = 50 x 20 =
⇒ Age of child = 60-56 = 4 Alternate :
1000
Alternate : Decrease in total age after the
Desired average =
With child average age of family 1000−31−46−59+26+36+64 = 19.8 exclusion of old man = 50
50 Actual decrease in the total age =
decrease by 8 years.
⇒ With child decrease in total 20 x 1 = 20
Alternate : ⇒ Age of the woman = 50-20 =
age of the family = 3 x 8 = 24
So, age of the child = 28-24 = 4 30
Decrease in sum=
Pi

(59+46+31)-(64+36+26) = 10
Sol 55. (d) 10 = 0.2
Sol 63. (d)
Decrease in average = 50
Sum of 4 numbers = 4 x 9 = 36 Let the average of last 3
⇒ desired average = 36+14 Desired average = 20-0.20 = 19.8 observations = k
4+1 = 10
Alternate : ⇒ the average of first 3
Sol 59. (d) observations = 2k
Average increase per number =
14−9 = 1 Sum of total numbers = 12 x 48 = Sum of six observation = 6 x 15 =
5 576
Desired average = 4+1 = 5 90
Total increase = 12 x 11 = 132 According to the question
Desired average = 576+132
12 = 59 (3 x 2k) + (3 x k) = 90
Sol 56. (b)
Alternate : ⇒ k = 10
Sum of the weight of the children
= 14 x 22 = 308 Sum of the last 3 observations = 3
Each number is increased by 11 x 10 = 30
Let the weight of the teacher = k
so increase in average = 11
According to the question
Desired average = 48+11 = 59 Alternate :

www.ssccglpinnacle.com support@ssccglpinnacle.com Ph. 09729327755, 09817390373 116


/
Days 13-16 Average

s
sse
la
_c
ob
Ratio of the average of first three Note : We can choose any Sum of their present age = 28 +
bo
ah
observations to the average of last number between 41 and 44 i.e 4(5) = 48
sm
ur
yo

three observations = 2 : 1 42, 43, 42.5 and 43.5. Sum of the present age of 5 girls
e/
t.m
://

This must be the ratio of their Average of first group is less than = 5 x 13 = 65
tp
ht

respective sums, as number of the assumed average by 1 Age of 5th girl = 65-48 = 17
observations are same. Total decrease due to group 1 = 1
Now, x7=7 Sol 71.(c)
Sum of all the six observations = Average of 2nd group is more Sum of 10 numbers = 10 x P =
6 x 15 = 90 than the assumed average by 2 10P
⇒ Sum of last three observations Total increase due to group 1 = 2 Sum of 4 numbers = 4 x Q = 4Q
1 = 30
= 90 x 2+1 x 5 = 10 Sum of remaining 6 numbers = 6
Clearly there is an increase of x R = 6R
Sol 64. (d) 10-7 = 3 Now, 10P = 4Q+6R
Average increase = 7+5 3 = 0.25 ⇒ 5P = 2Q+3R
Total weight of the group = 18 x
35 = 630 ⇒ Desired average = 42.25

e
Desired average = 630+20+22+26+28 Sol 72. (c)
18+4
= 33 Sol 68. (a) Total marks obtained = 9 x 98 =
Total sum = 11 x 32 = 352 982
Alternate : Sum of first six numbers = 6 x 26 Desired average = 882 − 86
9
+ 68 =

l
= 156 96
ac Sum of last six numbers = 6 x 36 Alternate :
= 216 Decrease in total marks = 86-68 =
⇒ Sixth number = (156+216) - 18
352 = 20 Decrease in average = 189 =2
Total decrease = 15+13+9+7 = 44 Desired average = 98-2 = 96
44 = 2
Average decrease = 22 Sol 69. (a)
Desired average = 35-2 = 33 Let the number of students in Sol 73. (d)
class M = m and Class N = n Let the average expanse = A
nn
Sol 65. (a) According to the question According to the question
25(m) + 40(n) 48 × 950 + 2 (A+1200)
Average of first n natural m+n = 30 =A
50
numbres is always (n+1). ⇒ 25m + 40n = 30m + 30n 48000 = 48A
⇒ Average of first n natural ⇒ m : n = 2:1 ⇒ A = 1000
numbers = 15+1 = 16 Total expanse = 50 x 1000 =
Alternate : 50,000
Sol 66. (c)
Sum of four numbers = 4 x 20 = Alternate :
Pi

80
Sum of first 2 numbers = 2 x 15 = Extra amount spent by two = 2 x
30 1200 = 2400
Sum of last 2 numbers = 80-30 = Average amount spent extra =
50 2400
48 = 50
Desired average = 50
2 = 25
Average expense of the group =
950+50 = 1000
Sol 67. (b)
(7 × 41) + (5×44)
Total expanse = 50 x 1000 =
Desired average = 7+5 = 50,000
42.25
Alternate : Sol 74. (d)
Let the overall average = 42 Sol 70. (d) Average of 25 numbers is zero,
Age of the 4 girls 5 years ago = 4 clearly sum of 25 numbers is
x 7 = 28 equal to zero.

www.ssccglpinnacle.com support@ssccglpinnacle.com Ph. 09729327755, 09817390373 117


/
Days 13-16 Average

s
sse
la
_c
ob
So there can be total 24 number Sum of first 12 numbers = 12 x 2 unit = 0.3
bo
ah
more than zero and the 25th 15 = 180 1 unit = 0.15
sm
ur
yo

number will be equal to the sum Sum of next 9 numbers = 756 ⇒ X = 3 unit = 0.45
e/
t.m
://

of 24 numbers but with the -180 = 576 ⇒ k = 5.5 + x


tp
ht

negative sign.{(i.e 25th number = Desired average = 576


9 = 64 = 5.5 + 0.45 = 5.95
-(sum of 24 numbers)}
Alternate : Sol 79. (a)
Sol 75. (c) Average increase = 2.5
Sum of 50 numbers = 50 x 75 = X : Y = 12 : 9 Total increase = 2.5 x 8 = 20
3750 = 4:3 Weight of the member replaced =
Sum of the first set of the 80-20 = 60
numbers = 25 x 65 = 1625
Sum of the second set of the Sol 80. (b)
numbers = 3750-1625 = 2125 Cost of one pencil = 30 = Rs 5
6
Desired average = 212525 = 85 Cost of one pen = 120 = Rs 10
12

e
Total cost of 50 pens and 50
Alternate : pencils = (50x10) + (50x5) = 750
750 =
Desired Average cost = 50+50
X : Y = 25 : 25 7.5

l
= 1:1
ac Now,
Y = 36-15 = 21
3 unit = 21
1 unit = 7
Sol 81. (b)
Month is started with monday so
total weekends = 2 x 4 = 8
Total number of visitors on
weekends = 750 x 8 = 6000
Total number of visitors on
⇒ X = 4 unit = 28
weekdays = 450 x 22 = 9900
⇒ k = 36 + x
Desired average = 6000+9900
30 = 530
nn
= 36 + 28 = 64
Sol 82. (d)
Now, Sol 78. (b)
Total amount spent in three years
Y = 75-65 = 10 Runs to be scored in last 20 years
= 3 x 32000 = 96000
1 unit = 10 = 275-156 = 119
Total petrol purchased in three
⇒ X = 10 Desired average = 119
20 = 5.95 years = 32000 32000 32000
64 + 80 + 320 =
⇒ k = 75 + x Alternate :
1000 litres
= 75 + 10 = 85 X : Y = 30 : 20
Average price of petrol per litre =
Pi

= 3:2 96000 = 96
Sol 76. (a) 1000

Total weight = 38 x 42 = 1596


Desired average = 1596−3846+26 = Sol 83. (c)
Total age of the students = 14 x
42.5 kg
12 = 168
Alternate :
Let the age of the teacher = k
Increase in total weight = 46-26 =
According to the question
20 168+k = 15
20 = 0.526
Increase in average = 38 12+1
⇒ k = 27
Desired average = 42 + 0.5 = 42.5
Alternate :
Average increase = 15-14 = 1
Sol 77. (c)
Now, Total increase = 13 x 1 = 13
Sum of 21 numbers = 21 x 36 =
756 Y = 5.5-5.2 = 0.3

www.ssccglpinnacle.com support@ssccglpinnacle.com Ph. 09729327755, 09817390373 118


/
Days 13-16 Average

s
sse
la
_c
ob
This increase is due the age of the Desired percentage = 5 x 100 = ⇒ the average score of boys =
bo
5+3
ah
teacher.
sm
62.5 3k
ur
yo

So, age of the teacher = 14+13 = Alternate : According to the question


e/
t.m
://

27 (3+2) x 54 = 3 x 2k + 2 x 3k
tp
ht

⇒ k = 22.5
Sol 84. (a) So, the average marks of the girls
Sum of the numbers = 14.8 x 10 = 2 x 22.5 = 45
= 148
Desired average = 148−5−23+13+26
10
= 15.9 Sol 90. (b)
Alternate : Let the fourth number = A
Increase / Decrease in total sum = ⇒ Average of first three numbers
26+13-5-23 = 11 = 4A
Clearly the sum is increasing so According to the question
11 = 1.1
average increase = 10 Let the students who passed = 5 A+3(4A)
4 = 87.75
unit and students who failed = 3

e
Desired average = 14.8+1.1 = ⇒ A = 27
unit
15.9 5 x 100 = Sum of first three numbers = 4 x
Desired percentage = 5+3
27 = 108
Sol 85. (c) 62.5

l
Total weight of the students and Sol 91. (c)
teacher = 24.5 x 14 = 343 Sol 88. (d)
ac Sum of 21 numbers = 21 x 44 =
Total weight of the students = Let the 19th number is k , 18th
924
343-31 = 312 number is k+3 and 20th number
Sum of first 11 numbers = 11 x
Desired average = 312 is k+6.
13 = 24 48 = 528
According to the question
Alternate : Sum of last 11 numbers = 11 x 42
20 x 80 = (10 x 76.5)+(7 x
Total average is less than the = 462
82)+(k+k+3+k+6)
teacher’s age by 31-24.5 = 6.5 ⇒ 11th number = (528+462)-924
1600 = 1348 + 3k
This reduction is due to age of = 66
⇒ k = 84
nn
students so average decrease per Desired average = 92420−66 = 42.9
student = 6.5 Desired average = 84+84+3 = 85.5
13 = 0.5
2

Average age of the students = Sol 92. (d)


24.5-0.5 = 24 Alternate :
60% = 53 and 30% = 3
10
Average of first 10 numbers is
Let the total students = 5 unit
Sol 86. (a) less than the total average by 3.5
Number of boys = 2 unit
According to the question and average of next seven number
Number of girls = 3 unit
( a+b b+c a+c is more than total average by 2.
2 )+( 2 )+( 2 ) =10+12+15 Let the average score of girls =
Pi

This will be managed by other


⇒ a+b+c = 37 10k
37
numbers.
Desired average = 3 ⇒ the average score of boys =
According to the question
(80 x 3) = {10 x (-3.5)} +{7 x 13k
Sol 87. (c) (2)}+ k + (k+3) + (k+6) + According to the question
Let the number of students who 252 = 3k (2+3) x 56 = 3 x 10k + 2 x 13k
passed = p and the number of ⇒ k = 84 ⇒k=5
students who failed = f Desired average = 84+84+3 = 85.5 So, average score of the girls = 5
2
According to the question x 10 = 50
69(p) + 61(f )
p+f = 66 Sol 89. (c)
⇒ 3p = 5f Sol 93. (c)
50 % = 21
Total marks in 4 subjects = 64 x 4
⇒p:f=5:3 Let the number of boys = 2 unit = 256
Let p = 5 unit and f = 3 unit Let the number of girls = 3 unit Marks in 5th subject = 69
Let the average score of girls = 2k

www.ssccglpinnacle.com support@ssccglpinnacle.com Ph. 09729327755, 09817390373 119


/
Days 13-16 Average

s
sse
la
_c
ob
Desired average = 325 = 65 Total pages in first 5 books = 5 x Desired average = 3384−714+810 =
bo
5 60−12+15
ah 430 = 2150
sm
Alternate : 55.2
ur
yo

Marks in 5th subject is more than Total pages in last 5 books = 5 x


e/
t.m
://

the average score by 69-64 = 5 380 = 1900 Sol 102. (c)


tp
ht

Average increase = 55 = 1 ⇒ Pages in 5th book = 75% = 43


Desired average = 64+1 = 65 (2150+1900)-3600 = 450 Let the total numbers are 4.
Total increase = 3x4 = 12
Sol 94. (c) Sol 99. (c) Total decrease = 1 x 6 = 6
Each number is multiplied by 3. 60% = 53 Average increase = 124−6 = 1.5
So the average will also be Let the number of boys = 5 unit Desired average = 48+1.5 = 49.5
multiplied by 3. and the number of girls= 8 unit
So the new average will be 27 x 3 Let the Average weight of the Sol 103. (b)
= 81 girls = A Total score = (60+70) x 63 =
⇒ average weight of the boys = 8190
Sol 95. (a) (A+2.6) Score of class B = 70 x 57 = 3990

e
Total amount spent = (15 x 70) + According to the question Score of class A = 8190 -3990 =
5(A+2.6) + 8(A)
(13 x 60) + (12 x 65) = 2610 5+8 = 50 4200
Average cost of the article = Desired average = 4200
⇒ A = 49 60 = 70
2610
15+13+12 = 65.25

l
Sol 100. (a) Alternate :
Sol 96. (d) ac Let the 8th number is k , 7th
Total temperature of first three number is 2k and 9th number is X : Y = 60 : 70
days = 3 x 23 = 69 2k-5. = 6:7
Total temperature of next three According to the question
days = 3 x 24 = 72 15 x 45 = (6 x
Total temperature of the week = 42)+(2k+k+2k-5)+(6 x 43)
23.5 x 7 = 164.5 675 = 505 + 5k
Temperature of the 7th day = ⇒ k = 34
nn
164.5 - (72+69) = 23.5 Desired average = 68+63
2 = 65.5
Alternate :
Temperature of first three days is Alternate :
less than average temperature of Average of first 6 numbers is less
the week by 0.5. than the total average by 3 and
Total decrease = 0.5 x 3 = 1.5 average of last 6 number is less Now,
Temperature of first three days is than total average by 2. This will X = 63-57 = 6
more than average temperature of be managed by other numbers. 6 unit = 6
Pi

the week by 0.5. According to the question 1 unit = 1


Total increase = 0.5 x 3 = 1.5 (45 x 3) = {6 x (-3)} + k + 2k + ⇒ Y = 7 unit = 7
Temperature of the 7th day = 2k-5 +{6 x (-2)} ⇒ k = 63 + y
23.5-1.5+1.5 = 23.5 135 = 5k-35 = 63 + 7 = 70
⇒ k = 34
Sol 97. (d) Desired average = 68+63 Sol 104. (a)
2 = 65.5
Total score of the students = (30 x Let the numbers are a, b and c.
65) + (36 x 35) + (34 x 45) = According to the question
Sol 101. (b)
4740 {( a+b b+c a+c
4740 Total weight of the class = 60 x 2 )+c}+{ ( 2 )+a}+{( 2
Desired average = 30+36+34 = 47.4 )+b}= 68+74+98
56.4 = 3384
Total weight of the students who ⇒ a+b+c =120
Sol 98. (d) Now, {( a+b
left the class = 12 x 59.5 = 714 2 )+c} = 68
Total pages in 9 books = 9 x 400 Total weight of the students who ⇒ a+b+2c = 136
= 3600 joined the class = 15 x 54 = 810

www.ssccglpinnacle.com support@ssccglpinnacle.com Ph. 09729327755, 09817390373 120


/
Days 13-16 Average

s
sse
la
_c
ob
Similarly This increase will be balanced by Sol 110. (b)
bo
ah
c+b+2a = 148 last three days. The average of a, b and c = 9
sm
ur
yo

And ⇒ Average temperature of last The average of b and c = 10


e/
t.m
://


c+a+2b = 196 three days = 31∘ − 43 = 28.67∘ c Average of b and c is 1 more than
tp
ht

Biggest number =(a+b+2c) - Alternate: the overall average.


(a+b+c) = 196-120 = 76 Average temperature on ⇒ Total decrease = 2 x 1 = 2
Smallest number = (c+a+2b) - remaining days = This increase will be managed by
(a+b+c) = 136-120 = 16 30×7−31×4 = 210−124 = 28.67° C a so the value of a = 9-2 = 7
3 3
Desired average = 76+16
2 = 46

Sol 108. (a) Alternate :


Sol 105. (d) Temperature of sixth day is 12
Given, a+b+c =9
Sum of 12 numbers = 12 x 18.5 = mm more than the average 3
222 temperature of first 5 days. And ⇒ a + b + c = 27 ---- (i)
Sum of first 6 numbers = 6 x 16.8 temperature of seventh day is 5 And, b+c
2 = 10
= 100.8 mm less than the average ⇒ b + c = 20 ---- (ii)

e
Sum of last 7 numbers = 7 x 17.4 temperature of the first 5 days. Subtracting (ii) from (i), we get
= 121.8 ⇒ Total increase = 12-5 = 7 mm a=7
⇒ 6th number = Average increase = 77 = 1 mm
121.8+100.8-222 = 0.6 Sol 111. (b)
Required average = 30+1 = 31

l
Desired average = 22211−0.6 = 20.1 Given, a+b
mm 2 = 36
Alternate : ac ⇒ a+b = 72 ---- (i)
Average of first six numbers is Alternate : Again, b+c = 42
2
less than the total average by 1.7 Average daily rainfall for the ⇒ b+c = 84 ---- (ii)
and average of last 7 numbers is week = 5×30+42+25 = 217
7 7 = 31 mm Now, Subtracting (i) from (ii), we
less than the total average by 1.1
get
Total decrease = 1.7 x 6 + 1.1 x 7
Sol 109. (b) c - a = 84 - 72 = 12
= 17.9
Let the required average is a
⇒ 6th number must be =
number between 20 and 35. Sol 112. (a)
nn
18.5-17.9 = 0.6 For example, let the average Let the required average is a
Desired average = 22211−0.6 = 20.1 weight of the coin = 30 gram number between 50 and 60.
Weight of gold coins is 10 gram For example, let the average cost
Sol 106. (b) less than the average weight. of the mixture = 20 gram
40% = 52 Total decrease = 8 x 10 = 80 gram Cost of first type of rice is Rs. 5
Let number of boys = 5 unit Weight of silver coins is 5 gram less than the average cost.
Number of girls = 7 unit more than the average weight. Total decrease = 30 x 50 = Rs.
Let the average score of girls = 5k Total increase = 12 x 5 = 60 gram 150
Pi

⇒ the average score of boys = Overall decrease = 80-60 = 20 Cost of second type of rice is Rs.
7k gram 5 more than the average cost.
According to the question 20 = 1
Average decrease = 8+12 Total increase = 20 x 5 = Rs. 100
(5+7) x 63 = 5 x 7k + 7 x 5k gram Overall decrease = 150-100 = Rs.
⇒ k = 10.8 Required average = 30-1 = 29 50
So, average score of the girls = 5 Average decrease = 30+2050 = Rs. 1
gram
x 10.8 = 54 Alternate : Required average = 55-1 = Rs. 54
Total weight of gold coin = 8 x 20
Sol 107. (d) = 160 gram Alternate :
Average of temperature in first Total weight of silver coin = 12 x Total price of 30 kg rice = 30 x 50
four days = 1∘ c 35 = 420 gram = 1500
Total increase due to first for days Required average = 160+420
8+12 = 29 Total price of 20 kg rice = 20 x 60
= 4 x 1 = 4∘ c gram = 1200

www.ssccglpinnacle.com support@ssccglpinnacle.com Ph. 09729327755, 09817390373 121


/
Days 13-16 Average

s
sse
la
_c
ob
Required average = 1500+1200 = Correct average = 35-2.5 = 32.5
bo
30+20
ah Sol 3. (c) For given five 32.5 × x = Sx-2+ 29+ 22 … (2)
sm
Rs. 54
ur
yo

consecutive even numbers, (1)-(2) ⇒ 2.5*x = 27.5


e/
t.m
://

Sol 113. (b) average = middle number x = 11 person


tp
ht

Present average age of Thus, M = middle number


grandparents = 72 + 2 = 74 years And their sum = 5 × M Sol 7. (d) The last three numbers
Present average age of parents = We can now conclude that first 5 are in ratio: 21 : 31 : 12
5 = 6a:4a:5a

36 + 1 = 37 years numbers are:M-4,M-2,M, M+2, According to question:


Present average of children = 12 M+4 24 × 56 = 10 × 71.7+ 11 × 42+
years And the next five numbers are 6a+4a+5a
Therefore, Average age of family M+6,M+8,M+10,M+12,M+14 1344 = 717+ 462 + 15a
= 74×2+37×2+12×4
8 = And their average = 5M5+50 = 1344 = 1179 + 15 a
148+74+48
8 = 270
8 = 33.75 years M + 10 a = 11
now , the average of all the 10 22nd +24th 66+55
2 = 2 = 60.5
Sol 114. (d) numbers = M +M2 +10 =M+5

e
Let the age of A and B be a and b Sol 8. (a) Let there be a total of
respectively. Sol 4. (a) Average of 12 numbers 4a numbers.
ATQ: a = 3b + 6 ----- (i) = 45.5 Suppose for 3a numbers, average
And, a + 3 = 2 (b + 3) + 8 Average of first four numbers = = x and for remaining a numbers,

l
⇒ 3b + 6 + 3 = 2b + 6 + 8 41.5 average = y
[From eq. (i)]
ac Average of next five numbers = According to given condition:
⇒b=5 48 3a × x + a × y = 4a × 54.6
Again, a = 3 x 5 + 6 = 21 3x + y = 4 × 54.6
Therefore, Average weight of A Let 12th number be ‘x’ Also, 3a × (x+5.6) + a × ( y- 8.4)
and B = 21+5 Then 10th number = x+9
2 = 13 years = 4a × A’
And 11th number = x+5
3 × (x+5.6) +( y- 8.4) = 4 × A’
Sol 115. (b) A’ = 54.6 × 4 + 43× 5.6 − 8.4 = 56.7
As per given data, it can be We get:
nn
deduced that the average weight 45.5 × 12 = 4 × 41.5 + 5 × 48 +
Sol 9. (d) Average height of 5
lies from 75kg to 77 kg. (x+9) + (x+5) + x
boys = 175 cm
Therefore, Average weight of the ⇒ 546 = 166 + 240 + 3x + 14
After addition of one more boy,
person = 75+76+77 = 228 ⇒ 546 = 420 + 3x
3 3 = 76 kg average height = 176 cm
increase ⇒ 126 = 3x New boys height = 175 + 6 ×
⇒ x = 42 (actual increase ) = 181 cm
SSC CGL TIER I
Thus, 12th number x = 42 Sol 10. (b) Let top scorers be ‘x’
Pi

Then 10th number = x+9 = 51 and ‘x+2’.


Sol 1. (d) As the sum of the first 5
Average of 10th and 12th number = 30 × 88 = 28 × 87.5 + x + x+2
numbers is 7 times the 6th number
42+51 = 93 = 46.5 x = 94 and x+2 = 96
and their average is 136. 2 2
We can say
Sol 5. (b) Let there be ‘x’ Sol 11. (b) According to question:
8 × sixth number = 6 × 136
24 × 50 + T = 25 × 56
Thus, students initially in the class.
T = ₹ 200
Sixth number is 102 According to given condition:
58.4 × x + 62.8 × 5 = 58.95(x+5) Sol 12. (b) According to question
Sol 2. (a) Let the fifth number be 3.85 × 5=0.55 × x :
‘x’ X =35 students 60 × 38 = 22 × 36+ 32 × 32 + 6 ×
Average of first four numbers= 3x A
4 × (3x) + x = 5 × 85.8 = 429 Sol 6. (a) Let there be ‘x’ persons ⇒ A = 77.33
13x = 429 in a group.
x= 33. 35 × x = Sx-2+ 38.5 + 40 … (1)

www.ssccglpinnacle.com support@ssccglpinnacle.com Ph. 09729327755, 09817390373 122


/
Days 13-16 Average

s
sse
la
_c
ob
Sol 13. (a) Average of five The average of the runs of a Sol:5.(d)
bo
ah
consecutive odd numbers = m cricket players in 20 matches = When serial number is added to
sm
ur
yo

Thus, middle number is ‘m’ 35 each then now average =59


e/
t.m

(1+2+3+.....+10)
Old average = 55-
://

First five odd numbers = Total runs = 35 × 20 = 700


tp

10
ht

m-4,m-2,m,m+2,m+4 The average of the first 12 =49.5


24+45+a+35+59+83+46+b+29+74 =495
Next three odd consecutive matches = 45 10
numbers = m+6,m+8,m+10 Total runs in first 12 matches = a+b=100
Average of these 8 consecutive Average of a+b=50
12 × 45 = 540
numbers = Total runs in last 8 matches =
m−4+m−2+m+m+2+m+4+m+6+m+8+m+10 Sol:6.(b)
8 = 700 - 540 = 160
Total sum of money=35 × 22=770
m+3 Average of the last 8 matches =
Sum of first 17 number=17 ×
Increase in average = 3 160/8 = 20
19=323
Sum of last 17 number=17 ×
Sol 14. (d) Number of students in Sol:3.(b)
20=340
A:B = 5:4 or 5x and 4x Three years ago, the average age
18th

e
respectively. of a husband, wife and child = 26
number=770-(323+340)=107
Total number of students = 90 years
5x = 50 and 4x = 40 Three years ago, the total age of a
Sol:7. (d)
Average scores of A and B are 5y husband, wife and child = 26
Present age of Ravi, Mohan and

l
and 6y respectively. × 3 = 78 years
Govind = 3 × 32 + 3 × 6 = 114
Total marks = 90 × 49 = 4410
ac Present total age of husband,
Sum of ages of four = 4 × 36 =
5y × 50+6y × 40 = 4410 wife and child = 78+9 = 87 years.
5 years ago, the average age of 144
y=9
wife and child = 20 years The present age of shyam =
Average score of class A = 5 × 9 =
5 years ago, the total age of wife 144-114 = 30 years
45
and child = 20 × 2 = 40 years
Sol:8.(c)
Sol 15. (b) According to question: Present total age of wife and
The sum of 17 consecutive
3 × 20 = 2 × 25 + A child = 40+10 = 50 years.
numbers = 289,
nn
A = 10 years The present age of the husband =
The average of 17 consecutive
87-50 = 37 years.
numbers = 289
17 = 17
Sol 16. (a) 1st item: sum of
Sol:4.(d) The first term of another series is
remaining three items = 1:3
Average of 35 consecutive natural 5 more than the average of the
1st item = x
numbers = N first set of consecutive
Sum of remaining 3 items = 3x
Dropping the first 10 numbers Numbers = 17 + 5 = 22
4x = 4 × 30
and including the next 10 The sum of another 10
x = 30
numbers, the average is consecutive numbers
Pi

changed to M, = 105 {2 × 22 + (10 − 1) 1}


Sol 17. (d) Actual average =
30×69 − 88+58 = 68 So we can say that the difference = 265
30
SSC CHSL 2019 between M and N is 10.
M - N = 10……...1 Sol:9.(b)
Sol:1. (b)
M 2 - N 2 = 600 Sum of 5 consecutive odd
Sum of ages of 25 men = 25 × 28
(M+N)(M-N) = 600 numbers = 5 × 75 = 375
= 700
(M + N) = 600/10 = 60……...2 Total average after adding = 6 ×
Sum of ages of 5 new men = 25 ×
Now we can get, from eq1 and 2 76 = 456
5 = 125
M = 35, N = 25 Required number = 456-375 = 81
Total ages = 70+125 = 825
required average = 825 the average of 3M and 5N =
30 = 27.5 3M +5N
2
Sol:10.(b)
= Sum of marks in five subjects = 5
Sol:2.(d)
3×35+5×25 = 115 × 150 = 750
2

www.ssccglpinnacle.com support@ssccglpinnacle.com Ph. 09729327755, 09817390373 123


/
Days 13-16 Average

s
sse
la
_c
ob
After calculation = 750-43+23 = Highest score = x+120 = 10+120 Son’s age = 7x = 7 × 6 = 42 years
bo
ah
770 = 130
sm
ur
yo

New average = 770 5 = 154 Sol:20. (a)


e/
t.m
://

Sol:15. (a) Let there be n numbers in the list


tp
ht

Sol:11.(a) Sum of 24 numbers = 26 × 24 = and let their average be x.


41 years 9 months = 41 43 = 167
4
624 Then, sum of n numbers = nx
Sum of first 15 numbers = 15 × 23 nx+40 = x + 4
Now, go through allegation, n+1
Male Female = 345 nx+40 = (n+1)(x+4)
42 41 Sum of last 8 numbers = 8 × 33 = 4n+x = 36……………..(1)
167 264 nx+70 = x + 5
4 n+2
______________________ 16th number = 624-(345+264) = nx+70 = (n+2)(x+5)
3 : 1 15 5n+2x = 60……………(2)
4 unit = 600 Solving (1) and (2), we get:
1 unit = 150 Sol:16. (a) n = 4; x = 20
So, number of female employee = There are total 4 sunday in a

e
150 month Sol 21. (d)
Total visitors on sunday = 4 × 265 Sum of ages of Sonu, Hari and
Sol:12. (c) = 1060 Govind = 3 × 30 = 90 years
Let his initial average = x Total visitors all day = 26 × 130 = Ratio of ages of Sonu, Hari and

l
Total runs in 15 innings = 15x 3380 Govind = 4:5:6
His run, after 16th innings =
ac Required average = 148 I.e. ages of sonu, hari and govind
15x+120 are 4x, 5x, 6x respectively
15x+120 = x+6 Sol:17. (a) 4x + 5x + 6x = 90
16
15x+120 = 16x+96 Let us consider by mistake he 15x = 90
x = 24 writes 9th number with its digit x=6
New average = 24+6 = 30 interchanged Difference between ages of Sonu
(10x+y)−(10y+x)
9 =6 and Govind = 6x - 4x = 2x = 12
Sol:13. (d) 9x-9y = 54 years
nn
Sum of first two numbers = 2 × 7 x-y = 6
= 14 Sol 22. (d)
Sum of last two numbers = Sol:18. (b) Total weight of P, Q and R = P +
2 × 10 = 20 Let the average of 4 consecutive Q + R = 62 × 3 = 186
Sum of first and last numbers = odd numbers = x R’ weight = 12 + P’s weight = 9 +
2 × 14 = 28 Average of 3 consecutive even Q’s weight
sum of all numbers = 14+20+28 = number = y S’s weight = R’s weight - 15
2
Sum of four consecutive odd ⇒ P + Q + R = 62 × 3 = 186
31
Pi

31 number = 4x ⇒ R - 12 + R - 9 + R = 186
required average = 3
Sum of 3 consecutive even ⇒ 3R = 186 + 21 = 207
number = 3y ⇒ R = 69
Sol:14. (b)
According to the question S’s weight = R - 15 = 69 - 15 =
Sum of runs in 20 matches = 20 ×
y -x =8 54 kg
52 = 1040
And 4x =3y Average weight of P,Q,R and S =
Sum of runs in 18 matches = 18 × P +Q+R+S
Solving both the equation gives = 186 + 54
= 240
=
50 = 900 4 4 4
x = 24 60 kg
Runs in 2 innings = 1040-900 =
140
Sol:19.(c) Sol 23. (c)
Let the lowest score = x and
Sum of ages of man to son = Let the average score of a
highest score = x+120
2*60 = 120 batsman for 10 innings = A
x+x+120 = 140
13x+7x = 120 According to question:-
x = 10
x=6 10 × A + 77 = 11 × (A+3)

www.ssccglpinnacle.com support@ssccglpinnacle.com Ph. 09729327755, 09817390373 124


/
Days 13-16 Average

s
sse
la
_c
ob
⇒ 10 × A + 77 = 11 × A+ 11 × 3 48 × (78 - 66) + 168 = 8 × T Sum of last 16 number = 43 × 16
bo
ah
sm
⇒ 11 × A - 10 × A = 77 - 33 = 44 48 × 12 + 168 = 8 × T Average of 40 numbers = 36
ur
yo

⇒ A = 44 T = 6 × 12 + 21 = 72 + 21 = 93 Sum of 40 numbers = 40 × 36
e/
t.m
://

Average after 11 innings = 44 + 3 Sum of first 25 numbers + Sum of


tp
ht

= 47 Sol 26. (a) last 16 numbers = Sum of 40


Let the four consecutive even numbers + 25th number
Sol: 24. (b) numbers = 2a, 2a + 2, 2a + 4, 2a 31 × 25 + 43 × 16 = 40 × 36 +
Sum of ages of 16 students = 16 +6 25th number
× 20 = 320 Sum of numbers = 2a + 2a + 2 + 25th number = 31 × 25 + 43 × 16
Sum of ages of 5 students = 5 × 2a + 4 + 2a + 6 = 8a + 12 - 40 × 36 = 775 + 688 - 1440 = 23
20= 100 Average = 8a +4 12 = 2a + 3 = 27
Sum of ages of 10 students = 10 2a = 27 - 3 = 24 Sol:29.(b)
× 20.4 = 204 a = 12 According to the que
a+b+c = 2 + c
Age of 16th students = Then, four numbers will be:- 24, 3
320-(100+204) = 16 26, 28, 30 a+b
2 = 48

e
Sum of four numbers = 8a + 12 = a + b = 96
Sol: 25. (c) 96 + 12 = 108 96+c = 2 + c
3
Let T students take part in an Let us add x to 108 to make
96 + c = 6 + 3c
examination. average = 28
c=45

l
Average of all students = A 108 + x = 28 × 5 = 140
d=35
For 48 students, average changes
ac x = 140 - 108 = 32 Average of a and b = 40
from 78 to 66.
While for remaining students i.e. Or, Average of four number, A1 = Sol:30.(b)
(T-48), average increase by 3.5 27 Number of students in A = 50
marks. Average of five numbers, A2 = 28 Number of students in B = 40
Overall change in average marks A2 × 5 = A1 + x Total marks = 90 × 63 = 5670
of students is a decrease of 4.5 ⇒ 28 × 5 = 27 × 4 + x
average score of students in B = x
marks. ⇒ x = 28 × 5 - 27 × 4 = 140 - Total marks of student in B = 40x
nn
This can be represented as 108 = 32 average score of students in A =
follows:-
1.3x
Sol: 27. (d) Total marks of student in A = 65x
Average of Average Overall Sum of ages of 65 men = 65 × 32 According to question = 40x +
(T - 48) of (48) average When 5 more men joined the 65x = 5670
of T group, average of 70 men x= 54
becomes = 34 years
X 78 A
Sum of ages of 70 men = 70 × 34 Sol:31.(d)
Pi

X + 3.5 66 A - 4.5 Sum of 65 men + Sum of new 5 Let the average score of student
men = Sum of 70 men in B = x
(T - 48) × X + 48 × 78 = T × A
65 × 32 + Sum of 5 new men = 70 Let the average score of student
… (i)
× 34 in A = 1.2x
(T - 48) × (X + 3.5) + 48 × 66 =
Sum of 5 new men = 70 × 34 - 65 According to the question
T × (A - 4.5) … (ii)
× 32 = 2380 - 2080 = 300 48x + 52x = 75 × 92
Average age of 5 men = 300
5 = 60 100x = 6900
In equation (i):- T × X - 48 × X +
years x= 69
48 × 78 = T × A …. (iii)
In equation (ii):- T × X - 48 × X +
SSC CGL 2019 TIER-II Sol:32.(c)
3.5 × T - 168 + 48 × 66 = T × A -
Sol: 28. (a) Total of 25 number = 54 × 25 =
T × 4.5 …. (iv)
Average of first 25 number = 31 1350
Sum of first 25 number = 31 × 25 Total of first 13 number = 13 ×
Subtract (iv) from (iii):--
Average of last 16 number = 43 52.8 = 686.4

www.ssccglpinnacle.com support@ssccglpinnacle.com Ph. 09729327755, 09817390373 125


/
Days 13-16 Average

s
sse
la
_c
ob
Total of last 13 number = 13 × 39 × 12=40 × 4+35 × Average weight of B, C, D and
bo
ah E is 59 kgs(given)
sm
62.2 = 808.6 5+x+x+6+x-5
ur
yo

Total of 26 number (13 is two 468=160+175+3x+1 B + C + D + E = 236 kgs


e/
t.m
://

times ) = 1495 133=3x+1 B + E = 236 - 30 - 35 = 171 kgs


tp
ht

13 number = 1495-1350 = 145 3x=132 So A = 210 - 171 = 39kg


Average of remaining 24 = x=44
1350− 145 = 50.20 Fifth number will be 44 Sol:39.(c)
24
Sixth number will be 50 Total weight of A, B and C = 70
Sol:33.(b) Average will be 44+50
2
× 3=210
Let the 1st no, is a and average of Average =47 Total weight of A, B C and D =
last four no. is x. 60 × 4= 240 …(a)
Sum of the last four no. is 4x. Sol:36.(b) so D = 30 kg
a= (¾) × 4x =3x Average weight of a, b and c is And E = 35kg
Sum of all five positive no.=56 × 65kgs(given) Total weight of B, C, D and E =
5=280 Average weight of a and b is 63.5 59 × 4= 236 …(b)

e
a+4x=280 kgs(given) From equation (a) and (b)
3x+4x=280 Total weight of a,b and c will be As, 240- 236 =4
x=40 65 × 3 = 195kg So, A is 4kg heavier than E
Total weight of a and b will be Weight of A = 39 kg

l
Sol:34.(c) 63.5 × 2=127kg Total weight of A, D and E =
39+ 30 + 35 = 34.66 =
Average age of Kishore, his wife
ac Weight of c will be 195-127=68 3
and their child 6 years ago =38 kgs approximately = 35
Years Average weight of a and c is 67.5
Present Average age of Kishore, kg Sol:40.(d)
his wife and their child =44 years Total weight of a and c will be Sum of ratio = 3 + 4 + 7 = 14
Present Total age of Kishore his 135 kg 7 × 14,014 =
Highest price = 14
wife and their child=132Years So the average weight of a will be 7,007
Average age of his wife and their (135-68)=67kg
nn
child 8 years ago =32 Years 41.Sol:(b)
Present Average age of his wife Sol:37.(a) Average temperature for monday
and their child =40 years According to the question wednesday and friday=41°
Present Total age of his wife and 6A = C The average for wednesday,
their child=80 Years 2A = B thursday and friday = 42°
Present Average age of Sum of number = A + 2A + 6A = Temperature on Thursday = 43°C
Kishore=132-80=52 Years 9A Let temperature on monday=X
Average = 9A3 = 3A Sum of temperatures on
Pi

SSC CPO 2019 3A = 30 wednesday, thursday and friday


Sol:35.(d) A = 10 =126
Average of twelve numbers=39 C = 60 Sum of temperatures on
Average of last five numbers is 35 C - A = 50 wednesday and friday=83°C
Average of first four numbers is Sum of temperatures on monday
40 Sol:38.(c) wednesday and friday=123°C
Let the fifth number be x Average weight of A, B and C is Temperature on monday=
Then sixth number will be x+6 60kgs(given) 123°C-83°C=40°C
seventh number will be x-5 A + B + C = 210
As we know sum of all the Average weight of A, B, C and D
numbers is equal to average is 70 kgs(given) 42.Sol:(d)
multiplied by total number of A + B + C + D = 240 Average of twelve numbers=39
terms So, D = 30kg Average of last five numbers is 35
And E = 35kgs

www.ssccglpinnacle.com support@ssccglpinnacle.com Ph. 09729327755, 09817390373 126


/
Days 13-16 Average

s
sse
la
_c
ob
Average of first four numbers is
bo
ah
40
sm
ur
yo

Let the fifth number be x


e/
t.m
://

Then sixth number will be x+6


tp
ht

seventh number will be x-5


As we know sum of all the
numbers is equal to average
multiplied by total number of
terms
39 × 12=40 × 4+35 ×
5+x+x+6+x-5
468=160+175+3x+1
133=3x+1
3x=132
x=44

e
Sixth number will be 50
Seventh number will be 39
Average will be 39+50
2
Average =44.5

l
43.Sol:(b) ac
Average weight of a, b and c is
65kgs(given)
Average weight of c and b is 61.5
kgs(given)
Total weight of a,b and c will be
65 × 3 = 195kg
Total weight of c and b will be
nn
61.5 × 2=123kg
Weight of a will be 195-123=72
kgs
Average weight of a and c is 68.5
kg
Total weight of a and c will be
137 kg
So the average weight of c will be
Pi

(137-72)=65kg

www.ssccglpinnacle.com support@ssccglpinnacle.com Ph. 09729327755, 09817390373 127


/
Days 17-20 Ratio and Proportion

s
sse
la
_c
ob
RATIO & PROPORTION / के पूववत से गुणा िकया जाता है और Equivalent investment = (Amount
bo
ah संबंिधत प रणाम भी गुणा िकया जाता Invested) × (Time for which the
sm
अनुपात-समानुपात
ur

amount is invested).
yo

है । यिद a:b, c:d और e: f तीन


e/
t.m

अनुपात ह, तो उनका यौिगक We will take some examples to


://
tp

Key-points:/ मुख िबंदु:


ht

अनुपात ace: bdf होगा। जैसे 2:3, understand the concepts of


partnership./साझेदारी की िस ां त
3:4 और 3:5 के िलए, यौिगक
के समझने के िलए हम कुछ
1) What are antecedent and अनुपात होगा :
उदाहरण लगे।
consequent?/ पूववत और अनुव 2×3×3 = 18 = 3 i.e. 3 : 10
3×4×5 60 10
ा ह?
In the form x : y ; x is called the Example 1: A invests Rs. 10,000
antecedent (पूववत ) and y is
rd
9) 3 Proportional: What is the and B invests Rs. 20,000 for a
called consequent.( अनुव ) 3rd proportion of 5 and 7. Answer period of 1 year each. The profit,
after 1 year will be divided in the
will be
ratio:
2) x : y can also be written as x
y 5 और 7 का तीसरा आनुपाितक ा A ने 10,000 पये का िनवेश िकया
3) Duplicate ratio (squaring)/ है । उ र होगा : और B ने 20,000 पये का िनवेश
5 : 7 :: 7 : x िकया 1 वष की अविध के िलए| 1 वष

e
वगानुपात. E.g. duplicate ratio of
49 के बाद लाभ िकस अनुपात म
2 : 3 will be 22 : 32 i.e. 4 : 9 Where, 5x = 49 ⇒ x = 5
िवभािजत िकया जाएगा:
P rof it of A 10000×12 1
P rof it of B = 20000×12 = 2
4) Sub-duplicate Ratio (square 10) 4th Propotional: 4th

l
root) / वगमूलानुपात. E.g. proportional of 2, 5 and 7.
Example 2: A invests Rs.100 in a
Sub-duplicate Ratio of 9 : 4 will
ac Answer will be 2 : 5 :: 7 : x
business. After 6 months B joins
be √9 : √4 i.e. 3 : 2 Where, 2x = 7 ×5 = 35 ⇒ x = 35
2 with an investment of Rs. 200. At
the end of 1 year C joins with an
5) Triplicate Ratio (cube) / investment of Rs.100. What will
11) Componendo and dividendo
be the ratio of profit after 3 years?
घनानुपात will be in x3 : y 3 rule:
A एक वसाय म .100 का िनवेश
format. Triplicate ratio of 4 : 5 Example: If a = 3 and b = 2. करता है | 6 महीने के बाद B 200
will be 64 : 125 Then, aa+b = c , ∴ c = 3+2 = 5 के िनवेश के साथ जुड़ जाता है | 1 वष
−b d d 3−2 1
के अंत म 100 पये के िनवेश के
nn
i.e. c = 5 and d = 1
6) Sub-triplicate Ratio (cube c+d a साथ C भी साथ जुड़ जाता है | 3 साल
Also, = [Check yourself]
root)/घनमूलानुपात will be in form
c−d b बाद लाभ का अनुपात ा होगा?
3
√x : √3 y
12) Mean Proportion
(म नुपाती): Mean proportion of
7) Inverse Ratio two numbers, a and b, is given by
( ुत मिन ि ) of x : y will be
y:x √ab .
िदये गये दो अनुपातों का
Pi

म पनुपाती उनके गुणनफल का


8) Compound Ratio (यौिगक वगमुल होता है | Example 3: In the previous case
अनुपात): If two or more ratios if the total profit after 3 years is
are given, then the antecedent of Partnership/ साझेदारी 10,000. What will be the share of
one is multiplied with antecedent A?
of other and respective Concept / िस ां त: िपछले सवाल म अगर 3 साल के बाद
consequents are also multiplied. The ratio of profits of two or कुल मुनाफ़ 10,000 पये है तो A
If a : b, c : d and e : f are three more partners will be equal to the का िह ा ा होगा?
ratios, then their compound ratio ratio of their respective Share of A in total profit =
3 3
will be ace : bdf. E.g. For 2 : 3, 3 equivalent investment ratio. 3+5+2 ×10, 000 = 10 ×10000 = 3×1000
: 4 and 4 : 5, compound ratio will दो या अिधक भागीदारों के मुनाफे = 3000
be : का अनुपात उनके संबंिधत समक
िनवेश के अनुपात के बराबर होगा
यिद दो या दो से अिधक अनुपात िदए Note: In case someone invest
जाते ह, तो एक के पूववत को दू सरे in-between the total period of
time, just take care for what time

www.ssccglpinnacle.com support@ssccglpinnacle.com Ph. 09729327755, 09817390373 128


Days 17-20 Ratio and Proportion

s
sse
la
_c
ob
an amount is invested. Similarly (c) 12 तीसरे आनुपाितक का अनुपात ात
bo
ah
there can be a case when (d) 14 कर |
sm
ur

someone opts out in that case


yo

SSC CGL 10 June 2019


e/
t.m

also, the time for which the


://

Q3. The prices of two articles are (Morning)


tp
ht

amount was invested will be


in the ratio 4 : 5. If the price of (a) 2 : 1
taken into consideration and not
the first article is increased by x% (b) 3 : 2
the full period.
and that of the other is decreased (c) 1 : 2
अगर कोई कुल अविध के
बीच म िनवेश करता है , तो ान रख by 30% , then the new prices of A (d) 2 : 3
िक रािश का िनवेश िकतने समय के and B will be in the ratio 10 : 7.
िलए िकया गया है । इसी तरह एक The value of x is : Q6. If a : b = 3 : 2 then (5a + 2b) :
ऐसा सवाल हो सकता है जब कोई दो व ुओं की कीमत 4 : 5 के (3a + 4b) is equal to:
बाहर िनकल जाता है , उस अनुपात म ह | यिद पहली व ु की यिद a : b = 3 : 2 है , तो (5a + 2b) :
सवाल म भी िनवेश का समय ही कीमत x % से बढ़ा दी जाए और (3a + 4b) का मान ा होगा ?
ान म रखा जाएगा और न की पूरा दू सरी व ु की कीमत 30% से कम SSC CGL 10 June 2019
समय|
कर दी जाए, तो A और B की नयी (Afternoon)
कीमतों का अनुपात 10 : 7 हो जाता

e
(a) 16 : 15
Variety Questions
है | x का मान है : (b) 8 : 7
SSC CGL 7 June 2019 (c) 19 : 17
Q1. The ratio of the ages of A and
(Afternoon) (d) 17 : 14
B, four years ago, was 4 : 5. Eight

l
(a) 24.5
years from now, the ratio of the
(b) 22.5 Q7. If a : b = 2 : 5 , c : b = 3 : 4 ,

ages?
ac
ages of A and B will be 11 : 13.
What is the sum of their present

A और B की उ का अनुपात चार
वष पहले 4 : 5 था | अब से आठ वष
(c) 25
(d) 20

Q4. A sum of Rs. x is divided


among A , B and C such that the
then a : b : c is equal to :
यिद a : b = 2 : 5 , c : b = 3 : 4 है ,
तो a : b : c का मान ात कर |
SSC CGL 12 June 2019
(Evening)
बाद, A और B की उ का अनुपात
ratio of shares of A and B is 7 : (a) 6 : 15 : 20
11 : 13 होगा | उनकी वतमान उ
12 and that of B and C is 8 : 5. If (b) 8 : 20 : 15
का योग ा है ?
the difference in the shares of A (c) 2 : 5 : 4
nn
SSC CGL 4 June 2019
and C is Rs. 214 , then the value (d) 2 : 5 : 3
(Morning)
of x is :
(a) 80 years
x पये की रािश A, B और C म इस Q8. If (5a - 3b) : (4a - 2b) = 2:3,
(b) 96 years
कार िवभािजत की जाती है िक A then a:b is equal to:
(c) 72 years
और B के िह े का अनुपात 7 : 12 यिद (5a - 3b) : (4a - 2b) = 2:3 है ,
(d) 76 years
है और B तथा C के िह े का तो a : b का मान ा होगा ?
अनुपात 8 : 5 है | यिद A और C के SSC CGL 13 June 2019
Q2. If x is subtracted from each
िह ों म 214 पये का अंतर है , तो x (Afternoon)
Pi

of 23, 39, 32 and 56, the numbers


का मान ात कर | (a)3:4
so obtained in this order are in
SSC CGL 7 June 2019 (b)2:3
proportion. What is the mean
(Evening) (c)5:8
proportional between (x + 4) and
(a) 11556 (d)5:7
(3x + 1)?
(b) 11128
यिद x को 23, 39, 32 और 56 म से
(c) 11770 Q9. A sum of Rs. 4360 was to be
ेक से घटाया जाता है , तो ा
(d) 11342 divided among A , B , C and D in
होने वाली सं ाएं समानुपात म आती
the ratio of 3 : 4 : 5 : 8, but it was
ह | (x + 4) और (3x + 1) के बीच
Q5. What is the ratio of the mean divided in the ratio of 31 : 41 : 51 : 81
म आनुपाितक ात कर |
proportional between 4.8 and by mistake. As a result:
SSC CGL 4 June 2019
10.8 and the third proportional to 4360 पये की एक रािश को A, B,
(Afternoon)
0.4 and 2.4? C तथा D के बीच 3 : 4 : 5 : 8 के
(a) 15
4.8 तथा 10.8 के बीच मा अनुपात म िवभािजत करना था,
(b) 10
समानुपाती तथा 0.4 और 2.4 के बीच

www.ssccglpinnacle.com support@ssccglpinnacle.com Ph. 09729327755, 09817390373 129


Days 17-20 Ratio and Proportion

s
sse
la
_c
ob
लेिकन इसे भूलवश 1 : 41 : 51 : 81 के failure ratio would have been 9 : Q15. If a:b:c=1:3:5, what is the
bo
3
ah 5. The total number of candidates 4a−b+2c ?
value of 3(a+b+c)
अनुपात म िवभािजत कर िदया गया |
sm
ur
yo

प रणाम प: who appeared for the examination यिद a:b:c=1:3:5 है , तो का


e/

4a−b+2c
t.m

3(a+b+c)
://

SSC CHSL 1 July 2019 was :


tp

मान ा होगा ?
ht

(Evening) एक परी ा म, सफलता और


SSC CHSL 8 July 2019
(a) A received Rs. 956 more/ A असफलता का अनुपात 5 : 2 था |
(Evening)
को 956 पये अिधक िमले | यिद असफल अ थ की सं ा 14 8
(a) 27
(b) B received Rs. 318 more/ B अिधक होती , तो सफलता और
(b) 10
को 318 पये अिधक िमले | असफलता का अनुपात 9 : 5 होता | 27
(c) 11
(c) D received Rs. 1144 less/ D इस परी ा म कुल िकतने अ थ 27

को 1144 पये कम िमले | शािमल ए थे ? (d) 1


3
(d) C received Rs. 132 less/ C को SSC CHSL 2 July 2019
132 पये कम िमले | (Evening) Q16. Rs. 8000 is distributed
(a) 210 among A, B and C such that they
Q10. Two numbers are in the (b) 196 receive notes of Rs. 500, Rs. 200

e
ratio 3:4. On increasing each of (c) 126 and Rs. 100 respectively. The
them by 30, the ratio becomes (d) 203 amounts received by them are in
9:10. The numbers are : the ratio 15:2:3. What was the
दो सं ाएं 3 : 4 के अनुपात म ह | Q13. Three numbers are in the ratio of the numbers of notes of

l
दोनों को 30 से बढ़ाने पर, यह ratio ½ : ⅔ : ¾ . The difference Rs. 500, Rs 200 and Rs 100 ?
अनुपात 9 : 10 हो जाता है | ये
ac between the greatest and the 8000 पये A, B और C के बीच इस
सं ाएं ह : smallest number is 27. The कार िवत रत िकये जाते ह िक उ
SSC CHSL 8 July 2019 smallest number is मशः 500, 200 और 100 पये के
(Afternoon) तीन सं ाएं : ½ : ⅔ : ¾ के नोट ा होते ह | उनके ारा ा
(a) 30,40 अनुपात म ह | सबसे बड़ी तथा सबसे की गयी रािश का अनुपात 15 : 2 : 3
(b) 15,20 छोटी सं ाओं के बीच 27 का अंतर है | 500, 200 तथा 100 पये के नोटों
(c) 12,16 है | सबसे छोटी सं ा है - की सं ा का अनुपात ात कर |
(d) 18,24 SSC CHSL 4 July 2019 SSC CHSL 10 July 2019
(Evening) (Morning)
nn
Q11. The ratio of incomes of A (a) 81 (a) 3:1:3
and B is 2 : 3 and that of their (b) 40 (b) 3:3:1
expenditure is 1 : 2. If 90% of B’s (c) 72 (c) 4:1:2
expenditure is equal to the (d) 54 (d) 3:2:2
income of A , then what is the
ratio of the savings of A and B? Q14. Three numbers are in the Q17. What is the ratio between
A और B की आय का अनुपात 2 : 3 ratio ½ : ⅔ : ¾ . The difference the fourth proportional of 3, 4, 9
है तथा उनके य का अनुपात 1 : 2 between the greatest and the and the mean proportional
Pi

है | यिद B का 90% य A की आय smallest number is 27. The between 2 and 98?


के बराबर है , तो A और B की बचत average of the three numbers is : 3, 4, 9 के चतुथ अनुपाती और 2 और
का अनुपात ात कर | तीन सं ाएं ½ : ⅔ : ¾ के अनुपात 98 के म ानुपाती के बीच का
SSC CHSL 2 July 2019 म ह | सबसे बड़ी तथा सबसे छोटी अनुपात ा है ?
(Morning) सं ा के बीच 27 का अंतर है | तीनों SSC CHSL 11 July 2019
(a) 1 : 1 सं ाओं का औसत ात कर | (Afternoon)
(b) 9 : 8 SSC CHSL 5 July 2019 (a) 7:8
(c) 8 : 7 (Morning) (b) 7:6
(d) 3 : 2 (a) 40 (c) 8:7
(b) 81 (d) 6:7
Q12. In an examination, the (c) 69
success to failure ratio was 5 : 2. (d) 54 Q18. Three partners A, B and C
Had the number of failures been share profit and losses in the ratio
14 more, then the success to 3:4:7 If the profit for the year

www.ssccglpinnacle.com support@ssccglpinnacle.com Ph. 09729327755, 09817390373 130


Days 17-20 Ratio and Proportion

s
sse
la
_c
ob
before charging 30% tax is Rs. Q21. A, B and C are partners. लाभ 1,81,400 पये के पम ा
bo
ah
1,10, 166 What is B’s share of They share profits in the ratio of 5 होता है | आपातकालीन िनिध के िलए
sm
ur
yo

profit after tax? तीन साझेदार A, B : 3: 6. If A earns rs 1,92,380 as अलग रखी गयी रािश ात कर |
e/
t.m

और C लाभ और हािन का अनुपात


://

his share of profit, then the share SSC CPO 15 March 2019
tp
ht

3: 4: 7 म बां टते ह यिद 30% कर of C is: (Morning)


लगाने से पहले वािषक लाभ 1,10, A, B तथा C साझेदार ह | वे 5 : 3 : 6 (a)Rs. 27848
166 है ,तो कर के बाद B का लाभां श के अनुपात म लाभ बां टते ह | यिद A (b)Rs.18140
ा होगा ? को लाभ म अपने िह े के पम (c)Rs.22675
SSC CPO 16 March (Morning) 1,92,380 पये ा होते ह, तो C (d)Rs.24500
(a) Rs.9442.80 का िह ा ात कर |
(b) Rs.31476 SSC CPO 14 March 2019 Q24. A ,B, and C started a
(c) Rs.22033.20 (Morning) business by investing Rs. 55,000 ,
(d) Rs. 24673.10 (a) 2,30,856 Rs. 65,000 and Rs. 75,000
(b)1,15,428 respectively. A is a working
Q19. In an office of 1200 (c)82,449 partner and gets 20% of the profit

e
employees, the ratio of urban to (d)1,60,317 as a working allowance and
rural members of staff is 8 : 7. remaining is distributed in the
After joining some new Q22. Rs 15,000 was invested by proportion of their investment. If
employees, out of which 20 are A and B together to start a small the money received by C is Rs.

l
rural, the ratio becomes 5 : 4. The business. They got a profit of Rs 27,000 what is total profit?
number of new urban employees 2,000 at the end of the year. B A, B और C ने मशः 55,000 पये
is:
ac
1200 कमचा रयों वाले एक कायालय
म शहरी और ामीण सद ों का
अनुपात 8 : 7 है | कुछ नए
कमचा रयों, िजनम से 20 ामीण ह,
took his profit share of Rs 600,
How much did A invest?
A और B ारा एक साथ िकसी छोटे
वसाय की शु आत करने के िलए
15,000 पये िनवेश िकये गए | उ
, 65,000 पये तथा 75,000 पये
िनवेश करने एक वसाय की
शु आत की | A कायशील साझेदार
है और उसे लाभ का 20% कायकारी
भ े के प म िदया जाता है तथा शेष
के शािमल होने के बाद यह अनुपात वष के अंत म 2000 पये का लाभ लाभ का िवतरण उनके िनवेश के
5 : 4 हो जाता है | नए शहरी आ | B ने लाभ म से अपना िह ा अनुपात म िकया जाता है | यिद C के
कमचा रयों की सं ा ात कर | 600 पये िलया, तो A का िनवेश ारा ा रािश 27000 पये है , तो
nn
SSC CPO 15 March (Morning) िकतना था ? कुल लाभ ा है ?
(a) 100 SSC CPO 16 March 2019 SSC CPO 12 March 2019
(b) 85 (Evening) (Evening)
(c) 76 (a)9,000 (a) Rs. 85500
(d) 108 (b)2,000 (b) Rs.87750
(c)10,500 (c) Rs.76850
Q20. Rs 4, 06, 736 is divided (d)10,000 (d) Rs.70200
among A, B and C such that the
Pi

ratio between A and B is 2 : 3 and Q23. A, B and C are partners in a SSC CGL 2018 TIER II
B and C is 1 : 2. The share of C firm sharing profit in the ratio of
is: 4,06,736 को A, B और C म इस 3:4:5. If they set aside 4% of the Q1. The ratio of the income of A
कार बां टा जाता है िक A और B का profits as emergency fund and to that of B is 5:7. A and B save
अनुपात 2 : 3 है तथा B और C का shared the rest of the profit and B Rs. 4000 and Rs. 5000
अनुपात 1 : 2 है | C का िह ा ात gets his share of profit as Rs. respectively. If the expenditure of
कर | 1,81,400, the amount of profit set A is equal to 66 32 % of the
SSC CPO 15 March (Morning) aside for emergency fund is: expenditure of B, then the total
(a) Rs 73,952 A, B तथा C िकसी फम म साझेदार income of A and B is :
(b) Rs 1,10,928 ह तथा लाभ को 3:4:5 के अनुपात म A की आय और B की आय म 5 : 7
(c) Rs 2,64,796 बां टते ह | यिद वे लाभ का 4% भाग का अनुपात है | A और B महीने म
(d) Rs 2,21,856 आपातकालीन िनिध के प म अलग मश 4000 और 5000 पये की
रख दे ते ह तथा शेष लाभ का बँटवारा बचत करते ह | यिद A का य B के
करते ह तो B को उसके िह े का

www.ssccglpinnacle.com support@ssccglpinnacle.com Ph. 09729327755, 09817390373 131


Days 17-20 Ratio and Proportion

s
sse
la
_c
ob
य के 66 32 % के बराबर है , तो A तथा वष के अंत म उनके लाभों का the ratio of the present age of A
bo
ah अनुपात 2 : 5 है | यिद A ने 3 माह के to that of C ?
sm
और B की कुल आय ात कर |
ur
yo

SSC CGL Tier II 11 September िलए रािश िनवेश की थी, तो B ारा 5 वष पहले, A और B की उ का
e/
t.m

िनवेश की गयी रािश की अविध ( अनुपात 4 : 5 था | अब से 5 वष बाद,


://

2019
tp
ht

(a) Rs. 25,200 महीने म ) िकतनी थी ? SSC CGL A और B की उ म 6 : 7 का


(b) Rs. 24,000 Tier II 11 September 2019 अनुपात होगा | यिद वतमान म, C, B
(c) Rs. 26,400 (a) 7 से 10 वष छोटा है | तो, A और C की
(d) Rs. 28,800 (b) 6 वतमान आयु म ा अनुपात है ?
(c) 5 SSC CGL Tier II 12 September
Q2. If (a+b) : (b+c) : (c+a) = 7 : 6 (d) 9 2019
: 5 and a+b+c = 27, then what (a) 3:2
will be the value of a1 : b1 : 1c ? Q5. One year ago, the ratio of the (b) 5:4
यिद (a+b) : (b+c) : (c+a) = 7 : 6 : age (in years) of A to that of B (c) 4:3
5 तथा a+b+c = 27 है , तो a1 : b1 : 1c was 4:3. The ratio of their (d) 5:3
respective ages, 3 years from
का मान ा होगा ?

e
now, will be 6:5. What will be the Q8. A sum of Rs. x is divided
SSC CGL Tier II 11 September
ratio of respective ages of A and among A, B and C such that the
2019
B, 9 years from now? ratio of the shares of A and B is
(a) 3:6:4
एक साल पहले, A और B की उ ( 6:7 and that of B and C is 3:2. If
(d) 3:2:4
वष म ) का अनुपात 4:3 था | अब से

l
the difference between the shares
(c) 4:3:6
3 वष बाद उनकी उ का अनुपात 6 of A and C is Rs. 540, then the
(d) 3:4:2 ac : 5 होगा | अब से 9 वष बाद A और value of x is :
B की उ म ा अनुपात होगा ? x पये की रािश A, B और C म इस
Q3. When x is added to each of
SSC CGL Tier II 11 September कार िवभािजत की जाती है िक A
2,3,30 and 35, then the numbers
2019 और B के िह ों का अनुपात 6 : 7 है
obtained in this order, are in
(a) 7:6 तथा B और C के िह ों का अनुपात
proportion. What is the mean
(b) 10:9 3 : 2 है | यिद A और C के िह ों म
proportional between (x+7) and
(c) 9:8 540 पये का अंतर है , तो x का मान
(x-2)?
(d) 8:7 ा होगा ?
nn
जब x को 2, 3, 30 और 35 म जोड़ा
SSC CGL Tier II 12 September
जाता है तो इस म म ा होने
Q6. What is the ratio of the third 2019
वाली सं ाएं समानुपात म ह | (x+7)
proportional to 0.4 and 0.8, to the (a) 7425
तथा (x-2) के बीच म समानुपाती
mean proportional between 13.5 (b) 7020
ात कर |
and 0.24 ? (c) 7155
SSC CGL Tier II 11 September
0.4 तथा 0.8 के तीसरे आनुपाितक (d) 7290
2019
और 13.5 तथा 0.24 के बीच मा
(a) 7
समानुपाती म ा अनुपात होगा ? Q9. A, B and C invested their
Pi

(b) 4
SSC CGL Tier II 12 September capitals in the ratio of 2:3:5. The
(c) 6
2019 ratio of months for which A, B
(d) 5
(a) 5:4 and C invested is 4:2:3. If C gets
(b) 7:8 a share of profit which is Rs.
Q4. The ratio of investment by A
(c) 8:9 1,47,000 more than that of A,
to that by B is a business is 14:15
(d) 9:10 then B’s share of profit is :
and the ratio of their respective
A, B और C ने अपनी पूँजी 2 : 3 : 5
profits at the end of a year is 2:5.
Q7. 5 years ago, the ratio of the के अनुपात म लगाईं | A, B और C
If A invested the money for 3
age of A to that of B was 4:5. के ारा िनवेश की गयी रािशयों की
months, then for how much time
Five years hence, the ratio of the अविध ( महीने म ) का अनुपात 4 : 2
(in months) B invested his
age of A to that of B will be 6:7. : 3 है | यिद C को लाभ का एक
money?
If, at present, C is 10 years िह ा ा होता है जो A के िह े से
एक वसाय म A और B के ारा
younger than B, then what will be 1,47,000 पये अिधक है , तो इस
िकये गए िनवेश का अनुपात 14:15 है
लाभ म B का िह ा ात कर |

www.ssccglpinnacle.com support@ssccglpinnacle.com Ph. 09729327755, 09817390373 132


Days 17-20 Ratio and Proportion

s
sse
la
_c
ob
SSC CGL Tier II 12 September वष के अंत म कुल 86,800 पये का A is equal to twice the investment
bo
ah
2019 लाभ आ, तो इस लाभ म A और C of B and also equal to four times
sm
ur
yo

(a) Rs. 1,26,000 के िह ों म अंतर ात कर | the investment of C. If C’s share


e/
t.m
://

(b) Rs. 1,68,000 SSC CGL Tier II 13 September out of the total profit is Rs. 4,863,
tp
ht

(c) Rs. 1,05,000 2019 then the share of A in the profit is


(d) Rs. 1,89,000 (a) Rs. 12,600 :
(b) Rs. 7,000 A, B और C ने एक वसाय की
Q10. The ratio of income of A (c) Rs. 9,800 शु आत की | A के िनवेश का ितगुना
and B last year was 4:3, (d) Rs. 8,400 B के िनवेश के दोगुने तथा C के
respectively. The ratios of their िनवेश के चार गुना के बराबर है | यिद
individual incomes of the last Q12. Two numbers are in the कुल लाभ म C का िह ा 4863 पये
year and the present year are 3:4 ratio 3:5. If 13 is subtracted from है , तो इस लाभ म A का िह ा ात
and 5:6, respectively. If their total each, the new numbers are in the कर |
income for the present year is Rs. ratio 10:21. If 5 is added to each SSC CGL Tier II 13 September
8.04 lakh, then the income of B of the original numbers, then the 2019

e
last year was : ratio becomes : (a) Rs. 7,272
िपछले वष A और B की आय का दो सं ाएं 3 : 5 के अनुपात म ह | (b) Rs. 6,484
अनुपात मशः 4 : 3 था | िपछले वष यिद ेक सं ा से 13 घटा िदया (c) Rs. 9,726
और इस वष की उनकी आय का जाए, तो नयी सं ाओं का अनुपात (d) Rs. 8,105
अनुपात मशः 3 : 4 और 5 : 6 है | 10 : 21 हो जाता है | यिद मूल

l
यिद वतमान वष के िलए उनकी कुल सं ाओं म 5 जोड़ा जाए, तो नया Q15. If (5x+2y) : (10x+3y) = 5:9,
ac
आय 8.04 लाख पये है , तो िपछले
वष B की आय िकतनी थी ?
SSC CGL Tier II 12 September
2019
(a) Rs. 2.7 lakh
अनुपात ा होगा ?
SSC CGL Tier II 13 September
2019
(a) 5:7
(b) 23:33
then (2 x2 + 3y 2 ):(4 x2 + 9y 2 ) = ?
यिद (5x+2y) : (10x+3y) = 5:9 है ,
तो (2 x2 + 3y 2 ):(4 x2 + 9y 2 ) = ?
SSC CGL Tier II 13 September
2019
(b) Rs. 3.6 lakh (c) 4:5 (a) 31:87
(c) Rs. 2.4 lakh (d) 24:35 (b) 10:27
(d) Rs. 2.8 lakh (c) 16:47
nn
Q13. A, B and C spend 80%, (d) 1:3
Q11. A, B and C started a 85% and 75% of their incomes,
business with their capitals in the respectively. If their savings are Q16. A sum is divided among A,
ratio 2:3:5. A increased his in the ratio 8:9:20 and the B, C and D such that the ratio of
capital by 50% after 4 months, B difference between the incomes shares of A and B is 2:3, that of B
increased his capital by 33 31 % of A and C is Rs. 18,000 , then and C is 1:2 and that of C and D
after 6 months and C withdrew the income of B is : is 3:4. If the difference between
50% of his capital after 8 months, A, B और C अपनी आय का मशः the shares of A and D is Rs. 648,
Pi

from the start of the business. If 80%, 85% और 75% खच करते ह | then the sum of their shares is :
the total profit at the end of a year यिद उनकी बचत 8 : 9 : 20 के एक रािश A, B, C और D म इस
was Rs. 86,800, then the अनुपात म है और A तथा C की आय कार िवभािजत की जाती है िक A
difference between the shares of म 18000 पये का अंतर है , तो B की और B के िह ों का अनुपात 2 : 3 है
A and C in the profit was : आय ात कर | तथा B और C के िह ों का अनुपात
A, B और C ने एक वसाय की SSC CGL Tier II 13 September 1 : 2 है और C एवं D के िह ों का
शु आत की िजनकी पूँजी का 2019 अनुपात 3 : 4 है | यिद A और D के
अनुपात मशः 2 : 3 : 5 है | (a) Rs. 24,000 िह ों म 648 पये का अंतर है , तो
वसाय के आरं भ से 4 महीने बाद A (b) Rs. 27,000 उनके िह ों का योग ात कर |
ने अपनी पूँजी 50% से बढ़ा ली, B ने (c) Rs. 30,000 SSC CGL Tier II 13 September
6 महीने बाद अपनी पूँजी 33 31 % से (d) Rs. 36,000 2019
बढ़ा ली तथा C ने 8 महीने बाद (a) Rs. 2,052
अपनी 50% पूँजी िनकाल ली | यिद Q14. A, B and C started a (b) Rs. 2,160
business. Thrice the investment of (c) Rs. 2,484

www.ssccglpinnacle.com support@ssccglpinnacle.com Ph. 09729327755, 09817390373 133


Days 17-20 Ratio and Proportion

s
sse
la
_c
ob
(d) Rs. 1,944 जब x को 21, 22, 60 तथा 64 म से (b) 23 : 22
bo
ah घटाया जाता है , तो इस कार ा (c) 10 : 9
sm
ur

सं ाएं समानुपात म ह | (x+1) तथा


yo

Q17. A started a business with a (d) 9 : 10


e/
t.m

(7x+8) के बीच मा समानुपाती


://

capital of Rs. 54000 and admitted


tp

ात कर |
ht

B and C after 4 months and 6 Q6. If a : b = 5 : 3, then (8a - 5b) :


SSC CGL 6 June 2019
months respectively. At the end (8a + 5b) is equal to :
(Morning)
of the year, the profit was divided (a) 27 यिद a : b = 5 : 3 है , तो (8a - 5b) :
in the ratio 1:4:5. What is the (b) 18 (8a + 5b) का मान ा होगा ?
difference between the capitals (c) 24 SSC CGL 11 June 2019
invested by B and C ? (d) 21 (Morning)
A ने 54000 पये की पूँजी से एक (a) 3 : 13
वसाय की शु आत की तथा Q3. The ratio of present ages of A (b) 2 : 5
मशः चार और छः महीने बाद B and B is 8 : 9. After 9 years, the (c) 3 : 11
एवं C को शािमल कर िलया | वष के ratio will become 19 : 21. C is 3 (d) 5 : 11
अंत म लाभों का िवतरण 1:4:5 के years younger to B. What is the
अनुपात म िकया गया | B और C के present age (In years) of C ?

e
Q7. If a : b = 2 : 3 and c : b = 5 :
ारा लगाई गयी पूँजी म ा अंतर है A और B की वतमान आयु का 6 , then a : b : c is equal to:
? अनुपात 8 : 9 है | 9 वष के बाद, यह यिद a : b = 2 : 3 तथा c : b = 5 : 6
SSC CGL TIER II (11 अनुपात 19 : 21 हो जाएगा | C, B से है , तो a : b : c का मान ात कर |
3 वष छोटा है | C की वतमान उ (

l
September 2019) SSC CGL 11 June 2019
(a) Rs. 1,08,000 वष म ) िकतनी है ? (Afternoon)
ac
(b) Rs. 1,62,000
(c) Rs. 2,16,000
(d) Rs. 3,24,000

Practice Questions
SSC CGL 6 June 2019
(Afternoon)
(a) 49
(b) 48
(c) 51
(a) 4 : 6 : 5
(b) 6 : 9 : 16
(c) 6 : 9 : 12
(d) 10 : 15 : 18

(d) 52 Q8. If a : b = 5 : 8 and c : b = 4 :


Q1. If x is added to each of 12, 3, then a : b : c is equal to:
28, 21 and 45, the numbers so Q4. The ratio of present ages of A यिद a : b = 5 : 8 और c : b = 4 : 3
nn
obtained, in this order, are in and B is 8 : 15. Eight years ago, है , तो a : b : c िकसके बराबर होगा ?
proportion. What is the mean the ratio of their ages was 6 : 13. SSC CGL 11 June 2019
proportional between (x+3) and What will be the ratio of ages of (Evening)
(4x + 1)? A and B after 8 years from now? (a) 15 : 24 : 28
यिद x को 12, 28, 21 और 45 म से A और B की वतमान उ का (b) 5 : 6 : 8
ेक म जोड़ा जाता है , तो इस अनुपात 8 : 15 है | आठ साल पहले, (c) 15 : 24 : 32
कार ा सं ाएं समानुपात म ह | उनकी उ का अनुपात 6 : 13 था | (d) 5 : 8 : 6
(x+3) और (4x + 1) का मा अब से 8 वष बाद A और B की उ
Pi

समानुपाती ात कर | म ा अनुपात होगा ? SSC CGL 7 Q9. If a : b = 2 : 3, then (5a - 2b) :


SSC CGL 4 June 2019 June 2019 (Morning) (5a + 2b) is equal to:
(Evening) (a) 5 : 8 यिद a : b = 2 : 3 है , तो (5a - 2b) :
(a) 15 (b) 9 : 14 (5a + 2b) का मान ात कर |
(b) 18 (c) 10 : 17 SSC CGL 12 June 2019
(c) 10 (d) 5 : 9 (Morning)
(d) 12 (a) 3 : 7
Q5. If a : b = 4 : 5, then (2a + 3b) (b) 2 : 7
Q2. When x is subtracted from : (3a + 2b) is equal to : (c) 1 : 3
each of 21, 22, 60 and 64, the यिद a : b = 4 : 5 है , तो (2a + 3b) : (d) 1 : 4
numbers so obtained, in this (3a + 2b) का मान ा होगा ?
order, are in proportion. What is SSC CGL 10 June 2019 Q10. If a : b = 2 : 3, then (5a +
the mean proportional between
(Evening) 3b) : (6a - 2b) is equal to :
(x+1) and (7x+8).
(a) 22 : 23

www.ssccglpinnacle.com support@ssccglpinnacle.com Ph. 09729327755, 09817390373 134


Days 17-20 Ratio and Proportion

s
sse
la
_c
ob
यिद a : b = 2 : 3 है , तो (5a + 3b) : (a) 56 SSC CHSL 5 July 2019
bo
ah
(6a - 2b) का मान ात कर | (b) 42 (Afternoon)
sm
ur
yo

SSC CGL 12 June 2019 (c) 50 (a) 75


e/
t.m
://

(Afternoon) (d) 48 (b) 40


tp
ht

(a) 19 : 6 (c) 25
(b) 3 : 2 Q14. Two numbers A and B are (d) 50
(c) 17 : 5 in the ratio 5 : 2. If 4 is added to
(d) 10 : 7 each number then this ratio Q17. Two numbers are in the
becomes 9 : 4. If 5 is subtracted ratio 7:5. On diminishing each of
Q11. If a : b = 5:7, then (5a - 3b) : from each of the original numbers them by 40, the ratio becomes
(4a - 2b) is equal to: , then the ratio of A and B will 27:17. The sum of the numbers is
यिद a : b = 5:7 है , तो (5a - 3b) : be: :
(4a - 2b) का मान ा होगा ? दो सं ाएं A और B 5 : 2 के दो सं ाएं 7 : 5 के अनुपात म ह |
SSC CGL 13 June 2019 अनुपात म ह | यिद ेक सं ा म 4 दोनों म से 40 घटाने पर, यह अनुपात
(Morning) जोड़ा जाए, तो यह अनुपात 9 : 4 हो 27 : 17 हो जाता है | इन दोनों
जाता है | यिद ेक मूल सं ा म से सं ाओं का योग ात कर |

e
(a)2 : 3
(b)5 : 4 5 घटाया जाए, तो A और B का SSC CHSL 5 July 2019
(c)4 : 3 अनुपात ा हो जाएगा ? SSC (Evening)
(d)3 : 2 CHSL 3 July 2019 (Afternoon) (a) 300

l
(a) 3 : 1 (b) 240
Q12. The ratio of the present ages (b) 8 : 3 (c) 325
ac
of A and B is 6 : 5. Four years
ago, this ratio was 5 : 4. What
will be the ratio of ages of A and
B after 12 years from now?
A और B की वतमान आयु का
(c) 7 : 2
(d) 4 : 1

Q15. Incomes of A and B are in


the ratio 5:3 and their expenditure
(d) 275

Q18. Two numbers are in the


ratio 3:4. On increasing each of
them by 30, the ratio becomes
अनुपात 6 : 5 है | चार वष पहले, यह are in the ratio 9:5. If income of 9:10. The sum of the numbers is :
अनुपात 5 : 4 था | अब से 12 वष बाद A is twice the expenditure of B, दो सं ाएं 3 : 4 के अनुपात म है |
A और B की उ म ा अनुपात then what is the ratio of savings दोनों को 30 से बढ़ाने पर, यह
nn
होगा ? of A and B? अनुपात 9 : 10 हो जाता है | इन
SSC CHSL 2 July 2019 A और B की आय 5 : 3 के अनुपात सं ाओं का योग है - SSC CHSL 8
(Afternoon) म है और उनके य 9 : 5 के July 2019 (Morning)
(a) 3 : 2 अनुपात म ह | यिद A की आय B के (a) 35
(b) 8 : 7 य से दोगुनी है , तो A और B की (b) 32
(c) 9 : 8 बचत का अनुपात ात कर | (c) 30
(d) 7 : 6 SSC CHSL 4 July 2019 (d) 25
(Morning)
Pi

Q13. Eight years ago, the ratio of (a) 2:3 Q19. What is the ratio of mean
ages of A and B was 9 : 10. The (b) 1:1 proportional of 14.4 and 3.6 and
ratio of their ages four years from (c) 3:2 the third proportional of 5 and 4?
now will be 12 : 13. What is the (d) 3:4 14.4 और 3.6 के बीच मा
age (In years) of C now, if his age समानुपाती और 5 तथा 4 के तीसरे
is 6 years more than that of A? Q16. Two numbers are in the आनुपाितक का अनुपात ा है ?
आठ वष पहले, A और B की उ म ratio 7:5. On diminishing each of SSC CHSL 11 July 2019
9 : 10 का अनुपात था | अब से चार them by 40, the ratio becomes (Evening)
वष बाद उनकी उ का अनुपात 12: 27:17. The difference between the (a) 4:9
13 हो जाएगा | C की वतमान आयु ( numbers is : (b) 5:8
वष म ) ात कर, यिद उसकी उ A दो सं ाएं 7 : 5 के अनुपात म ह | (c) 9:4
की उ से 6 साल अिधक है | दोनों म से 40 घटाने पर, यह अनुपात (d) 8:5
SSC CHSL 3 July 2019 27 : 17 हो जाता है | इन दोनों
(Morning) सं ाओं के बीच ा अंतर है ?

www.ssccglpinnacle.com support@ssccglpinnacle.com Ph. 09729327755, 09817390373 135


Days 17-20 Ratio and Proportion

s
sse
la
_c
ob
Q 20. What is the ratio of the (a) 3 : 5 (d) 80
bo
ah
mean proportional between 8.1 (b) 4 : 3
sm
ur
yo

and 3.6 and the third proportional (c) 3 : 4 Q27. Rs. 7,80,516 is divided
e/
t.m
://

of 2 and 3? (d) 5 : 3 between A, B, C and D in the


tp
ht

8.1 और 3.6 के बीच मा ratio of 2: 3: 4:3. What is the part


समानुपाती और 2 तथा 3 के तीसरे Q24. The ages of A and B are in of C?
आनुपाितक का अनुपात ा है ? the ratio 5:7. Five years ago, 7,80,516 को 2:3:4:3 के अनुपात म
SSC CPO 12 March 2019 their ages were in the ratio 5:8. A , B , C और D के बीच बां टा गया
(Evening) The respective present ages(in है | C का िह ा ा है ?
(a) 5:6 years) are: SSC CPO 14 March 2019
(b) 5:4 A और B की उ 5 : 7 के अनुपात म (Evening)
(c) 4:5 है | पां च वष पहले उनकी उ 5 : 8 (a) Rs 1,30,086
(d) 6:5 के अनुपात म थी | उनकी वतमान (b) Rs 2,60,172
आयु ( मशः ) है - (c) Rs 2,24,562
Q21. What is ratio of mean SSC CPO 14 March 2019 (d) Rs 1,95,129

e
proportional between 3.6 and (Morning)
12.1 and third proportional (a)10,14 Q28.Two numbers are in the ratio
between 2 and 11? (b)20,28 4:9. If both the numbers are
3.6 और 12.1 के बीच मा (c)25,40 increased by 12, the ratio
समानुपाती तथा 2 और 11 के बीच

l
(d)15,21 becomes 11:21, the sum of the
तीसरे आनुपाितक म ा अनुपात है original numbers is:
?

(Evening)
(a)36:5
(b)6:5
ac
SSC CPO 13 March 2019
Q25. Divide Rs 8,288 between A,
B and C such that the proportion
of their shares is 5 : 7 :9. The
share of C is:
8,288 पये को A, B और C म इस
दो सं ाएँ 4 : 9 के अनुपात म है |
यिद दोनों सं ाओं म 12 की वृ
होती है , तो अनुपात 11 : 21 हो जाता
है | मूल सं ाओं का योग है :
SSC CPO 15 March 2019
(c)11:36 कार िवभािजत कर िक उनके (Evening)
(d)6:55 िह ों का अनुपात 5 : 7 : 9 हो | C (a)52
का िह ा ात कर | (b)64
nn
Q22 What is the sum of the mean SSC CPO 16 March 2019 (c)128
proportional between 10.8 and (Afternoon) (d)104
4.8 and the third proportional of 2 (a)2032
and 4? / 10.8 तथा 4.8 के बीच मा (b)3552 Q29. The sum of three numbers is
समानुपाती तथा 2 और 4 के तीसरे (c)3872 777. The ratio between the first
आनुपाितक का योग ा है ? (d)2612 two numbers is 7:9 and the ratio
SSC CPO 12 March 2019 between the second and third
(Morning) Q26. In one bag 50 paisa, 25 number is 3:7. The second
Pi

(a) 15.2 paisa and 1 rupee coins are in the number is:
(b) 11.2 ratio of 5: 8: 1. If the total cost of तीन सं ाओं का योग 777 है | पहली
(c) 8.2 all the coins is 55 rupees then दो सं ाओं के बीच का अनुपात 7 :
(d) 10.2 how many coins of 25 paisa are 9 है तथा दू सरी और तीसरी सं ा के
in the bag? बीच का अनुपात 3 : 7 है | दू सरी
Q23. What is the ratio of mean एक बैग म 50 पैसे , 25 पैसे और 1 सं ा है :
proportional between 1.8 and 3.2 पए के िस े 5 :8 :1 के अनुपात SSC CPO 15 March 2019
and the third proportional of 5 म है | यिद सभी िस े की कुल (Evening)
and 3? कीमत 55 पए है तो बैग म 25 पैसे (a)252
1.8 तथा 3.2 के बीच मा के िकतने िस े है ? SSC CPO 14 (b)147
समानुपाती तथा 5 और 3 के तीसरे March 2019 (Evening) (c)63
आनुपाितक का अनुपात ा है ? (a) 25 (d)189
SSC CPO 13 March 2019 (b) 10
(Morning) (c) 50

www.ssccglpinnacle.com support@ssccglpinnacle.com Ph. 09729327755, 09817390373 136


Days 17-20 Ratio and Proportion

s
sse
la
_c
ob
Q 30. A, B and C started a पवन और सुनील की मािसक आय
bo
ah
business by investing Rs 1,37,500 Q32. A, B and C started a का अनुपात 4 : 3 है तथा उनके
sm
ur
yo

and Rs 1,62,500 and Rs 1,87,500 business by investing Rs 27,500, मािसक य का अनुपात 3 : 2 है |


e/
t.m

यिद पवन और सुनील मशः 4000


://

respectively. A is a working Rs 32,500 and Rs 37,500


tp
ht

partner and gets 20% of the profit respectively. A is a working पये और 6000 पये की बचत
as working allowance and partner and gets 20% of profit as करते ह, तो उनकी मािसक आय का
remaining is distributed in the working allowance and the योग ा होगा ?
proportion of their investment. If remaining is distributed in SSC MTS 2 August 2019
the total profit is Rs 2,19,375. proportion of their investments. If (Morning)
What is the share of C? the money received by C is Rs (a) Rs 60000
A, B और C ने मशः 1,37,500 13,500. What is total profit? (b) Rs 70000
पये , 1,62,500 पये तथा A, B और C ने मशः 27,500, (c) Rs 50000
1,84,500 पये िनवेश करके एक 32,500 तथा 37,500 पये िनवेश (d) Rs 36000
वसाय की शु आत की | A करके एक वसाय की शु आत की
कायशील साझेदार है तथा लाभ का | A कायशील साझेदार है तथा लाभ Q35. If U+V = 80 and U : V = 5 :
20% कायशील भ ा के प म लेता का 20% कायशील भ े के पम

e
3, then the value of U - V will be
है और शेष लाभ का िवतरण उनके लेता है और शेष रािश का िवतरण :
िनवेश के अनुपात म कर िदया जाता उनके िनवेश के अनुपात म कर िदया यिद U + V = 80 तथा U : V = 5: 3
है | यिद कुल लाभ 2,19,375 पये जाता है | यिद C के ारा ा रािश है , तो U - V का मान िकतना है ?
का है , तो C का िह ा ा होगा ? 13500 पये है , तो कुल लाभ ात

l
SSC MTS 2 August 2019
SSC CPO 13 March 2019 कर | (Afternoon)
(Evening)
(a) Rs 88,600
(b) Rs 64,500
(c) Rs 67,500
(d) Rs 62,700
ac SSC CPO 13 March 2019
(Morning)
(a) Rs 35,100
(b) Rs 38,425
(c) Rs 42,750
(a)20
(b)15
(c)30
(d)25

(d) Rs 43,875 Q36. The salary of Mahesh is Rs.


Q 31. A, B and C started a 12000 more than Sumit. If the
business by investing Rs. 55,000, SSC MTS ratio of Mahesh’s salary to
nn
Rs. 65,000 and Rs. 75,000 Sumit’s salary is 7 : 4, then what
respectively. A is a working Q33. If P : Q = 5:2,then (2P - 3Q) is the salary of Sumit ?
partner and gets 20% of the profit : (3P - 5Q) is equal to: महे श का वेतन सुिमत के वेतन से
and the remaining is distributed in यिद P : Q = 5 : 2 है , तो (2P - 3Q) : 12000 अिधक है | यिद महे श तथा
the proportion of their (3P - 5Q) का मान िकसके बराबर सुिमत के वेतन का अनुपात 7:4 है ,
investments. If total profit is Rs. होगा ? तो सुिमत का वेतन िकतना है ?
87,750. What is the share of A? SSC MTS 2 August 2019 SSC MTS 2 August 2019
A, B तथा C ने मशः 55,000, (Morning) (Afternoon)
Pi

65,000 तथा 75,000 पये िनवेश (a)5:6 (a)Rs 16000


करके एक वसाय की शु आत की (b)2:7 (b)Rs 12000
| A कायशील साझेदार है तथा लाभ (c)4:5 (c)Rs 18000
का 20% लेता है एवं शेष रािश का (d)3:4 (d)Rs 20000
िवतरण उनके िनवेश के अनुपात म
कर िदया जाता है | यिद कुल लाभ Q34. The ratio of monthly Q37. The ratio of monthly
87,750 पये है , तो A का िह ा incomes of Pawan and Sunil is incomes of Ram and Rahim is 4:3
ात कर | 4:3 and the ratio of their monthly and the ratio of their monthly
SSC CPO 12 March 2019 expenditures is 3:2. If Pawan and expenditures is 3:2. If each saves
(Morning) Sunil save Rs. 4000 and Rs 6000 Rs 5000 per month, then what is
(a) 27,000 respectively per month, then what the respective monthly incomes
(b) 37,500 is the sum of their monthly of Ram and Rahim?
(c) 23,000 incomes ? राम और रहीम की मािसक आय का
(d) 37,350 अनुपात 4 : 3 है तथा उनके मािसक

www.ssccglpinnacle.com support@ssccglpinnacle.com Ph. 09729327755, 09817390373 137


Days 17-20 Ratio and Proportion

s
sse
la
_c
ob
य का अनुपात 3 : 2 है | यिद उनम SSC MTS 5 August 2019 (b)14000
bo
ah
से ेक 5000 पये की मािसक (Morning) (c)16000
sm
ur
yo

बचत करता है , तो राम और रहीम (a) 120 (d)12000


e/
t.m

का वेतन ( मशः ) ात कर |
://

(b) 150
tp
ht

SSC MTS 2 August 2019 (c) 130 Q44. If x + y = 52 and x - y = 20,


(Evening) (d) 140 then find the value of x : y.
(a)Rs 12000 and Rs 9000 यिद x + y = 52 तथा x - y = 20 है ,
(b)Rs 10000 and Rs 7500 Q41. If x : y : z = 3:4:5 and x + y तो x : y का मान िकतना है ?
(c)Rs 16000 and Rs 12000 + z = 96, then what is the value of SSC MTS 6 August 2019
(d)Rs 20000 and Rs 15000 z? (Morning)
यिद x : y : z = 3:4:5 और x + y + z (a)3 : 2
Q38. For three numbers, the ratio = 96 है , तो z का मान िकतना है ? (b)3 : 4
of the first and the second number SSC MTS 5 August 2019 (c)9 : 4
is 2:3 and that of the second and (Afternoon) (d)7 : 5
the third number is 4:5. If the sum (a)42

e
of the three numbers is 140, then (b)36 Q45. The monthly incomes of A
what is the second number? (c)32 and B are Rs. 12000 and Rs. 8000
तीन सं ाओं के िलए, पहली और (d)40 respectively. Their monthly
दू सरी सं ाओं का अनुपात 2 : 3 है expenditures are in the ratio of 5 :
तथा दू सरी और तीसरी सं ाओं का

l
Q42. There are 150 students in a 3. If the save equal amount every
अनुपात 4 : 5 है | यिद इन तीनों school. If the ratio between the month, then What amount do they
ac
सं ाओं का योग 140 है , तो दू सरी
सं ा कौन सी है ?
SSC MTS 2 August 2019
(Evening)
(a)60
number of boys and girls is 4 : 1,
then find the mean proportional
between the number of boys and
girls.
िकसी िव ालय म 150 छा ह | यिद
save individually ?
A तथा B की मािसक आय मश:
12000 पए तथा 8000 पए है |
उनके मािसक य का अनुपात 5:3
है | यिद वे ित माह समान रािश
(b)48 लड़के तथा लड़िकयों की सं ा का बचाते है , तो उनमे से ेक हर माह
(c)96 अनुपात 4 : 1 है , तो िव ालय म िकतनी रािश बचाता है ?
(d)32 लड़कों और लड़िकयों की सं ा के SSC MTS 6 August 2019
nn
बीच म ानुपात िकतना है ? (Morning)
Q39. In a bag, the ratio of the SSC MTS 5 August 2019 (a) Rs. 2000
number of 2 rupee, 1 rupee and (Afternoon) (b) Rs. 1500
50 paise coins is 3:4:5. If the total (a)50 (c) Rs. 800
amount in the bag is Rs 250, then (b)40 (d) Rs. 1000
how many 1 rupee coins are (c)30
there? एक थैले म, 2 पये, 1 पये (d)60 Q46. If x+y+z = 360 and x:y:z =
और 50 पैसे के िस ों की सं ा का 4:3:2, then the value of y+z-x is :
Pi

अनुपात 3 : 4 : 5 है | यिद थैले म कुल Q43. The ratio between the यिद x+y+z = 360 तथा x:y:z =
रािश 250 पये है , तो 1 पये के monthly incomes of A and B is 3 4:3:2 है , तो y+z-x का मान िकतना
िस ों की सं ा ात कर | : 4 and their monthly है ?
SSC MTS 5 August 2019 expenditures are in the ratio of 2 : SSC MTS 6 August 2019
(Morning) 3. If each of them saves Rs. 4000 (Morning)
(a)70 monthly, then B’s income is : (a) 80
(b)100 A तथा B की मािसक आय का (b) 60
(c)60 अनुपात 3 : 4 है तथा उनके मािसक (c) 30
(d)80 य का अनुपात 2 : 3 है | यिद (d) 40
ेक 4000 ित माह बचाता है ,
Q40. If A:B is 2:3 and B-A = 28, तो B की आय िकतनी है ? Q47. If X : Y : Z = 1 : 2 : 3 and
then what is the value of B+A? SSC MTS 5 August 2019 X 2 + Y 2 + Z 2 = 224 , then the
यिद A:B = 2:3 तथा B-A = 28 है , (Evening) value of X+Y+Z is:
तो B+A का मान ात कर | (a)20000

www.ssccglpinnacle.com support@ssccglpinnacle.com Ph. 09729327755, 09817390373 138


Days 17-20 Ratio and Proportion

s
sse
la
_c
ob
यिद X : Y : Z = 1 : 2 : 3 तथा SSC MTS 6 August 2019 मोिहत तथा काश की मािसक आय
bo
ah (Evening) 2 : 3 के अनुपात म है | उनके
sm
2 2 2
X + Y + Z = 224 है , तो
ur
yo

X+Y+Z का मान िकतना है ? (a) 144 मािसक य 3 : 5 के अनुपात म ह |


e/
t.m

यिद उनम से ेक ितमाह 5000


://

(b) 120
tp

SSC MTS 6 August 2019


ht

(Afternoon) (c) 124 पये की बचत करता है , तो मोिहत


(a) 48 (d) 136 तथा काश की मािसक आय का
(b) 32 योग ात कर |
(c) 36 Q51. The monthly incomes of A SSC MTS 7 August 2019
(d) 24 and B are Rs12000 and Rs15000 (Morning)
respectively. The monthly (a) Rs 40000
Q48. The ratio in the number of expenditure of A is Rs8000. If the (b) Rs 50000
students in three classes is 3 : 4 : ratio of the monthly expenditures (c) Rs 60000
5. If 20 students are increased in of A and B is 2 : 3, then what is (d) Rs 42000
every class, then this ratio the sum of their monthly savings?
becomes 4 : 5 : 6. What was the A तथा B की मािसक आय मशः Q54. The ratio of two whole
12000 पये और 15000 पये है | A

e
original number of students in numbers is 5 : 7. Which of the
these classes taken together ? का मािसक य 8000 पये है | यिद following can be their sum?
तीन क ाओं म िव ािथयों का A और B के मािसक य का दो पूण सं ाओं का अनुपात 5 : 7 है
अनुपात 3 : 4 : 5 है | यिद ेक अनुपात 2 : 3 है , तो उनकी मािसक | उनका जोड़ िन म से ा हो
बचत का योग ा होगा ? सकता है ?

l
क ा म 20 छा बढ़ा िदये जाते ह तो
अनुपात बदलकर 4 : 5 : 6 हो जाता SSC MTS 6 August 2019 SSC MTS 7 August 2019

?
ac
है | तीनों क ाओं को िमलाकर उनम
छा ों की आरं िभक सं ा िकतनी है

SSC MTS 6 August 2019


(Afternoon)
(Evening)
(a) Rs 7000
(b) Rs 8000
(c) Rs 5000
(d) Rs 6000
(Morning)
(a) 54
(b) 60
(c) 46
(d) 68
(a) 280
(b) 240 Q52. The ratio of the ages of two Q55. If X 2 + Y 2 = 100 and X : Y
persons is 3 : 4. If the age of one = 4 : 3, then what is the value of
nn
(c) 320
(d) 300 of them is greater than the other X2 − Y 2 ?
by 8 years, then what is the sum यिद X 2 + Y 2 = 100 तथा X : Y =
Q49. The sum of two numbers is of their ages? 4 : 3 है , तो X 2 − Y 2 का मान ा
दो यों की उ म 3 : 4 का
77 and their ratio is 6 : 5. What is होगा ?
their difference ? अनुपात है | यिद उनम से एक की
SSC MTS 7 August 2019
दो सं ाओं का योग 77 है तथा आयु दू सरे से 8 वष अिधक है , तो
(Morning)
उनका अनुपात 6 : 5 है | उनका उनकी उ का योग ा होगा ?
(a) 18
अंतर िकतना है ? SSC MTS 6 August 2019
Pi

(b) 28
SSC MTS 6 August 2019 (Evening)
(c) 36
(Afternoon) (a) 54 years
(d) 24
(a) 4 (b) 58 years
(b) 6 (c) 60 years
Q56. If V1:V2 = 1:2 and
(c) 7 (d) 56 years
V1+V2=147, then what is the
(d) 5 value of V2-V1.
Q53. The monthly incomes of
यिद V1:V2 = 1:2 तथा
Mohit and Prakash are in the ratio
Q50. If X : Y = 13 : 12 and X - Y V1+V2=147 है , तो V2-V1 का मान
2 : 3. Their monthly expenditures
= 2, then what is the value of 2X ा होगा ?
+ 3Y? are in the ratio 3 : 5. If each saves
SSC MTS 7 August 2019
यिद X : Y = 13 : 12 तथा X-Y = 2 Rs5000 per month, then what is
(Afternoon)
है , तो 2X+3Y का मान ा होगा ? the sum of monthly incomes of
(a) 48
Mohit and Prakash?
(b) 56

www.ssccglpinnacle.com support@ssccglpinnacle.com Ph. 09729327755, 09817390373 139


Days 17-20 Ratio and Proportion

s
sse
la
_c
ob
(c) 98 SSC MTS 8 August 2019 िपता की वतमान उ ( वष म )
bo
ah
(d) 49 (Morning) िकतनी है ?
sm
ur
yo

(a) 200 SSC MTS 8 August 2019


e/
t.m
://

Q57. Rs 975 is distributed among (b) 280 (Afternoon)


tp
ht

Anuj, Bharat and Shekhar. The (c) 220 (a) 45


proportion of their shares is 3 : 4 : (d) 240 (b) 51
8 respectively. What is the share (c) 55
of Bharat? Q60. Ravi’s age is 3 of Shyam’s (d) 50
5
975 पये अनुज, भरत और शेखर के age. After x years the ratio of the
बीच बां टे गए | उनके िह ों का ages of Ravi and Shyam becomes Q63. Three positive numbers are
अनुपात मशः 3 : 4: 8 था | भरत 5 : 7. If initially the sum of their in the proportion 3 : 4 : 6. If the
का िह ा ात कर | ages is 32, then what is the value sum of their squares is 244, then
SSC MTS 7 August 2019 of x? what is the largest number?
(Afternoon) रिव की उ ाम की उ का 53 तीन धना क सं ाएँ 3 : 4 : 6 के
(a) Rs 198 भाग है | x वष के बाद, रिव और अनुपात म ह | यिद उनके वग का
योग 244 है , तो सबसे बड़ी सं ा

e
(b) Rs 260 ाम की उ का अनुपात 5 : 7 हो
(c) Rs 296 जाता है | यिद आरं भ म उनकी उ कौन सी है ?
(d) Rs 280 का योग 32 था, तो x का मान ात SSC MTS 8 August 2019
कर | (Afternoon)

l
Q58. A bag contains 5 rupee, 2 SSC MTS 8 August 2019 (a) 16
rupee, 1 rupee and 50 paise coins (Morning) (b) 8
ac
in the proportion 1 : 2 : 3 : 4. If
the total amount in the bag is Rs
168, then how many 50 paise
coins are there?
एक थैले म 5 पये, 2 पये, 1 पये
(a) 2
(b) 4
(c) 8
(d) 7
(c) 12
(d) 6

Q64. A bag contains one rupee,


50 paise and 25 paise coins in the
और 50 पैसे के िस े 1 : 2 : 3 : 4 Q61. What is the ratio of the proportion 5 : 7 : 9. If the total
के अनुपात म ह | यिद थैले म कुल mean proportional between 24 amount in the bag is Rs430, then
रािश 168 पये है , तो 50 पैसे के how many 25 paise coins are
nn
and 150 and the third proportional
िस ों की सं ा िकतनी है ? there?
between 12 and 6 √5 ?
SSC MTS 7 August 2019 िकसी थैले म एक पये, 50 पैसे और
24 तथा 150 के म ानुपाती और 12
(Evening) 25 पैसे के िस े 5 : 7 : 9 के
(a) 46 तथा 6 √5 के बीच तीसरे समानुपाती अनुपात म ह | यिद थैले म कुल रािश
(b) 52 म ा अनुपात है ? 430 पये है , तो 25 पैसे के िकतने
(c) 56 SSC MTS 8 August 2019 िस े ह ?
(d) 48 (Morning) SSC MTS 8 August 2019
(a) 2 : 1 (Afternoon)
Pi

Q59. The proportion of the (b) 1 : 2 (a) 400


number of students in three (c) 1 : 4 (b) 360
classes is 1 : 2 : 3. If 20 students (d) 4 : 1 (c) 380
are included in each class, then (d) 340
the proportion becomes 3 : 5 : 7. Q62. The sum of the present ages
What was initially the total of a father and his son is 78 years. Q65. The present age of a Manoj
number of students in the three After five years, the ratio of their is twice the sum of the ages of his
classes? ages becomes 7 : 4. What is the two children. After 20 years, the
तीन क ाओं म छा ों की सं ा का present age (in years) of the age of Manoj will become equal
अनुपात 1 : 2 : 3 है | यिद ेक father? to the sum of the ages of his two
क ा म 20 छा शािमल कर िलए िपता तथा पु की वतमान आयु का children. What is the present age
जाते ह, तो यह अनुपात 3 : 5 : 7 हो योग 78 वष है | पां च वष के बाद, of Manoj?
जाता है | इन तीन क ाओं म आरं भ उनकी उ का अनुपात 7 : 4 होगा | मनोज की वतमान आयु उसके दो
म िकतने छा थे ? ब ों की उ के योग से दोगुनी है |

www.ssccglpinnacle.com support@ssccglpinnacle.com Ph. 09729327755, 09817390373 140


Days 17-20 Ratio and Proportion

s
sse
la
_c
ob
20 वष के बाद, मनोज की उ उसके एक बैग म 3 : 2 : 3 के अनुपात म 1
bo
ah
दोनों ब ों की उ के योग के बराबर Q68. If the ratio of selected to पये, 50 पैसे और 10 पैसे के पम
sm
ur
yo

हो जायेगी | मनोज की वतमान आयु unselected candidates was 14 : 25.8 की रािश है | बैग म 50 पैसे के
e/
t.m

िकतनी है ? िकतने िस े ह?
://

25. If 35 less had applied and 10


tp
ht

SSC MTS 8 August 2019 less selected, the ratio of selected SSC MTS 9 August 2019
(Evening) to unselected would have been 3 : (Evening)
(a) 40 years 5. What is the Number of (a) 18
(b) 30 years candidates who had applied for a (b) 3
(c) 36 years job? (c) 36
(d) 35 years चयिनत और गैर-चयिनत उ ीदवारों (d) 12
का अनुपात 14 : 25 है | यिद 35 कम
Q66. The ratio of the age of a उ ीदवारों ने आवेदन िदया होता Q71. If 4A = 6B = 5C; then A : B
father and his son is 3 : 1. If the तथा 10 कम चयिनत ए होते, तो : C is :
product of their ages is 432, then चयिनत एवं गैर-चयिनत उ ीदवारों यिद 4A = 6B = 5C; तो A : B : C
what is the sum of their ages? का अनुपात 3 : 5 होता | इस नौकरी है -
िपता तथा पु की आयु 3 : 1 के के िलए िकतने उ ीदवारों ने आवेदन

e
SSC MTS 13 August 2019
अनुपात म है | यिद उनकी आयु का िदया था ? (Morning)
गुणनफल 432 है , तो उनकी आयु का SSC MTS 9 August 2019 (a) 20 : 15 : 24
योग ा है ? (Morning) (b) 15 : 10 : 12

l
SSC MTS 8 August 2019 (a) 200 (c) 15 : 20 : 24
(Evening) (b) 175 (d) 24 : 20 : 15
(a) 36 years
(b) 48 years
(c) 60 years
(d) 54 years
ac (c) 275
(d) 195

Q69. The employees of three


categories A, B and C get their
Q72. If A:B = 3:4, B:C = 2:3,
then the value of A + B : B + C :
C + A will be :
यिद A:B = 3:4, B:C = 2:3 है , तो A
Q67. The ratio of the sum of the wages in the ratio of 1 : 2 : 3. If + B : B + C : C + A का मान है :
salaries of A and B to the they get increment of 5%, 10% SSC MTS 13 August 2019
difference of their salaries is 11 : and 15% respectively, then what (Morning)
nn
1. The ratio of the sum of the will be the new ratio of their (a) 5:6:7
salaries of B and C to the wages ? (b) 7:10:9
difference of their salaries is also तीन ेणी के कमचारी A, B और C 1 (c) 7:5:6
11 : 1. If A’s salary is the highest : 2 : 3 के अनुपात म मजदू री ा (d) 3:4:6
and C’s is the lowest then what is करते ह, यिद वे मश: 5%, 10%
B’s salary (in Rs), given that the एवं 15% की वेतन-वृ ा करते Q73. Seventy eight is divided into
total of their salaries is Rs18,200? ह | उनकी मजदू रयों का नया two parts such that the ratio
A और B के वेतन के योग तथा उनके अनुपात ा होगा ? between four times the first part
Pi

वेतन के अंतर म 11 : 1 का अनुपात SSC MTS 9 August 2019 and five times the second part is
है | B और C के वेतन के योग तथा (Evening) 14 : 15. The first part is :
उनके वेतन के अंतर म भी 11 : 1 का (a) 21 : 44 : 69 अठह र को दो भागों म इस कार
ही अनुपात है | यिद A का वेतन (b) 7 : 22 : 23 िवभािजत िकया जाता है िक पहले
सवािधक तथा C का वेतन सबसे कम (c) 21 : 22 : 23 भाग के चौगुना और दू सरे भाग के
है , तो B का वेतन ात कर जब यह (d) 7 : 21 : 23 पां च गुना का अनुपात 14:15 है |
िदया आ है िक उनके वेतनों का पहला भाग है :
योग 18200 पये है | Q70. There are 1 rupee, 50 paise SSC MTS 13 August 2019
SSC MTS 9 August 2019 and 10 paise coins are in a bag in (Morning)
(Morning) the ratio of 3 : 2 : 3 and make the (a) 42
(a) 8500 amount Rs. 25.8. How many (b) 36
(b) 5500 50-paise coins are there in the bag (c) 30
(c) 6000 ? (d) 48
(d) 7200

www.ssccglpinnacle.com support@ssccglpinnacle.com Ph. 09729327755, 09817390373 141


Days 17-20 Ratio and Proportion

s
sse
la
_c
ob
Q74. A bag contains ten rupee, Q77. If a : b = 2 : 5, b : c = 4 : 7 Q80. In a school there are 550
bo
ah
five rupee and two rupee notes in and c : d = 9 : 14, then the value students. The ratio of the boys to
sm
ur
yo

the ratio 10 : 5 : 2. The total value of a : b : c : d is : that of the girls is 6 : 5. How


e/
t.m

यिद a : b = 2 : 5, b : c = 4 : 7 और
://

of five rupee notes in the bag is many more girls should join the
tp
ht

Rs 84 more than that of two rupee c : d = 9 : 14, तो a : b : c : d school so that the ratio becomes 5
notes. The total value of ten rupee िकतना है ? : 6?
notes in the bag (in Rs) is: SSC MTS 14 August 2019 एक िव ालय म, 550 छा ह |
एक थैले म दस पये, पां च पये (Morning) लड़कों और लड़िकयों की सं ा म 6
और दो पये के नोट 10 : 5 : 2 के (a) 72 : 180 : 245 : 490 : 5 का अनुपात है | इस अनुपात को 5
अनुपात म ह | थैले म पां च पये के (b) 72 : 180 : 315 : 490 : 6 करने के िलए िकतनी अित र
नोटों का कुल मान दो पये के नोटों (c) 72 : 144 : 315 : 490 लड़िकयों को िव ालय म दा खला
के कुल मान से 84 पये अिधक है | (d) 36 : 180 : 315 : 490 लेना चािहए ?
थैले म दस पये के नोटों का कुल SSC MTS 14 August 2019
मान ( पये म ) है : Q78. The number of students (Evening)
SSC MTS 13 August 2019 studying A, B and C in a school (a) 25

e
(Afternoon) are in the proportion 12 : 15 : 16. (b) 170
(a) 350 There is a proposal to increase the (c) 50
(b) 450 number of students studying A, B (d) 110
(c) 300 and C, respectively by 50%, 20%

l
(d) 400 and 50%. What will be the new Q81. Kartik’s father age is four
proportion of the number of times the age of kartik. Three
ac
Q75. If a : b = 2 : 3 and b : c = 2 :
3, then what is the value of (
3a2 + b2 − c2 ) : (a2 + 2b2 − c2 ) ?
यिद a : b = 2 : 3 है तथा b : c = 2 :
3 है , तो (
students studying A, B and C?
िकसी िव ालय म A, B तथा C की
पढ़ाई करने वाले छा ों की सं ा का
अनुपात 12 : 15 : 16 है | A, B तथा
C पढ़ने वाले छा ों की सं ा को
years ago, kartik’s father age was
seven times the age of kartik. The
present age of kartik is:
काितक के िपता की उ काितक की
उ से चार गुनी है | तीन साल पहले,
2 2 2 2 2
3a + b − c ) : (a + 2b − c ) 2 मशः 50%, 20% और 50% से काितक के िपता की उ काितक की
का मान ा होगा ? बढ़ाने का ाव है | A, B और C उ से सात गुना थी | काितक की
की पढ़ाई करने वाले छा ों की सं ा वतमान आयु है -
nn
SSC MTS 13 August 2019
(Afternoon) का नया अनुपात ा होगा ? SSC MTS 14 August 2019
(a) 3 : 5 SSC MTS 14 August 2019 (Evening)
(b) 4 : 5 (Afternoon) (a) 12 years
(c) 5 : 7 (a) 3 : 3 : 5 (b) 6 years
(d) 3 : 7 (b) 3 : 3 : 4 (c) 9 years
(c) 2 : 3 : 3 (d) 8 years
Q76. If three numbers are in the (d) 5 : 8 : 12
ratio 2 : 3 : 5 and the twice of Q82. The ratio of boys and girls
Pi

their sum is 200. The square of Q79. If A : B = 2 : 3, B : C = 4 : 5 in a college was 4 : 5. New


the largest of three numbers is: and C : D = 6 : 7, then the value students got admitted and the
यिद तीन सं ाएँ 2 : 3 : 5 के of A + BD + C is: number of boys went up by 50%
अनुपात म ह, तथा उनके योग का यिद A : B = 2 : 3, B : C = 4 : 5 and the number of girls went up
दोगुना 200 है , तो इन तीनों म सबसे और C : D = 6 : 7 है , तो A + BD + C by 60%. What is the new ratio of
बड़ी सं ा का वग होगा : का मान ा होगा ? boys and girls in the college?
SSC MTS 13 August 2019 SSC MTS 14 August 2019 एक कॉलेज म लड़कों तथा लड़िकयों
(Evening) (Evening) की सं ा म 4 : 5 का अनुपात है |
(a) 2500 (a) 3 नए छा ों का नामां कन आ और
(b) 1000 (b) 7 लड़कों की सं ा 50% तथा
(c) 625 (c) 5 लड़िकयों की सं ा 60% बढ़ गयी |
(d) 2250 (d) 2 इस कॉलेज म लड़कों और लड़िकयों
का नया अनुपात ा है ?

www.ssccglpinnacle.com support@ssccglpinnacle.com Ph. 09729327755, 09817390373 142


Days 17-20 Ratio and Proportion

s
sse
la
_c
ob
SSC MTS 16 August 2019 (c) Rs. 36100 दो सं ाओं x और y का योग 48 है
bo
ah
(Morning) (d) Rs. 38000 तथा उनका अंतर 6 है | तो x : y = ?
sm
ur
yo

(a) 3 : 5 SSC MTS 19 August 2019


e/
t.m
://

(b) 3 : 4 Q 86. If 0.8 : X :: 5 : 8, then X is (Morning)


tp
ht

(c) 5 : 8 equal to : (a) 8:9


(d) 2 : 3 यिद 0.8 : X :: 5 : 8, तो X िकसके (b) 7:6
बराबर है ? (c) 9:7
Q83. What is the proportion SSC MTS 16 August 2019 (d) 3:7
equivalent to xy : yz : xz , if X : Y : (Afternoon)
Z = 4 : 7 : 9? (a) 1.28 Q90. A sum is divided between
x : y : x के समतु समानुपात (b) 1.16 two people in the ratio 3:2. If one
y z z
(c) 1.32 person got Rs. 12 less than the
ात कर, यिद X : Y : Z = 4 : 7 : 9
(d) 1.24 other person, then what is the
है |
sum?
SSC MTS 16 August 2019
Q87. A number is divided into कोई रािश दो लोगों के बीच 3 : 2 के
(Morning)
अनुपात म िवभािजत की जाती है |

e
three parts such that three times
(a) 36 : 49 : 32
the first part, six times the second यिद एक को दू सरे से
(b) 16 : 21 : 14
part and eight times the third part 12 पये कम िमले, तो यह रािश
(c) 36 : 51 : 28
are equal. If the first part is Rs. िकतनी है ?
(d) 36 : 49 : 28

l
1600, then what is the third part ? SSC MTS 19 August 2019
एक सं ा को तीन भागों म इस (Morning)
Q84. The difference between the
ac कार िवभािजत िकया जाता है की (a) Rs. 60
cubes of two numbers is 999. The
पहला भाग का तीन गुना, दू सरे भाग (b) Rs. 50
numbers are in the ratio of 4 : 3.
का छह गुना और तीसरे भाग का (c) Rs. 72
What is the square of the smaller
आठ गुना बराबर है | यिद पहला भाग (d) Rs. 44
number ?
Rs. 1600 है , तो तीसरी भाग िकतना
दो सं ाओं के घनों का अंतर 999 है
है ? Q91. The ratio of two numbers is
| सं ाएं 4 : 3 के अनुपात म ह |
SSC MTS 16 August 2019 2:1. If each number is increased
छोटी सं ा का वग िकतना है ?
(Evening) by 5, then the new ratio becomes
nn
SSC MTS 16 August 2019
(a) Rs. 450 3:2. What is the sum of the
(Afternoon)
(b) Rs. 900 number?
(a) 49
(c) Rs. 600 दो सं ाओं का अनुपात 2 : 1 है |
(b) 81
(d) Rs. 750 यिद ेक सं ा को 5 से बढ़ा िदया
(c) 64
जाए, तो नया अनुपात 3 : 2 हो जाता
(d) 100
Q88. The sum and difference of है | सं ाओं का योग ात कर |
the two numbers are 27 and 3 SSC MTS 19 August 2019
Q85. A, B, C and D have to share
respectively. What is the ratio of (Morning)
Pi

profits in the ratio of 4 : 8 : 11 :


these two numbers ? (a) 15
15. If the difference between the
दो सं ाओं का योग और अंतर (b) 45
lowest and maximum proportions
मशः 27 और 3 है | दो सं ाओं (c) 30
is Rs. 22000, then what is the sum
का अनुपात िकतना है ? (d) 20
of the other two proportions ?
SSC MTS 16 August 2019
A, B, C और D को 4 : 8 : 11 : 15
(Evening) Q92. A sum is divided between
के अनुपात म लाभ बां टना है | यिद
(a) 5:3 Ram and Rahim in the ratio of 3 :
ूनतम और अिधकतम अनुपातों के
(b) 2:1 2. If Ram’s share is Rs. 36000,
बीच 22000 का अंतर है , तो अ
(c) 4:7 then the total amount is :
2 अनुपातों का योग ा है ?
(d) 5:4 िकसी रािश को राम तथा रहीम के
SSC MTS 16 August 2019
बीच 3:2 के अनुपात म बां टा जाता है |
(Afternoon)
Q89. The sum of two numbers x यिद राम का िह ा Rs. 36,000 है ,
(a) Rs. 39900
and y is 48 and their difference is तो कुल रािश िकतनी है ?
(b) Rs. 37050
6. Then x:y = ?

www.ssccglpinnacle.com support@ssccglpinnacle.com Ph. 09729327755, 09817390373 143


Days 17-20 Ratio and Proportion

s
sse
la
_c
ob
SSC MTS 19 August 2019 SSC MTS 19 August 2019 दोगुनी है , तब अिभनव और रे खा की
bo
ah
(Afternoon) (Evening) बचत का अनुपात है :
sm
ur
yo

(a) Rs. 60000 (a) 10:9 SSC MTS 20 August 2019


e/
t.m
://

(b) Rs. 40000 (b) 8:15 (Afternoon)


tp
ht

(c) Rs. 90000 (c) 9:10 (a) 21:16


(d) Rs. 72000 (d) 15:8 (b) 23:18
(c) 5:3
Q93. The ratio of the given two Q96. The ratio of present ages of (d) 3:2
numbers is 9 : 7. If 6 is added to A and B is 5 : 3. 9 years ago, the
each number, then the ratio ratio of their ages was 23 : 12. Q99. A person divides a certain
becomes 21 : 17. What is the What will be the ratio of their amount among his three sons in
difference between the given ages after 15 years ? the ratio of 2 : 3 : 8. If he had
numbers ? A और B की वतमान आयु का divided this amount in the ratio of
दी गयी दो सं ाओं का अनुपात 9:7 अनुपात 5:3 है | 9 वष पहले, उनकी 1 1 1
2 : 3 : 8 , then the son getting the
है | यिद ेक सं ा म 6 जोड़कर आयु का अनुपात 23:12 था | अब से lowest amount would have got
उसे बढ़ा िदया जाए तो अनुपात 15 वष बाद उनकी आयु का अनुपात

e
Rs. 2200 more. What is the total
21:17 हो जाता है | दी गयी दो ा होगा? amount ( in Rs ) ?
सं ाओं के बीच ा अंतर है ? SSC MTS 20 August 2019 एक एक िनि त रािश को
SSC MTS 19 August 2019 (Morning) अपने तीन पु ों म 2:3:8 के अनुपात

l
(Afternoon) (a) 35:24 म बां टता है | यिद उसने इस रािश को
(a) 4 (b) 7:5
2 : 3 : 8 के अनु पात म बां टा होता,
1 1 1

(b) 16
(c) 8
(d) 12
ac
Q94. A sum of Rs. 5200 is
(c) 34:25
(d) 3:2

Q97. Let x be the mean


proportional of 25.6 and 32.4 and
तो िजस पु को सबसे कम रािश


ई थी, उसे Rs. 2200 अिधक
होते | कुल रािश (Rs. म)
िकतनी थी?
SSC MTS 20 August 2019
divided amongst A, B, C and D y be the third proportional of 32 (Afternoon)
such that the ratio of shares of A and 48. Then, 3x:2y = ______ (a) 5980
and B is 2:3, that of B and C is मान लीिजए िक x, 25.6 और 32.4
nn
(b) 6556
4:5 and that of C and D is 1:2. का म ानुपाती है और y, 32 एवं 48 (c) 6578
What is the difference between का तृतीयानुपाती है | तब 3x:2y = (d) 5940
the shares of B and D? ___ .
5200 पये की रािश A, B, C और D SSC MTS 20 August 2019 Q100. Which number should be
म इस कार िवभािजत की जाती है (Morning) subtracted from the Numerator
िक A और B के िह ों का अनुपात 2 (a) 5:3 and Denominator of the fraction
: 3 है , B और C के िह ों का (b) 3:5 4
9 so that this fraction could be
अनुपात 4 : 5 है तथा C और D के (c) 4:5
Pi

made equal to the fraction 1 ?


िह ों का अनुपात 1 : 2 है | B और (d) 5:4 6
िभ 4 के अंश तथा हर ेक म
D के िह ों म ा अंतर है ? 9

SSC MTS 19 August 2019 Q98. The monthly incomes of कौन सी सं ा घटाई जानी चािहए
(Evening) Abhinav and Rekha are in the िजससे िभ को 61 के बराबर बनाया
(a) Rs. 1440 ratio of 6 : 5, while their monthly जा सके?
(b) Rs. 1360 expenditures are in the ratio of 9 SSC MTS 20 August 2019
(c) Rs. 1200 :8. If the income of Abhinav is (Evening)
(d) Rs. 1280 twice the expenditure of Rekha, (a) 3
then what is the ratio between the (b) 7
Q95. If a:b = 2:3, a:d = 3:4 and savings of Abhinav and Rekha ? (c) 2
e:d = 3:5 then e:b = ? अिभनव और रे खा की मािसक आय (d) 5
यिद a : b = 2 : 3 है , a : d= 3 : 4 है 6:5 के अनुपात म है , जबिक मािसक
तथा e : d = 3 : 5 है , तो e : b = ? य 9:8 के अनुपात म है | यिद
अिभनव की आय, रे खा के य से

www.ssccglpinnacle.com support@ssccglpinnacle.com Ph. 09729327755, 09817390373 144


Days 17-20 Ratio and Proportion

s
sse
la
_c
ob
Q101. If x, 8 and 27 are in between the number of girls in A
bo
ah
constant proportion, then what is and B ? Q107. The ratio of the number of
sm
ur
yo

the value of x ? एक क ा के अनुभाग A और B म males and females in a group is


e/
t.m

यिद x, 8 और 27 िनरं तर अनुपात म लड़कों की सं ा का अनुपात 2:3 है


://

6:7. Fifteen females leave the


tp
ht

है , तो x का मान ा है ? तथा अनुभाग A और B म लड़कों group. As a result, this ratio


SSC MTS 20 August 2019 और लड़िकयों की कुल सं ा का becomes 12:11. Now, if 6 males
(Evening) अनुपात 3:4 है | यिद B म लड़िकयों join the group, then what will be
(a) 32 की सं ा, B म लड़कों की सं ा the ratio of the number of males
(b) 8 की 50% है , तो A और B म and females in the group?
3
(c) 4 लड़िकयों की सं ा का अनुपात एक समूह म पु षों और मिहलाओं
3
64 िकतना होगा ? की सं ा का अनुपात 6 : 7 है |
(d) 27
SSC MTS 21 August 2019 पं ह मिहलाएं समूह से िनकल जाती
3a+4b
(Evening) ह | प रणाम प, यह अनुपात 12 :
Q102. If a : b = 2 : 3, then 4a+5b (a) 2:3 11 हो जाता है | अब, यिद 6 पु ष
is equal to : (b) 11:12 समूह म शािमल हो जाते ह, तो समूह
यिद a : b = 2 : 3, तो 3a+4b बराबर म पु षों और मिहलाओं की सं ा

e
4a+5b (c) 3:5
है : (d) 5:6 का अनुपात ा होगा ?
SSC MTS 21 August 2019 SSC MTS 22 August 2019
(Morning) Q105. Let y > 0 , If 5 : 15 :: x : (Morning)
(a) 6 : 7

l
90 and 162 : y :: y : 128, then 8x : (a) 3:2
(b) 20 : 23 y is equal to : (b) 6:5
(c) 19 : 23
(d) 18 : 23
ac
Q103. The ratio between the
divisor and remainder is 3 : 2 and
मान ले की, y > 0 , यिद 5 : 15 :: x :
90 और 162 : y :: y : 128 है , तब 8x
: y िकसके बराबर है :
SSC MTS 21 August 2019
(Evening)
(c) 4:3
(d) 5:4

Q108. The ratio of incomes of A


and B is 3:8 and the ratio of their
the ratio between the divisor and (a) 5:3 savings is 9:25. If the income of
the quotient is 7 : 12. If remainder (b) 10:7 A equals the expenditure of B,
is 14, then find the remainder (c) 3:2 then the ratio of expenditures of
nn
when the dividend is divided by (d) 4:3 A and B is:
9. A और B की आय म 3 : 8 का
भाजक तथा शेषफल का अनुपात Q106. The amount of Rs. 6859 is अनुपात है तथा उनकी बचत म 9 :
3:2 है और भाजक तथा भागफल का divided among A, B, C and D 25 का अनुपात है | यिद A की आय
अनुपात 7:12 है | यिद शेषफल 14 है , such that the shares of A and B B के य के बराबर है , तो A और B
तो भा को 9 से िवभािजत करने पर are in the ratio of 4 : 3, the shares के य म ा अनुपात है ?
ा शेषफल है : of B and C in 5 : 4 and the shares SSC MTS 22 August 2019
SSC MTS 21 August 2019 of C and D in the ratio of 6 : 5. (Morning)
Pi

(Morning) What is the share of B ? (a) 3:8


(a) 3 Rs. 6859 की रािश को A, B, C और (b) 6:17
(b) 6 D के बीच इस कार िवभािजत िकया (c) 5:12
(c) 4 जाता है की A और B के अंशों का (d) 2:5
(d) 5 अनुपात 4:3 होता है , B और C के
अंशों का अनुपात 5:4 होता है और C Q109. When x is added to each of
Q104. The ratio of the number of और D के अंशों का अनुपात 6:5 है | 8, 14, 20 and 30, then the number,
boys in sections A and B of a B का अंश िकतना है ? so obtained, in this order form a
class is 2 : 3 and the ratio SSC MTS 21 August 2019 proportion. What is the mean
between the total number of boys (Evening) proportion between (x-2) and
and girls in sections A and B is 3 (a) Rs. 1805 (7x+2)?
: 4. If the number of girls in B is (b) Rs. 1444 जब x को 8, 14, 20 और 30 म जोड़ा
50% of the number of boys in B, (c) Rs. 2508 जाता है , तो इस कार इस म म
then what will be the ratio (d) Rs. 2407 ा सं ाएँ समानुपात का िनमाण

www.ssccglpinnacle.com support@ssccglpinnacle.com Ph. 09729327755, 09817390373 145


Days 17-20 Ratio and Proportion

s
sse
la
_c
ob
करती है | (x-2) और (7x+2) का SSC MTS 22 August 2019 (a) 100
bo
ah
म ानुपाती ा है ? (Evening) (b) 150
sm
ur
yo

SSC MTS 22 August 2019 (a) 3:8 (c) 120


e/
t.m
://

(Morning) (b) 2:9 (d) 200


tp
ht

(a) 25 (c) 1:6


(b) 16 (d) 1:12 Q116. The ratio of the number of
(c) 24 boys and girls in a school 3:2. If
(d) 12 Q113. A, B and C are partners in there are 360 students in the
a firm sharing profit in the ratio school, then what is the number
Q110. x is the 4th proportional to of 3:4:5. If they set aside 4% of of girls in the school ?
12, 16 and 5; and 20, y, 15, 21 are the profits as emergency fund and एक िव ालय म लड़कों तथा
in proportion. Then the value of shared the rest of the profit and B लड़िकयों की सं ा म 3 : 2 का
(6x-y) is: gets his share of profit as Rs. अनुपात है | यिद िव ालय म 360
x, 12, 16 और 15 का चौथा 1,81,400, the amount of profit set छा ह, तो इस िव ालय म लड़िकयों
समानुपाती है ; तथा 20, y, 15, 21 aside for emergency fund is: की सं ा िकतनी है ?
समानुपात म ह | (6x-y) का मान है : A, B तथा C िकसी फम म साझेदार

e
SSC MTS 7 August 2019
SSC MTS 22 August 2019 ह तथा लाभ को 3:4:5 के अनुपात म (Evening)
(Afternoon) बां टते ह | यिद वे लाभ का 4% भाग (a) 144
(a) 9 आपातकालीन िनिध के प म अलग (b) 160
रख दे ते ह तथा शेष लाभ का बँटवारा

l
(b) 18 (c) 124
(c) 13 करते ह तो B को उसके िह े का (d) 142
(d) 12
ac
Q111. A sum of Rs. 3200 was to
be divided among A, B, C and D
in the ratio 4 : 6 : 7 : 3. But by
लाभ 1,81,400 पये के

SSC CPO 16 March 2019


(Afternoon)
पम ा
होता है | आपातकालीन िनिध के िलए
अलग रखी गयी रािश ात कर |
Q117. If l:m:n = 1:2:4, then
√5l2 + m2 + n2
यिद
is equal to :
l:m:n = 1:2:4 है , तो

mistake, it was divided in the (a)Rs. 27848


√5l2 + m2 + n2 का मान िकसके
बराबर है ?
ratio 3 : 5 : 6 : 2. As a result, who (b)Rs.18140
SSC MTS 13 August 2019
got 16 32 % less than her due? (c)Rs.22675
nn
(Evening)
3200 पये की रािश A, B, C और D (d)Rs.24500
(a) 4n
के बीच 4 : 6 : 7 : 3 के अनुपात म
(b) 5l
िवभािजत की जानी थी | लेिकन Q 114. What is the ratio of the
(c) 2m
भूल-वश, इसे 3 : 5 : 6 : 2 के mean proportional between 8.1
(d) 5
अनुपात म िवभािजत कर िदया गया | and 3.6 and the third proportional
प रणाम प, िकसे उसके िह े of 2 and 3?
Q118. The sum and difference
से 16 32 % कम िमला ? 8.1 और 3.6 के बीच मा
between two numbers are 27 and
समानुपाती और 2 तथा 3 के तीसरे
Pi

SSC MTS 22 August 2019 3 respectively. What is the ratio


(Evening) आनुपाितक का अनुपात ा है ?
of these two numbers ?
(a) C SSC CPO 12 March 2019
दो सं ाओं का योग और अंतर
(b) A (Evening)
मश: 27 और 3 है | दो सं ाओं
(c) B (a) 5:6
का अनुपात िकतना है ?
(d) D (b) 5:4
SSC MTS 16 August 2019
(c) 4:5
(Evening)
Q 112. What is the ratio of the (d) 6:5
(a) 5:3
mean proportional between 4.5 (b) 2:1
and 0.5, and the third proportional Q115. If x+y+z = 400 and x:y:z =
(c) 4:7
to 4.5 and 9.0? 1:1:2. Then find the value of z ?
(d) 5:4
4.5 और 0.5 के म ानुपाती और 4.5 यिद x+y+z = 400 तथा x:y:z =
और 9.0 के तृतीय समानुपाती म ा 1:1:2 है , तो z का मान ात कर |
Q119. If u+v = 84 and u-v = 4
अनुपात है ? SSC MTS 2 August 2019
then u:v is equal to
(Afternoon)

www.ssccglpinnacle.com support@ssccglpinnacle.com Ph. 09729327755, 09817390373 146


Days 17-20 Ratio and Proportion

s
sse
la
_c
ob
यिद u+v = 84 तथा u-v = 4 है , तो u Q126. The ratio of ages of A and
bo
ah
: v का मान होगा : Q123. The ratio between the B, four years ago, was 7:5. The
sm
ur
yo

SSC MTS 19 August 2019 present ages of a mother and her ratio of their ages, 6 years form
e/
t.m
://

(Afternoon) son is 4 : 1. 14 years from now, now will be 19:15. What is the
tp
ht

(a) 11:10 their ages will be in the ratio of 2 ratio of the present ages of A and
(b) 10:11 : 1. What is the present age ( in B?
(c) 10:9 years ) of the mother ? A और B की उ म चार वष पहले 7
(d) 9:10 िकसी माता और उसके पु की : 5 का अनुपात था | अब से छः वष
वतमान आयु का अनुपात 4:1 है | बाद उनकी उ म 19 : 15 का
Q120. The first, second and third आज से 14 वष के बाद उनकी आयु अनुपात हो जाएगा | A और B की
items of a proportion are 24, 32 का अनुपात 2:1 हो जायेगा | माता की वतमान उ म ा अनुपात है ?
and 48. What is the 4th item ? वतमान आयु (वष म ) है : SSC MTS 22 August 2019
िकसी समानुपात के थम, ि तीय SSC MTS 21 August 2019 (Evening)
और तृतीय पद 24,32,48 है | चौथा (Afternoon) (a) 2:1
पद ा है ? (a) 36 (b) 5:2

e
SSC MTS 20 August 2019 (b) 24 (c) 4:3
(Evening) (c) 32 (d) 3:2
(a) 54 (d) 28
(b) 84 Q127. If a:b:c = 1:3:5, what is the

l
(c) 64 Q124. If a:b=2:3 and b:c = 2:3, 4a−b+2c ?
value of 3(a+b+c)
(d) 72 then the value of ( 3a2 + b2 + c2 यिद a:b:c = 1:3:5 है , तो 4a−b+2c

2 2
ac
Q121. If (2x-y) : (5x+3y) = 3:8,
then find
2
the
(x + y ) : (x − y ) . 2
value of

यिद (2x-y) : (5x+3y) = 3:8 है , तो


):( a2 2
c2
+ 2b + ) = ?
यिद a : b = 2 : 3 तथा b:c = 2 : 3 है ,
तो ( 3a2 + b2 + c2 ):( a2 + 2b2 + c2
) का मान ा होगा ?
SSC MTS 21 August 2019
का मान ा होगा ?
SSC CHSL 11
(Evening)
8
(a) 27
10
July
3(a+b+c)

2019

(b)
(x2 + y 2 ) : (x2 − y 2 ) का मान ात (Afternoon)
27
(c) 11
कर | (a) 169:165 27
(d) 1
nn
SSC MTS 20 August 2019 (b) 165:169 3
(Afternoon) (c) 7:5
(a) 145:144 (d) 5:7 Q128. The price of a rubber is Rs.
(b) 257:255 3. The price of 20 rubbers is
(c) 144:143 Q125. Five year ago, the ratio of equal to that of three pens. The
(d) 133:122 the ages of A and B was 3:4. Five price of a pen is equal to that of
years from now, the ratio of their two pencils and the price of six
Q122. Ramesh is three times ages will be 4:5. What is the ratio pencils is equal to that of five
elder to Suresh. After two years markers. What is the price of one
Pi

of A and B, 10 years form now ?


Ramesh’s age will be twice the पां च साल पहले, A और B की उ marker ?
age of Suresh. What is the current का अनुपात 3 : 4 था | अब से पां च एक रबड़ का मू 3 पए है | 20
age of Ramesh. वष बाद, उनकी उ का अनुपात 4 : रबडो का मू तीन कलमों के मू
रमेश सुरेश से तीन गुना बड़ा है | दो 5 हो जाएगा | अब से 10 वष बाद A के बराबर है | एक कलम का मू दो
वष के बाद, रमेश की उ सुरेश की और B की उ म ा अनुपात होगा पिसल के मू के बराबर है और छह
उ से दोगुनी हो जाएगी | रमेश की ? पिसल का मू पां च माकर के मू
वतमान उ िकतनी है ? SSC MTS 22 August 2019 के बराबर है | एक माकर का मू
SSC MTS 21 August 2019 (Afternoon) िकतना है ?
(Morning) (a) 9:11 SSC MTS 14 August 2019
(a) 4 (b) 6:7 (Morning)
(b) 6 (c) 5:6 (a) Rs. 9
(c) 3 (d) 7:9 (b) Rs. 12
(d) 2 (c) Rs. 13.50
(d) Rs. 15

www.ssccglpinnacle.com support@ssccglpinnacle.com Ph. 09729327755, 09817390373 147


Days 17-20 Ratio and Proportion

s
sse
la
_c
ob
SSC CGL 3 March 2020 (a) 117
bo
ah
SSC CGL 2019 TIER I (Afternoon) (b) 78
sm
ur
yo

(a) 2:3 (c) 91


e/
t.m
://

Q1. If the base radius of 2 (b) 4:5 (d) 104


tp
ht

cylinders are in the ratio 3:4 and (c) 3:4


their heights are in the ratio of (d) 1:2 Q7. A sum of ₹x was divided
4:9, then the ratio of their between A, B, C and D in the
volumes is: Q4. Two numbers are in the ratio ratio 1 1 1 1
3 : 5 : 6 : 9 . If the
यिद दो बेलनों के आधार की ि ा 5:7. If the first number is 20, then difference between the shares of
का अनुपात 3 : 4 है तथा उनकी the second number will be: B and D is ₹ 832, then the value
ऊंचाई का अनुपात 4 : 9 है , तो उनके दो सं ाएँ 5 : 7 के अनुपात म ह | of x is:
आयतन का अनुपात ा होगा ? यिद पहली सं ा 20 है , तो दू सरी X पये की रािश का िवभाजन A, B,
SSC CGL 3 March 2020 सं ा िकतनी है ? C और D के बीच 31 : 51 : 61 : 91 के
(Morning) SSC CGL 3 March 2020 अनुपात म िकया गया | यिद B और
(a) 2:1 (Evening) D के िह ों म 832 पये का अंतर

e
(b) 1:4 (a) 18 है , तो x का मान ात कीिजए |
(c) 1:2 (b) 28 SSC CGL 4 March 2020
(d) 4:1 (c) 8 (Evening)
(d) 22 (a) ₹7,592

l
Q2. The ratio of the number of (b) ₹7,384
boys to the number of girls in a Q5. If 2x+1, x+2, 2 and 5 are in (c) ₹7,696
ac
school of 640 students is 5:3. If
30 more girls are admitted in the
school, then how many more
boys should be admitted so that
the ratio of boys to that of the
proportion, then what is the mean
proportional between 3.5(1-x)
and 8(1+x)? /
यिद 2x+1, x+2, 2 और 5 समानुपात
म ह, तो 3.5(1-x) एवं 8(1+x) के
(d) ₹7,488

Q8. The ratio of ages of A and B,


8 years ago, was 2:3. Four years
ago, the ratio of their ages was
girls, becomes 14:9 बीच मा समानुपाती ात कर | 5:7. What will be the ratio of their
640 छा ों वाले एक िव ालय म SSC CGL 4 March 2020 ages 8 years from now?
लड़कों की सं ा तथा लड़िकयों की (Morning) 8 वष पहले A और B की उ का
nn
सं ा म 5 : 3 का अनुपात है | यिद (a) 5.5 अनुपात 2 : 3 था | चार वष पहले,
इस िव ालय म 30 और लड़िकयों (b) 4.25 उनकी उ का अनुपात 5 : 7 था |
का नामां कन हो जाता है , तो िकतने (c) 5.25 अब से 8 वष के बाद उनकी उ का
अित र लड़कों का नामां कन होना (d) 4.5 अनुपात ा होगा ?
चािहए तािक लड़कों और लड़िकयों SSC CGL 5 March 2020
की सं ा का अनुपात 14 : 9 हो जाए Q6. The ratio of boys and girls in (Morning)
| a group is 7:6. If 4 more boys join (a) 7:8
SSC CGL 3 March 2020 the group and 3 girls leave the
Pi

(b) 4:5
(Morning) group, then the ratio of boys to (c) 3:4
(a) 15 girls becomes 4:3. What is the (d) 5:6
(b) 30 total number of boys and girls
(c) 25 initially in the group? / Q9. What x is added to each of
(d) 20 एक समूह म लड़कों तथा लड़िकयों 10,16, 22 and 32, the number so
का अनुपात 7 : 6 है | यिद 4 और obtained in this order are in
Q3. If x is the mean proportional लड़के समूह म आ जाते ह तथा 3 proportion? What is the mean
between 12.8 and 64.8 and y is लड़िकयाँ समूह से चली जाती ह, तो proportional between the numbers
the third proportional to 38.4 and लड़कों तथा लड़िकयों का अनुपात 4 (x+1) and (3x+1)?
57.6, then 2x:y is equal to: : 3 हो जाता है | आरं भ म इस समूह जब 10, 16, 22 तथा 32 म से ेक
यिद 12.8 तथा 64.8 का मा म लड़कों तथा लड़िकयों की कुल सं ा म x जोड़ा जाता है , तो इस
समानुपाती x है और y , 38.4 एवं सं ा िकतनी थी ? म म ा होने वाली सं ाएँ
57.6 का तीसरा समानुपाती है , तो SSC CGL 4 March 2020 समानुपात म होती ह | (x+1) तथा
2x:y का मान िकसके बराबर होगा ? (Afternoon)

www.ssccglpinnacle.com support@ssccglpinnacle.com Ph. 09729327755, 09817390373 148


Days 17-20 Ratio and Proportion

s
sse
la
_c
ob
(3x+1) के बीच मा समानुपाती अनुपात 1 : 2 है | इस वॉलेट म कुल Q16. If an amount of ₹990 is
bo
ah
ात कीिजए | िकतनी रािश है ? divided among A,B and C in the
sm
ur
yo

SSC CGL 5 March 2020 SSC CGL 6 March 2020 ratio 3:4:2, then B will get:
e/
t.m

यिद 990 पये की रािश A, B और C


://

(Afternoon) (Afternoon)
tp
ht

(a) 15 (a) ₹280 के बीच 3 : 4 : 2 के अनुपात म


(b) 12 (b) ₹360 िवभािजत की जाती है , तो B को
(c) 10 (c) ₹440 िकतनी रािश िमलेगी ?
(d) 9 (d) ₹110 SSC CGL 9 March 2020
(Evening)
Q10. A certain amount is divided Q13. The sum of the squares of 3 (a) ₹247.5
among Sunita, Amit and Vibha in natural numbers is 1029, and they (b) ₹440
the ratio of 2:3:4. If Vibha gets are in the proportion 1:2:4. The (c) ₹110
₹14,416, then the total amount is: difference between greatest (d) ₹350
सुनीता, अिमत तथा िवभा बके बीच
number and the smallest number SSC CHSL 2019
एक िनि त रािश 2 : 3 : 4 के अनुपात
is: Q1. Rs6,300 is divided between
म िवभािजत की जाती है | यिद िवभा
3 ाकृितक सं ाओं के वग का

e
को 14,416 पये िमलते ह, तो कुल X, Y, Z, such that X : Y = 7 : 5
रािश िकतनी है ? जोड़ 1029 है तथा वे 1 : 2 : 4 के and Y : Z = 4 : 3. Find the share
SSC CGL 5 March 2020 अनुपात म ह | सबसे बड़ी सं ा तथा of Y.
(Evening) सबसे छोटी सं ा के बीच ा अंतर 6300 पये को X, Y और Z के बीच
(a) ₹43,248 है ? इस कार िवभािजत िकया जाता है

l
(b) ₹3,604 SSC CGL 6 March 2020 िक X:Y = 7:5 तथा Y:Z = 4:3 है । Y
(c) ₹32,436
(d) ₹16,219
ac
Q11. If a sum of ₹ 1,180 is to be
divided among A, B and C, such
that 2 times A’s share, 5 times B’s
share and 7 times C’s share, are
(Evening)
(a) 18
(b) 15
(c) 31
(d) 21
का िह ा ात कीिजए।
CHSL 12-10-2020 (morning
shift)
(a) Rs 2,200
(b) Rs 2,000
(c) Rs 2,400
equal, then A’s share is:
Q14. If A:B = 3:5 and B:C = 2:3, (d) Rs 1,800
यिद 1,180 पये की एक रािश
then A:B:C is equal to:
nn
A,B,C और D के बीच इस कार
यिद A:B = 3:5 तथा B:C = 2:3 है , Q2. Rs 3,600 is divided between
िवभािजत की जाती है िक A के िह े
तो A:B:C का मान ा होगा ? Seema, Komal and Rita, such that
का दोगुना, B के िह े का 5 गुना
SSC CGL 7 March 2020 the ratios of the shares of Seema :
तथा C के िह े का 7 गुना बराबर है |
(Morning) Komal = 1.5 : 2 and Komal : Rita
A का िह ा िकतना है ?
(a) 6:10:15 = 2 : 2.5. Find Rita's share.
SSC CGL 6 March 2020
(b) 6:15:10 3,600 पये को सीमा, कोमल और
(Morning)
(c) 3:7:3 रीता म इस कार िवभािजत िकया
(a) ₹ 650
(d) 3:8:6 जाता है िक सीमा : कोमल = 1.5 : 2
Pi

(b) ₹ 500
और कोमल : रीता = 2 : 2.5 है । रीता
(c) ₹ 750
Q15. If 3A = 4B = 5C, then का िह ा ात कीिजए।
(d) ₹ 700
A:B:C is equal to: CHSL 12-10-2020 (afternoon
यिद 3A = 4B = 5C है , तो A:B:C shift)
Q12. In a wallet, there are notes
का मान िकसके बराबर है ? (a) Rs 1,500
of the denominations of ₹ 10 and
SSC CGL 9 March 2020 (b) Rs 1,300
₹ 50. The total notes is 12. The
(Morning) (c) Rs 1,400
number of ₹ 10 and ₹ 50 notes are
(a) 20:15:16 (d) Rs 1,200
in the ratio of 1:2. Total money in
(b) 10:7:6
the wallet is:
(c) 10:5:4 Q3. The sum of three numbers is
एक वॉलेट म, 10 पये तथा 50 पये
(d) 20:15:12 79. If the ratio of the first number
के नोट ह | कुल नोट 12 ह | 10 पये
to the second number is 4 : 7 and
तथा 50 पये के नोटों की सं ा का
that of the second number to the

www.ssccglpinnacle.com support@ssccglpinnacle.com Ph. 09729327755, 09817390373 149


Days 17-20 Ratio and Proportion

s
sse
la
_c
ob
third number is 4 : 5, then the CHSL 13-10-2020 (evening (c) Rs2,464
bo
ah
second number is: shift) (d) Rs2,386
sm
ur
yo

तीन सं ाओं का जोड़ 79 है । यिद (a) 6:10:15


e/
t.m

पहली से दू सरी सं ा का अनुपात


://

(b) 3:5:3 Q10. Two numbers are


tp
ht

4:7 है तथा दू सरी से तीसरी सं ा (c) 3:10:3 respectively 25% and 65% more
का अनुपात 4:5 है , तो दू सरी सं ा (d) 3:10:15 than a third number. The ratio of
कौन सी है ? the two numbers is:
CHSL 12-10-2020 (evening Q7. Six years ago, the average of दो सं ाएँ तीसरी सं ा से मशः
shift) the ages of Ravi, Mohan and 25% और 65% अिधक ह। दोनों
(a) 15 Govind was 32 years. If Shyam सं ाओं का अनुपात ा है ?
(b) 28 joins them now, the average of CHSL 14-10-2020 (evening
(c) 35 the ages of all four of them is 36 shift)
(d) 12 years. The present age of Shyam (a) 25 : 42
is: (b) 16 : 17
Q4. If 2145 : x :: 3003 : 42, then छह साल पहले रिव, मोहन और (c) 16 : 19
गोिवंद की उ का औसत 32 साल

e
the value of y so that x : 2508:: y: (d) 25 : 33
11704, is था। अगर ाम उनके साथ आ जाये,
यिद 2145 : x :: 3003 : 42 है , तो y तो उन चारों की उ का औसत 36 Q11. If an amount of 800 is
का मान इस कार ात कर िक x : साल है । ाम की वतमान आयु है : distributed between Ravi, Mohan
2508:: y: 11704 हो।

l
CHSL 14-10-2020 (morning and Govind in the proportions
CHSL 13-10-2020 (morning shift) 2:5:3, then the sum of the shares
shift)
(a)212
(b)156
(c)140
(d)96
ac (a) 35 years
(b) 32 years
(c) 40 years
(d) 30 years
of Mohan and Govind is:
यिद रिव, मोहन और गोिवंद के बीच
800 की रािश 2: 5: 3 के अनुपात म
बाटी जाती है , तो मोहन और गोिवंद
के िह े का योग है :
Q8. If x : y = 3 : 2 and x + y = 90. CHSL 15-10-2020 (morning
Q5. The ratio of man’s age to his Then the value 7(x - y) : (x + y) shift)
father’s age is 4:5, and the ratio of is: (a)Rs 560
nn
his age to his son’s age is 6:1. यिद x: y = 3: 2 और x+ y = 90 है । (b)Rs 400
Four years ago these ratios were तो मान 7 (x - y): (x y) मान है : (c)Rs 640
11:14 and 11:1, respectively. The
CHSL 14-10-2020 (morning (d)Rs 240
ratio of the age of the grandfather
shift)
to that of the grandson 12 years
from now will be: (a) 7 : 5 Q12. In a bag, white marbles and
एक की उ और उसके िपता (b) 2 : 3 red marbles are in the ratio of 3 :
की उ का अनुपात 4:5 है तथा (c) 7 : 9 5. If the number of red marbles
उसकी और उसके पु की उ म (d) 7 : 6 are 150, then how many white
Pi

6:1 का अनुपात है । चार वष पहले, ये marbles are there?


अनुपात मशः 11:14 और 11:1 थे। Q9. Dividing the amount एक बैग म, सफेद प र और लाल
दादा तथा उसके पोते की उ म अब Rs18,144 among three people A, प र 3: 5 के अनुपात म होते ह। यिद
से 12 वष बाद ा अनुपात होगा?
B, C in the ratio 3 : 5 : 8, the लाल प र की सं ा 150 है , तो
CHSL 13-10-2020 (afternoon
amount B gets more than A, is: सफेद प र िकतने ह?
shift)
तीन लोगों A, B, C के बीच की रािश CHSL 15-10-2020 (afternoon
(a) 12:5
Rs. 18,144 को 3: 5: 8 के अनुपात म shift)
(b) 14:3
(c) 18:5 िवभािजत करने पर रािश B के पास (a) 90
(d) 18:7 A से िकतनी अिधक रािश हो जाती (b) 60
है : (c) 30
Q6. If a:b = 3:5 and b:c = 2:3, CHSL 14-10-2020 (afternoon (d) 120
then the proportion a:b:c is: shift)
यिद a: b = 3: 5 और b: c = 2: 3 है , (a) Rs2,178
तो अनुपात a: b: c है : (b) Rs2,268

www.ssccglpinnacle.com support@ssccglpinnacle.com Ph. 09729327755, 09817390373 150


Days 17-20 Ratio and Proportion

s
sse
la
_c
ob
Q13. The ratio of the areas of two (a) 65 years
bo
ah
squares is 16 : 1. Find the ratio Q17. By adding 3 and 5 in (b) 58 years
sm
ur
yo

between their perimeters. numerator and denominator of a (c) 45 years


e/
t.m

दो वग के े ों का अनुपात 16: 1 है .
://

fraction it becomes 32 . If 1 and 3 (d) 78 years


tp
ht

उनके प रिध के बीच का अनुपात are subtracted and added from


ात कर। numerator and denominator Q20. The salaries of Vipin and
CHSL 15-10-2020 (evening respectively, it becomes 52 . Find Dinesh are in the ratio 5 : 8. If the
shift) the fraction. salary of each is increased by Rs
(a) 8 : 1 िकसी िभ के अंश और हर म 3 और 4,800, then new ratio becomes 7 :
(b) 3 : 1 5 जोड़ने पर , िभ 10. What is Vipin's salary?
3 हो जाता है ।
2

(c) 4 : 1 यिद 1 और 3 को मशः अंश और िविपन और िदनेश का वेतन 5: 8 के


(d) 12 : 1 हर म घटाया जाता है तो िभ 52 हो अनुपात म है । यिद ेक के वेतन म
4,800 पये की वृ ई है , तो नया
जाता है । अंश ात कीिजए।
Q14. The fourth proportional to अनुपात 7: 10 हो जाएगा। िविपन का
CHSL 19-10-2020 (morning
10, 12, 15 is: वेतन ा है ?
shift)
10, 12, 15 का चौथा आनुपाितक है :

e
CHSL 20-10-2020 (morning
(a) 75
CHSL 15-10-2020 (evening 6 shift)
(b) 7
shift) 7
(a) Rs 12,000
(c)
(a) 24 6 (b) Rs 10,000
(d) 7

l
(b) 22 5 (c) Rs 13,000
(c) 18 (d) Rs 12,500
(d) 20
ac
Q15. The proportion among three
numbers is 3 : 4 : 5 and their
LCM is 1800. The second
Q18. The ratio of the number of
men and women in a factory is 14
: 19. If the total number of
employees in the factory is 2145,
then the number of women in the
Q.21. When 50% of a number A
is added to B, the second number
B increases by 25%. The ratio
between the numbers A and B is:
number is: factory is: जब सं ा A का 50% B म जोड़
तीन सं ाओं का अनुपात 3: 4: 5 है एक कारखाने म पु षों और िदया जाता है , तो दू सरी सं ा B
और उनका LCM 1800 है । दू सरी मिहलाओं की सं ा का अनुपात 14: 25% बढ़ जाती है । सं ा A और B
nn
सं ा है : 19 है । यिद कारखाने म कमचा रयों के बीच का अनुपात है
CHSL 16-10-2020 (morning की कुल सं ा 2145 है , तो कारखाने CHSL 20-10-2020 (afternoon
shift) म मिहलाओं की सं ा है shift)
(a) 150 CHSL 19-10-2020 (evening (a) 3:2
(b) 30 shift) (b) 2:3
(c) 120 (a) 1367 (c) 1:2
(d) 90 (b) 1976 (d) 3:4
(c) 1645
Pi

Q16. What number must be (d) 1235


added to each of the numbers 8, Q.22.If (a+3b):(2a+4b)=3:5, then
13, 26 and 40 so that the numbers Q19. The ratio of present ages (in (a-b):(a+b) is equal to:
obtained in this order are in years) of a father and son is 15 : यिद (a+3b):(2a+4b)=3:5 तो
proportion? 8. Six years ago, the ratio of their (a-b):(a+b) के बराबर है :
िकस सं ा को 8, 13, 26 और 40 म ages was 13 : 6. What is the CHSL 20-10-2020 (evening
जोड़ा जाना चािहए तािक इस म म father's present age? shift)
ा सं ाएँ समानुपात म हों? एक िपता और पु की वतमान आयु (a) 1:2
CHSL 16-10-2020 (afternoon (वष म) का अनुपात 15: 8 है । छह (b) 2:1
shift) साल पहले, उनकी आयु का अनुपात (c) 3:2
(a) 2 13: 6 था। िपता की वतमान आयु ा (d) 2:3
(b) 3 है ?
(c) 1 CHSL 19-10-2020 (evening Q.23. The average age of a man
(d) 4 shift) and his son is 60 years. The ratio

www.ssccglpinnacle.com support@ssccglpinnacle.com Ph. 09729327755, 09817390373 151


Days 17-20 Ratio and Proportion

s
sse
la
_c
ob
of their ages is 13:7, respectively. आयु का योग 44 वष है । उनकी उ Q30. The length and breadth of a
bo
ah
What is the son’s age? का अंतर है : rectangle are in the ratio 5 : 3. If
sm
ur
yo

एक आदमी और उसके बेटे की CHSL 26-10-2020 (evening the length is 8 m more than the
e/
t.m

औसत आयु 60 वष है । उनकी आयु


://

shift) breadth, what is the area of the


tp
ht

का अनुपात मशः 13: 7 है । बेटे की (a) 30 years rectangle?


उ ा है (b) 11 years एक आयत की लंबाई और चौड़ाई 5:
CHSL 20-10-2020 (evening (c) 24 years 3 म है । 3. यिद लंबाई चौड़ाई से 8
shift) (d) 20 years मीटर अिधक है , तो आयत का
(a) 40 years े फल ा है ?
(b) 41 years Q.27. How much should be added CHSL 17-03-2020 (afternoon
(c) 42 years to each term of 4:7 so that it shift)
(d) 43 years become 2:3 (a) 240 m2
4: 7 के ेक पद पर िकतना जोड़ा (b) 380 m2
Q.24. Two numbers A and B are, जाना चािहए तािक यह 2: 3 हो जाए| (c) 360 m2
respectively, 80% and 20% more CHSL 17-03-2020 (morning (d) 400 m2

e
than a third number C. The ratio shift) Q31. The length and breadth of a
of the numbers A to B is: (a) 3 rectangle are in ratio 3:2. If its
दो सं ा A और B मशः , 80% (b) 4 perimeter is 730 cm, what is the
और 20% अिधक है एक तीसरी (c) 2 area of the rectangle?
सं ा C से । A से B की सं ा का

l
(d) 1 एक आयत की लंबाई और चौड़ाई 3:
अनुपात है 2 के अनुपात म है । यिद इसकी

shift)
(a) 5:4
(b) 4:5
(c) 3:2
ac
CHSL 26-10-2020 (morning Q.28. The perimeter of a
rectangle is 50 cm. Its area and
length are in the ratio of 5:1. Find
the length of the rectangle?
एक आयत की प रिध 50 सेमी है ।
प रिध 730 सेमी है , तो आयत का
े फल ा है ?
CHSL 17-03-2020 (evening
shift)
(a) 31,974 cm2
(d) 3;4 इसका े फल और लंबाई 5: 1 के (b) 24,452 cm2
अनुपात म है । आयत की लंबाई ात (c) 20,567 cm2
Q25. The perimeter of a कीिजये? (d) 28,976 cm2
nn
rectangular field is 32 meters and CHSL 17-03-2020 (morning
its sides are in the ratio 5:3, Then shift) Q32. The ratio of tables and
the sides of the field are:. (a) 15 cm chairs in a room is 7:9. If there
एक आयताकार े की प रिध 32 (b) 20 cm are 560 tables and chairs in the
मीटर है और इसके भुजा 5: 3 के (c) 18 cm room, then what is the number of
अनुपात म ह, िफर मैदान के भुजाएँ (d) 22 cm chairs?
ह एक कमरे म टे बल और कुिसयों का
CHSL 26-10-2020 (afternoon Q29. The average price of three अनुपात 7: 9 है । यिद कमरे म 560
Pi

shift) items is Rs14,265. If their prices टे बल और कुिसयां ह, तो कुिसयों की


(a) 10 m and 6 m are in the ratio 7 : 9 : 11, then the सं ा ा है ?
(b) 9 m and 7 m price of the costliest item is: CHSL 17-03-2020 (evening
(c) 12 m and 10 m तीन व ुओं का औसत मू 14,265 shift)
(d) 5 m and 3 m पये है । यिद उनकी कीमत 7: 9: 11 (a) 397
के अनुपात म ह, तो सबसे महं गी (b) 315
Q.26. The ages of Fatima and व ु की कीमत है : (c) 463
Ahmed are in the ratio of 3:8. The CHSL 17-03-2020 (afternoon (d) 489
sum of their present ages is 44 shift)
years. The difference of their ages (a) Rs 17,435 Q33. The incomes of two persons
is: (b) Rs 16,235 P and Q are in the ratio 5:6. If
फाितमा और अहमद की आयु 3: 8 (c) Rs 19,875 each of them saves Rs200 per
के अनुपात म है । उनकी वतमान (d) Rs 14,875 month, the ratio of their

www.ssccglpinnacle.com support@ssccglpinnacle.com Ph. 09729327755, 09817390373 152


Days 17-20 Ratio and Proportion

s
sse
la
_c
ob
expenditure is 3 : 4. Find the (b) 18 X और Y 3: 5 के अनुपात म पूँजी के
bo
ah
income of Q. (c) 16 साथ साझेदारी करते ह। 5 महीने के
sm
ur
yo

दो यों P और Q की आय 5: 6 (d) 14 बाद X अपनी पूँजी का 50% जोड़ता


e/
t.m

के अनुपात म है । यिद उनम से है , जबिक Y अपनी पूँजी का 60%


://
tp
ht

ेक ित माह 200 पये बचाता है , Q.37. How much will be िनकाल लेता है । ₹6.84 लाख के
तो उनके य का अनुपात 3: 4 है । decreased to each term of ratio वािषक लाभ म X का (₹लाख म)
Q की आय ात कीिजए। 16:19, so that it will be 7:6? िह ा ा है
CHSL 18-03-2020 (afternoon अनुपात 16:19 के ेक पद को CGL 2019 Tier-II (15-11-2020 )
shift) िकतना घटाया जाएगा, तािक यह 7: 6 (a) 3.72
(a) Rs750 हो जाए? (b) 4.2
(b) Rs800 CHSL 19-03-2020 (afternoon (c) 3.6
(c) Rs600 shift) (d) 3.12
(d) Rs740 (a) 37
(b) 36 Q.40: The sum of the present ages
Q34. If the perimeter and length (c) 35 of father and a son is 52 years.

e
of a rectangle are in the ratio 6:1 (d) 34 Four years hence, the son’s age
and the area of the rectangle is will be 41 that of the father. What
288 cm2 . Find the length of the SSC CGL2019 TIER-II will be the ratio of the ages of the
rectangle. Q.38. In a school, 83 of the son and father, 10 years from
यिद आयत की प रिध और लंबाई 6:

l
number of students are girls and now?
1 के अनुपात म है और आयत का the rest are boys, One -third of पु और िपता की वतमान आयु का

shift)
(a) 10 cm
ac
े फल 288 वग सेमी है । आयत की
लंबाई ात कीिजए।
CHSL 18-03-2020 (afternoon
the number of boys are below 10
years and 32 of the number of
girls are also below 10 years. If
the number of students of age 10
or more years is 260, then the
योग 52 वष है । चार साल बाद, बेटे
की उ िपता की उ की 41 होगी।
अब से 10 वष बाद पु और िपता की
आयु का अनुपात ा होगा?
CGL 2019 Tier-II (15-11-2020 )
(b) 12 cm number of boys in the school is: (a) 2 : 7
(c) 8 cm िकसी िव ालय म छा ों की सं ा (b) 2 : 5
(d) 9 cm का 83 भाग लड़िकयाँ और शेष
nn
(c) 3 : 8
लड़के है | लड़को की सं ा का एक (d) 1 : 3
Q35. Two numbers are
ितहाई 10 वष से कम है और
respectively 25% and 60% more Q41. The numerator of a fraction
लड़िकयों की सं ा का 32 भी 10 वष
than a third number. The ratio of is 3 more than the denominator.
से कम है | यिद 10 वष या उससे
the two numbers is: When 5 is added to the numerator
अिधक आयु वाले छा ों की सं ा
दो सं ाएँ , तीसरी सं ा से मशः and 2 is subtracted from the
260 है , तो िव ालय म लड़को की
25% और 60% अिधक ह। दो denominator, the fraction
सं ा ात कीिजए |
सं ाओं का अनुपात है :
Pi

8
becomes 3 . When the original
CGL 2019 Tier-II (15-11-2020 )
CHSL 18-03-2020 (evening
(a) 280 fraction is divided by 5 21 , the
shift)
(b) 300 fraction so obtained is:
(a) 20 : 30
(c) 234 एक िभ का अंश हर की अपे ा म 3
(b) 20 : 35
(d) 312 अिधक है । जब 5 को अंश म जोड़ा
(c) 25 : 32
जाता है और 2 को हर म से घटाया
(d) 21 : 31
Q39. X and Y enter into a जाता है , तो अंश 38 हो जाता है । जब
partnership with capital in ratio 3 मूल िभ को 5 21 से िवभािजत िकया
Q36. If x:y = 3:2 and x+y = 90,
: 5. After 5 months X adds 50% जाता है , तो ा होने वाला िभ ात
then the value of (x-y) is:
of his capital , while Y withdraws कीिजए
यिद x: y = 3: 2 और x y = 90 है , तो
60% of his capital. What is share CGL 2019 Tier-II (15-11-2020 )
(x-y) का मान है :
(in ₹ lakhs ) of X in the annual (a) 32
CHSL 19-03-2020 (morning
profit of ₹6.84 lakhs?
shift) (b) 41
(a) 12

www.ssccglpinnacle.com support@ssccglpinnacle.com Ph. 09729327755, 09817390373 153


Days 17-20 Ratio and Proportion

s
sse
la
_c
ob
(c) 1 िह ा िमला था, उसे ₹1,188 अिधक the ratio of the shares of A and B
bo
2
ah िमलता । रािश (₹ म) ात कीिजए is 1 : 3, that of B and C is 2 : 5
sm
(d) 3
ur

4
yo

CGL 2019 Tier-II (15-11-2020 ) and that of C and D is 2 : 3. If the


e/
t.m
://

(a) 6,840 difference between the shares of


tp

Q42. What x is added to each of


ht

9,15,21 and 31 the number so (b) 6,768 A and C is ₹ 3,510, then the share
obtained are in proportion . What (c) 7,008 of D is :
is the mean proportion between (d) 5,640 एक रािश A, B, C और D के बीच
the numbers (3x - 2) and (5x + 4) िवभािजत की जाती है , जैसे A और B
? Q45. The ratio of the monthly के शेयरों का अनुपात 1: 3 है , B और
9,15,21 और 31 ेक म x जोड़ा income of X and Y is 5 : 4 and C का अनुपात 2: 5 है और C और D
जाय तो ा सं ा अनुपात म ह। that of their monthly expenditure का अनुपात 2 : 3 है . यिद A और C
सं ा (3x - 2) और (5x 4) के बीच is 9 : 7 . If the income of Y is के शेयरों के बीच अंतर ₹ 3,510 है ,
मा अनुपात ा है ? equal to the expenditure of X, तो D का िह ा है
CGL 2019 Tier-II (15-11-2020 ) then what is the ratio of the CGL 2019 Tier-II (16-11-2020 )
(a) 35 saving of X and Y? (a) ₹ 6,075
X और Y की मािसक आय का

e
(b) 20 (b) ₹ 4,050
(c) 30 अनुपात 5: 4 है और उनके मािसक (c) ₹ 4,320
(d) 42 खच का अनुपात 9: 7 है । यिद Y की (d) ₹ 3,240
आय X के य के बराबर है , तो X
और Y की बचत का अनुपात ा है

l
Q43. The monthly incomes of A Q48. A,B and C invested capital
and B are in ratio 3 : 5 and the ? in ratio 5 : 7 : 4, the time of their
ac
ratio of their savings is 2 : 3 If the
income of B is equal to three
times the saving of A, then what
is the ratio of the expenditure of
A and B?
CGL 2019 Tier-II (16-11-2020 )
(a) 6 : 7
(b) 8 : 9
(c) 7 : 6
(d) 9 : 8
investment being in the ratio x : y
: z . If their profits are in the ratio
45 : 42 : 28, then x : y : z =?
A, B और C ने 5: 7: 4 के अनुपात म
पूँजी लगाई, उनके िनवेश का समय
A और B की मािसक आय 3: 5 के x: y: z के अनुपात म है । यिद उनका
अनुपात म है और उनकी बचत का Q46. The denominator of a लाभ 45: 42: 28 के अनुपात म है , तो
अनुपात 2 : 3 यिद B की आय A की fraction is 4 more than twice the
nn
x: y: z =?
बचत के तीन गुना के बराबर है , तो A numerator. When the numerator is CGL 2019 Tier-II (16-11-2020 )
और B के य का अनुपात ा है ? increased by 3 and the (a) 9 : 6 : 7
CGL 2019 Tier-II (15-11-2020 ) denominator is decreased by 3, (b) 6 : 7 : 9
(a) 5 : 8 the fraction becomes 32 . What is (c) 9 : 4 : 7
(b) 8 : 15 the difference between the (d) 7 : 9 : 4
(c) 7 : 11 denominator and numerator of the
(d) 3 : 7 original fraction ? Q49. If a : b : c = 1 : 1 : 1 ,
4 3 2
एक िभ का हर उसके अंश के
Pi

then a : b : c =?
b c a
Q44. A person divided a certain दोगुने से 4 अिधक है । जब अंश 3 से
यिद a : b : c = 1 : 1 : 1 तो
sum between his three sons in the बढ़ जाता है और हर म से 3 कम a b c
4 3 2
: : =?
ratio 3 : 4 : 5. Had he divided the िकया जाता है , तो िभ 3 हो जाता
2 b c a
CGL 2019 Tier-II (16-11-2020 )
sum in the ratio 31 , 41 , 51 the son, है । मूल िभ के हर और अंश के
(a) 12 : 9 : 8
who got the least share earlier, बीच ा अंतर होता है ?
(b) 8 : 9 : 24
would have got ₹1,188 more. The CGL 2019 Tier-II (16-11-2020 )
(c) 9 : 8 : 24
sum (in ₹) was : (a) 13
(d) 9 : 12 : 8
एक ने अपने तीनों बेटों के (b) 11
बीच 3: 4: 5. के अनुपात म एक (c) 12
Q50. When x is subtracted from
िनि त रािश को िवभािजत िकया था। (d) 10
each of 19, 28, 55 and 91, the
यिद उसने इस अनुपात को
numbers so obtained in this order
1 , 1 , 1 म िवभािजत िकया होता Q47. A certain sum is divided
3 4 5 are in proportion . What is the
,तो िजस बेटे को पहले सबसे कम between A, B, C and D such that

www.ssccglpinnacle.com support@ssccglpinnacle.com Ph. 09729327755, 09817390373 154


Days 17-20 Ratio and Proportion

s
sse
la
_c
ob
mean proportion between (x + 9) (b) 86 (c) ₹3,000
bo
ah
and x2 ? (c) 96 (d) ₹2,500
sm
ur
yo

जब x को 19, 28, 55 और 91 म से (d) 80


e/
t.m

घटाया जाता है , तो इस म म ा
://

SSC CPO 2019


tp
ht

सं ाएँ अनुपात म होती ह। (x + 9) Q53. The ratio of boys and girls Q56. A, B and C started a
और x2 के बीच मा अनुपात ा in a school is 27: 23. If the business by investing ₹13,750,
है ? difference between the number of ₹16,250 and ₹18,750,
CGL 2019 Tier-II (16-11-2020 ) boys and girls is 200, then find respectively. If B’s share in the
(a) 28 the number of boys . profit earned by them is ₹5,200,
(b) 24 एक ू ल म लड़कों और लड़िकयों What is the total profit (in ₹ )
(c) 32 का अनुपात 27: 23 है । यिद लड़कों earned by them together?
(d) 27 और लड़िकयों की सं ा म अंतर A, B और C ने मशः ₹13,750,
200 है , तो लड़कों की सं ा ात ₹16,250 और ₹18,750 िनवेश करके
Q51. A and B enter into a कीिजए। एक वसाय शु िकया। यिद
partnership with capital in the CGL 2019 Tier-II (18-11-2020 ) उनके ारा अिजत लाभ म से B का
िह ा ₹5,200 है , तो उनके ारा

e
ratio 5 : 6. After 4 months, A (a) 1250
withdraws 51 of his capital, while (b) 1200 अिजत कुल लाभ ( म) ात करे |
B increases his capital by 33 31 %. (c) 1350 CPO 23-11-2020 (Morning
(d) 1300 shift)
What is the share (in ₹lakhs ) of

l
(a) 15,600
B in the annual profit of ₹6.3
Q.54 The ratio between the (b) 17,500
lakhs? ac present ages of A and B is 3:5. If (c) 18,200
A और B, 5: 6 के अनुपात म पूँजी के
the ratio of their ages five years (d) 16,600
साथ साझेदारी करते ह। 4 महीने के
hence becomes 13:20, then the
बाद, A अपनी पूँजी म से 51 िह ा
present age of B is : Q57. Seven years ago,the ratio of
िनकाल लेता है , जबिक B अपनी पूँजी
A और B की वतमान आयु के बीच the ages of A and B was 4 : 5.
म 33 31 % की वृ करता है । ₹6.3
का अनुपात 3: 5 है । पाँ च वष बाद Eight years hence, the ratio of the
लाख के वािषक लाभ म B का उनकी आयु का अनुपात 13:20 हो ages of A and B will be 9 :10.
(₹लाख म) िह ा ा है जाता है , तो B की वतमान आयु है What is the sum of their present
nn
CGL 2019 Tier-II (18-11-2020 ) CGL 2019 Tier-II (18-11-2020 ) ages in years?
(a) 2.34 a)32 Years सात साल पहले, A और B की आयु
(b) 2.61 (b)35 Years का अनुपात 4: 5 था। आठ साल बाद
(c) 3.69 (c)30 Years A और B की आयु का अनुपात 9: 10
(d) 3.96 (d)40 Years होगा। वष म उनके वतमान युगों का
योग ा है |
Q55. Ramesh started a business CPO 23-11-2020 (Morning
Q52. The sum of three natural investing a sum of ₹40,000. Six shift)
Pi

numbers is 280. If the ratio months later, Kevin joined by (a) 41


between the first number and investing ₹20,000. If they make a (b) 82
second number is 2 : 3 and the profit of ₹10,000 at the end of (c) 32
ratio between second and third year, how much is the share of (d) 56
number is 4 : 5, then find the kevin?
second number. रमेश ने ₹40,000 की रािश का Q58. A and B started a business
तीन ाकृितक सं ाओं का योग 280 वसाय शु िकया। छह महीने investing amounts of Rs. 92,500
है । यिद पहली सं ा और दू सरी बाद, केिवन ₹20,000 िनवेश करके and Rs. 1,12,500, respectively. If
सं ा के बीच का अनुपात 2: 3 है शािमल हो गया। यिद वे वष के अंत B’s share in the profit earned by
और दू सरी और तीसरी सं ा के म ₹10,000 का लाभ कमाते ह, तो them is Rs. 9,000. What is the
बीच का अनुपात 4: 5 है , तो दू सरी केिवन का िह ा िकतना है total profit (in Rs) earned by them
सं ा ात करे । CGL 2019 Tier-II (18-11-2020 ) together?
CGL 2019 Tier-II (18-11-2020 ) (a) ₹2,000 A और B ने मशः 92,500 पये
(a) 90 (b) ₹4,000 और 1,12,500 पये िनवेश करके

www.ssccglpinnacle.com support@ssccglpinnacle.com Ph. 09729327755, 09817390373 155


Days 17-20 Ratio and Proportion

s
sse
la
_c
ob
एक वसाय शु िकया। यिद more than A . How much was C’s (b) 1,600
bo
ah
उनके ारा अिजत लाभ म B’s की alone share (in Rs)? (c) 1,200
sm
ur
yo

िह ेदारी 9,000 है । उनके ारा 8,200 पये की रािश A, B और C (d) 1,800


e/
t.m

अिजत कुल लाभ ( म) ा है के बीच इस तरह िवभािजत की गई


://
tp
ht

CPO 23-11-2020 (Evening थी | िक A के पास B से 500 अिधक Q64. Seven years ago, the ratio of
shift) है और C के पास A से Rs 300 the ages of A and B was 4 : 5.
(a) 19,000 अिधक है । तो C का शेयर िकतना था Eight years hence, the ratio of the
(b) 16,400 ( म) ages of A and B will be 9 : 10.
(c) 20,000 CPO 24-11-2020 (Morning What is the difference between
(d) 21,240 shift) their present ages in years?
(a) 2,000 सात साल पहले, A और B की आयु
Q59. A sum of Rs.8,200 was (b) 2,300 का अनुपात 4: 5 था। आठ वष बाद,
divided among A, B and C in (c) 3,100 A और B की आयु का अनुपात 9: 10
such a way that A has 500 more (d) 2,800 होगा। वष म उनकी वतमान आयु म
than B and C has Rs.300 more ा अंतर ा है ात करे ?

e
than A. How much was A’s share Q62. Three partners shared the CPO 25-11-2020 (Morning
(in Rs.)? profit in a business in the ratio 8 : shift)
8,200 पये की रािश A, B और C 7 : 5. They invested their capitals (a) 3
के बीच इस तरह िवभािजत की गई for 7 months, 8 months and 14 (b) 6
थी | िक A के पास B से 500 अिधक

l
months respectively. What was (c) 2
है और C के पास A से Rs 300 the ratio of their capitals? (d) 4

( म)

shift)
(a) 2,800
ac
अिधक है ।तो A का शेयर िकतना था

CPO 23-11-2020 (Evening


तीन भागीदारों ने 8: 7: 5. के अनुपात
म एक वसाय म लाभ साझा िकया
और उ ोंने मशः 7 महीने, 8 महीने
और 14 महीने के िलए अपना धन
िनवेश िकया। उनके धन का अनुपात
Q65. A,B,C subscribe a sum of
Rs.75,500 for a business. A
subscribes Rs.3,500 more than B,
and B subscribes Rs. 4,500 more
(b) 2,000 ा था ात करे | than C. Out of a total profit of
(c) 2,300 CPO 24-11-2020 (Evening Rs.45,300 how much (in Rs.)
(d) 3,100 shift) does A receive ?
nn
(a) 49 : 64 : 20 A, B, C एक वसाय के िलए
Q60. A and B started a business (b) 20 : 64 : 49 75,500 पये का िनवेश करता है ।
investing amount of 92,500 and (c) 64 : 49 : 20 A, B से 3,500 अिधक िनवेश
1,12,500 respectively. If B’s share (d) 20 : 49 : 64 करता है , और B C से 4,500
in the profit is 9,000. What is the अिधक िनवेश करता है , कुल लाभ
profit (in Rs.) earned by A? Q63. A, B and C started a म से 45,300 म से A को िकतना
A और B ने मशः 92,500 और business investing amounts of ा होगा |
1,12,500 पये िनवेश करके एक Rs13,750 Rs.16,250 and CPO 25-11-2020 (Evening
Pi

वसाय शु िकया। यिद लाभ म B Rs.18,500 respectively. If B’s shift)


की िह ेदारी 9,000 है । A ारा share in the profit is 5,200. What (a) 12,600
अिजत लाभ ( म) ा है is the difference in the profit (in (b) 15,000
CPO 24-11-2020 (Morning Rs.) earned by A and C? (c) 17,400
shift) A B और C ने मश 13,750 पये (d) 14,700
(a) 10,000 16,250 पये और 18,500 पये
(b) 7,400 िनवेश करके एक वसाय शु Q66. In a school, 5
12 of the
(c) 11,240 िकया। यिद लाभ म B का शेयर number of students are girls and
(d) 9,000 5,200 है । A और C ारा अिजत the rest are boys, 74 of the number
लाभ ( पये म) म ा अंतर है ात of boys are below 14 years of
Q61. A sum of Rs.8200 was करे | age, and 52 of the number of girls
divided among A, B and C in CPO 25-11-2020 (Morning
are 14 years or above 14 years of
such a way that A had Rs. 500 shift)
age. If the number of students
more than B and C had Rs.300 (a) 1,500

www.ssccglpinnacle.com support@ssccglpinnacle.com Ph. 09729327755, 09817390373 156


Days 17-20 Ratio and Proportion

s
sse
la
_c
ob
below 14 years of age is 1120,
bo
ah
then the total number in the
sm
ur
yo

school is:
e/
t.m

एक ू ल म, छा ों की सं ा म से
://
tp
ht

12 लड़िकयाँ ह और बाकी लड़के ह,


5

िजनम से 4
7 लड़कों की सं ा 14
वष से कम है , और 2
5 की सं ाम
लड़िकयाँ 14 वष या 14 वष से
अिधक ह। यिद 14 वष से कम आयु
के छा ों की सं ा 1120 है , तो ू ल
म कुल सं ा है :
CPO 25-11-2020 (Evening
shift)
(a) 1820
(b) 1900

e
(c) 1920
(d) 1290

l
ac
nn
Pi

www.ssccglpinnacle.com support@ssccglpinnacle.com Ph. 09729327755, 09817390373 157


Days 17-20 Ratio and Proportion

s
sse
la
_c
ob
SOLUTION =1:2 C’s share (24 unit) = 24 × 40 =
bo
ah
sm
960
ur
yo

Sol 1. (a) Let the ages of A and B Sol 6. (c) D’s share (15 unit) = 15 × 40 =
e/
t.m
://

4 years ago are 4x and 5x Given, a : b = 3 : 2 600


tp
ht

respectively. Let a = 3k and b = 2k Clearly D get 1744-600 =


According to the question (5a + 2b) : (3a + 4b) ⇒ Rs.1144 less
4x+12 = 11
5x+12 13 {5(3k)+2(2k)}:{3(3k)+4(2k)}
52x + 156 = 55x + 132 ⇒ 19k : 17K Sol 10. (b)
3x = 24 ⇒ 19 : 17 Let the numbers are 3k and 4k
X=8 According to the question
3k+30 9
Sum of their present age = Sol 7. (b) 4k+30 = 10
4x+4+5x+4 = 9 (8)+8 = 80 Given, a : b = 2 : 5 , c : b = 3 : 4 ⇒ 30k+300 = 36k+270
A:B:C ⇒ 6k=30
Sol 2. (c) 2:5 ⇒ k=5
Required proportion: 4:3 So, the numbers are 3 × 5 = 15

e
23−x = 32−x --- (i) Balancing the ratio for B
39−x 56−x and 4 × 5 = 20 ans
By hit and trial, we can find that A:B:C
x=5 satisfies the eq. (i) 2:5 ×4 Sol 11. (c)
Mean proportion of (x+4) and 4 : 3 ×5 Let, the incomes of A and B are

l
(3x+1) i.e. 9 and 16 is ⇒ A:B:C = 8 : 20 : 15 2I and 3I respectively.
√9 × 16 = 12 ac The expenditure of A and B
Sol 8. (d) are 1E and 2E respectively.
Sol 3. (c) Given, (5a - 3b) : (4a - 2b) = 2:3 According to the question
(5a − 3b)
Let the price of the articles are ⇒ (4a − 2b)
= 32 90
2I = 2E × 100
400 and 500. ⇒ 3(5a-3b) = 2(4a-2b) ⇒ I
= 9
E 10
According to the question ⇒ 15a-8a = -4b+9b Saving of A = 2(9)-1(10) = 8 unit
400× 100+x
100
500× 100100
−30 = 10
7
⇒ 7a=5b Saving of B = 3(9)-2(10) = 7 unit
4(100+x) ⇒ a:b = 5:7 Required ratio = 8 : 7 ans
nn
⇒ = 10
5×70 7
⇒ 400+4x = 500
Sol 9. (c) Sol 12. (b)
⇒ x = 25 %
Actual ratio = 3:4:5:8 Let the number of successful
⇒ (3+4+5+8) unit = 4360 candidates and failure candidates
Sol 4. (d) ⇒ 20 unit = 4360 5k and 2k.
Given, A:B = 7:12 and B:C = 8:5 According to the question
⇒ 1 unit = 218
Balancing the ratio for B 5k−14 9
A’s share (3 unit) = 3 × 218 = 654 2k+14 = 5
A:B:C = 14:24:15
B’s share (4 unit) = 4 × 218 = 872 ⇒ 25k - 70 = 18k + 126
Pi

⇒ A’s share = 14 unit


C’s share (5 unit) = 5 × 218 = ⇒ k = 28
B’s share = 24 unit
1090 Total number of students = 5k+2k
C’s share = 15 unit
D’s share (8 unit) = 8 × 218 = = 7k
According to the question
1744 = 7 × 28 = 196
(15-14) = 1 unit = 214
Sum (x) = 14+24+15 = 53 unit Mistaken ratio = 31 : 41 : 51 : 81
=40:30:24:15 Sol 13.(d)
= 53 × 214 = 11342
⇒ (40+30+24+15) unit = 4360 ½ : ⅔ : ¾ ⇒ 6:8:9
⇒ 1 unit = 40 Let the numbers 6k , 8k and 9k
Sol 5. (c)
A’s share (40 unit) = 40 × 40 = According to the question
Mean proportional between 4.8
1600 9k-6k = 27
and 10.8 = √4.8 × 10.8 =7.2
B’s share (30 unit) = 30 × 40 = ⇒k=9
Third proportional to 0.4 and 2.4
1200 Smallest number (6k)= 6 × 9 = 54
= 2.4×2.4
0.4 = 1.44
Required ratio = 7.2 : 14.4

www.ssccglpinnacle.com support@ssccglpinnacle.com Ph. 09729327755, 09817390373 158


Days 17-20 Ratio and Proportion

s
se
s
la
_c
ob
Sol 14. (c) x= 725-640 = 85 {(5 × 3 )-(7 × 2)} unit = (3 ×
bo
ah
½ : ⅔ : ¾ ⇒ 6:8:9
sm
4000)-(2 × 5000 )
ur
yo

Let the numbers 6k , 8k and 9k Sol 20. (d) 1 unit = 2000


e/
t.m
://

According to the question A:B = 2:3 Total income of A and B (5+7 =


tp
ht

9k-6k = 27 B:C = 1:2 12 unit) = 12 × 2000


⇒k=9 Balancing the ratio for B = 24000
Average of three numbers = A:B:C = 2:3:6
6k+8k+9k = 23k Share of C = 406736 × 6 =
3 3 2+3+6 Sol 2. (c)
= 23×9
3 = 69 2,21,856 Let (a+b) = 7 unit
(b+c) = 6 unit
Sol 15. (c) Sol 21. (a) (c+a) = 5 unit
Given, a:b:c=1:3:5 A:B:C = 5: 3 : 6 Adding all three
Let a = k, b=3k and c = 5k If 5----- 192380 (a+b)+(b+c)+(c+a) = (7+6+5)
4a−b+2c ⇒ 4(k)−3k+2(5k) Then, 6 ---- 192380
5 × 6 = 230856 unit
3(a+b+c) 3(k+3k+5k)
11k 11
(a+b+c) = 9 unit
= =

e
27k 27
Sol 22. (c) According to the question
Total profit = 2000 9 unit = 27
Sol 16.(a) 1 unit = 3
Share of B = 600
Ratio of amount received = ⇒ a = (a+b+c)-(b+c)
Share of A = 2000-600 = 1400
15:2:3

l
Ratio of shares of A : B = 7:3 = 9-6
Ratio of note’s worth = 500 : 200: = 3unit = 3(3) =9
ac Amount invested by A = 15000
100 7 = 10500 Similarly
× 7+3
=5:2:1 b = 12
Number of notes = Tnote
otal amount
′s worth And c = 6
15
Sol 23. (c)
Ratio of number of notes = : 22 ⇒ a1 : b1 : 1c = 91 : 12
1 : 1
5 Let total profit = P 6
: 13 = 3:1:3 According to question =4:3:6
96 ×
P × 100 4
3+4+5 = 181400
Sol 17. (d) P = 566875 Sol 3. (c)
nn
fourth proportional of 3, 4, 9 = amount of profit set aside for According to the question
9×4 = 12 (2+x) (30+x)
3 4
emergency fund = 566875 × 100 (3+x) = (35+x)
mean proportional between 2 and = 22675 ⇒ (2+x)(35+x) = (3+x)(30+x)
98 = √2 × 98 =14 ⇒ 70+2x+35x+ x2 =
Required ratio = 12 : 14 Sol 24. (b) 90+3x+30x+ x 2

=6:7 A : B : C ⇒ 4x = 20
55000 : 65000 : 75000 ⇒ x=5
Sol 18. (c) 11 : 13 : 15 mean proportional between (x+7)
Pi

B’s share after charging tax = Let total profit earned be x.


70% of( 14 4 × 110166) = Rs. Therefore,
and (x-2) = √(5 + 7)(5 − 2) = 6
22033.20 15 = 27000
x × 54 × 39
Sol 4.(a) Profit is always
Total profit = Rs. 87750 proportional to Amount invested
Sol 19. (b) × time period
Urban employee = 1200 × 8 =
8+7 SSC CGL TIER II Let amount invested by A = 14
640 unit, amount invested by B = 15
Rural employee = 1200 × 7
8+7 = Sol 1. (b) unit and the time period for which
560 66 32 % = 2
3 B invested the amount = t months
Let number of urban employee A : B ⇒ 14×3 2
15×t = 5
joined = x Income 5 : 7 ⇒t= 14×3×5 = 7 months
15×2
According to question Expenditure 2 : 3
640+x 5
560+20 = 4
Savings 4000 : 5000
Sol 5. (c)
Now,

www.ssccglpinnacle.com support@ssccglpinnacle.com Ph. 09729327755, 09817390373 159


Days 17-20 Ratio and Proportion

s
sse
la
_c
ob
Let the age of A and B one year 2× 4 : 3× 2: 5×3 Let initial capital of B = 3 unit
bo
ah
ago be 4x and 3x. ⇒ Increased capital of B = 4 unit
sm
8 : 6 : 15
ur
yo

According to the question According to the question 50% = 21


e/
t.m
://

4x+1+3 = 6
(15-8) unit = 147000 Let initial capital of C = 5 unit
tp

3x+1+3 5
ht

⇒ 20x+20 = 18x+24 1 unit = 21000 ⇒ Increased capital of C = 2.5


⇒ x=2 B’s share (6) = 6 × 21000 = unit
Required ratio 126000
= 4(2)+1+9 : 3(2)+1+9 Now,
=9:8 Sol 10. (a) A : B : C
Case I. Profit 2(4)+3(8) : 3(6)+4(6) : 5(8)
Sol 6. (c) A:B + 2.5(4)
the third proportional to 0.4 and Last year Income 32 : 42 : 50
0.8 = 0.8×0.8 = 1.6 4:3 16 : 21 : 25
0.4
mean proportional between 13.5 According to the question
Case II. (16+21+25) unit = 86800
and 0.24 = √13.5 × 0.24 = 1.8

e
Required ratio = 1.6 : 1.8 1 unit = 1400
Last Year : Present (25-16) unit = 9 × 1400 = 12600
=8:9
Year
Sol 7. (b)
A 3 : 4 Sol 12.(*) Let the numbers are 3k
Let the age of A and B five years

l
Last Year : Present and 5k
ago be 4x and 5x.
ac Year ⇒ 5k3k−13 = 10
According to the question −13 21
4x+5+5 = 6 B 5 : 6 63k - 273 = 50k - 130
5x+5+5 7
⇒ 28x+70 = 30x+60 k = 11
Balancing the ratio for last year Required ratio = 3(11)+5 :
⇒ x=5 income 5(11)+5
Present age of A = 4(5)+5 = 25 For A 3 unit (of case II) = 4 (of = 19 : 30
Present age of C = 5(5)+5-10 = Case I) Note : Given options are wrong
20 1 unit = 34
Required ratio = 25:20
nn
4 unit = 16
3 Sol 13.(b)
=5:4
For B 5 unit (of case II) = 3 (of Let the savings of A, B and C are
Case I) 8k, 9k and 20k respectively.
Sol 8. (c)
1 unit = 53 For A
A’s share : B’s share
6 unit = 18 80% = 54
6 : 7 5

B’s share : C’s share A : B Here 5 unit is income of A and 4


3 : 2 Present Income 16 : 18
3 5
unit is A’s expenditure
Balancing the ratio for B’s share 40 : (5-4) unit = 8k
Pi

A’s share : B’s share : C’s share 27 1 unit = 8k


18 : 21 : 14 According to the question Income of A(5 unit) = 5 × 8k =
According to the question (40+27) unit = 8.04 lakh 40k
(18-14) unit = 540 1 unit = 0.12 lakh For C
1 unit = 135 27 unit = 27 × 0.12 lakh 75% = 43
Total amount (x) = (18+21+14) Last year income of B = 27 × 0.12 Here 4 unit is income of C and 3
= 53 unit × 65 = 2.7 lakh unit is C’s expenditure
= 53 × 135 = (4-3) unit = 20k
7155 Sol 11. (a) 1 unit = 20k
50% = 21 Income of A(4 unit) = 4 × 20k =
Sol 9. (a) 80k
Let initial capital of A = 2 unit
Profit is always proportional to ⇒ Increased capital of A = 3 unit According to the question
Amount invested × time period 80k-40k = 18000
33 31 % = 31
A : B : C k = 450

www.ssccglpinnacle.com support@ssccglpinnacle.com Ph. 09729327755, 09817390373 160


Days 17-20 Ratio and Proportion

s
sse
la
_c
ob
For B 1 unit = 108 ⇒ 3x=18
bo
ah
85% = 17 (2+3+6+8) unit = 19 × 108 = 2052
sm
20 ⇒ x=6
ur
yo

Here 20 unit is income of B and So, Current age of C = 9(6)-3 =


e/
t.m
://

17 unit is B’s expenditure Sol 17.(c) 51


tp
ht

(20-17) unit = 9k Let the amount invested by B and


1 unit = 3k C be b and c respectively. Sol 4. (c)
Income of B(20 unit) = 20 × 3k = According to the question Let the current age of A and B be
54000 × 12 = 1
60k b×8 4 8x and 15x.
⇒ Income of B (60k)= 60 × 450 = ⇒ b = 54000 × 6 According to the question
8x−8 6
27000 Also, 15x−8 = 13
54000 × 12 = 1 ⇒ 104x-104 = 90x-48
c×6 5
Sol 14. (b) ⇒ c = 54000 × 10 ⇒ 14x=56
Let a,b and c be the invested Required difference = ⇒ x=4
amount of A, B and C. 54000 × 10 - 54000 × 6 = 216000 So, current age of A and B is 32
According to the question and 60 respectively.

e
3a=2b=4c Practice Sol Required ratio = 32+8 : 60+8
⇒ a:b:c = 4:6:3 = 10 : 17
This will also be the ratio of their Sol 1. (a)
profit shares. Let the mean proportional of Sol 5. (b) Let a = 4k and b = 5k

l
⇒ 3 unit = 4863 (x+3) and (4x+1) is b. (2a + 3b) : (3a + 2b) ⇒
1 unit = 1621
ac b2 = (x+3)(4x+1) {2(4k)+3(5k)}:{3(4k)+2(5k)}
A’s share (4 unit) = 4 × 1621 = ⇒ 23k : 22k
6484
b= √(x + 3)(4x + 1)
According to the question ⇒ 23:22
12+x = 21+x
28+x 45+x
Sol 15. (a) Sol 6. (d) Let a = 5k and b = 3k
(12+x)(45+x) = (21+x)(28+x)
Given, (5x+2y) : (10x+3y) = 5:9 (8a - 5b) : (8a + 5b) ⇒
(5x+2y) 540+57x+ x2 = 588+49x+ x2
⇒ (10x+3y) = 95 {8(5k)-5(3k)}:{8(5k)+5(3k)}
x=6
⇒ 45x+18y = 50x+15y ⇒ 25k : 55k
b= √(6 + 3)(24 + 1) = 15
nn
⇒ x:y = 3:5 ⇒ 5:11
Let x = 3 unit and y = 5 unit Sol 2. (c)
(2 x2 + 3y 2 ):(4 x2 + 9y 2 ) 21−x = 60−x Sol 7. (a)
22−x 64−x Given, a : b = 2 : 3 , c : b = 5 : 6
⇒ {2 (3)2 + 3(5)2 }:{4
(21-x)(64-x) = (22-x)(60-x) A:B:C
(3)2 + 9(5)2 } 1344 -21x-64x + x2 = 2:3
= 31 : 87 1320-22x-60x + x 2
6:5
24 = 3x Balancing the ratio for B
Sol 16. (a)
Pi

x=8 A:B:C
A:B:C:D x+1 = 9 2:3 ×2
2 :3 7x+8 = 64
1:2 6 : 5 ×1
Let the mean proportional a ⇒ A:B:C = 4 : 6 : 5
3:4 a2 = 64 × 9
Balancing the ratio for B and C
a=24 Sol 8. (c)
A:B:C:D
2 :3:3:3 Given, a : b = 5 : 8 , c : b = 4 : 3
Sol 3. (c) A:B:C
1 : 1:2:2
Let the present age of A and B be 5:8
3 : 3:3:4
8x and 9x respectively. 3:4
________________
Current age of C = 9x-3 Balancing the ratio for B
6 : 9 : 18 : 24
According to the question A:B:C
⇒ A:B:C:D = 2 : 3 : 6 : 8 8x+9 = 19
According to the question
9x+9 21 5:8 ×3
⇒ 168x+189 = 171x + 171 3:4 ×8
(8-2) unit = 648

www.ssccglpinnacle.com support@ssccglpinnacle.com Ph. 09729327755, 09817390373 161


Days 17-20 Ratio and Proportion

s
sse
la
_c
ob
⇒ A:B:C = 15 : 24 : 32 5k+4 = 49 third proportional of 5, 4 = 4×4 =
bo
2k+4 5
ah
sm
⇒ 20k+16 = 18k+36 3.2
ur
yo

Sol 9. (d) mean proportional between 14.4


e/

⇒ k=10
t.m
://

Let a = 2k and b = 3k Required ratio = 5(10)-5 : 2(10)-5 and 3.6 = √14.4 × 3.6 =7.2
tp
ht

(5a - 2b) : (5a + 2b) ⇒ = 3:1 Required ratio = 7.2 : 3.2


{5(2k)-2(3k)}:{5(2k)+2(3k)} =9:4
⇒ 4k : 16k Sol 15. (b)
⇒ 1:4 Let the income of A and B be 5I Sol 20. (d)
and 3I respectively. Mean Proportion =
Sol 10. (a) Let the expenditure of A and B be
√8.1 × 3.6 = 5.4
Let a = 2k and b = 3k 9E and 5E. Third Proportion = 3×3 = 9 = 4.5
2 2
(5a + 3b) : (6a - 2b) ⇒ According to the question
Therefore, Required Ratio =
{5(2k)+3(3k)}:{6(2k)-2(3k)} 5I = 2(5E) 5.4 = 6 : 5
⇒ 19k : 6k I 2 4.5
E = 1
⇒ 19:6 Savings of A = 5(2)-9(1) = 1 unit
Sol 21. (d) Mean proportion =

e
Savings of B = 3(2)-5(1) = 1unit
Sol 11. (a) Required ratio = 1:1 √3.6 × 12.1 = 6.6
Third Proportion = 11×11
Let a = 5k and b = 7k 2

(5a - 3b) : (4a - 2b) ⇒ Sol 16. (d) Therefore, Required Ratio =

l
6.6 = 6 : 55
{5(5k)-3(7k)} : {4(5k)-2(7k)} Let the numbers are 7k and 5k. 121
2
⇒ 4k : 6k ac Difference of the numbers =
⇒ 2:3 7k-5k = 2k Sol 22. (a) Mean proportion =
According to the question
7k−40 27
√10.8 × 4.8 = 7.2
Sol 12. (c) 5k−40 = 17 Third proportion = 8
Let the age of A and B be 6k and ⇒ 119k-680 = 135k - 1080 Therefore, Sum = 7.2+8 = 15.2
5k respectively. ⇒ k=25
According to the question Required difference = 2(25) = 50 Sol 23. (b) Mean proportion =
6k−4 = 5
5k−4 4
√1.8 × 3.2 = 2.4
nn
⇒ 24k-16 = 25k-20 Sol 17. (a) Third Proportion = 9
5
⇒ k=4 Let the numbers are 7k and 5k.
Therefore, Required Ratio = 2.4:
Required ratio = 6(4)+12 : Sum of the numbers = 7k+5k = 9
5 = 4:3
5(4)+12 12k
=9:8 According to the question 5x−5 5
7k−40 27 Sol 24. (d) ATQ, 7x−5 = 8
5k−40 = 17
⇒ 5x = 15
Sol 13. (c) ⇒ 119k-680 = 135k - 1080
Let the ages of A and B 8 years ⇒x=3
⇒ k=25
Pi

ago were 9k and 10k respectively. Therefore, Their ages are 15 and
Required sum = 12(25) = 300
According to the question 21.
9k+12 12
10k+12 = 13 Sol 18. (a)
⇒ 117k+156 = 120k+144 Sol 25. (b)
Let the numbers are 3k and 4k. 9
C’s share = 8288 × 5+7+9 = 3552
⇒ k=4 Sum of the numbers = 3k+4k =
Present age of A = 9(4)+8 = 44 7k
years According to the question Sol 26. (d)
3k+30 9 Value- 0.5 : 0.25 : 1
Present age of C = 44+6 = 50 4k+30 = 10
years Number- 5 : 8 :1
⇒ 30k+300 = 36k + 270
__________
⇒ k=5
Sol 14. (a) 2.5 : 2.00 : 1
Required sum = 7(5) = 35
Let the value of A and B be 5k
and 2k respectively. = 5.5 unit = 55
Sol 19. (c)
According to the question 1 unit = 10

www.ssccglpinnacle.com support@ssccglpinnacle.com Ph. 09729327755, 09817390373 162


Days 17-20 Ratio and Proportion

s
sse
la
_c
ob
Therefore, No. of 25 paise coins Sol 34.(b) Balancing the ratio for second
bo
ah
(8 unit)= 8x10 = 80 Pawan : Sunil number
sm
ur
yo

Income 4 : 3 First : Second : Third


e/
t.m
://

Sol 27. (b) C’s share = Expenditure 3 : 2 8 : 12 : 15


tp
ht

4
12 × 780516 = Rs. 260172 Savings 4000 : 6000 According to the question
Now, (8+12+15) unit = 140
Sol 28. (d) ATQ, 4x+12 = 11 {( 3 × 3 ) - ( 4 × 2)} unit = (3 × 1 unit = 4
9x+12 21
⇒x=8 6000)-(2 × 4000 ) Second number (12 unit) = 12 x 4
Therefore, Sum of numbers = 1 unit = 10000 = 48
(4+9)x8 = 104 Total income of Pawan and Sunil
(4+3 = 7 unit) = 7 × 10000 Sol 39. (d)
Sol 29. (d) Let the three numbers = 70000 Let the number of 2 rupee coins =
be a, b & c. 3 unit, the number of 1 rupee
a:b:c = 7:9:21 Sol 35. (a) coins = 4 unit and the number of
9 × 777 = 189
Therefore, b = 37 Let U = 5 unit and V = 3 unit 50 paise coins = 5 unit

e
According to the question According to the question
(5+3) unit = 80 [2(3) + 1(4) + 0.5(5)] units = 250
Sol 30. (c)
1 unit = 10 1 unit = 20
A : B : C
(5-3) unit = 10 x 2 = 20 unit 1 rupee coins = 4 unit = 4 x 20 =
137500 : 162500 : 187500

l
80
11 : 13 : 15
Therefore, C’s
ac share = Sol 36. (a)
Let the salary of Mahesh = 7 unit Sol 40. (d)
15 = Rs. 67500
219375 × 54 × 39
And Salary of Sumit = 4 unit Let A = 2 unit and B = 3 unit
According to the question So, B-A = (3-2) unit = 28
Sol 31. (d) ⇒ A+B = (3+2) unit = 5 x 28 =
(7-4) unit = 12000
A : B : C
1 unit = 4000 140
55000 : 65000 : 75000
Salary of sumit = 4 unit = 4 x
11 : 13 : 15
4000 = 16000 Sol 41. (d)
=39
Let x = 3 unit, y = 4 unit and z =
nn
Therefore, A’s share =
1 4 11 Sol 37.(d) 5 unit
87750 × 5 + 87750 × 5 × 39
Ram : According to the question
= 19800 + 17550 Rahim (3+4+5) unit = 96
= Rs. 37,350 Income 4 : 3 1 unit = 8
Expenditure 3 : 2 z = 5 unit = 5x8 = 40
Sol 32. (d) Savings 5000 :
A : B : C 5000 Sol 42. (d)
27500 : 32500 : Now, Let the total number of boys = 4
Pi

37500 {( 3 × 3 )-( 4 × 2)} unit = (3 × unit and the number of girls = 1


11 : 13 : 15 5000)-(2 × 5000 ) unit
C’s share = 15 ---- 13500 According to the question
1 unit = 5000
39---- 35100 (4+1) unit = 150
Income of Ram = 4 unit = 4 x
Therefore, Total Profit = 1 unit = 30
5000 = 20000
35100 × 80 = Rs. 43, 875/ −
100
Number of boys = 4 unit = 4x30
Income of Rahim = 3 unit = 3 x
5000 = 15000 = 120
Sol 33. (c) Number of girls = 1 unit = 30
Let P = 5 unit and Q = 2 unit Sol 38. (b) Mean proportion of number of
So, First : Second boys and number of girls =
(2P - 3Q) : (3P - 5Q) ⇒ {2(5) - 2 : 3 √120 × 30 = 60
3(2)} : {3(5) - 5(2)} Second : Third
⇒ 4:5 4 : 5 Sol 43.(c)

www.ssccglpinnacle.com support@ssccglpinnacle.com Ph. 09729327755, 09817390373 163


Days 17-20 Ratio and Proportion

s
sse
la
_c
ob
A : B k 2 = 16 Sol 53.(b)
bo
ah
Income 3 : 4 Mohit :
sm
⇒k=4
ur
yo

Expenditure 2 : 3 ⇒ (X+Y+Z) = (k+2k+3k) = 6 x Prakash


e/
t.m
://

Savings 4000 : 4000 4= 24 Income 2 : 3


tp
ht

Now, Expenditure 3 : 5
{( 3 × 3 )-( 4 × 2)} unit = (3 × Sol 48.(b) Savings 5000 : 5000
4000)-(2 × 4000 ) Let the number of students in the Now,
1 unit = 4000 classes are 3k, 4k and 5k {( 5 × 2 )-( 3 × 3)} unit = (5 ×
Income of B = 4 unit = 4 x 4000 respectively. 5000)-(3 × 5000 )
= 16000 According to the question 1 unit = 10000
3k+20 4
4k+20 = 5
Sum of the income of Mohit and
Sol 44. (c) ⇒ 15k + 100 = 16k + 80 Prakash = (2+3) x 10000 = 50000
Given, ⇒ k=20
x + y = 52 …….(1) Total number of students initially Sol 54. (b)
And = 3k+4k+5k = 12 x 20 = 240 Let the two numbers are 5 unit

e
x - y = 20 ………(2) and 7 unit.
Add (1) and (2) Sol 49. (c) Sum of the two numbers = 5+7 =
2x = 72 Let the numbers are 6 unit and 5 12 unit. Clearly there actual sum
⇒ x = 36 will be the multiple of 12 and
unit.

l
⇒ y = 52 − 36 = 36 − 20 = 16 According to the question going through options only option
Required ratio = 36 : 16
ac (6+5) unit = 77 B satisfies the condition.
=9:4 ⇒ (6-5) unit = 7
Sol 55. (b)
Sol 45. (a) Given,
Sol 50. (c)
Let the expenditure of A = 5 unit Let X = 13 unit and Y = 12 unit X 2 + Y 2 = 100 and X : Y = 4 : 3
and expenditure of B = 3 unit According to the question Let X = 4k and Y = 3k
According to the question (13-12) unit =1 unit = 2 X 2 + Y 2 = (4k)2 + (3k)2 = 25k 2
12000 - 5unit = 8000 - 3 unit ⇒ X = 13x2 = 26 ⇒ 25k 2 =100
nn
2 unit = 4000 And Y = 12x2 = 24 ⇒k=2
1 unit = 2000 2X + 3Y ⇒ 2(26)+3(24) = 124 So, X = 4x2 = 8 and Y = 3x2 = 6
Required savings = 12000 - X 2 − Y 2 ⇒ 82 − 62 = 28
5(2000) = 8000 - 3(2000) = 2000
Sol 51. (a)
Let expenditure of A = 2 unit and Sol 56.(d) Let V1 = 1 unit and V2
Sol 46. (d) = 2 unit
expenditure of B = 3 unit
Given, According to the question
According to the question
x+y+z = 360 and x:y:z = 4:3:2 V1+V2 = (1+2) unit = 147
2 unit = 8000
Let x = 4 unit, y = 3 unit and z =
Pi

1 unit = 4000 1 unit = 49


2 unit V1-V2 = (2-1) unit = 49
3 unit = 3x4000 = 12000
According to the question
sum of their monthly savings =
(4+3+2) unit = 360 Sol 57.(b)
(12000-8000) + (15000-12000) =
1 unit = 40 Let the shares of Anuj, Bharat
7000
(y+z-x) = (3+2-4) unit = 40 and Shekhar are 3 unit, 4 unit and
Sol 52. (d) 8 unit.
Sol 47. (d) According to the question
Let the ages of two persons are 3
Given, (3+4+8) unit = 975
unit and 4 unit.
X : Y : Z = 1 : 2 : 3 and 1 unit = 65
According to the question
X 2 + Y 2 + Z 2 = 224 (4-3) unit = 8 years Bharat’s share = 4 unit = 4x65 =
Let X = k, Y = 2k and Z = 3k Sum of their ages = (4+3) unit = 260
According to the question 7x8 = 56 years
( k 2 + (2k)2 + (3k)2 ) unit = 224 Sol 58.(d)

www.ssccglpinnacle.com support@ssccglpinnacle.com Ph. 09729327755, 09817390373 164


Days 17-20 Ratio and Proportion

s
sse
la
_c
ob
Let the number of 5 rupee, 2 Present age of father = 7x-5 = (A+B) + (A-B) = 11+1 unit
bo
ah
rupee, 1 rupee and 50 paise coins 7(8)-5 = 51 ⇒ A = 6unit and B = 5unit
sm
ur
yo

are 1unit, 2 unit, 3 unit and 4 unit. Similarly for B+C : B-C = 11:1
e/
t.m
://

According to the question Sol 63.(c) ⇒ B = 6unit and C = 5unit


tp
ht

[5(1) + 2(2) + 1(3) + 0.5(4)] unit Let the numbers are 3k, 4k and 6k But value of B must be same so
= 168 According to the question balancing the values for B
1 unit = 12 (3k)2 + (4k)2 + (6k)2 = 244 A = 36 unit, B = 30 unit and C =
the number of 50 paise coins = (61k)2 =244 25 unit
4x12 = 48 ⇒k=2 According to the question
The largest number = 6k = 6x2 = (36+30+25) unit = 18200
Sol 59.(d) 12 1 unit = 200
the number of students in three B’s salary = 30 unit = 30x200 =
classes is k, 2k and 3k. Sol 64. (b) 6000
According to the question Let the number of 1 rupee, 50
k+20 3
2k+20 = 5 paise and 25 paise coins are 5 Sol 68. (d)

e
5k+100=6k+60 unit, 7 unit and 9 unit. Let the selected candidates = 14
⇒ k = 40 [5(1) + 0.50(7) + 0.25(9)] unit = unit and the unselected candidates
Total number of students in three 430 = 25 unit
classes = k+2k+3k = 6x40 = 240 1 unit = 40 According to the question

l
(25+14) unit −35 5+3
Number of 25 paisa coins = 9 x 14 unit − 10 = 3
Sol 60.(c) ac 40 = 360 117 unit - 105 = 112 unit - 80
Let the age of shyam = 5 unit and 1 unit = 5
age of Ravi = 3 unit Sol 65. (a) Total number of candidates = 39
According to the question Let the age of Manoj = M and unit = 39x5 = 195
(5+3) unit = 32 sum of the ages of his son = S
1 unit = 4 According to the question Sol 69. (a)
Age of Shyam = 5x4 = 20 years M = 2S ………..(1) Let the salary of A = 1 unit, B = 2
and Age of Ravi = 3x4 = 12 And unit and C = 3 unit
nn
Now, M+20 = S+20+20 105 unit
New salary of A = 1 x 100
20+x = 7
Put the value of M from equation New salary of B = 2 x 110
100 unit
12+x 5
100+5x = 84+7x (1) 115
New salary of C = 3 x 100 unit
⇒ x=8 2S+20 = S+40 105
Required ratio = 1 x 100 : 2 x
⇒ S= 20
110 :3x 115
Sol 61.(d) So, age of Manoj = 2S = 2x20 = 100 100

Mean proportion of 24 and 150 = 40 = 21 : 44 : 69

√24 × 150 = 60 Sol 70.(d)


Pi

the third proportional between 12 Sol 66. (b)


Let the age of father = 3k and son Let the number of 1 rupee, 50
and 6 5 = √ √ = 15
6 5×6 5
√ 12 =k paise and 10 paise coins are 3
Required ratio = 60 : 15 According to the question unit, 2 unit and 3 unit.
= 4:1 3k x k = 432 [1(3) + 0.50(2) + 0.10(3)] unit =
3k 2 =432 25.8
Sol 62.(b) ⇒ k = 12 1 unit = 6
Let the age of father after 5 years Number of 50 paisa coins = 2 x 6
Required sum = 3k+k = 4x12 =
= 7x and age of the son after 5 = 12
48
years = 4x.
According to the question Sol 71. (b)
Sol 67. (c)
(7x-5)+(4x-5) = 78 Given,
Let A+B = 11 unit …..(1)
11x-10 = 78 4A = 6B = 5C
And A-B = 1 unit ……(2)
⇒x=8 LCM of 4,6 and 5 = 60
Add (1) and (2)
⇒ A:B:C = 15:10:12

www.ssccglpinnacle.com support@ssccglpinnacle.com Ph. 09729327755, 09817390373 165


Days 17-20 Ratio and Proportion

s
sse
la
_c
ob
(2+3+5) unit = 200 16:24:30:35
bo
2
ah
Sol 72. (b) ⇒ A + BD + C = 16 + 24 + 30. =2
sm
1 unit = 10 35
ur
yo

A:B Largest number = 5 unit = 5x10 =


e/
t.m
://

3: 4 50 Sol 80. (d)


tp
ht

B:C Required square of the number = Let the initial number of boys = 6
2:3 502 =2500 unit and initial number of girls =
Balancing the ratio for B 5 unit
A:B:C Sol 77.(b) According to the question
3:4:6 a:b:c:d (6+5) unit = 550
⇒ A + B : B + C : C + A = 3+4 : 2:5 1 unit = 50
4+6 : 6+3 4:7 Number of boys = 6x50 = 300
= 7 : 10 : 9 9 : 14 Number of girls = 5x50 = 250
Balancing the ratio for B and C Let the required number of girls =
Sol 73.(a) a:b:c:d k
Let the two parts are a and b. 2:5:5:5 300 = 5
⇒ 250+k 6

e
According to the question 4:4:7:7 ⇒ 360 = 250+k
4a 14
5b = 15 9 : 9 : 9 : 14 ⇒ k = 110
⇒a:b=7:6 ____________
According to the question 72:180:315:490 Sol 81.(b)

l
(7+6) unit = 78 Let the age of father = f and age
1 unit = 6 ac Sol 78.(b) of kartik = k
The first part = 7unit = 7x6 = 42 The number of students studying According to the question
in school A = 12 unit f = 4k ………(1)
Sol 74.(d) The number of students studying And
Let the number of 10 rupee, 5 in school B = 15 unit (f-3) = 7(k-3)
rupee and 2 rupee paise coins are The number of students studying Put the value of f from equation
10 unit, 5 unit and 2 unit. in school C = 16 unit (1)
According to the question Increased number of students (4k-3) = 7k-21
nn
5(5) unit - 2(2) unit = 84 150 =
studying in school A = 12 × 100 ⇒k=6
1 unit = Rs. 4 18 unit
Total number of 10 rupee notes = Increased number of students Sol 82. (b)
10 unit = 10x4 = 40 120 =
studying in school B = 15 × 100 Let the initial number of boys = 4
Total value of 10 rupee notes = 18 unit unit and initial number of girls =
40x10 = 400 Increased number of students 5 unit
Sol 75. (d) 150 =
studying in school C = 16 × 100 Increased number of boys = 4
a : b = 2 : 3 and b : c = 2 : 3 150 = 6 unit
× 100
24 unit
Pi

Balancing the ratio for b Increased number of girls = 5


Required ratio = 18:18:24 = 3:3:4
a:b:c = 4:6:9 160 = 8 unit
× 100
Let a = 4k, b= 6k and c = 9k
Sol 79.(d) Required ratio = 6:8
( 3a2 + b2 − c2 ) : (a2 + 2b2 − c2 ) =
a:b:c:d = 3:4
{ 3(4k)2 + (6k)2 − (9k)2 } : {(4k)2 2:3
+ 2(6k)2 - (9k)2 } 4:5 Sol 83. (d)
⇒ 6:7 Let X = 4 unit, Y = 7 unit and Z =
48k 2 + 36k 2 − 81k 2 : 16k 2 + 72k 2 − 81k 2 Balancing the ratio for B and C 9 unit
⇒ 3:7 a:b:c:d ⇒ xy : yz : xz = 74 : 97 : 94 = 36:49:28
2:3:3:3
Sol 76.(a) 4:4:5:5 Sol 84. (b)
Let the numbers are 2 unit, 3 unit 6:6:6:7 Let the numbers are 4k and 3k
and 5 unit ____________ According to the question
According to the question 48:72:90:105 (4k)3 − (3k)3 = 999

www.ssccglpinnacle.com support@ssccglpinnacle.com Ph. 09729327755, 09817390373 166


Days 17-20 Ratio and Proportion

s
sse
la
_c
ob
⇒ 37k 3 = 999 x+y=48 …….(1) 1:2
bo
ah and Balancing the ratio for B and C
sm
⇒k=3
ur
yo

⇒ Smaller number = 3k = 3x3 = x-y = 6 ……(2) A:B:C:D


e/
t.m
://

9 Add eq(1) and eq(2) 2:3:3 :3


tp
ht

Required square of the number = (x+y)+(x-y) = 48+6 4:4:5 :5


92 = 81 ⇒ x = 27 1:1:1 :2
⇒ y = 48 − 27 = 21 _______________
Sol 85. (d) Required ratio = 27:21 8 : 12 : 15 : 30
Given, = 9:7 According to the question
A : B : C : D = 4 : 8 : 11 : 15 (8+12+15+30) unit = 5200
Let A = 4 unit, B=8 unit, C=11 Sol 90.(a) 1 unit = 80
unit and D = 15 unit Let the sum got by first person = ⇒ D-B = (30-12) unit = 18x80 =
According to the question 3 unit and the sum got by 2nd 1440
(15-4) unit = 22000 person = 2 unit
1 unit = 2000 According to the question Sol 95.(b)

e
Required sum = (11+8) unit = 19 (3-2) unit = 12 Given,
x 2000 = 38000 Required sum = (3+2) unit = b:a:d:e
5x12 = 60 3:2
Sol 86. (a) 3:4

l
According to the question Sol 91.(a) 5:3
0.8 = 5 Let the two numbers are 2k and k. Balancing the ratio for a and d
X 8
⇒ X=1.28

Sol 87. (c)


ac
Let the three parts are a,b and c
According to the question
According to the question
2k+5 = 3
k+5 2
⇒ 4k + 10 = 3k + 15
⇒k=5
Required sum = 2k+k = 3x5 = 15
b:a:d:e
3:2:2:2
3:3:4:4
5:5:5:3
___________
45:30:40:24
3a=6b=8c
Sol 92.(a) Required ratio = 24 : 45
⇒a:b:c=8:4:3
= 8 : 15
nn
Let a = 8 unit, b = 4 unit and c = Let the Ram’s share = 3 unit and
3 unit Rahim’s share = 2 unit
According to the question Sol 96. (a)
According to the question
3 unit = 36000 Let the present age of A = 5k and
8 unit = 1600
1 unit = 12000 present age of B = 3k
1 unit = 200
Total sum = (3+2) unit = 5x12000 According to the question
c = 3 unit = 3x200 = 600 5k−9 = 23
= 60000 3k−9 12
⇒ 60k − 108 = 69k − 207
Sol 88. (d)
Pi

Let the numbers are a and b. Sol 93. (c) ⇒ k = 11


According to the question Let the two numbers are 9k and Present age of A =5k= 5x11 = 55
a+b=27 …….(1) 7k. Present age of B =3k= 3x11 = 33
and According to the question Required ratio = 55+15 : 33+15
9k+6 = 21 = 35 : 24
a-b = 3 ……(2) 7k+6 17
Add eq(1) and eq(2) ⇒ 153k + 102 = 147k + 126
(a+b)+(a-b) = 27+3 ⇒k=4 Sol 97. (b)
⇒ a = 15 Required difference = 9k-7k = Since x is the mean proportion.
⇒ b = 27 − a = 27 − 15 = 12 2x4 = 8 x = √25.6 × 32.4 = 28.8
Required ratio = 15:12 y is the third proportion of 32 and
= 5:4 Sol 94.(a) 48
⇒ 4832 = 48
A:B:C:D y
Sol 89. (c) 2:3 ⇒ y=72
According to the question 4:5 Required ratio = 3 x 28.8 : 2 x 72

www.ssccglpinnacle.com support@ssccglpinnacle.com Ph. 09729327755, 09817390373 167


Days 17-20 Ratio and Proportion

s
sse
la
_c
ob
=3:5 x = 8 ⇒ x = 30
bo
8 27
ah
sm
⇒x= 64 And
ur

27
yo

Sol 98. (a) 162 = y


e/
t.m

y 128
://

Let the salary of Abhinav = 6s ⇒ y 2 = 162 × 128


tp

Sol 102. (d)


ht

and salary of Rekha = 5s Given, a : b = 2 : 3 ⇒ y = 144


Let the expenditure of Abhinav = Let a=2 unit and b = 3 unit Required ratio = 8(30) : 144
9e and expenditure of Rekha = 8e According to the question =5:3
According to the question 3a+4b ⇒ 3(2)+4(3) 18
4a+5b 4(2)+5(3) = 23
6s = 2 × 8e
Sol 106.(a)
⇒ s:e = 8:3
Sol 103. (a) Given,
Let s = 8 unit and e = 3 unit A:B:C:D
Divisor : Dividend
Salary of Abhinav = 6x8 = 48 4:3
7 : 12
unit 5:4
Divisor : Remainder
Salary of Rekha = 5x8 = 40 unit 6:5
3 : 2
Expenditure of Abhinav = 9x3 = Balancing the ratio for B and C
Balancing the ratio for Divisor

e
27 unit A:B:C:D
Divisor : Dividend : Remainder
Expenditure of Rekha = 8x3 = 24 4:3: 3: 3
21 : 36 : 14
unit 5:5:4 : 4
According to the question
Required ratio = 48-27 : 40-24 6:6:6 :5
14 unit = 14

l
= 21 : 16 __________
1 unit = 1
ac 21 unit = 21 120 : 90 : 72 : 60
Sol 99. (a) 20 : 15 : 12 : 10
Required remainder = 21 9 =3
Case 1: According to the question
Son 1 : Son 2 : Son 3 : Total Sum (20+15+12+10) unit = 6859
2 : 3 : 8 : 13 Sol 104. (b)
57 unit = 6859
Case 2: A:B
Share of B = 15 unit =
Son 1 : Son 2 : Son 3 : Total Sum Total students 3:4 6859
Girls in class B is 50% of boys. 57 × 15 = 1805
1
2 : 31 : 81
Let total students in B = 4 unit
nn
12 : 8 : 3 : 23 Sol 107.(b)
50% = 21
Balancing the ratio for total sum Let the initial number of males =
So number of boys in B = 4 × 2
Son1 : Son2 : Son3 : Total 2+1
6k and initial number of females
= 8 unit
Sum 3 = 7k
Case 1 46 : 69 : 184 : 13 x Total students in A : Boys in B According to the question
23 3 : 8
6k 12
7k−15 = 11
3
Case 2 156: 104 : 39 : 23 x 9 : 8 66k = 84k-180
13 A:B ⇒ k = 10
Pi

According to the question Boys 2:3 Initial number of males = 6k =


(156-46) unit = 2200 Balancing the ratio for number of 6x10 = 60
1 unit = 20 boys in B Initial number of females = 7k =
Total sum = 23 x 13 x 20 = 5980 ⇒ Boys in B = 24 unit
7x10 = 70
Total students in A = 27 unit Required Ratio = 60+6 : 70-15
Sol 100.(a) Boys in A = 16 unit =6:5
Let the required number = x Number of girls in A = 27-16 =
According to the question 11 unit
4−x = 1 Sol 108. (d)
9−x 6 Girls in class B = 24
2 = 12 unit Let the income of A = 3x and B =
24-6x=9-x Required ratio = 11 : 12 8x
⇒x=3
Let the savings of A = 9y and B =
Sol 105.(a) 25y
Sol 101. (d) According to the question According to the question
According to the question 5 x
15 = 90 3x = 8x-25y

www.ssccglpinnacle.com support@ssccglpinnacle.com Ph. 09729327755, 09817390373 168


Days 17-20 Ratio and Proportion

s
se
s
la
_c
ob
⇒ x = 5 Share of C = 6 unit = 6x200 = (3+2) unit = 360
bo
y 1
ah 1200 1 unit = 72
sm
Let x = 5unit and y = 1 unit
ur
yo

Share of D = 2 unit = 2x200 = Number of girls = 2 unit = 72 x 2


e/

Expenditure of A = 3(5)-9(1) = 6
t.m
://

unit 400 = 144


tp
ht

Expenditure of B = 8(5)-25(1) = Only A and D got less amount


15 unit than the previous one. Sol 117.(b)
Required ratio = 6 : 15
−600 = 6 1 %
%age for A = 640640 4 Let l = k , m = 2k and n = 4k
=2:5 %age for B = 480−400
480 = 16 32 % ⇒ √5l2 + m2 + n2 =
Sol 109.(c)
Clearly option D is the desired
answer. √5(k) + (2k) + (4k) = 5k = 5l
2 2 2

According to the question


8+x 20+x Sol 118. (d) Let the numbers are x
14+x = 30+x Sol 112. (d)
and y.
⇒ (8 + x)(30 + x) = (20 + x)(14 + x) the mean proportional between
According to the question
⇒ 240 + 38x + x2 = 280 + 34x + x2 4.5 and 0.5 = √4.5 × 0.5 = 1.5
x+y = 27 …….(1)
⇒ x = 10 third proportional to 4.5 and 9.0 =
And x-y = 3 …….(2)

e
9.0× 9.0 = 18
x-2 = 10-2 = 8 and 7x+2 = 4.5 Adding equation (1) and (2)
7(10)+2 = 72 Required ratio = 1.5 : 18 2x=30
8 = x
x 72 = 1 : 12 ⇒ x = 15
x2 = 576

l
So, y = 27-x or x-3 = 12
⇒ x = 24 Sol 113. (c) Required ratio = 15:12

Sol 110. (d)

12

⇒x=
5
16 = x
ac
According to the question

20
3
Let total profit = P
According to question
96 ×
P × 100
P = 566875
4
3+4+5 = 181400

amount of profit set aside for


4
Sol 119.(a)
Given,
u+v = 84
= 5:4

………(1)
Also emergency fund = 566875 × 100 and u-v = 4 ……….(2)
20 = 15 = 22675 Adding equation (1) and (2)
y 21
2u = 84+4
nn
⇒ y = 28
Sol 114. (d) ⇒ u = 44
6x-y ⇒ 6( 20
3 ) - 28 = 12
Mean Proportion = √8.1 × 3.6 = So, v = 84-u or u-4 = 40
5.4 Required ratio = 44:40
Sol 111. (d)
Third Proportion of 2 and 3 = = 11 : 10
For A:B:C:D = 4:6:7:3 3×3 = 4.5
⇒ (4 + 6 + 7 + 3) unit = 3200 2
Required ratio = 5.4 : 4.5 Sol 120. (c)
⇒ 1 unit = 160
= 6:5 Let the fourth item = x
Share of A = 4 unit = 4x160 = According to the question
Pi

640 24 48
Sol 115. (d) 32 = x
Share of B = 6 unit = 6x160 =
Let x = 1 unit, y = 1 unit and z = ⇒ x = 48 24
× 32 = 64
960
Share of C = 7 unit = 7x160 = 2 unit
1120 According to the question Sol 121.(a)
Share of D = 3 unit = 3x160 = (1+1+2) unit = 400 Given,
480 1 unit = 100 (2x-y) : (5x+3y) = 3:8
The value of z = 2 unit = 100 x 2 (2x−y) 3
For A:B:C:D = 3:5:6:2 (5x+3y) = 8
⇒ (3 + 5 + 6 + 2) unit = 3200 = 200 16x-8y = 15x+9y
⇒ 1 unit = 200 x=17y
Sol 116. (a) ⇒ xy = 17
Share of A = 3 unit = 3x200 = 1
Let the number of boys = 3 unit
600 (x2 + y 2 ) : (x2 − y 2 ) =
and number of girls = 2 unit
Share of B = 5 unit = 5x200 = 2 2
(17 + 1 ) : (17 − 1 ) 2 2
According to the question
1000 = 145 : 144

www.ssccglpinnacle.com support@ssccglpinnacle.com Ph. 09729327755, 09817390373 169


Days 17-20 Ratio and Proportion

s
sse
la
_c
ob
A’s age after 10 years = 30+5+10 We know that,
bo
ah
Sol 122. (b) = 45 Volume of cylinder = Π r2h
sm
ur
yo

Let the age of Ramesh = r and the B’s age after 10 years = 40+5+10 Volume ∝ radius2
e/
t.m
://

age of Suresh = s = 55 Volume ∝ height


tp
ht

⇒ r = 3s ………(1) Required ratio = 45 : 55 Hence ratio of their volumes is


Also, = 9 : 11 3×3×4 = 1
4×4×9 4
(r+2) = 2(s+2)
⇒ r-2s = 2 Sol 126. (c) Sol 2. (d) Total students in school
Put the value of r from 3s Let 4 years ago the age of A = 7k = 640
⇒ 3s-2s = 2 and the age of B = 5k Ratio of number of boys to girls
⇒s=2 According to the question is 5:3 = 8 units in total
7k +4+ 6 = 19
So age of ramesh (r) = 3s = 3 x 2 5k+4 + 6 15 8 units = 640
= 6 years 105k + 150 = 95k + 190 1 unit = 80
⇒k=4 Boys are 5 units = 400
Sol 123. (d) Current age of A = 7k+4 = 7(4)+4 Girls are 3 units = 240

e
Let the present age of mother = = 32 On adding 30 more girls, Total
4k and age of son = k Current age of B = 5k+4 = 5(4)+4 girls = 270
According to the question = 24 Let x boys be added to make ratio
4k+14 = 2 Required ratio = 32 : 24 of boys to girls 14:9
k+14 1

l
=4:3 400 + x = 14
⇒k=7 270 9
the present age of mother = 4k =
ac x = 20
4x7 = 28 years Sol 127. (c)
Let a = k, b = 3k and c = 5k Sol 3. (a) Mean proportional, x=
4a−b+2c = 4(k)−3k+2(5k) = 11
Sol 124. (b) ⇒ 3(a+b+c) 3(k+3k+5k) 27 √12.8 × 64.8 = 28.8
Given, Third proportional, y= 57.6×57.6
38.4
a:b:c Sol 128. (b) =86.4
2:3 Rubber : Pen : Pencil : Marker Then,
2:3 20 : 3 2x = 2×28.8 = 2
y 86.4 3
nn
⇒ a:b:c = 4:6:9 1 : 2
Let a=4k, b = 6k and c = 9k 6 : 5
Sol 4. (b) Ratio of two numbers is
( 3a2 + b2 + c2 ):( a2 + 2b2 + c2 ) Balancing the ratio for pen and
5:7
⇒ { 3(4k)2 + (6k)2 + (9k)2 }:{ pencil
First number = 5a
2 2
(4k) + 2(6k) + (9k) } 2 Rubber : Pen : Pencil : Marker
Second number = 7a
⇒ 3 : 20 : 20 : 20
5a = 20 ⇒ a = 4
(48k2 + 36k 2 + 81k 2 ) : 16k 2 + 72k 2 + 81k 2 2 : 2 : 1 : 1 Second number = 7 × 4 = 28
⇒ 165k 2 : 169k 2 = 165:169 5 : 5 : 5 : 6
Pi

________________________
Sol 5. (c) 2x+1, x+2, 2 and 5 are
30 : 200 : 100 : 120
Sol 125. (a) in proportion.
6 : 40 : 20 : 24
Let the age of A five years ago = Thus:
According to the question 2x+1 = 2 ⇒ 10x+5 = 2x+4 ⇒ 8x
3k and the age of B five years ago x+2 5
6 unit = 3
= 4k = -1 ⇒ x =− 1
1 unit = ½ 8
According to the question 7
3k+5+5 = 4 24 unit = 12 3.5(1-x) = 2 [1 − (− 81 )] = 7
2 [ 89 ]
4k+5+5 5 = 7×9
16
⇒ 15k + 50 = 16k + 40
SSC CGL TIER I 8(1+x) = 8(1- 81 ) = 7
⇒ k = 10
Age of A five years ago = 3(10) =
30 years and age of B five years
Sol 1. (b) It is given that; base
Mean proportional =
√ 7×9
16 × 7=
radius of 2 cylinders are in the 5.25
ago = 4(10) = 40 years ratio 3:4 and their heights are in
the ratio of 4:9.

www.ssccglpinnacle.com support@ssccglpinnacle.com Ph. 09729327755, 09817390373 170


Days 17-20 Ratio and Proportion

s
sse
la
_c
ob
Sol 6. (d) Let boys be ‘7x’ and Sol 11. (d) According to question: Total share = 3+4+7 = 12 unit
bo
ah
girls be ‘6x’ 2A = 5B = 7C 5
Rita’s share = 12
sm
ur
yo

As per given question: A:B:C = 35:14:10 = 59 units × 3600 = Rs1500


e/
t.m
://

7x+4 = 4 ⇒ x = 8 59 units = ₹ 1180


tp

6x−3 3
ht

Initially total number of boys and 1 unit = ₹ 20 Sol3. (b)


girls = 13x = 104 A’s share = ₹ 700 Given, a : b = 4 : 7 , b : c = 4 : 5
A:B:C
Sol 7. (a) A:B:C:D = Sol 12. (c) Notes of ₹ 10 and ₹ 50 4:7
1
3 :
1
5 : 1
6 : 1
9
are in ratio = 1:2 = 3x 4:5
⇒ A:B:C:D = 30: 18: 15: 10 = 73 3x = 12 ⇒ x = 4 Balancing the ratio for B
There are 4 notes of ₹10 and 8 A : B :
units
notes of ₹50.
⇒ B-D = 8 units = ₹832 C
Total money in wallet = 40+400 =
⇒ 1 unit = 104 ₹ 440 16 : 28 :
73 units = 73*104 = ₹ 7,592 35
Sol 13. (d) Three numbers are in Sum of three numbers =

e
Sol 8. (b) Ratio 8 years ago was = ratio = 1a :2a :4a 16+28+35 = 79 Units → 79
2:3 …(i) ⇒ difference = 1 unit Their square are in ratio = 1a2:
Ratio 4 years ago was = 5:7 ⇒ 2 4a2:16a2 = 1 Unit → 1
unit …(ii) Sum of square of numbers = 21a2 Second number =28 units → 28

l
Difference in ratio must be the = 1029 ⇒ a2 = 49 ⇒ a= 7 × 1 = 28
ac
same as both have increased by Difference between biggest and
the same number of years. Thus, smallest number = 3a = 21
multiply (i) by 2; Sol4.(c)
we get: Sol 14. (a) A:B = 3:5 and B:C = 2145 : x :: 3003 : 42
2145 3003
Ratio 8 years ago was = 4:6 2:3 x = 42
Ratio 4 years ago was = 5:7 A:B:C = 6:10:15 x = 30
In 4 years, ratio increase by 1, x : 2508:: y: 11704
30 y
Thus, 1 ratio = 4 years Sol 15. (d) 3A=4B=5C ⇒ A:B:C 2508 = 11704
nn
And = 20:15:12 y = 140
Age 4 years ago = 20 years and Sol.5:(c)
28 years Sol 16. (b) A:B:C = 3x:4x:2x 4x−4 = 11
5x−4 14
Age after 8 years from now will 9x = 990 56x-56=55x-44
be 32 years and 40 years. x = 110 x=12
Required ratio = 4:5 B = 4x = ₹440 Age of man (4 years ago)=48
SSC CHSL 2019 year
22+x Age of father (4 years ago)=60
Sol 9. (a) 10+x
16+x = 32+x Sol1 . (b)
year
Pi

⇒ (10+x)(32+x) = (22+x)(16+x) X:Y = 7:5 and Y:Z = 4:3


Age of son (4 years ago)=8 year
⇒x=8 Therefore, x:y:z = 28:20:15 After12 years,
Mean proportional of (x+1) and x+y+z = 28+20+15 = 63 units Required ratio=(60+12):(8+12)
63 units = Rs. 6300 =18:5
(3x+1) is: √(9)(25) = 15 1 unit = Rs. 100
Y = 20 units = 20 × 100 = Rs. Sol.6:(a)
Sol 10. (c) Ratio of amount of
Sunita, Amit and Vibha = 2000 a:b : c
2x:3x:4x 3 :5
Total amount = 9x Sol2.(a) 2:3
(Directly check option which is Seema : Komal : Rita
multiple of 9) 1.5 : 2 6 : 10 : 15
4x = ₹ 14416 2 : 2.5
Then, 9x = 14416
4 × 9 = ₹ 32,436 ___________________________ Sol.7: (d)
3 : 4 : 5

www.ssccglpinnacle.com support@ssccglpinnacle.com Ph. 09729327755, 09817390373 171


Days 17-20 Ratio and Proportion

s
sse
la
_c
ob
Present age of Ravi, Mohan and Sol.15: (c) Sol. 23: (c)
bo
ah
Govind = 3 × 32 + 3 × 6 = 114 LCM of 3x, 4x and 5x = 60x Sum of ages of man to son =
sm
ur
yo

Sum of ages of four = 4 × 36 = 60x = 1800 2*60 = 120


e/
t.m
://

144 x = 30 13x+7x = 120


tp
ht

The present age of shyam = Second number = 4x = 4 × 30 = x=6


144-114 = 30 years 120 Son’s age = 7x = 7 × 6 = 42 years

Sol.8: (a) Sol.16: (a) Sol.24: (c)


8+x 26+x
x:y=3:2 13+x = 40+x A : B : C
7(x - y) : (x + y) Put x = 2 180 120 100
= 7(3-2): (3+2) 10 28 The ratio of the numbers A to B =
15 = 42
= 7:5 2 = 2 (satisfy) 3:2
3 3

Sol.9:(b) Sol 25. (a)


Sol.17: (a)
Amount divided among three Perimeter of rectangular field =
Go through option (a)

e
people A, B, C in the ratio 3 : 5 : 5 +3 8 2 32 m
7+5 = 12 = 3 (which satisfy)
8, 5 +1 6 2
Length : Breadth = 5 : 3
= = (which satisfy)
(3+5+8) units = 16 units → 7−3 4 3 Let length = 5x and breadth = 3x
Rs18,144 Perimeter of rectangle = 2(l + b)
Sol.18: (d)

l
The amount B gets more than A = = 2 (5x + 3x) = 32
2 units → Rs2,268 14x+19x = 2145 16x = 32

Sol.10:(d)
ac
Let, the third number = 100
Second number = 125, then third
number = 165
x = 2145
33 = 65
The number of women in the
factory = 19x = 19 × 65 = 1,235

Sol.19:(c)
x=2
Length = 5x = 10 m and breadth
= 3x = 6 m

Sol 26. (d)


15x−6 = 13 Age ratio of Fatima and Ahmed =
Required ratio = 125:165 = 25:33 8x−6 6
90x-36 = 104x-78 3:8
Let Fatima age = 3x and Ahmed
nn
Sol.11: (c) 42 = 14x
2x+5x+3x = 800 x=3 age = 8x
x = 80 Father's age = 15 × 3 = 45 Sum of their present ages = 11x =
Required sum = 5x+3x = 8x = 8 44
× 80 = 640 Sol.20:(a) x=4
5x+4800 = 7 Difference of their ages = 8x - 3x
8x+4800 10
Sol.12: (a) 56x+33,600 = 50x+48,000 = 5x = 5 × 4 = 20 years
ratio of white marbles to red 6x = 14,400
Sol 27. (c)
Pi

marbles = 3 : 5 x = 2400
5 unit = 150 Vipin's salary = 5x = 5 × 2400 = When a same number is added to
3 unit = 90 12,000 two different numbers, their
difference remains the same.
Sol.13: (c) Sol.21:(c) 4 : 7 → difference = 3
Side of the squares = √16 : √1 = 50% of A = 25% of B 2 : 3 → difference = 1
4:1 A :B=1:2 Make the difference same, by
Ratio of perimeter will be = 4 : 1 multiplying 1 by 3.
Sol.22: (b) Therefore,
Sol.14: (c) a+3b 3 4 : 7 → difference = 3
2a+4b = 5
fourth proportional = 5a+15b = 6a+12b 6 : 9 → difference = 3
10 15 Clearly, 2 must be added to each
12 = x 3b = a
x = 18 a 3 of 4:7, to make it equal to 2:3.
b = 1

www.ssccglpinnacle.com support@ssccglpinnacle.com Ph. 09729327755, 09817390373 172


Days 17-20 Ratio and Proportion

s
sse
la
_c
ob
Sol 28. (b) Expenditure ratio of P and Q = 5x Sol:38.(b)
bo
ah
Perimeter = 2(l+b) = 50 cm - 200 : 6x - 200 Number of boys = 5
sm
8
ur
yo

area 5
length = 1 Ratio of expenditure of P and Q = Below 10 boys = 5 × 31 = 24
5
e/
t.m

8
://

length × breadth 3:4 23 1


tp

5 Below 10 girls = ×
⇒ = 3 = 4
ht

length 1 5x − 200 = 3 8
⇒ 6x − 200 4 11
⇒ breadth = 5 Below 10 students = 24
⇒ 20x - 800 = 18 x - 600 13
2(l+5) = 50 cm Above 10 students = 24
⇒ 2x = 200
⇒ l + 5 = 25 24 = 480
Total students = 260 × 13
⇒ x = 100
⇒ l = 20 cm Number of boys =480 × 85 = 300
Income of Q = 6x = Rs. 600
Sol 29. (a)
Sol 34. (b)
Sum of prices = 14,265 × 3 =
⇒ PLength
erimeter = 6
1
Let the prices are 7x, 9x, 11x 39.Sol:(a)
2 × (L + B) 6
7x+9x+11x = 42,795 ⇒ =
L 1 X share in investment = 3 × 5 +
x = 1585 (L + B)
⇒ L = 13 4.5 × 7 = 46.5
The price of costliest item = 11x

e
⇒ L + B = 3L Y share in investment = 5 × 5 + 2
= 11 × 1585 = 17,435 ⇒ B = 2L × 7 =39
⇒ L : B = 1:2 Total investment = 85.5
Sol 30. (a) Length of rectangle = a 46.5 × 6.84 = 3.72
X share = 85.5
Length:Breadth of a rectangle =

l
Breadth of rectangle = 2a
5:3 Area of rectangle = 288 cm2

⇒ 2x = 8
⇒ x=4
ac
Length = 5x and Breadth = 3x
Length - Breadth = 8
⇒ 5x - 3x = 8
L × B = 288
a × 2a = 288
a2 = 144

Sol 35. (c)


Sol.40:(d)
Sum of Present age= 52
Sum of their age after 4 years =
60 years
Area of rectangle = Length × Let three numbers be a, b and c Son age after 4 year = 60 × 51 =
Breadth = 5x × 3x = 15x2 = 15 × a = 125% of c 12 years
(4)2 = 240 m2
nn
b = 160% of c Present age of son = 8 years
Sol 31. (a) a : b = 125 : 160 = 25 : 32 Present age of father = 44 years
2(3x+2x) = 730
After 10 years son = 18 years
x = 73 Sol 36. (b) After 10 years father = 54 years
Length = 3x = 3 × 73 = 219 x:y = 3:2 Their age ratios = 18 : 54 = 1 : 3
Breadth = 2x = 2 × 73 = 146 x = 3a
Required area = 219 × 146 = y = 2a Sol:41.(b)
319743 x+y = 90 Let the fraction = x+3
x
Pi

3a + 2a = 90 According to the question


Sol 32. (b) 5a = 90 x+8 = 8
x−2 3
Number of tables = 7x and chairs a = 90
5 = 18 3x+24 =8x -16
= 9x x - y = 3a - 2a = a = 18 x=8
16x = 560 11
Original Fraction = 8
x = 35 Sol 37. (a)
After dividing it by 11 =
9x = 9 × 35 = 315 16 − a = 7 2
19 − a 6 11 2 1
8 × 11 = 4
On Cross multiplying; we get:
Sol 33. (c) 6(16 -a) = 7(19 - a)
Ratio of income of P and Q = 5 : Sol:42.(a)
96 - 6a = 133 - 7a 9+x 21+x
6 a = 133 - 96 15+x = 31+x
Income of P = 5x a = 37 279 + x 2 + 40x = 315 + x 2 + 36x
Income of Q = 6x 4x = 36
Saving of each = Rs. 200 SSC CGL2019 TIER-II x=9

www.ssccglpinnacle.com support@ssccglpinnacle.com Ph. 09729327755, 09817390373 173


Days 17-20 Ratio and Proportion

s
sse
la
_c
ob
Mean proportion of 25 and 49 is Multiply income ratio by 9 and According to the question
bo
ah 19−x = 55−x
= √25 × 49 = 35 expenditure ratio by 4
sm
ur

28−x 91−x
yo

ratio of the monthly income of X 1729 - 110x + x 2 = 1540 - 83x


e/
t.m
://

Sol:43.(b) and Y = 45 : 36 +x 2
tp
ht

Income = saving + expenditure ratio of the monthly expenditure 189 = 27x


monthly incomes of A and B = 3 : = 36 : 28 x=7
5 Saving = income - expenditure mean proportion of 16 and 49 is =
monthly saving of A and B = 2 : Saving of X : Y = 9 : 8
3
√16 × 49 = 28
Given that Sol:46.(b)
x 51.Sol:(d)
income of B : saving of A = 3 : 1 Let the fraction be = 2x+4
Total capital of A = 4 × 5 + 8 × 4
Making income of B and saving According to the question
x+3 2
= 52
of A equal 2x +4 −3 = 3 Total capital of B = 4 × 6 + 8 × 8 =
monthly incomes of A and B = (3 3x +9 = 4x + 2 88
: 5 ) × 2 = 6 :10 x=7 88 × 6.3 = 3.96
Share of B = 140

e
monthly saving of A and B =( 2 : 7
Fraction = 18
3) × 5 = 10 : 15 Difference between the numerator Sol:52.(c)
Making income of B three times and denominator of the fraction Let three no. are a, b, c.
the saving of A 18 - 7 = 11 a 2 b 4
b = 3 , c = 5

l
monthly incomes of A and B =( 6
: 10 ) × 3 = 18 : 30 a:b:c
ac Sol47:(a)
8 : 12: 15
monthly saving of A and B = 10 : ratio of the shares of A : B = 1 :
a=8k, b=12k, c=15k
15 3
8k+12k+15k=35k
Ratio of expenditure of A and B = ratio of the shares of B : C = 2 : 5
35k=280
8 : 15 ratio of the shares of C : D = 2 :
k=8
3
b=96
Sol:44.(b) Ratio of A : B : C : D = 4 : 12 :
1 1 1 Second number =96
3 , 4 , 5 when multiplied by 30 : 45
nn
60(LCM of 3,4,5) Difference between A and C is =
Sol:53.(c)
Ratio = 20 : 15 : 12 26x = 3510
B/G=27/23
Sum of ratio = 20 + 15 + 12 = 47 x = 135
B=27K
multiply by 12 = Share of D = 135 × 45 = 6075
G=23K
240 : 180 : 144
B-G=4K
Sum of ratio = 3 + 4 + 5 = 12 Sol:48.(a)
4K=200
Multiply by 47= 141 : Profit = money invested ×
K=50
188 : 235 investment time
B=27K=1350
Pi

Profit ratio = 5x : 7y : 4z = 45 :
Difference of the least share son 42 : 28
Sol:54.(b)
= 240 - 141 = 99 Time = 9 : 6 : 7
Present age of A=3x
Total sum = 12×47
99 Present age of B=5x
× 1188 = 6, 768 Sol:49.(c) 3x+5 = 13
5x+5 20
LCM of 4,3 2 is 12
x=7
Sol:45.(d) Multiply the ratio by 12
Present age of B=35 Years
ratio of the monthly income of X a:b:c=3:4:6
a : b : c = 3 : 4 : 6
and Y = 5 : 4 b c a 4 6 3
55.Sol:(a)
ratio of the monthly expenditure LCM of 4,6 and 3 is 12
Total investment by Ramesh =
=9:7 Multiply the ratio by 12
a : b : c = 9 : 8 : 24
₹40000 × 12=480000
According to the question b c a Total investment by
income of Y = expenditure of X
Kevin=₹20,000 × 6=120000
Sol:50.(a)

www.ssccglpinnacle.com support@ssccglpinnacle.com Ph. 09729327755, 09817390373 174


Days 17-20 Ratio and Proportion

s
sse
la
_c
ob
Ratio of their investment = 9000 Total profit of A B and C=
Total profit = × 2,05,000
bo
1,12,500
ah
480000/120000=4:1 11+13+15=39 units
sm
= 16,400
ur
yo

Ratio of their share =4/1 13 units=5200


e/
t.m
://

Share by Ramesh =4k 59.Sol:.(a) 1 unit=400


tp
ht

Share by Kevin=1k Let the share of A = x Difference in profit earned by A


5k=10000 Then the share of B = x -500 and C is 4 units= 4 × 400= 1600.
1k=2000 Share of C = x + 300
Share of kevin=2000 Total share of A + B + C = 3x- 64.Sol:(c)
200 = 8,200 Ratio of A and B 7 years ago=4:5
SSC CPO 2019 3x = 8,400 Ratio of A and B 8 years
56.Sol:(a) x = 2,800 hence=9:10
As we know profit depends on Total difference in time = 8+7=15
two things time and amount Sol:60.(b) years
invested Total investment = 92,500 + 5 units=15 years
Here time is constant so the ratio 1,12,500 = 2,05,000 1 unit= 3 years

e
now will depend on investment 9000
Total profit = 1,12,500 × 92,000 = Difference in ages of A and B=1
only unit= 3 years
7,400
Investment ratio of A B and C is Always remember Difference in
13750:16250:18750 ages is always the same
61.Sol:.(c)

l
Simplifying it we get irrespective of time
Let the share of A = x
55:65:75 (dividing by 250)
ac Then the share of B = x -500
Which can be further simplified 65.Sol:.(c)
Share of C = x + 300
to Total of A, B and C = 75,500
Total share of A + B + C = 3x-
11:13:15 According to the question
200 = 8,200
Given B share of profit=5200 A = B + 3500
3x = 8,400
Total profit of A B and C= B = C + 4500
x = 2,800
11+13+15=39 units A = C + 8000
x + 300 = 3,100
13 units=5200 C + 8000 + C + 4500 + C =
39 units =5200✕3=15600 75,500
nn
62.Sol:(c)
3C + 12,500 = 75,500
Profit ratio=8:7:5
57.Sol(a) C = 21,000
Time of investment ratio=7:8:14
Ratio of Age of A and B 7years A = 24,500
Capital ratio=? 24,500
ago=4:5 Share of A = 75,500 × 45,300 =
We know that capital ×
8 years hence the ratio of their
investment=profit 14,700
ages=9:10
Capital ratio= 78 : 87 : 14
5
Difference in ages= 15 years
Multiplying by 56 we get 66.Sol:.(c)
Difference in ratio= 5 units
Pi

5
Number of girls = 12
So 1 unit=3 years 64:49:20
7
Number of boys = 12
The sum of ratios of their age 7
63.Sol:(b) Below 14 girls = 3
years ago=(4+5)=9 units=27 5 of number of

years Here time is constant so the ratio girls = 3 × 5 = 1


5 12 4
now will depend on investment Below 14 boys = 4 7 4
Now we have to find out the sum 7 × 12 = 12 =
of present ages so we will simply only 1
3
add 2 × 7=14 to 27 Investment ratio of A B and C is 1 1 7
Total below 14 = + =
The required answer will be 13750:16250:18750 4 3 12
Total student = 12 × 1120 =
27+14=41 years Simplifying it we get 7
55:65:75 (dividing by 250) 1,920
58.Sol:.(c) Which can be further simplified
Total investment = 92,500 + to
1,12,500 = 2,05,000 11:13:15
Given B share of profit=5200

www.ssccglpinnacle.com support@ssccglpinnacle.com Ph. 09729327755, 09817390373 175


Days 21-23 Mixture and Alligation

s
sse
la
_c
ob
MIXTURE / िम ण water are added to it, then what is 80% और 90% भाग दू ध और पानी
bo
ah the ratio of acid and water in the से भरता है जो मशः 3 : 2, 5 : 1
sm
ur
yo

resulting solution ? और 7 : 2 के अनुपात म ह | उसके


e/

Q1. The ratios of copper to Zinc


t.m

एक बतन म अ और पानी का 32 पास मौजूद दू ध और पानी की कुल


://

in alloys A and B are 3:4 and 5:9


tp
ht

respectively. A and B are taken in लीटर िवलयन है िजसम अ और मा ा का अनुपात ात कर |


the ratio 2:3 and melted to form a पानी का अनुपात 5 : 3 है | यिद 12 SSC CHSL 10 July 2019
new alloy C. What is the ratio of लीटर िम ण िनकाल िलया जाए और (Evening)
copper to Zinc in C? इसम 7 21 लीटर पानी िमला िदया (a) 7:2
िम धातु A और B म तां बा और जाए, तो इस कार बनने वाले (b) 31:12
ज ा का अनुपात मशः 3 : 4 और िवलयन म अ और पानी का (c) 35:9
5 : 9 है | A और B को 2 : 3 के अनुपात ात कर | (d) 5:2
अनुपात म िलया जाता है तथा SSC CGL Tier II 13 September
िपघलाकर एक नयी िम धातु C 2019
बनाई जाती है | C म तां बा और ज ा (a) 4:7 Q6. A mixture has milk and water
का ा अनुपात होगा ? (b) 5:6 in the ratio (by volume) of 8:3. If

e
SSC CGL Tier II 11 September (c) 4:9 3 litres of water is added to it,
2019 (d) 8:11 then the new ratio of milk and
(a) 8:13 water becomes 2:1. What are the
(b) 3:5 Q4. Fresh fruit contains 68% quantities of milk and water

l
(c) 9:11 water and dry fruit contains 20% respectively in the mixture
(d) 27:43 water. How much dry fruit can be initially?
ac
Q2. Alloy A contains copper and
Zinc in the ratio of 4:3 and alloy
B contains copper and Zinc in the
ratio of 5:2. A and B are taken in
obtained from 100 kg of fresh
fruit
ताज़े फल म 68% पानी और सूखे
फल म 20% पानी होता है | 100
िकलो ाम ताज़े फल से िकतने
एक िम ण म दू ध और पानी आयतन
के ि कोण से 8 : 3 के अनुपात म ह
| यिद इसम 3 लीटर पानी िमला िदया
जाए, तो दू ध और पानी का नया
अनुपात 2 : 1 हो जाता है | आरं भ म
the ratio of 5:6 and melted to िकलो ाम सूखे फल ा िकये जा िम ण म दू ध और पानी की मा ा
form a new alloy. The percentage सकते ह ? मशः िकतनी थी ?
SSC CPO 16 March 2019 SSC-MTS 2 August 2019
nn
of Zinc in the new alloy is closest
to : (Morning) (Evening)
िम धातु A म तां बा और ज ा 4 : 3 (a) 80 (a) 24 litres and 9 litres
के अनुपात म ह तथा िम धातु B म (b) 60 (b) 32 litres and 12 litres
तां बा और ज ा 5 : 2 के अनुपात म (c) 40 (c) 40 litres and 15 litres
ह | A और B को 5 : 6 के अनुपात म (d) 20 (d) 16 litres and 6 litres
िलया जाता है तथा उ िपघलाकर
एक नयी िम धातु का िनमाण िकया Q5. A milkman uses three Q7. The ratio of milk and water
जाता है | इस नयी िम धातु म ज ा containers for selling milk, their in a mixture is 4:3. If we add 2
Pi

का ितशत लगभग िकतना है ? capacities being 40L, 30L and litres of water, the ratio of milk
SSC CGL Tier II 12 September 20L respectively. He fills and water becomes 8:7. What is
2019 respectively 87.5%, 80% and the quantity of the final mixture?
(a) 54 90% of the containers with a mix िकसी िम ण म दू ध और पानी का
(b) 34.2 of milk and water in the ratios, अनुपात 4 : 3 है | यिद हम 2 लीटर
(c) 36.8 3:2, 5:1 and 7:2 respectively. पानी िमला द, तो दू ध और पानी का
(d) 35 What is the ratio of total quantity अनुपात 8 : 7 हो जाता है | अंितम
of milk to that of water carried by िम ण की मा ा िकतनी है ?
Q3. A vessel contains a 32 litre him ? SSC-MTS 5 August 2019
solution of acid and water in एक दू ध वाला दू ध बेचने के िलए तीन (Morning)
which the ratio of acid and water बतनों का इ ेमाल करता है , उनकी (a) 18 litres
is 5:3. If 12 litres of the solution धा रता मशः 40 ली०, 30 ली० और (b) 30 litres
are taken out and 7 21 litres of 20 ली० है | वह बतनों का 87.5%, (c) 24 litres
(d) 24 litres

www.ssccglpinnacle.com support@ssccglpinnacle.com Ph. 09729327755, 09817390373 176


Days 21-23 Mixture and Alligation

s
sse
la
_c
ob
जाता है | नए िम ण म पानी की मा ा so that the ratio of acid and water
bo
ah
Q8. In the 60 litre mixture of milk िकतनी है ? in the resulting solution is 1:2?
sm
ur
yo

and water, the quantities of milk SSC-MTS 7 August 2019 अ और पानी के 60 लीटर िवलयन
e/
t.m

म, अ और पानी का अनुपात 3 : 7
://

and water are in the ratio of 3 : 2. (Afternoon)


tp
ht

If we want to make the ratio of (a) 84 litres है | िवलयन म िकतना अ ( लीटर


the quantities of milk and water 1 (b) 24 litres म ) िमलाया जाना चािहए तािक इस
: 1, then how much extra water (c) 48 litres कार बनने वाले िवलयन म अ
should be put in the mixture ? (d) 36 litres और पानी का अनुपात 1 : 2 हो जाए
दू ध और पानी के 60 लीटर िम ण म ?
दू ध एवं पानी की मा ा का अनुपात SSC MTS 19 August 2019
3:2 है I यिद हम दू ध और पानी की Q11. In a mixture of 60 litres, the (Evening)
मा ा का अनुपात 1:1 करना चाहता ratio (by volume) of milk and (a) 6 21
है , तब िम ण म िकतना और पानी water is 2:1. If X litres of water is (b) 6
िमलाया जाना चािहए I added in the mixture, the ratio of (c) 3
SSC-MTS 5 August 2019 milk and water becomes 1:2, then (d) 5

e
(Afternoon) what is the value of X?
(a) 10 litre 60 लीटर के िम ण म, दू ध और पानी Q14. In a 729 litre solution of
(b) 16 litre का अनुपात ( मा ा म ) 2 : 1 है | यिद Acid and Water, the ratio of acid
(c) 12 litre िम ण म X लीटर पानी िमला िदया and water is 7 : 2. To get a
जाए, तो दू ध और पानी का अनुपात 1

l
(d) 14 litre solution with the ratio 5 : 3
: 2 हो जाता है | X का मान ा है ? between acid to water, how many
ac
Q9. The ratio of Milk and water
in a mixture ( by volume ) is 4 : 3.
If 4 litres of water is added to it,
then the ratio becomes 1 : 1.
What was the quantity of milk in
SSC-MTS 7 August 2019
(Evening)
(a) 40
(b) 56
(c) 20
litres of water should be mixed in
it ?
अ और पानी के 729 लीटर
िवलयन म अ और पानी का
अनुपात 7:2 है | ऐसा िवलयन ा
the initial ratio ? (d) 60 करने के िलए उसमे िकतने लीटर
एक िम ण म दू ध तथा पानी (मा ा पानी िमलाया जाना चािहए, िजसमे
म) का अनुपात 4:3 है I यिद इसम 4 अ और पानी का अनुपात 5:3 हो?
nn
लीटर पानी िमलाया जाये तो अनुपात Q12. In a mixture of 100 litres of SSC MTS 20 August 2019
1:1 हो जाता है | शु आती िम ण म milk and water, the ratio of milk (Morning)
दू ध की मा ा िकतनी थी? and water is 3 : 2. If this ratio is (a) 180.4
SSC-MTS 5 August 2019 to be 1 : 1, how much more water (b) 178.2
(Evening) is to be added to the mixture? (c) 182.4
(a) 16 litre दू ध और पानी के 100 लीटर िम ण (d) 187.2
(b) 20 litre म, दू ध और पानी का अनुपात 3 : 2 है
(c) 18 litre | यिद इस अनुपात को 1 : 1 करना है , Q15. The ratio of spirit and water
Pi

(d) 12 litre तो िम ण म िकतना अित र पानी in solutions in vessels A and B


िमलाना होगा ? are 3:4 and 5:9 respectively. The
Q10. The ratio (by volume) of SSC MTS 14 August 2019 contents of A and B are mixed in
milk and water in a mixture is (Afternoon) the ratio 2:3. What is the ratio of
2:1. If we add 12 litres of water in (a) 25 litres water and spirit in the resulting
the mixture, then the ratio of milk (b) 15 litres solution?
and water becomes 4:3. What is (c) 30 litres बतन A और B म मौजूद िवलयन म
the quantity of water in the new (d) 20 litres रट तथा पानी का अनुपात
mixture? मशः 3 : 4 और 5 : 9 है | A और B
एक िम ण म दू ध और पानी का Q13. In a 60 litre solution of acid की सामि यों को 2 : 3 के अनुपात म
अनुपात ( मा ा म ) 2 : 1 है | यिद हम and water, the ratio of acid and िमला िदया जाता है | इस कार बनने
िम ण म 12 लीटर पानी िमला द, तो water is 3:7. How much (in litre) वाले िवलयन म पानी और रट का
दू ध और पानी का अनुपात 4 : 3 हो acid is to be mixed in the solution अनुपात ा होगा ?

www.ssccglpinnacle.com support@ssccglpinnacle.com Ph. 09729327755, 09817390373 177


Days 21-23 Mixture and Alligation

s
sse
la
_c
ob
SSC MTS 22 August 2019 SSC CGL TIER I mixture becomes 8 : 13. What is
bo
ah
(Afternoon) x:y?
sm
ur
yo

(a) 43:27 Q1. 25 litres of a mixture contains A और B एिसड और पानी के िम ण


e/
t.m

ह। A और B म पानी और एिसड का
://

(b) 39:16 30% of spirit and rest water. If 5


tp
ht

(c) 8:13 litres of water be mixed in it, the अनुपात मशः 4: 5 और 1: 2 है ,


(d) 1:3 percentage of spirit in the new यिद x लीटर A को Y लीटर B के
mixture is: / 25 लीटर िम ण म साथ िमलाया जाता है , तो िम ण म
Q16. A mixture contains milk and 30% रट तथा शेष पानी है | यिद पानी और एिसड का अनुपात 8: 13.
water in the ratio (by volume) 5:3 5 लीटर पानी िमला िदया जाए, तो नए हो जाता है ।तो x: y ात कीिजए |
and another mixture, of the same िम ण म रट का ितशत िकतना CGL 2019 Tier-II (15/10/2020)
volume as that of the former, होगा ? SSC CGL 6 March 2020 (a) 5 : 6
contains water and milk in the (Evening) (b) 2 : 5
ratio (by volume) 1:3. In what (c) 3 : 4
ratio, two mixture be mixed in (a) 25% (d) 2 : 3
order to obtain a new mixture (b) 45%

e
consisting of milk and water in (c) 33 31 % Q2. How many kg of the rice
the ratio (by volume) 7:3 ? िकसी (d) 12 %1 costing ₹42 per kg should be
2
िम ण म दू ध एवं पानी, आयतन के mixed with 7 21 kg rice costing
ि कोण से, 5 : 3 के अनुपात म ह Q2. Two bottles of same capacity ₹50 per kg so that by selling the
तथा एक अ िम ण म, िजसका

l
are 35% and 33 31 % full of mixture at ₹53.10 per kg, there is
आयतन पहले िम ण के िजतना ही a gain of 18%?
ac orange juice, respectively. They
है , उसम पानी और दू ध 1 : 3 के िकतने िकलो चावल िजसकी लागत
are filled up completely with
अनुपात म ह | इन िम णों को िकस ₹42 ित िकलो ाम है , िजसे 7 21
apple juice and then the contents
अनुपात म िमलाना चािहए तािक िकलो ाम चावल , िजसकी लागत
of both bottles are emptied into
ा होने वाले नए िम ण म दू ध और ₹50 ित िकलो ाम है , के साथ
another vessel. The percentage of
पानी ( आयतन के अनुसार ) 7 : 3 के िमलाया जाना चािहए तािक िम ण
apple juice in the mixture is: /
अनुपात म हो ? को ₹53.10 पये ित िकलो ाम पर
समान धा रता वाले दो बोतल मशः
SSC MTS 13 August 2019 बेचकर 18% का लाभ हो|
35% और 33 31 % संतरे के जूस से
(Afternoon)
nn
भरे ए ह | उ पूरी तरह सेब के CGL 2019 Tier-II (15/10/2020)
(a) 2:3 (a) 12 21
जूस से भर िदया जाता है तथा िफर
(b) 3:4
दोनों बोतलों की सामि यों को एक (b) 10 21
(c) 5:6
अ बतन म उड़े ल िदया जाता है | (c) 8
(d) 3:2
इस िम ण म सेब के जूस का ितशत (d) 9
िकतना है ?
Q17. 57 sweets were distributed
SSC CGL 3 March 2020 Q3. In what ratio should sugar
among 10 children such that each
(Afternoon) costing ₹ 40 per kg be mixed
girl got 6 sweets and each boy got
Pi

5 sweets. What is the number of with sugar costing ₹ 48 per kg, so


(a) 34 61
boys ? that to earn 20% by selling the
10 ब ों म कुल 57 िमठाइयाँ इस (b) 64 31 mixture at ₹ 54 per kg ?
कार बां टी गई िक ेक लड़की (c) 60 32 चीनी िजसकी लागत ₹ 40 ित िक ा
को 6 िमठाइयाँ और ेक लड़के (d) 65 65 और चीनी िजसकी लागत ₹ 48 ित
को 5 िमठाइयाँ िमली | लड़कों िक िक ा को िकस अनुपात म िमलनी
सं ा है : SSC CGL TIER-II चािहए, तािक िम ण को ₹ 54 ित
SSC MTS 13 August 2019 Q1. A and B are solutions of acid िक ा बेचकर 20% कमाया जा सके?
(Morning) and water. The ratios of water and CGL 2019 Tier-II (16/10/2020)
(a) 3 acid in A and B are 4 : 5 and 1 : (a) 2 : 3
(b) 6 2, respectively If x liters of A is (b) 4 : 7
(c) 4 mixed with Y liters of B, then the (c) 3 : 5
(d) 5 ratio of water and acid in the (d) 5 : 8

www.ssccglpinnacle.com support@ssccglpinnacle.com Ph. 09729327755, 09817390373 178


Days 21-23 Mixture and Alligation

s
se
s
la
_c
ob
(c) 4L
bo
ah
Q4. A drink of chocolate and (d) 2L
sm
ur
yo

milk contains 8% pure chocolate


e/
t.m
://

by volume. If 10 liters of pure Q.7 The price of a variety of a


tp
ht

milk are added to 50 liters of this commodity is ₹ 7/kg and that of


drink, the percentage of chocolate another is ₹ 12/kg. Find the ratio
in the new drink is: in which two varieties should be
चॉकलेट और दू ध का एक पेय म 8% mixed so that the price of the
शु चॉकलेट होता है । यिद इस पेय mixture is ₹ 10/kg.
के 50 लीटर म 10 लीटर शु दू ध एक व ु की एक िक की कीमत
िमलाया जाता है , तो नए पेय म ₹ 7 / िक ा और दू सरे की ₹ 12 /
चॉकलेट का ितशत होता है : िक ा है । उस अनुपात का पता लगाएँ
CGL 2019 Tier-II (16/10/2020) िजसम दो िक ों को िमलाया जाना
(a) 5 31 चािहए तािक िम ण की कीमत 10 /
(b) 6 31 िक ा हो |

e
(c) 6 32 CGL 2019 Tier-II (18/10/2020)
(d) 5 32 (a) 3:4
(b) 4:5
(c) 2:3
Q5: Alloy A contains metal x and

l
(d) 2:5
y only in the ratio 5 : 2 and alloy
ac
B contains these metal in the ratio
3 : 4. Alloy C is prepared by
mixing A and B in the ratio 4 : 5.
The percentage of x in alloy C is:
िम धातु A म धातु x और y केवल
5: 2 के अनुपात म होते ह और िम
धातु B म ये धातु 3: 4 के अनुपात म
होते है । िम धातु C म A और B को
nn
4 : 5 म िम ण करके तैयार की जाती
है । िम धातु C म x का ितशत है :
CGL 2019 Tier-II (16/10/2020)
(a) 44 94
(b) 56
(c) 45
(d) 55 95
Pi

Q6. A container contains 20 L


mixture in which there is 10%
sulphuric acid. Find the quantity
of sulphuric acid to be added in it
to make the solution to contain
25% sulphuric acid.
एक कंटे नर म 20L िम ण होता है
िजसम 10% स ू रक एिसड होता
है । 25% स ू रक एिसड यु
घोल बनाने के िलए इसम िकतनी
स ू रक एिसड की मा ा िमलाएँ ।
CGL 2019 Tier-II (18/10/2020)
(a) 3L
(b) 5L

www.ssccglpinnacle.com support@ssccglpinnacle.com Ph. 09729327755, 09817390373 179


Days 21-23 Mixture and Alligation

s
se
s
la
_c
ob
SOLUTION Pulp : Water
bo
ah Fresh fruit 8 : 17 Sol 7. (b)
sm
ur
yo

Sol 1. (d) Dry fruit 4 : 1 Milk : Water


e/
t.m
://

Copper : Zinc : Total Balancing the ratio for pulp Initial 4 : 3


tp
ht

A 3 : 4 : 7 Pulp : Water New 8 : 7


B 5 : 9 : 14 Fresh fruit 32 : 68 Only water is added so quantity
Balancing the ratio for total Dry fruit 32 : 8 of milk in both cases will be
quantity and the ratio in which According to the question same. So, Balance the ratio for
mixtures are taken. (32+68) unit = 100 kg milk.
Copper : Zinc : Total 1 unit = 1 kg Milk : Water
A 12 : 16 : 7x2x2 40 unit = 40 kg Initial 8 : 6
B 15 : 27 : 14x1x3 New 8 : 7
__________ Sol 5.(d) Now,
27 : 43 87.5% of 40 = 35 kg (7-6) unit = 2
80% of 30 = 24 kg Final quantity of mixture =

e
Sol 2.(d) 90% of 20 = 18 kg (8+7)unit = 15x2 = 30 litres
Copper : Zinc : Total 3 +24 x
Total milk = [35 x 3+2 5 Alternate:
5+1
A 4 : 3 : 7 +18 x 7 = 55 kg Let the initial quantity of milk =
7+2
B 5 : 2 : 7 Total water = [35+24+18-55] = 4k and water = 3k

l
Balancing the ratio for the ratios 22 According to the question
in which mixtures are taken. 4k 8
ac Required ratio = 55 : 22 = 5:2 3k+2 = 7
Copper : Zinc : Total ⇒ 28k = 24k + 16
A 20 : 15 : 7x5 Sol 6. (a) ⇒k=4
B 30 : 12 : 7x6 Milk : Water the initial quantity of mixture =
_____________ Initial 8 : 3 (4+3)k= 7x4 = 28 litres
50 : 27 New 2 : 1 and the final quantity of mixture
Required %Age = 50+2727 x 100 ≈
Only water is added so quantity = 28+2 = 30 litres
35 of milk in both cases will be
nn
same. So, Balance the ratio for Sol 8. (c)
Sol 3. (b) milk. Milk : Water
Let the quantity of acid = 5 unit Milk : Water Initial 3 : 2
and quantity of water = 3 unit Initial 8 : 3 New 1 : 1
According to the question New 8 : 4 Only water is added so quantity
(5+3) unit = 32 Now, of milk in both cases will be
1 unit = 4 (4-3) unit = 3 same. So, Balance the ratio for
5 unit = 20 Initial quantity of milk = 8unit = milk.
3 unit = 12
Pi

8x3 = 24 litres Milk : Water


In 12 litres of taken out mixture Initial quantity of water = 3unit = Initial 3 : 2
quantity of acid = 12x 85 = 7 21 3x3 = 9 litres New 3 : 3
litres Alternate : Now,
In 12 litres of taken out mixture Let the initial quantity of milk = (3+2) unit = 60
quantity of water = 12- 7 21 =4 21 8k and water = 3k 1 unit = 12
litres According to the question Required quantity of water =
Required ratio = 20-7 21 : 12-4 21 8k 2
3k+3 = 1
(3-2)unit =1unit = 12 litres
+7 21 ⇒ 8k = 6k + 6 Alternate:
25 ⇒k=3 the initial quantity of milk = 60
= 2 :15 3 = 36 litres and water = 60
the initial quantity of milk = 8k= × 3+2
=5:6 2
8x3 = 24 litres × 3+2 = 24 litres. Let the quantity

Sol 4.(c) and the initial quantity of water = of water added = x litres
68% = 2017 and 20% = 1 3k = 3x3 = 9 litres According to the question
5

www.ssccglpinnacle.com support@ssccglpinnacle.com Ph. 09729327755, 09817390373 180


Days 21-23 Mixture and Alligation

s
se
s
la
_c
ob
36 = 11 x 2 = 40 litres. Let the amount
bo
24+x 2+3
ah ⇒ k = 24
sm
⇒ 36 = 24 + x of water added = x
ur
yo

the required quantity of water = According to the question


e/

⇒ x = 12
t.m
://

24+12 = 36 lites 60 1
40+x = 1
tp
ht

Sol 9. (a) ⇒ 60 = 40 + x
Milk : Water Sol 11.(d) ⇒ x = 20
Initial 4 : 3 Milk : Water
New 1 : 1 Initial 2 : 1 Sol 13. (c)
Only water is added so quantity New 1 : 2 Acid : Water
of milk in both cases will be Only water is added so quantity Initial 3 : 7
same. So, Balance the ratio for of milk in both cases will be New 1 : 2
milk. same. So, Balance the ratio for Only acid is added so quantity of
Milk : Water milk. water in both cases will be same.
Initial 4 : 3 Milk : Water So, Balance the ratio for water.
New 4 : 4 Initial 2 : 1 Acid : Water

e
Now, New 2 : 4 Initial 6 : 14
(4-3) unit = 4 Now, New 7 : 14
Required quantity of milk = 4 (2+1) unit = 60 Now,
unit = 16 litres 1 unit = 20 (6+14) unit = 60

l
Alternate: Required quantity of water x = 1 unit = 3
Let the initial quantity of milk = (4-1) unit = 3 unit = 3x20 =60 Required quantity of acid = (7-6)

⇒ 4k = 4 + 3k
⇒k=4
ac
4k and water = 3k.
According to the question
4k 1
3k+4 = 1
litres
Alternate:
the initial quantity of milk = 60 x
2
2+1 = 40 litres and water = 60 x
1
2+1 = 20 litres.
unit = 1 unit = 3 litres
Alternate:
the initial quantity of acid = 60 x
3
3+7 = 18 litres and water = 60 x
7 = 42 litres. Let the amount of
3+7
the initial quantity of milk = 4x4 According to the question acid added = x litres
40 1
= 16 lites 20+X = 2 According to the question
nn
⇒ 80 = 20 + X 18+x = 1
42 2
Sol 10.(d) ⇒ X = 60 ⇒ 42 = 36 + 2x
Milk : Water ⇒x=3
Initial 2 : 1 Sol 12. (d)
New 4 : 3 Milk : Water Sol 14.(b)
Only water is added so quantity Initial 3 : 2 Acid : Water
of milk in both cases will be New 1 : 1 Initial 7 : 2
same. So, Balance the ratio for Only water is added so quantity New 5 : 3
Pi

milk. of milk in both cases will be Only water is added so quantity


Milk : Water same. So, Balance the ratio for of acid in both cases will be
Initial 4 : 2 milk. same. So, Balance the ratio for
New 4 : 3 Milk : Water acid.
Now, Initial 3 : 2 Acid : Water
(4-3) unit = 12 New 3 : 3 Initial 35 : 10
Required quantity of water = 3 Now, New 35 : 21
unit = 3x12 =36 litres (2+3) unit = 100 Now,
Alternate: 1 unit = 20 (35+10) unit = 729
Let the initial quantity of milk = Required quantity of water = 1 unit = 16.2
2k and water = k. (3-2) unit = 1 unit = 20 litres Required quantity of water =
According to the question Alternate:
2k 4
(21-10) unit = 11 unit = 11x16.2
k+12 = 3 the initial quantity of milk = 100 = 178.2 litres
⇒ 6k = 48 + 4k 3 = 60 litres and water = 100
x 2+3 Alternate:

www.ssccglpinnacle.com support@ssccglpinnacle.com Ph. 09729327755, 09817390373 181


Days 21-23 Mixture and Alligation

s
se
s
la
_c
ob
the initial quantity of acid = 729 x Quantity of B = 7 21 × 5 = 12 21
bo
3
7 ah
2+7 = 567 litres and water = 729
sm
kg
ur
yo

x 2 = 162 litres. Let the amount


e/
t.m

2+7
://

Sol:3..(c)
tp

of water added = x litres


ht

Price after mixing = 54 × 100 =


According to the question 120
567 5
162+x = 3 45
⇒ 1701 = 810 + 5x A B
⇒ x = 178.5 40 48

Sol 15. (a) According to the question 45


Spirit : Water : Total (7+3) unit = 10
A 3 : 4 : 7 1 unit = 1 3 5
B 5 : 9 : 14 Number of boys (3 unit) = 3x1 =
Balancing the ratio for the ratios 3 Sol:4..(c)
in which mixtures are taken. % of pure chocolate in 50 litre =

e
Spirit : Water : Total SSC CGL TIER I 4
A 12 : 16 : 7x2x2 After addition of milk total drink
B 15 : 27 : 14x1x3 Sol 1. (a) In 25 litres mixture, =60litre
________ water = 70% of 25 = 17.5 litre % of pure chocolate now =

l
27 : 43 and spirit = 7.5 4 2
60 × 100 = 6 3 %
ac
Required ratio = 43 : 27 After adding 5 litres of water,
spirit % = 7.5
30 × 100 = 25% Sol:5..(d)
Sol 16.(a) x in Alloy A = 75
Amount of milk in first mixture = Sol 2. (d) x in alloy B = 3
7 and 33 1 % = 1 7
5 35% = 20
8 3 3 Taking A and B in ratio 4 : 5 =
20 15
Amount of milk in second 7 +7
= 95
SSC CGL TIER-II 9
mixture = 43
Sol:1.(c) Percentage = 95 × 100 = 55 95 %
Amount of milk in final mixture
nn
Water in solution A = 4
7
= 10 9
Water in solution B = 1 Sol:6..(c)
3
8 Amount of other components in
Water in mixture = 21 the mixture is the same in initial
A B and final mixture.
4 1
9 3 90% of initial(I) = 75% of
final(F)
8
21 I/F=5/6
Pi

I=5K, F=6K
3 4
Required ratio = 2:3 63 63 Amount of sulphuric acid to be
Ratio of mixture A and B = 3 : 4 added= 1k
Sol 17. (a) 5k=20L, 1k=4L
Average sweet distributed per Sol:2.(a)
57 = 5.7
children = 10 Sol:7..(c)
CP of mixture = 53.1 × 100 = 45
118
A B
42 50

45
5 3 Ratio=2:3
Ratio of mixture A and B = 5 : 3

www.ssccglpinnacle.com support@ssccglpinnacle.com Ph. 09729327755, 09817390373 182


Days 24-27 Percentage

s
sse
la
_c
ob
PERCENTAGE / ितशतता बढ़ाने से आता है , तो 210 का x%
bo
ah 180 के (x+20) % से िकतना ितशत Q4. The income of A is 50%
sm
ur
yo

कम है ? more than that of B. If the income


e/

Key Points:/ मुख िबंदु:


t.m

of A is increased by 40% and the


://

SSC CGL 4 June 2019


tp
ht

(Morning) income of B is increased by 90%,


What does a percentage, say 20% then the percentage increase in
(a) 33 31
mean? their combined income will be:
(b) 18 A की आय B की आय से 50%
20% simply means 20 of
100 (c) 16 32 अिधक है | यिद A की आय 40% से
something.Also, you can see (d) 20 बढ़ा दी जाए और B की आय 90% से
20 1
100 = 5 = 0.2 i.e. 0.2 of बढ़ा दी जाए, तो उनकी संयु आय
something is same as 20% of that Q2. The income of Raju is 20% म िकतने ितशत की वृ होगी ?
thing. SSC CGL 6 June 2019
more than his expenditure. If his
(Morning)
ितशत, मान लीिजए 20%, का ा income increases by 60% and his
(a) 64
मतलब है ? expenditure increases by 70% (b) 55
20% का अथ केवल, कुछ मू का then by what percent does his (c) 60

e
savings increase or decrease? (d) 70
100 है | इसके अलावा, आप दे ख
20
राजू की आय उसके य से 20%
सकते ह 20
100 = 1
5 = 0.2 अथात
अिधक है | यिद उसकी आय 60% Q5. When the price of an item
िकसी चीज़ का 0.2 उस चीज़ के
बढ़ जाती है और उसके य म 70% was reduced by 20% , then the
20% के समान है |

l
की वृ हो जाती है , तो उसकी बचत sale increased by x% . If there is
ac म िकतने ितशत की कमी या वृ an increase of 60% in receipt of
Important Fractions and their होगी ? the revenue , then the value of x
equivalent Percentage SSC CGL 4 June 2019 is :
(Afternoon) जब िकसी व ु की कीमत 20% कम
1) 1 (a) It decreases by 10%/ यह 10% कर दी गयी, तो इसकी िब ी x% से
2 = 50%
1 1 100 कम हो जाएगी बढ़ गयी | यिद आय की ा म
2) 3 = 33 3 % = 3 %
(b) It decreases by 2%/ यह 2% 60% की वृ होती है , तो x का मान
3) 2 2 200
3 = 66 3 % = 3 % कम हो जायेगी है :
1
(c) It increases by 10%/ यह 10%
nn
4) 4 = 25% SSC CGL 6 June 2019
5) 1 बढ़ जाएगी (Afternoon)
5 = 20%
3 (d) It increases by 2%/ यह 2% बढ़ (a) 120
6) 4 = 75%
जाएगी (b) 96
7) 5
4 = 125% (c) 100
8) 1
8 = 12.5% Q3. The income of A is 25% (d) 80
more than that of B and the
income of C is 65% less than the Q6. A is 20% less than B and C is
These fractions are very common
30% more than D. If D is 25%
Pi

and it will be very helpful to learn sum of the incomes of A and B.


Income of C is what percent less less than A, then which of the
them.
than the income of A? following is true?
ये िभ ब त आम ह और उ
A की आय B की आय से 25% A, B से 20% कम है और C, D से
सीखना ब त ही उपयोगी होगा।
अिधक है और C की आय A और B 30% अिधक है | यिद D, A से 25%
की आय के योग से 65% कम है | C कम है , तो िन म से ा सही है ?
Variety Questions
की आय A की आय से िकतना SSC CGL 7 June 2019
ितशत कम है ? (Morning)
Q1. If decreasing 120 by x% (a) B = 0.39C
SSC CGL 4 June 2019
gives the same result as (b) C = 0.78B
(Evening)
increasing 40 by x% then x% of (c) B = 0.78C
(a) 28
210 is what percent less than (x + (d) C = 0.39B
(b) 32
20)% of 180?
(c) 35
यिद 120 को x% कम करने से वही
(d) 37
प रणाम आता है जो 40 को x%

www.ssccglpinnacle.com support@ssccglpinnacle.com Ph. 09729327755, 09817390373 183


Days 24-27 Percentage

s
sse
la
_c
ob
Q7. The price of sugar has Q10. Sudha decided to donate अिधक िमले और A को िमले अंक
bo
ah
increased by 20%. A person 12% of her monthly income to an उनके अंकों के जोड़ का 55% थे | A
sm
ur
yo

wants to increase its expenditure orphanage. On the day of और B ारा ा िकये गए अंक (
e/
t.m

मशः ) ात कर |
://

by 8% only. By what percent donation, she changed her


tp
ht

should he decrease his decision and donated a sum of Rs. SSC CHSL 5 July 2019
consumption? 4800 which was equal to 80% of (Evening)
चीनी की कीमत 20% से बढ़ जाती है what she had decided earlier. (a) 44, 36
| एक अपना य केवल 8% What is 27% of her monthly (b) 36, 28
बढ़ाना चाहता है | उसे अपनी खपत income ? (c) 38, 30
म िकतने ितशत की कमी करनी सुधा ने अपनी मािसक आय का 12% (d) 40, 32
चािहए ? एक अनाथालय म दान दे ने का िनणय
SSC CGL 10 June 2019 िलया | दान दे ने के िदन, उसने अपना Q13. The sum of salaries of A
(Afternoon) िनणय बदल िलया और 4800 पये and B is Rs. 43000. A spends
(a) 10% दान िदए जो उसके ारा पूव म सोची 95% of his salary and B spends
(b) 11% गयी रािश का 80% था | उसकी 80% of his salary. If their savings
मािसक आय का 27% ात कर |

e
(c) 9% are the same, what is A’s salary
(d) 12% SSC CHSL 4 July 2019 (in Rs.) ? / A और B के वेतन का
(Morning) योग 43000 पये है | A अपनी 95%
Q8. If 85% of a number is added (a) Rs. 13959 आय और B अपनी 80% आय खच
कर दे ता है | यिद उनकी बचत समान

l
to 75, then the result is the (b) Rs. 11934
number itself. The number is: (c) Rs. 14040 है , तो A का वेतन ात कर |

है :
ac
यिद िकसी सं ा के 85% को 75 म
जोड़ा जाए, तो प रणाम
सं ा यं ा होती है | वह सं ा

SSC CGL 13 June 2019


प वह
(d) Rs. 13500

Q11. A number is increased by


30%, then decreased by 25%, and
then further increased by 25%.
SSC CHSL 8 July 2019
(Morning)
(a) 10600
(b) 8000
(c) 35000
(Evening) What is the net increase/decrease (d) 34400
(a)500 percent in the number (correct to
(b)200 the nearest integer) ? Q14. For an examination of a
nn
(c)300 एक सं ा को पहले 30% से बढ़ाया practical based subject, the total
(d)100 जाता है िफर 25% कम कर िदया marks is 100. The break-up for
जाता है और इसके बाद पुनः 25% theory, practical, project and viva
Q9. The price of an article बढ़ाया जाता है | सं ा म शु voce is 40%, 30%, 20%, 10%. A
increases by 20% every year. If ितशत वृ / कमी ( िनकटतम candidate scored 80% in theory,
the difference between the prices पूणाक म ) ात कर | 70% in practical, 60% in project
at the end of the third and fourth SSC CHSL 4 July 2019 and 50% in viva voce. What was
years is Rs. 259.20, then 40% of (Afternoon) her aggregate percentage ?
Pi

the price (In Rs.) at the end of the (a) 22% decrease ै कल आधा रत िवषय की एक
second year is : (b) 22% increase परी ा के िलए कुल अंक 100 ह |
एक व ु की कीमत हर वष 20% (c) 21% decrease ोरी, ै कल, ोजे और वाइवा
बढ़ जाती है | यिद तीसरे और चौथे (d) 21% increase के िलए इसम से 40%, 30% , 20%
वष की कीमतों म 259.20 पये का और 10% तय िकये गए ह | एक छा
अंतर है , तो दू सरे वष के अंत म Q12. Two students, A and B, को ोरी म 80%, ै कल म 70%,
इसकी कीमत का 40% होगा : appeared for an examination. A ोजे म 60% और वाइवा म 50%
SSC CHSL 3 July 2019 secured 8 marks more than B and अंक आए | उसका कुल ितशत ात
(Evening) the marks of the former was 55% कर |
(a) 484 of the sum of their marks. The SSC CHSL 9 July 2019
(b) 432 marks obtained by A and B, (Evening)
(c) 384 respectively, are : (a) 70
(d) 472 दो छा A और B एक परी ा म (b) 72
शािमल ए | A को B से 8 अंक (c) 68

www.ssccglpinnacle.com support@ssccglpinnacle.com Ph. 09729327755, 09817390373 184


Days 24-27 Percentage

s
sse
la
_c
ob
(d) 67 एक परी ा म, िव ान म 47% सफल April 2018 are Rs. 5,68,000. The
bo
ah रहे तथा गिणत म 51% असफल हो commission earned is
sm
ur
yo

Q15. What percentage of Rs.124 गए | यिद 42% दोनों िवषयों म िकसी िवपणन एजट को एक माह म
e/
t.m

असफल रहे , तो दोनों िवषयों म पहले 2,00,000 पये की िब ी पर


://

is Rs. 49.60 ?
tp
ht

49.60 पये 124 पये का िकतना िकतने ितशत सफल रहे ? 2%, अगले 200000 पये की िब ी
ितशत है ? SSC CPO 13 March 2019 पर 1.5% तथा िब ी की शेष रािश
SSC CPO 16 March 2019 (Evening) पर 1% कमीशन िमलता है | यिद
(Morning) (a)38% अ ैल 2018 माह म उस एजट ने
(a)123 (b)42% 5,68,000 पये की िब ी की है , तो
(b)250 (c)40% उसके ारा ा कमीशन ात कर |
(c)40 (d)36% SSC CPO 15 March 2019
(d)16 (Morning)
Q19. The population of a town (a)Rs.8680
Q16. The income of A is 24% has increased by 5% at the end of (b)Rs.7730
more than the income of B. By the first year and decreased by (c) Rs.8240

e
what percent is the income of B 4% at the end of second year. If (d)Rs.7105
less than the income of A? / A की the population at the end of
आय B की आय से 24% अिधक है | second year was 55,12,248, then Q22. 2.4 converted to percentage
B की आय A की आय से िकतना the population at the beginning of is:
ितशत कम है ? 2.4 को ितशत म बदलने से ा

l
first year was:
SSC CPO 12 March 2019 एक शहर की आबादी पहले वष के आएगा ?
(Evening)
(a) 150
(b)
(c)
(d)
7 %
600
29
500
31
600
%
%

%
ac अंत म 5% बढ़ी है और दू सरे वष के
अंत म 4% कम ई। यिद दू सरे वष
के अंत म जनसं ा 55,12,248 थी,
तो पहले वष की शु आत म
जनसं ा थी:
SSC CPO 16 March 2019
(Afternoon)
(a) 0.24%
(b) 24%
(c) 240%
31
SSC CPO 14 March 2019 (d) 2.4%
(Morning)
Q17. In an examination, 48% of
(a)55,72,950 Q23. In an examination, Shreya
nn
candidates passed in science and
(b)56,23,012 scored 84 out of 90 in
56% failed in mathematics. If
(c)54,68,500 Mathematics, 45 out of 50 in
32% failed in both subjects, then
(d)53,00,420 Science, 23 out of 25 in
what percent passed in both
Computer Science and 68 out of
subjects? / एक परी ा म 48% छा
Q20. If 20% of a = b, then b% of 80 in English. In which subject
िव ान म उ ीण ए जबिक 56%
20 is equal to: did Shreya score the highest
छा गिणत म अनु ीण रह गए | यिद
यिद a का 20% = b है , तो 20 का b% percentage?
32% छा दोनों िवषयों म असफल
िकसके बराबर होगा ? एक परी ा म ेया को गिणत म 90 म
Pi

रहे , तो दोनों िवषयों म िकतने ितशत


SSC CPO 16 March 2019 से 84 अंक, िव ान म 50 म से 45
उ ीण ए ?
(Evening) अंक, कं ूटर िव ान म 25 म से 23
SSC CPO 12 March 2019
(a)4% of a अंक तथा अं ेजी म 80 म से 68 अंक
(Evening)
(b)16% of a िमले | िकस िवषय म ेया का अंक
(a) 24%
(c)8% of a ितशत सवािधक है ?
(b) 32%
(d)2% of a SSC CPO 16 March 2019
(c) 28%
(Afternoon)
(d) 22%
Q21. A marketing agent earns a (a)Mathematics/ गिणत
commission of 2% on first Rs. (b)English/ अं ेजी
Q18. In an examination, 47%
2,00,000, 1.5% on next Rs (c)Science/ िव ान
passed in science and 51% failed
2,00,000 and 1% on the (d)Computer Science/ कं ूटर
in mathematics. If 42% failed in
remaining amount of sales made िव ान
both subjects, what percentage
in a month. If the sales achieved
passed in both subjects?
by the agent for the month of SSC CGL TIER II

www.ssccglpinnacle.com support@ssccglpinnacle.com Ph. 09729327755, 09817390373 185


Days 24-27 Percentage

s
sse
la
_c
ob
(a) 6% increase total number of permanent
bo
ah
Q1. If A is 28% more than B and (b) 7% decrease employees is 740, then 7/15 of
sm
ur
yo

C is 25% less than the sum of A (d) No decrease the total number of employees
e/
t.m
://

and B, then by what percent will (d) 9% decrease exceeds the number of temporary
tp
ht

C be more than A (correct to one female employees by :


decimal place) ? Q4. When the price of an item एक कंपनी के दो-ितहाई कमचारी
यिद A, B से 28% अिधक है और C, was reduced by 25%, then its sale पु ष ह तथा शेष मिहलाएं ह | यिद
A और B के योग से 25% कम है , तो was increased by x%. If there is पु ष कमचा रयों म से ⅜ और
C, A से िकतना ितशत ( एक an increase of 20% in the receipt मिहला कमचा रयों म से ⅖ कमचारी
दशमलव थान तक सही ) अिधक of the revenue, then the value of x अ थायी कमचारी ह और थायी
होगा ? will be : कमचा रयों की कुल सं ा 740 है ,
SSC CGL TIER II (11 जब एक व ु की कीमत 25% कम तो कमचा रयों की कुल सं ा का
September 2019) कर दी गयी, तो इसकी िब ी x% बढ़ 7/15 अ थायी मिहला कमचा रयों की
(a) 32.2% गयी | यिद आय ा म 20% की सं ा से _____ अिधक है |
(b) 28% वृ ई है , तो x का मान ा होगा ? SSC CGL TIER II (12

e
(c) 43% SSC CGL TIER II (11 September 2019)
(d) 33.6% September 2019) (a) 400
(a) 50 (b) 340
Q2. Raghav spends 80% of his (b) 60 (c) 308

l
income. If his income increases (c) 45 (d) 320
by 12% and the savings decrease (d) 75

expenditure ?
ac
by 10%, then what will be the
percentage increase in his

राघव अपनी आय का 80% खच


करता है | यिद उसकी आय 12% से
Q5. In a constituency, 55% of the
total number of voters are males
and the rest are females. If 40%
of the males are illiterate and
Q7. If 60% of a number is 120
more than 20% of the number, then
28% of the number is less than 33
1
3 % of the number by :
यिद िकसी सं ा का 60% उस सं ा
बढ़ जाती है और बचत 10% कम हो 40% of the females are literate, के 20% से 120 अिधक है , तो उस
जाती है , तो उसके य म ितशत then by what percent is the सं ा का 28% उस सं ा के 33 31 %
वृ ात कर | number of literate males more से िकतना कम है ?
nn
SSC CGL TIER II (11 than that of illiterate females ? SSC CGL TIER II (12 September
September 2019) एक िनवाचन े म, कुल मतदाताओं 2019)
(a) 20.5 म से 55% पु ष ह तथा शेष मिहलाएं (a) 14
(b) 16 ह | यिद 40% पु ष अिशि त ह तथा (b) 12
(c) 17.5 40% मिहलाएं िशि त ह, तो िशि त (c) 16
(d) 22 पु षों की सं ा अिशि त मिहलाओं (d) 15
की सं ा से िकतना ितशत अिधक
Q3. The number first increased है ?
Pi

Q8. A is 25% more than B and B


by 16% and then increased by SSC CGL TIER II (11 is 40% less than C. If C is 30%
14%. The number, so obtained, is September 2019) more than D, then by what
now decreased by 30%. What is 8
(a) 22 11 percent is A less than D?
the net increase or decrease (b) 18 92 A, B से 25% अिधक है और B, C से
percent in the original number (c) 22 92 40% कम है | यिद C, D से 30%
(nearest to an integer) ? 2 अिधक है , तो A, D से िकतना
(d) 18 11
एक सं ा पहले 16% से बढ़ा दी ितशत कम है ? SSC CGL TIER
जाती है और िफर 14% बढ़ा दी जाती II (12 September 2019)
Q6. Two-third of the number of
है | इस कार ा सं ा को 30% (a) 1.5
employees of a company are
कम कर िदया जाता है | मूल सं ा म (b) 2.5
males and the rest are females. If
शु ितशत वृ या कमी ( (c) 4
⅜ of the male employees and ⅖
िनकटतम पूणाक तक ) ात कर | (d) 5
of the female employees are
SSC CGL TIER II (11
temporary employees and the
September 2019)

www.ssccglpinnacle.com support@ssccglpinnacle.com Ph. 09729327755, 09817390373 186


Days 24-27 Percentage

s
se
s
la
_c
ob
Q9. In a class, 83 31 % of the तो उसे पेटोल की खरीदी गयी मा ा his wages per hour were
bo
ah को िकतने ितशत से कम करना increased by 20%. By how much
sm
number of students are girls and
ur
yo

the rest are boys. If 60% of the चािहए ? ( िनकटतम पूणाक तक ) percent did his daily earnings
e/
t.m
://

number of boys and 80% of the SSC CGL TIER II (12 increase ?
tp
ht

number of girls are present, then September 2019) बसीर के काय के दै िनक घंटे 15%
what percentage of the total (a) 7 बढ़ गए और उसकी मजदू री म ित
number of students in the class is (b) 6 घंटे 20% की वृ ई | उसकी
absent ? (c) 5 दै िनक आय म िकतने ितशत की
एक क ा म, छा ों की कुल सं ा म (d) 8 वृ ई?
से 83 31 % लड़िकयाँ ह तथा शेष SSC CGL TIER II (13
लड़के ह | यिद 60% लड़के और Q12. Monika spends 72% of her September 2019)
80% लड़िकयाँ उप थत ह, तो क ा income. If her income increases (a) 40
के कुल छा ों का िकतना ितशत by 20% and savings increase by (b) 38
अनुप थित है ? 15%, then her expenditure (c) 35
SSC CGL TIER II (12 increases by : (correct to 1 (d) 36

e
September 2019) decimal place)
(a) 26 32 मोिनका अपनी आय का 72% खच Q15. If 25% of half of x is equal
करती है | यिद उसकी आय 20% बढ़ to 2.5 times the value of 30% of
(b) 22 32
जाए और उसकी बचत 15% बढ़ one-fourth of y, then x is what per
(c) 23 31
जाए, तो उसके य म िकतने की

l
cent more or less than y ?
(d) 12 31 वृ होगी ? ( एक दशमलव थान यिद x के आधे का 25% y के
ac
Q10. A spends 65% of his
income. His income has increased
by 20.1% and his expenditure has
increased by 25%. His savings :
तक सही )
SSC CGL TIER II (13
September 2019)
(a) 20.8%
(b) 20.2%
एक-चौथाई के 30% मान के 2.5 गुना
के बराबर है , तो x, y से िकतना
ितशत अिधक या कम है ?
SSC CGL TIER II (13
September 2019)
A अपनी आय का 65% खच करता (c) 21.9% (a) 33 31 % more
है | उसकी आय 20.1% से बढ़ जाती (d) 19.8% (b) 50% more
है और य म 25% की वृ हो (c) 33 31 % less
nn
जाती है | उसकी बचत : Q13. The price of oil has (d) 50% less
SSC CGL TIER II (12 increased by 20%. However, its
September 2019) consumption decreased by 8 31 %. Q16. A,B and C start a business.
(a) Increase by 11% / 11% बढ़ What is the percentage increase A invests 33 31 % of the total
जाती है | or decrease in the expenditure on capital, B invests 25% of the
(b) Increase by 5%/ 5% बढ़ जाती it ? remaining and C invests the rest.
है | तेल की कीमत 20% बढ़ गयी है | If the total profit at the end of a
(c) Decrease by 5%/ 5% कम हो हालाँ िक, इसकी खपत भी 8 31 से
Pi

year is Rs. 1,62,000, then A’s


जाती है |
कम हो गयी है | इस पर िकये गए share in profit is :
(d) Decrease by 11%/ 11% कम हो
य म ितशत वृ या कमी ात A, B और C ने एक वसाय की
जाती है |
कर | शु आत की | A ने कुल पूँजी का 33
SSC CGL TIER II (13
3 % िनवे श िकया | B ने शे ष का
1
Q11. If the price of petrol
September 2019) 25% िनवेश िकया तथा शेष रािश C
increases by 19% and Sunitha
(a) Increase by 10%/ 10% की वृ के ारा िनवेश की गयी | यिद वष के
intends to spend only an
(b) Increase by 5% 5% की वृ अंत म कुल लाभ 1, 62, 000 पये है ,
additional 12% on petrol, by what
(c) Decrease by 10% / 10% की तो इस लाभ म A का िह ा है :
percent should she reduce the
कमी SSC CGL TIER II (12
quantity of petrol purchased
(d) Decrease by 5% / 5% की कमी September 2019)
(nearest to an integer) ?
यिद पेटोल की कीमत 19% बढ़ गयी (a) Rs. 81,000
Q14. Basir’s working hours per (b) Rs. 60,000
है और सुनीता पेटोल पर केवल 12%
day were increased by 15% and (c) Rs. 54,000
ही अित र खच करना चाहती है ,

www.ssccglpinnacle.com support@ssccglpinnacle.com Ph. 09729327755, 09817390373 187


Days 24-27 Percentage

s
sse
la
_c
ob
(d) Rs. 90,000 SSC CGL 7 June 2019 िकतने ितशत ( एक दशमलव थान
bo
ah (Afternoon) तक सही ) की कमी करनी चािहए ?
sm
ur
yo

Q17. In an examination, A (a) B = 0.675C SSC CGL 10 June 2019


e/
t.m
://

obtained 10% more marks than B, (b) C = 0.72B (Evening)


tp
ht

B obtained 20% more marks than (c) B = 0.72C (a) 6%


C and C obtained 32% less marks (d) C = 0.675B (b) 5.1%
that D. If A obtained 272 more (c) 5.3%
marks than C, then the marks Q3. Surbhi spends 75% of her (d) 5.6%
obtained by B is : income. If her income increases
िकसी परी ा म, A ने B से 10 % by 20% and savings decreases by Q6. The price of sugar has
अिधक अंक हािसल िकये | B ने C से 1%, then the percentage increase increased by 22%. A person
20% अिधक अंक ा िकया और C in her expenditure is : wants to increase his expenditure
ने D से 32% कम अंक ा िकया | सुरिभ अपनी आय का 75% खच by 12% only. By what percent
यिद A को C से 272 अंक अिधक करती है | यिद उसकी आय 20% से should he decrease his
िमले, तो B के ारा ा िकये गए बढ़ जाए और उसकी बचत म 1% consumption, nearest to one
अंक ात कर | की कमी हो जाए, तो उसके यम

e
decimal place?
SSC CGL TIER II (13 ई ितशत वृ ात कर | चीनी की कीमत 22% बढ़ जाती है |
September 2019) SSC CGL 7 June 2019 एक अपने य म केवल 12%
(a) 850 (Evening) की वृ करना चाहता है | उसे
अपनी खपत म िकतने ितशत की

l
(b) 816 (a) 27
(c) 1020 (b) 2.2 कमी ( एक दशमलव थान तक सही
(d) 952
ac
Practice Questions

Q1. Sudha saves 15% of her


(c) 22
(d) 2.7

Q4. The income of A is 40 %


more than that of B. A got a 25%
) करनी चािहए ?
SSC CGL 11 June 2019
(Morning)
(a) 10%
(b) 7.8%
income. If her expenditure rise in his income and B got a (c) 8.2%
increases by 20% and savings 40% rise in his income, then the (d) 8.6%
increases by 60% , then by what percentage increase in the
nn
percent has her income combined incomes of A and B is : Q7. The price of sugar has
increased? A की आय B की आय से 40% increased by 17%. A person
सुधा अपनी आय का 15% बचत अिधक है | A की आय 25% बढ़ गयी wants to increase his expenditure
करती है | यिद उसका य 20% से और B की आय 40% बढ़ गयी, तो A by 8% only. By what percent
बढ़ जाता है तथा बचत म 60% की और B की संयु आय म ितशत should he decrease his
वृ हो जाती है , तो उसकी आय म वृ ात कर | SSC CGL 10 consumption , nearest to one
िकतने ितशत की वृ ई है ? June 2019 (Morning) decimal place?
SSC CGL 6 June 2019 (a) 31.25 चीनी की कीमत 17% से बढ़ जाती है
Pi

(Evening) (b) 34.5 | एक अपना य केवल 8%


(a) 26 (c) 28.25 से बढ़ाना चाहता है | उसे अपनी
(b) 35 (d) 24.5 खपत म िकतने ितशत ( एक
(c) 24 दशमलव थान तक सही ) की कमी
(d) 30 Q5. The price of sugar has करनी चािहए ?
increased by 18%. A person SSC CGL 11 June 2019
Q2. A is 20% less than B and C is wants to increase the expenditure (Afternoon)
20% more than D. If D is 25% by 12% only. By what percent, (a) 7.9%
less than A , then which of the correct to one decimal place, (b) 8.3%
following is true? / B से 20% कम should he decrease his (c) 8.1%
है और C, D से 20% अिधक है | यिद consumption? (d) 7.7%
D, A से 25% कम है , तो िन म से चीनी की कीमत 18% बढ़ जाती है |
कौन सा सही है ? एक अपना य केवल 12% Q8. The price of sugar is
बढ़ाना चाहता है | उसे अपनी खपत म increased by 21%. A person

www.ssccglpinnacle.com support@ssccglpinnacle.com Ph. 09729327755, 09817390373 188


Days 24-27 Percentage

s
sse
la
_c
ob
wants to increase his expenditure (b) 7.1 SSC CGL 13 June 2019
bo
ah
by 12% only. By what percent (c) 6.9 (Afternoon)
sm
ur
yo

should he decrease his (d) 7.5 (a)10.3


e/
t.m
://

consumption , nearest to one (b)10.7


tp
ht

decimal place? Q11. The price of sugar has (c)10.9


चीनी की कीमत 21% से बढ़ जाती है increased by 24%. A person (d)9.9
| एक अपना य केवल 12% wants to increase his expenditure
बढ़ाना चाहता है | उसे अपनी खपत by 18% only. By approximately Q14. A man spends 72% of his
म िकतने ितशत ( एक दशमलव what percent should he decrease income. If his income increased
थान तक सही ) की कमी करनी his consumption? by 28% and his expenditure is
चािहए ? SSC CGL 11 June 2019 चीनी की कीमत 24% बढ़ जाती है | increased by 25%, then what is
(Evening) एक अपना य केवल 18% the percentage increase or
(a) 7.4% बढ़ाना चाहता है | उसे अपनी खपत decrease in his savings (correct to
(b) 7.2% म लगभग िकतने ितशत की कमी one decimal place)?
(c) 7.6% करनी चािहए ? SSC CGL 12 June एक अपनी आय का 72%
खच करता है | यिद उसकी आय

e
(d) 7.8% 2019 (Evening)
(a) 4.6% 28% से बढ़ जाती है और उसका
Q9. The price of sugar is (b) 5.1% य 25% से बढ़ जाता है , तो उसकी
increased by 17%. A person (c) 4.8% बचत म ितशत वृ या कमी ( एक
दशमलव थान तक सही ) ात कर |

l
wants to increase his expenditure (d) 5.3%
by 7% only. By what percentage SSC CHSL 1 July 2019

should
consumption?
ac
is correct to one decimal place,
he reduce

चीनी की कीमत 17% से बढ़ जाती है


| एक अपना य केवल 7%
his
Q12. The price of petrol has
increased by 28%. A person
wants to increase his expenditure
by 22% only. By approximately
what percent should decrease his
(Evening)
(a) 26.9% decrease
(b) 38.4% increase
(c) 35.7% increase
(d) 26.3% decrease
बढ़ाना चाहता है | उसे अपनी खपत consumption?
म िकतने ितशत ( एक दशमलव पेटोल की कीमत 28% से बढ़ जाती Q15. A is 20% more than B, B is
थान तक सही ) की कमी करनी है | एक अपना य केवल 25% more than C, C is 60% less
nn
चािहए ? SSC CGL 12 June 2019 22% से बढ़ाना चाहता है | उसे अपनी than D and D is 20% more than
(Morning) खपत म लगभग िकतने ितशत की E. Based on the above
(a) 8.1% कमी करनी चािहए ? information which of the
(b) 8.3% SSC CGL 13 June 2019 following is true?
(c) 8.5% (Morning) A, B से 20% अिधक है , B, C से
(d) 8.7% (a)5.3% 25% अिधक है , C, D से 60% कम है
(b)4.7% और D, E से 20% अिधक है | इस
Q10. The price of sugar is (c)4.9% जानकारी के आधार पर िन म से
Pi

increased by 24%. A person (d)5.1% कौन सा सही है ?


wants to increase his expenditure SSC CHSL 2 July 2019
by 15% only. By what Q13.The price of sugar has (Morning)
percentage, correct to one increased by 17%. A person (a) D is 60% less than B/ D, B से
decimal place, should he reduce wants to increase his expenditure 60% कम है |
his consumption? by 5% only. By approximately (b) E is 28% more than A/ E, A से
चीनी की कीमत 24% से बढ़ जाती है what percent should he decrease 28% अिधक है |
| एक अपना य केवल 15% his consumption? (c) A is 40% less than D/ A, D से
से बढ़ाना चाहता है | उसे अपनी चीनी की कीमत 17% से बढ़ जाती है 40% कम है |
खपत म िकतने ितशत ( एक | एक अपना य केवल 5% (d) C is 24% less than A/ C, A से
दशमलव थान तक सही ) की कमी से बढ़ाना चाहता है | उसे अपनी 24% कम है |
करनी चािहए ? SSC CGL 12 June खपत म लगभग िकतने ितशत की
2019 (Afternoon) कमी करनी चािहए ? Q16. In a constituency, 40% of
(a) 7.3 the voters are senior citizens.

www.ssccglpinnacle.com support@ssccglpinnacle.com Ph. 09729327755, 09817390373 189


Days 24-27 Percentage

s
se
s
la
_c
ob
40% of the senior citizen voters SSC CHSL 3 July 2019 the net increase/decrease percent
bo
ah
are illiterates and 25% of the (Morning) in the number (correct to the
sm
ur
yo

non-senior citizen voters are (a) Increased by 5%/ 5% से बढ़ nearest integer) ?


e/
t.m

जाती है | एक सं ा को 30% कम कर िदया


://

literates. By what percentage is


tp
ht

the number of literate senior (b) Decreased by 30%/ 30% कम जाता है , िफर 30% बढ़ा िदया जाता
citizens voters less than that of हो जाती है | है , एवं पुनः 30% बढ़ा िदया जाता है |
illiterate non-senior citizen voters (c) Decreased by 5%/ 5% कम हो सं ा म ई शु ितशत वृ या
? जाती है | कमी ( िनकटतम पूणाक तक सही )
एक िनवाचन े म, 40% मतदाता (d) Increased by 50%/ 50% बढ़ ात कर |
व र नाग रक ह | 40% व र जाती है | SSC CHSL 5 July 2019
नाग रक मतदाता अिशि त ह तथा (Morning)
25% गैर-व र नाग रक मतदाता Q19. The price of a commodity is (a) 18% increase/ 18% वृ
िशि त ह | िनर र गैर-व र increased by 36% and the (b) 18% decrease / 18% कमी
नाग रक मतदाताओं की तुलना म quantity purchased is decreased (c) 19% decrease/ 19% कमी
सा र व र व र मतदाताओं की by 30%. What is the percentage (d) 19% increase/ 19% वृ
सं ा िकतने ितशत कम है ?

e
increase/decrease in the amount
SSC CHSL 2 July 2019 spent on the commodity? Q22. A number is increased by
(Afternoon) एक व ु की कीमत 36% बढ़ जाती 30%, then decreased by 30%,
(a) 40 है और खरीदी गयी मा ा 30% कम then further decreased by 30%.
हो जाती है | इस व ु पर खच की

l
(b) 48 31 What is the net increase/decrease
(c) 50 गयी रािश म ितशत वृ या कमी percent in the number (correct to
(d) 46 32
ac
Q17. Anu spends 90% of her
income. If her expenditure
increases by 25% and savings
ात कर |
SSC CHSL 3 July 2019
(Afternoon)
(a) 6%, increase/ 6% वृ
(b) 4.8%, increase/ 4.8% वृ
the nearest integer) ?
एक सं ा को 30% बढ़ा िदया जाता
है , िफर 30% कम कर िदया जाता है ,
और पुनः 30% कम कर िदया जाता
है | सं ा म शु ितशत वृ या
increases by 30%, then by what (c) 6%, decrease/ 6% कमी कमी ( िनकटतम पूणाक तक सही )
percent does her salary increase ? (d) 4.8%, decrease/ 4.8 % कमी ात कर |
अनु अपनी आय का 90% भाग खच SSC CHSL 5 July 2019
nn
करती है | यिद उसका य 25% से Q20. A number is decreased by (Afternoon)
और उसकी बचत 30% बढ़ जाती है , 30%, then increased by 30%, then (a) 40% increase/ 40% वृ
तो उसके वेतन म िकतने ितशत की further decreased by 10%. What (b) 36% decrease/ 36% कमी
वृ ई है ? is the net increase / decrease (c) 36% increase/ 36% वृ
SSC CHSL 2 July 2019 percent in the number (correct to (d) 40% decrease/ 40% कमी
(Evening) the nearest integer) ?
(a) 25.5% एक सं ा को 30% कम कर िदया Q23. The sum of the salaries of A
जाता है , िफर 30% बढ़ा िदया जाता and B together is Rs. 43000. A
Pi

(b) 24%
(c) 22.5% है , और इसके बाद पुनः 10% कम spends 95% of his salary and B
(d) 20 कर िदया जाता है | सं ा म शु spends 80% of his salary. If now
ितशत वृ / कमी ( िनकटतम their savings are the same, what is
Q18. Sudha spends 80% of her पूणाक तक सही ) ात कर | B’s salary (in Rs.) ?
income. When her income is SSC CHSL 4 July 2019 A और B के वेतन का योग 43000
increased by 30%, She increases (Evening) पये है | A अपनी आय का 95%
her expenditure by 25%. Her (a) 18% increase/ 18% वृ तथा B अपनी आय का 80% भाग
savings are : (b) 19% decrease/ 19% कमी खच कर दे ता है | यिद उनकी बचत
सुधा अपनी आय का 80% खच (c) 19% increase/ 19% वृ समान है , तो B का वेतन ात कर |
करती है | जब उसकी आय 30% बढ़ (d) 18% decrease/ 18% कमी SSC CHSL 8 July 2019
जाती है , तो वह अपना य 25% से (Afternoon)
बढ़ा लेती है | उसकी बचत : Q21. A number is decreased by (a) 8000
30%, then increased by 30%, then (b) 34400
further increased by 30%. What is (c) 10600

www.ssccglpinnacle.com support@ssccglpinnacle.com Ph. 09729327755, 09817390373 190


Days 24-27 Percentage

s
sse
la
_c
ob
(d) 8600 SSC CHSL 9 July 2019 level of expenses on sugar, a
bo
ah (Morning) person reduced the consumption
sm
ur
yo

Q24. Two students, A and B, (a) 18,000 of sugar by 4% and also increased
e/
t.m
://

appeared for an examination. A (b) 12,600 his expenditure on sugar by x%.


tp
ht

secured 8 marks more than B and (c) 15,000 The value of x is :


the marks of the former was 60% (d) 30,000 चीनी की कीमत 25% से बढ़ गयी है |
of the sum of their marks. The चीनी पर समान र के य को
sum of the marks obtained by A Q27. The sum of the salaries of A बनाए रखने के िलए एक ने
and B is : and B is Rs. 42,000. A spends अपनी खपत म 4% की कमी कर ली
दो छा , A और B एक परी ा म 75% of his salary and B spends और चीनी पर अपना य भी x % से
शािमल ए | A को B से 8 अंक 90% of his salary. Now their बढ़ा िलया | x का मान है :
अिधक िमले और A का अंक उनके savings are the same. What is A’s SSC CHSL 10 July 2019
अंकों के योग का 60% था | A और B salary (in Rs) ? (Afternoon)
ारा ा िकये गए अंकों का जोड़ है A और B के वेतन का योग 42,000 (a) 19.5
: पये है | A अपनी आय का 75% (b) 21
तथा B अपनी आय का 90% खच

e
SSC CHSL 8 July 2019 (c) 20
(Afternoon) कर दे ता है | उनकी बचत समान है | (d) 18.75
(a) 45 A का वेतन ( पये म ) ात कर |
(b) 50 SSC CHSL 9 July 2019 Q30. The price of petrol is

l
(c) 40 (Afternoon) increased by 24%. A person
(d) 75 (a) 18,000 wants to increase his expenditure
ac
Q25. The sum of the salaries of A
and B together is Rs. 45000. A
spends 85% of his salary and B,
70% of his salary. If now their
(b) 12,000
(c) 30,000
(d) 15,000

Q28. Price of tea has increased by


by 14% only. By what percentage
(correct to one decimal place),
should he decrease
consumption in order to maintain
the same level of expenditure ?
his

savings are the same, what is B’s 20% but I have decided to पेटोल की कीमत 24% से बढ़ गयी है
salary (in Rs) ? / A और B के वेतन increase my expenditure towards | एक अपना य केवल 14%
का कुल योग 45000 पये है | A tea by 15% only. By what बढ़ाना चाहता है | उसे अपनी खपत
nn
अपनी आय का 85% तथा B अपनी percentage should I reduce my म िकतने ितशत ( एक दशमलव
आय का 70% भाग खच करता है | consumption (correct to one place थान तक सही ) की कमी करनी
यिद उनकी बचत समान है , तो B का of decimal) in order to be able to चािहए तािक समान र का य
वेतन ( पये म ) ात कर | maintain the same level of बना रहे ?
SSC CHSL 8 July 2019 expense towards tea ?vv SSC CHSL 11 July 2019
(Evening) चाय की कीमत 20% से बढ़ गयी है (Evening)
(a) 30,000 लेिकन मने चाय पर अपना य (a) 8.3
(b) 18,000 केवल 15% बढ़ाने का िनणय िलया है (b) 7.9
Pi

(c) 12,600 | चाय पर समान र के य को (c) 8.1


(d) 15,000 बनाए रखने के िलए मुझे अपनी खपत (d) 7.8
म िकतने ितशत की कमी ( एक
Q26. The sum of the salaries of A दशमलव थान तक सही ) करनी Q 31. A saves 12% of her
and B is Rs. 42,000. A spends चािहए ? income. If she spends Rs.
75% of his salary and B spends SSC CHSL 10 July 2019 2,16,128 her total income is :
90% of his salary. Now their (Morning) A अपनी आय का 12% धन संिचत
savings are the same. What is B’s (a) 5.4 करता है । यिद वह 2,16,128 पये
salary (in Rs) ? (b) 4.2 खच करती है तो उसकी कुल आय
A और B के वेतन का योग 42000 (c) 5.6 िकतनी है ?
पये है | A अपने वेतन का 75% (d) 4.8 SSC CPO 16 March 2019
तथा B अपने वेतन का 90% खच (Morning)
करता है | अब उनकी बचत समान है Q29. The price of sugar got raised (a) Rs. 2,45,600
| B का वेतन ात कर | by 25%. To maintain the same (b) Rs.2,43,560

www.ssccglpinnacle.com support@ssccglpinnacle.com Ph. 09729327755, 09817390373 191


Days 24-27 Percentage

s
sse
la
_c
ob
(c) Rs.2,48,000 (d) 5.5%
bo
ah
(d) Rs.2,42,063 Q38. The price of sugar has
sm
ur
yo

Q35. The price of sugar has increased by 18%. By what


e/
t.m
://

Q 32. The price of sugar is decreased by 15%. By what percentage can a person decrease
tp
ht

decreased by 10%. By percent percentage can a person increase the consumption so that, there is
can a person increase the the consumption so that there is no change in the expenditure? (
consumption so that there is no no change in the expenditure? correct to one decimal place)
change in the expenditure? चीनी की कीमत 15% कम हो गयी है चीनी की कीमत म 18% की वृ हो
चीनी की कीमत 10% से कम हो | एक को अपनी खपत म गयी है | एक को अपनी खपत
जाती है | एक अपनी खपत म िकतनी वृ करनी चािहए तािक िकतने ितशत से कम करनी चािहए
िकतने ितशत की वृ कर सकता य म कोई प रवतन ना हो ? तािक य म कोई भी प रवतन नहीं
है तािक य म कोई प रवतन ना SSC CPO 12 March 2019 हो ? (एक दशमलव थान तक ल )
आये ? (Morning) SSC CPO 13 March 2019
SSC CPO 12 March 2019 (a) 300
23 % (Morning)
(Evening) (b) 300 % (a) 15.9%
17

e
(a) 100
11 % (c) 50 % (b) 15.7%
3
(b) 109 % 20 % (c) 15.5%
11 (d) 3
(c) 100 % (d) 15.3%
9
(d) 10 % Q36. If A’s income is 40% more

l
Q39. In an examination, 53% of
than the income of B, then what
the candidates failed in science
Q33. If the income of A is 24%
ac percentage of B’s income is less
and 48% failed in mathematics. If
less than income of B, then what than income of A?
40% failed in both subjects, what
percentage is B’s income more यिद A की आय B की आय से 40%
percentage passed in both
than that of A ?/ यिद A की आय B अिधक है , तो B की आय A की आय
subjects?
की आय से 24% कम है , तो B की से िकतना ितशत कम है ? SSC
एक परी ा म 53% छा िव ान म
आय A की आय से िकतना ितशत CPO 12 March 2019 (Morning)
अनु ीण ए तथा 48% गिणत म
अिधक है ? (a) 27 74 %
अनु ीण रह गए | यिद 40% दोनों
SSC CPO 13 March 2019 (b) 28 75 % िवषयों म अनु ीण रहे , तो दोनों
nn
(Evening)
(c) 27 75 % िवषयों म िकतने ितशत उ ीण ए
(a) 600
31 %
600
(d) 28 74 % ?
(b) %
17 SSC CPO 13 March 2019
(c) 600 %
29 Q37. In an examination, 54% of (Morning)
(d) 600 %
19 the candidates passed in science (a) 51%
and 42% failed in mathematics. If (b) 39%
Q34. The price of sugar has 32% failed in both subjects, what (c) 49%
increased by 14%. By what percentage passed in both (d) 43%
Pi

percentage can a person decrease subjects?


the consumption so that there is एक परी ा म 54% छा िव ान म Q40. If the income of A is 40%
an increase in the expenditure by उ ीण ए तथा 42% गिणत म more than the income of B, then
8% only? (correct to the decimal अनु ीण रह गए | यिद 32% दोनों by what percentage is B’s income
place) िवषयों म अनु ीण रहे , तो दोनों less than that income of A?
चीनी की कीमत 14% से बढ़ गयी है | िवषयों म िकतने ितशत उ ीण ए यिद A की आय B की आय से 40%
एक अपनी खपत म िकतने ? अिधक है , तो B की आय A की आय
ितशत की कमी कर सकता है तािक SSC CPO 12 March 2019 से िकतना ितशत कम है ?
य म केवल 8% की वृ ही हो ? (Morning) SSC CPO 13 March 2019
SSC CPO 13 March 2019 (a) 56% (Morning)
(Evening) (b) 48% (a) 230
7 %
(a) 5.9% (c) 32% (b) 56%
(b) 5.3% (d) 44% (c) 25%
(c) 5.7%

www.ssccglpinnacle.com support@ssccglpinnacle.com Ph. 09729327755, 09817390373 192


Days 24-27 Percentage

s
sse
la
_c
ob
(d) 200 % करनी चािहए तािक इस पर िकये गए Science. What should be her
bo
7
ah य म वृ ना हो ? score in social science so that he
sm
ur
yo

Q41. A number which, when SSC CPO 15 March 2019 gets a total of 68%?
e/
t.m

िनिध ने गिणत म 74, अं ेजी म 62,


://

increased by 16% becomes 1914. (Morning)


tp
ht

The number is:


2 %
(a)9 11 िव ानं म 70 अंक ा िकये |
िकसी सं ा म 16% की वृ करने 3 %
(b)15 13 सामािजक िव ानं म उसका ोर
पर यह 1914 हो जाती है | यह सं ा 1 %
(c)13 23 ा होना चािहए तािक उसे कुल 68
है - % ा हो ?
(d)6 87 %
SSC CPO 14 March 2019 SSC CPO 14 March 2019
(Morning) (Evening)
Q45.What percent of 2.4 m is 3.2
(a)2220 (a) 67
cm?
(b)1650 (b) 65
3.2 सेमी 2.4 मी का िकतना ितशत
(c)2010 (c) 66
है ?
(d)1780 (d) 68
SSC CPO 16 March 2019
(Afternoon)

e
Q42. A number is first decreased Q49. What is 34% of 1.2
(a)75%
by 10% and then increased by kilometers?
(b)7.5%
10%. The number so obtained is 1.2 िकलोमीटर का 34% ा है ?
(c)1000 31 %
100 less than the original number. SSC CPO 15 March 2019
(d)1 31 %

l
The original number is: (Evening)
िकसी सं ा म पहले 10% की कमी (a)40,800 cm

इस कार ा
ac
तथा िफर 10% की वृ की जाती है |
सं ा वा िवक
सं ा से 100 कम है | वा िवक
सं ा ात कर |
SSC CPO 16 March 2019
Q46.3cm is how much percent of
12 m.
12m म 3 cm िकतने ितशत है
SSC CPO 14 March 2019
(Evening)
(b)4,08,000 cm
(c)4,080 cm
(d)408 cm

Q50.Salary of A increased by 8%
(Evening) (a) 0.025 in the year 2015 as compared to
(a)100000 (b) 0.25 2014 and decreased by 6% in the
(b)100 (c) 2.5 year 2016 as compared to 2015. If
nn
(c)1000 (d) 25 his salary was Rs.2,34,778 in
(d)10000 2016, his salary (round off to
Q47. A spends 24% of his nearest whole number) in 2014
Q43. 8% of 5 litres is : monthly income on household was :
5 लीटर का 8% है : expenses, 16% on entertainment, A के वेतन म वष 2014 के मुकाबले
SSC CPO 15 March 2019 12% on education, and saves the वष 2015 म 8% की वृ ई और
(Morning) rest. If the savings are 3288, then 2015 के मुकाबले वष 2016 म 6%
(a) 0.4 ml what is the monthly income of A? की कमी ई | यिद वष 2016 म
Pi

(b) 400 ml A अपनी मािसक आय से 24% घरे लू उसका वेतन 2, 34, 778 था तो
(c) 40 ml खच पर , 16% मनोरं जन पर ,12% 2014 म उसका वेतन (िनकटतम
(d) 4 ml िश ा पर य करता है और बािक संपूण सं ा म पूणािकत) िकतना था
बचा लेता है | यिद बचत 3288 है तो ?
Q44. The price of petrol was A की मािसक आय िकतनी होगी ? SSC CPO 15 March 2019
raised by 15%. By how much SSC CPO 14 March 2019 (Evening)
percentage should a motorist (Evening) (a) Rs 2,31,263
reduce the consumption of petrol (a) Rs 6,654 (b) Rs 2,38,347
so that the expenditure on it does (b) Rs 6,480 (c) Rs 2,34,987
not increase? पेटोल की कीमत (c) Rs 6,323 (d) Rs 2,36,402
15% से बढ़ गयी | एक वाहन मािलक (d) Rs 6,850
को पेटोल की खपत िकतनी कम
Q48. Nidhi received 74 marks in
Mathematics in 62 English and 70

www.ssccglpinnacle.com support@ssccglpinnacle.com Ph. 09729327755, 09817390373 193


Days 24-27 Percentage

s
sse
la
_c
ob
9 converted to percentage
Q51. 40 A, B से 20% अिधक है , जो C से (d) 6.14%
bo
ah 25% कम है | A और C के बारे म
sm
is: / 9 को ितशत म बदलने पर
ur

40
yo

_____ ा होगा | िन म से ा सही है ? Q7. 26% of A is 832. What is


e/
t.m
://

SSC MTS 2 August 2019 31% of A ?


tp

SSC CPO 15 March 2019


ht

(Evening) (Evening) A का 26% = 832 है | A के 31% का


(a)2 (a) A = 0.9 C मान िकतना है ?
(b)2 21 (b) A = 1.1 C SSC MTS 6 August 2019
(c) A = 0.95 C (Afternoon)
(c)22 21
(d) A = C (a) 968
(d)22
(b) 876
Q4. A number is first increased by (c) 854
SSC MTS
16 32 % and then decreased by (d) 992
15% to get 238. What is 37.5% of
Q1. A number is first increased
that number ? Q8. A number is first increased
by 20% and then reduced by
एक सं ा को पहले 16 32 % बढ़ाया by 40% and then it is increased
15%. If the final value is 2040,

e
जाता है और िफर 15% कम कर by 30%. What is the net
then what is the initial value of
िदया जाता है िजस से 238 ा होता percentage increase?
the numbers ?
है | इस सं ा का 37.5% ात कर | एक सं ा को पहले 40% बढ़ाया
एक सं ा को पहले 20% से बढ़ाया
SSC MTS 5 August 2019 जाता है और िफर इसे 30% बढ़ा
जाता है िफर 15% से कम िकया
िदया जाता है | शु ितशत वृ

l
जाता है | यिद अंितम मान 2040 है , (Morning)
(a) 150 ात कर |
तो सं ाओं का आरं िभक मान
ac ा
(b) 75 SSC MTS 6 August 2019
था ?
(c) 120 (Evening)
SSC MTS 2 August 2019
(d) 90 (a) 82%
(Morning)
(b) 96%
(a) 2100
Q5. A is 20% more than B. B is (c) 72%
(b) 1800
25% more than C. What percent (d) 70%
(c) 2000
(d) 1900 C is less than A ?
A, B से 20% अिधक है | B,C से Q9. 320 is how much percentage
nn
25% अिधक है | C, A से िकतने less than 400?
Q2. A person spends 10% of his
ितशत कम है ? 320, 400 से िकतना ितशत कम है
salary on food. He spends 20% of
SSC MTS 5 August 2019 ?
the remaining amount on fuel. If
(Evening) SSC MTS 7 August 2019
he has Rs. 4680 now, then what is
(a) 33.33% (Afternoon)
his salary ?
(b) 37.5% (a) 20%
एक अपने वेतन का 10%
(c) 50% (b) 18%
भोजन पर खच करता है | वह शेष
(c) 12%
Pi

भाग का 20% ईंधन पर खच करता है (d) 66.66%


(d) 15%
| यिद उसके पास 4680 पए शेष
है , तो उसका वेतन िकतना है ? Q6. A is 15% more than B. B is
what percent less than A ? ( Q10. If A is 200% more than B,
SSC MTS 2 August 2019
correct to the two decimal points then B is how much percentage
(Afternoon)
) less than A?
(a) 6000
A, B से 15% अिधक है | B, A से यिद A, B से 200% अिधक है , तो B,
(b) 5000
िकतने ितशत कम है (दशमलव के A से िकतना ितशत कम है ?
(c) 6500
दो थान तक ) ? SSC MTS 7 August 2019
(d) 5500
SSC MTS 6 August 2019 (Evening)
(Morning) (a) 33.33%
Q3. A is 20% more than B, which
(a) 9.17 % (b) 50%
is 25% less than C. Which of the
(b) 16.14 % (c) 100%
following is true about A and C ?
(c) 13.04 % (d) 66.67%

www.ssccglpinnacle.com support@ssccglpinnacle.com Ph. 09729327755, 09817390373 194


Days 24-27 Percentage

s
sse
la
_c
ob
Q11. Manish’s salary is half of Q14. Aman’s annual income has SSC MTS 13 August 2019
bo
ah
Ravi’s salary. Ravi’s salary is increased by Rs 20 lakhs but the (Afternoon)
sm
ur
yo

how much percentage more than tax on income that he has to pay (a) 80
e/
t.m
://

Manish’s Salary? has reduced from 20% to 16%. (b) 100


tp
ht

मनीष का वेतन रिव के वेतन का He now pays the same amount of (c) 90
आधा है | रिव का वेतन मनीष के tax as earlier. What is his new (d) 60
वेतन से िकतना ितशत अिधक है ? total income (in Rs lakhs)?
SSC MTS 8 August 2019 अमन की वािषक आय 20 लाख Q18. What is the difference
(Afternoon) पये से बढ़ गयी है लेिकन आय पर between 0.9 and 0.9% ?
(a) 100% उसके ारा िदया जाने वाला कर 0.9 और 0.9% के बीच ा अंतर है
(b) 25% 20% से कम हो कर 16% हो गया है | ?
(c) 50% वह अभी भी पहले िजतनी रािश ही SSC MTS 13 August 2019
(d) 75% कर दे ता है | नयी कुल आय ( लाख (Evening)
पये म ) ात कर | (a) 0.981
Q12. Rahul’s salary is 40% less SSC MTS 9 August 2019 (b) 0.891

e
than Rakesh’s salary. Deepak’s (Morning) (c) 0.198
salary is 80% more than Rahul’s (a) 100 (d) 8.91
salary. If Deepak’s salary is Rs (b) 120
34560, then what is the salary of (c) 80 Q19. A number is 50% less than

l
Rakesh? (d) 60 X and another number is 20%
रा ल का वेतन राकेश के वेतन से less than X. What is the ratio of
ac
40% कम है | दीपक का वेतन रा ल
के वेतन से 80% अिधक है | यिद
दीपक का वेतन 34560 पये है , तो
राकेश का वेतन ात कर |
SSC MTS 8 August 2019
Q15. If 40% of x equal 50% of y,
then y : x is:
यिद x का 40% y के 50% के बराबर
है , तो y : x है :
SSC MTS 9 August 2019
both numbers ?
कोई सं ा X से 50% कम है और
अ सं ा X से 20% कम है | दोनों
सं ाओं का अनुपात िकतना है ?
SSC MTS 14 August 2019
(Evening) (Afternoon) (Morning)
(a) Rs 32000 (a) 4 : 5 (a) 5 : 8
(b) Rs 24000 (b) 3 : 2 (b) 2 : 3
nn
(c) Rs 28000 (c) 2 : 3 (c) 3 : 8
(d) Rs 26000 (d) 5 : 4 (d) 3 : 5

Q13. The ratio of the number of Q16. 140 grams is what percent Q20. In an examination, there are
rural and urban workers (includes of 5.6 kg ? 800 boys and 600 girls. 40% boys
men and women only) in an 5.6 kg का िकतना ितशत 140 and 60% girls passed the
office is 3 : 2. If 20% of rural and gram है ? examination. The percentage
25% of urban are women, the SSC MTS 13 August 2019 (correct to two decimal places) of
Pi

percentage of men is: (Morning) failed students from the total


िकसी कायालय म ामीण और शहरी (a) 2.5 students is:
कमचा रयों ( केवल पु ष और (b) 1.8 एक परी ा म 800 लड़के और 600
मिहला शािमल ) की सं ा का (c) 2 लड़िकयाँ ह | 40% लड़के और 60%
अनुपात 3 : 2 है | यिद 20% ामीण (d) 1.5 लड़िकयों ने परी ा पास िकया | कुल
तथा 25% शहरी कमचारी मिहलाएं छा ों म असफल छा ों का ितशत (
ह, तो पु षों का ितशत ात कर | Q17. If 80% of X is 36 more than दो दशमलव थान तक सही ) ात
SSC MTS 9 August 2019 60% of Y and 120% of X is equal कर |
(Morning) to Y, then the difference between SSC MTS 14 August 2019
(a) 67.50% X and Y is: (Evening)
(b) 82.50% यिद X का 80% Y के 60% से 36 (a) 52.34%
(c) 78% अिधक है और X का 120% Y के (b) 50.36%
(d) 58% बराबर है , तो X और Y के बीच अंतर (c) 51.43%
ात कर | (d) 53.57%

www.ssccglpinnacle.com support@ssccglpinnacle.com Ph. 09729327755, 09817390373 195


Days 24-27 Percentage

s
sse
la
_c
ob
were 90% then, find his marks in
bo
ah
Q21. 75% of 260 + 30% of 320 = the fifth subject. Q28. In spite of an increase in
sm
ur
yo

? पां च िवषयों पर एक परी ा आयोिजत price of a commodity by 20% the


e/
t.m

260 का 75% + 320 का 30% = ? की गई जहाँ ेक िवषय के िलए


://

overall expenditure on it
tp
ht

SSC MTS 14 August 2019 अिधकतम अंक 100 थे | X ने 4 increases by 12%. What is the
(Evening) िवषयों म मश: 82, 97, 88 और 91 percentage decrease in the
(a) 301 अंक ा िकए | यिद उसके कुल quantity of commodity consumed
(b) 271 अंक 90% थे, तो पां चव िवषय म ?
(c) 281 उसने िकतने अंक ा िकए ? िकसी व ु की कीमत म 20% वृ
(d) 291 SSC MTS 16 August 2019 के बावजूद इस पर िकया जाने वाला
(Evening) कुल खच 12% बढ़ता है | व ु की
Q22. X and Y together have Rs (a) 92 खपत की मा ा म ितशत कमी ात
1300. If Rs 10 less than (b) 94 कर |
three-fifth of the amount of X is (c) 89 SSC MTS 19 August 2019
equal to half of the amount of Y, (d) 79 (Evening)

e
then how much does X have? (a) 7 31
X और Y के पास िमलाकर 1300 Q25. Two numbers are 80% and (b) 7 21
पये ह | यिद X की रािश के ⅗ से 10 35% respectively greater than the (c) 8
पये कम Y की रािश के आधे के third number. What is the ratio of (d) 6 32
बराबर है , तो X के पास िकतनी रािश

l
two numbers ?
है ? दो सं ाएँ िकसी तीसरी सं ा से
ac Q29. 60% of a number is 168,
SSC MTS 16 August 2019 मश: 80% और 35% बड़ी ह | दो
then what is the number ?
(Morning) सं ाओं का अनुपात िकतना है ?
िकसी सं ा का 60% 168 है | वह
(a) Rs 700 SSC MTS 16 August 2019
सं ा ा है ?
(b) Rs 600 (Evening)
SSC MTS 8 August 2019
(c) Rs 550 (a) 4 : 3
(Morning)
(d) Rs 650 (b) 17 : 6
(a) 280
(c) 8 : 5
(b) 320
Q23. A student scored 32% (d) 8 : 3
nn
(c) 240
marks and failed by 6 marks. If
(d) 200
he had scored 36% then he would Q26. 12.5% of A = 55, What is
have passed by 2 marks. What is the value of A?
Q30. 0.15 % of 33 31 % of 180000
the passing marks ? A का 12.5% = 55 है | A का मान
=?
एक छा ने 32% अंक ा िकए ा है ?
180000 के 33 31 % का 0.15% ात
और 6 अंकों से फेल हो गया रहा | SSC MTS 19 August 2019
यिद उसने 36% अंक ा िकए (Morning) कर |
होते, तो वह 2 अंकों से उ ीण हो (a) 480 SSC MTS 9 August 2019
Pi

गया होता | उ ीण अंक ( ोर) (b) 500 (Evening)


िकतना है ? (c) 440 (a) 9
SSC MTS 16 August 2019 (d) 550 (b) 90
(Afternoon) (c) 150
(a) 66 Q27. What is the value of 12.5% (d) 900
(b) 75 of 30% of 1440 ?
(c) 80 1440 के 30% के 12.5% का मान Q31. The sum of incomes of A
(d) 70 िकतना है ? and B is 165000. A spends 70%
SSC MTS 19 August 2019 of his income and B spends 80%
Q24. An examination was held on (Afternoon) of his income. If the savings of A
five subjects where the maximum (a)64 is 25% more than that of B, then
marks for each subject was 100. (b)44 find the difference between their
X scored 82, 97, 88 and 91 marks (c)50 incomes ?
in four subjects. If his total marks (d)54

www.ssccglpinnacle.com support@ssccglpinnacle.com Ph. 09729327755, 09817390373 196


Days 24-27 Percentage

s
sse
la
_c
ob
A और B की आय का योग 165000 (d) 61908 Q37. In an examination, 40% of
bo
ah
है | A अपनी आय का 70% और B the students who appeared were
sm
ur
yo

अपनी आय का 80% खच करते ह | Q34. There are 780 bananas in a boys and rest were girls. The pass
e/
t.m

यिद A की बचत, B की तुलना म


://

box out of which 130 bananas are percentage of the boys was 60%
tp
ht

25% अिधक ह, तो उनकी आय म rotten and the remaining ones are and overall pass percentage was
िकतना अंतर् ह? of good quality. The percentage ( 56%. What was the pass
SSC MTS 20 August 2019 upto two decimal places ) of good percentage of the girls?
(Morning) quality bananas is : एक परी ा म शािमल होने वाले 10%
(a)Rs. 12000 िकसी ब े म 780 केले ह िजनम से छा लड़के थे तथा शेष लड़िकयाँ थीं
(b)Rs. 18000 130 केले सड़े ए ह और शेष अ ी | लड़कों का उ ीणता ितशत 40%
(c)Rs. 15000 गुणव ा वाले ह | अ ी गुणव ा वाले था और कुल उ ीणता ितशत 56%
(d)Rs. 10000 केलों का ितशत (दशमलव के दो था | लड़िकयों का उ ीणता ितशत
थान तक) है : ात कर |
Q32. Sonu decided to donate 12% SSC MTS 21 August 2019 SSC MTS 22 August 2019
of his monthly income to a (Morning) (Morning)

e
charitable trust. On the day of (a) 71.12% (a) 54 32
donation, he changed his decision (b) 65.35% (b) 54 31
and donated Rs. 2160 which was (c) 83.33% (c) 52 32
equal to 125% of the amount he (d) 53.33%
(d) 53 31

l
intended to donate earlier. What
is the monthly income of Sonu ? Q35. If 40% of ⅖ of a number is

एक धमाथ ट
ac
सोनू ने अपनी मािसक आय का 12%
को दान दे ने का
फैसला िकया | दान दे ने के िदन ,
उसने अपना िनणय बदल िदया और
का 2160 पए का दान िदया , जो
24, then the number is :
यिद िकसी सं ा के 2/5 का 40%
24 है , तो सं ा है :
SSC MTS 21 August 2019
(Afternoon)
Q38. A is 25% more than B while
B is 20% less than C and C is
10% more than D. Which of the
following is not true?
A, B से 25% अिधक है जबिक B, C
से 20% कम है और C, D से 10%
उसके ारा पूव म दान दे ने के फैसले (a) 136
अिधक है | िन म से कौन सा सही
की रािश के 125% के बराबर है | (b) 148
नहीं है ?
सोनू की मािसक आय िकतनी है ? (c) 150
nn
SSC MTS 22 August 2019
SSC MTS 20 August 2019 (d) 154
(Afternoon)
(Afternoon)
(a) B is 22% less than A/ B, A से
(a) Rs. 11200 Q36. The price of a commodity
22% कम है |
(b) Rs. 9600 increases 15% every year. If the
(b) A is 10% more than D/ A, D
(c) Rs. 14400 difference between its prices at
से 10% अिधक है |
(d) Rs. 12800 the end of third and fourth year is
(c) B is 12% less than D/ B, D से
193.20, then which of the
12% कम है |
Q33. One year ago the population following was 90% of its price at
Pi

(d) A=C/ A = C
of a village was 72,000. Due to the end of second year ?
migration it decreases by 8% िकसी व ु का मू ितवष 15%
Q39. If (x + 10)% of 200 is 33 31
every year. Find the current बढ़ जाता है | यिद तीसरे वष और
population of the village. चौथे वष के अंत म उसके मू ों का % more than x % of 180, then
एक साल पहले िकसी गाँ व की अंतर 193.20 पए है , तो दू सरे वष 10% of (x + 20) is what percent
आबादी 72,000 थी | वास के कारण के अंत म उसके मू का 90% िन less than 40% of x?
यह हर साल 8% कम हो जाती है | म से कौन सा था : यिद 200 का (x+10)% 180 के x%
वतमान वष म इस गाँ व की आबादी SSC MTS 21 August 2019 से 33 31 % अिधक है , तो (x+20) का
िकतनी है ? (Evening) 10% x के 40% से िकतना ितशत
SSC MTS 20 August 2019 (a) Rs. 1008 कम है ?
(Evening) (b) Rs. 1120 SSC MTS 22 August 2019
(a) 68138 (c) Rs. 1064 (Evening)
(b) 60940 (d) Rs. 896 (a) 55
(c) 66240 (b) 60

www.ssccglpinnacle.com support@ssccglpinnacle.com Ph. 09729327755, 09817390373 197


Days 24-27 Percentage

s
sse
la
_c
ob
(c) 70 Q43. If x% of y is 150 and y% of (b) 62.5
bo
ah
(d) 65 z is 300, then the relation between (c) 64.5
sm
ur
yo

x and z is : (d) 66.7


e/
t.m

यिद y का x% 150 है और z का y%
://

Q40. If 60% of a number is equal


tp
ht

to 3/7 of another number, then 300 है , तो x और z म ा संबंध है ? Q46. For a number, greater than
what is the ratio of the two SSC CHSL 5 July 2019 one, the difference between itself
numbers ? (Evening) and its reciprocal is 20% of the
यिद िकसी सं ा का 60% एक अ (a) z=x sum of itself and its reciprocal.
सं ा के 3/7 के बराबर है , तो इन दो (b) z=x/3 By how much percentage (nearest
सं ाओं का अनुपात ा है ? (c) z=x/2 to an integer) is the square of the
SSC MTS 14 August 2019 (d) z=2x number less than its cube ?
(Afternoon) एक से बड़ी िकसी सं ा के संदभ म,
(a) 4:7 Q44. Two students, A and B, इसके (सं ा ) तथा इसके ु म
(b) 5:8 appeared for an examination. A के बीच का अंतर इसके तथा इसके
(c) 5:7 secured 8 marks more than B and ु म के योग का 20% है | सं ा
का वग इसके घन से िकतना ितशत

e
(d) 1:2 the marks of the former was 55%
of the sum of their marks. The ( दशमलव के एक थान तक ) कम
Q41. The price of a machine got sum of the marks obtained by A है ?
depreciated by 50% in the 1st and B is: SSC CHSL 11 July 2019
दो छा - A और B एक परी ा म

l
year, 33 31 % in the 2nd year and (Afternoon)
25% in the 3rd year. Overall, by शािमल ए | A ने B से 8 अंक (a) 122

depreciated ?
ac
how much has the machine

एक मशीन की कीमत पहले वष


50%, दू सरे वष 33 31 % और तीसरे
वष 25% कम हो गयी | कुल
अिधक हािसल िकये तथा A के अंक
उनके अंकों के जोड़ का 55% थे | A
और B के ारा ा िकये गए अंक
का जोड़ है :
SSC CHSL 8 July 2019
(b) 18
(c) 81
(d) 33

64 converted to percentage
Q47. 25
िमलाकर मशीन की कीमत म िकतनी (Morning) is :
कमी आई है ? (a) 100 64 को ितशत म बदला जाए तो :
25
(b) 80
nn
SSC MTS 14 August 2019 SSC CPO 16 March 2019
(Afternoon) (c) 75 (Evening)
(a) 66 32 % (d) 90 (a)2.56
(b) 80% (b)25.6
Q45. With reference to a number (c) 0.256
(c) 75%
greater than one, the difference (d)256
(d) 50%
between itself and its reciprocal is
25% of the sum of itself and its Q48. An alloy contains 32%
Q42. If 85% of a number is added
reciprocal. By how much
Pi

to 75, then the result is the copper, 24% nickel and rest zinc.
percentage (correct one decimal How much zinc is present in 12kg
number itself. The number is :
place) is the fourth power of the of the alloy? एक िम धातु म 32%
यिद िकसी सं ा के 85% को 75 म
number greater than its square? तां बा, 24% िनकेल और शेष ज ा है
जोड़ा जाए, तो प रणाम प वह
एक से बड़ी िकसी सं ा के संदभ म, | 12 िकलो ाम िम धातु म िकतना
सं ा खुद आ जाती है | वह सं ा है
इसके (सं ा ) तथा इसके ु म ज ा मौजूद होगा ?
-
के बीच का अंतर इसके तथा इसके SSC CPO 14 March 2019
SSC CGL 13 June 2019
ु म के योग का 25% है | सं ा (Morning)
(Evening)
का चौथा घात ( fourth power ) (a)5.28 kg
(a) 500
इसके वग से िकतना ितशत ( (b)6.72 kg
(b) 200
दशमलव के एक थान तक ) अिधक (c)528 gm
(c) 300
है ? (d)672 gm
(d) 100
SSC CHSL 11 July 2019
(Morning) SSC CGL TIER I
(a) 57.8

www.ssccglpinnacle.com support@ssccglpinnacle.com Ph. 09729327755, 09817390373 198


Days 24-27 Percentage

s
sse
la
_c
ob
(d) 24.2 Q6. In an examination in which
bo
ah
Q1. If the length of a rectangle is the full marks were 500, A scored
sm
ur
yo

increased by 40% and the breadth Q4. The income of A is 60% less 25% more marks than B. B
e/
t.m
://

is decreased by 20%, then the than that of B, and the scored 60% more marks than C
tp
ht

area of the rectangle is increased expenditure of A is equal to 60% and C scored 20% less marks
by x%. The value of x is: of B’s expenditure. If A’s income than D. If A scored 80% marks,
यिद िकसी आयत की लंबाई 40% is equal to 70% of B’s then the percentage of marks
बढ़ा दी जाती है तथा चौड़ाई 20% expenditure, then what is the ratio obtained by D is:
कम कर दी जाती है , तो इस आयत of the saving of A and B? एक परी ा िजसम पूणाक 500 है ,
का े फल x % बढ़ जाता है | x का A की आय B की आय से 60% कम उसम A को B से 25% अिधक अंक
मान है : है , तथा A का य B के य के आए ह | B को C से 60% अिधक
SSC CGL 3 March 2020 60% के बराबर है | यिद A की आय अंक िमले ह तथा C को D से 20%
(Morning) B के य के 70% के बराबर है , तो कम अंक िमले ह | यिद A को 80%
(a) 20 A और B की बचत का अनुपात ा अंक ा ए ह, तो D के ारा ा
(b) 12 है ? अंक का ितशत ात कीिजए

e
(c) 16 SSC CGL 4 March 2020 SSC CGL 5 March 2020
(d) 8 (Afternoon) (Morning)
(a) 3:8 (a) 60%
Q2. If the difference between (b) 5:9 (b) 54%

l
62% and 80% of a number is 198, (c) 4:7 (c) 65%
then the difference between 92% (d) 2:15 (d) 50%
ac
and 56% of the number will be:
यिद एक सं ा के 62% और 80% म
198 का अंतर है , तो उस सं ा के
92% और 56% म िकतना अंतर होगा
?
Q5. A,B and C donate 8%,7%
and 9%, of their salaries
respectively to a charitable trust.
The salaries of A and B are the
Q7. A and B spent 60% and 75%
of their incomes, respectively. If
the savings of A are 20% more
than that of B, then by what
SSC CGL 3 March 2020 same and the difference between percentage is the income of A
(Evening) their donations is ₹259. The total less than the income of B?
(a) 360 donation of A and B is ₹1,185 A तथा B ने अपनी आय का मशः
nn
(b) 1100 more than that of C. The total 60% तथा 75% िह ा खच िकया |
(c) 396 donation of A and C is what यिद A की बचत B की बचत से 20%
(d) 3564 percentage of the total salaries of अिधक है , तो A की आय B की आय
A,B and C? (Correct to one से िकतना ितशत कम है ?
Q3. Sonu saves 15% of her decimal place) SSC CGL 5 March 2020
income. If her income increases A, B तथा C अपने वेतन का मशः (Afternoon)
by 20% and she still saves the 8%, 7% तथा 9% भाग एक पु ाथ (a) decrease by 1.8%
same amount as before, then what ट म दान कर दे ते ह | A और B के (b) increase by 2.8%
Pi

is the percentage increase in her वेतन समान ह तथा उनके दान म (c) increase by 1.8%
expenditure?(correct to one 259 पये का अंतर है | A और B का (d) decrease by 2.8%
decimal place) / कुल दान C के दान से 1,185 पये
सोनू अपनी आय का 15% भाग बचत अिधक है | A और C का कुल दान Q8. The price of sugar is
करती है | यिद उसकी आय 20% बढ़ A, B तथा C के कुल वेतन का increased by 20%. By what
जाती है तथा तब भी वह पहले िजतनी िकतना ितशत है ? percentage must one cut down on
बचत ही कर पाती है , तो उसके य SSC CGL 4 March 2020 the consumption of sugar, so that
no extra amount has to be
म िकतने ितशत की वृ ई है ? ( (Evening)
incurred on sugar?
दशमलव के एक थान तक ) (a) 6.2%
चीनी की कीमत 20% बढ़ गयी है |
SSC CGL 4 March 2020 (b) 5.8% को चीनी की खपत म िकतने
(Morning) (c) 6.4% ितशत की कटौती करनी चािहए,
(a) 22.8 (d) 7.1% तािक चीनी पर ज़रा भी अित र
(b) 23.5 रािश खच ना हो ?
(c) 23.8

www.ssccglpinnacle.com support@ssccglpinnacle.com Ph. 09729327755, 09817390373 199


Days 24-27 Percentage

s
sse
la
_c
ob
SSC CGL 5 March 2020 (b) 2

bo
5
ah
(Evening) Q11. In an examination, Anita
sm
(c) 5
ur

4
(a) 80%
yo

scored 31% marks and failed by


e/

(d) 3
t.m

(b) 20% 4
://

16 marks. Sunita scored 40%


tp

(c) 16 32 %
ht

marks and obtained 56 marks


(d) 83 31 % Q14. Ravi scores 72% marks in
more than those required to pass.
examinations. If these are 360
Find the minimum marks
Q9. The population of a city marks, then the maximum marks
required to pass.
increased 30% in the first year are:
एक परी ा म, अनीता को 31% अंक
and decreased by 15% in the next रिव को परी ा म 72% अंक आए |
िमले तथा वह 16 अंकों से फेल हो
year. If the present population is यिद ये 360 अंक ह, तो अिधकतम
गयी | सुनीता को 40% अंक िमले
11,050 then the population 2 अंक िकतने ह ?
तथा उसके अंक उ ीण होने के िलए
years ago was: SSC CGL 9 March 2020
आव क अंक से 56 अिधक थे |
एक शहर की आबादी पहले साल (Morning)
उ ीण होने के िलए ूनतम िकतने
30% बढ़ी तथा अगले वष 15% कम (a) 500
अंकों की आव कता है ?
हो गयी | यिद वतमान आबादी (b) 350

e
SSC CGL 7 March 2020
11,050 है , तो 2 वष पहले िकतनी (c) 450
(Morning)
आबादी थी ? (d) 400
(a) 3116
SSC CGL 6 March 2020
(b) 3944
(Morning) Q15. The price of cooking oil

l
(c) 7100
(a) 10,000 increased by 25%. Find by how
(d) 264
(b) 10,050 ac much percentage a family must
(c) 99,000 reduce its consumption in order to
Q12. Anu spends 68% of her
(d) 99,500 maintain the same budget? / खाना
monthly income. If her monthly
पकाने के तेल की कीमत म 25% की
income increases by 20% and her
Q10. In a school, 4% of the वृ ई। उसी बजट को बनाए रखने
monthly saving increase by 9 83 %
students did not appear for the के िलए एक प रवार को अपनी खपत
then the percentage increase in म िकतने ितशत की कमी करनी
annual exams. 10% of the
her monthly expenditure is: चािहए?
students who appeared for the
अनु अपनी मािसक आय का 68%
nn
exams could not pass the exam. SSC CGL 9 March 2020
खच करती है | यिद उसकी मािसक (Evening)
Out of the remaining students,
आय 20% बढ़ जाती है तथा उसकी (a) 70%
50% got distinction marks and
मािसक बचत 9 83 % बढ़ जाती है , तो (b) 80%
432 students passed the exam but
could not get distinction marks. उसके मािसक य म िकतने (c) 30%
The total number of students in ितशत की वृ होगी ? (d) 20%
the school is: SSC CGL 7 March 2020 SSC CHSL 2019
एक िव ालय म, 4% छा वािषक (Afternoon) Q1. Kavita's attendance in her
Pi

परी ा म शािमल नहीं ए | परी ा म (a) 20% school for the academic session
शािमल होने वाले 10% छा परी ा (b) 25% 2018-2019 was 216 days. On
पास नहीं कर सके | शेष छा ों म से, (c) 22% computing her attendance, it was
50% को िड ं न अंक िमले तथा (d) 32% observed that her attendance was
432 छा परी ा तो पास कर गए 90%. The total working days of
लेिकन उ िड ं न अंक नहीं Q13. By what number must the the school were:
िमले | िव ालय म छा ों की कुल given number be multiplied to शै िणक स 2018-2019 म किवता
सं ा िकतनी है ? increase the number by 25%. के िव ालय म उसकी उप थित 216
SSC CGL 6 March 2020 िकसी सं ा म 25% की वृ करने िदन थी। उसकी उप थित की गणना
(Afternoon) के िलए उस सं ा को िकस सं ा म, यह पाया गया िक उसकी
(a) 960 से गुणा करना चािहए ? उप थित 90% थी। िव ालय के कुल
(b) 1000 SSC CGL 7 March 2020 काय िदवसों की गणना कीिजए।
(c) 878 (Evening) CHSL 12-10-2020 (Morning
(d) 1200 (a) 3 shift)

www.ssccglpinnacle.com support@ssccglpinnacle.com Ph. 09729327755, 09817390373 200


Days 24-27 Percentage

s
sse
la
_c
ob
(a) 250 Q5. The length and breadth of a
bo
ah
(b) 194 cuboid are increased by 10% and Q8. Sachin scored 120 runs,
sm
ur
yo

(c) 240 20%, respectively, and its height which included 6 boundaries and
e/
t.m
://

(d) 195 is decreased by 20%. The 4 sixes. What percentage of his


tp
ht

percentage increase in the volume total score did he make by


Q2. Mohan offers to sell his of the cuboid is: running between the wickets?
articles at a discount of 20%, but एक घनाभ की लंबाई और चौड़ाई सिचन ने 120 रन बनाए, िजसम 6
he marks his articles by मश 10% और 20% बढ़ जाती है , चौके और 4 छ े शािमल थे। उसके
increasing the price of each by और इसकी ऊंचाई 20% तक कम हो कुल रनों का िकतना ितशत िवकटों
35%. What percentage would his जाती है । घनाभ के आयतन म के बीच म दौड़ कर आया है ?
gain be? ितशत वृ है : CHSL 14-10-2020 (Evening
मोहन अपनी व ुओं को 20% की CHSL 14-10-2020 (Morning shift)
छूट पर बेचने की पेशकश करता है । shift) (a) 46 94 %
लेिकन उसने अपनी व ुओं का मू (a) 5 54 % (b) 33 ⅓ %
35% अिधक अंिकत िकया है । उसके 1
(b) 5 % (c) 60%
5
लाभ का ितशत ा होगा?

e
2
(c) 5 % (d) 45%
5
CHSL 12-10-2020 (Afternoon 3
(d) 5 %
5
shift) Q9. A student multiplied a
(a) 10% number with 43 instead of 34 .
Q6. A crate of fruits contains one

l
(b) 7% What is the error percentage?
spoiled fruit for every 25 fruits.
(c) 8% एक छा ने 43 की बजाय 34 के साथ
(d) 9%
ac
Q3. If each edge of a cube is
increased by 10%, then the
percentage increase in its surface
60% of the spoiled fruits were
sold. If the seller had sold 48
spoiled fruits, then the number of
fruits in the crate were:
फलों के एक टोकरे म हर 25 फलों
एक सं ा को गुणा िकया। ुिट
ितशत ा है ?
CHSL 15-10-2020 (Afternoon
shift)
(a) 59.67%
area is: पर एक खराब फल है । खराब हो
यिद िकसी घन के ेक िकनारे को चुके फलों के 60% की िब ी हो (b) 43.75%
10% बढ़ा िदया जाता है , तो इसके गयी। यिद िव े ता ने 48 खराब फलों (c) 67.45%
nn
पृ े फल म िकतने ितशत की की िब ी की थी, तो टोकरे म फलों (d) 39.34%
वृ होगी? की सं ा थी:
CHSL 13-10-2020 (Afternoon Q10. Rita's income is 15% less
CHSL 14-10-2020 (Afternoon
shift) than Richa's income. By what
shift)
(a) 21% percent is Richa's income more
(a) 2000
(b) 19% than Rita's income?
(b) 2400
(c) 22% रीता की आय ऋचा की आय से 15%
(d) 20% (c) 3000
(d) 1200 कम है । ऋचा की आय, रीता की आय
से िकतने ितशत अिधक है ?
Pi

Q4. If 40% of a number is less


Q7. Two numbers are CHSL 15-10-2020 (Evening
than its 60% by 30, then the 20%
respectively 25% and 65% more shift)
of that number is: 11 %
than a third number. The ratio of (a) 15 17
यिद िकसी सं ा का 40% उसके 11 %
the two numbers is: (b) 17 17
60% से 30 से कम है , तो उस सं ा
का 20% है : दो सं ाएँ तीसरी सं ा से मशः 11 %
(c) 16 17
CHSL 13-10-2020 (Evening 25% और 65% अिधक ह। दोनों 11 %
(d) 14 17
shift) सं ाओं का अनुपात ा है ?
(a)60 CHSL 14-10-2020 (Evening Q11. What percentage of the
(b)40 shift) numbers from 101 to 1000 have 9
(c)50 (a) 25 : 42 in the unit’s digit?
(d)30 (b) 16 : 17 101 से 1000 तक िकतने ितशत
(c) 16 : 19 सं ाओं म इकाई अंक 9 है ?
(d) 25 : 33

www.ssccglpinnacle.com support@ssccglpinnacle.com Ph. 09729327755, 09817390373 201


Days 24-27 Percentage

s
sse
la
_c
ob
CHSL 19-10-2020 (Morning CHSL 20-10-2020 (Afternoon (d) 15
bo
ah
shift) shift)
sm
ur
yo

(a) 10% (a) 15000 Q.18. The income of Renu is 10%


e/
t.m
://

(b) 12% (b) 12000 less than the income of Sudha,


tp
ht

(c) 20% (c) 8000 and the income of Sudha is 10%


(d) 15% (d) 13000 more than Rs.3000. The income
of Renu is:
Q12. 68 is 25% of which of the Q.15. When 50% of a number A रे णु की आय सुधा की आय से 10%
following numbers? is added to B, the second number कम है और सुधा की आय 3000
68 िन म से िकस सं ा का 25% B increases by 25%. The ratio पये से 10% अिधक है । रे णु की
है ? between the numbers A and B is: आय है :
CHSL 19-10-2020 (Evening जब सं ा A का 50% B म जोड़ CHSL 26-10-2020 (Afternoon
shift) िदया जाता है , तो दू सरी सं ा B shift)
(a) 272 25% बढ़ जाती है । सं ा A और B (a) 3300
(b) 285 के बीच का अनुपात है : (b) 3070

e
(c) 204 CHSL 20-10-2020 (Afternoon (c) 2700
(d) 136 shift) (d) 2970
(a) 3:2
Q13. The difference of two (b) 2:3 Q.19. The difference between two

l
positive numbers is 1020. If 7.6% (c) 1:2 positive numbers is equal to 30%
of the greater number is 12.4% of (d) 3:4 of the greater number. If the
ac
the smaller number, then the sum
of the two numbers is equal to:
दो धना क सं ाओं का अंतर
1020 है । यिद बड़ी सं ा का 7.6%
छोटी सं ा का 12.4% है , तो दोनों
Q.16. Richa invests in mutual
funds a sum of Rs.559968, which
is 19% of her annual income.
What is her monthly income?
smaller number is 28, then the
sum of both the number is:
दो धना क सं ाओं के बीच का
अंतर बड़ी सं ा के 30% के बराबर
है । यिद छोटी सं ा 28 है , तो दोनों
सं ाओं का योग ात कर | रचा ूचुअल फंड म 59968 पये सं ाओं का योग है :
CHSL 20-10-2020 (Morning की रािश का िनवेश करती है , जो CHSL 26-10-2020 (Evening
shift) उसकी वािषक आय का 19% है । shift)
nn
(a) 3250 उसकी मािसक आय ा है ? (a) 72
(b) 4250 CHSL 20-10-2020 (Evening (b) 65
(c) 4520 shift) (c) 68
(d) 3520 (a) 445600 (d) 64
(b) 145600
Q.14. Ravinder invests Rs.3,750 (c) 345600 Q.20. Sachin’s income is 25%
which is equal to 15% of his (d) 245600 more than Dileep’s income. By
monthly salary in a medical how much percentage is Dileep’s
Pi

insurance policy. Later he invests Q.17. A reduction of 20% in the Income less than Sachin’s
25% and 8% of his monthly price of sugar enables a purchaser income?
salary on a child education policy, to obtain 4 kg more for Rs.160. सिचन की आय िदलीप की आय से
and mutual funds, respectively. The original price of sugar per kg 25% अिधक है । िदलीप की आय
The total amount left with him is: is: सिचन की आय से िकतने ितशत
रिवंदर िचिक ा बीमा पॉिलसी म चीनी की कीमत म 20% की कमी कम है ?
3,750 पये का िनवेश करता है जो होने पर एक े ता को 160 पये म 4 CHSL 17-03-2020 (Morning
उसके मािसक वेतन के 15% के िकलो अिधक चीनी िमलती है । ित shift)
बराबर है । बाद म वह अपने मािसक िकलो चीनी का मूल मू ा है ? (a) 15%
वेतन के 25% और 8% को मश CHSL 20-10-2020 (Morning (b) 20%
बाल िश ा नीित और ूचुअल फंड shift) (c) 18%
म िनवेश करता है । उसके पास बची (a) 12 (d) 22%
ई कुल रािश है : (b) 10
(c) 14

www.ssccglpinnacle.com support@ssccglpinnacle.com Ph. 09729327755, 09817390373 202


Days 24-27 Percentage

s
sse
la
_c
ob
Q21. If he length of a rectangle is (b) 20 : 35 CHSL 19-03-2020 (Evening
bo
ah
increased by 12% and the breadth (c) 25 : 32 shift)
sm
ur
yo

is decreased by 8%, the net effect (d) 21 : 31 (a) 10%


e/
t.m
://

on the area is: (b) 15%


tp
ht

यिद आयत की लंबाई 12% बढ़ जाती Q25. In an election, candidate X (c) 18%
है और चौड़ाई 8% कम हो जाती है , got 70% of the overall valid (d) 21%
तो े फल पर शु भाव इस कार votes. If 20% of the overall votes
है : were declared invalid and the SSC CGL 2019 TIER-II
CHSL 18-03-2020 (Morning total number of votes is 640000, Q28. A is 80% more than B and
shift) then find the number of valid C is 48 74 % less than the sum of
(a) increase by 3.04%/ 3.04% की votes polled in favour of the A and B. By what percent is C
वृ candidate. less than A?
(b) increase by 2.6%/ 2.6% की एक चुनाव म, उ ीदवार ए को A, B से 80% अिधक है और C, A
वृ कुल वैध मतों का 70% िमला। यिद और B के योग से 48 74 % कम है । C,
(c) decrease by 3.04%/ 3.04% की कुल मतों म से 20% को अवैध घोिषत A से िकतना ितशत कम है ?
कमी िकया गया और कुल मतों की सं ा

e
CGL 2019 Tier-II (15-11-2020 )
(d) decrease by 2.6%/ 2.6% की 640000 है , तो उ ीदवार के प म (a) 20
कमी मतदान के वैध मतों की सं ा ात (b) 25
कीिजए। (c) 30

l
Q22. If a% of 240 is c% of a is CHSL 18-03-2020 (Evening (d) 15
117.6, then the value of a + c is: shift)

का मान है :
ac
यिद 240 का a% a का c% है , तो a+c

CHSL 18-03-2020 (Morning


shift)
(a) 238
(a) 358400
(b) 450000
(c) 400000
(d) 358000
Q29. If (x+20)% of 250 is 25%
more than x% of 220 then 10% of
(x+50) is what percent less than
15% of x ?
यिद 250 का (x +20)%, 220 के x%
(b) 144 Q.26. A woman earns से 25% अिधक है , तो (x +50) का
(c) 260 Rs.1,000/day. After some weeks, 10%, x के 15% से िकतना ितशत
(d) 196 she earns Rs.1,160/day. By how कम है ?
nn
much percentage did her earnings CGL 2019 Tier-II (15-11-2020 )
Q23. Find x, if 30% of 400 + x% increase? (a) 13 31
of 70 = 25% of 536. एक मिहला 1000 पये ित िदन
(b) 8 31
x ात कीिजए, यिद 70 के 400+x% कमाती है । कुछ ह ों के बाद, वह
(c) 16 32
का 30%= 536 का 25% है । 1,160 पये ित िदन कमाती है ।
CHSL 18-03-2020 (Afternoon उसकी कमाई म िकतने ितशत की (d) 33 31
shift) वृ ई?
(a) 20 CHSL 19-03-2020 (Afternoon Q30. A certain number of
Pi

(b) 30 shift) students from school X appeared


(c) 10 (a) 18% in an examination and 30%
(d) 40 (b) 15% students failed. 150% more
(c) 17% students than those from school
Q24. Two numbers are (d) 16% X, appeared in the same
respectively 25% and 60% more examination from school Y. If
than a third number. The ratio of Q.27. Each side of a rectangular 80% of the total number of
the two numbers is: field is increased by 10%. Then students who appeared from X
दो सं ाएँ तीसरी सं ा से मशः the percentage increase in the and Y passed, then what is the
25% और 60% अिधक ह। दोनों area of the field is: percentage of students who failed
सं ाओं का अनुपात है : एक आयताकार े की ेक भुजा from Y ?
CHSL 18-03-2020 (Evening म 10% की वृ ई है । िफर े के एक परी ा म ू ल X के कुछ छा
shift) े फल म ितशत वृ है : उप थत ए और 30% छा फेल हो
(a) 20 : 30 गए। ू ल X के छा ों की तुलना म

www.ssccglpinnacle.com support@ssccglpinnacle.com Ph. 09729327755, 09817390373 203


Days 24-27 Percentage

s
sse
la
_c
ob
150% अिधक छा , ू ल Y से उसी यिद A, B से 40% कम है , C, A और आय म 29% की वृ होती है , तो
bo
ah
परी ा म उप थत ए। यिद X और B के योग का 40% है , तो B, C से उसकी बचत म वृ होती है :
sm
ur
yo

Y से उप थत होने वाले छा ों की िकतने ितशत अिधक है ? CGL 2019 Tier-II (16-11-2020 )


e/
t.m

कुल सं ा का 80% उ ीण हो गया,


://

CGL 2019 Tier-II (16-11-2020 ) (a) 60%


tp
ht

तो Y से फेल ए छा ों का ितशत (a) 60 (b) 65%


ा है ? (b) 40 81 (c) 55%
CGL 2019 Tier-II (15-11-2020 ) (c) 56 41 (d) 54%
(a) 18 (d) 36
(b) 20 Q37. If A’s income is 60% less
(c) 16 than B’s income,then B’s income
Q34. 25% of (50% of 30% of 150)
40% of 2250 is
(d) 24 is what percentage more than that
Q31. Anuja owns 66 32 % of a equal to: of A’s income?
25% of (50% of 30% of 150)
का मान ा यिद A की आय B की आय से 60%
property. If 30 % of the property 40% of 2250

that she owns is worth ₹1,25,000 होगा कम है , तो B की आय A की आय से


then 45% of the value (in₹) of CGL 2019 Tier-II (16-11-2020 ) िकतने ितशत अिधक है ?

e
the property is. (a) 0.825% CGL 2019 Tier-II (18-11-2020)
अनुजा के पास संपि का 66 32 % (b) 0.25% (a) 40%
(c) 0.625% (b) 80%
िह ा है . यिद उसकी 30 % संपि
(d) 0.225% (c) 120%
की कीमत ₹1,25,000 है तो उसकी

l
(d) 150%
45% संपि की कीमत(₹ म) िकतनी
Q35. The monthly salaries of A
होगी ac and B are the same. A, B and C Q38. The sum of weight of A
CGL 2019 Tier-II (15-11-2020 )
donate 10%, 8% and 9% and B is 80kg, 50% of A’s weight
(a) 2,70,000
respectively, of their monthly is 65 times the weight of B. Find
(b) 2,62,500
salaries to a charitable trust. The the difference between their
(c) 2,81,250
difference between the donation weights.
(d) 2,25,000
of A and B is 400. The total A और B के वजन का योग 80 kg है ,
donation of A and B is 900 more A के वजन का 50% B के वजन का
Q32. Rishu saves x% of her
than that of C. What is the
6 गु ना है । उनके वजन के बीच का
5
nn
income. If her income increases
monthly salary of C? अंतर ात कर।
by 26% and the expenditure
A और B का मािसक वेतन समान CGL 2019 Tier-II (18-11-2020)
increases by 20%, then her
है । A, B और C अपने मािसक वेतन (a) 20kg
savings increase by 50%. What is
का 10%, 8% और 9%, एक धमाथ (b) 10kg
the value of x?
ट को दान करते ह। A और B के (c) 25kg
रशु अपनी आय का x% बचाती है ।
दान के बीच का अंतर 400 है । A (d) 15kg
अगर उसकी आय 26% बढ़ जाती है
और B का कुल दान C की तुलना म
और खच 20% बढ़ जाता है , तो
900 अिधक है । C का मािसक वेतन
Pi

उसकी बचत म 50% की वृ होती Q.39 In an examination, 92% of


ा है the students passed and 480
है । x का मान ा है
CGL 2019 Tier-II (16-11-2020 ) students failed. If so, how many
CGL 2019 Tier-II (15-11-2020 )
(a) ₹25,000 students appeared in the
(a) 30
(b) ₹36,000 examination?
(b) 20
(c) ₹30,000 एक परी ा म, 92% छा उ ीण ए
(c) 10
(d) ₹27,000 और 480 छा असफल रहे । परी ा
(d) 25
म िकतने छा उप थत ए ?
Q36.. Renu saves 20% of her CGL 2019 Tier-II (18-11-2020)
Q33. If A is 40% less than B and
income. If her expenditure (a) 5000
C is 40% of the sum of A and B,
increases by 20% and income (b) 6200
then by what percentage B is
increases by 29%, then her saving (c) 6000
greater than C ?
increased by (d) 5800
रे णु अपनी आय का 20% बचाती है ।
अगर उसका खच 20% बढ़ता है और

www.ssccglpinnacle.com support@ssccglpinnacle.com Ph. 09729327755, 09817390373 204


Days 24-27 Percentage

s
sse
la
_c
ob
Q.40. What is to be added to 15% यिद 110 को x% से घटने पर वही the percentage level of E, but T
bo
ah
of 180 so that the sum is equal to प रणाम आता है जो 50 को x% becomes 30% of E, and C
sm
ur
yo

20% of 360? बढ़ाने से आता है , तो 650 का x% becomes 20% of T. The


e/
t.m

180 के 15% म ा जोड़ा जाना 180 के (x + 20)% से िकतना


://

difference between two savings


tp
ht

चािहए है तािक योग 360 के 20% के ितशत अिधक है ? (in Rs) is:
बराबर हो? (correct to nearest integer) / एक का मािसक वेतन
CGL 2019 Tier-II (18-11-2020) (िनकटतम पूणाक म ) ₹50,000 था। वह प रवार के खच
(a) 40 CPO-2019 23-11-2020 (E), कर (T), दान (C) पर खच
(b) 60 (Morning shift) करता था और बाकी उसकी बचत
(c) 50 (a) 80% थी। E आय का 60% था, T, E का
(d) 45 (b) 90% 20% था, और C, T का 15% था। ।
(c) 136% जब उसका वेतन 40% बढ़ा, तो
SSC CPO 2019 (d) 154% उसने E का ितशत र बनाए रखा,
Q41. If each side of a rectangle is लेिकन T, E का 30% हो गया, और
decreased by 11%, then the area Q44. If A’s salary is 60% more C, T का 20% हो गया। दोनों बचत
के बीच का अंतर ( म) है :

e
will decrease by: than B’s salary, then by what
यिद आयत की ेक भुजा म 11% percentage is B’s salary less than CPO-2019 23-11-2020
की कमी होती है , तो े फल म that of A? (Evening shift)
िकतनी कमी आएगी? यिद A का वेतन B के वेतन से 60% (a) 128
अिधक है , तो B का वेतन, A के वेतन

l
CPO-2019 23-11-2020 (b) 130
(Morning shift) से िकतना ितशत कम है ? (c) 250
(a) 24.31%
(b) 25%
(c) 21.13%
(d) 20.79%
ac CPO-2019
(Morning shift)
(a) 47.7%
(b) 33.3%
(c) 37.5%
23-11-2020 (d) 220

Q47. If the area of a square is


decreased by 19%, then the
diagonal of the square is
Q42. The price of diesel (d) 45% decreased by :
increased by 16%, A person यिद वग का े फल 19% कम हो
wants to increase his expenditure Q45. A person’s salary increased जाता है तो वग का िवकण िकतना
nn
on diesel by 10% only. By what from ₹8,100 to ₹9,000. What is कम हो जाता है |
percentage, correct to one the percentage increase in his CPO-2019 23-11-2020
decimal place, should he reduce salary? (Evening shift)
his consumption? एक का वेतन ₹8,100 से (a) 15%
डीजल की कीमत म 16% की वृ बढ़कर ₹9,100 हो गया। उसके वेतन (b) 10%
ई, एक डीजल पर अपने म ितशत वृ ा है ? (c) 5%
खच को केवल 10% बढ़ाना चाहता CPO-2019 23-11-2020 (d) 12%
है । िकस ितशत तक, एक दशमलव (Evening shift)
Pi

थान तक, उसे अपनी खपत कम (a) 11 91 % Q48. A person’s salary has
करनी चािहए? (b) 13 97 % increased from Rs.7,000 to
CPO-2019 23-11-2020 (c) 9 91 % Rs.12,000. What is the percentage
(Morning shift) increase in his salary ?
(d) 6 91 %
(a) 6.5% एक का वेतन 7,000 पये से
(b) 5.2% बढ़कर 12,000 पये हो गया है ।
Q46. The monthly salary of a
(c) 4.5% उसके वेतन म ितशत वृ ा है ?
person was ₹50,000. He used to
(d) 3.7% CPO-2019 24-11-2020
spend on family expenses(E),
(Morning shift)
Taxes(T), charity(C), and the rest
Q43. If decreasing 110 by x% (a) 76 74 %
were his savings, E was 60% of
gives the same result as (b) 61 71 %
the income, T was 20% of E, and
increasing 50 by x%, then x% of (c) 71 73 %
C was 15% of T. When his salary
650 is what percentage more than
got raised by 40%, he maintained (d) 69 71 %
(x + 20) % of 180?

www.ssccglpinnacle.com support@ssccglpinnacle.com Ph. 09729327755, 09817390373 205


Days 24-27 Percentage

s
sse
la
_c
ob
यिद 110 को x% से घटाने पर 50 को
bo
ah
Q49. A man spends 75% of his Q51. If 25% of 400 + 35% of x% बढ़ाने के समान प रणाम िमलता
sm
ur
yo

income. If his income increases 1260 + 27% of 1800 = 1020 + x, है , तो 650 का x% 780 के (x-10)%
e/
t.m

से िकतना ितशत अिधक (िनकटतम


://

by 28% and his expenditure then the value of x lies between:


tp
ht

increases by 20%, then what is यिद 400 का 25% + 1260 का 35% पूणाक तक सही) है ?
the increase or decrease + 1800 का 27% = 1020 + x है , तो CPO-2019 25-11-2020
percentage in his saving ? x का मान िकसके बीच होगा ? (Morning shift)
एक आदमी अपनी आय का 75% CPO-2019 24-11-2020 (a) 14%
खच करता है । यिद उसकी आय म (Evening shift) (b) 12%
28% की वृ होती है और उसके (a) 16 to/से 20 (c) 17%
य म 20% की वृ होती है , तो (b) 6 to/से 10 (d) 18%
उसकी बचत म वृ या कमी का (c) 11 to/से 15
ितशत ा है ? (d) 0 to/से 5 Q55. If A’s salary is 30% more
CPO-2019 24-11-2020 than B's salary, then by what
(Morning shift) Q52. If 49% of X = Y, then Y% percentage is B’s salary less than
(a) 13% increase/ वृ

e
of 50 is : that of A? (correct to one decimal
(b) 52% decrease/ कमी यिद X का 49% = Y है , तो 50 का place)
(c) 52% increase/ वृ Y% ात करे | यिद A वेतन B वेतन से 30% अिधक
(d) 13% decrease/ कमी CPO-2019 24-11-2020 है , तो B वेतन A के मुकाबले िकतने
ितशत कम है ? (एक दशमलव

l
(Evening shift)
Q50. The monthly salary of a (a) 24.5% of X थान तक)
ac
person was 75,000. He used to
spend on family expenses(E),
Taxes(T), charity(C), and the rest
were his savings, E was 60% of
the income, T was 20% of E, and
(b) 24.5% of Y
(c) 40% of Y
(d) 50% of X

Q53. A number is first increased


CPO-2019
(Morning shift)
(a) 17.5%
(b) 25%
(c) 23.15
25-11-2020

C was 15% of T. When his salary by 40% and then decreased by (d) 19.7%
got raised by 40%, he maintained 25%, again increased by 15%
the percentage level of E, but T and then decreased by 20%, What Q56. The price of diesel is
nn
becomes 30% of E and C is the net increase decrease per increased by 26%, A person
becomes 20% of T. The ratio of cent in the number ? wants to increase the expenditure
the saving of earlier to that of his एक सं ा म पहले 40% की वृ by 15% only. By what
present salary is: ई है और िफर 25% की कमी ई, percentage, correct to one
एक का मािसक वेतन िफर से 15% की वृ ई और िफर decimal place, should he decrease
₹50,000 था। वह प रवार के खच 20% की कमी ई, सं ा म कुल his consumption?
(E), कर (T), दान (C) पर खच वृ या कमी के ितशत को ात डीजल की कीमत म 26% की वृ
करता था और बाकी उसकी बचत करे । ई है , एक केवल 15% तक
Pi

थी। E आय का 60% था, T, E का CPO-2019 24-11-2020 खच बढ़ाना चाहता है । िकस ितशत


20% था, और C, T का 15% था। । (Evening shift) तक (एक दशमलव थान तक ) उसे
जब उसका वेतन 40% बढ़ा, तो (a) 7.2% decrease/ कमी अपनी िकतनी खपत कम करनी
उसने E का ितशत र बनाए रखा, (b) 3.4% increase / वृ चािहए?
लेिकन T, E का 30% हो गया, और (c) 6.4% increase / वृ CPO-2019 25-11-2020
C, T का 20% हो गया। उसकी (d) 3.4% decrease/ कमी (Morning shift)
पहले की बचत तथा वतमान बचत म (a) 8.7%
ा अनुपात है ? (b) 6.5%
CPO-2019 24-11-2020 Q54. If decreasing 110 by x% (c) 7.2%
(Morning shift) gives the same result as (d) 9.5%
(a) 337 :325 increasing 50 by x%, then x% of
(b) 655 : 644 650 is what percentage (correct to 57.Q. Ramesh spends 40% of his
(c) 644 : 655 the nearest integer) more than monthly salary on food, 18% on
(d) 325 : 337 (x-10)% of 780? house rent, 12% on

www.ssccglpinnacle.com support@ssccglpinnacle.com Ph. 09729327755, 09817390373 206


Days 24-27 Percentage

s
sse
la
_c
ob
entertainment, and 5% on 40%, then the resultant fraction is
bo
ah
conveyance. But due to a family 16
63 The original fraction is:
sm
ur
yo

function he has to borrow Rs. यिद िकसी िभ का अंश 60% बढ़


e/
t.m
://

16,000 from a money lender to जाता है और हर 40% बढ़ जाता है ,


tp
ht

meet the expense of 20,000. His तो प रणामी िभ 63


16 होता है । मू ल

monthly salary (in Rs.)is: िभ ा है ?


रमेश अपने मािसक वेतन का 40% CPO-2019 25-11-2020
भोजन पर, 18% घर के िकराये पर, (Evening shift)
12% मनोरं जन पर और 5% वाहन (a) 95
पर खच करता है । लेिकन एक 2
(b)
पा रवा रक समारोह के कारण उसे 11
4
(c)
20,000 के खच को पूरा करने के 9
(d) 2
िलए एक ऋणदाता से 16,000 पये 9
उधार लेने पड़े । उसका मािसक वेतन
( पये म) ा है ? 60.Q A’s salary is 35% more than
B’s salary. How much percent is

e
CPO-2019 25-11-2020
(Evening shift) B’s salary less than that of
(a) 15,000 A’s?(correct to the nearest
(b) 18,000 integer)
A का वेतन B के वेतन से 35%

l
(c) 16,500
(d) 16,000 अिधक है । B का वेतन A से िकतना
ac
58.Q. In an entrance examination
at different centres, a total of 25,
30, 40, 45, 60 and 100 students
appeared. The pass percentage of
ितशत कम है ? (िनकटतम पूणाक
तक )
CPO-2019
(Evening shift)
(a) 26%
25-11-2020

the different centres are 20%, (b) 20%


30%, 35% 40% 50% and 75% (c) 35%
respectively. The pass percentage (d) 17.5%
nn
of the entrance examination
is:(correct to the nearest integer)
िविभ क ों पर वेश परी ा म कुल
25, 30, 40, 45, 60 और 100 छा
उप थत ए। िविभ क ों का
उ ीणता ितशत मशः 20%,
30%, 35% 40% 50% और 75% है ।
वेश परी ा का उ ीणता ितशत
Pi

ा है ात करे : (िनकटतम पूणाक


तक )
CPO-2019 25-11-2020
(Evening shift)
(a) 43%
(b) 59%
(c) 53%
(d) 50%

59.Q. If the numerator of a


fraction is increased by 60% and
the denominator is increased by

www.ssccglpinnacle.com support@ssccglpinnacle.com Ph. 09729327755, 09817390373 207


Days 24-27 Percentage

s
sse
la
_c
ob
SOLUTION
bo
ah Sol 5. (c) 20% = 1
sm
5
ur
yo

Variety Questions Let the original price = 50 unit


e/
t.m
://

⇒ decreased price = 40 unit


tp
ht

Sol 1. (c) Let the original sale = 100 unit


According to the question ⇒ New sale = (100+x) unit
120 × 100 −x 100+x
100 = 40 × 100 Initial revenue = 50 x 100 units
300-3x = 100 + x ⇒ Increased revenue = 40 × Desired percentage = 12 x 100 =
120
X = 50 (100+x) units 10%
50% of 210 = 105 According to the question
(50+20)% of 180 = 126 160 = 40 × (100+x)
(50 x 100) × 100 Sol 8. (a)
Required percentage = 126126
−105
⇒ 200 = 100 + x 17
85% = 20
× 100 = 16 32 % ⇒ x = 100 Let the original number = 20
According to the question
Sol 2. (c) Sol 6. (b) 75 + 17 unit = 20 unit

e
Income : Expenditure Savings 20% = 51 , 30% = 3
10 and 25% = 1 unit = 25
60 : 50 → 1
Original number (20 unit) = 25 x
4
10 ⇒ A:B = 4:5 and C:D = 13 : 10 20 = 500
After increase in Income and
and A:D = 4 : 3

l
Expenditure. Sol 9. (b)
Balancing the ratio for A and D
96 : ac 85 → 20% = 51 and 40% = 2
A:B:C:D = 40:50:39:30 5
11
Let Let the amount at the end of 2nd,
Therefore, Net Increase in
1 × 100 = 10%
B = 50 unit and C = 39 unit third and 4th years be a,b and c
Savings = 10 78 = 39
Now, 50 × 100 respectively.
⇒ C is 0.78 of B . a:b:c
Sol 3. (d) 5:6
25% = 41 5:6
Sol 7. (a)
⇒A:B=5:4 Balancing the ratio for b
We know that,
nn
65% = 13
20 Price x Consumption = a:b:c = 25 : 30 : 36
⇒ C : A+B = 7 : 20 Expenditure According to the question
Balancing the ratio for A + B 2
20% = 51 and 8% = 25 (36-30) unit = 259.20
A = 100, B = 80 and C = 63 Initial : 1 unit = 43.20
Required % age = 100100
−63 × 100 =
New Amount at the end of 2nd year
37 Price 5 : 6 (25 unit) = 43.20 x 25
Expenditure 25 : 40% of this amount = 43.20 x 25
Sol 4. (c) x 52 = 432
Pi

27
50% = 21
A:B = 3:2 _______________ Sol 10. (d)
Let income of A = 300 unit and B Consumption 25/5 : Let the income of sudha = A
= 200 unit 27/6 According to the question
12 × 80 = 4800
A × 100
Total = 300+200 = 500 unit 10 : 9 100
Increased income of A =300 × %age decrease in consumption = ⇒ A = 50,000
140 10−9 x 100 = 10% ans 27% of A = 50,000 × 27 =
100 = 420 10 100
Increased income of B = 200 13500
190 = 380
× 100 Alternate :
Total income of A and B = Sol 11.(b)
420+380 = 800 Direct Trick Initial :
Required % = 800500
−500 × 100 = New
30%increase 10 : 13
60%

www.ssccglpinnacle.com support@ssccglpinnacle.com Ph. 09729327755, 09817390373 208


Days 24-27 Percentage

s
se
s
la
_c
ob
25% decrease 4 : 3 31 : 25 Required percentage = 2.4 × 100
bo
ah
25% increase 4 : 5 Desired percentage = 31−25 x 100
sm
31 = 240
ur
yo

= 600 %
e/
t.m

31
://

________ Sol 23. (a)


tp
ht

160 : %age score of mathematics = 84


Sol 17. (a) Venn diagram for the 90
195 failed students: × 100 = 93.33 %
Desired percentage = 195−160 x 45
160 %age score of science = 50
100 = 21.87 ≈ 22% × 100 = 90 %
%age score of computer science =
Sol 12.(a) 23
25 × 100 = 92 %
A : A+B 68
%age score of English = 80
55 : 100
× 100 = 85 %
11 : 20
Total failed students = 20+32+24
Let A+B = 20 unit and A = 11
= 76% SSC CGL TIER II
unit
Therefore, Students passed in

e
⇒ B = 20-11 = 9 unit
both subjects = (100-76)% = 24% Sol 1. (d)
According to the question 7 and 25% = 1
28% = 25 4
(11-9) unit = 8
Sol 18. (a) Drawing a venn A:B
1 unit = 4
diagram:

l
Marks obtained by A (11 unit) = 32 : 25
11 x 4 = 44 A+B : C

Sol 13. (d)


ac
Marks obtained by B (9 unit) = 9
x 4 = 36

Let the salaries of A and B are a


4 : 3
Balancing the ratio for A+B
A : B : C : A+B
128 : 100 : 171 : 228
Desired %age = 171128
−128 x 100 =

and b respectively. 33.6 %


Percentage passed in both
According to the question subjects = 100 - (11+42+9) = 100
= 24000
nn
5 = b × 20
a × 100 100 - 62 = 38%
a = 4
b 1
Sol 2. (c)
⇒ (4+1) unit = 43000 Sol19. (c) Let the population at
80% = 54
1 unit = 8600 the beginning of first year be x.
A’s salary (4 unit) = 8600 x 4 = ATQ, Let the Income = 500,
34400 21 24 = 5512248
x × 20 × 25 Expenditure = 400
⇒ Savings = 500-400 = 100
Therefore, x = 5468500 112 =
Sol 14. (a) Increased income = 500 × 100
Sol 20. (a)
Pi

Aggregate percentage = 100 × [{ 20 =


Let A = 50, then B = 50 × 100 560
40 80 30 70 Decreased savings = 100 × 90 =
100 × 100 } +{ 100 × 100 } +{ 10 100
20 × 60 10 50 90
100 } +{ 100 × 100 } ] 20% of B = 10 × 20 =2
100 100
= 100 x 32+21+12+5 = 70% 2 New expenditure = 560-90 = 470
100 50 × 100 = 4%
Desired %age = 470400
−400 x 100 =
Clearly, 2 is 4% of A.
Sol 15. (c) 17.5%
Let x is the desired percentage Sol 21. (a)
⇒ 124 × 100x = 49.60 2 Sol 3. (b)
Total commission = 200000 × 100
49.60 Let the original number = 100
⇒x= 124 × 100 = 40 + 200000 × 1.5 +168000 × 1 = 116 × 114
100 100 New number = 100 × 100 100
8680 70
× 100 = 92.57
Sol 16. (d) Fractional value of
%age decrease = 100−92.57 x 100
24% = 25 6 100
Sol 22. (c)
Therefore, A : B ≈ 7%

www.ssccglpinnacle.com support@ssccglpinnacle.com Ph. 09729327755, 09817390373 209


Days 24-27 Percentage

s
sse
la
_c
ob
Alternate : Temporary female employee = ⇒ New savings = 2402-1625 =
bo
ah
%age change for 16% and 14% = 400 × 52 = 160
sm
777
ur
yo

16+14 + 16×14
100 = 32.24 7/15 of total employee = 1200 x Desired %age = 777−700 x 100 =
e/
t.m

700
://

Total change = 32.24 - 30 + 7


15 = 560 11%
tp
ht

32.24×(−30)
100 ≈ -7% Desired difference = 560-160 =
Here, -ve sign shows that 400 Sol 11. (b)
number is decreased We know that,
Sol 7. (c) Price x Consumption =
Sol 4. (b) According to the question Expenditure
19% = 10019 and 12% = 3
We know that, (60-20)% = 120 25
Price x sale = Revenue 1% = 3 Initial : New
25% = 41 and 20% = 51 (33 31 -28)% = 3 × 16
3 = 16 Price 100 : 119
Original : New Expenditure 25 : 28
Price 4 : 3 Sol 8. (b) _______________
Revenue 5 : 6 25% = 41 , 40% = 2
5 and 30% = Consumption 25/100 : 28/119

e
______________ 3 17 : 16
10
Sale 5/4 : 6/3 %age decrease in consumption =
A:B:C:D 17−16 x 100 ≈ 6 % ans
5 : 8 5:4 17
%age increase in sale (x) = 8−5 x 3:5

l
5
100 = 60% 13 : 10 Alternate :
ac Balancing the ratio for B and C Direct Trick
Sol 5. (c) A : B: C :D
Let the total number of voters = 195 : 156 : 260 : 200
1000 Desired percentage = 200200
−195 x

100 = 2.5

Sol 9. (c)
Let the total number of students =
nn
600 Desired percentage = 7 x 100
119
≈6 %
Desired %age = 330−270 x 100 =
270
22 92 % Sol 12. (c)
Let the income = 1000
Sol 6. (a) Expenditure = 720
Let the total employees = 3000 ⇒ Saving = 1000-720 = 280
Pi

Total number of absent students = 120 =


Increased income = 1000 × 100
unit
100+40 = 140 1200
140 x 100 = 23 1
Desired %age = 600 115
3 Increased saving = 280 × 100 =
% 322
New expenditure = 1200-322 =
Sol 10. (a) 878
65% = 2013
Desired percentage = 878720
−720 x

Total permanent employees = Let the income = 2000, 100 = 21.9


1250+600 = 1850 Expenditure = 1300
1850 unit = 740 ⇒ Savings = 2000-1300 = 700 Sol 13. (a)
1 unit = 52 Increased income = 2000 x 120.1
100 20% = 51 and 8 31 % = 1
12
Total number of employee = 3000 = 2402 Original : New
x 52 = 1200 Increased expenditure = 1300 x Price 5 : 6
125
100 = 1625 Consumption 12 : 11

www.ssccglpinnacle.com support@ssccglpinnacle.com Ph. 09729327755, 09817390373 210


Days 24-27 Percentage

s
se
s
la
_c
ob
11 : 10 : 10 : 10 Decreased saving of Surbhi = 100
bo
ah ____________ 6 : 6 : 5: 5 99 = 99 unit
× 100
sm
ur
yo

Expenditure 60 : 66 17 : 17 :17 : 25 New expenditure of surbhi =


e/
t.m
://

___________________ 480-99 = 381 unit


tp
ht

Desired %age = 66−60 x 100 = 1122 : 1020 : 850 : 1250


60 Desired %age = 381300
−300 × 100 =

10% According to the question 27 %


(1122-850) unit = 272
Sol 14. (b) 1 unit = 1 Sol 4. (a)
20% = 51 and 15% = 3 Marks of B = 1020 unit = 1020 x 40% = 52
20
Original : New 1 = 1020
Let the income of B = 500
Hours 5 : 6 ⇒ The income of A = 700
Wages 20 : 23 Practice Questions
Total income of A and B = 1200
Increased income of A = 700
Sol 1. (a) 125 = 875
3 × 100
____________ 15% = 20
Increased income of B = 500

e
Earning 100 : 138 Let sudha’s income = 2000 140 = 700
× 100
⇒ Sudha’s saving = 300
138−100 ⇒ Sudha’s expenditure = Total income of A and B =
Desired %age = 100 x 100 =
875+700 = 1575
38% 2000-300 = 1700
Desired %age = 15751200
−1200 x 100 =

l
Sudha’s increased saving = 300 x
160 31.25
Sol 15. (b) ac 100 = 480
According to the question Sudha’s increased expenditure =
120 = 2040 Sol 5. (b)
25 = y × 1 × 30 x 2.5
x × 21 × 100 4 100
1700 x 100
We know that,
x = 3 Sudha’s increased income =
y 2 Price x Consumption =
Desired percentage = 3−2 x 100 = 2040+480 = 2520
2 Expenditure
Desired percentage = 25202000
−2000 ×
9 and 12% = 3
50% 18% = 50 25
100 = 26 %
Initial : New
Sol 16. (c) Price 50 : 59
nn
33 31 % = 31 and 25 % = 1 Sol 2.(b)
4 Expenditure 25 : 28
20% = 51 and 25% = 1
Let the total amount = 12 unit 4 _______________
…(LCM of 3 and 4) ⇒ A:B = 4:5 and C:D = 6 : 5 and Consumption 25/50 : 28/59
⇒ Investment of A = 12 x 31 = 4 A:D = 4 : 3 59 : 56
Balancing the ratio for A and D %age decrease in consumption =
unit
A:B:C:D = 20:25:18:15 59−56 x 100 ≈ 5.1% ans
Profit will be distributed in the 59
Let Alternate :
ratio of amount invested as all
B = 25 unit and C = 18 unit
Pi

invested for the same period of Direct Trick


72 = 18
Now, 25 × 100
time.
Share of A in the profit = ⇒ C is 0.72 of B .
1,62,000 x 124 = 54000

Sol 3. (a)
Sol 17.(c) 75% = 43
10% = 101 , 20% = 1 and 32% = Let the income of Surbhi = 400
5
8 unit
25
⇒ saving of Surbhi = 100 unit
A : B : C:D Desired percentage = 6 x 100
So expenditure of Surbhi = 118
11 : 10
400-100 = 300 unit ≈ 5.1%
6 : 5
Increased income of surbhi = 400
17 : 25 120 = 480 unit
× 100 Sol 6. (c)
Balancing the ratio for B and C
We know that,
A : B : C:D

www.ssccglpinnacle.com support@ssccglpinnacle.com Ph. 09729327755, 09817390373 211


Days 24-27 Percentage

s
sse
la
_c
ob
Price x Consumption =
bo
ah
Expenditure Sol 8. (a)
sm
ur
yo

22% = 5011 and 12% = 3 We know that,


e/
t.m

25
://

Initial : New Price x Consumption =


tp
ht

Price 50 : 61 Expenditure
21% = 10021 and 12% = 3
Expenditure 25 : 28 25
_______________ Initial : New
Consumption 25/50 : 28/61 Price 100 : 121 10
Desired percentage = 117 x 100
61 : 56 Expenditure 25 : 28
≈ 8.5%
%age decrease in consumption = ______________
61−56 x 100 ≈ 8.2% ans _
61
Consumption 25/100 : 28/121 Sol 10. (a)
Alternate :
121 : 112 We know that,
Direct Trick
%age decrease in consumption = Price x Consumption =
121−112 x 100 ≈ 7.4% ans Expenditure
121 6 and 15% = 3
24% = 25

e
Alternate : 20

Direct Trick Initial : New


Price 25 : 31
Expenditure 20 : 23

l
______________
_

≈ 8.2%

Sol 7. (d)
ac
Desired percentage = 10
122 x 100

Desired percentage = 9 x 100


Consumption 20/25 : 23/31
124 : 115
%age decrease in consumption =
124−115 x 100 ≈ 7.3% ans
124
Alternate :
We know that, 121
Direct Trick
Price x Consumption = ≈ 7.4%
Expenditure
nn
17% = 10017 and 8% = 2 Sol 9. (c)
25
Initial : New We know that,
Price 100 : 117 Price x Consumption =
Expenditure 25 : 27 Expenditure
17% = 10017 and 7% = 7
_______________ 100

Consumption 25/100 : 27/117 Initial : New 9


Desired percentage = 124 x 100
117 : 108 Price 100 : 117
Expenditure 100 : 107 ≈ 7.3%
%age decrease in consumption =
Pi

117−108 x 100 ≈ 7.7% ans ______________


117
_ Sol 11. (c)
Alternate :
Consumption 100/100 : We know that,
Direct Trick
107/117 Price x Consumption =
117 : 107 Expenditure
24% = 25 6 and 18% = 9
%age decrease in consumption = 50
117−107 x 100 ≈ 8.5% ans
117
Initial : New
Alternate : Price 25 : 31
Direct Trick Expenditure 50 : 59
______________
Consumption 50/25 : 59/31
Desired percentage = 9 x 100 62 : 59
117
%age decrease in consumption =
≈ 7.7% 62−59 x 100 ≈ 4.8% ans
62

www.ssccglpinnacle.com support@ssccglpinnacle.com Ph. 09729327755, 09817390373 212


Days 24-27 Percentage

s
sse
la
_c
ob
Alternate : 117 : 105 ⇒ number of ilIiterate senior
bo
ah
Direct Trick %age decrease in consumption = 40
sm
citizens = 40 x = 16
ur

100
yo

117−105 x 100 ≈ 10.3% ans


e/

⇒ number of Iiterate senior


t.m

117
://

Alternate : citizens = 40 - 16 = 24
tp
ht

Direct Trick ⇒ number of Iiterate non-senior


25 = 15
citizens = 60 x 100
⇒ number of ilIiterate non-senior
citizens = 60 - 15 = 45
Desired %age = 4545 −24 x 100 = 46

2
Desired percentage = 6 x 100 3
124
≈ 4.8%
Sol 17. (a)
Desired percentage = 12 x 100 Income Expenditure Savings
Sol 12. (b) 117
≈ 10.3% 1000 : 900
We know that,
→ 100

e
Price x Consumption =
Sol 14. (c) After increase in Saving and
Expenditure
7 and 22% = 11 Income Expenditure Savings Expenditure.
28% = 25 50
100 : 72 → 1255 : 1125 → 130
Initial : New Therefore, Net Increase in

l
28
Price 25 : 32 Income =
After increase in Income and
Expenditure ac 50 : 61 1255−1000 × 100 = 25.5%
Expenditure. 1000
______________
128 : 90 →
_
38 Sol 18. (d)
Consumption 50/25 : 61/32
Therefore, Net Increase in Income Expenditure Savings
64 : 61
Savings = 3828−28 × 100 = 35.7% 1000 : 800
%age decrease in consumption =
64−61 x 100 ≈ 4.7% ans → 200
64 After increase in Saving and
Sol 15. (c)
Alternate : Expenditure.
20% = 51 , 25% = 1 and 60% =
nn
Direct Trick 4
3 1300 : 1000 → 300
5
Therefore, Net Increase in
A:B:C:D:E
Savings = 300200
−200 × 100 = 50%
6:5
5:4
2:5 Sol 19. (d)
6:5 Original : New
Balancing the ratio for C and D Price 25 : 34
Pi

A : B: C : D: E
6 18 : 15 : 12 : 30 : 25 Quantity 10 : 7
Desired percentage = 128 x 100
Now, ___________
≈ 4.7% Expenditure 250 : 238
30−18 x 100 = 40 %
30
Now, 250250
−238 x 100 = 4.8 %
Sol 13. (a) Clearly A is 40% less than D.
We know that,
Sol 16. (d) Sol 20. (d)
Price x Consumption =
Let total number of voters are Let the original number = 100
Expenditure 70 x 130 x
17 and 5% = 1 100. New number = 100 x 100 100
17% = 100 20 90
⇒ number of senior citizens = 100 = 81.9
Initial : New
40 = 40
100 x 100 Clearly the number is decreasing,
Price 100 : 117
⇒ number of non senior citizens desired %age
Expenditure 20 : 21
= 60 = 100100
−81.9 x 100 ≈ 18%
______________
Consumption 20/100 : 21/117

www.ssccglpinnacle.com support@ssccglpinnacle.com Ph. 09729327755, 09817390373 213


Days 24-27 Percentage

s
se
s
la
_c
ob
Sol 21. (a) According to the question Desired percentage = 5 x 100
bo
120
ah
Let the original number = 100 15 = b × 30
a × 100
sm
100 ≈ 4.2%
ur
yo

New number = 100 x 10070 x 130 x


⇒ a:b = 2:1
e/
t.m

100
://

130 = 118.3 Now, Sol 29. (c)


tp
ht

100
Clearly the number is increasing, (2+1) unit = 45000 We know that,
desired %age B’s salary (1 unit) = 15000 Price x Consumption =
= 118.3−100 x 100 ≈ 18%
100
Expenditure
Sol 26. (d) 25% = 41 and 4% = 251

Sol 22. (b) Let the salary of A = a and B = b Initial : New


Let the original number = 100 According to the question Price 4 : 5
25 = b × 10
a × 100
New number = 100 x 100130 x 70 x
100 100 Consumption 25 : 24
70 = 63.7 ⇒ a:b = 2:5 ______________
100
Now, Expenditure 100 : 120
Clearly the number is decreasing,
(2+5) unit = 42000 100 : 120
desired %age
B’s salary (5 unit) = 30000 %age increase in expenditure =
= 100100
−63.7 x 100 ≈ 36%

e
120−100 x 100 = 20 % ans
100
Sol 27.(b) Alternate :
Sol 23. (d)
Let the salary of A = a and B = b Direct Trick
Let the salaries of A and B are a
According to the question
and b respectively.

l
25 = b × 10
a × 100
According to the question 100

a
b =
5 = b × 20
a × 100
4
1
ac
100

⇒ (4+1) unit = 43000


B’s salary (1 unit) = 8600
⇒ a:b = 2:5
Now,
(2+5) unit = 42000
A’s salary (2 unit) = 12000

24
Desired percentage = 120 x 100 =
Sol 24. (c) Sol 28. (b)
Let the marks of B = k and A = 20 %
We know that,
nn
k+8 Price x Consumption =
According to the question Sol 30. (c)
Expenditure We know that,
60 = (k+8)
(k+k+8) × 100 20% = 51 and 15% = 203
Price x Consumption =
6k+24 =5k+ 40 Initial : New Expenditure
⇒ k = 16
Price 5 : 6 24% = 25 6 and 14% = 7
50
Sum of the marks of A and B = Expenditure 20 : 23
{16+(16+8)} = 40 ______________ Initial : New
Consumption 20/5 : 23/6 Price 25 : 31
Pi

Alternate : 24 : 23 Expenditure 50 : 57
%age decrease in consumption = ______________
60% = 3
5 24−23 x 100 ≈ 4.2% ans
24 _
According to the question Alternate : Consumption 50/25 : 57/31
A+B : A Direct Trick 62 : 57
5 : 3
%age decrease in consumption =
Let A+B = 5 unit and A = 3 unit 62−57 x 100 ≈ 8.1 % ans
⇒ B = 5-3 = 2 unit 62

According to the question


Alternate :
(3-2) unit = 8
Direct Trick
5 unit = 5 x 8 = 40

Sol 25.(d)
Let the salary of A = a and B = b

www.ssccglpinnacle.com support@ssccglpinnacle.com Ph. 09729327755, 09817390373 214


Days 24-27 Percentage

s
sse
la
_c
ob
57 : 54 Therefore, reduction in
bo
ah %age decrease in consumption = consumption =
sm
ur
yo

57−54 x 100 = 5.3 % ans 9


59 × 100 ≈ 15.3%
e/
t.m

57
://
tp
ht

Alternate : Sol 39. (b) Venn diagram for the


Direct Trick given data:

Desired percentage = 10 x 100 =


124
8.1 %

Sol 31. (a) Given,


88% = 216128
Desired percentage = 6 x 100 =
1 % = 2456 114 Percentage passed in both
⇒ Total income (100%) = 5.3 % subjects = 100 - (13+40+8) = 100

e
2,45,600 - 61 = 39%
Sol 35. (b)
Sol 32. (c) Original : New Sol 40. (d) Ratio of income :
Original : New Price 20 : 17 A : B

l
Price 10 : 9 Consumption 17 : 7 : 5
Expenditure ac 1 : 1 20 Therefore, B’s income
________ Therefore, % increase in less than A = 72 × 100 = 2007 %
consumption = 173 × 100 = 300
Consumption 1/10 : 1/9 17
9 : 10 Sol 41. (b) 16%=4/25
Desired %age = 9−9 x 100 = 100
10
9 Sol36. (d) According to the question
% A : B (4+25) unit = 1914
7 : 5 1 unit = 66
Note : If expenditure is same in Therefore, B’s income less than A Therefore, the no. is (25 unit) =
nn
both cases ratios of Price and = 72 × 100 = 28 74 % 25 x 66 = 1650
consumption will be reciprocal
to each other. Sol 37. (d) Venn diagram of the Sol 42. (d)
given data: Original : New
Sol 33. (d) 24% = 6
25 1st step 10 :
A : B 9
19 : 25 2nd step 10 :
Therefore, B’s income 11
Pi

6 × 100 = 600 %
more than A = 19 19
______________
Sol 34. (b) _
We know that, Therefore, Passed in both subjects 100 : 99
Price x Consumption = = 100 - (14+32+10) = 100-56 = According to question
Expenditure 44% (100-99) = 1 unit = 100
8% = 252 and 14% = 7 100 unit = 10000
50
Initial : New Sol 38. (d)
Original : Sol 43. (b) 5 litres = 5000 ml
Price 50 : 57 8 = 400
New 8% of 5 litres = 5000 × 100
Expenditure 25 : 27
Price 50 : 59 ml
______________ Consumption 59 : 50
_ Sol 44. (c)
15% = 20 3
Consumption 25/50 : 27/57

www.ssccglpinnacle.com support@ssccglpinnacle.com Ph. 09729327755, 09817390373 215


Days 24-27 Percentage

s
sse
la
_c
ob
SSC MTS 16 32 % = 1 and 15% = 3

bo
6 20
ah Old : Sol 1. (c)
sm
Original New
ur
yo

New Let the original value = x 6 7


e/
t.m
://

Price 20 : 23 According to the question 20 17


tp
ht

Consumption 23 : 120 × 85 = 2040


x × 100 100 _________________
20 ⇒ x = 2000 120 119
Alternate : According to the question
___________ 20% = 51 and 15% = 3 119 unit = 238
20
Expenditure 460 : Original New 1 unit = 2
460 5 6 120 unit = 240
20 17 37.5% of 240 = 240 x 37.5
100 = 90
Required %age = 3 × 100 = 13
23 _________________
1 %
23 100 102 Sol 5. (a)
According to the question 20% = 51 and 25% = 1
4
Sol 45. (d) 102 unit = 2040 A :B :C

e
Required percentage = 3.2 ×
240 1 unit = 20 6 : 5
100= 1 31 100 unit = 2000 5 :4
A: B :C
Sol 46. (b) Required percentage = Sol 2. (c) 6: 5 :4

l
3 Let the salary of the person = x Let A = 6 unit , B = 5 unit and C
12×100 × 100 = 0.25
ac According to the question = 4 unit
90 × 80 = 4680
x × 100
Sol 47. (d) % Savings = 100 - 100 Desired %age = 6−64 x 100 =
(24+16+12) = 100-52 = 48% ⇒ x = 6500 33.33 %
Now, 48% = 3288 Alternate :
1 and 20% =
10% = 10 1
1 % = 137 5 Sol 6. (c)
2
15% = 20 3
Therefore, Monthly income = Initial Remaining
100% = Rs. 6850 10 9 Let B = 20 unit ⇒ A = 23 unit
5 4 Desired %age = 2323
−20 x 100 =
nn
Sol 48. (c) _________________ 13.04 %
Let Nidhi’s marks in social 50 36
science be x. According to the question Sol 7. (d)
According to the question 36 unit = 4680 26 % = 832
74+62+70+x × 100 = 68 1 unit = 130 1% = 32
400
⇒ 206 + x = 272 50 unit = 6500 31% = 992
Therefore, x=66
Sol 3. (a) Sol 8. (a)
Pi

20% = 51 and 25% = 1


4 40% = 52 and 30% = 3
Sol 49. (a) 10
34% of 1.2km = 34 × 1.2 × 1000 × A :B :C Original
100
6 : 5 New
= 10040800cm
3:4 5 7
Balancing the ratio for B 10 13
Sol 50. (a)
A: B :C
Let A’s salary in 2014 = I
18 : 15 : 20 _________________
According to the question
108 × 94 = 234778 Let A = 18 unit , B = 15 unit and 50 91
I × 100 100 C = 20 unit 91−50
Desired %age = 50 x 100 =
⇒I= A 18
100 100 C = 20 82%
234778 × 108 × 94 = 231263
⇒ A = 0.9 C Alternate :
(approx)
Net percentage Increase =
Sol 4. (d) 40+30+ 40×30
Sol 51. (c) 9 × 100 = 22 21 % 100 = 82%
40

www.ssccglpinnacle.com support@ssccglpinnacle.com Ph. 09729327755, 09817390373 216


Days 24-27 Percentage

s
sse
la
_c
ob
Total Income = 80 lakh + 20 lakh Number of girls who failed = 600
bo
ah
Sol 9. (a) = 100 lakh 40 = 240
x 100
sm
ur
yo

Desired %age = 400−320 x 100 = Alternate : 480+240


Desired %age = x 100 =
e/
t.m

400 800+600
://

20% Let the Initial income = k


tp

51.43%
ht

Extra tax paid due to additional


Sol 10. (d) amount of 20 lakh = 20 lakh x Sol 21. (d)
16
200% = 12 100 = 320000 75% of 260 + 30% of 320 = 75
100
Let B = 1 unit This tax amount get adjusted due x 260 + 30 x 320
100
⇒ A= 3 unit to tax reduction
=
Desired %age = 3−1 x 100 = ⇒ k x (20100
−16)
= 320000
3 195+96 = 291
66.67% ⇒ k = 80 lakh
Total Income = 80 lakh + 20 lakh Sol 22. (b)
Sol 11. (a) = 100 lakh Let the amount X have = x and
Let Ravi’s salary = 2 unit the amount have Y = y
⇒ Manish’s salary = 1 unit Sol 15. (a) According to the question

e
Desired %age = 2−11 x 100 = 100 According to the question = x+y = 1300 …….(1)
40 = y × 50
x × 100
% 100 And
y 4 3 1
Sol 12. (a) ⇒ x = 5 5 × x -10= 2 × y
40% = 52 and 80% = 4
6x-5y = 100

l
5
Rakesh : Rahul : Deepak Sol 16. (a) …………(2)

Deepak
ac
5 : 3
5 :

Rakesh : Rahul :
9
Balancing the ratio for Rahul
5.6 kg = 5600 gram
Desired percentage =
= 2.5 %

Sol 17. (c)


140
5600 x 100
Multiply equation (1) by 5 add it
in equation (2)
11x = 6600
x = 600

25 : 15 : 27 According to the question Sol 23. (d)


27 unit = 34560 120 = Y
X × 100 Let the total marks = k and
1 unit = 1280 X = 5 passing marks = p
nn
Y 6
25 unit = 1280 x 25 = 32000 According to the question
Let X = 5 unit and Y and 6 unit 32 + 6 = p
80 = 6 unit × 60 + 36 k × 100
5 unit × 100 100
Sol 13. (c) …………(1)
⇒ 1 unit = 90
Let the total number of workers = And
Difference between X and Y = 36 - 2 = p
500 unit k × 100
(6-5) unit = 90
…………(2)
Sol 18. (b) From (1) and (2)
32 + 6 = k ×
k × 100 36 -2
Pi

0.9% = 0.009 100


8=k × 36 -k × 32
Desired difference = 0.9 - 0.009 = 100 100
0.891 ⇒ k = 200
Passing marks (p) = 200 × 36 -2
100
Desired %age = 240+150 x 100 = Sol 19. (a) 32
500 or k × 100 +6 = 70
78% Let X = 100 unit Alternate :
⇒ Other two numbers are 50 unit (36-32)% = 6+2
Sol 14. (a) and 80 unit respectively 1% = 2
Let the Aman’s original annual Desired ratio = 50:80 32% = 64
income = k = 5 : 8 ans 36% = 72
According to the question Pass marks = 64+6 or 72-2 = 70
16 = k x 20
(k+2000000) x 100 100
Sol 20. (c)
20k 16k = 320000
- 100 Number of boys who failed = 800 Sol 24. (a)
100 60 = 480
x 100 Total marks = 100 x 5 = 500
k = 8000000

www.ssccglpinnacle.com support@ssccglpinnacle.com Ph. 09729327755, 09817390373 217


Days 24-27 Percentage

s
sse
la
_c
ob
Total marks he got = 500 x 90 = ⇒ k = 280 Banana’s of good quality = 6-1 =
bo
100
ah 5 unit
sm
450
ur
yo

Marks in the fifth subject = Sol 30. (b) Desired %age = 65 x 100 = 83 31 %
e/
t.m
://

Desired value = 180000 x 100 x


450-82-97-88-91 = 92 or 83.33 %
tp

3×100
ht

0.15 = 90
100
Sol 25. (a) Sol 35. (c)
Let the third number is 100 unit Sol 31. (c) Let the number = K
⇒ other two numbers are 180 and Let the salary of A = a and B = b According to the question
135 According to the question
40 = 24
K × 52 × 100
Desired ratio = 180 : 135 a+b = 16500 ⇒ K = 150
= 4 : 3 And
30
Savings of A = a x 100 Sol 36. (a)
Sol 26. (c) Savings of B = b x 20 15% = 20 3
100
Given, 12.5% of A = 55 Let,
100 = 440
Now,
⇒ A = 55 × 12.5 30 = 125 20 End of 2nd / Beg. of 3rd = A
a x 100 x (b x )

e
100 100
3 =
a × 10 b × 41 End 3rd / Beg. of 4th = B:
Sol 27. (d) a 5 End of 4th = C
Desired value = 1440 × 30 × 12.5 b = 6
100 100 A : B : C
(5+6) unit = 165000
= 54 20 : 23

l
1 unit = 15000
20 : 23
Difference of their income =(6-5)
Sol 28. (d) ac ______________
unit = 1 unit = 15000
We know that, 400 : 460 : 529
Price x Consumption =
Sol 32. (c)
Expenditure (529-460) unit = 193.20
Let the monthly income of sonu =
20% = 51 and 12% = 253 1 unit = 2.8
I
Initial : New 400 unit = 1120
According to the question ⇒ Desired value = 1120 × 90
Price 5 : 6 12 × 125 = 2160 100 =
I × 100 100
Expenditure 25 : 28 1008
nn
⇒ I = 14400
______________
_ Sol 37. (d)
Consumption 25/5 : 28/6 Sol 33. (c) Let the total number of students =
8% = 252
15 : 14 100
%age decrease in consumption = Let the last year population of the ⇒ Number of boys = 100 x 100 40
15−14 x 100 = 6 2 % ans village = 25 unit = 40
15 3
Alternate : Current year population of the ⇒ Number of girls = 100-40 =
Direct Trick village = 23 unit
60
Pi

According to the question 56


Total students pass =100 x 100 =
25 unit = 72000
56
1 unit = 2880 60
Number of boys Pass = 40 × 100 =
23 unit = 66,240
24
Sol 34. (c) Number of girls pass = 56-24 =
Desired %age = 780−130 x 100 32
780 32 x 100 = 53 1
Desired percentage = 8 x 100 = 1 Desired %age = 60 3
120 = 83 % 3
2
6 % %
3
Alternate :
Sol 38. (a)
Sol 29. (a) Total : Rotten
25% = 41 , 20% = 1
5 and 10% =
Let the number = k 780 : 130
1
According to the question 6 : 1 10
60 = 168
k x 100 A:B:C:D

www.ssccglpinnacle.com support@ssccglpinnacle.com Ph. 09729327755, 09817390373 218


Days 24-27 Percentage

s
se
s
la
_c
y

ob
5:4 yx x = 150 and z x = 300 Area of rectangle is ‘ab’
bo
100 100
ah
4:5 y After 40% increase, length
sm
⇒ 2(y x x
100 ) = zx
ur

100
yo

11 : 10 becomes ‘1.4a’
e/

⇒ z = 2x
t.m
://

_____________ After 20% decrease, Breadth


tp
ht

55 : 44 : 55 : 50 Sol 44. (b) becomes ‘0.8b’


Here B is 5555−44 x 100 = 20% less
Let marks scored by B = k Hence, area of rectangle is
than A. ⇒ marks scored by A = k+8 (1.4a)(0.8b)= ‘1.12ab’
Clearly option A is wrong. Thus, %increase in area of
According to the question
55 x (k+k+8) rectangle is ‘12%’
(k+8) = 100
Sol 39. (d)
20k + 160 = 22k + 88
33 31 % = 31 Sol 2. (c) Let the number be ‘x’
⇒ k = 36
According to the question 80% x - 62% x = 198
Required sum = {36+(36+8)} =
200 x ( x+10 4 x 18% x = 198
100 )= 3 x(180 x 100 ) 80
1% x = 11
200(x+10) = 240x
92% x - 56% x = 36% x = 396
⇒ x = 50 Sol 45. (d)

e
So, 10% of (x + 20) ⇒ 10% of 25% = 41
Sol 3. (b) Let income of sonu =
(50 + 20) = 7 (x- 1x ) = 1 (x+ 1x )
4 ₹100.
So, 40% of x ⇒ 40% of 50 = 20 ⇒ 4x- 4x = (x+ 1x ) Her saving = ₹15
Desired %age = 2020−7 x 100 = 65

l
⇒ 3x = 5 Expenditure, E1= ₹ 85
x
⇒ x2 = 5 and x4 = 25 Increased income = ₹ 120
Sol 40. (c) ac
Let the two numbers are x and y.
According to the question
60 = 3 x y
x × 100
⇒ x : y=5:7
7
66.7

Sol 46. (b)


3

Required %age = 5
3

9
25 5
9 3
× 100 =
Increased expenditure, E2= ₹ 105
% increase in expenditure =
105−85 × 100 = 23.5%
85

Sol 4. (d) Income of A and B =


20% = 51 40:100 = 2:5
Sol 41. (c) (x- 1x ) = 1 (x+ 1x ) A’s income = 2a
5
50% = 21 , 33 31 % = 1 and 25% B’s income = 5a
nn
3 ⇒ 5x- 5x = (x+ 1x )
= 1
4 ⇒ 4x = 6 Expenditure of A and B = 60:100
x
= 3:5
and x3 = √
3 3
⇒ x2 = 3
Initial : New 2 2√2 A’s expenditure = 3b
First year 2 : 1 3√3
2√2
− 32 B’s expenditure = 5b
Required %age = × 100 =
2nd Year 3 : 2 3√3
2√2
As per given condition:
3rd Year 4 : 3 18.59 ≈ 18% 2a = 70% of 5b
⇒ 5b2a = 7
______________ 10
Pi

4 : 1 ⇒ a = 7
Sol 47. (d) b 4
Desired %age = 4−41 x 100 = 75 Required %age = 64 × 100 = 256 Let a= 7x and b= 4x
25
% Thus,
Sol48. (a) A’s income = 2a = 14x
Sol 42. (a) Percentage of zinc in the alloy = B’s income = 5a = 35x
85% = 2017
(100-32-24) = 44% A’s expenditure = 3b = 12x
Let the number = 20 unit Required Amount of Zinc = B’s expenditure = 5b = 20x
According to the question 44 = 5.28 kg
12 × 100 A’s saving = 2x
75 + 17 unit = 20 unit B’s saving = 15x
1 unit = 25 SSC CGL TIER I Required ratio = 2:15
⇒ So, the number = 20 unit = 20
x 25 = 500 Sol 1. (b) Let , Sol 5. (b) Let salaries of A and B
Length of a rectangle is a =₹x
Sol 43. (d) Breadth of rectangle is b

www.ssccglpinnacle.com support@ssccglpinnacle.com Ph. 09729327755, 09817390373 219


Days 24-27 Percentage

s
sse
la
_c
ob
As A donated 8% salary and B Sol 9. (a) Total change% in 125 C
100 = C ′
bo
ah
donated 7% salary and difference population = 30-15- 30×15
100 =
sm
5 C
4 = C′
ur
yo

between their donation is ₹ 259 10.5% increase


e/
t.m

%reduction = 5−4 × 100 = 20%


://

1% x = ₹ 259 5
Let two years ago, age = x
tp
ht

Salary of each of A and B = ₹ 110.5 × x = 11050 SSC CHSL 2019


100 Sol:1. (c)
25900 x = 10,000
Total donation of A and B = 15% 90% = 216
of ₹ 25900 = ₹ 3885 100% = 240
Sol 10. (b) Let total number of
C’s donation = ₹ (3885-1185) = ₹ students = x
2700 ⇒ according to question, Sol:2. (c)
C’s salary = ₹ 30000 students who passed but did not x + y + xy /100 %
A’s donation = ₹ 2072 get distinction can be represented 35 + (-20) + 35×(−20)
= 8%
100
B’s donation = ₹ 1813 as:
⇒ x × 10096 × 90 × 50 = 432
Total salary of A,B and C = ₹ 100 100
⇒ x = 1000 Sol:3.(a)
81,800 10×10
% increase=10+10+ =21%

e
Donation of A and C = ₹ 4772 100
4772 Sol 11. (d) Let M be the
Required % = 81800 × 100 = 5.8%
maximum marks Sol:4.(d)
According to question: 60%-40% = 30
Sol 6. (d) According to question: 20% = 30
31% of M +16 = 40% of M - 56

l
Marks scored by A:B:C:D = So, 20% of the number = 30
9% of M = 72
40:32:20:25 ac M = 800
A = 40 x = 80% Sol:5.(d)
Passing marks = 40% of M - 56 =
Then, D = 50% Let each side=10
320-56 = 264
volume= 10 × 10 × 10 =1000
Sol 7. (d) Let radius of cylinder New length, breadth and
Sol 12. (b) Income : Expenditure :
be ‘r’ and height of cylinder be height=11,12,8
Saving ⇒ 100: 68 : 32
‘h’ New volume= 11 × 12 × 8 =1056
Income increase by 20%, and
Change in ‘r’ ⇒ 10 → 9 56 ×
saving by 9 83 % Required percentage= 1000
nn
Change in ‘h’ ⇒ 10 → 12
New ratio, 100=5.6%
Volume of cylinder = Π × r2 × h
Income : Expenditure : Saving ⇒
Let v1 and v2 be volumes of
120: 85 : 35 Sol:6. (a)
before and after change.
v1 r 2 h % increase in monthly The seller sold 48 spoiled fruit
⇒ v2 = ( r 1 ) × ( h1 ) expenditure = which is 60% of the total spoiled
2 2
v2 9 2 12 85−68 × 100 = 25% fruits,
⇒ v1 = ( 10 ) × ( 10 ) 68
v2
So total spoiled fruits are
81 12
⇒ v1 = ( 100 ) × ( 10 ) = 60% → 48
Sol 13. (c) Let x be the number
Pi

v2 972
⇒ v1 = 1000 To increase it by 25%, we = 100% → 80
⇒ % decrease = 1000−972 × 100 = multiply it by 125 and divide by A crate of fruits contains one
972
100 spoiled fruit for every 25 fruits,
2.8%
125 × x = 5 × x
We get, 100 so
4
The number of fruits in the crate
Sol 8. (c) Price change in sugar =
10 → 12 Sol 14. (a) 72% of Maximum = 25 × 80 = 2000
1
Price ∝ consumption marks = 360
P1 C2 10 Then, maximum marks = 360 Sol:7.(d)
P2 = C1 = 12
× 100 Let, the third number = 100
% cut in consumption = 72 = 500
12−10 × 100 = 16 2 % Second number = 125, then third
12 3 number = 165
Sol 15. (d) price ∝ 1
consumption
Required ratio = 125:165 = 25:33
P′ C
P = C′
Sol:8.(c)

www.ssccglpinnacle.com support@ssccglpinnacle.com Ph. 09729327755, 09817390373 220


Days 24-27 Percentage

s
sse
la
_c
ob
Total runs = 120 Investment on Policy and funds = Then, Sachin’s income = Rs. 125
bo
ah
Run including boundaries and 33 = 8250
25000 × 100 Dileep’s income is less than
sm
ur
yo

sixes = 4 × 6 + 6 × 4 = 48 Remaining amount = Sachin’s income by = 125125− 100 ×


e/
t.m
://

Run made by running between 25,000-8250-3750 = 13,000 100 = 25 × 100 = 20%


tp
ht

125
the wickets = 120-48 = 72
Required percentage = 12072 × 100
Sol:15.(c) Sol: 21. (a)
= 60% 50% of A = 25% of B Length is increased by 12% and
A :B=1:2 breadth is decreased by 8%.
Sol:9. (b) Net effect on area = + 12 - 8 +
let the number be x Sol:16. (d) 12 × (−8)
= 4 - .96 = + 3.04
100
when multiplied by = 3x4
3 Let the annual income = x
4
19 = 559968
3.04% increase in area.
when multiplied by = 4x3 4 x × 100
3
difference = 4x3 − 3x4 = 12
7x x = 29,47,200 Sol 22. (a)
Monthly income = 2947200 =
7x
12 a% of 240 = c and c% of a =
percentage error = 12
4x × 100 =
3 245600 117.6

e
43.75% ⇒ 100a × 240 = c and c × a =
100
Sol:10.(b) Sol:17. (b) 117.6
Let Richa's income = 100 ⇒ ac = 100 = 5
240 12
Rita's income = 85
⇒ a = 5x and c = 12x

l
required percentage = c
100−85 × 100 = 17 11 % 100 × a = 117.6
85

Sol:11. (a)
ac 17

Number of terms = 1000-101 =


899+1 = 900
Number of unit digits = 90
The original price of sugar per kg
⇒ 12x
100 × 5x = 117.6
⇒ 60 x2 = 11760
⇒ x2 = 196
⇒ x = 14
⇒ a = 5x = 70 and c = 12x = 168
90 × 100 = = 160/16 = 10 a + c = 70 + 168 = 238
Required percent = 900
10%
Sol 18. (d)
nn
Sol 23. (a)
Sudha’s income = 11 × 3000 =
Sol:12. (a) 10 30% of 400 + x% of 70 = 25% of
Rs. 3300 536.
Let the number = x 9
Renu’s income = × 3300 = Rs. ⇒ 10030 × 400 + x × 70 = 25
25 = 68
x × 100 10 100 100
2970 × 536
x = 272
⇒ 120 + 7x = 134
10
Sol:19. (c) ⇒ 7x =134 - 120 = 14
Sol:13.(b) 10
Let two numbers be a and b of ⇒ 7x = 14
7.6% of a = 12.4% of b 10
Pi

a 124 31 which a be greater.


b = 76 = 19 30 × a
⇒ x = 20
a - b = 100
31x-19x = 1020
⇒ 10(a - b) = 3 × a
12x = 1020 Sol 24. (c)
⇒ 10 a - 10 b = 3a Let three numbers be a, b and c
x = 85
⇒ 7a = 10b a = 125% of c
so, numbers are = 31x = 31 × 85 =
⇒ ba = 10 b = 160% of c
2635, and 19x = 19 × 85 = 1615 7

sum of the two numbers = Smaller number, b = 28 a : b = 125 : 160 = 25 : 32


⇒ 28a = 10
2635+1615 = 4250 7
⇒ a = 40 Sol: 25. (a)
Sol:14.(d) Sum of both numbers = 40 + 28 = Total number of votes = 64000
Let his salary = 100% 68 Invalid votes = 20% of total votes
15% = 3750 ∴ Valid votes = 80% of total votes
100% = 25000 Sol: 20. (b) = 54 × 64000 = 51200
Let Dileep’s income = Rs. 100

www.ssccglpinnacle.com support@ssccglpinnacle.com Ph. 09729327755, 09817390373 221


Days 24-27 Percentage

s
sse
la
_c
ob
Candidate X got 70% of valid Total property = ( 12500 × 100 )× Expenditure = 0.8x
bo
30
ah 7 × 51200 = 35840
votes = 10 Expenditure after increment =
sm
3
ur

2
yo

120 = 0.96x
0.8x × 100
e/

45% of total property = (


t.m
://

Sol: 26. (d) 12500 × 100 3 45 New income = 1.29x


tp

30 ) × 2 × 100
ht

% increase in income = On solving total property New saving = 0.33x


1160 − 1000 × 100 = 160 × 100 =
1000 1000 =28,1,250 Increment in saving =
0.13 × 100 = 65%
16% 0.20

Sol:32.(b)
Sol: 27. (d) Let rishu income = 100 Sol:37.(d)
Required percentage = 10+10+ Saving = x Let B’s income = 100
10×10 = 21%
100 Expenditure = 100 - x A’s income = 40
New income = 126 B’s income is 60 more than A’s
SSC CGL 2019 TIER-II New expenditure = 1.2(100 - x) income
Sol:28.(a) New saving = 1.5(x) So, percentage = (60/40) × 100=
According to question Income = saving + expenditure 150%

e
A:B=9:5 126 = 120 - 1.2x + 1.5x
C = 51 73 % of (A+B) 6 = 0.3x Sol:38.(a)
C : (A + B) = 360 : 100 =36 : 70 x= 20% 50% A= (⅚) B
7
A : B : C = 45 : 25 : 36 A/2=5B/6

l
C%less than A = 45 9 × 100 =20 Sol:33.(c) A/B=5/3
ac Ratio of A : B = 3 : 5 A=5k, B=3K
%
Ratio of C : (A + B) = 2 : 5 A+B=80kg
Ratio A : B : C = 15 : 25 : 16 5k+3k=50
Sol:29.(c)
B is greater than C by = 8k=80kg
According to the question 9
16 × 100 = 56 4
1 k=10kg
( x+20 100 x
100 × 250) × 125 = 100 × 220 Difference ,A-B=2K=20KG
x = 200
Sol:34.(c)
10% of 250 = 25 25% of (50% of 30% of 150) Sol:39.(c)
15% of 200 = 30 40% of 2250
nn
25% of (50% of 45)
Failed = 8% = 480
Percentage less = 40% of 2250 Total student =
5 2
30 × 100 = 16 3 %
25% of (22.5) 100 × 480 = 6000
900 8
5.625
900
Sol:30.(c) Sol:40.(d)
0.625
Student from school X = x(let) 20% of 360 = 72
Student from school Y = x+ 15% of 180 = 27
Sol:34.(c)
150% of x Difference = 72 - 27 = 45
Let Monthly salary of A and B=
Pi

= 2.5x
x
Total students = 3.5x SSC CPO 2019
A’s donation = 0.1x
Passed from school X = 70% of x Sol:41. (d)
B’s donation = 0.08x
= 0.7x As we know Area ∝ side 2
2400= 0.1x-0.08x =0.02x
Total passed = 80% of 3.5x = Ratio of original side to decreased
x=20,000
2.8x side
Total donation of A and B = 18%
Passed from school Y = 2.8x 100:89
of 20000 =3600
-0.7x = 2.1x 100:89
Donation of C = 2700
Failed from school Y = 0.4x Solving this
Monthly salary of C = 2700
Failed percentage from school Y 10000:7921
0.4x × 100 = 16%
× 100
9 = 30, 000
= 2.5x 100:79.21 (by dividing by 100)
Sol:31(c) Net percentage
Sol:36(b)
30% of anuja property =125000 decrease=100-79.21=20.79%.
Let renu income =x
Anuja property = 12500 × 100
30 Saving = 0.2x

www.ssccglpinnacle.com support@ssccglpinnacle.com Ph. 09729327755, 09817390373 222


Days 24-27 Percentage

s
se
s
la
_c
ob
Sol:42.(b) C = 15% of T = 900 C = 15% of T = 1,350
bo
ah
Expenditure= price × saving = 50,000-(30,000 + 6,000 saving = 75,000-(45,000 + 9,000
sm
ur
yo

consumption + 900) = 13,100 + 1,350) = 19,650


e/
t.m
://

100:116 After increment After increment


tp
ht

X : Y New income = 70,000 New income =1,05,000


E = 60% of the income = 42,000 E = 60% of the income = 63,000
T = 30% of E = 12,600 T = 30% of E = 18,900
100:110
C = 20% of T = 2,520 C = 20% of T =3,780
X:Y=116:110
His saving now = 70000 -(42,000 His saving now = 1,05000
Percentage decrease in
6 × 100=5.17% + 12,600 + 2,520) = 12,880 -(63,000 + 18,900 + 3,780) =
consumption = 116
Change in saving = 13,100 - 19320
12,880 = 220 Ratio of saving = 19,650 : 19,320
43.Sol:(c)
= 655 : 644
According to the question
Sol:47.(b)
110( 100 −X 100+X
100 )=50( 100 ) Let the area of the square = 100 51.Sol:(b)
Solving the equation

e
units 25% of 400 + 35% of 1260 +
We get X=37.5% Side = 10 units 27% of 1800 = 1020 + x
Now we have to find out 37.5%
Diagonal = 10 √2 units 100+441+486=1020+x
of 650 is greater than 57.5% 1027=1020+x
After decrement = 81 units
of180 by how much

l
Side = 9 units x=7 which clearly lies between 6
So to make it simpler we will to 10
ac diagonal = 9 √2 units
simply take the ratio as we have
to find the percentage change Ratio = 10 √2 : 9 √2 = 10 : 9
52.Sol:(a)
Change in diagonal = 1 unit
49% of X = Y,
37.5✕650 Percentage change = 10%
57.5×180 then Y% of 50= 49% of X% of
243.75 50
103.50 50= 49% of X × 100
Sol:48.(c)
Required percentage increase=
243.75−103.50 × 100= 140.25 × Original salary = 7,000
103.50 103.50 =24.5% of X
Increment in salary = 12,000 -
nn
100=135.50%≅136%
7,000 = 5,000
53.Sol:(d)
Percentage increase = 5000
7000 × 100
Sol:44.(c) It’s easier to apply ratio in case of
= 5 = 71 73 %
Salary of A is 60% more than B’s 7 successive percent change
salary 100 :140
Let B’s Salary be 100 Sol:49.(c) 4 :3
A’s Salary will be 160 Let the earning be = 100x 20 : 23
B’s salary is less than A’s salary Saving = 25x 5:4
by 60 Rs Expenditure = 75x
Pi

Percentage reduction in salary = After increment


40000:38640
60 New earning = 128x
160 × 100 =37.5 120 × 75x = 100: 96.6
New expenditure = 100
Net decrease= 100-96.6=3.4%
Sol:45.(a) 90x
Original salary = 8,100 New saving = 128x- 90x = 38x
54.Sol:(a)
Increment in salary = 9,000 - Saving increased by = 38x -25x =
According to the question
8,100 = 900 13x
110( 100−X 100+X
100 )=50( 100 )
900 × 100
Percentage increase = 8100 Increase percentage = 13x
25x × 100 =
Solving the equation
= 1 = 11 91 % 52%
9 We get X=37.5%
Now we have to find out 37.5%
Sol:50.(b)
Sol:46.(d) of 650 is greater than 27.5% of
E = 60% of the income = 45,000
E = 60% of the income = 30,000 780 by how much
T = 20% of E = 9,000
T = 20% of E = 6,000

www.ssccglpinnacle.com support@ssccglpinnacle.com Ph. 09729327755, 09817390373 223


Days 24-27 Percentage

s
se
s
la
_c
ob
So to make it simpler we will
bo
ah
simply take the ratio as we have 59.Sol.(d)
sm
ur
yo

to find the percentage change Let the fraction be a


e/
t.m

b
://

Dividing by 13 and simplifying According to the question


tp
ht

we get a + 0.6a 16
b + 0.4b = 63
15×5
1.6a 16
1.4b = 63
6×11
75
a 2
b = 9
66
Required percentage increase=
9
66 × 100=13.64=14% 60.Sol(a)
Let the salary of B = 100x
55.Sol:(c) Then the salary of A = 135x
Let B’s salary=100 35x × 100
B is less than A by = 135x
A’s salary will be 130 = 25.92%
% reduction in B’s Salary than Approximately = 26%
A’s salary= 130130
−100 × 100

e
=23.15%

56.Sol:(a)
Expenditure= price ×

l
consumption
100:126
X : Y

100:115
X:Y=126:115
ac
Percentage decrease in
11 × 100=8.73%
consumption = 126
nn
57.Sol:.(d)
Total expenditure = 40 + 18 + 12
+ 5 = 75%
Saving = 25%
To meet the expenditure of
20,000 he took a loan of 16,000
Means he saves 4,000 per month
4,000 = 25%
Pi

100% = 16,000

58.Sol:(d)
The pass percentage of different
sections are given.
And the strength of students in
different sections is also given
Pass percentage of entire class=
average=
×sum of (individual pass%×section strength)
total section strength
=
20%×25+ 30%×30+ 35%×40+ 40%×45+60×50%+100×75%
25+30+35+40+50+75
≅50%

www.ssccglpinnacle.com support@ssccglpinnacle.com Ph. 09729327755, 09817390373 224


Days 28-32 Pro it and Loss

s
se
s
la
_c
ob
Profit and Loss/ लाभ और हािन एक व ु की अंिकत कीमत 315 (d)Rs. 1,200
bo
ah पये है | इसे 288 पये म बेचा जाता
sm
ur
yo

है | यिद 4% की हािन होती है , तो इस Q5. Anu allows a 20% discount


e/
t.m

1. There is a Profit when SP


व ु की अंिकत कीमत लागत कीमत
://

on the marked price of an article


tp

(selling price) is more than CP


ht

से िकतना ितशत अिधक रखी गयी and still makes a profit of 25%. If
(cost price)./ जब SP (िब ी मू ),
है ? she gains Rs. 44.80 on the sale of
CP ( य मू ) से अिधक है तो लाभ
SSC CGL 4 June 2019 the article, then the cost price of
होगा|
(Afternoon) the article is: अनु िकसी व ु के
2. Selling Price (SP) – Cost Price
(a)5 अंिकत मू पर 20% छूट दे ती है
(CP) = Profit
(b)8 और िफर भी 25% लाभ कमाती है |
3. Loss when CP is more than SP (c)6 21 यिद उसे व ु को बेचने पर 44.80
i.e. Loss = CP – SP पये का लाभ होता है , तो इस व ु
(d)5 21
4. % Profit = PCP
rof it
×100 का य मू ात कर |
5. % P rof it
Loss = CP ×100 Q3. Ram bought an article for Rs. SSC CGL 4 June 2019
1,240 and sold it at a loss of 25%. (Evening)
6. Profit or Loss is always

e
With this amount, she bought (a)Rs. 188.80
calculated on cost price.
another article and sold it at a gain (b)Rs. 192.80
it /loss
7. S P = C P × 100 ± prof
100 (c)Rs. 184. 20
of 40%. His overall percentage
100
8. C P = S P × 100 ± prof it/loss profit is: (d)Rs. 179.20

l
Note: In case of profit you must राम ने एक व ु 1,240 पये म
ख़रीदा और इसे 25% की हािन पर Q6. Abhi bought two articles for
add and in case of loss subtract in
ac बेच िदया | इस रािश से उसने एक Rs.624. He sold one at a loss of
above two formulas.
अ व ु ख़रीदा और इसे 40% लाभ 14% and the other at a profit of
पर बेच िदया | उसका कुल ितशत 14%. If the selling price of both
Variety Questions
लाभ है : the articles is equal, then the
SSC CGL 4 June 2019 difference between their cost
Q1. A person sold an article at a
(Afternoon) prices (in Rs) is:
loss of 15%. Had he sold it for Rs.
(a)12 अिभ ने दो व ुओं को 624 पये म
30.60 more, he would have gained
ख़रीदा | उसने एक को 14% की हािन
nn
9%. To gain 10%, he should have (b)6 32
तथा दू सरी को 14% लाभ पर बेच
sold it for: (c)5
िदया | यिद दोनों व ुओं का िव य
एक ने कोई व ु 15% हािन (d)15
मू समान है , तो उनके य मू म
पर बेची | यिद उसने इसे 30.60 पये
अंतर ( पये म ) ात कर |
अिधक म बेचा होता, तो उसे 9% का Q4. On selling an article for
SSC CGL 6 June 2019
लाभ होता | 10% का लाभ कमाने के Rs.800, a person loses 20% of its
(Afternoon)
िलए, उसे इस व ु को िकस कीमत selling price. At what price should
(a)89.64
पर बेचना चािहए था ? he sell it to gain 25% on its cost
(b)87.36
Pi

SSC CGL 4 June 2019 price?


(c)89.68
(Morning) िकसी व ु को 800 पये म बेचने
(d)88.84
(a)Rs. 140.25 पर, एक को इसके िव य
(b)Rs. 132 मू पर 20% की हािन होती है | य
Q7.Some fruits are bought at a
(c)Rs. 130 मू पर 25% लाभ कमाने के िलए
rate of 11 for Rs.100 and an equal
(d)Rs. 128.40 उसे इस व ु को िकस कीमत पर
number at a rate of 9 for Rs.100.
बेचना चािहए ?
If all the fruits are sold at a rate of
Q2. The marked price of an article SSC CGL 4 June 2019
10 for Rs.100, then what is the
is Rs. 315. It is sold for Rs. 288. If (Evening)
gain or loss percent in the entire
there is a loss of 4%, then by what (a)Rs. 1,280
transaction?
percent above the cost is the (b)Rs. 1,152
article marked? (c)Rs. 1,250

www.ssccglpinnacle.com support@ssccglpinnacle.com Ph. 09729327755, 09817390373

225
Days 28-32 Pro it and Loss

s
sse
la
_c
ob
कुछ फल 100 पये म 11 की दर से SSC CGL 10 June 2019
bo
ah
ख़रीदे गए तथा इतने ही फल 100 Q10. A person sells an article at a (Afternoon)
sm
ur
yo

पये म 9 की दर से ख़रीदे गए | यिद profit of 12%. If he had purchased (a)Rs.380 gain


e/
t.m

सभी फलों को 100 पये म 10 की


://

it for 12% less and sold it for Rs.9 (b)Rs.380 loss


tp
ht

दर से बेचा जाता है , तो पूरे लेन-दे न म less, he would have gained 27%. (c)Rs.360 loss
लाभ या हािन का ितशत ात कर | What is the original cost price of (d)Rs.360 gain
SSC CGL 6 June 2019 the article?
(Evening) एक िकसी व ु को 12% लाभ Q13. If the selling price of 40
(a)Gain, 5% पर बेचता है | यिद उसने इसे 12% articles is equal to the cost price
(b)Loss, 5% कम म ख़रीदा होता और इसे 9 पये of 50 articles, then the percentage
(c)Gain, 1% कम म बेचा होता, तो उसे 27% का loss or gain is:
(d)Loss, 1% लाभ होता | इस व ु का वा िवक यिद 40 व ुओं का िव य मू 50
य मू ात कर | व ुओं के य मू के बराबर है , तो
Q8. Sushma bought 6 tables and SSC CGL 7 June 2019 हािन या लाभ का ितशत है :
12 chairs for Rs.12000. She sold (Evening) SSC CGL 13 June 2019

e
the tables at a profit of 15% and (a)Rs.4,250 (Evening)
the chairs at a loss of 10%. If her (b)Rs.4,000 (a)25% gain
total gain was 300, then the total (c)Rs.4,500 (b)25% loss
cost of the tables was: (d)Rs.3,750 (c)20% gain
सुषमा ने 6 मेज और 12 कुिसयों को

l
(d)20% loss
12000 पये म य िकया | उसने Q11.The marked price of an
ac
मेजों को 15% लाभ पर तथा कुिसयों
को 10% हािन पर बेच िदया | यिद
उसका कुल लाभ 300 था, तो मेजों की
कुल कीमत ात कर |
SSC CGL 7 June 2019
article is Rs.550. A shopkeeper
allows a discount of 20% and still
gets a profit of 10%. If he sells it
for Rs.470, his profit percentage
will be:
Q14. A vendor bought 40 dozen
of fruits for Rs2400. Out of these,
30 fruits were rotten and thrown
away. At what rate per dozen
should he sell the remaining fruits
(Morning) एक व ु का अंिकत मू 550 पये to make a profit of 25%?
(a)Rs.6000 है | एक दु कानदार 20% की छूट दे ता एक िव े ता ने 40 दजन फल 2400
(b)Rs.5000 है और िफर भी 10% लाभ कमाता है | पये म ख़रीदे | इनम से 30 फल सड़े
nn
(c)Rs.5400 यिद वह इसे 470 पये म बेचता है , ए थे जो फक िदए गए | 25% का
(d)Rs.4800 तो लाभ का ितशत ा होगा ? लाभ कमाने के िलए उसे शेष फलों
SSC CGL 10 June 2019 को ित दजन िकस दर से बेचना
Q9. A person sold 25 articles for (Morning) चािहए ?
Rs.2500 and incurred a loss of (a)16.8 SSC CHSL 3 July 2019
10%. How many articles should (b)18 (Morning)
he sell for Rs2400 to make a (c)17.5 (a) Rs 84
profit of 20%? (d)16 (b) Rs 72
Pi

एक ने 2500 पये म 25 (c) Rs 90


व ुएँ बेची और उसे 10% की हािन Q12. Two articles are sold for (d) Rs 80
ई | उसे 2400 पये म िकतनी Rs.9,720 each. On one, the seller
व ुएँ बेचनी चािहए तािक 20% का gains 8% and on the other, he Q15. A shopkeeper marks his
लाभ हो ? loses 10%. What is his overall goods at 25% above the cost
SSC CGL 7 June 2019 gain or loss? price. He sells three-fourth of the
(Afternoon) दो व ुएँ 9720 पये म एक की दर goods at the marked price and the
(a)15 से बेची जाती ह | पहली व ु पर, remaining at 40% discount on the
(b)20 िव े ता को 8% का लाभ होता है तथा marked price. His gain/loss
(c)16 दू सरी पर उसे 10% की हािन होती है | percent is:
(d)18 उसका कुल लाभ या हािन ा है ?

www.ssccglpinnacle.com support@ssccglpinnacle.com Ph. 09729327755, 09817390373

226
Days 28-32 Pro it and Loss

s
sse
la
_c
ob
एक दु कानदार अपनी व ुओं की (c) 5% profit एक दु कानदार अपनी व ुओं की
bo
ah
कीमत य मू से 25% अिधक (d) No profit no loss कीमत इस कार रखता है िक 25%
sm
ur
yo

रखता है | वह तीन-चौथाई व ुएँ छूट दे ने के बाद उसे x% लाभ होता है


e/
t.m

अंिकत मू पर तथा शेष व ुएँ | यिद इस व ु का य मू तथा


://

Q18. A bought 38 kg. Rice at Rs.


tp
ht

अंिकत मू से 40% छूट पर बेचता 54.50/kg, 45 Kg rice at Rs. 62/kg, अंिकत मू मशः 460 पये और
है | उसके लाभ या हािन का ितशत है and 55 kg rice at Rs. 48/kg . He 736 पये है , तो x का मान ात कर |
: sold the mixture at Rs. 65/kg. His SSC CPO 13 March 2019
SSC CHSL 4 July 2019 loss or profit percentage is: (Morning)
(Morning) A ने 54.50 पये ित िकलो की दर (a) 20%
(a) Loss, 8.75% से 38 िकलो ाम चावल, 62 पये (b) 18%
(b) Loss, 12.5% ित िकलो की दर से 45 िकलो चावल (c) 24%
(c) Gain, 10.5% और 48 पये ित िकलो की दर से (d) 16%
(d) Gain, 12.5% 55 िकलो ाम चावल ख़रीदा | उसने
िम ण को 65 पये/िकलो ाम की Q21. A manufacturer sells the
Q16. A person purchased a दर से बेचा | उसके लाभ या हािन का product to a wholesaler at 6%
ितशत है -

e
vehicle for Rs. 5,90,828 and sold profit, the wholesaler sells the
it for Rs. 6,52,920. What is the SSC CPO 16 March 2019 product to a retailer at 8% profit
profit percent he earned on this (Morning) and the retailer sells the product to
vehicle (correct to two decimal (a) Loss 1.04 / हािन 1.04 his customer at 10% profit. The
(b) Loss 1.7/ हािन 1.7

l
places)? price paid by the customer is rs
एक ने 5,90,828 पये म एक (c) Profit 19.6/ लाभ 19.6 31,482. The cost of the product to
ac
वाहन ख़रीदा और इसे 6,52,920
पये म बेच िदया | इस वाहन पर उसे
िकतने ितशत ( दशमलव के दो
थान तक ) का लाभ आ ?
SSC CHSL 5 July 2019
(d) Profit 16.8 / लाभ 16.8

Q19. Three partners A, B and C


share profit and losses in the ratio
3:4:7 If the profit for the year
the manufacturer is:
एक िनमाता थोक ापारी को 6%
लाभ पर व ु बेचता है | थोक ापारी
इसे खुदरा ापारी को 8% लाभ पर
बेचता है तथा खुदरा ापारी इसे
(Afternoon) before charging 30% tax is Rs. ाहक को 10% लाभ पर बेच दे ता है |
(a) 10.51% 1,10, 166 What is B’s share of ाहक के ारा दी गयी कीमत 31,482
(b) 9.55% profit after tax? पये है , तो िनमाता के िलए इस
nn
(c) 9.51% तीन साझेदार A, B और C लाभ और उ ाद की कीमत ा होगी ?
(d) 11.55% हािन का अनुपात 3: 4: 7 म बां टते ह SSC CPO 14 March 2019
यिद 30% कर लगाने से पहले वािषक (Morning)
Q17. A man bought three articles लाभ 1,10, 166 है ,तो कर के बाद B (a)28,306
for Rs 3,000 each. He sold the का लाभां श ा होगा ? (b)26,524
articles respectively at 10% profit, SSC CPO 16 March 2019 (c)25,000
5% profit and 15% loss. The total (Morning) (d)26,980
percentage profit/loss he earned (a) Rs.9442.80
Pi

is: (b) Rs.31476 Q22. A cyclist sells 3 cycles at the


एक ने तीन व ुओं म से (c) Rs.22033.20 loss of Rs. 129 per bicycles and
ेक को 3000 पये म ख़रीदा | (d) Rs. 24673.10 12 cycles on the profit of 516 per
उसने इन व ुओं को मशः 10% cycle. If the total profit on all the
लाभ, 5% लाभ और 15% हािन पर Q20. A shopkeeper marks an bicycles sold is 15%, then what
बेचा | उसके ारा ा कुल article at a price such that after will be the cost per cycle?
लाभ/हािन ात कर | giving a discount of 25%, the एक साईिकल िव े ता 516 ित
SSC CHSL 8 July 2019 gains x%. If the cost price and the साईिकल के लाभ पर 12 साइिकलों
(Evening) marked price of the article are Rs और 129 ित साईिकल के नुकसान
(a) 10% loss 460 and Rs 736 respectively, what पर 3 साइिकले बेचता है | िब ी की
(b) 5% loss is the value of x? गयी सभी साइिकलों पर यिद कुल

www.ssccglpinnacle.com support@ssccglpinnacle.com Ph. 09729327755, 09817390373

227
Days 28-32 Pro it and Loss

s
sse
la
_c
ob
लाभ ितिशत 15% है तो ित A ने िकसी व ु को 15% लाभ पर (a)Rs 145
bo
ah
साईिकल का मू ा होगा ? 1495 म बेचा | उसने एक अ व ु (b)Rs 125
sm
ur
yo

SSC CPO 14 March 2019 खरीदी िजसका लागत मू पहले (c)Rs 115
e/
t.m

खरीदी गई व ु के बराबर है | उसने


://

(Evening) (d) Rs 135


tp
ht

(a) Rs 2960 इस नई व ु को 10% के लाभ पर


(b) Rs 3870 बेचा | A ने कुल िकतना लाभ कमाया? Q28. An article is marked 37.5%
(c) Rs 2580 SSC MTS 5 August 2019 above the cost price. If the
(d) Rs 4440 (Afternoon) discount of 9.09% is given, then
(a)Rs 325 find the profit percentage ( to the
Q23. If the selling price of an (b)Rs 450 nearest integer ).
article is 1 52 of its cost price, the (c)Rs 375 िकसी व ु पर लागत मू से 37.5%
percentage gain is : (d) Rs 425 अिधक कीमत अंिकत िकया जाता है |
यिद िकसी व ु का िव य मू यिद 9.09% की छूट दी जाती है तो
इसके य मू का 1 52 है , तो Q26. The cost prices of article A लाभ ितशत (िनकटतम पूणाक म)
ितशत म िकतना लाभ होगा ? and article B are Rs. 1200 and Rs. है :

e
SSC CPO 15 March 2019 1600 respectively. The selling SSC MTS 13 August 2019
(Evening) price of article A is Rs. 1380 and (Morning)
(a)40 the total profit after selling both (a)25
(b)25 the articles is 25%. What is the (b)15

l
(c)20 profit percentage on the article B ? (c)10
(d)47 व ु A और व ु B के लागत मू (d) 20
ac
Q24.An article is sold for Rs 6500
so as to earn a profit of 4%. A
second article whose cost price is
Rs 3750, is sold at a loss of 4%.
मशः 1200 एवं 1600 है | व ु
A का िव य मू 1380 है तथा
दोनों व ुओं को बेचने पर कुल लाभ
25% है | व ु B को बेचने पर लाभ
ितशत िकतना है ?
Q29. A shopkeeper earns the same
percent of profit as well as loss by
selling two similar pieces of
furniture for Rs 18000 and Rs
What is the overall gain or loss SSC MTS 5 August 2019 10000, respectively. At what price
percent in the whole transaction? (Afternoon) should he sell it to earn a profit of
एक व ु 6500 पये म बेची गयी (a)27.5 50%?
nn
तािक 4% का लाभ हो | एक अ (b)32.5 एक दु कानदार को दो समान फन चर
व ु िजसका य मू 3750 पये (c)29.5 मशः 18000 पये और 10000
है , उसे 4% की हािन पर बेचा गया | (d) 35.5 पये म बेचने पर समान ितशत का
पूरे लेन-दे न म कुल लाभ या हािन का लाभ और समान ितशत की हािन
ितशत ात कर | Q27. P sold an article to Q at a होती है | 50% का लाभ कमाने के
SSC MTS 5 August 2019 profit of 20%. Q sold the same िलए उसे इसे िकस कीमत पर बेचना
(Morning) article to R at a loss of 25%. R चािहए ?
sold the same article to T at a SSC MTS 14 August 2019
Pi

(a)Gain 45%
(b)Loss 1% profit of 50%. If P bought the (Afternoon)
(c)Loss 4% article at Rs 100, then at what (a)Rs 21,000
(d) Gain 1% price did T buy that article? (b)Rs 20,250
P ने एक व ु Q को 20% लाभ पर (c)Rs 21,750
Q25. A sold an article on 15% बेची | Q ने यही व ु R को 25% हािन (d)Rs 19,500
profit for Rs. 1495. He purchased पर बेच दी | R ने इसी व ु को T को
another article whose cost price 50% लाभ पर बेचा | यिद P ने यह Q30. An article is sold for Rs x. If
was equal to that of the first व ु 100 पये म खरीदी थी, तो T के it is sold at 33 31 % of this price,
article. He sold this new article at िलए इस व ु की कीमत ा रही ? there is a loss of 20%. What is the
10% profit. What is the total profit SSC MTS 6 August 2019 percentage profit when it is sold
of A ? (Evening) for Rs x?

www.ssccglpinnacle.com support@ssccglpinnacle.com Ph. 09729327755, 09817390373

228
Days 28-32 Pro it and Loss

s
se
s
la
_c
ob
कोई व ु x पये म बेची जाती है | (d) Rs 420 SSC CGL Tier 2 11 September
bo
ah
यिद इसे इस कीमत के 33 31 % पर 2019 (Morning)
sm
ur
yo

बेचा जाता, तो 20% की हािन होती | Q3. A shopkeeper allows a 28% (a)11.25
e/
t.m
://

इसे x पये म बेचने पर होने वाला discount on the marked price of an (b)10
tp
ht

ितशत लाभ ात कर | article and still makes a profit of (c) 11.75


SSC CHSL 3 July 2019 20%. If he gains Rs 30.80 on the (d) 12.75
(Evening) sale of one article, then what will
(a) 140 be the cost price of the article? Q6. An article is sold at a certain
(b) 125 एक दु कानदार िकसी व ु के अंिकत price. If it is sold at 33 31 % of this
(c) 130 मू पर 28% की छूट दे ता है पर price, there is a loss of 33 31 %.
(d) 120 िफर भी 20% का लाभ कमाता है | What is the percentage profit
यिद उसे एक व ु बेचने पर 30.80 when it is sold at 60% of the
SSC CGL TIER II पये का लाभ होता है , तो इस व ु original selling price?
का य मू ात कर | एक व ु िकसी िनि त कीमत पर
Q1. A shopkeeper bought 120 SSC CGL Tier 2 11 September बेची जाती है | यिद इसे इस कीमत के

e
quintals of wheat, 20% of it was 2019 (Morning) 33 31 % पर बेचा जाए, तो 33 31 % की
sold at 25% loss. At what percent (a)Rs 164
हािन होती है | यिद इसे आरं िभक
gain should he sell the rest to gain (b)Rs 145
िव य मू के 60% पर बेचा जाए, तो
25% on the whole transaction? (c) Rs 160
िकतने ितशत का लाभ होगा ?

l
एक दु कानदार ने 120 ं टल गे ँ (d) Rs 154
SSC CGL Tier 2 11 September
ख़रीदा | इसकी 20% मा ा 25% हािन
ac 2019 (Morning)
पर बेची गयी | पूरे लेन-दे न पर 25% Q4. When an article is sold for Rs
(a) 20
का लाभ कमाने के िलए उसे शेष 355, there is a loss of 29%. To
(b)30
मा ा को िकतने ितशत लाभ पर gain 21%, it should be sold for Rs:
(c) 33 31
बेचना चािहए ? जब कोई व ु 355 पये म बेची
(d)17 31
SSC CGL Tier 2 11 September जाती है , तो 29% की हािन होती है |
2019 (Morning) 21% का लाभ कमाने के िलए, इसे
िकस कीमत पर बेचा जाना चािहए ? Q7. An article is sold at a certain
(a)36 21
SSC CGL Tier 2 11 September price. If it is sold at 80% of this
nn
(b)40
2019 (Morning) price, then there will be a loss of
(c) 37 21
(a)629.20 10%. What is the percentage
(d) 35 profit when the article is sold at
(b)580.80
(c) 605 the original selling price?
Q2. Anu sold an article for Rs 480 एक व ु िकसी िनि त कीमत पर
(d) 635
at some profit. Had she sold it for बेची जाती है | यिद इसे इस कीमत के
Rs 400, then there would have 80% पर बेचा जाए, तो 10% की हािन
Q5. Radha marks her goods 25%
been a loss equal to one-third of होगी | व ु को आरं िभक िव य मू
above the cost price. She sells
Pi

the initial profit. What was the पर बेचने पर होने वाला ितशत लाभ
35% of goods at the marked price,
cost price of the article? ात कर |
40% at 15% discount and the
अनु ने एक व ु कुछ लाभ पर 480 SSC CGL Tier 2 12 September
remaining at 20% discount. What
पये म बेच दी | यिद उसने इसे 400 2019 (Morning)
is her overall percentage gain?
पये म बेचा होता, तो आरं िभक लाभ (a)15 21
राधा अपनी व ुओं की कीमत य
के एक-ितहाई के बराबर हािन ई
मू से 25% अिधक रखती है | वह (b)12 21
होती | व ु का य मू ात कर |
35% व ुएँ अंिकत मू पर, 40% (c) 15
SSC CGL Tier 2 11 September
व ुएँ 15% की छूट पर तथा शेष (d) 12
2019 (Morning)
व ुएँ 20% की छूट पर बेचती है |
(a)Rs 450
उसका कुल ितशत लाभ ात कर | Q8. The marked price of an article
(b)Rs 430
is Rs 800 and it is sold at a
(c) Rs 425

www.ssccglpinnacle.com support@ssccglpinnacle.com Ph. 09729327755, 09817390373

229
Days 28-32 Pro it and Loss

s
se
s
la
_c
ob
discount of 19%. If there is a gain Rs 536.25, then the profit/loss
bo
ah
of 8%, then by what percent Q11. A man sells his goods at a percent would have been:
sm
ur
yo

above the cost price was the certain price, 20% of which is his एक व ु 14% के लाभ पर बेची गयी |
e/
t.m

यिद इसे 121 पये कम पर बेचा


://

article marked? profit. If the price at which he


tp
ht

एक व ु का अंिकत मू 800 पये buys the goods increases by 10% जाता, तो 8% की हािन ई होती | यिद
है और इसे 19% की छूट पर बेचा and he sells them at an 8% higher इसी व ु को 536.25 पये म बेचा
जाता है | यिद 8% का लाभ होता है , price, then what will be his profit जाए, तो लाभ या हािन का ितशत
तो इस व ु की कीमत य मू से percent (correct to one decimal ा होगा ?
िकतना ितशत अिधक अंिकत की place)? SSC CGL Tier 2 13 September
गयी है ? एक अपनी व ुएँ एक िनि त 2019 (Morning)
SSC CGL Tier 2 12 September कीमत पर बेचता है , िजसका 20% वह (a)Profit, 5%
2019 (Morning) लाभ कमाता है | यिद िजस कीमत पर (b)Loss, 5%
(a)33 31 वह व ुएँ खरीदता है , वह 10% बढ़ (c) Loss, 2.5%
(b)35 जाए और वह इन व ुओं को 8% (d) Profit, 2.5%
(c) 27 अिधक कीमत पर बेचे, तो उसके
लाभ का ितशत ( एक दशमलव

e
(d) 36 32 Q14. A shopkeeper allows an 18%
थान तक सही ) ा होगा ? discount on the marked price of an
Q9. By selling two articles for Rs SSC CGL Tier 2 12 September article and still makes a profit of
800, a person gains the cost price 2019 (Morning) 23%. If he gains Rs 18.4040 on

l
of three articles. The profit (a)21.8 the sale of the article, then what is
percent is: (b)23.4 the marked price of the article?

एक
मू
ac
800 पये म दो व ुओं को बेचने पर
को तीन व ुओं के
के बराबर लाभ होता है | लाभ
का ितशत है :
SSC CGL Tier 2 12 September

(c) 21.4
(d) 22.7

Q12. 35% of goods were sold at a


profit of 65%, while the remaining
एक दु कानदार िकसी व ु के अंिकत
मू पर 18% की छूट दे ता है और
िफर भी 23% लाभ कमाता है | यिद
उसे व ु की िब ी पर 18.4040 का
लाभ होता है , तो इस व ु का अंिकत
2019 (Morning) were sold at x% loss. If the overall मू ात कर |
(a) 125 loss is 12%, then what is the value SSC CGL Tier 2 13 September
of x? (correct to one decimal 2019 (Morning)
nn
(b) 140
(c) 120 place) (a)Rs 140
(d) 150 35% व ुएँ 65% लाभ पर बेची गयीं (b)Rs 125
जबिक शेष व ुएँ x% हािन पर बेची (c) Rs 120
Q10. When an article is sold at its गयीं | यिद कुल हािन 12% है , तो x (d) Rs 146
marked price, it gives a profit of का मान ( दशमलव के एक थान तक
25%. If a discount of 9.6% is सही ) ात कर | Q15. Sudha bought 80 articles at
allowed on the marked price, then SSC CGL Tier 2 13 September the same price. She sold some of
2019 (Morning) them at 8% profit remaining at
Pi

the profit percent will be:


जब कोई व ु अंिकत मू पर बेची (a)51.8 12% loss resulting in an overall
जाती है , तो 25% का लाभ होता है | (b)50.6 profit of 6%. The number of items
यिद अंिकत मू पर 9.6% की छूट (c) 53.5 sold at 8% profit is:
दी जाए, तो लाभ का ितशत ा (d) 52.4 सुधा ने समान कीमत पर 80 व ुओं
होगा ? को य िकया | उसने उनम से कुछ
SSC CGL Tier 2 12 September Q13. An article was sold at a को 8% लाभ पर तथा शेष को 12%
2019 (Morning) profit of 14%. Had it been sold for हािन पर बेचा िजसके प रणाम प
(a)13 Rs 121 less, a loss of 8% would कुल 6% का लाभ आ | 8% लाभ पर
(b)15.4 have been incurred. If the same बेची गयी व ुओं की सं ा ात कर
(c) 15 article would have been sold for |
(d) 16.6

www.ssccglpinnacle.com support@ssccglpinnacle.com Ph. 09729327755, 09817390373

230
Days 28-32 Pro it and Loss

s
sse
la
_c
ob
SSC CGL Tier 2 13 September सुजाता अपनी व ुओं की कीमत य Practice Questions
bo
ah
2019 (Morning) मू से 36% अिधक रखती है और
sm
ur
yo

(a)64 अंिकत मू पर 40% की छूट दे ती Q1. A person sold an article at a


e/
t.m

है । हािन का ितशत है :
://

(b)60 loss of 8%. Had he sold it at a


tp
ht

(c) 72 SSC CGL Tier II 12 September gain of 10.5%. he would have


(d) 70 2019 received Rs.92.50 more. To gain
(a) 15 12%, he should have sold it for:
Q16.If the selling price of an (b) 16.8 एक ने कोई व ु 8% हािन पर
article is 32% more than its cost (c) 18.4 बेच दी | यिद उसने इसे 10.5% लाभ
price and the discount on its (d) 4 पर बेचा होता, तो उसे 92.50 पये
marked price is 12%, then what is अिधक िमले होते | 12% लाभ कमाने
the ratio of its cost price to the Q19. A person sells an article at के िलए, उसे इसे िकस कीमत पर
marked price? 16% below its cost price. Had he बेचना चािहए था ?
यिद िकसी व ु का िव य मू sold it for Rs.33 more, he would SSC CGL 6 June 2019
इसके य मू से 32% अिधक है have gained 14%. To gain 25%, (Morning)
और इसके अंिकत मू पर दी जाने

e
he should sell the article for : (a)Rs. 540.50
वाली छूट 12% है , तो इसके य मू एक िकसी व ु को इसके य (b)Rs.560
और अंिकत मू के बीच अनुपात मू से 16% कम पर बेचता है | यिद (c)Rs.580
ात कर | उसने इसे 33 पये अिधक म बेचा (d)Rs.537.40
होता, तो उसे 14% का लाभ होता |

l
SSC CGL Tier 2 13 September
2019 (Morning) 25% का लाभ कमाने के िलए, उसे Q2. A bought an article for
(a)4:5
(b)3:8
(c) 2:3
(d) 1:2
ac इस व ु को िकस कीमत पर बेचना
चािहए ?
SSC CGL TIER II 12
September 2019
(a) Rs. 128
Rs.5400 and sold it at a loss of
30%. With this amount, he bought
another article and sold it at a gain
of 60%. What was his overall
percentage gain or percentage
Q17. A person marks his goods (b) Rs. 137.5 loss?
x% above the cost price and (c) Rs. 135 A एक व ु 5400 पये म ख़रीदा
allows a discount of 30% on the (d) Rs. 130.5 और इसे 30% हािन पर बेच िदया |
nn
marked price. If his profit is 5%, इस रािश से उसने एक अ व ु
then the value of x will be: Q20. How many kg of salt costing ख़रीदा और उसे 60% लाभ पर बेच
एक अपनी व ुओं की कीमत Rs. 28 per kg must be mixed with िदया | उसका कुल ितशत लाभ या
य मू से x% अिधक रखता है 6.6 kg of salt costing Rs. 16 per हािन ात कर |
और अंिकत मू पर 30% की छूट kg, so that selling the mixture at SSC CGL 10 June 2019
दे ता है । यिद उसका लाभ 5% है , तो x Rs. 29.90, there is a gain of 15%? (Morning)
का मान ा होगा? 28 पये ित िकलो ाम लागत वाले (a)Gain, 1.2%
SSC CGL Tier II 11 September िकतने िकलो नमक को 16 पये ित (b)Gain, 12%
Pi

2019 िकलो ाम की लागत वाले 6.6 िकलो (c)Loss, 12%


(a) 50 नमक म िमलाना चािहए तािक िम ण (d)Loss, 1.2%
(b) 60 को 29.90 पये ित िकलो की दर से
(c) 45 बेचने पर 15% का लाभ हो ? Q3. Two articles are sold for Rs.
(d) 35 SSC CGL TIER II 13 10,384 each. On one, the seller
September 2019 gains 18% and on the other, he
Q18. Sujata marks an article 36% (a) 33 loses 12%. What is his overall
above the cost price and allows a (b) 31 gain or less ?
40% discount on the marked (c) 35 दो व ुओं म से ेक को 10384
price. The loss percentage is: (d) 32 पये म बेचा जाता है | पहली पर,
िव े ता को 18% का लाभ होता है

www.ssccglpinnacle.com support@ssccglpinnacle.com Ph. 09729327755, 09817390373

231
Days 28-32 Pro it and Loss

s
sse
la
_c
ob
तथा दू सरी पर उसे 12% की हािन Q6. Two articles are sold for हािन होती है | िव े ता को कुल
bo
ah
होती है | कुल लाभ या हािन ात कर | Rs.2508 each. On one, there is a िमलाकर िकतने ितशत लाभ अथवा
sm
ur
yo

SSC CGL 10 June 2019 gain of 14% and on the other, हािन ई?
e/
t.m
://

(Evening) there is a loss of 12%. What is the SSC CGL 12 June 2019
tp
ht

(a)Rs.178 loss overall gain or loss percent to (Afternoon)


(b)Rs.168 loss nearest one decimal place? दो (a)3.6% profit / लाभ
(c)Rs.178 gain व ुएँ 2508 पये म एक की दर से (b)3.4% loss / हािन
(d)Rs.168 gain बेची जाती ह | पहली व ु पर, 14% (c)3.6% loss/ हािन
का लाभ होता है तथा दू सरी पर 12% (d)3.4% profit / लाभ
Q4. Two articles are sold for की हािन होती है | एक दशमलव थान
Rs.10,005 each. On one, the seller के समीप कुल लाभ या हािन का Q9. Two articles are sold for
gains 15% and on the other, he ितशत ात कर | Rs.975 each. On one, the seller
loses 13%. What is his overall SSC CGL 11 June 2019 gains 30% and on the other, he
gain or loss percent, correct two (Evening) loses 25%. What is the overall
decimal places? (a)0.7% gain gain or loss percentage, correct to
दो व ुएँ 10,005 पये म एक की दर

e
(b)0.7% loss one decimal place?
से बेची जाती ह | पहली व ु पर, (c)0.5% gain दो व ुएँ 975 पये म एक की दर से
िव े ता को 15% लाभ होता है तथा (d)0.5% loss बेची जाती ह | पहली व ु पर िव े ता
दू सरी पर उसे 13% की हािन होती है | को 30% लाभ होता है तथा दू सरी
दो दशमलव थानों तक उसके कुल व ु पर उसे 25% की हािन होती है |

l
Q7. Two articles are sold for
लाभ या हािन का ितशत ात कर | Rs.4,956 each. On one, the seller कुल लाभ या हािन का ितशत ात

(Morning)
(a)1.42% gain
(b)1.42% loss
(c)0.94% loss
ac
SSC CGL 11 June 2019 gains 18% and on the other he
loses 16%. What is his overall
gain or loss percent to the nearest
one decimal place?
दो व ुएँ 4956 पये म एक की दर
कर | ( एक दशमलव थान तक सही
)
SSC CGL 12 June 2019
(Evening)
(a)4.9% loss
(d)0.94% gain से बेची जाती ह | पहली व ु पर (b)5.3% gain
िव े ता को 18% का लाभ होता है (c)4.9% gain
Q5. Two articles are sold for Rs. तथा दू सरी पर उसे 16% की हािन (d)5.1% loss
nn
5,104 each. On one, the seller होती है | एक दशमलव थान तक
gains 16% and on the other, he उसका कुल लाभ या हािन ितशत Q10. Two articles are sold for
loses 12%. What is his overall ात कर | Rs.4,752 each. On one, the seller
gain percent, nearest to two SSC CGL 12 June 2019 gains 32% and on the other he
decimal places? (Morning) loses 28%. What is his overall
दो व ुएँ 5104 पये म एक की दर (a)2.1% loss gain or loss percentage, correct to
से बेची जाती ह | पहली व ु पर (b)1.9% loss one decimal place?
िव े ता को 16% लाभ होता है तथा (c)2.1% gain दो व ुएँ 4752 पये म एक की दर
Pi

दू सरी पर उसे 12% की हािन होती है | (d)1.9% gain से बेची जाती ह | पहली पर, िव े ता
दो दशमलव थानों तक उसका कुल को 32% लाभ होता है और दू सरी पर
लाभ ितशत ात कर | Q8. Two articles are sold for Rs. उसे 28% की हािन होती है | उसके
SSC CGL 11 June 2019 4880 each. On one, the seller कुल लाभ या हािन का ितशत ( एक
(Afternoon) gains 22% and on the other he दशमलव थान तक सही ) ात कर |
(a)0.08% loses 20%. What is his overall SSC CGL 13 June 2019
(b)0.12% gain or loss percent ? (Morning)
(c)0.14% दो व ुओं म से ेक को 4880 (a)7.3% gain
(d)0.10% के मू पर बेचा जाता है | उनम से (b)7.3% loss
एक व ु पर िव े ता को 22% लाभ (c)6.8% loss
होता है और दू सरी व ु पर 20% की (d)6.8% gain

www.ssccglpinnacle.com support@ssccglpinnacle.com Ph. 09729327755, 09817390373

232
Days 28-32 Pro it and Loss

s
sse
la
_c
ob
(d) 11 91 % (a) Profit, Rs 140/ लाभ, 140 पये
bo
ah
Q11. Two articles are sold for
sm
ur
yo

Rs.962 each. On one, the seller Q14. A person sold an article at a (b) Loss, Rs 125/ हािन, 125 पये
e/
t.m

(c) Profit, Rs 180/ लाभ, 180 पये


://

gains 30% and on the other he loss of 8%. Had he sold it at a


tp
ht

loses 26%. What is his overall gain of 10.5%. He would have (d) Loss, Rs 130/ हािन, 130 पये
gain or loss percentage, nearest to received Rs 37 more. What is the
one decimal place? cost price of the article? Q17. By selling an article for
दो व ुएँ 962 पये म एक की दर से एक ने कोई व ु 8% की हािन Rs2,300, Rekha gains 25%. If she
बेची जाती ह | पहली पर, िव े ता को पर बेची | यिद उसने इसे 10.5% के sells it for Rs1955, then her
30% लाभ होता है और दू री व ु पर लाभ पर बेचा होता, तो उसे 37 पये loss/gain percent is:
उसे 26% की हािन होती है | उसके अिधक ा ए होते | इस व ु का िकसी व ु को 2300 पये म बेचने
कुल लाभ या हािन का ितशत ( एक य मू ा है ? पर रे खा को 25% का लाभ होता है |
दशमलव थान के समीप ) ात कर | SSC CHSL 2 July 2019 यिद वह इसे 1955 पये म बेचे, तो
SSC CGL 13 June 2019 (Morning) उसके लाभ/हािन का ितशत होगा :
(Afternoon) (a) Rs 200 SSC CHSL 3 July 2019

e
(a)6.0% gain (b) Rs 250 (Afternoon)
(b)5.7% loss (c) Rs 240 (a) Loss, 6.5%
(c)5.7% gain (d) Rs 210 (b) Gain, 6.5%
(d)6.0% loss (c) Gain, 6.25%

l
Q15. When an article is sold for (d) Loss, 6.25%
Q12. The marked price of an Rs 291, there is a loss of 3%.
ac
article is Rs 600. After allowing a
discount of 25% on the marked
price, there was a loss of Rs 30.
The loss percentage is:
एक व ु का अंिकत मू 600 पये
What will be the selling price of
the article, if it is sold at a gain of
8%?
जब िकसी व ु को 291 पये म बेचा
जाता है , तो 3% की हािन होती है |
Q18. The marked price of an
article is Rs400. After allowing a
discount of 20% on the marked
price, a shopkeeper makes a profit
of Rs 32. His gain percent is:
है | अंिकत मू पर 25% की छूट दे ने यिद इसे 8% लाभ पर बेचा जाए, तो िकसी व ु का अंिकत मू 400
के बाद, 30 पये की हािन होती है | इस व ु का िव य मू ा होगा ? पये है | अंिकत मू पर 20% की
हािन का ितशत है : SSC CHSL 2 July 2019 छूट दे ने के बाद एक दु कानदार को
nn
SSC CHSL 1 July 2019 (Afternoon) 32 पये का लाभ होता है | उसके
(Evening) (a) Rs308 लाभ का ितशत है :
(a) 7.50% (b) Rs332 SSC CHSL 3 July 2019
(b) 7.25% (c) Rs324 (Afternoon)
(c) 6.25% (d) Rs316 (a) 9
(d) 6.50% (b) 11 91
Q16. Abhi sold two articles for Rs (c) 12 91
Q13. By selling 72 articles, a loss 5,220 each. On one, he gained
Pi

(d) 8
equal to the selling price of 8 16% and on the other, he lost
articles was incurred. What is the 10%. His profit or loss on the Q19. Three articles are bought at
loss percentage? whole was: Rs200 each. One of them is sold
72 व ुएँ बेचने पर 8 व ुओं के अिभ ने दो व ुओं म से ेक को at a loss of 10%. If the other two
िव य मू के बराबर हािन होती है | 5220 पये म बेचा | पहली व ु पर articles are sold so as to gain 20%
हािन का ितशत ा है ? उसे 16% का लाभ आ तथा दु सुरी on the whole transaction, then
SSC CHSL 1 July 2019 व ु पर उसे 10% की हािन ई | पूरे what is the gain percent on the
(Evening) पर उसे िकतने लाभ या हािन की two articles?
(a) 12% ा ई? तीन व ुओं म से ेक को 200
(b) 10% SSC CHSL 2 July 2019 पये म ख़रीदा गया | उनम से एक
(c) 9 91 % (Evening) को 10% की हािन पर बेचा गया | यिद

www.ssccglpinnacle.com support@ssccglpinnacle.com Ph. 09729327755, 09817390373

233
Days 28-32 Pro it and Loss

s
sse
la
_c
ob
अ दो व ुओं को इस कार बेचा at 10% loss and another at 20% (a) 15.51%
bo
ah
गया िक पूरे लेन-दे न पर 20% का profit. The total profit/loss (b) 11.55%
sm
ur
yo

लाभ हो, तो इन दो व ुओं पर होने percentage he earned is: (c) 19.55%


e/
t.m

वाला ितशत लाभ ात कर | एक ने दो व ुओं म से ेक


://

(d) 12.65%
tp
ht

SSC CHSL 4 July 2019 को 3050 पये म ख़रीदा | उसने एक


(Morning) व ु 10% की हािन पर तथा दू सरी Q25. A person purchased a
(a) 28 व ु 20% लाभ पर बेच दी | उसके vehicle for Rs4,89,828 and sold it
(b) 32 ारा ा कुल लाभ / हािन का for Rs5,89,828. What is the profit
(c) 35 ितशत ात कर | percent he earned on this vehicle
(d) 30 SSC CHSL 5 July 2019 (correct to two decimal places)?
(Morning) एक ने 4,89,828 पये म एक
Q20. A man bought 2 articles for (a) 5% loss वाहन ख़रीदा और इसे 5,89,828 म
Rs 2650 each. He sold one article (b) 10% profit बेच िदया| इस वाहन पर उसे िकतने
at 10% profit and another at 5% (c) 5% profit ितशत का लाभ ( दो दशमलव
profit. The total profit percentage (d) 10% loss थानों तक सही ) आ ?

e
he earned is: SSC CHSL 8 July 2019
एक ने दो व ुओं म से ेक Q23. A man bought 2 articles for (Morning)
को 2650 पये म ख़रीदा | उसने एक Rs4158 each. He sold one article (a) 25%
व ु को 10% लाभ पर तथा दू सरी को at 15% loss. Then at what percent (b) 18.65%
5% लाभ पर बेच िदया | उसके ारा

l
profit the other article should be (c) 20.42%
ा कुल लाभ का ितशत ात कर | sold so that no profit/loss (d) 15%

(Afternoon)
(a) 8%
(b) 8.5%
(c) 7.5%
ac
SSC CHSL 4 July 2019 percentage is earned. एक
दो व ुओं म से ेक को 4158
पये म ख़रीदा | उसने एक व ु
ने

15% की हािन पर बेच दी | दू सरी व ु


को िकतने ितशत लाभ पर बेचना
Q26. A man bought three articles
for Rs6,000 each. He sold the
articles respectively at 15% profit,
12% profit and 15% loss. The
(d) 10% चािहए तािक ना तो लाभ हो ना ही total percentage profit/loss he
हािन ? earned is:
Q21. A man bought 2 articles for SSC CHSL 5 July 2019 एक ने तीन व ुओं म से
nn
Rs. 3050 each. He sold one article (Afternoon) ेक को 6000 पये म ख़रीदा |
at 10% profit and another at 20% (a) 15% उसने इन व ुओं को मशः 15%
profit. The total profit percentage (b) 10% लाभ, 12% लाभ और 15% हािन पर
he earned is: (c) 12% बेचा | उसे कुल िकतने ितशत का
एक ने दो व ुओं म से ेक (d) 18% लाभ या हािन आ/ ई ?
को 3050 पये म ख़रीदा | उसने एक SSC CHSL- 9 July 2019
व ु को 10% लाभ पर तथा दू सरी Q24. A person purchased a (Afternoon)
व ु को 20% लाभ पर बेच िदया | vehicle for Rs 4,90,828 and sold it (a) 4% profit
Pi

उसके ारा ा कुल लाभ का for Rs 5,52,920. What is the (b) 3% loss
ितशत ात कर | percent profit earned on this (c) 4% loss
SSC CHSL 4 July 2019 vehicle (correct to two decimal (d) No profit no loss
(Evening) places)?
(a) 10% एक ने 4,90,828 पये म एक Q27. Two items are sold for
(b) 18% वाहन ख़रीदा और इसे 5,52,920 Rs18,602 each. On one item there
(c) 15% पये म बेच िदया | इस वाहन पर has been a gain of 31% and on the
(d) 20% िकतने ितशत का लाभ ( दो second item a loss of 29%. What
दशमलव थानों तक सही ) आ ? was the overall loss or gain in the
Q22. A man bought 2 articles for SSC CHSL 5 July 2019 transaction?
Rs3050 each. He sold one article (Evening)

www.ssccglpinnacle.com support@ssccglpinnacle.com Ph. 09729327755, 09817390373

234
Days 28-32 Pro it and Loss

s
sse
la
_c
ob
दो व ुओं म से ेक को 18602 एक दु कानदार ने 9831 पये की दर (d) Rs 260
bo
ah
पये म बेचा जाता है | पहली व ु पर से दो व ुएं बेची | पहली पर उसे
sm
ur
yo

31% का लाभ होता है तथा दू सरी 13% लाभ जबिक दू सरी पर उसे 13 Q33. A shopkeeper sold two
e/
t.m

व ु पर 29% की हािन होती है | इस % की हािन ई | कुल िमलाकर


://

articles for Rs. 9471. On one,he


tp
ht

लेन-दे न म कुल हािन या लाभ ात िकतने ितशत का लाभ या हािन आ gained 23% and on the other,he
कर | ? lost 23%.What is the overall
SSC CHSL- 9 July 2019 SSC CPO 12 March 2019 percentage gain or loss? एक
(Evening) (Evening) दु कानदार ने 9471 पये म दो व ुएं
(a) Loss 7.91% (a) 6.5% loss बेची | पहली पर उसे 23% का लाभ
(b) Loss 8.25% (b) 6.5% gain आ तथा दू सरी पर उसे 23% की
(c) Gain 8.25% (c) 1.69% gain हािन ई | कुल िमलाकर लाभ या
(d) Gain 7.91% (d) 1.69% loss हािन का ितशत ा है ?
SSC CPO 12 March 2019
Q28. By selling an article for Q31. A shopkeeper marks his (Morning)
Rs144, a shopkeeper loses 28%. good at a price such that after (a) 5.29% loss

e
What should be the selling price giving a discount of 25%, he gains (b) 6.29% gain
for bringing down the loss to 20%. If the marked price of the (c) 5.29% gain
14%? article is Rs. 736, what is the cost (d) 6.29% loss
िकसी व ु को 144 पये म बेचने पर price of the article? एक दु कानदार
एक दु कानदार को 28% की हािन अपनी व ुओं की कीमत ऐसी रखता

l
Q34. A shopkeeper marks his
होती है | इस हािन को 14% करने के है िक 25% की छूट दे ने के बाद भी goods at a price such that after
िलए िव य मू

(Afternoon)
(a) Rs156
(b) Rs182
ac ा होना चािहए ?
SSC CHSL- 10 July 2019
उसे 20% लाभ होता है | यिद व ु का
अंिकत मू
व ु का य मू
736 पये है , तो इस
ात कर |
SSC CPO 12 March 2019
(Evening)
giving a discount of 25% , he
gains 20%. If the cost price of the
article is Rs. 460, what is its
marked price?
एक दु कानदार अपनी व ुओं की
(c) Rs172 (a) Rs.450 कीमत इस कार रखता है िक 25%
(d) Rs180 (b) Rs.465 छूट दे ने के बाद भी उसे 20% का
(c) Rs.460 लाभ होता है | यिद उस व ु का य
nn
Q29. If the cost price of 4 chairs is (d) Rs.440 मू 460 पये है , तो अंिकत मू
equal to the selling price of 3 ात कर |
chairs, then the profit or loss Q32. A shopkeeper marks the SSC CPO 12 March 2019
percentage is: यिद 4 कुिसयों का price of an article such that after (Morning)
य मू 3 कुिसयों के िव य मू giving a discount of 30%, he gains (a) 736
के बराबर है , तो लाभ या हािन का 20%. If the marked price of the (b) 748
ितशत ात कर | article is Rs 480, what is the cost (c) 725
SSC CPO 16 March 2019 price of the article? (d) 752
Pi

(Morning) एक दु कानदार िकसी व ु की कीमत


(a) 25% इस कार रखता है िक 30% की छूट Q35. A shopkeeper sold two
(b) 20% दे ने के बाद भी उसे 20% लाभ होता है articles for Rs 9639 each. On one,
(c) 33 31 % | यिद उस व ु का अंिकत मू 480 he gained 19% and on the other,
(d) 16 32 % पये है , तो उस व ु का य मू he lost 19%. What is the overall
ा है ? percentage gain or loss?
Q30. A shopkeeper sold two SSC CPO 13 March 2019 एक दु कानदार ने 9639 पये म एक
articles at Rs. 9831 each, On one (Evening) की दर से दो व ुएं बेची | पहली पर,
he gained 13% and on the other he (a) Rs 280 उसे 19% का लाभ आ जबिक दू सरी
lost 13% ,What is the overall (b) Rs 300 पर उसे 19% की हािन ई | कुल लाभ
percentage gain or loss? (c) Rs 250 या हािन का ितशत ात कर |

www.ssccglpinnacle.com support@ssccglpinnacle.com Ph. 09729327755, 09817390373

235
Days 28-32 Pro it and Loss

s
sse
la
_c
ob
SSC CPO 13 March 2019 जाता तो लाभ का ितशत 12% होता िकसी व ु को 25% लाभ तथा
bo
ah
(Morning) | इस घड़ी का य मू ात कर | 37.5% हािन पर बेचने से िव य मू
sm
ur
yo

(a) 3.81% loss SSC CPO 16 March 2019 का अंतर 1250 है | जब इसे 12.5%
e/
t.m

लाभ पर बेचा जाता है तो इसका


://

(b) 3.61% gain (Afternoon)


tp
ht

(c) 3.81% gain (a)3760 िव य मू िकतना होगा?


(d) 3.61% loss (b)3850 SSC MTS 2 August 2019
(c)3945 (Morning)
Q36. A sells a car to B at 10% (d)3900 (a) Rs 1800
loss. If B sells it for rs 5,40,000 (b) Rs 2500
and gains 20%, the cost price of Q39. The selling price of an (c) Rs 2400
the car for A was: article is 2,28,528. A shopkeeper (d) Rs 2250
A, B को एक कार 10% की हािन पर marks its price 15% above its cost
बेचता है | यिद B इसे 5,40,000 पये price and gives a discount of 10%. Q42. An article is sold for Rs.
म बेच कर 20% लाभ कमाता है , तो The cost price is- 1725 at 15% profit. If it is sold at
A के िलए इस कार का य मू एक व ु का िव य मू Rs 2, 28, 15% loss, then what will be its
ात कर | 528 है | एक दु कानदार इसकी कीमत

e
selling price ?
SSC CPO 16 March 2019 उसकी लागत मू से 15% अिधक िकसी व ु को 15% लाभ पर 1725
(Evening) अंिकत करता है और 10% की छूट म बेचा जाता है | यिद इसे 15% की
(a)5,00,000 दे ता है | व ु का लागत मू है : हािन पर बेचा जाता है तो िव य मू
िकतना होगा?

l
(b)5,40,000 SSC CPO 15 March 2019
(c)5,10,000 (Evening) SSC MTS 2 August 2019
(d)5,20,000
ac
Q37. The marked price of a dress
is Rs 2,340 which is 25% above
the cost price. If the dress is sold
(a)2,18,650
(b)2,87,390
(c)2,58,740
(d)2,20,800
(Morning)
(a) Rs 1275
(b) Rs 1475
(c) Rs 1025
(d) Rs 1325
at a profit of 10% the profit Q40.By selling an article for 320,
earned on the dress is: a man incurs a loss of 20%. What Q43. An article is sold for Rs
एक पोशाक अंिकत मू 2,340 should be the selling price of an 2070 at a 15% profit. IF the article
nn
पये है जो य मू से 25% अिधक article to gain 20%? is sold for Rs 1890, then what will
है । यिद पोशाक 10% के लाभ पर 320 पये म एक व ु को बेचने पर be the gain/loss percent?
बेची जाती है , तो पोशाक पर अिजत एक को 20% की हािन होती है एक व ु 15% लाभ पर 2070 पये
लाभ है : | 20% लाभ कमाने के िलए इस व ु म बेची जाती है | यिद इस व ु को
SSC CPO 15 March 2019 का िव य मू ा होना चािहए ? 1890 पये म बेचा जाए, तो
(Morning) SSC MTS 2 August 2019 लाभ/हािन का ितशत ा होगा ?
(a) Rs 234 (Morning) SSC MTS 2 August 2019
(b) Rs 187.20 (a)Rs 450 (Evening)
Pi

(c) Rs 197 (b)Rs 480 (a)10% loss


(d) Rs 175.50 (c)Rs 420 (b)5% loss
(d) Rs 500 (c)10% gain
Q38. A watch was sold at a profit (d) 5% gain
of 10%. Had it been sold at rs 77 Q41. The difference between the
more the profit percent would selling prices of an article when Q44.An article is sold for Rs 810
have been 12% . The cost price of sold at 25% profit and 37.5% loss at a loss of 10%. What should be
the watch is : is Rs. 1250. What will be its the selling price if the loss is
एक घड़ी 10% के लाभ पर बेची गयी | selling price when it is sold at 20%?
यिद इसे 77 पये अिधक पर बेचा 12.5% profit ? एक व ु 10% की हािन पर 810
पये म बेची जाती है | यिद हािन 20%

www.ssccglpinnacle.com support@ssccglpinnacle.com Ph. 09729327755, 09817390373

236
Days 28-32 Pro it and Loss

s
sse
la
_c
ob
है , तो िव य मू िकतना होना SSC MTS 5 August 2019 (d) 20%
bo
ah
चािहए ? (Evening)
sm
ur
yo

SSC MTS 2 August 2019 (a)Rs 450 Q51. If the selling price of an
e/
t.m
://

(Evening) (b)Rs 480 article is 25% of its cost price,


tp
ht

(a)Rs 750 (c)Rs 520 then what will be the loss


(b)Rs 630 (d) Rs 400 percentage?
(c)Rs 600 यिद िकसी व ु का िव य मू
(d) Rs 720 Q48. The difference between the इसके य मू का 25% है , तो हािन
selling prices of an article when का ितशत ा होगा ?
Q45. An article is sold at 14 72 % sold at 20% profit and 18% loss is SSC MTS 6 August 2019
profit. what is the ratio of the Rs. 570. What will be its selling (Evening)
selling price to the cost price? price, when it is sold at 12% loss ? (a)25%
एक व ु 14 72 % लाभ पर बेची जाती िकसी व ु को 20% लाभ पर तथा (b)60%
है | िव य मू और य मू का 18% हािन पर बेचने पर िव य मू (c)75%
अनुपात ात कर | का अंतर 570 है | यिद इसे 12% (d) 50%
हािन पर बेचा जाए, तो व ु का िव य

e
SSC MTS 5 August 2019
(Morning) मू िकतना होगा? Q52.The cost price of two articles
(a)7:5 SSC MTS 6 August 2019 is the same. One article among
(b)8:7 (Morning) them is sold at a profit of 15% and

l
(c)8:5 (a) Rs 1760 the other is sold at a profit of
(d) 7:6 (b)Rs 1540 12%. if the difference between
ac
Q46. An article was sold at a
profit of 22.5%. What is the ratio
of the cost price and the selling
price ?
(c)Rs 1320
(d) Rs 1650

Q49.The cost price of an article is


Rs. 1800. If the profit is 32%, then
their selling prices is Rs 18, what
is the cost price of each article?
दो व ुओं का य मू समान है |
इनम से एक व ु 15% के लाभ पर
तथा दू सरी व ु 12% के लाभ पर
िकसी व ु को 22.5% के लाभ पर find the selling price . बेची जाती है | यिद उनके िव य मू
बेचा गया | लागत मू तथा िव य िकसी व ु का लागत मू 1800 है म 18 पये का अंतर है , तो ेक
मू का अनुपात िकतना है ? | यिद लाभ 32% है , तो िव य मू व ु का य मू ा है ?
nn
SSC MTS 5 August 2019 िकतना है ? SSC MTS 7 August 2019
(Evening) SSC MTS 6 August 2019 (Morning)
(a)31:57 (Morning) (a)Rs 570
(b)39:69 (a)Rs 2288 (b)Rs 690
(c)40:49 (b)Rs 2376 (c)Rs 400
(d) 34:63 (c)Rs 2456 (d) Rs 600
(d) Rs 2496
Q53.The difference between the
Pi

Q47. Article 1 was sold for Rs.


180. Article 2 was sold for Rs. Q50. If the profit is 20% of the cost price and the selling price of
240. The profit on article 1 is 20% selling price, then find the profit is a bat is Rs 180. If there is a profit
and the loss on article 2 is 20% . what percent of the cost price ? of 20%, then what is the selling
What is the sum of the cost prices यिद लाभ, िव य मू का 20% है , price of the bat?
of these two articles ? तो लाभ, लागत मू का िकतना एक ब े के य मू और िव य
व ु 1 को 180 म बेचा गया | व ु ितशत है ? मू म 180 पये का अंतर है | यिद
2 को 240 म बेचा गया | व ु 1 पर SSC MTS 6 August 2019 लाभ 20% है , तो ब े का िव य
लाभ 20% है और व ु 2 पर हािन (Afternoon) मू िकतना है ?
20% है | दोनों व ुओं के लागत (a)15% SSC MTS 7 August 2019
मू ों का योग िकतना है ? (b)25% (Morning)
(c)22.5% (a)Rs 1080

www.ssccglpinnacle.com support@ssccglpinnacle.com Ph. 09729327755, 09817390373

237
Days 28-32 Pro it and Loss

s
sse
la
_c
ob
(b)Rs 1240 िकसी गद पर कमाया गया लाभ िकसी व ु का अंिकत मू 2600
bo
ah
(c)Rs 1040 इसके िव य मू का 53 है | यिद पये है , जो इसके य मू से 30%
sm
ur
yo

(d) Rs 1120 इस गद का िव य मू 120 पये है , ादा है | यिद लाभ 45% है , तो इस


e/
t.m

व ु का िव य मू ा होगा ?
://

तो इस गद पर िकतना लाभ कमाया


tp
ht

Q54.Ajay sold an article for Rs 84 गया ? SSC MTS 8 August 2019


at a loss of 30%. If he sells the SSC MTS 7 August 2019 (Morning)
same article at Rs 120, then what (Evening) (a)Rs 3100
will be his profit or loss (a)Rs 72 (b)Rs 3900
percentage? (b)Rs 48 (c)Rs 2700
अजय ने कोई व ु 30% की हािन पर (c)Rs 24 (d) Rs 2900
84 पये म बेच दी | यिद उसने इसी (d) Rs 96
व ु को 120 पये म बेचा होता, तो Q60.The cost price and the selling
उसके लाभ या हािन का ितशत ा Q57. The cost price of a cycle is price of a shirt are Rs 960 and Rs
होता ? Rs 24000. If the profit is 30%, 1392 respectively. If by way of
SSC MTS 7 August 2019 then what will be the selling bargaining a customer can bring

e
(Afternoon) price? the selling price down by 10% of
(a)10% loss एक साइिकल का य मू 24000 the cost price, then what is the
(b)15% loss पये है | यिद लाभ 30% है , तो िव य profit percentage?
(c)20% profit मू ा होगा ? एक शट का य मू और िव य
मू मशः 960 पये और 1392

l
(d) No profit no loss SSC MTS 7 August 2019
(Evening) पये है | यिद मोलभाव करके एक
ac
Q55.Two articles were sold of Rs
2400 each by a shopkeeper. The
shopkeeper incurred no profit and
no loss on the whole transaction.
If one of the two articles sold at a
(a)Rs 34100
(b)Rs 33400
(c)Rs 30800
(d) Rs 31200
ाहक िव य मू को य मू के
10% तक नीचे ले आता है , तो लाभ
का ितशत ा होगा ?
SSC MTS 8 August 2019
(Morning)
profit of 20%, then what was the Q58.The difference in selling (a)55%
loss incurred on the other article? prices of an article when sold at (b)35%
दो व ुओं म से ेक को एक 15% profit and 17% loss is Rs 96. (c)30%
nn
दु कानदार ने 2400 पये म बेचा | If it is sold at 10% profit, then (d) 45%
दु कानदार को पूरे लेन-दे न म ना तो what is the selling price?
लाभ आ ना ही हािन ई | यिद इनम जब कोई व ु 15% लाभ और 17% Q61.The cost price of an article is
से एक व ु 20% के लाभ पर बेची की हािन पर बेची जाती है , तो इसके
6
7 of its selling price. What will be
गयी, तो दू सरी व ु पर िकतने पये िव य मू म 96 पये का अंतर the profit or loss percentage?
की हािन ई ? आता है | यिद इसे 10% लाभ पर बेचा एक व ु का य मू इसके िव य
SSC MTS 7 August 2019 जाए, तो िव य मू ा होगा ? मू का 76 है | लाभ या हािन का
(Afternoon) SSC MTS 8 August 2019 ितशत ा होगा ?
Pi

(a)Rs 600 (Morning) SSC MTS 8 August 2019


(b)Rs 400 (a)Rs 345 (Afternoon)
(c)Rs 380 (b)Rs 360 (a)16.67% loss
(d) Rs 240 (c)Rs 315 (b)14.28% profit
(d) Rs 330 (c)16.67% profit
Q56.Profit earned on a ball is 3 of
5 (d) 14.28% loss
its selling price. If the selling Q59. Marked price of an article is
price of the ball is Rs 120, then Rs 2600, wdehich is 30% more Q62. The profit earned on an
how much profit is earned on this than the cost price. If the profit is article is 25%. If profit is
ball? 45%, then what will be the selling calculated on the selling price,
price of the article?

www.ssccglpinnacle.com support@ssccglpinnacle.com Ph. 09729327755, 09817390373

238
Days 28-32 Pro it and Loss

s
sse
la
_c
ob
then what will be the profit it after allowing a discount of (c)12 21 % Profit
bo
ah
percentage? 15%. The profit percentage of the
sm
(d) 15% Profit
ur
yo

िकसी व ु पर कमाया गया लाभ article is:


e/
t.m

25% है | यिद लाभ की गणना िव य शौय ने एक व ु की कीमत य मू


://

Q68. The cost price of an article is


tp
ht

मू पर की जाती है , तो लाभ का से 40% अिधक तय की | उसने इसे Rs. 480. If it is sold at 12.5%
ितशत ा होगा ? 15% छूट पर बेचा | इस व ु पर लाभ profit, then What will be its
SSC MTS 8 August 2019 का ितशत है : selling price ?
(Afternoon) SSC MTS 9 August 2019 िकसी व ु का लागत मू 480 है |
(a)30 (Afternoon) यिद उसे 12.5% के लाभ पर बेचा
(b)10 (a)20% जाता है , तो व ु का िव य मू
(c)20 (b)17% िकतना होगा?
(d) 50 (c)19% SSC MTS 13 August 2019
(d) 15% (Morning)
Q63.The ratio of the selling price (a)Rs 500
to the cost price in a transaction is Q66. If 20% of the goods are sold (b)Rs 560

e
4:5. If the selling price is Rs 80, at 50% profit, 40% of the goods at (c)Rs 540
then how much is the loss? 20% loss, 20% of the goods at 5% (d) Rs 492.5
एक लेन-दे न म िव य मू और य loss and the remaining at no loss
मू का अनुपात 4 : 5 है | यिद or no profit, then the overall profit Q69. A buys a water cooler at
िव य मू 80 पये है , तो हािन ात

l
percentage is: some price and sells it to B at 20%
कर | यिद 20% व ुएँ 50% लाभ पर बेची profit. B sells it to C at a 10%

(Evening)
(a)Rs 16
(b)Rs 15
(c)Rs 20
ac
SSC MTS 8 August 2019 जाती ह, 40% व ुएँ 20% हािन पर
बेची जाती ह, 20% व ुएँ 5% हािन
पर बेची जाती ह तथा शेष व ुएँ ना
तो लाभ ना ही हािन पर बेची जाती ह,
तो कुल िमलाकर लाभ का ितशत
profit. If C bought it at Rs. 6666,
then Find the cost price for B.
A एक वाटर कूलर कुछ मू
खरीदता है और उसे 20% लाभ पर B
पर

को बेच दे ता है | B इसे 10% लाभ पर


(d) Rs 30 ात कर | C को बेच दे ता है | यिद C ने उसे
SSC MTS 9 August 2019 6666 म खरीदा, तो B का लागत मू
Q64.The cost price of a pair of (Afternoon) है :
nn
shoes is Rs 12000. What should (a)5% SSC MTS 13 August 2019
be the marked price (in Rs) on a (b)8% (Morning)
pair of shoes such that after (c)4% (a)Rs 5400
allowing a discount of 16%, the (d) 1% (b)Rs 6060
shopkeeper earns 12% profit? (c)Rs 5600
एक जोड़ी जूते का य मू 12000 Q67. A trader gains 25% after (d) Rs 6400
पये है | एक जोड़ी जूतों का अंिकत selling an article for Rs. 2400. If
मू ( पये म ) ा होना चािहए he had sold the article for Rs. Q70. Parikh sold his pen at a
Pi

तािक 16% की छूट दे ने पर दु कानदार 2160, then What would be his profit of Rs 11. He calculated the
को 12% का लाभ हो ? profit or loss percentage ? profit percentage on selling price
SSC MTS 9 August 2019 एक ापारी िकसी व ु को 2400 and found it to be 25%. the cost
(Morning) पर बेचकर 25% लाभ ा करता है | price (in Rs) of the pen is:
(a)14,330 यिद उसने व ु को 2160 म बेचा पारीख ने अपना पेन 11 पये के लाभ
(b)16,000 होता, तो उसका लाभ या हािन पर बेचा | उसने अपने लाभ के
(c)13,440 ितशत िकतना था ? ितशत की गणना िव य मू पर
(d) 16,500 SSC MTS 9 August 2019 की और इसे 25% पाया | पेन का य
(Evening) मू ( पये म ) ात कर |
Q65. Shuarya marked an article at (a)12 21 % Loss SSC MTS 13 August 2019
40% above its cost price. He sells (b)15% Loss (Afternoon)

www.ssccglpinnacle.com support@ssccglpinnacle.com Ph. 09729327755, 09817390373

239
Days 28-32 Pro it and Loss

s
sse
la
_c
ob
(a)33 SSC CHSL 9 July 2019 SSC MTS 16 August 2019
bo
ah
(b)24 (Morning) (Morning)
sm
ur
yo

(c)36 (a) No profit no loss (a)Rs 3200


e/
t.m
://

(d)44 (b) 10
3 % profit (b)Rs 3600
tp
ht

(c) 10 % loss (c)Rs 4000


3
Q71.If an article is sold at 23% (d) 10% loss (d) Rs 1440
profit instead of 14% profit, then
the profit would be Rs 189 more. Q74. A shopkeeper gains 400% Q77. An article was sold for Rs.
What is the cost price? on a transaction. If the cost 500. If the selling price was 10%
यिद कोई व ु 14% लाभ के बजाय increases by 100% and there is no less, then the profit would be
23% लाभ पर बेची जाती है , तो 189 change in the selling price, then 12.5%. Find the cost price of this
पये का अित र लाभ होता है | find the ratio between new profit article.
य मू ात कर | and selling price. िकसी टां से न िकसी व ु को 500 म बेचा गया |
SSC MTS 13 August 2019 पर एक दु कानदार लागत पर 400% यिद िव य मू 10% कम रहता, तो
(Afternoon) मुनाफा कमाता है | यिद लागत 100% लाभ 12.5% होता | व ु का लागत
मू िकतना है ?

e
(a)Rs 2100 बढ़ जाती है और िव य मू म
(b)Rs 2105 प रवतन नहीं होता है , तो नए लाभ SSC MTS 16 August 2019
(c)Rs 2340 और िव य मू के बीच का अनुपात (Afternoon)
(d) Rs 1800 है : (a)Rs 360

l
SSC MTS 14 August 2019 (b)Rs 480
Q72.The ratio of cost price and (Morning) (c)Rs 400

एक व ु के
ac
selling price of an article is c:d. If
d is 150% of c then the percentage
of profit on cost price is:
य मू और िव य
मू म c : d का अनुपात है | यिद d,
(a)1:3
(b)2:5
(c)3:5
(d) 1:2
(d) Rs 420

Q78. 30 dozens of nuts were


purchased for Rs. 14400 and 32
packets of nuts ( 20 nuts in each
c का 150% है , तो य मू पर लाभ Q75. If the selling price of an packet ) were purchased for Rs.
का ितशत ात कर | article is 3/4 times its cost price, 57600. If these nuts are combined
SSC MTS 13 August 2019 and sold in a packet of 5 for Rs.
nn
the profit/loss percent is:
(Evening) यिद िकसी व ु का िव य मू 432, then find the profit
(a)150% इसके य मू का ¾ गुना है , तो percentage.
(b)50% लाभ/हािन का ितशत है : 30 दजन अखरोट (नट) 14400 म
(c)100% SSC MTS 14 August 2019 खरीदे गए और 32 पैकेट अखरोट
(d) 75% (Evening) ( ेक पैकेट म 20 अखरोट)
(a)25% profit 57600 म खरीदा गया | यिद अखरोटों
Q73. A man bought three articles (b)25% loss को िमलाकर एक पैकेट म पां च के
for Rs3,000 each. He sold the िहसाब से 432 पए म बेचा गया, तो
Pi

(c)33 31 % profit
articles respectively at 15% profit, लाभ ितशत िकतना था? SSC MTS
(d) 33 31 % loss
10% profit and 15% loss. The 16 August 2019 (Evening)
total percentage profit/loss he (a)10%
Q76. 30 dozens of nuts were
earned is: (b)15%
bought for Rs 14400. If the nuts
एक ने 3 व ुओं म से ेक (c)20%
are sold in a packet of 5 for Rs
को 3000 पये म ख़रीदा | उसने इन (d) 25%
250, then how much profit is
व ुओं को मशः 15% लाभ, 10%
earned?
लाभ और 15% हािन पर बेच िदया | Q79. The cost of two bikes is Rs.
30 दजन बादाम 14400 पये म
उसके ारा ा कुल लाभ/हािन का 40,000 each. One of them was
ख़रीदे गए | यिद इन बादामों को 5 के
ितशत ात कर | sold for Rs. 48000. At what price
पैकेट म 250 पये म बेचा जाता है ,
should the second bike be sold so
तो िकतना लाभ होता है ?

www.ssccglpinnacle.com support@ssccglpinnacle.com Ph. 09729327755, 09817390373

240
Days 28-32 Pro it and Loss

s
sse
la
_c
ob
that a total gain of 25% can be selling prices of A and B is Rs. ितशत हािन ( एक दशमलव थान
bo
ah
obtained ? 1390. If the profit of 10% and तक सही ) है :
sm
ur
yo

दो बाइक ेक की लागत 40000 20% is obtained on A and B SSC MTS 19 August 2019
e/
t.m

है | उनम से एक को 48000 म बेचा


://

respectively, then find the ratio (Evening)


tp
ht

गया | दू सरी बाइक को िकतने म बेचा between the cost prices of A and (a) 4.4
जाए तािक 25% का सम लाभ ा B. (b) 3.1
िकया जा सके? A तथा B के लागत मू ों का योग (c) 3.4
SSC MTS 16 August 2019 1200 है | A तथा B के िव य मू ों (d) 3.8
(Evening) का योग 1390 है | यिद A तथा B
(a)Rs 60000 पर मशः 10% तथा 20% लाभ Q85. An item is sold for Rs 702 at
(b)Rs 48000 कमाया जाता है , तो A तथा B के a profit of Rs 162. If an additional
(c)Rs 52000 लागत मू ों का अनुपात िकतना है ? profit of 10% is to be made, then
(d) Rs 50000 SSC MTS 19 August 2019 the new selling price will be
(Afternoon) एक व ु162 पये के लाभ पर 702
Q80. Cost price of an article is Rs (a)5:7 पये म बेची गयी। यिद 10% का
अित र लाभ कमाना है , तो नया

e
1440 and its selling price is Rs (b)5:9
1800. What is the profit (c)7:13 िव य मू ा होगा?
percentage? (d) 6:11 SSC MTS 19 August 2019
एक व ु का य मू 1440 पये है (Evening)
तथा इसका िव य मू 1800 पये

l
Q83. A bought an article for Rs. (a)Rs 756
है | लाभ ितशत ा है ? 1400. He sold it to B at the profit (b)Rs 729

(Morning)
(a)15%
(b)25%
(c)20%
ac
SSC MTS 19 August 2019 of 25%. B sold it to C at the profit
of 20%. C sold it to D at the loss
of 15%. What is the cost price for
D.
A ने िकसी व ु को 1400 म
(c)Rs 810
(d) Rs 750

Q86. A shopkeeper allows 10%


discount on the marked price of an
(d)12.5% खरीदा | उसने उसे B को 25% लाभ article and still makes a profit of
पर बेच िदया | B ने उसे C को 20% 8%. If the marked price is Rs 480,
Q81.The difference between the लाभ पर बेच िदया | C ने उसे D को then what is the cost price (in Rs)
nn
selling price of an article is Rs 15% हािन पर बेच िदया | D का य of the article?
1440, when it is sold at 20% profit मू िकतना है ? एक दु कानदार िकसी व ु के अंिकत
and 17.5% loss. What will be the SSC MTS 19 August 2019 मू पर 10% की छूट दे ता है तथा
selling price when it is sold at (Afternoon) िफर भी 8% का लाभ कमाता है | यिद
15% profit? एक व ु के िव य (a)Rs 1785 अंिकत मू 480 पये है , तो य
मू म 1440 पये का अंतर है जब (b)Rs 2025 मू ात कर |
इसे 20% लाभ और 17.5% हािन पर (c)Rs 1900 SSC MTS 19 August 2019
बेचा जाता है | इसका िव य मू (d) Rs 1665 (Evening)
Pi

ा होगा जब इसे 15% लाभ पर बेचा (a)350


जाता है ? Q84.A trader sells two articles for (b)360
SSC MTS 19 August 2019 Rs 9520 each, one at a profit of (c)420
(Morning) 12% and the other at a loss of (d) 400
(a) Rs 4108 15%. His overall percentage loss
(b) Rs 4218 (correct to one decimal place) is: Q87. If two-thirds of the articles is
(c) Rs 4612 एक ापारी दो व ुओं म से ेक sold at 25% profit, 20% of them is
(d) Rs 4416 को 9520 पये म बेचता है िजसम से sold at a loss of 20% and the
एक पर 12% लाभ तथा दू सरी पर remaining articles are sold at a
Q82. The sum of cost prices of A 15% की हािन होती है | उसकी कुल profit of 20%, then the profit of
and B is Rs. 1200. The sum of the

www.ssccglpinnacle.com support@ssccglpinnacle.com Ph. 09729327755, 09817390373

241
Days 28-32 Pro it and Loss

s
sse
la
_c
ob
Rs. 3312 is obtained. Find the cost (c)Rs 2000
bo
ah
price of the articles. Q90.When a shopkeeper sells an (d) Rs 1500
sm
ur
yo

यिद व ुओं का दो-ितहाई िह ा article for Rs. 230, he gets a loss


e/
t.m

25% लाभ पर, 20% िह ा 20% की


://

of 20%. Find his profit percentage Q93. The cost price of a camera is
tp
ht

हािन पर और शेष िह ा 20% के If he sells this article for Rs. 90% of its selling price. Find the
लाभ पर बेचा जाता है , तो 3,312 339.25. profit percentage. / एक कैमरे का
का लाभ होता है | व ुओं का लागत एक दु कानदार जब िकसी व ु को य मू उसके िव य मू का
मू है : 230 म बेचता है तो उसे 20% की 90% है | लाभ ितशत है :
SSC MTS 20 August 2019 हािन होती है | यिद वह उस व ु को SSC MTS 21 August 2019
(Morning) 339.25 म बेचता है तो उसे िकतने (Morning)
(a)Rs 18,600 ितशत का लाभ होगा? (a)11 91 %
(b)Rs 21,400 SSC MTS 20 August 2019 (b)10%
(c)Rs 21,600 (Afternoon) (c)12%
(d) Rs 20,000 (a)20 1 %
(d) 9 11
(b)18

e
Q88. Sujata sold 25 articles for (c)12 Q94. Two articles are purchased
Rs. 1250 and she incurred a loss (d) 15 on the same price. One is sold at a
of 10%. How many articles profit of 20% while the other is
should she sell in Rs. 2600 so that Q91. The cost of 21 cycles is sold at a loss of 10%. What is the

l
she can get a profit of 17% ? equal to the selling price of 20 total percentage of profit/loss ?
सुजाता ने 1,250 म 25 व ुएं बेचीं cycles. What is the percentage of दो व ुएं समान कीमत पर खरीदी
ac
और उसे 10% की हािन ई | उसे
2600 म िकतनी व ुएं बेचनी चािहए,
तािक वह 17% का लाभ कमा सक ?
SSC MTS 20 August 2019
(Morning)
loss or profit in the sale of one
cycle ?
21 साइिकलों की कीमत 20
साइिकलों के िव य मू के बराबर
है | एक साइिकल के िव य म हािन
जाती है | एक व ु को 20% लाभ पर
बेचा जाता है और दू सरे को 10% हािन
पर बेचा जाता है | कुल लाभ/हािन
ितशत है :
SSC MTS 21 August 2019
(a)26 या लाभ ितशत िकतना है ? (Morning)
(b)40 SSC MTS 20 August 2019 (a)5% loss / 5% हािन
(c)25 (Evening) (b)5%Profit / 5% लाभ
nn
(d) 42 (a)Profit 20% / लाभ 20% (c)10% Profit / 10% लाभ
(b)Loss 20% / हािन 20% (d)10% loss / 10% हािन
Q89. The marked price of an (c)Loss 5% / हािन 5%
article is 20% above its cost price. (d)Profit 5% / लाभ 5% Q95. A person bought a bike and
After giving x% discount on the a car for Rs. 500000. He sold the
marked price, the shopkeeper Q92. Abh ay sold his car at a loss bike at a profit of 20% and the car
incurred a loss of 10%. What is of 30%. If he had sold it for Rs. at a loss of 10% and in the whole
the value of x ? 950 more, then he would have got
Pi

deal he managed to get a profit of


िकसी व ु का अंिकत मू उसके the profit of 8%. What is the cost 5%. Find the cost ( in Rs ) of the
लागत मू से 20% अिधक है | price ? Bike.
अंिकत मू पर x% छूट दे ने के बाद अभय ने अपनी कार 30% की हािन िकसी ने 500000 म एक
दु कानदार को 10% की हािन होती है | पर बेच दी | यिद उसने उसे 950 बाइक और एक कार खरीदी | उसने
x का मान िकतना है ? अिधक म बेचा होता तो उसको 8 % बाइक को 20% लाभ पर और कार
SSC MTS 20 August 2019 का लाभ ा होता | लागत मू ा को 10% हािन पर बेच िदया और इस
(Morning) है ? कार पुरे सौदे म उसे 5% का लाभ
(a) 30 SSC MTS 20 August 2019 ा आ | बाइक की कीमत ( म)
(b) 28 (Evening) है :
(c) 20 (a)Rs 2500 SSC MTS 21 August 2019
(d) 25 (b)Rs 3500 (Afternoon)

www.ssccglpinnacle.com support@ssccglpinnacle.com Ph. 09729327755, 09817390373

242
Days 28-32 Pro it and Loss

s
sse
la
_c
ob
(a)180000 जब कोई व ु अपने अंिकत मू से हािन पर बेच िदया | इस रािश से
bo
ah
(b)250000 40% छूट पर बेची जाती है , तो 25% उसने एक व ु को ख़रीदा और उसे
sm
ur
yo

(c)200000 का लाभ होता है | इस व ु के य 40% लाभ पर बेच िदया | कुल लाभ


e/
t.m

मू और अंिकत मू म अनुपात या हािन का ितशत ात कर|


://

(d) 150000
tp
ht

ात कर | SSC MTS 22 August 2019


Q96. A green grocer bought some SSC MTS 22 August 2019 (Evening)
mangoes and sold 33 31 % of them (Morning) (a) Loss 2.5%
at 22.5 % profit, 25% at a loss of (a)12:25 (b) Gain 8%
25% and remaining at a loss of (b)4:5 (c) Loss 4%
3%. His overall gain or loss is : (c)5:8 (d) Gain 5%
एक स ी िव े ता ने कुछ आम खरीदे (d) 8:13
और उनम से 33 31 % को 22.5% Q102.The marked price of an
लाभ पर, 25% को 25% हािन पर Q99. By selling an article for Rs article is Rs 882. It is sold for Rs
और शेष को 3% हािन पर बेच िदया | 300, a person incurred a loss of 806.40. If there is a loss of 4%,
उसका सम लाभ या हािन है : 6.25%. What is his profit, if it is then by what percent above the

e
SSC MTS 21 August 2019 sold for Rs 352? cost was the article maked?
(Evening) िकसी व ु को 300 पये म बेचने पर िकसी व ु का अंिकत मू 882
(a) No profit and No loss / कोई एक को 6.25% की हािन होती पये है | इसे 806.40 पये म बेचा
लाभ या हािन नहीं है | यिद इसे 352 पये म बेचा जाए, जाता है | यिद 4% की हािन होती है ,
तो िकतना लाभ होगा ? तो इस व ु का अंिकत मू य

l
(b) 3 31 % of Profit / 3 31 % का लाभ
SSC MTS 22 August 2019 मू से िकतना ितशत अिधक था ?
(c) 5 21 % of loss / 5 21 % की हािन
ac (Afternoon) SSC MTS 22 August 2019
(d) 3 31 % of loss / 3 31 % की हािन
(a)Rs 38 (Evening)
(b)Rs 42 (a)8
Q97. Sonu sold his bike at a loss (c)Rs 32 (b)6
of 13.5%. If he had sold it for Rs. (d) Rs 28 (c)4
8900 more, then he would have (d) 5
obtained a profit of 31%. If he had Q100. By selling 30 articles, a
sold it for Rs. 20740, then what shopkeeper gains the selling price Q103. A shopkeeper wrongly
nn
profit or loss would he get ? of 9 articles. His gain percent is: calculates his profit on the selling
सोनू ने अपनी बाइक को 13.5% हािन 30 व ुओं को बेचने पर, एक price of an article and finds it to
पर बेचा | अगर वह उसे 8900 दु कानदार को 9 व ुओं के िव य be 40%. What is his actual profit
अिधक म बेचता तो उसे 31% लाभ मू के बराबर लाभ होता है | उसके percentage?
िमलता | अगर उसने उसे 20740 म लाभ का ितशत है : एक दु कानदार गलत तरीके से अपने
बेचा होता, तो उसका लाभ या हािन SSC MTS 22 August 2019 लाभ की गणना व ु के िव य मू
होता: (Afternoon) पर करता है तथा इसे 40%. पाता है |
SSC MTS 21 August 2019 (a)30 उसका वा िवक लाभ ितशत ात
Pi

(Evening) (b)39 कर |
(a)Profit of 3.9% / 3.9% का लाभ (c)42 76 SSC MTS 22 August 2019
(b)Profit of 3.7% / 3.7% का लाभ (Morning)
(d) 40 73
(c)Loss of 3.2% / 3.2% की हािन (a)64 31
(d) Loss of 3.7% / 3.7% की हािन
Q101. Amit bought an article for (b)56 31
Rs 310 and sold it at a loss of (c)66 32
Q98.When an article is sold at a
25%. With this money, he bought (d)60 32
discount of 40% on its marked
another article and sold it at a gain
price, the profit is 25%. What is
of 40%. What was his overall gain Q104. A purchased two articles
the ratio of the cost price to the
or loss percent? अिमत ने एक व ु for Rs 200 and Rs300 respectively
marked price of the article?
310 पये म ख़रीद कर इसे 25% की and sold at gains of 5% and 10%

www.ssccglpinnacle.com support@ssccglpinnacle.com Ph. 09729327755, 09817390373

243
Days 28-32 Pro it and Loss

s
sse
la
_c
ob
respectively. What was his overall the discount not been given, the
bo
ah
gain percentage? profit would have been 25%. Q110. A trader marks his goods at
sm
ur
yo

A ने मशः 200 और 300 पये म What is the cost price of the 40% above the cost price. He sells
e/
t.m

दो व ुएँ खरीदी और उ मशः


://

article? 70% of the goods at the marked


tp
ht

5% और 10% के लाभ पर बेच िदया | एक व ु के अंिकत मू पर 10% price and the rest, he sells by
उसका कुल लाभ ितशत ात कर | की छूट की दे ने के बाद, इसे 360 allowing a 40% discount on the
SSC CHSL 11 July 2019 पए म बेचा जाता है । अगर छूट नहीं marked price. His percentage
(Morning) दी गई होती, तो लाभ 25% होता। profit is:
(a) 6 व ु की लागत मू ा है ? एक िव े ता अपनी व ुओं की कीमत
(b) 9 SSC CHSL 2 July 2019 य मू से 40% अिधक रखता है |
(c) 5 (Morning) वह 70% व ुएँ अंिकत मू पर
(d) 8 (a) Rs350 बेचता है तथा शेष व ुएँ वह अंिकत
(b) Rs360 मू से 40% की छूट पर बेचता है |
Q105. A man loses 20% by (c) Rs320 उसका लाभ ितशत है :
selling an article for Rs 96. For (d) Rs325 SSC CGL 6 June 2019

e
what amount should he have sold (Afternoon)
the article to gain 15%? Q108. The cost price of an article (a) 23.4
एक को िकसी व ु को 96 is Rs 425. A shopkeeper gives a (b) 24.2
पये म बेचने पर 20% की हािन होती discount of 20% and still gains (c) 23.2
है | 15% का लाभ कमाने के िलए उसे

l
16%. What is the marked price of (d) 24.4
इस व ु को िकस कीमत पर बेचना the article?
चािहए ?

(Afternoon)
(a) Rs120
(b) Rs115
ac
SSC CHSL 11 July 2019
एक व ु की लागत मू 425 पए
है । एक दु कानदार 20% की छूट दे ता
है और िफर भी 16% ा करता है ।
व ु का अंिकत मू
SSC CHSL 11 July 2019
ा है ?
Q111. A trader marked up his
articles 25% more than the cost
price. If he offered a discount of
10%, then what will be his profit
percentage?
(c) Rs138 (Evening) एक ापारी अपनी व ुओं की
(d) Rs140 (a) Rs605.75 कीमत य मू से 25% अिधक
(b) Rs620.50 रखता है । यिद उसने 10% की छूट
nn
Q106. The selling price of 24 (c) Rs624.50 की पेशकश की, तो उसका लाभ
articles is equal to the cost price (d) Rs616.25 ितशत ा होगा?
of 26 articles. What is the gain SSC MTS 6 August 2019
percentage (correct to one decimal Q109. A shopkeeper marks his (Evening)
place) in such a situation? goods at 40% more than their cost (a) 10%
24 व ुओं का िव य मू 26 price and allows a discount of (b) 12.5%
व ुओं के य मू के बराबर है | 25% on the marked price. His (c) 25%
ऐसी थित म लाभ का ितशत ( एक gain or loss percent is: (d) 37.5
Pi

दशमलव थान तक सही ) ा है ? एक दु कानदार अपनी व ुओं की


SSC CHSL 11 July 2019 कीमत य मू से 40% अिधक Q112. On selling 36 mobiles, a
(Evening) रखता है और अंिकत मू पर 25% mobile vendor got a loss
(a) 25
3 % की छूट दे ता है । उसका लाभ या equivalent to the selling price of
(b) 105 % उसकी हािन ात कर। four mobiles. The loss percent is :
13
(c) 100 % SSC CGL 6 June 2019 36 मोबाइल बेचने पर, एक मोबाइल
13
26 % (Morning) िव े ता को चार मोबाइल के िव य
(d) 3
(a) 5% loss मू के बराबर हािन ई | हािन का
(b) 15% gain ितशत था :
Q107. After allowing a discount
(c) 10% loss SSC MTS 9 August 2019
of 10% on the marked price of an
(d) 5% gain (Morning)
article, it is sold for Rs360. Had

www.ssccglpinnacle.com support@ssccglpinnacle.com Ph. 09729327755, 09817390373

244
Days 28-32 Pro it and Loss

s
sse
la
_c
ob
(a) 10% have saved Rs 1,000 more. How (c) Rs. 2400
bo
ah
(b) 12.50% much did she pay for the mobile? (d) Rs. 2250
sm
ur
yo

(c) 8.50% सीमा ने एक मोबाइल ख़रीदा और


e/
t.m

उसे इस पर 20% की छूट िमली | यिद


://

(d) 8% Q118. An article is sold at a profit


tp
ht

उसे 25% की छूट िमली होती, तो of 15% for Rs. 1725. What will be
Q113. Renu bought an article for उसने 1000 पये और बचाए होते | its selling price if it is sold at a
Rs. 1240 and sold it at a loss of उसने मोबाइल के िलए िकतना loss of 15% ?
25%. With this amount, she भुगतान िकया ? िकसी व ु को 15% लाभ पर 1725
bought another article and sold it SSC CPO 16 March 2019 पए म बेचा जाता है | यिद इसे 15%
at a gain of 40%. Her overall (Evening) की हािन पर बेचा जाता है तो िव य
percentage profit is : (a)25,000 मू िकतना होगा ?
रे णु ने एक व ु 1240 पये म (b)22,000 SSC MTS 2 August 2019
खरीदकर उसे 25% की हािन पर बेच (c)16,000 (Afternoon)
िदया | इस रािश से, उसने एक अ (d)20,000 (a) Rs. 1275
व ु य िकया और उसे 40% के (b) Rs. 1475
लाभ पर बेच िदया | उसका कुल लाभ

e
Q116. A shopkeeper sold two (c) Rs. 1025
ितशत है : articles for Rs 9,879 each. On one, (d) Rs. 1325
SSC CGL 4 June 2019 he gained 11% and on the other,
(Afternoon) he lost 11%. What is the overall Q119. If the selling price of an

l
(a) 12 percentage gain or loss? article is 25% of its cost price,
(b) 6 32 एक दु कानदार ने 9879 पये ित then what will be the loss
(c) 5
(d) 15
ac
Q114. The total cost price of two
articles is Rs. 2000. One of them
व ु की दर से दो व ुएं बेची | पहली
पर उसे 11% का लाभ आ तथा
दू सरी पर उसे 11% की हािन ई |
कुल लाभ या हािन का ितशत ा है
?
percentage ?
यिद िकसी व ु का िव य मू
इसके य मू का 25% है , तो हािन
का ितशत ात कर |
SSC MTS 6 August 2019
is sold at a profit of 12% and the SSC CPO 13 March 2019 (Evening)
other at a loss of 12%. The overall (Evening) (a) 25%
gain in the transaction is 1.2%. (a) 1.21% loss (b) 60%
nn
The cost price of the article for (b) 1.21% gain (c) 75%
which there was a profit was : (c) 1.25% loss (d) 50%
दो व ुओं का कुल य मू 2000 (d) 1.25% gain
पये है | उनम से एक को 12% लाभ Q120. The cost price and the
पर तथा दू सरी को 12% हािन पर बेचा Q117. The difference between the selling price of a similar shirt are
जाता है | लेन-दे न म कुल लाभ 1.2% selling prices of an article is Rs. Rs. 960 and Rs. 1392 respectively.
का होता है | िजस व ु पर लाभ आ, 1250 when it is sold at 25% profit If by way of bargaining a
उसका य मू था : and 37.5% loss. What will be its customer can bring the selling
Pi

SSC CHSL 10 July 2019 selling price, if it is sold at a profit price down by 10% of the cost
(Morning) of 12.5% ? price, then what is the profit
(a) Rs. 1050 िकसी व ु को 25% लाभ तथा percentage ?
(b) Rs. 1100 37.5% हािन पर बेचने से िव य मू एक जैसी शट का य मू और
(c) Rs. 1120 का अंतर 1250 पए है | जब इसे िव य मू मशः 960 पये तथा
(d) Rs. 1080 12.5% लाभ पर बेचा जाता है तो 1392 पये है | यिद मोलभाव करके
इसका िव य मू िकतना होगा ? एक ाहक िव य मू को य मू
Q115. Seema purchased mobile SSC MTS 2 August 2019 के 10% तक कम करवा लेता है , तो
and got 20% discount on it. Had (Afternoon) लाभ का ितशत ा होगा ?
she got 25% discount, she would (a) Rs. 1800 SSC MTS 8 August 2019
(b) Rs. 2500 (Morning)

www.ssccglpinnacle.com support@ssccglpinnacle.com Ph. 09729327755, 09817390373

245
Days 28-32 Pro it and Loss

s
sse
la
_c
ob
(a) 56% (d) 15 म एक की दर से ख़रीदा | हालाँ िक,
bo
ah
(b) 35% 10 नाशपाती सड़े ए थे और उ
sm
ur
yo

(c) 30% Q124. A retailer bought 22 kg of फक िदया गया | शेष को 18 पये म


e/
t.m

एक की दर से बेचा गया | ितशत


://

(d) 45% rice at Rs. 35 per kg and bought


tp
ht

13 kg of rice at Rs. 30 per kg. If लाभ ा होगा ?


Q121. Shiva bought a table for Rs. he mixed the two varieties and SSC MTS 14 August 2019
135000 and sold it for Rs. 110000. sold it for Rs. 40 per kg, then what (Evening)
What is the loss percentage ? profit ( in the nearest integer ) did (a) 20%
िशव ने एक मेज Rs. 135000 म he get ? (b) 10%
खरीदी और उसे Rs. 110000 म बेच कोई खुदरा िव े ता 22 kg चावल 35 (c) 50%
िदया, हािन ितशत ा है ? पए ित की दर से ख़रीदा और 13 (d) 30%
SSC MTS 9 August 2019 kg चावल 30 पए ित kg की दर से
(Evening) ख़रीदा | जब उसने दोनों िक ो को Q127. For every Rs. 5 worth of
(a) 500
27 % िमि त कर 40 पए ित kg की दर revenue, a retailer earns Rs. 1. If
(b) 30% से बेच िदया तो उसने िकतना लाभ the cost of the goods was Rs.
(िनकटतम पूणाक म ) ा िकया ?

e
(c) 270 4800, then how much did he earn?
22 %
(d) 2200 SSC MTS 21 August 2019 ेक 5 पये की आय म, एक
27 %
(Afternoon) खुदरा िव े ता को 1 पये का लाभ
(a) 20 होता है | यिद व ुओं की कीमत
Q122. A scooter was bought for
4800 पये थी, तो उसे िकतना लाभ

l
(b) 21
Rs. 30000 and Rs. 3000 was spent
(c) 25 आ?
on its repairs. It was sold for Rs.
ac (d) 18 SSC MTS 16 August 2019
39600. The profit percentage was
(Morning)
:
Q125. A person sold an article at a (a) Rs. 600
एक ू टर 30000 पए म ख़रीदा
loss of 12%. Had he sold it for Rs. (b) Rs. 900
गया और उसकी मर त के िलए
162 more, he would have gained (c) Rs. 1200
3000 पए खच िकये गए | उसे
24%. If the article is sold for Rs. (d) Rs. 1120
39600 पए म बेच िदया गया | लाभ
360 then loss percent is :
ितशत िकतना था ?
एक ने को व ु 12% की हािन SSC CGL TIER I
nn
SSC MTS 9 August 2019
पर बेची | यिद उसने इसे 162 पये
(Morning)
अिधक म बेचा होता, तो उसे 24% Q1. A person sells an article at
(a) 15
लाभ आ होता | यिद इस व ु को 10% below its cost price. Had he
(b) 25
360 पये म बेचा जाए, तो हािन का sold it for ₹332 more, he would
(c) 20
ितशत ा होगा ? have made a profit of 20%. What
(d) 10
SSC MTS 22 August 2019 is the original selling price (in ₹)
(Evening) of the article?
Q123.If selling price of a product
(a) 18 एक िकसी व ु को इसके
Pi

is multiplied by 2, the resultant


(b) 20 य मू से 10% कम पर बेचता है |
profit is 6 times the original profit,
(c) 25 यिद उसने इसे 332 पये अिधक म
then find the original profit ?
(d) 15 बेचा होता, तो उसे 20% का लाभ
यिद िकसी उ ाद के िव य मू को
आ होता | इस व ु का आरं िभक
2 से गुना करने पर प रणामी लाभ मूल
Q126. A shopkeeper purchased िव य मू ( पये म ) िकतना था ?
लाभ का 6 गुना होता है , तो मूल लाभ
120 pears for Rs. 15 each. SSC CGL 3 March 2020
ितशत िकतना है ?
However, 10 pears were rotten (Morning)
SSC MTS 16 August 2019
and thrown away. The remaining (a) 896
(Afternoon)
sold at Rs. 18 each. What will be (b) 1,328
(a) 25
the percentage profit ? एक (c) 1,028
(b) 20
दु कानदार ने 120 नाशपाती 15 पये (d) 996
(c) 10

www.ssccglpinnacle.com support@ssccglpinnacle.com Ph. 09729327755, 09817390373

246
Days 28-32 Pro it and Loss

s
sse
la
_c
ob
को 10% लाभ पर तथा कुिसयों को (a) ₹36.80
bo
ah
Q2. Anu fixes the selling price of 20% लाभ पर बेच िदया | यिद सभी (b) ₹38.40
sm
ur
yo

an article at 25% above its cost of मेज एवं कुिसयों को बेचकर उसका (c) ₹42.60
e/
t.m

कुल लाभ 2,080 पये का था, तो 3


://

production. If the cost of (d) ₹39.20


tp
ht

production goes up by 20% and कुिसयों का य मू िकतना है ?


she raises the selling price by SSC CGL 4 March 2020 Q7. Sudha sold an article to Renu
10%, then her percentage profit is (Morning) for ₹576 at a loss of 20%. Renu
(correct to one decimal place): (a) ₹1,890 spent a sum of ₹224 on its
अनु िकसी व ु का िव य मू (b) ₹1,740 transportation and sold it to
इसके उ ादन लागत से 25% (c) ₹1,800 Raghu at a price which would
अिधक िनधा रत करती है | यिद (d) ₹1,860 have given Sudha a profit of 24%.
उ ादन की लागत 20% बढ़ जाती है The percentage of gain for Renu
एवं वह िव य मू 10% बढ़ा दे ती Q5. One-third of the goods are is:
है , तो उसका ितशत लाभ होगा : ( sold at a 15% profit, 25% of the सुधा ने 20% हािन पर रे णु को एक
दशमलव के एक थान तक ) goods are sold at a 20% profit and व ु 576 पये म बेच दी | रे णु ने
इसके प रवहन पर 224 पये खच

e
SSC CGL 3 March 2020 rest at a 20% loss. If the total
(Afternoon) profit of ₹138.50 is earned on the िकये तथा रघु को उस कीमत पर बेच
(a) 16.4% whole transaction, then the value िदया िजस कीमत पर सुधा को 24%
(b) 14.6% (in ₹) of the goods is: / का लाभ होता | रे णु के िलए ितशत
एक-ितहाई व ुएँ 15% लाभ पर बेची लाभ है :

l
(c) 13.8%
(d) 15.2% जाती ह, 25% व ुएँ 20% लाभ पर SSC CGL 5 March 2020
ac
Q3. A dealer sold 6 sewing
machines for ₹63,000 with a
profit of 5%. For how much
should he sell 8 machines if he
तथा शेष व ुएँ 20% हािन पर बेची
जाती ह | यिद पूरे लेनदे न पर कुल
138.50 पये का लाभ होता है , तो
व ुओं का मू ( पये म ) है :
SSC CGL 4 March 2020
(Morning)
(a) 10.5%
(b) 11.6%
(c) 12.9%
(d) 13.2%
intends to earn 15% profit? (Afternoon)
एक िव े ता ने 6 िसलाई मशीन (a) ₹8,587 Q8. Reena sold 48 articles for
63000 पये म बेची तथा उसे 5% (b) ₹8,030 ₹2,160 and suffered a loss of
nn
का लाभ आ | यिद वह 15% लाभ (c) ₹7,756 10%. How many articles should
कमाना चाहता है , तो उसे 8 मशीन (d) ₹8,310 she sell for ₹2,016 to earn a profit
िकस कीमत पर बेचनी चािहए ? of 12%?
SSC CGL 3 March 2020 Q6. A sells an article to B at a रीना ने 48 व ुओं को 2,160 पये म
(Evening) loss of 20%. B sells it to C at a बेचा तथा उसे 10% की हािन ई |
(a) ₹88,200 profit of 12.5% and C sells it to D 12% का लाभ कमाने के िलए उसे
(b) ₹92,000 at a loss of 8%. If D buys it for 2016 पये म िकतनी व ुओं को
(c) ₹69,300 ₹248.40, then what is the बेचना चािहए ?
Pi

(d) ₹92,400 difference between the loss SSC CGL 5 March 2020
incurred by A and C? (Afternoon)
Q4. A person buys 5 tables and 9 A ने एक व ु B को 20% की हािन (a) 36
chairs for ₹15,400. He sells the पर बेची | B ने उस व ु को C को (b) 40
tables at 10% profit and chairs at 12.5% लाभ पर बेच िदया तथा C ने (c) 32
20% profit. If his total profit on वह व ु D को 8% की हािन पर (d) 28
selling all the tables and chairs is बेची | यिद D ने उसे 248.40 पये म
₹2,080, what is the cost price of 3 ख़रीदा, तो A और C को ई हािन के Q9. Sudhir purchased a laptop for
chairs? / बीच ा अंतर है ? ₹42,000 and a
एक 5 मेज और 9 कुिसयों को SSC CGL 4 March 2020 scanner-cum-printer for ₹8,000.
15,400 पये म ख़रीदा | उसने मेजों (Evening) He sold the laptop for a 10%

www.ssccglpinnacle.com support@ssccglpinnacle.com Ph. 09729327755, 09817390373

247
Days 28-32 Pro it and Loss

s
sse
la
_c
ob
profit and scanner-cum-printer for खरीदता है , तो िविनमाण की लागत ‘B’. What are the cost prices of
bo
ah
a 5% profit. What is his profit िकतनी है ? the two watches? (two places
sm
ur
yo

percentage? SSC CGL 6 March 2020 after decimal).


e/
t.m

सुधीर ने 42,000 पये म एक एक दो घड़ी ‘A’ और ‘B’ को


://

(Afternoon)
tp
ht

लैपटॉप तथा 8,000 पये म ै नर (a) ₹ 6,500 कुल 800 पये की लागत म य
सह ि ंटर ख़रीदा | उसने लैपटॉप को (b) ₹ 6,000 करता है | वह दोनों घिड़यों को समान
10% लाभ पर तथा ै नर सह ि ंटर (c) ₹ 5,000 िव य मू पर बेच दे ता है तथा घड़ी
को 5% लाभ पर बेच िदया | लाभ का (d) ₹ 5,090 A पर 18% का लाभ कमाता है
ितशत िकतना है ? जबिक घड़ी B पर उसे 22% की
SSC CGL 5 March 2020 Q12. An article was sold at a gain हािन होती है | दोनों घिड़यों का य
(Evening) of 16%. If it had been sold for मू िकतना है ?
(a) 7 21 % ₹36 more, the gain would have SSC CGL 7 March 2020
(b) 9 52 % been 20%. The cost price of the (Afternoon)
(c) 15% article is: (a) A = ₹ 350.32 and B = ₹
(d) 9 51 % एक व ु 16% के लाभ पर बेची गयी 450.68
| यिद उसे 36 पये अिधक म बेचा

e
(b) A = ₹ 318.37 and B = ₹
गया होता, तो लाभ 20% का होता | 481.63
Q10. An article was sold at a gain
इस व ु का य मू िकतना है ? (c) A = ₹ 220 and B = ₹ 580
of 18%. If it had been sold for ₹
SSC CGL 6 March 2020 (d) A = ₹ 317 and B = ₹ 483
49 more, then the gain would

l
(Evening)
have been 25%. The cost price of
(a) ₹862 Q15. A man sold two gifts at ₹30
the article is:
ac (b) ₹720 each. On one gift he gained 18%
एक व ु 18% के लाभ पर बेची गयी
(c) ₹810 and on the other he lost 18%.
| यिद उसे 49 पये अिधक म बेचा
(d) ₹900 What is his overall gain/loss (in
गया होता, तो लाभ 25% होता | इस
व ु का य मू िकतना है ? ₹)?
Q13. By selling 18 table fans for एक ने दो उपहारों म से
SSC CGL 6 March 2020
₹ 11,664 a man incurs a loss of ेक को 30 पये म बेचा | पहले
(Morning)
10%. How many fans should he उपहार पर उसे 18% का लाभ आ
(a) ₹ 570
sell for ₹ 17,424 to earn 10% तथा दू सरे उपहार पर उसे 18% की
nn
(b) ₹ 890
profit? हािन ई | उसका कुल लाभ/हािन (
(c) ₹ 700
18 टे बल पंखों को 11,664 पये म पये म ) ा है ?
(d) ₹ 650
बेचने के बाद एक को 10% SSC CGL 7 March 2020
की हािन होती है | 10% का लाभ (Evening)
Q11. A manufacturer sells
कमाने के िलए उसे 17,424 पये म (a) Gain of ₹1.75
cooking gas stoves to
िकतने पंखे बेचने चािहए ? (b) Gain of ₹2.00
shopkeepers at 10% profit, and in
SSC CGL 7 March 2020 (c) Loss of ₹2.00
turn they sell the cooking gas
(Morning) (d) Loss of ₹2.50
Pi

stoves to customers to earn 15%


(a) 23
profit. If a customer gets a
(b) 18 Q16. Ram makes a profit of 30%
cooking gas stove for ₹ 7,590,
(c) 20 by selling an article. What would
then what is its manufacturing
(d) 22 be the profit percent if it were
cost?
एक िविनमाता कुिकंग गैस ोव को calculated on the selling price
Q14. A man buys two watches instead of the cost price? (Correct
दु कानदार को 10% लाभ पर बेचता
‘A’ and ‘B’ at a total cost of to one decimal place.)
है | तथा िफर दु कानदार इस कुिकंग
₹800. He sells both watches at राम िकसी व ु को बेचकर 30%
गैस ोव को ाहकों को 15% लाभ
the same selling price, and earns a लाभ कमाता है | यिद लाभ की गणना
पर बेच दे ते ह | यिद एक ाहक इस
profit of 18% on watch ‘A’ and य मू के बजाय िव य मू पर
कुिकंग गैस ोव को 7590 पये म
incurs a loss of 22% on watch की जाए, तो लाभ का ितशत ा

www.ssccglpinnacle.com support@ssccglpinnacle.com Ph. 09729327755, 09817390373

248
Days 28-32 Pro it and Loss

s
sse
la
_c
ob
होगा ? ( दशमलव के एक थान तक (b) ₹20,625 (c) Rs 2186.5
bo
)ah (c) ₹8,687.5 (d) Rs 2187.5
sm
ur
yo

SSC CGL 9 March 2020 (d) ₹47,750


e/
t.m
://

(Morning) SSC CHSL 2019 Q4. Kishore had a loss of 20% on


tp
ht

(a) 22.4% Q1. List the price of a bike is selling an article for Rs7,160. At
(b) 23.1% 15% more than its cost price. It is what price should he have sold
(c) 20.1% sold at a discount of 20%. Find the item to make a profit of 30%?
(d) 24.2% the dealer's loss or profit िकशोर को 7,160 पये म एक व ु
percentage. को बेचने पर 20% की हािन ई। उसे
Q17. A shopkeeper buys two एक बाइक का अंिकत मू उसके इस व ु को िकस कीमत पर बेचना
books for ₹300. He sells the first य मू से 15% अिधक है । इसे चािहए था तािक 30% का लाभ कमा
book at a profit of 20% and the 20% की छूट पर बेचा जाता है । सके?
second book at a loss of 10%. िव े ता के लाभ या हािन का ितशत CHSL 12-10-2020 ( Evening
What is the selling price of the ात कीिजए। shift)
first book, if in the whole CHSL 12-10-2020 (Morning (a) Rs11,635

e
transaction there is no profit no shift) (b) Rs12,007
loss? (a) Profit/ लाभ 8% (c) Rs10,678
एक दु कानदार ने दो पु कों को 300 (b) Loss/ हािन 9% (d) Rs8,988
पये म ख़रीदा | उसने पहली पु क (c) Profit/ लाभ 9%
20% के लाभ पर तथा दू सरी पु क (d) Loss/ हािन 8%

l
Q5. A person purchases 40 items
10% की हािन पर बेच दी | पहली at 10 each. He sells a part of them
ac
पु क का िव य मू

SSC CGL 9 March 2020


(Afternoon)
िकतना है ,
यिद पूरे लेनदे न म ना तो लाभ ना ही
हािन ई है ?
Q2. Ram sold a motorcycle for Rs
70000 at 25% profit. For what
price should he sell a motorcycle
to gain 30% profit?
राम ने 25% लाभ पर 70000 पये म
at 25% profit and the remaining
at 10% loss. The net profit is 4%
in this transaction. The number of
items he sold at a loss, is:
एक 40 व ुओं म से ेक
(a) ₹125 एक मोटरसाइिकल बेची। 30% लाभ को 10 म खरीदता है । वह उसका
(b) ₹115 कमाने के िलए उसे मोटरसाइिकल एक िह ा 25% लाभ पर तथा शेष
(c) ₹110 िकस कीमत पर बेचनी चािहए ? िह ा 10% हािन पर बेचता है । इस
nn
(d) ₹120 CHSL 12-10-2020 (Morning लेनदे न म शु 4% का लाभ होता है ।
shift) उसने हािन पर िकतनी व ुओं को
Q18. A car dealer purchased an (a) Rs 72,800 बेचा?
old car for ₹1,08,500 and spent (b) Rs 72, 700 CHSL 13-10-2020 (Morning
some amount on its maintenance. (c) Rs 72,900 shift)
He sold it for ₹1,56,250, thereby (d) Rs 72, 600 (a)16
earning a profit of 25%. How (b)22
much money did he spend on the Q3. On selling a bike for Rs 2500 (c)18
Pi

maintenance of the car? a seller incurs a loss of 20%. (d)24


एक कार िव े ता ने 1,08,500 पये What price would have caused
म एक पुरानी कार खरीदी तथा कुछ him to lose 30%? Q6. A sells an item at 20% profit
रािश उसकी मर त पर खच की | 2500 पये म एक बाइक बेचने के to B, B sells the same at 10%
उसने उस कार को ₹1,56,250 म बाद, िव े ता को 20% की हािन होती profit to C and receives 1,32,000.
बेचा, िजससे उसे 25% का लाभ आ है । िकतनी कीमत पर उसे 30% की Had C purchased the same item
| उसने कार की मर त पर िकतनी हािन होती ? from A, he would have spent 5%
रािश खच की है ? CHSL 12-10-2020 (Afternoon less than what he spent with B.
SSC CGL 9 March 2020 shift) What profit would A have made
(Evening) (a) Rs 2185.5 then?
(a) ₹16,500 (b) Rs 2188.5

www.ssccglpinnacle.com support@ssccglpinnacle.com Ph. 09729327755, 09817390373

249
Days 28-32 Pro it and Loss

s
sse
la
_c
ob
A िकसी व ु को B से 20% के लाभ of Rs 9 per article is Rs 150. If
bo
ah
पर बेचता है । B इसी व ु को C को the cost price of these articles is Q12. A man sold his furniture at a
sm
ur
yo

बेच दे ता है तथा 1,32,000 पये ा Rs250, then find the selling price 25% gain. Had he sold it at 15%
e/
t.m

करता है । यिद C ने इसी व ु को A


://

of 21 articles if profit earned is loss, he would have received


tp
ht

से य िकया होता, तो उसे B को 20%. Rs800 less. Find the cost price of
भुगतान की गयी रािश की तुलना म कुछ व ुओं की िव य की कीमतों the furniture.
5% कम खच करना पड़ता। A को के बीच अंतर, अगर 9 पये के एक आदमी ने अपने फन चर को
िकतना लाभ आ ? बजाय 12 पये ित व ु के िहसाब 25% लाभ पर बेचा। अगर उसने इसे
CHSL 13-10-2020 (Morning से बेचा जाता है तो 150 पये है । यिद 15% नुकसान पर बेचा होता, तो उसे
shift) इन व ुओं की लागत मू 250 800 पये कम िमलते। फन चर की
(a)Rs 25,400 है , तो लाभ अिजत होने पर 21 कीमत ात कर ।
(b)Rs 24,450 व ुओं की िव य मू 20% है । CHSL 15-10-2020 ( Evening
(c)Rs 25,540 CHSL 14-10-2020 (Afternoon shift)
(d)Rs 24,540 shift) (a) Rs2,500
(a) Rs 126 (b) Rs1,500

e
Q7. If Ravi sels an old scooter for (b) Rs 136 (c) Rs2,000
Rs 18,000 then his loss incurred (c) Rs 125 (d) Rs3,000
is 10%. If he sells it to gain a (d) Rs 130
profit of 15%, then the selling Q13. If the gain is one-fifth of the

l
price is: Q10.The cost price of 15 pens is selling price, then the gain
अगर रिव 18,000 पये म पुराना equal to the selling price of 20 percentage is:
ac
ू टर खरीदता है तो उसका
नुकसान 10% है । यिद वह इसे 15%
का लाभ ा करने के िलए बेचता
है , तो िव य मू है :
CHSL 13-10-2020 ( Evening
pens. The loss or profit
percentage is:
15 पेन की लागत मू 20 पेन की
िव य मू के बराबर है । नुकसान
या लाभ ितशत है :
यिद लाभ िव य मू
एक-पां चवाँ िह ा है , तो लाभ
ितशत है :
CHSL 16-10-2020 (Morning
shift)
का

shift) CHSL 15-10-2020 (Morning (a) 80%


(a)Rs 20,000 shift) (b) 20%
(b)Rs 22,000 (a)20% loss (c) 16%
nn
(c)Rs 24,000 (b)25% loss (d) 25%
(d)Rs 23,000 (c)20% profit
(d)25% profit Q14. A shopkeeper allows a
Q8. The cost price of 12 pens is discount of 20% on an article and
equal to the selling price of 8 Q11. If a saree is sold for Rs still makes a profit of 25%. What
pens. The profit percentage is: 3,060, the seller will face 15% does he pay for an article whose
12 पेन की लागत मू 8 पेन की loss, at what price should he sell marked price is ₹ 800?
िव य मू के बराबर है । लाभ the saree to gain a 20% profit? एक दु कानदार एक व ु पर 20%
Pi

ितशत है : यिद एक साड़ी 3,060 पये म बेची की छूट दे ता है और िफर भी 25%


CHSL 14-10-2020 (Morning जाती है , तो िव े ता को 15% हािन का लाभ कमाता है । वह उस व ु के
shift) का सामना करना पड़े गा, 20% लाभ िलये िकतना भुगतान करता है
(a) 45% ा करने के िलए उसे िकस कीमत िजसका अंिकत मू ₹ 800 है |
(b) 55% पर साड़ी बेचनी चािहए? CHSL 16-10-2020 (Morning
(c) 50% CHSL 15-10-2020 (Afternoon shift)
(d) 40% shift) (a) ₹492
(a) Rs 4,650 (b) ₹512
Q9. The difference between the (b) Rs 3,600 (c) ₹800
selling prices of some articles if (c) Rs 3,440 (d) ₹640
sold for Rs 12 per article instead (d) Rs 4,320

www.ssccglpinnacle.com support@ssccglpinnacle.com Ph. 09729327755, 09817390373

250
Days 28-32 Pro it and Loss

s
sse
la
_c
ob
Q15. Salma buys an article and (d) ₹ 15,400 CHSL 19-10-2020 (Afternoon
bo
ah
then sells it for ₹ 810. If she loses shift)
sm
ur
yo

10%, then at what price should Q18. The marked price of a shirt (a) 14 31 %
e/
t.m
://

she sell it to gain 4%? was ₹1,800. A man bought the (b) 14 32 %
tp
ht

सलमा एक व ु खरीदती है और same shirt for ₹ 1,200 after (c) 16 31 %


िफर उसे ₹ 810 बेचती है । यिद उसे getting two successive discounts.
(d) 16 32 %
10% की हािन होती है , िफर तो 4% If the first discount was 12%,
लाभ ा करने के िलए िकस कीमत what was the second discount
Q21. Prem purchased an old
पर बेचना चािहए? rate? (Correct to two decimal
printer for Rs 3,200 and spent Rs
CHSL 16-10-2020 (Afternoon places)
600 on its repair. He sold it for Rs
shift) एक शट का अंिकत मू ₹1,800 था
4,280. His profit per cent is
(a) ₹ 864 एक ने लगातार दो छूट ा
closest to: (correct upto two
(b) ₹ 729 करने के बाद उसी शट को ₹ 1,200
decimal places)
(c) ₹ 936 खरीदी। यिद पहली छूट 12% थी, तो
ेम ने 3,200 पये म एक पुराना
(d) ₹ 900 दू सरी छूट की दर ा थी? (दो
ि ंटर खरीदा और इसकी मर त पर

e
दशमलव थानों के िलए)
600 पये खच िकए। उ ोंने इसे
Q16. The cost price of 33 books CHSL 19-10-2020 (Morning
4,280 पये म बेचा। उसका लाभ
is the same as the selling price of shift)
ितशत िनकटतम है : (दो दशमलव
'x' books. If the profit is 10%, (a) 25.25%
थानों तक)

l
then the value of 'x' is: (b) 22.22%
CHSL 19-10-2020 ( Evening
33 पु कों की लागत मू 'x' (c) 24.24%
ac
पु कों के िव य मू के समान है ।
यिद लाभ 10% है , तो 'x' का मान है :
CHSL 16-10-2020 ( Evening
shift)
(a) 30
(d) 20.20%

Q19. Anil bought some articles at


6 for Rs 8 and sold them at 10 for
Rs12. His percentage loss or gain
shift)
(a) 16.92%
(b) 12.63%
(c) 15.78%
(d) 18.45%
(b) 20 is:
Q.22. When an article is sold for
(c) 10 अिनल ने 8 पये म 6 की दर से कुछ
Rs.720, there is a profit of x%.
(d) 40 व ु खरीदे और 12 पये म उ 10
nn
When the same article is sold for
की दर से बेच िदया। उसका
Rs.750, profit is (x+5)%. What is
Q17. The percentage profit नुकसान या लाभ ितशत है :
the value of x?
earned by selling a mobile for ₹ CHSL 19-10-2020 (Afternoon
जब कोई व ु Rs.720 म बेचा जाता
12,000 is equal to the percentage shift)
है , तो x% का लाभ होता है । जब एक
loss incurred by selling the same (a) 10% loss
ही व ु को Rs.750 म बेचा जाता है ,
mobile for ₹ 9,000. At what price (b) 8% loss
तो लाभ (x+ 5)% होता है । x का मान
should the mobile be sold to (c) 10% gain
ात करे ?
make 20% profit? (d) 8% gain
Pi

CHSL 21-10-2020 (Morning


₹ 12,000 म एक मोबाइल बेचकर
shift)
अिजत ितशत लाभ, ₹ 9,000 म उसी Q20. Marked price of an article is
(a) 20
मोबाइल बेचकर अिजत ितशत 20% more than its cost price. At
(b) 18
नुकसान के बराबर है । 20% लाभ what percent less should it be
(c) 25
कमाने के िलए मोबाइल को िकस sold for no profit and no loss?
(d) 15
कीमत पर बेचा जाना चािहए? एक व ु का अंिकत मू उसकी
CHSL 19-10-2020 (Morning लागत मू से 20% अिधक है । इसे
Q.23. On selling 26 balls for
shift) िकतने ितशत कम पर बेचा जाना
Rs.1350, there is a loss equal to
(a) ₹ 18,800 चािहए तािक न तो लाभ हो और न ही
the cost price of eight balls. The
(b) ₹ 16,200 हािन ?
cost price of a ball is:
(c) ₹ 12,600

www.ssccglpinnacle.com support@ssccglpinnacle.com Ph. 09729327755, 09817390373

251
Days 28-32 Pro it and Loss

s
sse
la
_c
ob
26 गदों को 1,350 पये म बेचने पर, selling price of the article are (c) 3,33,300
bo
ah
आठ गदों की लागत मू के बराबर decreased by Rs.100, the profit (d) 3,00,000
sm
ur
yo

नुकसान होता है । एक गद की लागत now would be 45%. The original


e/
t.m

मू है |
://

cost price of the article is: Q29. If the cost price of 25


tp
ht

CHSL 21-10-2020 (Afternoon एक व ु 30% के लाभ पर बेचा articles is equal to the selling
shift) जाता है । यिद व ु की लागत मू price of 35 articles, find the
(a) 70 और िव य मू दोनों को 100 पये profit/loss percentage.
(b) 65 कम िकया जाता है , तो अब लाभ यिद 25 व ुओं की लागत मू 35
(c) 60 45% होगा। व ु की मूल लागत मू व ुओं के िव य मू के बराबर है
(d) 75 ात कर | तो लाभ / हािन ितशत ात कर।
CHSL 26-10-2020 (Morning CHSL 17-03-2020 (Afternoon
Q.24. The marked price of an shift) shift)
article is 25% more than its cost (a) 400 (a) Loss - 28.57%
price. If a 10% discount is given (b) 250 (b) Profit - 28.57%
on the marked price, then what is (c) 300 (c) Loss - 18.93%

e
the profit percentage? (d) 500 (d) Profit - 18.93%
एक व ु की अंिकत मू उसकी
लागत मू से 25% अिधक है । यिद Q.27. The profit made by selling Q30. A shopkeeper purchased
अंिकत मू पर 10% की छूट दी an article for Rs.13400 is equal to pens in bulk for Rs28 each. He
जाती है , तो लाभ ितशत ा है |

l
the amount of loss incurred on sold each for Rs40. What was his
CHSL 21-10-2020 (Afternoon selling the same article at profit percentage?
shift)
(a) 12%
(b) 11.5%
(c) 10%
(d) 12.5%
ac Rs.11600. What will be the profit
if it is sold for Rs.14,750 (in Rs.)
एक व ु को 13400 पये म बेचने
से होने वाला लाभ, 11600 पये म
उसी व ु को बेचने पर ए नुकसान
एक दु कानदार ने थोक म
पए म पेन खरीदे । उ ोंने
को 40 पये म बेचा। उसका लाभ
ितशत ा था?
CHSL 17-03-2020 (Afternoon
ेक 28
ेक

के बराबर है । अगर इसे Rs.14,750 shift)


Q.25. The profit made by selling ( । म) बेचा जाता है तो ा लाभ (a) 42.85%
an article for Rs.8,800 is equal to होगा। (b) 28.40%
nn
the amount of loss incurred on CHSL 26-10-2020 ( Evening (c) 38.75%
selling the same article for shift) (d) 48.12%
Rs.7,200. What will be the profit (a) 2000 Q31. A chair was purchased for
per cent, if it was sold for (b) 2520 Rs785 and sold at a profit of
Rs.9,600? (c) 2500 22%. What was the selling price?
8,800 पये म एक व ु को बेचकर (d) 2250 एक कुस 785 पये म खरीदी गई
िकया गया लाभ, उसी व ु को 7,200 और 22% के लाभ पर बेची गई।
पये म बेचने पर ई हािन के बराबर Q.28. Sohan sold a plot for िव य मू ा था?
Pi

है । अगर इसे 9,600 पये म बेचा Rs.2,55,000 at a 15% loss. At CHSL 17-03-2020 (Afternoon
जाता तो लाभ ितशत िकतने होता | what price should he sell the plot shift)
CHSL 21-10-2020 ( Evening to gain a 10% profit? (a) Rs 987.4
shift) सोहन ने 15% की हािन पर 2,55,000 (b) Rs 768.3
(a) 20% पये म एक ॉट बेचा। 10% लाभ (c) Rs 857.9
(b) 25% ा करने के िलए उसे िकस कीमत (d) Rs 957.7
(c) 18% पर ॉट बेचना चािहए?
(d) 15% CHSL 17-03-2020 (Morning Q32. By selling an article for 600,
shift) a shopkeeper makes a profit of
Q.26. An article is sold at a profit (a) 3,33,000 20%. At what price should he sell
of 30%. If both cost price and (b) 3,30,000 the article to incur a loss of 20%?

www.ssccglpinnacle.com support@ssccglpinnacle.com Ph. 09729327755, 09817390373

252
Days 28-32 Pro it and Loss

s
sse
la
_c
ob
600 म एक व ु बेचकर एक (a) 27% रिव मोहन को 10% के लाभ पर एक
bo
ah
दु कानदार 20% का लाभ कमाता है । (b) 28% कुस बेचता है और मोहन उसे गोिवंद
sm
ur
yo

20% का नुकसान उठाने के िलए उसे (c) 25% को 20% के लाभ पर बेचता है । यिद
e/
t.m

िकस मू पर व ु बेचना चािहए? गोिवंद इसके िलए 1,320 का


://

(d) 24%
tp
ht

CHSL 17-03-2020 ( Evening भुगतान करता है तो रिव के िलए


shift) Q35. The selling price of a book, लागत मू है :
(a) ₹600 including the sales tax, is CHSL 19-03-2020 (Morning
(b) ₹400 Rs956.34. The rate of sales tax is shift)
(c) ₹300 10%. If the shopkeeper has made (a) 980
(d) ₹500 a profit of 15%, then the cost (b) 1,000
price of the book is: (c) 900
Q33. A shopkeeper pays 12% of िव य कर सिहत एक पु क का (d) 800
the cost price as tax while िव य मू 956.34 पये है । िव य
purchasing an item whose cost is कर की दर 10% है । यिद दु कानदार Q.38. A man sold his bike for
Rs 500. He wants to earn a profit ने 15% का लाभ कमाया है , तो Rs.25000 at 25% profit. At what
पु क का लागत मू है :

e
of 20% after giving a discount of price would it have sold if he had
16% on the marked price. So, the CHSL 18-03-2020 (Afternoon incurred a loss of 15%?
marked price should be: shift) एक आदमी ने अपनी बाइक 25%
एक दु कानदार िकसी व ु को (a) Rs756 लाभ पर 25000 पये म बेची। अगर
खरीदते समय लागत मू का 12% वह 15% का नुकसान उठाता है तो

l
(b) Rs797.34
कर के प म चुकाता है िजसकी (c) Rs845.98 वह िकस कीमत पर बेचेगा ?
ac
लागत 500 पये है । वह अंिकत मू
पर 16% की छूट दे ने के बाद 20%
का लाभ अिजत करना चाहता है । तो,
अंिकत मू होना चािहए:
CHSL 18-03-2020 (Morning
(d) Rs836

Q36.. A tradesman marks his


products 35% above the cost
price and allows his customers
CHSL 19-03-2020 ( Evening
shift)
(a) 16,000
(b) 19,000
(c) 18,000
shift) 15% reduction on their bills. (d) 17,000
(a) Rs 840 What percentage of profit does he
(b) Rs 780 make? SSC CGL TIER-II
nn
(c) Rs 960 एक ापारी लागत मू से 35% Q39. On selling an article for ₹
(d) Rs 800 ऊपर अपने उ ादों को अंिकत 123.40, the gain is 20% more
करता है और अपने ाहकों को than the amount of loss incurred
Q34. A person purchased 40 उनके िबलों पर 15% की छूट दे ता on selling it for ₹108. If the
items at some price. He sold some है । वह िकतने ितशत लाभ कमाता article is sold for ₹120.75, then
items at a profit of 30% by selling है ? what is the gain/loss per cent?
them at a price equal to the cost CHSL 18-03-2020 ( Evening ₹123.40 म एक व ु को बेचने पर,
price of 26 items. The remaining shift) लाभ ₹108 म इसे बेचने पर ए
Pi

items are sold at 18% profit. The (a) 14.25 नुकसान की रािश से 20% अिधक है ।
total profit percentage is: (b) 14 यिद व ु ₹120.75 म बेचा जाता है ,
एक ने कुछ कीमत पर 40 (c) 14.75 तो लाभ / हािन ितशत ा है ?
आइटम खरीदे । उ ोंने कुछ व ुओं (d) 14.10 CGL 2019 Tier-II (15-11-2020 )
को 30% के लाभ पर 26 व ुओं की (a) Gain 2.5% / 2.5% लाभ
कीमत के बराबर मू पर बेचकर Q.37. Ravi sells a chair to Mohan (b) Gain 5% / 5% लाभ
बेच िदया। शेष व ुओं को 18% at a profit 10% and Mohan sells it (c) Loss 2.5% / 2.5% हािन
लाभ पर बेचा जाता है । कुल लाभ to Govind at profit 20%. If (d) Loss 5% / 5% हािन
ितशत है | Govind pays Rs.1320 for it. Then
CHSL 18-03-2020 (Morning the cost price for Ravi is: Q40. The marked price of an
shift) article is 40% above the cost

www.ssccglpinnacle.com support@ssccglpinnacle.com Ph. 09729327755, 09817390373

253
Days 28-32 Pro it and Loss

s
sse
la
_c
ob
price. If its selling price is 73 21 % (d) 18 31 % CGL 2019 Tier-II (16-11-2020 )
bo
ah (a) ₹2,200
sm
of the marked price, then the
ur
yo

profit percentage is: Q43. A sold a watch to B at a (b) ₹2,430


e/
t.m
://

एक व ु का अंिकत मू लागत profit of 20%, B sold it to C at (c) ₹2,480


tp
ht

मू से 40% अिधक है । यिद इसका 30% profit C sold it to D at 10% (d) ₹2,268
िव य मू अंिकत मू का 73 21 % loss. If B’s profit is ₹80 more
है , तो लाभ ितशत ात कीिजए। than that of A, then D bought it Q.46.Anil bought two articles A
CGL 2019 Tier-II (15-11-2020 ) for: and B at a total cost of ₹ 10,000.
(a) 2.7% A ने B को 20% के लाभ पर एक He sold the article A at 15%
(b) 2.4% घड़ी बेची, B ने उसे C को 30% लाभ profit and the article B at 10%
(c) 2.9% पर बेच िदया, C ने उसे 10% हािन loss. In the whole deal, he made
(d) 3.1% पर D को बेच िदया। यिद B का लाभ no profit or no loss. Find the
A की तुलना म 80 पये अिधक है , selling price of the article A.
Q41. Remi earns a profit of 20% तो D ने इसे िकतने म ख़रीदा? अिनल ने ₹ 10,000 की कुल कीमत
on selling an article of a certain CGL 2019 Tier-II (15-11-2020 ) पर दो व ुओं A और B को य

e
price. If she sells the article for ₹8 (a) ₹652 िकया। उसने व ु A को 15% लाभ
more, she will gain 30%.What is (b) ₹702 पर और व ु B को 10% हािन पर
the original price of 16 such (c) ₹700 बेचा। पूरे सौदे म, उसने कोई लाभ
articles ? (d) ₹680 नहीं कमाया और न ही कोई नुकसान

l
रे मी एक व ु को एक िनि त कीमत आ। व ु A के िव य मू का
पर बेचने पर 20% का लाभ कमाती
ac Q44. A T.V is sold at 8% gain . पता लगाएं ।
है । यिद व ु को ₹8 अिधक म बेचती Had it been sold for ₹ 714 more, CGL 2019 Tier-II (18-11-2020 )
है , तो वह 30% लाभ ा करे गी। the gain would have been 15%. (a) ₹ 5400
ऐसी 16 व ुओं का मूल मू िकतना To gain 18% the selling price of (b) ₹ 4500
है ? the T.V should be: (c) ₹ 4600
CGL 2019 Tier-II (15-11-2020 ) एक T.V 8% लाभ पर बेचा जाता है । (d) ₹ 4200
(a) ₹1,280 अगर इसे ₹ 714 अिधक म बेचा
(b) ₹1,152 जाता, तो लाभ 15% होता। 18% Q.47. On selling 38 balls at ₹
लाभ हािसल करने के िलए T.V का
nn
(c) ₹1,120 2,240, there is a loss equal to the
(d) ₹1,200 िव य मू होना चािहए: cost price of 6 balls. The cost
CGL 2019 Tier-II (16-11-2020 ) price of a ball is equal to:
Q42. Shashi sells two articles for (a) ₹ 12,138 38 गदों को ₹ 2,240 म बेचने पर, 6
₹5,000 each with no loss and no (b) ₹ 12,036 गदों के य मू के बराबर नुकसान
profit in the overall transaction. If (c) ₹ 11,934 होता है । एक गद का य मू
one article is sold at 16 32 % loss, (d) ₹ 12,240 बराबर है
then the other is sold at a profit CGL 2019 Tier-II (18-11-2020 )
Pi

of: Q45. A sold an item to B at 20% (a) ₹ 50


शिश ने िबना िकसी नुकसान के gain, B sold it to C at 8% gain. C (b) ₹ 80
₹5,000 म दो व ुओं को बेचा और sold it to D at 25% loss. If the (c) ₹ 70
सम लेनदे न म उसे कोई लाभ या difference between the profit of A (d) ₹ 60
हािन नहीं ई। यिद एक व ु को 16 and B is ₹260, then D bought it
3 % हािन पर बे चा जाता है , तो दू सरे
2 for : Q.48. A dealer sold an article at a
पर लाभ ात कीिजए ? A ने एक व ु को B को 20% लाभ loss of 2%. Has he sold it for ₹ 44
CGL 2019 Tier-II (15-11-2020 ) पर बेचा, B ने इसे C को 8% लाभ पर more, he would have gained 20%.
(a) 24% बेचा। C ने इसे 25% नुकसान पर D Find the cost price of the article.
(b) 25% को बेच िदया। यिद A और B के लाभ एक डीलर ने 2% की हािन पर एक
(c) 16 32 % के बीच अंतर ₹260 है , तो D ने व ु को बेचा। यिद उसने इसे 44
िकतने का खरीदा है : पये अिधक म बेच िदया होता तो

www.ssccglpinnacle.com support@ssccglpinnacle.com Ph. 09729327755, 09817390373

254
Days 28-32 Pro it and Loss

s
sse
la
_c
उसे 20% का लाभ होता। व ु का (b) 16% loss/ हािन

ob
The cost price of the article (in
bo
ah
य मू ात कीिजए। Rs.) is: (c) 20% profit/ लाभ
sm
ur
yo

CGL 2019 Tier-II (18-11-2020 ) एक आदमी ने एक व ु को खरीदा (d) 10% loss/ हािन
e/
t.m

और उसे 10% की लाभ पर बेचा।


://

(a) ₹ 250
tp
ht

(b) ₹ 400 यिद उसने 20% कम म व ु खरीदा Q54. A person sold an article at a
(c) ₹ 200 होता और इसे 1,000 पये अिधक म loss of 16%. Had he sold it for
(d) ₹ 300 बेचा होता, तो उसने 40% का लाभ Rs. 660 more, he would have
कमाया होता। व ु का य मू gained 8%. If the article is sold at
Q49. A man sells two articles at ( पये म) ात करे । Rs. 3,080, then how much profit
₹ 9,975 each. He gains 5% on one CPO 2019 23-11-2020 percentage is gained?
article and loses 5% on the other. (Evening shift) एक ने 16% की हािन पर एक
Find his overall gain or loss. (a) 50,000 व ु को बेचा। अगर उसने इसे 660
एक आदमी 9,975 पये म दो (b) 25,000 पये अिधक म बेचा होता तो उसे
व ुओं को बेचता है । वह एक व ु (c) 60,000 8% का लाभ होता। यिद व ु को
पर 5% लाभ कमाता है और दू सरी (d) 40,000 3,080 पये म बेचा जाता है , तो
िकतना लाभ ितशत ा होता है

e
पर 5% हािन होती है । उसका सम
लाभ या हािन ात कर। Q52. A man bought an article and CPO 2019 25-11-2020
CGL 2019 Tier-II (18-11-2020 ) sold it at a gain of 10%. If he had (Morning shift)
(a) Profit ₹ 60 bought the article at 20% less and (a) 10%

l
(b) Loss ₹ 50 sold it for Rs1,000 more, he (b) 15%
(c) Loss ₹ 60 would have made a profit of 40%. (c) 20%

SSC CPO 2019


ac
(d) Profit ₹ 50

Q50. A person sold an article at a


loss of 16%. Had he sold it for
The earlier selling price of the
article (in Rs.) is:
एक आदमी ने एक व ु को ख़रीदा
और उसे 10% की लाभ पर बेचा।
यिद उसने 20% कम म व ु ख़रीदा
(d) 12%

Q55. An article was sold at a loss


of 13.5%, If it was sold for Rs.
1,104 more, then there would
होता और इसे 1,000 पये अिधक म have been a profit of 9.5%, The
Rs. 660 more, he would have
बेचता, तो उसने 40% का लाभ cost price (in Rs.)of the article
gained 8%. What should be the
कमाया होता। व ु का आरं िभक was:
nn
selling price (in Rs.) to gain a
िव य मू ( पये म) है : एक व ु को 13.5% की हािन पर
profit of 12%?
CPO 2019 24-11-2020 बेचा गया था, यिद इसे 1,104 पये
एक ने 16% की हािन पर एक
(Morning shift) अिधक म बेचा जाता, तो 9.5% का
व ु को बेचा। अगर उसने इसे 660
(a) 40,000 लाभ होता, तो व ु का लागत मू
पये अिधक म बेचा होता तो उसे
(b) 60,000 ( पये म) थी ात करे ।
8% का लाभ होता। 12% का लाभ
(c) 50,000 CPO 2019 25-11-2020
ा करने के िलए िव य मू (
(d) 55,000 (Evening shift)
म) ा होना चािहए?
(a) 4,800
Pi

CPO 2019 23-11-2020


Q53. If the selling price of 50 (b) 4,400
(Morning shift)
articles is equal to the cost price (c) 4,600
(a) 3,080
of 42 articles, then what is the (d) 4,200
(b) 3,200
approximate loss or profit
(c) 2,750
percentage ?
(d) 2,950
यिद 50 व ुओ का िव य मू 42
व ुओ के लागत मू के बराबर है ,
Q51. A man bought an article and
तो हािन या लाभ ितशत ा है |
sold it at a gain of 10%. If he had
CPO 2019 24-11-2020
bought the article at 20% less and
(Evening shift)
sold it for Rs1,000 more, he
(a) 8% profit/ लाभ
would have made a profit of 40%.

www.ssccglpinnacle.com support@ssccglpinnacle.com Ph. 09729327755, 09817390373

255
Days 28-32 Pro it and Loss

s
sse
la
_c
ob
Variety Questions Let CP of New article = 5 unit ,
bo
ah ⇒ SP = 7 unit 20% = 1 and 25% = 1
sm
5 4
ur
yo

Sol 1. (a) According to the question MP : SP : CP


e/
t.m
://

Let the CP = 100 unit 5 unit = 930 5 : 4


tp
ht

Old SP = 85 unit 1 unit = 186 5 : 4


New SP = 109 unit 7 unit = 1302 Balancing the ratio for SP
According to the question Desired Profit %age = 13021240
−1240 x
MP : SP : CP
109-85 unit = 30.60 100 = 5% 25 : 20 : 16
1 unit = 30.60
24 Alternate : According to the question
3060 (20-16) unit = 44.80
100 unit = 24 Desired Profit %age =
3060 110 (−25) × 40 1 unit = 11.20
Desired Sale Price = 24 x 100 = (-25)+(40)+ 100 = 5%
CP (16 unit) = 11.20 x 16 =
Rs. 140.25
Sol 4. (d) 179.20
Alternate :
(15+9)% = 30.60 Let the SP = 100 unit
Since loss is calculate on SP, CP Sol 6. (b)
1% = 30.60

e
24 7
3060 = 120 unit 14% = 50
110% = 24×100 x 110 = 140.25
According to the question CP : P/L : SP
100 unit = 800 1st article 50 : -7 : 43
Sol 2. (a)
1 unit = 8 2nd article 50 : 7 : 57
Let the CP = 100 unit

l
120 unit = 960 Balancing the ratio for SP
Loss = 4%
ac 125 =
Desired Sale Price = 960 × 100 CP : P/L :
Sale Price = 100 × 100−4
100 = 96 unit
1200 SP
According to the question 1st article 50 x 57 : -7 x 57 : 43
Alternate :
96 unit = 288 x 57
20% = 51 and 25% = 1
4
1 unit = 3 2nd article 50 x 43 : 7 x 43 : 57
100 unit = 300 Old SP : CP : New SP
x 43
Desired percentage = 315300
−300 x 5 : 6
According to the question
4 : 5
100 = 5% [50 x 57 + 50 x 43] unit = 624
Balancing the ratio for CP
nn
Alternate : 1 unit = 50 × 624
(57+43)
1 Old SP : CP : New SP
4% = 25 Desired difference = [50 x 57 - 50
10 : 12 : 15
Let the CP = 25 unit x 43] unit
According to the question
⇒ SP = 24 unit = [50 (57 - 43) ] x 50 × 624
10 unit = 800 (57+43) =
According to the question 1 unit = 80 87.36
24 unit = 288 15 unit = 15 x 80 = 1200
1 unit = 12
Sol 7. (d)
25 unit = 25 x 12 = 300 Sol 5. (d) Article → Price
Pi

Desired percentage = 315300


−300 x
Let the Mark price = 100 unit 11 100
100 = 5% ⇒ Sale price = 100 × 100 80 = 80
9 100
unit Balancing the number of articles
Sol 3. (c) ⇒ Cost Price = 80 × 100 = 64 Article → Price
125
25% = 41 11 x 9 100 x 9
unit
Let the CP = 4 unit and SP = 3 According to the question 9 x 11 100 x 11
unit (80-64) unit = 44.80 ___________________
4 unit = 1240 1 unit = 2.80 Total 198 2000
1 unit = 310 CP (64 unit) = 64 x 2.80 = 179.20 Now,
3 unit = 930 Article → Sale Price
Now, 40% = 52 Alternate : 10 100
⇒ 198 1980

www.ssccglpinnacle.com support@ssccglpinnacle.com Ph. 09729327755, 09817390373

256
Days 28-32 Pro it and Loss

s
se
s
la
_c
ob
Clearly there is a loss of 1st article 25 : 2 : 27
bo
ah
2000-1980 = Rs. 20 2nd article 30 : -3 : 27
sm
ur
yo

20 x 100 = 1%
%age loss = 2000 _______________________
e/
t.m
://

Total 55 : -1 : 54
tp
ht

Sol 8. (a) According to the question


Let the cp of a table = t and cp of 27 unit = 9720
a chair = c According to the question Total Loss (1 unit) = 360
According to the question 2.4 unit = 9
90
1 unit = 24 Sol 13. (a)
6t + 12c = 12000
…………(1) 1000 unit = 90 x 1000 = 3750 According to the question
24
Also 40 x SP = 50 x CP
15 - 12c × 10 = 300 ⇒ CPSP = 5
6t × 100 100 Sol 11. (c) 4
⇒ 9t-12c = 3000 Let the Mark price = 100 unit Let the SP = 5 unit and CP = 4
………..(2) ⇒ Sale price = 100 × 100 80 = 80 unit
Adding eq (1) and (2) ⇒ Profit = 5-4 = 1 unit
unit

e
15 t = 15000 ⇒ Cost Price = 80 × 100 = 800 %age profit = 5−44 x 100 = 25 %
110 11
t = 1000 unit
Total CP of 6 tables = 6 x 1000 = According to the question Sol 14. (d)
6000 100 unit = 550 Total fruits = 40 x 12 = 480

l
1 unit = 5.5 Total CP = 2400
Sol 9. (d) Total SP = 2400 x 100125 = 3000

Rs. 100
ac
Sale Price of one article =

Cost price of one article = 100 x


100
90 =
1000
9
2500
25 =
CP (800/11 unit) = 800
400
11 x 5.5 =

Profit earned = 470-400 = 70


70 x 100 = 17.5%
%age profit = 400
Fruits sold = 480-30 = 450
SP of fruit per dozen = 3000
= 80
450 x 12

Sale Price of the article at 20% Alternate : Sol 15. (d)


profit = 1000
9
120
x 100 = 400
3
Let the total number of articles =
⇒ Number of articles sold in 20% = 1 and 10% = 1 4 and CP of an article = 100 unit
nn
5 10
2400 = 2400
= 18 Total CP = 4 x 100 = 400 unit
400/3 MP : SP : CP 125 = 125 unit
MP = 100 × 100
Alternate : 5 : 4
Discounted Price = 125 × 60 =
Desired Number of Articles = 11 : 10 100
S N × N O × (100 ± K) Balancing the ratio for SP 75 unit
S O × (100 ± M )
MP : SP : CP Total SP = 3 x 125 + 1 x 75 = 450
2400×25×(100−10)
= 2500×(100+20) = 18 55 : 44 : 40 unit
According to the question Profit earned = 450-400 = 50 unit
Desired %age = 40050 x 100 = 12.5
SN = New Sale Price 55 unit = 550
Pi

⇒ 40 unit = 400
SO= Old Sale Price Profit earned = 470-400 = 70 Sol 16. (a)
70 x 100 = 17.5%
%age profit = 400 CP of a Vehicle = 5,90,828
NO = number of article
SP of a vehicle = 6,52,920
k = Old Profit/Loss Sol 12. (c) Profit = 652920-590828 = 62092
2 and 10% = 1 62092 x 100 =
Profit %age = 590828
8% = 25 10
M = New Profit/Loss CP : P/L : SP 10.51
1st article 25 : 2 : 27
Sol 10. (d) Sol 17. (d)
2nd article 10 : -1 : 9
Profit on first article = 3000 × 10
Balancing the ratio for SP 100
Let the CP = 1000
CP : P / L : SP = 300

www.ssccglpinnacle.com support@ssccglpinnacle.com Ph. 09729327755, 09817390373

257
Days 28-32 Pro it and Loss

s
sse
la
_c
ob
Profit on 2nd article = 3000 × 5 Let the SP = 5 unit and CP = 5 4 unit = 2800
bo
100
ah unit 1 unit = 700
sm
= 150
ur
yo

Loss on 3rd article = 3000 × 15 = Profit = 7-5 = 2 unit 5 unit = 5 x 700 = 3500
e/
t.m

100
://

Therefore, Profit% = 52 = 40% SP of 2nd article = 3500-1380 =


tp

450
ht

Total Profit or loss = 2120


300+150-450 = 0 Sol 24. (d) %age Profit earned on 2nd article
Clearly there is no profit and no 4% = 251 = 21201600
−1600 x 100 = 32.5

loss. Let cp of first article = 25 unit


⇒ SP of first article = 26 unit Sol 27. (d)
Sol 18. (c) According to the question 20% = 51 , 25% = 1
4 and 50% =
Average price of rice = 1
26 unit = 6500 2
38×54.50 + 45×62 + 55×48 7501
38+45+55 = 138 = 1 unit = 250 CP : SP
54.35 25 unit = 6250 5 : 6
Profit earned = 65-54.35 = 10.65 Similarly 4 : 3
10.65 x 100 = 19.59
%age profit = 54.35 Let cp of first article = 25 unit 2 : 3

e
or 19.6 ⇒ SP of first article = 24 unit _____________
According to the question 40 : 54
Sol 19. (c) 25 unit = 3750 According to the question
B’s share of the profit = 4 x 1 unit = 150 40 unit = 100

l
14
110166
24 unit = 3600 1 unit = 25
ac 4 Total SP = 6500+3600 = 10100 54 unit = 135
B’s share after charging tax = 14
70
Total CP = 6250+3750 = 10000 Alternate :
x 110166 × = Rs. 22033.20
100 Desired Profit % = 10100 −10000 x
10000
120 × 75 ×
100 × 100 150 = 135
100 100
100 = 1%
Sol 20. (a)
According to the question Sol 28. (a)
Sol 25. (a) 37.5 = 83 and 9.09 = 1
M P × 43 = C P × 100+x
100 3 and 10% = 1
11
15% = 20 10
⇒ 736 × 3 = 460 × 100+x
4 100 Let the CP = 20 unit SP : MP : CP
nn
⇒ 100 + x = 120 ⇒ SP = 23 unit 10 : 11
⇒ x = 20% According to the question 11 : 8
23 unit = 1495 _____________
Sol 21. (c) 1 unit = 65 10 : 11 : 8
Let the manufacturer’s cost = M 20 unit = 1300 Desired profit % = 108−8 x 100 =
According to the question Profit earned (3 unit) = 195
106 × 108 × 110 = 31482 25%
M × 100 100 100 Now,
Therefore, Manufacturer’s cost = CP of 2nd article (10 unit) = 1300 Sol 29. (a)
Pi

Rs. 25,000 Profit (1 unit) = 130 Since the both pieces are similar
SP of 2nd article (11 unit) = 1430 there cp must be same. Since CP,
Sol 22. (c) Net profit on 15 cycles Total profit earned = 195+130 = profit and loss % are same, profit
= Rs. 387 325 and loss amount will also be the
Now, 15% of CP= 387
same.
Therefore, C.P per cycle = Sol 26. (b)
387 × 100 = Rs. 2580
Let the profit amount = loss
15 Total CP = 1200+1600 = 2800 amount = k
25% = 41 According to the question
Sol 23. (a) Given, SP = 7 x CP
5 Let the total CP = 4 unit 18000-10000 = k+k
⇒ SP = 7 ⇒ Total SP = 5 unit
CP 5 k = 4000
According to the question

www.ssccglpinnacle.com support@ssccglpinnacle.com Ph. 09729327755, 09817390373

258
Days 28-32 Pro it and Loss

s
sse
la
_c
ob
CP of the articles = 18000-4000 ⇒ The loss occurred = 3 x 1 =1 Sol 6. (a)
bo
3
ah
or 10000+4000 = 14000 33 31 % = 1
sm
unit 3
ur
yo

Now, 50% = 21 According to the question Initial SP : New SP : CP


e/
t.m
://

CP (2 unit) = 14000 (3+1) unit = 480-400 3 : 1


tp
ht

1 unit = 7000 1 unit = 20 2 :


SP (3 unit) = 7000 x 3 = 21000 ⇒ CP of the article = (400 + 20) 3
or [480 - 3(20)] = 420 Balancing the ratio for New SP
Sol 30. (a) Initial SP : New SP :
33 31 %= 31 and 20% = 1
5 Sol 3. (d) CP
X : New SP : CP 20% of the CP = 30.80 6 : 2 :
3 : 1 ⇒ CP = 30.80
20 x 100 = 154
3
4 : 5 Let the initial SP = 6 unit
60% of the initial SP = 6 × 100 60 =
Balancing the ratio for SP Sol 4. (c)
X : New SP : CP Let the CP = 100 unit 3.60
12 : 4 : 5 SP of the article = 100 x 100−29 = CP = 3 unit
100

e
Desired profit % = 125−5 x 100 = 71 unit Desired profit % = 3.6−3
3 x 100 =
140 According to the question 20%
71 unit = 355
SSC CGL TIER II 1 unit = 5 Sol 7. (b)

l
100 unit = 500 80% = 54 and 10% = 1
10
Sol 1. (c) ac Desired Sale price = 500 x 121 =
100 Initial SP : New SP : CP
605 5 : 4
Let the CP per quintal = 100 unit 9 :
Total CP = 120 x 100 = 12000 Sol 5. (a) 10
Quantity sold on 25% loss = 120 35% = 20 7 and 40% = 2 Balancing the ratio for New SP
5
20 = 24
× 100 Let the total number of goods = Initial SP : New SP : CP
Sale price of 24 quintal = 24 x 75 20 unit and CP of per unit good = 45 : 36 : 40
= 1800 100 Let the initial SP = 45 unit
nn
Sale price of 120 quintal = 120 x 125 = 125
⇒ MP = 100 × 100 CP = 40 unit
125 = 15000 Total CP = 20 x 100 = 2000 Desired profit % = 4540 −40 x 100 =

Sale price of 96 quintal = 137.5 Goods sold at MP = 20 × 20 7 = 7 12 21 %


Profit %age = 137.5 −100 x 100 =
100 unit
37.5% Goods sold at 15% discount = 20 Sol 8. (a)
19% = 10019 and 8% = 2
x 52 = 8 25
Alternate : MP : SP : CP
Money earned from the goods
20% = 51 100 : 81
Pi

sold at MP = 7 x 125 = 875


Let the total quantity = 5 unit Money earned from the goods 27 : 25
Total profit on 5 unit = 5 x 25% = sold at 15% discount = 8 x 125 Balancing the ratio for SP
125% 85 = 850
× 100 MP : SP : CP
Loss on 1 unit = 1 x 25% = -25% 100 : 81 : 75
Money earned from the goods
Total Profit to be gained on Desired %age = 10075−75 x 100 = 33
sold at 20% discount = 5 x 125
remaining 4 units = 125 -(-25) = 80 = 500 1 %
× 100 3
150 %
Desired %age = 150 1 Total earning = 875 + 850 + 500
4 = 37 2 % Sol 9. (d)
= 2225
Desired %age gain = 22252000
−2000 x Let CP of the article = x
Sol 2. (d) Total CP of the 2 articles = 2x
Let the initial profit = 3 unit 100= 11.25
Profit earned = 3x

www.ssccglpinnacle.com support@ssccglpinnacle.com Ph. 09729327755, 09817390373

259
Days 28-32 Pro it and Loss

s
sse
la
_c
ob
Sale Price = Cost Price + Profit SP of 35% (7 unit) goods = 7 x
bo
ah
Sale price of two articles = 800 165 = 1155
100 × 100
sm
ur
yo

⇒ 2x+3x = 800 SP of remaining 13 goods =


e/
t.m
://

⇒ x = 160 1760-1155 = 605


tp
ht

Total CP (2x) = 2 x 160 = 320 SP of 1 good = 605


13 ≈ 46.5
Profit earned = 3 x 160 = 480 Value of x = 100-46.5 = 53.5
Desired percentage = 320480 x 100 =

150 % Sol 13. (c)


According to the question
Alternate : (14+8) % of CP = 121 (9+1) unit = 80
Let the CP of 1 article = 1 unit ⇒ CP = 12122 x 100 = 550
1 unit = 8
⇒ CP of 2 articles = 2 unit Loss = 550-536.25 = 13.75 9 unit = 72
⇒ Profit earned = 3 unit Desired %age = 13.75
550 x 100 = 2.5
Desired %age = 23 x 100 = 150 % % Sol 16. (c)
32% = 25 8 and 12% = 3
25

e
Sol 10. (a) Sol 14. (c) MP : SP : CP
12 and 25% =
9.6% = 125 1
4 18% = 50 9 and 23% = 23 25 : 22
100
SP : MP : CP MP : SP : CP 33 : 25
113 : 125 50 : 41 Balancing the ratio for SP

l
5 : 4 123 : 100 MP : SP : CP
ac
Balancing the ratio for MP Balancing the ratio for MP 75 : 66 : 50
MP : SP : CP MP : SP : CP Desired ratio = 50 : 75
125 : 113 : 100 150 : 123 : 100 = 2:3
Desired %age = 113100 −100 x 100 =
According to the question
123-100 unit = 18.4040 Sol 17. (a)
13 % 3 and 5% = 1
184040
1 unit = 10000 30%= 10 20
× 23
Sol 11. (d) MP (150 unit) = 184040 x 150 MP : SP : CP
10000 × 23
Let the SP = 100 unit 10 : 7
= 120.026 ≈ 120
nn
⇒ CP = 80 21 : 20
Balancing the ratio for SP
Sol 15. (c)
MP : SP : CP
Let the number of articles sold at
30 : 21 : 20
8% profit = x
Let the MP = 30 unit and CP = 20
According to the question
unit
6% of 80 = 8% of x - 12% of
⇒ x = 3020
−20 x 100 = 50 %
(80-x)
⇒ 480 = 8x - 960+12x
Pi

Sol 18. (c)


⇒ x = 72 9 and 40% = 2
36%= 25 5
Desired %age = 20 x 100 = SP : MP : CP
88 Alternate
22.72% 3 : 5
34 : 25
Sol 12. (c) Balancing the ratio for MP
35% = 20 7 MP : SP : CP
Let the total number of goods = 170 : 102 : 125
20 and CP of a good = 100 unit Let the SP = 102 unit and CP =
Total CP = 20 x 100 = 2000 unit 125 unit
Total SP = 2000 × 100100
−12 = 1760

www.ssccglpinnacle.com support@ssccglpinnacle.com Ph. 09729327755, 09817390373

260
Days 28-32 Pro it and Loss

s
sse
la
_c
ob
⇒ Loss % = 125−102 x 100 = 18.4 Desired Profit % = (-30) + 60 + CP P/L SP
bo
125
ah (−30)×60 25 4 29
sm
% = 12%
ur

100
yo

25 -3 22
e/
t.m
://

Sol 19. (b) Sol 3. (d) Balancing the ratio for SP


tp
ht

According to the question 18% = 50 9 and 12% = 3 CP P/L SP


25
(14+16)% of CP = 33 550 88
CP P/L SP
⇒ CP = 30 33 x100 = 110 638
50 9 59
Required SP = 110 x 125 = 725 -87
100 25 -3 22
638
137.50 Balancing the ratio for SP
CP P/L SP
__________________
Sol 20. (a) 1100 198
3 1275 1
15% = 20 1298
1276
Let cp = 20 unit and sp = 23 unit 1475 -177
According to the question
According to the question 1298
638 unit = 5104
23 unit = 29.90

e
1 unit = 8
1 unit = 1.3 __________________
1275 unit = 10200
20 unit = 26 2575 21 8 x 100
Desired loss %age = 10200
2596
According to the question ≈ 0.08%

l
1298 unit = 10384
ac 1 unit = 8 Sol 6. (b)
14% = 50 7 and 12% = 3
21 unit = 21 x 8 = 168 25
CP P/L SP
Sol 4. (c) 50 7 57
15% = 20 3 and 13% = 13 25 -3 22
100
CP P/L SP Balancing the ratio for SP
According to the question 20 3 23 CP P/L SP
1 unit = 6.6 kg 100 -13 87 1100 154
nn
5 unit = 6.6 x 5 = 33 kg Balancing the ratio for SP 1254
CP P/L SP 1425 -171
Practice Questions 1740 261 1254
2001 ___________________
Sol 1. (b) 2300 -299 2575 -17
According to the question 2001 2508
(10.5+8) % of CP = 92.50 According to the question
1 % of CP = 5 __________________ 1254 unit = 2508
Pi

112% of CP (Desired SP) = 112 x 4040 -38 1 unit = 2


5 = 560 4002 17 unit = 34
According to the question 2575 unit = 5150
Desired loss %age = 5150 34 x 100
Sol 2. (b) 2001 unit = 10005
CP = 5400 1 unit = 5 ≈ 0.07%
Loss % = 30 4040 unit = 20200
SP = 5400 × 70
100 = 3780 38 unit = 38 x 5 = 190 Sol 7. (b)
160 190 x 100
Desired loss %age = 20200 18% = 50 9 and 16% = 4
Final SP = 3780 × 100 = 6048 25

Desired Profit % = 6048−5400 x = 0.94% loss CP P/L SP


5400
50 9 59
100 = 12%
Sol 5. (a) 25 -4 21
Alternate : 4 and 12% = 3
16% = 25 25 Balancing the ratio for SP

www.ssccglpinnacle.com support@ssccglpinnacle.com Ph. 09729327755, 09817390373

261
Days 28-32 Pro it and Loss

s
sse
la
_c
ob
CP P/L SP 30 9 650 -169
bo
ah 1050 189 39 481
sm
ur
yo

1239 52 -13
e/
t.m
://

1475 -236 39 __________________


tp
ht

1239 1020 -58


__________________ 962
82 -4 According to the question
__________________ 78 481 unit = 962
2525 -47 According to the question 1 unit = 2
2478 39 unit = 975 58 unit = 116
According to the question 1 unit = 25 1020 unit = 2040
1239 unit = 4956 4 unit = 100 Desired loss %age = 2040116 x 100

1 unit = 4 82 unit = 2050 ≈ 5.7%


47 unit = 188 340 x 100
Desired loss %age = 10100
2525 unit = 10100 ≈ 4.9% Sol 12. (c)

e
188 x 100
Desired loss %age = 10100 25% = 41
≈ 1.9% Sol 10. (c) Let MP = 4 unit
32% = 25 8 and 28% = 7
25 ⇒ SP = 3 unit
Sol 8. (b) CP P/L SP Now,

l
22% = 50 11 and 20% = 1
5 25 8 33 4 unit = 600
ac CP P/L SP 25 -7 18 1 unit = 150
50 11 61 Balancing the ratio for SP SP (3 unit) = 450
5 -1 4 CP P/L SP CP = 450+30 = 480
Balancing the ratio for SP 150 48 Loss % = 48030 x 100 = 6.25 %

CP P/L SP 198
200 44 275 -77 Sol 13. (b)
244 198 Let the Sale Price of = 1 unit
305 -61 Total SP = 72 unit
nn
244 __________________ Loss = 8 unit
425 -29 Total CP = 72+8 = 80 unit
__________________ 396 Loss % = 80 8 x 100 = 10%

505 -17 According to the question


488 198 unit = 4752 Sol 14. (a)
According to the question 1 unit = 24 According to the question
244 unit = 4880 29 unit = 696 (8+10.5) % of CP = 37
1 unit = 20 425 unit = 10200 ⇒ CP = 18.537 x 100 = 200
Pi

17 unit = 340 696 x 100


Desired loss %age = 10200
505 unit = 10100 ≈ 6.8%
340 x 100 Sol 15. (c)
Desired loss %age = 10100 3 and 8% = 2
3% = 100 25
≈ 3.4% Sol 11. (b)
3 and 26% = 13
SP1 : CP : SP2
30% = 10 50 97 : 100
Sol 9. (a) CP P/L SP
3 and 25% = 1
25 : 27
30% = 10 4 10 3 13 Balancing the ratio for CP
CP P/L SP 50 -13 37 SP1 : CP : SP2
10 3 13 Balancing the ratio for SP 97 : 100 : 108
4 -1 3 CP P/L SP Now, 97 unit = 291
Balancing the ratio for SP 370 111 1 unit = 3
CP P/L SP 481

www.ssccglpinnacle.com support@ssccglpinnacle.com Ph. 09729327755, 09817390373

262
Days 28-32 Pro it and Loss

s
sse
la
_c
ob
108 unit = 324 ⇒ CP = 320-32 = 288 3 unit = 397.5
bo
ah 32 40 unit = 5300
sm
Desired Profit % = x 100 = 11
ur

288
yo

Alternate : 1 Desired Profit % = 397.5 x 100 =


e/

%
t.m

9 5300
://

7.5%
tp
ht

According to the question Sol 19. (c)


97% of CP = 291 Profit earned on the whole Sol 21. (c)
CP = 291
97 x 100 20 = 120
transaction = 600 x 100 10% = 10 1 and 20% = 1
5
Desired Sale price = 108% of CP Loss on one article = 200 x 10 = CP P/L SP
100
= 291 108
97 x 100 × 100 = 324 10 1 11
20
Profit earned on remaining two 5 1 6
Sol 16. (a) articles = 120+20 = 140 Balancing the ratio for CP
16% = 25 4 and 10% = 1 CP P/L SP
10 140 x 100 = 35
Desired profit % = 400
CP P/L SP 10 1 11
25 4 29 10 2 12
Alternate :
10 -1 9

e
Balancing the ratio for SP 1 1 __________________
10% = 10 and 20% = 5
CP P/L SP 20 3 23
CP P/L SP According to the question
225 36
One Article 10 -1 9 10 unit = 3050
261

l
Three Articles 5 1 6 1 unit = 305
290 -29
Balancing the ratio for CP 3 unit = 915
261 ac CP P/L SP 20 unit = 6100
10 -1 9 915 x 100 =
Desired Profit % = 6100
__________________
30 6
515 7 15%
36
522
According to the question
According to the question Sol 22. (c)
30 unit = 600 1 and 20% = 1
261 unit = 5220 10% = 10 5
1 unit = 20
1 unit = 20 CP P/L SP
6 unit = 120
nn
7 unit = 140 10 -1 9
Total profit earned on 2 articles =
120+20 = 140 5 1 6
Sol 17. (c) 140 x 100 = 35 Balancing the ratio for CP
Desired profit % = 400
According to the question CP P/L SP
125% of CP = 2300 10 -1 9
Sol 20. (c)
CP = 2300
125 x 100 = 1840 1 and 5% = 1 10 2 12
10% = 10 20
Profit earned = 1955-1840 = 115
115 x 100 = 6.25 CP P/L SP __________________
Desired %age = 1840
Pi

10 1 11 20 1 21
%
20 1 21 According to the question
Balancing the ratio for CP 10 unit = 3050
Sol 18. (b)
CP P/L SP 1 unit = 305
20% = 51
20 2 22 20 unit = 6100
MP : SP 20 1 21 305 x 100 =
Desired Profit % = 6100
5 : 4
According to the question 5%
__________________
5 unit = 400 40 3 43
1 unit = 80 Sol 23. (a)
According to the question 15
SP (4 unit) = 320 Loss on one article = 4158 x 100
20 unit = 2650
Profit = 32 1 unit = 132.5 = 623.70

www.ssccglpinnacle.com support@ssccglpinnacle.com Ph. 09729327755, 09817390373

263
Days 28-32 Pro it and Loss

s
se
s
la
_c
ob
Profit to be earned on 2nd article 13100 -3799 338 unit = 338
bo
ah
so that no profit/loss = 623.70 9301 20000 unit = 20000
sm
ur
yo

Profit % = 623.70
4158 x 100 = 15% Desired Profit % = 338 x 100 =
e/
t.m

20000
://

Alternate : __________________ 1.69%


tp
ht

Since CP is same for both the 20200 -1598 Alternate :


articles for our convenience we 18602 Trick : When SP, Loss and Profit
can assume any value of CP to According to the question percentage are same
find the profit/loss percentage. 9301 unit = 18602 Loss % = 13×13
100 = 1.69
For example assume CP = Rs. 1 unit = 2
100 1598 unit = 3196 Sol 31. (c)
Loss on one article = 100 x 10015 = 20200 unit = 40400 25% = 41 and 20% = 1
3196 5
15 Desired Loss % = 40400 x 100 =
MP : SP : CP
Profit to be earned on 2nd article 7.91% 4 : 3
so that no profit/loss = 15 6 : 5
15 x 100 = 15%
Profit % = 100 Sol 28. (c) Balancing the ratio for SP

e
According to the question MP : SP : CP
Sol 24. (d) 72% of CP = 144 8 : 6 : 5
CP = 4,90,828 CP = 144
72 x 100 According to the question
Desired SP = 86% of CP = 144 x
SP = 5,52,920 72 8 unit = 936

l
Profit earned = 62092 100 × 86 = 172 1 unit = 92
100
ac 62092 x 100
Desired Profit % = 4,90,828 5 unit = 460
= 12.65% Sol 29. (c)
According to the question Sol 32. (a)
30% = 10 3 and 20% = 1
Sol 25. (c) 4 × CP = 3 × SP 5
CP = 4,89,828 ⇒ SPCP = 3
4
MP : SP : CP
SP = 5,89,828 Let CP = 3 unit and SP = 4 unit 10 : 7
Profit earned = 1,00,000 ⇒ Profit earned = 4-3 = 1 unit 6 : 5
Desired Profit % = 1,00,000
x 100 Profit %age = 31 × = 33 31 % Balancing the ratio for SP
nn
4,89,828
MP : SP : CP
= 20.42%
60 : 42 : 35
Sol 30. (d)
13 According to the question
Sol 26. (a) 13% = 100
15 60 unit = 480
Total Profit earned = 6000 x ( 100 CP P/L SP
12 15
1 unit = 8
+ - ) = 720 100 -13 87
100 100 35 unit = 8 × 35 = 280
⇒ Total profit percentage = 720 100 13 113
18000
x 100 = 4% Balancing the ratio for SP
Sol 33. (a)
CP P/L SP
Pi

23% = 10023

Sol 27. (a) 11300 -1469


CP P/L SP
31% = 10031 and 29% = 29 9831
100 100 -23 77
8700 1131
CP P/L SP 100 23 123
9831
100 31 Balancing the ratio for SP
131
__________________
100 -29 71 CP P/L SP
20000 -338
Balancing the ratio for SP 12300 -2829 9471
19662
CP P/L SP 7700 1771 9471
According to the question
7100 2201 ________________
9831 unit = 9831
9301 20000 -1058 18942
1 unit = 1
According to the question

www.ssccglpinnacle.com support@ssccglpinnacle.com Ph. 09729327755, 09817390373

264
Days 28-32 Pro it and Loss

s
sse
la
_c
ob
9471 unit = 9471 Trick : When SP, Loss and Profit 207 unit = 228528
bo
ah
1 unit = 1 percentage are same 1 unit = 1104
sm
ur
yo

1058 unit = 1058 Net loss = 19×19


100 = 3.61 % 200 unit = 220800
e/
t.m
://

20000 unit = 20000


tp
ht

1058 Sol 40. (b)


Desired Profit % = 20000 x 100 = Sol 36. (a)
5.29% 20% = 51 20% = 51
10% = 1 SP1 : CP : SP2
10
Alternate : SP of A is cp of B and sp of B is 4 : 5
Trick : When SP, Loss and Profit cp of C 5 : 6
percentage are same 4 unit = 320
Loss % = 23×23
100 = 5.29 A:B:C 1 unit = 80
10: 9 6 unit = 480
Sol 34. (a) 5:6 Alternate :
25% = 41 and 20% = 1
Balancing the ratio for B According to the question
5
MP : SP : CP A: B :C 80% of CP = 320

e
4 : 3 CP 50 : 45 : 54 CP = 320
80 x 100
Desired SP = 120% of CP = 320 x
6 : 5 80
54 unit = 540000 100 x 120 = 480
Balancing the ratio for SP 100
MP : SP : CP 1 unit = 10,000

l
8 : 6 :5 50 unit = 5,00,000 Sol 41. (d)
According to the question
ac 25% = 41 and 37.5% = 3
8
5 unit = 460 Sol 37. (b) SP1 : CP : SP2
1 unit = 92 25% = 41 and 10% = 1
10 5 : 4
8 unit = 8 × 92 = 736 MP : CP : SP 8 : 5
5 : 4 Balancing the ratio for CP
Sol 35. (d) 10 : 11 SP1 : CP : SP2
19% = 10019
Balancing the ratio for CP 10 : 8 : 5
CP P/L SP MP : CP : SP According to the question
nn
100 -19 81 25 : 20 : 22 (10-5) unit = 1250
100 19 25 unit = 2340 1 unit = 250
119 1 unit = 93.6 CP (8 unit) = 8 x 250 = 2000
Balancing the ratio for SP Profit earned (2 unit) = 187.2 Desired SP = 2000 × 112.5 100 =
2250
CP P/L SP Sol 38. (b)
11900 -2261 9639 According to the question Alternate :
8100 1539 9639 (12-10) % of CP = 77 (37.5+25) % of CP = 1250
Pi

__________________ CP = 772 x 100 = 3850 ⇒ CP = 1250


62.5 x 100
20000 -722 19278
Desired SP = 112.5% of CP =
According to the question Sol 39. (d) 1250 112.5
3 and 10% = 1 62.5 x 100 x 100 = 2250
9639 unit = 9639 15% = 20 10
1 unit = 1 CP : MP : SP
Sol 42. (a)
722 unit = 722 20 : 23 3
15% = 20
20000 unit = 20000 10 : 9
722 x 100 =
Desired Profit % = 20000 SP1 : CP : SP2
Balancing the ratio for MP
23 : 20
3.61 % CP : MP : SP
20 : 17
200 : 230 : 207
Balancing the ratio for CP
Alternate : According to the question
SP1 : CP : SP2

www.ssccglpinnacle.com support@ssccglpinnacle.com Ph. 09729327755, 09817390373

265
Days 28-32 Pro it and Loss

s
sse
la
_c
ob
23 : 20 : 17 Sol 45. (b) CP of the article = 1800
bo
ah
According to the question Let the CP = 100 unit Profit % = 32
sm
ur
yo

23 unit = 1725 (100+ 100 ) Desired SP = 1800 x 100132 = 2376


= 800
e/

⇒ SP = 100 x 7
t.m

100 7
://

1 unit = 75 Alternate :
tp

Desired Ratio = 800 : 100


ht

7
Desired SP = 17 x 75 = 1275 8
32% = 25
=8:7
Alternate : According to the question
Alternate : 25 unit = 1800
14 72 % = 71
115 % of CP = 1725 1 unit = 72
⇒ CP = 1725 Let the CP = 7 unit
115 x 100 SP (33 unit) = 33 x 72 = 2376
1725
⇒ Profit = 1 unit
Desired SP = 85% of CP = 115 x
So, SP = 7+1 = 8 unit
100 x 85 = 1275 Sol 50. (b)
100 ⇒ Desired ratio = 8 : 7 ans
20% = 51
Sol 43. (d) Sol 46. (c) Let the SP = 5 unit and Profit = 1
15% = 20 3
Let the CP = 100 unit unit
23 unit = 2070 ⇒ CP = 5-1 = 4 unit
⇒ SP = 100 x (100+22.5) = 122.5

e
100
1 unit = 90 Profit % = 41 x 100 = 25 %
Desired Ratio = 100 : 122.5
CP (20 unit) = 20 x 90 = 1800 = 40 : 49
Profit earned =1890-1800 = 90 Sol 51.(c)
Alternate :
Desired gain % = 1800 90 x 100 = 25% = 41

l
9
22.5 % = 40
5% Let the CP = 4 unit and SP = 1
ac Let the CP = 40 unit
⇒ Profit = 9 unit unit
Alternate : ⇒ Loss = 4-1 = 3 unit
So, SP = 40+9 = 49 unit
115 % of CP = 2070 ⇒ Desired ratio = 40 : 49 ans Loss % = 43 x 100 = 75 %
⇒ CP = 2070
115 x 100 = 1800
Profit earned =1890-1800 = 90 Sol 47. (a) Sol 52. (d)
90 x 100 = 15% = 20 3 and 12% = 3
Desired gain % = 1800 20% = 51 25
5% CP : P/L : SP CP P/L SP
20 3 23
nn
Article 1 5 1 6
Sol 44. (d) Article 2 5 -1 4 25 3 28
10% = 10 1 and 20% = 1 Balancing the ratio for CP
5 For article 1
SP1 : CP : SP2 6 unit = 180 CP P/L SP
9 : 10 1 unit = 30 100 15 115
5 : 4 5 unit = 5 x 30 = 150 100 12 112
Balancing the ratio for CP For article 2 According to the question
SP1 : CP : SP2 4 unit = 240 (115-112) unit = 18
1 unit = 6
Pi

9 : 10 : 8 1 unit = 60
According to the question 5 unit = 5 x 60 = 300 CP (100 unit ) = 600
9 unit = 810 Total CP of two articles =
1 unit = 90 150+300 = 450 Sol 53. (a)
8 unit = 8 x 90 = 720 20% = 51
Sol 48. (c) Let the CP = 5 unit and Profit = 1
Alternate : According to the question unit
90 % of CP = 810 (18+20) % of CP = 570 ⇒ SP = 5+1 = 6 unit
⇒ CP = 810
90 x 100 CP = 570 According to the question
38 x 100 = 1500
Desired SP = 80% of CP = 810 x Desired SP = 1500 x 88 = 1320 (6-5) unit = 180
90 100
100 x 80 = 720 SP (6 unit) = 6 x 180 = 1080
100
Sol 49. (b)

www.ssccglpinnacle.com support@ssccglpinnacle.com Ph. 09729327755, 09817390373

266
Days 28-32 Pro it and Loss

s
sse
la
_c
ob
Sol 54. (d) Sol 58. (d) ⇒ Loss = 5-4 = 1 unit
bo
ah
30% = 10 3 According to the question
sm
According to the question
ur
yo

Let the CP = 10 unit and loss = 3 (17+15)% of CP = 96 4 unit = 80


e/
t.m
://

96 x 100
CP = 32
unit 1 unit = 20
tp
ht

⇒ SP = 10-3 = 7 unit Desired SP = 96 x 100 × 110 =


32 100
According to the question 330 Sol 64. (b)
16% = 25 4 and 12% = 3
7 unit = 84 25
1 unit = 12 Sol 59. (d) MP : SP : CP
CP (10 unit) = 10 x 12 = 120 30% = 10 3 and 45% = 9
20
25 : 21
Clearly if the article is sold at Rs. MP : CP : SP 28 : 25
120, there will be no profit and no 13 : 10 Balancing the ratio for CP
loss. 20 : 29 MP : SP : CP
Balancing the ratio for CP 100 : 84 : 75
Sol 55. (b) MP : CP : SP According to the question
20% = 51 26 : 20 : 29 75 unit = 12000

e
Let the CP = 5 unit and profit = 1 According to the question 1 unit = 160
unit 26 unit = 2600 MP (100 unit) = 160 x 100 =
⇒ SP = 5+1 = 6 unit 1 unit = 100 16000
According to the question 29 unit = 2900

l
6 unit = 2400 Sol 65. (c)
1 unit = 400 ac Sol 60. (b)
3
40% = 52 and 15% = 20
⇒ There was profit of Rs. 400 on 10% of CP = 960 × 10
100 =96 CP : MP : SP
one article but overall there was New SP = 1392-96 = 1296 5: 7
no profit and no loss so the loss Profit earned = 1296-960 = 336 20 : 17
on the other article must be Rs. Desired %age of profit = 960336 x Balancing the ratio for CP
400. CP : MP : SP
100 = 35%
100 : 140 : 119
Sol 56. (a) Desired Profit % = 119100−100 x 100
Sol 61. (c)
nn
Given, Given, = 19 %
Profit = 53 of Sale Price CP = 76 of Sale Price
Let the sale price = 5 unit Sol 66. (d)
Let the sale price = 7 unit and
⇒ Profit = 3 unit Let the total number of goods are
cost price = 6 unit
According to the question ⇒ profit = 1 unit 5 unit and Price of each good =
5 unit = 120 100
Profit % = 61 x 100 = 16.67 %
1 unit = 24 Total CP = 5 x 100 = 500
⇒ 3 unit = 72 20% of 5 = 1
Sol 62. (c)
Pi

40% of 5 = 2
25% = 41
Sol 57. (d) Sale Price of 20% of goods sold
3 Let the CP = 4 unit and Profit = 1 150 =
at 50% profit = 1 x 100 x 100
30% = 10
unit
Let the CP = 10 unit and Profit = 150
⇒ SP = 4+1 = 5 unit
3 unit Sale Price of 40% of goods = 2 x
Desired %age = 51 x 100 = 20% 80 = 160
100 x 100
⇒ SP = 13 unit
According to the question Sale Price of 20% of goods sold
Sol 63. (c) 95 = 95
at 5% loss = 1 x 100 x 100
10 unit = 24000
SP : CP
1 unit = 2400 Sale of remaining goods = 1 x
4 : 5
13 unit = 31200 100 = 100
Let the SP = 4 unit and CP = 5
unit

www.ssccglpinnacle.com support@ssccglpinnacle.com Ph. 09729327755, 09817390373

267
Days 28-32 Pro it and Loss

s
sse
la
_c
ob
Total Sale Price = 3 unit = 3 x 11 = 33 New CP = 100 + (100 × 100 ) =
bo
100
ah
150+160+95+100 = 505
sm
200 unit
ur
yo

Profit earned = 505-500 = 5 Sol 71. (a) Profit earned = 500-200 = 300
e/
t.m
://

%age profit = 5005 x 100 = 1% According to the question unit


tp
ht

(23-14) % of CP = 189 Desired ratio = 300 : 500


Sol 67. (c) ⇒ CP = 189 9 x 100 = 2100 3:5
25% = 41 Alternate :
Let the CP = 4 unit and Profit = 1 Sol 72. (b) 400% = 14
unit According to the question CP : SP
⇒ SP = 4+1 = 5 unit c:d = 100:150 1 5
According to the question Let the CP (c) = 100 unit Let the CP = 1 unit
5 unit = 2400 And SP (d) = 150 unit ⇒ SP = 5 unit
1 unit = 480 Profit earned = 150-100 = 50 unit After 100% increment new CP =
Desired Profit % = 10050 x 100 =
4 unit = 4 x 480 = 1920 2 unit
Profit earned = 2160-1920 = 240 50% Profit earned = 5-2 = 3 unit

e
240 x 100 =
Desired profit % = 1920 Desired ratio = 3 : 5
12 21 % Sol 73. (b)
Total Profit / Loss earned = 3000 Sol 75. (b)
15 + 10 - 15 ) = 300
× ( 100
Sol 68. (c) 100 100 According to the question

l
12.5% = 81 Note : Positive sign indicates CP : SP
ac that there was profit on the 4 : 3
Let the CP = 8 unit and Profit = 1
whole transaction. Let the CP = 4 unit
unit
Total CP = 3 x 3000 = 9000 And SP = 3 unit
⇒ SP = 8+1 = 9 unit
300 x 100 =
Desired Profit % = 9000 Loss = 4-3 =1 unit
According to the question
10 % Loss% = 41 x 100 = 25%
8 unit = 480 3
1 unit = 60
9 unit = 9 x 60 = 540 Alternate : Sol 76. (b)
3 and 10% =
15% = 20 1 Total number of nuts = 30 x 12 =
nn
10
Sol 69. (b) 360
CP : SP
1
20% = 51 and 10% = 10 Total CP = 14400
Article 1 20 23
SP of 5 nuts = 250
A:B:C Article 2 10 11
SP of 360 nuts = 250 x 72 =
5 :6 Article 3 20 17
18000
10 : 11 Balancing the ratio for CP
Profit earned = 18000-14400 =
____________ CP : SP
3600
50 : 60 : 66 Article 1 20 23
According to the question Article 2 20 22
Pi

Sol 77. (c)


66 unit = 6666 Article 3 20 17 1 and 12.5% = 1
10% = 10
1 unit = 101 + ___________ 8

60 unit = 6060 Total 60 62 According to the question


Desired Profit % = 6260−60 x 100 = SP1 : SP2 : CP
Sol 70. (a) 10 10 : 9
3 %
25% = 41 9 : 8
_________________________
Let the SP = 4 unit and Profit = 1 Sol 74. (c)
10 : 9 : 8
unit Let the CP = 100 unit
400
Now,
⇒ CP = 4-1 = 3 unit Profit earned = 100 × 100 = 400
10 unit = 500
According to the question unit 1 unit = 50
1 unit = 11 ⇒ SP = 100+400 = 500 unit 8 unit = 8 x 50 = 400

www.ssccglpinnacle.com support@ssccglpinnacle.com Ph. 09729327755, 09817390373

268
Days 28-32 Pro it and Loss

s
sse
la
_c
ob
And 985 unit = 19700
bo
ah
Sol 78. (c) 110 + b ×
a × 100 120 = 1390 33 unit = 660
sm
ur

100
yo

Total Nuts = ( 30 x 12 + 32 x 20) Desired Loss %age = 660 x 100


11a + 12b = 13900 ……….(2)
e/
t.m

19700
://

= 1000 Multiply equation 1 by 11 and = 3.35% ≈ 3.4%


tp
ht

Total CP = 14400+57600 = subtract it from (2) Note : To calculate %age data


72000 11a + 12b = 13900 we don’t need to find the exact
SP of 5 nuts = 432 11a + 11b = 13200 values. For example here
⇒ SP of 1000 nuts = 432 x 200 = -________________ CP = 985 unit
86400 b = 700 Loss = 33 unit
Profit earned = 86400-72000 = ⇒ a = 1200-700 = 500 33 x 100 =
Desired Loss %age = 985
14400 Desired ratio = 500 : 70 3.35% ≈ 3.4%
Desired Profit % = 14400
72000 x 100 = =5:7
20% Sol 85. (a)
Sol 83. (a) SP = 702
Sol 79. (c) Cost price of D = 1400 × 125
100 Profit = 162

e
Total CP = 2 x 40000 = 80000 × 120 × 85 = 1785 CP = 702-162 = 540
100 100
125 = 100000
Total SP = 80000 x 100 Alternate : Extra profit to be earned = 540
⇒ SP of second bike = 10 = 54
× 100
25% = 41 , 20% = 1
5 and 15% =
100000-48000 = 52000 3 Desired SP = 702 + 54 = 756

l
20
Alternate :
%age profit earned on first bike =
ac Sol 86. (d)
48000−40000 x 100 = 20%
4 5
10% = 10 1 and 8% = 2
40000 5 6 25
Let the %age profit earned on 2nd 20 17 MP : SP : CP
bike = x % ______________ 10 : 9
According to the question 400 510 27 : 25
20+x = 25
2 According to the question Balancing the ratio for SP
...(Cost Price of the bikes are 400 unit = 1400 MP : SP : CP
same) 1 unit = 3.5 30 : 27 : 25
nn
⇒ x = 30 % 510 unit = 1785 According to the question
130
SP of the 2nd bike = 40000 x 100 30 unit = 480
= 52000 Sol 84. (c) 1 unit = 16
12% = 25 3 and 15% = 3
20
25 unit = 400
Sol 80. (b)
Desired %age of profit = CP P/L SP Sol 87. (c)
1800−1440 x 100 = 25% Two third = 2 and 20% = 1
1440 25 3 28 3 5
20 -3 17 Let the total number of articles =
Pi

Sol 81. (d) 15 …..(LCM of 3 and 5)


According to the question Balancing the ratio for SP and the CP of each article = 100
(20+17.5) % of CP = 1440 unit
CP of the article = 1440 CP P/L SP CP of the two-third of the articles
37.5 x 100
1440 425 51 476 = 15 × 32 x 100 = 1000 unit
New SP of the article = 37.5 x
115 560 -84 476 Profit earned on two-third of the
100 × 100 = 4416 25 = 250 unit
__________________ articles = 1000 × 100
985 -33 952 Loss on the two-third of the
Sol 82. (a) According to the question articles (3 articles) = 3 x 100
Let the CP of A = a and B = b 476 unit = 9520 20 = 60 unit
× 100
According to the question 1 unit = 20
a+b = 1200 …….(1)

www.ssccglpinnacle.com support@ssccglpinnacle.com Ph. 09729327755, 09817390373

269
Days 28-32 Pro it and Loss

s
se
s
la
_c
ob
Profit earned on the remaining _____________
bo
ah
articles (2 articles) = 2 x 100 Sol 90. (b) 20 1 21
sm
ur
yo

20 = 40 unit
× 100 20% = 51 Desired Profit %age = 1 x 100 =
e/
t.m

20
://

Total profit earned = 250-60+40 = Let the CP = 5 unit 5


tp
ht

230 unit Loss = 1 unit


According to the question ⇒ SP =5-1 = 4 unit Sol 95. (b)
270 unit = 3312 According to the question Let the CP of Car = c and Bike =
1 unit = 14.4 4 unit = 230 b
Total CP (1500) = 14.4 x 1500 = 1 unit = 57.5 According to the question
21600 5 unit = 57.5 x 5 = 287.5 c + b = 500000 ……….(1)
New SP = 339.25 Also
Sol 88. (b) Profit earned = 339.25-287.50 = 20% of b - 10% of c = 5% of
1250 500000
Sale Price of one article = 25 = 51.75
51.75 x 100 = 18
%age Profit = 287.50 2b - c = 250000 ………(2)
Rs. 50
Cost price of one article = 50 x Adding (1) and (2)

e
100 = 500 Sol 91. (d) 3b = 750000
90 9
Sale Price of the article at 17% 21 x CP = 20 x SP b = 250000
CP = 20
profit = 500 117
9 x 100 = 65 SP 21
Let the CP = 20 unit and SP = 21 Alternate :
⇒ Number of articles sold in

l
2600 = 2600
= 40 nit
65
ac Profit = 21-20 = 1 unit
Alternate : 1 x 100 = 5 %
%age profit = 20
Desired Number of Articles =
S N × N O × (100 ± K) Sol 92. (a)
S O × (100 ± M )
According to the question
(30+8) % of CP = 950
2600×25×(100−10)
= = 40 ⇒ CP = 950
1250×(100+17) 38 x 100 = 2500
nn
SN = New Sale Price Sol 93. (a)
9
According to the question
90% = 10
SO= Old Sale Price (1+1) unit = 500000
Let the SP = 10 unit and CP = 9 1 unit = 250000
unit
NO = number of article
⇒ Profit = 10-9 = 1 unit Sol 96. (a)
k = Old Profit/Loss %age profit = 91 x 100 = 11 91 % 33 31 % = 31 and 25% = 1
4
Let the total number of mangoes
M = New Profit/Loss Sol 94. (b)
Pi

= 12 and cp of a mango = 100


20% = 51 and 10% = 1
10 unit
Sol 89. (d)
10% = 10 1 and 20% = 1 Mangoes sold at 22.5% of profit
5
CP P/L SP = 12 × 31 = 4
MP : CP : SP
5 1 6 Profit earned on the mangoes sold
6 : 5
10 -1 9 at 22.5% of profit = 400 x 22.5
10 : 9 100 =

Balancing the ratio for CP 90


Balancing the ratio for CP
MP : CP : SP Mangoes sold at 22.5% of profit
12 : 10 : 9 = 12 × 41 = 3
CP P/L SP
%age Discount = 1212−9 x 100 = Loss on the mangoes sold at 25%
10 2 12 25 = 75
25% of loss = 300 x 100
10 -1 9

www.ssccglpinnacle.com support@ssccglpinnacle.com Ph. 09729327755, 09817390373

270
Days 28-32 Pro it and Loss

s
sse
la
_c
ob
Loss on the remaining article = 4 : 3 Sol 106. (a)
bo
ah 3 = 15
500 x 100 5 : 7 According to the question
sm
ur
yo

Total profit/loss on the whole Balancing the ratio for SP 24 x sp = 26 x cp


e/
t.m

sp
://

CP : SP : New SP 13
transaction = 90-75-15 = 0 cp = 12
tp
ht

Clearly there is no profit and no 20 : 15 : 21 Let the cp = 12 unit and sp = 13


loss. Desired gain % = 2120 −20 x 100 =
unit
5% ⇒ profit = 13-12 = 1 unit
Sol 97. (b) Desired %age of profit = 12 1 x

According to the question Sol 102. (d) 100 = 25 %


1 3
(13.5 + 31) % of CP = 8900 4% = 25
⇒ CP = 8900
44.5 x 100 = 20000 Let the CP =25 unit and loss = 1 Sol 107. (c)
Profit earned= 20740-20000 = unit 10% = 101 and 25% = 1
4
740 ⇒ SP = 25-1 = 24 unit
SP : MP : CP
740 x 100 = 3.7
%age profit = 20000 According to the question 9 : 10
% 24 unit = 806.40 5 : 4

e
1 unit = 33.6 Balancing the ratio for MP
Sol 98. (a) 25 unit = 840 SP : MP : CP
40% = 52 and 25% = 1 Desired %age = 882840−840 x 100 =
9 : 10 : 8
4
According to the question 5% According to the question

l
MP : SP : CP 9 unit = 360
ac
5 : 3 Sol 103. (c) 1 unit = 40
5 : 4 40% = 52 8 unit = 8 x 40 = 320
Balancing the ratio for SP Since profit is calculated on the
MP : SP : CP sale price. Sol 108. (d)
25 : 15 : 12 Let SP = 5 unit and Profit = 2 unit 20% = 51 and 16% = 4
25
Desired ratio = 12 : 25 ⇒ CP = 5-2 = 3 unit MP : SP : CP
Actual %age of profit = 32 x 100 5 : 4
Sol 99. (c) = 66 32 % 29 : 25
nn
6.25% = 16 1
Balancing the ratio for SP
Let the CP = 16 unit and SP = 15 Sol 104. (d) MP : SP : CP
unit Total profit earned = 200 × 5 + 145 : 116 : 100
100
According to the question 300 × 10 = 40 According to the question
100
15 unit = 300 40 100 unit = 425
Desired %age of profit = 200 + 300
1 unit = 20 1 unit = 17
4
16 unit = 320 x 100 = 8% 17
145 unit = 4 x 145 = 616.25
Profit earned = 352-320 = 32
Pi

Sol 105. (c)


20% = 51 Sol 109.
Sol 100. (c) 40% = 52 and 25% = 1
Let the CP = 5 unit and loss = 1 4
Let the SP of a article = 1
unit SP : MP : CP
SP of 30 articles = 30
⇒ SP = 5-1 = 4 unit 3 : 4
SP of 9 articles = Profit = 9
According to the question 7 : 5
CP of 30 articles = 30-9 = 21
9 x 100 = 42 6 4 unit = 96 Balancing the ratio for MP
%age profit = 21 7
1 unit = 24 MP : SP : CP
5 unit = 120 28 : 21 : 20
Sol 101. (d) Let CP = 20 unit and SP = 21
115 =
Desired sale price = 120 x 100
25% = 41 and 40% = 2
5 unit
CP : SP : New SP 138

www.ssccglpinnacle.com support@ssccglpinnacle.com Ph. 09729327755, 09817390373

271
Days 28-32 Pro it and Loss

s
sse
la
_c
ob
Desired gain % = 21−20 x 100 = 20 : 15 : 21 CP : P/L : SP
bo
20
ah Required profit %age = 21−20 x 4 +1 5
sm
5% 20
ur
yo

100 = 5% 8 -3 5
e/
t.m
://

Sol 110. (c) Balancing the ratio for CP


tp
ht

Let the total number of goods = Alternate : CP : P/L : SP


10 and cp of a good = 100 unit Final sale price of the article = 8 +2 10
⇒ MP = 140 unit 75 x 140 = 1302
1240 x 100 8 -3 5
100
Total CP = 10 x 100 = 1000 unit Required profit %age = 1302−1240 According to the question
1240
Sp of 70% goods = 7 x 140= 980 (10-5) unit = 1250
x 100 = 5%
unit 1 unit = 250
SP of remaining goods = 3 x 140 8 unit = 250 x 8 = 2000
Sol 114. (b) 112.5
60 = 252 unit
x 100 Required SP = 2000 x 100 =
Had both articles were sold on
Total SP = 980+252 = 1232 unit 12% profit, total profit = 2000 x 2250
Desired %age of profit = 12
100 = 240
1232−1000 x 100 = 23.2 % Sol 118. (a)
1000 Had both articles were sold on

e
15% = 203
12
12% loss, total loss = 2000 x 100
Sol 111. (b) CP : SP1 : SP2
= 240
25% = 41 and 10% = 1 20 : 23 : 17
10 Overall profit on the both
SP : MP : CP 1.2 = 24 According to the question
transaction = 2000 x 100

l
9 : 10 23 unit = 1725
5 : 4 ac 1 unit = 75
Balancing the ratio for MP 17 unit = 1275
MP : SP : CP
10 : 9 : 8 Sol 119. (c)
Desired profit %age = 9−88 x 100 25% = 41

= 12.5 % Let the cost price = 4 unit


⇒ the sale price = 1 unit
Required %age = 4−1 × 100 =
Sol 112. (a) 4
nn
Let the sale price of a mobile = 1 75%
unit According to the question
Total sale price = 36 x 1 = 36 unit (11+9) unit = 2000 Sol 120. (b)
Loss amount = 4 unit 1 unit = 100 Cost Price = 960
Cost price of the article = 36+4 = 11 unit = 11x100 = 1100 Sale Price = 1392
40 unit Effective sale price = (1392 - 960
4 x 100 = 10 ) = 1296
x 100
Required loss % = 40 Sol 115. (c)Let the cp of mobile =
Required profit %age = 1296−960 x
10% x
Pi

960
According to question 100 = 35%
Sol 113. (c) x(25-20)% = 1000
25% = 41 and 40% = 2
5
X = 20000 Sol 121. (a)
Initial paid = 20000 × 100 80 =
CP : SP1 : SP2 Cost Price = 1,35,000
4 : 3 16000 Sale Price = 1,10,000
5 : 7 Required %age = 135000 −110000 x
135000
Balancing the ratio for SP1 Sol 116. (a) Overall loss % = 100 = 500 %
27
11×11 = 1.21%
CP : SP1 : SP2 100
4 : 3 : 3 Sol 122. (c) Total cost of the
5 : 5 : 7 Sol 117. (d) scooter = 30000+3000 = 33,000
___________ 25% = 41 and 37.5% = 3
8 Sale Price = 39,600

www.ssccglpinnacle.com support@ssccglpinnacle.com Ph. 09729327755, 09817390373

272
Days 28-32 Pro it and Loss

s
sse
la
_c
ob
Required %age = 39600−33000 x Sol 1. (d) Let the original cost Cost price of 3 chairs = ₹ 1,800
bo
33000
ah price of article is 100 units
sm
100= 20%
ur
yo

At 10% loss on CP, SP is 90 units Sol 5. (d) Let there be 12 goods


e/
t.m
://

Sol 123. (a) If article is sold at ₹332 more, of 100 unit each
tp
ht

Profit earned = Sale Price - Cost there is 20% profit, ie. SP = 120 Total CP = 1200 unit
Price units One third of goods= 4, which are
According to the question 120-90 = 30 units = ₹ 332 sold at 15% profit i.e. for 115 unit
6(SP-CP) = (2SP-CP) 1 unit = 332
30 each
⇒ 5 CP = 4 SP Original SP = 90 units = 332 SP1 = 460unit
30
⇒ CP 4 25% of goods = 3 which are sold
SP = 5 × 90 = ₹ 996
Required profit %age = at 20% profit i.e for 120 unit each
5−4 × 100 = 25% Sol 2. (b) Let cost price of article SP2 = 360unit
4
be ₹100 Remaining goods = 5 are sold at
Then SP after 25% increment = 20% loss i.e. for 80unit each
Sol 124. (b)
₹125 SP3 = 400unit
Total Cost price of Rice = (22x35

e
If CP raise by 20% = ₹120 Total Selling price = 1220units
+ 13x30) = 1160
Total Sale price of Rice = And SP raise by 10% = ₹ 137.50
% profit= 137.5120− 120 × 100 Total profit = 20unit
(22+13) x 40 = 1400
As per condition given in
Required %age = 1400 −1160 × 100 = 14.58%

l
1160 question:
≈ 20.7 = 21% 20 unit = ₹ 138.50
ac Sol 3. (b) Let CP of machine be x
1 unit = ₹ 6.925
Sol 125.(b) At 5% profit, on 6 sewing
1200 units = ₹ 8,310
According to the question machines
105 of x) = ₹63000
6 × ( 100
(12+24)% of CP = 162 Sol 6. (b) Let cost price of A
⇒ Cost Price = 450 X = ₹ 10000 =100 units
Required loss % = 450450
−360 x 100 Cost price for 8 machines= 8 × According to question:
= 20% ₹10000 = ₹80,000 At 20% loss, CP for B = 80 units
To earn 15% profit, At 12.5% profit, CP for C = 90
nn
115
Selling price of 8 machines = 100
Sol 126. (b) unit
Total Cost price = 120 x 15 = × 80,000 = ₹ 92,000 At 8% loss, CP for D = 82.80 unit
1800 82.80 units = ₹ 248.40
Total Sale price = 110 x 18 = Sol 4. (c) Let cost price of 1 table 1 units = ₹ 3
1980 be ‘t’ and cost price of 1 chair be Now, loss incurred by A = 20
Required profit %age = 19801800
−1800 ‘c’ units; and loss incurred by C =
x 100 = 10% 5t + 9c = ₹ 15,400 7.20 units
As 5 tables are sold at 10% profit Required difference = 12.80 units
Pi

Sol 127. (c) and 9 chairs at 20% profit, total = ₹38.40


Revenue = Rs. 5 profit is ₹ 2080
Earning = Rs. 1 If all the articles would have been Sol 7. (b) C.P for Sudha = ₹ 576
Cost price = 5-1 = Rs.4 sold at 10% profit, actual profit × 100
80 = ₹ 720
For CP of Rs. 4, earning = Rs. 1 would have been ₹ 15,40. But the C.P. for Renu = ₹ (576+224) = ₹
For CP of Rs.1, earning = Rs. 41 extra ₹ 5,40 profit is due to 10% 800
For CP of Rs. 4800, earning = more profit on 9 chairs. 124 × 720 = ₹
C.P for Raghu = ₹ 100
4800 x 41 = 1200 For 9 chairs, 1% of profit = ₹ 54
892.80
For 1 chairs, 1% of profit = ₹6 892.80 − 800
Renu profit % = 800 × 100
Thus, cost price of 1 chair = ₹
SSC CGL 2019 TIER I = 11.6%
600

www.ssccglpinnacle.com support@ssccglpinnacle.com Ph. 09729327755, 09817390373

273
Days 28-32 Pro it and Loss

s
se
s
la
_c
ob
Sol 8. (a) Cost price of 48 articles Selling price = 115 × 80 = 92
bo
100
ah
= 2160 Sol 14. (b) Let the cost price of
sm
90 × 100 = ₹ 2400 loss percent = 8%
ur
yo

two watches be ₹ A and ₹ B


e/

CP of 1 article = ₹ 50
t.m
://

Number of articles in ₹ 2016 = respectively. Sol:2. (a)


tp
ht

2016 118% of A = 78% of B According to the question


50 78 = 39
Therefore, BA = 118 125% of CP = 70,000
Profit% = 59
CostQuantitative − SellingQuantitative A = 39 units and B = 59 units 130 × 70000 =
130% of CP = 125
sellingQuantitative × 100
2016 2016
A+B = ₹ 800 72,800
12 −S 6 −S
⇒ 100 = 50
S
⇒ 50 = 50
S 98 units = ₹ 800
⇒ S = 36 articles 1 units = ₹ 8.163 Sol:3. (d)
A = ₹ 39 × 8.163 = ₹ 318.37 If CP of bike = 100
Sol 9. (d) CP of laptop = ₹ 42,000 B = ₹ 59 × 8.163 = ₹ 481.63 Then, SP = 80 (20% loss)
CP of scanner-cum-printer = ₹ And we have to find the price of
8,000 Sol 15. (c) SP1 = SP2 bike at a loss of 30%( it means SP
Total CP = ₹ 50,000 Overall gain/loss% = 18-18- = 70)
SP of laptop = ₹ 46,200

e
18×18 = 3.24 loss
100 If 80 → Rs 2500
SP of scanner-cum-printer = ₹
8,400 If total CP = 100, loss = 3.24 and Then 70 → Rs 2187.5
Total SP = ₹ 54,600 SP = 96.76
Total Profit = ₹ 4,600 But total SP = 60, then, Loss = Sol:4. (a)

l
4600
Profit % = 50000 × 100 = 9 51 % 60
96.76 × 3.24 ≈ ₹ 2 (approx.) Let CP of article = 100
ac Then, SP at a loss of 20% = 80
Sol 10. (c) Let CP of article = 100 Sol 16. (b) Let CP = 10 units, → Rs7,160
% then SP = 13 units SP at a profit of 30% = 130 →
₹ 49 change occur due to 7% profit% at SP = Rs 11,635
change in price 13−10 × 100 = 23.1%
13
1% = ₹ 7 Sol:5.(d)
100% = ₹ 700 Sol 17. (d) Let the cost price of
the first book = ₹x. Then, cost
nn
Sol 11. (b) Let the manufacturing price of second book = ₹(300-x)
cost = ₹ x 120% of x + 90% of (300-x) =
110 × 115 = ₹ 7590
⇒ x × 100 100 300
⇒ x = ₹ 6000
30% x + 270 = 300
Ratio of both types = 14 : 21 = 2 :
x = ₹100
Sol 12. (d) Due to 4% change in 3
Selling price of first book = 120%
profit%, there is a difference of Items at loss = 53 × 40 = 24
of x = ₹120
₹36.
1% of CP = ₹ 9
Pi

Sol 18. (a) Sol:6.(a)


CP = ₹ 900 Let A purchased the item = 100
Cost price of car = ₹1,08,500
Let maintenance cost = ₹ x B purchased the item from A at
Sol 13. (d) CP of 18 tables = price = 120 (A sells the item at
111664 × 100 = ₹12960 Selling price = ₹ 1,56,250
90 125 20% profit to B)
100 × (108500 + x) = 156250
CP of 1 table = ₹ 720 5 C purchased the item from B at
4 × (108500 + x) = 156250
Number of tables for ₹ 17,424 = price = 132 (B sells the item at
17424 = 24.2 x = ₹ 16,500
720
10% profit to C)
SSC CHSL 2019
To earn 10% profit, number of Had C purchased the item from
Sol:1. (d)
table that can be sold for ₹ 17,424 A at price = 125.4 (C spent 5%
Let the cost price = 100
10 = 24.2 − S
⇒ 100 less than what he spent with B)
S List price = 115
S = 22 tables

www.ssccglpinnacle.com support@ssccglpinnacle.com Ph. 09729327755, 09817390373

274
Days 28-32 Pro it and Loss

s
sse
la
_c
ob
So Profit made by A = 125.4 - Profit percentage = 1 × 100 =
bo
4
ah
100 = 25.4 Sol:20. (d)
sm
25%
ur
yo

We know that Let the cost price = 100


e/
t.m
://

132 → Rs 132000 Sol:14. (b) Marked price = 120


tp
ht

25.4 → Rs 25400 Selling price after discount = 800 Discount = 120-100 = 20


80 = 640 20 × 100 =
required percentage = 120
× 100
Sol:7.(d) Profit = 25% 16 32 %
90%=18000 Cost price = 640 × 100 = 512
125
115%=230000 Sol:21. (b)
Sol:15. (c) Total cost price of printer =
Sol:8. (c) Let cost price = 100 3200+600 = 3800
cost price × 12 pen = selling price 90% = 810 Profit = 4280-3800 = 480
×8 104% = 936
480 × 100 =
Profit percent = 3800
CP 2
SP = 3 So, selling price = 936 12.63%
profit percent = 3−2 × 100 = 50%
2

e
Sol:16. (a) Sol:22. (a)
Sol:9. (a) cost price × 33 = selling price × x (x+5)% - x% = 750-720 = 30
Number of articles = 150 = 50 CP = x 5% = Rs30
3 SP 33
Cost Price of one article = 250 = profit = 33-x CP(100%) = Rs.600

l
50
Rs 5 10 = 33x−x × 100 Profit on article when it is sold at
ac
the selling price of 21 articles if x = 30 Rs.720 = 720-600 = Rs.120
120 × 100 = 20%
Profit % = 600
profit earned is 20% = 21
× 5 × 120% = Rs126 Sol:17. (c)
Let the profit or loss = x Sol:23. (d) On selling 26 balls
Sol:10.(b) So, 12000-x = 9000+x for Rs.1350, there is a loss equal
cost price × 15 = selling price × x = 1500 to the cost price of eight balls, it
20 Cost price = 12000-1500 = 10500 means in this money I can
CP = 4 120 =
Selling price = 10,500 × 100 purchase 8 balls less than 26 balls
nn
SP 3
4−3 12,600 So in Rs.1350 I can purchase 18
Loss percent = 4 × 100 = 25%
balls.
Sol:18. (c) cost price of a ball = 1350/ 18 =
Sol:11. (d)
After first discount = 1800 × 88 = Rs.75
85% = 3060 100
120% = 4320 1584
Sol:24.(d)
selling price = 4320 Additional discount = 1584-1200
Let the CP is 100
= 384
Then, MP = 125 ( marked price
Sol:12. (c) Required percentage =
Pi

384 of an article is 25% more than its


Difference between 25% gain and 1584 × 100 = 24.24% cost price)
15% loss = 40%
Discount = 25%
40% = 800 Sol:19. (a) So, SP = 112.5
100% = 2000 6 in Rs.8 and 10 in Rs.12 Hence , Profit is 12.5%
so, the cost price = 2000 First, make quantity equal
Let, total quantity = 30 (LCM of Sol:25. (a)
Sol:13. (d) 6 and 10) CP of the article = (8800 +
If selling price = 5 unit, then if 6 in Rs.8 then 30 in Rs.40 7200)/2 = Rs.8000
profit = 1 unit if 10 in Rs.12 then 30 in Rs.36 Profit when article is sold at
Cost price = 5-1 = 4 unit Loss = 40-36 = 4 Rs.9600 = 9600-8000 = Rs.1600
Percent loss = 404 × 100 = 10%

www.ssccglpinnacle.com support@ssccglpinnacle.com Ph. 09729327755, 09817390373

275
Days 28-32 Pro it and Loss

s
sse
la
_c
ob
Profit % = 1600/8000 Selling price = 122 × 785 = Rs. Total selling price of 40 items =
bo
100
ah
× 100 = 20% 26 + 23.6 = Rs. 49.6
sm
957.7
ur
yo

Profit % = 49.640− 40 × 100 =


e/
t.m
://

Sol:26. (c) Sol:32. (b) 9.6 × 100 = 24%


tp
ht

40
Let the CP of the article is 10x Let the cost price be 100
and SP is 13x 120% = 600 Sol:35. (a)
According to the question 80% = 400 Let the cost price of the book =
13x-100 - 10x+100 = So, required selling price = 400 Rs. C
45%(10x-100) As the shopkeeper earned 15%
1.5x = 45 Sol:33. (d) 115 × C
profit, Selling price, S = 100
x = 30 Cost price = Rs. 500 …. (i)
The original cost price of the Tax paid by shopkeeper = 12% of Sales tax rate = 10%
article = 10x = Rs.300 12 × 500 = 60
CP = 100 Let the selling price = Rs. S
Actual cost of item for Selling price = Rs. 956.34
Sol:27. (d) shopkeeper = 500 + 60 = Rs. 560 including the sales tax

e
Let the cost price of an article = Profit %= 20% Therefore, 956.34 = S + 10 1 ×S =
Rs. C Discount = 16% of marked price 11
10 × S …. (ii)
According to the question:- Selling price = actual CP + profit
From (i) and (ii):-
13400 - C = C - 11600 = 560 + 10020 × 560 = 560 + 112 =
11
10 × S = 956.34

l
⇒ 2C = 13400 + 11600 = 25000
672 11 115
⇒ C = 12500 100 − 16 10 × 100 × C = 956.34
ac Selling price = 100 × MP = 100 10
Profit when selling price is Rs. 84
C = 956.34 × 115 × 11 = Rs.
100 × MP
14750 = 14750 − 12500 = 2250 84
756
672 = 100 × MP
Marked price, MP = 100 × 672 =
Sol:28. (b) 84 Sol 36. (c)
Let CP = C Rs. 800 Marks up in cost price = 35%
Loss = 15% Discount = 15%
85 = 255000
C × 100 Sol:34. (d) Profit % = 35 -15 + 35 × (−15)
=
100
Total items = 40
nn
C = 255000 × 100 = 300000 35 × (15)
85 20 - 100 = 20 - 5.25 =
Let the cost price of each item be
To earn 10% profit, Selling price 14.75%
110 = Rs. 330000 Rs. 1.
= 300000 × 100
Total cost of 40 items = Rs. 40
Sol 37. (b)
Cost price of x items = Rs. x
Sol:29. (a) Let the cost price of chair for
Let the man sell ‘x’ items at 30%
C.P × 25 = S.P × 35 Ravi = Rs. x
profit equal to cost price of 26
CP = 7 Cost price of chair for Mohan =
SP 5 items; i.e. 11 x = 1.1 x
loss = 7-5 = 2 Selling price of x items = Rs. 26 10
Pi

required loss = 2 × 100 = 28.57% 130 × x = 26 Cost price of chair for Govind =
7 100 12 (1.1x)
x = 20 10 = 1.32 x
Sol:30. (a) Therefore, 20 items are sold at 1.32x = Rs. 1320
Cost price of each pen = Rs. 28 30% profit. x = Rs 1000
Selling price of each pen = Rs. 40 Remaining items = 40 - x = 40 -
Profit % = 4028− 28 × 100 =
20 = 20 Sol 38. (d)
42.85% Remaining 20 items are sold at Let the cost price of bike = Rs. C
Sol:31. (d) 18% profit. At 25% profit, C × 125100 = 25000
C = 25000 × 100 = Rs. 20,000
Cost price = Rs. 785 Selling price of remaining 20 125
Profit % = 22% 118 × 20 = Rs. 23.6
items = 100 To incur a loss of 15%: SP =
85 = Rs. 17000
20000 × 100

www.ssccglpinnacle.com support@ssccglpinnacle.com Ph. 09729327755, 09817390373

276
Days 28-32 Pro it and Loss

s
sse
la
_c
ob
CP of D = SP of C = 90% of
bo
ah
SSC CGL TIER-II 1.56x = 1.404x Sol:47.(c)
sm
ur
yo

Sol39:(b) Profit of B= 0.36 x 38S.P. + 6C.P.=38C.P.


e/
t.m
://

Difference in the selling price = Profit of A = 0.2x C.P/ S.P=19/16


tp
ht

123.4 - 108 = 15.4 According to question S.P. of one ball =2240/38


Gain = loss + 20% of loss 0.36x-0.2x = 80 C.P of one ball = 2240×19
38×16 =70
Let loss = x x= 500
Then profit = 1.2x 1.404x= 702 Sol:48.(c)
Total profit + loss = 2.2x Let cost price of article is 100%
2.2x=15.4 Sol:44.(b) If he sold at 2%Loss, selling price
x=7 Let CP = x is 98%
Loss = 7 At 8% gain = 1.08x If he sold at 20% Profit, selling
Cost price = 108 + 7 = 115 At 15% gain = 1.15x price is 120%
Profit = 120.75 -115 = 5.75 According to the question 22%= 44
P% = 5.75
115 × 100 = 5% 1.15x-1.08x = 714 Cost price, 100%=200

e
0.07x = 714
Sol:40.(c) x = 10200 Sol:49.(b)
Let CP = x At 18% profit = CP of first article =
118
100 × 10200 = 12036
MP = 1.4 x 9975
105 × 100 = 9500

l
SP = 1.4x × 73.5 = 1.029x
100 CP of second article =
Profit = 0.029x Sol:45.(b) 9975
ac 95 × 100 = 10500
Profit percentage = 2.9% Let CP of A = x
Total CP = 20000
SP of A = CP of B = 1.2x
108
Total SP = 19950
Sol:41.(a) SP of B = CP of C = 1.2x × 100 Loss = 50
Let CP = x = 1.296x
At 20% profit = 1.2x SP of C = CP of D = 1.296x SSC CPO 2019
At 30% profit = 1.3x 75 = 0.972x
× 100 Sol:50.(a)
According to question 1.3x - Profit of A =1,2x -x =0.2x When the article is sold at 8%
nn
1.2x =8 Profit of B = 1.296x- 1.2x = gain instead of 16% loss the net
0.1x=8 0.096x change in sp is equal to 660 Rs
x = 80 Difference in profit = 260 = Net difference in percentage =
16x = 1280 0.104x 24%=660
0.104x= 260 1%=27.5
Sol:42.(b) x= 2500 100%=2750
CP of first article = 5000 CP of D 0.972x = 2500 × 0.972 = 112%=3080
× (10010016 2 ) = 6000 2,430
− 3
Pi

Loss = 1000 Sol:51.(a)


CP of another article =5000 - Sol:46(c) Let the CP be 100x
1000 = 4000 Then SP = 110x
P% = 1000 If CP was 20% less then CP =
4000 × 100 = 25%
80x
Sol:43.(b) If SP was 1000 more than SP =
Let CP of A = x 110x + 1000
CP of B = SP of A = 1.2x Now, SP = 140x
Ratio of C.P=2:3 According to the question
CP of C = SP of B = 130% of
C.P of A= 4000 110x + 1000 = 140x
1.2x = 1.56x 80x
Profit on A= 15%2000=600 x = 500
Selling Price=4600 So, CP = 50,000

www.ssccglpinnacle.com support@ssccglpinnacle.com Ph. 09729327755, 09817390373

277
Days 28-32 Pro it and Loss

s
sse
la
_c
ob
bo
ah
Sol:52.(d)
sm
ur
yo

Let the CP be 100x


e/
t.m
://

Then SP = 110x
tp
ht

If CP was 20% less then CP =


80x
If SP was 1000 more than SP =
110x + 1000
Now, SP = 140x
According to the question
110x + 1000 = 140x
80x
x = 500
So, CP = 50,000
Earlier selling price = 110x =
55,000

e
Sol:53.(b)
SP of 50 articles=CP of 42
articles

l
SP:CP=42:50
loss= 50-42=8ac
loss%= 508 × 100=16% loss

Sol:54.(d)
When the article is sold at 8%
gain instead of 16% loss the net
change in sp is equal to 660 Rs
Net difference in percentage =
nn
24%=660
1%=27.5
100%=2750
When the article is sold at 3080
then net profit= 3080-2750=330
330
Net profit percentage= 2750
× 100 =12%
Pi

Sol:55.(a)
According to the question
23% = 1,104
100% = 110423 × 100 = 4,800

www.ssccglpinnacle.com support@ssccglpinnacle.com Ph. 09729327755, 09817390373

278
Days 33-36 Discount

s
se
s
la
_c
ob
Discount / छूट मशः 20% और 10% छूट दी गई| marked price of an article, the
bo
ah total discount is Rs 259.20. If the
sm
इसका ा मतलब है ?
ur
yo

marked price of the article is


e/

Key Points:/ मुख िबंदु:


t.m

Let MRP or market price be Rs.


://

Rs720, then the value of x is:


tp
ht

100 एक व ु के अंिकत मू पर x% की
1. Market Price/बाज़ार-भाव: The
100 − 20% → 80 − 10% →72 दो िमक छूट दे ने के बाद कुल छूट
price on the label of a product,
259.20 पये है | यिद व ु का
also called printed price or ∴ Total discount is Rs.28 i.e.
अंिकत मू 720 पये है , तो x का
advertising price. 28% of 100. मान ात कर |
िकसी उ ाद के लेबल पर कीमत, ∴ Successive discount of 20% SSC CGL 4 June 2019
िजसे मुि त मू या िव ापन मू and 10% is equivalent to 28% ./ (Morning)
भी कहते ह| 20% और 10% की लगातार छूट (a) 18
28% के बराबर है | (b) 24
2. Discount/ छूट: Amount of The above concept can be (c) 20
rebate on a fixed price is called
interpreted as successive % (d) 25
discount./ िनि त कीमत पर छूट की

e
concept.
रािश को छूट कहा जाता है ।
Equivalent percentage = Q2. An article is sold for Rs.528
3. Selling Price = Market Price – (x + y − 100 ) %
xy after successive discounts of 20%
Discount and 12%. What is the marked
Here,

l
price of the article?
Let us say Rs. 100 is the market (20 + 10 − 200
100 ) % = 30 − 2 = 28%
price (MRP)./ मान लीिजये िक एक व ु 20% और 12% की िमक
ac
100 बाजार मू (एमआरपी) है ।
Discount is 10% i.e. 10 Rs/
िड ाउं ट 10% यानी 10 पये है ,
Therefore, selling price = 100 –
Example: What will be
equivalent discount for three
successive discounts of 20%,
20%, and 10%?
छूट के बाद 528 पये म बेची जाती
है | इस व ु का अंिकत मू
?
SSC CGL 10 June 2019
(Afternoon)
ा है

10 = Rs. 90 20%, 20% और 10% की लगातार (a)Rs.760


तीन छूट के समतु ा छूट होगी? (b)Rs.740
4. Discount is given/ calculated Solution: First calculate (c)Rs.750
nn
on market price./ िड ाउं ट बाजार equivalent discount for any two (d)Rs.780
मू पर िदया जाता है | of the three, lets take 20% and
Discount% 20%. Q3. A man purchases 100 copies
= M arketMP arket
rice−Selling P rice
×100 पहले तीन म से िकसी भी दो के िलए of a book from the publisher and
P rice
Discount समतु छूट की गणना कर, चलो gets a discount of 25%. He buys
Or, M arket P rice ×100
20% और 20% लेते ह| अब 36% 50 copies from a retailer at a
और 10% के समतु छूट की गणना discount of 10%. He got an
5. Selling Price = कर। overall discount of:
Pi

M arket P rice×(100−Discount%)
एक काशक से िकसी
M arket P rice (20 + 20 − 100 )
400 % = 40 4 = 36%

पु क की 100 ितयाँ खरीदता है
Now calculate equivalent और 25% की छूट ा करता है |
6. Successive Discount/ िमक
discount of 36% and 10%. वह खुदरा िव े ता से 50 ितयाँ
छूट: When two or more than two खरीदता है और 10% की छूट ा
(36 + 10 − 100 )
360 % = 46 3.6 = 42.4%

discount are given./ जब दो या दो करता है | उसे कुल िकतनी छूट ा
से अिधक छूट दी जाती है | 42.4% is the equivalent discount. ई?
SSC CGL 4 June 2019
20% and 10% discount given (Evening)
Variety Questions
successively. What does it mean? (a) 16.5%
Q1. After giving two successive (b) 17.5%
discounts, each of x%, on the (c) 20%

www.ssccglpinnacle.com support@ssccglpinnacle.com Ph. 09729327755, 09817390373

279
/
Days 33-36 Discount

s
sse
la
_c
ob
(d) 35% एक व ु की कीमत य मू से price. After allowing a certain
bo
ah िकतनी अिधक होनी चािहए तािक discount on the marked price, the
sm
ur
yo

Q4. A dealer buys an article at a इस पर 20% और 6.25% की दो profit reduces to 10%. The
e/
t.m

िमक छूट दे ने के बाद य मू


://

discount of 20% on its list price discount percent is:


tp
ht

and marks it at 25% above the list पर 20% का शु लाभ हो ? एक व ु की कीमत इसके य मू
price. If he allows a 20% discount SSC CGL 7 June 2019 से 1.2 गुना रखी जाती है | अंिकत
on the new list price, then his (Evening) मू पर एक िनि त छूट दे ने के बाद
profit percent is : (a) 66 32 % लाभ कम होकर 10% हो जाता है |
एक ापारी िकसी व ु को इसके (b) 46 41 % छूट का ितशत है :
सूची मू से 20% की छूट पर (c) 50% SSC CHSL 3 July 2019
खरीदता है और िफर इसका मू (d) 60% (Morning)
सूची मू से 25% अिधक तय करता (a) 8 31
है | यिद वह नए सूची मू पर 20% Q7. An article is sold for Rs 612 (b) 9
की छूट दे ता है , तो उसके लाभ का after successive discounts of 25% (c) 10
ितशत ात कर | and x%. If the marked price of (d) 8 32

e
SSC CGL 7 June 2019 the article is Rs 960, what is the
(Morning) value of x? Q10. A man gets a discount of
(a) 24 एक व ु 25% और x% की दो 30% and then 20% on his food
(b) 25 िमक छूट के बाद 612 पये म bill. How much equivalent single

l
(c) 20 बेची जाती है | यिद व ु का अंिकत discount does he get?
(d) 27 मू 960 पये है , तो x का मान ात एक को उसके भोजन के िबल
ac
Q5. A shopkeeper sells an item
for Rs 492 after allowing 18%
discount on its marked price. Had
he not allowed any discount, he
कर |
SSC CGL 12 June 2019
(Evening)
(a) 10
(b) 12
पर 30% और िफर 20% की छूट
िमलती है | उसे एकल छूट के
समतु िकतनी छूट िमली ?
SSC CHSL 3 July 2019
(Afternoon)
would have earned a profit of (c) 14 (a) 44%
20% on the cost price. What is (d) 15 (b) 50%
the cost price of the item?
nn
(c) 40%
एक दु कानदार एक व ु को इसके Q8. What is the difference (d) 35%
अंिकत मू पर 18% की छूट दे ने के between a single discount of 30%
बाद 492 पये म बेचता है | यिद वह and a single discount equivalent Q11. There was 25% off on
िकसी कार की छूट नहीं दे ता, तो to two successive discounts of shirts. A lady bought that shirt
उसे य मू पर 20% का लाभ 25% and 5%, being given on and got an additional 20%
आ होता | इस व ु का य मू shopping of Rs2,000? discount for paying in cash and
ात कर | 30% की एकल छूट और 25% तथा further 10% discount for being a
SSC CGL 7 June 2019 5% की दो िमक छूटों के समतु
Pi

loyal customer. She paid Rs324.


(Afternoon) एकल छूट के बीच अंतर ात कर, What was the price tag (in Rs) on
(a) Rs 500 यिद 2000 पये की ख़रीदारी की the shirt?
(b) Rs 640 जाती है | शट पर 25% की छूट थी | एक
(c) Rs 540 SSC CGL 13 June 2019 मिहला ने वह शट ख़रीदा और उसे
(d) Rs 600 (Evening) नकद म भुगतान करने के कारण
(a) Rs25 20% की अित र छूट ा ई
Q6. By how much above the cost (b) Rs15 तथा इसके बाद िन ावान ाहक होने
price should an article be marked (c) Rs20 के कारण उसे 10% की छूट और
up for sale so that after allowing (d) No difference िमली | उसने 324 पये का भुगतान
two successive discounts of 20% िकया | इस शट का अंिकत मू (
and 6.25% on it, a net gain of Q9. An article is marked at a पये म ) ात कर |
20% is made on the cost? price which is 1.2 times its cost

www.ssccglpinnacle.com support@ssccglpinnacle.com Ph. 09729327755, 09817390373

280
/
Days 33-36 Discount

s
sse
la
_c
ob
SSC CHSL 5 July 2019 SSC CPO 12 March 2019
bo
ah
(Afternoon) Q14. A shopkeeper normally (Evening)
sm
ur
yo

(a) 650 allows a discount of 10% on the (a) 46%


e/
t.m
://

(b) 725 marked price of each article. (b) 48%


tp
ht

(c) 600 During a sale season, he decides (c) 54%


(d) 750 to give two more discounts, the (d) 44%
first being at a rate of 50% of the
Q12. A dealer buys an article original and the second at a rate Q17.A trader gives a discount of
marked at Rs20000 with two of 40% of the first. What is the 4% for purchases above
successive discounts of 20% and percentage rate of the equivalent Rs.25,000, 6% for purchases
5%. He spends Rs1000 for its single discount (correct up to two above Rs.35000 and 8% for
repair and sells it for Rs20000. decimal places)? purchases above Rs. 50,000. If
What is his profit/loss percent एक दु कानदार आमतौर पर ेक the price of an item is marked
(correct to two decimal places)? व ु के अंिकत मू पर 10% की Rs.38,500. What would be the
एक िव े ता 20000 पये अंिकत छूट दे ता है | िब ी के मौसम म, वह amount of discount?
मू वाली कोई व ु 20% और 5% दो अित र छूट दे ने का िनणय लेता एक ापारी 25,000 पये से अिधक

e
की दो िमक छूट पर खरीदता है | है िजसम एक की दर ारं िभक छूट की खरीद पर 4%, 35000 पये से
वह इसके मर त पर 1000 पये का 50% तथा दू सरी छूट की दर अिधक की खरीद पर 6% तथा
खच करता है तथा इसे 20000 पये पहली छूट का 40% है | समतु 50,000 पये से अिधक की खरीद
म बेच दे ता है | उसके लाभ या हािन एकल छूट की ितशत दर ( दो पर 8% की छूट दे ता है | यिद िकसी

l
का ितशत ( दशमलव के दो थान दशमलव थानों तक सही ) ात कर व ु का मू 38,500 पये अंिकत
तक ) ात कर |

(Morning)
ac
SSC CHSL 8 July 2019

(a) 25.64% profit


(b) 23.64% loss
|
SSC CHSL 10 July 2019
(Afternoon)
(a) 11.25
(b) 16.21
िकया गया है , तो छूट की रािश
होगी ?
SSC CPO 15 March 2019
(Morning)
(a)Rs.3740

(c) 23.46% profit (c) 14.85 (b)Rs.1810


(d) 25.64% loss (d) 13.27 (c) Rs.3080
(d)Rs.2310
nn
Q13. Three successive discounts Q15. A customer gets a discount
on the marked price of an article of Rs. 90 which is 12%. The Q18.The price of a refrigerator is
turns out to be equivalent to a selling price is: Rs 22000. A shopkeeper marks
single discount of 19%. If the एक ाहक को 90 पये की छूट its price 15% above its cost price
rates of the first and second िमलती है जो िक 12% है । िव य and gives a discount of 8%. The
discount are 10% and 4% मू ा है ? discount is: िकसी रे ीिजरे टर का
respectively, what is the rate of SSC CPO 16 March 2019 मू 22,000 पये है | एक
the third discount? (Morning) दु कानदार इसका मू य मू से
Pi

िकसी व ु के अंिकत मू पर दी (a) Rs.660 15% अिधक िनधा रत करता है तथा


गयी तीन िमक छूट 19% के एकल (b) Rs.580 8% की छूट दे ता है | छूट की रािश है
छूट के समतु है | यिद पहली और (c) Rs.540 -
दू सरी छूट की दर मशः 10% और (d) Rs. 770 SSC CPO 15 March 2019
4% है , तो तीसरी छूट की दर ात (Morning)
कर | Q16. The successive discounts of (a) Rs.1960
SSC CHSL 10 July 2019 25% , 20% and 10% is (b) Rs. 1824
(Morning) equivalent to a single discount of: (c) Rs. 1672
(a) 7.50% 25%, 20% तथा 10% की िमक (d) Rs. 2024
(b) 5.25% छूट ______ एकल छूट के बराबर है
(c) 6.25% | SSC CGL TIER II
(d) 6.00%

www.ssccglpinnacle.com support@ssccglpinnacle.com Ph. 09729327755, 09817390373

281
/
Days 33-36 Discount

s
sse
la
_c
ob
Q1. The marked price of an (a)Rs.920 (d) Rs940
bo
ah
article is Rs 1500. If two (b)Rs.860
sm
ur
yo

successive discounts, each of x%, (c)Rs.900 Q6. An article is sold for Rs 612
e/
t.m
://

on the marked price is equal to a (d)Rs.880 after successive discounts of 25%


tp
ht

single discount of Rs 587.40, then and 15%. What is the marked


what will be the selling price of Q3. An article is sold for Rs1,680 price of the article?
the article if a single discount of after two successive discounts of एक व ु 25% और 15% की दो
x% is given on the marked price? 20% and 16%. What is the िमक छूटों के बाद 612 पये म
एक व ु का अंिकत मू 1500 marked price of the article? बेची जाती है | इस व ु का अंिकत
पये है | यिद अंिकत मू पर x % एक व ु 20% और 16% की दो मू ात कर |
की दो िमक छूट 587.40 पये की िमक छूटों के बाद 1680 पये म SSC CGL 12 June 2019
एकल छूट के बराबर है , तो व ु का बेची जाती है | इस व ु का अंिकत (Afternoon)
िव य मू ात कर यिद अंिकत मू ात कर | (a) Rs1000
मू पर x% की एकल छूट दी जाती SSC CGL 11 June 2019 (b) Rs940
है | (Afternoon) (c) Rs980

e
SSC CGL Tier 2 13 September (a) Rs2,300 (d) Rs960
2019 (Morning) (b) Rs2,200
(a)Rs 1025 (c) Rs2,500 Q7. An article is sold for Rs288
(b)Rs 1155 (d) Rs2,400 after successive discounts of 20%

l
(c)Rs 1170 and 25%. What is the marked
(d)Rs 1200 price of the article?
ac
Practice Questions

Q1. An article is sold for


Rs.642.60 after successive
Q4. An article is sold for
Rs535.50 after two successive
discounts of 25% and 15%. What
is the marked price of the article?
एक व ु 25% और 15% की दो
एक व ु 20% और 25% की दो

मू
िमक छूटों के बाद 288 पये म
बेची जाती है | इस व ु का अंिकत
ात कर |
SSC CGL 13 June 2019
discounts of 15% and 10%. What िमक छूटों के बाद 535.50 पये म (Morning)
is the marked price of the article? बेची जाती है | इस व ु का अंिकत (a) Rs520
एक व ु 15% और 10% की दो मू ात कर | (b) Rs480
nn
िमक छूट के बाद 642.60 पये म SSC CGL 11 June 2019 (c) Rs460
बेची जाती है | इस व ु का अंिकत (Evening) (d) Rs500
मू ात कर | (a) Rs800
SSC CGL 10 June 2019 (b) Rs830 Q8. An article is sold for Rs 288
(Evening) (c) Rs820 after successive discounts of 25%
(a)Rs.840 (d) Rs840 and x%. If the marked price of
(b)Rs.820 the article is Rs 480, what is the
(c)Rs.800 Q5. An article is sold for value of x?
Pi

(d)Rs.880 Rs547.40 after successive एक व ु 25% और x% की दो


discounts of 30% and 15%. What िमक छूटों के बाद 288 पये म
Q2.An article is sold for is the marked price of the article? बेची जाती है | यिद व ु का अंिकत
Rs.657.90 after successive एक व ु 30% और 15% की दो मू 480 पये है , तो x का मान ात
discounts of 15% and 10%. What िमक छूटों के बाद 547.40 पये म कर |
is the marked price of the article? बेची जाती है | इस व ु का अंिकत SSC CGL 13 June 2019
एक व ु 15% और 10% की दो मू ात कर | (Afternoon)
िमक छूट के बाद 657.90 पये म SSC CGL 12 June 2019 (a) 20
बेची जाती है | इस व ु का अंिकत (Morning) (b) 16
मू ात कर | (a) Rs920 (c) 15
SSC CGL 11 June 2019 (b) Rs960 (d) 18
(Morning) (c) Rs900

www.ssccglpinnacle.com support@ssccglpinnacle.com Ph. 09729327755, 09817390373

282
/
Days 33-36 Discount

s
sse
la
_c
ob
Q9. A man gets a discount of (c) 800 20% की अित र छूट िमली | इसके
bo
ah
30% and then 20% on his food (d) 925 बाद िन ावान ाहक होने के नाते
sm
ur
yo

bill of Rs1,250. How much does उसे 10% की छूट और िमली | उसने
e/
t.m

405 पये का भुगतान िकया | इस


://

he have to pay? Q12. A man gets a discount of


tp
ht

एक को उसके 1250 पये के 30% and then 20% on his food शट का अंिकत मू ( पये म ) ात
भोजन िबल पर 30% और िफर 20% bill of Rs1,500. How much कर |
की छूट िमलती है | उसे िकतने का discount, in rupees, did he get? SSC CHSL 5 July 2019
भुगतान करना पड़ा ? एक को उसके 1500 पये के (Evening)
SSC CHSL 3 July 2019 भोजन िबल पर 30% और िफर 20% (a) 650
(Evening) की छूट िमलती है | उसे िकतने पये (b) 600
(a) Rs700 की छूट िमली ? (c) 750
(b) Rs550 SSC CHSL 4 July 2019 (d) 725
(c) Rs350 (Evening)
(d) Rs500 (a) 700 Q15. A dealer buys an article
(b) 360 marked at Rs20000 with two

e
Q10. There was 29% off on bags. (c) 660 successive discounts of 10% and
A lady bought a bag and got a (d) 500 8%. He spends Rs1440 on repairs
12% discount for paying in cash. and sells it for Rs20000, what is
She paid Rs 312.40. What is the Q13. There was 25% off on bags. his profit/loss percent (correct to

l
price tag (in Rs) on the bag? A lady bought a bag and got one decimal place)?
थैलों पर 29% की छूट थी | एक additional 20% discount for एक ापारी ने 20000 पये अंिकत
ac
मिहला ने एक थैला ख़रीदा और उसे
नकद म भुगतान करने के िलए 12%
की छूट िमली | उसने 312.40 पये
का भुगतान िकया | थैले का अंिकत
मू ा था ?
paying in cash. She paid Rs480,
what was the price tag (in Rs) on
the bag?
थैलों पर 25% की छूट थी | एक
मिहला ने एक थैला ख़रीदा और उसे
मू वाली िकसी व ु को 10% और
8% की दो िमक छूट पर ख़रीदा |
उसने मर त पर 1440 पये खच
िकये और इसे 20000 पये म बेच
िदया | उसके लाभ या हािन का
SSC CHSL 4 July 2019 नकद म भुगतान करने के कारण ितशत ( एक दशमलव थान तक
(Afternoon) 20% की अित र छूट िमली | उसने सही ) ात कर |
(a) 600 480 पये का भुगतान िकया | इस SSC CHSL 8 July 2019
nn
(b) 625 थैले का अंिकत मू ( पये म ) ात (Afternoon)
(c) 450 कर | (a) 12.3% Profit
(d) 500 SSC CHSL 5 July 2019 (b) 12.3% loss
(Morning) (c) 11.1% Profit
Q11. There was 29% off on bags. (a) 800 (d) 11.1% loss
A lady bought a bag and got 13% (b) 825
discount for paying in cash. She (c) 750 Q16. A dealer buys an article
paid Rs 617.70. What was the (d) 950 marked at Rs20000 with two
Pi

price tag (in Rs) on the bag? successive discounts of 20% and
थैलों पर 29% की छूट थी | एक Q14. There was 25% off on shirt. 5%. He spends Rs1800 on repairs
मिहला ने एक थैला ख़रीदा और उसे A lady bought a shirt and got an and sells it for Rs20000, what is
नकद म भुगतान करने के कारण additional 20% discount for his profit/loss percent (correct to
13% की अित र छूट िमली | उसने paying in cash and further 10% one decimal place)?
617.70 पये का भुगतान िकया | इस discount for being a loyal एक ापारी ने 20000 पये अंिकत
थैले का अंिकत मू ( पये म ) ात customer. She paid Rs 405. What मू वाली िकसी व ु को 20% और
कर | was the price tag (in Rs) on the 5% की दो िमक छूट पर ख़रीदा |
SSC CHSL 4 July 2019 shirt? उसने मर त पर 1800 पये खच
(Evening) शट पर 25% की छूट थी | एक िकये और इसे 20000 पये म बेच
(a) 750 मिहला ने एक शट ख़रीदा और उसे िदया | उसके लाभ या हािन का
(b) 1000 नकद म भुगतान करने के कारण

www.ssccglpinnacle.com support@ssccglpinnacle.com Ph. 09729327755, 09817390373

283
/
Days 33-36 Discount

s
sse
la
_c
ob
ितशत ( एक दशमलव थान तक Q19. An article is sold for Rs (c) Rs703
bo
ah
सही ) ात कर 688.16 after two successive (d) Rs680
sm
ur
yo

SSC CHSL 8 July 2019 discounts of 12% and 8%


e/
t.m
://

(Evening) respectively. What is its marked Q22. An article having marked


tp
ht

(a) 23.46% profit price? price, Rs900, was sold for Rs648
(b) 17.65% profit एक व ु मशः 12% और 8% की after two successive discounts.
(c) 17.65% loss दो िमक छूट के बाद 688.16 पये The first discount was 20%. What
(d) 23.64% loss म बेची जाती है | इसका अंिकत मू was the percentage rate of the
ात कर | second discount?
Q17. A dealer buys an article SSC CHSL 9 July 2019 एक व ु, िजसका अंिकत मू 900
marked at Rs30000 with two (Evening) पये था, दो िमक छूट के बाद 648
successive discounts of 20% and (a) Rs800 पये म बेची गयी | पहली छूट 20%
5%. He spends Rs1200 on repairs (b) Rs900 की थी | दू सरी छूट की ितशत दर
and sells it for Rs30000, what is (c) Rs820 ा थी ?
his profit/loss percent? (d) Rs850 SSC CHSL 11 July 2019
एक ापारी ने 30000 पये अंिकत

e
(Morning)
मू वाली िकसी व ु को 20% और Q20. A shopkeeper bought an (a) 5
5% की दो िमक छूट पर ख़रीदा | article for Rs100 and marked its (b) 15
उसने मर त पर 1200 पये खच price 25% above the cost price. (c) 10
िकये और इसे 30000 पये म बेच

l
How much discount percentage (d) 12.5
िदया | उसके लाभ या हािन का should he announce in order to
ितशत ात कर

(Morning)
(a) 25% loss
ac
SSC CHSL 9 July 2019

(b) 25% profit


make a profit of 15%?
एक दु कानदार ने िकसी व ु को 100
पये म
कीमत य मू
य िकया और इसकी
से 25% अिधक
रखी | 15% का लाभ कमाने के िलए
Q23. A shopkeeper decides to
raise the marked price of an
article by 10%. How much
discount should he allow so as to
be able to sell the article at the
(c) 20% profit उसे िकतने ितशत छूट की घोषणा original marked price? एक
(d) 20% loss करनी चािहए ? दु कानदार िकसी व ु के अंिकत
SSC CHSL 10 July 2019 मू म 10% की वृ करने का
nn
Q18. A dealer buys an article (Evening) िनणय लेता है | व ु को ारं िभक
marked at Rs5000 with two (a) 8.25 अंिकत मू पर बेचने के िलए उसे
successive discounts of 20% and (b) 8.5 िकतने ितशत की छूट दे नी चािहए?
5%. He spends Rs200 on repairs (c) 8 SSC CHSL 11 July 2019
and sells it for Rs5000, what is (d) 10 (Afternoon)
his profit/loss percent? 1 %
(a) 9 11
एक ापारी ने 5000 पये अंिकत Q21. An article is subject to two (b) 8 91 %
मू वाली िकसी व ु को 20% और successive discounts of 10% and
Pi

(c) 9 21 %
5% की दो िमक छूट पर ख़रीदा | 5% before being sold. If its
(d) 10%
उसने मर त पर 200 पये खच marked price is Rs 800, then its
िकये और इसे 5000 पये म बेच selling price is:
Q24. The successive discounts of
िदया | उसके लाभ या हािन का िकसी व ु को बेचने से पहले 10%
30%, 25% and 15% is equivalent
ितशत ात कर | और 5% की दो िमक छूट दी जाती
to a single discount of:
SSC CHSL 9 July 2019 है | यिद इसका अंिकत मू 800
30%, 25% और 15% की िमक
(Afternoon) पये है , तो इसका िव य मू ा
छूट िकस एकल छूट के बराबर है ?
(a) 25% profit होगा ?
SSC CPO 13 March 2019
(b) 25% loss SSC CHSL 10 July 2019
(Evening)
(c) 20% profit (Evening)
(a) 60.275%
(d) 20% loss (a) Rs722
(b) 54.625%
(b) Rs684
(c) 55.375%

www.ssccglpinnacle.com support@ssccglpinnacle.com Ph. 09729327755, 09817390373

284
/
Days 33-36 Discount

s
sse
la
_c
ob
(d) 60.725% 12% और 8% के दो िमक छूट discount of Rs 345. The discount
bo
ah िकतनी एकल छूट के बराबर है percentage is:
sm
ur
yo

Q25. The successive discount of SSC CPO 14 March 2019 एक दु कानदार िकसी पु क को
e/
t.m

2500 पये म खरीदता है तथा इसकी


://

20%, 10% and 15% is equivalent (Morning)


tp
ht

to a single discount of: (a)17% कीमत लागत से 15% अिधक रखता


20%, 10% और 15% की िमक (b)18% है | वह 345 पये की छूट दे ता है |
छूट िकस एकल छूट के बराबर होगी (c)19% छूट का ितशत है -
? (d)20% SSC CPO 16 March 2019
SSC CPO 12 March 2019 (Afternoon)
(Morning) Q29. Seema purchased mobile (a)10
(a) 43.5% and got 20% discount on it. Had (b)12
(b) 42.2% she got 25% discount, she would (c) 11
(c) 38.8% have saved Rs 1,000 more. How (d)13
(d) 44.5% much did she pay for the mobile?
सीमा ने एक मोबाइल ख़रीदा और Q32. A trader allows a discount
उसे इस पर 20% की छूट िमली |

e
Q26. The successive discounts of of 8% on marked price. If the
20%, 10% and 8% is equivalent यिद उसे 25% की छूट िमली होती, selling price is Rs 667, the
to a single discount of: / 20%, तो उसने 1000 पये और बचाए होते discount in rupee is:
10% और 8% की िमक छूट िकस | उसने मोबाइल के िलए िकतना एक ापारी अंिकत मू पर 8% की
एकल छूट के बराबर है ? भुगतान िकया ? छूट दे ता है | यिद िव य मू 667

l
SSC CPO 13 March 2019 SSC CPO 16 March 2019 पये है , तो छूट ( पये म ) ात कर
(Morning)
(a) 66.24%
(b) 32.84%
(c) 38%
(d) 33.76%
ac (Evening)
(a)25,000
(b)22,000
(c)16,000
(d)20,000
|
SSC CPO 16 March 2019
(Afternoon)
(a)47
(b)54
(c)58
Q27. A trader marks the products Q30. A man could not decide (d)43
25% above the cost price and between discount of 30% or two
nn
allows a discount of 15%. If the successive discounts of 25% and Q33.Mobile cover with a cost of
cost price is rs 2,080, then the 5%, both given on a shopping of Rs.264 is available at a 12%
selling price is: Rs 2,000. What is the difference discount. What will be the sale
एक ापारी व ुओं का मू य between both the discounts? price of such 4 mobile covers
मू से 25% अिधक रखता है तथा एक 2000 की खरीदारी पर 264 पए की लागत वाला मोबाइल
15% की छूट दे ता है | यिद य मू िमलने वाली 30% की छूट अथवा कवर 12 % की छूट पर उपल है |
2080 पये है , तो िव य मू ात 25% एवं 5% की िमक छूट को ऐसे 4 मोबाइल कवरो का िव य
कर | लेकर िनणय नहीं ले पा रहा था | दोनों मू ा होगा ?
Pi

SSC CPO 14 March 2019 छूट के बीच अंतर ात कर | SSC CPO 14 March 2019
(Morning) SSC CPO 16 March 2019 (Evening)
(a)2210 (Evening) (a) 929.28
(b)1809 (a)20 (b) 940.46
(c)2392 (b)15 (c) 936.72
(d)2600 (c)No difference/ कोई अंतर नहीं (d) 934.04
(d)25
Q28.The single discount Q34. A car merchant marks 24%
equivalent to two successive Q31. A shopkeeper buys a book more than the cost of a car and
discounts of 12% and 8% for Rs 2,500 and marks its price gives a discount of 15%. If the
is(rounded off) at 15% above cost. He allows a discount is 2,23, 200, the cost of
the car will be:

www.ssccglpinnacle.com support@ssccglpinnacle.com Ph. 09729327755, 09817390373

285
/
Days 33-36 Discount

s
sse
la
_c
ob
एक ापारी कार की लागत से 24 % SSC MTS 2 August 2019 छूट 10% और 8% थी | तीसरी छूट
bo
ah
अिधक कीमत अंिकत करता है और (Morning) का ितशत िकतना था ?
sm
ur
yo

15 % की छूट दे ता है | यिद छूट 2,23 (a) 21.5% SSC MTS 5 August 2019
e/
t.m

,200 है , तो कार का लागत मू


://

(b) 23.5% (Evening)


tp
ht

होगा : (c) 25% (a) 5


SSC CPO 14 March 2019 (d) 26.5% (b) 3
(Evening) (c) 6.5
(a) Rs 11,60,000 Q38. An article is sold for (d) 4.5
(b) Rs 14,25,000 Rs7650 after two successive
(c) Rs 15,80,000 discounts of 15% and 25%. What Q41. The marked price of an
(d) Rs 12,00,000 is the marked price of the article? article is Rs. 2800. The selling
एक व ु 15% और 25% की दो price of the article is Rs. 2408.
Q35. A dealer allows a discount िमक छूट के बाद 7650 पये म Find the discount percentage.
of 12% on the marked price. If बेची जाती है | इस व ु का अंिकत एक व ु का अंिकत मू 2800 है
the selling price is 924, the मू ात कर | | व ु का िव य मू 2408 है |
छूट ितशत िकतना है ?

e
discount is: SSC MTS 2 August 2019
एक िवतरक अंिकत मू पर 12% (Evening) SSC MTS 6 August 2019
की छूट दे ता है | यिद िव य मू Rs (a) Rs15000 (Afternoon)
924 है , तो उसपर छूट िकतने पये (b) Rs10000 (a) 18%
की होगी ?

l
(c) Rs14000 (b) 24%
SSC CPO 15 March 2019 (d) Rs12000 (c) 16%
(Evening)
(a)126
(b)110.90
(c)119.20
(d)114
ac Q39. Marked price and cost price
of an article are in ratio 5 : 4. If
the profit earned by selling the
article is 12.5%, then what is the
(d) 14%

Q42. A man sold a watch at a


discount of 60% for Rs1560.
What is the marked price of the
discount percentage? watch?
Q36. The marked price of an एक व ु का अंिकत मू और य एक ने कोई घड़ी 60% की
article is Rs. 2500. It is sold at मू 5 : 4 के अनुपात म है | यिद छूट पर 1560 पये म बेच दी | इस
nn
two successive discounts of 12% व ु को बेचने पर 12.5% का लाभ घड़ी का अंिकत मू ात कर |
and 10%. What is the selling होता है , तो छूट का ितशत ात कर SSC MTS 7 August 2019
price ? | (Morning)
िकसी व ु का अंिकत मू 2500 SSC MTS 5 August 2019 (a) Rs3900
है | उसे मशः 12% तथा 10% की (Morning) (b) Rs3600
दो िमक छूटों पर बेचा जाता है | (a) 12.5 (c) Rs3300
िव य मू िकतना है ? (b) 15 (d) Rs3700
SSC MTS 2 August 2019 (c) 8
Pi

(Afternoon) (d) 10 Q43. If the discount offered on an


(a) Rs 1870 / 1870 article is 40%, then what will be
(b) Rs 2090 / 2090 Q40. The price of an article was the ratio of selling price and
(c) Rs 1760 / 1760 Rs. 10000. It was sold to a retailer marked price?
(d) Rs 1980 / 1980 after three successive discounts यिद िकसी व ु पर दी गयी छूट 40%
for Rs. 7866 out of which the first है , तो िव य मू और अंिकत मू
Q37. What is a single discount two discounts were 10% and 8%. का अनुपात ा होगा ?
equivalent to two successive Find the rate of third discount. SSC MTS 7 August 2019
discounts of 10% and 15%? िकसी व ु की कीमत Rs 10,000 थी (Afternoon)
10% और 15% की दो िमक छूटों | इसे तीन िमक छूटों के बाद (a) 2 : 5
के बराबर िकतनी एकल छूट होती है Rs7866 म िकसी खुदरा िव े ता को (b) 4 : 7
? बेच िदया गया, िजसम से पहली दो (c) 5 : 8

www.ssccglpinnacle.com support@ssccglpinnacle.com Ph. 09729327755, 09817390373

286
/
Days 33-36 Discount

s
sse
la
_c
ob
(d) 3 : 5 (c) Rs2432 SSC MTS 13 August 2019
bo
ah (d) Rs3132 (Evening)
sm
ur
yo

Q44. After offering a discount of (a) 2300


e/
t.m
://

20% on an article, a trader earned Q47. A buys an article at Rs1800 (b) 1800
tp
ht

a profit of 20%. If the cost price and sells it after giving two (c) 2233
is Rs300, then what will be the successive discounts of 10% and (d) 2144
selling price of the article after 20%. What will be the selling
offering a discount of 25%? price (in Rs) of the article? Q50. A customer was offered a
एक व ु पर 20% की छूट दे ने के A िकसी व ु को 1800 पये म discount of 40% on a piece of
बाद, एक ापारी को 20% का लाभ खरीदता है और 10% तथा 20% की cloth. On insisting further, the
होता है | यिद य मू 300 पये है , दो िमक छूट दे ने के बाद इसे बेच shopkeeper agreed to a further
तो 25% की छूट दे ने के बाद इस दे ता है | इस व ु का िव य मू discount of 20% on above. What
व ु का िव य मू ा होगा ? ा होगा ? was the effective discount offered
SSC MTS 7 August 2019 SSC MTS 9 August 2019 to the customer?
(Evening) (Morning) एक ाहक को िकसी कपड़े पर 40%
की छूट दी गयी | और अनुरोध करने

e
(a) Rs352.5 (a) 1296
(b) Rs375.5 (b) 1668 पर, दु कानदार 20% की अित र
(c) Rs432.5 (c) 1728 छूट दे ने को तैयार हो गया | ाहक
(d) Rs337.5 (d) 1336 को िमली भावी छूट ात कर |

l
SSC MTS 14 August 2019
Q45. Three successive discounts Q48. After allowing a discount of (Afternoon)
ac
of 20%, 20% and 30% are offered
on an article. If the marked price
of the article is Rs750, then what
will be selling price?
एक व ु पर 20%, 20% और 30%
12.5%, shopkeeper makes a profit
of 25% on a bag. At what percent
higher than the cost price did he
mark the bag? (correct to nearest
integer)
(a) 60%
(b) 48%
(c) 52%
(d) 54%

की तीन िमक छूट दी जाती है | 12.5% की छूट दे ने के बाद एक Q51. Marked price of a shirt is
यिद व ु का अंिकत मू 750 पये दु कानदार को िकसी थैले पर 25% Rs2000. If shopkeeper declares
है , तो िव य मू ा होगा ? का लाभ होता है | उसने थैले का मू successive discount on shirt of
nn
SSC MTS 8 August 2019 य मू से िकतना ितशत अिधक 10% & 25%, the selling price (in
(Afternoon) अंिकत िकया था ? ( िनकटतम Rs) of shirt is:
(a) Rs 326 पूणाक तक सही ) एक शट का अंिकत मू 2000 पये
(b) Rs 375 SSC MTS 13 August 2019 है | यिद दु कानदार 10% और 25%
(c) Rs 348 (Afternoon) की िमक छूट दे ता है , तो शट का
(d) Rs 336 (a) 35% िव य मू ( पये म ) ात कर |
(b) 43% SSC MTS 14 August 2019
Q46. The marked price of an (c) 41% (Evening)
Pi

article is Rs3040. If the discount (d) 38% (a) 1350


offered on this article is 20%, (b) 1250
then what will be the selling Q49. The price that Kamal should (c) 1650
price? mark on a chair which costs him (d) 1300
िकसी व ु का अंिकत मू 3040 Rs1500 to gain 15% after
पये है | यिद इस व ु पर दी गयी allowing a discount of 25% (in Q52. A vegetable seller bought
छूट 20% है , तो िव य मू ा Rs) is: 10 dozens of potatoes for Rs120,
होगा ? कमल को 1500 पये य मू another 5 dozens for Rs50 and
SSC MTS 8 August 2019 वाली अपनी कुस का मू िकतना another 5 dozens for Rs30. He
(Evening) अंिकत करना चािहए तािक 25% की sold the potatoes for Rs 9 a
(a) Rs2412 छूट दे ने के बाद भी 15% का लाभ हो dozen. How much discount did he
(b) Rs3262 ? overall offer?

www.ssccglpinnacle.com support@ssccglpinnacle.com Ph. 09729327755, 09817390373

287
/
Days 33-36 Discount

s
sse
la
_c
ob
एक स ी िव े ता ने 120 पये म 10 Q55. A person buys an article for की कमी के बाद उसका िब ी मू
bo
ah
दज़न, 50 पये म 5 दज़न और 30 Rs. 16. If he had to buy a dozen 420 है ?
sm
ur
yo

पये म पुनः 5 दज़न आलू ख़रीदे | of articles, then he would have to SSC MTS 20 August 2019
e/
t.m

उसने इन आलुओं को 9 पये म एक


://

pay a total amount of Rs. 160. (Evening)


tp
ht

दज़न की दर से बेच िदया | उसने What would be the discount (a) 10.5
कुल िकतनी छूट दी ? percentage ( correct to the nearest (b) 12.5
SSC MTS 16 August 2019 integer ) on buying a dozen of (c) 14.5
(Morning) articles ? (d) 13.5
(a) 11.11% कोई 16 म एक व ु
(b) 5% खरीदता है | यिद उसे एक दजन Q58. When an article is sold at a
(c) 7.5% व ुऍं खरीदनी होतीं तो उसे 160 discount of 40% on its marked
(d) 10% की कुल रािश का भुगतान करना price, the profit is 25%. What is
पड़ता | एक दजन व ुएं खरीदने पर the ratio of the cost price to the
Q53. A person bought a shirt उसे िमलने वाली छूट लगभग िकतनी marked price of the article?
marked Rs. 1000 and a pair of ितशत (िनकटतम पूणाक म ) जब कोई व ु अपने अंिकत मू से
होती? 40% की छूट पर बेची जाती है , तो

e
trousers marked Rs. 2000. The
discounts offered on the shirt and SSC MTS 21 August 2019 लाभ 25% होता है | इस व ु के य
the pair of trousers were 20% and (Morning) मू और अंिकत मू म अनुपात
50%. Overall, How much (a) 10% ात कर |

l
discount he was offered ? (b) 17% SSC MTS 22 August 2019
एक ने 1000 अंिकत मू (c) 12% (Morning)

अंिकत मू
ac
की एक कमीज और 2000
की एक जोड़ी पतलून
खरीदी | कमीज और पतलून पर दी
गई छूट मश: 20% और 50% थी |
कुल िमलाकर, उसे िकतनी छूट दी
(d) 22%

Q56. The marked price of an


article was 42% above its cost
price. If after selling the article, a
(a) 12 : 25
(b) 4 : 5
(c) 5 : 8
(d) 8 : 13

गई ? profit of 20.7% occurs, then find Q59. A shopkeeper allows 10%


SSC MTS 16 August 2019 the discount percentage on the discount on the marked price of
(Evening) marked price of this article. an article and still gains 17%. If
nn
(a) 25% िकसी व ु का अंिकत मू उसके he gives 15% discount on the
(b) 40% लागत मू से 42% अिधक था | यिद marked price, then his profit
(c) 60% व ु को बेचने पर 20.7% का लाभ percent is:
(d) 45% होता है , तो व ु के अंिकत मू पर एक दु कानदार िकसी व ु के अंिकत
दी गई छूट का ितशत है : मू पर 10% की छूट दे ता है और
Q54. Marked price of an article is SSC MTS 21 August 2019 िफर भी 17% का लाभ कमाता है |
Rs1500. If 16 32 % discount is (Evening) यिद वह अंिकत मू पर 15% की
given, then what is the selling (a)15.6 छूट दे ता है , तो उसके लाभ का
Pi

price? (b)16 ितशत ा होगा ? SSC MTS 22


िकसी व ु का अंिकत मू 1500 (c)15 August 2019 (Afternoon)
पये है | यिद 16 32 % की छूट दी (d)14.3 (a) 12
जाती है , तो िव य मू ात कर | (b) 10.5
SSC MTS 19 August 2019 Q57. What is the discount (c) 12.5
(Morning) percentage on the gift whose (d) 10
(a) Rs1000 selling price is Rs. 420 after
(b) Rs1300 deducting the marked price by Q60. By paying Rs. 20 less than
(c) Rs 1250 Rs. 60 ? the marked price, a person gets a
(d) Rs1150 उस उपहार पर छूट की दर ितशत discount of 25%. What amount (
ा है िजसके अंिकत मू पर 60 in Rs ) did he pay ?

www.ssccglpinnacle.com support@ssccglpinnacle.com Ph. 09729327755, 09817390373

288
/
Days 33-36 Discount

s
sse
la
_c
ob
िकसी व ु के अंिकत मू से 20 Q63. The marked price of an (b) 35%
bo
ah
पए कम भुगतान करने पर कोई article is 57 of its cost price. If a (c) 40%
sm
ur
yo

25% छूट ा करता है | discount of 20% is given on the (d) 25%


e/
t.m

उसने कुल िकतनी रािश ( पए म )


://

marked price, then find the profit


tp
ht

का भुगतान िकया : percentage. Q66. A discount of 20% is


SSC MTS 21 August 2019 एक व ु का अंिकत मू उसके य offered on an item. Rishi gets a
(Afternoon) मू का 57 है | यिद अंिकत मू पर cashback of 4% after using a
(a) 20 20% की छूट दी जाती है , तो लाभ promo code. What is the
(b) 80 ितशत िकतना है ? equivalent single discount ?
(c) 50 SSC MTS 19 August 2019 िकसी मद पर 20% छूट की पेशकश
(d) 60 (Afternoon) की गयी है | एक ोमो कोड का
(a) 22% योग कर ऋिष 4% नकद वापस
Q61. The marked price of an (b) 12% ा करता है | समतु एकल छूट
article is x. After allowing a (c) 18% िकतनी है ?
discount of 20% on the marked (d) 15% SSC MTS 9 August 2019

e
price, the shopkeeper still gains (Evening)
Rs. 40. If the profit is 11 91 %, then Q64. An article is marked 37.5% (a) 22.60%
find the value of x. above the cost price. If a discount (b) 33%
िकसी व ु का अंिकत मू x है | of 9.09% is given, then find the (c) 24%

l
अंिकत मू पर 20 % की छूट दे ने के profit percentage ( to the nearest (d) 23.20%
बाद भी दु कानदार को 40 पए का
ac integer ) .
लाभ होता है | यिद लाभ 11 91 % होता िकसी व ु पर लागत मू से Q67. A shopkeeper gains 30%
है तब x का मान िकतना है ? 37.5% अिधक कीमत अंिकत िकया after giving a discount of 20% on
SSC MTS 20 August 2019 जाता है | यिद 9.09% की छूट दी an article. The marked price is
(Afternoon) जाती है तो लाभ ितशत (िनकटतम what percent of the cost price ?
(a) 500 पूणाक म ) है : िकसी व ु पर 20% की छूट दे ने के
(b) 360 SSC MTS 13 August 2019 प ात कोई दु कानदार 30% का लाभ
(c) 450 (Morning) कमाता है | अंिकत मू , लागत मू
का िकतना ितशत है ?
nn
(d) 400 (a) 25
(b) 15 SSC MTS 6 August 2019
Q62. The marked price of an (c) 10 (Morning)
article is 20% above its cost price. (d) 20 (a) 122.5 %
After allowing a discount of x% (b) 137.5 %
on the marked price, the Q65. A shopkeeper sold an article (c) 150 %
shopkeeper loses 10%. Find the for Rs. 180 by making a profit of (d) 162.5 %
value of x. 20%. The shopkeeper offered a
िकसी व ु का अंिकत मू उसके Q68. The 4 successive discounts
Pi

discount of Rs. 30 on the marked


लागत मू से 20% अिधक है | price. How much percent above of 20% is equivalent to a single
अंिकत मू पर x% छूट दे ने के बाद the cost price the article is discount of:
दु कानदार को 10% की हािन होती है marked ? 20% के ेक 4 िमक छूटों की
| x का मान िकतना है ? एक दु कानदार ने कोई व ु 20% के भावी छूट ा है ?
SSC MTS 20 August 2019 लाभ पर 180 पये म बेची | SSC MTS 14 August 2019
(Morning) दु कानदार ने अंिकत मू पर 30 (Morning)
(a) 30 पये की छूट दी | व ु का मू य (a) 64%
(b) 28 मू से िकतना ितशत अिधक (b) 59.04%
(c) 20 अंिकत िकया गया था ? (c) 40.96%
(d) 25 SSC MTS 9 August 2019 (d) 48%
(Afternoon)
(a) 30%

www.ssccglpinnacle.com support@ssccglpinnacle.com Ph. 09729327755, 09817390373

289
/
Days 33-36 Discount

s
sse
la
_c
ob
Q69. The marked price of an Q72. A company bought a दे कर बेच दे ता है | यिद उसे 140
bo
ah
article is Rs. 2500. It is sold at machine for Rs. 40000 and spent पये की हािन होती है , तो इस व ु
sm
ur
yo

two successive discounts of 12% Rs. 10000 on its repairing. Later, का य मू ( पये म ) िकतना था
e/
t.m
://

and 10% respectively. What is the the machine was sold for Rs. ?
tp
ht

selling price ? 45000. The discount offered was : SSC CGL 3 March 2020
िकसी व ु का अंिकत मू 2500 एक कंपनी ने 40000 पए म एक (Afternoon)
पए है | उसे मश : 12% तथा मशीन खरीदी और 10000 पए (a) 900
10% की दो िमक छूटो पर बेचा उसकी मर त पर खच िकए | कुछ (b) 840
जाता है | िव य मू िकतना है ? समय बाद, मशीन को 45000 पए (c) 872
SSC MTS 2 August 2019 म बेच िदया गया | िकतनी छूट की (d) 875
(Afternoon) पेशकश की गयी ? SSC MTS 14
(a) Rs. 1870 August 2019 (Morning) Q3. A person marked his goods at
(b) Rs. 2090 (a) 5% a price that would give him 40%
(c) Rs. 1760 (b) 100
9 % profit. But he declared a sale and
(d) Rs. 1980 (c) 10% allowed a 20% discount on the

e
(d) 7 21 % marked price. What is the profit
Q70. The ratio of marked price to percentage of the person in the
the selling price of an article is 12 SSC CGL 2019 TIER I whole transaction?
: 7. What is the discount एक ने अपनी व ुओं की
percentage ? िकसी व ु के अंिकत कीमत इतनी रखी िक उसे 40% का

l
Q1. A shopkeeper marks the price
मू तथा िव य मू का अनुपात of the article in such a way that लाभ होता है | लेिकन उसने सेल की

(Afternoon)
(a) 41.67
(b) 37.5
ac
12:7 है | छूट ितशत िकतना है ?
SSC MTS 5 August 2019
after allowing a 28% discount, he
wants a gain of 12%. If the
marked price is 224, then the cost
price of the article is:
एक दु कानदार व ु की कीमत इस
घोषणा की तथा अंिकत मू

िकतना रहा ?
के लाभ का ितशत

SSC CGL 3 March 2020


पर
20% की छूट दे ने लगा | पूरे लेन-दे न
म उस

(c) 39.33 कार िनधा रत करता है िक 28% (Evening)


(d) 42.33 की छूट दे ने के बाद भी वह 12% का (a) 12%
लाभ कमाना चाहता है | यिद अंिकत (b) 32%
nn
Q71. The selling price of a मू 224 है , तो इस व ु का य (c) 20%
product was equal to 50% of its मू ात कर | (d) 30%
cost price. Two successive SSC CGL 3 March 2020
discounts were offered on the (Morning) Q4. A trader allows a discount of
product. If the first discount was (a) ₹ 168 18% on the marked price of an
20%, then What was the second (b) ₹ 144 article. How much percentage
discount ? (c) ₹ 196 above the cost price must he mark
उ ाद का िव य मू उसके लागत it so as to get a profit of 6.6%? /
Pi

(d) ₹ 120
मू के 50% के बराबर था | उ ाद एक िव े ता िकसी व ु के अंिकत
पर दो िमक छूट की पेशकश की Q2. A dealer marks an article मू पर 18% की छूट दे ता है | 6.6%
गयी | यिद पहली छूट 20% थी , तो 40% above the cost price and का लाभ कमाने के िलए उसे इस
दू सरी छूट िकतनी थी ? sells it to a customer, allowing व ु की कीमत य मू से िकतनी
SSC MTS 14 August 2019 two successive discounts of 20% अिधक ( ितशत म ) रखनी चािहए ?
(Morning) and 25% on the marked price. If SSC CGL 4 March 2020
(a) 37.5% he suffers a loss of ₹140, then the (Morning)
(b) 33.33% cost price (in ₹) of the article is: (a) 30
(c) 30% एक िव े ता िकसी व ु का मू य (b) 28
(d) 25% मू से 40% अिधक रखता है तथा (c) 25
इसे एक ाहक को अंिकत मू पर (d) 24
20% एवं 25% की दो िमक छूट

www.ssccglpinnacle.com support@ssccglpinnacle.com Ph. 09729327755, 09817390373

290
/
Days 33-36 Discount

s
sse
la
_c
ob
Q5. The marked price of an SSC CGL 5 March 2020 marked price is ₹ 888, then the
bo
ah
article is ₹740. After two (Morning) cost price of the article is:
sm
ur
yo

successive discounts of 15% and (a) 15 73 % एक दु कानदार िकसी व ु का मू


e/
t.m

इस कार रखता है िक 22% की छूट


://

x%, it is sold for ₹566.10. What (b) 16 32 %


tp
ht

is the value of x? / (c) 15 31 % दे ने के बाद भी उसे 11% लाभ होता


एक व ु का अंिकत मू 740 पये है | यिद अंिकत मू 888 पये है ,
(d) 14 72 %
है | 15% तथा x% की दो िमक तो इस व ु का य मू िकतना
छूटों के बाद, इसे 566.10 पये म होगा ?
Q8. The marked price of an item
बेचा जाता है | x का मान ा है ? SSC CGL 6 March 2020
is 25% above its cost price. A
SSC CGL 4 March 2020 (Morning)
shopkeeper sells it, allowing a
(Afternoon) (a) ₹ 782
discount of x% on the marked
(a) 12 (b) ₹ 550
price. If he incurs a loss of 8%,
(b) 5 (c) ₹ 895
then the value of x is:
(c) 10 (d) ₹ 624
एक व ु का अंिकत मू इसके य
(d) 20
मू से 25% अिधक है | एक

e
SSC CHSL 2019
दु कानदार इसे अंिकत मू पर x%
Q6. If the selling price of an Q1. Mohan offers to sell his
की छूट दे कर बेचता है | यिद उसे
article is 8% more than the cost articles at a discount of 20%, but
8% की हािन होती है , तो x का मान
price and the discount offered is he marks his articles by
ा है ?

l
10% on the marked price of the increasing the price of each by
SSC CGL 5 March 2020
article, then what is the ratio of 35%. What percentage would his
ac
cost price to the marked price?
यिद एक व ु का िव य मू
मू से 8% अिधक है तथा व ु के
अंिकत मू पर 10% की छूट दी
जाती है , तो य मू तथा अंिकत

(Afternoon)
(a) 25.6%
(b) 25.2%
(c) 26.8%
(d) 26.4%
gain be?
मोहन अपनी व ुओं को 20% की
छूट पर बेचने की पेशकश करता है ।
लेिकन उसने अपनी व ुओं का मू
35% अिधक अंिकत िकया है । उसके
मू के बीच ा अनुपात है ? लाभ का ितशत ा होगा?
Q9. A furniture shopkeeper
SSC CGL 4 March 2020 CHSL 12-10-2020 (Afternoon
allows a discount of 16% on the
(Evening) shift)
nn
marked price on the goods to his
(a) 5:6 customers and still gains 20%. (a) 10%
(b) 8:9 What is the marked price of a (b) 7%
(c) 3:4 dining table, which costs the (c) 8%
(d) 4:5 shopkeeper ₹11,900? (d) 9%
फन चर बेचने वाला दु कानदार अपने
Q7. A shopkeeper bought 80 kg ाहकों को व ुओं के अंिकत मू Q2. The marked price of a ceiling
of rice at a discount of 10%. पर 16% की छूट दे ता है तथा िफर भी fan is Rs3,250 and the
20% लाभ कमाता है | उस डाइिनंग
Besides 1 kg rice was offered free shopkeeper allows a discount of
Pi

टे बल का अंिकत मू ात कीिजए,
to him on the purchase of every 8% on it. What is the selling price
िजसका य मू दु कानदार के िलए
20 kg rice. If he sells the rice at 11,900 पये है ? of the fan?
the marked price, his profit SSC CGL 5 March 2020 एक सीिलंग पंखे का अंिकत मू
percentage will be: (Evening) 3250 पये है तथा दु कानदार इस पर
एक दु कानदार ने 10% छूट पर 80 (a) ₹15,000 8% की छूट दे ता है । पंखे का िव य
िकलो ाम चावल ख़रीदा | इसके (b) ₹17,000 मू िकतना है ?
अलावा, ेक 20 िकलो चावल की (c) ₹16,000 CHSL 12-10-2020 (Evening
ख़रीद पर उसे 1 िकलो चावल मु (d) ₹12,376 shift)
िदया गया | यिद वह चावल को बाज़ार (a) Rs3,070
Q10. A shopkeeper marks the
मू पर बेच दे ता है , तो उसके लाभ (b) Rs2,590
price of an article in such a way
का ितशत होगा : (c) Rs3,270
that after allowing a discount of
(d) Rs2,990
22%, he gets a gain of 11%. If the

www.ssccglpinnacle.com support@ssccglpinnacle.com Ph. 09729327755, 09817390373

291
/
Days 33-36 Discount

s
sse
la
_c
ob
एक व ु का मू x पये अंिकत 15% और 20% की छूट पर एक
bo
ah
Q3. The marked price and cost िकया गया था तथा उसे (x-40)% की कंबल 680 पये म बेचा जाता है ।
sm
ur
yo

price of a book are Rs 850 and Rs छूट पर बेचा गया। यिद ाहक ने कंबल के अंिकत मू का पता
e/
t.m

(x-32) पये का भुगतान िकया, तो लगाएँ ।


://

748 respectively. The discount in


tp
ht

percentage is: उस व ु का अंिकत मू िकतना था CHSL 15-10-2020 (Evening


एक पु क का अंिकत मू और ? shift)
य मू मशः 850 पये तथा CHSL 14-10-2020 (Evening (a) Rs800
748 पये है । छूट का ितशत shift) (b) Rs950
िकतना है ? (a) 75 (c) Rs900
CHSL 13-10-2020 (Evening (b) 60 (d) Rs1000
shift) (c) 80
(a)12% (d) 50 Q10. A mobile phone was sold
(b)10% for ₹ 31,500 after getting two
(c)15% Q7.If the marked price of a successive discounts of 30% and
(d)8% television set is Rs 24,500, then 10% respectively. What was the

e
its selling price after a 12% marked price of the mobile?
Q4. If the cost price of a book discount on it is: एक मोबाइल फोन को 30% और
after 10% discount is Rs.486, यिद टे लीिवज़न सेट का अंिकत मू 10% की िमक छूट पर ₹ 31,500
then the marked price of the book 24,500 पये है , तो उस पर 12% म बेचा जाता है । मोबाइल का अंिकत
छूट के बाद उसका िब ी मू है : मू ा था?

l
is:
यिद 10% छूट के बाद िकसी पु क CHSL 15-10-2020 (Morning CHSL 16-10-2020 (Afternoon
की लागत मू
ac 486 पये है , तो
पु क का अंिकत मू है :
CHSL 14-10-2020 (Morning
shift)
(a) Rs580
shift)
(a)Rs 21,460
(b)Rs 21,650
(c)Rs 21,640
(d)Rs 21,560
shift)
(a) ₹ 50,000
(b) ₹ 55,000
(c) ₹ 52,500
(d) ₹ 35,000
(b) Rs540
(c) Rs560 Q8. In a grocery store, an item Q11. In a 15% discount sale, the
(d) Rs600 with an MRP of Rs1,100 is on a cost of a book is Rs 2,150. What
nn
discount with a special price of was the original price of the
Q5. If the single discount Rs979. What is the percentage of book?
equivalent to successive discounts discount given for that item? (Correct to two decimal places)
of 20% and x% is 24%, then the िकराने की दु कान म, 1,100 की 15% छूट की सेल म, एक िकताब की
value of x is: एमआरपी के साथ एक व ु 979 कीमत 2,150 पये है । पु क की
यिद एकल छूट 24%, 20% और x% पये के िवशेष मू के साथ छूट पर आरं िभक कीमत िकतनी थी?
की िमक छूट के बराबर है , तो x है । उस व ु के िलए दी गई छूट का (दो दशमलव थानों के िलए)
का मान है : ितशत िकतना है ? CHSL 16-10-2020 (Evening
Pi

CHSL 14-10-2020 (Afternoon CHSL 15-10-2020 (Afternoon shift)


shift) shift) (a) Rs2,529.41
(a) 5% (a) 9% (b) Rs2,500.00
(b) 6% (b) 10% (c) Rs1,527.00
(c) 8% (c) 12% (d) Rs2,250.50
(d) 4% (d) 11%
Q12. The printed price of a
Q6. An article was marked at ₹ x Q9. A blanket is sold for Rs680 at cooker is Rs2,000, and discounts
and sold at a discount of (x-40)%. the discounts of 15% and 20%. are 30%, 20% and 10%,
If the customer paid ₹(x-32), then Find the printed price of the respectively. Find the selling
find the marked price of the blanket. price of the cooker.
article.

www.ssccglpinnacle.com support@ssccglpinnacle.com Ph. 09729327755, 09817390373

292
/
Days 33-36 Discount

s
sse
la
_c
ob
एक कुकर का अंिकत मू 2000 यिद अंिकत मू 6,000 पये की 15% on the marked price
bo
ah
पये है तथा छूट मशः 30%, 20% खाने की मेज़ ाहक को 5,520 पये Rs.12000 of an article is:
sm
ur
yo

और 10% है । कुकर का िव य मू म बेची जाती है , तो मेज़ के अंिकत 12000 पये अंिकत मू वाली एक
e/
t.m

ात कीिजए। मू पर छूट की दर है : व ु पर 45% की एकल छूट तथा


://
tp
ht

CHSL 19-10-2020 (Afternoon CHSL 21-10-2020 (Evening 30% और 15% की दो िमक छूटों
shift) shift) के बाद ा िव य मू के बीच का
(a) Rs 1,002 (a) 7% अंतर है :
(b) Rs 1,008 (b) 5% CHSL 26-10-2020 (Evening
(c) Rs 1,004 (c) 8% shift)
(d) Rs 1,006 (d) 6% (a) 440
(b) 560
Q13. If a shopkeeper sells an item Q.16. An article is listed at (c) 520
that is marked as Rs 3,685 for Rs Rs.8,500, and the discount (d) 540
2,845, then how much discount is offered is 12%. What additional
he offering? discount (percent) must be given Q.19. A dealer marks his goods at
यिद कोई दु कानदार 3685 पये

e
to bring the selling price to 30% above the cost price. Then
अंिकत मू वाली व ु को 2845 6,358? he allows a 35% discount on it.
पये म बेचता है , तो वह िकतनी छूट एक व ु का मू 8,500 पये पर What would be his loss
दे रहा है ? अंिकत िकया गया है और छूट की percentage?
पेशकश 12% है । िब ी मू 6,358 एक डीलर लागत मू से 30% ऊपर

l
CHSL 19-10-2020 (Evening
shift) पये तक लाने के िलए िकतनी अपने माल का मू अंिकत करता
(a) 22.795%
(b) 29.52%
(c) 26.59%
(d) 34.87%
ac अित र
चािहए?
छूट ( ितशत) दी जानी

CHSL 26-10-2020 (Morning


shift)
(a) 9%
है । िफर वह उस पर 35% की छूट
दे ता है । उसका नुकसान ितशत है :
CHSL 17-03-2020 (Morning
shift)
(a) 16.5%
Q.14. A discount of 30% on the (b) 20% (b) 17.5%
marked price of a book enables (c) 15% (c) 18.5%
Arun to get a pen worth Rs.60 (d) 10% (d) 15.5%
nn
free. How much did Arun pay for
the book? Q.17. The marked price of a book Q20. A product whose MRP is ₹
एक पु क के अंिकत मू पर 30% is Rs.540. After 10% discount on 978, is sold for ₹ 925 by a
की छूट से अ ण को 60 पये का the marked price, the selling price wholesale shop owner. What is
एक पेन मु म िमल गया। पु क of the book is: the percentage of discount given
के िलए अ ण ने िकतना भुगतान एक पु क का अंिकत मू 540 by him?
िकया | पये है । अंिकत मू पर 10% छूट एक उ ाद िजसका एमआरपी 978
CHSL 21-10-2020 (Morning के बाद, पु क का िव य मू है : पये है , एक थोक दु कान के मािलक
Pi

shift) CHSL 26-10-2020 (Afternoon ारा 925 पये म बेचा जाता है ।


(a) 140 shift) उसके ारा दी जाने वाली छूट का
(b) 160 (a) 386 ितशत िकतना है ?
(c) 150 (b) 590 CHSL 17-03-2020 ( Evening
(d) 130 (c) 486 shift)
(d) 530 (a) 6.5%
Q.15. If a dining table with (b) 9.2%
marked price Rs.6,000 was sold Q.18. The difference between the (c) 5.4%
to a customer for Rs.5,520, then selling price obtained after a (d) 7.8%
the rate of discount allowed on single discount of 45% and two
the marked price of the table is: successive discounts of 30% and Q21. A dozen pairs of gloves
worth Rs600 are available at a

www.ssccglpinnacle.com support@ssccglpinnacle.com Ph. 09729327755, 09817390373

293
/
Days 33-36 Discount

s
sse
la
_c
ob
discount of 10%. Find out how है , तो इसके िब ी मू का पता ₹ 840 पये की लागत वाली शट पर
bo
ah
many pairs of gloves can be लगाएँ । नीरज को िकतना मू अंिकत करना
sm
ur
yo

bought for Rs270. CHSL 19-03-2020 ( Evening चािहए, तािक अंिकत मू पर 16%
e/
t.m

600 पये के एक दजन जोड़े द ाने की छूट की अनुमित के बाद 18% का


://

shift)
tp
ht

10% की छूट पर उपल ह। ात (a) 65 लाभ कमाया जा सके ?


कीिजए िक 270 पये म िकतने जोड़े (b) 63 CGL 2019 Tier-II (16-11-2020 )
द ाने खरीदे जा सकते ह? (c) 66 (a) ₹ 1,160
CHSL 18-03-2020 (Afternoon (d) 64 (b) ₹ 1,200
shift) (c) ₹ 1,180
(a) Seven/ सात SSC CGL TIER-II (d) ₹ 1,240
(b) Five/ पाँ च Q.25. An article marked 35%
(c) Six/ छः above its cost. If a profit of 20% Q28. An article is marked 25%,
(d) Four/ चार is earned by selling the article, above its cost price. If x%
then the discount offered on the discount is allowed on the marked
Q22. A single discount equivalent marked price of the article is : price and still there is a profit of
एक व ु को उसकी लागत से 35%

e
to the discount series of 25%, 5.5%, then what is the value of
15% and 10% is: ऊपर अंिकत िकया गया। यिद व ु x%
25%, 15% और 10% की छूट को बेचकर 20% का लाभ अिजत एक व ु का अंिकत मू य मू
ृंखला के बराबर एक एकल छूट है : िकया जाता है , तो व ु के अंिकत से 25% अिधक है । यिद अंिकत मू
मू पर दी जाने वाली छूट है :

l
CHSL 18-03-2020 ( Evening पर x% छूट की अनुमित है और िफर
shift) CGL 2019 Tier-II (15-11-2020 ) भी 5.5% का लाभ होता है , तो x%
(a) 40.2%
(b) 25%
(c) 42.6%
(d) 30%
ac (a) 15%
(b) 12%
(c) 11 91 %
(d)10 91 %
का मान ा है ?
CGL 2019 Tier-II (16-11-2020 )
(a) 13.6
(b) 15.4
(c) 15.6
Q.23. The marked price of a book Q26. Amit sold an article for (d) 16.4
is Rs.900. What is the selling ₹369.60 after allowing 12%
price of the book after giving two
nn
discount on the marked price. Q29. A dealer marks his goods at
successive discounts each of 10% Had he not allowed any discount 40% above the cost price. He
on it? he would have earned a profit of sells 60% of the goods at the
एक पु क का अंिकत मू 900 20%. What is the cost price of the marked price giving 10%
पये है । उस पर ेक 10% की दो article? discount and the rest by giving
िमक छूट दे ने के बाद पु क का अिमत ने अंिकत मू पर 12% छूट 50% discount on the marked
िव य मू िकतना होगा? की दे ने के बाद ₹369.60 म एक व ु price. What is the overall profit
CHSL 19-03-2020 (Morning को बेच िदया। अगर उसने कोई छूट loss percent?
shift) नहीं दी होती तो उसे 20% का लाभ एक डीलर अपनी व ुओं पर य
Pi

(a) 800 होता।व ु का लागत मू ा है ? मू से 40% अिधक मू अंिकत


(b) 749 CGL 2019 Tier-II (15-11-2020 ) करता है । उसने 60% व ुओं को
(c) 729 (a) ₹350 10% की छूट दे ने के बाद अंिकत
(d) 820 (b) ₹320 मू पर बेच िदया तथा शेष व ुओं
(c) ₹380 को अंिकत मू पर 50% की छूट
Q.24. The printed price on a book (d) ₹400 दे कर बेचा। कुल लाभ-हािन ितशत
is Rs.150. If it is sold after two िकतना है ?
successive discounts of 30% and Q27. What price should Neeraj CGL 2019 Tier-II (16-11-2020 )
40%, then find its selling price. mark on a shirt that cost ₹ 840, so (a) Profit/ लाभ 36%
एक िकताब पर मुि त मू 150 as to earn a profit of 18% after (b) Loss/ हािन 2.8%
पये है । यिद इसे 30% और 40% allowing a discount of 16% on (c) Loss/ हािन 3.6%
की दो िमक छूट के बाद बेचा जाता the mark price? (d) Profit/ लाभ 2.8%

www.ssccglpinnacle.com support@ssccglpinnacle.com Ph. 09729327755, 09817390373

294
/
Days 33-36 Discount

s
sse
la
_c
ob
(c) 8% Q36. If house tax is paid before
bo
ah (d)15% the due date, one gets a reduction
sm
ur
yo

Q30.The selling price of one of 12% on the amount of the bill.


e/
t.m
://

article after allowing a discount SSC CPO 2019 By paying the tax before the due
tp
ht

of 15% on its cost price, is the Q33. Three successive discounts date, a person got a reduction of
same as the selling price of 22%, 17% and 11% are Rs.2100. The amount (in Rs.) of
another article after allowing a equivalent to a single discount of: house tax was:
discount of 25% on its cost price. 22%, 17% और 11% की तीन यिद दे य तारीख से पहले गृह कर का
If the sum of the cost prices of िमक छूट _____ की एकल छूट के भुगतान िकया जाता है , तो िकसी
both the articles in 640, then find बराबर ह। को िबल की रािश पर 12%
the selling price of each article. CPO 2019 23-11-2020 की कटौती िमलती है । िनयिमत
एक व ु के य मू पर 15% की (Morning shift) तारीख से पहले कर का भुगतान
छूट के बाद उसका िव य मू , (a) approximately / लगभग 45% करके, एक को 2100 पये
दू सरी व ु के य मू पर 25% की (b) approximately / लगभग 50% की कटौती ा ई। गृह कर की
छूट के बाद उसके िव य मू के (c) approximately / लगभग 42% रािश ( पये म) थी:
बराबर है । यिद दोनों व ुओ के य (d) approximately / लगभग 25%

e
CPO 2019 23-11-2020
मू ों का योग ₹640 है तो ेक (Evening shift)
व ु का िव य मू ात कीिजये। Q34. Two successive discounts, (a) 17,500
CGL 2019 Tier-II (18-11-2020 ) each of x% on the marked price (b) 25,000

l
(a) ₹340 of the article, are equal to a single (c) 21,000
(b) ₹255 discount of Rs. 331.20. If the (d) 18,000
(c) ₹280
(d) ₹250
ac
Q31. An umbrella is marked for
₹150 and sold for ₹138. The rate
marked price of the article is Rs.
920, then the value of x is:
व ु के अंिकत मू पर x%
की दो िमक छूट 331.20 पये की
एकल छूट के बराबर ह। यिद व ु
ेक
Q37. If house tax is paid before
the due date, one gets a reduction
of 12% on the amount of the bill.
By paying the tax before the due
of discount is: का अंिकत मू 920 पये है , तो x date, a person got a reduction of
एक छतरी का मू ₹150 अंिकत का मान िकतना होगा? Rs.2100. The amount (in Rs.) of
िकया गया है और इसे ₹138 म बेचा CPO 2019 23-11-2020 house tax paid was:
nn
जाता है । छूट की दर है : (Morning shift) यिद दे य तारीख से पहले गृह कर का
CGL 2019 Tier-II (18-11-2020 ) (a) 20 भुगतान िकया जाता है , तो िकसी
(a) 5% (b) 18 को िबल की रािश पर 12%
(b) 6% (c) 15 की कटौती िमलती है । िनयिमत
(c) 8% (d) 25 तारीख से पहले कर का भुगतान
(d) 9% करके, एक को 2100 पये
Q35. A single discount equivalent की कटौती ा ई। गृह कर की
Q.32 An article is listed at ₹ to three successive discount of रािश ( पये म) थी:
Pi

7,600 and the discount offered 8%, 15% and 12% is : CPO 2019 24-11-2020
unit is 10%. What additional 8%, 15% और 12% के तीन मागत (Morning shift)
discount must be given to bring छूट के बराबर एक एकल छूट ात (a) 17,500
the net selling price to ₹ 5,814? करे | (b) 15,400
एक व ु का अंिकत मू ₹ 7,600 है CPO 2019 23-11-2020 (c) 21,000
और छूट की पेशकश की इकाई (Evening shift) (d) 25,000
10% है । शु िब ी मू को ₹ (a) 35%
5,814 पर लाने के िलए अित र (b) 17.5% Q38. A single discount is
छूट िकतनी दी जानी चािहए? (c) 68.816% equivalent to three successive
CGL 2019 Tier-II (18-11-2020 ) (d) 31.184% discount of 6%, 15% and 14% is:
(a) 12%
(b) 10%

www.ssccglpinnacle.com support@ssccglpinnacle.com Ph. 09729327755, 09817390373

295
/
Days 33-36 Discount

s
sse
la
_c
ob
िकतनी एकल छूट 6%, 15% और equivalent to an approximate Q44. On the marked price of
bo
ah
14% की तीन िमक छूटों के बराबर single discount of: Rs.1,250 of an article, three
sm
ur
yo

है ? िकतनी एकल छूट, 12%, 11% और successive discount of 5%, 15%


e/
t.m

13% की तीन िमक छूट के बराबर


://

CPO 2019 24-11-2020 and 20% were offered, The


tp
ht

(Morning shift) है ? amount (in Rs.) of discount


(a) 31.286% CPO 2019 25-11-2020 received by a customer is:
(b) 34.357% (Morning shift) एक व ु के 1,250 पये के अंिकत
(c) 68.714% (a) 35% मू पर, 5%, 15% और 20% की
(d) 17.5% (b) 32% तीन िमक छूट की पेशकश की
(c) 42% गई, ाहक ारा ा छूट की रािश
Q39. On the marked price of an (d) 40% ( पये म) ा है ?
article, the sum of selling price CPO 2019 25-11-2020
with a discount of 35% and two Q42. If two successive discount, (Evening shift)
successive discounts of 20% and each of 20% on the marked price (a) 950.25
15% is Rs.1,995. The marked of an article, are equal to a single (b) 450

e
price of the article(in Rs.) is: discount of Rs. 331.20, then the (c) 442.50
एक व ु के अंिकत मू पर, 35% marked price(in Rs.) of the article (d) 807.50
की छूट और 20% और 15% के दो is:
िमक छूट के साथ िब ी मू का यिद एक व ु के अंिकत मू पर दो
योग 1,995 पये है । व ु का अंिकत िमक छूट, ेक 20% की,

l
मू ( म) है : 331.20 पये की एकल छूट के
CPO 2019

(a) 1,600
(b) 1,550
(c) 1,800
ac
(Evening shift)
24-11-2020 बराबर है , तो व ु का अंिकत मू
( पये म) ा है ?
CPO 2019
(Morning shift)
(a) 645
25-11-2020

(d) 1,500 (b) 1,200


(c) 920
Q40. On the marked price of Rs. (d) 750
nn
1,250 of an article, three
successive discounts of 5%, 15% Q43. On a marked price, the
and 20% are offered. What will difference of selling prices with a
be the selling price (in Rs) after discount of 35% and two
all discounts? successive discounts of 20% and
एक व ु के 1,250 पये के अंिकत 15% is Rs.504. The Marked price
मू पर, 5%, 15% और 20% की of the article (in Rs.) is:
तीन िमक छूट की दी जाती है । अंिकत मू पर, 35% की छूट और
Pi

सभी छूटों के बाद िव य मू ( म) 20% और 15% के दो िमक छूटों


ा होगा? के साथ िव य मू ों का अंतर 504
CPO 2019 24-11-2020 पये है । व ु का अंिकत मू
(Evening shift) ( पये म) है :
(a) 975.75 CPO 2019 25-11-2020
(b) 807.50 (Evening shift)
(c) 1,000 (a) 18,000
(d) 950.25 (b) 16,800
(c) 16,000
Q41. Three successive discount (d) 15,500
of 12%, 11% and 13% are

www.ssccglpinnacle.com support@ssccglpinnacle.com Ph. 09729327755, 09817390373

296
/
Days 33-36 Discount

s
sse
la
_c
ob
Variety Questions Balancing the ratio for List price Effective discount = 25 + 5 -
bo
ah and New list price 25 × 5 = 28.75%
sm
100
ur
yo

Sol 1. (c) CP : List Price : New List Price : Desired difference = 2000 x
e/
t.m
://

Effective Discount = 259.20 x 100 SP 30−28.75 = 25


tp

720
ht

100
= 36% 16 : 20 : 25 :
Effective Discount in terms of x = 20 Sol 9. (a)
x×x )
( x + x - 100 Desired gain % = 2016 −16 x 100 =
Let the CP = 100 unit
x×x ) = 36
⇒ ( x + x - 100 25%
⇒ x2 -200 x +3600 = 0
⇒ x2 -180 x -20 x +3600 = 0
Sol 5. (a)
⇒ x ( x -180) -20( x -180) = 0
20% = 51 and 18% = 9
50
⇒ x = 20
…….( x =/ 180 ) SP : MP : CP
41 : 50
6 : 5
Sol 2. (c)

e
Balancing the ratio for MP Discount % = 10 x 100 = 8 31 %
Effective discount = 20+12- 120
20 × 12 = 29.6 % SP : MP : CP
100
37
246 : 300 : 250
⇒ 29.6 % = 125
Sol 10. (a)
According to the question
Effective Discount = 30+20 -

l
Let the MP = 125 and Discount = 246 unit = 492 30×20 = 44%
37 1 unit = 2 100

88 unit = 528
1 unit = 6
ac
⇒ SP = 125 - 37 = 88
According to the question

125 unit = 125 x 6 = 750


CP (250) unit = 500

Sol 6. (d)
Effective discount = 20 + 6.25 -
20×6.25 = 25 %
100
Alternate :
30% = 103 and 20% =

10 ----- 7
5 ----- 4
_________
1
5

25% = 1 and 20% = 1 50 28


4 5 50−28
Sol 3. (c) Therefore, Net discount = 50 x
MP : SP : CP
Let the MP of a copy = 1 unit
nn
4 : 3 100 = 44
⇒ MP of 100 copies = 100 x 1 =
6 : 5
100 unit Sol 11. (c)
Balancing the ratio for SP
Discount from publisher = 100 Discount equivalent to (25% and
25 = 25 MP : SP : CP
× 100 20%)= 25+20 - 25×20
8 : 6 : 5 100 = 40%
⇒ MP of 50 copies = 50 x 1 = 50 Net discount = Equivalent
Desired %age = 8−55 x 100 = 60%
unit discount of (40% and 10%) =
Discount from Retailer = 50 40+10- 40×10
100 = 46%
Sol 7. (d)
Pi

10 = 5 unit
× 100
25% = 41 Let the MP = 100 unit
Total Discount = 25+5 = 30 unit ⇒ SP = 100 x 100100
−46 = 54 unit
30 x
Price of the article after 25%
Desired Discount % = 100+50 According to the question
discount = 960 × 43 = 720
100 = 20% 720−612 54 unit = 324
X= 720 x 100 = 15%
1 unit = 6
Alternate :
Sol 4. (b) 960−612 100 unit = 100 x 6 = 600
Effective discount = x 100
20% = 51 and 25% = 1
4
960 Alternate :
= 36.25 % 4 ----- 3
⇒ 25 + x - 25 × x = 36.25
CP : List Price : New List Price : 100 5 ----- 4
SP ⇒ 100 + 4x − x = 145 10 --- 9
4 : 5 ⇒ x = 15 50 : 27
4 : 5 According to the question
5 : 4 Sol 8. (a)

www.ssccglpinnacle.com support@ssccglpinnacle.com Ph. 09729327755, 09817390373

297
/
Days 33-36 Discount

s
sse
la
_c
ob
27 unit = 324 Discount equivalent to (10% and Discount amount (575-529) unit
bo
ah
1 unit = 12 5%) = 10+5 - 10×5
100 = 14.50 = 46 x 44 = 2024
sm
ur
yo

50 unit = 12 x 50 = 600 Discount equivalent to (14.50%


e/
t.m
://

and 2%) = 14.50+2 - 14.50×2 SSC CGL TIER II


100 =
tp
ht

Sol 12. (c) 16.21


Discount equivalent to (20% and Sol 1. (c)
5%)= 20+5 - 20×5
100 = 24% Sol 15. (a) Effective discount = 587.40
1500 x 100
CP of the dealer = 20000 x 12% = 25 3 = 39.16 %
100−24 = 15200 x×x = 39.16
⇒ x × x - 100
100 MP : SP
Total amount spent on the article 25 : 22 ⇒ 200x − x2 = 3916
= 15200+1000 = 16200 Let the MP = 25 unit ⇒ x2 - 200x +3916 = 0
Profit earned = 20000-16200 = SP = 22 unit ⇒ x2 - 178x − 22x +3916 = 0
3800 (25-22) unit = 90 ⇒ x(x − 178) − 22(x − 178) = 0
3800
Desired %age of profit = 16200 x 1 unit = 30 ⇒ x = 22%
100 = 23.46 % SP (22 unit) = 22 x 30 = 660 ………..( x =/ 178 )

e
Alternate : Desired SP = 1500 × 100100 −22 =

5% = 201 and 20% = 1 Sol16. (a)


5 1170
According to the question Discount equivalent to (20% and
20 ----- 19 25%)= 20+25 - 20×25
100 = 40% Practice Questions

l
5 ----- 4 Discount equivalent to (40% and

1 unit = 200
ac
100 : 76
According to the question
100 unit = 20000

76 unit = 15200
10%)= 40+10 - 40×10

Alternate :
100 = 46%

According to the question


4 ----- 3
Sol 1. (a)
Effective Discount = 15+10-
15×10 = 23.5%
100
23.5% = 47
200
Let the MP = 200 unit, Discount
Total amount spent on the article 5 ----- 4 = 47 unit
= 15200+1000 = 16200 10 --- 9 ⇒ SP = 200-47 = 153 unit
Profit earned = 20000-16200 = 50 : 27 According to the question
nn
3800 Therefore, Net discount = 50−27 x
3800 50 153 unit = 642.60
Desired %age of profit = 16200 x 100 = 46 1 unit = 4.20
100 = 23.46 % 200 unit = 840
Sol 17. (d) Amount discount =
Sol 13. (c) 6 = Rs. 2310
38500 × 100 Sol 2. (b)
Discount equivalent to (10% and Effective Discount = 15+10-
4%) = 10+4 - 10×4
100 = 13.60 Sol 18. (d)
15×10 = 23.5%
100
Let the third discount = x 47
Pi

Required discount = 22000 23.5% = 200


According to the question 115 × 8 = 2024
× 100 Let the MP = 200 unit, Discount
100
13.60 + x - 13.60
100
× x = 19
Alternate : = 47 unit
13.60 + 0.8640x = 19 3 and 8% =
15% = 20 2 ⇒ SP = 200-47 = 153 unit
25
⇒ x = 6.25 CP : MP : SP According to the question
20 : 23 153 unit = 657.90
Sol 14. (b) 25 : 23 1 unit = 4.30
Original Discount = 10% Balancing the ratio for MP 200 unit = 860
1st additional Discount = 50% of CP : MP : SP
10 = 5% 500 : 575 : 529 Sol 3. (c)
2nd additional Discount = 40% of According to the question Effective Discount = 20+16-
5=2% 500 unit = 22000 20×16 = 32.8%
100
1 unit = 44 32.8% = 41
125

www.ssccglpinnacle.com support@ssccglpinnacle.com Ph. 09729327755, 09817390373

298
/
Days 33-36 Discount

s
sse
la
_c
ob
Let the MP = 125 unit, Discount Let the MP = 5 unit, Discount = 2 According to the question
bo
ah
= 41 unit unit 50 unit = 1250
sm
ur
yo

⇒ SP = 125-41 = 84 unit ⇒ SP = 5-2 = 3 unit 1 unit = 25


e/
t.m
://

According to the question According to the question 28 unit = 700


tp
ht

84 unit = 1680 3 unit = 288


1 unit = 20 1 unit = 96 Sol 10. (d)
125 unit = 125 x 20 = 2500 5 unit = 5 x 96 = 480 Effective Discount = 29+12 -
29 × 12 = 37.52%
Alternate : 100
Sol 4. (d) 25% = 41 and 20% = 51 Let the MP =100 unit
Effective Discount = 25+15- 4 ------ 3 ⇒ Discount = 37.52 unit
25×15 = 36.25%
100 5 ------ 4 and
36.25% = 29 x _____________ CP = 100 -37.52 = 62.48 unit
80
Let the MP = 80 unit, Discount = 20 12 According to the question
29 unit According to the question 62.48 unit = 312.40
⇒ SP = 80-29 = 51 unit 12 unit = 288 1 unit = 5

e
According to the question 1 unit = 24 100 unit = 100 x 5 =500
51 unit = 535.50 20 unit = 20 x 24 = 480 Alternate :
29% = 10029 and 12% = 3
1 unit = 10.50 25
80 unit = 10.5 x 80 = 840 Sol 8. (a) 100 ------ 71

l
25% = 41 25 ------ 22
Sol 5. (a) ac Price of the article after 25% x _____________
Effective Discount = 30+15- discount = 480 × 43 = 360 2500 1562
30×15 = 40.50% 360−288 According to the question
100 X= 360 x 100 = 20%
40.50% = 81
Alternate : 1562 unit = 312.40
200
Let the MP = 200 unit, Discount Effective discount = 480−288 x 100 1 unit = 1/5
480
= 81 unit 2500 unit = 500
= 40 %
⇒ SP = 200-81 = 119 unit ⇒ 25 + x - 25 × x = 40
100
Sol 11. (b)
According to the question ⇒ 2500 + 100x − 25x = 4000
nn
Effective discount = 29 + 13 -
119 unit = 547.40 ⇒ 75x = 1500 29 × 13 = 38.23%
1 unit = 4.6 ⇒ x = 20
100

200 unit = 200 x 4.6 = 920 Let the MP =100 unit


⇒ Discount = 38.23 unit
Sol 9. (a)
Sol 6. (d) and
Effective Discount = 30+20 -
Effective Discount = 25+15- 30×20 = 44%
CP = 100 -38.23 = 61.77 unit
25×15 = 36.25% 100 According to the question
100 44% = 11
36.25% = 29 25 61.77 unit = 617.70
Pi

80 Let the MP = 25 unit and 1 unit = 10


Let the MP = 80 unit, Discount = Discount = 11 unit 100 unit = 100 x 10 = 1000
29 unit ⇒ CP = 25-11 = 14 unit
⇒ SP = 80-29 = 51 unit
According to the question Alternate :
According to the question 25 unit = 1250 29 and 13% = 13
29% = 100 100
51 unit = 612 1 unit = 50 100 ------ 71
1 unit = 12 14 unit = 14 x 50 =700 100 ------ 87
80 unit = 12 x 80 = 960 Alternate : x _____________
30% = 10 3 and 20% = 1
5 10000 1562
Sol 7. (b)
10 ------ 7 According to the question
Effective Discount = 25+20-
25×20 = 40% 5 ------ 4 6177 unit = 617.70
100 x _____________
2
1 unit = 1/10
40% = 5 50 28 10000 unit = 1000

www.ssccglpinnacle.com support@ssccglpinnacle.com Ph. 09729327755, 09817390373

299
/
Days 33-36 Discount

s
sse
la
_c
ob
Discount equivalent to (20% and Profit earned = 20000-18000 =
bo
ah
Sol 12. (c) 25%)= 20+25 - 20×25
100 = 40% 2000
sm
ur
yo

Effective Discount = 30+20 - 2000


Discount equivalent to (40% and Desired %age = 18000 = 11.1%
e/
t.m
://

30×20 = 44%
10%)= 40+10 - 40×10
100 = 46%
tp

100
ht

44% = 11
25 Let the MP = 100 unit Sol 16. (b)
Let the MP = 25 unit and ⇒ Discount = 46 unit and SP = Discount equivalent to (20% and
Discount = 11 unit 100-46 = 54 5%)= 20+5 - 20×5
100 = 24 %
⇒ CP = 25-11 = 14 unit According to the question CP of the dealer = 20000 x
According to the question 54 unit = 405 100−24 = 15200
100
25 unit = 1500 1 unit = 7.5 Total amount spent on the article
1 unit = 60 100 unit = 750 = 15200+1800= 17000
11 unit = 11 x 60 =660 Alternate : Profit earned = 20000-17000 =
Alternate : 25% = 41 , 20% = 51 and 10% = 3000
30% = 10 3 and 20% = 1 1 3000
5 10 Desired %age of profit = 17000 x
10 ------ 7 According to the question 100 = 17.65 %

e
5 ------ 4 4 ----- 3 Alternate :
x _____________ 5 ----- 4 20% = 51 and 5% = 1
20
50 28 10 ----- 9 According to the question
According to the question 50 : 27 5 ----- 4

l
50 unit = 1500 According to the question 20 ----- 19
1 unit = 30 ac 27 unit = 405 _________
Discount = 50-28 unit = 22 x 30 = 1 unit = 15 100 : 76
660 50 unit = 750 According to the question
100 unit = 20000
Sol 13. (a) Sol 15. (c) 1 unit = 200
Effective Discount = 25+20- Discount equivalent to (10% and
25×20 = 40%
76 unit = 15200
100 8%)= 10+8 - 10×8
100 = 17.20% Total investment = 15200+1800 =
40% = 2
5 CP of the dealer = 20000 x 17000
nn
Let the MP = 5 unit, Discount = 2 100−17.20 = 16560 Profit earned = 20000-17000 =
100
unit Total amount spent on the article 3000
⇒ SP = 5-2 = 3 unit Desired %age = 17000 3000
x 100=
= 16560+1440= 18000
According to the question Profit earned = 20000-18000 = 17.65%
3 unit = 480 2000
1 unit = 160 2000
Desired %age of profit = 18000 x Sol 17. (b)
5 unit = 5 x 160 = 800 Discount equivalent to (20% and
100 = 11.1 %
Alternate : 5%)= 20+5 - 20×5
100 = 24 %
Pi

25% = 41 and 20% = 51


Alternate : CP of the dealer = 30000 x
4 ------ 3 2 and 10% =
8% = 25 1 100−24 = 22800
10 100
5 ------ 4
According to the question Total amount spent on the article
x _____________
25 ----- 23 = 22800+1200= 24000
20 12
10 ----- 9 Profit earned = 30000-24000 =
According to the question
250 : 207 6000
12 unit = 480 6000
According to the question Desired %age of profit = 24000 x
1 unit = 40
250 unit = 20000 100 = 25 %
20 unit = 20 x 40 = 800
1 unit = 80
207 unit = 16560 Alternate :
Sol 14.(c)
Total investment = 16560+1440 = 20% = 51 and 5% = 1
20
18000 According to the question

www.ssccglpinnacle.com support@ssccglpinnacle.com Ph. 09729327755, 09817390373

300
/
Days 33-36 Discount

s
sse
la
_c
ob
5 ----- 4 80.96 unit = 688.16 1 unit = 4
bo
ah 20 ----- 19 1 unit = 8.5 171 unit = 684
sm
ur
yo

_________ 100 unit = 8.5 x 100 = 850


e/
t.m
://

100 : 76 Sol 22. (c)


tp
ht

According to the question Alternate : Let the second discount = x %


3 and 8% =
12% = 25 2 20% = 51
100 unit = 30000 25
1 unit = 300 According to the question Price of the article after 20%
76 unit = 22800 25 ----- 22 discount = 900 × 54 = 720
Total investment = 22800+1200 = 25 ----- 23 x= 720−648 x 100 = 10%
720
24000 _________ Alternate :
Profit earned = 30000-24000 = 625 : 506 900−648
Effective discount = 900 x 100
6000 According to the question
6000 = 28 %
Desired %age = 24000 x 100= 506 unit = 688.16 20 × x
⇒ 20 + x - 100 = 28
25% 1 unit = 1.36
⇒ 2000 + 100x − 20x = 2800
625 unit = 850
⇒ 80x = 800

e
Sol 18. (a)
⇒ x = 10
Discount equivalent to (20% and Sol 20. (c)
5%)= 20+5 - 20×5
100 = 24 %
100−24 Sol 23. (a)
CP of the dealer = 5000 x 100 1
10% = 10

l
= 3800
Let the original marked price =
ac
Total amount spent on the article
= 3800+200= 4000
Profit earned = 5000-4000 = 1000
Desired %age of profit = 1000
100 = 25 %
4000 x
10 unit
⇒ New price = 10+1 = 11 unit
Desired %age of discount = 1111−1
1 %
x 100 = 9 11

Alternate :
20% = 51 and 5% = 1 10 Sol 24 (c)
20 Desired %age = 125 x 100 = 8%
Discount equivalent to (30% and
According to the question
nn
25%)= 30+25 - 30×25
100 = 47.5%
5 ----- 4 Sol 21. (b)
20 ----- 19 Discount equivalent to (47.5%
Discount equivalent to (10% and
and 15%)= 47.5+15 - 47.5100× 15 =
_________ 5%)= 10 + 5 - 10×5
100 = 14.50 %
100 : 76 55.375%
Let the MP = 100 unit
According to the question ⇒ Discount = 14.50 and SP = Alternate :
30% = 103 , 15% = 3 and 25% =
100 unit = 5000 100-14.5 = 85.5 unit 20
1
1 unit = 50 According to the question 4
76 unit = 3800 100 unit = 800 According to question:
Pi

Total investment = 3800+200 = 1 unit = 8 10 : 7


4000 85.5 unit = 85.5 x 8 = 684 20 : 17
Profit earned = 5000-4000 = 1000 4 : 3
Desired %age = 10004000 x 100= 25% Alternate : 800 : 357
1 and 5% =
10% = 10 1 Equivalent Discount =
20
Sol 19. (a) 800−357 443 × 100 = 55.375%
× 100 = 800
According to the question 800
Discount equivalent to (12% and 10 ----- 9
8%)= 12+8 - 12×8
100 = 19.04 % 20 ----- 19 Sol 25. (c)
Let the MP = 100 unit _________ Discount equivalent to (20% and
⇒ Discount = 19.04 and SP = 200 : 171 10%)= 20+10 - 20×10
100 = 28%
100-19.04 = 80.96 unit According to the question Discount equivalent to (28% and
According to the question 200 unit = 800 15%)= 28+15 - 28× 15
100 = 38.8%

www.ssccglpinnacle.com support@ssccglpinnacle.com Ph. 09729327755, 09817390373

301
/
Days 33-36 Discount

s
sse
la
_c
ob
Alternate : Alternate : 1 unit = 29
bo
ah
20% = 51 , 15% = 3 and 10% = 3 , 8% =
12% = 25 2 Discount (2 unit) = 2 x 29 = 58
sm
20 25
ur
yo

1 According to question:
e/
t.m

10
://

Sol 33. (a)


tp

According to question:
ht

25 ------ 22 Let marked price = 100 unit


5 ------ 4 25 ---- 23 Discounted price = 100-12 = 88
10 ---- 9 625 ---- 506 unit
20 ---- 17 Desired Discount %age = 100 unit = 264
1000 ---- 612
625−506 × 100 ≈ 19% 1 unit = 2.64
625
Equivalent discount = 88 unit = 232.32
1000−612 × 100 = 194 = 38.8% Sol 29. (c)Let the cp of mobile = SP of such 4 covers = 4 x 232.32
1000 5
x = 929.28
According to question Alternate :
Sol 26. (d) 3
(25-20)% MP= 1000 12% = 25
Discount equivalent to (20% and
10%)= 20+10 - 20×10 MP = 20000 Let the MP = 25 unit and
100 = 28%

e
Initial paid = 20000 × 100 80 = Discount = 3 unit
Discount equivalent to (28% and
16000 ⇒ SP = 25-3 = 22 unit
8%)= 28+8 - 28× 8
100 = 33.76%
According to the question
Alternate :
Sol 30. (d) 25 unit = 264
20% = 51 , 8% = 2 and 10% =

l
25
Two successive discounts are 1 unit = 10.56
1
10 equivalent to 25+5- 25×5 SP (22 unit) = 22 x 10.56 =
ac 100 = 28.75
According to question: 232.32
%
SP of such 4 covers = 4 x 232.32
Desired difference = 2000 ×
5 ------ 4 = 929.28
(30-28.75)% = 25
10 ---- 9
Sol 31. (b)
25 ---- 23 3 Sol 34. (d)
15% = 20
1250 ---- 828 24% = 25 6 and 15% = 3
20
Equivalent discount = Let the CP = 20 unit
SP : MP : CP
1250−828 × 100 = 33.76% ⇒ MP = 20+3 = 23 unit
nn
1250 17 : 20
According to the question
31 : 25
20 unit = 2500
Balancing the rato for MP
1 unit = 125
Sol 27. (a) SP : MP : CP
23 unit = 2875
25% = 41 and 15% = 3
20 345 x 100 = 12 527 : 620 : 500
Desired %age = 2875
SP : MP : CP According to the question
17 : 20 (620-527) unit = 223200
Sol 32. (c) 1 unit = 2400
5 : 4
Let marked price = 100 unit
Pi

Balancing the ratio for MP CP (500 unit) = 2400 x 500 =


Discounted price = 100-8 = 92 1200000
SP : MP : CP
unit
17 : 20 : 16
92 unit = 667 Sol 35. (a) MP =
According to the question
1 unit = 7.25 100
924 × 88 = 1050
16 unit = 2080
8 unit = 58
1 unit = 130 Therefore, Discount =
Alternate :
17 unit = 2210 2
12 = 126
1050 × 100
8% = 25
Alternate : Alternate :
17 = 2210
SP = 2080 × 45 × 20 Let the MP = 25 unit and 3
12% = 25
Discount = 2 unit
⇒ SP = 25-2 = 23 unit Let the MP = 25 unit and
Sol 28. (c) Discount = 3 unit
Equivalent discount = 12 + 8 - According to the question
⇒ SP = 25-3 = 22 unit
12×8 = 20 - 0.96 = 19 (round-off) 23 unit = 667
100 According to the question

www.ssccglpinnacle.com support@ssccglpinnacle.com Ph. 09729327755, 09817390373

302
/
Days 33-36 Discount

s
sse
la
_c
ob
22 unit = 924 80 unit = 80 x 250 = 12000 60% = 3

bo
5
ah
1 unit = 42
sm
Let the MP = 5 unit and Discount
ur
yo

Discount (3 unit) = 3 x 42 = 126 Alternate : = 3 unit


e/
t.m
://

3 and 25% =
15 % = 20 1
⇒ SP = 5-3 = 2 unit
tp

4
ht

Sol 36. (d) 20 ----- 17 According to the question


Effective discount = 12+10 - 4 ---- 3 2 unit = 1560
12×10 = 20.80
100 _________ 1 unit = 780
20.80 % = 26 80 51
125 5 unit = 3900
Let the MP = 125 unit and Alternate:
Discount =26 unit According to the question According to the question
⇒ SP = 125-26 = 99 unit 51 unit = 7650 (100-60)% of MP = 1560
According to the question 1 unit = 150 ⇒ MP = 156040 x 100 = 3900
125 unit = 2500 80 unit = 80 x 250 = 12000
1 unit = 20 Sol 43. (d)
99 unit = 99 x 20 = 1980 Sol 39. (d) 40% = 52

e
Alternate : 12.5% = 81
Let the MP = 5 unit and Discount
12 % = 253 and 10% = 1 Now,
10 = 2 unit
25 ----- 22 MP : CP : SP ⇒ SP =5-2 = 3 unit
10 ---- 9 5 : 4 Desired ratio = 3:5

l
_________ 8 : 9 Alternate :
250 198 ac Balancing the ratio for CP
According to the question MP : CP : SP According to the question
250 unit = 2500 10 : 8 : 9 (100-40)% of MP = SP
1 unit = 10 Discount % = 1010−9 x 100 = 10% ⇒ MSPP = 100
60
198 unit = 1980 Alternate : ⇒ SP : MP = 3 : 5
Let the MP = 500 unit and CP =
Sol 37. (b) 400 unit Sol 44. (d)
Effective discount = 15+10 - ⇒ SP = 400 x 112.5
100 = 450 unit 20% = 51 and 20% = 1
nn
15×10 = 23.50 5
Discount % = 500−450 x 100 =
100 500 MP : SP : CP
Alternate : 10% 5 : 4
3 and 10% =
15 % = 20 1
10 6 : 5
20 ----- 17 Sol 40. (a) Balancing the ratio for SP
10 ---- 9 Discount equivalent to (10% and MP : SP : CP
_________ 8%) = 10+8- 10×8
100 = 17.20 15 : 12 : 10
200 153 ⇒ S P after this discount = 1000 According to the question
Effective discount = 200−153 x 100 × 100100
−17.20 = 8280 10 unit = 300
Pi

200
= 23.50 Further discount amount = 1 unit = 30
8280-7866 = 414 15 unit = 30 x 15 = 450
Sol 38. (d) 414 x
Desired Discount %age = 8280 Desired SP = 450 x 100100 −25 =

Effective discount = 25+15 - 100 = 5% 337.50


15×25 = 36.25
100
36.25 % = 29 Sol 45. (d)
80 Sol41. (d)
Let the MP = 80 unit and Discount offered = 2800-2408 = Discount equivalent to (20% and
Discount =29 unit 392 20%)= 20+20 - 20×20
100 = 36%
⇒ SP = 80-29 = 51 unit 392 x 100
Desired Discount % = 2800 Discount equivalent to (36% and
According to the question = 14 % 30%)= 36+30 - 36× 30
100 = 55.20%
51 unit = 7650 138
55.20 = 250
1 unit = 150 Sol 42. (a)

www.ssccglpinnacle.com support@ssccglpinnacle.com Ph. 09729327755, 09817390373

303
/
Days 33-36 Discount

s
sse
la
_c
ob
Let MP = 250 unit and Discount 5 ------ 4 13
32.5% = 40
bo
ah
= 138 10 ------ 9
sm
Let MP = 40 unit and Discount =
ur
yo

⇒ SP = 250-138 = 112 50 ----- 36 13


e/
t.m
://

According to the question Now, ⇒ SP = 40-13 = 27


tp
ht

250 unit = 750 50 unit = 1800 According to the question


1 unit = 3 1 unit = 36 40 unit = 2000
112 unit = 336 36 unit = 1296 1 unit = 50
27 unit = 1350
Alternate : Sol 48. (b)
20% = 51 , 20% = 1
5 and 30% = 12.5 % = 81 and 25% = 1
4 Alternate :
3 MP : SP : CP 25% = 41 and 10% = 1
10 10
According to question: 8 : 7 According to question:
5 : 4
5 ------ 4 Balancing the ratio for SP 4 ------ 3
5 ------ 4 MP : SP : CP 10 ------ 9

e
10 ---- 7 40 : 35 : 28 40 ----- 27
250 ---- 112 Desired percentage = 4028−28 x 100
Now,
According to the question ≈ 43% 40 unit = 2000
250 unit = 750 1 unit = 50

l
1 unit = 3 Sol 49. (a) 27 unit = 1350
112 unit = 336 15 % = 203 and 25% = 1

Sol 46. (c)


20% = 51
ac
Let the MP = 5 unit and Discount
= 1 unit
MP : SP : CP
4 : 3
23 : 20
Balancing the ratio for SP
MP : SP : CP
4
Sol 52.(d)
Total amount spent = 120+50+30
= 200
Total SP = (10+5+5) × 9 = 180
Discount offered = 200-180 = 20
Desired Discount % = 20020 x 100
⇒ SP = 5-1 = 4 unit 92 : 69 : 60
According to the question According to the question = 10%
nn
5 unit = 3040 60 unit = 1500
1 unit = 608 1 unit = 25 Sol 53. (b)
4 unit = 2432 90 unit = 25 x 92 = 2300 Total Discount offered = 1000
−20 + 2000 × 100−50 = 1800
× 100100 100
Sol 47. (a) Sol 50. (c) Desired Discount %age =
Discount equivalent to (20% and Effective discount = 40+20 - 3000−1800 x 100 = 40%
3000
40×20 = 52%
10%)= 20+10 - 20×10
100 = 28% 100
7
28% = 25 Alternate : Sol 54. (c)
Pi

Let MP = 25 unit and Discount = 40 % = 52 and 20% = 1


5 16 32 % = 61
7 5 ----- 3 Let the MP = 6 unit and Discount
⇒ SP = 25-7 = 18 5 ----- 4 = 1 unit
According to the question _________ ⇒ SP = 6-1 = 5 unit
25 unit = 1800 25 12 According to the question
1 unit = 72 6 unit = 1500
Effective discount = 25−12 x 100
18 unit = 1296 25 1 unit = 250
= 52 5 unit = 1250
Alternate :
20% = 51 and 10% = 1
10 Sol 51. (a) Sol 55. (b)
According to question: Discount equivalent to (25% and Total amount spent on one dozen
10%)= 25+10 - 25×10
100 = 32.5% articles = 16 x 12 = 192

www.ssccglpinnacle.com support@ssccglpinnacle.com Ph. 09729327755, 09817390373

304
/
Days 33-36 Discount

s
sse
la
_c
ob
Total Sale Price = 160 25% = 1 Alternate :
bo
4
ah
Desired Discount % = 192−160 x Let the MP = 700 unit and CP =
sm
192 Let the MP = 4 unit and Discount
ur
yo

100 ≈ 17% = 1 unit 500 unit


e/
t.m
://

⇒ SP = 4-1 = 3 unit According to the question


tp
ht

Sol 56. (c) 1 unit = 20 SP = 700 × 100100


−20 = 560 unit

42% = 5021 and 20.7% = 207 ⇒ 3 unit = 60 Profit earned = 560-500 = 60 unit
1000
Desired Profit %age = 50060 x 100
MP : CP : SP Alternate :
71 : 50 According to the question = 12%
1000 : 1207 25% of MP = 20
⇒ MP = 25 20 x 100 = 80
Balancing the ratio for CP Sol 64. (a)
MP : CP : SP Desired SP = 80-20 = 60 37.5% = 83 and 9.09% = 1
11
1420 : 1000 : 1207 SP : MP : CP
Desired Discount %age = Sol 61. (a) 10 : 11
1420−1207 x 100 = 15%
1420 20% = 51 and 11 91 % = 1
9 11 : 8
Now, so,

e
Sol 57. (b) MP : SP : CP SP : MP : CP
Discount = 60 5 : 4 10 : 11 : 8
Desired Profit %age = 10−8 x 100
SP = 420 10 : 9 8
⇒ MP = 420+60 = 480 Balancing the ratio for SP = 25%

l
Desired Discount % = 60 x 100 MP : SP : CP
480
= 12.5%

Sol 58. (a)


ac
40% = 52 and 25% =
MP : SP : CP
1
4
25 : 20 : 18
According to the question
(20-18) unit = 40
1 unit = 20
25 unit = 25 x 20 = 500
Sol 65. (c)
20% = 51
Let the CP = 5 unit and Profit = 1
unit
⇒ SP = 5+1 = 6 unit
5 : 3 According to the question
5:4 Sol 62. (d) 6 unit = 180
Balancing the ratio for SP 20% = 51 and 10% = 1
10 1 unit = 30
nn
MP : CP : SP Now, CP of article (5 unit) = 150
25 : 12 : 15 MP : CP : SP MP of the article = 180+30 = 210
6 : 5 Desired %age = 210150 −150 x 100 =

Desired ratio = 12 : 25 10 : 9 40%


Balancing the ratio for CP
Sol 59. (b) MP : SP : CP Sol 66. (d)
10% = 10 1 , 17% = 17 and 15% = 12 : 9 : 10 Effective Discount = 20+4 - 20×4
100 100
3 Desired Discount %age (x) =
= 23.20 %
Pi

20 12−9 x 100 = 25%


MP : SP : CP 12 Alternate :
10 : 9 20% = 51 and 4% = 1
25
117 : 100 Sol 63. (b)
5 ------- 4
Balancing the ratio for SP 20% = 51
25 ------ 24
MP : CP : SP SP : MP : CP __________
130 : 100 : 117 4 : 5 125 96
SP after 15% discount = 130 7: 5 Desired Discount %age = 125−96
125
17 = 110.5
× 20 Balancing the ratio for MP
x 100 = 23.20
Desired profit %age = 110.5−100 x SP : MP : CP
100
28 : 35 : 25
100 = 10.5% 28−25
Sol 67. (d)
Desired Profit %age = x
25 20% = 51 and 30% = 3
10
Sol 60. (d) 100 = 12%
MP : SP : CP

www.ssccglpinnacle.com support@ssccglpinnacle.com Ph. 09729327755, 09817390373

305
/
Days 33-36 Discount

s
sse
la
_c
ob
5 : 4 Sale price is 50% of cost price.
bo
ah 13 : 10 So, the effective discount = 50% Sol 4. (a) Let Marked price of
sm
ur
yo

Balancing the ratio for SP According to the question article = ₹ M


e/
t.m
://

MP : SP : CP 50 = 20 + x - 20100
×x Cost price = ₹ C
tp
ht

65 : 52 : 40 30 = 4x Selling price = ₹ S
5
Desired %age = 40 65 x 100 = 162.5 82% of Marked price = 106.6%
⇒ x = 37.5%
% Alternate : of cost price
M = 106.6 = 13
Let the Cost Price = 2 unit C 82 10
Sol 68. (b) ⇒ the sale price = 1 unit Thus, Marked price should be
Discount equivalent to (20% and According to the question 30% above Cost price.
20%)= 20+20 - 20×20
100 = 36% 80 x 100−x
1 = 2 × 100 100
Discount equivalent to (36% and Sol 5. (c) Marked price of article
125 = 200 - 2x
20%)= 36+20 - 36×20 = ₹740
100 = 48.8% ⇒ x = 37.5%
Discount equivalent to (48.8% Selling price of article = ₹566.10
85 × 100−x = 566.10
740 × 100
and 20%)= 48.8+20 - 48.80×20
100 = Sol 72. (c) 100

e
59.04% Total amount spent on machine = x= 10%
Alternate : 40000 + 10000 = 50000
According to the question Sale Price of machine = 45000 Sol 6. (a) According to question:
5 ----- 4 Required discount % = SP = 108% of CP = 90% of MP

l
50000−45000 x 100 = 10% Thus, M CP = 90 = 5
5 ----- 4 50000 P 108 6

625
ac
5 ----- 4
5 ----- 4
____________
256
Therefore, Net discount = 625−256
625
SSC CGL 2019 TIER I

Sol 1. (b) Marked price = ₹ 224


Discount% = 28%
Sol 7. (b) Let M.P of 1kg rice =
₹1
Then, M.P of 80 kg rice = ₹ 80
But after 10% discount, C.P for
customer = ₹ 72
x 100 = 59.04 Gain% = 12%
112 CP = 72 MP
Selling price = 100 Also 1 kg rice was given free for
100
Sol 69. (d) Cost Price = ( 72 ) × 224 = every 20 kg. Therefore, customer
nn
112
Effective Discount = 12+10- got 84 kg rice at ₹ 72
₹144
12×10 = 20.80 Selling price for 84 kg rice = ₹ 84
100
Required Sale Price = 2500 x
−72 × 100 = 16 2 %
Profit % = 8472 3
Sol 2. (d) Let the cost price of
100−20.80 = 1980
100 article be 100 units
Alternate : Marked price becomes 140 units Sol 8. (d) Let the cost price of an
Required Sale Price of the article Selling price after discount of article = 100 units.
88 x 90 = 1980
= 2500 x 100 20% and 25% becomes 84units Marked price at 25% increase =
100
Loss = (100-84) = 16 units 125 units
Pi

But according to question loss = Selling price at 8% loss = 92


Sol 70. (a)
₹140 units.
Let the Marked price of the
1 unit =₹ (140/16) Discount % = 125125 −92 × 100 =
article = 12 unit
100 units = 140 26.4%
⇒ Sale price of the article = 7 16 × 100 = ₹ 875
unit
Sol 3. (a) Let the CP of article be Sol 9. (b) According to question:
Discount offered = 12-7 = 5 unit 84 120
Required %age = 12 5 × 100 = ‘x’ 100 M P = 100 CP
M P = 120 = 10
140 of x
Marked Price = 100 CP 84 7
41.67 CP = 7x = ₹ 11,900
After 20% discount on MP, SP = Then, MP = ₹ 11900 × 10 = ₹
80 7 112 7
Sol 71. (a) 100 × 5 of x = 100 of x% profit in 17,000
Let the second discount = x % whole transaction=
12% Sol 10. (d) According to question:

www.ssccglpinnacle.com support@ssccglpinnacle.com Ph. 09729327755, 09817390373

306
/
Days 33-36 Discount

s
sse
la
_c
ob
78% of MP = 111% of CP Selling price after 12% discount = Discount = 12%
bo
ah
M P = 111 ⇒ M P = 37 88
100 × 24500 = 21,560 Then, SP of the article = Rs.7480
sm
CP 78 CP 26
ur
yo

MP = 37 units = ₹ 888 After another discount article is


e/
t.m
://

Then, CP = 26 units = ₹ 624 Sol:8. (d) sold at Rs.6358


tp
ht

SSC CHSL 2019 Discount = 1100-979 = 121 So, discount = 7480-6358 =


Sol:1. (c)
121 × 100 =
Discount percent = 1100 Rs.1122
11% Discount % = 1122
7480 × 100 = 15%
x + y + xy /100 %
35×(−20)
35 + (-20) + = 8%
100
Sol:9. (d) Sol :17. (c)
Let the printed price = x Marked price = Rs. 540
Sol:2.(d) discount = 8% = 2 85 × 80 = 680 Discount = 10%
25 x × 100 100
9 × 540 = 486
Selling price = 10
Here, MP = 25 Units → Rs3250 x = 1000
SP = 23 units → Rs2990
Sol:10. (a) Sol:18. (d)
Sol:3.(a) 70 ×
MP × 100 90 = 31,500 Two successive discount of 30%

e
100
% Discount = 850−748 × 100 = and 15%, gives single discount =
850 MP = 50,000
12%
× 15 = -45 + 450 = -
-30-15+ 30100 100

Sol:11.(a) 45 + 4.5 = - 40.5%


Sol:4. (b) Let the original price = x Difference between single

l
Marked price = 486 × 100 = 540 85 = 2,150 discount of 45% and two
90 x × 100
ac successive discount of 30% and
x = 2529.41
Sol.5 (a) 15% = 45% - 40.5% = 4.5%
4.5 ×
Required difference = 100
Equivalent percentage = Sol:12. (b)
(x + y − 100
xy
)% required selling price = 2000 × 12000 = Rs. 540
24 = 70 80 90
100 × 100 × 100 = 1,008
(x + 20 − 20x
)% Sol:19. (d)
100
x = 5% Let the cost price = Rs. C
Sol:13.(a)
Marked price = 130100 × C
Discount = 3685-2845 = 840
nn
Sol:6.(c) Required percentage = After 35% discount on marked
65 ×
price, Selling price = 100
(x-40)% of x = 32 840
3685 × 100 = 22.795%
x−40 × x = 32 130 ×C = 84.5 ×C
100 100 100
x2 - 40x = 3200 Sol:14. (a) Loss % = 100 - 84.5 = 15.5%
x2 - 40x - 3200 = 0 Let MP of the book = 100
(x - 80) (x + 40)=0 Discount = 30% = Rs. 60 Or, Net profit/loss = +30 - 35 -
30 × 35 = -5 - 1050 = -5-10.5 =
x = 80 or - 40 MP(100%) = Rs. 200 100 100
x = 80 (As x cannot be negative ) So, Arun pay for the book = -15.5%
Pi

ALTERNATE SOLUTION 200-60 = Rs.140


To save time, it is better to go Sol:20. (c)
through option: Sol:15. (c) Discount = 978-925 = 53
Go with option (c) Marked Price of dining table = 53 × 100
Required percentage = 978
Let x = 80 Rs.6000 = 5.4%
Price after discount = 80- Selling price of dining table = Rs.
(80−40)
100 × 80 = 48 (it will be paid 5520 Sol 21. (c)
by customer) Discount = 6000-5520 = Rs.480 Selling price of dozen pair of
480 × 100 = 8%
Discount % = 6000 9 = 540
Money paid by customer = gloves = 600 × 10
(80-32) = 48 (which satisfy) In Rs. 540, 12 pairs of gloves can
Sol:16. (c) be bought.
Sol:7. (d) MP of article = Rs.8500

www.ssccglpinnacle.com support@ssccglpinnacle.com Ph. 09729327755, 09817390373

307
/
Days 33-36 Discount

s
se
s
la
_c
ob
Thus, in Rs. 270, 6 pairs of Additional discount =
bo
ah
gloves will be bought. sol:28.(c) 6840-5814=1026
sm
ur
yo

Let CP = x Additional discount


e/
t.m
://

Sol 22. (c) MP = 1.25x %=(1026/6840) × 100=15%


tp
ht

Discount provided = 25% , 15% After x% discount = 100−x × 1.25


100
and 10% = 1.055 SSC CPO 2019
Let us initially have 100 units x = 15.6% Sol:33.(c)
After 25% discount, it becomes = Let MP be 100
75 units Sol:29.(a) After allowing a discount of 22%
Next after 15% disount, it Let CP = x SP would be 78%
becomes = 75 × 100 85 = 63.75 units
Let number of goods = y 100:78
Next a discount of 10% makes it Total cost price= xy 100:83
9 = 57.375 units
= 63.75 × 10 MP = 1.4x 100:89
Therefore, actual discount After discount of 10% MP= 1.26x After multiplying
received = 100 - 57.375 = 42.625 60% of the goods SP = 0.6y × 1000000:576186

e
units (approx) 1.26x = 0.756xy 100:57.61 (MP:SP)
Required discount % = 40% of the goods SP = 0.4y × discount= 100-57.61=42.39
100 − 57.375 × 100 = 42.6 % 0.7x = 0.28xy Approximately equal to 42%
100
Total selling price = 1.036xy

l
Profit = 0.036 Sol:34.(a)
Sol:23. (c) 1 × 100 = 3.6%
Marked price of book = Rs. 900 MP of the article = 920
ac
After two successive discount of
10% each, Selling price = 900
90 × 90 = Rs. 729
× 100 100

Sol 24. (b)


Sol:30.(b)
Let the cost price of first article =
100x
Cost price of another article =
100y
discount= 331.20
SP of article=588.80
As there is successive discount so
we will take under root to get the
required discount

Printed price of Book = Rs. 150 After discount of 15% on first SP √920 :√588.80
Selling price = 150 × 100100
− 30 × = 85x Taking square root and after
nn
100 − 40 70 60 After discount of 25% on second simplifying we get
100 = 150 × 100 × 100 = Rs.
SP = 75y 10:0.8
63
According to the question So required discount %= 0.2
10
85x = 75y × 100 =20%
SSC CGL TIER-II 15 y
x = 17
Sol:25.(c)
100x + 100y = 640 Sol:35.(d)
Let CP= x
15
17 y + y = 6.4
Let MP be 100
MP = 1.35x
y = 3.4 After allowing a discount of 8%
Profit = 20%
Pi

100y = 340 SP would be 92


SP = 1.2x
75y = 255 similarly
Discount = 1.35x −1.2x × 100
1.35x
0.15x 100
100:92
= 1.35x × 100 = 9 = 11 91 %
Sol:31.(c) 100:85
Discount = 150-138=12 100:88
Sol:26.(a) After multiplying
100 rate of discount=(12/150) ×
MP = 369.60 × 88 = 420 1000000:688160
100
100=8%
CP = 420 × 120 = 350 100:68.8160 (MP:SP)
Sol:32.(d) discount= 100-68.8160 =
Sol:27.(c) Discount offered = 10% of 31.184%
SP = 840 × 118
100 7600=760
Marked price = 840 × 118 × 100
Price after discount = Sol:36.(a)
100 84
MP = 1,180 7600-760=6840 According to the question

www.ssccglpinnacle.com support@ssccglpinnacle.com Ph. 09729327755, 09817390373

308
/
Days 33-36 Discount

s
sse
la
_c
ob
12% of bill= 2100 Sol:41.(b)
bo
ah
Bill = 2100 Let MP be 100
sm
12 × 100 = 17,500
ur
yo

After allowing a discount of 12%


e/
t.m
://

Sol:37.(b) SP would be 8%
tp
ht

According to the question 100:78


12% of bill = 2100 100:83
Bill = 2100 100:89
12 × 88 = 15,400
After multiplying
Sol:38.(a)
1000000:576186
Let MP be 100
100:57.61 (MP:SP)
After allowing a discount of 6%
discount= 100-57.61=42.39
SP would be 94
Approximately equal to 42%
similarly
100:94
Sol:42.(c)
100:85
Discount given=20%
100:86

e
MP:SP= 10:8
After multiplying
And here two successive
1000000:687,140
discounts are given so
100:68.7140 (MP:SP)
Ratio will be 100:64
discount= 100-68.7140 =

l
Discount offered on a MP of 100
31.286%
= 100-64=36
Sol:39.(d)
ac
Let the marked price= 100MP
After 35% discount SP= 65MP
After two successive discount of
20% and 15% net discount=
According to the question the
discount is 331.20
36 = 100
So comparing 331.20 X
So x = 920 which is our required
marked price
20+15- 20×15
100 =32%
Sol:43.(b)
So SP will be 68 MP
Let the marked price= 100MP
nn
Now sum of both the selling
After 35% discount SP= 65MP
prices=(65+68)MP=133 MP
After two successive discount of
As given 133 MP=1995
20% and 15% net discount=
100 MP=1500
20+15- 20×15
100 =32%

Sol:40.(b) So SP will be 68 MP
Marked price=1250 (given) Now difference of both the
Successive discount is offered so selling prices=(68-65)MP=3 MP
Pi

we will use ratio method to solve As given 3 MP=504


the problem 100 MP=16800
20 : 19
20 :17 Sol:44.(c)
5:4 MP = 1250
SP after Discount = 1250 ×
95 85 80
100 × 100 × 100 = 807.5
2000:1292
Discount = 1250 - 807.5 = 442.50
100 : 64.6

If MP =1250 then SP= 64.6 ×


12.5=807.5

www.ssccglpinnacle.com support@ssccglpinnacle.com Ph. 09729327755, 09817390373

309
/
Days 37-39 Simple Interest

s
sse
la
_c
ob
SIMPLE INTEREST / years. उदाहरण: यिद एसआई पये Q3. A sum of Rs. 12,800 is
bo
ah है 4 साल के िलए 400, तो यह पये invested partly at 15% per annum
sm

साधारण ाज
ur

हो जाएगा। 2 साल के िलए 200


yo

and the remaining at 12% per


e/
t.m
://

annum simple interest. If the total


tp
ht

Key Points:/ मुख िबंदु: Variety Questions interest at the end of 3 years is Rs.
5085. Then how much money
Q1. A borrows a sum of Rs 1000 was invested at 15% per annum.
1. Simple Interest on a sum
from his friend B on 31 12,800 पये की एक रािश अंशतः
(principal) of money at R% per
December 2015 on the condition 15% ित वष तथा अंशतः 12% ित
annum for T years is given by:
that he will return the same after वष साधारण ाज पर िनवेश की
T साल के िलए ितवष R% पर
one year with simple interest at जाती है | यिद 3 वष के अंत म कुल
िकसी रािश (मूलधन) पर साधारण
12%. However, A gets into a ाज 5085 पये है , तो 15% ित
ाज िन ानुसार िदया जाता है :
position of returning the money वष पर िकतनी रािश िनवेश की गयी
P rincipal×Rate×T ime P RT
SI = 100 = 100 on 1 May 2016. How much थी ?
2. Amount, A = Principal + amount he has to return to B? SSC CPO 12 March 2019
Simple Interest A ने अपने िम B से 1000 पये की

e
(Evening)
3. Also, P = SI×100 , T = SI×100 रािश 31 िदसंबर 2015 को इस शत (a) Rs.5200
R×T R×P
SI×100 पर उधार ली िक वह इसे एक वष के (b) Rs.7500
, R= P ×T
बाद 12% साधारण ाज के साथ (c) Rs.5800
लौटा दे गा | हालाँ िक, A 1 मई 2016

l
(d) Rs.5300
Example: A man lends Rs. 500 at को ही यह रािश लौटाने की थित म
ac
10% per annum for 2 years. His
interest will be:
एक आदमी 10% ित वष 5000
पये 2 साल के िलए उधार दे ता है |
उसका ाज ा होगा:
आ गया | वह B को िकतनी रािश
वापस करे गा ?
SSC CHSL 10 July 2019
(Morning)
(a)Rs 1,331.5
Q4. At what rate percent per
annum with simple interest will a
sum of money double in 12.5
years?
Solution: Here, P = Rs. 500, R = (b)Rs 1,045 ित वष िकतने ितशत साधारण
10% and T = 2 years. (c)Rs 1,120 ाज की दर से एक रािश 12.5 वष
SI = P RT = 500×10×2 = Rs. 100 (d)Rs 1,040 म दोगुनी हो जायेगी ?
nn
100 100
SSC CPO 16 March 2019
Important Note:/ मह पूण लेख: Q2.A certain sum was invested on (Evening)
simple interest. The amount to (a)8
i) Notice that in the formula,
which it had grown in five years (b)12.5
S I = P100
RT , we are just calculating
was 1 41 times the amount to (c)10
(T ×R) % of P.
which it had grown in three years. (d)6
ान द िक, S I = P100
RT , सू म हम
The percentage rate of interest
िसफ P के (T ×R) % गणना कर रहे
was: Q5.The simple interest on a
Pi

ह|
एक िनि त रािश साधारण ाज पर certain sum at 15% p.a. For three
िनवेश की गयी | पां च वष म यह years is Rs. 7200. The sum is
ii) Simple Interest increases बढ़कर जो रािश बनी वह तीन वष म िकसी िनि त रािश पर 15% ित वष
linearly with number of years i.e. इसम ई वृ का 1 41 गुना है | ाज की दर से तीन वष का साधारण ाज
you can directly divide by the
की दर है : 7200 पये है | यह रािश है -
number of years to get SI for 1
year. SSC CHSL 11 July 2019 SSC CPO 15 March 2019
साधारण ाज साल की सं ा के (Morning) (Morning)
साथ रै खक प से बढ़ता है अथात (a)10% (a)Rs.16000
आप 1 वष के िलए एसआई ा (b)20% (b)Rs.24000
करने के िलए साल की सं ा से सीधे (c)25% (c)Rs.32000
िवभािजत कर सकते ह। (d)15% (d)Rs.48000
Example: If SI is Rs. 400 for 4
years, then it will be Rs. 200 for 2

www.ssccglpinnacle.com support@ssccglpinnacle.com Ph. 09729327755, 09817390373


310
Days 37-39 Simple Interest

s
sse
la
_c
ob
Q6. The simple interest on a sum 480 पये साधारण ाज पर िनवेश invested in the first scheme. If the
bo
ah
for a certain number of years, the िकये गए | यह रािश 20 महीने बाद total interest earned after five
sm
ur
yo

same as the rate percentage of the 520 पये हो गयी | ित वष ाज years is Rs2800, then how much
e/
t.m

की दर ात कर |
://

interest, is equal to the sum itself. money was invested in the third
tp
ht

The number of years is equal to: SSC MTS 6 August 2019 scheme?
िकसी रािश पर कुछ िनि त वष, (Evening) 10000 पये की एक रािश साधारण
िजनकी सं ा ाज के दर ितशत (a) 6% ाज की तीन योजनाओं म िनवेश की
के बराबर है , का साधारण ाज (b) 5% जाती है | वािषक ाज दर मशः
यं रािश के बराबर है | वष की (c) 8% 4%, 6% और 10% ह | पहली योजना
सं ा ात कर | (d) 4% म 4000 पये िनवेश िकये गए | यिद
SSC MTS 2 August 2019 पां च वष के बाद ा कुल ाज
(Morning) Q10. A sum of Rs1500 is invested 2800 पये है , तो तीसरी योजना म
(a) 5 at simple interest for x months. If िकतनी रािश िनवेश की गयी थी ?
(b) 10 the rate of interest is 8x % per SSC MTS 8 August 2019
(c) 8 annum, then the sum grows to (Evening)

e
(d) 1 Rs1590. What is the value of x? (a) Rs1500
1500 पये की एक रािश x महीनों के (b) Rs5000
Q7. The simple interest on a िलए साधारण ाज पर िनवेश की (c) Rs1000
principal for 6 months at an गयी | यिद ाज की दर 8x % है , तो (d) Rs3000

l
interest rate of 10% per annum is यह रािश बढ़ कर 1590 पये हो
Rs100. What is the principal? जाती है | x का मान ात कर | Q13. If the ratio of principal and
ac
िकसी मूल धन पर 10% की दर से 6
माह का साधारण ाज 100 पये है |
यह मूलधन ात कर |
SSC MTS 5 August 2019
(Morning)
SSC MTS 7 August 2019
(Morning)
(a) 3.2
(b) 2.4
(c) 32
the simple interest for 5 years is
10 : 7, then the rate of interest
(per annum) is:
यिद मूल धन और 5 वष के साधारण
ाज म 10 : 7 का अनुपात है , तो
(a) Rs1000 (d) 24 ाज की दर ( ित वष ) ात कर |
(b) Rs2000 SSC MTS 9 August 2019
(c) Rs1500 (Morning)
nn
Q11. A sum of Rs 800 invested
(d) Rs2500 on simple interest becomes Rs (a) 15%
1200 in 8 years. What will be (b) 20%
Q8. If Rs. 1000 has been invested simple interest for 6 years on the (c) 10%
@ 12.5% simple interest per sum at the same rate of interest? (d) 14%
annum for two years, then what is साधारण ाज पर िनवेश की गयी
the amount ? 800 पये की एक रािश 8 वष म Q14. The Simple interest at the
यिद 1000 को 12.5% ित वष की 1200 पये बन जाती है | इसी ाज end of 3 years on a sum of Rs.
दर से साधारण ाज पर 2 वष के दर से इस रािश पर 6 वष का 2800 is Rs. 420. What will be the
Pi

िलए िनवेश िकया गया है , तो िम धन साधारण ाज िकतना होगा ? simple interest on Rs. 3200 for
िकतना है ? SSC MTS 8 August 2019 the same period at the same rate ?
SSC MTS 6 August 2019 (Afternoon) 3 वष के अंत म 2,800 की िकसी
(Afternoon) (a) Rs240 रािश पर साधारण ाज 420 है |
(a) 1125 (b) Rs210 इसी दर और इतनी ही अविध के िलए
(b) 1250 (c) Rs250 3200 पर साधारण ाज िकतना
(c) 1325 (d) Rs300 होगा ?
(d) 1275 SSC MTS 9 August 2019
Q12. A sum of Rs10000 is (Evening)
Q9. Rs 480 is invested at simple invested in three schemes of (a) Rs480
interest. It becomes Rs 520 after simple interest. The annual (b) Rs560
20 months. What is the interest interest rates are respectively, 4%, (c) Rs440
rate per annum? 6% and 10%. Rs4000 were (d) Rs640

www.ssccglpinnacle.com support@ssccglpinnacle.com Ph. 09729327755, 09817390373


311
Days 37-39 Simple Interest

s
sse
la
_c
ob
Q18. If the simple interest on Rs. जाती है िक पहले िह े पर 6 32 %
bo
ah
Q15. A certain sum amounts to 28000 at some rate for three years
sm
ित वष की दर से 4 51 वष का
ur
yo

Rs20720 in four years and is Rs. 225 more than the simple साधारण ाज दू सरे िह े पर 4%
e/
t.m
://

Rs24080 in six years at a certain interest on Rs. 27000 at the same ित वष की दर से 2 43 वष के


tp
ht

rate of simple interest. The sum rate for 3 years, then what will be
साधारण ाज से दोगुना है | इन दोनों
(in Rs) is: the simple interest on Rs. 35500
िह ों के बीच अंतर ात कर| SSC
कोई िनि त रािश साधारण ाज की for 2 53 years at the same rate ?
CGL Tier 2 11 September 2019
िनि त दर से चार वष म 20720 यिद 28000 की रािश पर 3 वष म (a) 680
पये तथा छः वष म 24080 पये हो िकसी दर पर साधारण ाज, 27000 (b) 600
जाती है | यह रािश है : की रािश पर 3 वष म उसी दर की (c) 560
SSC MTS 13 August 2019 साधारण ाज से 225 अिधक है , तो (d) 620
(Afternoon) उसी दर पर 35500 पर 2 53 वष की
(a) 11000 साधारण ाज िकतनी होगी ? Q3. A sum amounts to Rs
(b) 12000 SSC MTS 21 August 2019 14,395.20 at 9.25% p.a. simple
(c) 14000 (Evening) interest in 5.4 years. What will be

e
(d) 15000 (a) Rs 6966.50 / 6966.50 the simple interest on the same
(b) Rs 6922.50 / 6922.50 sum at 8.6% p.a. in 4.5 years?
Q16. A sum doubles in seven (c) Rs 6953.00 / 6953.00 कोई रािश 5.4 वष म 9.25% ित
years at simple interest. In how (d) Rs 6723.50 / 6723.50 वष साधारण ाज की दर से 14,395

l
many years will the sum become
पये हो जाती है | इसी रािश पर 4.5
five times the original sum? SSC CGL Tier 2
ac वष म 8.6% ित वष की दर से
एक रािश साधारण ाज पर सात
साधारण ाज ात कर |
वष म दोगुनी हो जाती है | िकतने Q1. A sum of Rs 8400 amounts to SSC CGL Tier 2 12 September
वष म यह रािश मूल रािश से पां च Rs 11,046 at 8.75% p.a. simple 2019
गुना हो जाएगी ? interest in certain time. What is (a)Rs 3715.20
SSC MTS 9 August 2019 the simple interest on the sum of (b)Rs 3627
(Afternoon) 9600 at the same rate for the same (c)Rs 3797.76
(a) 35 time? (d)Rs 3672
(b) 21 8400 पये की एक रािश िकसी
nn
(c) 28 िनि त समय म 8.75% ित वष Q4. A sum lent out at simple
(d) 30 साधारण ाज की दर से 11,046 interest amounts to Rs. 6076 in 1
पये हो जाती है | 9600 पये की year and Rs. 7504 in 4 years. The
Q17. The simple interest on Rs x रािश पर इसी दर से इसी समय के sum and the rate of interest p.a
for m years at a rate of r% is िलए साधारण ाज ात कर | are respectively
equal to the same on Rs y for n SSC CGL Tier 2 11 September साधारण ाज पर उधार दी गयी एक
years at the rate of s%, then xy is 2019 रािश 1 वष म 6076 पये तथा 4 वष
Pi

equal to: (a)Rs 2990 म 7504 पये हो जाती है | यह रािश


x पये पर r% की दर से m वष का (b)Rs 3012 तथा ाज की दर ( ित वष ) मशः
साधारण ाज y पये पर s% की (c)Rs 2686 है -
दर से n वष के साधारण ाज के (d)Rs 3024 SSC CGL Tier 2 12 September
बराबर है , तो xy िकसके बराबर होगा 2019
? Q2. A sum of Rs. 5,000 is divided (a) Rs. 5600 and 9%
SSC MTS 16 August 2019 into two parts such that the simple (b) Rs. 5600 and 8.5%
(Morning) interest on the first part for 4 51 (c) Rs. 5400 and 9%
nr
(a) ms years at 6 32 % p.a. Is double the (d) Rs. 5400 and 10%
(b) ns simple interest on the second part
mr
(c) ms for 2 43 years at 4% p.a. What is Q5.A person invested one-fourth
nr
mr the difference between the two of the sum of Rs 25000 at a
(d) ns
parts ? 5000 पये की एक रािश दो certain rate of simple interest and
िह ों म इस कार िवभािजत की the rest at 4% p.a. higher rate. If

www.ssccglpinnacle.com support@ssccglpinnacle.com Ph. 09729327755, 09817390373


312
Days 37-39 Simple Interest

s
sse
la
_c
ob
the total interest received for 2 शत पर उधार ली िक वह इस रािश 10% ित वष की दर से िकतनी रािश
bo
ah
years is Rs 4,125. What is the rate को एक वष के बाद 8% के साधारण िनवेश की गयी थी ?
sm
ur
yo

at which the second sum was ाज के साथ वापस कर दे गा | SSC CPO 12 March 2019
e/
t.m

हालाँ िक, A इस रािश को 1 जुलाई


://

invested? (Morning)
tp
ht

एक ने 25000 पये की 2008 को ही वापस करने की थित (a) 6,200


एक-चौथाई रािश साधारण ाज की म आ गया | उसे B को िकतनी रािश (b) 6,600
िकसी िनि त दर पर तथा शेष रािश वापस करनी होगी ? (c) 6,400
4% ित वष उ दर से िनवेश की | SSC CHSL 2018 10 July 2019 (d) 6,500
यिद 2 वष के िलए ा कुल ाज (Afternoon)
4,125 पये है , तो दू सरी रािश को (a)Rs 2200 Q4. A sum of Rs 15,600 is
िकस दर पर िनवेश िकया गया था ? (b)Rs 2080 invested partly at 7% per annum
SSC CGL Tier 2 13 September (c)Rs 2088 and the remaining at 9% per
2019 (Morning) (d)Rs 2,070 annum simple interest. If the total
(a)9.5% interest at the end of 3 years is Rs
(b)9.25% Q2. A borrows a sum of Rs 3000 3,738. How much money was

e
(c)5.255 from his friend B on 31 invested at 7% per annum?
(d)7.5% December 2011 on the condition 15600 पये की एक रािश का कुछ
that he will return the same after भाग 7% ित वष तथा शेष भाग 9%
Q6. A sum of Rs10,500 amounts one year with simple interest at ित वष साधारण ाज पर िनवेश
िकया जाता है | यिद 3 वष के अंत म

l
to Rs13,825 in 3 54 years at a 15%. However, A gets into a
certain rate per cent per annum position of returning the money कुल ाज 3738 पये है , तो 7%
ac
simple interest. What will be the
simple interest on the same sum
for 5 years at double the earlier
rate? 10,500 पये की एक रािश
ित वष साधारण ाज के एक
on 31 August 2012. How much
amount he has to return to B?
A ने अपने िम B से 31 िदसंबर
2011 को 3000 पये की रािश इस
शत पर उधार ली िक वह इस रािश
ित वष की दर से िकतनी रािश
िनवेश की गयी थी ?
SSC CPO 13 March 2019
(Morning)
(a) Rs 7,800
िनि त दर ितशत पर 3 54 वष म को एक वष के बाद 15% साधारण (b) Rs 7,900
13,825 पये हो जाती है | इसी रािश ाज के साथ लौटा दे गा | हालाँ िक, A (c) Rs 7.600
पर पहली दर से दोगुनी दर पर 5 वष 31 अग 2012 को ही यह रािश (d) Rs 7,700
nn
का साधारण ाज िकतना होगा ? लौटाने की थित म आ गया | उसे B
SSC CGL Tier 2 13 September को िकतनी रािश लौटानी होगी ? Q5. A borrowed a loan from B at
2019 (Morning) SSC CHSL 2018 10 July 2019 8% simple interest for 2 years and
(a) Rs8,470 (Evening) repaid the loan with interest
(b) Rs8,750 (a)Rs 3300 totaling Rs 1,91,864. The amount
(c) Rs8,670 (b)Rs 3200 of loan taken by A is:
(d) Rs8,560 (c)Rs 3310 A ने B से 2 वष के िलए 8% साधारण
(d)Rs 3305 ाज पर ऋण िलया तथा ाज
Pi

Practice Questions सिहत इस ऋण का भुगतान


Q3. A sum of Rs. 15,000 is 1,91,864 पये िकया | A ारा िलए
Q1.A borrows a sum of Rs 2000 invested partly at 12% per annum गए ऋण की रािश थी -
from his friend B on 31 and the remaining at 10% per SSC CPO 14 March 2019
December 2007 on the condition annum simple interest. If the total (Morning)
that he will return the same after interest at the end of 2 years is Rs. (a) Rs 1,68,920
one year with simple interest at 3,344 how much money was (b) Rs 1,66,540
8%. However, A gets into a invested at 10% per annum? (c) Rs 1,64,492
position of returning the money 15000 पये की एक रािश का कुछ (d) Rs 1,65,400
on 1 July 2008. How much भाग 12% ित वष तथा शेष भाग
amount he has to return to B? 10% ित वष साधारण ाज पर Q6. A sum at a simple interest of
A ने अपने िम B से 31 िदसंबर िनवेश िकया जाता है | यिद 2 वष के 8% p.a. Becomes 57 of itself in
2007 को 2000 पये की रािश इस अंत म कुल ाज 3344 पये है , तो how many years?

www.ssccglpinnacle.com support@ssccglpinnacle.com Ph. 09729327755, 09817390373


313
Days 37-39 Simple Interest

s
se
s
la
_c
ob
8% ित वष साधारण ाज पर एक िकस समय म एक रािश 8% ित वष SSC MTS 7 August 2019
bo
ah
रािश िकतने वष म 57 गुना हो जायेगी साधारण ाज पर दोगुनी हो जाएगी (Evening)
sm
ur
yo

? ? (a) 26%
e/
t.m
://

SSC CPO 16 March 2019 SSC CPO 16 March 2019 (b) 25%
tp
ht

(Afternoon) (Morning) (c) 20%


(a) 5 (a) 8 years (d) 24%
(b) 2 21 (b) 12.5 years
(c) 3 21 (c) 6 years Q13. A sum of Rs 2000 is
(d) 5 years invested on simple interest for
(d) 2
three years at the rate of 10% per
Q10. The simple interest for 9 annum, then the amount will be:
Q7. A sum of rs 10,200 is
years on a principal is 53 of the 2000 पये की एक रािश 10%
invested partly at 8% per annum
principal. What is the rate of साधारण ाज ित वष की दर से
and remaining at 6% per annum
interest per annum? तीन वष के िलए िनवेश की जाती है |
for 3 years at simple interest. If
िकसी मूल धन पर 9 वष का साधारण िम धन होगा :
the total interest is rs 2,124, how

e
ाज मूल धन का 53 है | ित वष SSC MTS 8 August 2019
much money was invested at 6%
ाज की दर ात कर | (Morning)
per annum?
SSC MTS 2 August 2019 (a) Rs2900
10200 पये की एक रािश 3 वष के
(Evening) (b) Rs2600
िलए अंशतः 8% ित वष और शेष

l
(a) 6% (c) Rs2300
6% ित वष साधारण ाज की दर
(b) 4% (d) Rs2500
से िनवेश की जाती है | यिद कुल
ac
ाज 2124 पया है , तो 6% ित वष (c) 6 32 %
Q14. Ramesh borrowed Rs.
पर िकतनी रािश िनवेश की गयी थी ? (d) 5 32 %
12000 at 13% p.a simple interest.
SSC CPO 13 March 2019
What amount will he pay in 5
(Evening) Q11. The simple interest on a
years to clear this loan ?
(a)4900 certain sum for two years is
रमेश ने 13% ितवष साधारण ाज
(b)5200 Rs1000 at the rate of 10% per
की दर से 12000 उधार िलए |
(c)4800 annum. What is the amount after
रमेश उधार चुकता करने के िलए 5
nn
(d)5400 these two years?
वष म िकतनी रािश का भुगतान
10% ित वष की दर से िकसी िनि त
करे गा ?
Q8. 2,64,000 is invested for 3 रािश पर दो वष का साधारण ाज
SSC MTS 9 August 2019
years on an annual rate of interest 1000 पये है | इन दो वष के बाद
(Evening)
8.25%. What will be the amount िम धन ात कर |
(a) Rs18800
of interest? SSC MTS 7 August 2019
(b) Rs20000
दर 8.25 % वािषक साधारण ाज (Afternoon)
(c) Rs19800
पर 3 वष के िलए 2,64,000 का
(d) Rs18600
Pi

िनवेश िकया जाता है | ाज की रािश (a) Rs6500


ा होगी ? (b) Rs6000
Q15. A sum of Rs50,000 is lent
SSC CPO 14 March 2019 (c) Rs7000
partly at 4% and remaining at 5%
(Evening) (d) Rs5500
per annum. If the yearly simple
(a) Rs 65,340
interest on the average is 4.6%,
(b) Rs 21,780 Q12. A sum of money becomes
the two parts are:
(c) Rs 87,120 double of itself in 50 months
50000 पये की एक रािश अंशतः
(d) Rs 43,560 when invested on simple interest.
4% तथा शेष 5% ित वष की दर से
What is the rate of interest per
उधार दी जाती है | यिद औसत पर
Q 9. In what time will a sum annum?
वािषक साधारण ाज 4.6% है , तो ये
double itself at 8% p.a. Simple साधारण ाज पर िनवेश की गयी
दोनों िह े ह :
interest. कोई रािश 50 महीनों म खुद से
SSC MTS 13 August 2019
दोगुनी हो जाती है | ित वष ाज की
(Evening)
दर ात कर |

www.ssccglpinnacle.com support@ssccglpinnacle.com Ph. 09729327755, 09817390373


314
Days 37-39 Simple Interest

s
sse
la
_c
ob
(a) Rs22500, Rs27500 िम धन बनने म िकतना समय लगेगा (d) 5 years /5 वष
bo
ah
(b) Rs15000, Rs35000 ?
sm
ur
yo

(c) Rs20000, Rs30000 SSC MTS 16 August 2019 Q22. A certain sum amounts to
e/
t.m
://

(d) Rs25000, Rs25000 (Afternoon) Rs 12096 at 8% p.a. in 5 21 years


tp
ht

(a) 10 years / 10 साल at simple interest. What will be


Q16. X took a loan of Rs5000 on (b) 8 years / 8 साल the simple interest on the same
simple interest, the rate of interest (c) 9 years / 9 साल sum at 10% p.a. in 8 years?
being the same as the number of (d) 11 years / 11 साल एक िनि त रािश 8% ित वष
years for which the loan was साधारण ाज की दर से 5 21 वष म
taken. If the interest paid was Q19. The simple interest on a sum 12096 पये हो जाती है | इसी रािश
Rs1800, then what was the rate of for the five years at 7% p.a is Rs. पर 10% ित वष की दर से 8 वष
interest? 700. What is the principal ? का साधारण ाज ात कर|
X ने साधारण ाज पर 5000 पये िकसी रािश पर 7% ितवष की दर से SSC MTS 19 August 2019
का ऋण िलया, िजसम ाज की दर पां च वष के िलए साधारण ाज (Evening)
िलए गए ऋण के वष की सं ा के 700 है | तो मूलधन िकतना है ? (a) Rs6760
समान थी | यिद 1800 पये के ाज

e
SSC MTS 16 August 2019 (b) Rs6810
का भुगतान िकया गया, तो ाज की (Evening) (c) Rs6680
दर ात कर | (a) Rs 1800 / 1800 (d) Rs6720
SSC MTS 14 August 2019 (b) Rs 1600 / 1600

l
(Afternoon) (c) Rs 2000 / 2000 Q23. A person borrowed a sum at
(a) 6.5% (d) Rs 2100 / 2100 8% p.a simple interest and in 8
(b) 6%
(c) 5%
(d) 5.5%
ac
Q17. At the rate of 8% the
Q20. A sum becomes Rs 500 in 5
years and Rs600 in 7 years at a
certain rate percent p.a. at of
simple interest. What is the sum?
years, the interest earned was Rs.
5,490 less than the amount of
loan. This amount was :
िकसी ने 8% वािषक साधारण
ाज पर एक रािश उधार ली और 8
amount invested earns a simple कोई रािश ित वष साधारण ाज वष म ा ाज, उधार की रािश से
interest of Rs240 after 3 years. If की िकसी िनि त दर से 5 वष म 500 5,490 कम था | यह रािश थी :
the rate of interest been 5% more, पये तथा 7 वष म 600 पये हो
nn
SSC MTS 20 August 2019
then how much more interest जाती है | यह रािश ात कर | (Morning)
would it have earned? SSC MTS 19 August 2019 (a) Rs 15,500 / 15,500
8% की दर से िनवेश की गयी रािश (Morning) (b) Rs 15,250 / 15,250
पर 3 वष के बाद 240 पये का (a) Rs300 (c) Rs 14280 / 14,280
साधारण ाज ा होता है | यिद (b) Rs400 (d) Rs 15600 / 15,600
ाज की दर 5% अिधक होती, तो (c) Rs200
इस पर िकतना अिधक ाज ा (d) Rs250 Q24. A certain sum is lent at x%
होता ?
Pi

p.a simple interest for X3 years.


SSC MTS 14 August 2019 Q21. A sum is invested on simple
The simple interest of this sum is
(Evening) interest. If the rate of interest is
equal to one-third of this sum.
(a) Rs105 20% p.a, then in what time the
What is the value of x ?
(b) Rs180 sum will be doubled ?
एक िनि त रािश को X3 वष के िलए
(c) Rs150 िकसी रािश को साधारण ाज पर
ित वष x% साधारण ाज की दर
(d) Rs135 िनवेिशत िकया जाता है | यिद ाज
से उधार िदया जाता है | रािश का
दर 20% ितवष है , तो िकतने समय
साधारण ाज, रािश के एक ितहाई
Q18. What time will it take for a म रािश दोगुनी हो जाएगी ?
के बराबर है | x का मान िकतना है ?
principal of Rs. 640 to become an SSC MTS 19 August 2019
SSC MTS 20 August 2019
amount of Rs. 768 at 2.5% p.a (Afternoon)
(Afternoon)
simple interest ? (a) 10 years /10 वष
(a) 12
640 के मूलधन को 2.5% वािषक (b) 8 years / 8 वष
(b) 10
साधारण ाज दर से 768 का (c) 4 years / 4 वष
(c) 9

www.ssccglpinnacle.com support@ssccglpinnacle.com Ph. 09729327755, 09817390373


315
Days 37-39 Simple Interest

s
sse
la
_c
ob
(d) 6 on another sum of Rs y at 9% per SSC MTS 5 August 2019
bo
ah annum for three years, then which (Afternoon)
sm
ur
yo

Q25. What is the amount that will of the following is true? (a) 15%
e/
t.m

यिद x पये की रािश पर 6% ित


://

become Rs. 1440 at 5% simple (b) 5%


tp
ht

interest in 4 years? वष की दर से दो वष का साधारण (c) 10%


वह रािश िकतनी है जो 5% साधारण ाज y पये की रािश पर 9% ित (d) 20%
ाज की दर से 4 वष म 1440 हो वष की दर से तीन वष के साधारण
जाएगी ? ाज से दोगुना है , तो िन म से कौन Q31. At the end of 3 years, the
SSC MTS 21 August 2019 सा सही है ? simple interest on any amount of
(Afternoon) SSC MTS 22 August 2019 2800 is 420. What will be the
(a) Rs 1180 / 1180 (Evening) simple interest on 3200 for the
(b) Rs 1080 / 1080 (a) x = 2y same rate and same period?
(c) Rs 1200 / 1200 (b) x = 4.5y 3 वष के अंत म 2800 की िकसी
(d) Rs 1240 / 1240 (c) 3x = 7y रािश पर साधारण ाज 420 है | इसी
(d) 2x = 5y दर और इतनी ही अविध के िलए
3200 पर साधारण ाज िकतना

e
Q26. What is the simple interest
on Rs35000 at 18
7 % per annum Q29. At a fixed simple interest होगा ?
for a period of 9 months? rate, the amount of Rs 500 SSC MTS 13 August 2019
35000 पये की रािश पर 18 % की becomes Rs 600 in 2 years. If the (Morning)
7

l
दर से 9 माह का साधारण ाज ात rate of interest is halved, then (a) 480
कर | what will be the amount of Rs (b) 560

(Morning)
(a) Rs675
(b) Rs600
(c) Rs875
ac
SSC MTS 22 August 2019 500 in 2 years.
िनि त साधारण ाज दर पर 500
पए की रािश 2 वष म 600 पए
हो जाती है यिद ाज दर को आधा
कर िदया जाता है तो 500 पए की
(c) 440
(d) 640

Q32. Pankaj invests an amount


after dividing in three different
(d) Rs700 रािश 2 वष म िकतनी होगी ? schemes A, B and C giving the
SSC MTS 5 August 2019 interest at the rate of 10%, 12%
(Afternoon) and 15% respectively and the
nn
Q27. A sum of Rs9000 amounts
to Rs13356 at a certain rate (a) Rs 500 accumulated interest for one year
percent per annum in 4 52 years at (b) Rs 520 is Rs. 3200. The amounts invested
(c) Rs 560 in A, B and C are in the ratio of 8
simple interest. What will be the
(d) Rs 550 : 5 : 12. What amount did he
simple interest on the same sum
invest in the scheme B ?
at double the rate for 2 31 years?
Q30. A person deposits Rs. 500 पंकज िकसी रािश को िवभािजत कर
9000 पये की रािश साधारण ाज
for 2 years, Rs. 600 for 5 years तीन अलग-अलग योजनाओ A, B
की एक िनि त दर से 4 52 वष म
and Rs. 1000 for 6 years at the और C म मशः 10%, 12% और
Pi

13356 पये हो जाती है | इसी रािश same rate of simple interest if he 15% ित वष ाज दर पर िनवेश
पर दोगुनी दर से 2 31 वष का receives a simple interest of Rs. करता है , और एक वष म संिचत कुल
साधारण ाज िकतना होगा ? 1000, then the rate of interest per ाज 3200 पए है | योजना A, B
SSC MTS 22 August 2019 year is ? और C म िनवेश की गयी रािश 8 : 5 :
(Afternoon) कोई 2 वष के िलए 500 12 के अनुपात म है | योजना B म वह
(a) Rs4640 पए 5 वष के िलए 600 पए तथा िकतनी रािश का िनवेश करता है ?
(b) Rs4760 6 वष के िलए 1000 पए सभी को SSC MTS 20 August 2019
(c) Rs4260 साधारण ाज की समान दर पर (Evening)
(d) Rs4620 जमा करता है यिद वह कुल 1000 (a) Rs 5500
पए का साधारण ाज ा करता (b) Rs 4500
Q28. If the simple interest on a है , तो ित वष ाज की दर िकतनी (c) Rs 5000
sum of Rs x at 6% p.a. for two है ? (d) Rs 4000
years is double the simple interest

www.ssccglpinnacle.com support@ssccglpinnacle.com Ph. 09729327755, 09817390373


316
Days 37-39 Simple Interest

s
sse
la
_c
ob
Q33. In how many years will a (b) ₹ 7,200 sum over a period of 10 years is
bo
ah
money become triple of itself at (c) ₹ 8,420 ₹1,850, then the sum is:
sm
ur
yo

the rate of 10% per annum simple (d) ₹ 9,260 एक धन रािश पर साधारण ाज की
e/
t.m

दर पहले 4 वष के िलए 5% ित वष
://

interest?
tp
ht

10% ित वष साधारण ाज की दर Q2. The difference in the तथा अगले 3 वष के िलए 8% ित


से कोई धन िकतने वष म यं का compound interest on a certain वष एवं 7 वष के बाद की अविध के
तीन गुना हो जायेगा ? sum at 10% p.a. for one year, िलए 10% ित वष है | यिद इस रािश
SSC MTS 21 August 2019 when the interest is compounded पर 10 वष म उपािजत साधारण
(Morning) half yearly and yearly, is ₹88.80. ाज 1850 पये है , तो यह रािश
(a) 25 What is the simple interest on the िकतनी है ?
(b) 20 same sum for 1 32 years at the SSC CGL 6 March 2020
(c) 15 same rate? / (Evening)
(d) 8 िकसी िनि त रािश पर 10% ित वष (a) ₹1,650
की दर से एक वष के च वृ ाज (b) ₹1,500
Q34. A person deposits Rs 500 म 88.80 पये का अंतर आता है , (c) ₹2,750

e
for 2 years, Rs 600 for 5 years जब ाज का संयोजन अधवािषक (d) ₹2,500
and Rs 1000 for 6 years at the और वािषक िकया जाता है | इसी
same rate of simple interest. If he रािश पर इसी दर से 1 32 वष का Q5. If in 13 years a fixed sum
earns a simple interest of Rs. साधारण ाज िकतना होगा ? doubles at simple interest, what

l
1000, then what is the rate of SSC CGL 4 March 2020 will be the interest rate per year?
interest per year? (Afternoon) (correct to one decimal places)
कोई

साधारण
ac
2 वष के िलए 500
पए, 5 वष के िलए 600 पए तथा
6 वष के िलए 1000 पए सभी को
ाज की समान दर पर
जमा करती है | यिद वह कुल 1000
(a) ₹5,916
(b) ₹5,986
(c) ₹5,980
(d) ₹5,920
यिद 13 वष म साधारण ाज पर
एक िनि त रािश दोगुनी हो जाती है ,
तो ित वष ाज की दर िकतनी
होगी ? ( दशमलव के एक थान तक
) SSC CGL 7 March 2020
पए का साधारण ाज ा करता Q3. The compound interest on a (Afternoon)
है , तो ित वष ाज की दर िकतनी certain sum at 16 32 % p.a. for 3
है ? (a) 8.69%
nn
years is ₹6,350. What will be the
SSC MTS 2 August 2019 (b) 7.69%
simple interest on the same sum
(Afternoon) (c) 7.29%
at the same rate for 5 32 years? /
(a) 15% (d) 7.92%
एक िनि त रािश पर 16 32 % ित वष
(b) 5%
(c) 10% की दर से 3 वष का च वृ ाज Q6. When two equal amounts are
(d) 20% 6,350 पये है | इसी रािश पर इसी deposited for 5 years and 3 years
दर से 5 32 वष का साधारण ाज at the rate of 7% and 9% per
SSC CGL TIER I िकतना होगा ? SSC CGL 5 March annum, respectively, the
Pi

2020 (Morning) difference of their simple interest


Q1. ₹4,300 becomes ₹4,644 in 2 (a) ₹7,620 is ₹475. Then find the deposited
years at simple interest. Find the (b) ₹9,600 amount. / जब दो समान रािश
principle amount that will become (c) ₹11,400 मशः 5 और 3 साल के िलए 7%
₹10,104 in 5 years at the same (d) ₹10,200 और 9% ित वष की दर से जमा की
rate of interest. जाती है , तो उनके साधारण ाज का
4300 पये साधारण ाज पर 2 वष Q4. The rate of simple interest on अंतर 475 है । तो जमा करी गयी रािश
म 4644 पये बन जाते ह | वह मूल a sum of money is 5% p.a. for the का पता लगाएं । SSC CGL 7
धन ात कर जो ाज की इसी दर से first 4 years, 8% p.a. for the next March 2020 (Evening)
5 वष म 10,104 पये बन जाएगा ? 3 years and 10% p.a. for the (a) ₹5,837.5
SSC CGL 3 March 2020 period beyond 7 years. If the (b) ₹5,937.5
(Morning) simple interest accrued by the (c) ₹5,992.5
(a) ₹ 5,710 (d) ₹6,037.5

www.ssccglpinnacle.com support@ssccglpinnacle.com Ph. 09729327755, 09817390373


317
Days 37-39 Simple Interest

s
sse
la
_c
ob
Rs.6000 more was borrowed and
bo
ah
SSC CHSL 2019 rate of interest on the total Q6. Find a simple interest of
sm
ur
yo

Q1. In how many years shall Rs principal was doubled than that of ₹74,000 at 18 ⅔ % per annum for
e/
t.m
://

3,500 invested at the rate of 10% the previous rate. At the end of a period of 8 months?
tp
ht

simple interest per annum, the year, 2800 was paid as the 74,000 पये पर 18 ⅔ % ित वष
amount to Rs 4,500? interest. Find the rate that was की दर से 8 माह का साधारण ाज
10% ित वष साधारण ाज की दर applicable in the initial. ात कीिजए।
से िनवेश िकये गए 3,500 पये 10000 पये की रािश साधारण ाज CHSL 14-10-2020 ( Evening
िकतने वष म 4,500 पये हो जाएं गे के िकसी दर पर उधार ली गई। चार shift)
? महीनों बाद, 6000 प े और उधार (a) ₹ 9,486.32
CHSL 12-10-2020 (Morning िलए गए और कुल मूल धन पर ाज (b) ₹ 8,956.74
shift) की दर को िपछली दर के मुकाबले (c) ₹ 8,458.96
(a) 2 75 years दोगुना कर िदया गया | साल के अंत (d) ₹ 9,208.88
(b) 2 76 years म, ाज के प म 2800 का
(c) 2 74 years भुगतान िकया गया, ारं भ म लागू की Q7. In how many years and
गयी ाज दर की गणना कर।

e
months will a sum of Rs24
(d) 2 73 years
CHSL 12-10-2020 (Morning become Rs56 at 16% simple
shift) interest per annum?
Q2. In how many years will the
(a)14% िकतने वष और महीनों म, ित वष
simple interest on a sum of
16% साधारण ाज पर 24 पये की

l
(b)16%
money be equal to the principle at
(c)12% रािश 56 पये हो जाएगी?
ac
rate of 12 42 % p.a.?
एक धनरािश पर 12 42 %
दर से साधारण ाज िकतने वष म
मूल धन के बराबर हो जाएगा ?
CHSL 12-10-2020 (Afternoon
ितवष की
(d)10%

Q5. A man takes a loan of some


amount at some rate of simple
interest. After three years, the
CHSL 15-10-2020 (Afternoon
shift)
(a) 7 years 8 months/ 7 वष 8
महीने
(b) 8 years 4 months/ 8 वष 4
shift) loan amount is doubled and the महीने
(a) 7 years/ वष rate of interest is decreased by (c) 6 years 11 months/ 6 वष 11
(b) 5 years/ वष 2%. After 5 years, if the total महीने
nn
(c) 8 years/ वष interest paid on the whole is (d) 6 years 5 months/ 6 वष 5
(d) 6 years/ वष Rs13,600, which is equal to the महीने
same when the first amount was
Q3. A person deposits Rs8,000 in taken for 11 31 years, then the Q8. In how many years will a
a bank which pays 8% p.a. simple loan taken initially is: sum of Rs5,000 yield a simple
interest. The amount after 8 years एक आदमी साधारण ाज की कुछ interest of Rs2,000 at an interest
will be: दर पर कुछ रािश का ऋण लेता है । rate of 10% p.a.?
एक एक बक म 8000 पये तीन साल के बाद, ऋण की रािश 5,000 पये पर 10% की ाज दर
Pi

जमा करता है , जो 8% ित वष दोगुनी हो जाती है और ाज की दर पर िकतने वष म साधारण ाज


साधारण ाज दे ता है । 8 वष के बाद 2% कम हो जाती है । 5 साल के बाद, 2000 पये िमलेगा?
िम धन िकतना होगा? यिद पूरे पर िदया गया कुल ाज CHSL 15-10-2020 ( Evening
CHSL 12-10-2020 ( Evening 13,600 पये है , जो उसी के बराबर shift)
shift) है जब पहली रािश 11 31 वष के िलए (a) 5 years/ वष
(a) Rs 12,600
उधार ली गई थी, तो शु म िलया (b) 3 years/ वष
(b) Rs 10,784
गया ऋण है : (c) 4 years/ वष
(c) Rs 13,120
CHSL 14-10-2020 (Afternoon (d) 6 years/ वष
(d) Rs 12,545
shift)
(a) Rs 13,600 Q9. A person invested a total of
Q4. A sum of Rs. 10,000 was ₹9,000 in three parts at 3%, 4%
(b) Rs 12,500
borrowed at a rate of simple and 6% per annum on simple
(c) Rs 10,000
interest. After four months, interest. At the end of a year, he
(d) Rs 12,000

www.ssccglpinnacle.com support@ssccglpinnacle.com Ph. 09729327755, 09817390373


318
Days 37-39 Simple Interest

s
sse
la
_c
ob
received equal interest in all three years and 8 years is Rs960. The certain rate of interest p.a. The
bo
ah
cases. The amount invested at 6% principal is: time in which the same sum will
sm
ur
yo

is: एक मूल धन पर 8% ित वष की दर be 4 times at the same rate of


e/
t.m

एक साधारण ाज पर से 4 वष तथा 8 वष के साधारण


://

interest at simple interest is:


tp
ht

₹9,000 को तीन भाग म 3%, 4% ाज का जोड़ 960 पये है । मूलधन ित वष साधारण ाज की एक


और 6% की दर से िनवेश करता है l िकतना है ? िनि त दर से एक रािश 8 वष म
एक वष के अंत म, उसे तीनों मामलों CHSL 19-10-2020 (Afternoon दोगुनी हो जाती है । वह अविध ात
म समान ाज ा आ। 6% पर shift) कीिजए जब यही रािश साधारण
िनवेिशत रािश िकतनी है ? (a) Rs 1000 ाज की समान दर से 4 गुना हो
CHSL 16-10-2020 (Morning (b) Rs 900 जाएगी?
shift) (c) Rs 1100 CHSL 21-10-2020 (Afternoon
(a) ₹2,000 (d) Rs 800 shift)
(b) ₹3,000 (a) 30 years
(c) ₹4,000 Q13. A man took a loan from a (b) 25 years
(d) ₹5,000 bank at the rate of 11% p.a. (c) 24 years

e
simple interest. After three years, (d) 20 years
Q10. At which rate of simple he had to pay Rs 9,570 interest
interest does an amount become only for the period. The principle Q.16. If the total simple interest
double in 12 years? amount borrowed by him was: on a sum of Rs.1400 for 4 years
साधारण ाज की िकस दर से रािश एक ने 11% ित वष साधारण

l
at rate of interest x% p.a. and on
12 वष म दोगुनी हो जाती है ? ाज की दर से एक बक से ऋण the same sum for two years at the

shift)
(a) 7 54 %
(b) 8%
(c) 8 31 %
ac
CHSL 16-10-2020 ( Evening िलया। तीन वष बाद, उ केवल
अविध के िलए 9,570 पये का ाज
दे ना पड़ा। उसके ारा उधार ली गई
रािश थी:
CHSL 19-10-2020 ( Evening
same rate, is Rs.672, then the
value of x is:
यिद 1400 पये की रािश पर x%
ित वष साधारण ाज की दर से 4
वष तथा उसी रािश पर उसी दर से 2
(d) 7 21 % shift) वष का कुल साधारण ाज 672
(a) Rs 27,685 पये है , तो x का मान ा है ?
(b) Rs 26,545 CHSL 21-10-2020 ( Evening
nn
Q11. A person borrowed
(c) Rs 25,000 shift)
Rs.1,200 at 8% p.a. andRs1,800
(d) Rs 29,000 (a) 9%
at 10% p.a. as simple interest for
(b) 8%
the same period. He had to pay
Q.14. A sum of money amounts (c) 6%
Rs1,380 in all as interest. Find the
to Rs.7500 in 5 years, and to (d) 10%
time period.
8,500 in 7 years at simple interest
एक ने साधारण ाज की दर
at the same rate of interest. The Q.17. The difference of simple
8% ित वष पर ₹ 1,200 उधार िलया
rate of interest per annum is: interest on a sum of money for 8
Pi

और उसी अविध के िलए 10% ित


एक रािश साधारण ाज की दर से 5 years and 10 years is Rs.200. If
वष पर ₹ 1,800 उधार िलया। उसे
वष म 7500 पये और उसी दर से 7 the rate of interest is 10% per
ाज के प म कुल ₹1,380 का
वष म 8,500 पये हो जाती है । ित annum then what is the sum of
भुगतान करना था। समय अविध ात
वष ाज की दर ा है ? money?
कीिजए।
CHSL 21-10-2020 (Morning 8 साल और 10 साल के िलए रािश
CHSL 19-10-2020 (Morning
shift) पर साधारण ाज का अंतर 200
shift)
(a) 12% पये है । यिद ाज की दर 10%
(a) 4 years/ वष
(b) 9% ित वष है तो रािश िकतनी है ?
(b) 10 years/ वष
(c) 8% CHSL 17-03-2020 (Morning
(c) 6 years/ वष
(d) 10% shift)
(d) 5 years/ वष
(a) 1000
Q.15. A sum at simple interest (b) 1400
Q12. The sum of simple interest
becomes two times in 8 years at a (c) 1600
on a principal at 8% p.a. for 4

www.ssccglpinnacle.com support@ssccglpinnacle.com Ph. 09729327755, 09817390373


319
Days 37-39 Simple Interest

s
sse
la
_c
ob
(d) 1200 (a) Rs78,456.34 दी। 5 वष के अंत म, राकेश ने रािश
bo
ah (b) Rs66,477.2 को चुकता करने के िलए सुरेश को
sm
ur
yo

Q18. Latha deposited an amount (c) Rs56,896.98 रािश का 9/8 भाग लौटा िदया। ित
e/
t.m

वष साधारण ाज की दर ात
://

of Rs35,000 in a bank with (d) Rs69,345.6


tp
ht

simple interest 11% per annum. कीिजए।


How much interest will she earn Q21. A person borrows Rs7,000 CHSL 19-03-2020 ( Evening
after one year? for 3 years at 5% p.a. simple shift)
लाठा ने साधारण ाज 11% ित वष interest. He immediately lends it (a) 3% p.a.
के साथ बक म 35,000 पये की to another person at 6 31 % p.a for (b) 2% p.a.
रािश जमा की। एक वष के बाद वह 3 years. Find his gain in the (c) 3.5% p.a.
िकतना ाज कमाएगा? transaction per year. (d) 2.5% p.a.
CHSL 17-03-2020 (Afternoon एक ने 5% ित वष साधारण
shift) ाज की दर से 3 वष के िलए 7,000 SSC CGL2019 TIER-II
(a) Rs 3,370 पये उधार िलए। उसने तुरंत इन Q24. A certain sum is lent at 4%
(b) Rs 3,500 पयों को एक अ को 3 वष p.a for 3 years 8% p.a for next 4

e
(c) Rs 3,850 के िलए 6 31 % ित वष की दर से years and 12 % p.a beyond 7
(d) Rs 3,220 उधार दे िदया। ित वष लेनदे न म years. If for a period of 11 years
उसका लाभ ात कीिजए। the simple interest obtained is
Q19. A man has Rs 10,000. He CHSL 17-03-2020 ( Evening ₹27,600, then the sum is (in ₹ ):
एक िनि त रािश 3 वष के िलए 4%

l
lent a part of it at 15% simple shift)
interest and the remaining at 10% (a) Rs90 ित वष, अगले 4 वष के िलए 8%
ac
simple interest. The total interest
he received after 5 years
amounted to Rs 6,500. The
difference between the parts of
the amounts he lent is:
(b) Rs93.33
(c) Rs92
(d) Rs95.33

Q.22. In a certain time, a sum of


ित वष तथा 7 वष के बाद 12%
ित वष की दर से उधार दी जाती है ।
यिद 11 वष की अविध के िलए
साधारण ाज 27,600 पये ा
होता है , तो यह रािश ( पये म)
एक के पास 10,000 पये ह। money becomes five times itself िकतनी है ?
उसने इसके एक िह े को 15% if the rate of the interest is 16% CGL 2019 Tier-II (15-11-2020 )
साधारण ाज तथा शेष िह े को (a) 27,000
nn
p.a. Then the certain time (in
10% साधारण ाज पर उधार दे years) is: (b) 25,000
िदया। 5 वष बाद उसे कुल 6500 एक िनि त समय म, एक धनरािश (c) 30,000
पये साधारण ाज ा आ। यं से पां च गुना हो जाती है , यिद (d) 32,000
उसके ारा उधार दी गयी रािशयों का ाज की दर 16% ित वष है । वह
अंतर है : िनि त समय ( वष म) कौन सा है ? Q25. The rate of interest for the
CHSL 18-03-2020 (Morning CHSL 19-03-2020 (Morning first 2 years is 6% p.a, for next 3
shift) shift) years is 10% p.a, and for the
(a) Rs 2,000 period beyond 5 years is 12% p.a,
Pi

(a) 32
(b) Rs 2,500 (b) 38 If a person gets ₹12,771 as simple
(c) Rs 1,500 (c) 25 interest after 7 years, then how
(d) Rs 1,750 (d) 30 money did he invest ?
ाज की दर पहले 2 वष के िलए
Q20. If the present amount is Rs Q.23. Suresh lent out a sum of 6% ित वष, अगले तीन वष के िलए
87,750 with 8% rate of interest in money to Rakesh for 5 years at 10% ित वष तथा 5 वष से बाद की
four years, then what was the simple interest. At the end of 5 अविध के िलए 12% है । यिद एक
principal amount? years, Rakesh paid 9/8 of the sum को 7 वष के बाद साधारण
यिद चार वष म 8% ाज दर से to Suresh to clear out the amount. ाज के प म 12,771 पये ा
वतमान िम धन 87,750 पये है , तो Find the rate of simple interest होते ह, तो उसने िकतनी रािश िनवेश
मूलधन िकतना था? per annum. की थी?
CHSL 18-03-2020 (Afternoon सुरेश ने राकेश को साधारण ाज CGL 2019 Tier-II (16-11-2020 )
shift) पर 5 वष के िलए कुछ रािश उधार (a) ₹ 19,450

www.ssccglpinnacle.com support@ssccglpinnacle.com Ph. 09729327755, 09817390373


320
Days 37-39 Simple Interest

s
sse
la
_c
ob
(b) ₹ 19,350 (c) 9 िकया गया है िक दो साल और चार
bo
ah (d) 8 साल बाद A और B पर ितवष 15%
sm
(c) ₹ 19,300
ur
yo

(d) ₹ 20,000 की दर से साधारण ाज बराबर है ।


e/
t.m

A और B पर एक साथ कुल िकतना


://

Q29. If the annual rate of simple


tp
ht

Q26. At what rate of interest will interest increases from 11% to 17 ाज ( पये म) ा होगा:
1
a sum of ₹4,500 amount to 2 % , a person’s yearly income CPO 2019 24-11-2020
₹6,525 at simple interest for 5 increases by 1,071.20. The (Evening shift)
years?/ साधारण ाज की िकस दर principal amount invested (in Rs) (a) 5,400
से 5 वष म ₹4,500 की रािश ₹6,525 is : (b) 9,600
का िम धन दे गी? यिद साधारण ाज की वािषक दर (c) 18,000
CGL 2019 Tier-II (18-11-2020 ) 11% से बढ़कर 17 21 % हो जाती है , (d) 10,800
(a) 8% तो एक की वािषक आय
(b) 12% 1,071.20 तक बढ़ जाती है । िनवेश Q32. A person invested Rs.
(c) 10% की गई मूल रािश ( म) है ? 12,000 on simple interest for 7
(d) 9% CPO 2019 23-11-2020 years to obtain a total amount of

e
(Evening shift) Rs. 20,400 on a certain annual
Q27. In how much time will the (a) 17,250 rate of interest. What was the rate
simple interest on a certain sum (b) 19,120 of interest to obtain the above
of money be 56 (c) 10,710 amount?
एक ने 7 वष के िलए साधारण

l
Times of the sum at 20% per (d) 16,480
annum? / ाज की एक िनि त दर पर 12,000
ac
एक िनि त धनरािश पर 20% ित
वष की दर से िकतने समय म
साधारण ाज धनरािश का 56 गुना
हो जाएगा?
CGL 2019 Tier-II (18-11-2020 )
Q30. If the annual rate of simple
interest increases from 11% to 17
1
2 % , then a person’s yearly
income increases by 1071.20. The
simple interest (in Rs.) on the
पये िनवेश िकये तथा उसे 20,400
पये का िम धन
उपरो िम धन ा करने के िलए
ाज की दर िकतनी थी?
CPO 2019
ा आ।

25-11-2020
(Morning shift)
(a) 5 Years sum at 10 % for 5 years is;
(a) 8%
(b) 8 Years यिद साधारण ाज की वािषक दर
(b) 7%
nn
(c) 6 Years 11% से बढ़कर 17 21 % हो जाती है ,
(c) 9%
(d) 7 Years तो एक की वािषक आय
(d) 10%
1071.20 बढ़ जाती है । 5 वष के
SSC CPO 2019 िलए 10% की दर से उस रािश पर
Q28. Sunita invested Rs. 12,000 साधारण ाज ( म) ात करे ।
on simple interest at the rate of CPO 2019 24-11-2020
10% per annum to obtain a total (Morning shift)
amount of Rs.20,400 after a (a) 16,480
Pi

certain period. For how many (b) 9,120


years did she invest to obtain the (c) 8,240
above amount (d) 7,250
सुनीता ने 10% ित वष साधारण
ाज की दर से एक िनि त अविध Q31. A sum of Rs.27,000 is
के बाद कुल 20,400 पये का divided into two parts A and B
िम धन ा करने के िलए 12,000 such that the simple interest at the
पये िनवेश िकये। उपरो िम धन rate of 15% per annum on A and
ा करने के िलए उसने िकतने वष B after two years and four years
तक िनवेश िकया था? respectively, is equal. The total
CPO 2019 23-11-2020 interest (in Rs.) received together
(Morning shift) from A and B is:
(a) 6 27,000 पये की रािश को दो भागों
(b) 7 A और B म इस तरह िवभािजत

www.ssccglpinnacle.com support@ssccglpinnacle.com Ph. 09729327755, 09817390373


321
Days 37-39 Simple Interest

s
sse
la
_c
ob
SOLUTION 5085 According to the question
Interest earned in one year =
bo
3
ah p×k×k
sm
= 1695 p=
ur

100
yo

Variety Questions
e/

Had complete sum been invested ⇒ k = 10


t.m
://

at the rate of 15% interest earned


tp
ht

Sol 1. (d) Principle = 1000 in one year = 12800×15×1 =1920 Sol 7. (b)
100
Rate of interest = 12% Had complete sum been invested Let the principal = P
Time Period = 4 months or 1
3 at the rate of 12% interest earned According to the question
years in one year = 12800×12×1 =1536 P ×10× 1
100 100 = 100 2
Interest earned =
P rinciple × Rate × T ime P eriod ⇒ P = 2000
100 =
Alternate :
1000×12× 13
= 40 15% 12% 1
10% = 10
100
Amount returned to B = 1000+40 Let the Principle = 10 unit and
= 1040 interest earned for 1 year = 1 unit
Alternate : ⇒ Interest earned in 6 months or
3
12% = 25 1 1 1

e
2 years = 1 x 2 = 2 unit
Let the principle = 25 unit and According to the question
interest earned in one year = 3 1
2 unit = 100
unit 1 unit = 200

l
⇒ Interest earned in ⅓ years = 3 10 unit = 200 x 10 = 2000
Money invested at 15% = 53 x
× 31 = 1 unit 53+75
ac
According to the question
25 unit = 1000
1 unit = 40
Desired Amount (26 unit) = 26 x
40 = 1040
12800 = 5300

Sol 4. (a) Let the principal = 1


unit, so si earned = 1 unit.
Let the rate of interest = r
Sol 8. (b)
Let the simple interest = I
According to the question
I = 1000 100
×12.5×2 = 250

⇒ Amount = 1000 + 250 = 1250


According to question
Alternate :
Sol 2. (b) 1 = 1×r×12.5
100
5th year : 3rd year
nn
r=8% 12.5% = 1
8
Amount 5 : 4
Let the Principle = 8 unit and
Let the amount after 5th year = 5 Sol 5. (a)
interest earned for 1 year = 1 unit
unit and 3rd year = 4 unit Let the principal = P ⇒ Interest earned in 2 years = 1 x
Interest earned in 2 yars = 5-4 = 1 According to the question
2= 2 unit
unit 7200 = P ×15×3
100 ⇒ Amount after 2 years = 8+2 =
Interest earned in 1 year = 21 unit ⇒ P = 16000
1
10 unit
⇒ Interest earned in 3 year = 1 Alternate :
2 According to the question
Pi

3
15% = 20
unit 8 unit = 1000
Principle = 4-1 21 = 2 21 Let the Principle = 20 unit and 1 unit = 125
1
Desired rate of interest = 2
×100
= interest earned for 1 year = 3 unit 10 unit = 1250
5
×1
2 ⇒ Interest earned in 3 years = 3 x
20%
3 = 9 unit Sol 9. (b)
According to the question Simple Interest earned in 20
Sol 3. (d)
9 unit = 7200 months = 520-480 = 40
Let the amount invested at the
1 unit = 800 Simple interest earned in 1 month
rate of 15% = x
20 unit = 20 x 800 = 16000 =2
According to the question
x×15×3 (12800−x) ×12×3 Simple interest earned in 12
100 + 100 = 5085 Sol 6. (b) months (1 year) = 24
15x + 153600 − 12x = 169500 Let the principle = p and time = 24 x
Desired rate of interest = 480
⇒ x = 5300 rate = k 100 = 5%
Alternate : ⇒ simple interest earned = p

www.ssccglpinnacle.com support@ssccglpinnacle.com Ph. 09729327755, 09817390373


322
Days 37-39 Simple Interest

s
sse
la
_c
ob
Sol 10. (d) Interest earned = Had the remaining sum been 20 unit = 3200
bo
ah
1590-1500 = 90 invested at the rate of 6% interest 1 unit = 160
sm
ur
yo

According to the question earned in one year = 6000×6×1 3 unit = 160 x 3 = 480
e/
t.m

100
x
× 8x
://

1500 × 12
90 = =360
tp
ht

100
⇒ x2 = 576 Had the remaining sum sum been Sol 15. (c)
⇒ x = 24 invested at the rate of 10% Interest earned in (6-4) years or 2
interest earned in one year = years = 24080-20720 = 3360
6000×10×1 =600 ⇒ Interest earned in 4 years =
Sol 11. (d) 100
3360 x 2 = 6720
Interest earned = 1200-800 = 400
Desired Sum = 20720-6720 =
Let the rate of interest = r
14000
According to the question
400 = 800100
×r×8

⇒r= 25 %
4 Sol 16.(c)
800× 25
4 ×6
Desired simple interest = 100 Let the principal = 1 unit and rate

e
= 300 of interest = r
Alternate : According to the question
Interest earned in 8 years = 400 1 = 1×7×r
100
Interest earned in 1 year = 4008 = r = 14 72 %

l
50 Now,
Money invested in third scheme =
Interest earned in 6 years = 50 x 6
ac 1 Let the desired time = t
= 300 5+1 x 6000 = 1000
In this period the sum amounts to
= 5 unit
Sol 12. (c) Sol 13. (d)
Interest earned in this period =
Interest earned in 5 years = 2800 Let the rate of interest = r
5-1 = 4 unit
⇒ Interest earned in one year = According to the question 1× 100 ×t
⇒4= 7
7 = 10 ×100r × 5 100
560 ⇒ t = 28 years
Interest earned from first scheme ⇒ r = 14%
Alternate : Alternate :
nn
in one year = 4000100
× 1×4 = 160

Principal : Simple Interest When money becomes 5 times of


Interest earned from other two the principal interest earned = 4
schemes in one year = 560-160 = 10 : 7
Let the principle = 10 unit time of the principle
400 Money become double in 7 years.
Let the amount invested in second Simple interest earned in 5 years
⇒ Time taken to earn the Interest
scheme = k = 7 unit
Simple interest earned in 1 year = equal to principal = 7 years
According to the question So, time taken to earn the interest
k×6×1 (6000−k) × 10 × 1 1.4 unit
100 + 100 = 400 equal to 4 times of the principal =
Desired rate = 1.4
10 x 100 = 14 %
Pi

⇒ k = 5000 7x4 = 28 years


Money invested in third scheme =
Sol 14. (a)
6000-5000 = 1000 Sol 17. (b)
Interest earned in 3 years = 420
Alternate : According to the question
Interest earned in 1 year = 140 x×m×r = y×n×s
Interest earned in 5 years = 2800 140 x 100 = 100 100
Rate of interest = 2800
⇒ Interest earned in one year = ⇒ xy = mrns
5%
560
Now,
Interest earned from first scheme 1 Sol 18. (b)
5% =
in one year = 4000100
× 1×4 = 160 20
Let r be the rate of interest
Let the Principle = 20 unit and the
Interest earned from other two According to the question
interest earned in 1 year = 1 unit
schemes in one year = 560-160 = (28000-27000) × 100 r × 3 = 225
⇒ interest earned in 3 years = 3
400 ⇒r= 15 %
unit 2
According to the question

www.ssccglpinnacle.com support@ssccglpinnacle.com Ph. 09729327755, 09817390373


323
Days 37-39 Simple Interest

s
se
s
la
_c
ob
Desired simple interest = Let the CP =400 unit and interest Let CP = 6 unit and interest
bo
ah
35500× 15 13
2 ×5 earned in one year = 37 unit earned in one year = 1 unit
sm
= 6922.50
ur

100
yo

⇒ Interest earned in 5.4 years = Interest earned in 5 years = 5 unit


e/
t.m
://

199.8 According to the question


tp

SSC CGL Tier 2


ht

⇒ Amount after 5.4 years = 6 unit = 10500


400+199.8 = 599.8 1 unit = 1750
Sol 1. (d)
According to the question 5 unit = 8750
Interest earned on the principle of
599.8 unit = 14395.20 Alternate :
8400 = 11046-8400 = 2646
400 unit = 9600 Interest earned in 3 54 years =
Let the time = t
⇒ 2646 = 8400×8.75×t Now, 13825-10500 = 3325
100 43
8.6% = 500 Interest earned in 1 year = 875
⇒ t = 3.6
Let the CP =500 unit and interest ⇒ Interest earned in 5 year at
Desired simple interest =
9600×8.75×3.6 = 3024 earned in one year = 43 unit double the rate of interest = 875 x
100
⇒ Interest earned in 4.5 years = 5 x 2 = 8750
Alternate :
43 x 4.5 = 193.5
Rs 8400 amounts to = 11046

e
⇒ 500 unit = 9600 Practice Questions
Rs 1 amounts to = 11046
8400
11046
Desired SI = 193.50 unit = 19.2 x
Rs 9600 amounts to = 8400 x Sol 1. (b) Principle = 2000
193.50 = 3715.20
9600= 12624 Sol 4. (b) Time period = 6 months or 1 year
2

l
Desired simple interest = Interest earned in (4-1) years or 3 Rate = 8%
12624-9600 = 3024 years = 7504-6076 = 1428 2000×8× 12

Sol 2. (b)
ac
Let the amount invested at one
part = x
According to the question
⇒ Interest earned in 1 year =
1428
3 = 476
⇒ Desired Sum = 6076-476 =
5600
476 x
Simple interest =

Alternate :
2
8% = 25
100
Desired amount = 2000+80 =
2080
= 80

x× 20 21
3 ×5 (5000−x)×4× 11 Desired rate of interest = 5600 Let the principle = 25 unit and
100 = 2× 100
4

100 = 8.5% interest earned in one year = 2


420x = 1650000-330x
unit
nn
750x = 1650000 Sol 5. (b) ⇒ Interest earned in ½ years = 2
⇒ x = 2200
Let r be the rate of interest. × 21 = 1 unit
Amount invested in the second According to the question According to the question
part = 5000-2200 = 2800 6250 × r × 2 18750 × (r + 4) × 2
100 + 100 = 25 unit = 2000
Desired difference = 2800-2200 =
4125 1 unit = 80
600
12500r + 37500r + 150000 = Desired Amount (26 unit) = 26 x
412500 80 = 2080
Sol 3. (a)
⇒ r = 5.25
Pi

Let P be the principle


Desired rate of interest = 5.25+4 Sol 2. (a)
According to the question
= 9.25 % Principle = 3000
(14395.20 - P) = P ×9.25×5.4
100 2
Time period = 8 months or 3 year
⇒ P = (14395.20 - P) × 9.25100
× 5.4 Sol 6. (b) Rate = 15%
⇒ 49950P = Let r be the rate of interest 3000×15× 23
1439520000-100000P Simple interest = 100 = 300
According to the question
⇒ 149950P =1439520000 10500×r× 19
Desired amount = 3000+300 =
3325 = 100
5
3300
⇒ P = 9600
⇒r=8 1
Desired SI = 9600 ×100
8.6 × 4.5 = 3 % Alternate :
⇒ New rate of interest = 16 32 3
15% = 20
3715.20
Alternate : Now, Let the principle = 20 unit and
37
9.25% = 400 16 32 % = 1
6
interest earned in one year = 3
unit

www.ssccglpinnacle.com support@ssccglpinnacle.com Ph. 09729327755, 09817390373


324
Days 37-39 Simple Interest

s
sse
la
_c
ob
⇒ Interest earned in ⅔ years = 3 Had complete sum been invested According to the question
bo
ah
× 32 = 2 unit at the rate of 12% interest earned (10200−x) ×6×3
sm
x×8×3
+ =2124
ur

100 100
yo

in one year = 15600×7×1 =1092


e/

According to the question 8x + 61200 − 6x = 70800


t.m

100
://

20 unit = 3000 Had complete sum been invested


tp

⇒ x = 4800
ht

1 unit = 150 at the rate of 10% interest earned ⇒ the amount invested at the
Desired Amount (22 unit) = 22 x in one year = 15600×9×1
100 =1404 rate of 6% = 10200-4800 = 5400
150 = 3300 Alternate :
Interest earned in one year = 2124
3
Sol 3. (c)
= 708
Let the amount invested at the
Had complete sum been invested
rate of 12% = x
at the rate of 12% interest earned
According to the question
(15000−x) ×10×2
in one year = 10200×8×1
100 =816
x×12×2
100 + 100 =3344
Had complete sum been invested
24x + 300000 − 20x = 334400 at the rate of 10% interest earned
⇒ x = 8600 in one year = 10200×6×1 =612

e
100
Amount invested at 10% = Money invested at 7 % =
79
15000-8600 = 6400 156 × 15600 = Rs. 7900
Alternate :
Interest earned in one year = 3344 Sol 5. (d)

l
2
= 1672 Let the Principle = 100
ac
Had complete sum been invested
at the rate of 12% interest earned
in one year = 15000×12×1
100 =1800
Had complete sum been invested
at the rate of 10% interest earned
Time period = 2 year
Rate = 8%
Simple interest = 100×8×2
100 = 16
Desired amount = 100+16 = 116
According to the question
in one year = 15000×10×1
100 =1500 116 unit = 1,91,864 Money invested at 6% =
1 unit = 1654 9 = 5400
10200 × 17
100 unit = 165400
nn
Alternate :
2
Sol 8. (a)
8% = 25 33
8.25 % = 400
Let the principle = 25 unit and
Let the CP = 400 unit and interest
interest earned in one year = 2
earned in 1 year = 33 unit
unit ⇒ Interest earned in 3 years = 33
⇒ Interest earned in 2 years = 2
x 3 = 99
× 2 = 4 unit
Therefore, Amount invested at According to the question
According to the question
Pi

32 = Rs. 6400
10% = 15000 × 75 400 unit = 26400
29 unit = 1,91,864 1 unit = 660
1 unit = 6616 99 unit = 660 x 99 = 65340
Sol 4. (b)
Desired Amount (25 unit) = 25 x
Let the amount invested at the
6616 = 165400 Sol 9. (b)
rate of 7% = x
According to the question Let the principal = 1 unit, so si
(15600−x) ×9×3
Sol 6. (a) earned = 1 unit.
x×7×3
100 + 100 =3738 Let CP = 5 unit Let the time period = t
7x + 140400 − 9x = 124600 Amount = 7 unit According to question
⇒ x = 7900 Interest = 7-5 = 2 unit 1 = 1×8×t
100
Alternate : Required time = 2×100
8×5 = 5 years
3738 t = 12.5 years
Interest earned in one year = 3 Alternate :
= 1246 Sol 7 (d) Let the amount invested 8 % = 252
at the rate of 8 % = x
Let the principal = 25 unit

www.ssccglpinnacle.com support@ssccglpinnacle.com Ph. 09729327755, 09817390373


325
Days 37-39 Simple Interest

s
sse
la
_c
ob
⇒ the interest earned in one year 1 unit = 200
bo
ah 13 unit= 2600 Sol 16. (b)
sm
= 2 unit
ur
yo

For the sum to be double the Let the rate of interest = time
e/
t.m
://

interest earned will be equal to Sol 14. (c) period = k


tp
ht

13% = 10013 According to the question


the principal.
⇒ 25 unit interest will be earned Let the CP = 100 unit and interest 1800 = 5000×k×k
100
in = 25
2 = 12.5 years earned in one year = 13 unit ⇒k=6
⇒ Interest earned in 5 years = 65
Sol 10. (c) unit Sol 17. (c)
Amount after 5 years = 100+65 = 8% = 252
Let the principal = 5 unit and
interest = 3 unit 165 unit Let the CP = 25 unit and interest
Let r be the rate of interest According to the question earned in one year = 2 unit
According to the question 100 unit = 12000 ⇒ Interest earned in 3 years = 6
3 = 5×r×9
100
1 unit = 120 unit
⇒ r = 6 32 % 165 unit= 19800 According to the question

e
6 unit = 240
Sol 11. (b) Sol 15. (c) 1 unit = 40
10% = 10 1 Let the amount invested at the 25 unit = 25 x 40 = 1000
rate of 4 % = x Now, New rate of interest = 8+5
Let the CP = 10 unit and interest

l
According to the question = 13%
earned in one year = 1 unit x×4×1 (50000−x) ×5×1 50000×4.6×1
+ = 13% = 10013

unit
ac
⇒ Interest earned in 2 years = 2

Amount after 2 years = 10+2 = 12


unit
According to the question
100

⇒ x = 20000
100
4x + 250000 − 5x = 230000

⇒ remaining part = 50000-20000


= 30000
100
Let the CP = 100 unit and interest
earned in one year = 13 unit
⇒ Interest earned in 3 years = 39
unit
2 unit = 1000 Alternate : 100 unit = 1000
1 unit = 500 Interest earned in one year = 1 unit = 10
50000×4.6×1 = 2300 39 unit = 390
12 unit= 6000 100
nn
Had complete sum been invested Extra interest earned = 390 - 240
Sol 12. (d) at the rate of 4% interest earned = 150
Let the principal = 1 unit, so si in one year = 50000×4×1 =2000 Alternate :
100
earned = 1 unit. Had complete sum been invested Extra interest earned at 5% in 3
Let the rate = r at the rate of 5% interest earned years = 5 x 3 = 15 unit
Time period = 50 months or 4 61 in one year = 50000×5×1 =2500 Now, (8% x 3 ) of Principal = 240
100
years (1% x 3 ) of Principal= 30
According to question ⇒ (5% x 3 ) of Principal= 150
Pi

1× 25
6 ×r
1= 100 Sol 18. (b)
r = 24% Total Interest earned = 768-640 =
128
Sol 13. (b) Let the time period = t
10% = 10 1
128 = 640×2.5×t
100
Let the CP = 10 unit and interest ⇒ t = 8 years
earned in one year = 1 unit
Alternate :
⇒ Interest earned in 3 years = 3 1
2.5% = 40
unit
Money invested at 4% = Let the principal = 40 unit and
Amount after 3 years = 10+3 = 13
2 = 20000
50000 × 2+3 interest earned per year = 1 unit
unit
Money invested at 5% = According to the question
According to the question
3 = 30000
50000 × 2+3 40 unit = 640
10 unit = 2000

www.ssccglpinnacle.com support@ssccglpinnacle.com Ph. 09729327755, 09817390373


326
Days 37-39 Simple Interest

s
sse
la
_c
ob
1 unit = 16 36 unit = 12096 350 unit = 35000
bo
ah
⇒ 128 = 8 unit 1 unit = 336 1 unit = 100
sm
ur
yo

25 unit = 336 x 25 = 8400 27 unit = 27 x 100 = 675


So time taken for the interest to
e/
t.m

4 4
://

become 8 unit = 1 x 8 = 8 years Now,


tp
ht

10% = 10 1
Sol 27. (d)
Sol 19. (c) Let the principal = 10 unit and Interest earned on the principle of
Let Principle = p interest earned per year = 1 unit 9000 = 13356-9000 = 4356
According to the question ⇒ interest earned in 8 years = 1 x Let the rate of interest = r
p×7×5 9000× 22
700 = 8= 8 unit 5 ×r
100 ⇒ 4356 = 100
⇒ p = 2000 10 unit = 8400 ⇒ r = 11
Alternate : 1 unit = 840
New rate of interest = 11 x 2 = 22
(7% x 5 ) of Principal = 700 8 unit = 6720
%
Principal= 700
35 x 100 = 2000 Desired simple interest =
Sol 23. (b) 9000×22× 73
8% = 252 100 = 4620
Sol 20. (d)

e
Let the principal = 25 unit and Alternate :
Interest earned in (7-5) years or 2 22
interest earned per year = 2 unit Interest earned In 5 years =
years = 600-500 = 100
⇒ Interest earned in 1 year = 100 ⇒ interest earned in 8 years = 2 x 4356
2 5
8= 16 unit In 1 years =4356x 22
= 50

l
In 7 years at double the rate
⇒ Interest earned in 5 year = 50 According to the question 3
ac (25-16) unit = 5490 percent = 4356 x 5 × 37 x 2=
x 5 = 250 22
⇒ Desired Sum = 500-250 = 250 1 unit = 610 4620
25 unit = 25 x 610 = 15250
Sol 21. (d) Sol 28. (b)
Let the principal = 1 unit, so si Sol 24. (b) According to the question
earned = 1 unit. Let the principal = 3 unit and x×6×2 =2x y×9×3
100 100
Let Time period = t simple interest = 1 unit
⇒ x = 4.5y
According to question According to the question
nn
3×x× x
1 = 1×20×t 1 = 100 3 Sol 29. (d)
100
t = 5 years ⇒ x = 10 Interest earned in two years =
Alternate : 600-500 = Rs 100
20% = 51 Sol 25. (c) ⇒ Interest earned in two years
5% = 201
Let the principal = 5 unit and with half the rate of interest =
Let the principal = 20 unit and 100 = Rs 50
interest earned per year = 1 unit 2
For the amount being double the interest earned per year = 1 unit Desired amount after 2 years =
principal simple interest earned = ⇒ interest earned in 4 years = 1 x 500+50 = 550
Pi

principal = 5 unit 4= 4 unit


So time taken for the sum to According to the question Sol 30. (c)
become double = 1 x 5 = 5 years (20+4) unit = 1440 Let the rate of interest = r%
1 unit = 60 According to the question
Sol 22. (d) 20 unit = 20 x 60 = 1200 500×2×r +
100
600×5×r + 1000×6×r =
100 100
8% = 252
1000
Let the principal = 25 unit and Sol 26. (a) 100r = 1000
18 % = 9
interest earned per year = 2 unit 7 350 ⇒ r = 10%
⇒ interest earned in 11/2 years = Let the principal = 350 unit and
2 x 11
2 = 11 unit
interest earned per year = 9 unit Sol 31. (a)
Amount after 11/2 years = 25+11 ⇒ interest earned in 9/12 years = Let the rate of interest = r%
9 = 27 unit
9 x 12
= 36 unit 4 According to the question
2800×3×r = 420
According to the question According to the question 100

www.ssccglpinnacle.com support@ssccglpinnacle.com Ph. 09729327755, 09817390373


327
Days 37-39 Simple Interest

s
sse
la
_c
ob
⇒ r = 5% P = ₹ 8,420 Therefore, 74% of P = 1,850
bo
ah 3200×3×5 P = ₹ 2500
sm
Desired SI = = 480
ur

100
yo

Sol 2. (d) Rate of interest is


e/
t.m
://

10%p.a. Sol 5. (b) SI = P ×r×t


Sol 32. (c)
tp

100
ht

Let the amount invested in A,B For half yearly, Rate= 5+5+ 5×5
100 In 13 years, sum doubles.
and C are 8k, 5k and 12k. =10.25% Therefore, SI in 13 years is equal
According to the question Difference in interest due to .25% to P.
8k×1×10 +
100
5k×1×12 + 12k×1×15 =
100 100
= ₹88.80 P = P ×r×13
100
3200 Then 1% = ₹355.20 r= 100 9 % = 7.69%
= 7 13
13
32k = 32000 So, principal = ₹35520
35520×10× 53
⇒ k = 1000 SI = = ₹5,920 Sol 6. (b) SI = P ×r×t
100 100
Amount invested in scheme B = 5 Let P amount by deposited at 7%
x 1000 = 5000 Sol 3. (d) Rate of interest = 16 and 9% 5 years and 3 years
2 1
3 % p.a. = 6 respectively.
Sol 33. (b) This question can be solved by ×7×5 - P ×9×3
475 = P 100

e
100
10% = 10 1
either of two methods: 475 = P ×8
100
Let the CP = 10 unit and interest Method 1: P = ₹ 5937.50
earned in one year = 1 unit Rate = 61 = a1 and time = 3 years.
For a sum to become three times Let Principal = (a)3= (6)3 = 216 SSC CHSL 2019

l
the simple interest earned is equal units Sol:1.(b)
ac
to the twice of the principal.
⇒ Simple interest earned = 20
unit
Time taken for SI to become 20
unit = 1 x 20 = 20 years
Interest = 4500-3500 = 1000
1000 = 3500×10×T
T= 20 %
7

Sol:2.(c)
100
= 2 76 years

12 42 % = 1
Sol 34. (c) 8

The rate of interest = r % Interest = 1 unit


nn
According to the question Principle = 8 unit
127 units = ₹ 6350
1000 = 500×2×r + 600×5×r + To obtain 1 unit interest, it takes 1
100 100 1 unit = ₹ 50
1000×6×r year ,
100 Principal = ₹ 216 × 50
Therefore, To obtain 8 unit
1000 = 10r + 30r + 60r Simple Interest = 216×50×17×1
3×6 =₹
interest (equal to principle), it
⇒ r = 10% 10200 will take,
8 × 1 = 8 years
Method 2:
SSC CGL TIER I For simple interest, time given =
Pi

17
Sol:3.(c)
3 Simple interest = P ×R×T
P ×R×T 100
Sol 1. (c) Simple Interest= 100 Check the option which is 8000×8×8
= 100 = 5120
For ₹4300 to become multiple of 17 and mark the
Amount = P + SI = 8000+5120 =
₹4644 in 2 years, interest gained answer.
Rs 13120
is ₹344 and rate of interest will be
344 = 4300×R×2
100 Sol 4. (d) We know that simple
Sol:4.(d)
R = 4% per annum interest remains the same if the
Let rate = r%
Now, the sum that will become rate does not change for each 12000×r×4 + 18000×2r×8
100×12 100×12 = 2800
₹10,104 in 5 years at the same year.
r = 10%
rate can be calculated using the SI
formula. According to question:
Here interest rate = rate × time = Sol:5. (c)
P(1+ 4×5
100 ) = 10,104
5% × 4+ 8% × 3+10% × 3 = 74% Let Principle = P and rate = R,
P( 56 ) = 10,104

www.ssccglpinnacle.com support@ssccglpinnacle.com Ph. 09729327755, 09817390373


328
Days 37-39 Simple Interest

s
sse
la
_c
ob
P×R× 34 = 13600 Sol:13. (d) X × 15 × 5 (10000 − X) × 10 × 5
+ =
bo
3×100 100 100
ah Let principle = P
sm
PR = 120000 6500
ur
yo

P × R% × 3 + 2P × (R-2)% × 5 = 9570 = P ×11×3


e/

⇒ X ×100
15 × 5 + 10000 × 10 × 5 -
t.m

100
100
://

13600 × 100 P = 29,000


tp

X × 10 × 5 = 6500
ht

100
3600+12000-20P = 13600 × 100 ⇒ X100× 25 + 10000 × 10 × 5 = 6500
100
15600-20P = 13600 × 100 Sol:14. (d)
⇒ X4 + 5000= 6500
20P = 2000 × 100 Interest in 2 years = 8500 - 7500
= Rs.1000 ⇒ X4 = 6500 - 5000 = 1500
P = 10000
Interest in 1 year = Rs.500 ⇒ X = 6000
Sol:6. (d) Interest in 5 years = Rs.2500 10000 - X = 4000
Simple interest = 74000×56×8 = So, sum of money = 7500 - 2500 Required difference = 6000 -
100×3×12
= Rs.5000 4000 = Rs. 2000
9208.88
Now, the rate of interest =
500/5000 × 100 = 10% Sol:20. (c)
Sol:7. (b)
Selling price of dozen pair of
interest = 56-24 = 32

e
gloves = 600 × 10 9 = 540
32 = 24×16×T Sol:15.(c)
100
In Rs. 540, 12 pairs of gloves can
T = 8 years 4 months
be bought.
Thus, in Rs. 270, 6 pairs of
Sol:8.(c)

l
If SI of 1 unit takes 8 years, gloves will be bought.
2000 = 5000×10×T
100
ac Then SI of 3 units will take = 3 Sol 21. (b)
T = 4 years × 8 = 24 years Total gain in 3 years = Interest
earned - Interest paid =
Sol:9. (a) Sol:16. (b) 7000 × 19 × 3 - 7000 × 5 × 3 = 7000
100 × 3 100 100
Rate ratio = 3 : 4 : 6 Total simple interest for 6 years at 19 × 3 7000
×( - 5 × 3) = ×
Interest ratio = 31 : 41 : 1
6 =4:3: x% = Rs.672
3 100
(19 - 5 × 3 ) = 70 × (19 - 15) =
2 Interest for 1 year = 672/6 = 70 × 4 = 280
4x+3x+2x = 9000 Rs.112
Interest per year = 280
3 = Rs.
x = 1000 112 × 100 = 8%
nn
Rate, x = 1400
Amount at 6% = 2x = 2 × 1000 = 93.33
2000 Sol:17. (a)
Sol 22. (c)
Let the principal = Rs. P
Sol:10. (c) Let sum = Rs. S
Rate = 10%
Let, P = 1 unit, Interest = 1 unit If in certain time, the amount
According to question:-
1 = 1×12×R becomes five times of itself; it
100 10 × 10 - P × 10 × 8 = 200
⇒ P × 100 100 means interest earned is four
R = 8 31 % ⇒ P × 10 × 2 = 200
100 times the sum.
Pi

⇒ P = 1000 Interest = 4S
Sol:11. (d)
4S = S ×10016 × t
Let the time = T years Sol 18. (c) ⇒ t = 25 years
12 × 8 × T + 18 × 10 × T = 1380 Interest earned = 35000 × 11 × 1 =
100
96T+180T = 1380
Rs. 3850 Sol: 23. (d)
276T = 1380
Let sum lent by Suresh = Rs. S
T = 5 years
Sol 19. (a) Time, t = 5 years
Man has Rs. 10,000. Amount paid by Rakesh = 98S
Sol:12. (a)
Let the man lent Rs. X at 15% Amount = Sum + Interest
Let the sum = P
interest and (10000-X) at 10% 9S = S + S×r×t
P × 8 × 4+P × 8 × 8 = 960 × 100 8 100
interest rate. S S×r×5
32P+64P = 960 × 100 8 = 100
Total interest received after 5 r×5 = 1
P = 1000 years = Rs. 6500 100 8
r = 20 8 = 2.5% p.a.

www.ssccglpinnacle.com support@ssccglpinnacle.com Ph. 09729327755, 09817390373


329
Days 37-39 Simple Interest

s
sse
la
_c
ob
At 10% for 5 year = 50%
bo
ah
SSC CGL2019 TIER-II 50% of 16,480 is 8,240
sm
ur
yo

Sol:24.(c)
e/
t.m
://

Interest for 3 years = 12% Sol:31.(d)


tp
ht

Interest for next 4 years = 32% sum=27000(given)


Interest for next 4 years = 48% Let A amount is invested for 2
Total interest = 92% years and B is invested for 4 year
92% of principle= 27600 Rate of interest=15%
P = 30000 Simple interest received is same
P ×R×T
As we know SI= 100

Sol:25.(b) A×2×15 B×4×15


100 = 100
Interest for first 2 years = 12%
From the above equation we get
Interest for next 3 years = 30%
A:B=2:1
Interest for next 2 years = 24%
A+B=27000
Total interest = 66%
So A=18000, B=9000

e
P = 12771 × 10066 = 19350
Interest received on A =
18000×15×2 =5400
100
Sol:26.(d)
And interest on A= interest
Interest = 6525 - 4500 = 2025
received on B(given)

l
interest ×100
Rate = principal × time =
So total interest received= 5400
2025×100 = 9%
4500×5

Sol:27.(c)
ac
Interest = 1.2x
Rate = 20%
Time = 1.2x ×100
× 2 =10800

Sol:32.(d)
Total amount invested=12000
Time period=7years
20× x = 6 years Amount received=20400
Interest = 8400
SSC CPO 2019 Let rate of interest = R%
nn
Sol:28.(b) Now applying formula of S.I=
Principal amount= 12000 P ×R×T
100
Total amount received after a 8400= 12400×R×7
100
certain period=20400 R=10%
Net interest received = 8400
Now applying formula of S.I=
P ×R×T
100
8400= 12400×10×T
Pi

100
Solving it we get T=7 years

Sol:29.(d)
Rate increased by = 17.5% - 11%
= 6.5%
Amount increased = 1071.20
Principle = 100×1071.2
6.5×1 = 16,480

Sol:30.(c)
Rate increased by = 17.5% - 11%
= 6.5%
Amount increased = 1071.20
Principle = 100×1071.2
6.5×1 = 16,480

www.ssccglpinnacle.com support@ssccglpinnacle.com Ph. 09729327755, 09817390373


330
Days 40-42 Compound Interest

s
sse
la
_c
ob
COMPOUND INTEREST Amount = 100×110×110×110 कोई रािश ाज की वािषक
bo
100×100×100
ah = 133.10; CI = Amount – च वृ पर ित वष एक िनि त दर
sm

/ च वृ ाज
ur
yo

Principal = 133.10 – 100 = 33.10 से 3 वष म 8028 पये तथा 6 वष


e/
t.m

Or, Amount = Principal म 12042 पये हो जाती है | यह


://
tp
ht

r t 10 3
Key-Points:/ मुख िबंदु: (1 + 100 ) = 100 (1 + 100 ) रािश है :
= 11 × 11 × 11 = 133.10;
100 × 10 SSC CGL 4 June 2019
10 10
1. For one year or one term simple CI = 33.10 (Morning)
interest and compound interest are (a) Rs 5,352
same. So all the above concepts have the (b) Rs 5,235
एक वष या एक अविध के िलए same result, but if you want to (c) Rs 5,325
साधारण ाज और च वृ ाज solve questions quickly and (d) Rs 5,253
समान होता ह। without using formulas then it is
advised to develop mastery on Q2. The compound interest on a
approximation technique. certain sum in 2 21 years at 10%
2. In simple interest you divide इसिलए सभी उपरो अवधारणाओं
the simple interest by number of p.a., interest compounded yearly,
का एक ही प रणाम है लेिकन अगर

e
years to calculate one year interest is Rs1,623. The sum is :
आप सू ों का इ ेमाल न करते ए िकसी िनि त रािश पर 10% ित वष
but in compound interest this is शी ता से हल करना चाहते ह तो
not applicable. वािषक च वृ की दर से 2 21 वष
उसे सि कटन तकनीक पर द ता
साधारण ाज म आप एक वष की का च वृ ाज 1623 पये है |
िवकिसत करने की सलाह दी जाती

l
ाज की गणना के िलए सालों की यह रािश है :
है ।
सं ा से साधारण ाज को िवभािजत
ac SSC CGL 4 June 2019
करते ह, लेिकन च वृ ाज म (Afternoon)
यह लागू नहीं होता है । Note: Notice that in CI, interest is (a) Rs 5,000
calculated on the amount at the
(b) Rs 6,000
end of every year while in SI
3. Compound Interest is the (c) Rs 6,500
interest for each year is calculated
interest on amount (principal + (d) Rs 7,200
on the Principal amount.
Interest); Symbol we will use
‘CI’.
4. If rate of interest is different for Q3. What will be the compound
च वृ ाज रािश (मूलधन +
nn
different year, let us say 10% for interest (nearest to Rs 1) on a
ाज) पर ाज होता है |
1st year, 20% for 2nd year, and 30% sum of Rs25,000 for 2 years at
for 3rd year then amount will be 12% p.a., if the interest is
Let us say one person borrows calculated as follows: compounded 8-monthly?
Rs.100 for 3 years at the rate of अगर ाज की दर अलग वष के िलए 25000 पये पर 12% ित वष की
10% annually. What amount अलग है , जैसे,1 साल के िलए 10%, दर से 2 वष का च वृ ाज ा
should he have to return after 3 दू सरे वष के िलए 20%, और तीसरे होगा यिद ाज की च वृ 8 माह
years? वष के िलए 30%, तो रािश की गणना
की है ?
मान लेते है िक एक ने सालाना िन ानुसार की जाएगी:
Pi

SSC CGL 4 June 2019


10% की दर से 3 साल के िलए 100
(Evening)
पये का उधार लेता है | 3 सालों के
(a) Rs 6,394
बाद उसे िकतनी रािश लौटानी होगी?
We can also do it as: Amount = (b) Rs 6.439
Here is the approximation 100×110×120×130 = 171.60 Rs. (c) Rs 6,493
technique (सि कटन तकनीक) to 100×100×100
(d) Rs 6.349
solve this problem:
Variety Questions
Q4. A sum of Rs15,000 is lent at
16% p.a. compound interest.
Q1. A sum amounts to Rs 8,028 in
What is the difference between
Compound interest will be = 3 years and to Rs 12,042 in 6
the compound interest for the
133.10 – 100 = 33.1 Rs. years at a certain rate percent per
second year and the third year?
The above concept can also be annum, when the interest is
written as: 15000 पये की रािश को 16% ित
compounded yearly. The sum is:
वष च वृ ाज पर उधार िदया

www.ssccglpinnacle.com support@ssccglpinnacle.com Ph. 09729327755, 09817390373


331
/
Days 40-42 Compound Interest

s
sse
la
_c
ob
जाता है | दू सरे वष और तीसरे वष के (b) 65,600 ाज 2940 पये है | इसी रािश पर
bo
ah
च वृ ाज म अंतर ात कर | (c) 64,800 इसी दर से 2 21 वष का च वृ
sm
ur
yo

SSC CGL 6 June 2019 (d) 65,400 ाज ात कर जब ाज की


e/
t.m
://

(Morning) च वृ वािषक है | ( एक पये के


tp
ht

(a) Rs 544 Q7. A sum of Rs 12,000 amounts िनकटतम )


(b) Rs 445.44 to Rs 20,736 in 3 years at a SSC CHSL 1 July 2019
(c) Rs 454.88 certain rate percent per annum, (Evening)
(d) Rs 548 interest compounded annually. (a)Rs 2,272
What will the amount of the (b)Rs 2,227
Q5. A person borrowed a certain same sum be in 2 years at the (c)Rs 2,327
sum at 10% p.a. for three years, same rate on compound interest? (d)Rs 2,372
interest being compounded 12000 पये की रािश ित वष एक
annually. At the end of two years, िनि त दर से 3 वष म 20,736 पये Q10.What is the difference
he repaid a sum of Rs. 6634 and हो जाती है िजसम ाज की between the compound interest,
at the end of the third year, he च वृ वािषक है | यह रािश when interest is compounded
च वृ ाज की इसी दर से दो

e
cleared off the debt by paying 5-monthly, and the simple
Rs. 13,200. What was the sum वष म िकतनी हो जाएगी ? interest on a sum of Rs 12,000
borrowed by him ? SSC CGL 10 June 2019 for 1 41 years at 12% per annum?
एक ने कोई िनि त रािश तीन (Morning) 12000 पये की रािश पर 12% की
वष के िलए 10% ित वष की दर से

l
(a) Rs15,640 दर से 1 41 वष के च वृ ाज (
उधार ली िजसम ाज की च वृ (b) Rs17,820
ac 5 माह की च वृ ) तथा साधारण
वािषक है | दो वष के अंत म, उसने (c) Rs17,280
ाज म अंतर ात कर |
6634 पये की रािश का भुगतान (d) Rs14,520
SSC CHSL 2 July 2019
कर िदया और तीसरे वष के अंत म
(Morning)
उसने 13,200 पये का भुगतान Q8. The difference between the
(a)Rs 90
करके ऋण चुकता कर िदया | उसके compound interest and simple
(b)Rs 91.50
ारा उधार ली गयी रािश िकतनी थी interest on Rs x at 8% per annum
(c)Rs 93
? for 2 years is Rs19.20. What is
(d)Rs 92.50
SSC CGL 6 June 2019 the value of x?
nn
(Evening) 8% ित वष की दर से x पये पर 2
Q11. Rs 60000 invested at a
(a) Rs. 16,400 वष के च वृ ाज और
certain rate for a certain even
(b) Rs. 15,400 साधारण ाज म 19.20 पये का
number of years, compounded
(c) Rs. 15,600 अंतर है | x का मान ा है ?
annually, grows to Rs 63,654. To
(d) Rs. 16,500 SSC CGL 10 June 2019
how much amount would it grow
(Afternoon)
if it is invested at the same rate
Q6. A sum of Rs x was borrowed (a) 2,500
for half period?
and paid back in two equal yearly (b) 3,200
Pi

एक िनि त दर पर सम सं ा के
instalments, each of Rs35,280. If (c) 2,800
वष के िलए िनवेश िकये गए 60000
the rate of interest was 5%, (d) 3,000
पये वािषक च वृ पर बढ़ कर
compounded annually, then the
63,654 पये हो जाते ह | यिद यह
value of x is: Q9.The simple interest on a
रािश आधी अविध के िलए िनवेश की
x पये की एक रािश उधार ली गयी certain sum for 3 21 years at 10%
जाए, तो यह बढ़ कर िकतनी हो
और इसका भुगतान दो बराबर per annum is Rs 2,940. What will जाएगी ?
वािषक िक ों म िकया गया िजसम be the compound interest on the SSC CHSL 4 July 2019
ेक िक़ 35,280 पये की थी | same sum for 2 21 years at the (Afternoon)
यिद वािषक च वृ ाज की दर same rate when interest is (a)Rs 61800
5% है , तो x का मान ात कर | compounded yearly (nearest to a (b)Rs 61809
SSC CGL 7 June 2019 rupee)? (c)Rs 61675
(Afternoon) िकसी िनि त रािश पर 10% ित वष (d)Rs 61827
(a) 64,400 की दर से 3 21 वष का साधारण

www.ssccglpinnacle.com support@ssccglpinnacle.com Ph. 09729327755, 09817390373


332
/
Days 40-42 Compound Interest

s
sse
la
_c
ob
Q12. The interest on Rs 24,000 (d) Rs750 (a)Rs 7800
bo
ah
in years compounded annually (b)Rs 7500
sm
ur
yo

when the rates are 8% p.a and Q15. A sum invested at 8% p.a. (c)Rs 8000
e/
t.m
://

10% p.a for two successive years amounts to Rs 20280 at the end (d)Rs 8500
tp
ht

is: of one year, when the interest is


24000 पये पर 8% ित वष तथा compounded half yearly. What Q18. The annual interest
10% ित वष की दर से वािषक will be the simple interest on the compounded on any amount is
च वृ पर दो लगातार वष का same sum for 4 53 years at double Rs 1,320 for the second year and
ाज ात कर | the earlier rate of interest? Rs 1,452 for the third year. What
SSC CPO 16 March 2019 8% ित वष की दर से िनवेश की will be the original amount at the
(Afternoon) गयी कोई रािश पहले वष के अंत म beginning of the first year?
(a) Rs 3994 20280 पये हो जाती है , जब ाज िकसी रािश पर वािषक संयोिजत
(b) Rs 4512 की च वृ अधवािषक है | इस च वृ ाज दू सरे वष के िलए Rs
(c) Rs 5040 रािश पर पहले की तुलना म ाज 1,320 है और तीसरे वष के िलए Rs
(d) Rs 5866 की दोगुनी दर से 4 53 वष का 1,452 है | पहले वष की शु आत म
मूल रािश िकतनी होगी ?

e
साधारण ाज ा होगा ?
Q13. A sum of money becomes 3 SSC CHSL 4 July 2019 SSC CPO 15 March 2019
times in 10 years at the rate of (Morning) (Evening)
compound interest (compounded (a)Rs 13500 (a)12,650

l
annually), In how many years (b)Rs 13800 (b)13,200
will it become243 times? (c)Rs 14200 (c)12,970
कोई रािश च वृ
च वृ
ac
म 243 गुना हो जाएगी ?
SSC MTS 2 August 2019
ाज ( वािषक
) की दर से 10 वष म
ितगुनी हो जाती है | यह िकतने वष
(d)Rs 14500

Q16.The difference between the


compound interest and the simple
interest on a sum at 10% p.a. for
(d)12,000

Q19. If the rate of interest is 20%


per annum, compounded yearly
and the interest on a certain sum
(Evening) three years is Rs 155. the sum (in in the second year is Rs250, then
(a) 40 years Rs) is: what will be the interest on the
(b) 50 years िकसी रािश पर 10% ित वष की दर same sum in the fifth year ?
nn
(c) 35 years से तीन वष के च वृ ाज और यिद ाज की दर ित वष 20% है
(d) 30 years साधारण ाज म 155 पये का तथा च वृ वािषक है और िकसी
अंतर है | यह रािश ( पये म ) है : िनि त रािश पर दू सरे वष का ाज
Q14. The simple interest on a SSC CHSL 11 July 2019 250 पये है , तो पां चव वष म इस
certain sum at 20% p.a for two (Afternoon) रािश पर ाज िकतना होगा ?
years is Rs. 250. What is the (a)5500 SSC MTS 20 August 2019
compound interest ( compounded (b)6000 (Morning)
annually ) on the same sum at (a) Rs518
Pi

(c)6600
the same rate for the same period (d)5000 (b) Rs400
? (c) Rs360
िकसी िनि त रािश पर 20% ितवष Q17.The difference between (d) Rs432
की ाज दर से दो वष का साधारण compound interest and simple
ाज 250 है | समान ाज दर पर interest on Rs x at 6.5% per Q20. If the compound interest in
समान अविध के िलए समान रािश annum for 2 years is Rs 33.80. the third year at 8% p.a. on a
पर च वृ ाज (वािषक प से What is the value of x? certain sum is Rs3600, then what
संयोिजत) िकतनी है ? x पये पर 6.5% ित वष की दर से is the difference between the
SSC MTS 5 August 2019 2 वष के च वृ ाज और compound interest in the 4th and
(Evening) साधारण ाज म 33.80 पये का 5th year? (nearest to an integer in
(a) Rs275 अंतर है | x का मान ात कर | Rs)
(b) Rs900 SSC CHSL 11 July 2019 यिद िकसी रािश पर 8% ित वष की
(c) Rs550 (Evening) दर से तीसरे वष का च वृ ाज

www.ssccglpinnacle.com support@ssccglpinnacle.com Ph. 09729327755, 09817390373


333
/
Days 40-42 Compound Interest

s
sse
la
_c
ob
3600 पये है , तो चौथे और पां चव 7200 पये की रािश पर 20% ित Q6. A certain sum amounts to
bo
ah
वष म च वृ ाज के बीच अंतर वष की दर से 2 52 वष का च वृ Rs4,205.55 at 15% p.a. In 2 52
sm
ur
yo

ात कर | ( िकसी पूणाक के ाज ात कर, िजसकी च वृ years, interest compounded


e/
t.m

िनकटतम पये म ) SSC MTS 22


://

वािषक है | ( एक पूणाक के yearly. The sum is:


tp
ht

August 2019 (Evening) िनकटतम ) एक िनि त रािश 15% ित वष की


(a) Rs304 SSC CGL Tier 2 12 September दर से 2 52 वष म 4205.55 पये हो
(b) Rs335 2019 जाती है , जब ाज की च वृ
(c) Rs288 (a) Rs4,290 वािषक है | यह रािश है :
(d) Rs311 (b) Rs3,960 SSC CGL Tier 2 13 September
(c) Rs4,205 2019
SSC CGL TIER II (d) Rs3,997 (a) Rs3,200
(b) Rs3,500
Q1. A sum of Rs 18000 is lent at Q4. A loan has to be returned in (c) Rs2,700
10% p.a. compound interest, two equal yearly instalments (d) Rs3,000
compounded annually. What is each of Rs44,100. If the rate of

e
the difference between the interest is 5% p.a., compounded Practice Questions
compound interest for 3rd year annually, then the total interest
and 4th year? paid is: Q1. What is the compound
18000 पये की रािश 10% ित वष िकसी ऋण को 44,100 पये की दो interest on a sum of Rs10,000 at
वािषक च वृ ाज पर उधार दी

l
बराबर वािषक िक ों म चुकता 14% p.a. for 2 75 years where the
जाती है | तीसरे और चौथे वष के करना है | यिद वािषक च वृ म
ac interest is compounded yearly?
च वृ ाज म अंतर ात कर | ाज की दर 5% ित वष है , तो (nearest to Rs1)
SSC CGL Tier 2 11 September कुल िकतने ाज का भुगतान िकया 10000 पये की रािश पर 14% ित
2019 गया ? वष की दर से 2 75 वष का च वृ
(a)Rs 220.60 SSC CGL Tier 2 12 September
ाज ात कर, जब ाज की
(b)Rs 217.80 2019
च वृ वािषक है |
(c)Rs 221.80 (a) Rs5,840
SSC CGL 6 June 2019
(d)Rs 215.40 (b) Rs6,000
(Afternoon)
(c) Rs6,200
nn
(a) Rs 4,259
Q2.What will be the compound (d) Rs6,280
(b) Rs 4,296
interest on a sum of Rs 31,250
(c) Rs 4,439
for 2 years at 12% p.a., if the Q5. A certain loan was returned
(d) Rs 4,394
interest is compounded in two equal half yearly
8-monthly? instalments each of Rs6,760. If
Q2. A sum amounts to Rs 18,600
31,250 पये की एक रािश पर 12% the rate of interest was 8% p.a.,
after 3 years and to Rs 27,900
ित वष की दर से 2 वष का compounded yearly, how much
after 6 years, at a certain rate
च वृ ाज ात कर, यिद ाज was the interest paid on the loan?
Pi

percent p.a., when the interest is


की च वृ 8 माह की है | एक िनि त ऋण को 6,760 पये की
compounded annually. The sum
SSC CGL Tier 2 11 September दो बराबर अधवािषक िक ों म
is:
2019 वापस िकया गया | यिद वािषक
एक रािश ित वष िकसी िनि त दर
(a)Rs 8106 च वृ ाज की दर 8% ित वष
से 3 वष म 18,600 पये तथा 6 वष
(b)Rs 8116 है , तो ऋण पर िकतने ाज का
म 27,900 पये हो जाती है , जब
(c)Rs 8016 भुगतान िकया गया ?
ाज की च वृ वािषक है | यह
(d)Rs 8156 SSC CGL Tier 2 13 September
रािश ात कर |
2019
SSC CGL 7 June 2019
Q3. What is the compound (a) Rs750
(Morning)
interest on a sum of Rs7200 for (b) Rs810
(a) Rs11,800
2 52 years at 20%p.a., interest (c) Rs790
(b) Rs12,400
compounded yearly (nearest to (d) Rs770
(c) Rs14,400
an integer)?
(d) Rs14,600

www.ssccglpinnacle.com support@ssccglpinnacle.com Ph. 09729327755, 09817390373


334
/
Days 40-42 Compound Interest

s
sse
la
_c
ob
Q6. The difference between the interest on Rs x at 8% per annum
bo
ah
Q3. What is the compound compound interest and simple for 2 years is Rs 48. What is the
sm
ur
yo

interest on a sum of Rs 8,100 for interest on Rs x at 7.5% per value of x?


e/
t.m

x पये पर 8% ित वष की दर से 2
://

1 41 years at 8% per annum, if the annum for 2 years is Rs 45. What


tp
ht

interest is compounded is the value of x? वष के च वृ ाज और


5-monthly? (Nearest to Rs1) x पये पर 7.5% ित वष की दर से साधारण ाज म 48 पये का अंतर
8100 पये की रािश पर 8% ित 2 वष के च वृ ाज और है | x का मान ात कर |
वष की दर से 1 41 वष का च वृ साधारण ाज म 45 पये का अंतर SSC CGL 12 June 2019
ाज ात कर, यिद ाज की है | x का मान ात कर | (Afternoon)
च वृ 5 माह की है | ( 1 पये के SSC CGL 11 June 2019 (a) 8000
िनकटतम ) (Afternoon) (b) 7500
SSC CGL 7 June 2019 (a) 7,000 (c) 7400
(Evening) (b) 8,000 (d) 7800
(a) Rs837 (c) 9,000
(b) Rs873 (d) 10,000 Q10. The difference between the

e
(c) Rs842 compound interest and simple
(d) Rs824 Q7. The difference between the interest on Rs x at 7% per annum
compound interest and simple for 2 years is Rs24.50. What is
Q4. The difference between the interest on Rs x at 12% per the value of x?
x पये पर 7% ित वष की दर से 2

l
compound interest and simple annum for 2 years is Rs 18.
interest on Rsx at 9% per annum What is the value of x? वष के च वृ ाज और
ac
for 2 years is Rs20.25. What is
the value of x?
x पये पर 9% ित वष की दर से 2
वष के च वृ
साधारण
ाज और
ाज म 20.25 पये का
x पये पर 12% ित वष की दर से
2 वष के च वृ

है | x का मान ात कर |
SSC CGL 11 June 2019
ाज और
साधारण ाज म 18 पये का अंतर
साधारण ाज म 24.50 पये का
अंतर है | x का मान ा है ?
SSC CGL 12 June 2019
(Evening)
(a) 5,400
अंतर है | x का मान ा है ? (Evening) (b) 4,800
SSC CGL 10 June 2019 (a) 1,250 (c) 5,000
(b) 1,280 (d) 6,000
nn
(Evening)
(a) 2,800 (c) 1,340
(b) 2,400 (d) 1,300 Q11. The difference between the
(c) 2,200 compound interest and simple
(d) 2,500 Q8. The difference between the interest on Rs x at 11% per
compound interest and simple annum for 2 years is Rs 60.50.
Q5. The difference between the interest on Rsx at 12% per What is the value of x?
compound interest and simple annum for 2 years is Rs43.20. x पये पर 11% ित वष की दर से
What is the value of x? 2 वष के च वृ ाज और
Pi

interest on Rs x at 8.5% per


annum for 2 years is Rs28.90. x पये पर 12% ित वष की दर से साधारण ाज म 60.50 पये का
The value of x is: 2 वष के च वृ ाज और अंतर है | x का मान ा है ?
x पये पर 8.5% ित वष की दर से साधारण ाज म 43.20 पये का SSC CGL 13 June 2019
2 वष के च वृ ाज और अंतर है | x का मान ात कर | (Morning)
साधारण ाज म 28.90 पये का SSC CGL 12 June 2019 (a) 4800
अंतर है | x का मान है : (Morning) (b) 4000
SSC CGL 11 June 2019 (a) 2,400 (c) 5000
(Morning) (b) 2,800 (d) 4500
(a) 3500 (c) 3,000
(b) 3800 (d) 2,500 Q12. The difference between
(c) 4000 compound interest and simple
(d) 4500 Q9. The difference between interest on Rs x at 15% per
compound interest and simple

www.ssccglpinnacle.com support@ssccglpinnacle.com Ph. 09729327755, 09817390373


335
/
Days 40-42 Compound Interest

s
sse
la
_c
ob
annum for 2 years is Rs9. What (b)Rs 8000
bo
ah
is the value of x? Q15. What is the compound (c)Rs 7600
sm
ur
yo

x पये पर 15% ित वष की दर से interest on a sum of Rs4,096 at (d)Rs 8400


e/
t.m

2 वष के च वृ ाज और
://

15% p.a. for 2 21 years. If the


tp
ht

साधारण ाज म 9 पये का अंतर interest is compounded Q18. The compound interest on a


है | x का मान ात कर | 10-monthly? certain sum for 3 years at 15%
SSC CGL 13 June 2019 4,096 पये पर 15% ित वष की p.a.,interest compounded yearly,
(Afternoon) दर से 2 21 वष का च वृ ाज is Rs 4167. What is the simple
(a) 600 ात कर, यिद ाज की च वृ interest on the same sum in 4 54
(b) 400 10 माह की है | SSC CHSL 2 July years at the same rate?
(c) 450 2019 (Evening) िकसी िनि त रािश पर 15% ित वष
(d) 500 (a) Rs1,726 की दर से 3 वष का च वृ ाज (
(b) Rs1,736 वािषक च वृ ) 4167 पये है |
Q13. If a sum amounts to (c) Rs1,636 इस रािश पर इसी दर से 4 54 वष
Rs2,190 in four years and Rs (d) Rs1,763 का साधारण ाज ात कर |

e
2,409 in five years at compound SSC CHSL 3 July 2019
interest, when the interest is Q16. A sum of Rs 10000 amounts (Evening)
compounded yearly, then the to Rs 11664 in 2 years, at a certain (a)Rs 6144
annual rate of interest is: rate percent per annum, when the (b)Rs 6000
यिद कोई रािश च वृ ाज पर

l
interest is compounded yearly. (c)Rs 4800
चार वष म 2190 पये तथा पां च What will be the simple interest (d)Rs 5760

ाज की च वृ
ac
वष म 2409 पये हो जाती है , जब

SSC CGL 13 June 2019


(Evening)
वािषक है , तो
ाज की वािषक दर ात कर |
on the same sum for 5 52 years at
the same rate?
10000 पये की एक रािश ित वष
वािषक च वृ ाज की एक
िनि त दर से 2 वष म 11664 पये हो
Q19. A certain amount invested
at a certain rate, compounded
annually, grows to an amount in
five years, which is a factor of
(a) 8% 1.1881 more than to what it
जाती है | इसी रािश पर इसी दर से 5
(b) 10% would have grown in three years.
5 वष का साधारण ाज ात कर |
2
(c) 9%
nn
SSC CHSL 3 July 2019 What is the rate percentage?
(d) 11% एक िनि त रािश वािषक च वृ
(Morning)
(a)Rs 4320 की िकसी िनि त दर पर िनवेश की
Q14. A sum of Rs 7,500 amounts जाती है जो पां च वष म बढ़ कर
(b)Rs 4160
to Rs 8,748 after 2 years at a उतनी हो जाती है जो तीन वष म
(c)Rs 3840
certain compound interest rate इसम ई वृ से 1.1881 अिधक का
(d)Rs 4040
per annum. What will be the एक गुणक है | दर ितशत ात कर
simple interest on the same sum |
Q17. A certain sum amounts to
for 4 53 years at double the earlier
Pi

Rs 29282 in 4 years at 10% per SSC CHSL 4 July 2019


interest rate? (Evening)
annum, when the interest is
7500 पये की रािश ित वष (a)9
compounded annually. What is
च वृ ाज की एक िनि त दर (b)8.1
the simple interest on the same
से 2 वष म 8,748 पये हो जाती है | (c)8
sum for same time at the same
इसी रािश पर पहले की तुलना म (d)9.2
rate?
दोगुनी दर से 4 53 वष का साधारण
िनि त रािश 10% ित वष की दर से
ाज िकतना होगा ? 4 वष म 29282 पये हो जाती है , Q20. A certain amount invested
SSC CHSL 2 July 2019 जब ाज की च वृ वािषक है | at a certain rate, compounded
(Afternoon) इसी रािश पर इसी दर से इतने ही annually, grows to an amount in
(a) Rs4,140 समय का साधारण ाज ात कर | five years, which is a factor of
(b) Rs5,520 SSC CHSL 3 July 2019 1.191016 more than to what it
(c) Rs8,180 (Afternoon) would have grown in two years.
(d) Rs2,760 (a)Rs. 8500 What is the rate percentage?

www.ssccglpinnacle.com support@ssccglpinnacle.com Ph. 09729327755, 09817390373


336
/
Days 40-42 Compound Interest

s
sse
la
_c
ob
एक िनि त रािश वािषक च वृ Q23. A sum of Rs 18,000 is Q26. A borrowed rs 28,500 at
bo
ah
की िकसी िनि त दर पर िनवेश की invested for 16 months at 8% per 8% pa interest compounded
sm
ur
yo

जाती है जो पां च वष म बढ़ कर annum compounded half-yearly. annually. If rs 5,780 was paid at


e/
t.m

उतनी हो जाती है जो दो वष म
://

What is the percentage gain at the end of first year, then the
tp
ht

इसम ई वृ से 1.191016 अिधक 18000 पये की रािश 8% ित वष outstanding amount at the end of
का एक गुणक है | दर ितशत ात अधवािषक च वृ पर 16 महीनों second year is:
कर | के िलए िनवेश की जाती है | ितशत A ने 8% ित वष वािषक च वृ
SSC CHSL 5 July 2019 लाभ ात कर | पर 28,500 पये उधार िलए | यिद
(Morning) SSC CPO 12 March 2019 पहले वष के अंत म 5,780 पये का
(a)5 (Evening) भुगतान िकया गया, तो दू सरे वष के
(b)4 (a) 9% अंत म बकाया रािश ा थी ?
(c)6 (b) 11% SSC CPO 14 March 2019
(d)8 (c) 10% (Morning)
(d) 12% (a)27,000.00
Q21.The compound interest on a (b)27,462.40

e
certain sum of money at 21% for Q24. A sum of Rs. 20,000 is (c)30,780.00
2 years is Rs 11,602.5 . Its simple invested for 15 months at the (d)33,242.40
interest (in Rs) at the same rate interest of 10% per annum
and for the same period is: compounded half yearly. What is Q27. If a sum becomes Rs. 1,460
िकसी िनि त रािश पर 21% की दर

l
the percentage gain, correct to in two years and Rs. 1606 in
से 2 वष का च वृ ाज one decimal place, at the end of three years due to the compound
ac
11,602.5 पये है | इस दर से इसी
अविध के िलए इसका साधारण
ाज ( पये म ) ात कर | SSC
CHSL 9 July 2019 (Afternoon)
(a)10,750
15 months?
20,000 पये की एक रािश 15 माह
के िलए 10% ित वष अधवािषक
च वृ पर िनवेश की जाती है | 15
माह के अंत म एक दशमलव थान
interest, then annual rate of
interest is:
यिद कोई रािश च वृ ाज की
वजह से दो वष म 1460 पये तथा
तीन वष म 1606 पये बन जाती है ,
(b)16,000 तक लाभ ितशत ात कर | तो ाज की वािषक दर ा है ?
(c)12,500 SSC CPO 12 March 2019 SSC CPO 16 March 2019
(d)10,500 (Morning) (Evening)
nn
(a) 12.5% (a)10%
Q22.A certain sum invested on (b) 13.6% (b)9%
compound interest grows Rs (c) 13.0% (c)8%
8000 and Rs 27,000 in three and (d) 13.4% (d)11%
six years, respectively when the
interest is compounded annually. Q25. A sum of Rs 12,000 is Q28. Rs 2,40,000 is taken as loan
What is the percentage rate of invested for 15 months at 10% for three years compounded
interest? per annum compounded half annually at 12.5%p.a. At the end
Pi

च वृ ाज पर िनवेश की गयी yearly. What is the percentage of first year, the interest is
एक िनि त रािश तीन और छः वष gain, at revised to 12% p.a. The total
म मशः 8000 पये और 27,000 12000 पये की एक रािश 15 माह amount to be repaid at the end of
पये बढ़ जाती है जब ाज की के िलए 10% ित वष अधवािषक third year is: 2,40,000 पये की
च वृ वािषक है | ाज का दर च वृ पर िनवेश की जाती है | रािश तीन वष के िलए 12.5% ित
ितशत ा है ? ितशत लाभ ात कर | वष वािषक च वृ पर उधार ली
SSC CHSL 9 July 2019 SSC CPO 13 March 2019 जाती है | पहले वष के अंत म ाज
(Evening) (Morning) म संशोधन करके 12% कर िदया
(a)25 (a) 13.0% जाता है | तीन वष के अंत म कुल
(b)0.5 (b) 13.1% िकतनी रािश का भुगतान करना
(c)50 (c) 12.8% होगा ? SSC CPO 15 March
(d)10 (d) 12.9% 2019 (Morning)
(a) Rs 3,26,400

www.ssccglpinnacle.com support@ssccglpinnacle.com Ph. 09729327755, 09817390373


337
/
Days 40-42 Compound Interest

s
sse
la
_c
ob
(b) Rs 3,34,800 Q32. What will be the compound Q35. The compound interest on a
bo
ah
(c) Rs 3,38,688 interest on a sum of Rs 1200 for certain sum of money at 11% for
sm
ur
yo

(d) Rs 3,42,648 2 years at the rate of 20% per 2 years is Rs. 6963. Its simple
e/
t.m
://

annum when the interest is interest (in Rs) at the same rate
tp
ht

Q29. On what amount, the compounded yearly? and for the same period is :
interest compounded 1200 पये पर 20% ित वष की दर िकसी िनि त रािश पर 11% की दर
semi-annually at 12% per annum से 2 वष का च वृ ाज ात कर से 2 वष का च वृ ाज 6963
for one year is 1545? िकस रािश जब ाज की च वृ वािषक है | पये है | समान दर से समान अविध
पर, एक वष के िलए 12 % ित वष SSC MTS 2 August 2019 के िलए इसका साधारण ाज
की दर से अधवािषक संयोिजत (Morning) िकतना होगा ?
च व ाज 1545 है ? (a) Rs624 SSC CHSL 8 July 2019
SSC CPO 14 March 2019 (b) Rs504 (Afternoon)
(Evening) (c) Rs576 (a) 6500
(a) Rs 12,500 (d) Rs528 (b) 6600
(b) Rs 25,750 (c) 6750

e
(c) Rs 24,300 Q33. A sum invested at compound (d) 6000
(d) Rs 12,875 interest (compounded annual)
amounts to Rs750 at the end of Q36 The compound interest on a
Q30. Find the compound interest first year and Rs900 at the end of certain sum of money at 21% for

l
at the rate of 7% p.a. second year. What is the sum ? 2 years is Rs. 9,282. Its simple
compounded annually for two च वृ ाज ( वािषक च वृ ) interest (in Rs) at the same rate
ac
years on the principal that yields
a simple interest of Rs 9450 for 3
years at 7% p.a. यिद िकसी रािश
पर 7% वािषक दर से 3 वष के िलए
साधारण ाज Rs 9450 है तो 7%
पर िनवेश की गयी कोई रािश पहले
वष के अंत म 750 पये और दू सरे
वष के अंत म 900 पये हो जाती है |
यह रािश िकतनी है ?
SSC MTS 5 August 2019
and for the same period is :
िकसी िनि त रािश पर 21% की दर
से 2 वष का च वृ ाज 9282
पये है | समान दर से समान अविध
के िलए इसका साधारण ाज ात
वािषक दर से दो वष के िलए उस (Morning) कर |
रािश का वािषक संयोिजत च वृ (a) Rs700 SSC CHSL 8 July 2019
ाज िकतना होगा ? (b) Rs625 (Evening)
nn
SSC CPO 15 March 2019 (c) Rs600 (a) 8750
(Evening) (d) Rs650 (b) 8400
(a)12,345.20 (c) 8000
(b)6,520.50 Q34. A sum of Rs. 2000 is (d) 8500
(c)10,127 invested at compound interest (
(d)12,678.40 compounded annually ). Find the Q37 The compound interest on a
amount after 30 months, if the certain sum of money at 21% for
Q31. If the compound interest at rate of interest is 10% p.a. 2 years is Rs. 6,961.5. Its simple
Pi

10% p.a. Compounded Rs 2000 की रािश को च वृ interest (in Rs) at the same rate
1
half-yearly for 1 2 years is Rs. ाज (वािषक प से संयोिजत) पर and for the same period is :
2,522. The principal amount is िनवेश िकया जाता है | यिद ाज की िकसी िनि त रािश पर 21% की दर
यिद 10% ित वष च वृ ाज दर 10% ितवष है , तो 30 माह के से 2 वष का च वृ ाज 6961.5
अधवािषक प से 1 2 वष के िलए
1 बाद िम धन िकतना होगा ? पये है | समान दर से तथा समान
2,522 पये है तो मूलधन ा होगा SSC MTS 5 August 2019 अविध के िलए इसका साधारण
? SSC CPO 16 March 2019 (Evening) ाज ात कर |
(Morning) (a) Rs 2538 SSC CHSL 9 July 2019
(a) Rs.18500 (b) Rs 2524 (Morning)
(b) Rs. 20000 (c) Rs 2541 (a) 6300
(c) Rs.15400 (d) Rs 2532 (b) 6500
(d) Rs. 16000 (c) 6000
(d) 6750

www.ssccglpinnacle.com support@ssccglpinnacle.com Ph. 09729327755, 09817390373


338
/
Days 40-42 Compound Interest

s
sse
la
_c
ob
(a) Rs1210
bo
ah
Q38. The difference between Q41. If Rs. 12000 is invested at (b) Rs2566
sm
ur
yo

simple interest and compound 20% p.a compound interest ( (c) Rs1800
e/
t.m
://

interest for 2 years on a principal compounded annually ) for 2 (d) Rs2000


tp
ht

at the rate of 5% per annum is years, then calculate the interest.


Rs. 25. Find the principal. यिद 12000 को 20% ित वष की Q44. A sum of Rs 900 is
िकसी मूलधन पर 5% वािषक ाज दर से च वृ ाज (वािषक प invested at compound interest
की दर से 2 वष के साधारण ाज से संयोिजत) पर 2 वष के िलए िनवेश (compounded annually) for 2
तथा च वृ ाज का अंतर 25 िकया गया है , तो ाज िकतनी है ? years. If the rate of interest is
पये है | मूलधन िकतना है ? SSC MTS 6 August 2019 10% per annum, then what will
SSC MTS 6 August 2019 (Afternoon) be the amount?
(Morning) (a) Rs5280 900 पये की रािश 2 वष के िलए
(a) Rs10000 (b) Rs4280 च वृ ाज ( वािषक प से
(b) Rs15000 (c) Rs4800 संयोिजत ) पर िनवेश की जाती है |
(c) Rs12000 (d) Rs5640 यिद ाज की दर 10% ित वष है ,
तो िम धन िकतना होगा ?

e
(d) Rs5000
Q42. A sum of Rs2400 becomes SSC MTS 7 August 2019
Q39. A sum of Rs 10,000 is Rs3600 in 6 years at a certain (Afternoon)
invested for 17 months at 8% per rate of compound interest (a) Rs1071

l
annum compounded half yearly. (compounded annually), What (b) Rs1089
What is the percentage gain at will be the amount after 12 years (c) Rs1289
ac
the end of 17 month, nearest to
one decimal place? 10,000 पये
की एक रािश को 17 माह के िलए
8% ित वष अधवािषक च वृ
पर िनवेश िकया जाता है | 17 माह के
at the same rate of interest?
2400 पये की रािश च वृ
( वािषक प से संयोिजत ) की
िकसी िनि त दर से 6 वष म 3600
ाज

पये हो जाती है | इसी ाज दर से


(d) Rs1121

Q45. The compound interest


(compounded annually) on a sum
of money invested for two years
अंत म लाभ ितशत ात कर | 12 वष बाद िम धन िकतना होगा ? is Rs10125. If the rate of interest
SSC CPO 13 March 2019 SSC MTS 6 August 2019 is 25% per annum, then what is
(Evening) (Evening) the amount after these two years?
nn
(a) 12.0% (a) Rs 6000 दो वष के िलए िनवेश की गयी िकसी
(b) 12.2% (b) Rs 4800 रािश पर च वृ ाज ( वािषक
(c) 12.4% (c) Rs 5400 प से संयोिजत ) 10125 पये है |
(d) 11.8% (d) Rs 4500 यिद ाज की दर 25% ित वष है ,
तो इन दो वष के बाद िम धन ात
Q40. A sum of Rs. 3000 is Q43. A certain sum invested on कर |
invested at 20% p.a compound compound interest (compounded SSC MTS 7 August 2019
interest ( compounded annually ). annually) grows to Rs5040 in (Evening)
Pi

What is the compound interest three years. If the rate of interest (a) Rs28125
for two years ? is 20% for the first year, 40% for (b) Rs32275
3000 की रािश को 20% ितवष the second and 50% for the third (c) Rs30625
के च वृ ाज दर (वािषक प year, then what is the sum? (d) Rs26275
से संयोिजत) पर िनवेश िकया गया है च वृ ाज ( वािषक प से
| 2 वष का च वृ ाज िकतना है संयोिजत ) पर िनवेश की गयी एक Q46. A sum of Rs1000 is
? िनि त रािश तीन वष म 5040 पये invested on compound interest
SSC MTS 6 August 2019 बढ़ जाती है | यिद ाज की दर (compounded annually) for three
(Morning) पहले वष के िलए 20%, दू सरे वष के years. If the rate of interest is
(a) Rs 1360 / 1360 िलए 40% और तीसरे वष के िलए 10% per annum for the first two
(b) Rs 1200 / 1200 50% है , तो यह रािश ात कर | years and 50% per annum for the
(c) Rs 1320 / 1320 SSC MTS 7 August 2019 third year, then what will be the
(d) Rs 1440 / 1440 (Morning) interest ?

www.ssccglpinnacle.com support@ssccglpinnacle.com Ph. 09729327755, 09817390373


339
/
Days 40-42 Compound Interest

s
sse
la
_c
ob
1000 पये की रािश तीन वष के Q49. The compound interest for (d) 11%
bo
ah
िलए च वृ ाज ( वािषक प two years at 12% per annum is
sm
ur
yo

से संयोिजत ) पर िनवेश की गयी है | Rs477. What is the Principal Q52. What will be the compound
e/
t.m

यिद ाज की दर पहले दो वष के
://

amount (in Rs) invested? interest for 3 years on Rs. 5120 at


tp
ht

िलए 10% ित वष और तीसरे वष 12% ित वष की दर से दो वष का the rate of 12.5% ( compounded


के िलए 50% ित वष है ,तो ाज च वृ ाज 477 पये है | िनवेश annually ) ?
िकतना होगा ? िकया गया मूल धन ( पये म ) ात 5120 की रािश पर 12.5% की दर
SSC MTS 8 August 2019 कर | से 3 वष का च वृ ाज (वािषक
(Morning) SSC MTS 9 August 2019 प से संयोिजत) िकतना होगा ?
(a) Rs612 (Morning) SSC MTS 13 August 2019
(b) Rs655 (a) 1875 (Morning)
(c) Rs815 (b) 1500 (a) Rs2280
(d) Rs756 (c) 2000 (b) Rs1960
(d) 1650 (c) Rs2120
Q47. Rs20000 is invested on (d) Rs2170

e
compound interest (compounded Q50. The simple interest on a
half yearly) at the rate of 20% sum of money for 2 years at Q53. A sum doubles in 4 years at
per annum, then what will be the certain rate of interest is Rs320. a certain rate of compound
interest after two years? The compound interest, interest. In how many years does
20000 पये की रािश 20% ित वष

l
compounded annually on the it amount to 8 times itself at the
की दर से च वृ ाज ( same sum for the same duration same rate?
ac
अधवािषक प से संयोिजत ) पर
िनवेश की गयी है | दो वष के बाद
ाज िकतना होगा ?
SSC MTS 8 August 2019
(Afternoon)
and at the same rate of interest is
Rs384. The sum (in Rs) is:
िकसी रािश पर िकसी िनि त दर से
दो वष का साधारण ाज 320 पये
है | इसी रािश पर इसी दर से इस
कोई रािश च वृ ाज की एक
िनि त दर से 4 वष म दोगुनी हो
जाती है | इसी दर से यह िकतने वष
म खुद से 8 गुना हो जाएगी ?
SSC MTS 13 August 2019
(a) Rs8800 अविध का च वृ ाज ( वािषक (Afternoon)
(b) Rs8824 प से संयोिजत ) 384 पये है | यह (a) 9
(c) Rs9282 रािश ( पये म ) है : (b) 12
nn
(d) Rs9428 SSC MTS 9 August 2019 (c) 15
(Afternoon) (d) 6
Q48. A sum of Rs1200 is (a) 400
invested at compound interest (b) 250 Q54. A man invested a sum of
(compounded half yearly). If the (c) 200 money at compound interest. It
rate of interest is 10% per (d) 309 amounted to Rs12100 in two
annum, then what will be the years and to Rs13310 in three
amount after 18 months? Q51. At what rate percent per years. The rate of interest per
Pi

1200 पये की रािश च वृ ाज annum, a sum of Rs. 6000 will annum is:
( अधवािषक प से संयोिजत ) पर become Rs. 7986 in 3 years, if एक ने कोई रािश च वृ
िनवेश की गयी है | यिद ाज की दर the interest is compounded ाज पर िनवेश की | यह दो वष म
10% ित वष है , तो 18 माह के बाद annually ? 12100 पये हो गयी और तीन वष
िम धन िकतना होगा ? 6000 की रािश 3 वष म िकतने म 13310 पये हो गयी | ित वष
SSC MTS 8 August 2019 ितशत ितवष पर 7986 हो ाज की दर है :
(Evening) जाएगी, यिद ाज वािषक प से SSC MTS 13 August 2019
(a) Rs1389.15 संयोिजत की जाती है ? (Evening)
(b) Rs1185.45 SSC MTS 9 August 2019 (a) 11%
(c) Rs1563.25 (Evening) (b) 9.5%
(d) Rs1295.35 (a) 10% (c) 12.5%
(b) 8% (d) 10%
(c) 12.5%

www.ssccglpinnacle.com support@ssccglpinnacle.com Ph. 09729327755, 09817390373


340
/
Days 40-42 Compound Interest

s
sse
la
_c
ob
Q55. If the rate of compound यिद िकसी रािश पर 5% ित वष की interest on Rs. 20000 for 3 years
bo
ah
interest, compounded half yearly, दर से दो वष के साधारण और at the same rate ?
sm
ur
yo

is 20% per annum, then calculate च वृ ाज म 125 पये का कोई मूलधन साधारण ाज दर पर
e/
t.m

अंतर है , तो यह रािश ( पये म ) है : पां च वष म 50% बढ़ जाता है |


://

the interest to be paid on Rs.


tp
ht

100000 for 2 years. SSC MTS 14 August 2019 समान ाज दर के साथ 20000
यिद च वृ ाज दर 20% (Evening) पर 3 वष के िलए च वृ ाज
ितवष है , अध वािषक प से (a) 50000 िकतना होगा ?
संयोिजत, तो 100000 के मूलधन (b) 5000 SSC MTS 16 August 2019
पर दो वष का ाज िकतना होगा ? (c) 10000 (Evening)
SSC MTS 14 August 2019 (d) 1000 (a) Rs 6620 / 6620
(Morning) (b) Rs 3310 / 3310
(a) Rs46,410 Q59. How much will a principal (c) Rs 5760 / 5760
(b) Rs44,000 of Rs5000 invested on compound (d) Rs 2800 / 2880
(c) Rs21,000 interest (compounded annually)
(d) Rs33,100 amount to, in three years at a rate Q62. What is the compound

e
of 50% per annum? interest on Rs 5000 in 2 years at
Q56. A principal increases 21% च वृ ाज ( वािषक प से the rate of 20% per annum?
with compound interest in two संयोिजत ) पर िनवेश िकया गया (interest compounded half
years. Calculate the simple 5000 पये का मूल धन 50% ित yearly)
वष की दर से तीन वष म िकतना हो 5000 पये पर 20% ित वष की दर

l
interest for 4 years on Rs. 1000 at
the same rate of interest. जाएगा ? से 2 वष का च वृ ाज ात

पर समान
ac
कोई मूलधन च वृ ाज के साथ
2 वष म 21% बढ़ जाता है | 1000
ाज दर से 4 वष म
साधारण ाज िकतना होगा ?
SSC MTS 14 August 2019
SSC MTS 16 August 2019
(Morning)
(a) Rs16,375
(b) Rs11,250
(c) Rs16,875
कर | ( ाज की दर च वृ
अधवािषक है ) SSC MTS 19
August 2019 (Morning)
(a) Rs2340.50
(b) Rs2275.50
(Morning) (d) Rs17,275 (c) Rs2290.50
(a) Rs320 (d) Rs2320.50
(b) Rs400 Q60. What is the difference
nn
(c) Rs360 between the compound interests Q63. A sum of Rs. 3600 invested
(d) Rs420 on Rs. 10000 for 2 years at 20% on Compound interest becomes
per annum when the interests are Rs. 4900 in 2 years . ( Interest is
Q57. What is the effective annual compounded half yearly and compounded annually ) What is
rate of interest corresponding to a yearly ? the rate of interest per annum ?
rate of 10% per annum 10000 की रािश पर 2 वष के िलए च व ाज पर िनवेिशत 3600
compounded half-yearly? 20% वािषक दर पर च वृ ाजों पए , 2वष म 4900 पए हो जाते
10% ित वष ( अध वािषक प से म ा अंतर् है , जब ाजों को है ( ाज वािषक प से संयोिजत )|
Pi

संयोिजत ) की दर के संगत भावी मश; अध-वािषक और वािषक ितवष ाज दर िकतनी है ?


वािषक ाज दर िकतनी होगी ? प से संयोिजत िकया जाता है ? SSC MTS 19 August 2019
SSC MTS 14 August 2019 SSC MTS 16 August 2019 (Afternoon)
(Afternoon) (Afternoon) (a) 18 31
(a) 10.75% (a) Rs 440 / 440 (b) 17 31
(b) 10.5% (b) Rs 241 / 241 (c) 15 32
(c) 10% (c) Rs 441 / 441
(d) 16 32
(d) 10.25% (d) Rs 240 / 240

Q64. What will be the compound


Q58. If the difference between Q61. A principal increases 50%
interest on a sum of Rs31250, at
simple and compound interest on at simple interest in 5 years.
12% compound interest (interest
a sum of money for 2 years at 5% What will be the compound
compounded yearly) in 2 32
p.a. is Rs125, the sum (in Rs) is:
years? 31250 पये की एक रािश

www.ssccglpinnacle.com support@ssccglpinnacle.com Ph. 09729327755, 09817390373


341
/
Days 40-42 Compound Interest

s
sse
la
_c
ob
पर 12% की दर से 2 32 वष का sum will become Rs. 24494.40 in ाज ( वािषक प से संयोिजत )
bo
ah 2 years at the rate of 8% 9327 पये है | यह रािश है :
sm
च वृ ाज ( वािषक प से
ur
yo

संयोिजत ) िकतना होगा ? compound interest ? SSC MTS 22 August 2019


e/
t.m

यिद च वृ ाज वािषक प से
://

SSC MTS 19 August 2019 (Morning)


tp
ht

(Evening) संयोिजत िकया जाता है , तो 8% (a) Rs24000


(a) Rs 11048 वािषक च वृ ाज की दर से (b) Rs25000
(b) Rs 11096 िकतनी रािश 2 वष म 24494.40 (c) Rs20000
(c) Rs 11068 हो जाएगी ? (d) Rs27000
(d) Rs 11086 SSC MTS 21 August 2019
(Morning) Q71. If an amount becomes 4000
Q65. Calculate the compound (a) Rs 21200 / 21200 rupees after two years and 6000
interest on Rs. 15625 for 2 years (b) Rs 22400 / 22400 rupees after four years at the
at the rate of 12% p.a , if the (c) Rs 21000 / 21000 same rate of compound interest
interest is compounded (d) Rs 22000 / 22000 (compounded annually), what is
8-monthly. the amount?
यिद कोई रािश च वत ाज

e
12% वािषक दर से 2 वष के िलए Q68. The difference between
15625 की रािश पर च वृ ाज compound interest and simple (वािषक संयोिजत ) की समान दर से
िकतना है , यिद ाज को 8-माह पर interest on a sum for 2 years at दो वष प ात् 4000 पए तथा चार
संयोिजत िकया जाता है ? 20% p.a is Rs. 200. The sum is : वष प ात 6000 पए हो जाती है ,
िकसी रािश पर 20% वािषक दर से 2 तो रािश िकतनी है ?

l
SSC MTS 22 August 2019
(Afternoon) वष के च वृ ाज तथा साधारण SSC MTS 2 August 2019
ac
(a) Rs 3075 / 3075
(b) Rs 4058 / 4058
(c) Rs 3675 / 3675
(d) Rs 4088 / 4088
है :
ाज दर का अंतर 200 है | रािश

SSC MTS 21 August 2019


(Afternoon)
(a) Rs 3000 / 3000
(Afternoon)
(a) Rs. 2888.88
(b) Rs. 2666.66
(c) Rs. 2777.77
(d) Rs. 2866.66
Q66. A sum invested at 10% (b) Rs 4500 / 4500
compound interest per annum (c) Rs 4000 / 4000 SSC CGL TIER I
becomes Rs. 19965 in 3 years. (d) Rs 5000 / 5000
nn
The same sum will become Rs. x Q1. The compound interest on a
in 2 52 years. If the interest is Q69. What is the present value of certain sum at 10% p.a. for 2 31
compounded annually in both the Rs. 14739 payable in 3 years at years is ₹1,201.60, interest
cases, then find the value of x . the rate of 6.25% yearly compounded yearly. The sum is:
10% ित वष च व ाज दर पर compound interest ? िकसी िनि त रािश पर 10% ित वष
िनवेिशत कोई रािश 3 वष म 19965 6.25% वािषक च वृ ाज की की दर से 2 31 वष का च वृ
पए हो जाती है | यही रािश समान दर से 3 वष म दे य 14739 की ाज ₹1,201.60 पये है तथा ाज
ाज दर पर 2 52 वष म x पए हो रािश का आज का मान िकतना है ? का संयोजन वािषक है | यह रािश
Pi

SSC MTS 21 August 2019 िकतनी है ?


जाएगी | यिद दोनों ही मामलो म
(Evening) SSC CGL 3 March 2020
ाज को वािषक प से संयोिजत
(a) Rs 12184 / 12184 (Afternoon)
िकया जाता है , तो x का मान िकतना
(b) Rs 12288 / 12288 (a) ₹4,200
है ?
(c) Rs 12473 / 12473 (b) ₹4,800
SSC MTS 20 August 2019
(d) Rs 12148 / 12148 (c) ₹5,400
(Afternoon)
(a) Rs 18855 (d) ₹4,500
Q70. The compound interest on a
(b) Rs 18768
certain sum at 15% p.a. Q2. The simple interest on a
(c) Rs 18867
Compounded yearly for 2 31 years certain sum at the end of three
(d) Rs 18876
is Rs9327. The sum is: years at 5% p.a. is ₹1,200. The
Q67. If the compound interest is िकसी िनि त रािश पर 15% ित वष compound interest on the same
compounded annually, then what की दर से 2 31 वष का च वृ sum for the same period at the

www.ssccglpinnacle.com support@ssccglpinnacle.com Ph. 09729327755, 09817390373


342
/
Days 40-42 Compound Interest

s
sse
la
_c
ob
same rate is (interest (b) ₹5,986 (d) 26,750
bo
ah
compounded yearly): (c) ₹5,980
sm
ur
yo

िकसी िनि त रािश पर 5% ित वष (d) ₹5,920 Q7. A certain amount of money at


e/
t.m

की दर से तीन वष के अंत म compound interest grows to


://
tp
ht

साधारण ाज 1200 पये है | इसी Q5. A sum of ₹8,000 invested at ₹66,550 in 3 years and ₹73,205 in
4 years. The rate per cent per
रािश पर इसी अविध के िलए इतने ही 10% pa amounts to ₹9,261 in a
annum is:
ाज दर से िकतने च वृ ाज certain time, interest compounded
एक िनि त धन रािश च वृ ाज
की ा होगी ? ( ाज का संयोजन half-yearly. What will be the पर 3 वष म 66,550 पये तथा 4 वष
वािषक है ) compound interest (in₹) on the म 73,205 पये हो जाती है | ित वष
SSC CGL 3 March 2020 same sum for the same time at दर ितशत ात कीिजए |
(Evening) double the earlier rate of interest, SSC CGL 5 March 2020
(a) ₹1,800 when interest is compounded (Evening)
(b) ₹1,260 annually? / 10% ित वष की दर से (a) 10%
(c) ₹820 िनवेश की गयी 8,000 पये की एक (b) 5%
(c) 9%
(d) ₹1,261 रािश अधवािषक च वृ ाज की
(d) 11%
दर से िकसी िनि त समय म 9,261

e
Q3. What is the compound पये हो जाती है | इसी रािश पर इतने
Q8. What will be the difference in
interest on a sum of ₹12,000 for ही समय के िलए पूव की तुलना म
compound interest on a sum of ₹
2 85 years at 8% p.a., when the दोगुने दर से च वृ ाज ात
7,800 at 8% for 1 year, when the
कीिजए, यिद ाज का संयोजन

l
interest is compounded annually? interest is paid yearly and half
/ वािषक है |
ac yearly?
12,000 पये की रािश पर 8% ित SSC CGL 4 March 2020
7,800 पये की रािश पर 8% की दर
वष की दर से 2 85 वष का च वृ (Evening)
से 1 वष के च वृ ाज म िकतना
ाज िकतना होगा, जब ाज का (a) ₹2,520
अंतर होगा, यिद ाज का भुगतान
संयोजन वािषक िकया जाता है ? (b) ₹2,480
वािषक तथा अधवािषक प से िकया
(c) ₹2,560
SSC CGL 4 March 2020 जाता है ?
(Morning) (d) ₹2,500
SSC CGL 6 March 2020
(a) ₹2,697 (Morning)
Q6. A and B together borrowed a
nn
(b) ₹2,654 (a) ₹ 24.72
(c) ₹2,712 sum of ₹51,750 at an interest rate
(b) ₹ 12.48
(d) ₹2,642 of 7% p.a. compound interest in
(c) ₹ 29.18
such a way that to settle the loan,
(d) ₹ 19.46
Q4. The difference in the A paid as much amount after three
compound interest on a certain years as paid by B after 4 years
Q9. Amit borrowed a sum of ₹
sum at 10% p.a. for one year, from the day of borrowing. The
25,000 on simple interest. Bhola
when the interest is compounded sum (in ₹) borrowed by A was: /
borrowed the same amount on
A तथा B ने एक साथ 7% ित वष
Pi

half yearly and yearly, is ₹88.80. compound interest (interest


What is the simple interest on the की दर से ₹51,750 पये की रािश
compounded yearly). At the end
same sum for 1 32 years at the इस कार उधार ली िक ऋण चुकता
of 2 years, Bhola had to pay ₹ 160
करने के िलए A ने उधार लेने वाले
same rate? / more interest than Amit. The rate
िदन से तीन वष के बाद उतनी ही
िकसी िनि त रािश पर 10% ित वष of interest charged per annum is:
रािश का भुगतान िकया िजतनी रािश
की दर से एक वष के च वृ ाज अिमत ने साधारण ाज पर 25,000
का भुगतान B ने 4 वष के बाद िकया
म 88.80 पये का अंतर आता है , जब पये की रािश उधार ली | भोला ने
| A के ारा उधार ली गयी रािश (
ाज का संयोजन अधवािषक और इतनी ही रािश च वृ ाज ( ाज
पये म ) है :
वािषक िकया जाता है | इसी रािश पर का संयोजन वािषक ) पर उधार ली | 2
SSC CGL 5 March 2020
इसी दर से 1 32 वष का साधारण वष के अंत म, भोला को अिमत से
(Afternoon)
ाज िकतना होगा ? 160 पये अिधक ाज का भुगतान
(a) 25,000
SSC CGL 4 March 2020 करना पड़ा | ित वष लगाए गए ाज
(b) 25,650
(Afternoon) की दर है :
(c) 24,860
(a) ₹5,916

www.ssccglpinnacle.com support@ssccglpinnacle.com Ph. 09729327755, 09817390373


343
/
Days 40-42 Compound Interest

s
sse
la
_c
ob
SSC CGL 6 March 2020 rounded off on the same sum at annum for one year and interest is
bo
ah
(Afternoon) the same rate for the same period, compounded half yearly. ₹2000 is
sm
ur

given at 40% per annum


yo

compounded annually is closest


e/
t.m

compounded quarterly for 1 year.


://

(a) 3 81 % to:
tp
ht

The total interest received is


(b) 16 % 6% ित वष साधारण ाज की दर से
25 nearest to:
िकसी रािश पर तीन वष का साधारण
(c) 8 % 4000 पये 5% ितवष की दर से
8 %
(d) 25 ाज 6750 पये है | इसी रािश पर एक वष के िलए िदए जाते ह तथा
इसी दर से वािषक संयोजन पर इतनी ाज की च वृ अधवािषक है ।
ही अविध का च वृ ाज ( 2000 पये 1 वष के िलए 40%
Q10. The simple interest on a sum
लगभग ) िकतना होगा ? ितवष की दर से िदए जाते ह तथा
of ₹50,000 at the end of two years
SSC CGL 9 March 2020 ाज की च वृ ैमािसक है । ा
is ₹4,000. What would be the होने वाला कुल ाज लगभग िकतना
(Morning)
compound interest on the same है ?
(a) ₹7,103
sum at the same rate for the same CHSL 13-10-2020 (Afternoon
(b) ₹7,133
period? shift)
(c) ₹7,163
50,000 पये की रािश पर दो वष के (a) ₹ 1,444.40

e
(d) ₹7,663
अंत म 4000 पये का साधारण ाज (b) ₹ 1888.80
िमलता है | इसी रािश पर इसी दर से (c) ₹ 1,130.70
Q13. A certain sum amounts to (d) ₹ 1,333.30
इसी अविध का च वृ ाज ात
₹280900 in 2 years at 6% per
कर |

l
annum, interest compounded Q2. The compound interest and
SSC CGL 7 March 2020
annually. The sum is: the amount obtained, on a certain
(Morning) ac एक िनि त रािश 6 ितशत ित वष sum of money are Rs 820 and
(a) ₹8,000
वािषक संयोजन की दर से 2 वष म 8,820 respectively after 2 years. If
(b) ₹4,040
280900 पये हो जाती है | यह रािश the rate of interest compounded
(c) ₹4,008
िकतनी है ? yearly, then the rate of interest is:/
(d) ₹4,080
SSC CGL 9 March 2020 एक िनि त रािश पर 2 साल बाद
(Afternoon) च वृ ाज और ा रािश
Q11. A sum lent out at compound
(a) ₹350000 मशः 820 और 8,820 पये है । यिद
interest amounts to ₹1,250 in one
(b) ₹200000 ाज की दर वािषक प से संयोिजत
nn
year and to ₹1,458 in 3 years at a
(c) ₹250000 है , तो ाज की दर है :
certain rate percentage p.a. What
(d) ₹550000 CHSL 13-10-2020 (Evening
is the simple interest on the same
sum for 5 32 years at the same rate shift)
Q14. The compound interest on a (a) 8%
of interest?
certain sum at the end of two (b) 6%
च वृ ाज पर उधार दी गयी एक
years is ₹408. The simple interest (c) 5%
रािश ित वष एक िनि त दर ितशत
on the same sum for the same (d) 7%
से एक वष म 1,250 पये तथा 3 वष
time is ₹400. The rate of interest
Pi

म 1,458 पये हो जाती है | इसी रािश


per annum is: Q3. The compound interest on
पर इसी ाज दर से 5 32 वष का
एक िनि त रािश पर दो वष के अंत Rs4,000 at the rate of 5% p.a. is
साधारण ाज िकतना होगा ? म 408 पये का च वृ ाज Rs630.50, then the time period is:
SSC CGL 7 March 2020 िमलता है | इसी रािश पर इसी अविध यिद 4000 पये पर 5% ित वष की
(Afternoon) का साधारण ाज 400 पये है | ित दर से च वृ ाज 630.50 पये
(a) ₹600 वष ाज की दर िकतनी है ? है , तो समय अविध िकतनी है ?
(b) ₹520 SSC CGL 9 March 2020 CHSL 14-10-2020 (morning
(c) ₹500 (Evening) shift)
(d) ₹480 (a) 4% (a) 2 years/ वष
(b) 80% (b) 3 21 years/ वष
Q12. The simple interest on a sum (c) 40% (c) 3 years/ वष
of money for 3 years at an interest (d) 8% (d) 1 21 years/ वष
rate of 6% p.a. is ₹6,750. What SSC CHSL 2019
will be the compound interest Q1. ₹4,000 is given at 5% per

www.ssccglpinnacle.com support@ssccglpinnacle.com Ph. 09729327755, 09817390373


344
/
Days 40-42 Compound Interest

s
sse
la
_c
ob
Q4.The compound interest on Rs ाज और साधारण ाज के बीच का at 10% p.a, if the interest is
bo
ah
4000 after 3 year is 630.50. Then अंतर ात करे । compounded half yearly (nearest
sm
ur
yo

the rate of interest compounded CHSL 26-10-2020 (Afternoon to ₹1) ?


e/
t.m

कोई िनि त रािश 12% वािषक


://

yearly is: shift)


tp
ht

3 साल बाद 4000 पये पर च वृ (a) 10 साधारण ाज की दर पर 2 वष म


ाज 630.50 है । िफर वािषक ाज (b) 30 ₹15,500 हो जाती है । यिद ाज की
की दर है : (c) 40 गणना अधवािषक च वृ आधार
CHSL 15-10-2020 (morning (d) 20 पर की जाती है , तो सामान रािश 10%
shift) वािषक दर पर 1 21 वष म िकतनी हो
(a)7% Q.8. What sum of money at जाएगी (₹1 के िनकटतम मान म )
(b)5% compound interest will amount to CGL 2019 Tier-II (15-11-2020 )
(c)8% Rs.4630.08 in three years if the (a) ₹14,470
(d)6% rate of interest is 4% for the first (b) ₹13,460
year, 5% for the second year and (c) ₹14,360
Q5. In how many years will a sum 6% for the third year? (d) ₹15,125
च वृ ाज पर िकस रािश का

e
of ₹320 amount to ₹405 if interest
is compounded at 12.5% per ाज तीन वष म 43030.08 पये का Q11. Surekha borrowed a sum of
annum? िम धन दे गा, यिद ाज की दर पहले money and returned it in two
िकतने वष म ₹320 रािश बढ़कर वष के िलए 4%, दू सरे वष के िलए equal annual installments of
₹405 हो जाएगी अगर ाज ित वष 5% और तीसरे वष के िलए 6% है ?

l
₹5547 each. If the rate of interest
12.5% पर च वृ है ? CHSL 26-10-2020 (Evening was 7 21 %p.a compounded yearly,

shift)
(a) 2 years
(b) 1 years
(c) 2 ½ years
ac
CHSL 16-10-2020 (Afternoon shift)
(a) 4500
(b) 4800
(c) 4000
(d) 3500
then the total interest paid by her
was:
सुरेखा ने कुछ रािश उधार ली और
इसे ेक वष ₹5547 की दो बराबर
वािषक िक ों म लौटा िदया। यिद
(d) 1 ½ years ाज की दर वािषक प से संयोजन
Q9. Ram deposited an amount of पर 7 21 ित वष थी, तो उसके ारा
Q.6. If the compound interest on a ₹ 8,000 in a bank’s savings
nn
िदया गया कुल ाज था:
certain sum of money for 2 years account with interest 6.5%
CGL 2019 Tier-II (15-11-2020 )
at 5% p.a. is Rs.328, then the sum compounded monthly. What
(a) ₹1,144
is equal to: amount will he get at the end of
(b) ₹1,134
यिद 2 वष के िलए 5% ितवष की दर 18 months?
(c) ₹1,096
से एक रािश पर च वृ ाज 328 राम ने बक के बचत खाते म 8000
(d) ₹1,126
पये है , तो रािश ात करे | पये की रािश जमा की िजस पर
CHSL 26-10-2020 (morning मािसक संयोजन के अनुसार 6.5%
Q12. The compound interest on a
shift) ाज िमलता है । 18 माह के अंत म
Pi

sum of ₹5,500 at 15% p.a for 2


(a) 3600 उसे िकतना िम धन ा होगा?
years, when the interest
(b) 3500 CHSL 17-03-2020 ( Evening
compounded 8 monthly is:
(c) 3000 shift)
₹5,500 की रािश पर वािषक 15% की
(d) 3200 (a) ₹ 8816.97
दर से 2 वष म ा च वृ ाज
(b) ₹ 8788.98 ात कीिजए, जब ाज की गणना हर
Q.7. The difference between the (c) ₹ 8790.54 8 महीने पर च वृ आधार पर की
compound interest and the simple (d) ₹ 8907.56 जाती है ।
interest on a sum of Rs.8,000 for 2
CGL 2019 Tier-II (15-11-2020 )
years at the rate of 5% per annum SSC CGL 2019 TIER-II (a) ₹1,880
is: Q10. A certain sum amounts to (b) ₹1,820.50
5% ित वष की दर से 2 वष के िलए ₹15,500 in 2 years at 12% p.a. (c) ₹1,773.75
8,000 पये की रािश पर च वृ simple interest. The same sum (d) ₹1,850
will amount to what in 1 21 years

www.ssccglpinnacle.com support@ssccglpinnacle.com Ph. 09729327755, 09817390373


345
/
Days 40-42 Compound Interest

s
sse
la
_c
ob
Q13. The compound interest on a (d) ₹12,143 years at a certain rate percent per
bo
ah
sum of ₹20, 000 at 15% p.a for annum. When the interest is
sm
ur
yo

2 32 years, interest compounded Q.16 At what rate per cent per compounded annually, The rate of
e/
t.m
://

yearly is : annum will a sum of ₹15,625 interest is:


tp
ht

2 32 वष के िलए ₹20,000 पर 15% amount to ₹ 21,952 in three एक रािश ित वष एक िनि त दर


की दर से च वृ ाज ात years. If the interest is ितशत से 4 वष म 7,562 पये तथा
कीिजए, यिद ाज का संयोजन compounded annually? 5 वष म 8,469 पये हो जाती है ।
वािषक है । ित वष िकस ाज दर से, 15,625 ाज का संयोजन वािषक है । ाज
CGL 2019 Tier-II (16-11-2020 ) पये की एक रािश तीन वष म की दर िकतनी है ?
(a) ₹ 9,098 21,952 पये हो जाएगी, यिद ाज CPO 2019 23-11-2020
का संयोजन वािषक है ? (Morning shift)
(b) ₹ 9,095
CGL 2019 Tier-II (18-11-2020 ) (a) 8%
(c) ₹ 8,896
(a) 12% (b) 12%
(d) ₹ 9,000
(b) 8% (c) 20%
(c) 9% (d) 15%
Q14. A loan is to be returned in

e
(d) 10%
two equal instalments, If the rate
Q19. If the difference between the
of interest is 10% p.a.
Q.17. Rahul invested equal sums compound interest and simple
Compounded annually, and each
of money at compound interest interest at 17% on a sum of
instalment is ₹5,808, the the total

l
under two schemes A and B. money for 2 years (compounded
interest charged in this scheme is:
Under scheme A, the interest rate annually) is Rs. 433.50 then the
एक ऋण दो समान िक ों म वापस
ac was 10% per annum and under sum (in Rs.) is:
िकया जाना है , यिद ाज की दर
scheme B, the interest rate was यिद एक धनरािश पर 17% की दर से
10% ित वष वािषक संयोजन है और
12%p.a. The compound interest 2 वष म ा च वृ ाज और
ेक िक ₹5,808 की है , तो इस
after two years on the sum साधारण ाज का अंतर (वािषक
योजना म िलया जाने वाला कुल ाज
invested in scheme A was ₹1,050. संयोजन) 433.50 पये है , तो यह
है :
How much is the interest earned रािश ( पये म) िकतनी है ?
CGL 2019 Tier-II (16-11-2020 )
under scheme B after two years, if CPO 2019 23-11-2020
(a) ₹1,536
the interest is compounded (Evening shift)
nn
(b) ₹1,632
annually in both schemes? (a) 15,000
(c) ₹1,602
रा ल ने दो योजनाओं A और B म (b) 12,000
(d) ₹1,563
च वृ ाज पर बराबर रािशयाँ (c) 20,000
िनवेश की। योजना A म, ाज की (d) 25,000
Q15. A sum of ₹10,500 amounts
दर 10% ित वष थी तथा योजना B
to ₹13,650 in 2 years at a certain
म, ाज की दर 12% थी। योजना A Q20. If the difference between
rate per cent per annum simple
म िनवेिशत रािश पर दो वष के बाद the compound interest and simple
interest. The same sum will
च वृ ाज 1050 पये था। दो interest at 17% on a sum for 2
Pi

amount to what in 1 year at the


वष के बाद योजना B म िकतना years (compounded annually ) is
same rate, if the interest is
ाज ा होगा, यिद दोनों ही Rs.433.50. Then the compound
compounded half yearly(nearest to
योजनाओं म ाज का संयोजन interest (in Rs.) is:
₹1)?
वािषक है । यिद एक धनरािश पर 17% की दर से
10,500 पये की एक रािश ित वष
CGL 2019 Tier-II (18-11-2020 ) 2 वष म ा च वृ ाज और
एक िनि त दर ितशत से 2 वष म
(a) ₹1,270 साधारण ाज का अंतर (वािषक
13,650 पये हो जाती है । यही रािश
(b) ₹1,372 संयोजन) 433.50 पये है तो च वृ
इसी दर से 1 वष म िकतनी होगी, यिद
(c) ₹ 1,722 ाज ात करे ।
ाज का संयोजन अधवािषक है ? ( 1
(d) ₹1,272 CPO 2019 24-11-2020
पये के िनकटतम)
(Morning shift)
CGL 2019 Tier-II (16-11-2020 )
SSC CPO 2019 (a) 5,533.50
(a) ₹12,134
Q18. A Sum amounts to Rs.7,562 (b) 2,735.50
(b) ₹12,314
in 4 years and to Rs.8,469.44 in 5 (c) 5,100
(c) ₹12,124

www.ssccglpinnacle.com support@ssccglpinnacle.com Ph. 09729327755, 09817390373


346
/
Days 40-42 Compound Interest

s
sse
la
_c
ob
(d) 2,500 15% p.a. When the interest is
bo
ah compounded yearly?
sm
ur
yo

Q21. The simple interest on a sum 2 साल के िलए ित वष 10% की दर


e/
t.m

से एक धन पर साधारण ाज 8,100
://

of money at 10% per annum for 2


tp
ht

years is Rs. 8,100, Compounded पये है । 15% ित वष की दर से


annually. What would be the इसी अविध के िलए इसी रािश पर
compound interest (in Rs.) on the च वृ ाज ( पये म) िकतना
same sum for the given period at होगा, यिद ाज वािषक प से
the rate of interest ? संयोिजत होता है ?
एक धनरािश पर 10% ित वष की CPO 2019 25-11-2020
दर से 2 वष का साधारण ाज 8100 (Evening shift)
पये है । दी गयी अविध के िलए ाज (a) 13,061
दर पर इसी रािश का च वृ ाज (b) 8,100
( पये म) िकतना होगा? (c) 14,671
CPO 2019 24-11-2020 (d) 12,751

e
(Evening shift)
(a) 9,000
(b) 8,715
(c) 8,505

l
(d) 8,100
ac
Q22. A sum amounts to ₹7,562 in
4 years and to ₹8,469.44 in 5
years, at a certain rate percent per
annum when the interest is
compounded yearly. If ₹10,000 at
the same rate of interest is
borrowed for two years , then
what will be the compound
nn
interest (in ₹)?
जब ाज का संयोजन वािषक है , तब
एक रािश ित वष एक िनि त दर
ितशत से 4 वष म 7,562 पये तथा
5 वष म 8,469 पये हो जाती है ।
यिद इसी दर से 10000 पये दो वष
के िलए उधार िलए जाते ह, तो
च वृ ाज िकतना होगा?
Pi

CPO 2019 25-11-2020


(Morning shift)
(a) 1,736
(b) 1,965
(c) 2,544
(d) 2,764

Q23. The simple interest on a sum


of money at 10% per annum for 2
years is Rs. 8,100. What would be
the compound interest (in Rs.) on
the same sum for the period at

www.ssccglpinnacle.com support@ssccglpinnacle.com Ph. 09729327755, 09817390373


347
/
Days 40-42 Compound Interest

s
sse
la
_c
ob
Sol 1. (a) 15625 unit = 25000 100 unit = 15400
bo
ah A1 ×A1 1 unit = 1.6
sm
Principle =
ur

A2
yo

4058 unit = 4058 x 1.6 ≈ 6493 Sol 6. (b)


e/
t.m

Here, A1 = first amount and A2 =


://

5% = 20 1
Alternate :
tp

Second amount
ht

8% = 25 2
⇒ Desired sum = 8028 × 8028 20 ---------- 21
12042 =
Principal Amount 202 ---------- 212
5352
25 ------- 27 Installment for 2 years must be
1 same so balancing the ratio for
Sol 2. (b) 10% = 10 25 ------- 27 installment
Let the principle = 25 ------- 27 420 ------- 441
103 = 1000 unit ___________ 400 ------- 441
15625 19683 _______________
15625 unit = 25000 820 882
1 unit = 1.6
(19683-15625) 4058 unit = 4058 x According to the question

e
1.6 ≈ 6493 441 unit = 35280
1 unit = 80
Sol 4. (b) 820 unit = 65600
16% = 25 4

l
Let the principle = Sol 7. (c)
1
1
Interest earned in 2 years = 100
ac 2 253 = 15625 unit Rate of interest = [ ( PAmount
rinciple ) − 1
t

+(100+10)+ 21 (100+10+10+1) = ] x 100


270.50 Where t = time interval
270.5 unit = 1623 1
= ( 20736
12000 ) − 1] x 100=
3

1 unit = 6
20%
1000 unit = 6000
Now,
20% = 1
Sol 3. (c) 5
nn
Interest is compounded 8 monthly 5 ---------- 6
So, effective rate of interest = 12 5 ---------- 6
8 = 8%
× 12 Desired difference = _____________
12 (2500+400+400+64) - 25 36
Effective period of time = 2 × 8 =
(2500+400) = 464 25 unit = 12000
3
2 15625 unit = 15000 1 unit = 480
8% = 25
1 unit = 0.96 36 unit = 17280
Let the principle =
464 unit = 464 × 0.96 = 445.44
253 = 15625 unit
Pi

Sol 8. (d)
8% = 25 2
Sol 5. (b)
10% = 10 1 Let X = 252 = 625 unit
10 ---------- 11
10 ---------- 11
_______________
100 121
Let the principal = 100 unit
⇒ amount after 2 years = 121 unit
According to the question
Interest earned = 11 = 13200
(121 unit - 6634) × 10
1250+1250+1250+100+100+100+
8 121 unit = 12000+6634 According to the question
= 4058 unit 1 unit = 154 4 unit = 19.20

www.ssccglpinnacle.com support@ssccglpinnacle.com Ph. 09729327755, 09817390373


348
/
Days 40-42 Compound Interest

s
se
s
la
_c
ob
1 unit = 4.8 250 unit = 24000
bo
ah
625 unit = 3000 1 unit = 96
sm
ur
yo

Alternate : (297-250) unit = 47 x 96 = 4512


e/
t.m
://

Difference between CI and SI =


tp
ht

R2 Sol 13. (a)


100 % of Principal
⇒ 19.20 =
2
8 The amount becomes 3 times in
100 % of X
10 years.
⇒ X = 3000
Time take for the amount to
become ( 34 times = 81 times) =
Sol 9. (a)
1 10 x 4 = 40 years
10% = 10 CI-SI = 20+20+20+1 = 61 unit
Let the Principle = 10 unit so, According to the question
Sol 14. (a)
interest earned in one year = 1 8000 unit = 12000
Simple interest earned in 2 years =
unit 1 unit = 23
250
⇒ Interest earned in 27 years = 1 61 unit = 91.50 ⇒ simple interest earned in one
× 7 = 3.5 unit

e
2 year = 250
2 = 125
3.5 unit = 2940 Sol 11. (a) Let the rate of interest
We know that
1 unit = 840 = r and time period = 2t
Compound interest earned in two
10 unit = 840 x 10 = 8400 According to the question
2t years = simple interest of two

l
r
Now, 63654 = 60000 (1 + 100 ) years + interest earned on the
Let the principle = 103 = 1000

t
⇒ (1 + r 63654 simple interest of first year
ac 100 ) = 60000
unit Desired Compound interest = 250
………..(1)
+ 125100
× 20 = 275
Amount earned in t years = 60000
r t
(1 + 100 ) Sol 15. (b)
Put the value from equation (1) Since interest is compounded half
r t
60000 (1 + 100 )
⇒ 60000 x yearly, effective rate of interest =
8 × 21 = 4 % and effective time
√ 63654
nn
60000
period = 1x2= 2 years
= 61800 1
Now, 4% = 25
Sol 12. (b) Let the principal = 252 = 625

Interest earned in 2 21 years = 100


+(100+10)+ 21 (100+10+10+1) =
270.50
1000 unit = 8400
Pi

1 unit = 8.4
270.5 unit ≈ 2272

Sol 10. (b) According to the question


Since interest is compounded 5 (625+51) unit = 20280
Compound Interest =
monthly, effective rate of interest 1 unit = 30
1920+2400+192 = 4512
5 = 5 % and effective time
= 12 × 12 625 unit = 625 x 30 = 18750
Alternate :
period = 5 × 12 = 3 years 2 and 10% = 1
New rate of interest = 2 x 8 = 16%
4 5 8% = 25
1
10 Desired SI = 18750×16×23
5×100 = 13800
Now, 5% = 20 Principal Amount
3 Alternate :
Let the principal = 20 = 8000 25 27 1
4% = 25
10 11
Principal Amount
____________
25 ------- 26
250 297

www.ssccglpinnacle.com support@ssccglpinnacle.com Ph. 09729327755, 09817390373


349
/
Days 40-42 Compound Interest

s
sse
la
_c
ob
25 ------- 26 121 unit = 217.8
bo
ah ___________ Note : Whenever rate % = 10
sm
ur
yo

625 676 and time period = 4 years.


e/
t.m
://

676 unit = 20280 CI = 46.41% of Principle. To


tp
ht

1 unit = 30 save your precious time just


625 unit = 625 x 30 = 18750 crame it.
New rate of interest = 2 x 8 = 16%
16% = 25 4 Sol 2. (b)
Let CP = 25 unit and interest Since interest is compounded 8
earned in one year = 4 unit monthly, effective rate of interest
8 = 8 % and effective time
= 12 × 12
⇒ Interest earned in 23
5 years =
period =2 × 12 = 3 years
18.4 unit 8
110 unit = 1320 or 121 unit = 2
According to the question Now, 8% = 25
1452
25 unit = 18750 Let the principal = 253 = 15625
1 unit = 12
1 unit = 750
1000 unit = 12000

e
18.4 unit = 750 x 18.4 = 13800
Sol 19. (d)
Sol 16. (d)
20% = 51
Difference between 3rd year CI
CI after 5th year = 250 × 56 x 56 x 56

l
R R 2
and SI = P ( 100 ) ( 100 + 3)
= 432

⇒ P = 5000
ac
Here P = principal and R is the
rate of interest
10
⇒ 155 = P ( 100 10
) [ 100 + 3]
2
Sol 20. (d)
8% = 252

4th year’s CI = 3600 x


5th years’s CI = 3888
27
25 = 3888
27 =
x 25
Interest earned =
1250+1250+1250+100+100+100+
Sol 17. (c) 8
4199.04 = 4058 unit
Difference between 2 year’s CI
2
Desired difference = 15625 unit = 31250
R
nn
and SI = P ( 100 ) 4199.04-3888 ≈ 311 1 unit = 2
Here P = principal and R is the
4058 unit = 4058 x 2 =8116
rate of interest SSC CGL TIER II Alternate :
⇒ 33.80 = P 6.5 2
( 100 ) 8% = 25 2

⇒ P = 8000 Sol 1. (b)


1
Principal Amount
Sol 18. (d) Difference of SI = 10% = 10
25 ------- 27
1452 - 1320 = 132 Let the principal = 104 = 10000 25 ------- 27
r = 132
So, 1320 × 100 25 ------- 27
Pi

⇒ r = 10% ___________
1
10% = 10 15625 19683
Let the principal = 103 = 1000 15625 unit = 31250
1 unit = 2
(19683-15625) 4058 unit = 4058 x
2 = 8116

Sol 3. (d)
Difference between 4th year and 20% = 51
3rd year CI = 100+10+10+1 = 121 Let the principle = 53 = 125 unit
unit
According to the question
10000 = 18000
1 unit = 1.8

www.ssccglpinnacle.com support@ssccglpinnacle.com Ph. 09729327755, 09817390373


350
/
Days 40-42 Compound Interest

s
sse
la
_c
ob
15% = 3 ⇒ Desired sum = 18600 × 18600 =
bo
20 27900
ah
sm
3 12400
Let the principal = 20 = 8000
ur
yo
e/

unit
t.m
://

Sol 3. (a)
tp
ht

Since interest is compounded 5


monthly, effective rate of interest
5 = 10 % and effective time
= 8 × 12 3
period = 5 × 12 = 3 years
4 5
Now, 10 %= 1
3 30
Total interest earned = 25+25+5+ Let the principal = 303 = 27000
2
5 x (25+5+5+1) = 69.4
125 unit = 7200
1 unit = 57.6 Total interest earned =
69.4 unit = 69.4 x 57.6 = 3997 1200+1200+180+ 52
(1200+180+180+27) = 3214.8

e
Sol 4. (c) unit
5% = 20 1 According to the question
20 -------- 21 (8000+3214.8) unit = 4205.55
202 ------- 212 1 unit = 0.375

l
8000 unit = 8000 x 0.375 = 3000
Installment must be same so

Installment
ac
balancing the equation for

420 -------- 441


400 -------- 441
________________
Practice Questions

Sol 1. (b)
14% = 50 7

Let the principle =


Total interest earned =
900+900+30+900+30+30+1 =
2791 unit
820 882 According to the question
According to the question 503 = 125000 unit 27000 unit = 8100
441 unit = 44100 1 unit = 0.3
nn
1 unit = 100 2791 unit = 2791 x 0.3 ≈ 837
(882-820) unit = 62 x 100 = 6200 Alternate :
10 % = 1
3 30
Sol 5. (d) Principal Amount
Effective rate of interest = 8 = 4%
2 30 ------- 31
4% = 1
25 30 ------- 31
25 --------- 26 30 ------- 31
252 ------- 262 ___________
Pi

Desired difference =
Installment must be same so 27000 29791
(17500+17500+2450)+ 75
balancing the equation for According to the question
(17500+2450+2450+343) =
Installment 27000 unit = 8100
53695
650 -------- 676 1 unit = 0.3
125000 unit = 10000
625 -------- 676 2
(29791-27000) unit = 2791 x 0.3
1 unit = 25
________________ ≈ 837
53695 unit = 53695 × 2 ≈ 4296
1275 1352 25

According to the question Sol 4. (d)


676 unit = 6760 Sol 2. (b) 9% = 1009
A1 ×A1
1 unit = 10 Principle = A2 Let X = 1002 = 10000 unit
(1352-1275) unit = 77 x 10 = 770 Here, A1 = first amount and A2 =
Second amount
Sol 6. (d)

www.ssccglpinnacle.com support@ssccglpinnacle.com Ph. 09729327755, 09817390373


351
/
Days 40-42 Compound Interest

s
sse
la
_c
ob
bo
ah
sm
ur
yo
e/
t.m
://
tp
ht

According to the question According to the question According to the question


81 unit = 20.25 9 unit = 45 9 unit = 43.20
1 unit = 0.25 1 unit = 5 1 unit = 4.8
10000 unit = 10000 x 0.25 = 2500 1600 unit = 1600 x 5 = 8000 625 unit = 625 x 4.8 = 3000
Alternate : Alternate : Alternate :
Difference between CI and SI = Difference between CI and SI = Difference between CI and SI =
R2 R2 R2
100 % of Principal % of Principal 100 % of Principal
100

e
2
⇒ 20.25 = 9 % of X 7.52 ⇒ 43.20 = 122
100 ⇒ 45 = % of X 100 % of X
100
⇒ X = 2500 ⇒ X = 8000 ⇒ X = 3000

Sol 5. (c) Sol 7. (a) Sol 9. (b)

l
8.5% = 20017 2
12% = 25 3 8% = 25
Let X = 2002 = 40000 unit
ac Let X = 252 = 625 unit Let X = 252 = 625 unit
nn
According to the question According to the question
289 unit = 28.90 9 unit = 18 According to the question
1 unit = 0.1 1 unit = 2 4 unit = 48
40000 unit = 40000 x 0.1 = 4000 625 unit = 625 x 2 = 1250 1 unit = 12
Alternate : Alternate : 625 unit = 7500
Difference between CI and SI = Difference between CI and SI = Alternate :
R2 R2 Difference between CI and SI =
100 % of Principal % of Principal
Pi

100 R2
⇒ 28.90 = 8.52
% of X 122 100 % of Principal
100 ⇒ 18 = 100 % of X
⇒ 48 = 82 % of X
⇒ X = 4000 ⇒ X = 1250 100
⇒ X = 7500
Sol 6. (b) Sol 8. (c)
7.5% = 40 3
12% = 25 3 Sol 10. (c)
2 7% = 1007
Let X = 40 = 1600 unit Let X = 252 = 625 unit
Let X = 1002 = 10000 unit

www.ssccglpinnacle.com support@ssccglpinnacle.com Ph. 09729327755, 09817390373


352
/
Days 40-42 Compound Interest

s
sse
la
_c
ob
bo
ah
sm
ur
yo
e/
t.m
://
tp
ht

According to the question


49 unit = 24.50 According to the question
1 unit = 0.5 9 unit = 9 Interest earned =
10000 unit = 10000 x 0.5 = 5000 1 unit = 1 64+64+64+8+8+8+1 = 217
Alternate : 400 unit = 400 x 1 = 400 Now,
Difference between CI and SI = Alternate : 512 unit = 4096
R2
% of Principal Difference between CI and SI = 1 unit = 8
100
R2
⇒ 24.5 = 72 % of X % of Principal 217 unit = 1736

e
100
100
⇒9= 152
⇒ X = 5000 100 % of X
⇒ X = 400 Alternate :
50 % = 1
Sol 11. (c) 4 8
Principal Amount

l
11% = 10011 Sol 13. (b)
1 8 -------------- 9
Let X = 1002 = 10000 unit
ac Rate of interest = [ ( PAmount
rinciple ) − 1
t
8 -------------- 9
] x 100 8 -------------- 9
Where t = time interval ___________
1
= ( 2409 512 729
2190 ) − 1] x 100=
3

10% According to the question


512 unit = 4096
Sol 14. (b) 1 unit = 8
1 (729-512) unit = 217 x 8 = 1736
nn
Rate of interest = [ ( PAmount
rinciple ) − 1
t

According to the question


121 unit = 60.50 ] x 100 Sol 16. (a)
1 unit = 0.5 Where t = time interval 1

1 Rate of interest = [ ( PAmount


rinciple ) − 1
t

10000 unit = 10000 x 0.5 = 5000 = ( 8748


7500 ) − 1] x 100= 8%
2
] x 100
Alternate : New rate of interest = 8 x 2 = 16% Where t = time interval
Difference between CI and SI = Desired Simple Interest = 1

R2 7500×16×23 = 5520 = [ ( 11664


10000 ) − 1] x 100= 8%
2

100 % of Principal 5×100


112
Desired Simple Interest =
Pi

⇒ 60.5 = 100 % of X 10000×8×27 = 4320


Sol 15.(b) 5×100
⇒ X = 5000
Since interest is compounded 10
monthly, effective rate of interest Sol 17. (b)
Sol 12. (b) 10% = 10 1
10 = 50 % and effective
= 15 × 12
15% = 20 3 4
2 time period = 5 × 12 = 3 years Let the principal = 104 = 10000
Let X = 20 = 400 unit 2 10
Now, 50 %= 1
4 8
Let the principal = 83 = 512

www.ssccglpinnacle.com support@ssccglpinnacle.com Ph. 09729327755, 09817390373


353
/
Days 40-42 Compound Interest

s
sse
la
_c
ob
bo
ah Sol 20. (c)
sm
ur
yo

Let the amount after 2 years =


e/
t.m
://

1000000
tp
ht

⇒ the amount after 5 years =


1191016
1
Rate of interest = [ ( PAmount
rinciple ) − 1
t

] x 100
Interest earned = Where t = time interval
Total interest earned 1

=1000+1000+100+1000+100+100 1200+1200+180+1200+180+180+ = ( 1191016


1000000 ) − 1] x 100=
3

+10+1000+100+10+100+1+10+1 27 = 4167 unit 6%


0+100 = 4641 unit According to the question
According to the question 4167 unit = 4167 Sol 21. (d)
(10000+4641) unit = 29282 1 unit = 1 21% = 10021
8000 unit = 8000

e
1 unit = 2 Let the CP = 1002 = 10000 unit
10000 unit = 20000 Desired Simple Interest =
8000×15×24 = 5760
Desired Simple Interest = 5×100
20000×10×4 = 8000
100
Alternate :
3

l
Alternate : 15% = 20
1
10% = 10 ac 20 -------- 23
10 -------- 11 20 -------- 23
10 -------- 11 20 -------- 23
10 -------- 11 _______________
8000 ----- 12167 Compound Interest earned =
10 -------- 11
According to the question 2100+2100+441= 4641
_______________
(12167-8000) unit = 4167 Simple interest earned =
10000 --- 14641
1 unit = 1 2100+2100 = 4200
According to the question
8000 unit = 8000 According to the question
nn
14641 = 29282
Let the principal = 20 unit and 4641 unit = 11602.5
1 unit = 2
interest earned in one year = 3 1 unit = 2.5
10000 unit = 20000
unit 4200 unit = 4200 x 2.5 = 10500
Let the principal = 10 unit and
interest earned in one year = 1 ⇒ Interest earned in 24
5 years =
Alternate :
unit 14.4 unit 21
21% = 100
⇒ Interest earned in 4 years = 4 Now,
20 unit = 8000 100 -------- 121
unit
100 -------- 121
Pi

Now, 1 unit = 400


14.4 unit = 5760 _______________
10 unit = 20000
10000 ----- 14641
1 unit = 2000
Sol 19. (a) According to the question
4 unit = 8000
Let the amount after 3 years = (14641-10000) unit = 11602.5
10000 1 unit = 2.5
Sol 18. (d)
3 ⇒ the amount after 5 years = 10000 unit = 25000
15% = 20
11881 Let the principal = 100 unit and
Let the principal = 203 = 8000 1 interest earned in one year = 21
Rate of interest = [ ( PAmount
rinciple ) − 1
t
unit
] x 100 ⇒ Interest earned in 2 years = 42
Where t = time interval unit
1
= ( 11881
10000 ) − 1] x 100=
2
Now,
9% 100 unit = 25000

www.ssccglpinnacle.com support@ssccglpinnacle.com Ph. 09729327755, 09817390373


354
/
Days 40-42 Compound Interest

s
se
s
la
_c
ob
1 unit = 250 Total interest gained =
bo
ah
42 unit = 10500 1000+1000+50+ 21
sm
ur
yo

(1000+50+50+2.5)= 2601.25
e/
t.m
://

Sol 22. (c) Gain % = 2601.25


20000 × 100 = 13.00
tp
ht

the amount after 3 years = 8000


the amount after 6 years = 27000 Sol 25. (a)
1
Rate of interest = [ ( PAmount
rinciple ) − 1
t
Since interest is compounded half
] x 100 yearly, effective rate of interest =
10 = 5% and effective time period
Where t = time interval 2
1
= 15 x 2= 2 21 years
= ( 27000
8000 ) − 1] x 100=
3 12
Interest earned =
50% 30000+28800+28800+3456+3600
+3600+432 = 98688
Sol 23. (b ) Amount earned = 2,40,000+98688
Since interest is compounded half = 3,38,688

e
yearly, effective rate of interest =
8
2 = 4% and effective time period Alternate :
= 16
12 x 2= 2 2
3 years 12.5% = 81 and 12% = 3
25
Now, 4% = 1
25
8 -------- 9

l
Let the principal = 253 = 15625 25 -------28
Total interest gained = 25 ------ 28
ac 600+600+30+ 21
(600+30+30+1.5)= 1560.75
Therefore, % Gain =
1560.75 × 100 = 13.0%
12000
_____________
5000 ----- 7056
According to the question
5000 unit = 240000
1 unit = 48
Sol 26. (a) Amount to be paid 7056 unit = 338688
27 =
after 1st year = 28500 × 25
nn
30780 Sol 29. (a)
Interest earned = 625+625+25+ 32 Amount remaining after Since interest is compounded half
payment of Rs. 5780 = 25000 yearly, effective rate of interest =
(625+25+25+1) = 1725.67 unit 12 = 6% and effective time period
Desired %age = 1725.67 Amount to be paid at the 2
15625 x 100 = 27 =
end of 2nd year = 25000 × 25 =1x 2= 2 years
11.04 % ≈ 11% 3
Rs. 27000 6%= 50

Sol 24. (c) Let the principal = 502 = 2500


Sol 27. (a) unit
Since interest is compounded half
Pi

yearly, effective rate of interest = Rate of interest = [ ( PAmount


rinciple ) − 1
t

10 = 5% and effective time period


2 ] x 100
= 15 x 2= 2 1 years Where t = time interval
12 2
1
= ( 1606
1460 ) − 1] x 100=
1

10%

Sol 28. (c)

Total interest earned = 150+150+9


= 309 unit
According to the question

www.ssccglpinnacle.com support@ssccglpinnacle.com Ph. 09729327755, 09817390373


355
/
Days 40-42 Compound Interest

s
sse
la
_c
ob
309 unit = 1545 Sol 31. (d) According to the question
bo
ah
1 unit = 5 Since interest is compounded half 25 unit = 1200
sm
ur
yo

2500 unit = 2500 x 5 = 12500 yearly, effective rate of interest = 1 unit = 48


e/
t.m
://

10 = 5% and effective time period 11 unit = 11 x 48 = 528


tp

2
ht

Alternate : =1 21 x 2= 3 years
3
6% = 50 Alternate :
1
5%= 20
3
6% = 50
50 -------- 53 Let the principal = 203 = 8000
50 -------- 53 50 -------- 53
unit
_____________ 50 -------- 53
2500 ----- 2809 _____________
According to the question 2500 ----- 2809
(2809-2500) unit = 1545 According to the question
1 unit = 5 (2809-2500) unit = 1545
(2500) unit = 2500 x 5= 12500 1 unit = 5
(2500) unit = 2500 x 5= 12500

e
Sol 30. (b)
7% = 1007 Sol 33. (b)
1
Let the CP = 100 unit and the Rate of interest = [ ( PAmount
rinciple ) − 1
t

interest earned in one year = 7 Total interest earned =


] x 100

l
unit 400+400+400+20+20+20+1 =
1261 unit Where t = time interval
⇒ the interest earned in three
ac 1
900 1
years = 3x7 = 21 unit According to the question = ( 750 ) − 1] x 100=
According to the question 1261 unit = 2522 20%
21 unit = 9450 1 unit = 2 Now,
1 unit = 450 8000 unit = 16000 20% = 1
5
100 unit = 45000 Let the CP = 5 unit and interest
Alternate : earned = 1 unit
1
5% = 20 ⇒ Amount after one year = 5+1 =
20 -------- 21
nn
6 unit
20 -------- 21 According to the question
20 -------- 21 6 unit = 750
_____________ 1 unit = 125
8000 ----- 9261 5 unit = 625
Now, According to the question
100 unit = 45000 (9261-8000) unit = 2522 Sol 34. (c)
1 unit = 450 1 unit = 2 10% = 10 1

14.49 unit = 6520.50 8000 unit = 16000


Pi

Let the CP = 103 = 1000 unit

Alternate : Sol 32. (d)


7
7% = 100 20%= 51
100 -------- 107 Let the principal = 52 = 25 unit
100 -------- 107
_____________
10000 ----- 11449
According to the question
10000 unit = 45000
1 unit = 4.5
(11449-10000) unit = 1449 x 4.5=
6520.50 Total interest earned = 5+5+1 = 11
unit

www.ssccglpinnacle.com support@ssccglpinnacle.com Ph. 09729327755, 09817390373


356
/
Days 40-42 Compound Interest

s
sse
la
_c
ob
Total interest earned = Compound Interest earned = (10000) unit = 10000 x 1.5=
bo
ah
100+100+10+ 21 (100+10+10+1) = 21+21+4.41 = 46.41 unit 15000
sm
ur
yo

270.5 unit Simple interest earned = 21+21 = Desired SI = 15000×21×2 = 6300


e/
t.m

100
://

Amount after 5 years = 42 unit


tp
ht

2
1000+270.5 = 1270.5 unit According to the question Sol 38. (a)
According to the question 46.41 unit = 9282 5% = 201

1000 unit = 2000 1 unit = 200 Let X = 202 = 400 unit


⇒ 1270.5 unit = 2541 42 unit = 8400

Sol 35. (b) Alternate :


21
21% = 100
Let the Principal = 100 unit
100 -------- 121
100 -------- 121
_____________
10000 ----- 14641

e
According to the question
(14641-10000) unit = 9282 According to the question
1 unit = 2 1 unit = 25
(10000) unit = 10000 x 2= 20000 400 unit = 10000
Compound Interest earned = Alternate :

l
Desired SI = 20000×21×2
100 = 8400
11+11+1.21 = 23.21 unit Difference between CI and SI =

22 unit
ac
Simple interest earned = 11+11 =

According to the question


23.21 unit = 6963
1 unit = 300
Sol 37. (a)
Let the Principal = 100 unit
R2
100
⇒ 25 =
% of Principal

Sol 39. (d)


52
100
⇒ Principal = 10000
% of Principal

22 unit = 6600
Since interest is compounded half
Alternate : yearly, effective rate of interest =
nn
8
2 = 4% and effective time period
11
11% = 100
17 x 2= 2 5 years
= 12
100 -------- 111 6
100 -------- 111 Compound Interest earned =
_____________ 21+21+4.41 = 46.41 unit
10000 ----- 12321 Simple interest earned = 21+21 =
According to the question 42 unit
(12321-10000) unit = 6963 According to the question
1 unit = 3 46.41 unit = 6961.5
Pi

(10000) unit = 10000 x 3= 30000 1 unit = 150


Desired SI = 30000×11×2
100 = 6600 42 unit = 6300

Alternate : Total interest earned =


Sol 36.(b)
21
21% = 100 400+400+16+ 65
Let the Principal = 100 unit
100 -------- 121 (400+16+16+0.64) = 1176.54
100 -------- 121 Therefore, Gain % =
1176.54 × 100 = 11.76
_____________ 10000

10000 ----- 14641


According to the question Sol 40. (c)
(14641-10000) unit = 6961.5 20% = 51
1 unit = 1.5 Let the principal = 52 = 25 unit

www.ssccglpinnacle.com support@ssccglpinnacle.com Ph. 09729327755, 09817390373


357
/
Days 40-42 Compound Interest

s
sse
la
_c
ob
3600
The amount becomes = 1.5
bo
2400
ah
sm
times in 6 years.
ur
yo
e/

⇒ In 12 years or (6 x 2 ) years the


t.m
://

amount will become (1.5)2 of the


tp
ht

initial amount.
⇒ Desired amount = 2400 x 2.25
= 5400
According to the question Total interest earned = 10+10+1 =
25 unit = 3000 Sol 43. (d) 21 unit
1 unit = 120 Let the principal = 1000 unit Amount after 2 years = 100+21 =
11 unit = 1320 121 unit
Alternate : According to the question
20% = 51 100 unit = 900
5 -------- 6 1 unit = 9
5 -------- 6 121 unit = 1089

e
_____________
25 ----- 36 Alternate :
1 ,
10% = 10
According to the question
25 unit = 3000 10 -------- 11

l
1 unit = 120 10 -------- 11
ac
(36-25) unit = 120 x 11= 1320 _____________
Total interest earned = 100 ----- 121
Sol 41. (a) 200+400+80+500+200+100+40 = According to the question
20% = 51 1520 100 unit = 900
Let the principal = 52 = 25 unit Amount earned in 3 years = 1 unit = 9
1000+1520 = 2520 121 unit = 121 x 9= 1089
According to the question
2520 unit = 5040 Sol 45. (a)
nn
1 unit = 2 25% = 41
1000 unit = 2000 Let the CP = 42 = 16 unit
Alternate :
20% = 51 , 40% = 52 and 50% = 21
5 -------- 6
According to the question 5 -------- 7
25 unit = 12000 2 -------- 3
1 unit = 480 _____________
Pi

11 unit = 5280 50 ----- 126


Alternate : Total interest earned = 4+4+1 = 9
According to the question
20% = 51 unit
126 unit = 5040
Amount after 2 years = 16+9 = 25
5 -------- 6 1 unit = 40
unit
5 -------- 6 50 unit = 50 x 40= 2000
According to the question
_____________ Sol 44. (b)
9 unit = 10125
25 ----- 36 10% = 10 1
1 unit = 1125
According to the question
Let the CP = 102 = 100 unit 25 unit = 28125
25 unit = 12000
1 unit = 480
Alternate :
(36-25) unit = 480 x 11= 5280
25% = 41 ,
Sol 42. (c) 4 -------- 5
4 -------- 5

www.ssccglpinnacle.com support@ssccglpinnacle.com Ph. 09729327755, 09817390373


358
/
Days 40-42 Compound Interest

s
sse
la
_c
ob
_____________ Total interest earned = (4 x 1000) 8000 unit = 1200
bo
ah 16 ----- 25 + (6 x 100) + (4 x 10) + 1 = 4641 3
1 unit = 20
sm
ur
yo

According to the question unit (9261) unit = 9261 × 3 =


e/
t.m

20
://

(25-16) unit = 10125 According to the question


tp

1389.15
ht

1 unit = 1125 10000 unit = 20000


25 unit = 25 x 1125 = 28125 1 unit = 2 Sol 49. (a)
4641 unit = 9282 12% = 25 3
Sol 46. (c) Note : Whenever rate % = 10
Let X = 252 = 625 unit
and time period = 4 years.
CI = 46.41% of Principle. To
save your precious time just
crame it.

Sol 48. (a)


Since interest is compounded half

e
yearly, effective rate of interest =
10 = 5% and effective time period
2 Total interest earned = 75+75+9 =
Total interest earned =
=1 21 x 2= 3 years 159 unit
100+100+10+500+50+50+5 = 1
5%= 20 According to the question
815 unit

l
Let the principle = 203 = 8000 159 unit = 577
ac unit 1 unit = 3
Alternate :
1 and 50% = 1 625 unit = 625 x 3 = 1875
10% = 10 2
Alternate :
10 -------- 11 3
12% = 25
10 -------- 11
25 -------- 28
2 --------- 3
25 -------- 28
_____________
_____________
200 ----- 363
625 ----- 784
According to the question
nn
According to the question
200 unit = 1000
784-625 unit = 577
1 unit = 5
1 unit = 3
(363-200) unit = 163 x 5= 815
(625) unit = 625 × 3= 1875
Sol 47.(c) Total interest earned =
Since interest is compounded half 400+400+20+400+20+20+1 = Sol 50. (a)
yearly, effective rate of interest = 1261 2 year compound interest = 2 year
20 = 10% and effective time According to the question simple interest + interest on 1st
Pi

2
8000 unit = 1200 year’s simple interest
period =2 x 2= 4 years
3
1 unit = 20 ⇒ interest on 1st year’s simple
Let the principle = 104 = 10000
(8000+1261) unit = 9261 × 3 = interest = 384-320 = 64
unit 20 64 x 100 =
Rate of interest = 160
1389.15
40%
Alternate : Now,
1
5% = 20 Difference between 2 year’s CI
R 2
20 -------- 21 and SI = P ( 100 )
20 -------- 21 Here P = principal and R is the
20 --------- 21 rate of interest
2
_____________ 40
⇒ 64 = P ( 100 )
8000 ----- 9261 ⇒ P = 400
According to the question

www.ssccglpinnacle.com support@ssccglpinnacle.com Ph. 09729327755, 09817390373


359
/
Days 40-42 Compound Interest

s
sse
la
_c
ob
1
Sol 51. (a) Rate of interest = [ ( PAmount Since interest is compounded half
bo
rinciple ) − 1
t

ah 1
yearly, the rate of interest = 10
2 =
sm
Rate of interest = [ ( PAmount ) −1
t
ur

rinciple ] x 100
yo

5% and time period =1 x 2= 2


e/
t.m

] x 100 Where t = time interval


://

years
tp

1
Where t = time interval
ht

1
= ( 13310
12100 ) − 1] x 100=
3
5%= 201
= ( 7986
6000 ) − 1] x 100=
3
10% Let the principle = 202 = 400 unit
10%
Sol 55. (a)
Sol 52. (d) Since interest is compounded half
12.5% = 81 yearly, effective rate of interest =
20 = 10% and effective time
Let the principle = 83 = 512 unit 2
period =2 x 2= 4 years
Let the principle = 104 = 10000
unit

Interest earned = 20+20+1 = 41

e
unit
the effective annual rate of interest
41 x 100 =10.25
= 400

l
Alternate :
ac
Total interest earned =
64+64+8+64+8+8+1 = 217 unit
According to the question
Total interest earned = (4 x 1000)
+ (6 x 100) + (4 x 10) + 1 = 4641
unit
1
5% = 20
20 -------- 21
20 -------- 21
_____________
400 ----- 441
512 unit = 5120 the effective annual rate of interest
According to the question
1 unit = 10 = 441400
−400 x 100 =10.25
10000 unit = 100000
217 unit = 2170
nn
1 unit = 10 Also,
4641 unit = 46410 the effective annual rate of interest
Alternate :
Note : Whenever rate % = 10 = 5+5+ 5×5
100 = 10.25
12.5% = 81
and time period = 4 years.
8 -------- 9 CI = 46.41% of Principle. To Sol 58. (a)
8 -------- 9 save your precious time just 1
5% = 20
8 -------- 9 crame it.
_____________ Let X = 202 = 400 unit
512 ----- 729
Pi

Sol 56.(b)
According to the question Let the principle = 100 unit
512 unit = 5120 ⇒ Amount after 2 years = 121
1 unit = 10
unit
(729-512) unit = 217 × 10 = 2170 1
Rate of interest = [ ( PAmount
rinciple ) − 1
t

Sol 53. (b) ] x 100


The amount becomes double in 4 Where t = time interval
1
years. 121 2
= ( 100 ) − 1] x 100= According to the question
Time take for the amount to 10% 1 unit = 125
become ( 23 times = 8 times) = 4 x Desired Simple Interest = 400 unit = 50000
3 = 12 years 1000×10×4 = 400 Alternate :
100
Difference between CI and SI =
Sol 54. (d) R2
Sol 57. (d) 100 % of Principal

www.ssccglpinnacle.com support@ssccglpinnacle.com Ph. 09729327755, 09817390373


360
/
Days 40-42 Compound Interest

s
sse
la
_c
ob
2
⇒ 125 = 5 % of Principal Case 2 : When interest is
bo
100
ah compounded semi-annually.
sm
⇒ Principal = 50000
ur
yo

Since interest is compounded half


e/
t.m
://

yearly, effective rate of interest =


tp

Sol 59. (c)


ht

20 = 10% and effective time


50% = 21 2

Let the principal = 22 = 4 unit period =2 x 2= 4 years


Let the principle = 104 = 10000
unit

Total interest earned =


2000+2000++200+2000+200+200
+20 = 6620

e
Alternate :
1
10% = 10
Total interest earned = (4 x 1000) 10 --------11
Total interest earned =
+ (6 x 100) + (4 x 10) + 1 = 4641 10 -------- 11
2+2+1+2+1+1+0.5 = 9.5

l
unit 10 -------- 11
According to the question
ac According to the question ____________
4 unit = 5000
10000 unit = 10000 1000 ------- 1331
1 unit = 1250
1 unit = 1 According to the question
(4+9.5) unit = 13.5 x 1250 =
4641 unit = 4641 1000 unit = 20000
16875
Note : Whenever rate % = 10 1 unit = 20
and time period = 4 years. (1331-1000) unit = 331 × 20 =
Alternate :
CI = 46.41% of Principle. To 6620
50% = 21
save your precious time just
2 -------- 3
nn
crame it. Sol 62. (d)
2 -------- 3
Note : Whenever rate % = 10
2 -------- 3 Desired difference = 4641-4400 = and time period = 4 years.
__________ 241 CI = 46.41% of Principle.
8 ------- 27
Desired CI = 46.41% of 5000
According to the question Sol 61.(a) ⇒ CI =5000 x 1004641
× 100 = 2320.50
8 unit = 5000 Let the principal = 100 unit and
1 unit = 625 rate of interest = r
27 unit = 27 × 625 = 16875 Sol 63. (d)
Pi

⇒ Interest earned in 5 years = 50 1


Rate of interest = [ ( PAmount
rinciple ) − 1
t

unit
Sol 60. (b) According to the question ] x 100
Case 1 : When interest is 50 = 100×r×5 Where t = time interval
100
compounded annually. 1

r = 10% = ( 4900
3600 ) − 1] x 100= 16
2

2 %
3

Sol 64. (d)


12% = 25 3

Let X = 253 = 15625 unit

Total interest earned =


2000+2000+400 = 4400

www.ssccglpinnacle.com support@ssccglpinnacle.com Ph. 09729327755, 09817390373


361
/
Days 40-42 Compound Interest

s
sse
la
_c
ob
25 ------- 27
bo
ah 25 ------- 27
sm
ur
yo

___________
e/
t.m
://

15625 19683
tp
ht

15625 unit = 15625


1 unit = 1
(19683-15625) 4058 unit = 4058 x
1 = 4058

Sol 66. (d)


10% = 10 1 Total interest earned =
Total interest earned = 1500+1500+150+ 52
Let the CP = 103 = 1000 unit
1875+1875+225+ 32 (1500+150+150+15)=3876
(1875+225+225+27) = 5543 Total Amount = 15000+3876 =
According to the question 18876

e
15625 unit = 31250
1 unit = 2 Sol 67. (c)
8% = 252
5543 unit = 5543 x 2 = 11086
Let the principal = 252 = 625 unit

l
Sol 65. (b)
Since interest is compounded 8
ac
monthly, effective rate of interest
8 = 8 % and effective time
= 12 × 12
period =2 × 12 = 3 years
8
Total interest earned =
100+100+10+100+10+10+1 = Interest earned = 50+50+4 = 104
Let the principle = unit
331 unit
253 = 15625 unit According to the question
Amount after 3 years = 1000+331
(625+104) unit = 24494.40
= 1331 unit
nn
1 unit = 33.6
According to the question
625 unit = 625 x 33.6 = 21000
1331 unit = 19965
1 unit = 15
Alternate :
1000 unit = 1000 x 15 = 15000 2
8% = 25
Alternate : 25 -------- 27
1
10% = 10 25 -------- 27
_____________
10 --------11
Pi

Interest earned = 625 ----- 729


10 -------- 11
1250+1250+100+1250+100+100+ According to the question
10 -------- 11
8 729 unit = 24494.40
____________
= 4058 unit 1 unit = 33.6
1000 ------- 1331
According to the question 625 unit = 625 x 33.6 = 21000
According to the question
15625 unit = 15625
1331 unit = 19965
1 unit = 1 Sol 68. (d)
1 unit = 15
4058 unit = 4058 Difference between 2 year’s CI
1000 unit = 1000 × 15 = 15000 2
R
Now, and SI = P ( 100 )
Alternate : Here P = principal and R is the
2
8% = 25 rate of interest
Principal Amount 20
⇒ 200 = P ( 100
2
)
25 ------- 27
⇒ P = 5000

www.ssccglpinnacle.com support@ssccglpinnacle.com Ph. 09729327755, 09817390373


362
/
Days 40-42 Compound Interest

s
sse
la
_c
ob
Total interest earned = As per given data, SI = P ×5×3 =
bo
100
ah
Sol 69. (b) 1200+1200+180+ 31
sm
₹1200
ur
yo

6.25% = 16 1
(1200+180+180+27) = 3109 unit P = 1200×100 = ₹ 8,000
e/
t.m

15
://

3 According to the question


Let the CP = 16 = 4096 5 3
tp

Amount = P(1+ 100 )


ht

3109 unit = 9327 =8000( 21 3


20 )
1 unit = 3
= ₹ 9261
8000 unit = 8000 x 3 = 24000
Compound interest = ₹
(9261-8000) = ₹ 1,261
Sol 71. (b)
Amount after two years ( A1 ) =
Sol 3. (a) Sum = ₹ 12,000
4000 Rate = 8%
Amount after four years ( A2 ) = Time = 2 85 years
6000
Required amount or Principle =
A1 ×A1 4000 × 4000
= = Rs. 2666.67

e
A2 6000
Total interest earned =
256+256+16+256+16+16+1 =
817 unit SSC CGL TIER I
According to the question

l
(4096+817) unit = 14739 Sol 1. (b) Rate per cent = 10%=
1 unit = 3 ac 1 Thus compound interest for 2 85
10
4096 unit = 4096 x 3 = 12288 Let principal be ₹ 1000 years = 960+960+76.8+699.84 =
Alternate : ₹2696.64 ~ ₹ 2697
6.25% = 16 1

16 --------17 Sol 4. (d) Rate of interest is


16 -------- 17 10%p.a.
16 -------- 17 For half yearly,
____________ Rate= 5+5+ 5×5
100 =10.25%
nn
4096 ------- 4913 Difference in interest due to .25%
According to the question = ₹88.80
4913 unit = 14739 Then 1% = ₹355.20
1 unit = 3 So, principal = ₹35520
4096 unit = 4096 × 3 = 12288 35520×10× 53
SI = 100 = ₹5,920

Sol 70. (a) For 2 years, compound interest = 10 2t


3
Sol 5. (c) 9261 = 8000(1+ 2×100 )
15% = 20 ₹210
Pi

⇒ t= 1.5 years
Let the principal = 203 = 8000 For 1/3year, CI= 121
3 = ₹40.33 Now at 20% rate, CI for 1.5 years
unit Total CI for 2 31 years at ₹1000 is :
₹250.33

But it is given that CI = ₹1201.60


Hence Principal = 1201.6
250.3 × 1000

= ₹ 4800

Sol 2. (d) Simple interest =


P ×R×T
100

www.ssccglpinnacle.com support@ssccglpinnacle.com Ph. 09729327755, 09817390373


363
/
Days 40-42 Compound Interest

s
se
s
la
_c
ob
Total compound interest = ⇒ P = ₹37500
bo
ah
1600+960 = 2560 Sol:3. (c)
sm
ur
yo

Amount = 4000+630.5 = 4630.5


e/
t.m

4630.5 = ( 1 + 5 )n
://

107 )3 = B( 107 )4
Sol 6. (d) A( 100
tp

100 4000 100


ht

9261 21 n
A:B = 107:100 8000 = ( 20 )
A+B = 207 units = ₹ 51750 21 )3 = ( 21 )n
( 20 20
1 unit = ₹ 250 n = 3 years
A borrowed = 107 units = ₹ 26750
Sol:4. (b)
Sol 7. (a) 73205 = 66550(1+ r )
100 Amount = 4000+630.5 = 4630.5
⇒ 1011 = 1+ r
100 4630.5 = (1 + r )3
4000 100
⇒ r = 10% 9261 r 3
8000 = (1 + 100 )

Sol 8. (b) Rate of interest is 8% r = 5%


Total CI = ₹ 7163 (approx)
per annum.
Sol:5.(a)

e
For half yearly, interest rate = 4% 6 )2
Sol 13. (c) 280900 = P(1+ 100 Amount = 405
Compound interest rate for half
106 )2 = 280900
⇒ P( 100 405 12.5 n
year = 4+4+ 4×4 320 = ( 1 + 100 )
16
100 = 8 100 %
⇒ P = ₹250000 81 9 n
64 = ( 8 )
Difference between rate = 16 %
100

l
Interest difference = 16 × 7800 = ( 89 )2 = ( 89 )2
100
Sol 14. (a) SI for 2 years = ₹400 n = 2 years
₹ 12.48 ac Then, SI for 1 year = ₹ 200
Sol 9. (c) For 2 years, SI rate = Sol:6. (d)
r×r = (2r+
2r% and CI rate = r+r+ 100 CI for 2 years = ₹ 408 5% = 1/20
r 2 This implies that ₹8 extra interest
100 ) %
Difference between interest rate = as CI is r% of ₹200.
r
r2 100 × 200 = 8
100 %
r2
25000 × 100 = 160 r = 4%
SSC CHSL 2019
nn
r = 8%
Sol:1.(c)
Sol 10. (d) rate = 4000×100 = 4% Effective rate when interest is
50000×2
compounded half yearly =
Compound interest = SI for 2 2.5+2.5+ 2.5×2.5
100 CI = 41 units = Rs.328
years+ 4% of SI for 1 year
1 units = Rs.8
SI remains equal for each year. =5+0.0625=5.0625% Sum = 400 units = 400
Thus SI for 1 year = ₹ 2000 Interest=4000 × 5.06%=202.5
4 × 2000 = ₹4080
× 8 = Rs.3200
CI = 4000+ 100 When interest is compounded
Pi

quarterly then effective rate for 4


Sol 7. (d)
Sol 11. (c) A1 = P(1+ r ) = 1250 years=46.41%
100
Interest=2000 × 46.41 Principle = Rs. 8000, Time = 2
r 3 100 =928.2
A3 = P(1+ 100 ) = 1458 years and Rate = 5% per annum
Total interest
r 2
⇒ (1+ 100 ) = 1458
1250 =928.2+202.5=1130.70 At 5% per annum, Simple interest
r = 27
⇒ 1+ 100 rate = 5% + 5% = 10%
25
⇒ r = 8% Sol:2.(c) At 5% per annum, Compound
8 ) = 1250
⇒ A1 = P(1+ 100 P=8820-820 = 8000 interest rate = 5% + 5% + 5100
×5 =

Ratio of P:A = 8000:8820 10.25%


⇒ P = ₹ 1250×100
108
1250×100
= 4000:4410 Interest rate difference = 10.25 -
P ×r×t ×8× 27
SI = 100 = 108
100
5
= ₹ 500 For 2 years 10 = 0.25%

P ×r×t
√4000 : √4410 Required interest difference =
Sol 12. (c) SI = 25
100 20:21 100 × 100 × 8000 = Rs. 20
⇒ 6750 = P 100
×6×3 1 × 100 =5%
R= 20

www.ssccglpinnacle.com support@ssccglpinnacle.com Ph. 09729327755, 09817390373


364
/
Days 40-42 Compound Interest

s
se
s
la
_c
ob
Sol: 8. (c) Formulae =>Interest for 2 year =
bo
ah
A = P (1 + r t ab
100 ) Sol:12.(b) a+b+ 100
sm
ur
yo

⇒ 4630.08 = P (1 + 4 )(1 + 5 P = 5500 At 10% amount = 121%


e/
t.m

100 100
://

)(1 + ) 6 T = 2 × 23 = 3 Interest = 21% = 1050


tp
ht

100
104 105 Rate = 15 × 2 = 10 Amount = 5000
⇒ 4630.08 = P ( 100 )( 100 )( 3
amount = 5500 × (1 + 10 3 At 12%
100 ) =
106 )
100
100 100 7320.5 Interest = 12 +12 12×12
100 = 25.44
⇒ P = 4630.08 × ( 104 )( 105 )(
100 CI = 7320.5 - 5500 = 1820.50 Interest = 25.44% of 5000 = 1272
106 ) = 4000

Sol:13.(b) SSC CPO 2019


Sol:9 . (a) Sol:18.(b)
Compound interest for first 2 year
Monthly compounding factor for r ) 2 = 26450 Amount at the end of 4
= 20000 (1 + 100
Rs.1 years=7,562
For next 8 month
((1+ (0.065÷12)) = 1.0054166667 Amount at the end of 5
r=10%
Maturity factor for 18 months. years=8,469.44
t=1

e
1.0054166667 × 18= 10 Net interest= 907.44
Interest = 26450 × 100 = 2645
1.1021214212 Rate of interest= 907.44
7562 × 100
Total interest = 2645+ 6450 =
This is value for ₹1. =12%
9095
Maturity value for ₹8,000,.

l
Sol:19..(a)
1.1021214212 x 8000 = 8,816.97 Sol:14.(a)
ac SI for 2 years = 34
10% in ratio = 10: 11 289
CI for 2 years = 17 + 17 + =
SSC CGL 2019 TIER-II P : I for first year after making 100

installment equal = 110 :121 36.89%


Sol:10.(a)
15500×100 P : I for second year = 100 : 121 Total difference = 2.89%
P = 100+(2×12)
Interest in second year = 21 According to the question
P= 1550000 = 12500
124 Interest for first year = 11 2.89% = 433.5
Time for CI = 3 Total interest = 32 100% = 15,000
Rate for CI = 5% Interest = 12132 × 5808 = 1536
nn
r )n
A = P (1 + 100 Sol:20.(a)
= 105 ) 3
12500( 100 SI for 2 years = 34
Sol:15(a) 289
= 21 × 21
12500 × 20 × 21 CI for 2 years = 17 + 17 + =
20 20 Interest = 13650-10500 = 3150 100

Rate = si×100 3150×100 36.89%


= 12500 × 9261
8000 p×t = 10500×2 = 15%
Total difference = 2.89%
= 14470.31 ≈ ₹14470 For CI
According to the question
R =7.5%
2.89% = 433.5
Sol:11.(b) t=2
107.5 107.5 100% = 15,000
Pi

107 21 % in ratio = 43 : 40 Amount = 10500 × 100 × 100 =


36.89% = 5,533.50
For first year = 40 : 43 12134.06
For second year = 40 2 : 43 2 Option A is correct Sol:21.(c)
=1600 : 1849 Simple interest of 2 years=8100
To make the installment equal Sol:16.(a) Simple interest of 1 years=4050
Amount = p × (1+ r n
multiply first year by 43 = 1720 : 100 ) Compound interest for 2nd
1849 21952 = 15625( 100+r ) 3
100 year=4050+(10% of 4050)
According to question 21952 = ( 100+r 3 =4050+405=4455
15625 100 )
1849x = 5547 112 3 3 Total compound interest for 2
( 100 ) = ( 100+r
100 )
x=3 years =4050+4455=8505
Interest of first year = 129x r = 12%
Interest of second year = 249x Sol:22.(c)
Total interest = 378x Sol:17.(d)
Amount received at the end of 4
Interest = 378 × 3 = 1134 years=7562

www.ssccglpinnacle.com support@ssccglpinnacle.com Ph. 09729327755, 09817390373


365
/
Days 40-42 Compound Interest

s
sse
la
_c
ob
Amount received at the end of 5
bo
ah
years=8469.44
sm
ur
yo

Total interest earned in 1 year=


e/
t.m
://

907.44
tp
ht

Rate of interest= 907.44


7562 × 100
=12%
Compound interest received in 2
years at the rate of 12% = 12+12+
12×12
100
=25.44%
Compound interest on 10000 rs
will be 25.44% × 10000
=2540

Sol:23.(a)

e
Simple interest of 2 years=8100
Simple interest of 1 years=4050
Rate of interest=10%
Now rate of interest =15%

l
So Simple interest = 4050
10 ×
15=6075 ac
Compound interest for 2nd
year=6075+(15% of 6075)
=6075+911.25=6986.25
Total compound interest for 2
years =6075+6986.25=13061.25
nn
Pi

www.ssccglpinnacle.com support@ssccglpinnacle.com Ph. 09729327755, 09817390373


366
/
Days 43-46 Work and Time

s
sse
la
_c
ob
WORK AND TIME / काय िदनों म कर सकता है | यिद काय 11 31 Q6. A earns Rs 180 per hour and
bo
ah works for 7 hours per day. B earns
sm
और समय िदनों म समा होता है , तो B अकेले 4
ur
yo

गुना काय िकतने िदनों म करे गा ? Rs. 160 per hour and works for 5
e/
t.m
://

SSC CGL 7 June 2019 (Afternoon) hours per day. what is the ratio of per
tp

Variety Questions
ht

(a) 24 days day wages of A and B? / A 180


(b) 32 days पये ित घंटे कमाता है तथा एक िदन
Q1. The efficiencies of A , B and C
(c) 27 days म 7 घंटे काय करता है | B 160 पये
are in the ratio of 2 : 3 : 5. Working
(d) 30 days ित घंटे कमाता है और ितिदन 5 घंटे
together, they can complete a task in
काय करता है | A और B के दै िनक
6 days. In how many days will A
Q4. A is 40% more efficient than B वेतन का अनुपात ा है ?
alone complete 20% of the task?
and C is 20% less efficient than B. SSC CGL 13 June 2019 (Evening)
A, B और C की काय मता का
Working together, they can finish a (a)40:61
अनुपात 2 : 3 : 5 है | एक साथ काय
work in 5 days. In how many days, (b)33:20
करते ए, वे िकसी काय को 6 िदनों म
will A alone complete 70% of the (c)20:30
पूरा कर सकते ह | A 20% काय िकतने
work? (d)63:40
िदनों म पूरा करे गा ?

e
SSC CGL 12 June 2019 A, B से 40% अिधक काय कुशल है
तथा C, B से 20% कम काय कुशल है | Q7. A and B can complete a task in
(Afternoon)
एक साथ काय करते ए, वे िकसी काय 25 days. B alone can complete
(a) 8
को 5 िदनों म पूरा कर सकते ह | A इस 33 31 % of the same task in 15 days.
(b) 5

l
(c) 6 काय का 70% भाग अकेले िकतने िदनों In how many days can A alone
(d) 4 ac म पूरा करे गा ? 4 th of the same task? / A
complete 15
SSC CGL 10 June 2019 और B िकसी काय को 25 िदनों म पूरा
Q2. The ratio of the efficiencies of (Afternoon) कर सकते ह | B इसी काय का 33 31 %
A, B and C is 2 : 5 : 3. Working (a) 9 भाग अकेले 15 िदनों म पूरा कर सकता
together, they can complete a work (b) 7 है | A इस काय का 15 4 th भाग िकतने

in 27 days. B and C together can (c) 10


िदनों म पूरा करे गा ?
complete 4/9th part of that work in: (d) 8
SSC CHSL 1 July 2019 (Evening)
A, B और C की काय मता का (a) 15
Q5. 3 men, 4 women and 6 boys
nn
अनुपात 2 : 5 : 3 है | एक साथ काय (b) 10
करते ए, वे िकसी काय को 27 िदन म together can complete a work in 6
(c) 18
पूरा कर सकते ह | B और C एक साथ days. A woman does triple the work
(d) 12
उस काय के 4/9 भाग को िकतने िदनों म a man does and a boy does half the
पूरा करगे ? work a man does. How many women
Q8. A , B and C can finish a task in
SSC CGL 4 June 2019 (Morning) alone will be able to complete this
42 days, 84 days and 28 days,
(a) 27 days work in 4 days?
respectively. A started the work. B
(b) 15 days 3 पु ष 4 मिहलाएं तथा 6 लड़के एक
joined him after 3 days. If C joined
साथ िकसी काय को 6 िदन म कर सकते
Pi

(c) 17 71 days them after 5 days from the


ह| एक मिहला एक पु ष की तुलना म
(d) 24 days beginning, then for how many days
ितगुना काय करती है तथा एक लड़का
did A work till the completion of the
एक पु ष की तुलना म आधा काय
Q3. To do a certain work , A and B task?
करता है | इस काय को 4 िदनों म करने
work on alternate days, with B A, B और C िकसी काय को मशः 42
के िलए िकतनी मिहलाओं की
beginning the work on the first day. िदन, 84 िदन और 28 िदन म कर सकते
आव कता होगी ?
A can finish the work alone in 48 ह | A ने काय शु िकया तथा B 3 िदन
SSC CGL 13 June 2019 (Evening)
days. If the work gets completed in बाद उसम शािमल आ | यिद C ने
(a)9
11 31 days, then B alone can finish 4 आर से 5 िदनों के बाद काय करना
(b)6
times the same work in : / िकसी शु िकया, तो A काय पूरा होने तक
(c)8
िनि त काय को करने के दौरान A और िकतने िदनों तक काय करता रहा ?
(d)7
B एक के बाद एक िदन काय करते ह SSC CHSL 2 July 2019
तथा पहले िदन काय की शु आत B (Afternoon)
करता है | A इस काय को अकेले 48 (a) 20

www.ssccglpinnacle.com support@ssccglpinnacle.com Ph. 09729327755, 09817390373 367


/
Days 43-46 Work and Time

s
se
s
la
_c
ob
(b) 15 मिहलाओं को िकसी काय को समा (d) Rs. 2700
bo
ah
(c) 17 करने म 2 िदन लगते ह, तो एक मिहला
sm
ur
yo

(d) 18 अकेले काय करते ए इस काय को Q14. A can complete a piece of


e/
t.m

िकतने िदनों म समा कर सकती है ?


://

work in 20 days and B can complete


tp
ht

Q9. A and B , working together, can SSC CHSL 10 July 2019 (Evening) 20% of the work in 6 days. If they
complete a work in 16 days, C and A (a) 12 21 work together in how many days can
together can complete it in 32 days (b) 16 they finish 50% of the work, if they
and C and B together can complete it (c) 10 31 work together ?
in 24 days. They worked together for (d) 8 A िकसी काय को 20 िदनों म तथा B
12 days. In how many days will C इस काय के 20% भाग को 6 िदनों म
can complete the remaining work ? / Q12. A and B can complete a piece पूरा कर सकता है | यिद वे साथ काय
A और B एक साथ काय करते ए of work in 15 days and 10 days कर, तो वे इस काय का 50% भाग
िकसी काय को 16 िदनों म कर सकते ह respectively. They got a contract to िकतने िदनों म पूरा करगे ? SSC CHSL
, C और A इस काय को 32 िदनों म कर complete the work for Rs. 35,000. 11 July 2019 (Morning)
सकते ह और C और A इस काय को 32 The share of A in the contracted (a) 12
िदनों म कर सकते ह | उ ोंने 12 िदनों

e
money will be : (b) 6
तक साथ काय िकया | शेष काय समा A और B िकसी काय को मशः 15 (c) 8
करने म C को िकतने िदन लगगे ? SSC िदन और 10 िदन म कर सकते ह | उ (d) 9
CHSL 2 July 2019 (Evening) 35000 पये म यह काय पूरा करने का

l
(a) 40 अनुबंध िमला | इस रािश म A का िह ा Q15. A, B and C, alone can do a
(b) 36 होगा : piece of work in 9, 12 and 18 days
(c) 45
(d) 32
ac
Q10. 18 men can complete a work in
9 days. After they have worked for 5
SSC CHSL 8 July 2019 (Morning)
(a) 7000
(b) 15000
(c) 14000
(d) 21000
respectively. They all started the
work together, but A left after 3
days. In how many days, was the
remaining work completed ?
A, B और C अकेले िकसी काय को
days, 6 more men join them. How मशः 9, 12 और 18 िदनों म कर सकते
many days will they take to Q13. Rs. 10,000 has to be distributed ह | उन सभी ने साथ काय शु िकया,
complete the remaining work ? लेिकन 3 िदन के बाद A ने काय छोड़
nn
among 3 craftsmen, 5 helpers and 6
18 पु ष िकसी काय को 9 िदनों म पूरा labourers such that each helper िदया | शेष काय िकतने िदनों म समा
कर सकते ह | उनके 5 िदनों तक काय receives the amount twice as much आ?
कर लेने के बाद, 6 अित र पु ष as a labourer receives and each SSC CHSL 11 July 2019 (Evening)
उनके साथ हो गए | शेष काय करने म craftsman receives the amount thrice (a) 2
उ िकतना समय लगेगा ? as much as a labourer receives. What (b) 25
SSC CHSL 3 July 2019 (Evening) is the amount received by the three (c) 11
4
(a) 3 craftsmen ? (d) 9
5
(b) 2 21 10000 पये का िवतरण 3 िश कारों, 5
Pi

(c) 2 सहायकों तथा 6 िमकों के बीच इस Q16. A and B can finish a work
(d) 3 21 कार करना है िक ेक सहायक को together in 30 days, B and C can
िमलने वाली रािश एक िमक को िमलने finish the same work together in 24
Q11. It is given that men are twice as वाली रािश से दोगुनी हो तथा ेक days and A and C can finish the
efficient than women in respect to िश कार को िमलने वाली रािश एक same work together in 40 days. If all
doing work. If three men and two िमक को िमलने वाली रािश से ितगुनी three work together, how long will it
women can complete the work in 2 हो | तीनों िश कारों ारा ा की गयी take them to complete the work?
days, then in how many days can a रािश ात कर | A और B िकसी काय को 30 िदनों म
woman working alone complete the SSC CHSL 10 July 2019 समा कर सकते ह | B और C इसी
work? (Afternoon) काय को 24 िदनों म तथा A और C इसी
यह िदया गया है िक िकसी काय को (a) Rs. 2400 काय को एक साथ 40 िदनों म समा
करने म पु ष मिहलाओं से दोगुने काय (b) Rs. 4000 कर सकते ह | यिद ये तीनों साथ काय
कुशल ह | यिद तीन पु ष और दो (c) Rs. 3600

www.ssccglpinnacle.com support@ssccglpinnacle.com Ph. 09729327755, 09817390373 368


/
Days 43-46 Work and Time

s
sse
la
_c
ob
कर, तो उ काय समा करने म काय को शु िकया | कुछ िदनों बाद C (d) 15
bo
ah
िकतना समय लगेगा ? उनके साथ शािमल हो गया और इस
sm
ur
yo

SSC CPO 16 March 2019 कार उन सभी ने इस पूरे काय को 10 SSC CGL TIER II
e/
t.m

िदनों म पूरा िकया | उन सभी को कुल


://

(Morning) Q1. To do a certain work, the ratio of


tp
ht

(a) 20 days 700 पये का भुगतान िकया गया | C का efficiency of A to that of B is 3:7.
(b) 5 days िह ा ा है ? Working together, they can complete
(c) 10 days SSC CPO 16 March 2019 the work in 10 21 days. They work
(d) 15 days (Evening) together for 8 days. 60% of the
(a) 55 remaining work will be completed by
Q17. A can do work in 12 days. B (b) 65 A alone in : िकसी काय को करने म, A
can do work in 18 days. After 5 days (c) 75 की काय मता तथा B की काय मता
of working together, how much work (d) 70 का अनुपात 3: 7 है | एक साथ काय करते
will be left? ए, वे इस काय को 10 21 िदनों म कर
A िकसी काय को 12 िदनों म कर सकता Q20. The ratio of efficiencies of A , सकते ह | वे 8 िदनों तक साथ काय
है | B िकसी काय को 18 िदनों म कर B and C is 7 : 5 : 8. Working करते ह | शेष काय का 60% िह ा A
सकता है | 5 िदन तक एक साथ काय

e
together, they can complete a piece अकेले िकतने िदनों म पूरा करे गा ? SSC
करने के बाद िकतना काय शेष रह of work in 42 days. B and C worked CGL TIER II (11 September 2019)
जाएगा ? together for 21 days and the (a) 5 21 days
SSC CPO 16 March 2019 remaining was completed by A
(b) 5 days

l
(Morning) alone. The whole work was
5
(c) 6 21 days
(a) 12 completed in :
(b)
(c)
(d)
5
13
7
25
11
36
ac A, B और C की काय मता का
अनुपात 7 : 5 : 8 है | एक साथ काय
करते ए, वे िकसी काय को 42 िदनों म
कर सकते ह | B और C ने एक साथ 21
िदनों तक काय िकया और शेष काय A
(d) 4 days

Q2. A and B can do a piece of work


in 6 days and 8 days, respectively.
With help of C, they completed the
Q18. 6 men or 5 women earn Rs work in 3 days and earned Rs. 1848.
ने पूरा िकया | पूरा काय िकतने िदनों म
14,820 in two days. How much will What was the share of C ?
समा आ?
4 women and 6 men earn in one A और B िकसी काय को मशः 6 और
SSC CGL 7 June 2019 (Morning)
nn
day? 8 िदन म कर सकते ह | C की सहायता
(a) 96
6 पु ष अथवा 5 मिहलाएं दो िदनों म से, उ ोंने इस काय को 3 िदनों म पूरा
(b) 99
14,820 पये कमाते ह | एक िदन म 4 िकया और 1848 पये कमाए | C का
(c) 102
मिहलाएं तथा 6 पु ष िकतना कमाएं गे ? िह ा ात कर | SSC CGL TIER II
(d) 93
SSC CPO 14 March 2019 (11 September 2019)
(Morning) (a) Rs. 231
Q21. A can do 40% of a work in 6
(a)13,338 (b) Rs. 924
days and B can do 30% of the same
(b)13,832 (c) Rs. 462
work in 3 days. They started the
Pi

(c)26,676 (d) Rs. 693


work together but B left after 2 days
(d)27,664
and A continued to work. In how
Q3. A certain number of persons can
many days was the entire work
Q19. A can do a work in 20 days, complete a work in 34 days working
completed ?
while B can do the same work in 25 9 hours a day. If the number of
A िकसी काय का 40% भाग 6 िदनों म
days. They started the work jointly. persons is decreased by 40%, then
तथा B इसी काय का 30% भाग 3 िदनों
Few days later C also joined them how many hours a day should the
म पूरा कर सकता है | उ ोंने एक साथ
and thus all of them completed the remaining persons work to complete
काय शु िकया लेिकन 2 िदनों के बाद
whole work in 10 days. All of them the work in 51 days ?
B हट गया और A ने काय जारी रखा |
were paid total of rs 700. What is the कुछ िकसी काय को एक िदन म
पूरा काय िकतने िदनों म समा आ?
share of C? 9 घंटे काय करते ए 34 िदनों म पूरा
SSC CHSL 4 July 2019 (Morning)
A िकसी काय को 20 िदनों म कर सकता कर सकते ह | यिद यों की सं ा
(a) 10
है जबिक B इसी काय को 25 िदनों म 40% से कम कर दी जाए, तो शेष
(b) 12
कर सकता है | उ ोंने एक साथ इस यों को इस काय को 51 िदनों म
(c) 9

www.ssccglpinnacle.com support@ssccglpinnacle.com Ph. 09729327755, 09817390373 369


/
Days 43-46 Work and Time

s
sse
la
_c
ob
पूरा करने के िलए िदन म िकतने घंटे Q6. A can do 40% of a work in 12 A िकसी काय का एक-ितहाई भाग 15
bo
ah
काय करना पड़े गा ? days, whereas B can do 60% of the िदनों म पूरा कर सकता है | B इसी काय
sm
ur
yo

SSC CGL TIER II (11 September same work in 15 days. Both work का 75% भाग 18 िदनों म तथा C यही
e/
t.m

काय 36 िदनों म कर सकता है | B और


://

2019) together for 10 days. C completes


tp
ht

(a) 9 the remaining work alone in 4 days. C 8 िदनों तक साथ काय करते ह | A
(b) 8 A, B and C together will complete अकेला शेष काय िकतने िदनों म पूरा
(c) 12 28% of the same work in : करे गा ?
(d) 10 A िकसी काय का 40% भाग 12 िदनों म SSC CGL TIER II (13 September
कर सकता है जबिक B इसी काय का 2019)
Q4. 4 men and 5 women can 60% भाग 15 िदनों म कर सकता है |
complete a work in 15 days, दोनों 10 िदनों तक एक साथ काय करते (a) 24 days
whereas 9 men and 6 women can ह | C शेष काय 4 िदनों म पूरा करता है | (b) 18 days
do it in 10 days. To complete the A, B और C एक साथ इसी काय का (c) 20 days
same work in 7 days, how many 28% भाग िकतने िदनों म पूरा करगे ? (d) 16 days
women should assist 4 men? SSC CGL TIER II (12 September
4 पु ष और 5 मिहलाएं िकसी काय को

e
2019) Q9. 25 persons can complete a work
15 िदनों म समा कर सकती ह जबिक (a) 2 21 days in 60 days. They started the work. 10
9 पु ष और 6 मिहलाएं इसी काय को (b) 3 days people left the work after x days. If
10 िदनों म कर सकती ह | इसी काय को (c) 1 21 days the whole work was completed in 80
7 िदनों म पूरा करने के िलए, िकतनी

l
(d) 2 days days, then what is the value of x ?
मिहलाओं को 4 पु षों की सहायता 25 िकसी काय को 60 िदनों म
करनी चािहए ?

2019)
(a) 11
(b) 14
ac
SSC CGL TIER II (12 September
Q7. A is as efficient as B and C
together. Working together A and B
can complete a work in 36 days and
C alone can complete it in 60 days.
A and C work together for 10 days.
पूरा कर सकते ह | उ ोंने काय शु
िकया | x िदनों के बाद 10 लोगों ने काय
छोड़ िदया | यिद पूरा काय 80 िदनों म
पूरा आ, तो x का मान ात कर |
SSC CGL TIER II (13 September
(c) 12 B alone will complete the remaining 2019)
(d) 13 work in :
A की काय मता B और C की कुल (a) 9
nn
Q5. To do a certain work, the ratio of काय मता के बराबर है | एक साथ (b) 8
the efficiencies of X and Y is 5:4. काय करते ए, A और B िकसी काय (c) 12
Working together, they can complete को 36 िदनों म कर सकते ह तथा C (d) 15
the same work in 10 days. Y alone अकेले इस काय को 60 िदनों म कर
starts the work and leaves after 5 सकता है | A और C 10 िदनों तक एक Practice Questions
days. The remaining work will be साथ काय करते ह | B अकेले शेष काय
completed by X alone in : िकतने िदनों म समा करे गा ? Q1. The ratio of the efficiencies of
िकसी िनि त काय को करने के िलए, X A, B and C is 4 : 5 : 3. Working
Pi

SSC CGL TIER II (13 September


और Y की काय मताओं का अनुपात 5 2019) together, they can complete that
: 4 है | एक साथ काय करते ए वे इस (a) 110 days work in 25 days. A and C together
काय को 10 िदनों म समा कर सकते (b) 88 days will complete 35% of that work in:
ह | Y अकेले काय शु करता है और 5 (c) 84 days A, B तथा C की काय मता का
िदनों के बाद छोड़ दे ता है | X अकेले (d) 90 days अनुपात 4 : 5 : 3 है | एक साथ काय
शेष काय िकतने िदनों म पूरा करे गा ? करते ए वे 25 िदनों म काय पूरा कर
SSC CGL TIER II (12 September Q8. A can do one-third of a work in लेते ह | इस काय का 35% भाग A और
2019) 15 days, B can do 75% of the same C िकतने िदनों म पूरा करगे ?
(a) 14 days work in 18 days and C can do the SSC CGL 4 June 2019 (Afternoon)
(b) 12 days same work in 36 days. B and C work (a) 12 days
(c) 15 days together for 8 days. In how many (b) 10 days
(d) 10 days days will A alone complete the (c) 18 days
remaining work ? (d) 15 days

www.ssccglpinnacle.com support@ssccglpinnacle.com Ph. 09729327755, 09817390373 370


/
Days 43-46 Work and Time

s
sse
la
_c
ob
Q2. The ratio of the efficiencies of (a) 60 days A, B से 40% अिधक काय कुशल है
bo
ah
A, B and C is 7:5:4. Working (b) 57 days तथा C, B से 20% कम काय कुशल है |
sm
ur
yo

together, they can finish work in 35 (c) 54 days एक साथ काय करते ए, वे िकसी काय
e/
t.m

को 15 िदनों म समा कर सकते ह | B


://

days. A and B work together for 28 (d) 50 days


tp
ht

days. The remaining work will be अकेले इस काय का 75% भाग िकतने
completed (in days) by C alone: Q5. The ratio of efficiencies of A , B िदनों म पूरा करे गा ?
A, B और C की काय मता का and C is 3 : 5 : 1. Working together, SSC CGL 11 June 2019 (Morning)
अनुपात 7 : 5 : 4 है | एक साथ काय they can complete a piece of work in (a) 36
करते ए, वे िकसी काय को 35 िदनों म 5 days. A and B worked together for (b) 48
समा कर सकते ह | A और B एक 3 days. The remaining will be (c) 32
साथ 28 िदनों तक काय करते ह | C शेष completed by C alone in : (d) 44
काय िकतने िदनों म पूरा करे गा ? A, B और C की काय मता का
SSC CGL 4 June 2019 (Evening) अनुपात 3 : 5 : 1 है | एक साथ काय Q8. A is 40% more efficient than B
(a) 56 करते ए, वे िकसी काय को 5 िदनों म and C is 20% less efficient than B.
(b) 63 पूरा कर सकते ह | A और B ने एक साथ Working together, they can complete
3 िदनों तक काय िकया | C को अकेले

e
(c) 49 a task in 20 hours. In how many
(d) 60 शेष काय करने म िकतना समय ( िदन ) hours, will A alone can complete
लगेगा ? 35% of the task?
Q3. The efficiences of A , B and C SSC CGL 7 June 2019 (Evening) A, B से 40% अिधक काय कुशल है
तथा C, B से 20% कम काय कुशल है |

l
are in ratio 5 : 3 : 8. Working (a) 18 Days
together they can complete a work in (b) 24 Days एक साथ काय करते ए, वे िकसी काय
ac
30 days. A and B worked together
for 20 days. The remaining work
will be completed by C alone in :
A, B और C की काय मता 5 : 3 : 8 है
| एक साथ काय करते ए, वे िकसी काय
(c) 21 Days
(d) 15 Days

Q6. The ratio of the efficiencies of A


, B and C to do a certain work is 7 :
को 20 घंटों म पूरा कर सकते ह | A को
अकेले इस काय का 35% भाग पूरा
करने म िकतने घंटे लगगे ?
SSC CGL 11 June 2019
(Afternoon)
को 30 िदनों म कर सकते ह | A और B 3 : 5. Working together, they can (a) 13
ने 20 िदनों तक काय िकया | शेष काय complete work in 21 days. A and C (b) 15
पूरा करने म C को अकेले िकतने िदन worked together for 15 days. The (c) 16
nn
लगगे ? remaining work will be completed (d) 14
SSC CGL 6 June 2019 (Afternoon) by B alone in :
(a) 40 days A, B और C की काय मता का Q9. A is 50% more efficient than B
(b) 36 days अनुपात 7 : 3 : 5 है | एक साथ काय and C is 40% less efficient than B.
(c) 30 days करते ए, वे िकसी काय को 21 िदनों म Working together, they can complete
(d) 32 days पूरा कर सकते ह | A और C ने एक साथ a task in 10 days. In how many days,
15 िदनों तक काय िकया | शेष काय पूरा will A alone complete 150% of that
Q4. To do a certain work, the ratio of करने म B को अकेले िकतने िदन लगगे task?
Pi

the efficiencies of A , B and C is 7 : ? A, B से 50% अिधक काय कुशल है


5 : 6. Working together, they can SSC CGL 10 June 2019 (Morning) और C, B से 40% कम काय कुशल है |
complete the same work in 35 days. (a) 54 days एक साथ काय करते ए वे िकसी काय
B and C worked together for 21 (b) 45 days को दस िदनों म पूरा कर सकते ह | इस
days. The remaining work will be (c) 60 days काय का 150% भाग पूरा करने म A को
completed by A alone in : (d) 63 days अकेले िकतने िदन लगगे ?
एक िनि त काय करने के िलए, A, B SSC CGL 11 June 2019 (Evening)
और C की काय मता का अनुपात 7 : 5 Q7. A is 40% more efficient than B (a) 33
: 6 है | एक साथ काय करते ए, वे इसी and C is 20% less efficient than B. (b) 35
काय को 35 िदनों म पूरा कर सकते ह | Working together, they can finish a (c) 28
B और C ने 21 िदनों तक काय िकया | task in 15 days. In how many days, (d) 31
शेष काय पूरा करने म A को अकेले will B alone will complete 75% of
िकतने िदन लगगे ? the task? Q10. A is 50% more efficient than B
SSC CGL 6 June 2019 (Evening) and C is 40% less efficient than B.

www.ssccglpinnacle.com support@ssccglpinnacle.com Ph. 09729327755, 09817390373 371


/
Days 43-46 Work and Time

s
sse
la
_c
ob
Working together, they can complete Q13. The efficiencies of A, B and C (b) 3 51 Days
bo
ah
a task in 20 days. In how many days are in the ratio of 5:3:2. Working
sm
(c) 16 Days
ur
yo

will C alone complete 30% of that together, they can complete a task in (d) 5 31 Days
e/
t.m
://

task? 21 hours. In how many hours will B


tp
ht

A, B से 50% अिधक काय कुशल है alone complete 40% of that task? Q16. A, B and C can complete a
तथा C, B से 40% कम काय कुशल है | A, B और C की काय मता का piece of work in 10, 20 and 60
एक साथ काय करते ए, वे िकसी काय अनुपात 5 : 3 : 2 है | एक साथ काय respectively. Working together, they
को 20 िदनों म पूरा कर सकते ह | C करते ए, वे िकसी काय को 21 घंटों म can complete the same work in how
अकेले इस काय का 30% भाग िकतने कर सकते ह | इस काय का 40% भाग many days ?
िदनों म पूरा करे गा ? पूरा करने म B को अकेले िकतने घंटे A, B और C िकसी काय को मशः 10,
SSC CGL 12 June 2019 (Morning) लगगे ? 20 और 60 िदनों म पूरा कर सकते ह |
(a) 31 SSC CGL 13 June 2019 एक साथ काय करते ए, वे इसी काय
(b) 33 (Afternoon) को िकतने िदनों म पूरा करगे ?
(c) 35 (a)28 SSC CHSL 4 July 2019
(d) 29 (b)24 (Afternoon)

e
(c)35 (a) 5
Q11. The efficiencies of A , B and C (d)21 (b) 6
are 2 : 5 : 3. Working together, they (c) 10
can complete a task in 9 days. In Q14. If 30 persons take 10 days to (d) 8

l
how many days will C alone can complete a certain work working 8
complete 40% of that task? hours a day. Then 40 persons should Q17. A, B and C can complete a
ac
A, B और C की काय मता का
अनुपात 2 : 5 : 3 है | एक साथ काय
करते ए वे िकसी काय को 9 िदनों म
पूरा कर सकते ह | C को अकेले इस
काय का 40% भाग पूरा करने म िकतने
work how many hours a day so that
the work is completed in 6 days?
यिद 30 लोग ितिदन 8 घंटे काय करके
िकसी काय को 10 िदनों म पूरा करते ह,
तो इस काय को 6 िदनों म पूरा करने के
piece of work in 4, 28 and 56
respectively. Working together, they
can complete the same work in how
many days ?
A, B और C िकसी काय को मशः 4,
िदन लगगे ? िलए 40 िदन म िकतने घंटे काय 28 और 56 िदनों म पूरा कर सकते ह |
SSC CGL 12 June 2019 (Evening) करगे ? एक साथ काय करते ए, वे इस काय को
(a) 14 SSC CHSL 2 July 2019 (Morning) िकतने िदनों म पूरा करगे ?
nn
(b) 16 (a) 6 SSC CHSL 4 July 2019 (Evening)
(c) 15 (b) 10 5
(a) 5 17
(d) 12 (c) 8 1
(b) 3 17
(d) 12 1
(c) 5 17
Q12. The efficiencies of A, B and C
5
(d) 3 17
are in the ratio 2:5:3. Working Q15. A and B together can do a
together, they can complete a task in piece of work in 10 days, B and C
12 days. In how many days can A together can do it in 15 days while C Q18. A, B and C can complete a
Pi

alone complete 30% of that task? and A together can do it in 20 days. piece of work in 5, 20 and 60 days
A, B और C की काय मता का They work together for 8 days. C respectively. Working together, they
अनुपात 2 : 5 : 3 है | एक साथ काय alone will complete the remaining can complete the same work in how
करते ए, वे िकसी काय को 12 िदनों म work in: many days ?
पूरा कर सकते ह | A अकेले इस काय A और B एक साथ िकसी काय को 10 A, B और C िकसी काय को मशः 5,
का 30% भाग िकतने िदनों म पूरा करे गा िदनों म कर सकते ह | B और C इसे एक 20 और 60 िदनों म पूरा कर सकते ह |
? साथ 15 िदनों म तथा C और A इसे एक एक साथ काय करते ए, वे इस काय को
SSC CGL 13 June 2019 (Morning) साथ 20 िदनों म कर सकते ह | वे 8 िदनों िकतने िदनों म पूरा करगे ?
(a)15 तक साथ काय करते ह | शेष काय पूरा SSC CHSL 5 July 2019 (Morning)
(b)16 करने म C को अकेले िकतने िदन लगगे (a) 3 41
(c)20 ? (b) 3 43
(d)18 SSC CHSL 3 July 2019 (c) 5 41
(Afternoon) (d) 5 43
(a) 12 Days

www.ssccglpinnacle.com support@ssccglpinnacle.com Ph. 09729327755, 09817390373 372


/
Days 43-46 Work and Time

s
sse
la
_c
ob
(d) 21000
bo
ah
Q19. A, B and C can complete a Q25. A earns Rs. 100 per hour and
sm
ur
yo

piece of work in 4, 20 and 60 days Q22. A and B can complete a piece works for 8 hours per day. B earns
e/
t.m
://

respectively. Working together, they of work in 15 days and 10 days Rs. 120 per hour and works for 6
tp
ht

can complete the same work in how respectively. They got a contract to hours per day. The ratio of per day
many days ? complete the work for Rs. 75,000. wages of B to that of A is :
A, B और C िकसी काय को मशः 4, The share of B in the contracted A 100 पये ित घंटे कमाता है तथा
20 और 60 िदनों म पूरा कर सकते ह | money will be : िदन म 8 घंटे काय करता है | B 120
एक साथ काय करते ए वे इस काय को A और B िकसी काय को मशः 15 पये ित घंटे कमाता है और ितिदन 6
िकतने िदनों म समा करगे ? िदन और 10 िदन म कर सकते ह | उ घंटे काय करता है | B और A के दै िनक
SSC CHSL 5 July 2019 75000 पये म यह काय पूरा करने का वेतन म ा अनुपात है ?
(Afternoon) अनुबंध िमला | इस रािश म B का िह ा SSC CHSL 9 July 2019
3
(a) 5 19 ात कर | (Afternoon)
1
(b) 3 19 SSC CHSL 8 July 2019 (Evening) (a) 10 : 9
1
(c) 5 19 (a) 35000 (b) 4 : 5

e
3 (b) 40000 (c) 5 : 4
(d) 3 19
(c) 45000 (d) 9 : 10
(d) 30000
Q20. A, B and C can complete a
Q26. A and B can complete a piece
piece of work in 4, 20 and 60 days

l
Q23. A earns Rs. 640 per day and of work in 15 days and 20 days
respectively. Working together, they
works for 8 hours. B earns Rs. 360 respectively. They got a contract to
ac
can complete the one third work in
how many days ?
A, B और C िकसी काय को मशः 4,
20 और 60 िदनों म पूरा कर सकते ह |
एक साथ काय करते ए, वे एक ितहाई
per day and works for 6 hours. The
ratio of per day wages of A to that of
B is :
A हर िदन 640 पये कमाता है तथा 8
घंटे काय करता है | B हर िदन 6 घंटे
complete the work for Rs. 77,000.
The share of A in the contracted
money will be :
A और B िकसी काय को मशः 15
िदन और 20 िदनों म कर सकते ह | उ
काय िकतने िदनों म समा करगे ?
काय करके 360 पये कमाता है | A 77000 पये म यह काय समा करने
SSC CHSL 5 July 2019 (Evening)
3
और B के दै िनक वेतन का अनुपात ात का अनुबंध िमला | इस रािश म A का
(a) 1 19
कर | िह ा ात कर |
nn
1
(b) 1 19 SSC CHSL 8 July 2019 (Evening) SSC CHSL 9 July 2019
(c) 3
19 (a) 5:4 (Afternoon)
1
(d) 2 19 (b) 9:16 (a) 45000
(c) 16:9 (b) 40000
(d) 4:5 (c) 44000
Q21. A and B can complete a piece (d) 42000
of work in 15 days and 10 days Q24. A and B can complete a piece
respectively. They got a contract to of work in 15 days and 10 days Q27. A, B and C can complete a
Pi

complete the work for Rs. 35,000. respectively. They got a contract to piece of work in 20, 24 and 36 days
The share of B in the contracted complete the work for Rs. 75,000. respectively. Working together, they
money will be : The share of A in the contracted can complete the same work in how
A और B िकसी काय को मशः 15 money will be : many days ?
िदन और 10 िदन म पूरा कर सकते ह | A और B िकसी काय को मशः 15 A, B और C िकसी काय को मशः 20,
उ 35000 पये म यह काय पूरा करने और 10 िदनों म पूरा कर सकते ह | उ 24 और 36 िदनों म पूरा कर सकते ह |
का अनुबंध िमला | इस रािश म B का 75000 पये म यह काय पूरा करने का एक साथ काय करते ए, वे इसी काय
िह ा ात कर | अनुबंध िमला | इस रािश म A का िह ा को िकतने िदनों म पूरा करगे ?
SSC CHSL 8 July 2019 ात कर | SSC CHSL 9 July 2019 (Evening)
SSC CHSL 9 July 2019 (Morning) 16
(a) 8 43
(Afternoon)
(a) 7000 (a) 35000 (b) 6 41
(b) 15000 (b) 40000 (c) 9 41
(c) 14000 (c) 45000 19
(d) 7 20
(d) 30000

www.ssccglpinnacle.com support@ssccglpinnacle.com Ph. 09729327755, 09817390373 373


/
Days 43-46 Work and Time

s
sse
la
_c
ob
(b) 55 पूरा करने म B को अकेले िकतने घंटे
bo
ah
Q28. If 16 men working 12 hours a (c) 60 लगगे ?
sm
ur
yo

day can complete a work in 27 days, (d) 45 SSC CPO 13 March 2019
e/
t.m
://

then working for how many hours a (Evening)


tp
ht

day can 18 men complete the work Q 31. The efficiency of A, B and C (a)114
in 24 days ? are in the ratio 5:6:9 working (b)152
यिद 16 पु ष ितिदन 12 घंटे काय together, they can complete a work (c)182.4
करके िकसी काय को 27 िदनों म पूरा in 18 days. In how many days can B (d)167.2
करते ह, तो इस काय को 24 िदनों म पूरा alone can complete 25% of that
करने के िलए 18 पु षों को िदन म work? Q34. The efficiencies of A,B and C
िकतने घंटे काय करना पड़े गा ? A, B और C की काय मता 5 : 6 : 9 are in the ratio 4 : 5 : 6. Working
SSC CHSL 10 July 2019 के अनुपात म है | एक साथ काय करते together, they can complete a work
(Morning) ए वे िकसी काय को 18 िदनों म समा in 12 days. In how many days will A
(a) 9 days कर सकते ह | B अकेले इस काय का alone be able to complete that work?
(b) 18 days 25% भाग िकतने िदनों म पूरा कर A, B तथा C की काय मता 4 : 5 : 6
सकता है ? के अनुपात म है | एक साथ काय करते

e
(c) 16 days
(d) 12 days SSC CPO 12 March 2019 ए वे िकसी काय को 12 िदन म पूरा कर
(Evening) सकते ह | A अकेले इस काय को िकतने
Q29. If 40 men working 12 hours a (a) 18 िदन म कर पाएगा ? SSC CPO 12

l
day can complete a work in 8 days, (b) 10 March 2019 (Morning)
then how many men working 4 (c) 15 (a) 45
ac
hours a day can complete the same
work in 16 days ?
यिद 40 लोग ितिदन 12 घंटे काय
करके िकसी काय को 8 िदनों म समा
कर सकते ह, तो इस काय को ितिदन 4
(d) 16

Q32. 16 persons working 6 hours a


day can complete a work in 10 days.
In how many days 24 persons
(b) 36
(c) 30
(d) 40

Q35. 24 persons working 8 hours a


घंटे काय करते ए 16 िदनों म समा working 8 hours a day will complete day can complete 2 units of a work
करने के िलए िकतने पु षों की 80% of that work? in 10 days. How many persons are
आव कता होगी ? एक िदन म 6 घंटे काय करके 16 required to complete 4 units of that
nn
SSC CHSL 10 July 2019 िकसी काय को 10 िदनों म work, if they work 6 hours a day for
(Afternoon) समा कर सकते ह | एक िदन म 8 घंटे 16 days?
(a) 50 काय करके 24 इस काय का एक िदन म 8 घंटे काय करते ए 24
(b) 60 80% भाग िकतने िदनों म समा कर िकसी काय की 2 इकाई 10 िदनों
(c) 54 सकते ह ? SSC CPO 13 March म समा कर सकते ह | इस काय की 4
(d) 45 2019 (Evening) इकाई समा करने के िलए िकतने लोगों
(a)3 की आव कता होगी यिद वे 16 िदनों
Q30. 36 persons working 8 hours a (b)6 तक एक िदन म 6 घंटे काय करते ह ?
Pi

day can do 3 units of work in 12 (c)4 SSC CPO 12 March 2019


days. How many persons are (d)8 (Morning)
required to do 5 units of that work in (a) 48
16 days, if they work for 6 hours a Q33. The efficiencies of A,B and C (b) 36
day? are in the ratio 5 : 6 : 8, working (c) 40
ितिदन 8 घंटे काय करके 36 लोग together, they can complete a piece (d) 32
िकसी काय की 3 इकाई 12 िदनों म कर of work in 120 hours. In how many
सकते ह| यिद वे एक िदन म 6 घंटे काय hours will, B alone be able to Q36. 18 persons working 8 hours a
कर तो 16 िदन म इस काय की 5 इकाई complete 40% of that work? day can complete 3 units of works in
समा करने के िलए िकतने लोगों की A, B तथा C की काय मता 5 : 6 : 8 के 10 days. How many persons are
आव कता होगी ? अनुपात म है | एक साथ काय करते ए required to complete 5 units of that
SSC CPO 12 March 2019 वे िकसी काय को 120 घंटों म समा work in 16 days working 6 hours a
(Evening) कर सकते ह | इस काय का 40% भाग day?
(a) 50

www.ssccglpinnacle.com support@ssccglpinnacle.com Ph. 09729327755, 09817390373 374


/
Days 43-46 Work and Time

s
se
s
la
_c
ob
एक िदन म 8 घंटे काय करते ए 18 does and a child does half the work a (c) 38 days
bo
ah िकसी काय की 3 इकाई 10 िदनों man does. How many women alone (d)16 days
sm
ur
yo

म समा कर सकते ह | ितिदन 6 घंटे can complete this work in 7 days?


e/
t.m

काय करते ए 16 िदन म इस काय की 3 पु ष, 4 मिहलाएं तथा 6 ब े िकसी


://

Q42. If 15 men can do a piece of


tp
ht

5 इकाई समा करने के िलए िकतने काय को 7 िदनों म पूरा कर सकते ह | work in 14 days, how many men will
लोगों की आव कता होगी ? एक मिहला एक पु ष की तुलना म be needed to do the work in 30
SSC CPO 13 March 2019 दोगुना काय करती है तथा एक ब ा days?
(Morning) एक पु ष की तुलना म आधा काय यिद 15 लोग िकसी काय को 14 िदनों म
(a) 25 करता है | िकतनी मिहलाएं अकेले इस कर सकते ह, तो इस काय को 30 िदनों
(b) 15 काय को 7 िदनों म पूरा कर लगी ? म करने के िलए िकतने लोगों की
(c) 20 SSC CPO 16 March 2019 आव कता होगी ?
(d) 9 (Evening) SSC CPO 16 March 2019
(a)6 (Afternoon)
Q37. The efficiencies of A, B and C (b)7
are in the ratio 2 : 5 : 7 working (c)9 (a)8

e
together, they can complete a work (d)8 (b)10
in 10 days. In how many days will, (c)7
A alone be able to complete 30% of Q40. Three painters have to spend 6 (d)9
that work? hours a day for 12 days to finish a
A, B और C की काय मता 2 : 5 : 7 के

l
work. If after 3 days one painter Q43. 21 typist can complete a task in
अनुपात म है | एक साथ काय करते ए leaves, in how many days the 8 days. Then in how many days 15
ac
वे िकसी काय को 10 िदनों म समा कर
सकते ह | इस काय का 30% भाग
समा करने म A को अकेले िकतना
समय (िदन ) लगेगा ?
SSC CPO 13 March 2019
remaining work will be completed?
तीन िच कारों को ित िदन 6 घंटे खच
करके िकसी काय को समा करने म
12 िदन लगते ह| यिद तीन िदन बाद एक
िच कार छोड़ दे , तो शेष काय िकतने
typist can complete that task?
21 टाइिप 8 िदनों म एक काय पूरा
कर सकता है । िफर 15 टाइिप िकतने
िदनों म उस काय को पूरा करगे ?
SSC CPO 14 March 2019
(Morning) िदनों म समा होगा ? SSC CPO 15 (Evening)
(a) 20 March 2019 (Morning) (a) 11.2
(b) 28 (b) 5.8
nn
(c) 30 (a)15 32 (c) 9.3
(d) 21 (b)13 21 (d) 7
(c)11
Q38. A can do a work in 12 days (d)8 Q44. The efficiency of A is three
while B can do same work in 18 times of B and efficiency of B is
days. How long(in days) will it take Q41.A can do a work in 30 days, B twice the efficiency of C. If B alone
if they do the work together? can do the same work in 48 days. can complete one task in 15 days,
A िकसी काय को 12 िदनों म कर then how many days will A and C
Pi

After working alone for 20 days A


सकता है जबिक B इसी काय को 18 left and B started working, how long take to complete this task
िदनों म कर सकता है | यिद वे साथ काय will B take to complete the work? A की द ता B से तीन गुनी है और B
कर तो इसे पूरा होने म िकतने िदन लगगे A िकसी काय को 30 िदनों म कर सकता की द ता C से दोगुनी है | यिद B
? SSC CPO 14 March 2019 है , B इसी काय को 48 िदनों म कर अकेला एक काय को 15 िदनों म पूरा
(Morning) सकता है | 20 िदन तक अकेले काय कर सकता है , तो A और C िमलकर
(a) 5 43 करने के बाद A ने छोड़ िदया तथा B ने इस काय को िकतने िदनों म पूरा करगे
(b) 7 51 काय करना शु कर िदया | B को इस SSC CPO 14 March 2019
(c) 6 51 काय को पूरा करने म िकतना समय (Evening)
लगेगा ? (a) 4 31
(d) 6 32
SSC CPO 16 March 2019 (b) 6 21
Q39. 3 men, 4 women and 6 children (Afternoon) (c) 4 72
can complete a work in 7 days. A (a)24 days (d) 7 54
woman does double the work a man (b)28 days

www.ssccglpinnacle.com support@ssccglpinnacle.com Ph. 09729327755, 09817390373 375


/
Days 43-46 Work and Time

s
sse
la
_c
ob
Q45. Both A and B working together (c)4 : 5 in 15 days. In how many days A
bo
ah
can complete a task in 3 32 days. C (d)5 : 4 alone can do the work?
sm
ur
yo

and D complete the same task in 3 71 A तथा B िमलकर िकसी काय को 20


e/
t.m

िदन म कर सकते ह | B तथा C िमलकर


://

Q48.Two teachers A and B can


tp

days. If A, B, C and D work


ht

together, how long will it take to complete an academic work in 10 उसी काय को 25 िदन म कर सकते ह |
complete half the work? days and 15 days respectively.They C तथा A िमलकर उसी काय को 15
A और B दोनों िमलकर एक काय को 3 started the work together, but A left िदन म कर सकते ह | A अकेला उस
after 5 days and another teacher C काय को िकतने िदनों म कर सकता है ?
3 िदनों म पू रा करते ह | C और D उसी
2
joined, who alone can complete the SSC MTS 2 August 2019
काय को 3 71 िदनों म पूरा करते ह | यिद
work in 60 days. In how many days (Afternoon)
A, B, C और D एक साथ िमलकर काय
the work got completed? (a) 600
17
करते ह तो आधे काय को पूरा करने म
दो िश क A और B मशः 10 िदनों (b) 600
उ िकतना समय लगेगा ? 23
और 15 िदनों म एक शै िणक काय पूरा (c) 1200
SSC CPO 15 March 2019 23
कर सकते ह। उ ोंने एक साथ काम 1200
(Evening) (d)
11 Days / िदन
शु िकया, लेिकन A 5 िदनों के बाद 17
(a) 13
छोड़ िदया और एक अ िश क C इस

e
1 Days / िदन
(b)1 13 Q51. A can do a piece of work alone
काय म शािमल हो गया, जो अकेले 60
(c) 8 Days / िदन िदनों म इस काम पूरा कर सकता है । यह in 8 days. B can do the same work
13
(d) 2 Days / िदन काय िकतने िदनों म पूरा होगा? alone in 21 days. If they work
13
together for 3 days, then how much

l
SSC MTS 2 August 2019
(Morning) work is completed?
Q46.A can do one fifth part of a task
ac (a)7 A अकेले िकसी काय 8 िदनों म पूरा कर
in 4 days, B can do one sixth part of
the same work in 5 days. If they (b)5 सकता है , B वही काय अकेले 21 िदनों
work together, how many days can (c)6 म कर सकता है । यिद वे 3 िदनों के िलए
this task be completed? (d)2 एक साथ काम करते ह, तो िकतना काम
A, िकसी काय का ⅕ वां भाग 4 िदनों म पूरा होता है ?
कर सकता है , B उसी काय का 1/6 वां SSC MTS 2 August 2019
Q49. A alone can do a piece of work
भाग 5 िदनों म कर सकता है | यिद वे (Evening)
in 30 days. B alone can do the same
एक साथ िमलकर काय करते ह तो इस 29
(a) 67
काय को िकतने िदनों म पूरा कर सकते work in 40 days. If they work
nn
31
(b) 65
ह? alternately (Alternate Days) starting
from A, then in how many days 27
(c) 64
SSC CPO 15 March 2019
(Evening) (upto two decimal places) the work 29
(d) 56
(a)12 will be completed?
(b)30 A अकेला िकसी काय को 30 िदन म कर Q52. A, B and C, working alone can
(c)20 सकता है | B अकेला उसी काय को 40 do a piece of work in 15, 30 and 75
(d)15 िदन म कर सकता है | यिद A से days respectively. They work
शु आत करके , वे बारी बारी काम
Pi

together and get Rs 1615 for


Q47. A earns Rs 40 per hour and करते ह, तो काय िकतने िदनों म पूरा हो completing the work. What is the
works for 12 hours. B earns Rs 60 जाएगा? difference in shares of A and C?
per hour and works for 10 hours. SSC MTS 2 August 2019 A, B और C, अकेले िकसी काय को
Find the ratio of their per day wages. (Afternoon) मशः 15, 30 और 75 िदनों म कर
A 40 पये ित घंटा कमाता है तथा 12 (a)34.25 सकते ह। वे एक साथ काम करते ह और
घंटे काय करता है | B 60 पये ित घंटा (b)32.33 काम पूरा करके 1615 . ा करते है ।
कमाता है तथा 10 घंटे काय करता है | (c)16.33 A और C के िह ों म िकतना अंतर है ?
उनकी ितिदन मजदू री का अनुपात (d)17.25 SSC MTS 5 August 2019
ात कर | (Morning)
SSC CPO 16 March 2019 Q50.A and B together can do a piece (a)Rs 760
(Evening) work in 20 days. B and C together (b)Rs 620
(a)15 : 4 can do the same work in 25 days. C (c)Rs 680
(b)6 : 5 and A together can do the same work (d)Rs 540

www.ssccglpinnacle.com support@ssccglpinnacle.com Ph. 09729327755, 09817390373 376


/
Days 43-46 Work and Time

s
sse
la
_c
ob
SSC MTS 5 August 2019 ू िडयो A के 5 तकनीिशयन िकसी
bo
ah
Q 53. A can do a piece of work (Evening) काय को 30 िदनों म कर सकते ह |
sm
ur
yo

alone in 10 days. A and B together (a) Rs. 350 ू िडयो B के 8 तकनीिशयन उसी काय
e/
t.m

को 15 िदनों म कर सकते ह | ू िडयो A


://

can do the same work in 20/3 days. (b) Rs. 1400


tp
ht

A, B and C can do the same work (c) Rs. 1050 के 1 तकनीिशयन और ू िडयो B के 1
together in 40/7 days. In how many (d) Rs. 700 तकनीिशयन िमलकर उसी काय को
days can B and C do the same work िकतने िदनों म कर सकते ह?
together? Q56. X can do a work alone in 15 SSC MTS 6 August 2019
A िकसी काय को अकेले 10 िदन म कर days. Y can do the same work in 30 (Afternoon)
सकता है | A तथा B उसी काय को days alone. X, Y and Z together can (a) 190 3
िमलकर 20/3 िदनों म कर सकते ह | A, do that work in 9 days. In how many (b) 200
3
B तथा C उसी काय को िमलकर 40/7 days Z alone can do that work? (c) 250
9
िदनों म कर सकते ह | B तथा C उसी X िकसी काय को अकेला 15 िदन म कर
(d) 220
काय को िमलकर िकतने िदनों म कर सकता है | Y उसी काय को अकेला 30 9

सकते ह? िदन म कर सकता है | X, Y तथा Z एक


Q59. A can do a work alone in 20
साथ िमलकर उस काय को 9 िदन म कर

e
SSC MTS 5 August 2019
days. A and B together can do the
(Afternoon) सकते ह | Z उस काय को अकेला िकतने
same work in 15 days. B and C
(a) 44
3 िदन म कर सकता है ?
together can do the same work in
(b) 20 SSC MTS 6 August 2019
3 752 days. What is the ratio of

l
(c) 40 (Morning)
3 efficiency of A, B and C in terms of
(a)120
(d) 44
7 ac working?
(b)90
A िकसी काय को अकेला 20 िदनों म
(c)45
Q54. 10 men can complete a work in कर सकता है | A तथा B िमलकर उसी
(d)60
30 days by working for 8 hours per काय को 15 िदनों म कर सकते ह | B
day. In how many days 12 men तथा C िमलकर उसी काय को 75 2 िदनों
Q57. A alone can do a work in 30
working 4 hours a day will complete म कर सकते ह | काय करने की ि से
days. B alone can do the same work
the work? A, B तथा C की द ता का अनुपात
in 60 days. If they work together for
िकसी काय को 10 पु ष ितिदन 8 घंटों िकतना है ?
5 days, then what part of the work
nn
तक काय कर 30 िदनों म पूरा कर सकते SSC MTS 6 August 2019
will be left?
ह | 12 पु ष ितिदन 4 घंटे काय कर (Afternoon)
A िकसी काय को अकेला 30 िदन म कर
उसी काय को िकतने िदनों म पूरा करगे? (a)5:3:1
सकता है | B उसी काय को अकेला 60
SSC MTS 5 August 2019 (b)10:4:1
िदन म कर सकता है | यिद वे 5 िदन तक
(Evening) (c)5:4:3
एक साथ काय करते ह, तो काय का
(a)30 Days / िदन (d)15:5:3
िकतना िह ा शेष रह जाएगा?
(b)50 Days / िदन
SSC MTS 6 August 2019
(c)40 Days / िदन Q60. Mohit is five times more
(Morning)
Pi

(d)60 Days / िदन efficient as Rohit. If Mohit can


(a) 32
complete a piece of work in 28 days
(b) 21
Q 55. A, B and C can complete a less than Rohit, then in how many
work alone - in 12, 24 and 36 days (c) 43
days can Rohit alone complete the
respectively. Together they complete (d) 65 same work?
the same work and get Rs. 3850. मोिहत रोिहत से पां च गुना अिधक काय
What is the difference between the Q58. 5 technicians of Studio A can कुशल है । अगर मोिहत िकसी काय को
share of B and C ? do a piece of work in 30 days. Studio पूरा करने म रोिहत की तुलना म 28 िदन
A, B और C िकसी काय को अकेले - B's 8 technicians can do the same कम का समय लेता है ,तो रोिहत अकेले
अकेले कर उसे मश: 12, 24 और 36 work in 15 days. In how many days इस काय को िकतने िदन म पूरा करे गा ?
िदनों म पूरा कर सकते है | वे िमलकर can 1 technician from Studio A and SSC MTS 6 August 2019
उसी काय को पूरा कर 3850 पए 1 technician from Studio B together (Evening)
ा करते है | B और C के िह े की do the same work? (a)35 days
रािशयों म िकतना अंतर है ? (b)32 days

www.ssccglpinnacle.com support@ssccglpinnacle.com Ph. 09729327755, 09817390373 377


/
Days 43-46 Work and Time

s
se
s
la
_c
ob
(c)45 days in how many days will the total work complete the same work in 14 days.
bo
ah
(d)40 days be completed ? If they both get Rs 44000 to
sm
ur
yo

स ाट अकेले 10 िदनों म एक काम पूरा complete the work, then what is the
e/
t.m

कर सकता है और िवराट अकेले उसी


://

Q61. Ravika alone can complete share of Piyush?


tp
ht

three-fifth of a work in 105 days. काम को 40 िदनों म पूरा कर सकता पीयूष अकेले 8 िदनों म एक काम को
Mallika can complete one-third of है ।स ाट इस काय को आर करता है पूरा कर सकता है और अ ण अकेले
work in 50 days. In how many days और िफर दोनों इसे बारी बारी से करते उसी काम को 14 िदनों म पूरा कर
Ravika and Mallika working है , इस कार यह काय िकतने िदनों म सकता है । अगर इन दोनों को काम पूरा
together, can complete 35 26 of the समा हो जाएगा ? करने के िलए 44000 पये िमलते ह, तो
work ? SSC MTS 7 August 2019 पीयूष का िह ा िकतना है ?
िकसी काय के 3 भाग को रवीका 105 (Morning) SSC MTS 7 August 2019
5
िदनों म पूरा कर सकती है । म का 50 (a) 14 days (Evening)
िदनों म इसका एक ितहाई काम पूरा कर (b) 16 days (a)Rs 24000
सकती ह। रािवका और म का दोनों (c) 8 days (b)Rs 16000
िमलकर िकतने समय म इस काय का (d) 12 days (c)Rs 28000

e
(d)Rs 21000
35 भाग पू रा कर दे गी ?
26
Q64. C and D together can make a
SSC MTS 6 August 2019
chair in 4 days and C alone can Q 67. E, F and G together can
(Evening)
make this chair in 12 days. In how complete a work in 12 days. If E and
(a) 63 days

l
many days D alone can make this F together can complete the same
(b) 70 days
chair? work in 30 days, then in how many
(c) 60 days ac C और D िमलकर 4 िदन म एक कुस days can G alone complete the same
(d) 80 days
बना सकते ह और C अकेले इस कुस work?
को 12 िदनों म बना सकता ह। D िकतने E,F और G एक साथ 12 िदनों म एक
Q62. A man finish a piece of work in
िदनों म अकेले इस कुस को बना सकता काम पूरा कर सकते ह। यिद E और F
15 days. A woman can complete the
है ? िमलकर एक ही काम को 30 िदनों म
same work in 10 days. Both work
SSC MTS 7 August 2019 पूरा कर सकते ह, तो G िकतने िदनों म
together for 5 days, then the man
(Afternoon) अकेले उसी काम को पूरा कर सकता
leaves. How many days will be taken
(a)10 days है ?
nn
by the woman to finish the
(b)6 days SSC MTS 8 August 2019
remaining work?
(c)4 days (Morning)
एक आदमी िकसी काय 15 िदनों म पूरा
(d)8 days (a)18 days
करता है । एक मिहला इसी काय को 10
(b)20 days
िदनों म पूरा कर सकती है । दोनों 5 िदन
Q65. If Dev can make 40 chairs in (c)12 days
तक साथ काम करते ह, िफर आदमी
20 days, working 10 hours per day (d)24 days
इस काय को छोड़ दे ता है । शेष काय को
then in how many days can he make
पूरा करने म मिहला को िकतने िदन 1
10 chairs working 8 hours per day? Q68. Vishal alone can complete
Pi

लगगे? 3
ितिदन 10 घंटे काम करके यिद दे व 20 part of a work in 60 days and Ashok
SSC MTS 7 August 2019
िदनों म 40 कुिसयाँ बना सकता है ,तो alone can complete 41 part of the
(Morning)
ितिदन 8 घंटे काम करके 10 कुिसयाँ same work 30 days. In how many
(a)2 21
िकतने िदन म बनाएगा ? days Vishal and Ashok together can
(b)1 31
SSC MTS 7 August 2019 complete the same work?
(c)2 32 (Afternoon) िवशाल अकेले एक काम के 13 व िह े
(d)1 32 (a)8 को 60 िदनों म पूरा कर सकता है और
(b)6 अकेले अशोक उसी काम के 14 व िह े
Q 63. Samrat alone can complete a (c)6 41 को 30 िदनों म पूरा कर सकता है ।
work in 10 days and Virat alone can (d)7 21 िवशाल और अशोक एक साथ िकतने
complete the same work in 40 days. िदनों म इस काम पूरा कर सकते ह?
If they are working on alternate days Q66. Piyush alone can complete a SSC MTS 8 August 2019
with Samrat starting the work, then work in 8 days and Arun alone can (Morning)

www.ssccglpinnacle.com support@ssccglpinnacle.com Ph. 09729327755, 09817390373 378


/
Days 43-46 Work and Time

s
sse
la
_c
ob
(a)64 (a) 1 करता है , तो B को काय पूरा करने म
bo
5
ah
(b)20 िकतने िदन लगगे?
sm
(b) 1
ur

3
yo

(c)72 SSC MTS 9 August 2019


e/

(c) 1
t.m

10
://

(d)56 (Afternoon)
tp

(d) 1
ht

4
(a)12
Q69. Anil can make 20 bags in 4 (b)24
Q72. A can do 50% of the job in 16
days and Manoj can make 10 bags in (c)30
days, B can do one-fourth of the
5 days. How many bags both Anil (d)15
same job in 24 days. Working
and Manoj together can make in one
together, in how many days they can
day? Q75. A contractor takes a contract to
do seven-fourth of the job?
अिनल 4 िदन म 20 बैग और मनोज 5 complete a road in 60 days and
A िकसी काय का 50% भाग 16 िदनों म
िदन म 10 बैग बना सकता ह। अिनल employs 70 labours. After 25 days,
कर सकता है , B इस काय का एक
और मनोज दोनों िमलकर एक िदन म he found that one fourth work is
चौथाई भाग 24 िदनों म पूरा कर सकता
िकतने बैग बना सकते ह? completed. How many more labours
है । एक साथ काम करके व िकतने िदनों
SSC MTS 8 August 2019 he requires to complete the
म वे सात-चौथाई काय कर सकते ह?

e
(Afternoon) remaining work in time?
SSC MTS 9 August 2019
(a)11 bags एक ठे केदार 60 िदनों म एक सड़क को
(Morning)
(b)7 bags पूरा करने का ठे का लेता है और 70
(a)24
(c)9 bags मजदू रों को इस काय म लगाता है । 25
(b)28
िदनों के बाद,एक चौथाई काम पूरा हो

l
(d)5 bags
(c)27
जाता है । शेष काम को िदए गए समय
ac
Q70. N and K together can complete
a work in 240 days, K and G
together can complete the same work
in 72 days and N and G together can
complete the same work in 80 days.
(d)42

Q73. Some persons can do a piece of


work in 84 days. Two times the
number of such persons will do half
पर पूरा करने के िलए उसे िकतने और
मजदू रों की आव कता पड़े गी?
SSC MTS 9 August 2019
(Afternoon)
(a)90
of the same work in how many days?
In how many days K alone can (b)82
कुछ 84 िदनों म एक काम को
complete the same work? (c)80
पूरा कर सकते ह। ऐसे यों की
N और K िमलकर एक काम को 240 (d)85
nn
दु गनी सं ा िकतने िदनों म आधा काम
िदनों म पूरा कर सकते ह, K और G
पूरा कर दे गी ?
िमलकर उसी काम को 72 िदनों म पूरा Q76. X takes 10 days less than the
SSC MTS 9 August 2019
कर सकते ह और N और G िमलकर time taken by Y to finish a piece of
(Morning)
उसी काम को 80 िदनों म पूरा कर सकते work, if both X and Y together finish
(a)21 days
ह। K अकेले िकतने िदनों म इस काम a work in 12 days. Find the time
(b)14 days
पूरा कर सकता है ? taken by Y to finish the work.
(c)16 days
SSC MTS 8 August 2019 X, िकसी काय को पूरा करने म Y की
(d)15 days
(Evening) तुलना म 10 िदन का कम समय लेता है |
Pi

X और Y दोनों िमलकर काय को 12


Q74. Working together, A and B can
(a)280 days िदन म पूरा कर सकते ह | Y िकतने िदनों
complete a work in 12 days. They
(b)240 days म अकेले काय को पूरा करे गा?
work together for 9 days after which
(c)360 days SSC MTS 9 August 2019
B leaves. If A finishes the remaining
(d)180 days (Evening)
work in 5 days, then the number of
(a)22.5 Days / िदन
days that B alone would take to
Q71. Vijay can complete a work in (b)25 Days / िदन
complete the work is:
50 days. How much part of work (c)32 Days / िदन
एक साथ काम करते ए, A और B एक
will be completed in 10 days ? (d)30 Days / िदन
काय को 12 िदनों म पूरा कर सकते ह।
िवजय 50 िदनों म एक काम को पूरा कर
वे 9 िदनों तक साथ म काय करते है और
सकता है । 10 िदनों म काम का िकतना Q77. A is twice as efficient as B in
उसके बाद B उस काय को छोड़ दे ता
िह ा पूरा होगा? writing novels. Together they can
है । यिद A शेष काय को 5 िदनों म पूरा
SSC MTS 8 August 2019 complete the work of writing a novel
(Evening)

www.ssccglpinnacle.com support@ssccglpinnacle.com Ph. 09729327755, 09817390373 379


/
Days 43-46 Work and Time

s
sse
la
_c
ob
in 24 days. In how many days A can completed. How many more labours SSC MTS 14 August 2019
bo
ah
write the novel alone? he requires to complete the (Morning)
sm
ur
yo

A उप ास िलखने म B से दोगुना द है remaining work in time? (a)9


e/
t.m

| वे एक साथ िमलकर 24 िदनों म एक एक ठे केदार 60 िदनों म एक सड़क को


://

(b)12
tp
ht

उप ास लेखन का काय पूरा कर सकते पूरा करने का ठे का लेता है और 105 (c)6


ह | A िकतने िदनों म उप ास अकेले मजदू रों को इस काय म लगाता है । 25 (d)8
िलख सकता है ? िदनों के बाद, एक ितहाई काम पूरा हो
SSC MTS 13 August 2019 जाता है । शेष काम को समय पर पूरा Q 83.C can do a piece of work alone
(Morning) करने के िलए उसे िकतने और मजदू रों in 120 days. B is twice efficient than
(a)24 की आव कता पड़े गी? C and A is thrice efficient than B. In
(b)18 SSC MTS 13 August 2019 how many days they can complete
(c)32 (Evening) the work together?
(d)36 (a)45 C िकसी काय को 120 िदनों म अकेले
(b)150 कर सकता ह | B, C से दोगुना कुशल है
Q78. A can do a work in 15 days. B (c)75 और A, B से ितगुना कुशल है | वे एक
साथ िमलकर काय को िकतने िदनों म

e
is 25% more efficient than A. In how (d)105
many days, working together A and पूरा कर सकते ह?
B will complete the same work? Q81. A and B together can complete SSC MTS 14 August 2019
A िकसी काय को 15 िदनों म पूरा कर some work in 36 days, B and C (Morning)
सकता है , B, A की तुलना म 25%

l
together can complete the same work (a)13.33
अिधक काय कुशल है । यिद दोनों एक in 60 days and A and C together can (b)16.67

(Afternoon)
(a) 21
4
ac
साथ काय करते है तो इसी काय को पूरा
करने म िकतने िदन का समय लगेगा ?
SSC MTS 13 August 2019
complete the same work in 45 days.
In how many days, B alone can
complete the same work?
A और B िमलकर िकसी काम को 36
िदनों म पूरा कर सकते ह, B और C
(c)12.5
(d)15

Q84. X did a task for 24 days and


left it incomplete. He could have
(b) 24 िमलकर उसी काम को 60 िदनों म पूरा finished the task all by himself in a
5
(c) 20 कर सकते ह और A और C िमलकर total of 36 days. Y, who can finish
3
उसी काम को 45 िदनों म पूरा कर सकते the task alone in 18 days, will take
nn
(d) 25
7
ह। िकतने िदनों म, B अकेले उसी काम how many more days to complete
को पूरा कर सकता है ? the task?
Q79. A work can be completed by
SSC MTS 13 August 2019 X ने 24 िदनों के िलए एक काय िकया
35 workers in 30 days. If 5 workers
(Evening) और उसे अधूरा छोड़ िदया। वह कुल 36
leave after every 10 days then in
(a)180 िदनों म अकेला सारा काम पूरा कर
how many days will the work be
(b)60 सकता था। Y, जो 18 िदनों म अकेले
completed ?
(c)100 काय पूरा कर सकता है , उसे इस काय
एक काम 30 िदनों म 35 िमकों ारा
(d)90 को पूरा करने म िकतने और िदन लगगे?
Pi

पूरा िकया जा सकता है । यिद ेक 10


SSC MTS 14 August 2019
िदनों के बाद 5 िमक काय छोड़ दे ते ह
Q82. A alone can complete a piece (Afternoon)
तो काय िकतने िदनों म पूरा होगा?
of work in 30 days.He left after the (a)9
SSC MTS 13 August 2019
completion of half of the work.If B (b)6
(Afternoon)
alone can complete the entire work (c)12
(a) 35.5 days
in 16 days, then how many days will (d)8
(b) 37.5 days
B take to complete the remaining
(c) 40 days
work? Q85. A, B and C can complete a
(d) 50 days
A िकसी काय को अकेले 30 िदनों म पूरा work alone in 400, 600 and 900 days
कर सकता है | उसने आधा काय करके respectively. In how many days can
Q80. A contractor takes a contract to
छोड़ िदया | यिद B अकेले पूरे काय को the work be completed if it is started
complete a road in 60 days and
16 िदनों म सकता है , तो B को शेष काय by A and he is assisted by B and C
employed 105 labours. After 25
को पूरा करने म िकतने िदन लगगे? on every second and third day
days, he found that one-third work is
respectively?

www.ssccglpinnacle.com support@ssccglpinnacle.com Ph. 09729327755, 09817390373 380


/
Days 43-46 Work and Time

s
sse
la
_c
ob
A, B और C मशः 400, 600 और 900 (d)6 (b)36
bo
ah
िदनों म अकेले एक काम पूरा कर सकते (c)42
sm
ur
yo

ह। यिद A ारा काय शु िकया जाता है Q88. A and B can do a piece of work (d)27
e/
t.m

एवं B और C उसे मशः दू सरे और


://

alone in 15 and 30 days respectively.


tp
ht

तीसरे िदन सहायता दान करते है ,तो In how many days can they together Q91.A, B and C can do a work in 16
िकतने िदनों म काय पूरा िकया जा complete the work ? days, while A and B can do the same
सकता है ? A और B मशः 15 और 30 िदनों म work in 40 days. C alone can do that
SSC MTS 14 August 2019 अकेले िकसी काम को पूरा कर सकते work in how many days?
(Afternoon) ह। वे िकतने िदनों म एक साथ िमलकर A, B और C िकसी काय को 16 िदनों म
(a)292 इस काम पूरा कर सकते ह? कर सकते ह, जबिक A और B उसी
(b)270 SSC MTS 16 August 2019 काय को 40 िदनों म कर सकते ह | C
(c)240 (Morning) उस काय को अकेले िकतने िदनों म कर
(d)293 (a) 10 सकता है ?
(b) 8 SSC MTS 16 August 2019
Q86. A is 1.5 times as efficient as B. (c) 9 (Evening)

e
A & B together complete the work in (d) 12 (a)26.67
12 days. In how many days, A alone (b)25
can complete the work? Q89.A, B and C alone can do a piece (c)23.33
A, B की तुलना म 1.5 गुना अिधक काय of work in 40, 120 and 36 days (d)30
कुशल है और A और B दोनों िमलकर

l
respectively. A and B work together
12 िदनों म इस काय को पूरा कर सकते for 20 days and leave it incomplete. Q92.A alone can complete a piece of

पूरा करे गा ?

(Evening)
(a)20
ac
है ,A अकेला इस काय को िकतने िदनों म

SSC MTS 14 August 2019


C carries out the work and finishes it
alone. How many days did it take to
complete the work?
A, B और C अकेले िकसी काय को
मशः 40, 120 और 36 िदनों म कर
work in 96 days. B is three times
efficient than A is . A worked alone
for 24 days after which B joined
him. In how many days will they
complete the remaining work
(b)26 सकते ह | A और B एक साथ िमलकर together?
(c)24 20 िदनों तक काय कर उसे अधूरा छोड़ A िकसी काय को अकेले 96 िदनों म पूरा
(d)27 दे ते ह | C काय को आगे बढ़ाता है और कर सकता है | B, A से तीन गुना कुशल
nn
उसे अकेले पूरा करता है | काय पूरा है | A ने 24 िदनों तक अकेले काय िकया
Q87. An experienced carpenter can करने म चको िकतने िदन लगे? िजसके बाद B उससे जुड़ गया | वे एक
complete a woodwork in 8 days. A SSC MTS 16 August 2019 साथ िमलकर शेष काय को िकतने िदनों
young carpenter can complete the (Afternoon) म पूरा करगे?
same work in 12 days. Both work (a)12 SSC MTS 16 August 2019
together for 3 days, and then the (b)18 (Evening)
latter leaves. How many days will be (c)20 (a)18
taken by the former to complete the (d)16 (b)16
Pi

remaining work? (c)21


एक अनुभवी बढ़ई 8 िदनों म एक लकड़ी Q90.A and B together can complete (d)14
का काम पूरा कर सकता है । एक युवा a work in 21 days. If A takes 42 days
बढ़ई उसी काम को 12 िदनों म पूरा कर to complete the work alone, how Q93. A can do a piece of work alone
सकता है । दोनों 3 िदनों के िलए एक many days will B take to complete in 10 days, whereas B alone can do it
साथ काम करते ह, और िफर युवा बढ़ई the same work alone? in 15 days. They work together and
इस काय को छोड़ दे ता है । शेष काय A और B िमलकर िकसी काय को 21 get Rs 2000 for their work. What is
पूरा करने म अनुभवी बढ़ई िकतने िदन िदनों म पूरा कर सकते ह | यिद A काय the share of B?
का समय लेगा? पूरा अकेले करने म 42 िदन लेता है , तो A अकेले 10 िदनों म एक काम को कर
SSC MTS 16 August 2019 B को उसी काय को अकेले पूरा करने म सकता है , जबिक B अकेले इसे 15 िदनों
(Morning) िकतने िदन लगगे? म कर सकता है । वे एक साथ काम
(a)3 SSC MTS 16 August 2019 करके 2000 कमाते है , B का िह ा
(b)4 (Afternoon) िकतना है ?
(c)2 (a)32

www.ssccglpinnacle.com support@ssccglpinnacle.com Ph. 09729327755, 09817390373 381


/
Days 43-46 Work and Time

s
sse
la
_c
ob
SSC MTS 19 August 2019 They worked together for 6 days and होने के 9 िदनों के बाद A ने काय करना
bo
ah
(Morning) the remaining work was completed छोड़ िदया और काय पूरा होने के 3 िदन
sm
ur
yo

(a)Rs 1200 by C alone in 6 days. A, B and C पहले B ने भी काय करना छोड़ िदया | C
e/
t.m

ने िकतने िदन काय िकया?


://

(b)Rs 800 together will do the same work in:


tp
ht

(c)Rs 1000 A िकसी काय को 15 िदनों म पूरा कर SSC MTS 20 August 2019
(d)Rs 1600 सकता है एवं B उसे िदनों म पूरा कर (Afternoon)
सकता है । उ ोंने 6 िदनों तक एक साथ (a)29
Q94.A can do a piece of work alone काम िकया और शेष काय 6 िदनों म (b)33
in 10 days. B can do the same work अकेले C ने पूरा िकया। A, B और C (c)31
in 15 days alone. They start the work िमलकर इस काय को िकतने िदन म पूरा (d)30
together but B leaves the work 2 कर दगे?
days after the start and A alone SSC MTS 19 August 2019 Q99.6 men can repair a road in 14
completes the remaining work. How (Evening) hours. How many men will be
many days the work will be (a)8 required to repair the road in 4
completed? (b)10 hours?
A िकसी काय को अकेला 10 िदन म कर 6 पु ष िकसी सड़क की मर त 14 घंटों

e
(c)6
सकता है | B उसी काय को अकेला 15 (d)12 म कर सकते ह | 4 घंटों म सड़क की
िदन म कर सकता है | वे एक साथ मर त करने के िलए िकतने पु षों की
िमलकर काय शु करते ह पर ु B Q97.A can do a piece of work in 30 आव कता होगी?
काय आरं भ होने के 2 िदन बाद काय

l
days. He completed 40% of the work SSC MTS 20 August 2019
छोड़ दे ता है तथा A शेष काय को and left it. B completed the (Evening)

(Afternoon)
(a) 17
3
ac
अकेला पूरा करता है | काय को पूरा होने
म कुल िकतने िदन लगगे?
SSC MTS 19 August 2019
remaining work in 21 days. In how
many days will A and B complete
65% of the same work together?
A िकसी काय को 30 िदनों म कर
सकता है | उसने 40% काय पूरा कर
(a)21
(b)7
(c)28
(d)14
(b) 31 उसे छोड़ िदया | B ने शेष काय 21 िदनों
3
(c) 26 म पूरा िकया | एक साथ िमलकर काय Q100. A and B can complete a piece
3
करते ए, A और B उसी काय का 65% of work in 20 days and 30 days
nn
(d) 13
3
िकतने िदनों म पूरा करगे? respectively. A works full time while
SSC MTS 20 August 2019 B works half of the time. In how
Q95. Aditya can complete a piece of
(Morning) many days the entire work will be
work in 24 days alone. Raman can
(a)13 completed?
complete the same work in 45 days
(b)10 A तथा B िकसी काय को मशः 20
alone. In how many days they can
(c)10 21 िदनों तथा 30 िदनों म पूरा कर सकते ह |
complete the same work together?
(d)13 21 A पूरे समय काय करता है जबिक B
आिद िकसी काय को अकेला 24 िदन
आधे समय काय करता है | पूरा काय
Pi

म पूरा कर सकता है | रमन उसी काय


Q98.A, B and C can complete a िकतने िदनों म समा हो जाएगा?
को अकेला 45 िदन म पूरा कर सकता है
piece of work in 36 days, 54 days SSC MTS 21 August 2019
| वे दोनों िमलकर काय को िकतने िदन म
and 108 days respectively. They (Morning)
पूरा कर सकते ह?
started working together, but 9 days (a)9
SSC MTS 19 August 2019
after the start of work, A quit (b)15
(Afternoon)
working and B also stopped working (c)12
(a) 720
13
3 days before the work was (d)10
1080
(b) 43 completed. How many days did C
(c) 360
23 work? Q101. Satyam and Shivam can do a
(d) 960
33 A, B और C, एक काय को मशः 36 piece of work in 10 days and 15 days
िदनों, 54 िदनों और 108 िदनों म पूरा respectively. They both started
Q96. A can do a piece of work in 15 कर सकते ह | उ ोंने एक साथ काय together. After 4 days Satyam quits
days and B can do it in 22 21 days. करना शु िकया, लेिकन काय शु working. The remaining work is to
be completed by Shivam. In how

www.ssccglpinnacle.com support@ssccglpinnacle.com Ph. 09729327755, 09817390373 382


/
Days 43-46 Work and Time

s
se
s
la
_c
ob
many days the entire work will be (c)12 18 days. C alone will do the same
bo
ah
completed? (d)10 work in:
sm
ur
yo

स म और िशवम िकसी काय को A एक काम का एक ितहाई 5 िदनों म


e/
t.m

मशः 10 िदनों और 15 िदनों म कर पूरा कर सकता है और B उसी काम के


://

Q104. A is more efficient than B and


tp
ht

सकते ह | दोनों साथ िमलकर शु करते they together can complete a work in 5 भाग को 10 िदनों म पू रा कर सकता
2

ह | 4 िदन बाद स म काय करना छोड़ 24 days. Had A done 50% of the है । वे 6 िदनों तक एक साथ काम करते
दे ता है | शेष काय िशवम को पूरा करना work and then B, the remaining ह। शेष काय C ारा 18 िदनों म पूरा
है | पूरा काय िकतने िदनों म समा हो work, then the work would have िकया जाता है । अकेले C इस काम को
जाएगा? been done in 50 days. B alone will िकतने िदनों म पूरा करे गा ?
SSC MTS 21 August 2019 complete 40% of the same work in: SSC MTS 22 August 2019
(Morning) A, B की तुलना म अिधक काय कुशल है (Afternoon)
(a)9 और वे एक साथ 24 िदनों म एक काम (a)50
(b)12 को पूरा कर सकते ह। यिद A ने 50% (b)30
(c)10 काम िकया और िफर B ने शेष काय, तो (c)25
(d)6 वह काय 50 िदनों म पूरा हो जाता है । B (d)45
अकेले उसी काम का 40% भाग िकतने

e
Q102.A, B and C can do a work in िदनों म पूरा करे गा? Q107.To do a certain work, the ratio
12, 15 and 20 days respectively. In SSC MTS 22 August 2019 of efficiencies of A and B is 3:7,
how many days will they complete (Morning) working together, they can complete

l
the same work together? a work in 14 days. B started the
A, B तथा C िकसी काय को मशः 12, (a)16 days work and after working for 8 days,
ac
15 तथा 20 िदनों म कर सकते ह | एक
साथ िमलकर वे उसी काय को िकतने
िदनों म पूरा करगे?
SSC MTS 21 August 2019
(Afternoon)
(b)24 days
(c)21 days
(d)20 days

Q105. 36 men and 48 women can do


he left and A completed the
remaining work. For how many days
did A work?
एक िनि त काय करने के िलए, A और
B की मता का अनुपात 3: 7 है , एक
(a) 5 a certain work in one day whereas 6 साथ काम करते ए, वे 14 िदनों म इस
(b)4 men and 12 women can do it in 5 काम पूरा कर सकते ह। B ने काम शु
(c)7.5 days. The number of women िकया और 8 िदनों तक काम करने के
nn
(d)6 required to do the same work in 8 बाद, उसने छोड़ िदया और A ने शेष
days is: काय पूरा कर िलया। A ने िकतने िदनों
Q103. A can complete a work in 20 36 पु ष और 48 मिहलाएं एक िदन म तक काम िकया?
days and B in 22 21 days. They both एक काम को पूरा कर सकते ह जबिक 6 SSC MTS 22 August 2019
work together for 6 days and the पु ष और 12 मिहलाएं इसे 5 िदनों म (Evening)
remaining work is completed by C in पूरा कर सकते ह। 8 िदनों म इस काम (a)28
26 days. In how many days the work को करने के िलए आव क मिहलाओं (b)30
will be completed, if all the three की सं ा िकतनी है ? (c)24
Pi

work together? / िकसी काय को A, 20 SSC MTS 22 August 2019 (d)27


िदनों म पूरा कर सकता है और B उसे (Afternoon)
22 21 िदनों म पूरा कर सकता है | वे दोनों (a)10 Q108. A and B can complete a
िमलकर 6 िदनों तक एक साथ काय (b)15 certain work in 24 days and 42 days,
करते ह और शेष काय को C ारा 26 (c)18 respectively. A started the work and
िदनों म पूरा िकया गया है | तीनों एक (d)12 after x days, B joined him and the
साथ िमलकर उसी काय को िकतने िदनों whole work was completed in 20
म पूरा करगे? Q106. A can complete one-third of a days. The value of x is:
SSC MTS 21 August 2019 work in 5 days and B can do 52 th of A और B एक काय को मशः 24 िदनों
(Evening) the same work in 10 days. They और 42 िदनों म पूरा कर सकते है । A ने
work together for 6 days. The काम शु िकया और x िदनों के बाद, B
(a)8 remaining work is completed by C in भी उस काय म लग गया,और सारा काय
(b)9

www.ssccglpinnacle.com support@ssccglpinnacle.com Ph. 09729327755, 09817390373 383


/
Days 43-46 Work and Time

s
sse
la
_c
ob
20 िदनों म पूरा हो गया। x का मान minutes, what will be the ratio of काय अकेले पूरा करने म B को िकतने
bo
ah
िकतना है ? chemical B in the tank? िदन लगगे ?
sm
ur
yo

SSC MTS 22 August 2019 तीन पाइप x , y , z एक टक म तीन िभ SSC CGL 3 March 2020
e/
t.m

िभ रसायन A , B , C भरते है | ये पाइप


://

(Evening) (Afternoon)
tp
ht

(a)11 मश : 20 , 25 और 40 िमनट म टक (a) 20 43 days


(b)13 को भर सकते है | यिद सभी पाइपों को (b) 16 51 days
(c)7 10 िमनट के िलए खुला छोड़ िदया जाता (c) 16 53 days
(d)9 है तो टक म रसायन B का अनुपात ा
(d) 20 21 days
होगा ?
Q109. A and B have to do 13 part of SSC CPO 14 March 2019
15
Q3. A can finish work in 20 days
a work together and B and C have to (Evening)
and B can finish the same work in 25
11 th part of the work together. If
do 20 (a) 74
13
days. They began together, but B left
the difference between the wages of (b) 23 the work after 5 days. How many
A and C is Rs. 7600, then the total (c) 8
23 more days will A take to finish the
wages of A, B and C is : (d) 11
remaining work?
15

e
A तथा B को एक साथ िमलकर 15 13 भाग
A 20 िदनों म काय समा कर सकता है
काय करना है और B तथा C को SSC CGL 2019 TIER I तथा B इसी काय को 25 िदनों म पूरा
िमलकर 20
11 भाग काय करना है | यिद A
कर सकता है | उ ोंने एक साथ काय
तथा C की मजदू री का अंतर 7600 है , तो Q1. A, B and C can individually करना शु िकया, लेिकन B ने 5 िदनों

l
A, B तथा C की कुल मजदू री है : complete a piece of work in 24 days, के बाद काय छोड़ िदया | शेष काय पूरा
SSC MTSac 21 August 15 days and 12 days respectively. B करने म A को िकतने अित र िदन
2019(Afternoon) and C started the work and worked लगगे ?
(a) Rs. 24000 for 3 days and left. The number of SSC CGL 3 March 2020 (Evening)
(b) Rs. 18000 days required by A alone to (a) 8
(c) Rs. 36000 complete the remaining work is: (b) 21
(d) Rs. 56000 A, B और C अकेले-अकेले िकसी काय (c) 16
को मशः 24, 15 और 12 िदनों म पूरा (d) 11
Q110. 12 buckets of water fill a tank कर सकते ह | B और C ने काय करना
nn
when the capacity of each bucket is शु िकया तथा 3 िदनों तक काय करके Q4. A can complete a certain work
13.5 litres. How many buckets will छोड़ िदया| शेष काय अकेले पूरा करने in 30 days. B is 25% more efficient
be needed to fill the same tank, if the म A को िकतने िदन लगगे ? than A and C is 20% more efficient
capacity of each bucket in 9 litres? / SSC CGL 3 March 2020 than B. They all worked together for
िकसी टं की को भरने म 12 बा ी पानी (Morning) 3 days. B alone will complete the
लगता है जब ेक बा ी की धा रता (a) 15 21 remaining work in: /
13.5 लीटर है | यिद ेक बा ी की (b) 18 A िकसी िनि त काय को 30 िदनों म पूरा
धा रता 9 लीटर हो, तो इसी टं की को (c) 13 51 कर सकता है | B, A की तुलना म 25%
Pi

भरने के िलए िकतनी बा यों की (d) 11 अिधक काय कुशल है तथा C, B की


आव कता होगी ? तुलना म 20% अिधक काय कुशल है |
SSC CPO 16 March 2019 Q2. A can complete a certain piece उन सभी ने एक साथ तीन िदनों तक
(Evening) of work in 40 days. B is 25% more काय िकया | शेष काय पूरा करने म B
(a)15 efficient than A and C is 28% more को अकेले िकतने िदन लगगे ?
(b)16 efficient than B. They work together SSC CGL 4 March 2020
(c)18 for 5 days. The remaining work will (Morning)
(d)17 be completed by B alone, in: (a) 15 days
A िकसी िनि त काय को 40 िदनों म पूरा (b) 12 days
Q111. Three pipes x, y, z leave three कर सकता है | B, A की तुलना म 25% (c) 20 days
different chemicals A, B, C in a tank. अिधक काय कुशल है तथा C, B की (d) 18 days
These pipes can fill the tank in 20, तुलना म 28% अिधक काय कुशल है | वे
25 and 40 minutes, respectively. If 5 िदनों तक एक साथ काय करते ह | शेष Q5. A and B, working together can
all the pipes are left open for 10 complete a work in d days. Working

www.ssccglpinnacle.com support@ssccglpinnacle.com Ph. 09729327755, 09817390373 384


/
Days 43-46 Work and Time

s
sse
la
_c
ob
alone, A takes (8+d) days and B काय कुशल है | एक साथ काय करते ए, Q10. Amit and Sunil together can
bo
ah
takes (18+d) days to complete the वे इस काय को 21 िदनों म पूरा कर complete a work in 9 days, Sunil and
sm
ur
yo

same work. A works for 4 days. The सकते ह | Y तथा Z ने 35 िदनों तक एक Dinesh together can complete the
e/
t.m

साथ काय िकया | शेष काय A अकेला


://

remaining work is completed by B same work in 12days, and Amit and


tp
ht

alone, in: / िकतने िदनों म पूरा करे गा ? Dinesh together can complete the
एक साथ काय करते ए A और B SSC CGL 5 March same work in 18 days. In how many
िकसी काय को d िदनों म पूरा कर सकते 2020(Afternoon) days will they complete the work if
ह | अकेले काय करते ए इसी काय को (a) 8 days Amit, Sunil and Dinesh work
पूरा करने म A को ( 8+d ) तथा B को ( (b) 6 days together? /
18+d ) िदन लगते ह | A 4 िदनों तक (c) 5 days अिमत और सुनील एक साथ िकसी काय
काय करता है | शेष काय B अकेले (d) 4 days को 9 िदनों म पूरा कर सकते ह | सुनील
िकतने िदनों म पूरा करे गा ? तथा िदनेश एक साथ इस काय को 12
SSC CGL 4 March 2020 Q8. Sixteen men can finish a work in िदनों म पूरा कर सकते ह तथा अिमत
(Afternoon) 8 days. Eight men and nine women और िदनेश इस काय को एक साथ 18
(a) 24days working together can finish the same िदनों म पूरा कर सकते ह | यिद अिमत,
work in 10 days. In how many days
सुनील तथा िदनेश एक साथ काय कर,

e
(b) 16days
will twenty women finish the same
(c) 18days तो वे इस काय को िकतने िदनों म पूरा
work?
(d) 20days सोलह पु ष िकसी काय को 8 िदनों म करगे ?
पूरा कर सकते ह | आठ पु ष तथा नौ SSC CGL 6 March 2020

l
Q6. Four men and 6 women can मिहलाएं एक साथ काय करके इसी काय (Afternoon)
complete a certain piece of work in 5 को 10 िदनों म पूरा कर सकती ह | इस (a) 14 days
ac
days whereas three men and 4
women can complete it in 7 days.
How many should assist 25 women
to complete 2 21 times the same work
in 5 days? / 4 पु ष और 6 मिहलाएं
काय को 20 मिहलाएं िकतने िदनों म पूरा
करगी ?
SSC CGL 5 March 2020 (Evening)
(a) 13
(b) 12
(c) 9
(b) 16 days
(c) 12 days
(d) 8 days

Q11. A, B and C can individually


एक िनि त काम को 5 िदनों म पूरा कर (d) 11 complete a task in 20 days, 16 days
सकते ह, जबिक तीन पु ष और 4 and 30 days, respectively. If A and B
मिहलाएं इसे 7 िदनों म पूरा कर सकती started working on the task, and they
nn
Q9. A, B and C can individually
ह। समान काय के 2 21 गुना काय को 5 complete a task in 24 days, 20 days worked for 4 days and left, then the
िदनों म पूरा करने के िलए िकतनी और and 18 days respectively. B and C number of days required by C to
मिहलाओं को 25 मिहलाओं की सहायता start the task, and they work for 6 finish the remaining tasks is:
करनी होगी? days and leave. The number of days A, B तथा C अकेले काय करते ए
SSC CGL 4 March 2020(Evening) required by A alone to finish the िकसी काय को मशः 20, 16 तथा 30
remaining task is: िदनों म पूरा कर सकते ह | यिद A और
(a) 8 A, B और C अकेले काय करते ए B ने यह काय शु िकया तथा उ ोंने 4
िकसी काय को मशः 24 िदन, 20 िदन िदन काय करके छोड़ िदया, तो शेष
Pi

(b) 10
(c) 4 तथा 18 िदनों म पूरा कर सकते ह | B काय पूरा करने म C को िकतने िदन
(d) 5 और C ने काय शु िकया और उ ोंने 6 लगगे ?
िदन काय करके छोड़ िदया | शेष काय SSC CGL 6 March 2020 (Evening)
Q7. To complete a certain task, X is अकेले पूरा करने म A को िकतने िदन (a) 16 21 days
40% more efficient than Y, and Z is लगगे ? (b) 13 days
40% less efficient than Y. Working SSC CGL 6 March 2020 (c) 12 21 days
together, they can complete the task (Morning) (d) 10 days
in 21 days. Y and Z together worked (a) 12 21 days
for 35 days. The remaining work (b) 15 32 days Q12. Ten men or twelve women can
will be completed by A alone in: (c) 10 days finish the same work in 10 days. If 5
िकसी िनि त काय को पूरा करने के (d) 8 54 days men and 2 women undertake the
िलए, X, Y की तुलना म 40% अिधक work together; how many days will
काय कुशल है तथा Z, Y से 40% कम they take to complete the work?

www.ssccglpinnacle.com support@ssccglpinnacle.com Ph. 09729327755, 09817390373 385


/
Days 43-46 Work and Time

s
sse
la
_c
ob
दस पु ष या बारह मिहलाएं एक काय काय िकया तथा िफर राम चला गया | Q1. 15 men can complete a task in
bo
ah
को 10 िदनों म पूरा कर सकती ह | यिद राम के जाने के िकतने िदनों बाद ाम 10 days. In how many days can 20
sm
ur
yo

5 पु ष तथा 2 मिहलाओं ने उस काय को अकेला इस काय को पूरा कर पायेगा ? men complete the same task?
e/
t.m

शु िकया, तो काय पूरा करने म उ 15 पु ष िकसी काय को 10 िदनों म पूरा


://

SSC CGL 9 March 2020(Morning)


tp
ht

िकतने िदन लगगे ? (a) 2 days कर सकते ह। इसी काय को 20 पु ष


SSC CGL 7 March 2020 (b) 3 days िकतने िदनों म म पूरा करगे?
(Morning) (c) 4 days CHSL 12-10-2020 (morning shift)
(a) 40 (d) 1 21 days (a) 7.5 days
(b) 15 (b) 5.5 days
(c) 60 Q16. A, B and C can individually (c) 6.5 days
(d) 20 complete a task in 24 days, 16 days (d) 8.5 days
and 32 days respectively. If A and C
Q13. If 18 men can cut a field in 35 start the work and worked for 6 days Q2. 30 men working 8 hours per day
days, then in how many days can 21 and left, then the number of days can dig a pond in 16 days. By
men cut the same field? / यिद 18 required by B to complete the working how many hours per day
पु ष 35 िदनों म एक खेत को काट

e
remaining task is: can 32 men dig two same ponds, in
सकते ह, तो 21 पु ष यह खेत िकतने A, B और C गत प से िकसी 20 days?
िदनों म काट सकते ह? काय को मशः 24, 16 और 32 िदनों म 30 पु ष ितिदन 8 घंटे काय करके 16
SSC CGL 7 March पूरा कर सकते ह | यिद A और C ने िदनों म एक तालाब खोद सकते ह। 32
पु ष ितिदन िकतने घंटे काय करके 20

l
2020(Afternoon) काय शु िकया तथा 6 िदनों तक काय
(a) 27 करके छोड़ िदया, तो शेष काय पूरा िदनों म दो समान तालाब खोद सकते ह?
(b) 28
(c) 30
(d) 32
ac
Q14. A can do a piece of work in 6
करने म B को िकतने िदन लगगे ?
SSC CGL 9 March 2020
(Afternoon)
(a) 17 21
(b) 7 21
CHSL 12-10-2020 (afternoon shift)
(a) 6 hours per day/ ितिदन 6 घंटे
(b) 5 hours per day/ ितिदन 5 घंटे
(c) 7 hours per day/ ितिदन 7 घंटे
(d) 8 hours per day/ ितिदन 8 घंटे
days. B can do it in 9 days. With the
(c) 12 21
assistance of C they completed the Q3. Shyam can complete a task in 12
(d) 9
work in 3 days. In how many days days by working 10 hours a day.
nn
can C alone do the work? / A 6 िदनों How many hours a day should he
Q17. Eight persons can finish a work
म एक काम कर सकता है । B इसे 9 work to complete the task in 8 days?
in 20 days. After 5 days they were
िदनों म कर सकता ह। C की सहायता से ाम एक िदन म 10 घंटे काय करके
requested to complete the work in
उ ोंने 3 िदनों म काम पूरा कर िलया। िकसी काय को 12 िदनों म पूरा कर
the next 8 days. How many more
तो C अकेले िकतने िदनों म यह काम सकता है । इस काय को 8 िदनों म पूरा
persons should join the group to
करे गा? करने के िलए उसे ितिदन िकतने घंटे
fulfill the requirement?
SSC CGL 7 March 2020(Evening) काय करना चािहए?
आठ 20 िदनों म एक काम पूरा
(a) 16 CHSL 12-10-2020 (Evening shift)
Pi

कर सकते ह। 5 िदनों काम करने के


(b) 8 (a) 14
बाद उ अगले 8 िदनों म काम पूरा
(c) 18 (b) 16
करने का अनुरोध िकया गया।
(d) 12 (c) 12
आव कता को पूरा करने के िलए
(d) 15
िकतने और यों को समूह म
Q15. Ram and Shyam can complete
शािमल करना चािहए?
a task in 6 32 days and 15 days Q4. 5 men and 8 women can
SSC CGL 9 March 2020(Evening)
respectively. They work together for complete a task in 34 days, whereas
(a) 23
4 days and then Ram leaves. In how 4 men and 18 women can complete
(b) 15
many days after Ram leaves, will the same task in 28 days. In how
(c) 7
Shyam complete the remaining task many days can the same task can be
(d) 12
alone? / राम तथा ाम िकसी काय को completed by 3 men and 5 women?
SSC CHSL 2019
मशः 6 32 िदनों म तथा 15 िदनों म पूरा 5 पु ष तथा 8 मिहलाएं िकसी काय को
कर सकते ह | उ ोंने 4 िदनों तक साथ 34 िदनों म पूरा कर सकते ह। जबिक 4
पु ष और 18 मिहलाएं इसी काय को 28

www.ssccglpinnacle.com support@ssccglpinnacle.com Ph. 09729327755, 09817390373 386


/
Days 43-46 Work and Time

s
sse
la
_c
ob
िदनों म पूरा कर सकते ह। िकतने िदनों Q7. A can do a work in 12 days and
bo
ah
म यही काय 3 पु षों तथा 5 मिहलाओं B can do the same work in 16 days. Q10. A and B separately can build a
sm
ur
yo

के ारा िकया जा सकता है ? If they work on it together for 4 wall in 12 days and 16 days,
e/
t.m
://

CHSL 13-10-2020 (Morning Shift) days, then the fraction of the work respectively. If they work
tp
ht

(a)64 that is left is: alternatively, starting with A, in how


(b)72 A एक काय को 12 िदनों म कर सकता many days will the wall be built?
(c)56 है और B उसी काय को 16 िदनों म कर A और B अलग-अलग मशः 12 िदनों
(d)36 सकता है । यिद वे 4 िदनों के िलए इस पर और 16 िदनों म एक दीवार का िनमाण
एक साथ काय करते ह, िफर जो काय कर सकते ह। यिद वे एक िदन छोड़कर
Q5. A can finish a piece of work in a बचा है उसका अंश है : काय करते ह, तो A से शु करके,
certain number of days. B takes 45% CHSL 15-10-2020 (Afternoon िकतने िदनों म दीवार बनाई जाएगी?
more number of days to finish the shift) CHSL 16-10-2020 (Afternoon
same work independently. They (a) 53 shift)
worked together for 58 days and (b) 9 (a) 7 32 days
16
then the remaining work was done (c) 5 (b) 12 32 days
12

e
by B alone in 29 days. In how many 7 (c) 13 32 days
(d) 8
days could A have completed the
(d) 6 43 days
work, had he worked alone?
A िकसी काय को कुछ िनि त िदनों म
समा कर सकता है जबिक B को Q8. How many men will be required Q11. A can complete a task in 18

l
तं प से इसी काय को करने म to plough 50 acres of land in 10 days days while B can complete the same
ac
45% अिधक िदन लगते ह। उ ोंने एक
साथ 58 िदनों तक काय िकया तथा शेष
काय B ने अकेले 29 िदनों म पूरा िकया।
A इस काय को अकेले िकतने िदनों म
समा कर सकता था?
CHSL 13-10-2020 (afternoon
if 15 men required 6 days to plough
10 acres of land?
अगर 10 आदिमयों को 10 एकड़ ज़मीन
जोतने के िलए 15 िदन की ज़ रत होती
है तो 10 िदनों म 50 एकड़ ज़मीन जोतने
task in 12 days. If both work
together for 6 days, and then A
leaves. In how many days will B
complete the task?
A िकसी काय को 18 िदनों म पूरा कर
के िलए िकतने आदिमयों की ज़ रत सकता है जबिक B उसी काय को 12
Shift)
होगी? िदनों म पूरा कर सकता है । यिद दोनों 6
(a) 110 days/ िदन
CHSL 15-10-2020 (Evening shift) िदनों के िलए एक साथ काय करते ह,
(b) 118 days/ िदन
nn
(a) 45 और िफर A काय छोड़ दे ता है , तो B
(c) 98 days/ िदन
(d) 120 days/ िदन (b) 40 काय को िकतने िदनों म पूरा करे गा?
(c) 55 CHSL 19-10-2020 (Morning shift)
(d) 50 (a) 3 days
Q6. Ramu works 4 times as fast as (b) 6 days
Somu. If Somu can complete a work (c) 2 days
in 20 days independently, then the Q9. Raju can finish a piece of work (d) 4 days
number of days in which Ramu and in 20 days. He worked at it for 5
Pi

Somu together can complete the days and then Jakob alone finished
work is: the remaining work in 15 days. In Q12. Amir and Akbar can finish a
रामू सोमू की तुलना म 4 गुणा अिधक how many days can both finish it task in 30 days and 15 days,
तेज़ी से काय करता है । यिद सोमू िकसी together? respectively. Akbar worked on the
काय को तं प से 20 िदनों म कर राजू 20 िदनों म काय ख कर सकता task for 8 days and left the job. In
सकता है , तो रामू और सोमू िमलकर है । उसने 5 िदनों तक इस पर काय िकया how many days can Amir alone
उस काय को िकतने िदनों म पूरा करगे ? और िफर जेकॉब ने अकेले 15 िदनों म finish the remaining work?
CHSL 14-10-2020 (Evening Shift) शेष काय पूरा कर िलया। दोनों इसे आिमर और अकबर मशः 30 िदनों
(a) 5 days िकतने िदनों म पूरा कर सकते ह? और 15 िदनों म एक काय पूरा कर
(b) 4 days CHSL 16-10-2020 (Morning Shift) सकते ह। अकबर ने 8 िदनों तक काय
(c) 6 days (a) 10 days िकया और नौकरी छोड़ दी। शेष काय
(d) 3 days (b) 12 days को आिमर अकेले िकतने िदनों म पूरा
(c) 16 days कर सकता है ?
(d) 20 days CHSL 19-10-2020 (Evening shift)

www.ssccglpinnacle.com support@ssccglpinnacle.com Ph. 09729327755, 09817390373 387


/
Days 43-46 Work and Time

s
sse
la
_c
ob
(a) 14 days (b) 2/3 CHSL 26-10-2020 (Morning Shift)
bo
ah
(b) 15 days (c) 1/3 (a) 15 days
sm
ur
yo

(c) 16 days (d) 1/4 (b) 18 days


e/
t.m
://

(d) 17 days (c) 12 days


tp
ht

Q16. A is twice as good a workman (d) 10 days


as B and together they finish a piece
Q13. A and B together can complete of work in 22 days. In how many Q19. Ravi and Mohan together can
a piece of work in 15 days. B and C days will A alone finish the same complete a task in 3 days. Ravi alone
together can do it in 24 days. If A is work? can complete the same task in 7
twice as good a workman as C, then A, B से दोगुना अ ा कारीगर है और days. How many days will Mohan
in how many days can B alone साथ म वे 22 िदनों म एक काय पूरा कर alone take to complete the same
complete the work? लेते ह। A अकेले िकतने िदनों म काय task?
A और B िमलकर 15 िदनों म एक काय पूरा करे गा रिव और मोहन एक साथ 3 िदनों म एक
पूरा कर सकते ह। B और C िमलकर CHSL 21-10-2020 (Afternoon काय पूरा कर सकते ह। अकेले रिव 7
इसे 24 िदनों म कर सकते ह। यिद A, C Shift) िदनों म उस काय पूरा कर सकता है ।
से दोगुना अ ा काय करने वाला है , तो उसी काय को पूरा करने म मोहन को

e
(a) 11 days
B अकेले िकतने िदनों म काय पूरा कर (b) 44 days अकेले िकतने िदन लगगे?
सकता है (c) 30 days CHSL 26-10-2020 (Afternoon
CHSL 20-10-2020 (Morning shift) (d) 33 days Shift)

l
(a) 40 days (a) 5 41 days
(b) 60 days (b) 4 51 days
(c) 52 days
(d) 45 days
ac
Q14. Antony and Vikash together
can complete a piece of work in 20
Q17. A and B can do a work in 12
days, B and C can do it in 15 days
and C and A can do it in 20 days. If
A, B and C work together, then they
will complete the same work in:
(c) 10 days
(d) 4 days

Q20. A and B working together can


days and vikash alone can complete A और B एक काय को 12 िदनों म कर do 45% of the work in 9 days. A
it in 25 days. In how many days can सकते ह, B और C इसे 15 िदनों म कर alone can do the work in 30 days.
Antony alone complete the same सकते ह और C और A इसे 20 िदनों म
nn
How many days will B alone take to
work? कर सकते ह। यिद A, B और C एक do the same work?
एं टनी और िवकास एक साथ 20 िदनों म साथ काय करते ह, तो वे उसी काय को A और B एक साथ काय करते ए 9
काय पूरा कर सकते ह और अकेले िकतने समय म पूरा करगे? िदनों म 45% काय कर सकते ह। एक
िवकास इसे 25 िदनों म पूरा कर सकता CHSL 21-10-2020 (Evening Shift) अकेला 30 िदनों म काय कर सकता है ।
है । एं टनी अकेले इसी काय को िकतने (a) 12 days B अकेले उसी काय को करने म िकतने
िदनों म पूरा कर सकता है ? (b) 5 days िदन लगाएगा?
CHSL 20-10-2020 (Evening shift) (c) 10 days CHSL 26-10-2020 (Evening Shift)
(a) 90 days (d) 14 days
Pi

(a) 60 days
(b) 80 days (b) 70 days
(c) 100 days Q18. A and B can do a work in 15 (c) 48 days
(d) 110 days days and 10 days respectively. They (d) 50 days
begin the work together but B leaves
Q15. A can do work in 15 days and after two days. Now A completes the Q21. X and Y together can finish a
B can do it in 10 days. If they work remaining work. The total number of piece of work in 15 days, while Y
together for 4 days, then the fraction days needed for the completion of alone can finish it in 40 days.X alone
of the work left is: the work is: can finish the work in:
A 15 िदनों म एक काय कर सकता है A और B मशः 15 िदनों और 10 िदनों X और Y िमलकर 15 िदनों म एक काय
और B इसे 10 िदनों म कर सकता है । म एक काय कर सकते ह। वे एक साथ पूरा कर सकते ह, जबिक Y अकेले इसे
यिद वे 4 िदनों के िलए एक साथ काय काय शु करते ह लेिकन B दो िदनों के 40 िदनों म पूरा कर सकता है । X अकेले
करते ह, तो बचे ए काय का िह ा है : बाद छोड़ दे ता ह। अब A शेष काय को काय पूरा कर सकता है :
CHSL 21-10-2020 (Morning Shift) पूरा करता है । काय पूरा होने के िलए CHSL 17-03-2020 (Morning Shift)
(a) 3/4 आव क कुल िदनों की सं ा है :

www.ssccglpinnacle.com support@ssccglpinnacle.com Ph. 09729327755, 09817390373 388


/
Days 43-46 Work and Time

s
sse
la
_c
ob
(a) 24 days CHSL 18-03-2020 (Morning shift) िमलकर िकतने िदनों म उसी काय को
bo
ah
(b) 26 days (a) 12 days पूरा कर सकते ह?
sm
ur
yo

(c) 25 days (b) 10 days CHSL 19-03-2020 (Morning Shift)


e/
t.m
://

(d) 23 days (c) 15 days (a) 8


tp
ht

(d) 18 days (b) 4


Q22. P and Q can finish a work in 10 (c) 2
days and 5 days, respectively. Q Q25. P can work thrice as fast as Q. (d) 6
worked for 2 days and left the job. In Working independently, Q can
how many days can P alone finish complete a task in 24 days. In how Q28. Kamal and Anil can dig a pond
the remaining work? many days can P and Q together in 8 days and 14 days, respectively.
P और Q मशः 10 िदनों और 5 िदनों म finish the same task? If the total expense of digging is
एक काय ख कर सकते ह। Q ने 2 P, Q से तीन गुना तेजी से काय कर Rs.4400, then how much money will
िदन काय िकया और नौकरी छोड़ दी। P सकता है । तं प से काय करते ए, Anil earn?
शेष काय को िकतने िदनों म पूरा कर Q एक काय को 24 िदनों म पूरा कर कमल और अिनल मशः 8 िदन और
सकता है ? सकता है । P और Q एक साथ उसी काय 14 िदन म एक तालाब खोद सकते ह।
को िकतने िदनों म पूरा कर सकते ह? यिद खुदाई का कुल खच 4400 पये है ,

e
CHSL 17-03-2020 (Afternoon
shift) CHSL 18-03-2020 (Afternoon तो अिनल िकतना पैसा कमाएगा?
(a) 4 days Shift) CHSL 19-03-2020 (Afternoon
(b) 10 days (a) 5 Shift)

l
(c) 6 days (b) 4 (a) 1500
(d) 8 days (c) 6 (b) 1300
ac
Q23. Smith and Ajit can complete a
task in 12 days and 18 days
respectively. If they work together
on a task for 4 days, then the fraction
(d) 8

Q26. P can do a work in 10 days and


Q can do the same work in 15 days.
(c) 1600
(d) 1400

Q29. Ravi can complete a task in 6


days and Mohan can complete the
of task that will be left is: If they work on it together for 3 same task in 9 days. In how many
थ और अजीत मशः 12 िदनों और days, then the fraction of the work days can Ravi and Mohan together
18 िदनों म एक काय पूरा कर सकते ह। that is left is: complete the same task?
nn
यिद वे 4 िदनों के िलए एक काय पर एक P एक काय को 10 िदनों म कर सकता रिव एक टा को 6 िदनों म पूरा कर
साथ काय करते ह, तो जो काय शेष है और Q वही काय 15 िदनों म कर सकता है और मोहन उसी टा को 9
होगा वह है : सकता है । यिद वे 3 िदनों तक इस पर िदनों म पूरा कर सकता है । रिव और
CHSL 17-3-2020 (Evening Shift) एक साथ काय करते ह, तो जो काय बचा मोहन एक साथ उसी टा को िकतने
(a) 59 है उसका अंश है : िदनों म पूरा कर सकते ह?
(b) 2 CHSL 18-03-2020 (Evening Shift) CHSL 19-03-2020 (Evening Shift)
9
(c) 1 (a) 31 (a) 15 days
9
(b) 2 (b) 3 52 days
Pi

(d) 4 3
9
(c) 1 (c) 3 53 days
2

Q24. 25 men can complete a task in (d) 4 (d) 9 days


3
16 days. Four days after they started
working, 5 more men, with equal Q27. Ravi, Mohan and Govind can SSC CGL 2019 TIER-II
workmanship, joined them. How complete a task in 12 days, 10 days Q30. A can do a piece of work in 15
many days will be needed by all to and 15 days respectively. In how days, B is 25% more efficient than A
complete the remaining task? many days can Ravi, Mohan and and C is 40% more efficient than B.
25 पु ष एक काय को 16 िदनों म पूरा Govind together complete the same A and C work together for 3 days
कर सकते ह। काय शु करने के चार task? and then C leaves A and B together
िदन बाद, 5 और पु ष, समान कारीगरी रिव, मोहन और गोिवंद मशः 12 िदनों, will complete the remaining work in:
के साथ, उनके साथ शािमल हो गए। शेष 10 िदनों और 15 िदनों म एक काय पूरा A 15 िदनों म एक काय करता है । B, A
काय को पूरा करने के िलए सभी को कर सकते ह। रिव, मोहन और गोिवंद की तुलना म 25% अिधक कुशल है और
िकतने िदनों की आव कता होगी? C, B की तुलना म 40% अिधक कुशल

www.ssccglpinnacle.com support@ssccglpinnacle.com Ph. 09729327755, 09817390373 389


/
Days 43-46 Work and Time

s
sse
la
_c
ob
है । A और C एक साथ 3 िदनों के िलए एक काय करने के िलए, A और B की respectively. They started the work
bo
ah
काय करते ह और िफर C चला जाता है । मता का अनुपात 7: 5 है । एक साथ together, but B and C left 5 days and
sm
ur
yo

A और B एक साथ िमलकर शेष काय काय करते ए, वे 17 21 िदनों म काय 10 days, respectively, before the
e/
t.m

को पूरा करते ह:
://

पूरा कर सकते ह, अकेले A 60% काय completion of the work. In How


tp
ht

CGL 2019 Tier-II (15-11-2020) को कब तक पूरा करगे : many days was the work finished?
(a) 2 21 days CGL 2019 Tier-II (16-11-2020) A, B और C अलग-अलग मश 18, 36
(b) 3 21 days (a) 16 days और 54 िदनों म एक काय कर सकते ह।
(c) 4 days (b) 18 days उ ोंने एक साथ काय शु िकया,
(d) 3 days (c) 21 days लेिकन B और C ने काय पूरा होने से
Q31. A can do 20% of the work in 4 (d) 15 days मश: 5 िदन और 10 िदन पहले छोड़
days, B can do 33 31 % of the same िदया। िकतने िदनों म काय पूरा हो गया
Q34. Two men and 7 women can था?
work in 10 days. They worked
complete a work in 28 days, whereas CGL 2019 Tier-II (18-11-2020)
together for 9 days. C completed the
6 men and 16 women can do the (a) 13 days
remaining work in 6 days. B and C
work in 11 days. In how many days (b) 12 days
together will complete 75% of the

e
will 5 men and 4 women, working (c) 14 days
work in :
together, will complete the work? (d) 15 days
A 4 िदनों म 20% काय कर सकता है , B
वही काय का 33 31 % भाग 10 िदनों म दो पु ष और 7 मिहलाएं 28 िदनों म एक
काय को पूरा कर सकते ह, जबिक 6 Q37. A and B can do a piece of work
कर सकता है । उ ोंने 9 िदनों तक एक

l
पु ष और 16 मिहलाएं 11 िदनों म काय in 18 days. B and C together can do
साथ काय िकया। C ने शेष काय 6 िदनों
को पूरा कर सकते ह। 5 पु ष और 4 it in 30 days. If A is twice as good a
म पूरा कर िलया। B और C िमलकर
ac मिहलाएं एक साथ िकतने िदनों म काय workman as C. B alone can do the
75% काय पूरा करगे
को पूरा करगे work in
CGL 2019 Tier-II (15-11-2020)
CGL 2019 Tier-II (16-11-2020) A और B 18 िदनों म एक काय को कर
(a) 12 days
(a) 18 सकते ह। B और C िमलकर इसे 30
(b) 15 days
(b) 14 िदनों म कर सकते ह। यिद A, C से
(c) 10 days
(c) 22 दोगुना कुशल है तो B अकेले िकतने िदन
(d) 9 days
(d) 20 म काय कर सकता है ?
CGL 2019 Tier-II (18-11-2020)
nn
Q32. Three men and 4 women can
Q35. A can do 1 of work in 30 (a) 80 days
do a piece of work in 7 days, 3
days, B can do 2 of the same work (b) 100 days
whereas 2 men and 1 woman can do 5
(c) 75 days
it in 14 days. Seven women will in 24 days. They worked together for
(d) 90 days
complete the same work in: 20 days. C completed the remaining
तीन पु ष और 4 मिहलाएं एक काय को work in 8 days. Working together A,
Q38. A and B together can do a
7 िदनों म कर सकते ह, जबिक 2 पु ष B and C will complete the work in:
piece of work in 12 days. A alone
और 1 मिहला 14 िदनों म कर सकते ह। A 30 िदनों म 31 काय कर सकता है , B,
can do it in 18 days. In how many
Pi

उसी काय को 7 मिहलाएं िकतने िदन म 24 िदनों म उसी काय के 2 भाग को


5 days B alone can do the work?
पूरा करगी कर सकता है । उ ोंने 20 िदनों तक साथ A और B िमलकर 12 िदनों म एक काय
CGL 2019 Tier-II (15-11-2020) काय िकया। C ने शेष काय 8 िदनों म कर सकते ह। A अकेला इसे 18 िदनों म
(a) 10 days पूरा िकया। A, B और C के एक साथ कर सकता है । B िकतने िदनों म अकेले
(b) 12days काय करने से काय िकतने िदनों म पूरा इस काय को कर सकता है ?
(c) 8 days हो जाएगा.? CGL 2019 Tier-II (18-11-2020)
(d) 9 days CGL 2019 Tier-II (16-11-2020) (a) 32 days
(a) 10 days (b) 30 days
Q33. To do a certain work, the ratio (b) 12 days (c) 36 days
of the efficiencies of A and B is 7 : 5 (c) 18 days (d) 24 days
. Working together, they can (d) 15 days
complete the same work in 17 21
Q39. A and B can do a work
days, A alone will complete the 60% Q36. A, B and C can do work together in 18 days. A is three times
of the same work in: separately in 18,36 and 54 days

www.ssccglpinnacle.com support@ssccglpinnacle.com Ph. 09729327755, 09817390373 390


/
Days 43-46 Work and Time

s
sse
la
_c
ob
as efficient as B. In how many days many days can A alone complete the Q45. A and B can do a piece of work
bo
ah
can B alone complete the work ? work ? in 25 days, B alone can do 66 32 % of
sm
ur
yo

A और B एक काय को 18 िदनों म एक A और B 36 िदनों म एक काय को कर the same work in 30 days. In how


e/
t.m

साथ कर सकते ह। A, B से तीन गुना सकते ह, B और C 60 िदनों म उसी


://

many days A alone can do 154 part of


tp
ht

अिधक कुशल है । B अकेले िकतने िदनों काय को कर सकते ह। A और C उसी the same work ?
म काय पूरा कर सकता है ? काय को 45 िदनों म कर सकते ह। A A और B एक काय 25 िदनों म कर
CGL 2019 Tier-II (18-11-2020) अकेले िकतने िदनों म वह काय पूरा कर सकते ह, B अकेले 30 िदनों म 66 32 %
(a) 60 days सकता है ?
काय कर सकता है । िकतने िदनों म A
(b) 54 days CPO 2019 23-11-2020 (Evening Shift)
अकेले उसी काय का 154 भाग पू रा कर
(c) 72 days (a) 90
सकता है ?
(d) 64 days (b) 120
CPO 2019 24-11-2020 (Morning Shift)
(c) 45
(a) 12
SSC CPO 2019 (d) 60
(b) 15
Q40. A is twice as good a workman
(c) 18
as B and together they finish a piece Q43. X can do a work in 3 day, Y
(d) 20

e
of work in 13 days . does three times the same work in 8
In how many days will B alone finish thedays,
workand? Z does five times the same
Q46. A can complete some work in
A, B से दोगुना अ ा काय करने वाला है और साथinम12
work वे 13 िदनों
days. If म एकhave
they कायtoपूरा कर लेते ह। B
35 days and B can complete the
अकेले work together for 6 hours in a day,
same work in 15 days. They worked
िकतने िदनों म काय पूरा करे गा?

l
then in how much time can they
together for 8 days, then B left the
CPO 2019 23-11-2020 (Morning Shift) complete the work?
(a) 18.5
(b) 21
(c) 39
(d) 42
ac X एक काय को 3 िदन म कर सकता है ,
Y, उसी काय का तीन गुना 8 िदन म
करता है , और Z, उसी काय का पां च
गुना 12 िदनों म करता है । यिद उ
ितिदन 6 घंटे काय करना है , तो वे
work. In how many days will A
alone complete 60% of the
remaining work?
A िकसी काय को 35 िदनों म पूरा कर
सकता है और B उसी काय को 15 िदनों
म पूरा कर सकता है । उ ोंने 8 िदनों
Q41. A and B can do a job in 10 िकतने समय म काय पूरा कर सकते ह?
तक एक साथ काय िकया, िफर B ने
days and 5 days respectively. They CPO 2019 23-11-2020 (Evening Shift)
काय छोड़ िदया। िकतने िदनों म A शेष
worked together for two days, after (a) 4 hours 10 minutes
nn
काय का 60% पूरा करे गा?
which B was replaced by C and the (b) 4 hours
CPO 2019 24-11-2020 (Evening Shift)
work was finished in the next three (c) 5 hours
(a) 15
days. How long will C take to finish (d) 5 hours 20 minutes
(b) 10
60% of the job?
(c) 5
A और B मशः 10 िदनों और 15 िदनों Q44. A and B can do a piece of work
(d) 8
म एक काय कर सकते ह। उ ोंने दो in 36 days, B and C can do the same
िदनों तक एक साथ काय िकया, िजसके work in 60 days. A and C can do the
Q47.P and Q together can do a work
बाद B को C से बदल िदया गया और same work in 45 days. In how many
Pi

in 12 days. P alone can do the same


अगले तीन िदनों म काय ख कर िदया days can B alone complete the same
work in 18 days. In How many days
गया। C को 60% काय पूरा करने म work?
can Q alone complete two-third part
िकतना समय लगेगा? A और B एक काय 36 िदनों म कर
of the same work?
CPO 2019 23-11-2020 (Morning Shift) सकते ह, B और C उसी काय को 60
P और Q िमलकर एक काय को 12 िदनों
(a) 30 days िदनों म कर सकते ह। A और C उसी
म कर सकते ह। P अकेले 18 िदनों म
(b) 25 days काय को 45 िदनों म कर सकते ह। B
यह काय कर सकता है । िकतने िदनों म
(c) 18 days अकेले िकतने िदनों म काय पूरा कर
Q अकेले इसी काय का दो-ितहाई िह ा
(d) 24 days सकता है ?
पूरा कर सकता है ?
CPO 2019 24-11-2020 (Morning Shift)
CPO 2019 24-11-2020 (Evening Shift)
Q42. A and B can do a piece of (a) 90
(a) 21
work in 36 days, B and C can do the (b) 45
(b) 36
same work in 60 days. A and C can (c) 120
(c) 24
do the same work in 45 days. In how (d) 60
(d) 30

www.ssccglpinnacle.com support@ssccglpinnacle.com Ph. 09729327755, 09817390373 391


/
Days 43-46 Work and Time

s
se
s
la
_c
ob
CPO 2019 25-11-2020 (Evening Shift)
bo
ah
Q48. A is twice as good as a (a) 10
sm
ur
yo

workman as B, and together they (b) 7


e/
t.m
://

finish a piece of work in 13 days. In (c) 8


tp
ht

how many days will A alone finish (d) 15


the work?
A, B से दोगुना अ ा काय करता है , Q51. P and Q together can do a work
एक साथ वे 13 िदनों म एक काय ख in 12 days, P alone can do the same
कर दे ते ह। A अकेले िकतने िदनों म work in 36 days, In how many days
काय पूरा करे गा ? can Q alone complete two-third part
CPO 2019 25-11-2020 (Morning of the same work?
Shift) P और Q िमलकर एक काय को 12 िदनों
(a) 9 41 म कर सकते ह, P अकेले उसी काय को
(b) 39 36 िदनों म कर सकता ह, Q अकेले उसी
(c) 19 21 काय का दो-ितहाई िह ा िकतने िदनों म
कर सकता ह ?

e
(d) 41
CPO 2019 25-11-2020 (Evening Shift)
Q49. A and B can do a job in 10 (a) 12
days and 5 days, respectively. They (b) 18

l
worked together for two days, after (c) 15
which B was replaced by C and the (d) 21
ac
work was finished in the next three
days. How long will C alone take to
finish 40% of the job?
A और B मशः 10 िदनों और 5 िदनों
म एक काय कर सकते ह। उ ोंने दो
िदनों तक एक साथ काय िकया, िजसके
बाद B को C से बदल िदया गया और
अगले तीन िदनों म काय समा हो
nn
गया। C को अकेले 40% काय पूरा करने
म िकतना समय लगेगा ?
CPO 2019 25-11-2020 (Morning Shift)
(a) 18 days
(b) 10 days
(c) 15 days
(d) 12 days
Pi

Q50. A can complete a certain work


in 35 days and B can complete the
same work in 15 days. They worked
together for 7 days, then B left the
work. In how many days will A
alone complete 60% of the
remaining work?
A िकसी काय को 35 िदनों म पूरा कर
सकता है और B उसी काय को 15 िदनों
म पूरा कर सकता है । उ ोंने 7 िदनों
तक एक साथ काय िकया, िफर B ने
काय छोड़ िदया। िकतने िदनों म A शेष
काय का 60% पूरा करे गा?

www.ssccglpinnacle.com support@ssccglpinnacle.com Ph. 09729327755, 09817390373 392


/
Days 43-46 Work and Time

s
se
s
la
_c
ob
SOLUTION Efficiency 2 : 1 Efficiency of C = 3 unit
bo
ah Balancing the ratio for man Word done by A in first 3 days = 3
sm
ur
yo

Variety Solutions Man : Woman : Child × 2 = 6 days


e/
t.m
://

Efficiency 2 : 6 : 1 Work done by A and B in next 2


tp
ht

Sol 1.(c) Let the efficiency of A, B Total work = 6{3(2)+4(6)+6(1)} days = 2(2+1) = 6 days
and C are 2 unit , 3 unit and 5 unit = 216 unit Work remaining = 84-6-6 = 72 unit
respectively. Let the required number of woman = Total time taken by all three to finish
Total work = 6(2+3+5) = 60 unit w 72 = 12 days
this work = 2+1+3
20% of work = 60 × 10020 = 12 unit 216 = 4 × w × 6 A worked = 12+2+3 = 17 days
Time taken by A = 12 = 6 days ⇒w=9
2

Sol 2. (b) Let the efficiency of A, B Sol 6. (d)


and C are 2 unit , 5 unit and 3 unit Per day wages of A = 180 × 7 = Sol 9. (b)
respectively. 1260
Total work = 27 × (2 + 5 + 3) = 270 Per day wages of B = 160 × 5 = 800

e
unit Required Ratio = 1260 : 800
Time taken by B and C to complete = 63 : 40
4 270× 49
9 th part of that work = 5+3 = 15
Sol 7. (a)
days

l
33 31 % = 1
3
ac 1 unit work is done by B in 15 days Total work = 96 unit
Sol 3. (c)
3 unit work is done by B in 3 × 15 = 2(A+B+C) = 13 unit
Let the efficiency of A = 1 unit
Total work = 48 × 1 = 48 unit 45 days A+B+C = 6.5 unit
From 11 31 days, A worked Efficiency of C = 6.5-6 = 0.5 unit
Work done in 12 days = 12 × 6.5 =
for 5 31 days and B worked for 6 days
78 unit
…...( B started the
Work remaining = 96-78 = 18 unit
work) 18 = 36
Required number of days = 0.5
Work done by A in 5 31 days = 16 ×1
nn
3
= 16 unit
3 Sol 10. (a)
Work done by B = 48 - 16 = 128
3 3 Total work = 225 unit Total work = 18 × 9 = 162 unit
unit Efficiency of A+B = 9 unit Work done = 18 × 5 = 90 unit
Efficiency of B = 128 = 64 unit
3×6 9 Efficiency of B = 5 unit Remaining work = 162-90 = 72 unit
Time taken by B to finish 4 times of Efficiency of A = 9-5 = 4 unit 72 = 3
Required number of days = 18+6
4
total work = 4×48
64/9 = 27 days 15 th of the task will be completed
by A = 225×4 = 15 unit Sol 11. (b)
Pi

4×15
Sol 4. (d) Man = 2 × Woman
40% = 52 ⇒ A : B = 7 : 5 Sol 8. (c) an = 2
⇒ WMoman 1
20% = 1 ⇒B:C=5:4
5 Total Work = 2 × {3(2)+2(1)} = 16
⇒ A:B:C = 7 : 5 : 4 unit
Total work = 5 (7+5+4) = 80 unit No of days required for a woman =
16 = 16 days
Time taken by A to complete the 1
80×70 = 8 days
70% of work = 7×100
Sol 12.(c)
Sol 5. (a) A : B
According to the question Time 15 : 10
Man : Woman Let total work = 84 unit Efficiency 4:6
Efficiency 1 : 3 Efficiency of A = 2 unit A’s share in the money = 35000 ×
Efficiency of B = 1 unit 4
Man : Child 4+6 = 14000

www.ssccglpinnacle.com support@ssccglpinnacle.com Ph. 09729327755, 09817390373 393


/
Days 43-46 Work and Time

s
sse
la
_c
ob
Work done by All three in 3 days = Let money earned by 6 men or 5
bo
ah
Sol 13. (c) 3(8+6+4) = 54 unit women in 2 days = 30 unit
sm
ur
yo

According to the question Remaining work = 72-54 = 18 unit Money earned by one man in 1 days
e/
t.m
://

Helper : Labourer 18 = 9
Required number of days = 6+4 30 = 2.5 unit
= 6×2
tp

5
ht

Wages 2 : 1 Money earned by one woman in 1


Craftsman : Labourer Alternate : 30 =3 unit
day = 5×2
Wages 3 : 1 Work done by A in 3 days = 8 × 3 = 4 women and 6 men earn in one day
⇒ 24 unit = 6(2.5)+4(3) = 27 unit
Helper : Craftsman : Labourer Work remaining = 72-24 = 48 unit Now,
Wages 2 : 3 : 1 Time taken by B and C to finish this 30 unit = 14820
48 =4 4
work = 6+4 5 1 unit = 494
Total amount received = Number of days taken by B and C 27 unit = 27 × 494 = 13338
3(3)+5(2)+6(1) = 25 unit after exclusion of A = 4 54 -3 = 1 54 or
Now, 9 Sol 19. (d)
5 days
25 unit = 10000

e
1 unit = 400
Sol 16. (a)
Amount received by 3 craftsman (9
unit) = 9 × 400 = 3600
Sol 14. (b)

l
Number of days taken by B to
complete 20%( 51 ) of Work = 6
ac
Number of days taken by B to
complete the work = 6 × 5 = 30 days

Work done by A+B in 10 days =


Total work = 120 unit 10(5+4) = 90 unit
2(A+B+C) = 4+5+3 Work done by C = 100-90 = 10 unit
A+B+C = 6 unit 10 = 70
Share of C = 700 × 100
nn
Required number of days = 120 = 20
6
Sol 20. (c)
Sol 17. (d) Total work = 42 × (7+5+8) = 840
unit
Work done in 21 days by C and B =
21 × (5+8) = 273 unit
Required number of days =
60 × 50 Remaining work = 840-273 = 567
(3+2) × 100 = 6 days
Time required by A to finish this
Pi

work = 5677 = 81 days


Sol 15. (d)
Total number of days required to
finish the work = 81+21 = 102
Alternate :
C and B worked 21 less number of
days. So extra to be done by A =
Total work = 36 unit
(5+8) × 21 = 273 unit
Work completed in 5 days = 5 ×
Extra days required by A to finish
(3+2) = 25 unit
this work = 273
7 = 39 days
Remaining work = 3636−25 = 11
36
Total number of days taken by A =
42+39 = 81
Sol 18. (a)
Total number of days required to
finish the work = 81+21 = 102
Let total work = 72 unit

www.ssccglpinnacle.com support@ssccglpinnacle.com Ph. 09729327755, 09817390373 394


/
Days 43-46 Work and Time

s
sse
la
_c
ob
Efficiency of A = 4 unit Time taken by B to complete the
bo
ah
Sol 21. (b) Efficiency of B = 3 unit work = 15 x 35 = 25 days
sm
ur
yo

40% ( 52 ) work is done in 6 days. Efficiency of C = 8-4-3 = 1 unti


e/
t.m
://

⇒ 100% of the work will be done by Since, all worked for the same time
tp
ht

A in 6×5 = 15 days period, Share will be distributed


2
30% 3 ) of work
( 10 is done by B in 3 according to the efficiency of worker
According to the question
days
8 unit = 1848
⇒ 100% of the work will be done by
1 unit = 231
B in 3×10
3 = 10 days

Sol 3. (d)
Let the number of Person = 100 and
H be the desired number of hours. Let the total work = 150 unit
⇒ Remaining persons = 60 Work done in 10 days = 10 x (5+6) =
According to the question 110 unit

e
100 x 34 x 9 = 60 x 51 x H Work left = 150-110 = 40 unit
⇒ H = 10 hours Time taken by C = 4 days
Efficiency of C = 40
4 = 10 unit
Sol 4. (d) Time taken to finish 28% of the

l
Let the efficiency of a man = m and 28
100 ×150
work = = 2 days
Let total work = 30 unit
ac efficiency of the woman = w and W 5+6+10

Work done by B in 2 days = 2 × 3 = be the desired number of woman.


6 unit {4(m)+5(w)} x 15 ={9(m)+6(w)} x Sol 7. (a)
Work done by A = 30-6 = 24 unit 10 A : B+C
Number of days taken by A = ⇒ 60m + 75w = 90m + 60w Efficiency 1: 1
Number of days required for the ⇒ wm = 21 A+B : C
completion of work = 24 Time 36 : 60
2 = 12 Total work = {4(1)+5(2)} x 15 or
Efficiency 5 :3
{9(1)+6(2)} x 10 = 210 unit
__________
nn
SSC CGL TIER II According to question
Work 180 : 180
210 = 7 x {4(1)+W(2)
Balancing the ratio for total
Sol 1. (b) ⇒ 30 = 4 + 2W
efficiency
A:B ⇒ W = 13
A : B+C : A+B : C : Total
Efficiency 3:7
4: 4 : 5 : 3 : 8
Total work = 21
2 x (3+7) = 105 unit Sol 5. (a)
Efficiency of B =8-4-3 = 1 unit
Work done in 8 days = 8 x (3+7) = X:Y
Work done by A and C in 10 days =
80 unit Efficiency 5:4
10 x (4+3) = 70 unit
Pi

Work left = 105-80 = 25 unit Total work = 10 x (5+4) = 90 unit


Work left = 180-70 = 110 unit
Time taken by A of finish 60 % of Work done by Y = 5 x 4 = 20 unit
60
Time taken by B to finish this work
remaining work = 100 ×25
= 5 days Work left = 90-20 = 70 unit
3 = 110
1 = 110 days
Time taken by X to finish this work
= 70
5 = 14 days
Sol 2. (a) Sol 8. (c)
Let the total work = 24 unit Time taken by A to finish 1/3rd of
Sol 6. (d)
work = 15 days
40% ( 52 of work) is done by A = 12
Time taken by A to finish the
days complete work = 15 x 3 = 45 days
Time taken by A to complete the Time taken by B to finish
work = 12 x 25 =30 days 75%(3/4th) of work = 18 days
60% ( 53 of work) is done by B = 15 Time taken by B to finish the
days complete work = 18 x 34 = 24 days

www.ssccglpinnacle.com support@ssccglpinnacle.com Ph. 09729327755, 09817390373 395


/
Days 43-46 Work and Time

s
sse
la
_c
ob
Sol 3. (a)
bo
ah Total work = 30 × (5+3+8) = 480 Sol 6. (b)
sm
ur
yo

unit Total work = 21 × (7+3+5) = 315


e/
t.m
://

Work done in 20 days by A and B = unit


tp
ht

20 × (5+3) = 160 unit Work done in 15 days by A and C =


Remaining work = 480-160 = 320 15 × (7+5) = 180 unit
Time required by C to finish this Remaining work = 315-180 = 135
work = 320 8 = 40 days Time required by B to finish this
Work done by B and C in 8 days = 8 work = 135
Alternate : 3 = 45 days
x (15+10) = 200 units
A and B worked 10 fewer days. So Alternate :
Remaining work = 360-200 = 160
extra to be done by C = (5+3) × 10 = A and C worked 6 fewer days. So
unit
80 unit extra to be done by B = (7+5) × 6 =
Time taken by A to finish this work
Extra days required by C to finish 72 unit
= 160
8 = 20 days this work = 80
8 = 10 days Extra days required by B to finish
Total number of days taken by C = this extra work = 72
3 = 24 days

e
Sol 9. (*)
30+10 = 40 Total number of days taken by C =
Total work = 25 x 60 = 1500 unit
21+24 = 45 days
According to the question
Sol 4. (b)
1500 = 25x + 15(80-x)
Total work = 35 × (7+5+6) = 630 Sol 7. (a)

l
1500 = 25x + 1200 - 15x 2 1
unit ⇒ 40% = 5 and 20% = 5
x = 30
ac Work done in 21 days by C and B = A:B
21 × (5+6) = 231 unit Efficiency 7:5
Note : None of the given options is
Remaining work = 630-231 = 399 B:C
correct.
Time required by A to finish this Efficiency 5:4
work = 3997 = 57 days A:B:C=7:5:4
Practice Solutions
Alternate : Total work = 16 × 15 = 240 unit
Sol 1. (d) C and B worked 14 less number of Number of days taken by B to finish
Efficiency of A : B : C = 4 : 5 : 3 days. So extra to be done by A = 75% of work = 240×75
100×5 = 36
nn
Total work = (4+5+3)x25 = 12x25 = (5+6) × 14 = 154 unit
300 Extra days required by A to finish Sol 8. (c)
300×35 = 15 days
Required time = (4+3)×100 this work = 154
7 = 22 days ⇒ 40% = 2
5 and 20% = 1
5
Total number of days taken by A = A:B
Sol 2. (a) 35+22 = 57 Efficiency 7:5
Total work = 35 × (7+5+4)unit = B:C
Sol 5. (c) Efficiency 5:4
560
Total work = 5 × (3+5+1) = 45 unit A:B:C=7:5:4
Pi

Work done in 28 days by A and B =


28 × (7+5) = 336 unit Work done in 3 days by A and B = 3 Total work = 16 × 20 = 320 unit
× (3+5) = 24 unit Number of days taken by A to finish
Remaining work = 560-336 = 224
Time required by C to finish this Remaining work = 45-24 = 21 35% of work = 320×35
100×7 = 16

work = 224 Time required by C to finish this


4 = 56 days
work = 21 1 = 21 days Sol 9. (d)
Alternate :
Alternate : ⇒ 50% = 1 and 40% = 2
A and B worked 7 fewer days. So 2 5

extra to be done by C = (7+5) × 7 = A and B worked 2 less days. So A:B


extra to be done by C = (3+5) × 2 = Efficiency 3:2
84 unit
Extra days required by C to finish 16 unit B:C
this work = 84 Extra days required by C to finish Efficiency 5:3
4 = 21 days
this extra work = 16
1 = 16 days Balancing the ratio for B
Total number of days taken by C =
Total number of days taken by C = A : B : C = 15 : 10 : 6
35+21 = 56
16+5 = 21 days

www.ssccglpinnacle.com support@ssccglpinnacle.com Ph. 09729327755, 09817390373 396


/
Days 43-46 Work and Time

s
sse
la
_c
ob
Total work = (15+10+6) × 10 = 310
bo
ah
sm
unit
ur
yo

Number of days taken by A to finish


e/
t.m
://

150% of work = 310×150


100×15 = 31
tp
ht

Sol 10. (a)


⇒ 50% = 1 and 40% = 2
2 5
A:B Total Work = 60 unit
Efficiency 3:2 2 × efficiency of (A+B+C) = 6+4+3
Total work = 60 unit
B:C efficiency of (A+B+C) = 6.5 60
Required number of days =
Efficiency 5:3 Efficiency of C = 6.5-6 = 0.5 12+3+1

Balancing the ratio for B Work done in 8 days = 8 × (A+B+C) =3 43


A : B : C = 15 : 10 : 6 = 8 × 6.5 = 52
Total work = (15+10+6) × 20 = 620 unit Sol 19. (d)
unit Remaining work = 60-52 = 8 unit

e
Number of days taken by A to finish Number of days taken by C = 0.5 8 =

30% of work = 620×30


100×6 = 31 16

Sol 11. (d) Sol 16.(b)

l
Total work = 9 × (2 + 5 + 3) = 90 unit

Sol 12. (d)


ac
Number of days taken by C to finish
90×40 = 12
40% of work = 100×3

Total work = 12 × (2 + 5 + 3) = 120


Total work = 60 unit
Required number of days =
3
60
15+3+1
unit =3 19
Number of days taken by A to finish
30% of work = 120×30
100×2 = 18 Sol 20. (b)
nn
Total work = 60 unit
Required number of days = 60 =
6+3+1
Sol 13. (a)
6
Total work = 21 × (5 + 3 + 2) = 210
unit
Sol 17.(d)
Number of days taken by B to finish
40% of work = 210×40
100×3 = 28

Sol 14. (b)


Pi

Total work = Number of men ×


Number of days × Number of hours Total work = 60 unit
Let H be required the number of 60× 13
Required number of days = 15+3+1
hours 1
=1 19
⇒ 30 × 10 × 8 = 40 × 6 × H
⇒ H = 10 ans
Total work = 56 unit Sol 21. (d)
Required number of days = 56 A : B
14+2+1
=3 5 Time 15 : 10
Sol 15. (c) 17
Efficiency 4:6
Sol 18. (b) A’s share in the money = 35000 ×
6
4+6 = 21000

Sol 22. (c)

www.ssccglpinnacle.com support@ssccglpinnacle.com Ph. 09729327755, 09817390373 397


/
Days 43-46 Work and Time

s
sse
la
_c
ob
A : B According to the question
bo
ah
Time 15 : 10 Sol 29. (b) 24×8×10 = M ×6×16
sm
2 4
ur
yo

Efficiency 2 : 3 Total work = Number of men × Therefore, M=40


e/
t.m
://

A’s share in the money = Number of days × Number of hours


tp
ht

75000 × 2+33 = 45000


Let M be the required number of Sol 36. (a)
men Let M be the required number of
Sol 23. (c) ⇒ 40 × 8 × 12 = M × 16 × 4 men
Required Ratio = 640 : 360 ⇒ M = 60 ans According to the question
= 16 : 9 18×8×10 = M ×6×16
3 5
Sol 24. (d) Sol 30. (c) ⇒ M = 25
A : B Let M be the required number of
Time 15 : 10 men Sol 37. (d)
Efficiency 4:6 According to the question Total work = 10 × (2+5+7) = 140
A’s share in the money = 75000 × 36×8×12 = M ×16×6
3 5 unit
4
4+6 = 30000 Therefore, M = 60 men Number of days taken by A to finish

e
30% of work =
Sol 25.(d) Sol 31. (c) A : B : C = 5 : 6 : 9 10×(2+5+7)×30
2×100 = 21 days
Total salary of A = 100 × 8 = 800 Total Work = 18x(5+6+9) = 18x20 =
Total salary of B = 120 × 6 = 720 360 Sol 38. (b)

l
Required ratio = 720 : 800 Number of days taken by B to finish
25
360× 100
ac= 9 : 10 25% of work = = 15
6
Sol 26. (c)
A:B Sol 32. (c)
Days 15 : 20 Let D be the required number of
Efficiency 4 : 3 days
A’s share = 77000 × 4+34 = 44000
According to the question
16×6×10 = 24×8×D
100 80
Sol 27.(a) Therefore, D = 4 days
nn
Sol 33. (b) Total work =
120x(5+6+8) = 120x19 = 2280 unit
Number of days taken by B to finish Required number of days = 36 =7
3+2
40
2280× 100 1
40% of work = 6 = 152 5

Sol 33. (b) Let total capacity of tank


Sol 39. (b)
Pi

be 240units
Efficiency of A=+15, B=+10 and Woman : Man
C=-6 Efficiency 2 : 1
Total work = 360 Therefore, Time to fill remaining Man : Child
Required number of days = 360 tank = 240−4152 = 88
4 = 22 hrs
Efficiency 2 : 1
18+15+10
16
=8 43 Balancing the ratio for man
Sol 34. (a) Woman : Man : Child
c Efficiency 4 : 2 : 1
Sol 28. (d)
Total work = Number of men × Therefore, time taken by A alone =
180 = 45days Total work = [3(2) + 4(4) + 6(1)]7 =
Number of days × Number of hours 4
196
Let H be required the number of 196 =
Required number of women = 4×7
hours Sol 35. (c)
⇒ 16 × 27 × 12 = 18 × 24 × H Let M be the required number of 7
⇒ H = 12 ans men

www.ssccglpinnacle.com support@ssccglpinnacle.com Ph. 09729327755, 09817390373 398


/
Days 43-46 Work and Time

s
se
s
la
_c
ob
Sol 40. (b) Balancing the ratio for B Sol 47. (c)
bo
ah Required ratio = 40 × 12 : 60 × 10
sm
ur
yo

Total work = Number of men × A : B : C =4:5


e/
t.m
://

Number of days × Number of hours 6 : 2 : 1


tp
ht

= 3 × 12 × 6 Sol 48. (a)


= 216 unit Total work = 2x15 = 30 Total work = 60 unit
Work done in 3 days = 3 × 3 × 6 Time taken by A & C together =
30 = 4 2 days
= 54 unit 7 7
Remaining work = 216-54 = 162
unit Sol 45. (a)
Required number of days = 2×6 162 =

13 21
Alternate :
Let the required number of days = D
According to the question

e
3×9×6 = 2× D × 6 Work done by A and B in 5 days =
D = 13 21 (6+4) x 5 = 50 unit
Work left = 60 - 50 = 10 unit
Sol 41. (d) Time taken by B and C to finish this

l
10 = 2 days
= 4+1
ac Let Total work = 22 unit
Total efficiency of A and B = 6
Total efficiency of C and D = 7
Required time =
22× 12
6+7
11
= 13
Total time taken = 5+2 = 7 days

Sol 49. (a)

Sol 46. (a)


Work done by A in 20 days = 20 × 8 Number of days taken by A to
nn
complete 1/5th of work = 4 days
= 160 unit
Number of days taken by A to
Work left = 240-160 = 80 unit
complete the work = 4 × 5 = 20 days
Time taken by B = 80
5 = 16
Number of days taken by B to
complete 1/6th of work = 5 days
Sol 42 (c)
Number of days taken by B to
Let the required number of Men = M
complete the work = 5 × 6 = 30 days Work done in one cycle (2 days) =
According to the question
15 × 14 =M × 30 4+3 = 7 unit
Pi

M=7 Work done in 17 cycles (34 days) =


17 x 7 = 119 unit
Sol 43. (a) Work to be done in 35th day = 41 =
Let D be the required number of 0.25 days
days Total time taken = 34 + 0.25 = 34.25
According to the question days
21x8=15xD
D = 11.2 Sol 50. (b)
Let the total work = 300 unit
Sol 44. (c)
A:B
Efficiency 3:1 Let total work = 60 unit
Required number of days = 60 = 12
B:C 3+2

Efficiency 2:1

www.ssccglpinnacle.com support@ssccglpinnacle.com Ph. 09729327755, 09817390373 399


/
Days 43-46 Work and Time

s
sse
la
_c
ob
Desired difference = 1615 x 10−2 =
bo
10+5+2
ah
sm
760
ur
yo
e/
t.m
://

Sol 53. (c)


tp
ht

Total work = 40 unit

Efficiency of Z = 10-6-3 = 1 unit


15+12+20
⇒ efficiency of (A+B+C) = 2 Number of days taken by Z = 90 1 =
= 23.5 90 days
⇒ efficiency of A = 23.5-12 = 11.5
Desired number of days = 300 = 600 Sol 57. (c)
11.5 23
Let the total work = 60 unit

e
Sol 51. (d)
⇒ Efficiency of B+C = 7-4 = 3
Let the total work = 168 unit 40
Desired number of days = 3

Sol 54. (b)

l
Total work = Number of men ×
ac Number of days × Number of hours
Let D be the required number of
days
⇒ 10 × 30 × 8 = 12 × 4 × D
⇒ D = 50 ans

Sol 55. (a) Work done in 5 days = (2+1) x 5 =


Work done in 3 days = (21+8) x 3 = Total work = 72 unit 15 unit
nn
87 unit Work left = 60-15 = 45 unit
87 = 29
⇒ Work done in fraction = 168 45 = 3
56 Desired fraction = 60 4

Sol 52. (a) Sol 58. (b)


Total work = 150 unit Let the efficiency of a technician in
studio A = a
And the efficiency of a technician in
studio B = b
Pi

According to the question


5 x a x 30 = 8 x b x 15
Ratio of the efficiency of A, B and C ⇒ a:b = 4 : 5
=6:3:2 Total work = 5 x 4 x 30 = 8 x 5 x 15
Since, they worked for the same = 600 unit
600 = 200
Desired number of days = 4+5
period of time their share will be 3
distributed in the ratio of their
efficiency. Sol 59. (d)
Ratio of the efficiency of A, B and C
3−2 =
Desired difference = 3850 x 6+3+2 Total work = 300 unit
= 10 : 5 : 2
Since, they worked for the same 350
period of time their share will be
distributed in the ratio of their Sol 56. (b)
efficiency. Let the number of days = 90 unit

www.ssccglpinnacle.com support@ssccglpinnacle.com Ph. 09729327755, 09817390373 400


/
Days 43-46 Work and Time

s
sse
la
_c
ob
Work done in one cycle (2 days ) =
bo
ah 4+1 = 5 unit
sm
ur
yo

⇒ 40 unit will be done in = 40


5 = 8
e/
t.m
://

cycles
tp
ht

So, required number of days = 8 x 2


= 16

Sol 64. (b)


Let the total work = 12 unit

Efficiency of A = 15 unit
Efficiency of A+B = 20 unit 26 26
35 of the work = 1050 x 35 = 780
⇒ efficiency of B = 20-15 = 5 unit
Required number of days = 780 = 60
Efficiency of B+C = 8 unit 6+7

⇒ efficiency of C = 8-5 = 3 unit days

e
Desired ratio = 15 : 5 : 3
Sol 62. (d)
Sol 60. (a) Total work = 30 unit
Mohit :

l
Rohit
Efficiency ac 5 : 1
Time 1 : 5 Efficiency of D = 3-1 = 2 unit
According to the question Desired number of days = 12 = 6
2
(5-1) unit = 28 days
1 unit = 7
Number of days taken by Rohit = 5 x Sol 65. (c)
7 = 35 Let D be the required number of
days.
nn
Sol 61. (c) According to the question
Number of days taken by Ravika to ⇒ 10 40 10
× 20 = 8 × D
complete three-fifth of the work = Work done in 5 days = (2+3) x 5 =
⇒ D = 6 41 days ans
105 days 25 unit
Number of days taken by Ravika to Work left = 30-25 = 5 unit
Desired number of days = 35 = 1 32 Sol 66. (c)
complete the work = 105 x 35 =175
Piyush : Arun
days days
Time 8 : 14
Number of days taken by Mallika to Efficiency 14 : 8
Sol 63. (b)
Pi

complete one-third of the work = 50 According to the question


days Total work = 40 unit
(14+8) unit = 44000
Number of days taken by Mallika to 1 unit = 2000
complete the work = 50 x 3=150 Share of piyush = 14 unit = 28000
days
Sol 67. (b)
Let the total work = 60 unit

www.ssccglpinnacle.com support@ssccglpinnacle.com Ph. 09729327755, 09817390373 401


/
Days 43-46 Work and Time

s
sse
la
_c
ob
Let the efficiency of A = a and B = b
bo
ah According to the question
sm
ur
yo

(a+b) x 12 = (a+b) x 9 + 5a
e/
t.m
://

⇒ ba = 23
tp
ht

Total work = (3+2) x 12 = 60 unit


Desired number of days = 60 2 = 30
days
Alternate :
B Worked three days less so A has to
Efficiency of N, K and G = 3+10+9 = work 2 days more
2
⇒bx3=ax2
11 unit
Efficiency of G = 5-2 = 3 unit ⇒ a:b = 3:2
⇒ Efficiency of K = 11-9 = 2 unit
Desired number of days = 60 Total work = (3+2) x 12 = 60 unit
3 = 20 Desired number of days = 720 = 360
2 ⇒ Desired number of days = 60 2 =
days
Sol 68. (c) 30 days

e
1
3 part of a work is completed by Sol 71. (a)
Sol 75. (c)
Vishal = 60 days Total work is done = 50 days
10 = 1 Let M be the total number of men
Total work is completed by Vishal = Work done in 10 days = 50 5
According to the question

l
60 x 3 = 180 days 70×25 = M ×35
1
4 part of a work is completed by
1 3
ac Sol 72. (d)
Therefore, M = 150 men
Ashok = 30 days 50% of work is done by A = 16 days
Additional number of men required
Total work is completed by Ashok = Total work is done by A = 16 x 2 =
= 150 - 70 = 80 men
30 x 4 = 120 days 32 days
1/4th of work is done by B = 24 days
Sol 76. (d)
Total work is done by B = 24 x 4 =
Let the number of days taken by x =
96 days
d
⇒ the number of days taken by y =
nn
d+10
According to the question
d × (d+10)
d + (d+10) = 12
⇒ 2 d + 10d = 24d+ 120
⇒ d2 -14d - 120 = 0
⇒ d2 -20d + 6d - 120 = 0
⇒ d(d-20) + 6(d-20) = 0
Desired number of days = 360 = 72
Pi

2+3 ⇒ d = 20
days Number of days taken by y = 20+10
96× 74 = 30
Sol 69. (b) Desired number of days = 3+1 = 42
Bags made by Anil in one day = 20
4 days Sol 77. (d)
= 5 bags A:B
Bags made by Manoj in one day = Sol 73. (a) Efficiency 2 :1
10 = 2 bags
5
Let the number of person = n and Total work = (2+1) x 24 = 72 unit
Total bags made in one day = 5+2 = efficiency = k Number of days taken by A = 72 2 =
7 bags Total work = n × k × 84
36
Desired number of days =
1
× (n × k × 84)
Sol 70. (c) 2
2n × k = 21 Sol 78.(c)
Total work = 720 unit 25% = 41
Sol 74. (c) A:B

www.ssccglpinnacle.com support@ssccglpinnacle.com Ph. 09729327755, 09817390373 402


/
Days 43-46 Work and Time

s
sse
la
_c
ob
Efficiency 4 :5
bo
ah
Total work = 4 x 15 = 60 unit
sm
ur
yo

Number of days taken by A+B =


e/
t.m
://

60 20
4+5 = 3
tp
ht

Sol 79. (b)


Total work = 35 x 30 = 1050 unit
Work done in first 10 days = 35 x 10
= 350 unit
Work done in next 10 days = 30 x 10
Work done in one cycle (3 days ) = 9
= 300 unit
+ (9+6) + (9+4) = 37 unit
Work done in next 10 days = 25 x 10
Work done in 97 cycles (291 days )
= 250 unit Work remaining = 240 x 1 = 120
2 = 37 x 97 = 3589 unit
Work left = 1050 - 350-300-250 =
unit Work done on day 292 = 3589+9 =
150 unit 120
Desired number of days = =8 3598 unit

e
15
Time taken to finish this work = 150
20
Work left = 3600-3598 = 2 unit
= 7.5 days 2 days
Sol 83. (a) Time taken to finish it = 15
Total time taken = 10 + 10 + 10 +
A:B ⇒ The work will be finished on 293
7.5 = 37.5 days
Efficiency 3:1

l
day.
B:C
Sol 80. (a) ac Efficiency 2:1 Sol 86. (a)
Let M be the total number of men
Balancing the ratio for B A :B
According to the question
105×25 = M ×35
A:B:C Efficiency 1.5 : 1
1 2 Efficiency 6:2:1 Total work = (1.5+1) x 12 = 30
Therefore, M = 150 men Total work = 120 x 1 = 120 unit 30
Number of days taken by A = 1.5 =
Additional number of men required Required number of days = 120 3 = 20
= 150 - 105 = 45 men
13.33
Sol 81. (d)
Sol 87. (a)
nn
Let the total work = 180 unit
Sol 84. (b) Let the total work = 24 unit
Let the efficiency of X = x and Y =
y
According to the question
36 × x = 18 × y
x:y = 1:2
Work left by x = 12 x 1 = 12 unit
Time required by y to finish it = 12
Pi

2
= 6 days
5+3+4
Desired answer = 12-6 = 6 days
⇒ Efficiency of A+B+C = =6
2 Note : Efficiency of x is half of y so
unit number of days taken by x will be
So, Efficiency of B = 6-4 = 2 unit double.
Desired number of days = 180 2 = 90 Work done in 3 days = (3+2) x 3 =
days Sol 85. (d) 15
Let the total work = 3600 unit Remaining work = 24-15 = 9 unit
Sol 82. (d) Desired number of days = 39 = 3
Let the total work = 240 unit
days

Sol 88. (a)


Let the total work = 30 unit

www.ssccglpinnacle.com support@ssccglpinnacle.com Ph. 09729327755, 09817390373 403


/
Days 43-46 Work and Time

s
sse
la
_c
ob
bo
ah
sm
ur
yo
e/
t.m
://
tp
ht

Work done in 2 days = (3+2) x 2 =


10 unit
Desired number of days = 30 = 10 Work left = 30-10 = 20 unit
2+1 Efficiency of C = 5-2 = 3 unit
days Desired number of days = 80 = Time taken by A to finish this work
3
= 20
3
26.67
Sol 89. (a) Total time taken to finish this work =

e
20 +2 = 26
Let the total work = 360 unit Sol 92. (a) 3 3

A:B Alternate :
Efficiency 1:3 Since A worked for the complete
period total number of days taken to

l
Total work = 96 x 1 = 96 unit
Work done by B in 24 days = 24 x 1 finish the work = number of days
ac = 24 unit
Work left = 96-24 = 72 unit
Required number of days = 1+3

Sol 93. (b)


72 = 18
taken by A .
Work done by B in 2 days = 2 x 2 =
4 unit
Work done by A = 30-4 = 26 unit
Number of days taken by A = 263

Let the total work = 30 unit


Work done in 20 days = (9+3) x 20 = Sol 95. (c)
240 unit Let the total work = 360 unit
nn
Work left = 360-240 = 120 unit
Desired number of days = 12010 = 12
days

Sol 90. (c)


Let the efficiency of A = a and
efficiency of B = b
(a+b) x 21 = 42 x a
Pi

Since, Both worked for same


⇒ a:b = 1:1
number of days the amount will be
Total work = (1+1) x 21 = 42 x 1 = Desired number of days = 360 = 360
distributed in the ratio of their 15+8 23
42 unit
efficiency.
Number of days taken by B = 42 1 = ⇒ (3+2) unit = 2000 Sol 96. (c)
42 days 1 unit = 400 Let the total work = 45 unit
Note : Efficiency of A and B is Share of B (2 unit) = 2 x 400 = 800
same so number of days taken will
also be same. Sol 94. (c)
Let the total work = 30 unit
Sol 91. (a)
Let the total work = 80 unit

www.ssccglpinnacle.com support@ssccglpinnacle.com Ph. 09729327755, 09817390373 404


/
Days 43-46 Work and Time

s
sse
la
_c
ob
Had B worked 3 days more extra
bo
ah work done = 3 x 2 = 6 unit
sm
ur
yo

Total work = 54+6 = 60 unit


e/
t.m
://

Number of days taken to finish this


tp
ht

60 = 20 days
work = 2+1
Total days taken to finish this work =
20 + 9 = 29 days

Sol 99. (a)


Work done in 6 days = (3+2) x 6 = Let M be the required number of
30 unit persons
⇒ Work left = 45-30 = 15 unit According to the question 60
Desired number of days = 5+4+3 =5
Efficiency of C = 15
6 = 2.5 6 x 14 = 4 x M days
Desired number of days = 45 =6 ⇒ M = 21
3+2+2.5

Sol 103. (b)

e
Sol 97. (c) Sol 100. (b) Total work = 180 unit
100% work is done by A = 30 days Let the total work = 60 unit
60% work is done by A = 18 days
According to the question

l
18 x A = 21 x B
⇒ A:B = 7:6 ac
Total work = 7 x 30 = 210 unit
Work to be finished by A and B
65
together = 210 x 100
65
210 × 100
Desired number of days = 7+6
Work done by both in one day = 3+
=10 21 days 2 Work done in 6 days = 6 x (9+8) =
2 = 4 unit
Desired number of days = 60 = 15 102 unit
4
Sol 98. (a)
nn
days Work left = 180-102 = 78 unit
Let the total work = 108 unit 78 = 3 unit
Efficiency of C = 26
Desired number of days = 180 =9
Sol 101. (a) 9+8+3
days

Sol 104. (b)


Let the efficiency of A = a and B = b
Total work = 24 (a+b)
Pi

According to the question


24(a+b)
2a + 24(a+b)
2b = 50
(a+b)2
⇒ ab = 25 6
Work done in 9 days = 9 x (3+2+1) = Work done in 4 days = (6+4) x 4 =
Only 3 and 2 can be the values of a
54 unit 40 unit
and b to satisfy the given condition.
Let D be the number of days in Work left = 60-40 = 20 unit
So a=3 and b = 2
which this work was done. Number of days taken by Shivam to
…….(a ≻ b)
⇒ 54 = 2 x (D-3) + 1 x D finish this work = 20
4 = 5 days Total work = (3+2) x 24 = 120
⇒ 60 = 3D Desired number of days = 4+5 = 9 40
120× 100
⇒ D = 20 Desired number of days = 2 =
days
Total days taken to finish this work = 24 days
20 + 9 = 29 days Sol 102. (a)
Alternate : Let total work = 60 unit Sol 105. (b)

www.ssccglpinnacle.com support@ssccglpinnacle.com Ph. 09729327755, 09817390373 405


/
Days 43-46 Work and Time

s
sse
la
_c
ob
Let efficiency of Man = m and Total number of days = 20+8 = 28 According to the question
bo
ah
efficiency of Woman = w days (27-8) unit = 7600
sm
ur
yo

According to the question 1 unit = 400


e/
t.m
://

(36m + 48w) = 5 x (6m+12w) Alternate : Total wages of A, B and C = 60 unit


tp
ht

36m + 48w = 30m + 60w Total work = (3+7) x 14 = 140 unit = 60 x 400 = 24000
6m = 12w Work done by B in 8 days = 7 x 8 =
⇒ m:w = 2:1 56 unit Sol 110. (c) Let the required number
Total work = 36(2) + 48(1) = 30(2) + Work remaining = 140-56 = 84 unit of buckets = n
60 = 120 This work will be finished by A = According to the question
84 = 28 days
Desired number of woman = 8120 ×1 = 3 12 x 13.5 = 9 x n
15 Since A worked for the complete ⇒ n = 12 ×913.5 = 18
period total number of days taken to
Sol 106. (a) finish the work = number of days Sol111. (c)
Number of days taken by A to taken by A . Let the total work = 200 unit
complete one-third of a work = 5

e
days Sol 108. (b)
Number of days taken by A to Let the total work = 168 unit
complete the full work = 5 x 3 = 15
days

l
Number of days taken by B to
complete 2/5th of a work = 10 days
ac
Number of days taken by B to Total amount filled in 1 minute =
complete the full work = 10 x 25 = (10+8+5) unit
25 days Total amount filled in 10 minutes =
(10+8+5) x 10 = 230 unit
Amount of chemical B filled in 10
minutes = 10 x 8 = 80 unit
Work done by A in 20 days = 20 x 7 80 = 8
Required ratio = 230 23
= 140 unit
nn
Work done by B = 168-140 = 28 unit
⇒ B worked for 28
4 = 7 days
SSC CGL TIER I
⇒ X = 20-7 = 13 days
Alternate : Sol 1. (c) A can complete the work
168 = 7x + (7+4)(20-x) in 24 days
Let the total work = 75 unit 168 = 7x + 220 - 11x B can complete the work in 15 days
Work done in 6 days = (5+3) x 6 = 4x = 52 C can complete the work in 12 days
⇒ x = 13
Pi

48 unit
Work remaining = 75-48 = 27 unit
27 = 3
Efficiency of C = 18 Sol 109. (a)
2
A+B : A+B+C
Required number of days = 375/2 = 50
Work 13 : 15
days
C+B : A+B+C
Work 11 : 20
Sol 107. (a) Here, total work done be
Balancing the ratio for total work
Total work = (3+7) x 14 = 140 unit lcm(24,15,12) = 120 units
A+B : B+C : A+B+C 120
Work done in 8 days = (3+7) x 8 = Thus, efficiency of A,B and C is 24
52 : 33 : 60 120 120
80 unit =5, =8, =10 respectively.
Work done by A = 60-33 = 27 unit 15 12
Work remaining = 140-80 = 60 unit
Work done b C = 60-52 = 8 unit
This work will be finished by A = Work done by B and C in 3 days is
60 = 20 days
Amount will be distributed
3 according to the amount of work (18
done by each person. × 3)= 54 units

www.ssccglpinnacle.com support@ssccglpinnacle.com Ph. 09729327755, 09817390373 406


/
Days 43-46 Work and Time

s
sse
la
_c
ob
Work remaining = 120-54= 66 units Remaining work = 75 units. ⇒ 16 × 8 × m = 10 × (8 × m+ 9 × w)
bo
ah
A will do 66 units work in 66 75 ⇒ 48 × m = 90 × w
5 days = B complete remaining work in =
sm
5
ur

⇒ m:w = 15:8
yo

13 51 days 15 days.
e/
t.m

20 women finish the work in =


://
tp

16×8×15 = 12 days.
ht

20×8
Sol 2. (c) A can complete the work Sol 5. (a) Working together A and B
in 40 days. takes ‘d’ days.
Sol 9. (d)
Working alone, A takes ‘8+d’ days
A, B and C can individually
We know that, And B takes ‘18+d’ days.
1 1 1
complete a task in 24 days, 20 days
efficiency ∝ time 1taken d = 8+d + 18+d and 18 days respectively
It is given that B is 25% more d= 12 days. Total work = lcm(24,20,18) = 360
efficient than A and C is 28% more Now, working alone A takes ‘20 units
efficient than B days’ and B takes ‘30 days’. Efficiency of A,B and C =
So, their efficiency can be stated as Total work = lcm(20,30) = 60 units
60 = 3 24 /
360 = 15units day ,

follows: Efficiency of A = 20 360


20 = 18units/day ,
A:B:C = 4 : 5 : 6.4 Efficiency of B = 60 =2
30 360 = 20units/day respectively.

e
18
Total work done by A in 40 days= 4 In 4 days, A does 12 units of work
Work done by B and C in 6 days = 6
× 40 = 160 units B complete work in 60−2 12 = 24 days.
× (38) = 228 units
Total work done in 5 days = 5 ×
Remaining work = 360-228 = 132
(15.4) = 77 units Sol 6 (d) Let efficiency of 1men = m

l
units.
Remaining work= 83 units units and efficiency of 1 woman = w
ac A can complete remaining work in
B can complete remaining work in units 132 = 8 4 days
83 = 16 3 days . 15 5
5 5 According to question:
5(4m+6w)=7(3m+4w)
Sol 10. (d)
Sol 3. (d) A can finish the work in ⇒ Efficiency of 1 men = 2women
Time taken by Amit+Sunil = 9 days
20 days Total work = 5[4(2w)+6w] = 70w Time taken by Sunil+Dinesh = 12
B can finish the work in 25 days units. days
Total work = LCM(20,25) = 100 Let x men assist 25 women to Time taken by Dinesh+Amit = 18
units complete 25 times 70w units of work days
nn
in 5 days. Total work = lcm (9,12,18) = 36
Efficiency of A = 100 = 5unit/day 5 units
20 2 × 70w = 5[25w + x(2w)] efficiency (Amit+Sunil) = 36
Efficiency of B = 100 = 4unit/day 9 = 4
25 ⇒ 175w = 125w+ 10wx
unit/day
⇒ 50 = 10x 36 = 3
efficiency (Sunil+Dinesh) = 12
Work done by A and B together in 5 ⇒x=5 unit/day
days = (5+4) × 5 = 45 units 36 = 2
efficiency (Amit+Dinesh) = 18
Sol 7. (c) According to question, unit/day
Remaining work = 100-45=55 units
Pi

X:Y:Z = 7:5:3 = 15 units


A complete remaining work in 55
5 = Total work = 21 × (15) = 315 units Amit’s efficiency = 1.5 unit/day
11 days. In 35 days, Y and Z completed = 35 Sunil’s efficiency = 2.5 unit/day
Dinesh’s efficiency = 0.5 unit/day
× 8 = 280 units
Sol 4. (a) A can complete work in 30 Remaining work = 35 units 36
Together they complete work in 4.5
days. X can complete 35 units work in 35 7 = 8 days
Efficiency of A:B = 4:5 = 5 days.
Efficiency of B:C = 5:6 Sol 11. (a) A alone can complete
Efficiency A:B:C= 4:5:6 Sol 8. (b) Let efficiency of a man = work in = 20 days
Total work done by A in 30 days= 4 ‘m’ and efficiency of a woman = ‘w’ B alone can complete work in = 16
× 30 = 120 units Sixteen men can finish the work in 8 days
In 3 days, they will together days, thus, total work = 16 × 8 × m C alone can complete work in = 30
complete (3*[4+5+6])= 45 units of 8 men and 9 women can finish work
days
work in 10 days, implies, total work =
10 × (8 × m+ 9 × w)

www.ssccglpinnacle.com support@ssccglpinnacle.com Ph. 09729327755, 09817390373 407


/
Days 43-46 Work and Time

s
sse
la
_c
ob
Total work = lcm(20,16,30) = 240 Sol 16. (d) A can complete the task task completed by 3 men and 5
bo
ah
units in 24 days 28×34 = 56 days
women in = 3×4
sm
+ 5×1
ur
yo

Individual efficiency of A, B and C B can complete the task in 16 days


e/
t.m
://

= 240 = 12, 240 = 15 and 240 = 8 C can complete the task in 32 days Sol:5.(b)
tp

20 16 30
ht

respectively. Total work = lcm(24,16,32) = 96 Efficiency ratio of A to B=29:20


Work done by A and B in 4 days = units B does remaining work in 29 days
96 = 4 units/day
Efficiency of A = 24 Then work done by B=29 × 20=580
27 × 4 = 108 units
Remaining work = 132 units Efficiency of B = 96 = 6 units/day Total work = (29+20)×58+580
29
16
Days required by C to finish Efficiency of C = 96 = 3 units/day =118 days
32
remaining work = 132 1
8 = 16 2 days
Sol:6.(b)
Work done by A and C in 6 days =
Ratio of efficiency of Ramu to Samu
Sol 12. (b) 10 men = 12 women 42 units
=4:1
⇒ m:w = 6:5 Remaining work = 96-42 = 54 units
Ratio of time of Ramu to Samu = 1 :
⇒ Total work = work done by 10 Time required by B to complete
4
men in 10 days = 10 × 6 × 10 = 600 remaining work = 54
6 = 9 days 4 unit = 20 days

e
units 1 unit = 5 days
⇒ Work done by 5 men and 2 Sol 17. (c) Total work = 8 × 20 = Total work = 20 × 1 = 20 unit
women in 1 day = 5 × 6+2 × 5 = 40 160 units Time taken by both = 1+420 = 4 days
units Work done by 8 workers in 5 days =

l
⇒ time to complete 600 units work 8 × 5 = 40 units
Sol:7.(c)
= 600 ac
40 days = 15 days

Sol 13. (c) M1D1 = M2D2


⇒ 18 × 35 = 21 × D2
⇒ D2 = 30 days
Remaining work = 120 units
Let x more workers be employed to
complete remaining work in the next
8 days.
120 = (8+x) × 8
Let the total work = 48 unit (LCM of
12 and 16)
efficiency of A = 48/12 = 4
efficiency of B = 48/16 = 3
Work done by them in 4 days =
x=7 (4+3) × 4 = 28
Sol 14. (c) A can do work in 6 days, SSC CHSL 2019 Remaining work = 48-28 = 20
B can do the same work in 9 days Sol:1.(b) 20 = 5
left work in fraction = 48
nn
12
and (A,B and C) together complete 15 × 10 = 20 × x
work in 3 days x = 7.5 days
Sol:8.(a)
Total work = lcm(6,9,3) = 18 units Let number of men required = x
A’s efficiency = 18
6 = 3 unit/day
Sol:2.(a) x×10 = 15×6
M 1 D1 H 1 M 2 D2 H 2 50 10
B’s efficiency = 18 = 2 unit/day =
9 W1 W2 x = 45
(A+B+C)’s efficiency = 18 =6 30×16×8 32×20×H 2
3 1 = 2
unit/day H 2 =12 hours per day Sol:9.(a)
Pi

C’s efficiency = (6-3-2) = 1 unit/day Let the total work = 20 unit


C alone can do work in 18 1 = 18 days Sol:3.(d) Efficiency of Raju = 20/20 = 1
Total work completed by shyam in 5 days work of Raju = 1 × 5 = 5 unit
Sol 15. (a) Ram can complete work 12 days by working 10 hours a day = Remaining work = 20-5 = 15 unit
in 6 32 days = 20
3 days 12 × 10 = 120 units Required Hours Efficiency of Jakob = 15/15 = 1
20 = 10 days
Time taken by both = 1+1
Shyam can complete work in 15 a day to complete the task in 8 days
days = 120/8 = 15 hours.
Total work = lcm( 20
3 ,15) = 60 units Sol:10.(c)
Efficiency of Ram = 9 units/day Sol:4.(c) Let the total work = 48 unit (LCM of
Efficiency of Shyam = 4 units/day (5m + 8w)34 = (4m + 18w)28 12 and 16)
Work done in 4 days = 52 units m 4
w = 1
Efficiency of A = 48/12 = 4
Shyam complete remaining work in Total work = ( 5 × 4 + 8 × 1) 34 Efficiency of B = 48/16 = 3
8 Total work in 2 days = (4+3) unit
4 = 2 days = 28 × 34
Total work in 12 days = 42 unit

www.ssccglpinnacle.com support@ssccglpinnacle.com Ph. 09729327755, 09817390373 408


/
Days 43-46 Work and Time

s
sse
la
_c
ob
On the day of 13th, work done = 46 Sol:15.(c) Efficiency (Ravi + Mohan) = 21 =7
bo
3
ah
unit
sm
units/day
ur
yo

Remaining 2 unit work will be done Efficiency (Ravi) = 21 = 3 units/day


e/
t.m

7
://

by B in 32 days
tp

Efficiency (Mohan) = 7-3 = 4


ht

So, total time = 13 32 days units/day


Mohan alone can complete work in
Sol:11.(c) Work done by A and B in 4 days = = 21 1
4 = 5 4 days
Let the total work = 36 unit (LCM of (2+3)4 = 20
12 and 18) the fraction of the work left = 10/30 Sol:20.(a)
Efficiency of A = 36/12 = 3 = 1/3 (A + B) can finish 45% of work in 9
Efficiency of B = 36/18 = 2 days.
Total work in 6 days = (3+2) × 6 = Sol:16.(d) ∴ A + B can complete 100% of work
30 Efficiency ratio of A and B = 2 : 1 in 9 × 100
45 = 20 days
Remaining work = 36-30 = 6 unit Total work = (2+1) × 22 = 66 A can complete work in 30 days
Time taken by B to complete the A alone finish the work = 66
2 = 33 Total work = LCM (20, 30) = 60

e
remaining work = 36 = 2 days units
days
60 = 3
Efficiency (A+B) = 20
Sol:12.(a) Sol:17.(c) units/day
Let the total work = 30 unit Efficiency (A) = 60 = 2 units/day

l
30
Efficiency of Amir = 30/30 = 1
Efficiency (B) = 3 - 2 = 1 unit/day
Efficiency of Akbar = 30/15 = 2
ac B can complete work in = 60 1 = 60
Work done by Akbar in 8 days = 8*2
days
= 16 unit
Remaining work = 30-16 = 14 unit
2(A+B+C) = 12 Sol:21.(a)
Time taken by Amir = 14/1 = 14
A+B+C = 6 X + Y can finish work in 15 days
days
A, B and C work together, then they Y can finish work in 40 days.
will complete the same work = 60/6 Total work = LCM (15, 40) = 120
Sol:13.(b)
Efficiency (X+Y) = 12015 = 8
nn
Let the total work = 120 unit (LCM = 10 days
Efficiency (Y) = 120 =3
of 15 and 24) 40

Efficiency of A and B together = Sol:18.(c) Therefore, efficiency (X) =


120 = 8 Efficiency (X+Y) - Efficiency (Y) =
15
Efficiency of B and C together = 8-3=5
120 = 5 X can finish work in 120
5 = 24 days.
24
Efficiency ratio of A to C = 2 : 1 or
6:3 Sol:22.(c)
Work done by A and B together in 2
Pi

then, efficiency of B = 2 P can finish the work in 10 days


days = 2 × 5 = 10 Q can finish the work in 5 days.
Time taken by B to complete the
Remaining work done by A = 20/2 = Total work = LCM (10,5) = 10 units
work = 1202 = 60 days 10 days 10 = 1 unit per day
Efficiency (P) = 10
The total number of days needed for 10
Sol:14.(c) Efficiency (Q) = 5 = 2 units per
the completion of the work = 2+10 =
Let the total work = 100 unit day
12 days
Efficiency of Antony and VIkash = Work done by Q in 2 days = 2 × 2 =
100 = 5 4 units
20 Sol:19.(a)
Efficiency of VIkas = 100 =4 Work left = 10 - 4 = 6 units
25 Ravi + Mohan can complete work in
P can finish remaining work in = 16
Efficiency of Atony = 5-4 = 1 → 3 days
Time taken by Atony to complete the Ravi alone can complete work in → = 6 days
work = 100
1 = 100 days 7 days
Sol:23.(d)
Total work = LCM(3, 7) = 21 units
Let the total work = 36 unit

www.ssccglpinnacle.com support@ssccglpinnacle.com Ph. 09729327755, 09817390373 409


/
Days 43-46 Work and Time

s
se
s
la
_c
ob
Efficiency of Smit = 36/12 = 3 Efficiency of A : B : C = 4 : 5 : 7
bo
ah
Efficiency of Ajit = 36/18 = 2 Sol:27.(b) If A can do work in 15 days then
sm
ur
yo

Total work in 4 days = (3+2) × 4 = Ravi can complete work in 12 days (let) total work is = 15 × 4 = 60
e/
t.m
://

20 unit Mohan can complete work in 10 One day work of A and C = 4+7 =11
tp
ht

Remaining work = 36-20 = 16 unit days 3 day work of A and C = 33


Required fraction = 3616 = 4 Govind can complete work in 15 Work left = 60 - 33 = 27
9
days One day work of A and B = 4+5 = 9
Sol:24.(b) Total work = LCM (12, 10, 15) = 60 Time required = 27 9 = 3 days
25 men can complete a task in 16 units
60 = 5 units
Efficiency (Ravi) = 12
days. Sol:31.(c)
Let efficiency of 1 men = a units/day per day A can do complete work in =
Efficiency (Mohan) = 60 = 6 units 100 × 4 = 20 days
Total work = 25a × 16 a units = 400a 10 20
units per day B can do complete work in =
25 men worked for four days, then 5 Efficiency (Govind) = 60 = 4 units 100 × 10 = 30 days
15 33 1
3
more men joined the work; i.e. 25 per day Efficiency of A : B = 3 : 2

e
men in four days completed = 25a × Total efficiency = (5 + 6 + 4) units Let total work = 20 × 3 = 60 units
4 = 100a units per day = 15 units per day 9 day work of A and B = 9 × (2+3) =
Remaining work = 400a- 100a = Time taken by them together to
45 units
300a units complete the work = 15 60 = 4 days
Remaining work = 60- 45 = 15

l
Now, 5 more men joined the work,
Efficiency of C = 15
6 = 2.5
total men = 30
ac Sol:28.(c)
30 men can complete 300a units of 75% of work = 45 units
Kamal can finish work in 8 days
work in = 300 a One day work of B and C =2+2.5 =
30 a = 10 days Anil can finish the same work in 14
4.5
days.
B and C can complete the work in =
Sol:25.(c) Total work = LCM ( 8, 14) = 56 45
Ratio of efficiency of P and Q = 3 : 1 4.5 = 10 days
units
Thus, efficiency of P = 3a and Efficiency (Kamal) = 568 = 7 units
efficiency of Q = a Sol:32.(a)
per day
According to question = 3M + 4W =
nn
Q can complete a task in 24 days. Efficiency (Anil) = 56 = 4 units per
Total work = Time taken by Q ×
14 7 days
day 21M + 28W can do it one day
Efficiency of Q = 24 × a
Amount paid will be in the same According to question 2M + W = 14
P and Q can together finish the work ratio as the efficiency of Kamal and days
in = sum of ef ftotal work 24 a
iciency of P and Q = 3a + a = Anil respectively. 28M + 14W can do it in one day
24 a = 6 days
4a Total efficiency = 11 units per day Comparing one day work
Anil earnings = 440011 × 4 = Rs. 1600 21M + 28W = 28M + 14W
Sol 26. (c) Efficiency of M : W = 2 : 1
Pi

P can complete work in 10 days. Sol:29.(c) Total work = 21 × 2 + 28 × 1 = 70


Q can complete work in 15 days. Let the total work = 36 unit (LCM of 7 women can do it in= 70
7 = 10 days
Total work = LCM (10,15) = 60 6 and 9)
units Efficiency of Ravi = 36
6 = 6 unit/d
60 = 6 units per
Sol:33.(b)
Efficiency of P = 10 Efficiency of Mohan = 36 = 4 unit/d
9 efficiencies of A : B = 7 : 5
day Time taken by Ravi and Mohan = They can complete the work in 5x :
Efficiency of Q = 60 = 4 units per 36 18 3
15 6+4 = 5 = 3 5 days 7x days
day Together they can complete the work
Work done by P and Q together in 3 SSC CGL 2019 TIER-II in 35x
12
days = (6 + 4) × 3 = 30 units According to the question
Work left = 60 - 30 = 30 units Sol:30.(d) 35x = 35
work lef t 12 2
30
Required fraction = total work = 60 = Efficiency of A : B = 4 : 5 x=6
1 Efficiency of B : C = 5 : 7
2

www.ssccglpinnacle.com support@ssccglpinnacle.com Ph. 09729327755, 09817390373 410


/
Days 43-46 Work and Time

s
se
s
la
_c
ob
A can complete the work in = 30 It’s given in the question that A
bo
daysah works for 5 days and B is replaced
sm
ur
yo

60% work can be completed by A in by C after 2 days


e/
t.m
://

60 =18 days
= 30 × 100 Total work done by A =5 × 2=10
tp
ht

units
Sol:34.(c) Total work done by B = 4 × 2=8 units
2M + 7W = 28 Efficiency of a = 2 × efficiency of c Work done by C in 3 days = 2 units
6M + 16W = 11 2c + b = 5 Therefore 20 units of work will be
Comparing both the equations c+b=3 done by C in 30 days
56M + 196W =66M + 176W B=1 60% of work will be done by C in
20W = 10M hence, .60 × 30= 18 days
Ratio of efficiency of M : W = 2 : 1 B can complete the work in 90 days
Total work =56 × 2 + 196 × 1 = 308 Sol:42.(d)
5M + 4W = 10 + 4 = 14 Sol:38.(c)
Time require = 308
14 = 22

e
Sol:35.(b)
A can do complete work in = 90
days

l
B can do complete work in = 60
Efficiency of b = 1 Combined efficiency 2(A + B + C )
days ac B can complete the work in = 36 = 12
A and B 8 days work = 20
days A+B+C=6
1 )
× ( 901 + 60
So A = 3
5 )=
A and B 8 days work = 20 × ( 180 Sol:39(c) Time needed by A = 180
3 = 60 days
5
9 Efficiency ratio of a : b = 3 : 1
Work left = 4 A and B can complete the work in
9 Sol:43.(d)
C can do the complete work in = 8 18 days A can do in 3 days
× 49 = 18 days Total work = 18 × 4 = 72 B can do in 8/3 days
nn
Working together their one day work B can complete the work in = 72 C can do in 12/5 days
1 + 1 ) = 15 = 1
= ( 901 + 60 days
18 180 12
They can complete the work in = 12
SSC CPO 2019
days
Sol:40.(c)
Let efficiency of A= 2x
Sol:36.(a)
Efficiency of B= X
Total work= 108 unit(LCM of
Total efficiency=2x+x=3x
Pi

18,36,54)
Total work= efficiency × time
Efficiency of A=6 Combined efficiency = 27
= 13 × 3x= 39x (total work) 24
Time taken = 27
Efficiency of B=3
Efficiency of C=2 Number of days in which B alone 24 16
Time taken (in hours) = 27 ×6 = 3
Let, The number of days to finish the finishes the work=total work
= 5 hours 20 minutes
work=x efficiency of B
Number of days B’s work=x-5 =39x ÷ x=39.
Sol:44.(a)
Number of days C’s work=x-10 LCM of 36, 60 and 45 is 180
6 × x+3(x-5)+2(x-10)=108 Sol:41.(c)
x=13 days A can do a job in 10 days
B can do the job in 5 days
Sol:37.(d) Let total work be 20units
A and B will do 2 units and 4 units
of work in a day respectively.

www.ssccglpinnacle.com support@ssccglpinnacle.com Ph. 09729327755, 09817390373 411


/
Days 43-46 Work and Time

s
se
s
la
_c
ob
Sol:48.(c) To complete two- third of the work =
bo
ah Let efficiency of B=x 18 × 32 = 12
sm
ur
yo

Then efficiency of A will be 2x


e/
t.m
://

Together they finish the work in 13


tp
ht

days
So (x+2x) × 13 = total work

A alone4 will finish the work in


Combined efficiency 2(A + B + C ) T otal work
Ef f iciency of A
= 12
= 39x
2x
A+B+C=6
So, efficiency of B = 2 =19.5 days
Time taken by B = 1802 = 90 days
Sol:49.(d)
Sol:45.(b)

e
A worked for 5 days and B for 2
days

l
Efficiency of A = 4 Total work completed by A and B is
Time taken by A = 225 5 × 2 + 4 × 2= 18 units
4
15
= 4

Sol:46.(c)
×
ac15 = 15 days
4
of the work will be completed in
225 4
Remaining 2 units of work was done
by C in 3 days
So 40%of Total work= 8 units of
work will be done by C in 23 × 8 =12
days
A and B worked for 8 days so total
work done =(7+3) × 8=80
Work left to be completed=105-80= Sol:50.(b)
nn
25 units
25 units of work will be done by A
in 25
3 days
60% of this work will be completed
in 53 × 25
3 =5 days Total efficiency = 3 + 7 = 10
In 7 days work done = 7 × 10 = 70
units
Work left = 35units
Pi

60 ×
60 % of the remaining work = 100
35 = 21 units
Sol:47.(c) A can complete it in = 21 = 7 days
3
P and Q can together do a work in 12
days Sol:51.(a)
P can do same work in 18 days
Let total work=36
Efficiency of P and Q =3 units
Efficiency of P=2 units
Then efficiency of Q= 1 unit
Q can complete 32 of the same work Efficiency of Q = 3 - 1 = 2
in 2 × 36=24 days. Time taken by Q = 36
3 2 = 18

www.ssccglpinnacle.com support@ssccglpinnacle.com Ph. 09729327755, 09817390373 412


/
Days 47-48 Pipes and Cistern

s
sse
la
_c
ob
PIPE AND CISTERN / पाइप (d) 3 43 hours many hours will the empty tank
bo
ah
और टं की be filled?
sm
ur
yo

Q3. An inlet pipe can fill a tank in दो पाइप A और B िकसी खाली टं की


e/
t.m

को मशः 8 और 12 घंटे म भर दे ते
://

Variety Questions 10 hours and an outlet pipe can


tp
ht

empty the completely filled tank ह | पाइप A से शु आत करते ए


Q1. Pipes A and B can fill a tank in 20 hours. Both the pipes are उ एक-एक करके 1 घंटे के िलए
in one hour and two hours opened at 6.30 a.m. When will खोला जाता है | िकतने घंटों म यह
respectively while pipe C can the tank get filled ? खाली टं की भर जायेगी ?
empty the filled up tank in one एक अंतगम पाइप िकसी टं की को 10 SSC CPO 12 March 2019
hour and fifteen minutes. A and C घंटों म भर सकता है जबिक एक (Evening)
are turned on together at 9 a.m. बिहगम पाइप पूरी तरह से भरी ई (a) 9 31
After 2 hours, only A is closed टं की को 20 घंटों म खाली कर सकता (b) 9
and B is turned on. When will the है | दोनों पाइपों को सुबह 6 : 30 बजे (c) 9 21
tank be emptied ? चालू िकया जाता है | टं की कब भर (d) 9 41
पाइप A और B िकसी टं की को जाएगी ?

e
मशः एक घंटे और दो घंटे म भर SSC CHSL 3 July 2019 Q6. Pipes A and B can fill a tank
सकते ह लेिकन पाइप C भरी ई (Afternoon) in 16 hours and 24 hours
टं की को एक घंटे 15 िमनट म खाली (a) 2.30 a.m. next day/ अगले िदन respectively whereas pipe C can
कर सकता है | A और C को एक 2.30 am बजे empty the full tank in 40 hours.

l
साथ सुबह 9 बजे चालू िकया जाता है (b) 2 a.m. next day/ अगले िदन All three pipes are opened
| 2 घंटे बाद, केवल A को बंद िकया
ac 2am बजे together, but pipe A is closed
जाता है और B को चालू िकया जाता (c) 1 a.m. next day/ अगले िदन after 8 hours. After how many
है | टं की कब खाली हो जाएगी ? 1a.m बजे hours, the remaining part of the
SSC CGL 6 June 2019 (d) 12:00 midnight/ म राि tank will be filled?
(Morning) 12:00 बजे पाइप A तथा पाइप B िकसी टं की को
(a) 12:10 p.m. मशः 16 और 24 घंटों म भर सकते
(b) 11:30 a.m. Q4. One tap can fill a tank in ह जबिक पाइप C भरी ई टं की को
(c) 10:30 a.m. 3hours and a leak can empty it in 40 घंटे म खाली कर सकता है | इन
5 hours. If the tap and the leak
nn
(d) 12:20 p.m. तीनों पाइप को एक साथ खोल िदया
(which was half closed) were left जाता है लेिकन 8 घंटे बाद पाइप A
Q2. Pipes A and B can fill a tank open, how long will it take for the बंद कर िदया गया | टं की का शेष
in 8 hours and 12 hours, tank to fill? भाग िकतने घंटों म भरे गा ?
respectively whereas pipe C can एक नल िकसी टं की को 3 घंटे म भर SSC CPO 13 March 2019
empty the full tank in 6 hours. A सकता है और एक छे द इसे 5 घंटे म (Evening)
and B are opened for 3 hours and खाली कर सकता है | यिद नल एवं (a) 28
then closed and C is opened छे द (जो आधा बंद था ) को खुला (b) 22
instantly. C will empty the tank in छोड़ िदया जाए, तो टं की को भरने म
Pi

(c) 26
: पाइप A और B िकसी टं की को िकतना समय लगेगा ? (d) 30
मशः 8 घंटे और 12 घंटे म भर SSC CPO 16 March 2019
सकते ह जबिक पाइप C भरी ई (Morning) SSC CGL TIER II
टं की को 6 घंटों म खाली कर सकता (a) 4 72 hours
है | A और B को 3 घंटे के िलए खोला (b)5 31 hours Q1. Pipes A, B and C can fill a
जाता है तथा िफर बंद कर िदया जाता (c) 6 32 hours tank in 30h, 40 h and 60 h
है और C को तुरंत चालू कर िदया respectively. Pipes A, B and C are
(d)7 21 hours
जाता है | C टं की को िकतने समय म opened at 7 a.m., 8 a.m., and 10
खाली कर दे गा ? SSC CHSL 3 a.m., respectively on the same
Q5. Two pipes A and B can fill an
July 2019 (Morning) day. When will the tank be full?
empty tank in 8 hours and 12
(a) 4 21 hours पाइप A, B और C िकसी टं की को
hours respectively. They are
(b) 4 41 hours मशः 30, 40 और 60 घंटों म भर
opened alternately for 1 hour each
(c) 3 21 hours starting with pipe A first. In how सकते ह | पाइप A, B और C को

www.ssccglpinnacle.com support@ssccglpinnacle.com Ph. 09729327755, 09817390373 413


/
Days 47-48 Pipes and Cistern

s
sse
la
_c
ob
एक ही िदन मशः सुबह 7, 8 और एक साथ चालू कर िदया जाता है , तो SSC CPO 12 March 2019
bo
ah
10 बजे चालू िकया जाता है | टं की टं की 2 घंटे म भर जाती है | A और B (Evening)
sm
ur
yo

िकस समय भर जाएगी ? को 12 िमनट तक एक साथ चालू (a) 9


e/
t.m

िकया जाता है , िफर बंद करके C को


://

SSC CGL TIER II (11 (b) 12


tp
ht

September 2019) चालू िकया जाता है | टं की िकतने (c) 11


(a) 10.00 p.m. समय बाद खाली हो जाएगी ? (d) 10
(b) 10.20 p.m. SSC CGL TIER II (13
(c) 9.20 p.m. September 2019) Q3. Two pipes A and B can fill a
(d) 9.40 p.m. (a) 15 minutes tank in 16 hours and 20 hours
(b) 18 minutes respectively. They are opened
Q2. Pipes A and B can fill a tank (c) 12 minutes alternatively for 1 hour each,
in 16 hours and 24 hours, (d) 16 minutes starting with pipe A first. In how
respectively, and pipe C alone can many hours with the empty tank
empty the full tank in x hours. All Practice Questions be filled ?
the pipes were opened together at दो पाइप A और B िकसी टं की को
मशः 16 और 20 घंटे म भर सकते

e
10.30 a.m., but C was closed at Q1. Pipe A can fill a cistern in 4
2:30 p.m. If the tank was full at hours and another pipe B is ह | पाइप A से शु आत करते ए
8:30 p.m. on the same day, then installed. Both the pipes together उ 1 घंटे के िलए एक के बाद एक
what is the value of x ? fill the cistern in 2 21 hours. How करके खोला जाता है | खाली टं की
पाइप A और B िकसी टं की को िकतने घंटों म भर जायेगी ?

l
long will it take for B alone to fill
मशः 16 और 24 घंटे म भर सकते the cistern. SSC CPO 13 March 2019
ac
ह , तथा पाइप C अकेला भरी ई
टं की को x घने म खाली कर सकता
है | सभी पाइपों को सुबह 10 : 30
बजे एक साथ चालू िकया गया,
लेिकन 2 : 30 pm म C को बंद कर
पाइप A िकसी टं की को 4 घंटे म
भर सकती है | एक अ पाइप B
लगाई गयी | दोनों पाइप एक साथ
टं की को 2 21 घंटे म भर सकती है | B
को अकेले इस टं की को भरने म
(Evening)
(a) 17 53
(b) 17 51
(c) 17 41
(d) 17 43
िदया गया | यिद टं की उसी िदन 8 : िकतना समय लगेगा ? SSC CPO
30 pm म भर गई, तो x का मान ात 16 March 2019 (Morning)
कर |
nn
(a) 5 hours Q4. Two pipes A and B can fill a
SSC CGL TIER II (12 (b) 6 83 hours tank in 6 hours and 9 hours
September 2019)
(c) 6 32 hours respectively. They are opened
(a) 64
(d) 5 61 hours alternately for 1 hour each,
(b) 48
Q2 . Pipes A and B can fill a tank starting with pipe A first. In how
(c) 45
in 6 hours and 8 hours many hours will the tank be
(d) 96
respectively and pipe C can filled?
empty the full tank in 12 hours. दो पाइप A और B िकसी टं की को
Q3. Pipes A and B are filling
Pi

All three pipes are opened मशः 6 और 9 घंटे म भर सकते ह |


pipes while C is an emptying
together, but pipe A is closed पाइप A से शु आत करते ए उनम
pipe. A and B can fill a tank in 72
after 3 hours. In how many hours से ेक को एक-एक करके 1 घंटे
and 90 minutes respectively.
will the remaining part of the tank के िलए खोला जाता है | टं की को
When all the three pipes are
be filled? भरने म िकतने घंटे लगगे ? SSC
opened together, the tank gets
पाइप A और पाइप B िकसी टं की CPO 12 March 2019 (Morning)
filled in 2 hours. A and B are
को मशः 6 और 8 घंटे म भर सकते (a) 5
opened together for 12 minutes,
ह तथा पाइप C भरी ई टं की को 12 (b) 4
then closed and C is opened. The
घंटे म खाली कर सकता है | इन तीनो (c) 6
tank will be empty after :
पाइप को एक साथ खोल िदया जाता (d) 7
पाइप A और B भरने वाले पाइप ह
जबिक पाइप C खाली करने वाला है लेिकन 3 घंटे बाद पाइप A बंद कर
िदया गया | टं की का शेष िह ा Q5. Pipes A and B can fill a tank
पाइप है | A और B िकसी टं की को
िकतने घंटे म भरे गा ? in 6 hours and 9 hours
मशः 72 और 90 िमनट म भर
respectively and pipe C can
सकते ह | जब इन तीनों पाइप को

www.ssccglpinnacle.com support@ssccglpinnacle.com Ph. 09729327755, 09817390373 414


/
Days 47-48 Pipes and Cistern

s
se
s
la
_c
ob
empty the full tank in 12 hours. If पाइप A और पाइप B िकसी टं की (c)12 31 h
bo
ah
all three pipes are opened को मशः 12 घंटे एवं 16 घंटे म भर
sm
(d)6 32 h
ur
yo

together when a tank is empty. In सकते ह जबिक पाइप C भरी ई


e/
t.m

टं की को 24 घंटों म खाली कर सकता


://

how many hours will 35% of the


tp

Q10. Pipe A can fill a tank in 16


ht

tank be filled? है | इन तीनों पाइप को एक साथ चालू minutes and pipe B empties it in
पाइप A तथा पाइप B िकसी टं की को कर िदया जाता है तथा 4 घंटे बाद 24 minutes. If both the pipes are
मशः 6 घंटे और 9 घंटे म भर सकते पाइप A बंद कर िदया जाता है | टं की opened together, after how many
ह और पाइप C भरी ई टं की को 12 को भरने म कुल िकतने घंटे लगगे ? minutes should B be closed, so
घंटे म खाली कर सकता है | यिद SSC CPO 13 March 2019 that the tank is filled in 30
टं की खाली होने पर तीनों पाइप एक (Morning) minutes?
साथ चालू कर िदए जाते ह, तो टं की (a) 24 पाइप A िकसी टं की को 16 िमनट म
का 35% भाग िकतने घंटों म भर (b) 28 भर सकता है तथा पाइप B इसे 24
जाएगा ? (c) 30 िमनट म खाली कर सकता है | यिद
SSC CPO 12 March 2019 (d) 32 इन दोनों पाइप को एक साथ चालू
(Morning) िकया गया है , तो B को िकतने िमनट

e
(a) 1.9 Q8. One-fourth of a tank can be बाद बंद करना होगा तािक टं की 30
(b) 1.5 filled in 3 hours by pipe A and िमनट म भर जाए ?
(c) 1.6 one-third of the same tank can be SSC CPO 16 March 2019
(d) 1.8 filled in 2 hours by pipe B. How (Evening)

l
long will it take for the tank to be (a)21 min
Q6. Two pipes A and B can fill an filled if both are pipes are kept (b)20 min
ac
empty tank in 10 hours and 16
hours respectively. They are
opened alternatively for 1 hour
each, starting with pipe A first. In
how many hours, the empty tank
open?
िकसी टं की का एक चौथाई भाग
पाइप A के ारा 3 घंटे म तथा इसी
टं की का एक-ितहाई भाग पाइप B के
ारा 2 घंटे म भरा जा सकता है | यिद
(c)18 min
(d)15 min

Q11. Two pipes A and B can fill a


tank 45 minutes. If pipe A can fill
will be filled? दोनों पाइप चालू कर िदए जाते ह, तो an empty tank in 1 hour, how
दो पाइप A और B िकसी खाली इस टं की को भरने म िकतना समय long will it take pipe B to fill the
टं की को मशः 10 घंटे एवं 16 घंटे लगेगा ?
nn
empty tank?
म भर सकते ह | A से शु आत करते SSC CPO 14 March 2019 दो पाइप A तथा B िकसी टं की को
ए उ एक-एक करके हर बार 1 (Morning) 45 िमनट म भर सकते ह | यिद पाइप
घंटे के िलए खोला जाता है | िकतने (a) 5 h A खाली टं की को 1 घंटे म भर
घंटों म खाली टं की भर जायेगी ? (b) 2 h सकता है , तो इस खाली टं की को
SSC CPO 13 March 2019 (c) 4 h भरने म पाइप B को िकतना समय
(Morning) (d) 2 21 h लगेगा ? SSC CPO 15 March
(a) 12 31 2019 (Morning)
(b) 12 81 Q9. A tank can be filled by pipe
Pi

(a) 2 hours
(c) 12 41 A in 5 hours and emptied by pipe (b) 3 hours
(d) 12 61 B in 8 hours respectively. How (c) 1 hour
much time will it take for the tank (d)4 hours
to be half full?
Q7. Pipes A and B can fill a tank
एक टक को पाइप A ारा 5 घंटे म Q12. A pipe can fill a tank in 30
in 12 hours and 16 hours
भरा जा सकता है और पाइप B ारा minutes. Due to two leakages A
respectively and pipe C can
मशः 8 घंटे म खाली िकया जा and B, the filled tank would be
empty the full tank in 24 hours.
सकता है । टक को आधा भरा होने म drained off in 1 21 hour and 1 41
All three pipes are opened
िकतना समय लगेगा? hour respectively. How long will
together, but after 4 hours pipe A
SSC CPO 14 March 2019 it take to fill the tank if the pipe,
is closed. In how many hours
(Morning) A and B left open?
from the beginning the tank be
(a)3 31 h एक पाइप िकसी टं की को 30 िमनट
filled?
(b)8 32 h म भर सकता है | दो िछ ों A एवं B

www.ssccglpinnacle.com support@ssccglpinnacle.com Ph. 09729327755, 09817390373 415


/
Days 47-48 Pipes and Cistern

s
sse
la
_c
ob
के कारण भरी ई टं की मशः 1 21 Q15. A pipe can fill a tank in 32 को 1 घंटे म खाली कर सकता है |
bo
ah minutes. Due to a leak the tank is यिद सभी चार पाइपों को एक साथ
घंटे तथा 1
sm
1 घंटे म खाली हो सकती
ur

4
yo

है | यिद A और B को खुला छोड़ filled in 48 minutes. How long खुला रखा जाता है तो उस टक की
e/
t.m

भरने म िकतना समय लगेगा? SSC


://

िदया जाए, तो पाइप ारा टं की को will it take to empty the tank due
tp
ht

भरने म िकतना समय लगेगा ? SSC to leakage? CPO 15 March 2019 (Evening)
CPO 15 March 2019 (Morning) एक पाइप 32 िमनट म एक टक भर (a)9 81
(a)1 87 hour सकता है | रसाव के कारण टक 48 12
(b)5 25
िमनट म भर पाता है | रसाव के 16
(c)8 43
(b)1 31 hour
कारण भरा आ टक खाली होने म
(c)1 54 hour (d)7 32
िकतना समय लगेगा ?
(d)1 65 hour SSC CPO 14 March 2019
SSC MTS
(Evening)
Q13. Two pipes can fill a cistern (a) 1 hour 36 minute
in 72 and 90 minutes respectively. Q18. Pipe A can fill a tank in 6
(b) 1 hour 42 minute
If both the pipes are left open hours. Pipe B can fill the same
(c) 1 hour 20 minute
how long will it take for the tank in 8 hours. Pipe A, B and C

e
(d) 1 hour 56 minute
cistern to be half full? together can fill the same tank in
दो पाइप िकसी टं की को मशः 72 12 hours. then which of the
Q16. Two pipes A and B fill tank
और 90 िमनट म भर सकते ह | यिद following statements is true for
in 20 min and 30 min.
दोनों पाइप चालू कर िदए जाए, तो pipe C?

l
respectively. If A is kept open for
टं की को आधा भरने म िकतना समय पाइप A, 6 घंटे म एक टक को भर
only the 1/5th of starting time and
ac
लगेगा ? SSC CPO 16 March सकता है । पाइप B उसी टक को 8
A and B are kept open for
2019 (Afternoon) घंटे म भर सकता है । पाइप A, B
remaining time; then how much
(a)40 minutes और C िमलकर उसी टक को 12 घंटे
time is taken by both pipe to fill
(b)24 minutes म भर सकते ह। िफर पाइप C के
tank.
िलए िन िल खत म से कौन सा कथन
(c)48 minutes दो पाइप A और B एक टक को
सही है ?
(d)20 minutes मश: 20 िमनट और 30 िमनट म
SSC MTS 2 August 2019
भर सकते ह| यिद शु म कुल समय
(Morning)
Q14. A pipe can fill a tank in 4 के ⅕ वे भाग के िलए केवल पाइप A
nn
(a)It can fill the tank in 4 hours
hours and a leak at the bottom can को और शेष समय के िलए पाइप A
40 minutes/यह टक को 4 घंटे 40
empty that full tank in 6 hours. If और B दोनों को खुला रखा गया था,
िमनट म भर सकता है
after the tank is 1/3 full, the leak तो टक को भरने म दोनों पाइपों को
(b)It can fill the tank in 4 hours
is completely closed, how much िकतने िमनट लगे होंगे?
time from beginning will it take 48 minutes/यह 4 घंटे 48 िमनट म
SSC CPO 15 March 2019
for tank to get filled completely? टक को भर सकता है
(Evening)
एक पाइप िकसी टं की को 4 घंटे म 5 (c)It can empty the tank in 4
(a)16 23
भर सकता है तथा तल पर मौजूद एक 1
hours 48 minutes/यह 4 घंटे 48
(b)13 23
Pi

िछ इस भरी ई टं की को 6 घंटे म िमनट म टक को खाली कर सकता है


5
(c)13 23
खाली कर सकता है | यिद टं की के (d)It can empty the tank in 4
1
(d)16 23
1/3 भाग भर जाने के बाद इस छे द hours 40 minutes/ यह 4 घंटे 40
को पूरी तरह से बंद कर िदया जाता िमनट म टक को खाली कर सकता
है , तो टं की को पूणतः भरने म शु Q17. Three pipes A, B and C can है |
से कुल िकतना समय लगेगा ? fill a cistern in 15,24 and 36
SSC CPO 16 March 2019 minutes respectively. If pipe D Q19. Pipe P can fill a tank alone
(Afternoon) can drain a full tank in 1 hour. in 7 hours. Pipe Q can fill the
(a)12 hours how long will it take for the tank same tank alone in 13 hours. In
(b)4 hours to be filled if all the four pipes are how much time can they together
(c)9 hours kept open together ? fill the tank?
(d) 20 तीन पाइप A, B और C मश: 15, पाइप P, 7 घंटे म अकेले एक टक भर
3 hours
24 और 36 िमनटों म टं की को भर सकता है । पाइप Q उसी टक को 13
सकते ह| वही, पाइप D पूरे भरे टक घंटे म भर सकता है । वे एक साथ

www.ssccglpinnacle.com support@ssccglpinnacle.com Ph. 09729327755, 09817390373 416


/
Days 47-48 Pipes and Cistern

s
sse
la
_c
ob
िकतने समय म टक को भर सकते time will the empty tank be time will the tank get completely
bo
ah
ह? filled? filled?
sm
ur
yo

SSC MTS 2 August 2019 पाइप A िकसी खाली टक को 11 पाइप VI एक खाली टक को 8 घंटे म
e/
t.m

घंटे म पूण प से भर सकता है | भर सकता है । पाइप V2 उस टक को


://

(Evening)
tp
ht

(a)3 hours 18 minutes/3 घंटे 18 पाइप B इस पूण प से भरे गए टक 16 घंटे म भर सकता है । पाइप V3
िमनट को 15 घंटे म खाली कर सकता है | उसी भरे ए टक को 12 घंटों म
(b)3 hours 45 minutes/3 घंटे 45 यिद दोनों पाइपों को एक साथ खोला खाली कर सकता है । यिद तीन पाइप
िमनट जाता है , तो खाली टक िकतने समय एक साथ खोले जाते ह, तो टक
(c)4 hours 12 minutes/4 घंटे 12 म भरे गा? िकतने समय म पूरी तरह भर जाएगा?
िमनट SSC MTS 5 August 2019 SSC MTS 8 August 2019
(d)4 hours 33 minutes/4 घंटे 33 (Afternoon) (Afternoon)
िमनट (a) 41
3 hours
(a) 41 hours and 15 minutes / 41 42 hours
(b) 11
घंटे 15 िमनट (c) 38
5 hours
(b) 49 hours and 45 minutes / 49 48
(d) hours
घंटे 45 िमनट 5

e
Q20. An inlet pipe A originating
from a river can fill a reservoir in (c) 47 hours and 30 minutes / 47
Q24.Pipe A can fill a tank in 36
30 days. And an outlet pipe B, घंटे 30 िमनट
minutes and pipe B is able to
which is capable of emptying the (d) 39 hours and 15 minutes / 39
empty this in 45 minutes. If both
घंटे 15 िमनट

l
completely filled reservoir in 50
pipes are opened simultaneously,
days, drains out the water from
ac
the reservoir to an irrigation
canal. The pipes are opened on
alternate days starting with A. On
which day from the beginning
will the reservoir get completely
Q22. Pipe D1 can fill an empty
tank in 80 minutes and Pipe D2
then the tank will filled half in
_______hours.
पाइप A िकसी टक को 36 िमनट म
भर सकता है I और पाइप B उसे 45
िमनट म खाली कर सकता है I यिद
दोनों पाइपों को एक साथ खोल िदया
filled for the first time? can empty the same tank in 100
है , तो टक की आधी मता को भरने
एक आं त रक पाइप एक नदी से minutes. If pipes are opened
म लगने वाला समय (घंटो म) है :
उ होने वाला एक जलाशय 30 simultaneously, then in how much
nn
SSC MTS 13 August 2019
िदनों म भर सकता है । और एक बा time the tank gets completely
(Morning)
पाइप B, जो 50 िदनों म पूरी तरह से filled?
(a) 2
भरे जलाशय को खाली करने म पाइप D1 एक खाली टक को 80
(b) 1.5
स म है , जलाशय से एक िसंचाई िमनट म भर सकता है और पाइप D2
(c) 1.25
नहर ारा पानी बाहर िनकाला जाता उसी टक को 100 िमनट म खाली
(d) 1.75
है । A से आर होते ए ये पाइप कर सकता है । यिद दोनों पाइप एक
बारी बारी से खोले जाते है । शु आत साथ खोले जाते ह, तो टक िकतने
Q25. Two pipes P and Q can fill
के िकस िदन के बाद जलाशय पूरी समय म पूरी तरह से भर जाएगा?
Pi

an empty tank in 20 minutes and


तरह से भर जाएगा? SSC MTS 7 August 2019
10 minutes respectively. R can
SSC MTS 5 August 2019 (Evening)
empty a full tank in 15 minutes. If
(Morning) (a)320 minutes
all three pipes are opened
(a)75th (b)400 minutes
together, how much time (in
(b)147th (c)200 minutes
minutes) will they take to fill the
(c)150th (d)240 minutes
tank?
(d) 74th
दो पाइप P और Q मशः 20 िमनट
Q23. Pipe VI can fill an empty
और 10 िमनट म एक खाली टक भर
Q21.Pipe A can fill an empty tank tank in 8 hours. Pipe V2 can fill
सकते ह। R, 15 िमनट म एक पूरा
completely in 11 hours. Pipe B the same tank in 16 hours. Pipe
टक खाली कर सकता है । यिद सभी
can empty this fully filled tank in V3 can empty the same,
तीन पाइप एक साथ खोले जाते ह, तो
15 hours. If both the pipes are completely filled, tank in 12
टक को भरने म िकतना समय
opened simultaneously, in what hours. If three pipes are opened
लगेगा?
simultaneously, then in how much

www.ssccglpinnacle.com support@ssccglpinnacle.com Ph. 09729327755, 09817390373 417


/
Days 47-48 Pipes and Cistern

s
sse
la
_c
ob
SSC MTS 14 August 2019 (a) 36 hours. If the inlet and outlet pipe
bo
ah
(Evening) (b) 21 are opened simultaneously, in
sm
ur
yo

(a) 12 (c) 45 what time will the empty tank be


e/
t.m
://

(b) 18 (d) 27 filled?


tp
ht

(c) 10 कोई आं त रक पाइप 2 घंटे म िकसी


(d) 15 Q28. A pipe can fill a tank in 56 टं की को भर सकता है और बाहरी
hours, but due to leakage in its पाइप 3 घंटे म पूरी भरी ई टं की को
Q26. Pipe A can fill a tank in 10 base, it takes 16 hours more to fill खली कर सकता है . यिद आं त रक
hours. Pipe B can fill the same it. Only in how much time (in और बाहरी पाइप को एक साथ खोल
tank in 12 hours. Pipe C can hours) can the 83 31 % portion of िदया जाता है , तो खाली टं की िकतने
empty a full tank in 16 hours. All the leakage tank be consumed? / समय म भर जाएगी?
the pipes are opened at 8:00 AM एक पाइप 56 घंटे म िकसी टक को SSC MTS 20 August 2019
and pipe A and B are closed at भर सकता है , लेिकन इसकी तली म (Evening)
10:00 AM. After how much time रसाव के कारण उसे भरने म 16 घंटे (a) 2 h
from starting will the tank be अिधक लगते है । मा रसाव के (b) 23 h

e
empty? कारण टक के 83 31 % िह े को (c) 3 h
पाइप A, 10 घंटे म एक टक भर िकतने समय (घंटों म) म खIली कर (d) 6 h
सकता है । पाइप B उसी टक को 12 सकता है | SSC MTS 20 August
घंटे म भर सकता है । पाइप C, 16 2019 (Morning) Q31. Pipes A and B can empty a
घंटे म पूरा टक खाली कर सकता है ।

l
(a) 126 filled tank in 20 hours and 15
सभी पाइप सुबह 8:00 बजे खोले (b) 210 hours respectively, while pipe C
ac
जाते ह और पाइप A और B 10:00
AM पर बंद होते ह। आर
िकतने समय म टक खाली हो जाएगा
?
SSC MTS 19 August 2019
से
(c) 252
(d) 105

Q29. Pipes A and B can empty a


fully filled tank in 8 and 12 hours
alone can fill the same tank in x
hours. The three pipes have been
opened simultaneously and they
took 40 minutes to finish the 1/18
(one-eighteenth) part of the tank.
(Morning) respectively. C is a filling pipe. The value of x is:
(a) 5 hours 52 minutes/5 घंटे 52 These three pipes were opened at पाइप A और B एक भरे टक को
िमनट मश: 20 घंटे म और 15 घंटे म
nn
the same time and in one hour the
(b) 5 hours 24 minutes/5 घंटे 24 sixth part of the tank became खIली कर सकते है , जबिक उसी टक
िमनट empty in one hour. In how much को पाइप C अकेला x घंटों म भर
(c) 4 hours 30 minutes/4 घंटे 30 time can C fill the tank alone? सकता है . तीनो पाइप एक साथ खोल
िमनट पाइप A और B मशः 8 और 12 िदए गए है और उ ोंने टक के 1/18
(d) 4 hours 8 minutes/4 घंटे 8 घंटे म एक पूरी तरह से भरे टक को (one-eighteenth) भाग को खIली
िमनट खाली कर सकते ह | C भरने वाला करने म 40 िमनट का समय िलया I x
पाइप है | इन तीनों पाइपों को एक ही का मान है :
Q27. A pipe can fill a tank in 18 समय पर खोल िदया गया और एक SSC MTS 21 August 2019
Pi

hours. In how much time (in घंटे म उस टक का छठवां भाग एक (Evening)


hours) will another pipe operating घंटे म खाली हो गया | C अकेले टक (a) 21
at one-third the efficiency of the को िकतने समय म भर सकता है ? (b) 30
first pipe fill a tank whose SSC MTS 20 August 2019 (c) 26
capacity is 50% of that of the first (Afternoon) (d) 24
tank? (a) 25 hours
एक पाइप 18 घंटे म एक टक भर (b) 20 hours Q32.Pipes A and B can fill a tank
सकता है । िकतने समय म (घंटों म) (c) 30 hours in 24 hours and 30 hours,
पहले पाइप की एक ितहाई काय (d) 24 hours respectively, while pipe C can
कुशलता पर चलने वाला दू सरा पाइप emptyl it in x hours. A and B are
उस टक को भर दे गा िजसकी धा रता Q 30.An inlet pipe can fill a tank kept open for 10 hours at the
पहले टक की तुलना म 50% है ? in 2 hours and an outlet pipe can same time and then C is opened.
SSC MTS 19 August 2019 empty a fully filled tank in 3 If the tank become empty in 90
(Morning) hours, then what is the value of

www.ssccglpinnacle.com support@ssccglpinnacle.com Ph. 09729327755, 09817390373 418


/
Days 47-48 Pipes and Cistern

s
sse
la
_c
ob
x? / पाइप A और B मशः 24 घंटे Q1. Tap A can fill a tank in 6 CHSL 15-10-2020 (Morning
bo
ah
और 30 घंटे म िकसी टक को भर hours, tap B can fill the same task shift)
sm
ur
yo

सकते है , जबिक पाइप C इसे x घंटे in 8 hours and tap C can empty (a)15
e/
t.m

म खIली कर सकता है । A और B
://

the same tank in 4 hours. If all (b)18


tp
ht

को एक ही समय पर 10 घंटे क िलए three taps A, B and C are opened (c)20


खुला रखा जाता है और िफर C को together, then how much time (in (d)16
भी खोल िदया जाता है । यिद टक 90 hours) will be taken to fill the
घंटे म खIली होता है , तो x का मान tank? SSC CGL 2019 TIER-II
िकतना है ? टै प A 6 घंटे म एक टक भर सकता Q4. Pipe A and B fill a tank in
SSC MTS 22 August 2019 है , टै प B उसी टक को 8 घंटे म भर 43.2 minutes and 108 minutes,
(Morning) सकता है और C उसी टक को 4 घंटे respectively, Pipe C can empty it
(a) 15 म खाली कर सकता है । यिद सभी at 3 litres minutes. When all the
(b) 16 तीन टै प A, B और C को एक साथ three pipes are opened together,
(c) 18 खोला जाता है , तो टक को भरने म they will fill the tank in 54
(d) 20 िकतना समय (घंटों म) लगेगा? minutes. The capacity (in litres)

e
CHSL 13-10-2020 (Evening of the tank is:
SSC CGL TIER I shift) पाइप A और B मशः 43.2 िमनट
(a)24 और 108 िमनट म एक टक भरते ह,
Q1. Pipes A and B can fill a tank (b)30 पाइप C इसे 3 लीटर िमनट पर खाली
कर सकता है । जब तीनों पाइप एक

l
in 10 hours and 40 hours, (c)28
respectively. C is an outlet pipe (d)20 साथ खोले जाएं गे, तो वे 54 िमनट म

three
ac
attached to the tank. If all the
pipes are opened
simultaneously, it takes 80
minutes more time than what A
and B together take to fill the
Q2. Tap A can fill a tank in 20
hours and tap B can fill the same
tank in 30 hours. If both taps are
opened together, then how much
टक को भर दगे। टक की मता
(लीटर म) है :
CGL 2019 Tier-II (15/10/2020)
(a) 160
(b) 180
tank. A and B are kept open for 7 time will be taken to fill the tank? (c) 216
hours and then closed and C is टै प A 20 घंटे म एक टक भर सकता (d) 200
opened. C will now empty the है और टै प B 30 घंटे म एक ही टक
nn
tank in: / पाइप A तथा B िकसी भर सकता है । यिद दोनों नल एक Q5. Pipes A and B can fill a tank
टं की को मशः 10 घंटे तथा 40 घंटे साथ खोले जाते ह, तो टक को भरने in 12 minutes and 15 minutes,
म भर सकते ह | C एक िनकास म िकतना समय लगेगा? respectively. The tank when full
पाइप है जो टं की से जुड़ा आ है | CHSL 14-10-2020 (Morning can be emptied by pipe C in x
यिद सभी तीन पाइपों को एक साथ shift) minutes. When all the three pipes
चालू कर िदया जाए, तो टं की को (a) 10 hours are opened simultaneously, the
भरने म A और B के ारा एक साथ (b) 16 hours tank is full in 10 minutes. The
िलए गए समय की तुलना म 80 िमनट (c) 12 hours value of x is:
Pi

अिधक लगते ह | A और B को 7 घंटों (d) 24 hours पाइप A और B मश 12 िमनट


तक चालू छोड़ा जाता है तथा िफर और 15 िमनट म एक टक भर सकते
बंद करके पाइप C को चालू िकया Q3.Tap A can fill a tank in 6 ह। जब टक भरी हो तो पाइप C ारा
जाता है | C इस टं की को िकतने hours and tap B can empty the x िमनट म खाली िकया जा सकता है ।
समय म खाली करे गा ? same tank in 10 hours. If both जब तीनों पाइप एक साथ खोले जाते
SSC CGL 5 March 2020 taps are opened together, then ह, तो टक 10 िमनट म भर जाता है ।
(Morning) how much time (in hours) will be X का मान है
(a) 45.5 hours taken to fill the tank? CGL 2019 Tier-II (16/10/2020)
(b) 38.5 hours नल A, 6 घंटे म एक टक भर सकता (a) 18
(c) 42 hours है और नल B,10 घंटे म उसी टक (b) 15
(d) 49 hours को खाली कर सकता है । यिद दोनों (c) 20
नल एक साथ खोले जाते ह, तो टक (d) 24
SSC CHSL 2019 को भरने म िकतना समय (घंटों म)
लगेगा? SSC CPO 2019

www.ssccglpinnacle.com support@ssccglpinnacle.com Ph. 09729327755, 09817390373 419


/
Days 47-48 Pipes and Cistern

s
sse
la
_c
ob
Q6. Two pipes A and B can fill a और जब दोनों पाइप एक साथ (b) 3
bo
ah
tank in 15 hours and 18 hours, संचािलत होते ह, तो टं की 6 घंटे म (c) 5
sm
ur
yo

respectively. Both pipes are भर जाता है । संचािलत होने पर (d) 4


e/
t.m

िकतने घंटे (घंटों म) पाइप B, टं की


://

opened simultaneously to fill the


tp
ht

tank. In how many hours will the को भर दे गा। Q11. Pipes A,B and C can fill a
tank be filled? CPO 2019 23/11/2020(Evening) tank in 15, 30 and 40 hours
दो पाइप A और B मशः 15 घंटे (a) 9 respectively. Pipes A,B and C are
और 18 घंटे म एक टक भर सकते (b) 18 opened at 6 am 8 am and 10 am
ह। टक को भरने के िलए दोनों पाइप (c) 10 respectively on the same day,
एक साथ खोले जाते ह। िकतने घंटे म (d) 15 when will be tank be full?
टक भर जाएगा| पाइप A, B और C मशः 15, 30
CPO 2019 23/11/2020(morning) Q9. Two pipes can fill a tank in और 40 घंटे म एक टक भर सकते
2
(a) 7 11 15 hours and 4 hours, ह। पाइप A, B और C को उसी िदन
2
(b) 9 11 respectively , while a third pipe मशः सुबह 6 बजे सुबह 8 बजे और
2
(c) 10 11 can empty it in 12 hours. How सुबह 10 बजे खोला जाता है , जब
टक कब भर जाएगा

e
2 long (in hours) will it take to fill
(d) 8 11
the empty if all the three pipes CPO 2019 24/11/2020(Evening)
are opened simultaneously? (a) 3:20 p.m.
Q7. Pipes A and B can fill a tank
दो पाइप मशः 15 घंटे और 4 घंटे म (b) 5:20 p.m.
in 16 hours and 24 hours,
एक टक भर सकते ह, जबिक एक

l
(c) 7 :20 p.m.
respectively, whereas pipe C can
तीसरा पाइप 12 घंटे म इसे खाली (d) 11:20 p.m
ac
empty the full tank in 40 hours.
All three pipes are opened
together, but pipe A is closed
after 10 hours. After how many
hours will the remaining tank be
कर सकता है । यिद तीनों पाइप एक
साथ खोले जाते ह तो खाली टक को
भरने म िकतना समय लगेगा |
CPO 2019 23/11/2020(Evening)
(a) 15
7
Q12. Pipes A and B can fill a tank
in 16 hours and 24hours
respectively, whereas pipe C can
empty the tank in 40 hours. All
filled?
(b) 30 three pipes are opened together,
पाइप A और B मशः 16 घंटे और 7
(c) 20 but pipe C is closed after 10
24 घंटे म एक टक भर सकते ह, 7
hours. After how many hours will
nn
(d) 50
जबिक पाइप C, 40 घंटे म पूरा टक 7
the remaining tank be filled ?
खाली कर सकता है । सभी तीन पाइप
पाइप A और B मशः 16 घंटे और
एक साथ खोले जाते ह, लेिकन 10 Q10. Pipe A, B and C can fill an
24 घंटे म एक टक भर सकते ह,
घंटे के बाद पाइप A बंद कर िदया empty tank in 30
7 hours, if all the जबिक पाइप C टक को 40 घंटे म
जाता है । िकतने घंटे बाद शेष टक three pipes are opened खाली कर सकता है । सभी तीन पाइप
भर जाएगा? simultaneously A and B are एक साथ खोले जाते ह, लेिकन पाइप
CPO 2019 23/11/2020(morning) filling pipe and C is an emptying C 10 घंटे बाद बंद हो जाता है ।
(a) 15.5 pipe. Pipe a can fill the tank in 15 िकतने घंटे के बाद शेष भाग भर
Pi

(b) 12.5 hours and pipe C can empty it in जाएगा


(c) 20 12 hours. In how many hours can CPO 2019 25/11/2020(morning)
(d) 10 pipe B alone fill the empty tank ? (a) 5 21
पाइप A, B और C 30 7 घं टे म एक (b) 5
Q8. When operated separately , खाली टक भर सकते ह, यिद तीनों (c) 2
pipe A takes 5 hours less than पाइप एक साथ खुलते ह A और B (d) 2 21
pipe B to fill a cistern, and when पाइप भर रहे ह और C पाइप खाली
both pipes are operated together, कर रहा है । पाइप A टक को 15 घंटे
the cistern gets filled in 6 hours. Q13. Two pipes a and b can fill a
म भर सकता है और पाइप C टक
In how much time (in hours) will tank in 12 hours and 18 hours,
को 12 घंटे म खाली कर सकता है ।
pipe B fill the cistern, if operated. respectively. Both pipes are
पाइप B िकतने घंटे म अकेले खाली
पाइप A, जब अलग से संचािलत opened simultaneously In how
टक भर सकता है ?
िकया जाता है तो टं की को भरने के much time will the empty tank be
CPO 2019 24/11/2020(morning)
िलए पाइप B से 5 घंटे कम लेता है , filled completely
(a) 6

www.ssccglpinnacle.com support@ssccglpinnacle.com Ph. 09729327755, 09817390373 420


/
Days 47-48 Pipes and Cistern

s
sse
la
_c
ob
दो पाइप A और B मशः 12 घंटे (d) 50 hours
bo
ah
और 18 घंटे म एक टक भर सकते
sm
ur
yo

ह। दोनों पाइप एक साथ खोले जाते


e/
t.m

ह। खाली टक को पूरी तरह से भरने


://
tp
ht

म िकतना समय लगेगा?


CPO 2019 25/11/2020(morning)
(a) 9 hours 30 minutes
(b) 7 hours 12 minutes
(c) 6 hours
(d) 10 hours 24 minutes

Q14. A pump can fill a tank with


water in 1 hour, Because of a
leak, it took 1 31 hours to fill the
tank. In how many hours can the

e
leak alone drain all the water of
the tank when it is full ?
एक पंप एक टक को पानी से 1 घंटे
म भर सकता है , छे द के कारण, टक

l
को भरने म 1 31 घंटे लग गए। छे द
िकतने घंटे म टं की का सारा पानी
ac
खाली कर सकता है ?
CPO 2019 25/11/2020(Evening)
(a) 5
(b) 1
(c) 4
(d) 2
nn
Q15. Two pipes A and B can fill a
cistern in 12 21 hours and 25
hours, respectively. The pipes are
opened simultaneously and it is
found that due to a leakage it the
bottom, It took 1 hour and 40
minutes more to fill the cistern.
When the cistern is full, in how
Pi

much time will the leak empty the


cistern?
दो पाइप A और B मशः 12 21 घंटे
और 25 घंटे म एक टं की भर सकते
ह। पाइप एक साथ खोले जाते ह और
यह पाया जाता है िक एक नीचे िछ
के कारण, 1 घंटे और 40 िमनट
अिधक समय लगा। जब टं की भर
जाती है , तो टं की को िछ िकतने
समय म खाली कर दे गा?
CPO 2019 25/11/2020(Evening)
(a) 42 hours
(b) 48 hours
(c) 45 hours

www.ssccglpinnacle.com support@ssccglpinnacle.com Ph. 09729327755, 09817390373 421


/
Days 47-48 Pipes and Cistern

s
sse
la
_c
ob
Variety Questions
bo
ah
Sol 1. (d)
sm
ur
yo
e/
t.m
://
tp
ht

Sol 4. (a)

Tank filled in 2 hours( at 11am ) Efficiency of A=+15, B=+10 and


= 2 {10+(-8)} = 4 unit. C=-6
Time taken to empty it = 5+(4−8) = Therefore, Time to fill remaining
tank = 240−4152 = 88
4 = 22 hrs
1 hour 20 minutes
Required time = 11 am + 1 hour
20 min = 12:20 pm SSC CGL TIER II

e
Let the total capacity of tank = 15
Sol 2. (d) unit Sol 1. (c)
Let total capacity of tank = 24 Desired time = 5153 =4 72 hours Let total capacity of tank = 120
−2
unit unit

l
Sol 5. (c) Let the total volume of
ac empty tank be 24 units.

Quantity filled by pipe A till


nn
Quantity filled in 3 minutes = 3 x 10:00 AM = 4 x 3 = 12 unit
(3+2) = 15 unit Quantity filled by pipe B till
Time taken to empty this quantity 10:00 AM = 3 x 2 = 6 unit
= 15 3
4 = 3 4 hours
Efficiency of A=3 and B=2 Remaining quantity = 120-12-6 =
Part filled in one cycle (2 hrs) 102 unit
Sol 3. (a) =3+2 = 5 units Time taken to fill this quantity =
Part filled in 4 cycle (8 hrs) =4 x 102 1
Let total capacity of tank = 20 4+3+2 = 11 3 hours
5 = 20 units
Pi

unit ⇒ Tank will be filled at 10:00


In 9th hr quantity filled = 3 unit AM + 11 31 hr = 9:20 PM
Quantity remaining = 24-20-3 = 1
unit
Sol 2. (d)
Time taken to fill this quantity =
1
2 hour
Therefore,to fill it completely 9 21
hour time is required.

Sol 6. (b) Let total capacity of


Time taken to fill the tank = 20
2−1 tank be 240 units
= 20 hours
Desired time = 6:30 a.m. + 20
hours = 2:30 a.m. next day

www.ssccglpinnacle.com support@ssccglpinnacle.com Ph. 09729327755, 09817390373 422


/
Days 47-48 Pipes and Cistern

s
sse
la
_c
ob
Let total capacity of tank = 48 Therefore, In 17 43 hrs the entire
bo
ah
unit
sm
tank will be filled.
ur
yo

Quantity filled by pipe A and B


e/
t.m
://

(from 10:30 a.m. to 8:30 pm)=10


tp
ht

x (3+2)=50 unit
Quantity drained by pipe c (from
10:30 a.m. to 2:30 pm) = 50-48 =
2 unit
Efficiency of pipe C = 42 = 21 unit Sol 4. (d) Let the capacity of tank
Time taken by pipe C to drain the be 18 units.
tank = 148/2 = 96 hours
Let the total capacity = 20 unit
Efficiency of B = 8-5 = 3 unit
Number of days taken by B = 20 3
= 6 32 hours

e
Sol 2. (a) Let the total capacity of
Sol 3. (b) the tank be 24 units

l
ac Quantity filled in one cycle ( 2
hrs) = 3+2 = 5units
Quantity filled in 3 cycle (6 hrs) =
5 x 3 = 15 units
Quantity filled in 7th hour = 3
unit
Therefore, total time taken to fill
Efficiency of A=+4, B=+3 and the tank = 6+1 = 7 hours.
Efficiency of pipe C = 3-5-4 = -6
nn
C=-2
unit
After 3 hrs, quantity remaining = Sol 5. (d) Let the capacity of tank
Note : Negative sign indicates
24 - (4+3-2)x3 = 24 - 15 = 9 be 36 units.
that it is an outlet pipe.
Therefore, Time to fill remaining
Quantity filled by pipe A and B in
tank
12 minutes = 12 x (5+4) = 108
= 3−92 = 9 hours
unit
Time taken by pipe C to drain it =
108 = 18 minutes Sol 3. (d) Let the total capacity of
Pi

6
the tank be 80 units.

Practice Questions

Sol 1. (c)

Efficiency of A=6, B=4 and C=-3


Therefore, Required time to fill
35
36× 100
35% of the tank = (6+4−3) = 1.8 hr

Sol 6. (c) Let the capacity of the


Efficiency of A=5 and B=4 tank be 80 units.
In 2 hours 9units is filled.
In 16hours 72 units will be filled.

www.ssccglpinnacle.com support@ssccglpinnacle.com Ph. 09729327755, 09817390373 423


/
Days 47-48 Pipes and Cistern

s
sse
la
_c
ob
bo
ah
sm
ur
yo
e/
t.m
://
tp
ht

Efficiency of A = 8, B = 5 Let total capacity of tank = 12


Let the total capacity of tank =
Quantity filled in one cycle ( 2 unit
12 = 4 hours
180 unit
hrs) = 13 units Desired time = 1+2
Total efficiency of A and B = 4
Quantity filled in 6 cycle ( 12 hrs)
Efficiency of A = 3
= 13x6 = 78 units Sol 9. (d) Efficiency of B = 4-3 = 1
Quantity to be filled in 13th hour
Time taken by B = 180 1 = 180

e
= 80-78 = 2 unit
minutes or 3 hours
Time taken to fill this quantity =
2 1
8 = 4 hour
Sol 12. (a)
Total time taken = 12+ 41 = 12 41 hr

l
Sol 7. (d) Let the capacity of tank
ac
be 96 units.

Let total capacity of tank = 40


unit
1
×40
Desired time = 2
8−5 = 6 32 hr

Sol 10. (a) Let total capacity of the tank =


nn
450 unit
Desired time = 15450
−5−6
= 450
4 min
or 1 87 hr
Efficiency of A=+8, B=+6 and
C=-4 Sol 13. (d)
In 4 hours 4x(8+6-4) = 40 units
filled.
Pi

Time taken to fill remaining tank


Let both the pipes were opened
by B and C = 96−2 40 = 28 hours
for x minutes
Total time taken = 28+4 = 32 48 = (3-2)x + 3(30-x)
hours 48 = x + 90 -3x
2x = 42
Sol 8. (c) x = 21
Time taken by A to fill ¼ th of
tank = 3 hr Sol 11. (b)
Time taken by A to fill complete
tank =3 x 4 =12 hr Let the total capacity of cistern =
Time taken by B to fill ⅓ rd of 360 unit
1
×360
tank = 2 hr Required time = 2
5+4 = 20
Time taken by B to fill complete minutes.
tank =2 x 3 =6 hr

www.ssccglpinnacle.com support@ssccglpinnacle.com Ph. 09729327755, 09817390373 424


/
Days 47-48 Pipes and Cistern

s
sse
la
_c
ob
Sol 14. (d) Let the total capacity of tank =
bo
ah 150 unit
sm
ur
yo
e/
t.m
://
tp
ht

Total capacity = 24 unit


Let total capacity of the tank =
Time taken to fill 1/3rd or 8 unit
360 unit
= 6−84 = 4 hours 360 16
Desired time = 24+15+10 −6
= 8 43
Tank to be filled = 24-8 = 16 Quantity filled in one cycle (2
Time taken = 16 6 = 3
8
days ) = 5-3 = 2 unit
Sol 18. (c)
Total time = 8 +4 = 20 hours
3 3 Let the capacity of tank = 24 unit 146 unit will be filled in 146
2 = 73
cycles of 146 days
Sol 15. (a)

e
Quantity to be filled till 147th day
= 150-146 = 4 unit
Time taken to fill this quantity =
4
5 day

l
⇒ the reservoir will get
ac completely filled on 147th day.

Sol 21. (a)


Efficiency of A+B > Efficiency
Total capacity of the tank = 165
of A+B+C
⇒ C is an outlet pipe.
Let total capacity of the tank = 96
Efficiency of C = 4+3-2 = 5 unit
unit
Time taken by C to empty the
Desired time = 396−2
= 96 minutes
tank = 24
5 =4 5
4
nn
or 1hr 36 min.
Clearly option ‘c’ is the right
answer.
Sol 16. (b)

Sol 19. (d)


Time taken to fill the tank =
Let the total capacity of tank = 91 165 1
unit 15−11 = 41 4 hours or 41 hours 15
minutes
Pi

Sol 22. (b)


Total capacity of the tank = 800
unit
Let total time taken be 5x.
ATQ, 3x+(5 × 4x) =60
23x=60
Therefore, Total time, 5x =
300 = 13 1
23 23

Time taken to fill the tank = 91


Sol 17. (c) 13+7
11 hours or 4 hours 33
= 4 20
minutes
Required time = 800 = 400
10−8
Sol 20. (b) minutes

www.ssccglpinnacle.com support@ssccglpinnacle.com Ph. 09729327755, 09817390373 425


/
Days 47-48 Pipes and Cistern

s
sse
la
_c
ob
Let the total capacity of tank = Required time = 420 = 210 hours
bo
2
ah
Sol 23. (d) 240 unit
sm
ur
yo

Let the total capacity of tank = 48 Sol 29. (d)


e/
t.m
://

unit Let the quantity to be filled = 24


tp
ht

unit

Quantity filled till 10 AM = 2 x


48 48
(24+20-15) = 58 unit
Required time = = hours 58 = 3
6+3−4 5 Time taken to empty this = 15
13 hours or 3 hours and 52
15

e
Sol 24. (b) Quantity drained = 24 x 1 =4
minutes 6
Let the total capacity of tank = unit
Total time taken = 2 hours + 3
180 unit Total efficiency of A, B and C =
hours and 52 minutes
4
= 5 hours and 52 1 = 4 unit

l
minutes ⇒ Efficiency of C = 5-4 = 1 unit
ac Required time = 24 = 24 hours
1
Sol 27. (d)
Let the efficiency of the pipe = 3 Sol 30. (d)
unit Let the total capacity of tank = 6
⇒ Total capacity of tank = 18 x 3 unit
= 54 unit
Quantity to be filled = 21 x 54 =
Quantity to be filled = 1 x 180 = 27 unit
nn
2
90 unit Efficiency of new pipe = 31 x 3 = 1
Time taken to fill this = 90 = 90 unit
5−4
Required time = 27 = 27 hours
minutes or 1.5 hours 1

Sol 25. (a) Sol 28. (b)


Let the total capacity of tank = 60 Let the capacity of tank = 504 6
Required time = 3−2 = 6 hours
unit unit
Pi

Sol 31. (b)


Let the capacity of tank = 60 unit

Required time = 60 = 12 Quantity leaked per hour = 9-7 =


3+6−4
minutes 2 unit
83 31 % = 65 Quantity drained = 60 x 1
18 = 10
3
Quantity to be drained = 504 x 5
Sol 26. (a) 6 unit
= 420 unit

www.ssccglpinnacle.com support@ssccglpinnacle.com Ph. 09729327755, 09817390373 426


/
Days 47-48 Pipes and Cistern

s
sse
la
_c
ob
Total efficiency of A, B and C = ⇒ Efficiency of C = 1 unit Overall efficiency = 60 =6
bo
10
ah
10/3
sm
40/60 = 5 unit Total capacity of tank = 56 units Efficiency of c = 5 + 4 - 6 = 3
ur
yo

In 7 hours, A and B fills = 7 × 7 Time taken by C = 60 3 = 20


e/

⇒ Efficiency of C = 7-5 = 2 unit


t.m
://

= 49 units
tp

So, Time taken by C to fill the


ht

tank = x = 60 Time taken by C to empty 49


2 = 30 hours SSC CPO 2019
units = 49 hours Sol:6..(d)
Sol 32. (a) Time taken by pipe A to Fill the
SSC CHSL 2019 tank is 15 hours
Sol:1.(a) Time taken by pipe B to Fill the
LCM of 6,8,4 = 24 units = Total Tank is 18 hours
work Let Total work be 90 units
Efficiency of A = 4 90 = 6units
Efficiency of Pipe A= 15
Efficiency of B = 3 per hour
Efficiency of C = -6 90 =5 units
Efficiency of Pipe B = 18
24 = 24 hours
Total time= 4+3 −6 per hour

e
Combined efficiency of A and B
Quantity filled in 10 hours = Sol:2. (c) is 11 units per hour
(5+4) x 10 = 90 unit Let the total work = 60 unit Talk will be filled by A and B
Total efficiency of A, B and C = (LCM of 20 and 30) combined in 11 90 units per hour= 8

l
90 60 = 3
Efficiency of A = 20
90 = 1 unit 2
11
Efficiency of B = 60 =2
unit
ac
So, efficiency of C = 5+4-1 = 8

⇒ Time taken by C to empty the


tank = x = 120
8 = 15 hours
Time taken by both =
hours

Sol:3. (a)
30
60
2+3 = 12 Sol:7..(b)

Let the total work = 30 unit


SSC CGL TIER I (LCM of 6 and 10)
Efficiency of A = 30/6 = 5
nn
Sol 1. (d) Pipe A can fill a tank in Efficiency of B = 30/10 = -3
10 hours Time taken by both tap = 302 = 15
Pipe B can fill a tank in 40 hours hour Total work done by A,B,C in 10
Total capacity = l cm (10,40) = 40 hours is (15+10-6) is 19 units
units SSC CGL 2019 TIER-II Total work done in 10 hours is
In 1 hour, Pipe A fills = 4 units Sol:4.(c) 190 units
and pipe B fills = 1 unit LCM of 43.2,108 and 54 = 432 Remaining work to be done is 50
40
A and B together fills tank in 4+1 Then let the capacity of the tank = units.which will be done by B and
Pi

432 x C
= 8 hour
Efficiency of A = 10x So total time to complete the
Together A, B and C fill the tank
80 = 28 hours Efficiency of B = 4x remaining work will be 504 =12.5
in 8+ 60 3
Efficiency of A, B and C hours
combined = 8x
Efficiency of C = 6X Sol:8..(d)
C can empty the tank in = Let time taken by A be x hours
432 = 72 min According to the question
6
x(x + 5)
Capacity of the tank = 72 × 3 = 2x + 5 =6
2
216 l x + 5x = 12x + 30
x2 - 7x + 30 = 0
Sol:5.(c) (x - 10)(x + 3)= 0
LCM of 10,12,15 is = 60 x = 10 hours
Efficiency of A and B = 5 : 4

www.ssccglpinnacle.com support@ssccglpinnacle.com Ph. 09729327755, 09817390373 427


/
Days 47-48 Pipes and Cistern

s
sse
la
_c
ob
Time taken by B = x + 5 = 15 Efficiency of Pipe A= 240 = Sol:15.(d)
bo
16
ah
hours
sm
15units per hour
ur
yo

Efficiency of Pipe B = 240 =10


e/
t.m

24
://

Sol:9.(b)
tp

units per hour


ht

Efficiency of Pipe C = 240 =6


40
units per hour Without leakage tank filling time
Combined efficiency of A , B and = 25
3 = 8 hr 20 min
C is 19(15+10-6) units per hour
With leakage = 8 hr + 20 min = 1
Tank will be filled by all the three
hr + 40 min = 10 hr
pipes in 10 hours is equal to 25 =
Efficiency with leakage = 10
15+10-6=19 units × 10=190 units
2.5
Work left to be done is 50 units
Total efficiency = 4 + 15 -5 = 14 Efficiency of leakage = 0.5
Combined efficiency of A And B
Time required = 1460 = 30 Time required to empty the tank =
7 =25 units 25
Sol:10..(d) Time taken by A and B to fill 0.5 = 50 hr

e
remaining tank=50 ÷ 25=2 hours

Sol:13.(b)
Time taken by pipe A to Fill the

l
tank is 12 hours
ac
Efficiency of B = 14 - 4 + 5 = 15
Time taken by B = 15
Sol:11..(a)
60 = 4
Time taken by pipe B to Fill the
Tank is 18 hours
Let Total work be 36 units
(L.C.M. of 12,18)
36 = 3units
Efficiency of Pipe A= 12
per hour
36 =2 units
Efficiency of Pipe B = 18
per hour
nn
A worked alone for 4 hrs
B worked alone for 2 hrs When both pipes are opened
Total work done by A and B= 8 × simultaneously then total units of
work done by them in 1 hour=
4+4 × 2=40
2+3=5 units
Work left to be done by A,B and
Total time required to complete
C=80units
36 units of work= 365
Time taken for the work to be
80 = 80 = 16 = 5 hrs
completed= 8+4+3 = 7 51 hours = 7 hours 12 minutes
15 3
Pi

20 minutes
The tank will be fill in 10 am Sol:14.(c)
+5hrs 20 min= 3:20 pm Pump can fill theTank in 1 hours
Due to leakage time taken = 1 31 =
4
Sol:12.(c) 3
Time taken by pipe A to Fill the Let capacity of tank be 4 units
tank is 16 hours Efficiency of the pump without
Time taken by pipe B to Fill the leak=3
Tank is 24 hours Efficiency of the pump with
Time taken by pipe C to empty leak=2
the Tank is 40 hours Efficiency of leak= 4-3=1
Let Total work be 240 units Time taken by leak to empty the
(L.C.M. of 16,24,40) tank= 14 =4 hours

www.ssccglpinnacle.com support@ssccglpinnacle.com Ph. 09729327755, 09817390373 428


/
Days 49-52 Speed and Distance

s
sse
la
_c
ob
SPEED AND DISTANCE / 5). TRAINS/ रे लगाड़ी
bo
ah
sm
चाल और दु री 3). Relative Speed/ सापे गित Case 1: Train crossing a person
ur
yo

or, pole./ रे ल िकसी या पोल


e/

i. Same Direction -: When


t.m
://

को पार करे |
tp

two things are moving in


ht

KEY- POINTS/ मुख िबंदु


the same direction, the In such case the length of
relative speed is given by the person or pole is taken to be
1). T ime = Distance
Speed the difference between negligible. To cross the person the
We just need to remember this their respective speeds. train has to cover distance equal
formula. If two values are given जब दो चीजे एक िदशा म to its length.
the third can be found easily. चलती है तो उनकी सापे इस थित म या पोल की
हमे ये सू याद रखना चािहए| अगर गित एक दू सरे की गित के ल ाई को नग मान िलया जाता है |
दो मू िदए गए हो तो हम तीसरा अंतर से िदया जाता है | को पार करने के िलए रे ल को
िनकाल सकते है | Ex. A and B are going to अपनी ल ाई
Ex. A man travels from Delhi to a same point Z at speeds िजतनी दू री तय करनी पड़े गी
Chandigarh at a speed of 50 km/h of 80 km/h and 60 km/h Ex. A train, of length 100 m,

e
covering a distance of 300 km. respectively. travelling at 90 km/hr will take
Find the time taken? Relative speed of A with how much time to cross a man?
एक िद ी से चंडीगढ़ की respect to B will be = 80 Here, Speed = 90 km/hr
300 km की दु री को 50 km/hr की – 60 = 20 km/h. 5 m = 25 m
= 90 × 18

l
s s
गित से पूरा करता है | उसे िकतना Distance = Length of train =
समय लगा?

km/h.
∴T ime (T ) =
ac
Here, we are given, Distance(D)
= 300 km and Speed(S) = 50

D
S = 300
50 = 6 hours
ii. Opposite direction :-
When two things are
moving in opposite
directions, the relative
speed is given by the
100 m
Time = Distance / Speed
= 100
25 = 4 seconds

addition of their Case 2: Train crossing a


respective speeds. platform./ रे ल िकसी ेटफाम को
T otal Distance
2). Average Speed = T otal T ime पार करे |
जब दो चीजे िवपरीत िदशा
nn
Ex. A person travels from point A म चलती है तो उनकी सापे In such case the distance covered
to point B at a speed of 30 km/h गित एक दू सरे की गित के by train to cross the platform will
and comes back at a speed of 20 योग से िदया जाता है | be equal to sum of length of train
km/h. What is his average speed? and platform.
Ex. A is moving towards
एक पॉइं ट A से पॉइं ट B तक point Z at 60 km/h and B Ex. A train, of length 100 m,
30 km/hr की गित से जाता है और is going away from point travelling at 90 km/hr will take
20 km/hr की गित से वापस आता है | Z at 80 km/h. how much time to cross a
उसकी औसत गित ा है ? platform of length 150 m?
Pi

Relative speed of A with


Let’s take the distance between A respect to B = 60 + 80 = Here, Speed = 90 km/hr
and B be 60 km. (LCM of 30 and 5 m = 25
= 90 × 18 m
140 km/h. s s
20.) Distance = Length of train +
4). Unit Conversion:
Time taken to travel from A to B km Length of platform = 100 + 150 =
1 = 1× 1000 m
3600 s =
5 m
= Distance 60
Speed = 30 = 2 hours
hr 18 s
250 m
So, to convert km/hr to m/sec
Time taken to travel from B to A 5 Time =
just multiply by 18 and to Distance 250
= Distance 60
Speed = 20 = 3 hours Speed = 25 = 10 seconds
convert m/sec to km/hr multiply
Total distance covered = 60 + 60 by 18
5 .
= 120 km km = 90 × 5 m = 25 m Case 3: Train crossing another
Ex. 90 hr 18 s s
Total time taken = 2 + 3 = 5 hours train moving in same direction./
Also, 25 ms = 25 × 18 km = 90 km
5 hr hr रे ल िकसी दू सरी रे ल को पार करे जो
∴ Average Speed =
120 = 24 km/h उसी िदशा म चल रही हो|
5

www.ssccglpinnacle.com support@ssccglpinnacle.com Ph. 09729327755, 09817390373 429


Days 49-52 Speed and Distance

s
sse
la
_c
ob
In such case the distance covered Time = Ex. A train, of length 100 m,
bo
ah
by train to cross the other train Distance = 150 = 30 seconds travelling at 90 km/hr will take
sm
Speed 35 7
ur
yo

will be equal to sum of lengths of how much time to cross a man


e/
t.m
://

the two trains. To calculate time sitting in another train, of length


tp

Case 5: Train crossing a man


ht

we need to consider relative 50 m, moving in opposite


sitting in another train moving
speed of one train with respect to direction at speed of 54 km/hr?
in same direction./ रे ल िकसी
the other. Speed of the man = 54 km/hr
को पार करे जो िकसी दू सरी
Ex. A train, of length 100 m, रे ल म बैठा हो जो उसी िदशा म चल Here, Relative speed of first train
travelling at 90 km/hr will take रही हो| with respect to the man = (90 +
how much time to cross another 54) km 5 m m
hr = 144 × 18 s = 40 s
train, of length 50 m, moving in
In such case the distance covered Distance = Length of first train =
same direction at speed of 36
by train to cross the man will be 100 m
km/hr?
equal to length of the train. To Time =
Here, Relative speed of first train Distance 100 5
calculate time we need to Speed = 40 = 3 seconds
with respect to second train =
consider relative speed of train

e
(90 – 36) with respect to the man. Length
km 5 m m Variety Questions
hr = 54 × 18 s = 15 s of the second train will not be Q1. A truck covers a distance of
Distance = Length of first train + considered. 384 km at a certain speed. If the
Length of second train = 100 + 50

l
Ex. A train, of length 100 m, speed is decreased by 16 km/h, it
= 150 m travelling at 90 km/hr will take will take 2 hours more to cover
Time
= 10 seconds
ac =

Case 4: Train crossing another


train moving in opposite
Distance
Speed = 150
15 how much time to cross a man
sitting in another train, of length
50 m, moving in same direction at
speed of 36 km/hr?
Speed of the man = 36 km/hr
the same distance. 75% of its
original speed (in km/h) is :
एक टक 384 िकमी की दू री एक
िनि त चाल से तय करता है | यिद
चाल 16 िकमी/घंटा से कम कर दी
direction./ रे ल िकसी दू सरी रे ल को Here, Relative speed of first train जाए, तो इसे इसी दू री को तय करने
पार करे जो िवपरीत िदशा म चल रही with respect to the man = (90 – म दो घंटे अिधक लगगे | इसकी
आरं िभक चाल (िकमी/घंटा म ) का
nn
36) km 5 m m
हो| hr = 54 × 18 s = 15 s
75% है :
In such case the distance covered Distance = Length of first train =
SSC CGL 4 June 2019
by train to cross the other train 100 m
(Morning)
will be equal to sum of lengths of Time =
(a) 45
the two trains. To calculate time Distance = 100 = 20 seconds
Speed 15 3 (b) 54
we need to consider relative
(c) 48
speed of one train with respect to
Case 6: Train crossing a man (d) 42
the other.
sitting in another train moving
Pi

Ex. A train, of length 100 m,


in opposite direction./ रे ल िकसी Q 2. A takes 30 minutes more
travelling at 90 km/hr will take
को पार करे जो िकसी दू सरी than B to cover a distance of 15
how much time to cross another
रे ल म बैठा हो जो िवपरीत िदशा म km at a certain speed. But if A
train, of length 50 m, moving in
चल रही हो| doubles his speed, he takes one
opposite direction at speed of 36 hour less than B to cover the
km/hr? same distance. What is the speed
In such case the distance covered
Here, Relative speed of first train (in km/h) of B?
by train to cross the man will be
with respect to second train = (90 A को एक िनि त चाल से 15 िकमी
5 m m equal to length of the train. To
+ 36) km
hr = 126 × 18 s = 35 s की दू री तय करने म B से 30 िमनट
calculate time we need to
Distance = Length of first train + अिधक लगते ह | लेिकन यिद A
consider relative speed of train
Length of second train = 100 + 50 अपनी चाल दोगुनी कर ले, तो उसे
with respect to the man. Length
= 150 m इसी दू री को तय करने म B से एक
of the second train will not be
घंटा कम लगेगा | B की चाल ( िकमी/
considered.
घंटा म ) ात कर |

www.ssccglpinnacle.com support@ssccglpinnacle.com Ph. 09729327755, 09817390373 430


Days 49-52 Speed and Distance

s
se
s
la
_c
ob
SSC CGL 4 June 2019 (b) 320 does the train stop on an average
bo
ah
(Afternoon) (c) 300 per hour?
sm
ur
yo

(a) 6 (d) 360 िबना के ए कोई टे न 50 िकमी/घंटा


e/
t.m

की औसत चाल से चलती है तथा


://

(b) 5
tp
ht

(c) 6 21 Q5. A train travelling at 44 km/h क- क कर यह 40 िकमी/घंटा की


(d) 5 21 crosses a man walking with a औसत चाल से चलती है | हर घंटे यह
speed of 8 km/h, in the same टे न औसतन िकतने िमनट कती है
Q3. A and B are travelling towards direction, in 15 seconds. If the ?
each other from the points P and Q train crosses a woman coming SSC CGL 10 June 2019
respectively. After crossing each from the opposite direction in 10 (Afternoon)
other A and B take 6 81 hours and seconds, then what is the speed ( (a) 12
In km/h ) of the woman? (b) 13
8 hours, respectively, to reach their
44 िकमी/घंटा की चाल से चल रही (c) 14
destinations Q and P, respectively.
कोई टे न उसी िदशा म 8 िकमी/घंटा (d) 15
If the speed of B is 16.8 km/h then
की चाल से चल रहे एक को
the speed ( in km/h) of A is:
15 सेकंड म पार करती है | यिद यह

e
Q8. Walking at 3/4 of his usual
A और B मशः िबंदु P और Q से
टे न सामने से आ रही एक मिहला को speed, a person reaches his office
एक-दू सरे की तरफ आ रहे ह |
10 सेकंड म पार करती है , तो मिहला 18 minutes later than the usual
एक-दू सरे को पार करने के बाद, A
की चाल ( िकमी/घंटा म ) ात कर | time. His usual time in minutes is:
और B को अपने-अपने गंत ों अथात
अपनी सामा चाल की तुलना म ¾

l
SSC CGL 6 June 2019
Q और P तक प ँ चने म मशः 6 81
(Evening) चाल से चलते ए एक
घंटे और 8 घंटे लगते ह | यिद B की
ac (a) 10.5 सामा समय से 18 िमनट दे र से
चाल 16.8 िकमी/घंटा है , तो A की
(b) 10 कायालय प ँ चता है | उसका सामा
चाल ( िकमी/घंटा म ) ात कर |
(c) 9 समय ( िमनट म ) ा है ?
SSC CGL 6 June 2019
(d) 8.5 SSC CGL 12 June 2019
(Morning)
(Afternoon)
(a) 20.8
Q6. The speed of train A is 25 (a) 60
(b) 19.8
km/h more than the speed of train (b) 54
(c) 19.2
B. A takes 4 hours less time to (c) 72
nn
(d) 20.4
travel a distance of 300 km than (d) 45
what train B takes to travel 250
Q4. The distance between two
km. What is the speed (In km/h) Q9. A starts walking at 4 kmph
stations A and B is 800 km. A
of A? and after 4 hours, B starts cycling
train X starts from point A and
टे न A की चाल टे न B की चाल से 25 from the same point as that of A,
moves towards point B at a speed
िकमी/घंटा अिधक है | A को 300 in the same direction at 10 kmph.
of 40 km/h and another train Y
िकमी की दू री तय करने म B ारा After how much distance from
starts from point B and moves
250 िकमी की दू री तय करने म िलए the starting point will B catch up
Pi

towards A at 60 km/h. How far


गए समय से 4 घंटे कम लगते ह| A with A (correct to two decimal
from A will they cross each
की चाल ( िकमी/घंटा म ) ात कर | places)?
other?
SSC CGL 10 June 2019 A, 4 िकमी/घंटा की चाल से चलना
दो े शनों A और B के बीच की दू री
(Morning) शु करता है और 4 घंटे के बाद, B,
800 िकमी है | एक टे न X िबंदु A से
(a) 60 A के ही आरं िभक िबंदु से उसी िदशा
चलती है और B की तरफ 40
(b) 50 म 10 िकमी/घंटा की चाल से
िकमी/घंटा की चाल से जाती है तथा
(c) 65 साइिकल चलाता है | आरं िभक िबंदु
दू सरी टे न Y िबंदु B से A की ओर 60
(d) 55 से िकतनी दू री ( दो दशमलव थान
िकमी/घंटा की चाल से चलना शु
तक सही ) पर B, A को पकड़ लेगा
करती है | A से िकतनी दू री पर वे
Q7. A train without stoppage ?
एक-दू सरे को पार करगी ?
travel with an average speed of SSC CGL 13 June 2019
SSC CGL 6 June 2019
50 km/h and with stoppage, it (Evening)
(Afternoon)
travels with an average speed of (a)24.67 km
(a) 380
40 km/h. For how many minutes (b)26.67 km

www.ssccglpinnacle.com support@ssccglpinnacle.com Ph. 09729327755, 09817390373 431


Days 49-52 Speed and Distance

s
sse
la
_c
ob
(c)25.67 km अिमत ने A से B तक 80 िकमी/घंटा
bo
ah
(d)23.67 km की औसत चाल से या ा की | उसने Q15. The ratio between the
sm
ur
yo

पहली 75% दू री दो-ितहाई समय म speeds of two trains is 2 : 5. If the


e/
t.m

तथा शेष दू री x िकमी/घंटा की चाल


://

Q10. A man travels a certain first train covers 350 km in 5


tp
ht

distance at 12km/h and returns to से तय की | x का मान ा है ? hours, then the speed (in km/h) of
the starting point at 9km/h. The SSC CHSL 2 July 2019 the second train is:
total time taken by him for the (Afternoon) दो टे नों की चाल म 2 : 5 का अनुपात
entire journey is 2 31 hours. The (a)56 है | यिद पहली टे न 350 िकमी की
total distance (In km) covered by (b)60 दू री 5 घंटे म तय करती है , तो दू सरी
him is: / एक कोई िनि त (c)64 टे न की चाल ( िकमी/घंटा म ) ात
दू री 12 िकमी/घंटा की चाल से तय (d)54 कर |
करता है और आरं िभक िबंदु पर 9 SSC CHSL 8 July 2019
िकमी/घंटा की चाल से लौटता है | पूरी Q13. A 360 m long running at a (Morning)
या ा म उसे कुल 2 31 घंटे का समय uniform speed, crosses a platform (a)175
लगता है | उसके ारा तय की गयी in 55 seconds and a man standing (b)150

e
कुल दू री ात कर | on the platform in 24 seconds. (c)180
SSC CHSL 1 July 2019 What is the length (in metre) of (d)165
(Evening) the platform?
(a) 25 360 मी लंबी एक टे न एक समान Q16. Given that the lengths of the
चाल से चलते ए िकसी ेटफ़ॉम को

l
(b) 12 paths of a ball thrown with
(c) 24 55 सेकंड म तथा ेटफ़ॉम पर खड़े different speeds by two boys are
(d) 28
ac
Q11. Two trains of the same
length are running on parallel
tracks in the same direction at 54
एक
करती है |
को 24 सेकंड म पार
ेटफ़ॉम की लंबाई (
मीटर म ) िकतनी है ?
SSC CHSL 3 July 2019
(Morning)
the same, if they take 0.6 seconds
and 1 second respectively to
cover the said length, what is the
average speed of travel for the
first throw, if the same for the
km/h and 42 km/h respectively. (a) 480 second is 96 km/h?
The faster train passes the other (b) 445 िदया गया है िक दो लड़कों के ारा
(c) 410 अलग-अलग चाल से फकी गयी गदों
nn
train in 63 seconds. What is the
length (In metres) of each train? (d) 465 के माग की लंबाई समान है | यिद
समान लंबाई की दो टे न समानां तर उ इस लंबाई तक जाने म मशः
पट रयों पर एक ही िदशा म मशः Q14.Two trains of equal length 0.6 सेकंड और 1 सेकंड लगते ह, तो
54 िकमी/घंटा और 42 िकमी/घंटा की travelling in opposite directions at पहली ो म गद की औसत चाल ात
चाल से चल रही ह | तेज़ टे न दू सरी 72 km/h and 108 km/h cross each कर, यिद इसी के िलए दू सरी गद की
टे न को 63 सेकंड म पीछे छोड़ दे ती other in 10 seconds. In how much औसत चाल 96 िकमी/घंटा है | SSC
है | ेक टे न की लंबाई ( मीटर म ) time (in seconds) does the first CHSL 10 July 2019 (Afternoon)
ात कर | train cross a platform of length (a)100 km/h
Pi

SSC CHSL 2 July 2019 350 m? (b)150 km/h


(Morning) िवपरीत िदशाओं म मशः 72 (c)160 km/h
(a) 90 िकमी/घंटा और 108 िकमी/घंटा की (d)200 km/h
(b) 81 चाल से चल रही बराबर लंबाई की दो
(c) 105 टे न एक-दू सरे को 10 सेकंड म पार Q17.The platform of a station 400
(d) 210 करती ह | पहली टे न 350 मी लंबे m long starts exactly where the
ेटफ़ॉम को िकतने समय ( सेकंड last span of a bridge 1.2 km long
Q12.Amit travelled from A to B म ) पार करे गी ? ends. How long will a train 200 m
at an average speed of 80 km/h. SSC CHSL 3 July 2019 long and travelling at the speed of
He travelled the first 75% of the (Evening) 72 km/h take to cover the
distance in two-third of the time (a)30 distance between the starting
and the rest at a constant speed of (b)32 point of the span of the bridge
x km/h. The value of x is: (c)36 and the far end of the platform?
(d)24

www.ssccglpinnacle.com support@ssccglpinnacle.com Ph. 09729327755, 09817390373 432


Days 49-52 Speed and Distance

s
sse
la
_c
ob
िकसी े शन का 400 मीटर लंबा औसत चाल ( िकमी/घंटा म ) ात
bo
ahेटफ़ॉम ठीक वहीं से शु होता है कर | SSC CHSL 11 July 2019 Q2. A train travelling at the speed
sm
ur
yo

जहाँ 1.2 िकमी लंबे पुल का अंितम (Evening) of x km/h crossed a 200 m long
e/
t.m

पाट समा होता है | 72 िकमी/घंटा


://

(a) 500 platform in 30 seconds and


tp

11
ht

की चाल से चल रही 200 मीटर लंबी (b) 600 overtook a man walking in the
13
एक टे न को पुल के पाट के आरं िभक (c) 500 same direction at the speed of 6
13
िबंदु तथा ेटफ़ॉम के अंितम िबंदु 600 km/h in 20 seconds. What is the
(d)
तक जाने म िकतना समय लगेगा ? 11
value of x ?
SSC CHSL 11 July 2019 x िकमी/घंटा की चाल से चलती ई
Q20. The speed of a car increases
(Morning) कोई टे न 200 मीटर लंबे ेटफ़ॉम
by 2km/h after every one hour. If
(a)1.6 min को 30 सेकंड म पार करती है तथा
the distance travelled in the first
(b)1.5 min उसी िदशा म 6 िकमी/घंटा की चाल
one hour was 35 km, what was
(c)1.8 min से चल रहे एक को 20 सेकंड
the total distance travelled in 12
(d)1.2 min म पीछे छोड़ दे ती है | x का मान ा
hours?
है ?
एक कार की चाल हर एक घंटे के

e
Q18. A train goes from P to Q SSC CGL TIER II (11
बाद 2 िकमी/घंटा से बढ़ जाती है |
with a speed μ km/h, then from Q September 2019)
यिद पहले एक घंटे म तय की गयी
to R (QR = 2PQ) with a speed 3 (a) 50
दू री 35 िकमी थी, तो 12 घंटे म
μ km/h, and returns from R to P (b) 54
िकतनी दू री तय की गयी थी ?

l
with a speed μ/2 km/h. What is (c) 56
SSC CPO 16 March 2019
the average speed (in km/h) of the (d) 60
ac
train for the entire journey
starting from P and back to P?
एक टे न P से Q तक μ िकमी/घंटा
की चाल से जाती है तथा िफर Q से R
तक ( QR = 2PQ) 3 μ िकमी/घंटा
(Evening)
(a)560 km
(b)650 km
(c)558 km
(d)552 km
Q3. A and B started their
journeys from X to Y and Y to X,
respectively. After crossing each
other, A and B completed the
की चाल से जाती है और R से P तक remaining parts of their journey
SSC CGL TIER II
μ/2 िकमी/घंटा की चाल से वापस in 6 81 h and 8h respectively. If the
आती है | P से शु होकर P तक speed of B is 28 km/h, then the
nn
Q1. Travelling at 60 km/h, a
वापस आने के दौरान इस पूरी या ा speed (in km/h), then the speed
person reaches his destination in a
म टे न की औसत चाल ( िकमी/घंटा म (in km/h) of A is :
certain time. He covers 60% of
) ात कर | A और B ने मशः X से Y और Y
his journey in 2/5th of the time.
SSC CHSL 11 July 2019 से X तक की अपनी या ाएं शु की |
At what speed (in km/h) should
(Afternoon) एक-दू सरे को पार करने के बाद, A
he travel to cover the remaining
(a) 18μ और B ने अपनी या ा का शेष भाग
23 journey so that he reaches the
4μ मशः 6 81 घंटे एवं 8 घंटे म पूरा
(b) 3
destination right on time?
िकया | यिद B की चाल 28
Pi

(c) 16μ 60 िकमी ित घंटा की चाल से चलते


23
ए एक अपने गंत थल पर िकमी/घंटा है , तो A की चाल (

(d) िकमी/घंटा म ) ात कर |
2 िकसी िनि त समय म प ँ चता है | वह
अपनी 60% या ा ⅖ समय म कर SSC CGL TIER II (11
Q19.A person covers 40% of a लेता है | शेष या ा ( िकमी/घंटा म ) September 2019)
distance with a speed of 60 km/h उसे िकस चाल से करनी चािहए (a) 40
and the remaining with a speed of तािक वह गंत थल पर सही समय (b) 42
40 km/h. What is his average पर प ं चे ? (c) 32
speed for the whole journey in SSC CGL TIER II (11 (d) 36
km/h? September 2019)
एक िकसी दू री का 40% (a) 40 Q4. Renu was sitting inside train
िह ा 60 िकमी/घंटा की चाल से तथा (b) 48 A, which was travelling at 50
शेष दू री 40 िकमी/घंटा की चाल से (c) 42 km/h. Another train, B, whose
तय करता है | पूरी या ा म उसकी (d) 36 length was three times the length

www.ssccglpinnacle.com support@ssccglpinnacle.com Ph. 09729327755, 09817390373 433


Days 49-52 Speed and Distance

s
sse
la
_c
ob
of A crossed her in the opposite distance (in km) from his house (a) 9 85
bo
ah
direction in 15 seconds. If the to his office is :
sm
(b) 9 32
ur
yo

speed of train B was 58 km/h, एक अपने घर से िनकलता है


e/

(c) 9 83
t.m

और 30 िकमी/घंटा की चाल से चलता


://

then the length of train A (in m)


tp

(d) 9 31
ht

is : है | वह अपने कायालय 10 िमनट की


रे णु एक टे न के भीतर बैठी ई थी, जो दे री से प ँ चता है | जब वह 24
Q9. Walking at 60% of his usual
50 िकमी/घंटा की चाल से चल रही िकमी/घंटा की चाल से चलता है , तब
speed, a man reaches his
थी | A की लंबाई से ितगुनी लंबाई की वह 18 िमनट की दे री से कायालय
destination 1 hour 40 minutes
एक अ टे न B ने उसे िवपरीत िदशा प ँ चता है | उसके घर से उसके
late. His usual time (in hours) to
से 15 सेकंड म पार िकया | यिद टे न कायालय की दू री ( िकमी म ) है :
reach the destination is :
B की चाल 58 िकमी/घंटा थी, तो टे न SSC CGL TIER II (12
अपनी सामा चाल की तुलना म
A की लंबाई ( मीटर म ) ात कर | September 2019)
60% चाल से चलते ए एक
SSC CGL TIER II (12 (a) 18
अपने गंत थल पर 1 घंटे 40
September 2019) (b) 16
िमनट की दे री से प ँ चता है | गंत
(a) 210 (c) 12
थल तक प ँ चने म उसे लगने वाला

e
(b) 180 (d) 20
सामा समय ( घंटे म ) है : SSC
(c) 160
CGL TIER II (13 September
(d) 150 Q7. To cover a distance of 416
2019)
km, a train A takes 2 32 hours
(a) 2 21

l
Q5. Places A and B are 396 km more than train B. If the speed of
apart. Train X leaves from A (b) 2 41
ac A is doubled, it would take 1 31
for B and train Y leaves from B (c) 3 81
hours less than B. What is the
for A at the same time on the speed (in km/h) of train A ? (d) 3 41
same day on parallel tracks. 416 िकमी की दू री तय करने म, टे न
Both trains meet after 5 21 A को टे न B से 2 32 घंटे अिधक लगते Practice Questions
hours. The speed of Y is 10 ह | यिद A की चाल दोगुनी कर दी
km/h more than that of X. जाए, तो इसे B से 1 31 घंटे कम लगगे Q1. A journey of 96 km takes one
What is the speed (in km/h) of hour less by a fast train (A) than
| टे न A की चाल ( िकमी/घंटा म )
Y?
nn
by a slow train (B) If the average
िकतनी है ? SSC CGL TIER II
थान A और B 396 िकमी दू र ह | speed of B is 16 km/h less than
(13 September 2019)
टे न X, A से B के िलए तथा टे न Y, that of A, then the average speed
(a) 56
B से A के िलए एक ही िदन एक ही (in km/h) of A is:
(b) 54
समय पर समानांतर पट रयों पर 96 िकमी की एक या ा म तेज़ टे न (
(c) 52
रवाना होती ह | दोनों टे न 5 21 घंटे A) धीमी टे न ( B) की तुलना म एक
(d) 65
के बाद िमलती ह | Y की चाल X घंटा कम लेती है | यिद B की औसत
की चाल से 10 िकमी/घंटा अिधक चाल A की औसत चाल से 16 िकमी
Q8. A person covers 40% of the
है | Y की चाल ( िकमी/घंटा म ) कम है , तो A की औसत चाल ात
Pi

distance from A to B at 8 km/h,


िकतनी है ? SSC CGL TIER II कर |
40% of the remaining distance at
(12 September 2019) SSC CGL 7 June 2019
9 km/h and the rest at 12 km/h.
(a) 41 (Evening)
His average speed (in km/h) for
(b) 54 (a) 64
the journey is :
(c) 31 (b) 48
एक A से B तक की 40%
(d) 56 (c) 54
दू री 8 िकमी/घंटा की चाल से, शेष
(d) 60
दू री का 40%, 9 िकमी/घंटा की चाल
Q6. A man starts from his house से तथा शेष दू री 12 िकमी/घंटा की
and travels at 30 km/h, he reaches Q2. A train without stoppage
चाल से तय करता है | इस या ा म
his office late by 10 minutes, and travels with an average speed of
उसकी औसत चाल ( िकमी/घंटा म )
travelling at 24 km/h, he reaches 70 km/h, and with stoppage, it
िकतनी रही है ?
his office late by 18 minutes. The travels with an average speed of
SSC CGL TIER II (13
56 km/h. How many minutes
September 2019)

www.ssccglpinnacle.com support@ssccglpinnacle.com Ph. 09729327755, 09817390373 434


Days 49-52 Speed and Distance

s
sse
la
_c
ob
does the train stop on an average (c) 6 कायालय प ँ चता है | उसका सामा
bo
ah
per hour? (d) 8 समय ( िमनट म ) ा है ?
sm
ur
yo

िबना के कोई टे न 70 िकमी/घंटा की SSC CGL 12 June 2019


e/
t.m

औसत चाल से चलती है तथा


://

Q5. A train without a stoppage (Evening)


tp
ht

क- क कर, यह 56 िकमी/घंटा की travels with an average speed of (a) 25


औसत चाल से चलती है | ित घंटे 80 km/h and with a stoppage , it (b) 30
यह टे न औसतन िकतने िमनट travels with an average speed of (c) 20
कती है ? 72 km/h. For how many minutes (d) 40
SSC CGL 10 June 2019 does the train stop on an average
(Evening) per hour? Q8. Walking 5/7 of his usual
(a) 12 िबना के कोई टे न 80 िकमी/घंटा की speed, a person reaches his office
(b) 14 औसत चाल से चलती है जबिक 10 minutes later than the usual
(c) 16 क- क कर यह 72 िकमी/घंटा की time. His usual time in minutes is:
(d) 15 औसत चाल से चलती है | हर घंटे यह अपनी सामा चाल की तुलना म 5/7
टे न औसतन िकतने िमनट कती है चाल से चलते ए, एक
सामा समय से 10 िमनट की दे री से

e
Q3. A train without stoppage ?
travels with an average speed of SSC CGL 11 June 2019 कायालय प ँ चता है | उसका सामा
65 km/h, and with stoppage, it (Evening) समय ( िमनट म ) है -
travels with an average speed of (a) 8 SSC CGL 13 June 2019

l
52 km/h. For how many minutes (b) 6 (Morning)
does the train stop on an average (c) 7 (a)28
per hour?
ac
िबना के कोई टे न 65 िकमी/घंटा की
चाल से चलती है और क- क कर
यह 52 िकमी/घंटा की चाल से चलती
है | हर घंटे यह टे न औसतन िकतने
(d) 9

Q6. A train without stoppage


travels with an average speed of
80 km/h and with stoppage, it
(b)30
(c)25
(d)35

Q9. Walking at 7/9 of his usual


िमनट कती है ? travels with an average speed of speed, a person reaches his office
SSC CGL 11 June 2019 64 km/h. For how many minutes 10 minutes later than the usual
(Morning) does the train stop on an average time. His usual time in minutes is:
nn
(a) 13 per hour? अपनी सामा चाल की तुलना म 7/9
(b) 15 िबना के एक टे न 80 िकमी/घंटा की चाल से चलते ए एक
(c) 12 औसत चाल से चलती है जबिक कायालय सामा समय से 10 िमनट
(d) 14 क- क कर यह 64 िकमी/घंटा की की दे री से प ँ चता है | उसका
औसत चाल से चलती है | हर घंटे यह सामा समय ( िमनट म ) है -
Q4. A train without stoppage टे न औसतन िकतने िमनट कती है SSC CGL 13 June 2019
travels with an average speed of ? (Afternoon)
72 km/h and with stoppage, it SSC CGL 12 June 2019 (a)35
Pi

travels with an average speed of (Morning) (b)27


60 km/h. For how many minutes (a) 12 (c)42
does the train stop on an average (b) 8 (d)30
per hour? (c) 10
िबना के एक टे न 72 िकमी/घंटा की (d) 14 Q10.The ratio of speeds of A and
औसत चाल से चलती है तथा B is 3:5. If A takes 24 minutes
क- क कर यह 60 िकमी/घंटा की Q7. Walking at ⅗ of his usual more than B to cover a certain
औसत चाल से चलती है | हर घंटे यह speed , a person reaches his office distance, then how much time (in
टे न औसतन िकतने िमनट कती है 20 minutes later than the usual minutes) will B take to cover the
? time. His usual time in minutes is same distance?
SSC CGL 11 June 2019 : A और B की चाल का अनुपात 3 : 5
(Afternoon) अपनी सामा चाल की तुलना म ⅗ है | यिद A को एक िनि त दू री तय
(a) 10 चाल से चलते ए, एक करने म B से 24 िमनट अिधक लगते
(b) 12 सामा समय से 20 िमनट दे री से

www.ssccglpinnacle.com support@ssccglpinnacle.com Ph. 09729327755, 09817390373 435


Days 49-52 Speed and Distance

s
sse
la
_c
ob
ह, तो इसी दू री को तय करने म B को taken 10 hours less for the same
bo
ah
िकतना समय ( िमनट म ) लगेगा ? journey. What is the speed of the Q16. The ratio between the
sm
ur
yo

SSC CHSL 3 July 2019 train (in km/h)? speeds of two trains is 2 : 5. If the
e/
t.m

एक टे न 60 िकमी की दू री एक
://

(Afternoon) first train runs 350 km in 5h, then


tp
ht

(a)40 समान चाल से तय करती है | यिद the difference between the speeds
(b)36 इसकी चाल 8 िकमी/घंटा अिधक (in km/h) of both the trains is:
(c)20 होती, तो इसे इस या ा म 10 घंटे कम दो टे नों की चाल म 2 : 5 का अनुपात
(d)18 लगते | इस टे न की चाल ( िकमी/घंटा है | यिद पहली टे न 5 घंटे म 350
म ) ात कर | िकमी की दू री तय करती है , तो दोनों
Q11. A train covers a certain SSC CHSL 4 July 2019 टे नों की चालों म अंतर ात कर |
distance in 45 minutes. If its (Evening) SSC CHSL 8 July 2019
speed is reduced by 5 km/h, it (a)4 (Evening)
takes 3 minutes more to cover the (b)2.5 (a)165
same distance. The distance (in (c)3 (b)180
km) is: (d)5 (c)350
एक टे न कोई िनि त दू री 45 िमनट म

e
(d)105
तय करती है | यिद इसकी चाल 5
िकमी/घंटा से कम कर दी जाए, तो Q14.A car covers 25 km at a Q17.The ratio between the speeds
इसे यह दू री तय करने म 3 िमनट uniform speed. If the speed had of two trains is 2 : 5. If the first
अिधक लगगे | यह दू री है -

l
been 8 km/h more, it would have train runs 250 km in 5 h, then the
SSC CHSL 4 July 2019 taken 10 hours less for the same difference between the speeds (in
(Morning)
(a)64
(b)60
(c)54
(d)80
ac journey. What is the speed of the
car (in km/h)?
एक कार 25 िकमी की दू री एक
समान चाल से तय करती है | यिद
चाल 8 िकमी/घंटा अिधक होती, तो
km/h) of both the trains is:
दो टे नों की चाल म 2 : 5 का अनुपात
है | यिद पहली टे न 5 घंटे म 250
िकमी की दू री तय करती है , तो दोनों
टे नों की चालों म अंतर ात कर |
इसे इस या ा म 10 घंटे कम लगते | SSC CHSL 9 July 2019
कार की चाल ( िकमी/घंटा म)िकतनी (Morning)
Q12. A train covers 360 km at a है ? (a)75
nn
uniform speed. If the speed had SSC CHSL 5 July 2019 (b)180
been 10 km/h more, it would have (Morning) (c)65
taken 3 hours less for the same (a)2.5 (d)150
journey. What is the speed of the (b)4
train (in km/h)? (c)3 Q18. The ratio between the
एक टे न 360 िकमी की दू री एक (d)2 speeds of two trains is 2 : 5. If the
समान चाल से तय करती है | यिद first train runs 250 km in 5 h, then
इसकी चाल 10 िकमी/घंटा अिधक Q15. The ratio between the the sum of the speeds (in km/h) of
Pi

होती, तो इसे इस या ा म 3 घंटे कम speeds of two trains is 2 : 5. If the both the trains is:
लगते | टे न की चाल ( िकमी/घंटा म ) first train runs 350 km in 5 hours, दो टे नों की चाल म 2 : 5 का अनुपात
ात कर | then the sum of the speed (in है | यिद पहली टे न 5 घंटे म 250
SSC CHSL 4 July 2019 km/h) of both the trains is: िकमी की दू री तय करती है , तो दोनों
(Afternoon) दो टे नों की चाल म 2 : 5 का अनुपात टे नों की चालों का योग ात कर |
(a)40 है | यिद पहली टे न 5 घंटे म 350 SSC CHSL 9 July 2019
(b)25 िकमी की दू री तय करती है , तो दोनों (Afternoon)
(c)30 टे नों की चालों का योग ात कर | (a)175
(d)50 SSC CHSL 8 July 2019 (b)150
(Afternoon) (c)180
(a)180 (d)165
Q13. A train covers 60 km at a (b)265
uniform speed. If the speed had (c)245 Q19. If I travel by bus, I reach my
been 8 km/h more. it would have (d)350 office 15 min late, and if I travel

www.ssccglpinnacle.com support@ssccglpinnacle.com Ph. 09729327755, 09817390373 436


Days 49-52 Speed and Distance

s
sse
la
_c
ob
by car, I reach 10 min early. If the km/h, then what is the time taken Q24. If a train runs with the speed
bo
ah
distance between my home and by the first throw to cover the of 48 km/hr, it reaches its
sm
ur
yo

my office is 25 km, then the length if the same for the second destination late by 12 minutes.
e/
t.m
://

difference of the reciprocals of thrown is one second? However, if its speed in 64 km/hr
tp
ht

average speeds of the bus and the िदया गया है िक दो लड़कों के ारा it is late by 3 minutes only. The
car, is second per metre is: अलग-अलग चाल से फकी गयी गदों right time for the train to cover
यिद म बस से या ा करता ँ , तो म के माग की लंबाई समान है तथा the journey (in minutes) is:
अपने कायालय 15 िमनट दे र से पहले और दू सरे फकी गयी गदों की यिद कोई टे न 48/घंटा की चाल से
प ँ चता ँ तथा यिद म कार से या ा औसत गित मशः 90 िकमी / घंटा चलती है , तो यह अपने गंत पर 12
करता ँ , तो म 10 िमनट पहले प ँ च और 162 िकमी / घंटा है , अगर दू सरे िमनट की दे री से प ँ चती है | हालाँ िक
जाता ँ | यिद मेरे घर से मेरे कायालय फकी गयी गद ने इस लंबाई को कवर जब इसकी चाल 64 िकमी/घंटा होती
की दू री 25 िकमी है , तो बस और करने के िलए एक सेकंड का समय है , तो यह केवल 3 िमनट की दे री से
कार की औसत चाल के पार रक िलया तो पहली फकी गयी गद ारा प ँ चती है | इस या ा को पूरा करने म
का अंतर ( मीटर ित सेकंड म ) है : इस लंबाई को कवर करने के िलए टे न ारा िलया गया सही समय है -
SSC CHSL 9 July 2019 िलया गया समय ा है ? SSC CPO 12 March 2019

e
(Evening) SSC CHSL 10 July 2019 (Evening)
3
(a) 25 (Evening) (a) 24
3
(b) 50 (a) 23 sec (b) 18
3
(c) 20 (b)1 sec (c) 20

l
3 (c) 59 sec (d) 22
(d) 10
ac (d) 32 sec
Q25. If a train runs with the speed
Q20. A boy standing by the side
of 52km/h, it reaches its
of a railway track that an Up train Q22. A train travels at a speed of
destination late by 15 minutes.
crosses him in 8 seconds and a 76 km/hr. If it crosses a pole in 36
However, if its speed is 65km/h,
Down train of twice the length of sec. The length of the train is:
it is late by 5 minutes only. The
that of the Up train crosses him in एक टे न 76 िकमी/घंटा की चाल से
right time for the train to cover its
20 seconds. How long (in चलती है | यिद यह एक खंभे को 36
journey is:
seconds) will the two trains take सेकंड म पार करती है , तो टे न की
यिद कोई टे न 52 िकमी/घंटा की चाल
nn
to cross each other? लंबाई ा है ?
से चलती है , तो यह अपने गंत
रे लवे टै क के िकनारे खड़ा एक SSC CPO 16 March 2019
थल पर 15 िमनट की दे री से
लड़का दे खता है िक अप-टे न उसे 8 (Morning)
प ँ चती है | लेिकन, जब इसकी चाल
सेकंड म पार कर जाती है जबिक (a) 760m
65 िकमी/घंटा होती है , तो यह केवल
अप-टे न से दोगुनी लंबाई की डाउन (b) 675 m
5 िमनट दे र करती है | टे न को या ा
टे न को उसे पार करने म 20 सेकंड (c) 720m
पूण करने म लगने वाला सही समय
लगते ह | दोनों टे नों को एक दू सरे को (d) 630m
ा है ?
पार करने म िकतना समय ( सेकंड म
SSC CPO 13 March 2019
Pi

) लगेगा ? Q23. If an airplane covers a


(Evening)
SSC CHSL 10 July 2019 distance of 980 km in 35 minutes,
(a)45 minutes
(Morning) then what time it will take to
(b)40 minutes
(a)13 31 cover a distance of 1470 km?
(c)35 minutes
(b)15 यिद कोई हवाई जहाज 35 िमनट म
(d)30 minutes
(c)20 980 िकमी की दू री तय करता है , तो
(d)12 31 1470 िकमी की दू री तय करने म
Q26. If a train runs at 60 Km/h, it
िकतना समय लगेगा? SSC CPO 16
reaches its destination 15 late.
March 2019 (Morning)
Q21. Given that the lengths of the But, if it runs at 80 Km/h, it is
(a) 21 hours
paths of a ball thrown with late by 7 minutes only. The right
different speeds by two boys are (b) 1 81 hours
time for the train to cover its
the same, and the average speed (c) 1 61 hours journey is:
7
for the first and second throws are (d) 8 hours यिद एक टे न 60 िकमी/घंटा की चाल
respectively 90 km/h and 162 से चलती है तो यह अपने गंत थल

www.ssccglpinnacle.com support@ssccglpinnacle.com Ph. 09729327755, 09817390373 437


Days 49-52 Speed and Distance

s
sse
la
_c
ob
पर 15 िमनट दे र से प ँ चती है | (a)8 km 65 km/h, how much time would
bo
ah
लेिकन, यिद यह 80 िकमी/घंटा की (b)4 km be saved?
sm
ur
yo

चाल से चलती है , तो इसे केवल 7 (c)5 km दो शहरों के बीच की दू री 52


e/
t.m

िमनट की दे री होती है | इस या ा को िकमी/घंटा की चाल से 3 41 घंटे म तय


://

(d)6 km
tp
ht

पूण करने म टे न को लगने वाला सही की जाती है | यिद चाल बढ़ाकर 65


समय ात कर | Q29. A and B at a distance of 1.7 िकमी/घंटा कर दी जाये, तो िकतना
SSC CPO 12 March 2019 km apart and they start running समय बचेगा ?
(Morning) towards each other at a speed of SSC CPO 15 March 2019
(a) 18 minutes 8m/s and 9 m/s respectively. (Morning)
(b) 17 minutes After how much time, will they (a) 39 minutes
(c) 20 minutes meet each other? (b) 45 minutes
(d) 21 minutes A और B 1.7 िकमी की दू री पर (c) 40 minutes
अलग ह और वे मशः 8 मीटर / (d) 42 minutes
Q27. If a train runs with the speed सेकंड और 9 मीटर / सेकंड की गित
of 36 km/h, it reaches its से एक दू सरे की ओर दौड़ना शु Q32.A drives at the rate of
करते ह। िकतने समय के बाद, वे

e
destination 15 minutes late. 45km/h and reaches its
However, if its speed is 45 km/h, एक दू सरे से िमलगे? destination 4 minutes late. If
it is late by only 4 minutes. The SSC CPO 14 March 2019 speed is 60 km/h, A reaches 5
correct time to cover its journey (Morning) minutes early.The distance

l
in minutes is : travelled by A is :
यिद एक टे न 36 िकमी/घंटा की चाल (a)1 minute 40 seconds / 1 िमनट A 45 िकमी/घंटा की चाल से वाहन
ac
से चलती है तो यह अपने गंत
पर 15 िमनट दे र से प ँ चती है |
लेिकन, यिद यह 45 िकमी/घंटा की
चाल से चलती है , तो इसे केवल 4
थल

िमनट की दे री होती है | इस या ा को
40 सेकड
(b)1 minute 4 seconds / 1 िमनट 4
सेकड
(c)14 minute / 14 िमनट
(d)14 seconds / 14 सेकड
चलाता है और अपने गंत थल पर
4 िमनट दे र से प ँ चता है | यिद चाल
60 िकमी/घंटा होती है , तो A 5 िमनट
पहले प ँ चता है | A ारा तय की गयी
दू री ात कर |
पूण करने म टे न को लगने वाला सही SSC CPO 15 March 2019
समय ात कर | Q30. A boy increases his speed to (Morning)
SSC CPO 13 March 2019 9/5 times of his original speed. (a)24 km
nn
(Morning) By this he reaches his school 30 (b)21 km
(a) 22 minutes before the usual time. (c)27 km
(b) 27 How much time does he takes (d)30 km
(c) 25 usually?
(d) 40 एक लड़का चाल म वृ करके Q33. A boy walks 15 m in 7
वा िवक चाल का 9/5 गुना कर लेता seconds and then walks back in 5
Q28. A goes to a mall from his है | इसके कारण वह अपने िव ालय seconds. His average speed (in
house on a cycle at 8km\h and सामा समय से 30 िमनट पहले m/s) is:
Pi

comes back to his house on a प ँ च जाता है | उसे आमतौर पर एक लड़का 7 सेकंड म 15 मीटर
cycle at 6km/h. If he takes 1 hour िकतना समय लगता है ? चलता है तथा िफर 5 सेकंड म वापस
10 minutes in all, what is the SSC CPO 16 March 2019 आ जाता है | उसकी औसत चाल (
distance between his house and (Evening) मीटर/सेकंड म ) ात कर |
the mall? (a) 67.75 min SSC CPO 16 March 2019
A अपने घर से 8 िकमी/घंटा की चाल (b) 67.50 min (Afternoon)
से साइिकल चला कर मॉल गया और (c) 67.25 min (a)6
वापस अपने घर साइिकल से 6 (d) 67.10 min (b)2.5
िकमी/घंटा की चाल से आया | यिद (c) 3.25
उसे कुल िमलाकर 1 घंटा 10 िमनट Q31. The distance between two (d)4
का समय लगा, तो उसके घर एवं cities in 3 41 hours at a speed of 52
मॉल के बीच की दू री ात कर | km/h. If the speed is increased to Q34. A part of the journey is
SSC CPO 14 March 2019 covered in 31.5 minutes at 80
(Morning)

www.ssccglpinnacle.com support@ssccglpinnacle.com Ph. 09729327755, 09817390373 438


Days 49-52 Speed and Distance

s
sse
la
_c
ob
km/h and the remaining part in 16 Q37. A boy goes from his home Q40. A travels 15 km with a
bo
ah
minutes at 75 km/h. The total to the park 500 m away in 5 speed of 30 km/h. He travels
sm
ur
yo

distance of the journey is: minutes and comes back in 7 another 25 km with a speed of
e/
t.m

या ा का एक िह ा 80 िकमी/घंटा
://

minutes. Their average move : 10km/h. What is his average


tp
ht

की चाल से 31.5 िमनट म तथा शेष एक लड़का अपने घर से पाक तक speed for the entire journey?
भाग 75 िकमी/घंटा की चाल से 16 500m दू र 5 िमनट म जाता है और 7 A 15 km की दू री को 30 km/h की
िमनट म पूण होता है | या ा की कुल िमनट म वािपस आ जाता है | उनकी चाल से तय करता है । वह 10 िकमी /
दू री ात कर | औसत चाल है : SSC CPO 14 घंटा की गित के साथ 25 िकमी की
SSC CPO 16 March 2019 March 2019 (Evening) या ा और करता है । पूरी या ा के
(Afternoon) (a) 1 िलए उसकी औसत गित ा है ?
(a)45 km (b) 12 SSC MTS 2 August 2019
(b)38 km (c) 2.5 (Morning)
(c) 62 km (d) 5 (a) 40
3 km/h
(d)54 km (b) 80 km/h
3
Q38. A person completes 210 km (c) 20 km/h

e
Q35. A train 100 m long running of his journey at 60 km/h and (d) 12 km/h
running speed crosses a station completes the next 198 km at 66
which is 500 m long in 25 km/h. What is the average speed Q41. A man leaves from P at 6
seconds. How long will it take for during the whole trip? am and reaches Q at 2 pm on the
एक अपनी या ा के पहले

l
the train to pass a station that is same day. Another man leaves Q
380 m long? 210 km को 60 km/h की गई से और at 8 am and reaches P at 3 pm on
ac
100 मीटर लंबी एक टे न एक े शन
को 25 सेकंड म पार करती है जो
500 मीटर लंबा है | 380 मीटर लंबे
े शन को पार करने म टे न को
िकतना समय लगेगा ?
अगले 198 km को 66 km/h की गित
से पूरा करता है | पूरी या ा के दौरान
औसत गित ा है ?
SSC CPO 14 March 2019
(Evening)
the same day. At what time do
they meet?
एक आदमी P से सुबह 6 बजे
िनकलता है और उसी िदन दोपहर 2
बजे Q पर प ं चता है । एक अ
SSC CPO 16 March 2019 (a) 68.5 km/h सुबह 8 बजे Q को छोड़ता है
(Afternoon) (b) 64 km/h और उसी िदन दोपहर 3 बजे P पर
(a)21 seconds (c) 63 km/h प ं चता है । वे िकस समय िमलते ह?
nn
(b)20 seconds (d) 62.8 km/h SSC MTS 2 August 2019
(c)19 seconds (Morning)
(d)22 seconds Q39. A train of length 230m has (a) 11 : 46 am
to cross a platform of length (b) 11 : 24 am
Q36. At the speed of 5 km/h a 750m. If train is moving at the (c) 10 : 48 am
girl, walks from her home to speed of 72 km/h. Then find the (d) 11 : 00 am
school in 24 minute, and in the time taken to cross the platform?
return at 8 km/h from the cycle. It 230 m ल ी एक टे न को 750 m
Pi

Q42.A car can cover 300 meters


took time to come back from a लंबे ेटफॉम को पार करना है | यिद in 6 minutes. What is its average
cycle: टे न 72 km/h की गित से चल रही है speed in km / h?
एक लड़की 5 km/h की गित से 24 और की छूट दे ता है | व ु का लागत एक कार 6 िमनट म 300 मीटर की
िमनट म अपने घर से ू ल पैदल मू है : दू री तय कर सकती है | इसकी
प ँ चती है और वापसी म वही दू री SSC CPO 14 March 2019 औसत गित km/h म िकतनी है ?
साईिकल से 8 km/h की गित से तय (Evening) SSC MTS 2 August 2019
करती है | साईिकल से वापस आने से (a)72 Seconds / सेकंड (Afternoon)
समय लगा : SSC CPO 14 March (b)58 Seconds / सेकंड (a) 50
2019 (Evening) (c)64 Seconds / सेकंड (b) 3
(a) 15 minute (d)49 Seconds / सेकंड (c) 12
(b) 10 minute (d) 6
(c) 8 minute SSC MTS
(d) 21 minute

www.ssccglpinnacle.com support@ssccglpinnacle.com Ph. 09729327755, 09817390373 439


Days 49-52 Speed and Distance

s
sse
la
_c
ob
Q43. A train crosses a 600 metres आलोक P िबंदु से Q की ओर 6 km/h SSC MTS 5 August 2019
bo
ah
long platform in 50 seconds. It की गित से चलना शु करता है । (Afternoon)
sm
ur
yo

crosses another 900 metres long रमन 9 km/h की गित के साथ P से (a) 660
37 km/h
e/
t.m

की Q की ओर चलना आर करता
://

platform in 60 seconds. What are (b) 480 km/h


tp
ht

37
the length and the speed of the है । रमन Q पर प ँ चता है , वापस (c) 460 km/h
37
train? मुड़ता है और P की ओर चलना शु 540
(d) km/h
एक टे न 50 सेकंड म 600 मीटर लंबे कर दे ता है । वह आलोक से R पर 37

ेटफाम को पार करती है । यह 60 िमलता है । यिद PQ 15 km है , तो PR


Q48. A train takes 1 minute to
सेकंड म 900 मीटर लंबा एक और िकतना है ?
cross a stationary pole. How long
ेटफाम पार करती है । टे न की SSC MTS 5 August 2019
(in seconds) will the train take to
लंबाई और गित ा ह? (Morning)
cross a bridge whose length is
SSC MTS 2 August 2019 (a) 20 km
twice the length of the train?
(Evening) (b) 12 km
एक रे लगाड़ी िकसी थर खंभे को
(a) 900 metres, 96 km/h (c) 15 km
पार करने म 1 िमनट का समय लेती
(b) 900 metres, 108 km/h (d) 18 km
है | िकसी पुल, िजसकी लंबाई

e
(c) 600 metres, 108 km/h
रे लगाड़ी की लंबाई से दोगुनी है , को
(d) 700 metres, 96 km/h Q46.A train crosses a 900 meter
पार करने म रे लगाड़ी को िकतना
long platform in 300 seconds. If
समय लगेगा (सेकंड म)?
Q44. A train leaves P at 9 am the length of the train is 600
SSC MTS 5 August 2019

l
with speed of 30 km/h. Another meters, then in what time can it
(Evening)
train leaves Q at 11 am with cross a stationary pole (stationary
ac
speed of 45 km/h. The trains are
travelling towards each other on
parallel tracks. Distance between
P and Q is 300 km. When they
meet, what is the ratio of the
pole)?
एक रे लगाड़ी 900 मीटर लंबे
ेटफॉम को 300 सेक म पार
करती है | यिद रे लगाड़ी की लंबाई
600 मीटर है , तब वह एक थर खंभे
(a) 150
(b) 90
(c) 120
(d) 180

Q49. The average speed of a car


distances covered by them? ( े शनरी पोल) को िकतने समय म
is 600 m / min. How much slower
एक टे न सुबह 9 बजे 30 km/h की पार कर सकती है ?
does the car run (in meters /
गित से P से िनकलती है । एक अ SSC MTS 5 August 2019
nn
second) than a runner who travels
टे न सुबह 11 बजे 45 km/h की गित (Afternoon)
100 meters in 9.6 seconds?
से Q से िनकलती है । टे न समानां तर (a) 5 Minutes
एक कार की औसत गित 600
पट रयों पर एक-दू सरे की ओर जा (b) 2 Minutes
मीटर/िमनट है | कोई धावक, जो 9.6
रही ह। P और Q के बीच की दू री (c) 3 Minutes
सेकंड म 100 मीटर की दू री तय
300 km है । वे कब िमलगीं, एवं (d) 4 Minutes
करता है , की तुलना म कार िकतनी
उनके ारा तय की गई दू रयों का
धीमी गित (मीटर/सेकंड म) से चलती
अनुपात िकतना होगा? Q47. A travels a certain distance
है ?
SSC MTS 5 August 2019 with three different speeds; first
Pi

SSC MTS 5 August 2019


(Morning) covers 3 km with a speed of 9 km
(Evening)
(a) 8 : 5 / h, the second 6 km with a speed 5
(a) 24
(b) 13 : 12 of 15 km / h and the third 9 km
(b) 1
(c) 17 : 14 with a speed of 18 km / h. What 2
(c) 7
(d) 11 : 9 is the average speed of A's entire 12
journey? (d) 5
12
Q45. Alok starts walking from P A िकसी िनि त दू री को तीन िभ
with speed of 6 km/h towards Q. गितयों के साथ तय करता है ; पहली 3 Q50. A car can cover a distance
Raman starts at same time from P km की दू री 9 km/h की गित के साथ, of 18 km in 24 minutes. The
towards Q with speed of 9 km/h. दू सरी 6 km की दू री 15 km/h की speed of a bus is twice that of a
Raman reaches Q, turns back and गित के साथ और तीसरी 9 km की car. In how many minutes can the
starts walking towards P. He दू री 18 km/h की गित के साथ तय bus cover a distance of 135 km?
meets Alok at R. If PQ is 15 km, करता है | A की पूरी या ा की औसत
then what is PR? गित िकतनी है ?

www.ssccglpinnacle.com support@ssccglpinnacle.com Ph. 09729327755, 09817390373 440


Days 49-52 Speed and Distance

s
sse
la
_c
ob
एक कार 24 िमनट म 18 km की दू री एक कार की गित 36 km/h है ।एक A िकसी िनि त दू री को तय करने म
bo
ah
तय कर सकती है | िकसी बस की बस िजसकी गित कार की गित का ⅕ B से 10 िमनट अिधक लेता है । यिद
sm
ur
yo

गित कार की दु गुनी है | बस 135 km है , 900 मीटर की दू री तय करने म उनकी गित 3: 4 के अनुपात म है , तो
e/
t.m

की दू री िकतने िमनट म तय कर िकतना समय लेगी ? B ारा समान दू री तय करने म


://
tp
ht

सकती है ? SSC MTS 6 August 2019 िकतना समय लगेगा?


SSC MTS 6 August 2019 (Evening) SSC MTS 7 August 2019
(Morning) (a) 5 21 (Afternoon)
(a) 48 Minutes (b) 7 21 (a)40 minutes
(b) 90 Minutes (c) 10 21 (b)30 minutes
(c) 50 Minutes (c)50 minutes
(d) 8 21
(d) 75 Minutes (d)20 minutes

Q54. A train, 700 m long crosses


Q 51. A train crosses a stationary Q57. A train moves at the speed
a pole in 35 seconds. How much
pole in 3 minutes. it crosses the of 80 km/h and crosses a platform
time does it take to cross a
600 meter long platform in 5 in 0.75 minutes. If the length of
platform of length 740 m?

e
minutes. The Length and speed of the train is equal to the length of
एक रे लगाड़ी , 700 मीटर लंबी 35
train is- the platform, then what is the
सेकंड म एक ख े को पार करती
कोई रे लगाड़ी िकसी थर खंभे को 3 length of the platform?
है । 740 मीटर के ेटफॉम को पार
िमनट म पार करती है | वह 600 एक टे न 80 km/h की गित से चलती
करने म इसे िकतना समय लगेगा ?
मीटर लंबे ेटफॉम को 5 िमनट म है और एक ेटफॉम को 0.75 िमनट

l
SSC MTS 6 August 2019
पार करती है | रे लगाड़ी की लंबाई म पार करती है । यिद टे न की लंबाई

(Afternoon)
ac
तथा गित िकतनी है ?
SSC MTS 6 August 2019

(a) 3600 m, 18 km/h


(b) 900 m, 18 km/h
(Evening)
(a) 1 min 24 sec
(b) 1 min 30 sec
(c) 1 min 12 sec
(d) 1 min 20 sec
ेटफ़ॉम की लंबाई के बराबर है , तो
ेटफ़ॉम की लंबाई ा है ?
SSC MTS 7 August 2019
(Afternoon)
(a)400 m
(c) 900 m, 15 km/h (b)480 m
Q55. A bus covers first 200 km of
(d) 1200 m, 15 km/h (c)450 m
a journey in 4 hours and the next
(d)500 m
nn
600 km in 6 hours. What is the
Q 52. A person travels from A to
average speed of the bus for the
B with a speed of 30 km /h. He Q58. A bus covers a distance of
whole journey?
returns from B to A at with speed 1.8 km in 3 minutes. How much
एक बस पहले 200 km की या ा 4
of 20 km / h. What is his average faster (in m/s) is it than a young
घंटे म और अगली 600 km की दू री 6
speed in the entire journey? athlete who runs 200 m in 25
घंटे म तय करती है । पूरी या ा के
एक A से B तक 30 km/h की seconds ?
िलए बस की औसत गित ा है ?
गित से जाता है | वह B से A तक 20 एक बस 3 िमनट म 1.8 िकमी की
SSC MTS 7 August 2019
km/h की गित से वापस लौट आता है दू री तय करती है । 25 सेकंड म 200
Pi

(Morning)
| पूरी या ा म उसकी औसत गित मीटर दौड़ने वाले युवा धावक की
(a) 100 km/h
िकतनी है ? तुलना म यह िकतना तेज (m/s) है ?
(b) 90 km/h
SSC MTS 6 August 2019 SSC MTS 7 August 2019
(c) 60 km/h
(Afternoon) (Evening)
(d) 80 km/h
(a) 26.5 km/h (a) 1.5
(b) 24 km/h (b) 1
(c) 25 km/h (c) 2
Q56. A takes 10 minutes more
(d) 27.5 km/h (d) 2.5
than B in covering a certain
distance. If their speeds are in the
Q53. The speed of a car is 36 Q59. A train with 72 km/h speed
ratio of 3:4, then what is the time
km/h. How much time (in crosses a stationary pole in 35
taken by B to cover the same
minutes) will a bus traveling at seconds. How much time (in
distance?
one-fifth of the speed of the car minutes) does it take to cross a
take to cover a distance of 900 m? 1.1 km long bridge?

www.ssccglpinnacle.com support@ssccglpinnacle.com Ph. 09729327755, 09817390373 441


Days 49-52 Speed and Distance

s
sse
la
_c
ob
72 km/h की गित वाली टे न 35 Q62. A man goes from C to D at पूरा करने के िलए िकतनी गित (km
bo
ah
सेकंड म एक थर ख े को पार 40 km/h and he returns from D to /h) बढ़ाई जानी चािहए?
sm
ur
yo

करती है । 1.1 km लंबे पुल को पार C at x km/h. If the average speed SSC MTS 9 August 2019
e/
t.m

करने म िकतना समय (िमनटों म)


://

of the man for the whole journey (Morning)


tp
ht

लगेगा? is 60 km/h, then what is the value (a) 180


SSC MTS 8 August 2019 of x? (b) 108
(Morning) एक आदमी 40 km/h की गित से C (c) 72
(a) 2.5 से D तक जाता है और वह X km/h (d) 100
(b) 3 की गित से D से C पर लौटता है ।
(c) 2 यिद पूरी या ा के िलए आदमी की Q65. In 100m race A beats B by
(d) 1.5 औसत गित 60 km/h है , तो x का 10 m and B beats C by 10m. By
मान ा है ? what distance A beat C (in m) ?
Q60. A man travels from P to Q SSC MTS 8 August 2019 100 m की दौड़ म A, B से 10 m
at the speed of 60 km/h and (Evening) आगे हो जाता है , B, C से 10 m आगे
travels from Q to P at the speed (a) 100 हो जाता है । A, C से िकतना आगे
िनकला है ?

e
of 90 km/h. What is the average (b) 120
speed of the man for the whole (c) 110 SSC MTS 9 August 2019
journey? (d) 80 (Afternoon)
एक आदमी 60 km/h की गित से P (a) 19
से Q की ओर या ा करता है और 90

l
Q63. A motor car moves at a (b) 18
km/h की गित से Q से P की ओर speed of 72 km/h and 54 km/h (c) 20

(Afternoon)
(a) 75 km/h
ac
या ा करता है । पूरी या ा के िलए
आदमी की औसत गित ा है ?
SSC MTS 8 August 2019
after and before repairing
respectively. It covers X distance
in 6 hours after repairing. How
much time (in hours) will it take
to cover 5X distance before
(d) 21

Q66. A fighter aircraft runs at a


speed of 1440 km / hr. How many
meters does it cover in one
(b) 78 km/h repairing? second?
(c) 70 km/h एक मोटर कार मशः मर त करने एक लड़ाकू िवमान 1440 km/hr की
(d) 72 km/h से पहले और बाद म 72 km/h और गित से चलता है | एक सेकंड म वह
nn
54 km/h की गित से चलती है । यह िकतने मीटर की दू री तय करता है ?
Q61. A train starts from A at 6 मर त के बाद 6 घंटे म x दू री तय SSC MTS 9 August 2019
AM and reaches B at 11 AM on करती है । मर त से पहले 5x दू री (Evening)
the same day. Another train starts को तय करने म िकतना समय (घंटों (a)394
from B at 8 AM and reaches A at म) लगेगा? (b)385
3 PM on the same day. At what SSC MTS 9 August 2019 (c)374
time the two trains will have (Morning) (d)400
crossed each other? (a) 40
Pi

एक टे न सुबह 6 बजे शु होती है (b) 45 Q67. A man is walking at a speed


और उसी िदन सुबह 11 बजे B पर (c) 30 of 12 km / h. After every
प ं चती है । एक अ टे न B से सुबह (d) 36 kilometer, he takes rest for 4
8 बजे शु होती है और उसी िदन minutes. How much time (in
दोपहर 3 बजे A प ं चती है । िकस Q64. A sports car going at an minutes) will it take to cover a
समय पर दोनों टे न एक-दू सरे को पार average speed of 108 km/h takes distance of 8 kilometers?
कर लेगी ? 15 minutes to complete a lap on a एक आदमी 12 िकमी/घंटा की गित
SSC MTS 8 August 2019 racing track. How much speed (in से चल रहा है | ेक िकलोमीटर के
(Evening) km/h) should be increased to बाद, वह 4 िमनट आराम करता है | 8
(a) 9:45 AM complete the lap in 9 minutes? िकलोमीटर की दू री तय करने म वह
(b) 8:45 AM 108 km/h की औसत गित से चलने िकतना समय (िमनट म) लेगा?
(c) 10:30 AM वाली एक ोट् स कार को रे िसंग टै क SSC MTS 13 August 2019
(d) 7:45 AM पर एक घेरे को पूरा करने म 15 (Morning)
िमनट लगते ह। 9 िमनट म घेरे को (a)68

www.ssccglpinnacle.com support@ssccglpinnacle.com Ph. 09729327755, 09817390373 442


Days 49-52 Speed and Distance

s
sse
la
_c
ob
(b)64 SSC MTS 13 August 2019 तय करने के िलए गित िकतनी बढ़ानी
bo
ah
(c)60 (Evening) होगी?
sm
ur
yo

(d)72 SSC MTS 14 August 2019


e/
t.m
://

(a)2:3 (Morning)
tp
ht

Q68. A woman travelling at (b)9:7 (a)7.5 km/h


130% of her usual speed reaches (c)5:8 (b)10 km/h
her office 12 minutes early. Her (d)3:4 (c)8 km/h
usual time to cover the journey is (d)5 km/h
: Q71. A man goes to a place on
अपनी सामा गित से 130% या ा car at speed of 160 km/h and Q74. A train's average speed is 72
करने वाली मिहला अपने कायालय म comes back at lower speed. If the km/h but is reduced to 60 km/h
12 िमनट पहले प ँ चती है । या ा को average speed is 64 km/h in total, due to stoppages. For how much
तय करने के िलए उसका सामा them the return speed (in km/h) time does the train stop in an
समय ा है ? is: hour?
SSC MTS 13 August 2019 एक कार से िकसी थान को एक टे न की औसत चाल 72
160 िकमी/घंटा की चाल से जाता है िकमी/घंटा है लेिकन कने के कारण

e
(Afternoon)
(a) 0.52 hr तथा कम चाल से वापस आता है | यह कम हो कर 60 िकमी/घंटा रह
(b) 0.48 hr यिद कुल िमलाकर औसत चाल 64 जाती है | एक घंटे म टे न िकतने समय
(c) 2 hrs िकमी/घंटा है , तो वापसी की चाल ( के िलए कती है ?
िकमी/घंटा म ) है :

l
(d) 1 hr SSC MTS 14 August 2019
SSC MTS 13 August 2019 (Afternoon)
ac
Q69. Each wheel of a bus is
making 7 revolutions per second.
If the diameter of a wheel is 56
cm, then the speed of the bus (in
cm/sec) would be:
(Evening)
(a)25
(b)80
(c)40
(d)60
(a)10 minutes
(b)12 minutes
(c)15 minutes
(d)8 minutes

एक बस का ेक पिहया ित Q75. If a student walks with


सेकंड 7 च र लगा रहा है । यिद Q72. The ratio in the speeds of speed 30% more than the usual
पिहये का ास 56 cm है , तो बस की two aeroplanes is 11 : 18. If the speed, he reaches 15 min earlier
nn
गित िकतनी (cm/s) होगी? first covers a distance of 1650 to his destination. How much
SSC MTS 13 August 2019 Km in 3 hours, then find the time (in minutes) does he take to
(Evening) speed of the second one (in m/s ). reach his destination normally?
(a)616 दो हवाई जहाज़ों की गितयों का यिद कोई छा सामा चाल से 30%
(b)1232 अनुपात 11:18 है | यिद पहला हवाई तेज़ चाल से चलता है , तो वह अपने
(c)1000 जहाज 3 घंटों म 1650 km की दू री गंत थल पर 15 िमनट पहले प ँ च
(d)176 तय करता है , तो दू सरे हवाई जहाज जाता है | आमतौर पर गंत थल
की गित (m/s म) िकतनी है ? तक प ँ चने म उसे िकतना समय (
Pi

Q70. Two trains each having SSC MTS 14 August 2019 िमनट म ) लगता है ?
length of 160 meters moving in (Morning) SSC MTS 14 August 2019
opposite direction crossed each (a)270 (Evening)
other in 9 seconds. If one train (b)200 (a) 45
crossed a 200 meters long (c)225 (b) 65
platform in 18 seconds, then the (d)250 (c) 50
ratio of the speeds is: (d) 39
िवपरीत िदशा म चलते ए 160 मीटर Q73. A car can cover a distance
ल ाई वाली दो टे न एक-दू सरे को 9 of 420 km in 7 hours. How much Q76. A man on a tour travels first
सेकंड म पार करती है । यिद एक टे न speed should be increased to 360 km by train at 72 km/h, the
18 सेकंड म 200 मीटर लंबे cover this distance in 6 hours ? next 160 km on a motor cycle at
ेटफाम को पार करती है , तो इनकी कोई कार 7 h म 420 km की दू री 12.80 km/h and the last 200 km
गित का अनुपात है : तय कर सकती है | दू री को 6 h म by on a bicycle at 16 km/h.
Ignoring the buffer times between

www.ssccglpinnacle.com support@ssccglpinnacle.com Ph. 09729327755, 09817390373 443


Days 49-52 Speed and Distance

s
sse
la
_c
ob
the different modes of travel, गित से तय िकया | उसने कुल िकतनी (in seconds) can it cross a bridge
bo
ah
what is the average speed (in m/s) दू री तय की ? 2200 metres long?
sm
ur
yo

for his tour? SSC MTS 16 August 2019 600 मीटर लंबी एक टे न 1200 मीटर
e/
t.m

िकसी या ा म एक पहली 360 लंबे ेटफ़ॉम को 36 सेकंड म पार


://

(Afternoon)
tp
ht

िकमी की दू री 72 िकमी/घंटा की चाल (a) 384 km कर सकती है | यह 2200 मीटर लंबे


से टे न से तय करता है , अगली 160 (b) 768 km पुल को िकतने समय ( सेकंड म ) म
िकमी की दू री मोटरसाइिकल पर (c) 576 km पार करे गी ?
12.80 िकमी/घंटा की चाल से तथा (d) 960 km SSC MTS 19 August 2019
अंितम 200 िकमी की दू री एक (Morning)
साइिकल पर 16 िकमी/घंटा की चाल Q79. A car travels for 6 hours at (a) 56
से तय करता है | प रवहन के िविभ different speeds. For the first two (b) 52
साधनों के बीच म वत समय को hours at 60 m/s, for the next 2 (c) 48
नज़रअंदाज़ करते ए, इस या ा म hours at 50 m/s and for the (d) 44
उसकी औसत चाल ( मी/सेकंड म ) remaining time at x m/s. If the
ात कर | average speed for the entire Q82. The speed of a car is 20 m/s.

e
SSC MTS 16 August 2019 journey is 52 m/s, what is the What distance (in km) will it
(Morning) value of x? cover in 2.5 hours ?
(a) 6.67 एक कार अलग-अलग चाल से 6 घंटे एक कार की गित 20 m/s है | कार
(b) 7.33 तक या ा करती है | पहले दो घंटे वह 2.5 घंटे म िकतनी दू री (km म) तय
60 मी/सेकंड की चाल से चली, अगले करे गी ?

l
(c) 4.33
(d) 5.67 दो घंटे 50 मी/सेकंड और शेष समय SSC MTS 19 August 2019
ac
Q77. The ratio in the speeds of
two aeroplanes is 7 : 15. If the
first aeroplane covers a distance
of 1050 km in 3 hours, then what
वह x मी/सेकंड से चली | यिद पूरी
या ा के िलए औसत चाल 52
मी/सेकंड रही है , तो x का मान ात
कर |
SSC MTS 22 August 2019
(Afternoon)
(a) 108
(b) 180
(c) 144
(d) 72
is the speed of the second (Evening)
aeroplane ? (a) 46 Q83. A person travels a certain
दो हवाई जहाजों की गितयों का (b) 44 21 distance at a speed of 18 km/h
nn
अनुपात 7 : 15 है | यिद पहला हवाई (c) 48 and returns to the starting point at
जहाज 3 घंटों म 1050 km की दू री (d) 45 21 12 km/h. If he takes 2 hours 55
तय करता है , तो दू सरे हवाई जहाज minutes for the whole journey,
की गित िकतनी है ? Q80. Speed of a man is 45 km/h. the distance one way is:
SSC MTS 16 August 2019 In how much time (in seconds) एक कोई िनि त दू री 18
(Afternoon) will he cover a distance of 225 िकमी/घंटा की चाल से तय करता है
(a) 720 km/h metres? और आरं िभक िबंदु पर 12 िकमी/घंटा
(b) 675 km/h एक की चाल 45 िकमी/घंटा है की चाल से वापस आता है | यिद पूरी
Pi

(c) 900 km/h | वह 225 मीटर की दू री िकतने समय या ा म उसे 2 घंटे 55 िमनट लगते ह,
(d) 750 km/h ( सेकंड म ) म तय करे गा ? तो एक तरफ की दू री है :
SSC MTS 19 August 2019 SSC MTS 19 August 2019
Q78. A person goes on a journey. (Morning) (Evening)
He travelled 16 hours in all. He (a) 24 (a) 21 km
covered the first half of the (b) 16 (b) 25 km
distance @ 40 km/h and the other (c) 18 (c) 18 km
half @ 60 km/h. Find the total (d) 22 (d) 24 km
distance covered by him.
एक आदमी या ा पर िनकलता है | Q81. A 600 metres long train can Q84. A train X travelling at 72
उसने कुल 16 घंटों की या ा की | दू री cross a 1200 metres long platform km/h crosses another train Y
का पहला आधा भाग उसने 40 km/h in 36 seconds. In how much time travelling at 63 km/h (in opposite
और दू सरा आधा भाग 60 km/h की direction) in 18 seconds. If the
length of train Y is two-third the

www.ssccglpinnacle.com support@ssccglpinnacle.com Ph. 09729327755, 09817390373 444


Days 49-52 Speed and Distance

s
sse
la
_c
ob
length of X, then length of train X समान दू री तय करने म िकतना समय taken by him to cover a distance
bo
ah
is: (िमनटों म) लगता है ? of 665 m.
sm
ur
yo

72 िकमी/घंटा की चाल से चल रही SSC MTS 20 August 2019 यिद िशवान 19 km/h की गित से
e/
t.m

कोई टे न X , 63 िकमी/घंटा की चाल चलता है , तो 665 m की दू री तय


://

(Afternoon)
tp
ht

से िवपरीत िदशा म चल रही दू सरी (a) 135 करने म उसे िकतना समय (िमनट म)
टे न Y को 18 सेकंड म पार करती है | (b) 126 लगेगा ?
यिद टे न Y की लंबाई टे न X की (c) 120 SSC MTS 21 August 2019
लंबाई की दो-ितहाई है , तो टे न X की (d) 96 (Morning)
लंबाई ात कर | (a) 3
SSC MTS 19 August 2019 Q87. Two trains running in the 1
(b) 2 10
(Evening) opposite directions cross each (c) 1 21
(a) 300 m other in 12 seconds and they take (d) 2
(b) 420 m 60 seconds when they run in the
(c) 405 m same direction. The possible Q90. A 180 m long train running
(d) 270 m speeds ( m/s) of these two trains at 20 m/s will take what time ( in

e
can be : seconds ) to cross a child walking
Q85. A and B start travelling दो रे लगािड़यां िवपरीत िदशा म चलते at 10 m/s in the same direction ?
towards each other at the same ए एक दू सरे को 12 सेकंड म पार 20 m/sec की गित से चल रही 180 m
time at the respective speeds of करती ह, और समान िदशा म चलने ल ी एक रे लगाड़ी 10 m/sec की
पर वे 60 सेकंड का समय लेती ह |

l
10 km/h and 20 km/h. The गित से उसी िदशा म चल रहे एक
distance between them is 49 km. दोनों रे लगािड़यों की संभािवत गितयां ब े को पार करने म िकतना समय
ac
What is the distance ( in meters )
between them before 5 minutes of
meeting ?
A और B एक ही समय पर मश:
10 km/h और 20 km/h की गित से
(m/sec म) हो सकती ह:
SSC MTS 20 August 2019
(Afternoon)
(a) 18, 40
(b) 15, 45
(sec म) लेगी ?
SSC MTS 21 August 2019
(Morning)
(a) 12
(b) 36
एक दू सरे की ओर चलना शु करते (c) 20, 30 (c) 15
ह और उनके बीच की दू री 49 km है (d) 15, 30 (d) 18
| आपस म िमलने से 5 िमनट पहले,
nn
उनके बीच की दू री, m म, िकतनी है Q88. A train with the average Q91. A person travels 75 km at a
? speed of 54 km/h, covers a speed of 25 km/h, next 60 km at a
SSC MTS 20 August 2019 distance in 200 minutes. To speed of 20 km/h and the last 90
(Morning) reduce this time to 90 minutes, at km at a speed of 15 km/h. His
(a) 1600 what speed ( km/h ) the train average speed is :
(b) 1800 should run ? एक 75 km की या ा 25
(c) 2500 एक रे लगाड़ी िजसकी औसत गित 54 km/h की गित से, अगली 60 km की
(d) 2000 km/h है , िकसी दू री को 200 िमनट म या ा 20 km/h की गित से और
Pi

तय करती है | या ा समय को कम उसके बाद 90 km की या ा 15


Q86. The speed of A for walking कर 90 िमनट करने हे तु रे लगाड़ी को km/h की गित से तय करता है |
two times is equal to the speed of िकस गित (km/h म) से चलना उसकी औसत गित है :
B for walking three times. To चािहए? SSC MTS 21 August 2019
cover a distance, A takes 42 SSC MTS 20 August 2019 (Afternoon)
minutes less than B to cover the (Evening) (a) 25.5 km/h
same distance. What is the time ( (a) 120 (b) 18.75 km/h
in minutes ) taken by B to cover (b) 60 (c) 20.25 km/h
the same distance ? (c) 180 (d) 15 km/h
A की दो बार चलने की गित B के (d) 220
तीन बार चलने की गित के बराबर है | Q92. A bus covers a distance of
िकसी दू री को तय करने म A को B Q89. If Shivan runs at 19 km/h, 10 km in 12 minutes. If its speed
की तुलना म समान दू री तय करने म then find the time ( in minutes ) is reduced by 25 km/h, then what
42 िमनट कम समय लगता है | B को

www.ssccglpinnacle.com support@ssccglpinnacle.com Ph. 09729327755, 09817390373 445


Days 49-52 Speed and Distance

s
se
s
la
_c
ob
time ( in minutes ) will it take to travelled by him in three modes टै न A और B की लंबाइओं का
bo
ah
cover the same distance? is: अनुपात 5:3 है और उनकी गितयों
sm
ur
yo

एक बस 10 km की दू री 12 िमनट म िकसी टे न की चाल एक कार की चाल का अनुपात 2:3 है | िवपरीत िदशा म


e/
t.m

तय करती है | यिद उसकी गित 25 से ितगुनी तथा एक ीमर की चाल से चलते ए, टै न A ारा B को पार
://
tp
ht

km/h कम कर दी जाए, तो उसी दू री 1.5 गुनी है | एक ने ीमर से करने म 2 21 िमनट का समय लगा |
को तय करने म बस को िकतना x िकमी, टे न से 3.75x िकमी तथा एक थर ( े शनरी पोल ) को
समय (िमनट म) लगेगा ? कार से 2x िकमी की या ा की | यिद पार करने म टै न A ारा िलया गया
SSC MTS 21 August 2019 कार की चाल 40 िकमी/घंटा है और समय (िमनट म ) है :
(Afternoon) उसके ारा िलया गया कुल समय SSC MTS 21 August 2019
(a) 36 4 2 घंटा है , तो इन तीन साधनों की
1 (Evening)
(b) 13 सहायता से उसके ारा तय की गयी (a) 2
(c) 15 कुल दू री ात कर | (b) 3
(d) 24 SSC MTS 22 August 2019 (c) 125
32
(Morning) (d) 25
16
Q93. A person covers a distance (a) 450 km

e
of 390 km in 3.25 hours. His (b) 520 km Q98. Two trains are moving in
speed in m/s is : (c) 480 km opposite directions with the speed
एक 390 km की दू री 3.25 (d) 420 km of 35 m/s and 45 m/s respectively.
घंटों म तय करता है | उसकी गित,
From the time they are 12 km
m/sec म, है :

l
Q96. A and B started walking in apart, how much time would they
SSC MTS 21 August 2019 the opposite direction from X and
ac take to cross each other ?
(Evening) Y respectively at the same time. दो टे न िवपरीत िदशाओं म मशः 35
(a) 33 31 After meeting, A and B took 2.7 मी/सेकंड और 40 मी/सेकंड की चाल
(b) 24 hours and 1.2 hours to reach Y से या ा कर रही ह | जब उनके बीच
(c) 30 32 and X respectively. If the speed of की दू री 12 िकमी है , उस समय से
(d) 25 B is 48 km/hr, then find the speed उ एक-दू सरे को पार करने म
of A ( in km/h ) . िकतना समय लगेगा ?
Q94. The diameter of a wheel is A और B ने मश X और Y से एक SSC MTS 14 August 2019
ही समय पर िवपरीत िदशा म चलना
nn
70 cm. It completes 600 (Afternoon)
revolutions in 1 minute. The शु िकया | िमलने के बाद, A और (a) 2 minutes 30 seconds
speed, in km/h, of the vehicle is: B को Y और X पर प ं चने म मश: (b) 2 minutes
(Take π = 22
7 ) 2.7 घंटे और 1.2 घंटे लगे | यिद B की (c) 3 minutes
एक पिहये का ास 70 सेमी है | यह गित 48 km/h है , तो A की गित (d) 3 minutes 30 seconds
1 िमनट म 600 च र लगाता है | (km/h म) िकतनी है ?
वाहन की चाल ( िकमी/घंटा म ) है : SSC MTS 22 August 2019
SSC MTS 22 August 2019 (Afternoon) Q99. A train running at a speed of
Pi

(Morning) (a) 32 108 km/h crosses a pole in 32


(a) 78.4 (b) 36 seconds. The length of the train is
(b) 79.2 (c) 40 :
(c) 77.8 (d) 30 108 km/h की गित से चल रही कोई
(d) 78.2 टे न िकसी ख े को 32 सेकंड म पार
Q97. The ratio of the lengths of करती है | टे न की ल ाई ( m म ) है
Q95. The speed of a train is 3 train A and B is 5 : 3 and their :
times that of a car and 1.5 times speeds are in the ratio of 2 : 3. SSC MTS 13 August 2019
that of a steamer. A person Walking in the opposite direction, (Evening)
travelled x km by steamer, 3.75x Train A took 2 21 minutes to cross (a) 960
km by train and 2x km by car. If B. The time ( in minutes ) taken (b) 1024
the speed of the car is 40 km/h by train A to cross a stationary (c) 1200
and total time taken by him is 4 21 pole is : (d) 1240
hours, then the total distance

www.ssccglpinnacle.com support@ssccglpinnacle.com Ph. 09729327755, 09817390373 446


Days 49-52 Speed and Distance

s
sse
la
_c
ob
Q100. Train A takes 9 hours (c) 100
bo
ah
more than train B to cover a Q103. A train x running at 84 (d) 120
sm
ur
yo

distance of 612 km. If its speed is km/h crosses another train y


e/
t.m
://

doubled then it takes 3 hours less running at 52 km/h in opposite Q106. A car consumes 5.4 litres
tp
ht

than the time taken by train B. direction in 12 seconds. If the of petrol to cover 60.48 km. how
The speed of the train B is : length of y is two-third that of x, many kilometers be covered with
612 km की या ा करने के िलए टे न then what is the length of x ? / 84 22 litres of petrol?
B की तुलना म, टे न A 9 घंटे अिधक िकमी/घंटा की चाल से चल रही एक एक कार 60.48 िकमी की दू री तय
समय लेती है | यिद टे न की गित को टे न x सामने से 52 िकमी/घंटा की करने म 5.4 लीटर पेटोल खपत
दोगुना िकया जाता है , तो वह टे न B चाल से आ रही दू सरी टे न y को 12 करती है | 22 लीटर पेटोल म िकतनी
की तुलना म 3 घंटे का समय कम सेकंड म पार करती है | यिद y की दू री तय की जायेगी ?
लेती है | टे न B की गित (km/h म) है लंबाई x की लंबाई से दो-ितहाई है , तो SSC CPO 15 March 2019
:- x की लंबाई िकतनी है ? (Morning)
SSC MTS 9 August 2019 SSC CHSL 2 July 2019 (a) 246.4
(Evening) (Evening) (b) 238.62

e
(a) 51 km/h (a) 250 m (c) 240.24
(b) 1.2 km/h (b) 242 m (d) 243.5
(c) 40.8 km/h (c) 272 m Q107. A person completes 210
(d) 30.6 km/h (d) 408 m km of his journey at 60 km/h and

l
completes the next 198 km at 66
Q101. A train crosses a stationary Q104. The ratio between the km/h. What is the average speed
ac
object in 25 seconds. If the speed
of the train is 25 m/s, then find
the length of the train.
एक रे लगाड़ी िकसी
( े शनरी ऑ े
थर व ु
) को 25 सेकंड
speeds of two trains is 5:7. If the
first train covers 300 km in 3
hours, then the speed (in km/h) of
the second train is :
दो टे नों की चाल म 5 : 7 का अनुपात
during the whole trip? / एक
अपनी या ा के पहले 210 km
को 60 km/h की गई से और अगले
198 km को 66 km/h की गित से पूरा
करता है | पूरी या ा के दौरान औसत
म पार करती है | यिद रे लगाड़ी की है | यिद पहली टे न 300 िकमी की गित ा है ?
गित 25 m/s है | तो रे लगाड़ी की दू री 3 घंटे म तय करती है , तो दू सरी SSC CPO 15 March 2019
ल ाई िकतनी है ? टे न की चाल ( िकमी/घंटा म ) िकतनी (Evening)
nn
SSC MTS 9 August 2019 है ? (a) 68.5 km/h
(Evening) SSC CHSL 5 July 2019 (b) 64 km/h
(a) 625 m (Afternoon) (c) 63 km/h
(b) 500 m (a) 150 (d) 62.8 km/h
(c) 300 m (b) 140
(d) 620 m (c) 120 Q108. A train of length 230m has
(d) 100 to cross a platform of length
Q102. A train crosses two 750m. If train is moving at the
Pi

platform of length 1000 m and Q105. The ratio between the speed of 72 km/h. Then find the
600 m in 80 seconds and 60 speeds of two trains is 5:7. If the time taken to cross the platform? /
seconds respectively. What is the first train covers 300 km in 3 230 m ल ी एक टे न को 750 m लंबे
length of the train ? hours, then the speed (in km/h) of ेटफॉम को पार करना है | यिद टे न
कोई टे न 1000 मी तथा 600 मी लंबाई the first train is : 72 km/h की गित से चल रही है तो
वाले दो ेटफ़ॉम को मशः 80 दो टे नों की चाल 5 : 7 के अनुपात म ेटफॉम पार करने म टै न ारा िलया
सेकंड और 60 सेकंड म पार करती है | यिद पहली टे न 300 िकमी की गया समय ात करे : SSC CPO 15
है | टे न की लंबाई िकतनी है ? दू री 3 घंटे म तय करती है , तो पहली March 2019 (Evening)
SSC MTS 8 August 2019 टे न की चाल ( िकमी/घंटा म ) िकतनी (a)72 Seconds / सेकंड
(Afternoon) है ? (b)58 Seconds / सेकंड
(a) 720 m SSC CHSL 5 July 2019 (c)64 Seconds / सेकंड
(b) 600 m (Evening) (d)49 Seconds / सेकंड
(c) 540 m (a) 150
(d) 400 m (b) 140

www.ssccglpinnacle.com support@ssccglpinnacle.com Ph. 09729327755, 09817390373 447


Days 49-52 Speed and Distance

s
sse
la
_c
ob
Q109. Each wheel of a bus is (a) 30.75 km/h simultaneously, one from A and
bo
ah
making 7 revolutions per second. (b) 10.8 km/h the other from B. If they move in
sm
ur
yo

If the diameter of a wheel is 56 (c) 25.5 km/h the same direction, they meet
e/
t.m
://

cm, then the speed of the bus (in (d) 32.45 km/h after 12 hours, but if they move
tp
ht

cm/sec) would be : / एक बस का towards each other they meet


ेक पिहया ित सेकंड 7 च र Q2. A and B start moving toward after 89 hours . The speed(in km/h)
लगाता है | यिद एक पिहये का ास each other from places X and Y, of the car moving at a faster
56 सेमी है , तो बस की चाल ( respectively at the same time. The speed, is: / थान A और B 144
सेमी/सेकंड म ) होगी : speed of A is 20% more than that िकमी दू र ह | दो कार एक ही समय
SSC MTS 13 August 2019 of B. After meeting on the way, A चलना शु करती ह, पहली कार A
(Evening) and B take 2 21 hours and x hours, से तथा दू सरी कार B से | ये वे समान
(a) 616 now to reach Y and X, िदशा म चलती ह, तो वे 12 घंटों के
(b) 1232 respectively. What is the value of बाद िमलती ह, लेिकन यिद वे
(c) 1000 x? / A और B मशः थान X और एक-दू सरे की ओर चलती ह, तो 89
(d) 176 Y से एक ही समय म एक-दू सरे की घंटे के बाद िमलती ह| ती गित से

e
ओर चलना शु करते ह | A की चलने वाली कार की चाल (
Q110. A train runs 50% faster चाल B की चाल से 20% अिधक है | िकमी/घंटा म ) िकतनी है ? SSC
than a car. Both start running रा े म िमलने के बाद, A और B को CGL 4 March 2020 (Evening)
from the same point and meet Y और X तक प ँ चने म मशः 2 21

l
after covering a distance of 360 घंटे और x घंटे लगते ह | x का मान (a) 70
km. In the middle of this journey, ा है ? SSC CGL 3 March 2020 (b) 72
ac
the train stops for two hours due
to some faults in engine. What is
the speed of the train ? / कोई
रे लगाड़ी एक कार से 50% तेजी से
चलती है | दोनों एक ही िबंदू से चलना
(Afternoon)

(a) 3 53
(b) 3 32
(c) 60
(d) 64

Q5. A train takes 2 21 hours less


for a journey of 300 km, if its
(c) 3 21
शु करते ह और 360 km की या ा
(d) 3 52 speed is increased by 20km/h
के बाद आपस म िमलते ह | या ा के
from its usual speed . How much
बीच रे लगाड़ी इं जन म खराबी के
nn
time will it take to cover a
कारण िकसी े शन पर 2 घंटों के Q3. Two racers run at a speed of
distance of 192 km at its usual
िलए क जाती है | रे लगाड़ी की गित 100m/min and 120m/min
speed? / यिद एक टे न की चाल
िकतनी है ? respectively. If the second racer
उसकी सामा चाल से 20
SSC MTS 16 August 2019 takes 10 minute less than the first
िकमी/घंटा बढ़ा दी जाए, तो इसे 300
(Evening) to complete the run, then how
िकमी की या ा म 2 21 घंटे कम लगते
(a) 80 km/h long is the race?
दो धावक मशः 100 मीटर/िमनट ह | अपनी सामा चाल से इसे 192
(b) 60 km/h
तथा 120 मीटर / िमनट की चाल से िकमी की दू री तय करने म िकतना
(c) 72 km/h
Pi

दौड़ते ह | यिद दू सरे धावक को पहले समय लगेगा ? SSC CGL 5 March
(d) 90 km/h
धावक की तुलना म दौड़ पूरी करने म 2020 (Morning)
10 िमनट कम लगते ह, तो दौड़ की
लंबाई िकतनी है ? (a) 4.8 hours
SSC CGL TIER I
SSC CGL 3 March 2020 (b) 2.4 hours
(Evening) (c) 3 hours
Q1. A train crosses a pole in 12
(d) 6 hours
sec., and a bridge of length 170m
in 36 sec. Then the speed of the (a) 4 cm
Q6. The distance between two
train is: / एक टे न िकसी खंभे को 12 (b) 6 cm
stations, A and B is 428 km. A
सेकंड म तथा 170 मीटर लंबे पुल को (c) 1 cm
train starts from station ‘A’ at
36 सेकंड म पार करती है | इस टे न (d) 2 cm
6:00 a.m. and moves towards
की चाल है :
Q4. Places A and B are 144 km station ‘B’ at an average speed of
SSC CGL 3 March 2020
apart. Two cars start 48km/h . Another train starts
(Morning)

www.ssccglpinnacle.com support@ssccglpinnacle.com Ph. 09729327755, 09817390373 448


Days 49-52 Speed and Distance

s
sse
la
_c
ob
from station ‘B’ at 6:20 a.m. and distance between the places A (b) 45 km
bo
ah
moves towards station ‘A’ at an and B is: (c) 100 km
sm
ur
yo

average speed of 55km/h. At दो साइिकिल X और Y थान A से (d) 5 km


e/
t.m

एक ही समय पर चलते ह तथा B की


://

what time will the train meet?


tp
ht

दो े शनों - A और B के बीच की तरफ मशः 6 िकमी/घंटा और 8 Q11. The two trains leave
दू री 428 िकमी है | एक टे न े शन A िकमी/घंटा की चाल से जाते ह | Varanasi for Lucknow at 11:00
से सुबह 6 बजे िनकलती है तथा अपनी या ा के दौरान 15 िमनट तक a.m. and at 11:30 a.m.,
े शन B की ओर 48 िकमी/घंटा की कने के बाद भी, Y, X से 10 िमनट respectively and travel at a speed
औसत चाल से जाती है | एक अ पहले प ँ च जाता है | थान A और B of 110 km/h and 140 km/h,
टे न े शन B से सुबह 6 : 20 म के बीच की दू री है : SSC CGL 6 respectively. How many
चलती है तथा े शन A की ओर 55 March 2020 (Afternoon) kilometers from Varanasi will
िकमी/घंटा की चाल से जाती है | ये both the trains meet?
टे न िकस समय िमलगी ? (a) 10 km दो टे न वाराणसी से लखनऊ के िलए
SSC CGL 5 March 2020 (b) 16.5 km मशः 11 : 00 am तथा 11 : 30 a.m
(Evening) (c) 8 km म खुलती ह तथा मशः 110
िकमी/घंटा और 140 िकमी/घंटा की

e
(a) 10:20 a.m. (d) 6 km
(b) 9:40 a.m. चाल से या ा करती ह| वाराणसी से
(c) 10:00 a.m. Q9. A train crosses a platform िकतने िकलोमीटर की दू री पर ये
(d) 10:40 a.m. 180 m long in 60 sec at a speed of दोनों टे न िमलगी ?

l
72 km/h. The time taken by the SSC CGL 7 March 2020
Q7. A student takes 1.25 hours to train to cross an electric pole is: (Afternoon)
ac
travel from home to school at a
speed of 4 km/h. By what
percentage should he increase his
speed to reduce the time by 25%
to cover the same distance from
एक टे न 72 िकमी/घंटा की चाल से
180 मीटर लंबे ेटफ़ॉम को 60
सेकंड म पार करती है | िबजली के
खंभे को पार करने म टे न को िकतना
समय लगेगा ?
(a) 255 31 km
(b) 238 32 km
(c) 246 31 km
(d) 256 32 km
school to home? SSC CGL 6 March 2020
एक छा को 4 िकमी/घंटा की चाल (Evening)
Q12. X and Y are two stations
से चलते ए घर से िव ालय प ँ चने (a) 0.51 min
nn
which are 280 km apart. A train
म 1.25 घंटे लगते ह | िव ालय से घर (b) 5.1 min
starts at a certain time from X and
तक इतनी ही दू री तय करने के (c) 51 sec
travels towards Y at 60 km/h.
दौरान समय म 25% कटौती करने के (d) 5.1 sec
After 2 hours, another train starts
िलए उसे अपनी चाल म िकतने
from Y and travels towards X at
ितशत की वृ करनी चािहए ? Q10. Two cars A and B leave
20 km/h. After how many hours
SSC CGL 6 March 2020 Delhi at 8:30 a.m. and 9 a.m. for
does the train leaving from X
(Morning) Shimla, respectively. They travel
meets the train which left from
at the speeds of 40km/h and 50
Pi

Y? / X और Y दो े शन ह जो एक
(a) 45% km/h respectively. How many
दू सरे से 280 िकमी दू र ह। एक टे न
(b) 50% kilometres away from Delhi will
े शन X से एक िनि त समय पर
(c) 25% the two cars be together?
शु होती है और 60 िकमी / घंटा की
(d) 33 31 % दो कार A तथा B िद ी से मशः 8
गित से े शन Y की ओर जाती है । 2
: 30 a.m एवं 9 a.m म िशमला जाने
घंटे के बाद, एक और टे न े शन Y
Q8. Two cyclists X and Y start at के िलए खुलती है | वे मशः 40
से शु होती है और 20 िकमी / घंटा
the same time from place A to go िकमी/घंटा तथा 50 िकमी/घंटा की
की गित से े शन X की ओर जाती
towards place B at a speed of 6 चाल से या ा करती ह | दोनों कार
है । े शन X से िनकलने वाली टे न
km/h and 8 km/h, respectively. िद ी से िकतने िकलोमीटर की दू री
िकतने घंटों के बाद े शन Y से
Despite stopping for 15 minutes पर एक साथ होंगी ?
रवाना ई टे न से िमलेगी?
during the journey, Y reaches 10 SSC CGL 7 March 2020
SSC CGL 9 March 2020
minutes earlier than X. The (Morning)
(Morning)
(a) 200 km

www.ssccglpinnacle.com support@ssccglpinnacle.com Ph. 09729327755, 09817390373 449


Days 49-52 Speed and Distance

s
sse
la
_c
ob
(a) 3 hours (d) 225 km एक धावक 40 िकमी/घंटा की चाल से
bo
ah
(b) 6 hours दौड़ रहा है । यिद वह 30 िकमी/घंटा
sm
ur
yo

(c) 4 hours SSC CHSL 2019 की चाल से दौड़ता है , तो उसकी चाल


e/
t.m

म िकतने ितशत की कमी आएगी ?


://

(d) 2 hours
tp
ht

Q1. Richa travels from A to B at CHSL 12-10-2020 (Afternoon


Q13. Amit travelled a distance of the speed of 15 km/h, from B to C shift)
50 km in 9 hours. He travelled at 20 km/h, and from C to D at 30 (a) 15%
partly on foot at 5km/h and partly km/h. If AB = BC = CD, then (b) 20%
by bicycle at 10km/h. The find the Richa's average speed. (c) 25%
distance travelled on the bicycle रचा ने A से B तक 15 िकमी/घंटा (d) 30%
is: / अिमत ने 9 घंटे म 50 िकमी की की चाल से, B से C तक 20
दू री तय की। उ ोंने आं िशक प से िकमी/घंटा की चाल से तथा C से D Q4. A train of length 350 m
5 िकमी / घंटा की पैदल या ा की तक 30 िकमी/घंटा की चाल से या ा crosses a bridge of length 250 m
और आं िशक प से 10 िकमी / घंटा की। यिद AB= BC= CD है , तो रचा in 20 seconds. What is the speed
की दर से साइिकल से या ा की। की औसत चाल ात कीिजए। of the train (in km/h)?
साइिकल पर तय की गई दू री है : 350 मीटर लंबी रे लगाड़ी 250 मीटर

e
CHSL 12-10-2020 (morning
SSC CGL 9 March 2020 shift) लंबे पुल को 20 सेकंड म पार करती
(Afternoon) (a) 19 km/h है । इस रे लगाड़ी की चाल (िकमी/घंटा
(b) 18 km/h म) िकतनी है ?

l
(a) 12 km (c) 17 km/h CHSL 12-10-2020 (Evening
(b) 10 km (d) 20 km/h shift)
(c) 11 km
(d) 13 km
ac
Q14. The distance between two
stations, A and B is 575 km. A
Q2. A man divided his journey
into three parts of distances of 18
km, 20 km and 27 km. He
travelled the distances at the
(a) 95
(b) 72
(c) 108
(d) 88

train starts from station ‘A’ at speeds of 6 km/h, 5 km/h and 9 Q5. The distance between two
3:00 p.m. and moves towards km/h, respectively. What was his railway stations is 1176 km. To
station ‘B’ at an average speed of average speed during the entire cover this distance, an express
nn
50 km/h. Another train starts journey? train 5 hours less than a passenger
from station ‘B’ at 3:30 p.m. and एक ने अपनी या ा को 18 train while the average speed of
moves towards station ‘A’ at an िकमी, 20 िकमी तथा 27 िकमी की the passenger train is 70 km/h
average speed of 60 km/h. How दू रयों म िवभािजत िकया। उसने इन less than that of the express train.
far from station ‘A’ will the trains दू रयों को मशः 6 िकमी/घंटा, 5 The time taken by the passenger
meet? िकमी/घंटा और 9 िकमी/घंटा की train to complete the travel is:
दो े शनों A और B के बीच की दू री चाल से तय िकया। पूरी या ा के दो रे लवे े शनों के बीच की दू री
575 िकमी है | एक टे न े शन A से 3 दौरान उसकी औसत चाल िकतनी 1176 िकमी है । इस दू री को तय
Pi

: 00 p.m म खुलती है तथा े शन B रही ? करने के िलए, एक ए ेस टे न या ी


की तरफ 50 िकमी/घंटा की औसत CHSL 12-10-2020 (Afternoon टे न की तुलना म 5 घंटे कम लेती है
चाल से जाती है | एक अ टे न shift) जबिक या ी टे न की औसत चाल
े शन B से 3:30 p.m म खुलती है (a) 4.5 km/h ए ेस टे न की औसत चाल की
तथा े शन A की तरफ 60 (b) 5.5 km/h तुलना म 70 िकमी/घंटा कम है । या ा
िकमी/घंटा की औसत चाल से जाती (c) 6.5 km/h को पूण करने म या ी टे न ारा िलया
है | दोनों टे न े शन A से िकतनी दू री (d) 7.5 km/h गया समय िकतना है ?
पर िमलगी ? CHSL 13-10-2020 (Morning
SSC CGL 9 March 2020 Q3. A runner is running at a Shift)
(Evening) speed of 40 km/h. If he runs at a (a)18 hours/ घंटे
speed of 30 km/h, then what will (b)17 hours/ घंटे
(a) 275 km the decrease in the percentage of (c)23 hours/ घंटे
(b) 325 km his speed be? (d)12 hours/ घंटे
(c) 300 km

www.ssccglpinnacle.com support@ssccglpinnacle.com Ph. 09729327755, 09817390373 450


Days 49-52 Speed and Distance

s
sse
la
_c
ob
Q6. The distance between the एक टे न 126 िकमी/घंटा की चाल से 40 िमनट म एक िनि त दू री तय
bo
ah
places H and O is D units. The े शनों P और Q के बीच की दू री तय करने के िलए एक कार 64
sm
ur

average speed that gets a person


yo

करती है , जबिक िवपरीत िदशा म यह िकमी/घंटा की चाल से चल रही है ।


e/
t.m

from H to O in a stipulated time 90 िकमी/घंटा की चाल से वापस या ा के समय को 30 िमनट कम


://
tp
ht

is S units. He takes 20 minutes


आती है । दू सरी टे न पहले टे न की करने के िलए कार को िकस चाल से
more time than usual if he travels
औसत चाल से समान दू री तय करती चलना चािहए?
at 60 km/h, and reaches 44
minutes early if he travels 75 है । 525 िकमी की या ा के िलए दू सरी CHSL 15-10-2020 (Afternoon
km/h. The sum of the numerical टे न ारा िलया गया समय है : Shift)
values of D and S is: CHSL 14-10-2020 (Afternoon (a) 85.33 km/h
H और O थानों के बीच की दू री D shift) (b) 74.65 km/h
इकाई है । H से O तक जाने म एक a) 5 hours/ घंटे (c) 54 km/h
िनधा रत समय म S इकाई की (b) 4 hours/ घंटे (d) 84.78 km/h
औसत चाल को ा करता है । यिद (c) 5 hours 20 min/ 5 घंटे 20 िमनट
वह 60 िकमी/घंटा की चाल से या ा
(d) 4 hours 20 min/ 4 घंटे 20 िमनट Q12. Rakesh walking at the speed
करे , तो उसे सामा की तुलना म 20
of 8km/h crosses a bridge in 30
िमनट अिधक समय लगेगा और वह

e
यिद वह 75 िकमी/घंटा की चाल से minutes. What is the length of the
या ा करे , तो वह 44 िमनट पहले Q9. An athlete crosses a distance bridge in kilometers?
प ँ चेगा। D और S के सं ा क of 900 m in 10 minutes. What is 8 िकमी/घंटा की चाल से चलने वाला
मान का जोड़ है : his speed in km per hour? राकेश 30 िमनट म एक पुल को पार
CHSL 13-10-2020 (Afternoon एक एथलीट 900 मीटर की दू री 10 कर जाता है । िकलोमीटर म पुल की

l
Shift) िमनट म तय करता है । िकमी/घंटा म लंबाई िकतनी है ?
a) 358
(b) 384
(c) 376
(d) 344
ac
Q7.A train that is running at the
उसकी चाल िकतनी है ?
CHSL 14-10-2020 (Evening
shift)
(a) 5.4 km/h
(b) 3.6 km/h
CHSL 15-10-2020 (Evening
Shift)
(a) 4 km
(b) 2 km
(c) 3 km
(c) 4.8 km/h (d) 5 km
speed of 72 km/h crosses an
(d) 6.9 km/h Q13. A train, 150 m long, is
electric pole in 36 seconds. The
running at 90 km/h. How long (in
nn
length of the train (in metres) is:
Q10.Mohan covers a distance of seconds) will it take to cross a
एक टे न जो 72 िकमी/घंटा की चाल
2.5 km by scooter at the rate of 30 platform that is 300 m long?
से चल रही है और 36 सेकंड म एक
km/h. The time taken by Mohan 150 मीटर लंबी एक टे न, 90
िवद् युत पोल को पार करती है । टे न
to cover the given distance (in िकमी/घंटा की चाल से चल रही है ।
की लंबाई (मीटर म) है :
minute) is: 300 मीटर लंबाई के एक ेटफॉम
CHSL 13-10-2020 (Evening
मोहन ने ू टर ारा 30 िकमी/घंटा को पार करने म इसे िकतना समय
Shift)
की चाल से 2.5 िकमी की दू री तय (सेकंड म) लगेगा?
(a)360 m
की। मोहन ारा दी गयी दू री को CHSL 16-10-2020 (Morning
Pi

(b)460 m
िमनटों म तय करने का समय Shift)
(c)620 m
(िमनटों म ) है : (a) 6
(d)720 m
CHSL 15-10-2020 (Morning (b) 18
Shift) (c) 12
Q8. A train travels the distance (a)10 (d) 50
between stations P and Q at a (b)5
speed of 126 km/h, while in the (c)6 Q14. Two trains start at the same
opposite direction it comes back (d)8 time from stations A and B, 1800
at 90 km/h. Another train travels km apart, and
the same distance at the average Q11. A car is running at a speed proceed towards each other at an
speed of the first train. The time of 64 km/h to cover a certain average speed of 44km/h and 46
taken by the second train to travel distance in 40 min. At what speed km/h, respectively. Where will
525 km is: should the car run to reduce the the trains meet?
time of the journey to 30 min?

www.ssccglpinnacle.com support@ssccglpinnacle.com Ph. 09729327755, 09817390373 451


Days 49-52 Speed and Distance

s
sse
la
_c
ob
दो टे न एक ही समय म े शनों A
bo
ah
और B से शु होती ह, जो 1800 Q17. A man travelled a distance Q20. Rahul and Mithun travel a
sm
ur
yo

िकमी की दू री पर ह और मशः 44 of 1200 km in 16 hours. He distance of 30 km. The sum of


e/
t.m

िकमी/घंटा और 46 िकमी/घंटा की
://

travelled partly by car at a speed their speeds is 70 km/h and the


tp
ht

औसत चाल से एक दू सरे की ओर of 40 km/h and partly by train at a total time taken by both travel the
बढ़ती ह। टे न कहां िमलगी ? speed of 80 km/h. What is the distance is 2 hours 6 minutes. The
CHSL 16-10-2020 (Afternoon distance travelled by car? difference between their speeds
Shift) एक आदमी ने 16 घंटे म 1200 िकमी is:
(a) 920 km from station A/ े शन की दू री तय की। उ ोंने आं िशक दू री रा ल और िमथुन 30 िकमी की दू री
A से 920 िक.मी. 40 िकमी/घंटा की चाल से कार से तय करते ह। उनकी चाल का योग 70
(b) 900 km from station B/ े शन और आं िशक दू री 80 िकमी/घंटा की िकमी/घंटा है और दोनों के ारा दू री
Bसे 920 िक.मी. चाल से टे न से तय की। कार ारा तय तय करने म लगने वाला कुल समय 2
(c) 880 km from station A/ े शन की गई दू री िकतनी है ? घंटे 6 िमनट है । उनकी चाल के बीच
A से 880 िक.मी. CHSL 19-10-2020 (Morning का अंतर है :
(d) 880 km from station B/ े शन Shift) CHSL 20-10-2020 (Morning
B से 880 िक.मी.

e
(a) 100 km Shift)
(b) 120 km (a) 30 km/h
Q15. A person walks a distance (c) 96 km (b) 25 km/h
from point A to B at 15km/h, and (d) 80 km (c) 20 km/h

l
from point B to A at 30km/h. If (d) 35 km/h
he takes 3 hours to complete the Q18. Mohan travels three equal

एक
ac
journey, then what is the distance
from point A to B?
िबंदु A से B की दू री 15
िकमी/घंटा और िबंदु B से A की दू री
30 िकमी/घंटा की चाल से तय करता
distances at speeds of 12 km/h, 18
km/h and 24 km/h. If he takes a
total of 13 hours, then what is the
total distance covered?
मोहन 12 िकमी / घंटा, 18 िकमी/घंटा
Q.21. A man travelled a distance
of 35 km in 5 hours. He travelled
partly on foot at the rate of 4km/h
and the rest on bicycle at the rate
है । यिद उसे या ा पूरी करने म 3 घंटे और 24 िकमी/घंटा की चाल से तीन of 9 km/h. The distance travelled
लगते ह, तो िबंदु A से B की दू री ा समान दू री तय करता है । यिद उसे on foot is:
है ? कुल 13 घंटे लगते ह, तो कुल दू री एक आदमी ने 5 घंटे म 35 िकमी की
nn
CHSL 16-10-2020 (Evening िकतनी है ? दू री तय की। उसने आं िशक प से
Shift) CHSL 19-10-2020 (Afternoon 4 िकमी/घंटा की चाल से पैदल या ा
(a) 25km Shift) की और बाकी साइिकल 9 िकमी/घंटा
(b) 10km (a) 212 km की चाल से या ा की। पैदल या ा की
(c) 15km (b) 216 km दू री ात करे ।
(d) 30km (c) 214 km CHSL 20-10-2020 (Afternoon
(d) 218 km Shift)
Q16. The distance covered by a (a) 8 km
Pi

train in (x + 1) hours is ( x3 + 1) Q19. How much time (in (b) 12 km


km. What is the speed of the minutes) will a dog take to run (c) 10 km
train? around a square field of side 75 m (d) 15 km
(x + 1) घंटे म टे न ारा तय की गई if it runs at the rate of 6 km/hr?
दू री ( x3 + 1) ) िकमी है । टे न की चाल 6 िकमी/घंटा की चाल से दौड़ने वाले Q.22 How many minutes does
ा है ? एक कु े को 75 मीटर भुजा के वग Shyam take to over a distance of
CHSL 16-10-2020 (Evening के चारो तरफ दौड़ने म िकतना समय 500m if he runs at a speed of 30
Shift) (िमनट म) लगेगा? km/h?
(a) (x3 − 1) km/h CHSL 19-10-2020 (Evening 30 िकमी/घंटा की चाल से चलने पर
(b) ( x2 − x + 1) km/h Shift) ाम को 500 मीटर की दू री को तय
(c) ( x2 + x + 1) km/h (a) 2.5 करने म िकतने िमनट लगते ह
(d) (x + 1) km/h (b) 3.6 CHSL 20-10-2020 (Evening
(c) 3 Shift)
(d) 1.8 (a) 2

www.ssccglpinnacle.com support@ssccglpinnacle.com Ph. 09729327755, 09817390373 452


Days 49-52 Speed and Distance

s
se
s
la
_c
ob
(b) 2 21 (a) 42 या ा के िलए लगने वाले घंटों की
bo
ah (b) 40 सं ा िकमी/घंटा म चाल का
sm
(c) 1 1
ur

2
yo

(c) 48 ितिनिध करने वाली सं ा का


e/

(d) 1
t.m

2/3 है । दू री तय करने म लगने वाला


://

(d) 45
tp
ht

Q.23. A train covers a distance of Q.26. A train takes 45 minutes to समय है :


12 km in 12 minutes, if its speed cover a certain distance at a speed CHSL 26-10-2020 (Evening
is decreased by 5 km/h, then the of 80 km/h. If the speed is Shift)
time taken by it to cover the increased by 125%, then how (a) 15 hours/ घंटे
distance of 22 km will be: long will it take the train to cover (b) 18 hours/ घंटे
एक टे न 12 िमनट म 12 िकमी की 8/5 of the same distance? (c) 24 hours/ घंटे
दू री तय करती है , अगर इसकी चाल एक टे न को 80 िकमी/घंटा की चाल (d) 20 hours/ घंटे
5 िकमी/घंटा कम हो जाती है , तो से एक िनि त दू री तय करने म 45
इसके ारा 22 िकमी की दू री तय िमनट का समय लगता है । यिद चाल Q.29. A man walks from point X
करने म लगने वाला समय होगा: 125% बढ़ जाती है , तो टे न को उसी to Y at a speed of 20 km/h, but
CHSL 21-10-2020 (Morning दू री के 8/5 भाग को तय करने म comes back from point Y to X at
िकतना समय लगेगा?

e
Shift) a speed of 25 km/h. Find his
(a) 24 minutes/ िमनट CHSL 26-10-2020 (Morning average speed.
(b) 20 minutes/ िमनट Shift) एक आदमी 20 िकमी/घंटा की चाल
(c) 22 minutes/ िमनट (a) 25 minutes/ िमनट से िबंदु X से Y तक जाता है , लेिकन
(b) 30 minutes/ िमनट 25 िकमी/घंटा की चाल से िबंदु Y से

l
(d) 18 minutes/ िमनट
(c) 32 minutes/ िमनट X तक वापस आता है । उसकी
ac
Q.24. A man is walking at a
speed of 12km/h. After every km,
he takes a rest for 3 minutes. How
much time will he take to cover a
distance of 6 km?
(d) 28 minutes/ िमनट

Q.27. Abhi finishes a journey by


car in 9 hours. He travels the first
half of the journey at a speed of
औसत चाल ात कीिजए।
CHSL 17-03-2020 (Morning
Shift)
(a) 22 92
(b) 23 92
एक आदमी 12 िकमी/घंटा की चाल 40 km/h and the second half of (c) 25 92
से चल रहा है । ेक िकमी के बाद, the journey at a speed of 50 km/h.
(d) 24 92
वह 3 िमनट के िलए आराम करता The total distance covered is:
nn
है । 6 िकमी की दू री तय करने म उसे अिभ 9 घंटे म कार से या ा समा
Q30. How much time will a horse
िकतना समय लगेगा? करता है । वह या ा का पहला आधा
take to run around a square field
CHSL 21-10-2020 (Afternoon भाग 40 िकमी/घंटा की चाल से और
of side 175 m if it runs at the
Shift) दू सरा आधा भाग 50 िकमी/घंटा की
speed of 15 km/hr?
(a) 42 चाल पूरा करता है । तय की गई कुल
यिद घोड़ा 15 िकमी/घंटा की चाल से
(b) 40 दू री है :
दौड़ता है , तो 175 मीटर भुजा के वग
(c) 48 CHSL 26-10-2020 (Afternoon
का च र लगाने म घोड़े को िकतना
Shift)
Pi

(d) 45 समय लगेगा?


(a) 350 km
CHSL 17-03-2020 (Afternoon
Q.25. A man is walking at a (b) 450 km
Shift)
speed of 12km/h. After every km, (c) 400 km
(a) 180 sec
he takes a rest for 3 minutes. How (d) 300 km
(b) 175 sec
much time will he take to cover a (c) 155 sec
distance of 6 km? Q.28. A car moves a distance of
(d) 168 sec
एक आदमी 12 िकमी/घंटा की चाल 600 km with uniform speed. The
से चल रहा है । ेक िकमी के बाद, number of hours taken for the
Q31. A car travels 105 km in 3
वह 3 िमनट के िलए आराम करता journey is 2/3 of the number
hours and a train travels 252 km
है । 6 िकमी की दू री तय करने म उसे representing speed in km/h. The
in 4 hours. The ratio of speed of
िकतना समय लगेगा? time taken to cover the distance
the car to that of the train is:
CHSL 21-10-2020 (Evening is:
एक कार 3 घंटे म 105 िकमी की
Shift) एक कार एकसमान चाल के साथ
या ा करती है और एक टे न 4 घंटे म
600 िकमी की दू री तय करती है ।

www.ssccglpinnacle.com support@ssccglpinnacle.com Ph. 09729327755, 09817390373 453


Days 49-52 Speed and Distance

s
sse
la
_c
ob
252 िकमी की या ा करती है । टे न (a) 8:50 a.m. (c) 12 hours/ घंटे
bo
ah
की चाल का कार की चाल से अनुपात (b) 8:40 a.m. (d) 8 hours/ घंटे
sm
ur
yo

है : (c) 9:00 a.m.


e/
t.m
://

CHSL 17-03-2020 (Afternoon (d) 9:40 a.m. Q.37. Mohan finishes a journey
tp
ht

Shift) by scooter in 5 hours. He travels


(a) 2 : 7 Q34. A person covers 700 m the first half of the journey at 30
(b) 3 : 5 distance in 6 minutes. What is his km/h and the second half of the
(c) 9 : 11 speed in km/h? journey at 20 km/h. The distance
(d) 5 : 9 एक 6 िमनट म 700 मीटर की covered by him is:
दू री तय करता है । िकमी/घंटा म मोहन ने 5 घंटे म ू टर से या ा पूरी
Q32. Two cars start from the उसकी चाल ा है ? की। वह या ा का पहला आधा भाग
same place at the same time at CHSL 18-03-2020 (Afternoon 30 िकमी/घंटा और दू सरा आधा भाग
right angles to each other. Their Shift) 20 िकमी/घंटा की चाल से पूरा करता
speeds are 54 km/h and 72 km/h, (a) 6 km/h है । उसके ारा तय की गई दू री है :
respectively. After 20 seconds, (b) 3.45 km/h CHSL 19-03-2020 (Evening

e
the distance between them will (c) 4.23 km/h shift)
be: (d) 7 km/h (a) 140 km
दो कार एक ही थान से एक ही (b) 100 km
समय म एक दू सरे से समकोण पर Q35. A train covers a distance in (c) 120 km
चलना शु करती ह। उनकी चाल

l
30 min if it runs at a speed of 54 (d) 130 km
मशः 54 िकमी/घंटा और 72 km/h on an average. The speed at

Shift)
(a) 480 m
ac
िकमी/घंटा है । 20 सेकंड के बाद,
उनके बीच की दू री होगी:
CHSL 17-03-2020 (Evening
which the train must run to
reduce the time of the journey to
20 min is:
एक टे न 30 िमनट म एक दू री तय
करती है , यिद यह औसतन 54
SSC CGL 2019 TIER-II
Q38. A train of length 287m,
running at 80km/h, crosses
another train moving in the
opposite direction at 37km/h in
(b) 540 m िकमी/घंटा की चाल से चलती है । 18 seconds. What is the length of
(c) 720 m या ा के समय को 20 िमनट तक कम the other train ?
(d) 500 m करने के िलए टे न को िकस चाल से एक रे लगाड़ी िजसकी ल ाई 287
nn
चलना चािहए? मी, चाल 80 िकमी/घंटा है , दू सरी
Q33. Ravi starts for his school CHSL 18-03-2020 (Evening रे लगाड़ी को, िजसकी चाल 37
from his house on his cycle at 8 : Shift) िकमी/घंटा है ,
20 a.m. If he runs his cycle at a (a) none of these/ इनम से कोई नहीं 18 सेकंड म पार करती है । तो दू सरी
speed of 10 km/h, he reaches his (b) 81 km/h रे लगाड़ी की चाल ा है ?
school 8 minutes late, and if he (c) 60 km/h CGL 2019 Tier-II (15-11-2020 )
drives the cycle at a speed of 16 (d) 75 km/h (a) 300m
km/h, he reaches his school 10 (b) 298m
Pi

minutes early. The school starts Q.36. A car covered 150 km in 5 (c) 285m
at: hours. If it travels at one-third its (d) 289m
रिव अपने ू ल के िलए अपनी usual speed, then how much more
साइिकल से 8: 20 बजे अपने घर से time will it take to cover the same Q39. A and B start moving
िनकलता है । यिद वह 10 िकमी/घंटा distance? towards each other from X and Y,
की चाल से अपनी साइिकल चलाता 5 घंटे म एक कार 150 िकमी की दू री respectively, at the same time on
है , तो वह अपने ू ल 8 िमनट दे री से तय करती है । यिद यह अपनी the same day. The speed of A is
प ं चता है और अगर वह 16 सामा चाल से एक-ितहाई चाल से 20% more than the speed of B.
िकमी/घंटा की चाल से साइिकल या ा करे , तो समान दू री तय करने म After meeting on the way, A and
चलाता है , तो वह अपने ू ल 10 िकतना अिधक समय लगेगा? B take p hours and 7 51 hours,
िमनट पहले प ँ चता है । ू ल शु CHSL 19-03-2020 (Afternoon respectively to reach Y and X
होता है : shift) respectively. What is the value of
CHSL 18-03-2020 (Morning (a) 14 hours/ घंटे p?
Shift) (b) 10 hours/ घंटे

www.ssccglpinnacle.com support@ssccglpinnacle.com Ph. 09729327755, 09817390373 454


Days 49-52 Speed and Distance

s
se
s
la
_c
ob
A और B एक ही िदन को एक ही CGL 2019 Tier-II (15-11-2020 ) speed of A is 33km/hr, then the
bo
ah
समय म मशः X और Y से एक (a) 6 speed (in km/hr) of B is:
sm
ur
yo

दू सरे की ओर बढ़ने लगते ह। A की (b) 6.5 A और B एक ही िदन को एक ही


e/
t.m

चाल B की चाल से 20% अिधक है । समय म मश X से Y और Y से X


://

(c) 6.4
tp
ht

रा े म िमलने के बाद A और B, (d) 8 तक चलना शु करते ह। एक दू सरे


मशः Y और X तक प ँ चने के को पार करने के बाद, A और B
िलए p घंटे और 7 51 घंटे लेते है । p Q42. A train travelling at 36km/h अपने संबंिधत गंत तक प ं चने के
मान ा है ? crosses a pole in 25 seconds. How िलए मश: 5 94 घंटे और 9 घंटे लेते
CGL 2019 Tier-II (15-11-2020 ) much time (n seconds) will it ह। यिद A की चाल 33 िकमी/घंटा है ,
(a) 5 make to cross a bridge 350m long तो B की चाल (िकमी/घंटा म) है :
(b) 5.5 ? CGL 2019 Tier-II (16-11-2020 )
(c) 6 36 िकमी/घंटा की चाल से या ा करने (a) 22
(d) 4.5 वाली टे न 25 सेकंड म एक पोल को (b) 25 32
पार करती है । 350 मीटर लंबे पुल को (c) 24 31
Q40. A takes 2 hours more than B पार करने म िकतना समय (सेकंड
(d) 2
म) लगेगा?

e
to cover a distance of 40km. If A
doubles his speed he takes 1 21 CGL 2019 Tier-II (16-11-2020 )
Q.45 A delivery boy started from
hours more than B to cover 80 (a) 48
his office at 10a.m. to deliver an
km. To cover a distance of 90 km, (b) 56
article. He rode his scooter at a

l
how much time will B take (c) 72
speed of 32 km/h. He delivered
travelling at his speed? (d) 60
ac the article and waited for 15
A को 40 िकमी की दू री तय करने म minutes to get the payment. After
B से 2 घंटे अिधक लगते ह। यिद A Q43. Amita travels from her
the payment was made, he
अपनी चाल को दोगुना कर दे ता है तो house at 3 21 km/h and reaches her
reached his office at 11.25a.m.,
उसे 80 िकमी की दू री तय करने म B school 6 minutes late. The next travelling at a speed of 24km/h.
से1 21 घंटे अिधक लगते ह। 90 िकमी day she travels at 4 21 km/h and Find the total distance travelled
की दू री तय करने के िलए, B को reaches school 10 min early. by the boy.
अपनी चाल से या ा करने म िकतना What is the distance between her एक िडलीवरी बॉय सुबह 10 बजे
nn
समय लगेगा ? house and the school ? अपने कायालय से एक व ु प ँ चाने
CGL 2019 Tier-II (15-11-2020 ) अिमता अपने घर से 3 21 िकमी/घंटा के िलए िनकला। उसने अपने ू टर
(a) 1 81 hours/ घंटे की चाल से या ा करती है और 6 को 32 िकमी/घंटा की चाल से
(b) 1 hours/ घंटे
3 िमनट की दे री से अपने ू ल प ँ चती दौड़ाया। उ ोंने व ु को िदया और
8
(c) 1 hours/ घंटे
1 है । अगले िदन वह 4 21 िकमी/घंटा भुगतान ा करने के िलए 15 िमनट
6
की चाल से या ा करती है और ू ल तक इं तजार िकया। भुगतान िकए
(d) 1 hours/ घंटे
1
3
10 िमनट पहले प ँ चती है । उसके जाने के बाद, वह 24.2 िकमी/घंटा की
घर और ू ल के बीच की दू री ा चाल से या ा करते ए अपने
Pi

है ? कायालय म 11.25 बजे प ं चा।


Q41. A person has to cover a
CGL 2019 Tier-II (16-11-2020 ) लड़के ारा तय की गई कुल दू री का
distance of 160km in 15 hours. If
(a) 5.4km ात करे ।
he covers 54 of the distance in 32
(b) 5.6km CGL 2019 Tier-II (18-11-2020 )
of the time, then what should be (a) 32km
(c) 4.8km
his speed (in km/hr) to cover the (b) 30km
(d) 4.2km
remaining distance in remaining (c) 35km
time ? (d) 40km
Q44. A and B start moving from
एक को 15 घंटे म 160 िकमी
places X and Y and Y to X,
की दू री तय करनी होती है । यिद वह Q46. An athlete runs an 800 m
respectively, at the same time on
3 समय म 5 दू री को तय करता है ,
2 4
the same day. After crossing each race in 96 seconds. His speed (in
तो शेष समय म शेष दू री को तय other, A and B take 5 94 hours and km/h) is:
करने के िलए उसकी चाल
9 hours, respectively, to reach
(िकमी/घंटा म) ा होनी चािहए?
their respective destination. If the

www.ssccglpinnacle.com support@ssccglpinnacle.com Ph. 09729327755, 09817390373 455


Days 49-52 Speed and Distance

s
sse
la
_c
ob
एक धावक 96 सेकंड म 800 मीटर 74 िकमी/घंटा की चाल से चलने वाली CPO 23-11-2020 (Evening
bo
ah
की दौड़ लगाता है । उसकी चाल एक टे न x, 12 सेकंड म िवपरीत shift)
sm
ur
yo

(िकमी/घंटा म) है : िदशा से 52 िकमी/घंटा की चाल से (a) 24


e/
t.m

चलने वाली एक अ टे न y को पार


://

CGL 2019 Tier-II (18-11-2020 ) (b) 15


tp
ht

(a) 40 km/h करती है । यिद y की लंबाई x की (c) 20


(b) 20 km/h दो-ितहाई है , तो x की लंबाई (मीटर (d) 30
(c) 25 km/h म) ा है ?
(d) 30 km/h CPO 23-11-2020 (Morning Q52. A person covers a distance
shift) of 300 km and then returns to the
Q.47 A man travelled a distance (a) 168 starting point. The time taken by
of 42 km in 5 hours. He travelled (b) 252 him for the outward journey is 5
partly on foot at the rate of 6 (c) 210 hours more than the time taken
km/h and partly on bicycle at the (d) 200 for the return journey. If he
rate of 10 km/h. The distance returns at a speed of 10km/h more
travelled on foot is: Q50. A bus covers a than the speed of going, What
एक आदमी ने 5 घंटे म 42 िकमी की

e
50-kilometer distance in 1 hour was the speed (in km/h) for the
दू री तय की। उसने आं िशक प से 15 minutes, whereas the same outward journey?
6 िकमी/घंटा की चाल से पैदल या ा distance is covered by a car in 45 एक 300 िकमी की दू री तय
की और आं िशक प से 10 minutes, what is the ratio of the करता है और िफर शु आती िबंदु पर
िकमी/घंटा की चाल से साइिकल पर लौटता है । जाने की या ा के िलए

l
bus to the speed of the car?
या ा की। पैदल या ा की दू री है : एक बस 1 घंटे 15 िमनट म 50 उसके ारा िलया गया समय वापसी

(a) 10 km
(b) 12 km
(c) 18 km
(d) 15km
ac
CGL 2019 Tier-II (18-11-2020 ) िकलोमीटर की दू री तय करती है ,
जबिक वही दू री 45 िमनट म कार
ारा तय की जाती है । कार की चाल
से बस की चाल का अनुपात ा है ?
CPO 23-11-2020 (Evening
की या ा के िलए, िलए गए समय से 5
घंटे अिधक है । अगर वह जाने की
चाल से 10 िकमी/घंटा अिधक की
चाल से लौटता है , तो जाने की या ा
के िलए चाल (िकमी/घंटा म) ा है ?
shift) CPO 24-11-2020 (Morning
Q.48. A man walks at a speed of (a) 3 : 1 shift)
8km/h. After every kilometer, he (b) 5 : 3 (a) 20
nn
takes a rest for 4 minutes. How (c) 1 : 3 (b) 15
much time will he take to cover a (d) 3 : 5 (c) 25
distance of 6km? (d) 30
एक आदमी 8 िकमी/घंटा की चाल से Q51. A person travels a distance
चलता है । हर िकलोमीटर के बाद, of 300km and then returns to the Q53. A train travelling at the
वह 4 िमनट के िलए आराम करता starting point. The time taken by speed of x km/h crossed a 300m
है । 6 िकमी की दू री तय करने म उसे him for the outward journey is 5 long platform in 30 sec. And
िकतना समय लगेगा? hours more than the time taken overtook a man walking in the
Pi

CGL 2019 Tier-II (18-11-2020 ) for the return journey. If he same direction at 6km/h in 20 sec.
(a) 60 minutes/ िमनट returns at a speed of 10km/h more What is the value of x?
(b) 65 minutes/ िमनट than the speed of going, what is X िकमी/घंटा की चाल से या ा करने
(c) 70 minutes/ िमनट the average speed (in km/h) for वाली एक टे न ने 30 सेकंड म 300
(d) 69 minutes/ िमनट the entire journey ? मीटर लंबा ेटफॉम पार करती है
एक 300 िकमी की दू री तय और 20 सेकंड म 6 िकमी/घंटा की
SSC CPO 2019 करता है और िफर शु आती िबंदु पर चाल से उसी िदशा म चलने वाले एक
49. A train x running at 74 km/h लौटता है । जाने की या ा के िलए को पीछे छोड़ दे ती है । X का
crosses another train y running at उसके ारा िलया गया समय वापसी मान ात करे ।
52 km/h in the opposite की या ा के िलए, िलए गए समय से 5 CPO 24-11-2020 (Evening
direction in 12 seconds. If the घंटे अिधक है । अगर वह जाने की shift)
length of y is two-thirds that of x , चाल से 10 िकमी/घंटा अिधक की (a) 48
then what is the length of x (in चाल से लौटता है , तो पूरी या ा म (b) 60
m)? औसत चाल (िकमी / घंटा) ा है ? (c) 102

www.ssccglpinnacle.com support@ssccglpinnacle.com Ph. 09729327755, 09817390373 456


Days 49-52 Speed and Distance

s
sse
la
_c
ob
(d) 96 taken 2 hrs less for the same
bo
ah journey. What is the usual time
sm
ur
yo

Q54. A train x running at taken (in hours) by it to complete


e/
t.m
://

74km/hr crosses another train y the journey ?


tp
ht

running at 52km/h in the opposite एक टे न एक समान चाल से 400


direction in 12 seconds. If the िकमी की दू री तय करती है । यिद
length of y is two-third that of x, चाल 10 िकमी/घंटा अिधक होती तो
then what is the length of y(in m) उसी या ा के िलए टे न को 2 घंटे कम
? लगते । या ा को पूरा करने के िलए
74 िकमी/घंटा की चाल से चलने वाली इसके ारा (घंटों म) सामा प से
एक टे न x, 12 सेकंड म िवपरीत िकतना समय लगता है ?
िदशा म 52 िकमी/घंटा की चाल से CPO 25-11-2020 (Evening
चलने वाली एक अ टे न y को पार shift)
करती है । यिद y की लंबाई x की (a) 10
दो-ितहाई है , तो y की लंबाई (मीटर (b) 8
म) ा है ?

e
(c) 15
CPO 25-11-2020 (Morning (d) 12
shift)
(a) 168

l
(b) 200
(c) 180
(d) 252
ac
Q55. A takes 2 hours 30 minutes
more than B to walk 40km, If A
doubles his speed, then he can
make it in 1 hours less than B.
What is the average time taken by
A and B to walk a 40 km distance
nn
?
40 िकमी तक चलने म A को B से 2
घंटे 30 िमनट अिधक लगते ह। यिद
A अपनी चाल को दोगुना कर दे ता है ,
तो वह B से 1 घंटे कम लगाता है । A
और B ारा 40 िकमी की दू री तय
करने म औसत िकतना समय लगता
है ?
Pi

CPO 25-11-2020 (Morning


shift)
(a) 5 hours 15 minutes/ 5 घंटे 15
िमनट
(b) 7 hours 15 minutes/ 7 घंटे 15
िमनट
(c) 6 hours / घंटे
(d) 5 hours 45 minutes/ 5 घंटे 45
िमनट

Q56. A train covers 400km at a


uniform speed. If the speed had
been 10km/h more. It would have

www.ssccglpinnacle.com support@ssccglpinnacle.com Ph. 09729327755, 09817390373 457


Days 49-52 Speed and Distance

s
sse
la
_c
ob
a×b 30 2a×b
SOLUTION ⇒ × = ×1 ⇒ 50{ 5(s+25)−6s
s(s+25) }= 4
bo
b−a 60 2a−b
ah
sm
⇒ 2a-b = 4(b-a) ⇒ -50s+(125 × 50) = 4s(s+25)
ur
yo

Sol 1. (c) Let the original speed =


e/

⇒ 6a = 5b ⇒ 4s2 +150s-6250 = 0
t.m
://

s km/h, decreased speed = (s-16) ⇒ ba = 65


tp

⇒ 2s2 +75s-3125 = 0
ht

km/h
Let a = 5 unit and b= 6 unit ⇒ 2s2 +125s-50s-3125 = 0
We know that
Put the values in either of the
D=
S 1 ×S 2
xt ⇒ 2s(s +62.5) -50(s+62.5) = 0
S 1 −S 2 given equations
a×b 30
⇒ s = 25 km/h and f = s+25 ⇒
Here D = distance b−a × 60 = 15
⇒ 65×6
−5
30 = 15
× 60
25+25 = 50 km/h
s1 and s2 = two speeds ⇒ 15 unit =15
( s1 > s2 ) ⇒ 1 unit =1
Sol 7. (a)
t = time difference Speed of B (6 unit) = 6 x 1 = 6 Tricky approach :
km/h f −s
t= f hours
According to the given question
s(s−16) Sol 3. (c) Here, t = time of stoppage
s−(s−16) × 2 = 384
We know that, if time taken to f = speed without stoppage

e
s(s-16) = 384 × 8 s = speed with stoppage
cover same distance at speed S 1
s(s-16) = 48 × 8 × 8 t = 5050
−40
is T 1 and time taken at speed S 2
s(s-16) = 48 × 64 = 1 hours = 12 minutes
Clearly s = 64 km/h is T 2 then, 5

l
75 S1 T2
75% of original speed = 64 × =
100 S2 T1 Sol 8. (b)
= 48 km/h ansac Here S 2 = 16.8 km/h, T 1 = 49 h Original : New
8
and T 2 = 8 h Speed 4 : 3
Sol 2. (a) Time 3 : 4

S1 8
Let the speed of A and B be a and ⇒ 16.8 = 49/8 According to the question
b respectively, ⇒ S1 = 8 × 16.8 = 19.2 km/h
7 (4-3) unit = 18 minutes
ATQ: 15 15 1
a − b = 2 ----- (i) Original Time (3 unit) = 3 x 18 =
15 = 1 ----- (ii)
15
− 2a
b Sol 4. (b) 54
Adding (i) and (ii) Distance between A and B = 800
nn
15 3
2a = 2
⇒a=5 km Sol 9. (b)
Putting the value of a in eq. (i), Time taken to cover this distance Distance travelled by A in 4
800 = 8 hours
= 60+40
we get hours = 4 x 4 = 16 km
3 − 15 1 Distance covered by train X in 8 Relative speed = (10-4) km/h
b = 2
⇒ b = 6 km/h hours = 8 x 40 = 320 km Time taken by B to catch A =
16 16
10−4 = 6 h

Alternate : Sol 5. (b) Distance covered by B before


Let the speed of A = a km/h and b Let the speed of the woman = w catching A = 16
6 x 10 = 26.67 km
Pi

= b km/h km/h
We know that ⇒ (44-8) × 15 = (44+w) x 10 Sol 10. (c)
S 1 ×S 2 ⇒ 540 = 440 + 10w Average speed of the journey =
D= S 1 −S 2 xt
⇒ w = 10 km/h 2×s1 ×s2 2×12×9
Here D = distance s1 +s2 = 12+9 km/h
s1 and s2 = two speeds ( Total time = 2 31 h
Sol 6. (b)
s1 > s2 ) Total distance travelled = 2×12×9
Let speed of slower train = s and 12+9
t = time difference faster train = f × 7
3 = 24 km
According to first condition According to the question
a×b 30
b−a × 60 = 15 ………..(1) f = (s+25) km/h Sol 11. (c)
According to the second Also Let the length of each train = a
condition 250 - 300 = 4
s f Relative speed = (54-42) = 12
2a×b
2a−b × 1 = 15 …………(2) ⇒ 250 - 300 =4 km/h
s s+25
From (1) and (2) Total distance covered = a+a = 2a

www.ssccglpinnacle.com support@ssccglpinnacle.com Ph. 09729327755, 09817390373 458


Days 49-52 Speed and Distance

s
sse
la
_c
ob
According to the question The time in which train covered Sol 18.(a)
bo
ah 5 x 63
2a = 12 × 18 its length and length of the Let PQ = 1 unit
sm
ur
yo

⇒ a = 105 m platform =55 seconds ⇒ QR = 2 unit


e/
t.m
://

The time in which Train covered ⇒ PR = 1+2 = 3 unit


tp
ht

Sol 12. (b) its length = 24 seconds Average speed of the journey =
Let the distance = 4D ⇒ Time in which train covered T otal distance = 3+3
T otal time 1 2
+ + 3 μ 3μ μ/2
Time = 3 unit platform = 55-24 = 31 seconds 6 18μ
= =
According to the question Speed of train = 360
24 = 15 m/s
3+2+18

23

3D D Length of platform = 15 x 31 =
2 - 1 = 80
⇒ D= 160 unit 465 metre Sol 19.(b)
According to the question 40% = 52
Total time = 4D 4×160 Sol 14. (a) Let total distance = 5 unit
80 = 80 = 8 unit
Relative speed = (72+108) = 180 Distance covered at the speed of
Time for which amit travelled at x 5 m/s
km/h or 180 × 18 60 km/h = 2 unit
km/h = 8 × 31 = 38 unit
Total length of two trains = 180 Distance covered at the speed of
Now, x = 160
8/3 = 60 km/h

e
5 × 10 = 500 metre
× 18 40 km/h = 3 unit
Length of each train = 500 = 250 ⇒ average speed of the journey =
Alternate : 2
5 = 5×120 = 600
m 2
60+3 40
4+9 13

Time taken by first train to cross

l
Let total time = 3 unit
Total distance = 80 x 3 = 240 unit the platform = 250+350
72× 5
= 30 Sol 20. (d)
18

km

= 240 × 100
ac
Distance covered in 2 unit of time
75 = 180 unit km

Speed at which this distance was


covered = 180 unit
2 unit = 90 km/h
seconds

Sol 15. (a)


Let the speed of first train = 2
unit and second train = 5 unit
Initial speed =

35+2 = 37 and so on.


First term = 35
35
1
Distance covered in 1st hour = 35
Distance covered in 2nd hour =
= 35 kmph

Speed of first train = 350 Common difference = 37-35 = 2


Time for which amit travelled at 5 = 70
km/h Number of terms = 12
90 km/h = 2 unit
Total distance covered = 12
nn
Time for which amit travelled at x 2 unit = 70 2

km/h = 1 unit 1 unit = 35 [2(35)+(12-1)2]


5 unit = 5 x 35 = 175 km/h = 552 km

Sol 16. (c) SSC CGL TIER II


Ball 1 : Ball 2
Time 0.6 : 1 Sol 1.(a)
3 : 5 Let the total time = 5 unit
Speed 5 : 3 Total distance to be travelled = 60
Pi

According to the question x 5 = 300 unit


3 unit = 96 km/h According to the question
1 unit = 32 Distance to be travelled in 3 unit
time = 300 × 100 40 = 120 unit
b = 90-80 = 10 5 unit = 160 km/h
Desired Speed = 120 = 40 kmph
⇒ a = 10 x 2 = 20 3
Sol 17. (b)
also
Total distance to be covered = Sol 2.(d)
a = 80 - x
⇒ 20 = 80 - x 400 +1200+200 = 1800 metres Let ‘t’ metres be the length of the
Speed of the train = 72 km/h or train.
⇒ x = 60 km/h 5 = 20 m/s
72 × 18 According to the question
t+200 = 30
Sol 13. (d) Time taken to cover this distance x× 518
= 1800
20 = 90 s or 1.5 m 5x )
t+200 = 30 ( 18
⇒t= 150x -200 ……(1)
18

www.ssccglpinnacle.com support@ssccglpinnacle.com Ph. 09729327755, 09817390373 459


Days 49-52 Speed and Distance

s
sse
la
_c
ob
Also t = time difference Here D = distance
bo
ah t = 20 s1 and s2 = two speeds ( s1 > s2
sm
ur

5
(x−6)× 18
yo

Let D be the distance )


e/

⇒ t = (x-6) x 100
t.m

18
://

⇒ D = 3030×24 × 18−10 = 16 km
t = time difference
tp

Put the value of t −24 60


ht

150x -200 = (x-6) x 100 According to the question


18 18 s(s+16)
150x 100x = 200 - 100
Sol 7.(c) { (s+16)−s }x 1 = 96
18 - 18 3 Let the speed of train A = a km/h ⇒ s(s+16) = 96 x 16
x= 500 x 18 = 60
3 50 and train B = b km/h ⇒ s(s+16) = 48 x 32
According to the question
⇒ s(s+16) = (32+16) x 32
Sol 3.(c) 416 - 416 = 8 ……….(1)
a b 3 Clearly s = 32 km/h
We know that, if time taken to Also ⇒ speed of faster train = 32+16 =
cover same distance at speed S 1 416 - 416 = 4 …………(2)
b 2a 3 48 km/h
is T 1 and time taken at speed S 2 Adding (1) and (2)
is T 2 then, 416 - 416 + 416 - 416 = 8 + 4
a b b 2a 3 3 Sol 2. (a)

S1 T2
= 1 )=4
416( a1 - 2a Tricky approach :

e
S2 T1
1 1 f −s
Here S 2 = 28 km/h, T 1 = 49 h ⇒ 2a = 104 t= f hours
8
and T 2 = 8 h ⇒ a = 52 Here, t = time of stoppage
f = speed without stoppage

S1 8
⇒ 28 = 49/8

l
Sol 8.(c) s = speed with stoppage
⇒ S1 = 8 × 28 = 32 km/h
7 Let the total distance = 50 unit 70
t = 70−56

Sol 4. (d)
ac
Let the length of train A = L
⇒ the length of train B = 3L
Relative speed of the trains =
Time taken to cover first 40 % of
the distance =
40
50× 100
8
Time taken to cover first 40 % of
the remaining distance =
12
= 20
8

9
40
30× 100
=
= 1
5 hours = 12 minutes

Sol 3. (c)
Tricky approach :
t= f −s
hours
(50+58) km/h f
9
According to the question Time taken to cover the Here, t = time of stoppage
3L = 15 remaining distance = 50−12
20−12 = f = speed without stoppage
nn
(50+58)× 5 18
18 s = speed with stoppage
⇒ 3L = 15 x 30 12 65−52
50 t = 65
⇒ L = 150 m Average speed = 20 12 18
+ 9 + 12
= 9 83
8 = 1 hours = 12 minutes
5

Sol 5. (a) Sol 9.(a)


Sol 4.(a)
Let speed of train X = x Original : New
Tricky approach :
⇒ Speed of train Y = x+10 Speed 5 : 3 f −s
t= hours
Relative speed = x+x+10 Time 3 : 5 f
Pi

According to the question According to the question Here, t = time of stoppage


396 11 f = speed without stoppage
(x+x+10) = 2 (5-3) unit = 1 h 40 min
1 unit = 50 min s = speed with stoppage
72 = 2x+10 72
t = 72−60
⇒ x = 31 km/h Original time (3 unit) = 3 x 50 =
1
Speed of train y = 31+10 = 41 150 min or 2 21 h = 6 hours = 10 minutes
km/h Practice Questions
Sol 5. (b)
Sol 6.(b) Sol 1. (b) Tricky approach :
f −s
We know that Let the speed of slower train = s t= f hours
D=
S 1 ×S 2
xt km/h and faster train = (s+16) Here, t = time of stoppage
S 1 −S 2
km/h f = speed without stoppage
Here D = distance
We know that s = speed with stoppage
s1 and s2 = two speeds ( S 1 ×S 2
D= S 1 −S 2 xt t = 8080
−72
s1 > s2 )

www.ssccglpinnacle.com support@ssccglpinnacle.com Ph. 09729327755, 09817390373 460


Days 49-52 Speed and Distance

s
se
s
la
_c
ob
= 1 hours = 6 minutes Sol 11. (b) We know that
bo
10
ah Let the original speed = s km/h S 1 ×S 2
sm
D= xt
ur

S 1 −S 2
yo

Sol 6. (a) ⇒ new speed = (s-5) km/h


e/
t.m

Here D = distance
://

Tricky approach : We know that


tp

s1 and s2 = two speeds (


ht

f −s S 1 ×S 2
t= f hours D= S 1 −S 2 xt s1 > s2 )
Here, t = time of stoppage Here D = distance t = time difference
f = speed without stoppage s1 and s2 = two speeds ( ⇒ s(s+8)
x 10 = 60
(s+8)−s
s = speed with stoppage s1 > s2 )
80−64
⇒ s(s+8) = 48
t = 80 t = time difference
1
⇒ s(s+8) = 4 x 12
= 5 hours = 12 minutes According to the question
(45+3)
⇒ s(s+8) = 4 x (4+8)
45
sx 60 = (s-5) x 60 45s = 48s - Clearly s = 4 km/h
Sol 7. (b) 240
Original : New s=80 km/h Sol 14. (d)
Speed 5 : 3 ⇒ Desired distance = 80 × 45 or
60 Let original speed = s km/h and
Time 3 : 5

e
(45+3)
(s-5) x 60 = 60 km increased speed = (s+8) km/h
According to the question
Alternate : We know that
(5-3) unit = 20 minutes S 1 ×S 2
Original : New D= S 1 −S 2 xt
1 unit = 10 minutes
Original time (3 unit) = 3 x 10 = Time 45 : 48 Here D = distance

l
30 15 : 16 s1 and s2 = two speeds
ac Speed 16 : 15 t = time difference
Sol 8. (c) According to the question ⇒ s(s+8)
(s+8)−s x 10 = 25
Original : New (16-15) unit = 5 km/h
1 unit = 5 km/h ⇒ s(s+8) = 20
Speed 7 : 5 ⇒ s(s+8) = 2 x 10
Time 5 : 7 16 unit = 80 km/h
15 unit = 75 km/h ⇒ s(s+8) = 2 x (2+8)
According to the question
⇒ Desired distance = 80 × 45 or Clearly s = 2 km/h
(7-5) unit = 10 minutes 60
(45+3)
1 unit = 5 minutes (s-5) x = 60 km
nn
60
Sol 15. (c)
Original time (5 unit) = 5 x 5 = 350
Speed of first train = 5 = 70
25 Sol 12. (c)
km/h
Let the original speed = s km/h
Sol 9. (a) Train 1 : Train 2
⇒ new speed = (s+10) km/h
Original : New Speed 2 : 5
We know that
Speed 9 : 7 S 1 ×S 2 According to the question
D= S 1 −S 2 xt 2 unit = 70 km/h
Time 7 : 9
Here D = distance 1 unit = 35 km/h
According to the question
Pi

s1 and s2 = two speeds ( Sum of the speed of both the


(9-7) unit = 10 minutes
1 unit = 5 minutes s1 > s2 ) trains (2+5) unit = 7 x 35 = 245
Original time (7 unit) = 7 x 5 = t = time difference km/h
35 Now,
s(s+10)
x 3 = 360 Sol 16. (d)
(s+10)−s 350
Sol 10.(b) Speed of first train = 5 = 70
s(s+10) = 120 x 10
A : B ⇒ s(s+10) = 40 x 30 km/h
Speed 3 : 5 Train 1 : Train 2
⇒ s(s+10) = (30+10) x 30
Time 5 : 3 Speed 2 : 5
⇒ s = 30 km/h
According to the question
(5-3) unit = 24 minutes According to the question
Sol 13. (a)
1 unit = 12 minutes 2 unit = 70 km/h
Let original speed = s km/h and
B’s time (3 unit) = 3 x 12 = 36 1 unit = 35 km/h
increased speed = (s+8) km/h

www.ssccglpinnacle.com support@ssccglpinnacle.com Ph. 09729327755, 09817390373 461


Days 49-52 Speed and Distance

s
s se
la
_c
ob
Difference of the speed of both Had length of the second train ⇒ Time taken to cover 490 km =
bo
ah
the trains (5-2) unit = 3 x 35 = been same as the first train, time 35
sm
min
ur

2
yo

105 km/h taken by it to cross the person =


e/

⇒ Time taken to cover 1470


t.m
://

20 = 10 seconds 35
(980+490) km =35+ min
tp

2
ht

2
Sol 17.(a) Up Train : Down
Speed of first train = 250 = 50
5 Train = 105 min or 87 hours
2
km/h Time 8 : 10
Train 1 : Train 2 Speed 10 : 8
Sol 24.(a)
Speed 2 : 5
We know that
__________________ S 1 ×S 2
D= S 1 −S 2 xt
According to the question Distance 80 : 80
2 unit = 50 km/h Here D = distance
1 unit = 25 km/h Distance travelled = length of the s1 and s2 = two speeds (
Difference of the speed of both train s1 > s2 )
the trains (5-2) unit = 3 x 25 = 75 Length of down train = 80 unit t = time difference

e
km/h Length of up train = 2 x 80 = 160 Let D be the distance travelled
unit ⇒ D = 64 × 48 12−3
64−48 × 60 = 28.8 km
Sol 18. (a) Relative speed of the trains = Time taken by train to travel this
Speed of first train = 250 = 50 10+8 = 18 unit
5 distance = 28.8
48 = 0.6 hours or 36

l
km/h Desired time = 80+160
18 = 13 3
1
min
Train 1 : Train 2
ac seconds The right time for the train to
Speed 2 : 5 cover the journey = 36-12 = 24
According to the question Sol 21. (c) min
2 unit = 50 km/h Speed of first ball = 90 km/h or Alternate :
1 unit = 25 km/h 5 = 25 m/s
90 × 18 Let ‘t’ minutes be the desired
Sum of the speed of both the Speed of 2nd ball = 162 km/h or time.
trains (2+5) unit = 7 x 25 = 175 5 = 45 m/s
162 × 18 According to the question
km/h Distance covered by the second 48 x (t+12) = 64 (t+3)
nn
ball = 45 x1 = 45 m 48t + 576 = 64t + 192
Sol 19. (b) Time taken by first ball to travel 384 = 16t
Let the speed of bus = b and 45 = 9 sec
this distance = 25 t = 24
5
speed of car = c
We know that Sol 25. (c)
S 1 ×S 2
Sol 22. (a)
D= S 1 −S 2 xt Speed of the train = 76 km/h or We know that
S 1 ×S 2
Here D = distance 5 m/s
76 × 18 D= S 1 −S 2 xt
s1 and s2 = two speeds ( Time taken = 36 seconds Here D = distance
Pi

s1 > s2 ) Length of the train = 76 × 5 ×


18 s1 and s2 = two speeds (
t = time difference 36 = 760 m s1 > s2 )
b×c × 15+10
60 = 25
⇒ c−b t = time difference
⇒ b×c = 60 km/h Sol 23. (d)
c−b
5 =
= 60 × 18 50
3 ……..(1) Distance covered = 980 km Let D be the distance travelled
35 h
Time taken = 35 min or 60
the difference of the reciprocals ⇒ D = 52 × 65 15−5
65−52 × 60 = 3 km
130

Speed of the train = 980 = 28 x


of average speeds of the bus and 35/60 Time taken by train to travel this
the car = b1 - 1c = cb×c
−b
60 kmph distance = 3130 10
× 52 = 12 hours or 50
1470 7
From equation (1) Desired time = 28×60 = 8 hours min
c−b = 1 = 3 The right time for the train to
b×c 50 50
3
Alternate : cover the journey = 50-15 = 35
Sol 20. (a) Time taken to cover 980 km = 35 min
min Alternate :

www.ssccglpinnacle.com support@ssccglpinnacle.com Ph. 09729327755, 09817390373 462


Days 49-52 Speed and Distance

s
sse
la
_c
ob
Let ‘t’ minutes be the desired Let ‘t’ minutes be the desired 5 unit = 13 hour or 195 min
bo
4
ah
time. time.
sm
Time saved 1(5-4) unit = 39 min
ur
yo

According to the question According to the question


e/
t.m
://

52 x (t+15) = 65 (t+5) 36 x (t+15) = 45 (t+4) Sol 32. (c)


tp
ht

52t + 780 = 65t + 325 36t + 540 = 45t + 180


455 = 13t 360 = 9t
t = 35 t = 40

Sol 26. (b) Sol 28. (b)


We know that Time taken = 1h 10 min or 7 h
6
S 1 ×S 2
D= S 1 −S 2 xt
Here D = distance
s1 and s2 = two speeds ( 4-3 = 1unit = 9
60
s1 > s2 ) 180 unit = 180 × 9 = 27 km
60
t = time difference

e
Let D be the distance travelled Alternate :
⇒ D = 80 × 60 15−7
80−60 × 60 = 32 km According to the question We know that
Time taken by train to travel this (3+4) unit = 67 S 1 ×S 2
D= S 1 −S 2 xt
32 = 8 hours or 32
distance = 60 1 unit = 1

l
15 6
1
Here D = distance
min Distance (24 unit) = 24 x 6 =4
ac s1 and s2 = two speeds (
The right time for the train to km
s1 > s2 )
cover the journey = 32-15 = 17
min t = time difference
Sol 29. (a)
Alternate : Distance travelled = 1.7 km or
Let ‘t’ minutes be the desired Let D be the desired distance.
1700 60×45 × 9 = 27 km
time. D = 60 −45 60
Time taken =
According to the question 1700
8+9 = 100 sec = 1min 40sec
60 x (t+15) = 80 (t+7) Sol 33. (b)
nn
Average speed = 15+15 = 2.5
60t + 900 = 80t + 560 Sol 30. (b) 7+5
340 = 20t Let the original speed = 5 unit
t = 17 Sol 34. (c)
New speed = 9 unit 31.5
Total distance = 80 × 60 + 75 ×
Sol 27. (d) 16
60 = 62 km
We know that
S 1 ×S 2
D= S 1 −S 2 xt Sol 35. (b)
Speed of train = 500+100 = 24m/s
Pi

Here D = distance 25
Required time = 380+100 = 20
s1 and s2 = two speeds ( 24
s1 > s2 ) seconds
t = time difference According to question
9-5 unit = 4 unit = 30 Sol 36. (a)
Let D be the distance travelled 1 unit = 30
4
9 unit = 30 × 9 = 67.50 minute
⇒ D = 36 × 45 15−4
45−36 × 60 = 33 km 4
Time taken by train to travel this
33 = 11 hours or 55
distance = 36 12
Sol 31. (a)
min Case 1 : Case 2
The right time for the train to Speed 52 : 65
cover the journey = 55-15 = 40 4 : 5
Time 5 : 4 According to the question
min
According to the question 8 unit = 24
Alternate :

www.ssccglpinnacle.com support@ssccglpinnacle.com Ph. 09729327755, 09817390373 463


Days 49-52 Speed and Distance

s
sse
la
_c
ob
1 unit = 3 Sol 43. (b)
bo
ah
5 unit = 15 min Let the length of the train = k Sol 46. (b)
sm
ur
yo

According to the question Total distance covered = 900 +


e/
t.m
://

k+600 = k+900 600 = 1500 meters


tp

50 60
ht

Sol 37. (d) Average speed = 6k + 3600 = 5k + 4500 Time taken = 300 seconds
5+7 3 /
500+500 = 250 m min or 250 ×
3
3
50 ⇒ k = 900 metres Speed of the train = 1500
300 = 5 m/s
= 5km/h Speed of the train = 900+600
50 = Required time = 600
5 = 120
900+900 = 30 m/s seconds or 2 minutes
60
Sol 38. (d) Average speed = Required speed = 30 x 18 = 108
5
210 198
60
+ 66
6.5 /
210+198 = 408 = 62.8 km h
km/h Sol 47. (d)
Total time taken = 3 6 + 9 =
+ 15
(approx) Alternate : 9 18
37 hours
Distance covered in 60-50 30
Sol 39. (d) Required time = seconds = 900-600 Total distance = 3+6+9 = 18 km
230+750 = 49 s
72× 5 Speed of train = 300
10 = 30 m/s or Required speed = 3718/30 = 540
37
18

30 x 18 km/h = 108 km/h km/h

e
5
Sol 40. (a) Length of the train = (30x50)-600
Total time taken = 15 25
30 + 10 or (30x60)-900 = 900 metres Sol 48. (d)
Required speed = 15+25 = 40
15 25
Let the length of the train = t
+ 3
meters

l
30 10
Sol 44. (b)
km/h
Distance covered in till 11 a.m. (2 Time taken to cover this length =

Sol 41. (c)


ac
Time taken by the man to cover
the distance from P to Q = 8
hours
Time taken by the man to cover
hours) = 30 x 2 = 60 km
Relative speed = (30+45) = 75
km/h
Both will meet after 240
hours
75 = 5
16
1 minute or 60 seconds
Length of the new train = 2t
Total distance to be covered = t +
2t = 3t
Required time = 3 x 60 = 180
Distance covered by the other seconds
the distance from Q to P = 7
hours train = 16
5 x 45 = 144 km
Sol 49. (d)
nn
Man 1 : Man 2 Distance covered by the first train
Average speed of the car = 600
Time 8 : 7 = 300-144 = 156 km
meters/minute or 10 m/s
Speed 7 : 8 Desired ratio = 156 : 144
Speed of the runner = 100 125
9.6 = 12
x = 13 : 12
Desired difference = 125 -10 =
_______________ 12
5 m/s
Distance 56 : 56 Sol 45. (b) 12

Distance covered by the first man Time taken by Raman to reach Q Sol 50. (b)
in 2 hours = 7 x 2 = 14 unit = 15 5
9 = 3 hours Speed of the car = 18
24/60 = 45
Pi

Remaining distance = 56-14 = 42 Distance covered by Alok in this km/h


unit time = 6 x 35 = 10 km Speed of the bus = 2 x 45 = 90
Relative speed = 8+7 = 15 unit Relative speed of Raman and km/h
Time taken to cover this distance Alok = 6+9 = 15 km/h Time taken by bus = 135 1
90 = 1 2
42 = 2 hours and 48 minutes
= 15 Distance to be covered with this hours or 30 minutes
⇒ Both will meet at = 8:00 AM + speed = 15-10 = 5 km
2 hours and 48 minutes = 10:48 Time taken to cover this distance Sol 51. (b)
5 = 1 hours
= 6+9
AM 3 Distance of 600 meters is covered
Distance covered by Alok in this in 5-3 = 2 minutes
Sol 42. (b) period = 6 x 31 = 2 km Speed of the train = 2 600
× 60 = 5
Speed of the car = 300 = 50 m/m 18
6 Distance covered by Raman in m/s or 5 x 5 = 18 km/h
6
Required speed = 50 x 100 =3 this period = 9 x 31 = 3 km Length of the train = 5 x (3 x 60)
km/h Required distance (PR) = 10+2 or = 900 meters
15-3 = 12 km

www.ssccglpinnacle.com support@ssccglpinnacle.com Ph. 09729327755, 09817390373 464


Days 49-52 Speed and Distance

s
sse
la
_c
ob
Sol 52. (b) Time taken = 15 minutes or 1

bo
4
ah
Average speed of the man = Sol 59. (d)
sm
hours
ur
yo

2×30×20 = 24 km/h Speed of the train = 72 km/h or 1


Total distance = 108 x 4 = 27 km
e/
t.m

30+20
://

5 = 20 m/s
72 x 18 27×60 = 180
tp

Increased speed =
ht

9
Sol 53. (b) Length of the train = 20 x 35 = km/h
Speed of the car = 36 km/h 700 meters Desired increase = 180 - 108 = 72
Speed of the bus = 36 x 51 = 7.2 Required time = 1100+700
20 = 90 km/h
km/h or 2 m/s seconds or 1.5 minutes
Required time = 900 = 450
2 Sol 65. (a)
seconds or 7 minutes 1 Sol 60. (d) A:B
2
Required average speed = Distance 100 : 90
2×90×60 = 72 km/h
Sol 54. (c) 90+60 B:C
Length of the train = 700 meters Distance 100 : 90
Time taken = 35 seconds Sol 61. (a) Balancing the ratio for B
Speed of the train = 700 Time taken by first train = 5 A : B: C
35 = 20 m/s

e
Required Time = 700+740 = 72 hours Distance 1000 : 900 : 810
20
Time taken by other train = 7
seconds or 1 minute and 12
hours ⇒ In a race of 1000 meters A
seconds
First train : Second beats C by 190 meters

l
Train So, in race of 100 meters A beats
Sol 55. (d)
ac Time 5 : 7 C by 19 meters.
Total time taken = 4+6 = 10 hours
Speed 7 : 5
Desired average = 200+600
10 = 80
x ___________ Sol 66. (d)
km/h
Distance 35 : 35 Speed of fighter plan = 1440
Sol 56. (b) 5 = 400 m/s
Distance covered by first train in km/h or 1440 x 18
A:B
2 hours = 7 x 2 = 14 unit Distance covered in one seconds
Speed 3 :4
Relative speed of two trains = = 400 x 1 = 400 meters
Time 4 :3
7+5 = 12 km/h
According to the question
nn
Time by both trains to meet = 12 21
Sol 67. (a)
(4-3) unit = 10 minutes 7 1
= 4 hours or 1 hours and 45 Time taken to cover 1 km = =
3 unit = 3 x 10 = 30 minutes 12
minutes 300 seconds or 5 minutes
⇒ Both trains will meet at 8 : 00 Time taken to cover 8 km = 8 =
Sol 57. (d) 12
Speed of the train = 80 km/h AM + 1 hours and 45 minutes = 2400 seconds or 40 minutes
Time taken = 0.75 minutes or 1 9 : 45 AM ⇒ the person will stop 7 times
80
hours after every 5 minutes.
1 Sol 62. (b) Total time of stoppage = 7 x 4 =
Distance covered = 80 x =1
Pi

80
According to the question 28 minutes
km or 1000 meters
1000
60 = 2×40×x
40+x Total time of journey = 40+28 =
Length of the platform = 2 =
240 + 6x = 8x 68 minutes
500 meters
⇒ x = 120 km/h
Sol 68. (a)
Sol 58. (c)
Sol 63. (a)
Distance covered by bus = 1.8 km
Distance covered after repairing = Original :
or 1800 meters
72 x 6 = 432 km New
Time taken = 3 minutes or 180
Required time = 5×432
54 = 40 km/h
Speed 10 :
seconds
13
Speed of the bus = 1800
180 = 10 m/s
Sol 64. (c) Time 13 :
Speed of the athlete = 200 = 8 m/s
25 Speed of the train = 108 km/h 10
Required difference = 10-8 = 2 According to the question
m/s (13-10) unit = 12 minutes

www.ssccglpinnacle.com support@ssccglpinnacle.com Ph. 09729327755, 09817390373 465


Days 49-52 Speed and Distance

s
sse
la
_c
ob
1 unit = 4 minutes Required difference = 70-60 = 10
bo
ah
13 unit = 52 minutes or 0.52 km/h Sol 79. (a)
sm
ur
yo

hours According to the question


e/
t.m
://

Sol 74. (a) 2 × 60 + 2 × 50 + 2 × x = 6 x 52


tp
ht

Sol 69. (b) Tricky approach : 220 + 2x = 312


Circumference of the wheel = π d f −s
t= f hours ⇒ x = 46
= 22
7 x 56 = 176 Here, t = time of stoppage
Distance covered in one second = f = speed without stoppage Sol 80. (c)
7 x 176 = 1232 cm s = speed with stoppage Distance = 225 meters
Speed of the bus = 1232 cm/sec 72
t = 72−60 Speed of the man = 45 km/h or 45
5 = 12.5 m/s
x 18
= 1 hours = 10 minutes
6
Sol 70. (b) Desired time = 225
12.5 = 18 seconds
Total distance covered along with
Sol 75. (b)
platform by the train = 200+160 = 3 Sol 81. (a)
30% = 10
360 meters Total distance covered =
Original : New

e
Time taken = 18 seconds 600+1200 = 1800 meters
Speed 10 : 13
Speed of the train = 360
18 = 20 m/s Time = 36 seconds
Time 13 : 10
Let the speed of other train = x Speed of the train = 1800
According to the question 36 = 50 m/s
m/s Desired time = 600+2200 = 56
3 unit = 15 minutes

l
50
According to the question
160 + 160 = 9
1 unit = 5 minutes seconds
20 + x

⇒ 140 = 9x
⇒ x = 140
9
ac
⇒ 320 = 180 + 9x

Desired ratio = 20 : 140


9
13 unit = 65 minutes

Sol 76. (a)


Total distance = 360 + 160 + 200
= 720 km
Sol 82. (b)
Speed of car = 20 m/s = 20 x
72 km/h
Time = 2.5 hours
18
5 =

=9:7 Total Time = 360 160 200


72 + 12.8 + 16 = Distance covered = 72 x 2.5 =
30 180 km
Sol 71. (c) Required average speed = 720 =
nn
30
Let the speed of car on return 24 km/h or 24 x 5 = 6.67 m/s Sol 83. (a)
18
journey = x km/h Average speed of the journey =
According the question 2×18×12 = 72 km/h
Sol 77. (d) 18+12 5
64 = 2 160
× 160 ×x
+x Plane 1 : Plane 2 Time = 2 hours and 55 minutes or
⇒ x = 40 km/h 35
Speed 7 : 15 12 hours
1050 Total distance covered = 72 x 35
Speed of plane 1 = 3 = 350 5 12
Sol 72. (d) km/h = 42 km
1650
Speed of the first plane = =
Pi

⇒ One way journey = 42 = 21


3 According to the question 2
550 km/h 7 unit = 350 km/h km
11 unit = 550 km/h 1 unit = 50 km/h
1 unit = 50 km/h 15 unit = 750 km/h Sol 84. (c)
18 unit = 900 km/h Relative speed of the trains = 72
Required speed = 900 x 5 = 250
18 Sol 78. (b) + 63 = 135 km/h or 135 x 18 5 =

m/s Let the person travelled for t 37.5 m/s


hours at the speed of 40 km/h. Time = 18 seconds
Sol 73. (b) According to the question Total distance covered = 37.5 x
Total distance = 420 km 40 x t = 60 x (16-t) 18 = 675
Time = 7 hours 40t = 960 - 60t Let the length of train X = 3 unit
Speed of the car = 420
7 = 60 km/h t = 9.6 ⇒ And train Y = 2 unit
To cover the distance in 6 hours Total Distance = 40 x 9.6 + 60 x According to the question
speed of the car = 420 6.4 = 768 km
6 = 70 km/h (3+2) unit = 675

www.ssccglpinnacle.com support@ssccglpinnacle.com Ph. 09729327755, 09817390373 466


Days 49-52 Speed and Distance

s
sse
la
_c
ob
1 unit = 135 Total Distance = 75+60+90 = 225 x = 80
bo
ah
3 unit = 3 x 135 = 405 meters km x
sm
Total Distance = 3.75x + x + =
ur

2
yo

75 + 60 + 90 =
Total time taken = 25
e/

5.25x
t.m

20 15
://

Sol 85. (c) 12 hours Required distance = 5.25 x 80 =


tp
ht

Relative speed of A and B = Required speed = 225 = 18.75 420 km


12
10+20 = 30 km/h or 30 x 185 m/s
km/h
Distance covered in 5 minutes = Sol 96. (a)
5 x 5 x 60 = 2500 meters
30 x 18 Sol 92. (d) We know that, if time taken to
Clearly they are 2500 meters Distance = 10 km cover same distance at speed S 1
apart. Time = 12 minutes or 1
5 hours is T 1 and time taken at speed S 2
⇒ speed of the bus = 10
1/5 = 50 is T 2 then,
Sol 86. (b)

km/h S1 T2
S2 = T1
According to the question New speed = 50-25 = 25 km/h
2xA=3xB 10 hours or 24 Here S 2 = 48 km/h, T 1 = 2.7 h
Required time = 25
A:B and T 2 = 1.2 h
minutes

e
Speed 3: 2

S1 1.2
⇒ 48 = 2.7
Time 2:3
⇒ S1 = 2 × 48 = 32 km/h
Now, 3
Sol 93. (a)
(3-2) unit = 42 minutes 390
Speed of the person = = 120

l
Time taken by B (3 unit) = 126 3.25 Sol 97. (c)
minutes km/h Let the length of train A = 5L and
Sol 87. (c)
ac
Let the speed of one train = x
km/h and other train y km/h
According to the question
(x-y) x 60 = (x+y) x 12
= 120 x

Sol 94. (b)


5
18 = 33 31 m/s

Perimeter of the wheel = π d


= 22
7 x 70 = 220
length of train B = 3L
Let Speed of train A = 2 unit and
train B = 3 unit
According to the question
5L + 3L = 5
2+3 2
L= 25 unit
60x - 60y = 12x + 12y cms 16
48x = 72y Total distance covered = 600 x Length of train A = 5 x 25
16 = 125
16
nn
⇒ x:y = 3:2 220 cms = 1320 meters unit
Going through the options only Time = 1 minutes or 60 seconds Required time = 125 = 125
16 × 2 32
option C satisfy this condition. Speed of the vehicle = 1320
60 = 22 minute
m/s
Sol 88. (a) = 22 x 18 = 79.2 Sol 98. (a)
5
Let the required speed = k km/h km/h Distance to be covered = 12 km
According to the question or 12000 m
54 x 200 = 90 x k Sol 95. (d) Relative speed = 35+45 = 80 m/s
Pi

⇒ k = 54 90
×200 = 120 km/h
According to the question Required time = 12000
80 = 150
Train : Steamer : Car seconds or 2 minute 30 seconds
Sol 89. (b) Speed 3 : 2 : 1
Speed of shivan = 19 km/h or 19 Sol 99. (a)
5 = 95 m/s
x 18 18 Now, Speed of the train = 108 km/h or
Required time = 665 x 18 = 126 1 unit = 40 km/h 5 m/s = 30 m/s
95 108 x 18
seconds or 2 minutes 1 seconds ⇒ 2 unit = 80 km/h
10 Time = 32 seconds
⇒ 3 unit = 120 km/h Length of the train = 32 x 30 =
Sol 90. (d) So, 960 meters
3.75x + x + x = 9
Relative speed = (20-10) = 10 m/s 120 80 2×40 2
3.75x + 40x + 40x = 9
Required time = 180
10 = 18 seconds 60 Sol 100. (c)
7.5x + 3x + 3x = 9 x 120 Let the speed of train A = a and
Sol 91. (b) 13.5x = 1080 speed of Train B = b
According to the question

www.ssccglpinnacle.com support@ssccglpinnacle.com Ph. 09729327755, 09817390373 467


Days 49-52 Speed and Distance

s
sse
la
_c
ob
612 - 612
b = 9 …..(1) 5 unit = 100 SSC CGL TIER I
bo
a
ah 1 unit = 20
sm
612 - 612 = 3 …..(2)
ur

b 2a
yo

7 unit = 7 x 20 = 140 km/h Sol 1. (c) Train crosses a pole in


e/

Add eq (1) and (2)


t.m
://

1024−612 = 12 12 sec Means;


tp
ht

2a
Sol 105. (c) length of train is= 12 × speed of
⇒ a = 25.5 km/h
Distance covered = 300 km train =12 × S
Put this value in eq (1)
612 612 Time taken =3 hours Train crosses a bridge of length
25.5 - b = 9 Speed of first train = 300
612 3 = 100 170 m in 36 sec
⇒ 24 - 9 = b km/h Means;
⇒ b = 40.8 km/h Length of train+ 170 m = 36 ×
Sol 106. (a) speed of train = 36 × S
Sol 101. (a) Distance covered in 1 litre petrol Thus,
Time = 25 s = 60.48
5.4 = 11.2 km 12 × S + 170 = 36 × S
Speed = 25 m/s
Required distance = 11.2 × 22 = 170 = 24 × S
Length of the train = 25 x 25 =
246.4 km S= 170
24 m/s
625 meters

e
= 170 × 18
24 5
Sol107. (d) = 25.5 km/hour
Sol 102. (b)
Total distance covered = 210+198
Distance covered in (80-60)
= 408 km Alternate :
seconds = (1000-600) meters

l
Total time taken to cover the Trick approach, Distance covered
⇒ Speed of the trains = 400
20 = 20
ac distance = 210 198
60 + 66 hours in 24 (36-12) sec = 170m
m/s 408 408
Average speed = 210 198
+ 66
= 6.5 speed of train = 170
24 m/s
Length of the train = [ (80 x 20) - 60
170 18
1000] or [(60 x 20) - 600] = 600 = 62.8 km/h (approx) = 24 × 5
meters = 25.5 km/hour
Sol108. (d)
Sol 103. (c) Total distance to be covered = Sol 2. (a) The ratio of speed of A
Let the length of the train x = 3 230+750 = 980 m and B = 6:5
unit Speed of the train = 72 km/h = 72 After meeting, time taken by A
nn
5 = 20 m/s
x 18 and B are inversely proportional
And the length of the train y = 2
unit Required time = 980 = 49 s to square of their speed
20
Relative speed of trains = S 2a t
Hence, S 2b
= tab
(84+52) km/h = 136 × 12 5 = 170
3 Sol 109. (b) 36 = 2x
⇒ 25 5
m/s Distance covered in one ⇒ x = 18
5 =3 5
3 hours.
Distance covered in 12 minutes = revolution = π d = 22
7 × 56 = 176
170 x 12 = 680 meters
3 meters
Sol 3. (b) Speed of A and B are
Pi

According to the question Distance covered in 7 revolutions


100m/min and 120m/min
5 unit = 680 = 7 x 176 = 1232 meters
respectively.
1 unit = 136 Required speed = 1232
1 = 1232 Distance = LCM(100,120)=
Length of train y = 2 unit = 2 x m/s 600units.
136 = 272 meters
Time taken by A to cover 600
Sol 110. 600 = 6min
unit = 100
Sol 104. (b) 50% = 21 Time taken by B to cover 600
Let the speed of first train = 5unit
Car : Train 600 = 5 min
unit = 120
and the speed of second train = 7
Speed 2 : 3 Time difference is 1 min which is
unit
Time 3 : 2 given as 10 min
Now,
According to the question Thus, 600 unit distance= 6000 m
Speed of first train = 300
3 = 100 (3-2) unit = 2 hours = 6 km
km/h 2 unit = 2x2 = 4 hours
According to the question ⇒ Speed of train = 3604 = 90 km/h

www.ssccglpinnacle.com support@ssccglpinnacle.com Ph. 09729327755, 09817390373 468


Days 49-52 Speed and Distance

s
sse
la
_c
ob
Sol 4. (a) Distance between A and Speed of B = 8 km/h X = 10 km
bo
ah
B = 144 km Let ‘D’ be distance between A
sm
ur

and B
yo

Let speed of A = a km/h Alternate :


e/
t.m

Time taken by B to cover D


://

Speed of B = b km/h Had he travelled all the distance


tp
ht

distance is 25 minutes less than


For same direction, meeting time on foot, time taken = 50
5 = 10
time taken by A to cover the same
= 12 hour hours
distance.
⇒ 12 × (a-b) = 144 …(i) Had he travelled all the distance
Therefore, D6 = D8 + 60
25
For opposite direction, meeting ⇒ D = 10 km
50 = 5
by bicycle, time taken = 10
time = 89 hour hours
⇒ 9 × (a+b) = 144 …(ii) Sol 9. (c) Let the length of the
8 Now,
We get: 12 × (a-b) = 9 × (a+b) train = L m
8
a:b = 35:29 Speed of train = 72 km/h = 20m/s
a = 35x L + 180 = 20 × 60 = 1200
b = 29x L = 1020 m
Put values in (i): 12 × (6x) = 144 Time taken by train to cross the

e
x=2 pole = 1020
20 sec = 51 sec

a = 70 km/h
Sol 10. (c) Speed of car A =
Sol 5. (a) For such question, we 40km/h

l
Speed of car B = 50km/h According to question, (4+1) unit
directly put values in given
Distance travelled by car A in 30 = 50 km
formula:
Distance =
ac
Speed×(Speed± increase in speed)
increase in speed × time dif f erence
Let usual speed of train = S km/h
⇒ 300 = S×(S+20
20)
× 5
2
minutes = 20 km

Meeting time = 5020

3hours = 100km
−40
= 2 hour
Distance travelled by car B in 2
1 unit = 10 km
⇒ Distance travelled by bicycle =
10 km ans

Sol 14. (a) Distance between


⇒ S = 40 km/h station A and B = 575 km
⇒ 192 = 40 × time Distance travelled by train from
Sol 11. (d) Distance travelled by
station A in 30 minutes = 25 km
nn
⇒ Time = 4.8 hours first train in 30 min = 55 km
Remaining distance = 550 km
Relative speed of two trains = 30
Relative speed of two trains =
Sol 6. (a) Distance between km/h
station A and B = 428 km 55 h 50+60 = 110 km/h
Meeting time = 30 550 = 5 hours
Speed of trains from station A 55
Meeting time = 110
Meeting distance = 140 × 30 =
and B is 48km/h and 55 km/h Meeting distance from station A
2
256 km
respectively. 3 = 50 × 5 21 = 275 km
Distance travelled by train from Sol 12. (c) Distance travelled by
SSC CHSL 2019
station A in 20 min. = 16 km first train in 2 hours = 60 × 2=
Pi

1.Sol: (d)
Remaining distance = 412 km 120 km
412 = 412 = 4
Meeting time = 48+55 Let the AB = BC = CD = 60 km
103 Remaining distance to be Total distance = 180 km
hour travelled = 280-120 = 160
At 10:20 a.m. both the trains Time taken from A to B = 60/15
Relative speed of trains after two = 4 hours
meet.
hours = 60+20 = 80km/h Time taken from B to C = 60/20 =
Sol 7. (d) Distance between home Meeting time = 160 80 = 2 hours 3 hours
and school = 1.25 × 4 = 5 km Total time taken by first train to Time taken from C to D = 60/30
1
Speed ∝ time meet the second train = 4 hours = 2 hours
Decrease by 25%, i.e. t → 4 : 3 Total time = 4+3+2 = 9 hours
Sol 13. (b) Let Amit travelled x so, average speed = 180
Thus, speed change from 3 : 4 9 = 20
% increase = 33 31 % km on bicycle and (50-x) km on km/h
foot.
x 50−x
Sol 8. (a) Speed of A = 6 km/h 10 + 5 = 9 2.Sol (c)

www.ssccglpinnacle.com support@ssccglpinnacle.com Ph. 09729327755, 09817390373 469


Days 49-52 Speed and Distance

s
se
s
la
_c
ob
Average Speed = (Total length of train=72 × 5 × 36=720 (x3 +1) (x+1)(x2 −x+1)
Speed = = =
bo
18
ah
(x+1) (x+1)
distance)/(Total Time)
sm
m
(x2 − x + 1) km/h
ur
yo

Average Speed = (18+20+27)/(


e/
t.m
://

18 + 20 + 27 )
8.Sol. (a)
tp

6 5 9
ht

2xy
17.Sol: (d)
Average Speed = 65/10 Average speed = = 2×126×90 = 1200
x+y 216 Average speed = 16 = 75
Average Speed = 6.5 km/h 105 km/h Now, go through allegation
The time taken by the second 40 80
3.Sol (c) 525 = 5
train to travel 525 km = 105
Decrease in speed = 10 km/h 75
hours ________________
Percentage decrease in speed =
10 5 : 35 or 1 : 7
40 × 100 =25% 9.Sol: (a) Time ratio = 1 : 7
900 ×
Speed = 10×60 18 = 5.4 km/h
5 8 unit = 16 hours
4.Sol(c)
1 unit = 2 hours
Speed = distance
time 10.Sol: (b) Distance travel by Car = 2 × 40 =
= 350+250 m/s
20 time= Distance 80 km

e
speed
= 30m/s 2.5
Time = 30 × 60 = 5 min
Speed of the train (in km/h) = 30 18.Sol: (b)
× 18
5 = 108 km/h Let the total distance = 3x
11.Sol: (a)
x x x
12 + 18 + 24 = 13

l
Distance = 64 × 40 = 42.67 km
60
5.Sol: (d) 42.67
6x+4x+3x = 13
ac Speed = × 60 = 85.33 km/h 72
Let the speed of faster train = x 30
x = 72 km
km/h, speed of passenger train =
12.Sol: (a) Total distance = 3x = 3 × 72 = 216
x-70 km/h
1176 1176 Length of the train = 8 × 30 =4
x−70 − x = 5 60
19.Sol: (c)
1176x−1176x+1176×70 = 5 km
x(x−70) Perimeter of Square = 4 × 75 =
1176 × 14 = x2 70x
− 300 meter
13.Sol: (b)
x2 − 70x -16464=0 Speed = 6 km/h
90 km/h = 25 m/s
nn
2
x − 168x +98x -16464=0 Time = 6×300 = 180 sec or 3 min
Time = 150+300
25 = 18 5
18
x(x-168)+98(x-168)=0
(x-168) (x+98)=0 20.Sol: (a)
14.Sol: (c)
So x=168 Distance = 30 km
Distance = 1800 km
Now speed of passenger Let their speed = a km/h and b
Relative speed = 44+46 = 90
train=168-70=98 Km/hr km/h
km/h
Time taken by train to travel 1176 a+b = 70 km/h
Time = 1800
90 = 20 km/h
km distance= 1176
98 =12 hours Time = 2 hours 6 min
Distance travelled by first train =
Pi

30 + 30 = 2 6
20 × 44 = 880 km from A a b 60
6.Sol:(b) 30(a+b)
So, option (C) is the correct = 126
Let distance=300km ab 60
answer. 30×70 = 126
Time= 300
60 =5h ab 60
Time= 300
75 =4h ab = 1,000
Time difference=5-4=1h or 60 15.Sol: (d) 2 2
(a − b) = (a + b) − 4ab
min Let the distance = x km
2
60 minute=64 min ⇒ 15x + 30x = 3 (a − b) = 4900-4000
300 km=320 km ⇒ 3x =3 a-b = 30
30
Sum=320+64=384 so, difference in their speed = 30
⇒ x = 30 km
km/h
7.Sol:(d)
Using formula Distance=Speed × 21.Sol: (a)
Time 16.Sol: (b) 35
Average speed = 5 = 7 km/h

www.ssccglpinnacle.com support@ssccglpinnacle.com Ph. 09729327755, 09817390373 470


Days 49-52 Speed and Distance

s
sse
la
_c
ob
Now, go through the allegations. If speed is increased by 125%,
bo
ah
4 km/h 9 km/h then speed of train = 180 km/h
sm
ur
yo

Time taken by train to cover 8/5


e/
t.m
://

7 km/h of the same distance =


tp
ht

___________________________ 60 8
180 × 5 × 60 = 32 min
_____
2 : 3 27.Sol . (c)
5 unit = 5 hour Time = Distance AB = 54 × 5 × 20 = 300 m
Speed 18
2 unit = 2 hour BC = 72 × 5 × 20 = 400 m
Total time = 9 hrs, Total distance 18
Distance travelled on foot = 2 × 4
=D AC = √3002 + 4002 = 500 m
= 8 km 9 = 2 ×D40 + 2 ×D50 = 5 2D×+200
4D =

9D
22.Sol: (d) 2 × 200
5 25 ⇒ D = 400 km
30 km/h = 30 × 18 = 3 m/s
500 33.Sol . (c)
Time = 25 × 3 = 60 second or 1
28.Sol . (d) Let the distance between school

e
minute
Distance = 600 km and house = D km and the school
Let speed = S starts after t-minutes if he runs at
23.Sol.(a) :
Time, t = 32 × S his usual speed, S.
Speed of the train = 12/12
D=S × t D = S ×t

l
× 60 = 60 km/h.
600 = 23 × t × t Condition 1:- He runs cycle at 10
the time taken by it to cover the
ac t2=400 so t=20 hours km/h; he reaches school 8
distance of 22 km, when its speed
minutes late;
is decreased by 5 km/h 8 ) …. (i)
29.Sol (a) D = 10 (t + 60
= 22/55 = ⅖ hours = 24 min
Condition 2:- He runs cycle at 16
km /h; he reaches school 10
24.Sol (d)
minutes early;
Time taken by man to cover a 10 ) …. (ii)
D = 16 (t - 60
distance of 6 km = 6/12 = 30min
(i) = (ii) :---
nn
After every km, he takes rest for Average Speed = T otal Distance =
T otal T ime 8 ) = 16 (t - 10
3 minutes, so in distance of 6km 2D 2 200 = 200 10 (t + 60 60 )
D D
+ 25
= 1
+ 1 = 5+4 9 =
20 20 25 ⇒ 5 (t + 8 ) = 8 (t - 10 )
he rests for 5 times = 3 2 60 60
22 km/h ⇒ 5t + 40 80
× 5 = 15 min 9
60 = 8t - 60
Total time taken by man = 30+15 ⇒ 8t - 5t = 80 + 40 = 120 =2
60 60 60
30.Sol . (d)
= 45 min ⇒ 3t = 2
Side of square field = 175 m
⇒ t = 32 × 60 = 40 minutes
Perimeter of square field = 4 ×
25.Sol (d) Therefore, School starts at = 8:20
175 = 700 m = 0.7 km
Pi

Time taken by man to cover a + 40 min = 9:00 am


distance of 6 km = 6/12 = 30min Speed of horse = 15 km/h
After every km, he takes rest for Time = 0.715 hr =
0.7 × 3600 sec
15
34.Sol. (d)
3 minutes, so in distance of 6km = 168 cm
Speed = Distance
time = 700 m
6 min =
he rests for 5 times = 3 31.Sol . (d)
700 m × 60 = 7 km/h
Speed of car = 105 = 35 km/h 6 min × 1000
× 5 = 15 min 3
Total time taken by man = 30+15 Speed of train = 252
4 = 63 km/h
35.Sol. (b)
= 45 min Required ratio ⇒ 35 : 63 ⇒ 5 : 9 1
Speed ∝ time
26.Sol: (c) Ratio of time when it changes
32.Sol. (d)
Distance covered by train at a from - 30 → 20 = 3 : 2
speed of 80 km/h in 45 min = 60 Therefore, ratio of speed should
km be 2 : 3
2 → 54 km/h

www.ssccglpinnacle.com support@ssccglpinnacle.com Ph. 09729327755, 09817390373 471


Days 49-52 Speed and Distance

s
sse
la
_c
ob
Then, 3 → 81 km/h x = 80 Speed = 800 × 18 = 30km/h
bo
96 5
ah Time taken to travel 90 km=
sm
36.Sol . (b) 47.Sol:(b)
ur
yo

90 1
Speed of car, S = 150
5 = 30 km/h 80 = 1 8 Let time by foot = x
e/
t.m
://

S’ = S = 10 km/h Time by bicycle = 5-x


tp
ht

3
Time to cover 150 km = 150 = 41.Sol:(c) According to the question
10
4
15 hr 5 of the distance = 128km 6x + 10(5 - x) = 42
Required time difference = 15 - 5
2 of the time = 10 hours x=2
3
= 10 hour Remaining distance = 32km Distance travelled by foot = 6 × 2
Remaining time = 5 km = 12
37.Sol: (c) Speed required = 32
5 = 6.4
Let the distance = 2D 48.Sol:(b)
D D To walk 6 km he will stop 5 times
30 + 20 = 5 42.Sol:(d)
2D+3D = 20min
60 =5 Speed of train in (m/s)= 36
5 = 10 To travel 6km time = 86 × 60 = 45
D = 60 km × 18
Total time = 65min
Therefore, Distance = 2 × 60 = Length of train = 25 × 10 = 250

e
120 km m
SSC CPO 2019
Total distance to cover 350m long
49.Sol:(b)
SSC CGL 2019 TIER-II bridge = 250m + 350m = 600m
Speed of train of X=74km/h
38.Sol: (b) To cover 600m = 60010 = 60sec

l
Speed of train of Y= 52km/h
Total speed = 117km/h
ac 5 Relative speed=
Speed in (m/s) = 117 × 18 43.Sol:(d)
74+52=126km/h=126 × (5 ÷
Time =18 sec Ratio of both speed = 7 : 9
18)=35m/sec
Distance covered = speed × time Ratio of Time required to travel
Time=12s
=585m equal distance in both case = 9 : 7
Total distance travelled= 35 ×
Length of second train = 585-287 Difference in time =9x-7x=2x=
12=420metres
= 298m 16min
Let length of x be 3a
x= 8
Then length of y= 2a
39.Sol:(a) Actual time at 3.5 km/hr speed =
nn
72 hr Total length = 5a
Speed of A and B = 6 : 5 9 × 8 =72 min = 60
I.e 5a=420metres
Speed and time ratio after Total distance = So a=84metre
meeting 72
60 × 3.5 = 4.2km Then length of x= 3a i.e 252
T1 2 S2
T2 2
= S1 metres
44.Sol:(b)
time of A and B = 49 :9
S1 = 25 × 36 = 5 9 Sol:50.(d)
36 5
Speed and time ratio after For the same distance
Pi

meeting Ratio of speed is opposite to the


40.Sol:(a)
T1 2 S2 ratio of time taken
Speed of B = x T2 2
= S1
Time taken by B = 40 Ratio of time taken = 75 min : 45
x T2 2 = 7 × 33 = 77 = 25 32
40
3 3 3 min = 5 : 3
Time taken by A = x +2
Then,
Speed of A = 40
40
+2 45.Sol:(a) Ratio of speed = 3 : 5
x

Double speed of A = 80
40 = Average speed of boy= 2×32×24
(32+24) =
x
+2
2×32×24 = 8×24 51.Sol:.(a)
80x 56 7
40+2x
80 Total time taken by boy=1 hours Let speed for going = x
Time taken by A = 80x
40+2x
25 minutes Time for going = 300
x
According to que Net time = 1 hours 10 minutes= 67 Speed for coming = x + 10
80 80
80x = x + 1.5 Time for coming = 300x -5
Total distance = 8×24 × 7 =32km
40+2x
7 6
3200+ 160x = 80+ 1.5x According to question
80x x
On solving the above eq we get 46.Sol:(d) (x + 10) × ( 300
x - 5) = 300

www.ssccglpinnacle.com support@ssccglpinnacle.com Ph. 09729327755, 09817390373 472


Days 49-52 Speed and Distance

s
sse
la
_c
ob
3000 So length of Y = 168 metres
300 - 5x + - 50 = 300
bo
x
ah
sm
2
3000= 5x + 50
ur
yo

55.Sol:(d)
e/

X2 + 10x - 600 = 0
t.m
://

(x + 30)(x - 20) = 0 Let the speed of a be A km/hr


tp
ht

x = 20, as speed cannot be And speed of B is B km/hr


negative From the question 40 40
A − B = 2.5
And 40 +1= 40
Speed for coming = 30 2A B
Average speed = a+b2ab = 2×20×30
50 Solving both the above equations
= 24 we get A= 40
7 km/hr
B= 80 km/hr
9
52.Sol:.(a) Average time taken by A and B to
Let speed for going = x cover a distance of 40kms is
Time for going = 300
x distance
Average speed
Speed for coming = x + 10 Since distance is constant in both
Time for coming = 300x -5 the cases to we can directly apply

e
According to question 2ab
the formula Avg speed= a+b
(x + 10) × ( 300
x - 5) = 300 Average speed = 160
23
3000
300 - 5x + x - 50 = 300 = 23
4

l
2
3000= 5x + 50 = 5 43
X2 + 10x - 600 = 0
ac = 5 hours 45 minutes.
(x + 30)(x - 20) = 0
x = 20, as speed cannot be
Sol: 56.(a)
negative
Let the speed of train = x
Time taken = 400
x
53.Sol:(d)
When speed increases
Let the length of the train be L
Speed = x + 10
metres
Time = 400
x -2
According to the question 30 × X
nn
400 = (x + 10 )( 400 - 2)
5 =300+L
× 18 x
4000
In second case both train and man 400 = 400 - 2x + x - 20
is travelling in the same direction 2x2 + 20 x = 4000
so relative speed = X-6 Km/hr 2x2 + 20 x - 4000 =0
So (X-6) × 18 5 × 20=L (distance= x2 + 10x - 2000 = 0
Speed × Time) x = 40 or -50
Solving the above two equations Speed cant be negative
So , x = 40km/hr
Pi

we get X=96km/hr
Time = 400
40 = 10 h

54.Sol:(a)
Speed of train A=74Km/hr
Speed of train B=52Km/hr
Relative speed of A and B = 126
Km/hr= 126 × 18 5 = 35 m/sec

Total time taken= 12 secs


Total distance travelled= 35 × 12=
420 metres.
Let length of X = 3t
Length of Y= 2t
Total length= 5t=420
t=84 metres

www.ssccglpinnacle.com support@ssccglpinnacle.com Ph. 09729327755, 09817390373 473


Day 53 : Boat and Stream

s
sse
la
_c
ob
BOAT & STREAM / नाव और Still water speed of the man = (c) 3
bo
ah
धारा /
30+10 = 40 = 20 km h
(d) 2.5
sm
2 2
ur
yo

Speed of the current


e/
t.m

Important Terms/ मह ूण बात: = 2 = 2 = 10 km/h


30 10 20
://

− Q3. A boat can go 30 km


tp
ht

1). Still Water Speed: Speed of a Note: Changing k m/h to m/s downstream and 24 km upstream
thing in water which is not km = 1000 m = 5 m in 2 hours 27 minutes. Also, it
h 3600 s 18 s
flowing. / के ए पानी म िकसी can go 10 km downstream and 4
∴ To change k m/h to m/s ,
व ु के चलने की गित| 5 . km upstream in 37 minutes. What
multiply it by 18
2). Speed of Current: The speed is the speed of the boat upstream
at which water is flowing./ पानी To change m/s to k m/hr ,
multiply it by 18 (in km/h)?
के बहाव की गित| 5 .
एक नाव 2 घंटे 27 िमनट म धारा के
3). Downstream speed: Speed of
अनुकूल 30 िकमी तथा धारा के
a thing, moving in the same Q1. A boat can cover a distance
direction as that of the current./ ितकूल 24 िकमी जा सकती है | साथ
of 7.2 km downstream and 3.2
बहाव के साथ चलने म िकसी व ु ही, यह 37 िमनट म 10 िकमी धारा के
km upstream in 2 hours. It can
की गित अनुकूल तथा 4 िकमी धारा के
also cover 1.5 km downstream
ितकूल जा सकती है | धारा के

e
Downstream speed and 0.6 km upstream in 24
ितकूल नाव की चाल ( िकमी/घंटा
minutes. What is the speed of the
= S till water speed + S peed of current म) ा है ?
boat when going downstream (in
SSC CPO 12 March 2019
4). Upstream Speed: Speed of a km/h) ? / एक नाव धारा के अनुकूल

l
(Evening)
thing, moving in opposite 7.2 िकमी और धारा के िवपरीत 3.2
[a] 24
direction as that of the current. /
ac िकमी की दू री 2 घंटे म तय कर
बहाव के िव िकसी चीज़ के चलने (b) 18
सकती है | यह धारा के अनुकूल 1.5
की गित| िकमी और धारा के िवपरीत 0.6
(c) 22
U pstream speed (d) 20
िकमी की दू री भी 24 िमनट म तय
कर सकती है | धारा के अनुकूल नाव
= S till water speed − S peed of current Q4. A boat can go 10 km
की चाल िकतनी ( िकमी/घंटा म) है ?
downstream and 8 km upstream
Let’s take an example: SSC CGL 7 June 2019
in 49 minutes. Also it can go 12
A man can swim at 20 km/h in (Morning)
km downstream and 4 km
nn
still water. He is swimming in a (a) 6
river flowing at 10 km/h. upstream in 42 minutes. What is
(b) 4.5
एक के ए पानी म 20 the speed of stream in km/h?
(c) 5
km/h की गित से तैर सकता है | वह एक नाव 49 िमनट म 10 िकमी धारा
(d) 7.5
की गित से बहने वाली एक नदी म तैर के अनुकूल और 8 िकमी धारा के
रहा है | ितकूल जा सकती है | साथ ही, यह
Q2. A person can row a distance
Here, Speed of current is = 10 42 िमनट म 12 िकमी धारा के
of 4 km upstream in 1 hour 20
km/h अनुकूल और 4 िकमी धारा के
minutes and can row back to the
If the man swims downstream i.e. ितकूल जा सकती है | धारा की चाल
Pi

starting point in just 24 minutes.


with the flow of the river (बहाव ( िकमी/घंटा म ) ात कर |
के साथ), his speed = 20 + 10 = 30 How much time (in hours) will he
SSC CPO 13 March 2019
km/h take to row 13 km in still water? /
(Evening)
If the man swims upstream i.e. एक धारा के िवपरीत 4 िकमी
(a)1.5
against the flow of the river की दू री 1 घंटा 20 िमनट म तय कर
(b)1
(बहाव के िव ), his speed = 20 – सकता है और अपने आरं िभक िबंदु
(c)2
10 = 10 km/h तक केवल 24 िमनट म वापस प ँ च
Remember/याद रख: (d)2.5
सकता है | थर जल म 13 िकमी की
1). S till water speed दू री तय करने म उसे िकतना व (
Q5. A boat can go 30 km
= Downstream speed+U pstream Speed
2
घंटे म ) लगेगा ?
downstream and 24 km upstream
2). Speed of the current SSC CGL 7 June 2019
in 2 hours 27 minutes.Also, it can
= Downstream speed2−U pstream Speed (Afternoon)
go 20km downstream and 8 km
Let’s verify in the given example: (a) 2
upstream in 74 minutes. What is
(b) 3.5

www.ssccglpinnacle.com support@ssccglpinnacle.com Ph. 09729327755, 09817390373 474


Day 53 : Boat and Stream

s
sse
la
_c
ob
the speed of the boat in still water SSC CPO 14 March 2019 (अप ीम) गित 10km/h है वह नाव
bo
ah
in km/h? (Evening) शां त जल म 72 km की दू री िकतने
sm
ur
yo

एक नाव 2 घंटे 27 िमनट म 30 िकमी (a) 2.5 km/h समय म तय कर सकती है ? SSC
e/
t.m

धारा के अनुकूल और 24 िकमी धारा


://

(b) 5 km/h MTS 6 August 2019 (Morning)


tp
ht

के ितकूल जा सकती है | साथ ही, (c) 4.2 km/h (a) 8 Hours / घंटे
यह 74 िमनट म 20 िकमी धारा के (d) 3.6 km/h (b) 6 Hours / घंटे
अनुकूल और 8 िकमी धारा के (c) 4 Hours / घंटे
ितकूल जा सकती है | थर जल म SSC MTS (d) 12 Hours / घंटे
नाव की चाल ( िकमी/घंटा म ) ात
कर | Q8. The speed of a boat in still Q11. A boat covers 64 km
SSC CPO 12 March 2019 water is 8 km/h. If the speed of upstream in 8 hours and 120 km
(Morning) current is 2 km/h, then what downstream in 12 hours. What is
(a)18 distance this boat will cover in 2 the speed (in m/s) of the boat in
(b)20 hours going upstream ? still water?
(c)24 शां त जल म एक नाव की गित 8 एक नाव धारा के ितकूल 8 घंटे म
km/h है | यिद धारा की गित 2 km/h 64 िकमी तथा धारा के अनुकूल 12

e
(d)22
है , तो धारा के ितकूल जाते ए 2 घंटे म 120 िकमी की दू री तय कर
Q6.A boat can go 20 km घंटे म नाव िकतनी दू री तय करे गी ? सकती है | थर जल म नाव की चाल
downstream and 30 km upstream SSC MTS 2 August 2019 ( मीटर/सेकंड म ) ा होगी ?

l
in 2 hours 20 minutes. Also, it (Afternoon) SSC MTS 7 August 2019
can go 10 km downstream and 8 (a) 16 km (Morning)

in km/h?
ac
km upstream in 49 minutes. What
is the speed of boat downstream

एक नाव 2 घंटे 20 िमनट म 20 िकमी


धारा के अनुकूल और 30 िकमी धारा
(b) 14 km
(c) 20 km
(d) 12 km

Q9. A boat can travel 60 km in 3


(a) 2.5
(b) 2
(c) 3.5
(d) 3

के ितकूल जा सकती है | साथ ही, hours while going downstream. It Q12. The speed of a boat in still
यह 49 िमनट म 10 िकमी धारा के can travel 90 km in 5 hours while water is 30 km/hr. If the boat
अनुकूल और 8 िकमी धारा के going upstream. What is the ratio covers 60 km downstream in 1
nn
ितकूल जा सकती है | धारा के of the speed of boat in still water hour 30 minutes, then what is the
अनुकूल नाव की चाल ( िकमी/घंटा म to the speed of the stream? time taken by the boat to cover 60
) ात कर | एक नाव धारा के अनुकूल 3 घंटे म km upstream?
SSC CPO 13 March 2019 60 िकमी जा सकती है | यह धारा के थर जल म िकसी नाव की चाल 30
(Morning) ितकूल 5 घंटों म 90 िकमी की दू री िकमी/घंटा है | यिद यह नाव धारा के
(a)18 तय कर सकती है | थर जल म नाव अनुकूल 60 िकमी की दू री 1 घंटे 30
(b)20 की चाल तथा धारा की चाल म ा िमनट म तय करती है , तो धारा के
(c)16 अनुपात है ? िवपरीत 60 िकमी की दू री तय करने
Pi

(d)24 SSC MTS 2 August 2019 म इसे िकतना समय लगेगा ?


(Evening) SSC MTS 7 August 2019
Q7. The speed of a boat in still (a) 6 : 1 (Evening)
water is 6 km/h. If it takes four (b) 4 : 1 (a)3 hours
times more time to go against the (c) 19 : 1 (b)5 hours
current than to go in the direction (d) 10 : 9 (c)4 hours
of the current, what will be the (d)1 hour
speed of the current ? / शां त जल Q10. The downstream speed of a
म एक नाव की गित 6 km /h है | यिद boat is 14 km/h. The upstream Q13. The speed of a boat in still
इसे धारा की िदशा म जाने की तुलना speed of this boat is 10 km/h. In water is 6 km/h. Time taken by
म धारा के िवपरीत जाने म चार गुना what time it can cover a distance the boat to cover a certain
अिधक समय लगता है तो धारा की of 72 km in still water ? / नाव की distance upstream is 3 hours more
गित ा होगी ? अनु वाह (डाउन ीम) गित 14 than the time taken to cover the
km/h है | नाव की ऊ व वाह same distance downstream. If the

www.ssccglpinnacle.com support@ssccglpinnacle.com Ph. 09729327755, 09817390373 475


Day 53 : Boat and Stream

s
sse
la
_c
ob
speed of the stream is 2 km/h, SSC MTS 13 August 2019 Q18. A boat goes at 20 km/h up
bo
ah
then what is the total distance, (Afternoon) stream and at 30 km/h down the
sm
ur
yo

upstream and downstream, (a) 2 hours stream. What is the speed of this
e/
t.m
://

covered by the boat? (b) 3 hours boat in still water ?


tp
ht

थर जल म िकसी नाव की चाल 6 (c) 4 hours कोई नाव 20 km/h की गित से


िकमी/घंटा है | धारा के िवपरीत िकसी (d) 2.5 hours उ व वाह (अप ीम) और 30 km/h
िनि त दू री को तय करने म नाव को की गित से अनु वाह (डाउन ीम)
लगने वाला समय धारा के अनुकूल Q16. In one hour, a man rows his चलती है | थर जल ( ल वाटर) म
इसी दू री को तय करने म लगने वाले canoe against the stream at 11 नाव की गित िकतनी है ?
समय से 3 घंटा अिधक है | यिद धारा km/h and along the stream at 23 SSC MTS 16 August 2019
की चाल 2 िकमी/घंटा है , तो नाव के km/h. What is the speed (in km/h) (Evening)
ारा धारा के िवपरीत तथा धारा के of stream? (a) 26 km/h
अनुकूल तय की गयी कुल दू री ात एक घंटे म, कोई अपनी डोंगी (b) 24 km/h
कर | धारा के ितकूल 11 िकमी/घंटा की (c) 25 km/h
SSC MTS 8 August 2019 चाल से तथा धारा के अनुकूल 23 (d) 22.50 km/h
िकमी/घंटा की चाल से चलाता है |

e
(Morning)
(a) 72 km धारा की चाल ( िकमी/घंटा म ) Q19. A boat can go 10 km
(b) 24 km िकतनी है ? upstream and 20 km downstream
(c) 48 km SSC MTS 14 August 2019 in 7 hours. It can go 20 km

l
(d) 36 km (Evening) upstream and 10 km downstream
(a) 6 in 11 hours. What is the speed of
ac
Q14. If the speed of the stream is
20% of the speed of boat in still
water and it covers 120 km
upstream in 150 minutes, then
what is the downstream speed of
(b) 5
(c) 17
(d) 16

Q17. A man can row a boat in


this boat in still water ?
एक नाव 7 घंटे म 10 km. उ व वाह
(अप ीम) तथा 20 km. अनु वाह
(डाउन ीम) चल सकती है | वह 11
घंटे म 20 km. उ व वाह (अप ीम)
the boat? still water at a speed of 5 m/s. He तथा 10 km. अनु वाह (डाउन ीम)
यिद धारा की चाल थर जल म नाव covers a stretch of 200 m in a चल सकती है | थर जल म नाव की
की चाल का 20% है और यह धारा के river downstream during high and गित िकतनी है ?
nn
ितकूल 120 िकमी की दू री 150 low tides in 10 s and 25 s SSC MTS 19 August 2019
िमनट म तय करती है , तो नाव की respectively. What is the ratio of (Afternoon)
धारा के अनुकूल चाल ात कर | the speed (in m/s) of the water (a) 2 km/h
SSC MTS 9 August 2019 flowing in the river during high (b) 8 km/h
(Afternoon) and low tides? (c) 6 km/h
(a)75 km/hr एक अपनी नाव थर जल म (d) 4 km/h
(b)72 km/hr 5 मी/सेकंड की चाल से चलाता है |
(c)80 km/hr वह धारा के अनुकूल उ और िन Q20. A boat takes 45 minutes to go 3
Pi

(d)64 km/hr ार म नदी म 200 मी का एक km


पड़ाव मशः 10 सेकंड और 25 upstream and 4 21 km downstream
Q15. A boat takes 80 minutes to सेकंड म पार करता है | उ तथा while it
12 km upstream and 60 minutes िन ार के दौरान नदी म बहने covers a distance of 3.6 km upstream
to row 15 km downstream. How वाले पानी की चाल ( मीटर/सेकंड ) and
long will it take to row a distance म अनुपात ात कर | 2.4 km downstream in 39 minutes. The
of 36 km in still water? SSC MTS 16 August 2019 speed ( km/h ) of this boat in
िकसी नाव को धारा के ितकूल 12 (Morning) downstream
िकमी जाने म 80 िमनट लगते ह तथा (a) 5 : 3 is :
धारा के अनुकूल 15 िकमी जाने म 60 (b) 5 : 1 एक नाव को 3 km उ व वाह
िमनट लगते ह | थर जल म इसे 36 (c) 3 : 2 (upstream) और 4 21 अनु वाह
िकमी की दू री तय करने म िकतना (d) 4 : 3 (downstream) दू री तय करने म 45
समय लगेगा ? िमनट लगते ह, जबिक वह 3.6 km

www.ssccglpinnacle.com support@ssccglpinnacle.com Ph. 09729327755, 09817390373 476


Day 53 : Boat and Stream

s
se
s
la
_c
ob
उ व वाह (upstream) और 2.4 km SSC CGL TIER II SSC CGL TIER II (13
bo
ah
अनु वाह (downstream) दू री 39 September 2019)
sm
ur
yo

िमनट म तय कर लेती है | नाव की Q1. Abhi rows upstream a (a) 4 21


e/
t.m

अनु वाह (downstream) चलने की


://

distance of 28 km in 4 hours and (b) 6


tp
ht

गित (km/h म) है : rows downstream a distance of 50 (c) 5 41


SSC MTS 20 August 2019 km in 2 hours. To row a distance (d) 5
(Morning) of 44.8 km in still water, he will
(a) 12 take : / अिभ धारा के ितकूल 28 SSC CGL 2019 TIER I
(b) 16 िकमी 4 घंटे म जाता है तथा धारा के
(c) 9 अनुकूल 50 िकमी 2 घंटे म तय करता Q1. A boat can go 3 km upstream
(d) 10 है | थर जल म 44.8 िकमी की दू री and 5 km downstream in 55
तय करने म, उसे िकतना समय minutes. It can also go 4 km
Q21. The speed of a boat in still लगेगा ? upstream and 9 km downstream
water is 15 km/h. The speed of SSC CGL TIER II (11 in 1 hour 25 minutes. In how
the current is 3 km/h. In how September 2019) much time (in hours) will it go

e
much time (in hours) will the boat (a) 2.8 hours 43.2 km downstream? / एक नाव
travel a distance of 54 km (b) 3.2 hours 55 िमनट म 3 िकमी धारा के ितकूल
upstream and the same distance (c) 2.4 hours तथा 5 िकमी धारा के अनुकूल जा
downstream? (d) 2.2 hours सकती है | यह 1 घंटे 25 िमनट म 4
थर जल म िकसी नाव की चाल 15

l
िकमी धारा के ितकूल तथा 9 िकमी
िकमी/घंटा है | धारा की चाल 3 Q2. The speed of boat in still धारा के अनुकूल जा सकती है | धारा
ac
िकमी/घंटा है | यह नाव धारा के
ितकूल 54 िकमी तथा धारा के
अनुकूल इतनी ही दू री िकतने समय (
घंटे म ) म तय करे गी ?
SSC MTS 22 August 2019
water is 18 km/h and the speed of
the current is 6 km/h. In how
much time (in hours) will the boat
travel a distance of 90 km
upstream and the same distance
के अनुकूल 43.2 िकमी जाने म इसे
िकतना समय ( घंटे म ) लगेगा ? SSC
CGL 4 March 2020 (Morning)

(a) 4.8
(Afternoon) downstream ? / थर जल म एक (b) 5.4
(a) 7 21 नाव की चाल 18 िकमी/घंटा है तथा (c) 3.6
(b) 7 धारा की चाल 6 िकमी/घंटा है | यह
nn
(d) 4.4
(c) 6 नाव धारा के ितकूल 90 िकमी तथा
(d) 6 21 इतनी ही दू री धारा के अनुकूल िकतने Q2. The time taken by a boat to
समय ( घंटा म ) म तय करे गी ? travel 13 km downstream is the
Q22. A boat goes a distance of 4 SSC CGL TIER II (12 same as time taken by it to travel
km upstream in 2 hours and the September 2019) 7km upstream. If the speed of the
same distance downstream in 20 (a) 9 21 stream is 3km/h, then how much
minutes. How long will it take to (b) 11 41 time(in hours) will it take to
Pi

go 10 21 km in still water? (c) 12 travel a distance of 44.8 km in


एक नाव 2 घंटे म 4 िकमी धारा के (d) 10 still water? / धारा के अनुकूल 13
ितकूल जाती है और धारा के िकमी की दू री तय करने म एक नाव
अनुकूल इतनी ही दू री 20 िमनट म Q 3. A man can row a distance of को लगने वाला समय धारा के
तय करती है | थर जल म 10 21 900 meters against the stream in ितकूल 7 िकमी की दू री तय करने
िकमी तक जाने म इसे िकतना समय 12 minutes and returns to the म इसे लगने वाले समय के बराबर है |
लगेगा ? starting point in 9 minutes. What यिद धारा की चाल 3 िकमी/घंटा है ,
SSC MTS 22 August 2019 is the speed (in km/h) of the man तो थर जल म इसे 44.8 िकमी की
(Evening) in still water ? / एक धारा के दू री तय करने म िकतना समय ( घंटे
(a) 1 21 hours िवपरीत 900 मीटर की दू री नाव से म ) लगेगा ?
12 िमनट म तय करता है तथा SSC CGL 4 March 2020
(b) 48 minutes
आरं िभक िबंदु पर 9 िमनट म प ँ चता (Afternoon)
(c) 1 41 hours
है | थर जल म की चाल (
(d) 1 hour
िकमी/घंटा म ) िकतनी होगी ? 12
(a) 4 25

www.ssccglpinnacle.com support@ssccglpinnacle.com Ph. 09729327755, 09817390373 477


Day 53 : Boat and Stream

s
se
s
la
_c
ob
(b) 5 53 Q1. A boat covered a distance of (c) 36
bo
ah 15 km upstream in 5 hours and a (d) 32
sm
(c) 5 2
ur

5
yo

distance of 42 km downstream in
e/

(d) 4 13
t.m

25
://

6 hours. The speed of the stream SSC CPO 2019


tp
ht

in km/h is: Q4. The speed of a boat in still


Q3. A boat can go 3.6 km
एक नाव धारा के िवपरीत 15 िकमी water is 15 km/h, and the speed
upstream and 5.4 km downstream
की दू री 5 घंटे म और धारा के साथ of the current is 5 km/h, in how
in 54 minutes, while it can go 5.4
42 िकमी, 6 घंटे म दु री तय िकया। much time (in hours) will the boat
km upstream and 3.6 km
िकमी / घंटा म धारा की गित है : travel a distance of 60 km
downstream in 58.5 minutes. The
CHSL 14-10-2020 (Morning upstream and the same distance
time (in minutes) taken by the
shift) downstream?
boat in going 10 km downstream
(a) 3 थर नदी म एक नाव की चाल 15
is:
(b) 2 km/h और धारा की चाल 5 km/h है ।
एक नाव 54 िमनट म धारा के
(c) 2.5 नाव 60 िकलोमीटर की दु री धारा के
ितकूल 3.6 िकमी तथा धारा के
(d) 1.5 साथ और उतनी ही दु री धारा के
अनुकूल 5.4 िकमी जा सकती है |
िवपरीत िकतने समय (घंटे म ) म तय

e
जबिक यह 58.5 िमनट म धारा के
Q.2. A motorboat goes 24 km in 2 करे गी
ितकूल 5.4 िकमी तथा धारा के
hour along the stream and 10 km CPO 25-11-2020 (Evening
अनुकूल 3.6 िकमी जा सकती है |
in 1 hour against the stream. The shift)
धारा के अनुकूल 10 िकमी जाने म

l
speed of the motorboat in (a) 12
नाव को िकतना समय लगेगा ?
Kilometers per hour, is: (b) 9
SSC CGL 5 March 2020
ac एक मोटरबोट धारा के साथ 2 घंटे म (c) 10
(Afternoon)
24 िकमी और धारा के खलाफ 1 घंटे (d) 20
म 10 िकमी जाती है । ित घंटे
(a) 48
िकलोमीटर म मोटरबोट की गित है : Sol 1. (a)
(b) 54
CHSL 19-03-2020 (Morning Upstream speed = u km/h
(c) 50
shift) Downstream speed = d km/h
(d) 45
(a) 14 According to the question
(b) 10 7.2 + 3.2 = 2 ………..(1)
nn
Q4. In a stream running at 3 d u
(c) 11 And
km/h, a motorboat goes 12 km
(d) 12 1.5 + 0.6 = 24 = 52
upstream and back to the starting d u 60

point in 60 min. Find the speed of ………..(2)


SSC CGL 2019 TIER-II Multiply equation (1) by 5 and eq
the motorboat in still water. (in
Q3. A boat can go 5km upstream (2) by 2.4 and subtract eq (2)
km/h) / एक मोटरबोट 12 िकमी
and 7 21 km downstream in 45 from (1)
धारा की िदशा म जाती है और 60
minutes. It can also go 5km ( 36 16 36 14.4 48
िमनट म शु आती िबंदु पर वापस d + u )-( d + u )= 10- 5
downstream and 2.5km upstream
Pi

आती है । यिद धारा की गित 3 िकमी / ⇒ 1.6


u = 52
घंटा है , शां त जल म मोटरबोट की in 25minutes. How much time (in
⇒ u = 4 km/h
गित का पता लगाएं । (िकमी / घंटा म) minutes) will it take to go 6 km
Put this value in either of the
SSC CGL 7 March 2020 upstream ?
example
(Evening) एक नाव 5 िकमी ऊ वाह और 7
1 िकमी अनु वाह 45 िमनट म जा For example
2 7.2 + 3.2 = 2 ⇒ 7.2 + 3.2 = 2
d u d 4
(a) 2(2+ √17 ) सकती है । वह 25 िमनट म 5 िकमी
⇒ d = 6 km/h
अनु वाह और 2.5 िकमी ऊ वाह
(b) 3(4+ √17 )
पर भी जा सकती है । 6 िकमी
(c) 2(4+ √15 ) ऊ वाह जाने म िकतना समय
Sol 2. (a)
Let the speed of boat = b km/h
(d) 3(2+ √17 ) (िमनटों म) लगेगा ?
And speed of stream = s km/h
CGL 2019 Tier-II (18-11-2020 )
Upstream speed (b-s) = 44/3 = 3
SSC CHSL 2019 (a) 30
(b) 24 km/h ……(1)

www.ssccglpinnacle.com support@ssccglpinnacle.com Ph. 09729327755, 09817390373 478


Day 53 : Boat and Stream

s
sse
la
_c
ob
Downstream speed (b+s) = 4 Now, Subtract (i) from 2x(ii), we
bo
24/60
ah get u=20 Sol 12. (a)
sm
= 10 km/h ..….(2)
ur
yo

Putting the value of u=20 in eq. Distance covered = 60 km


e/

Add (1) and (2)


t.m
://

(b-s)+(b+s) = 3+10 (ii), we get d=24 km/h Time = 1 21 hours


tp
ht

Downstream speed = 60 = 40
2b = 13 3/2
⇒ b = 6.5 km/h Sol 7. (d) Let the speed of stream km/h
Time taken to travel the distance be a. Speed of the current = 40-30 = 10
13 = 2 h
= 6.5 Given, Upstream : km/h
Downstream Upstream speed = 30-10 = 20
Time= 4 : 1 km/h
Sol 3. (d) Let the speed of boat
Speed= 1 : 4 60 = 3 hours
Required time = 20
upstream be a and downstream be 6 a 1

So, 6+a = 4
b.
Therefore, ⇒ a = 3.6 km/h
Sol 13. (c)
ATQ: The speed of a boat in still water
30 + 24 = 2 27 = 49 ………(1) Sol 8. (d) = 6 km/h
b a 60 20

e
And, 10 + a4 = 37 …………(2) Speed of the boat = 8 km/h the speed of the stream = 2 km/h
b 60
Speed of the current = 2 km/h Downstream speed = 6+2 = 8
Multiply eq. (ii) by 3 and subtract
Speed of the boat against the km/h
from eq. (i)
current = 8-2 = 6 km/h Upstream speed = 6-2 = 4 km/h
We get, 12 12
a = 20

l
Required distance = 6 x 2 = 12 Let the time taken to cover the
Therefore, a = 20 km/h (speed of km
ac
the boat upstream) distance downstream = t hours.
According to the question
Sol 9. (c) 8 x t = 4 (t+3)
Sol 4.(c) Let the speed of boat Downstream speed of the boat = ⇒ 4t = 12
upstream be a and downstream be 60 = 20 km/h
b.
3 ⇒ t = 3 hours
Upstream speed of the boat = 90
Therefore, ATQ: 10 8 49 5 One side distance = 8 x 3 = 4 x
a + b = 60 ----
12 4 42 = 18 km/h (3+3) = 24 km
(i) a + b = 60 ---- (ii)
Speed of the boat in still water = Total distance covered = 24+24 =
Subtract (i) from (ii)x2, we get
nn
20+18 = 19 km/h
2 48 km
a=24 and b=20 20−18
Speed of the current = 2 =1
Therefore, Speed of stream =
km/h Sol 14. (b)
24−20 = 2km h
2 / 20% = 51
Desired ratio = 19 : 1
Let the speed of boat in still water
Sol 5. (d) Let the speed of boat
Sol 10. (b) = 5 unit
upstream be u and downstream be
Downstream speed of the boat= Speed of stream = 1 unit
d.
14 km/h Downstream speed = 5+1 = 6 unit
Pi

ATQ: 30 24 49
d + u = 20 ----- (i) Upstream speed of the boat = 10 Upstream speed = 5-1 = 4 unit
20 + 8 = 37 ----- (ii)
d u 30 km/h Distance covered = 120 km
Now, Subtract (i) from {3 x (ii)} , Speed of the boat in still water = Time taken = 150 minutes or 2 21
we get, d=24 14+10 = 12 km/h
hours
2
Again, putting the value of d in 72 Upstream Speed = 120
Required time = 12 = 6 hours 5/2 = 48 km/h
eq. (ii), we get, u = 20
According to the question
Therefore, Speed of boat in still
Sol 11. (a) 4 unit = 48
water = 20+24
2 = 22km/h
Downstream speed of the boat = 1 unit = 12
120 = 10 km/h 6 unit = 12 x 6 = 72 km/h
12
Sol 6.(d) Let the speed of the boat 64
Upstream speed of the boat = 8
upstream and downstream be u Sol 15. (b)
and d respectively. = 8 km/h
Speed of the boat in still water = Distance = 12 km
ATQ: 20 30 140
d + u = 60 ---- (i) Time = 80 minutes or 1 31 hours
10+8 = 9 km/h = 9 x 5 = 2.5 m/s
10 + 8 = 49 ---- (ii) 2 18
d u 60

www.ssccglpinnacle.com support@ssccglpinnacle.com Ph. 09729327755, 09817390373 479


Day 53 : Boat and Stream

s
sse
la
_c
ob
Upstream speed = 412/3 = 9 km/h Put the value y in eq (3) Downstream speed of boat = 50 =
bo
2
ah ⇒ x = 21
sm
Downstream speed = 15 1 = 15
25 km/h
ur
yo

So, u = 2 and d = 10 Speed of boat in still water =


e/

km/h
t.m
://

Speed of the boat in still water = 25+7 = 16 km/h


tp

Speed of boat in still water =


ht

2
15+9 = 12 km/h 10 + 2 = 6 km/h 44.8
2 2 Required time = 16 = 2.8 hours
Required time = 36 = 3 hours
12
Sol 20. (a) Sol 2. (b)
Sol 16. (a) Let the upstream speed = u and Upstream speed of boat = 18-6 =
Upstream speed = 11 km/h downstream speed = d 12 km/h
Downstream speed = 23 km/h According to the question Downstream speed of boat =
3 4.5 45
Speed of stream = 23−2 11 = 6 km/h u + d = 60 …….(1) 18+6 = 24 km/h
2.4 + 3.6 39 90 + 90 = 11 1
d u = 60 …….(2) Required time = 12 24 4
Let 1 = x and 1 =y hours
Sol 17. (b) u d
3x + 4.5y = 45
Speed of boat in still water = 5 60
m/s 2x + 3y = 0.5 ……….(3) Sol 3. (c)

e
Downstream speed = 900 = 5
Distance = 200 m 2.4y + 3.6x = 39
60
9×60 3
Upstream speed in high tide = 8y + 12x = 13/6 …….(4) m/s
200 = 20 m/s Upstream speed = 900 = 5 m/s
10 Multiply eq (3) by 6 and subtract 12×60 4
5 5
+

l
Downstream speed in low tide = eq (4) from it Speed of man in still water = 3 4
2
200 = 8 m/s
25 12x + 18y = 3 = 35 m/s

= 15 m/s
ac
Stream speed in high tide = 20-5

Stream speed in low tide = 8-5 =


3 m/s
Required ratio = 15 : 3
8y + 12x = 13/6
- _________
10y = 5/6
1
⇒ y = 12
Put the value y in eq (3)
=
24
35
24 x 18
5

SSC CGL 2019 TIER I


= 5 41 km/h

Sol 1. (c) Let speed of boat = x


=5:1 ⇒ x = 81
km/h and speed of stream = y
So, u = 8 km/h and d = 12 km/h km/h
Sol 18. (c)
nn
distance = time
speed
Upstream speed = 20 km/h Sol 21. (a)
Downstream speed = 30 km/h According to question:
Downstream speed of boat = 3 5 55
Speed of boat in still water = x−y + x+y = 60
(15+3) = 18 km/h
30+20 = 25 km/h 4 + 9 = 85
2 Upstream speed of boat = (15-3) x−y x+y 60

= 12 km/h x+y = 12 km/h


Sol 19. (c) Required time = 1854 + 54 = 7 1
12 2
Time taken to 43.2 km
Let the upstream speed = u and downstream = 43.2= 3.6 hour
km/h 12
Pi

downstream speed = d
According to the question Sol 22. (a) Sol 2. (a) distance = speed * time
10 + 20 = 7 …….(1) 4
u d Upstream speed = 2 = 2 km/h Let speed of boat = a km/h
10 + 20
d u = 11 …….(2) Downstream speed = 4
1/3 = 12 7 km = (a-3)* t
Let 1 = x and 1 =y 13 km = (a+3)* t
u d km/h
10x + 20y = 7 ……….(3) Speed of boat in still water =
10y + 20x = 11 …….(4) 12+2 = 7 km/h 7*(a+3) = 13*(a-3)
2
Multiply eq (3) by 2 and subtract 21
6a = 60
Required time = 2×7 = 1 21 hours
eq (4) from it a = 10 km/h
20x + 40y = 14 Time to travel 44.8 km in still
SSC CGL TIER II 48 = 4 12
water = 4.48 hour = 4 100
10y + 20x = 11 25
Sol 1. (a)
- _________ 28
hours
Upstream speed of boat = 4 =7
30y = 3
⇒ y = 101 km/h

www.ssccglpinnacle.com support@ssccglpinnacle.com Ph. 09729327755, 09817390373 480


Day 53 : Boat and Stream

s
se
s
la
_c
ob
Sol 3. (c) Let speed of boat = x 5 +15 = 3

bo
a 2b 4
ah
km/min
sm
5 + b5 = 5
ur

2a 12
yo

Speed of stream = y km/min


e/

Multiplying eq 2 by 3 and then


t.m

2
://

Speed upstream = (x-y) km/min


tp

Subtracting eq 1 from eq 2
ht

Speed downstream = (x+y)


a =10
km/min 6 × 60 = 36
Time for 6 km = 10
Time = Distance
speed
minutes
54 = 3.6 5.4
x−y + x+y … (i)
58.5 = x5.4
−y
3.6 … (ii)
+ x+y SSC CPO 2019
Multiply (i) by 3 and (ii) by 2, Sol: 4.(b)
and solve the equations. speed of a boat in still water = 15
We get: km/h
x = 30 5 km/min and y = 1 speed of the current = 5 km/h
30
km/min Downstream speed = 15 + 5 =
For 10 km downstream; time 20km/h

e
taken = 10×30 Upstream speed = 15 - 5 = 10
6 = 50 min
km/h
Time for downstream = 20 60 = 3hr
Sol 4. (b) Speed of stream = 3
Time for upstream = 60 = 6hr

l
km/h 10

Let speed of boat = x km/h Total time = 3 + 6 = 9hr

2
12 12

x -24x-9 = 0
x=
ac
x−3 + x+3 = 1
12(x−3+x+3)
x2 −9

24±√576+36
2 =
=1

24±√612
2
24±√36×17 24±6√17
= 2 = 2 =
12 ± 3√17 = 3( 4 ± √17 )
nn
SSC CHSL 2019
1.Sol:(b)
x-y = 15
5 = 3 km/h
x+y = 42 = 7 km/h
6
on solving,
y = 2 km/h
Pi

2.Sol. (c)
Let the speed of motorboat = x
km/h
Let the speed of stream = y km/h
When boat goes along the stream,
x + y = 24 2 = 12 …. (i)
When boat goes against the
stream, x - y = 10
1 = 10 …. (ii)
Add (i) and (ii), we get :-
2x = 22
x = 11 km/h

SSC CGL 2019 TIER-II


3.Sol:(c)

www.ssccglpinnacle.com support@ssccglpinnacle.com Ph. 09729327755, 09817390373 481


s
se
Days 54-60 Mensuration / े िमित

s
la
_c
ob
bo
ah
sm
MENSURATION / े िमित 4) Right angled Triangle/समकोण
ur
yo

ि भुज :- One of the angles is 90 ° .


e/
t.m
://
tp

2- D: Under 2 dimensions we
ht

will study/ ि -आयामी आकृितयों के


अंतगत हम अ यन करगे :
● Triangles/ ि भुज
● Quadrilateral/ चतुभुज
Area / े फल = √4 a2
3
● Polygons/ ब भुज ●
● Circle/ वृ
Height / ऊंचाई = √2 a
3

● Perimeter / प रमाप = 3a Here, p = perpendicular/ ल , b
3 - D: Under 3 dimensions we
= base/ आधार and h =
will study/ ि -आयामी आकृितयों के
2) Scalene Triangle / िवषमबा hypotenuse/ कण
अंतगत हम अ यन करगे :
ि भुज – Sides are of unequal ● Area / े फल= 21 ×b×p
● Cube/घन
● Cuboids/ घनाभ lengths. / सभी भुजाओं की ल ाई ● Perimeter / प रमाप =

e
● Box/ बॉ असमान होंगी | p+b+h
● Cylinder/ बेलन ● Pythagoras Theorem/
● Prism/ि पाइथागोरस मेय :
2 2
● Cone/ शंकु 2
h =p +b

l
● pyramid/ िपरािमड ● Area / े फल =
● Frustum/ िछ क
ac
● Sphere/गोला
√ s(s − a)(s − b )(s − c) , Note: Common Pythagoras
This formula is called triplets:
● Hemisphere/ अधगोला
Hero’s formula. (1, 1, √2 ); (1, 2, √5 ); (3, 4, 5);
● Tetrahedral/ चतु लक
Where, Semi-perimeter, (6, 8, 10); (9, 12, 15); (12, 16,
s = a+b+c
2
20); (5, 12, 13); (7, 24, 25); (7,
Note: For 2D figures, area and 40, 41); (8, 15, 17) .
perimeter are calculated and for ● Perimeter / प रमाप =
a+b+c Try remembering them.
3D figures, volume and surface
area are calculated.
QUADRILATERAL/ चतुभुज
nn
2D आकृित के िलए, े फल और 3) Isosceles Triangle / समि बा
प रमाप की गणना की जाती है और ि भुज : Any two sides and two A figure enclosed by four sides is
3D आकृित के िलए, आयतन और angles are equal. Altitude bisects called a quadrilateral. A
पृ ीय े फल की गणना की जाती the base. / कोई दो भुजा और दो quadrilateral has four angles and
है । कोण सामान होंगे | शीष-ल ,आधार sum of these angles is equal to
को समान भागो म बां टता है | 360 ° .
चार भुजाओं से बनी एक आकृित को
TWO DIMENSIONAL
चतुभुज कहा जाता है । एक चतुभुज
FIGURES/ ि -आयामी आकृितयाँ
म चार कोण होते ह और इन कोणों
Pi

का योग 360 िड ी के बराबर होता है |


TRIANGLE/ ि भुज :
For a triangle with height, h, and
Special Cases:
base, b/ िकसी ि भुज िजसकी
1) Parallelogram/ समा र चतुभुज
ऊंचाई, h है और आधार, b है ;
- It is a quadrilateral with
Area / े फल = 21 ×b×h and ● Area / े फल = opposite sides parallel and equal.
Perimeter / प रमाप = Sum of all b
√ 2
4 4a2 −b एक चतुभुज िजसकी िवपरीत भुजाएं
sides / सभी प ों का योग
● Height / ऊंचाई = समानां तर और बराबर हो|
Special cases:
1) Equilateral Triangle / समभुज
ि भुज - All sides are equal and
√a − ( )2 b 2
2

each angle is 60° . / सभी भुजा


= 1
2 √4a2 − b2
बराबर होगी और सभी कोण 60° के ● Perimeter / प रमाप =
होंगे | a+a+b
= 2a + b

www.ssccglpinnacle.com support@ssccglpinnacle.com Ph. 09729327755, 09817390373 482


/
s
se
Days 54-60 Mensuration / े िमित

s
la
_c
ob
bo
ah
3) Trapezium/ समल चतुभुज – It
sm
● Area =
ur
yo

is a quadrilateral with any one (side)2 = a2 = 21 d2


e/
t.m

pair of opposite sides parallel.


://

● Perimeter = 4 × side =
tp
ht

एक चतुभुज िजसकी िवपरीत भुजाओं 4a


की एक जोड़ी सामानां तर हो|
● Diagonal (d) = a √2

NOTE: Important points about


● Area / े फल = base x Quadrilaterals/चतुभुज के बारे म
height = b x h मह पूण िबंदु
● Perimeter / प रमाप =
1. The diagonals of a
2 (a+ b)
parallelogram bisect each other.
● Opposite angles are equal
Here, a and b are parallel sides एक समा र चतुभुज के िवकण एक
in parallelogram but they
and h is the height or दू सरे को ि िवभािजत करते ह|
are not right angles. /
perpendicular distance between a
समां तर चतभुज म आमने

e
and b. 2. Diagonal of a parallelogram
सामने की भुजाये बराबर
● Area = 21 ×height divides it into two triangles of
होती है पर ु उनके बीच
का कोण समकोण नहीं × (sum of parallel sides) equal area.
होता | एक समा र चतुभुज का िवकण उसे

l
= 21 ×(a + b)×h बराबर छे फल के दो ि भुज म
2) Rhombus / समचतुभुज - It is a
ac िवभािजत करता है |
parallelogram with all four sides 4) Rectangle/ आयत – It is a
equal. The opposite angles in a 3. The diagonals of a rectangle
parallelogram with equal opposite
rhombus are equal./ यह एक sides and each angle is 90 ° .
are of equal lengths and bisect
समा र चतुभुज िजसकी चारो भुजाएं each other.
एक समां तर चतुभुज िजसकी िवपरीत
बराबर होती है | एक समचतुभुज म आयत के िवकण बराबर होते है और
भुजाएं बराबर हो और सभी कोण 90
िवपरीत कोण बराबर होते ह। एक दू सरे को ि िवभािजत करते ह|
° हो.
● Area = Length × Breadth
4. The diagonals of a square are
nn
=L × B
equal and bisect each other at
● Perimeter = 2 (L + B) right angles.
● Diagonal (d) = √L2 + B 2 वग के िवकण बराबर होते है और
● Area of the walls of a एक दू सरे को 90 ° पे ि िवभािजत
rectangular room = करते ह|
2×(L + B )×H
Here, a = side; d1 and d2 are 5. A rhombus has equal diagonals
diagonals. 5) Square/ वग – It is a and they bisect each other at right
Pi

● Area / े फल= parallelogram with all four sides angles.


1 ×d ×d
2 1 2
equal and each angle is equal to समचतुभुज के िवकण बराबर होते है
● Side / भुजा (a) = 90 ° . और एक दू सरे को 90 ° पे
एक समां तर चतुभुज िजसकी भुजाएं ि िवभािजत करते ह|
1
2 √d 1
2
+ d2 2
बराबर हो और सभी कोण 90 ° हो.
● Perimeter / प रमाप = 4a 6. A parallelogram and a
● 4a2 = d1 2 + d2 2 rectangle have equal areas if they
● Diagonals bisect each other are on the same base and between
the same parallel lines.
at right angles and divide into
एक समा र चतुभुज और आयत
two equal parts.
बराबर छे फल के होंगे यिद वो एक
िवकण एक दू सरे को ही आधार और सामानां तर रे खाओं के
समकोण पर काटते है और बीच बने हो|
दो समान भागो म बां टते है |

www.ssccglpinnacle.com support@ssccglpinnacle.com Ph. 09729327755, 09817390373 483


/
s
se
Days 54-60 Mensuration / े िमित

s
la
_c
ob
bo
ah
प रिध
sm
REGULAR POLYGON / ● Circumference/ area will change by
ur
yo

समब भुज (perimeter/ प रमाप ) = (2a + 100 )


a2 %
e/
t.m

2πr
://

In a regular polygon all sides and Whenever there is a


tp
ht

all interior angles are equal. A ● Diameter/ ास = 2r decrease, use negative


polygon is called a pentagon, ● Length of Arc/ चाप की value for ‘a’./ यिद िकसी
hexagon, octagon, nonagon and a लंबाई (AB) = 2πr × 360°
θ ि -आयामी आकृित की
decagon as they have 5, 6, 8, 9,
● Area of sector AOB/ वृ सभी भुजाओं म a% का
10 sides, respectively.
खंड AOB का े फल = प रवतन हो, तो े फल म
एक समब भुज म सभी प और π r2 × 360°
θ (2a + 100 )
a2 % का प रवतन

सभी आं त रक कोण बराबर होते है | होगा ।


एक ब भुज को पंचभुज, षट् भुज,
CIRCULAR RING / सकुलर रं ग
अ भुज, नौभुज तथा दसभुज कहा 3. If all the sides of any two
जा सकता है ोंिक उनम मशः 5, dimensional figure has
6, 8, 9, 10 भुजाएँ होती ह । changed (increased or
If each side of a regular polygon decreased) by a% then its
of ‘n’ sides is equal to ‘a’ then: perimeter also changes by

e
यिद n भुजाओं वाले एक सम ब भुज a%. In case of circle such
की ेक भुजा a के बराबर है , तो : changes takes place
● Area of regular pentagon/ because of the change in
सम पंचभुज का े फल = radius (or diameter)./ यिद

l
एक ि -आयामी आकृित की
5a2 √
3
ac 4
Here, R = radius of bigger ring/ सभी भुजाओं म a % का
● Area of regular hexagon/ प रवतन ( वृ या कमी )
बड़े वलय की ि ा , r = radius of
सम षट् भुज का े फल = आ है , तो प रमाप भी a%
smaller ring/ छोटे वलय की ि ा
6a2 √ से बदल जाएगा । वृ के
3
4 ● Area / े फल =
● Each exterior angle/ 2 2
π(R − r ) मामले म, ऐसे प रवतन
ेक बाहरी कोण = 360° ि ा ( या ास ) म
● Difference in
n
प रवतन के कारण होते ह ।
● Each interior angle/ circumference of both the
ेक आतं रक कोण = rings/ दोनों वलयों की
4. If area of a square is ‘a’
180 ° − Exterior angle / प रिध म अंतर =
nn
square units. Then the
बाहरी कोण 2πR − 2πr
area of the circle formed
● Number of diagonals/ with the same perimeter
िवकण की सं ा = Short Cut methods/Tricks is given by square units./
{ n(n−1)
2 −n} यिद एक वग का े फल a
1. If the length and breadth वग इकाई है , तो इसी
CIRCLE/ वृ of a rectangle are प रमाप के साथ िनिमत वृ
increased by a% and b%, के े फल की गणना वग
It is a plane figure enclosed by a the area of the rectangle इकाइयों से की जाती है ।
Pi

line on which every point is will be increased by


equally distant from a fixed point (a + b + 100
ab %
) 5. Area of a square
(centre) inside the circle. If any of the two sides of inscribed in a circle of
rectangle is decreased radius ‘r’ is equal to 2r2 .
यह उस रे खा से िघरी सपाट आकृित
then use ‘–ve’ values for /ि ा ‘r’ वाले एक वृ के
होती है , िजसके ेक िबंदु की दू री
a. / यिद िकसी आयत की भीतर थत वग का े फल
वृ के भीतर एक थर िबंदु ( क )
लंबाई और चौड़ाई a% तथा 2r2 होता है ।
से बराबर होती है ।
b% बढ़ा दी जाती है , तो इस
आयत का े फल 6. The area of the largest
(a + b + 100 )
ab % से बढ़ triangle inscribed in a
जाएगा । semi-circle of radius r is
equal to r2 . / ि ा r वाले
2. All the sides of any two अधवृ के भीतर सबसे बड़े
dimensional figure ि भुज का े फल r2 होगा
● Area / े फल = πr2 changed by a%, then its ।

www.ssccglpinnacle.com support@ssccglpinnacle.com Ph. 09729327755, 09817390373 484


/
s
se
Days 54-60 Mensuration / े िमित

s
la
_c
ob
bo
ah
Here, a = length of the side/ भुजा
sm
7. If a pathway of width x is volume – Internal
ur
yo

made inside or outside a की ल ाई । volume( or capacity) /


e/
t.m

rectangular plot of length बॉ म लगी साम ी का


://


tp
ht

L and breath B, then area CUBOID/ घनाभ आयतन = बा आयतन -


of the pathway is/ यिद आतं रक आयतन ( या
लंबाई L तथा चौड़ाई B वाले A rectangular body having 3D धा रता ) =
आयताकार भू-खंड के rectangular shape, is called a lbh − (l − 2t)(b − 2t)
भीतर या बाहर चौड़ाई x cuboid./ एक आयताकार ठोस, जो (h − 2t)
वाला एक माग बनाया जाता ि -आयामी होता है , घनाभ कहलाता
है , तो इस माग की लंबाई है । ROOM/ म
होगी :
A rectangular room has four walls
(i) 2x(L + B + 2x) , if
(surfaces) and opposite walls
path is made outside the
have equal area.
plot./ यिद माग को भूखंड
के बाहर बनाया गया है । एक आयताकार म की चार दीवार
(ii) 2x(L + B − 2x) , if होती है |

e
path is made inside the ● Total Area of walls/
● Volume/ आयतन =
plot./ यिद माग को भूखंड दीवारों का कुल े फल =
l×b×h
के भीतर बनाया गया है । 2(l×b)×h
● Total surface area/ कुल
● Total volume of the
पृ े फल =
room/ कमरे का कुल

l
8. If two paths, each of 2(lb + bh + lh)
width x are made parallel आयतन = l×b×h
ac ● Diagonal/ िवकण =
to length (L) and breadth ● Area of floor or roof/ छत
(B) of the rectangular √l2 + b2 + h2 की फश का े फल =
plot in the middle of the l×b
plot crossing each other, BOX/ बा
then/ िकसी आयताकार A box has its shape like a cube or CYLINDER/ बेलन
भूखंड के बीच म, यिद cuboid. The amount that a box
उसकी लंबाई L तथा चौड़ाई can hold or contain is called the
B के समानां तर दो माग capacity of the box. Capacity
बनाए गए ह, जो एक-दू सरे
nn
means internal volume.
को ित े द करते ह, तो एक बा का आकार घन या घनाभ
Area of the path/ माग का जैसा होता है | बा का आं त रक
े फल = x(L + B − x) आयतन उसकी मता दशाता है |

THREE DIMENSIONAL
FIGURES/ ि -आयामी आकृितयाँ
● Volume of cylinder = area
of base × height / बेलन
CUBE/ घन का आयतन = आधार का
Pi

● Surface area of an open


े फल x ऊंचाई = π r2 h
box/ खुले बॉ का पृ
े फल = 2 ( length/ ● Curved surface area/ व
लंबाई + breadth/ चौड़ाई ) पृ े फल = Perimeter
× height/ ऊंचाई + of base/ आधार का प रमाप
length/ लंबाई × breadth/ × height/ ऊंचाई = 2πrh
चौड़ाई = ● Total surface Area / कुल
All sides are equal. It has six पृ े फल = curved
faces. / सभी भुजाएं बराबर होती है | 2 (l + b) ×h + l×b
● Capacity of box/ बॉ की surface area/ व पृ
इसकी छे सतह होती है | े फल + area of both
● Volume/ आयतन = a3 धा रता/ मता =
(l − 2t)(b − 2t)(h − 2t) ; the circles/ दोनों वृ ों का
● Total surface area/ कुल े फल =
where, t = thickness of
पृ े फल = 6 a2 2πrh + 2πr2 = 2π (r + h) r
box/ बॉ की मोटाई
● Diagonal/ िवकण = a√3 ● Volume of the material of
the box = External

www.ssccglpinnacle.com support@ssccglpinnacle.com Ph. 09729327755, 09817390373 485


/
s
se
Days 54-60 Mensuration / े िमित

s
la
_c
ob
bo
ah
िनमाण चार समबा ि भुजों को
sm
HOLLOW CYLINDER /
ur
yo

खोखला बेलन िमलाकर िकया जाता है ।


e/
t.m
://
tp
ht

● Volume/ आयतन =
1 ×base area×height
3 / 3
1

× आधार का े फल ×
● Volume/ आयतन (V) =
ऊंचाई = 31 πr2 h 1
3 ×base area×height या
● Slant height/ ितयक ऊंचाई 1 × आधार का े फल ×
● Volume of hollow 3
cylinder/ खोखले बेलन का (l) = √r2 + h2 ऊंचाई

e
आयतन = π (R2 − r2 )h ● Curved surface area/ व ⇒ V =
पृ े फल
● Curved surface area/ व = 1 √3 2
(
√6 √6
3 × 4 a × 12 a + 4 a )
पृ े फल = πrl = πr√r2 + h2
2π(R + r)h ● Total surface area / कुल ⇒ V = √122 a3 !! Remember

l
● Total surface area/ कुल पृ े फल = πrl + πr2 this formula/ सू याद कर !!
पृ ac े फल = = πr(l + r) ● Total surface area/ कुल
2
2π (R + r) h + 2π(R − r ) 2 Note: If the base is not round, it पृ े फल =
3× √ a2 + √ a2
3 3
Where, R = External radius of will be called a pyramid. A 4 4
box, r = internal radius box, h pyramid can have various shapes ● Lateral surface area/ पा
= height/ जहाँ , R = बॉ की of the base example: square,
े फल = 3× √ a2
3
rectangular, triangular etc. / यिद पृ
बाहरी ि ा, r = बॉ की 4

आतं रक ि ा, h = ऊंचाई आधार गोल नहीं हो, तो इसे िपरािमड


कहा जाएगा । एक िपरािमड म FRUSTUM OF CONE / शंकु का
आधार के कई आकार हो सकते ह िछ क
nn
PRISM/ ि
जैसे : वग, आयत, ि भुज आिद If a cone is cut by a plane parallel
to its base, so as to divide the
● Volume of prism/ ि
PYRAMID/ िपरािमड cone into two parts: upper part
का आयतन = area of
and lower part, then the lower
base/ आधार का े फल
● Volume/ आयतन = part is called frustum./ यिद एक
× height/ ऊंचाई
1 ×base area×height या शंकु को दो िह ों म िवभािजत करने
● Lateral surface area/ पा 3
के िलए, उसके आधार के समानां तर
1 × आधार का े फल ×
पृ े फल = Perimeter 3 काटा जाए, तो िनचले िह े को
of base/ आधार का प रमाप ऊंचाई िछ क कहते ह ।
Pi

× height / ऊंचाई ● Slant height / ितयक


● Total surface Area = ऊंचाई (l) = √r2 + h2
Lateral surface area + ● Lateral surface area/ पा
area of base and top पृ े फल =
surfaces/ कुल पृ े फल P erimeter × slant height
= पा पृ े फल + 2

आधार का े फल तथा ● कुल पृ े फल = पा


शीष का े फल पृ े फल + आधार का
े फल ● Slant height/ ितयक ऊंचाई
CONE / शंकु
A solid and round body with a
TETRAHEDRON/ चतु लक
(l) = √h + (R − r)
2 2

It is a 3-D figure made by joining ● Curved Surface Area/ व


round base and pointed beek./ एक पृ े फल = π (R + r) l
आकृित िजसका आधार गोलाकार है four equilateral triangles./ यह एक
और ऊंचाई नुकीली| ि -आयामी आकृित है , िजसका

www.ssccglpinnacle.com support@ssccglpinnacle.com Ph. 09729327755, 09817390373 486


/
s
se
Days 54-60 Mensuration / े िमित

s
la
_c
ob
bo
ah
Total surface area/ कुल
sm
● surface area of the original cube (d) 128 cm3
ur
yo

पृ े फल = to the sum of the surface areas of


e/
t.m

2 2
://

π (R + r) l + πR + πr three of the smaller cubes is : Q4. The area of a triangle is 15 sq


tp
ht

2
= π{(R + r) l + R + r2 } 13824 घन सेमी आयतन वाले एक cm and the radius of its incircle is
● Volume/ आयतन = ठोस घन को बराबर आयतन वाले 3 cm. Its perimeter is equal to:
πh (r 2 + R2 + r R)
3 आठ घनों म काटा जाता है | आरं िभक एक ि भुज का े फल 15 वग सेमी
घन के पृ े फल तथा तीन छोटे है और इसके वृ की ि ा 3 सेमी
SPHERE/ गोला घनों के पृ े फलों के योग का है । इसकी प रिध िकतनी है ?
● Volume of sphere/ गोला अनुपात ा है ? SSC CGL 4 June
का आयतन = 34 πr3 SSC CGL 4 June 2019(Evening)
● Total surface area/ कुल 2019(Morning) (a) 12 cm
पृ े फल = 4πr2 (a) 2 : 3 (b) 20 cm
(b) 4 : 3 (c) 5 cm
HOLLOW SPHERE OR (c) 8 : 3 (d) 10 cm

e
SPHERICAL SHELL/ खोखला (d) 2 : 1
गोला : Q5. The volume of a metallic
● Volume of hollow sphere/ Q2. How much iron sheet (in m2 ) cylindrical pipe is 7480 cm3 . If
खोखले गोले का आयतन = will be needed to construct a its length is 1.4 m and its external
4 3 3
3 π(R − r )

l
rectangular tank measuring radius is 9 cm, then its thickness
● Internal surface area/ 10 m × 8 m × 6 m , if a circular (given π = 22
ac 7 ) is:
आतं रक पृ े फल = opening of radius one metre is to
2 एक धातु के बेलनाकार पाइप का
4πr be left at the top of the tank? आयतन 7480 घन सटीमीटर है । यिद
● External surface area/ (correct to one decimal place) इसकी लंबाई 1.4 मीटर है और
बा पृ े फल = 4πR2 10 मीटर 8 मीटर 6 मीटर िवमाओं इसकी बाहरी ि ा 9 सटीमीटर है ,
Here R = external radius/ वाले आयताकार टक के िनमाण के तो इसकी मोटाई ात कीिजये?
बाहरी ि ा and r = internal
िलए लोहे की िकतनी शीट (वग मीटर SSC CGL 6 June 2019
radius/ आतं रक ि ा
म ) की आव कता होगी, अगर एक (Morning)
मीटर ि ा वाला वृताकार मुख टक
nn
HEMISPHERE/ अधगोल (a)1 cm
के शीष पर छोड़ा जाना है ? (b)0.8 cm
SSC CGL 4 June 2019 (c)0.9 cm
(Afternoon) (d)1.2 cm
(a) 371.6
(b) 370.4 Q6. The length of the metallic
(c) 372.9 pipe is 7.56 m. Its external and
(d) 370.8 internal radii are 2.5 cm and 1.5
● Volume of the
Pi

cm respectively. If 1 cubic cm of
hemisphere/ अधगोले का Q3. The areas of the three the metal weigh 7.5 g, then the
आयतन = 32 πr3 adjacent faces of a cuboid are 32 weight of the pipe is : ( Take 𝜋 =
● Total surface area/ कुल cm 2 ,24 cm 2 and 48 cm 2 . What 22/7)
पृ े फल = 3πr2 is the volume of the cuboid? धातु के पाइप की लंबाई 7.56 मीटर
● Curved surface area/ व एक घनाभ के तीन आस फलकों है । इसकी बाहरी और आं त रक
पृ े फल = 2πr2 का े फल 32 cm 2 ,24 cm 2 और ि ा मशः 2.5 सेमी और 1.5 सेमी
Where, r = radius/ ि ा है । यिद धातु के 1 घन सेमी का वजन
48 cm 2 ह। घनाभ का आयतन ात
कीिजये? 7.5 ाम है , तो पाइप का भार ात
Variety Questions कीिजये ?
SSC CGL 4 June
2019(Evening) SSC CGL 6 June 2019
Q1. A solid cube of volume (Afternoon)
(a) 192 cm3
13824 cm3 is cut into 8 cubes of (a) 72.82 kg
(b) 256 cm3
equal volumes. The ratio of the (b) 70.14 kg
(c) 288 cm3

www.ssccglpinnacle.com support@ssccglpinnacle.com Ph. 09729327755, 09817390373 487


/
s
se
Days 54-60 Mensuration / े िमित

s
la
_c
ob
bo
ah
sm
(c) 71.28 kg (b) 5 : 4 (d) 16
ur
yo

(d) 69.68 kg (c) 7 : 6


e/
t.m
://

(d) 3 : 2 Q13. Six cubes, each of edge 2


tp
ht

Q7. A sector is cut out from a cm, are joined end to end. What is
circle of diameter 42 cm. If the Q10. The radius of a sphere is the total surface area of the
angle of the sector is 150°, then reduced by 40%. By what resulting cuboid in cm 2 ?
its area ( in square cm ) is : (Take percent, will its volume decrease? 6 घन, िजनका ेक का फलक 2
𝜋 = 22/7 ) एक गोले की ि ा को 40% कम सेमी है ,सभी को फलकों के अनु प
42 cm ास के एक वृत म से एक कर िदया जाता है , इसका आयतन जोड़े जाते है , प रणामी घनाभ का
भाग काटा जाता है ,यिद इस भाग का िकतने ितशत घटे गा? े फल ात कीिजये ?
कोण 150° तो इसका े फल SSC CGL 10 June 2019 SSC CGL 13 June
िकतना होगा ? (Afternoon) 2019(Morning)
SSC CGL 7 June (a) 60% (a)96
2019(Morning) (b) 64% (b)144

e
(a) 564 (c) 72.5% (c)104
(b) 574 (d) 78.4% (d)128
(c) 580.6
(d) 577.5 Q11. The radii of two circular Q14. The area of a field in the

l
faces of the frustum of a cone of shape of a hexagon is 2400√3
Q8. The area of a field in the
ac height 21 cm are 3 cm and 2 cm square metre. What will be the
shape of a triangle with each side respectively. What is the volume cost of fencing it at Rs. 18.50 per
x metre is equal to the area of of the frustum of the cone in metre?
another triangular field having cubic cm? (Take 𝜋 = 22/7) एक षट् भुज का े फल 2400 वग
sides 50m , 70m and 80m. The 21 सेमी ऊंचाई वाले शंकु के िछ क मीटर है । 18.50 ित मीटर पये के
value of x is closest to : ेक के दो वृताकार फलकों की ि ा िहसाब से इस पर बाड़ लगाने की
भुजा x मीटर वाले एक ि भुज का मशः 3 सेमी और 2 सेमी है । घन लागत ा होगी ?
े फल एवं िवमाओं 50m , 70m सेमी म शंकु के िछ क का आयतन SSC CHSL 1 July
और 80m वाले ि भुज का े फल िकतना होगा ?
nn
2019(Evening)
समान है , x का मान ात कीिजये ? SSC CGL 11 June (a) Rs. 4440
SSC CGL 7 June 2019(Morning) (b) Rs. 5920
2019(Afternoon) (a) 154 (c) Rs. 5550
(a) 65.5 (b) 286 (d) Rs. 5180
(b) 63.2 (c) 345
(c) 62.4 (d) 418 Q15. The volume of a right
(d) 61.8 circular cone is 924 cubic cm. If
Q12. A sphere of radius 4 cm is it's height is 18 cm, then the area
Pi

Q9. The curved surface area and melted and recast into smaller of it’s base (In square cm) is:
volume of a cylinder are 264 spheres of radii 2 cm each. How एक ल वृतीय शंकु का आयतन
square cm and 924 cubic cm, many such spheres can be made? 924 घन सेमी है । यिद इसकी ऊंचाई
respectively. What is the ratio of 4 सेमी ि ा वाले एक गोले को 18 सेमी है , तो इसके आधार का
its radius to height? (Take 𝜋 = िपघलाया जाता है और ेक 2 सेमी े फल (वग सेमी म) है :
22/7 ) ि ा के छोटे गोले म बदल िदया SSC CHSL 2 July
एक बेलन का पृ ीय े फल और जाता है । ऐसे िकतने गोले बनाए जा 2019(Morning)
आयतन मशः 264 वग सेमी और सकते ह? (a) 154
924 घन सेमी है । इसकी ि ा और SSC CGL 12 June (b) 132
ऊँचाई का अनुपात ा है ? 2019(Evening) (c) 176
SSC CGL 10 June 2019 (a) 4 (d) 198
(Morning) (b) 8
(a) 4 : 3 (c) 32

www.ssccglpinnacle.com support@ssccglpinnacle.com Ph. 09729327755, 09817390373 488


/
s
se
Days 54-60 Mensuration / े िमित

s
la
_c
ob
bo
ah
बीच के दो वृ ों के बीच के े फल
sm
Q16. The curved surface area and
ur
yo

volume of a cylindrical pole are Q19. A wire is in the shape of a का अनुपात ा है ?


e/
t.m
://

132 square metres and 528 cubic rectangle whose sides are in the SSC CGL 6 June
tp
ht

metres, respectively. What is the ratio 7 : 4. It was initially in the 2019(Evening)


height (In m) of the pole? (Take 𝜋 shape of a circle of radius , very (a) 4:7
= 22/7) nearly equal to 31.5 cm. The (b) 5:9
एक बेलनाकार का व पृ ीय length of smaller side of the (c) 4:5
े फल और आयतन मशः 132 rectangle is : (Take 𝜋 = 22/7) (d) 3:8
वग मीटर और 528 घन मीटर है । एक तार एक आयत के भीतर है
की ऊंचाई (मीटर म) ा है ? िजसकी भुजाएं 7: 4 के अनुपात म Q22. The parallel sides of a
SSC CHSL 2 July ह।आर म यह वृत की ि ा के trapezium are 20 cm and 10 cm
2019(Afternoon) बराबर था, लगभग 31.5 सेमी। and its non-parallel sides are equal
(a) 2 21 आयत की छोटी भुजा की लंबाई to each other. If its area is 180
(b) 3 85 िकतनी है : cm2 , then what is the length (in

e
(c) 3 21 SSC CHSL 3 July cm) of each non parallel side? /
2019(Afternoon) एक समलंब चतुभुज की समानां तर
(d) 2 85
(a) 44 cm भुजाएं 20 सेमी तथा 10 सेमी की ह
(b) 36 cm तथा इसकी गैर-समानां तर भुजाएं एक
Q17. The radius of the base of a

l
(c) 40 cm दू सरे के बराबर ह | यिद इसका
cylinder is 7 cm and its curved
ac (d) 32 cm े फल 180 वग सेमी है , तो ेक
surface area is 440 square cm. Its
गैर-समानां तर भुजा की लंबाई ात
volume (In cubic cm) will
Q20. The length of a rectangular कर |
be:(Take 𝜋 = 22/7)
park is 20m more than its breadth. SSC CHSL 04 July
एक बेलन के आधार की ि ा 7
If the cost of fencing the park at 2019(Morning)
सेमी है और इसका व पृ ीय
Rs. 53 per metre is Rs. 21,200, (a) 11
े फल 440 वग सेमी है । इसका
then what is the area (In square (b) 13
आयतन िकतना होगा?
metres) of the park? (c) 12
SSC CHSL 2 July
एक आयताकार पाक की लंबाई (d) 15
nn
2019(Evening)
इसकी चौड़ाई से 20 मीटर अिधक
(a) 1760
है । यिद 53 ित मीटर की दर से Q23. Diagonals of a rhombus are
(b) 1430
पाक म बाड़ लगाने की लागत respectively 4 cm and 12 cm. Its
(c) 1540
21,200 है । तो पाक का े फल (वग area (in cm2 ) is equal to :
(d) 1650
मीटर म) ा है ? एक समचतुभुज के िवकण मश 4
SSC CHSL 3 July सेमी और 12 सेमी ह | इसका
Q18. A circle circumscribes a
2019(Evening) े फल ( वग सेमी म ) ात कर |
rectangle whose sides are in the
(a) 9504 SSC CHSL 09 July
Pi

ratio 4 : 3. If the perimeter of the


(b) 8925 2019(Evening)
rectangle is 56 cm, then what is
(c) 9240 (a) 12
the area (In square cm) of the
(d) 9900 (b) 24
circle?
(c) 36
एक वृ एक आयत को घेरे ए है
Q21. The radii of three concentric (d) 8
िजसकी भुजाएँ 4: 3 म ह। यिद आयत
circles are in the ratio of 4:5:7.
की प रिध 56 सेमी है , तो वृ का
What is the ratio of the area Q24. In triangle ABC, the length
े फल (वग सेमी) ा है ?
between the two inner circles to of BC is less than twice the length
SSC CHSL 3 July
that between the two outer circles of AB by 2 cm. The length of AC
2019(Morning)
? exceeds the length of AB by 10
(a) 70𝜋
तीन संकि त वृ ों की ि ा 4: 5: 7 cm. The perimeter is 32cm. The
(b) 96𝜋
के अनुपात म है । दो बाहरी वृ ों के length (in cm) of the smallest side
(c) 90𝜋
of the triangle is :
(d) 100𝜋

www.ssccglpinnacle.com support@ssccglpinnacle.com Ph. 09729327755, 09817390373 489


/
s
se
Days 54-60 Mensuration / े िमित

s
la
_c
ob
bo
ah
ि भुज ABC म, BC की लंबाई AB (a) 13.13% increase/ बढ़ जाएगा
sm
SSC CHSL 10 July 2019(Afternoon)
ur
yo

की लंबाई के दोगुने से 2 कम है | AC (b) 1.21% increase/ बढ़ जाएगा (a) 41


e/
t.m
://

की लंबाई AB की लंबाई से 10 सेमी (c) 1.21% decrease/ कम हो जाएगा (b) 1


tp
ht

2
अिधक है | प रमाप 32 सेमी है | (d) 13.13% decrease/ कम हो (c) 1
ि भुज की सबसे छोटी भुजा की लंबाई जाएगा √2
(d) 1
(सेमी म ) ात कर | SSC CHSL 05 2
√2
July 2019 (Afternoon) Q28. What is the area of a
(a) 4 rhombus (in cm2 ) whose side is Q31. The area of a sector of a
(b) 10 10 cm and the smallest diagonal circle with central angle 60∘ is A.
(c) 8 is 12 cm? The circumference of the circle is
(d) 6 उस समचतुभुज का े फल (वग C. Then A is equal to :
सेमी म ) ात कर िजसकी भुजा 10 िकसी वृ के खंड का े फल A है
Q25. If each side of a rectangle is सेमी है तथा सबसे छोटा िवकण 12 िजसका क ीय कोण 60∘ है | इस
increased by 22%, then its area सेमी का है | वृ की प रिध C है | तो A िकसके

e
will increase by : SSC CHSL 04 July 2019 बराबर है ?
यिद एक आयत की ेक भुजा को (Afternoon) SSC CHSL 10 July 2019
22% से बढ़ा िदया जाए, तो इसका (a) 120 (Evening)
े फल िकतना बढ़ जाएगा ? (b) 192 c2
(a) 6π

l
SSC CHSL 08 July (c) 96 c2
(b) 18π
2019(Morning)ac (d) 50
(c) c2
(a) 44% 24π
(d) c2
(b) 50% Q29. Twelve sticks, each of 4π
(c) 46.65% length one unit, are used to form
(d) 48.84% an equilateral triangle. The area Q32. The two diagonals of a
of the triangle is : rhombus are respectively, 14 cm
Q26. If each side of a rectangle is 12 छिड़याँ , िजनम से ेक की and 48 cm. The perimeter of the
decreased by 11%, then its area लंबाई एक इकाई है , का इ ेमाल rhombus is equal to :
will decrease by : िकसी समबा ि भुज को बनाने के एक समचतुभुज के दो िवकण मशः
nn
यिद िकसी आयत की ेक भुजा िलए िकया जाता है | इस ि भुज का 14 सेमी और 28 सेमी के ह |
को 11% से कम कर िदया जाए, तो े फल होगा : समचतुभुज का प रमाप ात कर |
इसका े फल िकतना कम हो SSC CHSL 10 July 2019 SSC CHSL 11 July 2019
जाएगा ? (Morning) (Morning)
SSC CHSL 09 July 2019 (a) 3 √3 sq units (a) 120 cm
(Morning) (b) 160 cm
(b) 2 √3 sq units
(a) 21.69% (c) 80 cm
(c) 4 √3 sq units
(b) 20.79% (d) 100 cm
Pi

(c) 13.13% (d) 8 √3 sq units


(d) 26.78% Q33. The volume of a right
Q30. Equilateral triangles are circular cone is equal to that of a
Q27. If the length of a rectangle drawn on the hypotenuse and one sphere, whose radius is half the
is decreased by 11% and the of the perpendicular sides of a radius of the base of the cone.
breadth is increased by 11%, its right-angled isosceles triangle. What is the ratio of the radius of
area will undergo : Their areas are H and A the base to the height of the cone
यिद िकसी आयत की लंबाई को 11% respectively. HA is equal to : / ?
से कम कर िदया जाए और इसकी िकसी समकोण समि बा ि भुज के एक ल वृ ीय शंकु का आयतन
चौड़ाई 11% से बढ़ा दी जाए, तो कण तथा एक ल भुजा पर समबा उस गोले के आयतन के बराबर है ,
इसका े फल : ि भुज खींचे जाते ह | उनके े फल िजसकी ि ा शंकु के आधार की
SSC CHSL 09 July 2019 मश H और A ह | HA का मान ि ा से आधी है | शंकु के आधार
(Afternoon) िकसके बराबर होगा ?

www.ssccglpinnacle.com support@ssccglpinnacle.com Ph. 09729327755, 09817390373 490


/
s
se
Days 54-60 Mensuration / े िमित

s
la
_c
ob
bo
ah
की ि ा और शंकु की ऊंचाई म
sm
(b) 11 cm2 dimension 30cm × 25cm × 12cm
ur
yo

ा अनुपात है ? (c) 10 cm2 can be painted with the liquid in


e/
t.m
://

SSC CHSL 11 July 2019 (d) 12 cm2 this container.


tp
ht

(Afternoon) एक बतन म 11.28 m2 े फल को


(a) 1:4 Q37. Find the inner surface area रं गने के िलए पया तरल है ।
(b) 1:2 of all walls of a rectangular room िवमाओं 30cm × 25cm × 12cm के
(c) 4:1 with length 7 m breadth 5 m and िकतने ब े इस बतन के तरल ारा
(d) 2:1 height 3.5 m रं गे जा सकते है ?
एक आयताकार कमरे की सभी SSC CPO 16 March 2019
Q34. ABCD is a rhombus with दीवारों का आतं रक पृ े फल (Morning)
each side being equal to 8 cm. If ात कर िजसकी लंबाई 7 मी. चौड़ाई (a) 40
BD = 10 cm, AC = 2 √x cm, what 5 मी और ऊंचाई 3.5 मी है | (b) 24
is the value of √x + 10 ? SSC CPO 16 March 2019 (c) 32
ABCD एक समचतुभुज है िजसकी (Morning) (d) 12

e
ेक बराबर भुजा 8 सेमी की है | (a) 84 m2
यिद BD = 10 सेमी, AC = 2 √x (b) 168 m2 Q41. The radius of a cylinder is
सेमी है , तो √x + 10 का मान ा (c) 126 m2 increased by 120% and its height
होगा ? (d) 42 m2 is decreased by 40%. What is the

l
SSC CHSL 11 July 2019 percentage increase in is volume?
(Afternoon) ac Q38. A square cardboard with / एक बेलन की ि ा 120% बढ़ा दी
side 3 m is folded through one of जाती है तथा इसकी ऊंचाई 40%
(a) 2 √3
its diagonal to make a triangle, कम कर दी जाती है | इसके आयतन
(b) 3 √2 म िकतने ितशत की वृ होगी ?
the height of the triangle is:
(c) 7 SSC CPO 12 March
3 मी भुजा वाले एक मोटे कागज़ को
(d) 5 2019(Evening)
इसके एक िवकण पर से मोड़कर
ि भुज बनाया जाता है | इस ि भुज (a) 180.6%
Q35. The area of parallelogram is (b) 212.8%
की ऊंचाई ा होगी
338 m2 . If its altitude is twice the
nn
SSC CPO 16 March 2019 (c) 190.4%
corresponding base, its base is: (d) 175.4%
(Morning)
समां तर चतुभुज का े फल 338
(a) 3 m
m2 है । यिद इसकी ऊंचाई इसके √2 Q42. The sides of a triangle are 8
आधार से दोगुनी है , तो इसका आधार (b) 2 √3 m cm, 15 cm, and 17 cm
िकतना है ? (c) 3 √2 m respectively. At each of its
SSC CPO 16 March 2019 (d) 2 m vertices, a circle of radius 3.5 cm
(Morning) √3
is drawn. What is the area of the
(a) 13
Pi

triangle excluding the portion


Q39. The surface area of a cube is
(b) 14 covered by the sectors of the
1176 cm2 Its volume is:
(c) 26 circle ? π = 22 एक ि भुज की
एक घन का पृ ीय े फल 1176 7
(d) 28 भुजाएं मशः 8 सेमी, 15 सेमी और
cm2 है ,इसका आयतन ा होगा ?
SSC CPO 16 March 2019 17 सेमी की ह | इसके ेक शीष पर
Q36. The base of an isosceles 3.5 सेमी ि ा वाला एक वृ खींचा
triangle is 6 cm and its perimeter (Morning)
(a) 3486 cm3 जाता है | वृ के खंडो ारा घेरे गए
is 16 cm. Its area is भाग को छोड़ते ए इस ि भुज का
एक समि बा ि भुज का आधार 6 (b) 3964 cm3
े फल ात कर |
सेमी और इसका प रमाप 16 सेमी है (c) 3206 cm3
SSC CPO 12 March 2019
| इसका े फल ात कर | (d) 2744 cm3
(Evening)
SSC CPO 16 March 2019 (a) 23.5 cm2
(Morning) Q40. The liquid in a container is
(b) 21.5 cm2
(a) 9 cm2 sufficient to paint an area of 11.28
(c) 47 cm2
m2 . How many boxes of

www.ssccglpinnacle.com support@ssccglpinnacle.com Ph. 09729327755, 09817390373 491


/
s
se
Days 54-60 Mensuration / े िमित

s
la
_c
ob
bo
ah
sm
(d) 40.75 cm2 SSC CPO 14 March 2019 (c)175 rs
ur
yo

(Morning) (d)186 rs
e/
t.m
://

Q43. The sides of a triangle are (a)128 cm


tp
ht

24cm,45cm and 51cm. At each of (b)80 cm Q49. A steel vessel has a base of
its vertices, circles of radius (c)70 cm length 60 cm and breadth 30 cm.
10.5cm are drawn. What is the (d)256 cm Water is poured in the vessel. A
area of the triangle, excluding the cubical steel box having edge of
portion covered by sectors of the Q46. The length of diagonal of a 30 cm is immersed completely in
circles? ( π = 22
7 )
square whose area is 64 m2 is: the vessel. By how much will the
एक ि भुज की भुजाएं 24 सेमी, 45 उस वग के िवकण की लंबाई ात water rise?
सेमी तथा 51 सेमी की ह | इसके कर िजसका े फल 64 वग मीटर है इ ात के एक बतन के आधार की
ेक शीष पर 10.5 सेमी ि ा वाले | लंबाई 60 सेमी तथा चौड़ाई 30 सेमी
वृ खींचे जाते ह | वृ के खंडो ारा SSC CPO 14 March 2019 है | इस बतन म पानी डाला जाता है |
घेरे गए भाग को छोड़ कर इस ि भुज (Morning) 30 सेमी िकनारा वाले इ ात के एक

e
का े फल ा होगा ? (a)4 √2 m घनीय िड े को इस बतन म पूणतः
SSC CPO 13 March डूबा िदया जाता है | पानी िकतना
(b)8 √2 m
2019(Evening) बढ़े गा ?
(c)4 m
(a)244.75 cm2 SSC CPO 16 March 2019
(d)8 m

l
(b)366.75 cm2 (Evening)
(c)464.75 cm2ac (a)12 cm
Q47. The unequal side of an
(d)327.75 cm2 (b)15 cm
isosceles triangle is 2cm. The
(c)10 cm
medians drawn to the equal sides
(d)9 cm
Q44.The radius of a cylinder is are perpendicular. The area of the
increased by 150 % and its height triangle is :
Q50. The perimeter of a square is
is decreased by 20 %. What is the एक समि बा ि भुज की असमान
equal to the perimeter of a
percentage increase in its भुजा 2 सेमी की है | बराबर भुजाओं
rectangle of length 16cm and
volume? एक बेलन की ि ा 150 पर खींची गयी म काएं ल ह |
breadth 14 cm. Find the
nn
% बढ़ा दी जाती है तथा इसकी इस ि भुज का े फल ात कर |
circumference of a semicircle
ऊंचाई 20 % कम कर दी जाती है | SSC CPO 16 March 2019
whose diameter is equal to the
इसके आयतन म िकतने ितशत की (Evening)
side of the square.
वृ होगी ? (a) 2 cm2
एक वग का प रमाप िकसी आयत के
SSC CPO 12 March 2019 (b) 3 cm2
प रमाप के बराबर है िजसकी लंबाई
(Morning) (c) 5 cm2
16 सेमी और चौड़ाई 14 सेमी है | उस
(a) 400% (d) 1 cm2
अधवृ की प रिध ात कर िजसका
(b) 600%
ास वग की भुजा के बराबर है ?
Pi

(c) 500% Q48. What will be total cost of SSC CPO 16 March 2019
(d) 80% polishing curved surface of a (Evening)
wooden cylinder at rate of Rs 20 (a)38.57 cm
Q45. The area of each square of a per m2 , if its diameter is 40 cm (b)21.57 cm
chessboard having 64 squares is 4 and height is 7m? (c)23.57 cm
cm2 . If there is a border on all लकड़ी की बेलन की व सतह को (d)25.57 cm
the sides of the chessboard of 2 20 पये ित वग मीटर की दर से
cm, then the perimeter of the पॉिलश करने की कुल लागत ा Q51. Original breadth of a
chessboard is: 64 वग वाले एक होगी, यिद इसका ास 40 सेमी तथा rectangular box is 20cm. The box
शतरं ज की िबसात के ेक वग का ऊंचाई 7 मीटर है | was then remade in such a way
े फल 4 वग सेमी है | यिद इस SSC CPO 16 March 2019 that its length increased by 30%
िबसात की सभी भुजाओं पर 2 सेमी (Evening) but the breadth decreased by 20%
का िकनारा है , तो इस शतरं ज की (a)176 rs and the area increased by 100
िबसात का प रमाप ात कर | (b)184 rs

www.ssccglpinnacle.com support@ssccglpinnacle.com Ph. 09729327755, 09817390373 492


/
s
se
Days 54-60 Mensuration / े िमित

s
la
_c
ob
bo
ah
sm
cm2 . What is the new area of the Q54.A tall rectangular vessel is SSC CPO 16 March 2019
ur
yo

box? half filled with water. The base (Afternoon)


e/
t.m
://

एक आयताकार िड े की वा िवक dimension of the vessel is 62 cm (a)12,375


tp
ht

चौड़ाई 20 सेमी है | इस िड े का × 45 cm. A heavy metal cube of (b)13,280


पुनिनमाण इस कार िकया गया िक edge 15 cm is dropped into the (c)11,695
इसकी लंबाई 30% बढ़ा दी गयी vessel. The rise in level of the (d)12,040
लेिकन चौड़ाई म 20% की कमी कर vessel is:
दी गयी तथा े फल 100 वग सेमी से एक लंबा आयताकार बतन पानी से Q57. A field is 119m × 18 m in
बढ़ गया | इस िड े का नया े फल आधा भरा आ है । इस पा की मूल dimension. A tank 17m × 6m ×
ा है ? लंबाई-चौड़ाई 62 सेमी x 45 सेमी है । 3m is dug out in the middle and
SSC CPO 16 March 15 सेमी िकनारा वाले एक भारी the soil removed is evenly spread
2019(Evening) धा क घन को को बतन म िगरा over the remaining part of the
(a)2500 cm2 िदया जाता है । बतन के जल र म field. The increase in the level on
(b)2200 cm2 + वृ ात कर | the remaining part of the field is:

e
(c)2600 cm2 + SSC CPO 15 March 2019 एक मैदान की लंबाई-चौड़ाई 119मी
(d)2400 cm2 (Morning) × 18मी है | बीच म 17मी x 6 मी x 3
(a)1.21 cm मी आकार वाला एक तालाब खोदा
Q52. 12 buckets of water fill a (b)1.15 cm जाता है तथा इससे िनकली िम ी को

l
tank when the capacity of each (c) 1.07 cm मैदान के शेष भाग म फैला िदया
bucket is 13.5 litres. How many
ac (d)1 cm जाता है | मैदान के शेष भाग के र
buckets will be needed to fill the म ई वृ ात कर | SSC CPO 16
same tank, if the capacity of each Q55.The dimensions of a March 2019 (Afternoon)
bucket in 9 litres? िकसी टं की को swimming pool are 66m (a)14cm
भरने म 12 बा ी पानी लगता है जब × 35m × 3m . How many hours (b)13cm
ेक बा ी की धा रता 13.5 लीटर will it take to fill the pool by a (c)15cm
है | यिद ेक बा ी की धा रता 9 pipe of diameter 35 cm with (d)12cm
लीटर हो, तो इसी टं की को भरने के water flowing at speed 8m/s ?
एक तरणताल के आयाम 66मी × 35
nn
िलए िकतनी बा यों की Q58. A swimming pool is 40 m in
आव कता होगी ? मी × 3 मी ह | 35 सेमी ास वाले length, 30 m in breadth and 2.2 m
SSC CPO 16 March 2019 पाइप से इस ताल को भरने म िकतना in depth. The cost of cementing
(Evening) समय लगेगा यिद जल वाह की दर its floor and the four sides at Rs.
(a)15 8 मी/सेकंड है | 25/ m 2 is:
(b)16 SSC CPO 15 March 2019 एक तरणताल की लंबाई 40 मी,
(c)18 (Morning) चौड़ाई 30 मी तथा गहराई 2.2 मीटर
(d)17 (a) 2.75 है | 25 पये ित वग मीटर की दर से
इसके तल तथा इसकी चारों भुजाओं
Pi

(b) 3.5
Q53. If the height of an (c) 2.5 पर सीमट चढ़ाने की लागत ात कर |
equilateral triangle is 10√3 cm, (d) 3.2 SSC CPO 16 March 2019
the area is: (Afternoon)
यिद िकसी समबा ि भुज की ऊंचाई Q56. Find the cost of carpeting a (a)Rs.43,980
room which is 11m long and 6m (b)Rs.37540
10√3 सेमी है , तो े फल ात कर |
broad by a carpet which is 60cm (c) Rs.34260
SSC CPO 15 March 2019
broad at the rate of rs 112.50 per (d)Rs.37700
(Morning)
meter.
(a) 124 √3 cm2 11 मीटर लंबाई तथा 6 मीटर चौड़ाई Q59. If the area of a regular
(b) 75 √3 cm2 वाले िकसी कमरे म 112.50 पये hexagon is 108 √3 cm2 , its
(c) 80√3 cm2 ित मीटर की दर से कालीन िबछाने perimeter is:
की लागत ात कर यिद कालीन 60
(d) 100√3 cm2
सेमी चौड़ी है |

www.ssccglpinnacle.com support@ssccglpinnacle.com Ph. 09729327755, 09817390373 493


/
s
se
Days 54-60 Mensuration / े िमित

s
la
_c
ob
bo
ah
यिद िकसी सम षट् भुज (regular 16 m ल े, 8 m चौड़े और 11 m
sm
volume in cubic cm? (Take 𝜋 =
ur
yo

hexagon) का े फल 108 √3 है तो ऊंचे कमरे म िकतना सबसे अिधक 22/7)


e/
t.m
://

इसका प रमाप (perimeter) है : ल ा ख ा रखा जा सकता है ? एक शंकु के िछ क के दो वृ ाकर


tp
ht

SSC CPO 15 March 2019 SSC CPO 15 March 2019 फलकों की ि ा 5 सेमी और 4 सेमी
(Evening) (Evening) है । यिद िछ क की ऊंचाई 21 सेमी
(a)36 √2 cm (a)21 m है , तो घन सेमी म इसका आयतन
(b)20 m िकतना होगा ?
(b)42 √3 cm
(c)18 m SSC CGL 11 June 2019
(c)28 √3 cm (d)19 m (Afternoon)
(d)24 cm (a) 902
Practice Questions (b) 1056
Q60. A square piece of cardboard (c) 1342
with side 12 cm has a small square Q1. A cylindrical road roller (d) 638
of 2 cm cut out from each of the made of metal is 1 m long. Its
corners. The resulting flaps are

e
inner radius is 27 cm and the Q4. The radii of two circular
turned up to make a box 2 cm thickness of the metal sheet rolled faces of the frustum of a cone of
deep. The volume of the box is: into it is 9 cm. What is the weight height 14 cm are 5 cm and 2 cm.
12 cm सतह वाले एक वगाकार ग े of the roller if 1 cubic cm of What is its volume in cubic cm?
के टु कड़े के ेक कोने से 2 cm

l
metal weighs 8 g? (Take 𝜋= 22/7)
छोटे वग काटे जाते ह | 2 cm गहरा धातु से बना एक बेलनाकार रोड 14 सेमी ऊंचाई के शंकु के िछ क के
ac
िड ा तैयार करने के िलए प रणामों
पि याँ (resulting flaps) को मोड़ा
जाता है | िड े के घनफल का मान
ा है :
SSC CPO 15 March 2019
रोलर 1 मीटर लंबा है । इसकी
आं त रक ि ा 27 सेमी है और धातु
की शीट की मोटाई 9 सेमी है । यिद 1
घन सेमी धातु का वजन 8 ाम हो तो
रोलर का वजन िकतना होगा?
दो वृताकार फलकों की ि

आयतन िकतना होगा ?


SSC CGL 11 June 2019
(Evening)
ा 5 सेमी
और 2 सेमी है । घन सेमी म इसका

(Evening) SSC CGL 7 June 2019 (a) 572


(a) 128 cm3 (Evening) (b) 560
(b) 94 cm3 (a) 441 𝜋 kg (c) 520
nn
(c) 102 cm3 (b) 442.4 𝜋 kg (d) 540
(d) 112 cm3 (c) 449 𝜋 kg
(d) 453.6 𝜋 kg Q5. The radii of the two circular
Q61. The area of a right angled face of the frustum of a cone of
triangle having base 24 cm and Q2. The radius of a sphere is height 21 cm are 5 cm and 3 cm.
hypotenuse 25 cm is: increased by 140%. By what What is its volume in cm3 ? (
उस समकोण ि भुज का े फल ा percent is the volume increased? Π = 22
7 )
होगा िजसका आधार 24 cm और एक गोले की ि ा 140% बढ़ा दी 21 सेमी ऊंचाई के शंकु के िछ क के
Pi

कण (hypotenuse) 25 cm है : जाती है । आयतन िकतने ितशत दो वृताकार फलकों की ि ा 5 सेमी


SSC CPO 15 March 2019 बढ़े गा ? और 3 सेमी है । घन सटीमीटर म
(Evening) SSC CGL 10 June 2019 इसका आयतन िकतना होगा ?
(a)72 cm2 (Evening) SSC CGL 12 June 2019
(b)108 cm2 (a) 174.4% (Morning)
(c)92 cm2 (b) 1382.4% (a) 1078
(d)84 cm2 (c) 274.4% (b) 1020
(d) 1282.4% (c) 1058
Q62. The length of the longest (d) 1025
pole that can be placed in a room Q3. The radii of two circular
16 m long, 8 m wide and 11 m faces of frustum of a cone are 5 Q6. The radii of two circular
high is: cm and 4 cm. If the height of the faces of the frustum of a cone of
frustum is 21 cm, what is its height 10.5 cm are 5 cm and 3 cm

www.ssccglpinnacle.com support@ssccglpinnacle.com Ph. 09729327755, 09817390373 494


/
s
se
Days 54-60 Mensuration / े िमित

s
la
_c
ob
bo
ah
उस समचतुभुज का े फल ( वग
sm
respectively. What is its volume AC exceeds the length of AB by
ur
yo

in cm3 ( π = 22
7 )? सेमी म ) ात कर िजसकी भुजा 13 1 cm. The perimeter of the
e/
t.m
://

10.5 सेमी ऊंचाई वाले शंकु के सेमी है तथा सबसे छोटा िवकण 10 triangle is 34 cm. The length (in
tp
ht

िछ क के दो वृताकार फलकों की सेमी का है | cm) of the smaller side of the


ि ा मशः 5 सेमी और 3 सेमी है । SSC CHSL 04 July 2019 triangle is :
घन सटीमीटर म इसका आयतन (Evening) ि भुज ABC म, BC की लंबाई AB
िकतना होगा ? (a) 192 की लंबाई के दोगुने से 3 सेमी कम है
SSC CGL 12 June 2019 (b) 96 | AC की लंबाई AB की लंबाई से 1
(Afternoon) (c) 120 सेमी अिधक है | ि भुज का प रमाप
(a) 552 (d) 50 34 सेमी है | ि भुज की छोटी भुजा की
(b) 539 लंबाई (सेमी म ) ात कर |
(c) 545 Q10. What is the area of a SSC CHSL 08 July
(d) 564 rhombus (in cm2 ) whose side is 2019(Morning)
20 cm and one of the diagonal is (a) 10

e
Q7. Five cubes, each of edge 3 24 cm? (b) 8
cm are joined end to end. What is उस समचतुभुज का े फल (वग (c) 7
the total surface area of the सेमी म ) ात कर िजसकी भुजा 20 (d) 9
resulting cuboid, in cm 2 ? सेमी है और उसका एक िवकण 24

l
पां च घन, िजनमे ेक का फलक 3 सेमी है | Q13. If each side of a rectangle is
सेमी है , फलकों के अनु प जोड़े
ac SSC CHSL 05 July 2019 increased by 13%, then its area
जाते है , वग सटीमीटर म प रणामी (Morning) will increase by :
घनाभ का कुल े फल िकतना है ? (a) 384 यिद िकसी आयत की ेक भुजा
SSC CGL 13 June 2019 (b) 350 को 13% से बढ़ा िदया जाए, तो
(Afternoon) (c) 396 इसका े फल िकतना बढ़ जाएगा ?
(a)244 (d) 392 SSC CHSL 08 July 2019
(b)280 (Afternoon)
(c)270 Q11. In triangle ABC, the length (a) 21.69%
BC is less than twice the length of (b) 13%
nn
(d)198
AB by 3 cm. The length of AC (c) 27.69%
Q8. A sphere of radius 6 cm is exceeds the length of AB by 9 (d) 26%
melted and recast into spheres of cm. The perimeter of the triangle
radius 2 cm each. How many is 34 cm. The length (in cm) of Q14. If each side of a rectangle is
such spheres can be made? the smallest side of the triangle is decreased by 13%, then its area
6 सेमी ि ा के एक गोले को ि भुज ABC म, BC की लंबाई AB will decrease by :
िपघलाया जाता है और ेक 2 सेमी की लंबाई के दोगुने से 3 सेमी कम है यिद िकसी आयत की ेक भुजा
ि ा के गोले म िफर से ढाला जाता | AC की लंबाई AB की लंबाई से 9 को 13% कम कर िदया जाए, तो
Pi

है । ऐसे िकतने गोले बनाए जा सकते सेमी अिधक है | ि भुज का प रमाप इसका े फल िकतना कम हो
ह? 34 सेमी है | ि भुज की सबसे छोटी जाएगा ?
SSC CGL 13 June 2019 भुजा की लंबाई है - SSC CHSL 08 July 2019
(Evening) SSC CHSL 05 July 2019 (Evening)
(a)27 (Evening) (a) 26%
(b)25 (a) 10 (b) 21.69%
(c)36 (b) 9 (c) 13%
(d)24 (c) 7 (d) 24.31%
(d) 8
Q9. What is the area of a Q15. Twelve sticks, each of
rhombus (in cm2 ) whose side is Q12. In triangle ABC, the length length one unit, are used to form
13 cm and the smallest diagonal of BC is less than twice the length an right-angled triangle. The area
is 10 cm? of AB by 3 cm. The length of the of the triangle is :

www.ssccglpinnacle.com support@ssccglpinnacle.com Ph. 09729327755, 09817390373 495


/
s
se
Days 54-60 Mensuration / े िमित

s
la
_c
ob
bo
ah
बारह छिड़याँ , िजनम से ेक की वाले गोले बनाए जाते ह | ऐसे िकतने
sm
SSC CPO 13 March
ur
yo

लंबाई एक इकाई है , का इ ेमाल गोले बनाए जा सकते ह ? 2019(Evening)


e/
t.m
://

एक समकोण ि भुज बनाने के िलए SSC CPO 12 March 2019 (a)576 cm2
tp
ht

िकया जाता है | इस ि भुज का (Evening) (b)768 cm2


े फल ात कर | (a) 17 (c)640 cm2
SSC CHSL 10 July 2019 (b) 18 (d)544 cm2
(Afternoon) (c) 16 Q22. One side of a rhombus is 6.5
(a) 4 sq units (d) 15 cm and one of it’s diagonal is 12
(b) 6 sq units cm. What is the area of the
(c) 8 sq units Q19. One side of a rhombus is 13 rhombus?
(d) 10 sq units cm and one of its diagonal is 24 िकसी समचतुभुज की एक भुजा 6.5
cm. What is the area of rhombus? सेमी की है तथा इसका एक िवकण
Q16. Four cubes, each of edge 5 िकसी समचतुभुज की एक भुजा 13 12 सेमी का है | इस समचतुभुज का
cm are joined end to end. What is सेमी तथा इसका एक िवकण 24 सेमी े फल ात कर |

e
the total surface area of the का है | इस समचतुभुज का े फल SSC CPO 13 March 2019
resulting cuboid ? ा है ? (Evening)
चार घन िजनम से ेक का िकनारा SSC CPO 12 March 2019 (a) 78cm2
5 सेमी है , उ शु से अंत तक (Evening) (b) 15cm2

l
जोड़ा जाता है | इस कार बनने वाले (a) 312 cm2
(c) 30cm2
घनाभ का कुल पृ
ac े फल ात कर (b) 156 cm2
(d) 60cm2
| (c) 120 cm2
SSC CHSL 11 July (d) 130 cm2
Q23. A sphere of radius 9 cm is
2019(Evening)
melted and recast into small
(a) 475 Q20. 6 cubes, each of edge 4 cm,
spheres of radius 2 cm each. How
(b) 450 are joined end to end. What is the
many such sphere can be made?
(c) 600 total surface area of the resulting
9 सेमी ि ा वाले एक गोले को
(d) 500 cuboid?
िपघलाया जाता है और 2 सेमी ि ा
6 घन, िजनम से ेक का िकनारा 4
nn
वाले गोले बनाए जाते ह | ऐसे िकतने
Q17. Find the weight of solid सेमी है , शु से अंत तक जुड़े ए ह |
गोले का िनमाण िकया जा सकता है ?
cylinder of height 35 cm and इस कार बनने वाले घनाभ का कुल
SSC CPO 13 March 2019
radius 14 cm. If the material of पृ े फल ात कर |
(Evening)
the cylinder weighs 8 gm / cm3 . SSC CPO 12 March
(a) 90
ऊंचाई 35 सेमी और ि ा 14 cm 2019(Evening)
(b) 92
के ठोस िसलडर का भार ात कर। (a) 416 cm2
(c) 93
यिद िसलडर की साम ी का वजन 8 (b) 496 cm2 (d) 91
gm / cm3 है ।
Pi

(c) 576 cm2


SSC CPO 16 March 2019 (d) 208 cm2 Q24. The radius of a cylinder is
(Morning) increased by 60% and height is
(a) 160 Kg Q21. A cuboid of edges 32cm, decreased by 20%, what is
(b) 172.48 Kg 4cm and 4cm is cut to form cubes percentage increase in it’s
(c) 166 Kg of edge 4cm each. What is the volume?
(d) 177.44 Kg sum of total surface areas of all एक बेलन की ि ा 60% बढ़ा दी
cubes formed? जाती है और ऊंचाई 20% कम कर
Q18. A sphere of radius 5 cm is 32 सेमी, 4 सेमी तथा 4 सेमी िकनारों दी जाती है | इसके आयतन म िकतने
melted and recast into spheres of वाले एक घनाभ को 4 सेमी िकनारा ितशत की वृ होगी ?
radius 2 cm each. How many वाले घनों का िनमाण करने के िलए SSC CPO 13 March 2019
such spheres can be made? काटा जाता है | इस कार बने सभी (Evening)
5 सेमी ि ा वाले एक गोले को घनों के कुल पृ े फल का योग (a) 105.2%
िपघलाया जाता है और 2 सेमी ि ा ा होगा ? (b) 104.8%

www.ssccglpinnacle.com support@ssccglpinnacle.com Ph. 09729327755, 09817390373 496


/
s
se
Days 54-60 Mensuration / े िमित

s
la
_c
ob
bo
ah
sm
(c) 105.8% (a) 36 (a) 172 cm2
ur
yo

(d) 105.6% (b) 25 (b) 163 cm2


e/
t.m
://

(c) 27 (c) 196 cm2


tp
ht

Q25. 5 cubes, each of edge 4cm, (d) 24 (d) 86 cm2


are joined end to end. What is the
total surface area of resulting Q28. The side of a rhombus is 5 Q31. The radius of a cylinder is
cuboid? cm and one of its diagonal is 8 increased by 150% and its height
5 घन, िजनम से ेक का िकनारा 4 cm. What is the area of the is increased by 50%. What is the
सेमी का है , को एक के बाद एक rhombus? percentage increase in its
जोड़ा जाता है | इस कार बनने वाले एक समचतुभुज की भुजा 5 सेमी तथा volume?
घनाभ का कुल पृ े फल ा इसका एक िवकण 8 सेमी का है | इस एक बेलन की ि ा 150% बढ़ा दी
होगा ? समचतुभुज का े फल ात कर | जाती है और इसकी ऊंचाई म 50%
SSC CPO 12 March 2019 SSC CPO 12 March 2019 की वृ कर दी जाती है | इसके
(Morning) (Morning) आयतन म िकतने ितशत की वृ

e
(a) 352 cm2 (a) 30 cm2 होगी ?
(b) 486 cm2 (b) 20 cm2 SSC CPO 13 March 2019
(c) 720 cm2 (c) 40 cm2 (Morning)
(d) 526 cm2 (d) 24 cm2 (a) 375%

l
(b) 625.5%
Q26. The sides of a triangle are
ac Q29. One side of a rhombus is 26 (c) 775.75%
10cm, 24cm and 26 cm. At each cm and one of the diagonal is 48 (d) 837.5%
of its vertices, circles of radius cm. What is the area of the
3.5 cm are drawn. What is the rhombus? Q32. 8 cubes, each of edge 5 cm,
area of the triangle excluding the िकसी समचतुभुज की एक भुजा 26 are joined end to end. What is the
portion covered by the sectors of सेमी की तथा इसका एक िवकण 48 total surface area of the resulting
the circle? सेमी का है | इस समचतुभुज का cuboid?
एक ि भुज की भुजाएं 10 सेमी, 24 े फल ात कर | SSC CPO 13 8 घनों को एक-एक करके जोड़ िदया
सेमी तथा 26 सेमी की ह | इसके जाता है िजनम से ेक का िकनारा
nn
March 2019 (Morning)
ेक शीष पर 3.5 सेमी ि ा वाले (a) 624 cm2 5 सेमी का है | इस कार बनने वाले
वृ खींचे जाते ह | वृ खंड ारा घेरे (b) 580 cm2 घनाभ का कुल पृ े फल ात कर
गए भाग को छोड़ कर इस ि भुज का (c) 520 cm2 |
े फल ात कर | (d) 480 cm2 SSC CPO 13 March 2019
SSC CPO 12 March 2019 (Morning)
(Morning) Q30. The sides of a triangle are (a) 850 cm2
(a) 81.5 cm2 16 cm, 30 cm and 34 cm (b) 825 cm2
Pi

(b) 100.75 cm2 respectively. At each vertices, (c) 1200 cm2


(c) 75.75 cm2 circles of radius 7 cm are drawn. (d) 800 cm2
(d) 78.25 cm2 What is the area of the triangle,
excluding the portion covered by Q33. A sphere of radius 7 cm is
Q27. A sphere of radius 6 cm is the sectors of the triangle? ( melted and recast into small
melted and recast into sphere of π = 22
7 ) spheres of radius 2 cm each. How
radius 2 cm each. How many एक ि भुज की भुजाएं मशः 16 many such spheres can be made?
such spheres can be made? सेमी, 30 सेमी तथा 34 सेमी की ह | 7 सेमी ि ा वाले एक गोले को
6 सेमी ि ा वाले एक गोले को ेक शीष पर 7 सेमी ि ा वाले िपघलाकर 2 सेमी ि ा वाले
िपघलाकर 2 सेमी ि ा वाले गोले वृ खींचे जाते ह | वृ के खंडो ारा छोटे -छोटे गोले बनाए जाते ह | ऐसे
बनाए जाते ह | ऐसे िकतने गोले का घेरे गए भाग को छोड़ कर ि भुज का िकतने गोले बनाए जा सकते ह ?
िनमाण िकया जा सकता है ? े फल ात कर | SSC CPO 13 March 2019
SSC CPO 12 March 2019 SSC CPO 13 March 2019 (Morning)
(Morning) (Morning) (a) 40

www.ssccglpinnacle.com support@ssccglpinnacle.com Ph. 09729327755, 09817390373 497


/
s
se
Days 54-60 Mensuration / े िमित

s
la
_c
ob
bo
ah
1 मी लंबी भुजा वाले िकसी घन को
sm
(b) 42 SSC CPO 15 March 2019
ur
yo

(c) 41 10 सेमी भुजा वाले ेक छोटे -छोटे (Morning)


e/
t.m
://

(d) 43 घनों म काटा जाता है | ऐसे िकतने (a)720 cm2


tp
ht

छोटे घन ा िकये जा सकते ह ? (b)2880 cm2


Q34. The volume of a conical tent SSC CPO 16 March (c)600 cm2
is 924 m3 and its area is 154 m2 . 2019(Evening) (d)1440 cm2
The height of the tent is: (a)1000
एक शं ाकार तंबू का आयतन 924 (b)10000 Q41.The sides of a triangle are in
घन मीटर है तथा इसका े फल 154 (c)10 the ratio 3 : 2 : 4 and the
घन मीटर है | इस तंबू की ऊंचाई ात (d)100 perimeter is 72 cm, The sides are:
कर | SSC CPO 14 March 2019 एक ि भुज की भुजाएं 3 : 2 : 4 के
(Morning) Q38. The sides of a triangle are in अनुपात म ह तथा इसका प रमाप 72
(a)24 m the ratio 3 : 4 : 5. If the perimeter सेमी है | भुजाएं ात कर |
(b)6 m of the triangle is 24 cm, its area SSC CPO 16 March 2019

e
(c)18 m is: (Afternoon)
(d)12 m एक ि भुज की भुजाएं 3 : 4 : 5 के (a)24, 16, 32
अनुपात म ह | यिद इस ि भुज का (b)48, 24, 12
Q35. Three cubes with edges 6 प रमाप 24 सेमी है , तो इसका (c) 36, 24, 12

l
cm each are joined end to end to े फल ात कर | (d)36, 18, 9
form a cuboid. The total surface
ac SSC CPO 15 March 2019
area of the cuboid is: (Morning) Q42. A river is 3 m deep and 36
6 सेमी भुजा वाले तीन घनों को (a) 18cm2 m wide flows at the rate of 5km/h
एक-एक करके जोड़ा जाता है और (b) 24cm2 in to sea. The volume of water
घनाभ बनाया जाता है | इस घनाभ का (c) 20cm2 that runs into the sea per minute
कुल पृ े फल ात कर | (d) 22.89cm2 is:
SSC CPO 14 March 2019 3 मीटर गहरी तथा 36 मीटर चौड़ी
(Morning) Q39. How many soap cakes of नदी समु म 5 िकमी/घंटा की चाल
(a)432 cm2 से बहती है | ित मीटर समु म जाने
nn
size 8 cm × 4.5 cm × 2 cm can
(b)504 cm2 be kept in a carton of size 11 m × वाले पानी का प रमाण ात कर |
(c)648 cm2 0.82 m × 0.63 m? SSC CPO 16 March 2019
(d)720 cm2 8 सेमी × 4.5 सेमी × 2 सेमी आकार (Afternoon)
वाली साबुन की िकतनी िटिकया को (a)8300 m 3
Q36. The volume of a solid 11 मी x 0.82 मी x 0.63 मी आकार (b)9000 m 3
cylinder with height 6 cm is 231 वाले एक िड े म रखा जा सकता है (c)7600 m 3
cm3 . The radius of cylinder is : ? (d)6400 m 3
Pi

6 सेमी ऊंचाई वाले एक ठोस बेलन SSC CPO 15 March 2019


का आयतन 231 घन सेमी है | इस (Morning) Q43. A rectangular solid is 20 cm
बेलन की ि ा ात कर | (a) 81052 long and 12 cm wide. If its
SSC CPO 14 March 2019 (b) 75626 volume is 2160 cm3 , the height
(Morning) (c) 73498 is:
(a)21 cm (d) 78925 एक आयताकार ठोस 20 सेमी लंबा
(b)2.1 cm तथा 12 सेमी चौड़ा है | यिद इसका
(c)35 cm Q40. If a cuboid has l=24cm, आयतन 2160 घन सेमी है , तो इसकी
(d)3.5 cm b=16cm, h=7.5 cm, its lateral ऊंचाई ात कर |
surface area is: SSC CPO 16 March 2019
Q37. A cube of side 1 m length is यिद िकसी घनाभ म l = 24 सेमी, b = (Afternoon)
cut into small cubes of side 10 cm 16 सेमी, h = 7.5 सेमी है , तो इसका (a)11 cm
each. How many such small पा पृ े फल ात कर | (b)10 cm
cubes can be obtained? (c) 12 cm

www.ssccglpinnacle.com support@ssccglpinnacle.com Ph. 09729327755, 09817390373 498


/
s
se
Days 54-60 Mensuration / े िमित

s
la
_c
ob
bo
ah
ास वाला 18 m गहरा एक
sm
(d) 9 cm Q47. Which of the following has 7 m
ur
yo

the maximum number of vertex? कुआं खोदा गया है | खुदाई से


e/
t.m
://

Q44. A diagonal of a िन िल खत म से िकस ठोस म सबसे िनकली िम ी को 18 m × 14 m का


tp
ht

quadrilateral is 40 cm. The length अिधक शीष होते है समतल ेटफॉम तैयार करने के
of the perpendicular to the SSC CPO 14 March िलए समान प से फैलाया जाता है |
opposite ends is 7.5 cm and 8.6 2019(Evening) ेटफॉम की ऊंचाई िकतनी होगी ?
cm. What is the area of (a) Cuboid SSC CPO 15 March 2019
quadrilateral ? (b) Triangular Prism (Evening)
िकसी चतुभुज का एक िवकण 40 cm (c) Hexagonal pyramid (a)2.6 m
है | िवपरीत िसरों से ल की ल ाई (d) Tetrahedron (b)3.2 m
7.5 cm और 8 .6 cm है | चतुभुज का (c)2.75m
े फल ा है ? Q48. How much volume will the (d)3.05 m
SSC CPO 14 March 2019 wood need to make a closed box
(Evening) of 2.5cm thickness with outer SSC CGL TIER II 2018

e
(a) 434 cm2 measurements 90 cm × 75 cm × 50
(b) 322 cm2 cm ? Q1. A right circular cylinder of
(c) 368 cm2 बाहरी माप 90cm × 75cm × 50cm maximum volume is cut out from
(d) 288 cm2 के साथ 2.5cm मोटाई के एक बंद a solid wooden cube. The

l
बॉ बनाने के िलए िकतने आयतन material left is what percent of
Q45. The floor of the square
ac की लकड़ी की आव कता होगी ? the volume (nearest to an integer)
room has a perimeter of 230 m SSC CPO 14 March 2019 of the original cube ?
and the height of the room is 5m. (Evening) एक ठोस लकड़ी के घन से
At the rate of 7.50 / m2 , what (a) 46720 cm3 अिधकतम आयतन वाला एक ल
amount will be spent on dyeing (b) 69750 cm3 वृ ीय बेलन काटा जाता है | बची ई
the walls of the room? (c) 49050 cm3 साम ी आरं िभक घन के आयतन (
वगाकार कमरे के फश की प रिमित (d) 36170 cm3 एक पूणाक के िनकटतम ) का
230 m और कमरे की ऊंचाई 5 m है िकतना ितशत है ?
SSC CGL TIER II (11
nn
| 7.50 / m2 की दर से कमरे की Q49. To make two triangles, a
दीवारों पर रं गाई करवाने पर िकतनी rectangular paper is cut slant. If September 2019)
रािश खच होगी ? (a) 19
the diagonal is 4√5 and the
SSC CPO 14 March 2019 (b) 28
length is twice the width, then the
(Evening) (c) 23
area of the rectangle is:
(a) Rs 3,450 (d) 21
दो ि भुज बनाने के िलए एक
(b) Rs 17,250 आयताकार कागज को ितरछा काटा
(c) Rs 8,625 Q2. The ratio of the volumes of
जाता ह | यिद िवकण 4√5 ह और
two cylinders is x:y and the ratio
Pi

(d) Rs 4,312.50
चौड़ाई तुलना म ल ाई दोगुनी है , तो of their diameters is a:b. What is
आयत का े फल है : the ratio of their heights?
Q46. If the total surface area of a
SSC CPO 15 March 2019 दो बेलनों के आयतन का अनुपात x :
cube is 1944 m2 , then find the
(Evening) y है और उनके ास का अनुपात a:
volume of the cube : (a)54 cm2
यिद एक घन का पृ े फल 1944 b है | उनकी ऊंचाई का अनुपात ात
(b)72 cm2 कर |
m है , तो इसका आयतन है :
2
(c)32 cm2 SSC CGL TIER II (11
SSC CPO 14 March
(d)80 cm2 September 2019)
2019(Evening)
(a) 1648 m3 (a) xb:ya
Q50. An 18 m deep well with (b) xa:yb
(b) 4912 m3
diameter 7 m is dug and the earth (c) x b2 :y a2
(c) 2744 m3
from digging is spread evenly to (d) x a2 :y b2
(d) 5832 m3 form a platform 18 m × 14 m
.The height of the platform is:

www.ssccglpinnacle.com support@ssccglpinnacle.com Ph. 09729327755, 09817390373 499


/
s
se
Days 54-60 Mensuration / े िमित

s
la
_c
ob
bo
ah
एक बेलन का पा पृ े फल 352
sm
Q3. If the radius of a right (b) 19404
ur
yo

circular cylinder is decreased by (c) 12936 वग सेमी है | यिद इसकी ऊंचाई 7


e/
t.m
://

20% while its height is increased (d) 38808 सेमी है , तो इसका आयतन ( घन सेमी
tp
ht

by 40%, then the percentage म ) ात कर | ( π = 22


7 )
change in its volume will be : Q6. If a cuboid of dimensions SSC CGL TIER II (11
यिद िकसी ल वृ ीय बेलन की 32cm × 12cm × 9cm is cut into September 2019)
ि ा 20% कम कर दी जाए और two cubes of same size, what will (a) 1408
इसकी ऊंचाई 40% बढ़ा दी जाए, तो be the ratio of the surface area of (b) 1078
इसके आयतन म आने वाला ितशत the cuboid to the total surface (c) 1243
प रवतन होगा : area of the two cubes ? (d) 891
SSC CGL TIER II (11 यिद 32 सेमी x 12 सेमी x 9 सेमी
September 2019) िवमाओं वाले एक घनाभ को बराबर Q9. The internal and external
(a) 1.04% increase आकार के दो घनों म काटा जाए, तो radii of a hollow hemispherical
(b) 10.4% decrease घनाभ के पृ े फल और दोनों घनों vessel are 6 cm and 7 cm

e
(c) No increase के कुल पृ े फल म ा अनुपात respectively. What is the total
(d) 10.4% increase होगा ? surface are (in cm2 ) of the vessel
SSC CGL TIER II (11 ?
Q4. If the radius of the base of a September 2019) एक खोखले अधगोलीय बतन की

l
cone is doubled, and the volume (a) 65:72 आतं रक और बा ि ा मश 6
of the new cone is three times the
ac (b) 37:48 सेमी और 7 सेमी है | इस बतन का
volume of the original cone, then (c) 24:35 कुल पृ े फल ( वग सेमी म ) ात
what will be the ratio of the (d) 32:39 कर |
height of the original cone to that SSC CGL TIER II (11
of the new cone ? Q7. The base of right prism is a September 2019)
यिद िकसी शंकु के आधार की ि ा trapezium whose parallel sides (a) 183 π
दोगुनी कर दी गयी है , और नए शंकु are 11cm and 15cm and the (b) 189 π
का आयतन आरं िभक शंकु के distance between them is 9 cm. If (c) 177 π
आयतन का ितगुना है , तो आरं िभक the volume of the prism is 1731.6
nn
(d) 174 π
शंकु और नए शंकु की ऊँचाइयों के cm3 , then the height (in cm) of
बीच ा अनुपात होगा ? the prism will be : एक ल ि Q10. Three solid metallic spheres
SSC CGL TIER II (11 का आधार समलंब है िजसकी whose radii are 1cm, x cm and 8
September 2019) समानां तर भुजाएं 11 सेमी और 15 cm, are melted and recast into a
(a) 1:3 सेमी ह तथा उनके बीच की दू री 9 single solid sphere of diameter 18
(b) 4:3 सेमी है | यिद ि का आयतन cm. The surface area (in cm2 ) of
(c) 2:9 1731.6 घन सेमी है , तो ि की the sphere with radius x cm is :
(d) 9:4 ऊंचाई होगी :
Pi

तीन ठोस धा क गोले िजनकी


SSC CGL TIER II (11 ि ाएँ 1 सेमी, x सेमी और 8 सेमी
Q5. If the diameter of the base of September 2019) ह, उ िपघलाया जाता है और िफर
a cone is 42 cm and its curved (a) 15.6 18 सेमी ास वाला एक ठोस गोला
surface area is 2310 cm2 , then (b) 15.2 बनाया जाता है | उस गोले का पृ
what will be its volume (in cm3 ) (c) 14.8 े फल ( वग सेमी म ) ात कर
? (d) 14.2 िजसकी ि ा x सेमी थी |
यिद िकसी शंकु के आधार का ास SSC CGL TIER II (11
42 सेमी है और इसका व पृ September 2019)
े फल 2310 वग सेमी है , तो इसका Q8. The lateral surface area of a (a) 144 π
आयतन ( घन सेमी म ) ा होगा ? cylinder is 352 cm2 . If its height
(b) 72 π
SSC CGL TIER II (11 is 7 cm, then its volume (in cm3 )
(c) 64 π
September 2019) is : (Take π = 22
7 ) (d) 100 π
(a) 25872

www.ssccglpinnacle.com support@ssccglpinnacle.com Ph. 09729327755, 09817390373 500


/
s
se
Days 54-60 Mensuration / े िमित

s
la
_c
ob
bo
ah
sm
Q11. If the radius of a sphere is (d) 560 π (a) 6
ur
yo

increased by 4 cm, its surface (b) 5


e/
t.m
://

area is increased by 464 π cm2 . Q14. The base of a right prism is (c) 5 31
tp
ht

What is the volume (in cm3 ) of a triangle with sides 20 cm, 21 (d) 6 32
the original sphere ? cm and 29 cm. If its volume is
यिद िकसी गोले की ि ा 4 सेमी से 7560 cm3 , then its lateral surface Q17. The sides of a triangle are
बढ़ा दी जाए, तो इसका पृ े फल area (in cm2 ) is : एक ल ि 56 cm, 90 cm and 106 cm. The
464 π वग सेमी से बढ़ जाता है | का आधार एक ि भुज है िजसकी circumference of its circumcircle
आरं िभक गोले का आयतन ( घन भुजाएं 20 सेमी, 21 सेमी और 29 is : एक ि भुज की भुजाएं 56 सेमी,
सेमी म ) ा था ? सेमी की ह | यिद इसका आयतन 90 सेमी और 106 सेमी ह | इसके
SSC CGL TIER II (11 7560 घन सेमी है , तो इसका पा पृ प रवृ की प रिध ात कर |
September 2019) े फल ( वग सेमी म ) ात कर | SSC CGL TIER II (12
(a) 15625
6 π SSC CGL TIER II (12 September 2019)
(b) 35937 π September 2019) (a) 106 π
8

e
(c) 11979 π (a) 2484 (b) 109 π
2
15625 (b) 2556 (c) 108 π
(d) 8 π
(c) 2520 (d) 112 π
(d) 2448
Q12. The volume of a right

l
Q18. The base of a right pyramid
pyramid is 45 √3 cm3 and its base Q15. A cylindrical vessel of
ac
is an equilateral triangle with side
6 cm. What is the height (in cm)
of the pyramid ?
एक ल िपरािमड का आयतन 45
√3 घन सेमी है और इसका आधार
radius 3.5 m is full of water. If
15400 litres of water is taken out
from it, then the drop in the water
level in the vessel will be :
3.5 मी ि ा वाला एक बेलनाकार
is an equilateral triangle with side
8 cm, and the height of pyramid
is 24 √3 cm. The volume (in
cm3 ) of the pyramid is :
एक ल िपरािमड का आधार एक
समबा ि भुज है िजसकी भुजा 8
एक समबा ि भुज है िजसकी भुजा बतन पानी से भरा आ है | यिद इस
सेमी की है | िपरािमड की ऊंचाई 24
6 सेमी है | इस िपरािमड की ऊंचाई ( से 15400 लीटर पानी िनकाल िलया
सेमी म ) ात कर | जाए, तो इस बतन के जल र म √3 सेमी है | इस िपरािमड का
nn
SSC CGL TIER II (11 िकतनी िगरावट आएगी ? आयतन ( घन सेमी म ) ात कर |
September 2019) SSC CGL TIER II (12 SSC CGL TIER II (12
(a) 15 September 2019) September 2019)
(b) 18 (a) 72 cm (a) 1152
(c) 12 (b) 40 cm (b) 480
(d) 20 (c) 35 cm (c) 576
(d) 60 cm (d) 384
Pi

Q13. The area of the base of a


right circular cone is 400 π and Q16. A solid metallic sphere of Q19. If the diameter of the base
its height is 15 cm. The curved radius 8 cm is melted and drawn of a right circular cylinder is
surface area of the cone (in cm2 ) into a wire of uniform reduced by 33 31 % and its height
is : cross-section. If the length of the is doubled, then the volume of the
एक ल वृ ीय शंकु के आधार का wire is 24 m, then its radius (in cylinder will :
े फल 400 π है और इसकी ऊंचाई mm) is : यिद िकसी ल वृ ीय बेलन के
15 सेमी है | इस शंकु का व पृ 8 सेमी ि ा वाले एक ठोस धा क आधार का ास 33 31 % कम कर
े फल ( वग सेमी म ) है : गोले को िपघलाया जाता है और एक िदया जाए और इसकी ऊंचाई दोगुनी
SSC CGL TIER II (12 समान अनु थ काट वाला एक तार कर दी जाए, तो इस बेलन का
September 2019) बनाया जाता है | यिद तार की लंबाई आयतन :
(a) 480 π 24 मी है , तो इसकी ि ा होगी : SSC CGL TIER II (12
(b) 500 π SSC CGL TIER II (12 September 2019)
(c) 450 π September 2019)

www.ssccglpinnacle.com support@ssccglpinnacle.com Ph. 09729327755, 09817390373 501


/
s
se
Days 54-60 Mensuration / े िमित

s
la
_c
ob
bo
ah
(a) increase by 1 91 % / 1 91 % बढ़
sm
CGL TIER II (12 September
ur
yo

जाएगा Q22. A solid cube is cut into 2019)


e/
t.m
://

(b) remain unchanged/ अप रवितत three cuboids of same volumes. (a) 3960
tp
ht

रहे गा What is the ratio of the surface (b) 3600


(c) increase by 11 91 %/ 11 91 % area of the cube to the sum of the (c) 3300
बढ़ जाएगा surface areas of any two of the (d) 3560
(d) decrease by 11 91 % / 11 91 % cuboids so formed ?
एक ठोस घन को समान आयतन Q25. If the volume of a sphere is
कम हो जाएगा
वाले तीन घनाभों म काटा जाता है | 4851 cm3 , then its surface area
घन के पृ े फल तथा िकसी भी दो (in cm2 ) is : (Take π = 22
7 )
Q20. A right circular solid cone
घनाभ के पृ े फलों के जोड़ के यिद एक गोले का आयतन 4851 घन
of radius 3.2 cm and height 7.2
बीच अनुपात ात कर | सेमी है , तो इसका पृ े फल ( वग
cm is melted and recast into a
SSC CGL TIER II (12 सेमी म ) होगा : ( π = 7 )
22
right circular cylinder of height
September 2019) SSC CGL TIER II (12
9.6 cm. What is the diameter of

e
(a) 9:10 September 2019)
the base of the cylinder ?
(b) 27:16
3.2 सेमी ि ा और 7.2 सेमी ऊंचाई (a) 1386
(c) 27:10
वाले एक ल वृ ीय ठोस शंकु को (b) 2772
(d) 9:8
िपघलाया जाता है तथा 9.6 सेमी (c) 1323

l
ऊँचाई वाला एक ल वृ ीय बेलन (d) 1337
Q23. If the curved surface area of
बनाया जाता है | बेलन के आधार का
ac a solid cylinder is one-third of its
ास िकतना है ? Q26. N solid metallic spherical
total surface area, then what is the balls are melted and recast into a
SSC CGL TIER II (12
ratio of its diameter to its height ? cylindrical rod whose radius is 3
September 2019)
/ यिद िकसी ठोस बेलन का व पृ times that of a spherical ball and
(a) 4.2 cm
े फल इसके कुल पृ े फल का height is 4 times the radius of a
(b) 4.5 cm
एक-ितहाई है , तो इसके ास और spherical ball. The value of N is :
(c) 3.5 cm
इसकी ऊंचाई म अनुपात ात कर | N ठोस धा क गोलाकार गदों को
(d) 3.2 cm
SSC CGL TIER II (12
nn
िपघलाकर एक बेलनाकार छड़
September 2019) बनाया जाता है िजसकी ि ा एक
Q21. A hemispherical bowl of
(a) 5:2 गोलाकार गद की ि ा से ितगुनी है
internal diameter 36 cm is full of
(b) 1:1 और ऊंचाई एक गोलाकार गद की
a liquid. This liquid is to be filled
(c) 2:1 ि ा से चौगुनी है | N का मान है :
into cylindrical bottles each of
(d) 4:1 SSC CGL TIER II (13
radius 3 cm and height 12 cm.
How many such bottles are September 2019)
Q24. A field roller, in the shape
required to empty the bowl ?
of a cylinder, has a diameter of 1
Pi

36 सेमी आतं रक ास वाला (a) 30


m and length of 1 41 m. If the
अधगोलीय कटोरा िकसी तरल पदाथ (b) 27
से भरा आ है | इस तरल पदाथ को speed at which the roller rolls is (c) 24
बेलनाकार बोतलों म डाला जाता है 14 revolutions per minute, then (d) 36
िजनम से ेक की ि ा 3 सेमी the maximum are (in m2 ) that it
और ऊंचाई 12 सेमी है | कटोरे को can roll in 1 hour is : (Take π = Q27. The radius of the base of a
22 )
खाली करने के िलए ऐसी िकतनी 7 right circular cylinder is increased
बोतलों की आव कता है ? एक मैदानी रोलर, जो बेलन के by 20%. By what per cent should
SSC CGL TIER II (12 आकार का है , उसका ास 1 मी its height be reduced so that its
September 2019) और लंबाई 1 41 मी है | यिद इस रोलर volume remains the same as
(a) 72 के घूमने की चाल 14 च र ित before ?
(b) 54 िमनट है , तो 1 घंटे म यह अिधकतम एक ल वृ ीय बेलन के आधार की
(c) 36 िकतने े फल ( वग मीटर म ) पर ि ा 20% बढ़ा दी जाती है | इसकी
(d) 27 घूम सकता है ? ( π = 22 ऊंचाई को िकतने ितशत से कम
7 ) SSC

www.ssccglpinnacle.com support@ssccglpinnacle.com Ph. 09729327755, 09817390373 502


/
s
se
Days 54-60 Mensuration / े िमित

s
la
_c
ob
bo
ah
िकया जाना चािहए तािक इसका पा फलक का े फल 30 वग सेमी
sm
length of the platform ? (Take
ur
yo

आयतन पहले िजतना ही बना रहे ? है , तो इसकी ऊंचाई ात कर | π = 22


7 )
e/
t.m
://

SSC CGL TIER II (13 SSC CGL TIER II (13 2.8 मी ि ा वाला 15 मी गहरा एक
tp
ht

September 2019) September 2019) कुआँ खोदा जाता है तथा इससे


(a) 25
(b) 30 92
(a)
√ 739
12
िनकली िम ी को समान
फैलाकर 8 मी चौड़ाई तथा 1.5 मी
प से

(c) 30 95 (b)
√ 209
12 ऊंचाई वाला एक चबूतरा बनाया
जाता है | इस चबूतरे की लंबाई ा
(d) 28 (c)
√ 611
12
होगी ? ( π = 22
7 )
Q28. The radius and the height of
a right circular cone are in the
(d)
√ 643
12 SSC CGL TIER
September 2019)
II (13

ratio 5:12. Its curved surface area Q31. A sphere of maximum (a) 28.4 m
is 816.4 cm2 . What is the volume volume is cut out from a solid (b) 28.8 m
( in cm3 ) of the cone ? (Take π = hemisphere. What is the ratio of (c) 30.2 m

e
3.14) the volume of the sphere to that (d) 30.8 m
एक ल वृ ीय शंकु की ि ा तथा of the remaining solid ?
ऊंचाई 5 : 12 के अनुपात म है | िकसी ठोस अधगोले से अिधकतम Q34. A tank is in the form of a
इसका व पृ े फल 816.4 वग आयतन वाला एक गोला काटा जाता cuboid with length 12m. If 18

l
सेमी है | इस शंकु का आयतन ( घन है | गोले के आयतन एवं शेष बचे ठोस kilolitre of water is removed from
सेमी म ) ात कर | ( π = 3.14)
ac के आयतन के बीच अनुपात ात कर it, the water level goes down by
SSC CGL TIER II (13 | SSC CGL TIER II (13 30 cm. What is the width (in m )
September 2019) September 2019) of the tank ?
(a) 2512 (a) 1:4 एक टं की घनाभ के आकार की है
(b) 1256 (b) 1:2 िजसकी लंबाई 12 मी है | यिद इससे
(c) 3140 (c) 1:3 18 िकलोलीटर पानी िनकाल िदया
(d) 628 (d) 1:1 जाए, तो जल र 30 सेमी नीचे चला
जाता है | टं की की चौड़ाई ( मीटर म )
nn
Q29. The sides of a triangle are Q32. A right prism has height िकतनी है ?
12 cm, 35 cm and 37 cm. What is 18cm and its base is a triangle SSC CGL TIER II (13
the circumradius of the triangle ? with sides 5cm, 8cm and 12cm. September 2019)
एक ि भुज की भुजाएं 12 सेमी, 35 What is the lateral surface area (a) 4
सेमी और 37 सेमी की ह | इस ि भुज (in cm2 ) ? (b) 5
की प रि ा ात कर | एक ल ि की ऊंचाई 18 सेमी (c) 5.5
SSC CGL TIER II (13 है तथा इसका आधार एक ि भुज है (d) 4.5
September 2019) िजसकी भुजाएं 5 सेमी, 8 सेमी और
Pi

(a) 19 cm 12 सेमी की ह | पा पृ े फल ( Q35. The radius of the base of a


(b) 17.5 cm वग सेमी म ) ात कर | right circular cylinder is 3 cm and
(c) 17 cm SSC CGL TIER II (13 its curved surface area is 60 π
(d) 18.5 cm September 2019) cm2 . The volume of the cylinder
(a) 450 (in cm3 ) is : एक ल वृ ीय बेलन
Q30. The base of a right pyramid (b) 468 के आधार की ि ा 3 सेमी है तथा
is an equilateral triangle with area (c) 432 इसका व पृ े फल 60 π वग
16 √3 cm2 . If the area of one of (d) 486 सेमी है | इस बेलन का आयतन होगा
its lateral faces is 30 cm2 , then :
its height (in cm) is: Q33. A 15m deep well with SSC CGL TIER II (13
एक ल िपरािमड का आधार एक radius 2.8 m is dug and the earth September 2019)
समबा ि भुज है िजसका े फल taken out from it is spread evenly (a) 90 π
to form a platform of breadth 8m (b) 72 π
16 √3 वग सेमी है | यिद इसके एक
and height 1.5m. What will be the (c) 60 π

www.ssccglpinnacle.com support@ssccglpinnacle.com Ph. 09729327755, 09817390373 503


/
s
se
Days 54-60 Mensuration / े िमित

s
la
_c
ob
bo
ah
इसे मोड़कर वृ खंड की दो
sm
(d) 81 π Q3. Radius of the base of a right
ur
yo

सीमां कन ि ाओं को जोड़ कर एक circular cone and a sphere is each


e/
t.m
://

Q36. A solid cylinder of base शंकु का िनमाण िकया जाता है | इस equal to r. If the sphere and the
tp
ht

radius 12cm and height 15 cm is कार िनिमत शंकु का आयतन ( घन cone have the same volume, then
melted and recast into n toys each सेमी म ) ा होगा ? what is the height of the cone?
in the shape of a right circular SSC CGL TIER II (13 एक ल वृ ीय शंकु के आधार एवं
cone of height 9 cm mounted on a September 2019) एक गोले की ि ा दोनों r के बराबर
hemisphere of radius 3 cm. The (a)
343√2
π है | यिद गोले और शंकु का आयतन
6
value of n is : आधार ि ा 12 343√3
समान है , तो शंकु की ऊंचाई िकतनी
सेमी तथा ऊंचाई 15 सेमी वाले एक (b) 6 π है ?
343√3
ठोस बेलन को िपघलाकर n खलौने (c) 12 π SSC MTS 2 August 2019
बनाए जाते ह िजनम से ेक 3 सेमी 343√2 (Morning)
(d) π
ि ा वाले अधगोले के शीष पर 12 (a) 7r
SSC MTS
थत एक ल वृ ीय शंकु के (b) 4r

e
आकार म ह िजसकी ऊंचाई 9 सेमी (c) 2r
Q1. 0.1 percent of 1.728 x 106
है | n का मान ात कर | (d) 3r
SSC CGL TIER II (13 spherical droplets of water, each
September 2019) of diameter 2 mm, coalesce to Q 4. The perimeter of a rhombus
form a spherical bubble. What is

l
(a) 27 is 20 cm. The length of its one of
(b) 64 the diameter (in cm ) of the the diagonals is 6 cm Find the
(c) 48
(d) 54
ac
Q37. The internal diameter of a
hollow hemispherical vessel is 24
bubble ?/ 1.728 x 106
पानी की 1.728 x 106 गोलाकार बू
, िजनम से ेक का ास 2 िममी
है , उन का 0.1 ितशत, एक साथ
िमलकर एक गोलाकार बुलबुला
length of the other diagonal .
िकसी समचतुभुज की प रमाप 20
cm है | इसके एक िवकण की ल ाई
6 cm है | दू सरे िवकण की ल ाई
िकतनी है ?
cm. It is made of a steel sheet बनाते ह। इस बुलबुले का ास (सेमी SSC MTS 2 August 2019
which is 0.5 cm thick. What is the म ) ा है ? 1.728 x 106 (Afternoon)
total surface area (in cm2 ) of the SSC MTS 2 August 2019 (a) 10 cm
nn
vessel ? एक खोखले अधगोलीय (Morning) (b) 8 cm
बतन का आतं रक ास 24 सेमी है | (a) 1.2 (c) 9 cm
यह इ ात की चादर से बना आ है (b) 1.6 (d) 6 cm
िजसकी मोटाई 0.5 सेमी है | इस (c) 1.8
बतन का कुल पृ े फल ( वग सेमी (d) 2.4 Q5. The side of a square is 6 cm.
म) ात कर | Find the area of the largest circle
SSC CGL TIER II (13 Q2. If the volumes of two cubes that can be drawn inside the
September 2019) are in the ratio 64:125, then what square. ( π = 22
7 )
Pi

(a) 612.75 π is the ratio of their total surface एक वग की भुजा 6 cm है | वग के


(b) 468.75 π areas? अंदर बनने वाले सबसे बड़े वृ का
यिद दो घनों के आयतन 64 : 125 के े फल िकतना है ? ( π = 22
(c) 600.2 π 7 )
(d) 600.5 π अनुपात म ह, तो उनके कुल पृ SSC MTS 2 August 2019
े फलों का अनुपात ा होगा ? (Afternoon)
SSC MTS 2 August 2019 (a) 198 2
Q38. A sector of radius 10.5 cm 7 cm
with the central angle 120∘ is (Morning) 318
(b) 7 cm2
folded to form a cone by joining (a) 9:16 252
(c) cm2
the two bounding radii of the (b) 4:5 7
(d) 156 cm2
sector. What is the volume (in (c) 16:25 7

cm3 ) of the cone so formed ? (d) 64:125


Q6. The length, breadth and
10.5 सेमी ि ा वाला एक वृ खंड
height of a cuboid is 15 cm, 12
है िजसका क ीय कोण 120∘ है |
cm and 11 cm respectively.

www.ssccglpinnacle.com support@ssccglpinnacle.com Ph. 09729327755, 09817390373 504


/
s
se
Days 54-60 Mensuration / े िमित

s
la
_c
ob
bo
ah
12 सेमी है | इस शंकु का व पृ
sm
Length is reduced by 6 32 % and curved surface area of the
ur
yo

े फल ात कर | cylinder ?
e/

breadth is increased by 8 % 1
t.m

3
://

SSC MTS 2 August 2019 िकसी गोले की ि ा, एक लंबवत


tp

while the height remains the


ht

same. What is the change in the (Evening) बेलन (िसिलंडर) के आधार की ि ा


total area of four side faces of the (a) 132 π cm2 तथा ऊंचाई के बराबर है | गोले की
cuboid ? ( considering the (b) 143 π cm2 पृ भूिम े फल (सरफेस ए रया)
rectangle formed by the length (c) 156 π cm2 तथा बेलन (िसिलंडर) के व पृ के
and breadth as base) (d) 168 π cm2 े फल का अनुपात िकतना है ?
िकसी घनाभ की ल ाई, चौड़ाई और SSC MTS 5 August 2019
ऊंचाई मशः 15 cm, 12 cm तथा Q9. The length, breadth and (Afternoon)
11 cm है | ल ाई को 6 32 % घटा height of a cuboid are 5 cm, 2 cm (a) 2:1
and 4 cm respectively. What is (b) 1:2
िदया जाता है और चौड़ाई 8 31 % को
the total surface area of the (c) 1:1
बढ़ा िदया जाता है जबिक ऊंचाई को
cuboid? (d) 2:3
अप रवितत रखा जाता है | घनाभ के

e
िकसी घनाभ की लंबाई, चौड़ाई और
(ल ाई और चौड़ाई से बने आयत
ऊंचाई मशः 5 सेमी, 2 सेमी और 4 Q12. The length of a cuboid is
को आधार मानते ए) चार
सेमी है | इस घनाभ का कुल पृ ीय double of its breadth and its
पा -फलकों के कुल े फल म
े फल ात कर | height is half of its breadth. If the
िकतना प रवतन है ?

l
SSC MTS 5 August 2019 height of the cuboid is 2 cm, then
SSC MTS 2 August 2019
ac (Morning) what will be the edge of a cube
(Afternoon)
(a) 84 cm2 whose volume is the same as that
(a) 22 sq. cm decrease
(b) 152 cm2 of the cuboid mentioned above ?
(b) 22 sq. cm increase
(c) 38 cm2 िकसी घनाभ की ल ाई अपनी
(c) No change
(d) 76 cm2 चौड़ाई की दोगुनी है तथा उसकी
(d) 33 sq. cm increase
ऊंचाई, उसकी चौड़ाई की आधी है |
यिद घनाभ की ऊंचाई 2 cm है , तब
Q7. The side of a cube is 15cm. Q10. 70 sticks each of unit length
िकसी घण का कोर (एज) िकतना
What is the base area of a cuboid are combined to form a right
होगा िजसका आयतन उ घनाभ के
nn
whose volume is 175 cm3 less angle triangle without breaking
समान है ?
than that of the cube and whose any stick. What is the area (in
SSC MTS 5 August 2019
height is 32 cm? square units) of the triangle? / 70
(Afternoon)
एक घन की भुजा 15 सेमी है | उस तीिलयाँ , िजनम से ेक इकाई
(a) 4 cm
घनाभ के आधार का े फल ात लंबाई की ह, उ संयु करके िबना
(b) 6.4 cm
कर िजसका आयतन घन के आयतन एक भी तीली तोड़े एक समकोण
(c) 8 cm
से 175 घन सेमी कम है तथा िजसकी ि भुज बनाया जाता है | इस ि भुज
(d) 7.2 cm
ऊंचाई 32 सेमी है | का े फल ( वग इकाई म ) ा
Pi

SSC MTS 2 August 2019 होगा ?


Q13. The diameter of a solid
(Evening) SSC MTS 5 August 2019
hemisphere is 35 cm. What is its
(a) 200 cm2 (Morning)
total surface ? ( π = 22
7 )
(b) 100 cm2
(a) 210 िकसी ठोस अध-गोले का ास 35
(c) 160 cm2
(b) 180 cm है | उसका कुल तल िकतना है | (
(d) 325 cm2 π = 22
(c) 240 7 )
(d) 350 SSC MTS 5 August 2019
Q8. The height of a right circular
(Afternoon)
cone is 5 cm and its base radius is
Q11. The radius of a sphere is (a) 2981 cm2
12 cm. What is the curved surface
equal to the base radius and (b) 2887.5 cm2
area of the cone?
height of a right circular cylinder. (c) 3163 cm2
एक ल वृ ीय शंकु की ऊंचाई 5
What is the ratio between the (d) 2198.5 cm2
सेमी है तथा इसके आधार की ि ा
surface area of the sphere and

www.ssccglpinnacle.com support@ssccglpinnacle.com Ph. 09729327755, 09817390373 505


/
s
se
Days 54-60 Mensuration / े िमित

s
la
_c
ob
bo
ah
sm
Q14. The side of an equilateral (d) 604 cm2 Q20. What is the total surface
ur
yo

triangle is 4 cm. What is its area ? area and the curved surface area
e/
t.m
://

एक समबा ि भुज की भुजा 4 cm है Q17. What is the ratio between of a solid hemisphere with a
tp
ht

| उसका े फल िकतना है ? the inradius and the circumradius radius of 14 cm.


SSC MTS 5 August 2019 of a square ? 14 cm की ि ा वाले िकसी ठोस
(Evening) िकसी वग की अ ः ि ा (इन अधगोले का संपूण पृ े फल तथा
(a) 8√3 cm2 रे िडयस) तथा प रि ा व पृ े फल िकतना है ?
(b) 6√3 cm2 (िसरकम-रे िडयस ) का अनुपात SSC MTS 6 August 2019
िकतना है ? (Afternoon)
(c) 9√3 cm2
SSC MTS 6 August 2019 (a) 5544 cm2 , 1848 cm2
(d) 4√3 cm2 (Morning) (b) 1848 cm2 , 1232 cm2
(a) 1:2 (c) 924 cm2 , 616 cm2
Q15. A ribbon of equal width and
(b) √2 :3 (d) 2772 cm2 , 1848 cm2
length l is wrapped around the
(c) 1:3
curved surface of a right circular

e
(d) 1: √2 Q21. The radius of a right circular
cylinder to cover it completely. If
cylinder is 7 cm. Its height is
the base circumference of this
Q18. The curved surface area of a double of its radius. What is the
cylinder is c, then how many
cylinder is 25344 cm2 and its curved surface area of the
times the ribbon was wrapped

l
height is 32 cm. Find the volume cylinder ? ( π = 22
7 )
around the cylinder ?
ac π
of cylinder whose capacity is 792 िकसी लंब वृ ीय बेलन (िसिलंडर )
एक समान चौड़ाई वाले िकसी रबन,
times the capacity of the cylinder की ि ा 7 cm है | उसकी ऊंचाई
िजसकी ल ाई l है , को एक ल
given. उसकी ि ा की दोगुनी है | बेलन
वृ ीय बेलन (िसिलंडर) के व पृ
िकसी बेलन (िसिलंडर ) का व पृ (िसिलंडर) का व पृ े फल
(कवड सरफेस ) पर पूण प से
े फल 25344 cm2 है तथा उसकी िकतना है ? ( π = 22 )
कवर करने के िलए लपेटा जाता है | 7

यिद बेलन (िसिलंडर) के आधार की ऊंचाई 32 cm है | उस बेलन SSC MTS 6 August 2019
प रिध c है , तो बेलन (िसिलंडर ) पर (िसिलंडर ) का आयतन िकतना है (Afternoon)
रबन से िकतनी बार लपेटा गया : िजसकी धा रता िदए गए बेलन (a) 476 cm2
nn
SSC MTS 5 August 2019 (िसिलंडर) के आयतन की 792 π गु नी (b) 1232 cm2
(Evening) है ? (c) 616 cm2
(a) 4cl SSC MTS 6 August 2019 (d) 896 cm2
(b) l
c
(Morning)
l (a) 3168 cm3 Q22. The side of a hexagon is 4
(c) 2c
2l (b) 6336 cm3 cm. The side of a square is 4 √2
(d) c
(c) 1584 cm3 cm. Find the ratio of their areas.
(d) 9504 cm3 िकसी षट् भुज की भुजा 4 cm है |
Pi

Q16. The volume of a right


circular cone is 1232 cm3 and its िकसी वग की भुजा 4 √2 cm है |
height is 24 cm. Find its curved
Q19. The diagonal of a square is उनके े फल का अनुपात िकतना
surface area. ( π = 22
24 cm. Find its perimeter. है ?
7 )
एक वग का िवकण 24 cm है | SSC MTS 6 August 2019
एक ल वृ ीय शंकु का आयतन
उसका प रमाप िकतना है ? (Afternoon)
1232 cm है तथा उसकी ऊंचाई 24
3
SSC MTS 6 August 2019 (a) 3 √3 :2
cm है | उसका व पृ े फल
(Morning)
िकतना है ? ( π = 7 )
22 (b) 2 √3 :1
(a) 28 cm
SSC MTS 5 August 2019 (c) 3 √3 :4
(b) 48 √2 cm
(Evening)
(c) 36 √2 cm
(d) √3/2 :1
(a) 354 cm2
(b) 550 cm2 (d) 46 √2 cm Q23. The radii of a right circular
(c) 430 cm2 cone and a right circular cylinder
are in the ratio 2:3. If the ratio of

www.ssccglpinnacle.com support@ssccglpinnacle.com Ph. 09729327755, 09817390373 506


/
s
se
Days 54-60 Mensuration / े िमित

s
la
_c
ob
bo
ah
यिद िकसी समचतुभुज के िवकण की
sm
heights of the cone and the Q26. What is the area of a
ur
yo

cylinder is 3:4, then what is the triangle whose sides are 7cm, लंबाई 24 सेमी और 18 सेमी है , तो
e/
t.m
://

ratio of the volumes of the cone 24cm and 25 cm? इस समचतुभुज का े फल ात कर


tp
ht

and the cylinder? उस ि भुज का े फल ात कर |


एक ल वृ ीय शंकु तथा ल िजसकी भुजाएँ 7 सेमी, 24 सेमी और SSC MTS 7 August 2019
वृ ीय बेलन की ि ाएँ 2 : 3 के 25 सेमी की ह | (Afternoon)
अनुपात म ह | यिद शंकु तथा बेलन SSC MTS 7 August 2019 (a) 196 cm2
की ऊंचाई म 3 : 4 का अनुपात है , तो (Morning) (b) 188 cm2
शंकु तथा बेलन के आयतन म ा (a) 72 cm2 (c) 204 cm2
अनुपात होगा ? (b) 108 cm2 (d) 216 cm2
SSC MTS 6 August 2019 (c) 84 cm2
(Evening) (d) 42 cm2 Q30. The base and hypotenuse of
(a) 1:6 a right angle triangle is 9 cm and
(b) 1:3 Q27. What is the area of the 41 cm respectively. What is the

e
(c) 1:9 largest square which can be ratio of triangle ?
(d) 2:3 inscribed in a circle of radius 14 िकसी समकोण ि भुज का आधार
cm? ( π = 22
7 ) तथा कण मशः 9 सेमी एवं 41 सेमी
Q24. The area of rhombus is 300 उस सबसे बड़े वग का े फल ात का है | ि भुज का े फल ा होगा

l
cm2 . If the length of one of the कर िजसे 14 सेमी ि ा वाले वृ के ?
diagonals of the rhombus is 30 भीतर खींचा जा सकता है | ( π = 22 SSC MTS 7 August 2019
ac 7 )
cm, then what is the length (in (Afternoon)
SSC MTS 7 August 2019
cm) of the second diagonal? (a) 180 cm2
(Morning)
िकसी समचतुभुज का े फल 300 (b) 170 cm2
(a) 392 cm2
वग सेमी है | यिद समचतुभुज के एक (c) 190 cm2
(b) 484 cm2
िवकण की लंबाई 30 सेमी है , तो दू सरे (d) 210 cm2
(c) 196 cm2
िवकण की लंबाई िकतनी होगी ? (
(d) 784 cm2
सेमी म ) Q31. What is the area of the
SSC MTS 6 August 2019
nn
Q28. The height of a right largest square which can be
(Evening) inscribed in a circle of radius 28
(a) 25 circular cone is 24 cm and the
radius of its base is 7 cm. What is cm?
(b) 10 उस सबसे बड़े वग का े फल ात
(c) 20 the cost of painting the curved
surface area of the cone at the rate कर िजसे 28 सेमी ि ा वाले वृ के
(d) 30 भीतर खींचा जा सकता है |
of Rs. 6 per cm2 ? ( π = 22
7 )
SSC MTS 7 August 2019
Q25. If the height of an एक ल वृ ीय शंकु की ऊंचाई 24
(Afternoon)
equilateral triangle is 20 √2 cm, सेमी है तथा इसके आधार की ि ा
Pi

(a) 3136 cm2


7 सेमी है | 6 पये ित वग सेमी की
then what is its area (in cm2 )? (b) 1568 cm2
दर से शंकु के व पृ े फल को
यिद िकसी समबा ि भुज की ऊंचाई (c) 784 cm2
रं गने की लागत िकतनी आएगी ?
20 √2 cm है , तो इसका े फल ( (d) 196 cm2
SSC MTS 7 August 2019
वग सेमी म ) ात कर | (Morning)
SSC MTS 6 August 2019 (a) Rs. 3600 Q32. A room is in the shape of a
(Evening) (b) Rs. 3300 cuboid, with dimensions 12m x
(a) 800
3 (c) Rs. 880 10m x 3m. What is the cost of
(b) 1
√3 painting the four walls of the
3 400 (d) Rs. 4200
room at the rate of Rs. 50 per
(c) 1
3 800 √3 sq.m?
400 Q29. If the lengths of the
(d) 3 एक कमरा घनाभ के आकार म है
diagonals of a rhombus are 24 cm
and 18 cm, then what is the area िजसकी लंबाई, चौड़ाई और ऊंचाई
of the rhombus? 12 मी x 10 मी x 3 मी है | 50 पये

www.ssccglpinnacle.com support@ssccglpinnacle.com Ph. 09729327755, 09817390373 507


/
s
se
Days 54-60 Mensuration / े िमित

s
la
_c
ob
bo
ah
ित वग मीटर की दर से इस कमरे
sm
cm. What is the area of the volume (in cm3 ) through which
ur
yo

की चार दीवारों को रं गने की लागत triangle? the rectangle moves?


e/
t.m
://

ात कर | एक समि बा ि भुज की भुजाएँ 10 एक आयत की लंबाई और चौड़ाई


tp
ht

SSC MTS 7 August 2019 सेमी, 10 सेमी और 12 सेमी की ह | मशः 15 सेमी और 12 सेमी है |
(Evening) इस ि भुज का े फल ात कर | यिद आयत की चौड़ाई को धुरी
(a) Rs. 15000 SSC MTS 8 August 2019 मानकर इसे एक बार पूरा घुमाया
(b) Rs. 15600 (Morning) जाता है , तो उसका आयतन ( घन
(c) Rs. 6600 (a) 60 cm2 सेमी म ) ात कर िजससे होकर
(d) Rs. 7500 (b) 48 cm2 आयत घूमता है ?
(c) 40 cm2 SSC MTS 8 August 2019
Q33. The radius of a solid right (d) 44 cm2 (Afternoon)
circular cylinder is 21 cm and its (a) 2160 π
height is 40 cm. What is the cost Q36. The area of a square is 144 (b) 1440 π
of painting the curved surface cm2 . What is the perimeter of the (c) 1800 π

e
area of the cylinder at the rate of square formed by the diagonal of (d) 2700 π
Rs. 20 per sq.cm? (Take π = 22 7 ) the original square as its side?
एक ठोस ल वृ ीय बेलन की एक वग का े फल 144 वग सेमी है Q39. The volume of a cube is 216
ि ा 21 सेमी है तथा इसकी ऊंचाई | आरं िभक वग के िवकण को भुजा cm3 . What is the area of one face

l
40 सेमी है | 20 पये ित वग मीटर मानकर बनाए गए वग का प रमाप of the cube?
की दर से इस बेलन के व पृ
ac े ा होगा ? िकसी घन का आयतन 216 cm3 है |
को रं गने की लागत ात कर | SSC MTS 8 August 2019 इस घन के एक फलक का े फल
SSC MTS 7 August 2019 (Morning) ात कर |
(Evening) (a) 48 √2 cm SSC MTS 8 August 2019
(a) Rs. 108600 (b) 48 cm (Afternoon)
(b) Rs. 105600 (a) 36 cm2
(c) 24 √2 cm
(c) Rs. 26400 (b) 12 cm2
(d) 24 cm
(d) Rs. 5280 (c) 24 cm2
nn
Q37. The radius of hemisphere is (d) 30 cm2
Q34. The area of a rectangle is
14cm. What is the cost of
180 cm2 . If the ratio of length
painting the outer curved surface Q40. What is the area of the
and breadth of the rectangle is largest circle that can be inscribed
of the hemisphere at the rate of
5:4, then what is the length (in in a square of side 28 cm? (Take
Rs. 45 per sq.cm? (Take π = 22
7 )
cm) of diagonal of the rectangle? π = 22
एक अधगोले की ि ा 14 सेमी है | 7 )
एक आयत का े फल 180 वग सेमी
इस अधगोले के बाहरी व पृ को उस सबसे बड़े वृ का े फल ात
है | यिद आयत की लंबाई और चौड़ाई
45 पये ित वग सेमी की दर से कर िजसे 28 सेमी भुजा वाले एक वग
का अनुपात 5 : 4 है , तो इस आयत
Pi

रं गने की लागत ात कर | ( π = 22 के भीतर खींचा जा सकता है | (


के िवकण की लंबाई ( सेमी म ) 7
) π = 22
7 )
िकतनी होगी ?
SSC MTS 7 August 2019 SSC MTS 8 August 2019 SSC MTS 8 August 2019
(Evening) (Morning) (Afternoon)
(a) Rs 53160 (a) 308 cm2
(a) √423 (b) Rs 55440 (b) 616 cm2
(b) √317
(c) Rs 56820 (c) 476 cm2
(c) √369 (d) Rs 58280 (d) 512 cm2
(d) √371
Q38. The length and breadth of a Q41. What is the circumference
Q35. The sides of an isosceles rectangle are 15 cm and 12 cm of the largest circle that can be
triangle are 10 cm, 10 cm and 12 respectively. If the rectangle is inscribed in a square of side 14
given one full rotation about its cm? (Take π = 227 )
breadth as the axis, what is the

www.ssccglpinnacle.com support@ssccglpinnacle.com Ph. 09729327755, 09817390373 508


/
s
se
Days 54-60 Mensuration / े िमित

s
la
_c
ob
bo
ah
उस सबसे बड़े वृ की प रिध िकतनी
sm
SSC MTS 9 August 2019 SSC MTS 9 August 2019
ur
yo

होगी िजसे 14 सेमी भुजा वाले वग म (morning) (Afternoon)


e/
t.m
://

समािहत िकया जा सकता है |( (a) 240 cm2 (a) 18 √2


tp
ht

π = 22
7 ) (b) 260 cm2 (b) 18
SSC MTS 8 August 2019 (c) 65 cm2 (c) 36
(Evening) (d) 130 cm2 (d) 36 √2
(a) 66 cm
(b) 88 cm Q45. How many balls of radius Q48. A room is in the shape of a
(c) 22 cm 45 cm can be made by melting a cube and the length of the longest
(d) 44 cm bigger ball of diameter 360 cm? rod placed in it is 21 √3 cm. The
360 सेमी ास वाली एक बड़ी गद
area of the floor is:
Q42. Edge of a cube is 8cm. को िपघलाकर 45 सेमी ि ा वाली
कोई कमरा घन के आकार म है तथा
What is the total surface area of िकतनी गदों का िनमाण िकया जा
इसम रखी गयी सबसे लंबी छड़ की
the cube? सकता है ?
लंबाई 21 √3 सेमी है | तल का

e
िकसी घन का िकनारा 8 सेमी का है | SSC MTS 9 August 2019
इस घन का कुल पृ ीय े फल ात े फल है :
(morning)
कर | SSC MTS 9 August 2019
(a) 64
SSC MTS 8 August 2019 (Afternoon)
(b) 128
(a) 441 cm2

l
(Evening) (c) 32
(a) 128 cm2 (d) 48 (b) 144 cm2
(b) 256 cm2
(c) 384 cm2
(d) 484 cm2
ac
Q43. The length of one of the
Q46. A cuboid of size 100 cm x
80 cm x 60 cm cut into eight
identical parts by three cuts.
Where is the total surface area (in
(c) 169 cm2
(d) 961 cm2

Q49. If the height of a circular


cone is decreased by 10% and its
diagonals of a rhombus is 48 cm, square cm.) of all the eight parts? radius is increased by 10%, then
If side of the rhombus is 26 cm, 100 सेमी x 80 सेमी x 60 सेमी what will be the change in its
आकार वाले एक घनाभ को तीन बार volume?
nn
then what is the area of the
rhombus? काट कर आठ सम प िह ों म यिद िकसी वृ ाकार शंकु की ऊंचाई
िकसी समचतुभुज के एक िवकण की काटा जाता है | सभी आठ िह ों का 10% कम कर दी जाती है और
लंबाई 48 सेमी है | यिद समचतुभुज कुल पृ ीय े फल ( वग सेमी म ) इसकी ि ा को 10% बढ़ा िदया
की भुजा 26 सेमी की है , तो िकतना होगा ? जाता है , तो इसके आयतन म ा
समचतुभुज का े फल ा होगा ? SSC MTS 9 August 2019 प रवतन आएगा ?
SSC MTS 8 August 2019 (morning) SSC MTS 9 August 2019
(Evening) (a) 22500 (Afternoon)
(a) Decreases by 8.9%
Pi

(a) 540 cm2 (b) 84100


(c) 50750 (b) Decreases by 3.2%
(b) 420 cm2
(d) 75200 (c) Increases by 8.9%
(c) 360 cm2
(d) Increases by 3.2%
(d) 480 cm2
Q47. If in a triangle angles are in
the ratio 1:1:2 and the length of Q50. The hypotenuse of a
Q44. The length of the diagonal
its longest side is 6 √2 cm, then right-angled triangle is 39 cm and
of a rectangle is 26 cm and one
the difference between the other
side is 10 cm. The area of the what is the area (in cm2 ) of the
two sides is 21 cm. The area of
rectangle is: triangle?
this triangle is :
एक आयत के िवकण की लंबाई 26 यिद िकसी ि भुज म, कोण 1 : 1 : 2
िकसी समकोण ि भुज का कण 39
सेमी है तथा इसकी एक भुजा 10 के अनुपात म ह तथा इसकी सबसे
cm है और अ दो भुजाओं का अंतर
सेमी की है | इस आयत का े फल लंबी भुजा 6 √2 सेमी की है , तो इस
21 cm है , तो ि भुज का े फल है ?
है : ि भुज का े फल ात कर |

www.ssccglpinnacle.com support@ssccglpinnacle.com Ph. 09729327755, 09817390373 509


/
s
se
Days 54-60 Mensuration / े िमित

s
la
_c
ob
bo
ah
sm
SSC MTS 9 August 2019 SSC MTS 13 August 2019 (b) 156 cm2
ur
yo

(Evening) (Morning) (c) 412 cm2


e/
t.m
://

(a) 360 cm2 (a) 3080 (d) 139 cm2


tp
ht

(b) 270 cm2 (b) 3160


(c) 280 cm2 (c) 2920 Q57. The volume of prism is 308
(d) 540 cm2 (d) 3240 cm3 and height is 11 cm. The
base area of prism is:
Q51. There is a path of equal Q54. The volume of a prism is ि का आयतन 308 cm3 है तथा
width of 3.5 m along with a 288 cm3 and height is 24 cm. The ऊंचाई 11 सेमी है | इस ि के
building measuring 25 m in base area ( in cm2 ) of the prism आधार का े फल है :
length and 15 m in breadth. Find is : SSC MTS 13 August 2019
the flooring cost of this path at ि का आयतन 288 cm3 और (Afternoon)
the rate of Rs. 25.50 per sq. m. ऊंचाई 24 cm है | ि के आधार (a) 21 cm2
एक 25 मीटर ल ा तथा 15 मीटर का े फल ( cm म) है :
2
(b) 14 cm2

e
चौड़ा माप के भवन के साथ 3.5 SSC MTS 13 August 2019 (c) 28 cm2
मीटर एक समान चौड़ाई का माग है | (Morning) (d) 22 cm2
Rs. 25.50 ित वग मीटर की दर से (a) 10
माग की ो रं ग लागत है : (b) 12 Q58. The diagonal of a square

l
SSC MTS 9 August 2019 (c) 15
measures 6 √2 cm. The measure
(Evening) ac (d) 14
of the diagonal of a square whose
(a) Rs. 8389.50
area is twice that of the first
(b) Rs. 4186.50 Q55. The length, breadth and
square is:
(c) Rs. 8146.50 height of a cuboid are 18 cm, 24
िकसी वग का िवकण 6 √2 सेमी का
(d) Rs. 9149.50 cm, and 4 cm respectively. The
volume of a cube is equal to that है | उस वग के िवकण का माप ात
Q52. If the length of a side of a of the given cuboid. The side of कर िजसका े फल पहले वग के
square is equal to the diameter of the cube is : े फल से दोगुना है ?
एक घनाभ की ल ाई, चौड़ाई और SSC MTS 13 August 2019
nn
a circle, then the ratio between
the areas of square and circle is ऊंचाई मसः 18 cm, 24 cm और 4 (Afternoon)
(π = 22 cm है | घन का आयतन िदए गए (a) 12 cm
7 )
यिद िकसी वग की भुजा की ल ाई घनाभ के आयतन के बराबर है | घन (b) 12 √2 cm
िकसी वृ के ास के बराबर है , तब की भुजा है : (c) 6 cm
वग तथा वृ के े फल का अनुपात SSC MTS 13 August 2019 (d) 6 √2 cm
है : ( π = 22 (Morning)
7 )
SSC MTS 9 August 2019 (a) 9 cm Q59. How many cubes with a
Pi

(Evening) (b) 16 cm side 10 cm can be cut out of a


(a) 14:11 (c) 12 cm cube having a side of 10 metre ?
(b) 28:11 (d) 8 cm 10 मीटर भुजा वाले एक घन से 10
(c) 11:14 सेमी भुजा वाले िकतने घन काटे जा
(d) 11:7 Q56. The area of the sheet metal सकते ह ?
needed to make a box of size 7cm (a) 10,000
Q53. The radius and height of a x 8cm x 9cm is: (b) 1,00,00,000
cone are 20 cm and 21 cm 7 सेमी x 8 सेमी x 9 सेमी आकार के (c) 1,00,000
respectively. The total surface एक िड े को बनाने के िलए (d) 10,00,000
area ( cm2 ) of the cone is : आव क धा क चादर का े फल
िकसी शंकु की ि ा 20 cm तथा होगा : Q60. Find the curved surface area
ऊंचाई 21 cm है , शंकु का कुल पृ SSC MTS 13 August 2019 of a right circular cylinder whose
े फल ( cm2 म) है : ( π = 22 (Afternoon) height and diameter are 10 cm
7 )
(a) 382 cm2 and 8 cm respectively.

www.ssccglpinnacle.com support@ssccglpinnacle.com Ph. 09729327755, 09817390373 510


/
s
se
Days 54-60 Mensuration / े िमित

s
la
_c
ob
bo
ah
एक ल वृ ीय बेलन, िजसकी जाता है और 9 सेमी ि ा वाले बेलन
sm
Q63. The total surface area of a
ur
yo

ऊंचाई और ास मशः 10 cm hemisphere is 462 cm2 . What is का िनमाण िकया जाता है | इस बेलन
e/
t.m
://

और 8 cm है , के व पृ का the diameter? का आयतन है :


tp
ht

े फल ( cm म) िकतना है ?
2
एक अधगोले का कुल पृ े फल SSC MTS 14 August 2019
SSC MTS 14 August 2019 462 वग सेमी है | ास ात कर | ( (Evening)
(Morning) π = 22
7 )
(a) 1525 cm3
(a) 20 π SSC MTS 14 August 2019 (b) 900 cm3
(b) 80 π (Afternoon) (c) 1750 cm3
(c) 160 π (a) 14 cm (d) 1782 cm3
(d) 40 π (b) 7 cm
(c) 17.5 cm Q67. The radii of two cylinders A
Q61. If two cubes, each having (d) 10.5 cm and B are in the ratio 5:6 and the
the side 10 mm, are put in a box heights are in the ratio 7:4
containing 200 c.c water, then Q64. A hemisphere of radius 30 respectively. The ratio of curved

e
what will be the volume of the cm is moulded to form a cylinder surface area of cylinder B to that
material (in ml) in the box ? of height 180 cm. The diameter of of A is:
यिद लोहे से िनिमत ेक 10 mm the cylinder is: दो बेलन A और B की ि ाएँ 5 : 6
की भुजावाले 2 ूब को 1 ऐसे िड े 30 सेमी ि ा वाले एक अधगोले को के अनुपात म ह तथा ऊंचाई 7 : 4 के

l
म डाला जाता है , िजसमे 200 c.c. िपघलाकर 180 सेमी ऊंचाई वाला अनुपात म है | बेलन B तथा बेलन A
पानी है , तो िड े की साम ी का
ac एक बेलन बनाया जाता है | इस बेलन के व पृ े फल म अनुपात ात
आयतन (ml म) ा होगा? का ास है : कर |
SSC MTS 14 August 2019 SSC MTS 14 August 2019 SSC MTS 14 August 2019
(Morning) (Afternoon) (Evening)
(a) 201 (a) 15 cm (a) 35:24
(b) 202 (b) 10 cm (b) 24:35
(c) 200002 (c) 5 cm (c) 49:35
(d) 200001 (d) 20 cm (d) 35:49
nn
Q62. A solid cube, in which the Q65. What is the total surface Q68. The diagonal of a square is
diagonal of each face is 128 √2 area of a cone which has a radius equal to the side of an equilateral
cm, has been molded to make a of 21 cm and a height of 28 cm? ( triangle. If the area of the square
cuboid. The length and breadth of π = 22
7 ) is 18 √3 sq.cm. What is the area
this cuboid are 512 cm and 160 उस शंकु का कुल पृ े फल ात (in cm2 ) of the equilateral
cm respectively. What is the कर िजसकी ि ा 21 सेमी और triangle?
height of this cuboid ? ऊंचाई 28 सेमी है | िकसी वग का िवकण एक समबा
Pi

एक ठोस घन, िजसका ेक सतह SSC MTS 14 August 2019 ि भुज की भुजा के बराबर है | यिद
(फेस) िवकण 128 √2 cm है , को (Afternoon) वग का े फल 18 √3 वग सेमी है ,
घनाभ बनाने के िलए मो िकया (a) 3696 cm2 तो समबा ि भुज का े फल ( वग
गया | घनाभ की ल ाई और चौड़ाई (b) 1848 cm2 सेमी म ) ात कर |
मसः 512 cm और 160 cm है | (c) 5544 cm2 SSC MTS 14 August 2019
घनाभ की ऊंचाई िकतनी है ? (d) 7392 cm2 (Evening)
SSC MTS 14 August 2019 (a) 54 √2
(Morning) Q66. A rectangular paper of
(b) 27 √2
(a) 25.6 cm width 7 cm is rolled along its
(c) 54
(b) 16 cm width and a cylinder of radius 9
(d) 27
(c) 20.8 cm cm is formed. The volume of the
(d) 16.4 cm cylinder is:
Q69. What is the volume of the
7 सेमी चौड़ाई वाले एक आयताकार
sphere whose diameter is 42 cm?
कागज़ को इसकी चौड़ाई से लपेटा

www.ssccglpinnacle.com support@ssccglpinnacle.com Ph. 09729327755, 09817390373 511


/
s
se
Days 54-60 Mensuration / े िमित

s
la
_c
ob
bo
ah
उस गोले का आयतन ात कर िकसी बेलन (िसिलंडर) के आयतन (
sm
10%. By how much did the
ur
yo

िजसका ास 42 सेमी है | volume change? cm3 म) तथा व पृ े फल (


e/
t.m
://

( π = 22
7 ) एक घनाभ की लंबाई और चौड़ाई ( cm2 म) के बीच अनुपात सं ा क
tp
ht

SSC MTS 16 August 2019 ेक को ) 10% बढ़ा िदया जाता है प से 14:1 है I यिद बेलन
(Morning) जबिक ऊंचाई को 10% कम कर (िसिलंडर) की ऊंचाई 50 cm है , तो
(a) 9702 cm3 िदया जाता है | आयतन म िकतना बेलन का आयतन िकतना है ? (
(b) 38808 cm3 प रवतन आया ? π = 22
7 )
(c) 19404 cm3 SSC MTS 16 August 2019 SSC MTS 16 August 2019
(d) 58212 cm3 (Morning) (Afternoon)
(a) 10% decrease (a) 61600 cm3
Q70. A spherical ball of diameter (b) 8.9% increase (b) 92400 cm3
35 cm rolls 20 times. How much (c) 8.9% decrease (c) 123200 cm3
(d) 10% increase
is the distance (in m) covered by (d) 184800 cm3
it? ( π = 22
7 )

e
Q73. If the radius of a cylinder is
35 सेमी ास वाली एक गोलीय गद Q76. If the height of a right
doubled and the height is reduced
20 बार घूमती है | इसके ारा तय की circular cylinder is 10 cm, and its
by 50%, then the volume
गयी दू री ( मीटर म ) ात कर | ( curved surface area is 440 cm2 ,
increases/decreases by what
π = 22
7 ) then what is its radius ?

l
percent ?
SSC MTS 16 August 2019 यिद िकसी ल वृ ीय बेलन
यिद िकसी बेलन (िसिलंडर) की
(Morning) ac (िसिलंडर) की ऊंचाई 10 cm है , और
ि ा को दोगुना िकया जाता है और
(a) 22 उसका व पृ े फल 440 cm2
ऊंचाई को 50% काम िकया जाता है ,
(b) 20 है , तो उसकी ि ा िकतनी है ?
तो आयतन म िकतने ितशत
(c) 35 SSC MTS 16 August 2019
वृ /िगरावट होती है ?
(d) 44 (Evening)
SSC MTS 16 August 2019
(a) 17.5 cm
(Afternoon)
Q71. A rectangular park was (b) 10.5 cm
(a) 100% increase/ वृ
redesigned and as a result of (c) 14 cm
(b) 66.67% decrease/ कमी
nn
which its length increased by (d) 7 cm
(c) 75% increase/ वृ
50%. If the area of the park, (d) 50% decrease/ कमी
remained unchanged, then by Q77. The volumes of a sphere
how much percentage had the and a right circular cylinder is
Q74. Find the volume of a cube
breadth been reduced? equal. The radii of the sphere and
whose side is 7.5 cm.
एक आयताकार उ ान को िफर से एक घन, िजसकी भुजा 7.5 cm है ,
the cylinder are 21 cm and 14 cm
बनाया गया तथा इसके का आयतन िकतना है ?
respectively. The height of the
प रणाम प लंबाई 50% से बढ़ SSC MTS 16 August 2019
cylinder is :
Pi

गयी | यिद उ ान का े फल िकसी गोले और िकसी ल ृ ीय


(Afternoon)
अप रवितत रहा, तो चौड़ाई को बेलन (िसिलंडर) का आयतन सामान
(a) 421.875 cm3
िकतने ितशत से कम िकया गया ? है | गोला और बेलन की ि ा
(b) 759.375 cm3
SSC MTS 16 August 2019 मसः 21 cm और 14 cm है | बेलन
(c) 631.81 cm3
(Morning) की ऊंचाई िकतनी है ?
(d) 210.94 cm3
(a) 25 SSC MTS 16 August 2019
(b) 50 (Evening)
Q75. The ratio between the (a) 63 cm
(c) 33.33
volume (in cm3 ) and curved (b) 56 cm
(d) 40.33
surface area ( in cm2 ) of a (c) 42 cm
Q72. The length and breadth of a cylinder is numerically 14 : 1. If (d) 49 cm
cuboid are increased 10% each, the height of the cylinder is 50
whereas the height is reduced by cm, then find its volume. (
π = 22
7 )

www.ssccglpinnacle.com support@ssccglpinnacle.com Ph. 09729327755, 09817390373 512


/
s
se
Days 54-60 Mensuration / े िमित

s
la
_c
ob
bo
ah
sm
Q78. The total surface area of a of a hemisphere of radius 21cm? (c) 7 π cm2
ur
yo

cube is 864 cm2 . What is its (π = 22


7 ) (d) 16 π cm2
e/
t.m
://

volume (in cm3 ) ? ि ा 21 सेमी वाले अधगोले का व


tp
ht

िकसी घन का कुल पृ े फल 864 पृ े फल एवं आयतन ( मशः ) Q85. A sphere is inscribed in a


cm है | उसका आयतन ( cm3 म)
2
ात कर | cube. What is the ratio of the
िकतना है ? SSC MTS 19 August 2019 volume of the cube to the volume
SSC MTS 16 August 2019 (Morning) of the sphere?
(Evening) (a) 2772 cm2 , 19404 cm3 एक गोले को िकसी घन म डाला गया
(a) 216 (b) 4158 cm2 , 19404 cm3 है | घन के आयतन तथा गोले के
(b) 1728 (c) 2772 cm2 , 4158 cm3 आयतन म अनुपात ात कर |
(c) 729 (d) 2772 cm2 , 9702 cm3 SSC MTS 19 August 2019
(d) 512 (Evening)
Q82. What is the area of a (a) 6: π
Q79. The volume of a right triangle with sides 35 cm, 84 cm (b) 8: π

e
circular cylinder is 3 times the and 91 cm ? (c) 11:2 π
volume of a right circular cone. 35 cm, 84 cm तथा 91 cm भुजाओं (d) 9:2 π
The radius of the cone and the वाले िकसी ि भुज का े फल
cylinder are 3cm and 6cm िकतना है ? SSC MTS 19 August Q86. The radius of a solid right

l
respectively. If the height of the 2019 (Afternoon) circular cylinder is 66 32 % of its
cylinder is 1cm, then what is the
ac (a) 2160 cm2 height. If height is h centimeters
slant height of the cone? (b) 1530 cm2 then its total surface area (in cm2
िकसी ल वृ ीय बेलन का आयतन (c) 1470 cm2 ) is:
एक ल वृ ीय शंकु के आयतन से (d) 1880 cm2 ठोस ल वृ ीय बेलन की ि ा
तीन गुना है | शंकु तथा बेलन की इसकी ऊंचाई का 66 3 % है | यिद
2
ि ा मशः 3 सेमी और 6 सेमी है |
Q83. If the lateral surface area of ऊंचाई h सेमी है , तो इसका कुल पृ
यिद बेलन की ऊंचाई 1 सेमी है , तो
a cube is 144 cm2 , then find the े फल ( वग सेमी म ) ात कर |
शंकु की ितयक ऊंचाई ात कर |
length of its side. SSC MTS 19 August 2019
nn
SSC MTS 19 August 2019
यिद िकसी घन का पाशव (lateral) (Evening)
(Morning)
पृ े फल 144 cm2 है , तो (a) 20
9 πh
2
(a) √13 cm उसकी भुजा की ल ाई िकतनी है ? 40 πh2
(b) 9
(b) 4 cm SSC MTS 19 August 2019
(c) 44 πh2
(c) 5 cm (Afternoon) 9
2
(d) √15 cm (a) 4 cm (d) 4πh
(b) 8 cm
Q80. The volume of a sphere is (c) 5 cm Q87. What is the area (in m2 ) of
Pi

36 π cm3 . What is the radius of (d) 6 cm a triangle field whose sides


the sphere? measure 25 m, 39 m and 56 m?
एक गोले का आयतन 36 π घन सेमी Q84. The height and the radius of एक ि भुजाकार मैदान का े फल (
है | इस गोले की ि ा ात कर | a right circular cone are 4 cm and वग मीटर म ) ात कर िजसकी
SSC MTS 19 August 2019 3 cm respectively. What is the भुजाओं का माप 25 मी, 39 मी और
(Morning) total surface area of this cone ? 56 मी है |
(a) 4 cm िकसी लंब वृ ीय शंकु की ऊंचाई SSC MTS 19 August 2019
(b) 2 cm तथा ि ा मश: 4 cm तथा 3 cm (Evening)
(c) 5 cm है . शंकु का कुल पृ े फल (a) 210
(d) 3 cm िकतना है ? (b) 240
SSC MTS 19 August 2019 (c) 420
Q81. What are respectively the (Afternoon) (d) 480
curved surface area and volume (a) 12 π cm2
(b) 24 π cm2

www.ssccglpinnacle.com support@ssccglpinnacle.com Ph. 09729327755, 09817390373 513


/
s
se
Days 54-60 Mensuration / े िमित

s
la
_c
ob
bo
ah
sm
Q88. If the radius of a right SSC MTS 20 August 2019 maximum possible volume is cut.
ur
yo

circular cone is reduced by 10% (Morning) The side of the cube is :


e/
t.m
://

and its height is increased by (a) 64 π लकड़ी के बने 6 cm ि ा के एक


tp
ht

40%, then find the percentage (b) 96 π ठोस से, अिधकतम संभािवत प रमाण
increase or decrease in its (c) 80 π (वॉ ूम) के एक घन को हटाया जाता
volume. (d) 40 π है | घन की भुजा ात कीिजये|
यिद एक लंब वृ ीय शंकु की ि ा SSC MTS 20 August 2019
को 10 % कम िकया जाता है और Q91. The length of a rectangle is (Afternoon)
उसकी ऊंचाई को 40 % बढ़ाया 6 cm more than its breadth. And (a) 4√3 cm
जाता है , तब उसके आयतन म िकतने its perimeter is 100 cm. If the (b) 3√3 cm
ितशत की वृ या िगरावट होती है ? area of this rectangle is nearest (c) 6√3 cm
SSC MTS 20 August 2019 equal to the area of a circle, then
(Morning) (d) 2√3 cm
what will be the circumference of
(a) 13.4 % of decrease this circle ?
Q94. The volumes of two spheres

e
(b) 1.34 % of decrease एक आयत की ल ाई, उसकी
(c) 1.34 % of increase are in the ratio of 64:125. The
चौड़ाई से 6 cm अिधक है और
(d) 13.4 % of increase ratio between their surface areas
उसका प रमाप 100 cm है | यिद इस
is :
आयत का े फल एक वृ के
दो गोलों का आयतन 64:125 के

l
Q89. The area of a circular park े फल के िनकटतम बराबर है , तो
is approximately equal to the अनुपात म है तो उनके पृ े फल
ac उस वृ की प रिध िकतनी है ?
seven-fifteenth of the area of a का अनुपात है :
SSC MTS 20 August 2019
triangular park with sides 110 m, SSC MTS 20 August 2019
(Afternoon)
600 m, and 610 m. Find the (Evening)
(a) 88 cm
diameter of the park. (a) 4:25
(b) 110 cm
एक वृ ाकार पाक का े फल एक (b) 4:5
(c) 132 cm
ऐसे ि भुजाकार मैदान िजसकी (c) 16:25
(d) 66 cm
भुजाएं 110 m, 600 m और 610 m ह (d) 25:16
, के े फल के 7/15
nn
Q92. The total surface area of a
(seven-fifteenth) के लगभग बराबर Q95. What is the volume ( in
solid cylinder is 1155 cm2 . Its
है | पाक का ास िकतना है ? cm3 ) of a sphere with diameter
curved surface area is equal to
SSC MTS 20 August 2019 18 cm. ( correct to two decimal
two-fifths of its total surface area.
(Morning) places )
What is the height of the cylinder
(a) 160 m 18 cm ास वाले िकसी गोले का (दो
?
(b) 120 m द बलव थानों तक) आयतन (
एक ठोस बेलन (िसिलंडर) का कुल
(c) 150 m cm3 म) ा है ? ( π = 22
7 )
पृ े फल 1155 cm2 है | उसका
(d) 140 m SSC MTS 20 August 2019
Pi

व पृ े फल, उसके कुल पृ


े फल के 5 के बराबर है | बेलन
2 (Evening)
Q90. A solid metallic cylinder, (a) 3504.58
(िसिलंडर) की ऊंचाई (cm म)
whose base radius is 4 cm and (b) 4503.86
िकतनी है ? ( π = 22
7 )
height is 5 31 cm, is melted and (c) 4530.58
SSC MTS 20 August 2019
converted into a sphere. What is (d) 3054.86
(Afternoon)
the surface area of the sphere. (in
(a) 7
cm2 ) Q96. The central angle of a sector
(b) 5
एक ठोस धातु के बेलन (िसिलंडर ), of a circle with a radius of 30 cm
(c) 6
िजसकी आधार ि ा 4 cm है और measures 210°. What is the area (
(d) 10.5
ऊंचाई 5 3 cm है , को िपघलIकर एक
1 cm2 ) of the sector given ?
गोले के प म प रवितत िकया जाता 30 cm ि ा वाले िकसी वृ के
Q93. From a solid made of wood
है | गोले का पृ े फल, cm2 म, वृ खंड के क ीय कोण की माप
with radius 6 cm, a cube of
िकतना है ?

www.ssccglpinnacle.com support@ssccglpinnacle.com Ph. 09729327755, 09817390373 514


/
s
se
Days 54-60 Mensuration / े िमित

s
la
_c
ob
bo
ah
210° है | िदए गये वृ खंड का
sm
(b) 1:2 (d) 480
ur
yo

े फल ( cm2 म) ा है ? ( π = 22
7 ) (c) 1: √2
e/
t.m
://

SSC MTS 20 August 2019 Q103. The radius of a circle is


tp

(d) 2:1
ht

(Evening) equal to the length of a rectangle.


(a) 1650 Q100. A solid sphere of brass The circumference of the circle
(b) 1645 with a radius 15 cm is drawn into and the breadth of the rectangle
(c) 1649 a wire of diameter 6 mm. The are 132 cm and 20 cm
(d) 1647 length of the wire ( in cm ) is : respectively. The diagonal of the
15 cm ि ा वाले पीतल के एक rectangle is :
Q97. If the wheel of a bicycle ठोस गोले को 6 mm ास के तार म ( π = 22
7 )
takes 160 rounds to cover a खंचा जाता है | तार की ल ाई (cm िकसी वृ की ि ा िकसी आयत
distance of 1.8 km, then find its म) है : की ल ाई के बराबर है | वृ की
radius ( in m ). SSC MTS 21 August 2019 प रिध और आयत की चौड़ाई मशः
यिद िकसी साइिकल का पिहया 1.8 (Morning) 132 cm और 20 cm है | आयत का

e
km की दू री तय करने म 160 च र (a) 50000 िवकण है : ( π = 22
7 )
लगता है , तो उसकी ि ा (m म) (b) 60000 SSC MTS 21 August 2019
ा है ? (c) 55000 (Afternoon)
SSC MTS 20 August 2019 (d) 45000 (a) 28 cm

l
(Evening) (b) 29 cm
45
(a) 8π ac Q101. A solid metallic cuboid (c) 25 cm
(b) 4.5

measuring 343 cm x 49 cm x 7 (d) 27 cm
(c) 15 cm is melted and converted into

45 cubes with side 7 cm. The sum of Q104. 2541 metallic sphere balls,
(d) 4π
the total surface areas ( cm2 ) of each with a radius of 1 cm, are
all the cubes is : melted to form a cube. The total
Q98. If the area of a circle is 154
343 cm x 49 cm x 7 cm के िकसी surface area of the cube ( in cm2 )
sq. cm, then find the ratio
ठोस धा क घनाभ को िपघलाकर 7 in the nearest form will be :
between the circumferences of
nn
cm भुजा वाले घन िनिमत िकये जाते 2541 गोलाकार धातुमय गदों,
this circle and another circle with
है | सभी िनिमत घणो के कुल पृ िजनकी ेक की ि ा 1 cm है ,
a radius 21 cm.
े फलों ( cm2 म) का योग है : को िपघलIकर एक घन बनाया जाता
यिद िकसी वृ का े फल 154 वग
SSC MTS 21 August 2019 है | घन का स ूण पृ े फल (वग
cm है , तो उस वृ की प रिध और 21
(Afternoon) cm2 म) िनकटतम प म होगा:
cm ि ा वाले िकसी दू सरे वृ की
(a) 16807 SSC MTS 21 August 2019
प रिधयों का अनुपात है :
(b) 10842 (Evening)
SSC MTS 21 August 2019
(c) 120506 (a) 1936
(Morning)
Pi

(d) 100842 (b) 2904


(a) 1:3
(b) 2:3 (c) 2992
Q102. The length, breadth and (d) 3168
(c) 2:1
height of a cuboid are 6 cm, 8 cm
(d) 1:2
and 10 cm respectively. Its Q105. A cylindrical bucket,
volume ( in cm3 ) is : whose height is 27 cm and base
Q99. The side of a square is a cm.
िकसी घनाभ की ल ाई, चौड़ाई और radius is 48 cm, is filled with
The ratio between its diagonal
ऊंचाई मशः 6 cm, 8 cm और 10 sand. When the bucket is emptied
and its side is :
cm है | इसका आयतन ( cm3 म) है : on the ground, a conical heap of
िकसी वग की भुजा a cm है , इसके
िवकण और इसकी भुजा का अनुपात SSC MTS 21 August 2019 radius 54 cm is formed. What is
है : (Afternoon) the height ( in cm ) of the heap ?
SSC MTS 21 August 2019 (a) 128 एक बेलनाकार (िसिलंिडकल) बा ी,
(Morning) (b) 256 िजसकी ऊंचाई 27 cm और आधार
(c) 960 ि ा 48 cm है , को रे त (सड) से
(a) √2 :1
www.ssccglpinnacle.com support@ssccglpinnacle.com Ph. 09729327755, 09817390373 515
/
s
se
Days 54-60 Mensuration / े िमित

s
la
_c
ob
bo
ah
भरा जाता है | जब बा ी को जमीन
sm
(b) 1.8frustum of a right circular (b) 36√2π
ur
yo

पर खाली िकया जाता है और 54 cm cone. The radii of its circular ends


e/

(c) 48√2π
t.m
://

ि ा का एक शंकुकार ढे र बन are 4 m and 8 m and its depth is 7


tp

(d) 54√2π
ht

जाता है | ढे र की ऊंचाई (cm म) m. How many kilolitre of water


िकतनी है ? (correct up to one decimal place)
Q111. A wire, is in the form of a
SSC MTS 21 August 2019 can it hold? ( π = 22
7 )
circle, encloses an area 3118.5
(Evening) एक जलाशय िकसी ल वृ ीय शंकु
cm2 . It is now bent to form a
(a) 32 के िछ क के आकार का है | इसके
(b) 56 rectangle whose length and
वृ ाकार िसरों की ि ाएँ 4 मी और
(c) 54 breadth are very nearly in the
8 मी की ह तथा इसकी गहराई 7 मी
(d) 64 ratio 7:4. The length of the
है | यह िकतने िकलोलीटर पानी (
rectangle, in cm, is:
दशमलव के दो थान तक ) को
Q106. The area of a circular एक तार, जो वृ के प म है ,
धारण कर सकता है ?
ground is approximately equal to 3118.5 वग सेमी े को घेरता है |
SSC MTS 22 August 2019
अब इसे मोड़कर एक आयत बनाया

e
73 31 % of the area of a triangular (Morning)
जाता है िजसकी लंबाई और चौड़ाई
ground with sides 400 m, 420 m (a) 821.3
लगभग 7 : 4 के अनुपात म है | इस
and 580 m. What is the (b) 815.7
आयत की लंबाई ( सेमी म ) है :
circumference ( in m ) of the (c) 792.3
(Take π = 227 )

l
circular ground ? ( π = 22
7 ) (d) 775.7
SSC MTS 22 August 2019
िकसी वृताकार मैदान का े फल,
ac Q109. 1000 solid spherical balls (Afternoon)
िकसी ऐसे ि भुजाकार मैदान,
each of radius 0.6 cm are melted (a) 56
िजसकी भुजाएं 400 m, 420 m और
and recast into a single ball. What (b) 49
580 m है , के े फल के 73 31 % के
is the surface area (in cm2 ) of (c) 7
लगभग बराबर है | वृताकार मैदान (d) 63
की प रिध (m म) िकतनी है ? ( ball so formed?
π = 22 ेक 0.6 सेमी ि ा वाली 1000
7 ) Q112. A copper wire of radius 0.5
ठोस गोलीय गदों को िपघलाकर एक
SSC MTS 21 August 2019 mm and length 42 32 m is melted
गद बनाई जाती है | इस कार बनी
nn
(Evening) and converted into a sphere of
गद का पृ े फल ( वग सेमी म )
(a) 880 m radius R cm. What is the value of
ा होगा ?
(b) 440 m R?
SSC MTS 22 August 2019
(c) 1056 m ि ा 0.5 mm और लंबाई 42 32 मी
(Morning)
(d) 528 m
(a) 144 π वाले एक ता े के तार को िपघलाकर
(b) 128 π ि ा R सेमी वाले एक गोले का प
Q107. The sides of a triangular
(c) 124 π िदया जाता है | R का मान ा है ?
park are 200 m, 210 m and 290
SSC MTS 22 August 2019
Pi

m. The area of the park (in (d) 108 π


(Afternoon)
hectares) is:
Q110. The height of a cone is (a) 3
िकसी ि भुजाकार उ ान की भुजाएँ
equal to its base radius and its (b) 2
200 मी, 210 मी और 290 मी की ह |
volume is 72 π cm3 . What is its (c) 1.5
इस उ ान का े फल ( हे े यर म )
curved surface area in cm2 ? (d) 1.8
है :
SSC MTS 22 August 2019 एक शंकु की ऊंचाई इसके आधार
Q113. The diameter of the base of
(Morning) की ि ा के बराबर है तथा इसका
a right circular cone is 10 cm and
(a) 2.4 आयतन 72 π घन सेमी है | इसका
its height is 12 cm. What is the
(c) 2.1 व पृ े फल ( घन सेमी म ) ात
total surface area (in cm2 ) of the
(d) 2.9 कर |
cone?
SSC MTS 22 August 2019
एक ल वृ ीय शंकु के आधार का
Q108. A reservoir is in the shape (Afternoon)
ास 10 सेमी है तथा इसकी ऊंचाई
of a (a) 72√2π

www.ssccglpinnacle.com support@ssccglpinnacle.com Ph. 09729327755, 09817390373 516


/
s
se
Days 54-60 Mensuration / े िमित

s
la
_c
ob
bo
ah
12 सेमी है | इस शंकु का कुल पृ
sm
length of the longest stick that
ur
yo

े फल ( वग सेमी म ) िकतना है ? can be fitted inside the cuboid ? Q1. The radius of a circular
e/
t.m
://

SSC MTS 22 August 2019 एक खोखले घनाभ का कुल पृ garden is 42 m. The distance (in
tp
ht

(Evening) े फल 340 वग सेमी है | यिद घनाभ m) covered by running 8 rounds


(a) 90 π की लंबाई और चौड़ाई 10 सेमी तथा around it, is: (Take Π = 22
7 )
(b) 70 π 8 सेमी है , तो उस सबसे लंबी छड़ी एक वृ ाकार उ ान की ि ा 42
(c) 84 π की लंबाई ात कर िजसे इस घनाभ मीटर है | इसके 8 च र लगाने पर
(d) 65 π म रखा जा सकता है ? िकतनी दू री तय की जाएगी ? (
SSC MTS 2 August 2019 Π = 22
7 )
Q114. The areas of three adjacent (Evening) SSC CGL 3 March 2020
faces of a cuboid are 18 cm2 , 20 (a) 10 cm (Morning)
cm2 and 40 cm2 . What is the (b) 4 √41 cm (a) 1124
volume (in cm3 ) of the cuboid? (c) 3 √21 cm (b) 4262
एक घनाभ के तीन सि कट फलकों (d) 21 cm (c) 2112

e
के े फल 18 cm2 , 20 cm2 और (d) 3248
40 cm2 ह | इस घनाभ का आयतन ( Q117. The diameter of a right
घन सेमी म ) ात कर | circular cylinder is decreased to Q2. If the base radius of 2
SSC MTS 22 August 2019 one third of its initial value. If the cylinders are in the ratio 3:4 and

l
(Evening) volume of the cylinder remains their heights are in the ratio of
(a) 144 ac the same, then the height 4:9, then the ratio of their
(b) 100 becomes how many times of the volumes is:
(c) 120 initial height ? यिद दो बेलनों के आधार की ि ा
(d) 125 एक ल वृ ीय बेलन के ास को का अनुपात 3 : 4 है तथा उनकी
इसके आरं िभक मान से एक ितहाई ऊंचाई का अनुपात 4 : 9 है , तो उनके
Q115. A wire encloses an area of कम कर िदया जाता है | यिद बेलन आयतन का अनुपात ा होगा ?
616 cm2 when it is bent in the का आयतन समान बना रहता है , तो SSC CGL 3 March 2020
form of a circle. If the wire is इसकी ऊंचाई अपने आरं िभक मान (Morning)
nn
bent in the form of a square. Then से िकतना गुना हो जाएगी ? (a) 2:1
its area (in cm2 ) is very nearly SSC MTS 5 August 2019 (b) 1:4
equal to: (Take π = 22 (Morning) (c) 1:2
7 )
(a) 1 (d) 4:1
एक तार 616 वग सेमी का े घेरता
है जब इसे वृ के आकार म मोड़ (b) 9
(c) 6 Q3. If the length of a rectangle is
िदया जाता है | यिद इस तार को वग
(d) 3 increased by 40% and the breadth
के आकार म मोड़ा जाए, तो इसका
is decreased by 20%, then the
े फल ( वग सेमी म ) िनकटतम
Pi

Q118. How many cubes with a area of the rectangle is increased


प म िकतना होगा ? (Take
side 10 cm can be cut out of a by x%. The value of x is:
π = 22
7 )
cube having a side of 10 metre ? यिद िकसी आयत की लंबाई 40%
SSC MTS 22 August 2019
10 मीटर भुजा वाले एक घन म से 10 बढ़ा दी जाती है तथा चौड़ाई 20%
(Evening)
सेमी भुजा वाले िकतने घन काटे जा कम कर दी जाती है , तो इस आयत
(a) 400
सकते ह ? का े फल x % बढ़ जाता है | x का
(b) 576
SSC MTS 13 August 2019 मान है :
(c) 441
(Evening) SSC CGL 3 March 2020
(d) 484
(a) 10,000 (Morning)
(b) 1,00,00,000 (a) 20
Q116. The total surface area of a
(c) 1,00,000 (b) 12
hollow cuboid is 340 cm2 . If the
(d) 10,00,000 (c) 16
length and the breadth of the
(d) 8
cuboid are 10 cm and 8 cm
SSC CGL 2019 TIER I
respectively, then what is the

www.ssccglpinnacle.com support@ssccglpinnacle.com Ph. 09729327755, 09817390373 517


/
s
se
Days 54-60 Mensuration / े िमित

s
la
_c
ob
bo
ah
sm
Q4. A race track is in the shape of Q7. A cylindrical vessel of radius SSC CGL 5 March 2020
ur
yo

a ring whose inner and outer 30cm and height 42 cm is full of (Morning)
e/
t.m
://

circumferences are 440m and water. Its contents are emptied (a) (a + b)2
tp
ht

506m, respectively. What is the into a rectangular tub of length (b) 2 (a + b)2
cost of levelling the track at 75cm and breadth 44cm. The
(c) 4(a + b)2
₹6/m2?(Take Π = 22 7 ) height(in cm) to which the water
(d) 8(a + b)2
एक धावन पथ छ े के आकार म है rises in the tub is: (take Π = 22
7 )
िजसकी आतं रक और बाहरी प रिध ि ा 30 सेमी तथा ऊंचाई 42 सेमी
Q10. If the radius of a right
मशः 440 मीटर तथा 506 मीटर है | वाला एक बेलनाकार बतन पानी से
circular cylinder is decreased by
इस पथ को 6 पये ित वग मीटर पूणतः भरा आ है | इसकी साम ी
10% and height is increased by
की दर से समतल करने की लागत को एक आयताकार टब म उड़े ल
20%, then the percentage
िकतनी आएगी ? िदया जाता है िजसकी लंबाई 75 सेमी
increase/ decrease in its volume
SSC CGL 3 March 2020 तथा चौड़ाई 44 सेमी है | टब म पानी
is:
(Afternoon) का र िकतनी ऊंचाई ( सेमी म )
यिद एक ल वृ ीय बेलन की ि ा

e
(a) ₹18,966 तक बढ़े गा ?
10% कम कर दी जाए तथा ऊंचाई
(b) ₹24,832 SSC CGL 4 March 2020
20% बढ़ा दी जाए, तो इसके आयतन
(c) ₹19,866 (Afternoon)
म िकतने ितशत की वृ या कमी
(d) ₹29,799 (a) 36
होगी ?

l
(b) 30
SSC CGL 5 March 2020
Q5. The curved surface area of a
ac (c) 40
(Afternoon)
hemisphere with radius 7cm is: (d) 45
(a) decrease by 1.8%/ 1.8% कमी
7 सेमी ि ा वाले एक अधगोले का
(b) increase by 2.8%/ 2.8% वृ
व पृ े फल िकतना होगा ? Q8. The area of field in shape of a
(c) increase by 1.8%/ 1.8% वृ
SSC CGL 3 March 2020 regular hexagon is 2400 √3 m2. (d) decrease by 2.8%/ 2.8% कमी
(Evening) The cost of fencing the field at
(a) 308 cm2 ₹16.80 metre is: Q11. The perimeter of a square is
(b) 616 cm2 सम षट् भुज के आकार वाले एक 64 cm. Its area will be:
nn
(c) 462 cm2 मैदान का े फल 2400 √3 वग एक वग का प रमाप 64 सेमी है |
(d) 385 cm2 इसका े फल िकतना होगा ?
मीटर है | 16.80 पये ित मीटर की
दर से इस मैदान पर बाड़ लगाने की SSC CGL 5 March 2020
Q6. The circumference of the (Evening)
लागत ात कीिजए |
base of a conical tent is 66 m. If (a) 32 cm2
SSC CGL 4 March 2020 (b) 128 cm2
the height of the tent is 36 m,
(Evening) (c) 8 cm2
what is the area (in m2) of the
(a) ₹4,536 (d) 256 cm2
canvas used in making the tent?
(b) ₹3,024
(Take Π = 22
Pi

7 )/ (c) ₹4,032 Q12. A metallic sphere of


एक शं ाकार तंबू के आधार की (d) ₹3,528 diameter 40 cm is melted into a
प रिध 66 मीटर है | यिद तंबू की smaller sphere of radius 0.5 cm.
ऊंचाई 36 मीटर है , तो उस ितरपाल Q9. The diagonal of a square A is How many such small balls can
का े फल ात कर, िजसका योग (a+b) units. What is the area (in be made?
तंबू के िनमाण म िकया गया है | ( Π = square units) of the square drawn 40 सेमी ास वाले धातु के एक गोले
22 )
7 on the diagonal of square B को 0.5 सेमी ि ा वाले एक छोटे
SSC CGL 4 March 2020 whose area is twice the area of गोले के प म िपघलाया जाता है |
(Morning) square A? ऐसी िकतनी छोटी गद बनाई जा
(a) 1155 एक वग A का िवकण (a+b) इकाई सकती ह ?
(b) 1237.5 है | वग B, िजसका े फल वग A के SSC CGL 6 March 2020
(c) 1171.5 े फल से दोगुना है , उसके िवकण (Morning)
(d) 1254 पर खींचे गए वग का े फल ( वग (a) 64,000
इकाई म ) िकतना होगा ? (b) 32,000

www.ssccglpinnacle.com support@ssccglpinnacle.com Ph. 09729327755, 09817390373 518


/
s
se
Days 54-60 Mensuration / े िमित

s
la
_c
ob
bo
ah
यिद िकसी आयत का प रमाप 50
sm
(c) 3200
ur
yo

(d) 6400 Q16. The area of four walls of a इकाई है तथा इसका े फल 150
e/
t.m
://

room having length 6 m, breadth वग इकाई है , तो इसकी छोटी भुजा


tp
ht

Q13. A circular disc of area 0.64 4 m and height 4 m, is: की लंबाई िकतनी इकाई है ?
Π m2 rolls down a length of 1.408 6 मीटर लंबाई, 4 मीटर चौड़ाई तथा 4 SSC CGL 7 March 2020
km. The number of revolutions it मीटर ऊंचाई वाले कमरे की चार (Afternoon)
make is: दीवारों का े फल िकतना होगा ? (a) 9
एक वृ ाकार िड , िजसका SSC CGL 7 March 2020 (b) 12
े फल 0.64 Π वग मीटर है , 1.408 (Morning) (c) 10
िकमी की लंबाई तक घूमती है | (a) 50 m2 (d) 15
इसके ारा लगाए गए च रों की (b) 60 m2
सं ा है : (c) 80 m2 Q20. What is the area of a
(Take Π = 22 7 )
(d) 40 m2 triangle whose sides are 3 cm, 4
SSC CGL 6 March 2020 cm and 5 cm?

e
(Morning) Q17. The perimeter of an उस ि भुज का े फल ात कर
(a) 280 isosceles triangle is 50 cm. If the िजसकी भुजाएँ 3 सेमी, 4 सेमी तथा 5
(b) 360 base is 18 cm, then find the सेमी की ह |
(c) 140 length of the equal sides. SSC CGL 7 March 2020

l
(d) 180 एक समि बा ि भुज का प रमाप (Evening)
ac 50 सेमी है | यिद इसका आधार 18 (a) 2 √6 cm2
Q14. The length of the diagonals सेमी है , तो बराबर भुजाओं की लंबाई (b) 2 √3 cm2
of a rhombus are 16 cm and 12 ात कर |
(c) 3 cm2
cm. Its area is: SSC CGL 7 March 2020
(d) 6 cm2
एक समचतुभुज के िवकण की लंबाई (Afternoon)
16 सेमी और 12 सेमी है | इसका (a) 18 cm
Q21. Find the area and
े फल होगा : (b) 25 cm
circumference of a circle if the
SSC CGL 6 March 2020 (c) 16 cm
radius is 14 cm. (Take Π = 22
7 )
(d) 32 cm
nn
(Afternoon) एक वृ का े फल तथा प रिध ात
(a) 28 cm2 कर यिद ि ा 14 सेमी है | ( Π =
(b) 96 cm2 Q18. If radius of a circle is 22 )
(c) 69 cm2 decreased by 11%, then the total 7

decrease in the area of the circle SSC CGL 7 March 2020


(d) 48 cm2
is given as: (Evening)
यिद िकसी वृ की ि ा 11% कम (a) Area/ े फल = 616 cm2,
Q15. The inner and outer radius
कर दी जाए, तो वृ के े फल म Circumference/ प रिध = 88 cm
of a circular track are,
िकतने ितशत की कमी होगी ? (b) Area / े फल = 308 cm2,
Pi

respectively, 29 m and 23 m. The


SSC CGL 7 March 2020 Circumference/प रिध = 44 cm
cost of levelling the track at
(Afternoon) (c) Area/ े फल = 44 cm2,
₹7/m2 is:
(a) 19.50% Circumference/प रिध = 308 cm
एक वृ ाकार पथ की आतं रक तथा
(b) 20.79% (d) Area/ े फल = 88 cm2,
बाहरी ि ा मशः 29 मीटर तथा
(c) 20.50% Circumference/प रिध = 616 cm
23 मीटर है | 7 पये ित वग मीटर
की दर से इस पथ को समतल (d) 21%
Q22. Triangle PDC is drawn
करवाने की लागत िकतनी होगी ?
Q19. If the perimeter of a certain inside the square ABCD of side
SSC CGL 6 March 2020
rectangle is 50 units and its area 24 cm where P lies on AB. What
(Evening)
is 150 sq. units, then how many is the area of the triangle?
(a) ₹ 7,215
units is the length of its shorter ि भुज PDC को एक वग ABCD के
(b) ₹ 5,300
side?/ भीतर खींचा गया है िजसकी भुजा 24
(c) ₹ 6,864
सेमी है तथा P, AB पर थत है | इस
(d) ₹ 3,284
ि भुज का े फल िकतना है ?

www.ssccglpinnacle.com support@ssccglpinnacle.com Ph. 09729327755, 09817390373 519


/
s
se
Days 54-60 Mensuration / े िमित

s
la
_c
ob
bo
ah
sm
SSC CGL 9 March 2020 (b) 1600 Π Q1. The diagonal of the rectangle
ur
yo

(Morning) (c) 800 Π is 15 cm and length is 12 cm.


e/
t.m
://

(a) 298 cm2 (d) 1024 Π Find the area of the rectangle.
tp
ht

(b) 200 cm2 एक आयत का िवकण 15 सेमी तथा


(c) 288 cm2 Q26. Find the height of a cuboid लंबाई 12 सेमी है । आयत का े फल
(d) 280 cm2 whose volume is 330 cm3 and ात कीिजए।
base area is 15 cm2. CHSL 12-10-2020 (Morning
Q23. What is the area of a sector एक घनाभ की ऊंचाई ात कर shift)
of a circle of radius 14 cm and the िजसका आयतन 300 घन सेमी तथा (a) 114 cm2
central angle 45°? (Take Π = 227 ) आधार का े फल 15 वग सेमी है | (b) 108 cm2
एक वृ के खंड का े फल ात SSC CGL 9 March 2020 (c) 112 cm2
कीिजए िजसकी ि ा 14 सेमी तथा (Afternoon) (d) 116 cm2
क ीय कोण 45° है | ( Π = 22
7 ) (a) 19 cm
SSC CGL 9 March 2020 (b) 24 cm Q2.In the given figure, chords PQ

e
(Morning) (c) 22 cm and RS intersect each other at
(a) 11cm2 (d) 21 cm point L. Find the length of RL.
(b) 77cm2 दी गयी आकृित म, जीवाएँ PQ और
(c) 67cm2 Q27. A wheel covers a distance RS एक-दू सरे को िबंदु L पर

l
(d) 70cm2 of 1,100 cm in one round. The ित े द करती ह। RL की लंबाई
ac diameter of the wheel is: ात कीिजए।
Q24. The length, breadth and एक पिहया एक च र म 1,100
height of a cuboidal box are in the सेमी की दू री तय करता है | इस
7:5:3 and its whole surface area is पिहये का ास िकतना है ?
27832 cm2. Its volume is: SSC CGL 9 March 2020
एक घनाभाकार ब े की लंबाई, (Afternoon)
चौड़ाई तथा ऊंचाई 7 : 5 : 3 के (a) 175 cm
अनुपात म है तथा इसका पूण पृ (b) 100 cm
nn
े फल 27832 वग सेमी है | इसका (c) 125 cm
आयतन है : (d) 150 cm CHSL 12-10-2020 (Morning
SSC CGL 9 March 2020 shift)
(Morning) Q28. The perimeter of a square (a) 6 cm
(a) 280120 cm3 plot is the same as that of a (b) 8 cm
(b) 288120 cm3 rectangular plot with sides 35 m (c) 3 cm
(c) 208120 cm3 and 15 m. The side of the square (d) 2 cm
(d) 288100 cm3 plot is/ एक वगाकार भूखंड का
प रमाप उतना ही है िजतना प रमाप
Pi

Q3. In the given figure, O is the


Q25. The volumes of sphere A एक आयताकार भूखंड का है centre of the circle. Its two chords
and B are in the ratio 125:64. If िजसकी भुजाएँ 35 मीटर तथा 15 AB and CD intersect each other
the sum of radii A and B is 36 मीटर ह | वगाकार भूखंड की भुजा है at the point P within the circle. If
cm, then the surface area (in cm2) : AB = 15 cm, PB = 9 cm, CP = 3
of A is: SSC CGL 9 March 2020 cm, then find the length of PD.
A और B गोले का आयतन 125 : 64 (Evening) दी गयी आकृित म, O वृ का क
के अनुपात म है | यिद A और B की (a) 25 m है । इसकी दो जीवाएँ AB और CD
ि ाओं का जोड़ 36 सेमी है , तो A (b) 20 m एक-दू सरे को वृ के भीतर िबंदु P
का पृ े फल ( वग सेमी म ) (c) 100 m पर ित े द करती ह। यिद AB =
िकतना है ? (d) 50 m 15 सेमी, PB = 9 सेमी, CP = 3 सेमी
SSC CGL 9 March 2020 है , तो PD की लंबाई ात कीिजए।
(Afternoon) SSC CHSL 2019
(a) 512 Π

www.ssccglpinnacle.com support@ssccglpinnacle.com Ph. 09729327755, 09817390373 520


/
s
se
Days 54-60 Mensuration / े िमित

s
la
_c
ob
bo
ah
sm
which touches the smaller circle
ur
yo

is:
e/
t.m
://

दो संक ी वृ ों की ि ा 13 सेमी
tp
ht

तथा 5 सेमी है । बड़े वृ की जीवा की


लंबाई ात कीिजए, जो छोटे वृ को
श करती है ।
CHSL 12-10-2020 (Evening
shift)
(a) 24 cm
(b) 15 cm
CHSL 12-10-2020 (Afternoon
(c) 13 cm
CHSL 12-10-2020 (Afternoon shift)
(d) 10 cm
shift) (a) 70 π
(a) 16 cm (b) 72 π
Q9. If the three sides of a triangle
(b) 18 cm (c) 66 π

e
are 11 cm, 12 cm and 13 cm, then
(c) 20 cm (d) 68 π what is the area of the given
(d) 22 cm
triangle (in cm2 )?
Q4. Length and breadth of यिद एक ि भुज की तीन भुजाएँ 11
Q6. What is the radius of the

l
rectangular field are in the ratio 5 सेमी, 12 सेमी तथा 13 सेमी ह, तो
circle whose area is equal to the िदए गए ि भुज का े फल ( वग
: 2. If the perimeter of the field is
ac sum of the areas of two circles
238 m. Find the length of the सेमी म ) िकतना है ?
whose radii are 15 cm and 8 cm? CHSL 12-10-2020 (Evening
field. उस वृ की ि ा िकतनी है ,
एक आयताकार मैदान की लंबाई shift)
िजसका े फल दो वृ ों के े फलों
और चौड़ाई 5 : 2 के अनुपात म है । (a) 15 √13
के योग के बराबर है , िजनकी ि ाएँ
यिद मैदान का प रमाप 238 मीटर है , 15 सेमी तथा 8 सेमी ह? (b) 13√26
तो मैदान की लंबाई ात कीिजए। CHSL 12-10-2020 (Evening (c) 6√105
CHSL 12-10-2020 (Afternoon shift) (d) 17√42
nn
shift) (a) 14 cm
(a) 85 m (b) 15 cm Q10. A cylindrical vessel with
(b) 83 m (c) 13 cm radius 6 cm and height 5 cm is to
(c) 82 m (d) 17 cm be made by melting a number of
(d) 84 m
spherical metal balls of diameter
Q7. What is the distance between 2 cm. The minimum number of
Q5. In the figure, L is the centre two parallel tangents of a circle of balls needed is:
of the circle, and ML is the radius 8 cm? एक बेलनाकार बतन, िजसकी ि ा
Pi

perpendicular to LN. If the area 8 सेमी ि ा वाले वृ की दो 6 सेमी और ऊंचाई 5 सेमी है , उसका
of the triangle MLN is 36, then समानां तर श रे खाओं के बीच की िनमाण कई गोलीय धातु की गदों से
the area of the circle is: दू री िकतनी है ? िकया जाना है , िजसका ास 2 सेमी
इस आकृित म, L वृ का क है तथा CHSL 12-10-2020 (Evening है । ूनतम िकतनी गदों की
ML, LN पर ल है । यिद ि भुज shift) आव कता होगी?
MLN का े फल 36 है , तो इस वृ (a) 8 cm CHSL 13-10-2020 (Morning
का े फल ात कीिजए। (b) 4 cm Shift)
(c) 16 cm (a)125
(d) 12 cm (b)135
(c)115
Q8. Two concentric circles are of (d)105
radii 13 cm and 5 cm. The length
of the chord of the larger circle Q11. A triangle has sides 25, 39
and 34 units. If the area of a

www.ssccglpinnacle.com support@ssccglpinnacle.com Ph. 09729327755, 09817390373 521


/
s
se
Days 54-60 Mensuration / े िमित

s
la
_c
ob
bo
ah
sm
square exceeds the area of this CHSL 14-10-2020 (Afternoon
ur
yo

triangle by 21 units, then the side Q14. The radius of a circular shift)
e/
t.m

of the square is:


://

cylinder is 5 cm and its height is (a) 4 : 1


tp
ht

एक ि भुज की भुजाएँ 25, 39 और 14 cm. Then the volume of (b) √2 : 1


34 इकाई ह। यिद एक वग का
cylinder in cm3 is: (Take π = 22
7 ) (c) 3 : 1
े फल इस ि भुज के े फल से 21
इकाई अिधक है , तो वग की भुजा एक गोलाकार बेलन की ि ा5 (d) 2 : 1
ात कीिजए। सेमी है और इसकी ऊंचाई 14 सेमी
CHSL 13-10-2020 (Afternoon है । िफर बेलन का आयतन ( cm3 म) Q17. Water flows into a tank 180
Shift) है : m × 140 m through a rectangular
(a) 22 units CHSL 14-10-2020 (Morning pipe of 1.2m × 0.75m at a rate of
(b) 21 units shift) 15 km/h. In what time will the
(c) 18 units
(a) 1504 water rise by 4 m?
(d) 25 units’
(b) 1120 15 िकमी / घंटा की दर से 1.2 मीटर
Q12. In a circle two equal and (c) 1100 0.75 मीटर के आयताकार पाइप के

e
parallel chords are 6 cm apart and (d) 1254 मा म से एक टक 180 मीटर 140
lie on the opposite sides of the मीटर म पानी बहता है । िकस समय
centre of the circle.If the length of Q15. The areas of two similar म पानी 4 मीटर बढ़ जाएगा?
each chord is 8 cm, than the triangles are 324 cm2 and 225 CHSL 14-10-2020 (Afternoon

l
radius of the circle is: cm2 . If the altitude of the smaller shift)
एक वृ म दो बराबर और समा र
ac triangle is 10 cm, then the altitude (a) 6 hours 42 minutes
जीवाएँ 6 cm के दु री पर ह और वृ of the bigger triangle in (b) 7 hours 28 minutes
के क के िवपरीत भाग पर थत ह। centimetres, is: (c) 5 hours 46 minutes
यिद ेक जीवाएँ की लंबाई 8 सेमी दो समान ि भुजों के े 324 cm2 (d) 8 hours 12 minutes
है , तो वृ की ि ा है : और 225 cm2 ह। यिद छोटे ि भुज
CHSL 13-10-2020 (Evening की ऊंचाई 10 सेमी है , तो सटीमीटर Q18. The perimeter and length of
Shift) म बड़े ि भुज की ऊंचाई है : a rectangle are in the ratio of 8:1,
(a)5 cm CHSL 14-10-2020 (Morning and the area of the rectangle is
nn
(b)7 cm shift) 1323 cm2. Find the length of the
(c)3 cm (a) 12 rectangle.
(d)2 cm (b) 16 एक आयत का प रमाप और उसकी
(c) 14 लंबाई 8:1 के अनुपात म है । आयत
Q13. The length and breadth of a (d) 18 का े फल 1323 वग सेमी है ।
cuboid are increased by 10% and आयत की लंबाई ात कीिजए।
20%, respectively, and its height CHSL 14-10-2020 (Evening
is decreased by 20%. The Q16. In the given figure, the ratio shift)
Pi

percentage increase in the volume of the area of the largest square to (a) 22 cm
of the cuboid is: that of the smallest square is: (b) 23 cm
एक घनाभ की लंबाई और चौड़ाई दी गयी आकृित म, सबसे बड़े वग के (c) 25 cm
मशः 10% और 20% बढ़ जाती है , े फल और सबसे छोटा वग के (d) 21 cm
और इसकी ऊंचाई 20% तक कम हो े फल का अनुपात है
जाती है । घनाभ के आयतन म Q19. Find the area of an
ितशत वृ है : equilateral triangle whose sides
CHSL 14-10-2020 (Morning are 12 cm.
shift) उस समबा ि भुज का े फल ात
(a) 5 54 % कीिजए िजसकी भुजाएँ 12 सेमी की
(b) 5 51 % ह।
CHSL 14-10-2020 (Evening
(c) 5 52 %
shift)
(d) 5 53 % (a) 38

www.ssccglpinnacle.com support@ssccglpinnacle.com Ph. 09729327755, 09817390373 522


/
s
se
Days 54-60 Mensuration / े िमित

s
la
_c
ob
bo
ah
sm
(b) 29√5 (d)a - b (c) 84
ur
yo

(d) 21
e/

(c) 45√2
t.m
://

Q23.The radius of a circular cone


tp

(d) 36√3
ht

is 6 cm and its height is 7 cm. Q26. The ratio of the areas of two
Then the volume of cone in cm3 squares is 16 : 1. Find the ratio
Q20. Two parallel chords are
is: (take π = 22
7 )
between their perimeters.
drawn in a circle of diameter 20
एक गोलाकार शंकु की ि ा 6 cm दो वग के े फल का अनुपात 16: 1
cm. The length of one chord is 16
है और इसकी ऊंचाई 7 सेमी है । िफर है . उनके प रिध के बीच का अनुपात
cm and the distance between the
शंकु का आयतन cm3 म ा होगा ? ात कर।
two chords is 12 cm. The length
( π = 22 CHSL 15-10-2020 (Evening
of the other chord is : 7 )
shift)
एक वृ म दो समानां तर जीवाओं को CHSL 15-10-2020 (Morning
(a) 8 : 1
खींचा जाता है , िजसका ास 20 shift)
(b) 3 : 1
सेमी है । एक जीवा की लंबाई 16 सेमी (a)264
(c) 4 : 1
है तथा दोनों जीवाओं के बीच की दू री (b)188

e
(d) 12 : 1
12 सेमी है । दू सरी जीवा की लंबाई (c)216
िकतनी है ? (d)154
Q27. The area of the largest
CHSL 14-10-2020 (Evening
triangle that can be inscribed in a
shift) Q24.In a circle, two equal and

l
semicircle of radius 4 cm in
(a) 20 parallel chords are 6 cm apart and
square centimeters is
(b) 18 ac they lie on the opposite sides of
सबसे बड़ा ि भुज का े फल ात
(c) 16 the centre of the circle, whose
कीिजए िजसे 4 सेमी के ि ा वाले
(d) 12 radius is 5 cm. The length of each
अधवृ म अंिकत िकया गया है ।
chord (in cm), is
CHSL 15-10-2020 (Evening
Q21. In a circle, PQ and RS are एक वृ म, दो समान और
shift)
two diameters that are समाना र जीवा 6 सेमी के दु री पर ह
(a) 16 cm2
perpendicular to each other. Find और वे वृ के क के िवपरीत
िकनारों पर थत ह, िजसकी ि ा5 (b) 14 cm2
the length of chord PR.
(c) 12 cm2
nn
एक वृ म, PQ तथा RS दो ास ह सेमी है । ेक जीवा की लंबाई
(सेमी म) है : (d) 18 cm2
जो एक-दू सरे पर ल ह। जीवा PR
की लंबाई ात कीिजए। CHSL 15-10-2020 (Afternoon
shift) Q28. In the given figure XYZ is a
CHSL 14-10-2020 (Evening
(a)12 secant and ZT is a tangent. What
shift)
PQ (b)6 is the value of x.
(a) 2
(c)8 िदए गए आकृित म XYZ एक छे दक
(b) √2 PQ रे खा है और ZT एक शरे खा है । x
(d)10
(c) 2PQ का मान ात कीिजए।
Pi

PQ
(d) Q25. The breadth of a rectangle is
√2
four times of its length. If the area
Q22.The area of a rectangle is a2 of the rectangle is 1764, then
- b2 and its length is a + b, what what is the breadth of the
will be its breadth? rectangle?
एक आयत का े फल a2 - b2 है एक आयत की चौड़ाई इसकी लंबाई
का चार गुना है । यिद आयत का
और इसकी लंबाई a + b है , इसकी CHSL 15-10-2020 (Evening
े फल 1764 है , तो आयत की
चौड़ाई ा होगी? shift)
चौड़ाई ा है ?
CHSL 15-10-2020 (Morning (a) 9
CHSL 15-10-2020 (Afternoon
shift) (b) 5
shift)
(a)2ab (c) 8
(a) 44
(b)a + b (d) 7
(b) 56
(c)ab

www.ssccglpinnacle.com support@ssccglpinnacle.com Ph. 09729327755, 09817390373 523


/
s
se
Days 54-60 Mensuration / े िमित

s
la
_c
ob
bo
ah
एक वग पाक का े फल 16x2 + 8x
sm
Q29. The area of a square field is (a) 5√5 cm
ur
yo

7200 m2. What is the length of its + 1. पाक की लंबाई िकतनी है ?


e/

(b) 5√3 cm
t.m
://

diagonal? CHSL 16-10-2020 (Afternoon


tp

(c) 5√2 cm
ht

एक वग े का े फल 7200m2 है । shift)
इसके िवकण की लंबाई ा है ? (a) (4x - 1) units (d) 3√3 cm
CHSL 16-10-2020 (Morning (b) (4x + 1) units
shift) (c) (4x + 1)2 units Q36. What will be the area of a
(a) 60 m (d) 4x units circle whose radius is √5 cm?
(b) 1800 m एक वृ का े फल ा होगा
(c) 180 m Q33. What is the area of an िजसकी ि ा √5 सेमी है ?
(d) 120 m equilateral triangle of side 4 √3 CHSL 16-10-2020 (Evening
cm? shift)
Q30. The volume of a tank is 72 43 सेमी के समबा ि भुज का (a) 3π cm2
cubic meters. Water is poured into े फल िकतना है ? (b) π cm2

e
it at the rate of 60 litres per CHSL 16-10-2020 (Evening (c) 2π cm2
minute. How much time will it shift) (d) 5π cm2
take to fill the tank? (a) 15√3 cm2
एक टक की आयतन 72 घन मीटर
(b) 16√3cm2 Q.37.If the area of the triangle

l
है । इसम 60 लीटर ित िमनट की दर
(c) 20√ 3cm2 DGC is 20cm2, then the area of
से पानी डाला जाता है । टक को भरने
ac (d) 12√3cm2 triangle AGF + the area of
म िकतना समय लगेगा?
triangle BGF is equal to:
CHSL 16-10-2020 (Afternoon
ि भुज ABC म, AD, BE और CF
shift) Q34. If two concentric circles are
म रे खा ह और G ि भुज का
(a) 6 hours of radii 13 cm and 12 cm,
के क है । यिद ि भुज DGC का
(b) 20 hours respectively, then the length of
े फल 20cm2 है , तो ि भुज AGF
(c) 12 hours the chord of the larger circle
का े फल+ ि भुज BGF का
(d) 10 hours which touches the smaller circle
े फल िकसके बराबर है :
Q31. In a circle of radius 10 cm, is:
nn
CHSL 19-10-2020 (Morning
PQ and RS are two parallel यिद दो संकि त वृ मशः ि ा
shift)
chords of length 16 cm and 12 cm 13 सेमी और 12 सेमी के ह, तो बड़े
(a) 30 cm2
respectively. What is the distance वृ के जीवा की लंबाई जो छोटे वृ
(b) 20 cm2
between the chords if they are on को श करती है वह है :
(c) 25 cm2
opposite sides of the centre? CHSL 16-10-2020 (Evening
(d) 40 cm2
ि ा 10 सेमी के एक वृ म, PQ shift)
और RS मशः 16 सेमी और 12 (a) 35 cm
Q38. A wire in the shape of a
सेमी लंबाई के दो समानां तर जीवा ह। (b) 10 cm
Pi

circle of radius 28 cm is bent in


यिद वे क के िवपरीत िकनारों पर ह, (c) 15 cm
the form of a square. What is the
तो जीवा के बीच की दू री ा है ? (d) 25 cm
difference of their areas? (Take
CHSL 16-10-2020 (Afternoon
Π = 227 )
shift) Q35. The radius of a circle is 15
(a) 6 cm cm and the length of one chord of 28 सेमी ि ा के वृ के आकार का
(b) 8 cm the circle is 20 cm. What is the एक तार एक वग के प म मुड़ा
(c) 14 cm distance of the chord from the आ है । उनके े ों म ा अंतर है ?
(d) 2 cm centre of the circle? ( Π = 227 )
एक वृ की ि ा 15 सेमी है और CHSL 19-10-2020 (Morning
Q32. The area of a square park is वृ के जीवा की लंबाई 20 सेमी है । shift)
16x2 + 8x + 1. What is the length वृ के क से कॉड की दू री ा है ? (a) 530 cm2
of the park? CHSL 16-10-2020 (Evening (b) 532 cm2
shift) (c) 538 cm2
(d) 528 cm2

www.ssccglpinnacle.com support@ssccglpinnacle.com Ph. 09729327755, 09817390373 524


/
s
se
Days 54-60 Mensuration / े िमित

s
la
_c
ob
bo
ah
sm
(b) ₹ 1800 AC = 15 cm and DE = 3 cm, then
ur
yo

Q39. The radii of two circles are (c) ₹ 1600 the length of AE is:
e/
t.m
://

20 cm and 13 cm, respectively. (d) ₹ 2400 िदए गए आकृित म, AB, AC और


tp
ht

Find the radius of the circle EF एक वृ के शरे खा ह। यिद


which has a circumference equal Q42. The diameter of a wheel is AC = 15 सेमी और DE = 3 सेमी है ,
to the sum of the circumference 49 cm. The number of revolutions तो AE की लंबाई है :
of two circles. in which it will have to cover a
दो वृ की ि ा मशः 20 सेमी distance of 770 m, is:
और 13 सेमी है । उस वृ की ि ा एक पिहया का ास 49 सेमी है ।
ात कीिजए िजसम दो वृ की प रिध च र की सं ा िजसम इसे 770
के योग के बराबर एक प रिध है । मीटर की दू री तय करनी होगी, वह
CHSL 19-10-2020 (Morning है :
shift) CHSL 19-10-2020 (Afternoon
(a) 33 cm shift)

e
(b) 32 cm (a) 400
(c) 30 cm (b) 600
(d) 28 cm (c) 700 CHSL 19-10-2020 (Evening
(d) 500 shift)
(a) 24 cm

l
Q40. A triangle PQR is a right
angled triangle at Q. E and F are Q43. The centroid of an (b) 9 cm
ac
the mid points of QR and PR
respectively. What will be the
ratio of the area of quadrilateral
PQEF to the area of triangle PQR.
एक समकोण ि भुज PQR, E पर
equilateral Δ XYZ is L. If XY =
12 cm, then the length of XL (in
cm), is:
एक समबा XYZ का के क L है ।
यिद XY = 12 सेमी है , तो XL की
(c) 18 cm
(d) 12 cm

Q46. A circle is drawn


circumscribing a rectangle of
समकोण है E और F मशः QR sides 24 cm and 7 cm. Find the
लंबाई (सेमी म) है ,
और PR के म िबंदु ह। चतुभुज area of the circle. (Take
CHSL 19-10-2020 (Afternoon
PQEF के े फल और ि भुज PQR π = 3.14)
shift)
nn
के े फल का अनुपात ा होगा एक वृ 24 cm और 7 cm भुजा वाले
(a) 5 √3
CHSL 19-10-2020 (Morning आयत के प रगत है । वृ का े फल
(b) 4 √3 ात कीिजए।( π = 3.14)
shift)
(c) 2 √3 CHSL 19-10-2020 (Evening
(a) 2/3
(b) 3/4 (d) 3 √3 shift)
(c) 23 (a) 490.625 cm2
(d) 34 Q44. The sum of the squares of (b) 420.545 cm2
the sides of a rhombus is 900 m2 . (c) 397.982 cm2
Pi

Q41. The breadth of a rectangular What is the side of the rhombus? (d) 478.967 cm2
field is two-third of its length. If एक समभुज की भुजाओं के वग का
its area is 864 m2, then find the योग 900 m2 है । समभुज का भुजा Q47. Three sides of a triangle
cost of fencing it all around at ₹ ा है ? measure 6 cm, 10 cm and x cm.
15/m. CHSL 19-10-2020 (Afternoon The minimum integral value of x
एक आयताकार े की चौड़ाई shift) is:
इसकी लंबाई का दो-ितहाई है । यिद (a) 17 m ि भुज की तीन भुजाएँ 6 सेमी, 10
इसका े फल 864 m2 है ,तो ₹ 15/m (b) 15 m सेमी और x सेमी मापती ह। X का
के दर से चारों ओर बाड़ लगाने की (c) 14 m ूनतम मान है
लागत का पता लगाएं (d) 16 m CHSL 19-10-2020 (Evening
CHSL 19-10-2020 (Morning shift)
shift) Q45. In the given figure, AB, AC (a) 2
(a) ₹ 2000 and EF are tangents to a circle. If (b) 1
(c) 3

www.ssccglpinnacle.com support@ssccglpinnacle.com Ph. 09729327755, 09817390373 525


/
s
se
Days 54-60 Mensuration / े िमित

s
la
_c
ob
bo
ah
sm
(d) 5 the area of triangle BDE is equal cm and 44 cm respectively. The
ur
yo

to: width of the ring is:


e/
t.m
://

Q48 A is a point at a distance 26 ABC और BDE दो समबा ि भुज दो संकि त वृ एक वलय का


tp
ht

cm from the centre O of a circle ह जैसे D, BC का म -िबंदु है । यिद िनमाण करते ह। वलय की आं त रक
of radius 10 cm. AP and AQ are ि भुज ABC का े फल 136 cm2 , और बाहरी प रिध मशः 22 सेमी
the tangents to the circle at the है , तो ि भुज BDE का े फल ात और 44 सेमी है । वलय की चौड़ाई
point of contacts P and Q. If a करे ात करI
tangent BC is drawn at a point R CHSL 20-10-2020 (Morning CHSL 20-10-2020 (afternoon
lying on the minor are PQ to shift) shift)
intersect AP at B and AQ at C, (a) 36 cm2 (a) 3.5 cm
then the perimeter of Δ ABC is: (b) 34 cm2 (b) 1.5 cm
ि ा 10 सेमी के वृ के क O से (c) 38 cm2 (c) 2.5 cm
26 सेमी की दू री पर A एक िबंदु है । (d) 24 cm2 (d) 3 cm
AP और AQ िबंदु P और Q पर Q51. In an isosceles triangle ABC

e
शरे खा ह। यिद शरे खा BC वृ with AB = AC and AD is Q.54. A, B and C are three points
के PQ माइनर को R पर कटती है perpendicular to BC, if AD = 6 on the circle. If AB = AC = 7√2
और PA को B पर और PQ को C पर cm and the perimeter of Δ ABC cm and ∠BAC = 90°, then the
ितछे िदत करती है , तो ABC की is 36 cm, then the area of Δ ABC radius is equal to:

l
प रिध है is: वृ पर A, B और C तीन िबंदु ह।
CHSL 20-10-2020 (Morning
ac एक समि बा ि भुज ABC म AB = यिद AB = AC = 72cm और ∠BAC
shift) AC और AD, BC पर लंबवत है , = 90° है , तो ि ा है
(a) 40 cm यिद AD = 6 सेमी और ABC की CHSL 20-10-2020 (afternoon
(b) 48 cm प रिध 36 सेमी है , तो ABC का shift)
(c) 46 cm े फल है (a) 14
(d) 42 cm CHSL 20-10-2020 (Morning (b) 7
shift) (c) 7√2
Q49. ABC is a right angled (a) 45 cm2 (d) 6
triangle, right angled at A. A
nn
(b) 48 cm2
circle is inscribed in it. The
(c) 54 cm2 Q.55. The centroid of an
lengths of two sides containing equilateral triangle PQR is L. If
(d) 64 cm2
the right angle are 48 cm and 14 PQ = 18 cm, then the length of
cm. The radius of the inscribed PL is:
Q52. If 3.96 cubic dm of lead is
circle is: एक समबा ि भुज PQR का के क
to be drawn in to a cylindrical
ABC एक समकोण ि भुज है , A L है । यिद PQ = 18 सेमी है , तो PL
wire of diameter 0.6 cm, then the
कोण पर समकोण है । समकोण वाले की लंबाई ात कर
length of the wire (in metres), is:
दो भुजाओं की लंबाई 48 सेमी और
Pi

यिद सीसा 3.96 घन डे सीमीटर है CHSL 20-10-2020 (Evening


14 सेमी है । अ िनिहत वृ की shift)
िजससे एक 0.6 सेमी ास के
ि ा है
बेलनाकार तार को बनाना है , तो तार (a) 3√3
CHSL 20-10-2020 (Morning
की लंबाई (मीटर म) है : (b) 5√3
shift)
CHSL 20-10-2020 (Morning
(a) 4 cm (c) 6√3
shift)
(b) 6 cm (d) 4√3
(a) 130 m
(c) 8 cm
(b) 125 m
(d) 5 cm Q.56. Two circles with centres P
(c) 140 m
(d) 120 m and Q of radii 7 cm and 3cm,
Q50. ABC and BDE are two respectively, touch each other
equilateral triangles such that D is externally at a point A. BC is a
Q.53. Two concentric circles form
the mid-point of BC. If the area direct common tangent to these
a ring. The inner and outer
of triangle ABC is 136 cm2 , then two circles where B and C are the
circumference of the ring are 22

www.ssccglpinnacle.com support@ssccglpinnacle.com Ph. 09729327755, 09817390373 526


/
s
se
Days 54-60 Mensuration / े िमित

s
la
_c
ob
bo
ah
Δ ABC म, AB = AC = 13 सेमी के
sm
point on the circles respectively. Q.59. AB is the diameter of a
ur
yo

The length of BC is: circle. C is a point on a tangent साथ एक समि बा ि भुज है । AD,
e/
t.m
://

क P और Q के साथ दो दो वृ , drawn on A. If AB = 24cm and A से BC पर मा का इस कार है


tp
ht

िजनकी ि ाएँ मशः 7 सेमी और AC = 7cm, then what is the की AD = 12 सेमी। BC की लंबाई
3 सेमी है , एक दू सरे को बाहरी प length of BC? है :
से श करते ह। BC इन दो वृ म AB एक वृ का ास है । C, A से CHSL 21-10-2020 (Morning
एक सीधा शरे खा है जहां मशः B खींची गई शरे खा पर एक िबंदु shift)
और C वृ पर िबंदु ह। BC की है ,यिद AB = 24 cm और AC = 7 (a) 5 cm
लंबाई है cm तो BC की लंबाई ा है (b) 7.5 cm
CHSL 20-10-2020 (Evening CHSL 21-10-2020 (Morning (c) 10 cm
shift) shift) (d) 6 cm
(a) 3 √21 cm (a) 50 cm
(b) 15 cm Q.62. Two concentric circles form
(b) √21 cm (c) 25 cm a ring. The inner and outer
(c) 2√21 cm

e
(d) 26 cm circumference of the ring are 22
(d) 4√21 cm cm and 44 cm respectively. The
Q.60. In the given figure, TB is a width of the ring is:
Q.57. If the breadth and perimeter chord which passes through the दो संकि त वृ एक वलय का
of a rectangle are in the ratio 1:8

l
centre of the circle. PT is a िनमाण करते ह। वलय की आं त रक
and the area of rectangle is 363 tangent to the circle at the point. और बाहरी प रिध मशः 22 सेमी

rectangle is:
ac
cm3 , then the breadth of the

यिद आयत की चौड़ाई और प रिध 1:


8 के अनुपात म है और आयत का
े फल 363 cm3 है , तो आयत की
T on the circle. If PT = 10 cm, PA
= 5 cm and AB = x cm, then the
radius of the circle is:
िदए गए िच म, TB एक जीवा है जो
वृ के क से होकर गुजरता है । PT,
और 44 सेमी है । वलय की चौड़ाई
ात करे
CHSL 21-10-2020 (Afternoon
shift)
(a) 3.5 cm
चौड़ाई है वृ के िबंदु T पर वृ की शरे खा (b) 1.5 cm
CHSL 20-10-2020 (Evening है । यिद PT= 10 सेमी, PA= 5 सेमी (c) 2.5 cm
shift) और AB = x सेमी, तो वृ का ि ा (d) 3 cm
nn
(a) 11 cm है
(b) 12 cm Q.63. In ΔABC , E and D are
(c) 10 cm points on sides AB and AC,
(d) 13 cm respectively, such that ∠ABC =
∠ADE, if AE = 6cm, AD = 4 cm
Q.58. The perimeter of a and EB = 4 cm, then the length of
rectangle is 80 cm and its area is DC is:
375 cm2 . What is the difference ΔABC म, E और D मश AB
Pi

between the length and the CHSL 21-10-2020 (Morning और AC पर एक िबंदु ह, जैसे िक
breadth of the rectangle? shift) ∠ABC = ∠ADE, यिदAE = 6cm,
एक आयत की प रिध 80 सेमी है और (a) 5√3 AD = 4 cm और EB = 4 cm है , तो
इसका े फल 375 cm2 है । आयत (b) 6√5 DC की लंबाई है :
की लंबाई और चौड़ाई म ा अंतर CHSL 21-10-2020 (Afternoon
(c) 3√5
है ? shift)
CHSL 21-10-2020 (Morning (d) 10√3
(a) 11 cm
shift) (b) 8 cm
(a) 12 cm Q.61. Δ ABC is an isosceles
(c) 9.5 cm
(b) 20 cm triangle with AB = AC=13 cm. (d) 10 cm
(c) 10 cm AD is the median on BC from A
(d) 16 cm such that AD= 12 cm. The length Q.64. A, B and C are three points
of BC is equal to:
on the circle. If AB = AC = 7√2

www.ssccglpinnacle.com support@ssccglpinnacle.com Ph. 09729327755, 09817390373 527


/
s
se
Days 54-60 Mensuration / े िमित

s
la
_c
ob
bo
ah
AB, OQ ⊥ CD और AB, CD के
sm
cm and ∠BAC = 90°, then the cm, BP = 28 cm and CD = 25 cm,
ur
yo

radius is equal to: समा र है । यिद AB= 40 सेमी और then what is the radius of the
e/
t.m
://

वृ पर A, B और C तीन िबंदु ह। CD = 42 सेमी है , तो PQ की लंबाई circle is:


tp
ht

यिद AB = AC = 72cm और ∠BAC है ABCD एक चतुभुज है जैसे िक ∠D


= 90 then है , तो ि ा का मान ात = 90°। क O वाला एक वृ मशः
करे AB, BC और CD और DA को P,
CHSL 21-10-2020 (Afternoon Q, R और S से छूता है । यिद BC =
shift) 40 सेमी, BP = 28 सेमी और CD =
(a) 14 25 सेमी है , तो वृ का ि ा ा है
(b) 7
(c) 7√2 CHSL 21-10-2020 (Evening
(d) 6 shift)
(a) 32
Q.65. A chord 10 cm long is (b) 20

e
drawn in a circle of diameter 26 (c) 41
cm. The perpendicular distance of (d) 21
the chord from the centre is: CHSL 26-10-2020 (Morning
एक तार 10 सेमी लंबी, ास 26 सेमी Q.68. In a Δ ABC, 2 ∠ABC = 9 ∠ shift)

l
के एक वृ म खींचा जाता है । क से (a) 12
ACB and 2 ∠ BAC = 7 ∠ ACB. If
जीवा की लंबवत दू री है
ac (b) 13
AB = 8 cm, AC = 17 cm, then the
CHSL 21-10-2020 (Evening (c) 5
length of BC is:
shift) (d) 8
Δ ABC म, 2 ∠ABC = 9 ∠ ACB
(a) 12 cm Q71 The perimeter of a
और 2 ∠ BAC = 7 ∠ ACB । यिद
(b) 7 cm rectangular field is 32 meters and
AB = 8 सेमी, AC = 17 सेमी है , तो
(c) 5 cm its sides are in the ratio 5:3, Then
BC की लंबाई है :
(d) 8 cm the sides of the field are:.
CHSL 26-10-2020 (Morning
एक आयताकार मैदान की प रमाप
shift)
32 मीटर है और इसके भुजा 5: 3 के
nn
Q.66. The ratio of the length and (a) 8
the perimeter of a rectangle is 2:7. अनुपात म ह, िफर मैदान के भुजाएँ
(b) 25
What is the ratio of the length and ह
(c) 15
breadth of the rectangle? CHSL 26-10-2020 (afternoon
(d) 9
एक आयत की लंबाई और प रिध का shift)
अनुपात 2: 7 है । आयत की लंबाई (a) 10 m and 6 m
Q.69. The total surface area of a
और चौड़ाई का अनुपात ा है (b) 9 m and 7 m
solid cube is 2400 cm2 . The
CHSL 21-10-2020 (Evening (c) 12 m and 10 m
volume of the cube is:
(d) 5 m and 3 m
Pi

shift)
एक ठोस घन का कुल े फल 2400
(a) 4:3
cm2 है । घन की आयतन है
(b) 4:5 Q.72. In a circle centred at O, a
CHSL 26-10-2020 (Morning tangent AP is drawn from an
(c) 5:4
shift) external point A. If OA = 13 cm
(d) 5:3
(a) 6000 cm3 and OP = 5cm, then the length of
(b) 8200 cm3 tangent AP is:
Q.67. In the figure, O is the
(c) 8400 cm3 O क वाले एक वृ म, एक श
centre of the circle of radius 29
(d) 8000 cm3 रे खा AP को बाहरी िबंदु A से खींचा
cm, OP ⊥ AB, OQ ⊥ CD and AB
is parallel to CD. If AB = 40 cm जाता है । यिद OA = 13 सेमी और
Q.70. ABCD is a quadrilateral OP = 5 सेमी है , तो श रे खा AP
and CD = 42 cm, then the length
such that ∠D = 90°. A circle with की लंबाई ात कर
of PQ is:
दी गयी आकृित म, O वृ का क है centre O touches the sides AB, CHSL 26-10-2020 (afternoon
िजसकी ि ा 29 सेमी है , OP ⊥ BC and CD and DA at P, Q, R shift)
and S respectively. If BC = 40

www.ssccglpinnacle.com support@ssccglpinnacle.com Ph. 09729327755, 09817390373 528


/
s
se
Days 54-60 Mensuration / े िमित

s
la
_c
ob
bo
ah
sm
(a) 10 cm (b) 124 (b) 20 cm
ur
yo

(b) 8 cm (c) 120 (c) 18 cm


e/
t.m
://

(c) 18 cm (d) 125 (d) 22 cm


tp
ht

(d) 12 cm
Q.76. Two tangents PA and PB Q.79. A 5 cm long perpendicular
Q.73. If the diagonal of the cube are drawn from an external point is drawn from the centre of a
is√27 cm, then its volume is: P to a circle with centre O at the circle to a 24 cm long chord. Find
point A and B respectively on it, the diameter of the circle.
यिद घन का िवकण √27 सेमी है ,
such that ∠APB = 120°, and AP= एक वृ के क से 5 सटीमीटर ल ी
तो इसकी आयतन है :
12.5 cm. The length of OP is: एक लंबवत रे खा 24 सेमी लंबे जीवा
CHSL 26-10-2020 (Evening
दो शरे खाएँ PA और PB एक तक खींचा जाता है । वृ का ास
shift)
बाहरी िबंदु P से एक िबंदु पर मशः ात कीिजए।
(a) 27
क A और B पर O के साथ वृ की CHSL 17-03-2020 (Morning
(b) 32
ओर खींची जाती ह, जैसे िक∠APB = shift)
(c) 30

e
120° और AP = 12.5 cm OP की (a) 32 cm
(d) 25
लंबाई है : (b) 13 cm
CHSL 26-10-2020 (Evening (c) 30 cm
Q.74. A circle inscribed in a
shift) (d) 26 cm
triangle ABC touches its sides

l
(a) 24 cm
AB, BC and AC at the points D,
(b) 25 cm Q.80. In the given figure, chords
E and F, respectively. If AB =
ac (c) 26 cm AB and CD are intersecting each
18cm, BC = 15cm and AC = 13
(d) 20 cm other at point L. Find the length
cm then the value of of
of AB
AD+BE+CF is:
Q.77. If M is the mid point of the दी गयी आकृित म, जीवा AB और
एक ि भुज ABC म अ िनिहत एक
side BC of Δ ABC, and the area CD िबंदु L पर एक दू सरे को
वृ मशः AB, BC और AC के
of Δ ABM is 18 cm2 , then the ित े द कर रहे ह। AB की लंबाई
भुजाओं D, E और F को श करता
area of ΔABC is: ात कर
है । यिद AB = 18 सेमी, BC = 15
यिद M Δ ABC के भुजा BC के
nn
सेमी और AC = 13 सेमी है तो AD+
BE+ CF का मान है : म िबंदु है , और Δ ABM का
CHSL 26-10-2020 (Evening े फल 18 cm2 है , तो ΔABC का
shift) े फल है :
(a) 25 CHSL 17-03-2020 (Morning
(b) 33 shift)
(c) 23 (a) 30 cm2 CHSL 17-03-2020 (Morning
(d) 20 (b) 34 cm2 shift)
Pi

(c) 36 cm2 (a) 22.5 cm


Q.75. In an isosceles triangle (d) 32 cm2 (b) 21.5 cm
ABC, AB=AC and AD is (c) 24.5 cm
perpendicular to BC at D. If AD Q.78. The perimeter of a (d) 23.5 cm
= 8 cm and perimeter of Δ ABC rectangle is 50 cm. Its area and
is 64cm, then the area of Δ ABC length are in the ratio of 5:1. Find Q81. Find the circumference of a
एक समि बा ि भुज ABC म, AB the length of the rectangle? circle whose diameter is 12
= AC और AD, BC पर लंबवत है । एक आयत की प रिध 50 सेमी है । inches.
यिद AD = 8 सेमी और ABC की इसका े फल और लंबाई 5: 1 के एक वृ की प रिध ात कर िजसका
प रिध 64 cm है , तो ABC का अनुपात म है । आयत की लंबाई ात ास 12 इं च है ।
े फल है : कीिजये? CHSL 17-03-2020 (Afternoon
CHSL 26-10-2020 (Evening CHSL 17-03-2020 (Morning shift)
shift) shift) (a) 87.4672 cm
(a) 130 (a) 15 cm (b) 95.7072 cm

www.ssccglpinnacle.com support@ssccglpinnacle.com Ph. 09729327755, 09817390373 529


/
s
se
Days 54-60 Mensuration / े िमित

s
la
_c
ob
bo
ah
एक ित े द रे खा को एक िबंदु P से
sm
(c) 88.1876 cm Q85. If the height of an
ur
yo

(d) 90.2348 cm equilateral triangle is 12 cm, then एक वृ तक खींचा जाता है तािक वह


e/
t.m
://

what is the area of the triangle? वृ म पहले A से िमल जाए, िफर


tp
ht

Q82. The length and breadth of a यिद एक समबा ि भुज की ऊंचाई क से होकर जाता है , और B पर
rectangle are in the ratio 5 : 3. If 12 सेमी है , तो ि भुज का े फल वृ छोड़ता है । यिद P से वृ की
the length is 8 m more than the िकतना है ? शरे खा की लंबाई है 12 सेमी, और
breadth, what is the area of the CHSL 17-03-2020 (Evening वृ की ि ा 5 सेमी है , िफर P से A
rectangle? shift) की दू री है :
एक आयत की लंबाई और चौड़ाई 5: (a) 89.567 cm2 CHSL 17-03-2020 (Morning
3 म है । 3. यिद लंबाई चौड़ाई से 8 (b) 96.897 cm2 shift)
मीटर अिधक है , तो आयत का (c) 67.9843 cm2 (a) 8 cm
े फल ा है ? (d) 83.1384 cm2 (b) 12 cm
CHSL 17-03-2020 (Afternoon (c) 18 cm
shift) Q86. What is the area of a (d) 10 cm

e
(a) 240 m2 triangle whose sides measure 5
(b) 380 m2 cm, 6 cm and 7 cm? Q89. If one side of a triangle is 7
(c) 360 m2 एक ि भुज का े फल ा है with its perimeter equal to 18, and
(d) 400 m2 िजसकी भुजाएँ 5 सेमी, 6 सेमी और 7 area equal to √108, then the

l
सेमी ह? other two sides are:
Q83. Find the volume (in cm3 )
ac CHSL 17-03-2020 (Evening यिद ि भुज की एक भुजा 7 के साथ
of a sphere whose radius is 7.5 shift) प रिध 18 है , और े फल 108 के
cm. (a) 10.9797 cm2 बराबर है , तो अ दो भुजाएँ ह:
एक गोले का आयतन (in cm3 ) ात (b) 12.8484 cm2 CHSL 18-03-2020 (Morning
कर िजसका ि ा 7.5 सेमी है । (c) 16.4545 cm2 shift)
CHSL 17-03-2020 (Afternoon (d) 14.6969 cm2 (a) 6 and 5
shift) (b) 3.5 and 7.5
(a) 1767.85 Q87. A chord of length 24 cm is (c) 7 and 4
at a distance of 5 cm from the
nn
(b) 1985.23 (d) 3 and 8
(c) 1683.25 centre of a circle. What is its
(d) 1489.12 area? Q90. If he length of a rectangle is
एक वृ के क से 5 सेमी की दू री increased by 12% and the breadth
पर 24 सेमी लंबाई का एक जीवा है । is decreased by 8%, the net effect
Q84. The length and breadth of a इसका े फल ा है ? on the area is:
rectangle are in ratio 3:2. If its CHSL 17-03-2020 (Evening यिद आयत की लंबाई 12% बढ़ जाती
perimeter is 730 cm, what is the shift) है और चौड़ाई 8% कम हो जाती है ,
(a) 120 cm2 तो इस े का शु भाव है :
Pi

area of the rectangle?


एक आयत की लंबाई और चौड़ाई 3: (b) 480.67 cm2 CHSL 18-03-2020 (Morning
2 के अनुपात म है । यिद इसकी (c) 531.14 cm2 shift)
प रिध 730 सेमी है , तो आयत का (d) 389.28 cm2 (a) increase by 3.04%
े फल ा है ? (b) increase by 2.6%
CHSL 17-03-2020 (Evening Q88. A secant is drawn from a (c) decrease by 3.04%
shift) point P to a circle so that it meets (d) decrease by 2.6%
(a) 31,974 cm2 the circle first at A, then goes
(b) 24,452 cm2 through the centre, and leaves the Q91.
(c) 20,567 cm2 circle at B. If the length of the
(d) 28,976 cm2 tangent from P to the circle is 12
cm, and the radius of the circle is
5 cm, then the distance from P to
A is:

www.ssccglpinnacle.com support@ssccglpinnacle.com Ph. 09729327755, 09817390373 530


/
s
se
Days 54-60 Mensuration / े िमित

s
la
_c
ob
bo
ah
sm
and the area of the rectangle is
ur
yo

Q93. Find the perimeter of a right 288 cm2 Find the length of the
e/
t.m
://

angle triangle whose sides have rectangle.


tp
ht

sizes of 5 cm and 12 cm. यिद आयत की प रिध और लंबाई 6:


समकोण ि भुज की प रिध ात कर 1 के अनुपात म है और आयत का
िजसके भुजाओं का आकार 5 सेमी े फल 288 cm2 है । आयत की
और 12 सेमी हो। लंबाई ात कीिजए।
CHSL 18-03-2020 (Afternoon CHSL 18-03-2020 (Afternoon
PA and PB are tangents to the
shift) shift)
circle and O is the centre of the
(a) 30 cm (a) 10 cm
circle. The radius is 5 cm and PO
(b) 25 cm (b) 12 cm
is 13 cm. If the area of the
(c) 17 cm (c) 8 cm
triangle PAB is M, then the value
(d) 18 cm (d) 9 cm
of
√ M
15 is:

e
PA और PB वृ के शरे खा ह और Q94. Two circles touch each other Q97. The perimeter of an
O वृ का क है । ि ा 5 सेमी और externally. The distance between isosceles triangle is 125 cm. If the
PO 13 सेमी है । यिद ि भुज PAB का their centres is 8 cm. If the radius base is 33 cm, find the length of
of one circle is 3 cm, then the the equal sides.
े फल M है , तो
√ M का मान है :

l
15 radius of the other circle is: समि बा ि भुज की प रिध 125
CHSL 18-03-2020 (Morning दो वृ एक दू सरे को बा प से सेमी है । यिद आधार 33 सेमी है , तो
shift)
(a)
(b)
(c)

24
13
12
13
24
13
ac ि
ि
श करते ह। उनके क ों के बीच
की दू री 8 सेमी है । यिद एक वृ की
ा 3 सेमी है , तो दू सरे वृ की
ा है :
CHSL 18-03-2020 (Afternoon
समान भुजाओं की लंबाई ात करे ।
CHSL 18-03-2020 (Afternoon
shift)
(a) 32 cm
(b) 46 cm
(d)
√ 12
13 shift)
(a) 6 cm
(c) 34 cm
(d) 42 cm
(b) 4 cm
nn
Q92.
(c) 3 cm Q98.Two circles with the same
(d) 5 cm centre P have radii 7.5 cm and 4.4
cm. Through a point A of the
Q95. The wheel of a car has 210 larger circle, a tangent is drawn to
cm diameter. How many the smaller circle touching it at B.
A circle is inscribed in the revolutions per minute must the Find AC (Approximate in cm).
triangle ABC whose sides are wheel make so that the speed of एक ही क P के साथ दो वृ बनी है
the car is kept at 120 km/h? िजसकी ि ा 7.5 सेमी और 4.4
Pi

given as AB = 10, BC = 8, CA =
12 unit as shown in the figure. एक कार के पिहये का ास 210 cm सेमी है । बड़े वृ के िबंदु A से एक
The value of AD × BF is: है । पिहया ित िमनट िकतने च र शरे खा छोटे वृ से B पर िमलती है
ि भुज ABC म एक वृ अ िनिहत लगाना चािहए तािक कार की गित ।AC का मान ात कीिजए (सेमी म
है िजसकी भुजाएं AB = 10, BC = 8, 120 िकमी / घंटा रखी जाए? अनुमािनत)।
CA = 12 इकाई के प म िदए गए CHSL 18-03-2020 (Afternoon CHSL 18-03-2020 (Evening
ह जैसा िक िच म िदखाया गया है । shift) shift)
AD × BF का मान है : (a) 326.42 (a) 14 cm
(b) 245 (b) 12.14 cm
CHSL 18-03-2020 (Morning
(c) 303.03 (c) 14.27 cm
shift)
(d) 289 (d) 13 cm
(a) 18 units
(b) 21 units
Q96. If the perimeter and length Q99. The length of a rectangle
(c) 16 units
of a rectangle are in the ratio 6:1 plot is five times of its breadth. If
(d) 15 units

www.ssccglpinnacle.com support@ssccglpinnacle.com Ph. 09729327755, 09817390373 531


/
s
se
Days 54-60 Mensuration / े िमित

s
la
_c
ob
bo
ah
sm
the area of the rectangular plot is Q102. In an isosceles right-angled S, respectively. The length of AB
ur
yo

2000 m2 , then what is the triangle, the perimeter is 30 m. is equal to:


e/
t.m
://

breadth of the rectangular plot? Find its area (Approximate.) दो वृ िजनकी ि ाएँ मशः 20
tp
ht

एक आयताकार भूखंड की लंबाई समि बा समकोण ि भुज म, प रिध सेमी और 5 सेमी है , िबंदु P पर बा
इसकी चौड़ाई का पां च गुना है । यिद 30 मीटर है । इसका े फल ात प से एक दू सरे को श करते ह,
आयताकार भूखंड का े फल 2000 कर (लगभग) AB मशः क ों R और S वाले दो
m2 , है , तो आयताकार भूखंड की CHSL 18-03-2020 (Evening वृ ों के शरे खा है । AB की
चौड़ाई ा है ? shift) लंबाई के बराबर है :
CHSL 18-03-2020 (Evening (a) 38.63 m2 CHSL 19-03-2020 (afternoon
shift) (b) 39.60 m2 shift)
(a) 40 m (c) 37.86 m2 (a) 15 cm
(b) 10 m (d) 40 m2 (b) 5 cm
(c) 20 m (c) 10 cm
(d) 30 m Q.103. The three medians AX, (d) 20 cm

e
BY and CZ of Δ ABC intersect at
Q100. Two small circular grounds point L. If the area of Δ ABCis Q.106. The area of the quadrant
of diameters 42 m and 26 m are 30 cm2 , then the area of the of a circle whose circumference is
to be replaced by a bigger circular quadrilateral BXLZ is: 22 cm, will be:

l
ground. What would be the radius एक ि भुज ABC के तीन मा काएँ एक वृ का चतुथाश का े फल,
of the new ground if the new
ac AX, BY और CZ िबंदु L पर िजसकी प रिध 22 सेमी है , वह होगा:
ground has the same area as the ितछे िदत करते है , यिद Δ ABC CHSL 19-03-2020 (afternoon
two small grounds? shift)
का े फल 30 cm2 है , तो चतुभुज
42 मीटर और 26 मीटर ास के दो (a) 3.5 cm2
BXLZ का े फल है :
छोटे गोलाकार मैदानों को एक बड़े CHSL 19-03-2020 (afternoon
(b) 10 cm2
गोलाकार मैदान से बदलना है । यिद shift) (c) 38.5 cm2
नए मैदान का े फल, दो छोटे (a) 12 cm2 (d) 9.625 cm2
मैदानों के े फल के समान है , तो
(b) 16 cm2
नई मैदान की ि ा ा होगी ?
nn
(c) 10 cm2 Q.107. The distance between the
CHSL 18-03-2020 (Evening centres of two equal circles each
(d) 14 cm2
shift) of radius 4 cm is 17 cm. The
(a) 24.69 m length of a transverse tangent is:
Q.104. The area of an isosceles
(b) 23 m दो समान वृ ों के क ों के बीच की
right angles triangle is 121 cm2 .
(c) 25 m दू री, ेक ि ा 4 सेमी, 17 सेमी
Find its hypotenuse
(d) 25.01 m है । अनु थ शरे खा की लंबाई है :
समि बा समकोण ि भुज का
Q101. The perimeter of an CHSL 19-03-2020 (afternoon
े फल 121 cm2 है । इसका कण
isosceles triangle is 90 cm. If the
Pi

shift)
base is 26 cm, then find the ात कर.
(a) 15 cm
length of the equal sides. CHSL 19-03-2020 (afternoon
(b) 16 cm
समि बा ि भुज की प रिध 90 सेमी shift)
(c) 14 cm
है । यिद आधार 26 सेमी है , तो समान (a) 23
(d) 19 cm
भुजाओं की लंबाई ात करे । (b) 22
CHSL 18-03-2020 (Evening (c) 21
Q.108. The radius a circular cone
shift) (d) 20
is R and its height is H. the
(a) 30 cm volume of cone is:
Q.105. Two circles of radii 20 cm
(b) 40 cm एक गोलाकार शंकु की ि ा R है
and 5 cm respectively, touch each
(c) 42 cm और इसकी ऊंचाई H है । शंकु का
other externally at the point P, AB
(d) 32 cm आयतन है
is the direct common tangent of
CHSL 19-03-2020 (Morning
these two circles of centres R and
shift)

www.ssccglpinnacle.com support@ssccglpinnacle.com Ph. 09729327755, 09817390373 532


/
s
se
Days 54-60 Mensuration / े िमित

s
la
_c
ob
bo
ah
sm
πR2 H the percentage increase in the the sum of the surface area of the
(a)
ur

3
yo

area of the field is: balls?


e/

(b) 2
t.m

3πR2 H
://

एक आयताकार मैदान के ेक 15 सेमी ि ा का एक ठोस धा क


tp
ht

2
(c) π R H
भुजा म 10% की वृ ई है । िफर े को िपघला कर ि ा 3 सेमी के
(d) π R2 H मैदान के े फल म ितशत वृ है : ेक गोलाकार गद बनाई जाती है ।
CHSL 19-03-2020 (Evening मूल सतह के े फल और गदों के
Q.109. The length of tangents shift) सतह े के योग का अनुपात ा
drawn from an external point to (a) 10% है ?
the circle are: (b) 15% (CGL MAINS 15-10-2020)
एक बाहरी िबंदु से वृ तक खींची गई (c) 18% (a) 1 : 5
शरे खा की लंबाई ह: (d) 21% (b) 1 : 10
CHSL 19-03-2020 (Morning (c) 5 : 27
shift) Q.113. In a circle centred at O, (d) 3 : 40
(a) not equal AB is a chord and C is any point
(b) perpendicular

e
on AB, such that OC is Q116. The radii of two right
(c) parallel perpendicular to AB. If the length circular cylinders are in the ratio
(d) equal of the chord is 16 cm and OC = 6 3 : 2 and the ratio of their
cm, the radius of circle is : volumes is 27:16. What is the
Q.110. A pair of tangents AB and

l
O क वाले एक वृ म, AB एक ratio of their heights?
AC are drawn from a point which जीवा है और C, AB पर कोई िबंदु है , दो गोलाकार िसलडरों की ि ा 3: 2
ac
is at a distance of 10 cm from the
centre O of a circle of radius 6
cm, then the area in cm2 of
quadrilateral ABOC, is:
एक जोड़ी शरे खा AB और AC
जैसे OC, AB पर लंबवत है । यिद
जीवा की लंबाई 16 सेमी और OC =
6 सेमी है , तो वृ की ि
CHSL 19-03-2020 (Evening
shift)
ा है :
के अनुपात म है और उनके आयतन
का अनुपात 27:16 है । उनकी ल ाई
का अनुपात ा है
(CGL MAINS 15-10-2020)
(a) 3 : 4
एक िबंदु से खींची जाती है , जो 6 सेमी (a) 10 cm (b) 8 : 9
ि ा वाले वृ के क O से 10 सेमी (b) 8 cm (c) 4 : 3
की दू री पर है , िफर चतुभुज ABOC (c) 6 cm (d) 9 : 8
nn
का े फल है (d) 12 cm
CHSL 19-03-2020 (Morning Q117. The base of a right
shift) SSC CGL 2019 TIER-II pyramid is a square of a side
(a) 52 Q114. The areas of three adjacent 10cm. If its height is 10 cm, then
(b) 72 faces of a cuboidal tank are the area (in cm 2 ) of its lateral
(c) 60 3m 2 , 12m 2 and 16m 2 . The surface is:
(d) 48 capacity of the tank in litres is : एक िपरािमड का आधार 10 सेमी
घनाकार टक के तीन आस भुजाओं भुजा वाला वग है । यिद इसकी
Pi

Q.111. The volume of a cube is के े फल 3m 2 , 12m 2 और 16m 2 ऊंचाई 10 सेमी है , तो इसकी पा


343 cm3 . The edge of cube is: सतह का े फल (सेमी 2 म) है ?
ह। टक की मता है (लीटर म)
एक घन का आयतन 343 cm3 है । (CGL MAINS 15-10-2020) (CGL MAINS 15-10-2020)
घन का िकनारा है : (a) 48000 (a) 100
CHSL 19-03-2020 (Evening (b) 24000 (b) 100(√5 + 1)
shift) (c) 72000
(a) 6 cm (c) 50√5
(d) 36000
(b) 5 cm (d) 100√5
(c) 7 cm Q115. A solid metallic sphere of
(d) 4 cm radius 15cm melted and recast Q118. The height of a cylinder is
into spherical balls of radius 3 cm 30 cm and the diameter of its base
Q.112. Each side of a rectangular each. What is the ratio of the is 10 cm. Two identical conical
field is increased by 10%. Then surface area of the original and holes each of radius 5 cm and 12
cm are drilled out. what is the

www.ssccglpinnacle.com support@ssccglpinnacle.com Ph. 09729327755, 09817390373 533


/
s
se
Days 54-60 Mensuration / े िमित

s
la
_c
ob
bo
ah
sm
surface area(in cm 2 ) of the Q121. The perimeter of Δ ABC increased by 20% and the height
ur
yo

and Δ DEF are 43.2 cm and 28.8 is decreased by 30% then what is
e/

remaining solid?
t.m
://

एक िसलडर की ऊंचाई 30 सेमी है cm, respectively, and ΔABC the percentage increase/decrease
tp
ht

और इसके आधार का ास 10 सेमी ~ ΔDEF . If DE =12cm, then the in the volume ?


है । दो समान शं ाकार िछ ेक length of AB is: यिद िसलडर के आधार की ि ा
ि ा 5 सेमी और 12 सेमी तक िडल Δ ABC और Δ DEF की प रिध 20% बढ़ जाती है और लंबाई 30%
कर बाहर िनकाल िदया जाता है शेष मशः 43.2 सेमी और 28.8 सेमी है , तक कम हो जाती है तो आयतन म
ठोस का पृ ीय े फल ा है ? और ΔABC ~ ΔDEF । यिद DE = ितशत वृ / कमी ा है ?
(CGL MAINS 15-10-2020) 12 सेमी है , तो AB की लंबाई है (CGL MAINS 15-10-2020)
(a) 230 π (CGL MAINS 15-10-2020) (a) Increase 2%
(b) 430 π (a) 20 cm (b) Decrease 0.8%
(c) 330 π (b) 18.4 cm (c) Increase 0.8%
(d) 120 π (c) 18 cm (d) Decrease 2%
(d) 20.4 cm

e
Q119. Given that Δ DEF ~ Δ Q125.. A spherical metallic shell
ABC, If the area of Δ ABC is 9 Q122. The area of the base of a with 6 cm external radius weighs
2 right circular cone is 81 π cm 2 6688 g, What is the thickness of
cm and that of Δ DEF =12
the shell if the density of metal is
2
cm , BC=2.1 cm, then the length and its height is 12cm. What is

l
10.5 g per cm 3 ?(take π = 227 )
of EF is : the curved surface area ( in cm 2 )
ac 6 सेमी के बाहरी ि ा के साथ एक
िदया आ है की, Δ DEF ~ Δ ABC, of the cone ?
एक गोलाकार शंकु के आधार का गोलाकार धा क शैल का वजन
यिद Δ ABC का े फल 9 cm 2 है
े फल 81π cm 2 सेमी है और 6688 ाम है , यिद धातु का घन
और Δ DEF = 12 cm 2 ,BC = 2.1
10.5 ाम ित सेमी है तो शेल की
सेमी है , तो EF की लंबाई है इसकी ऊंचाई 12 cm है । शंकु का
मोटाई ा है ? (मािनए = 227 )
(CGL MAINS 15-10-2020) घुमावदार सतह े फल ( cm 2 म)
ा है (CGL MAINS 15-10-2020)
(a) √ cm
8 3
5
(CGL MAINS 15-10-2020) (a) 2 cm
3√7 (b) 3 cm
(b) 5 cm (a) 108 π
nn
4√7 (c) 2 21 cm
(c) (b) 135 π
5 cm
(c) 126 π (d) 4 cm
7√3
(d) 5 cm (d) 144 π
Q126.. The sum of the radii of
spheres A and B is 14 cm, the
Q123. The base of a right prism is
radius of A being larger than that
a regular hexagon of side 5 cm. If
Q120. The curved surface area of of B. The difference between
its height is 12√3 cm, then its
a right cylinder is 3696 cm 2 , Its their surface area is 112 π . What
volume ( in cm 3 ) is:
Pi

height is three times its radius. is the ratio of volumes of A and


What is the capacity(in liters) of एक ि का आधार 5 सेमी का एक B?
the cylinder?(Take π = 22 िनयिमत षट् भुज है । यिद इसकी गोला A और B के ि ा का योग 14
7 )
एक िसलडर का घुमावदार सतह का ऊंचाई 12√3 सेमी है , तो इसकी सेमी है , A की ि ा B की तुलना म
े फल 3696 cm 2 है , इसकी आयतन बड़ी है । उनके सतह े फल के बीच
( स◌े म◌ीम)3 है का अंतर 112 π है । A और B के
ऊंचाई, ि ा से तीन गुना है ।
िसलडर की मता (लीटर म) ा है ? (CGL MAINS 15-10-2020) आयतन का अनुपात ा है ?
(मािनये π = 22 (a) 900 (CGL MAINS 15-10-2020)
7 )
(b) 1800 (a) 64 : 27
(CGL MAINS 15-10-2020)
(c) 1350 (b) 8 : 1
(a) 25.872
(d) 675 (c) 125 : 64
(b) 30.87
(c) 19.008 (d) 27 : 8
Q124. If the radius of the base of
(d) 29.75
a right circular cylinder is

www.ssccglpinnacle.com support@ssccglpinnacle.com Ph. 09729327755, 09817390373 534


/
s
se
Days 54-60 Mensuration / े िमित

s
la
_c
ob
bo
ah
sm
Q127. The radius and height of a Q130. A cylindrical roller made (b) 416.25 π
ur
yo

right circular cone are in ratio 3 : up of iron is 1.2m long. Its (c) 472.5 π
e/
t.m
://

4. If its curved surface area (in internal radius is 24cm and (d) 270 π
tp
ht

cm 2 ) is 240 π , then its volume thickness of the iron sheet used in


(in cm 3 ) is: making the roller is15 cm. What Q133. A metallic solid spherical
एक गोलाकार शंकु की ि ा और is the(in kg ) of the roller, if 1 ball of radius 3 cm is melted and
ऊंचाई अनुपात 3: 4 म है । यिद cm3 of the iron has 8 g mass? recast into three spherical balls.
इसकी व सतह का े फल ( cm 2 लोहे से बना एक बेलनाकार रोलर The radii of two of these balls are
म) 240 π है , तो इसकी आयतन ( 1.2 मीटर लंबा है । इसकी आं त रक 2 cm and 1.5cm . What is the
स◌े म◌ीम)3 है ? ि ा 24 सेमी है और रोलर बनाने म surface area (in cm2 ) of the third
(CGL MAINS 15-10-2020) उपयोग की जाने वाली लोहे की शीट ball?
(a) 768 π की मोटाई 15 सेमी है । रोलर का ि ा 3 सेमी की एक ठोस गोलाकार
वजन (िकलो म) ा है , यिद 1 cm3 गद को िपघलाया जाता है और तीन
(b) 384 π
लोहे का मान 8 ाम है ? गोलाकार गदों म बदल िदया जाता
(c) 2304 π

e
(d) 1536 π
(CGL MAINS 16-10-2020) है । इन गदों म से दो की ि ा 2
(a) 892.8 π सेमी और 1.5 सेमी है । तीसरी गद का
(b) 907.2 π सतह े (सेमी 2 म) ा है
Q128. The length of two sides of
(c) 846.72 π (CGL MAINS 16-10-2020)
a parallelogram are 3cm and 10

l
(d) 845.75 π (a) 25
2 π
cm. What is the sum of the
ac Q131. The base of a solid prism (b) 25 π
squares of diagonals of the 4
parallelogram ? of height 10cm is a square and its (c) 50 π
एक समां तर चतुभुज के दो भुजाओं volume is 160 cm3 , What is its (d) 25 π
की लंबाई 3 सेमी और 10 सेमी है । total surface area of the prism (in
समां तर चतुभुज के िवकण के वग cm2 )?
का योग ा होगा ? ऊंचाई 10 सेमी के एक ठोस ि Q134.. The ratio of the radii of
(CGL MAINS 15-10-2020) का आधार एक वग है और इसकी two cones is 5 : 6 and their
(a) 218 cm 2 आयतन 160 cm3 है , ि की कुल volume is in the ratio 8 : 9. The
nn
(b) 169 cm 2 सतह का े फल ( cm2 म) ा है ratio of their height is :
(c) 206 cm 2 (CGL MAINS 16-10-2020) दो शंकु की ि ा का अनुपात 5: 6
(d) 109 cm 2 (a) 200 है और उनकी आयतन 8: 9 के
(b) 192 अनुपात म है । उनकी ऊंचाई का
(c) 180 अनुपात ा होगा
Q129. If the radius of a sphere is
(d) 176 (CGL MAINS 16-10-2020)
increased by 2.5 decimeter (dm),
(a) 25 : 27
then its surface area increases by
Q132.. From a solid cylinder (b) 27 : 20
110 dm2 . What is the volume(in
Pi

wooden block of height 18cm and (c) 20 : 27


dm2 ) of the sphere?(take π = 22
7 )
radius 7.5cm, a conical cavity of (d) 32 : 25
यिद एक गोले का ि ा 2.5
same radius and same height is
डे सीमीटर (dm) बढ़ जाता है , तो Q135. The circumference of the
taken out. What is total surface
इसका सतह े से 110 dm2 बढ़
area (in cm2 ) of the remaining base of a cylindrical vessel is
जाता है । गोले का आयतन ( dm2 म) solid? 158.4cm and its height is 1 m .
ा है ? ऊंचाई 18 सेमी और ि ा 7.5 सेमी How many liters of water can it
(CGL MAINS 16-10-2020) के एक ठोस िसलडर लकड़ी के hold(correct to one decimal
(a) 73 ॉक से, समान ि ा और समान place)?
(b) 11
21 ऊंचाई का एक शं ाकार गुहा बाहर एक बेलनाकार पतीले के आधार की
(c) 13
21 िनकाला जाता है । शेष ठोस का कुल प रिध 158.4 सेमी है और इसकी
(d) 4 े फल ( स◌े म◌ीम)
2
ा है ऊंचाई 1 मी है । इसके िकतने लीटर
7
(CGL MAINS 16-10-2020) पानी की मता है (एक दशमलव
(a) 326.25 π थान तक )

www.ssccglpinnacle.com support@ssccglpinnacle.com Ph. 09729327755, 09817390373 535


/
s
se
Days 54-60 Mensuration / े िमित

s
la
_c
ob
bo
ah
sm
22
(Take π = ) 30√3 . The volume( in cm2 ) of
ur

7
yo

Q141. The base of a pyramid is


e/

(CGL MAINS 16-10-2020) the pyramid is :


t.m
://

(a) 199.6 िपरािमड का आधार 8 सेमी के साथ an equilateral triangle of side is


tp
ht

(b) 198.2 एक समबा ि भुज है , और इसकी 10m. If the height of the pyramid
(c) 200.8 ऊंचाई 30√3 है । िपरािमड का is 40 √3 m, then the volume of
(d) 186.4 आयतन ( cm2 म) है : the pyramid is :
(CGL MAINS 16-10-2020) एक िपरािमड का आधार 10 m भुजा
Q136. In Δ ABC, D and E are (a) 480 वाला एक समबा ि भुज है । यिद
points on the sides AB and AC,
(b) 360√3 िपरािमड की ऊंचाई 40 √3 मीटर है ,
respectively, such that DE ∥ BC तो िपरािमड का आयतन ा है
(c) 360
and DE : BC = 6 : 7, (Area of Δ (CGL MAINS 18-10-2020)
(d) 240√3
ADE) : (Area of trapezium (a) 800 m3
BCED) =? (b) 900 m3
Δ ABC म, D और E, मशः AB (c) 1000 m3

e
Q139. The circumference of the
और AC पर िबंदु ह, जैसे िक DE ∣ ∣ (d) 1200 m3
base of a right circular cone is
BC और DE : BC = 6 : 7, तो ( Δ 44cm and its height is 24cm. The
ADE का े फल): (समल curved surface area ( in cm2 ) of
Q142. The curved surface area of

l
BCED का े फल) = ? the cone is :
a cylinder is five times the area of
(CGL MAINS 16-10-2020)
ac एक गोलाकार शंकु के आधार की
a base. Find the ratio of radius
(a) 49 : 13 प रिध 44 cm है और इसकी ऊंचाई
and height of the cylinder.
(b) 36 : 13 24 cm है । शंकु की घुमावदार सतह
एक िसलडर का घुमावदार सतह
(c) 13 : 36 े ( cm2 म) है
े फल उसके आधार के े फल
(d) 13 : 49 (CGL MAINS 16-10-2020)
का पां च गुना है । िसलडर के ि ा
(a) 572
और ऊंचाई के अनुपात का पता
Q137. A hemispherical tank full (b) 550
लगाएं ।
of water is emptied by a pipe at (c) 528
(CGL MAINS 18-10-2020)
(d) 440
nn
the rate of 7.7 liters per seconds.
(a) 2 : 5
How much time(in hours) will it
(b) 2 : 3
take to empty 32 part of the tank if Q140. A solid metallic cuboid of
(c) 3 : 4
the internal radius of the tank is dimensions 18 cm × 36cm ×
(d) 3 : 5
10.5 m ? 72cm is melted and recast into 8
पानी से भरा एक गोलाध टक 7.7 cubes of the same volume. What
Q143. The volume of a
लीटर ित सेकंड की दर से एक is the ratio of the total surface 1 3
hemisphere is 2425 2 cm . Find
पाइप ारा खाली िकया जाता है । टक area of the cuboid to the sum of
its radius. (take π = 22 )
the lateral surface area of all 8
Pi

7
के 32 िह े को खाली करने म
cubes? एक गोलाध का आयतन 2425 21 cm3
िकतना समय लगेगा(घंटे म), यिद
18 सेमी × 36 सेमी × 72 सेमी आयाम है तो उसकी ि ा ात करे (
टक की आं त रक ि ा 10.5 मीटर
वाले एक ठोस धा क घनाभ को π = 22
7 )
है ?
(CGL MAINS 16-10-2020) िपघलाकर समान आयतन वाले 8 घन (CGL MAINS 18-10-2020)
(a) 185 म पुनगिठत िकया जाता है । घनाभ (a) 10 cm
6
175 के कुल सतह े फल से सभी 8 घन (b) 9.5 cm
(b) 3 के पा सतह े फल का अनुपात (c) 12 cm
(c) 185
3 ा है ? (d) 10.5 cm
(d) 175
2 (CGL MAINS 16-10-2020)
(a) 2 : 3 Q144. If the radius of a cylinder
Q138. The base of a right (b) 7 : 8 is decreased by 20% and the
pyramid is an equilateral triangle (c) 4 : 7 height is increased by 20% to
with side 8 cm, and its height is (d) 7 : 12

www.ssccglpinnacle.com support@ssccglpinnacle.com Ph. 09729327755, 09817390373 536


/
s
se
Days 54-60 Mensuration / े िमित

s
la
_c
ob
bo
ah
वाले ठोस लेड गोले से बनाया जा
sm
form a new cylinder then the (d) 256 cm2
ur
yo

volume will be decreased by: सकता है ?


e/
t.m
://

यिद िकसी िसलडर की ि ा 20% (CGL MAINS 18-10-2020) Q.151 If the surface area of a
tp
ht

तक कम हो जाती है और उसकी (a) 2742 sphere is 1386 cm2 , then its


ऊँचाई 20% तक बढ़ जाती है तो नए (b) 2744 volume is: ( π = 22
7 )
िसिलंडर की आयतन म िकतनी कमी (c) 4722 यिद एक गोले का सतह े 1386
होगी (d) 7244 cm2 है , तो इसका आयतन है : ( π =
(CGL MAINS 18-10-2020) 22 )
7
(a) 23.2% Q.148 The base of a right prism is
(CGL MAINS 18-10-2020)
(b) 32.2% a square having side of 15 cm. If
(a) 8451 cm3
(c) 22.3% its height is 8 cm, then find the
(b) 5418 cm3
(d) 20.5% total surface area.
(c) 4581 cm3
एक ि का आधार एक वग होता
Q145.The ratio of the height and है , जो 15 सेमी की भुजा का है । यिद (d) 4851 cm3

e
the diameter of a right circular इसकी ऊंचाई 8 सेमी है , तो कुल
cone is 6:5 and its volume is 2200 सतह के े को ात करे Q.152. In the figure, chords AB
7
(CGL MAINS 18-10-2020) and CD of a circle intersect
cm2 .What is its slant height?
(a) 920 cm2 externally at P. If AB = 4cm, CD
(Take π = 22
7 ) =11cm and PD=15cm, then the

l
(b) 930 cm2
एक गोलाकार शंकु का ऊंचाई और length of PB is:
(c) 900 cm2
ास का अनुपात 6: 5 है और इसकी
ac दी गयी आकृित म, जीवा AB और
(d) 940 cm2
आयतन 22007 cm
2
है । तो इसकी CD एक वृ को बाहरी प से P पर
ऊंचाई िकतनी है ?( π = 22
7 )
काटते ह। यिद AB = 4cm, CD =
Q.149. If the perimeter of an
(CGL MAINS 18-10-2020) 11cm और PD = 15cm है , तो PB
isosceles right triangle is 8( √2 की लंबाई ात करे |
(a) 25cm
+1) cm, then the length of the
(b) 26cm
hypotenuse of the triangle is:
(c) 13cm
यिद समि बा समकोण ि भुज की
(d) 5cm
nn
प रिध 8( √2 +1) सेमी है , तो ि भुज
Q.146 The radii of two cylinders के िवकण की लंबाई है
are in the ratio 3:4 and their (CGL MAINS 18-10-2020)
(a) 10cm (CGL MAINS 18-10-2020)
heights are in the ratio 8:5. The
(b) 8cm (a) 14cm
ratio of their volumes is .
(c) 24cm (b) 12cm
दो िसलडरों की ि ा 3: 4 के
(d) 12cm (c) 8cm
अनुपात म है और उनकी ऊँचाई 8: 5
(d) 10cm
के अनुपात म है । उनके आयतन का
Pi

अनुपात है : Q.150. The sum of length,


breadth and height of a cuboid is Q153. In a triangle ABC, AB =
(CGL MAINS 18-10-2020)
20 cm. If the length of the AC and the perimeter of Δ ABC
(a) 8:9
(b) 9:10 diagonal is 12cm, then find the is 8(2 + √2 )cm . If the length of
(c) 7:10 total surface area of the cuboid. BC is √2 times the length of AB,
(d) 9:11 एक घनाभ की लंबाई, चौड़ाई और then find the area of Δ ABC.
ऊंचाई का योग 20 सेमी है । यिद एक ि भुज ABC म AB = AC और
िवकण की लंबाई 12 सेमी है , तो
Q.147. The number of lead balls, Δ ABC की प रिध 8(2 + √2 ) सेमी
घनाभ का कुल सतह े फल ात
each 3cm in diameter, that can be है । यिद BC की लंबाई AB की लंबाई
made from a solid lead sphere of कर |
से √2 गुना है , तो Δ ABC का
diameter 42 cm is : (CGL MAINS 18-10-2020)
(a) 264 cm2 े फल ात कीिजए|
ेक 3 cm ास वाले लेड गदों की (CGL MAINS 18-10-2020)
सं ा ा होगी िजसे 42 cm ास (b) 364 cm2
(a) 28 cm2
(c) 356 cm2

www.ssccglpinnacle.com support@ssccglpinnacle.com Ph. 09729327755, 09817390373 537


/
s
se
Days 54-60 Mensuration / े िमित

s
la
_c
ob
bo
ah
को िपघलाया जाता है और एक ही
sm
(b) 36 cm2 (b) 25%
ur
yo

(c) 32 cm2 (c) 21.13% घन म पुनगिठत िकया जाता है । नए


e/
t.m
://

(d) 16 cm2 (d) 20.79% घन का कुल े फल ( cm2 म) ा


tp
ht

है ?
SSC CPO 2019 Q157. The radius of the base of a CPO-2019 23-11-2020
Q154. One side of a rhombus is cylinder is 14cm and its curved (Evening shift)
13cm and one of its diagonal is surface area is 880 cm2 , Its (a) 2744
10 cm. What is the area of the volume (in cm3) is: (Take π = 22
7 ) (b) 1362
rhombus (in cm2 ) ? एक िसलडर के आधार की ि ा 14 (c) 865
िकसी समचतुभुज की एक भुजा 13 सेमी है और इसकी घुमावदार सतह (d) 1176
cm और एक िवकण 10 cm है । का े फल 880 cm2 ह◌ै, िसिलंडर
समचतुभुज का े फल ात करे | ( का आयतन(cm3) ात करे | (Take Q160 The two parallel sides of a
cm2 म) π = 22 trapezium are 27cm and 13cm,
7 )
CPO-2019 23-11-2020 CPO-2019 23-11-2020 respectively. If the height of the

e
(Morning shift) (Morning shift) trapezium is 7cm, then what is the
(a) 30 (a) 1078 area in m2 ?
(b) 60 (b) 3080 यिद एक समलंब चतुभुज के दो
(c) 120 (c) 9240 समां तर भुजाए मशः 27 सेमी और

l
(d) 90 (d) 6160 13 सेमी ह। यिद समल चतुभुज की
ac ऊंचाई 7 सेमी है , तो इसका े फल
Q158. The sides of a triangle are ात कर ( m2 म◌े ◌ं )
Q155. The perimeter of a square 24cm, 26cm and 10cm. At each CPO-2019 23-11-2020
is the same as the perimeter of a of its vertices, circles of radius (Evening shift)
rectangle. The perimeter of the 4.2cm are drawn. What is the area (a) 0.14
square is 40m. If its breadth is (in cm2 ) of the triangle, (b) 140
two-third of its length, then the excluding the portion covered by (c) 1.4
area (in m2) of the rectangle is: the sectors of the circle? (d) 0.014
nn
एक वग की प रमाप एक आयत की एक ि भुज की भुजाएँ 24 सेमी, 26 Q161. If a wheel has diameter
प रमाप के समान है । वग की सेमी और 10 सेमी ह। इसके ेक 42cm, then how far does the
प रमाप 40 मी है । यिद इसकी कोने पर ि ा 4.2 सेमी के वृ खींचे wheel go (in meters) in 12
चौड़ाई इसकी लंबाई का दो-ितहाई गए ह। वृ के े ों ारा कवर िकए revolutions?(Take π = 22 7 )
है , तो आयत का े फल (m2) ात गए िह े को छोड़कर, ि भुज का यिद िकसी पिहये का ास 42 सेमी
कर | े फल ( cm2 ) ात करे | है , तो 12 च रो म पिहया िकतनी
CPO-2019 23-11-2020 CPO-2019 23-11-2020 दु री (मीटर म) तय करे गा ? (मािनए
(Morning shift) (Morning shift) π = 22 7 )
Pi

(a) 96 (a) 27.72 CPO-2019 23-11-2020


(b) 84 (b) 120 (Evening shift)
(c) 100 (c) 105.86 (a) 17.64
(d) 121 (d) 92.28 (b) 15.84
(c) 23.27
Q156. If each side of a rectangle Q159. A 9 cm solid metallic cube (d) 21.45
is decreased by 11%, then the and a solid metallic cuboid
area will decrease by: having dimensions 5 cm, 13cm,
यिद आयत के ेक भुजा म 11% 31cm are melted and recast into a Q162. A rectangular lawn whose
की कमी होती है , तो े फल म single cube. What is the total length is twice of its breadth is
िकतनी कमी आएगी ? surface area(in cm2 ) of the new extended by having four
CPO-2019 23-11-2020 cube? semi-circular portions on its
(Morning shift) 5 सेमी, 13 सेमी, 31 सेमी आयाम sides. What is the total cost (in
(a) 24.31% वाले घनाभ और 9 सेमी ठोस धातु घन Rs.) of levelling the entire lawn at

www.ssccglpinnacle.com support@ssccglpinnacle.com Ph. 09729327755, 09817390373 538


/
s
se
Days 54-60 Mensuration / े िमित

s
la
_c
ob
bo
ah
5 सेमी, 13 सेमी, 31 सेमी आयाम
sm
the rate of Rs.100 per square Q167. A solid lead sphere of
ur
yo

metre if the smaller side of the वाले घनाभ और 9 सेमी ठोस धातु घन radius 11cm is melted and recast
e/
t.m
://

rectangle lawn is 12 m?(Take को िपघलाया जाता है और एक ही into small solid of radius 2 cm


tp
ht

π = 3.14) घन म पुनगिठत िकया जाता है । each. How many maximum (in


एक आयताकार लॉन िजसकी लंबाई Rs.10 ित cm2 की दर से नए घन integer) of such sphere can be
उसकी चौड़ाई से दोगुनी है , उसके को पॉिलश करने की लागत ा होगी made?
भुजाओ पर चार अध-वृ ाकार िह े CPO-2019 24-11-2020 11 सेमी की ि ा का एक ठोस गोले
बनाकर इसे बढ़ाया जाता है । यिद (Morning shift) को िपघलाया जाता है और ेक 2
आयताकार लॉन की चौड़ाई 12 मीटर (a) 13,620 सेमी ि ा के छोटे गोले म
है तो पूरे लॉन को 100 पये ित वग (b) 11,760 पुनगिठत िकया जाता है । ऐसे िकतने
मीटर की दर से समतल करने म (c) 8,650 गोले (अिधकतम पूणाक) बनाए जा
कुल लागत ( म) ा है ? ( म◌ानि◌ए π (d) 27,440 सकते ह
= 3.14) CPO-2019 24-11-2020
CPO-2019 23-11-2020 Q165. The two parallel sides of a (Morning shift)

e
(Evening shift) trapezium are 17cm and 15cm (a) 100
(a) 85,320 respectively. If the height of the (b) 30
(b) 86,540 trapezium is 6 cm, then its area (c) 166
(c) 78,650 (in m2 ) is: (d) 125

l
(d) 97,625 यिद एक समलम चतुभुज के दो
ac समानां तर भुजाए मशः 17 सेमी Q168. The internal measures of a
Q163. A rectangular lawn whose और 15 सेमी ह। यिद समल cuboidal room are with length as
length is twice of its breadth is चतुभुज की ऊंचाई 6 सेमी है , तो 12m, breadth as 8m and height as
extended by having four इसका े फल ात करे ( m2 म◌े ◌ं ) 10m. The total cost (in Rs.) of
semi-circular portions on its CPO-2019 24-11-2020 whitewashing all four walls of the
sides. What is the total area (in (Morning shift) room and also the ceiling of the
m 2 ) of the lawn if the smaller (a) 960 room, If the cost of whitewashing
side (b) 0.96 is Rs.25 per m2 is:
घनाकार कमरे के आं त रक माप की
nn
of the rectangle is 12m? (Take (c) 0.0096
π = 3.14) (d) 9.6 लंबाई 12 मीटर, चौड़ाई 8 मीटर और
एक आयताकार लॉन िजसकी लंबाई ऊंचाई 10 मीटर है । कमरे की सभी
इसकी चौड़ाई से दोगुनी है , उसके Q166. The area of a field in the चार दीवारों और कमरे की छत को
िकनारों पर चार अध-वृ ाकार िह े shape of a hexagon is 1944√3m3 . भी सफेद करने की कुल लागत (
होते ह। यिद आयत की चौड़ाई 12 What will be the cost (in Rs) of म) ा होगी यिद सफेदी की लागत
मीटर है तो लॉन का कुल े फल ( fencing it at the rate of Rs.11.50 ित m2 25 है :
m 2 ) म ा है ? ( म◌ानि◌ए=π 3.14) per metre? CPO-2019 24-11-2020
(Evening shift)
Pi

CPO-2019 24-11-2020 समषट् भुज के आकार वाले एक


(a) 12,400
(Morning shift) मैदान का े फल 1944√3m3 है ।
(a) 444 (b) 12,000
11.50 पये ित मीटर की दर से
(b) 548.32 (c) 18,000
फिसंग लगाने की लागत ( म) ा
(c) 308.64 (d) 13,600
होगी?
(d) 853.2 CPO-2019 24-11-2020
Q169. The base of a triangle is
(Morning shift)
Q164. A 9cm solid metallic cube equal to the perimeter of a square
(a) 2,256
and a solid metallic cuboid (b) 2,785 whose diagonal is 6 √2 cm , and
having dimensions 5 cm, 13cm, (c) 3,200 its height is equal to the side of a
31cm are melted and recast into a (d) 2,484 square whose area is 144 cm2 .
single cube. How much (in Rs.) is The area of the triangle (in cm2 )
the cost to polish the new cube at is :
the rate of Rs.10 per cm2

www.ssccglpinnacle.com support@ssccglpinnacle.com Ph. 09729327755, 09817390373 539


/
s
se
Days 54-60 Mensuration / े िमित

s
la
_c
ob
bo
ah
एक ि भुज का आधार, एक वग की
sm
24cm. What is the area (in cm2 ) Q175. If each side of a square is
ur
yo

प रिध के बराबर है िजसका िवकण 6 of rhombus? decreased by 17%, then by what


e/
t.m
://

√2 सेमी है , और इसकी ऊंचाई एक िकसी समचतुभुज की एक भुजा 13 percentage does its area decrease?
tp
ht

वग के भुजा के बराबर है िजसका cm और एक िवकण 24 cm है । यिद िकसी वग का ेक भुजा को


े फल 144 cm2 ..है । ि भुज का समचतुभुज का े फल ात करे | ( 17% घटाया जाता है , तो उसके
े फल ( cm2 म): cm2 म) े फल म िकतने ितशत की कमी
CPO-2019 24-11-2020 CPO-2019 25-11-2020 आती है ?
(Evening shift) (Morning shift) CPO-2019 25-11-2020
(a) 144 (a) 30 (Morning shift)
(b) 216 (b) 120 (a) 31.11%
(c) 288 (c) 60 (b) 30.79%
(d) 72 (d) 90 (c) 44.31%
(d) 25%
Q170. The area of a square and Q173. The radius of the base of a

e
rectangle are equal. The length of cylinder is 14cm and its volume Q176. Let A and B be two
the rectangle is greater than the is 6160 cm3 . The curved surface cylinders such that the capacity of
length of a side of the square by area(in cm2 ) of the cylinder is: A is the same as the capacity of
(Take π = 22 B. the ratio of diameter of A and
10cm and the breadth is less than 7 )

l
5cm. The perimeter (in cm ) of एक िसलडर के आधार की ि ा 14 B is 1 : 4. What is the ratio of
the rectangle is:
ac cm है और इसका आयतन 6160 heights of A and B?
एक वग और आयत का े फल cm3 है । िसलडर का व पृ ीय A और B दो िसलडर इस कार है
बराबर होता है । आयत की लंबाई वग की A की मता B की मता के
े फल
के एक भुजा की लंबाई से 10 सेमी समान है । A और B के ास का
( cm2 म) ा है ?
अिधक और चौड़ाई 5 सेमी कम है । अनुपात 1:4 है तो A और B की
CPO-2019 25-11-2020
आयत की प रमाप (सेमी म) ात करे ऊँचाई का अनुपात ात करे |
(Morning shift)
: CPO-2019 25-11-2020
(a) 940
CPO-2019 24-11-2020 (Evening shift)
(b) 660
(a) 16 : 3
nn
(Evening shift) (c) 778
(a) 40 (b) 3 : 16
(d) 880
(b) 100 (c) 1 : 16
(c) 80 (d) 16 : 1
Q174. The perimeter of a square
(d) 50 is half the perimeter of a
Q177. The ratio of the total
rectangle. The perimeter of the
Q171. The perimeter of a right surface area and volume of a
square is 40m. If its breadth is
triangle is 60 cm and its sphere is 2: 7. Its radius is:
two-thirds of its length, then what
एक गोले की कुल सतह के े फल
Pi

hypotenuse is 26 cm. What is the is the area of the rectangle ?


area (in cm2 ) of the triangle ? और आयतन का अनुपात 2: 7.
एक वग की प रिध एक आयत की
एक समकोण ि भुज की प रिध 60 इसका ि ा है :
प रिध का आधा है । वग की प रिध 40
cm है और इसका कण 26 cm है । CPO-2019 25-11-2020
मीटर है । यिद इसकी चौड़ाई इसकी
ि भुज का े फल ( cm2 ) ा है ? (Evening shift)
लंबाई का दो-ितहाई है , तो आयत का
(a) 10cm
CPO-2019 24-11-2020 े फल ा है ?
(b) 7.5cm
(Evening shift) CPO-2019 25-11-2020
(c) 7cm
(a) 120 (Morning shift)
(d) 10.5cm
(b) 96 (a) 321
(c) 90 (b) 196
(d) 60 (c) 400
Q178. If the volume of a sphere
(d) 384
is 4851 cm3 , then the surface area
Q172. One side of a rhombus is
13 cm and one of its diagonals is (in cm2 ) is: (Take π = 22
7 )

www.ssccglpinnacle.com support@ssccglpinnacle.com Ph. 09729327755, 09817390373 540


/
s
se
Days 54-60 Mensuration / े िमित

s
la
_c
ob
bo
ah
यिद एक गोले का आयतन 4851
sm
ur
yo

cm3 है , तो सतह े फल (cm2 म)


e/
t.m
://

ात कर
tp
ht

CPO-2019 25-11-2020
(Evening shift)
(a) 1268
(b) 1427
(c) 1399
(d) 1386

Q179. The internal length,


breadth and height of a cuboidal
room are 12m, 8m and 10m,
respectively. The total cost (in

e
Rs.) of whitewashing only all
four walls of the room at the cost
of Rs. 25 per m2 , is :
एक घनाकार कमरे की आं त रक

l
लंबाई, चौड़ाई और ऊंचाई मश 12
मीटर, 8 मीटर और 10 मीटर है ।
ac
कमरे की सभी चार दीवारों को सफेद
करने की कुल लागत 25/m2 की दर
से ा है ?
CPO-2019 25-11-2020
(Evening shift)
(a) 18,000
(b) 11,400
nn
(c) 12,600
(d) 10,000
Pi

www.ssccglpinnacle.com support@ssccglpinnacle.com Ph. 09729327755, 09817390373 541


/
s
se
Days 54-60 Mensuration / े िमित

s
la
_c
ob
bo
ah
sm
SOLUTION: Where R = external radius 2 π rh = 264 ⇒ 2 × 22 ×7×h =
ur

7
yo

r = internal radius
e/

264
t.m
://

Sol 1. (b) h = height or length of the ⇒ h=6


tp
ht

Let side of bigger cube = R cylinder Required ratio = 7:6


R3 = 13824 Here, volume =
2
R = √13824 =24 (2.52 − 1.5 ) × 756
3 22
7 ×
Sol 10. (d)
Let side of smaller cubes = r = 22 × (2.5+1.5)(2.5-1.5) × 108 40% = 52
13824 = 8 × r3 = 9504 cm3 Old : New
1728 = r3 Total Weight of pipe = Volume of Radius 5 : 3
3
Volume( ∝ r ) 125 : 27
12 = r pipe × weight of metal
Required ratio = 6 × 242 : 3 × 6 × = 9504 × 7.5 Let the volumes are 125 unit and
122 = 71280 g or 71.28 kg 27 unit
=4:3 Required %age = 125125
−27 × 100 =

Sol 7. (d) 78.4%

e
Sol 2. (c) Area of Sector = θ × πr2
360
Required area of sheet = = 150 × 22 2
× 21 = 577.5 cm2 Sol 11. (d)
360 7 2
2 × (80 + 48 + 60) − π × 12 Volume of Frustum = 1
3π (R +
2
= 376 − 3.1 = 372.9 m r2 +R.r)h

l
Sol 8. (b)
Area of triangular field with side = 1
3 × 22
7 × ( 32 + 22 +3.2) × 21 =
Sol 3. (a)

three faces
=
ac
Volume of cuboid = √a × b × c
Here a,b and c are the area of

√32 × 24 × 48 = 192 cm3


x m = √ x2

Where s =
3

50+70+80
4
Area of other triangular field =
√s(s − a)(s − b)(s − c)
a+b+c
2
418

Sol 12. (b)


Volume of bigger sphere =
R3
4

⇒ 2 = 100m
⇒ 4
3 × π × 43 = 256
3 π
Sol 4. (d) Area of field =
Volume of smaller sphere
√100(100 − 50)(100 − 70)(100 − 80)
nn
We know that = 34 × π × 23 ⇒ 32
3 π
area
Inradius = semiperimeter = √100 × 50 × 30 × 20
Let n be the required number of
3= 15
semiperimeter
=1000 √3 m2 spheres
Semiperimeter = 5 According to question According to the question
√3 x2 = 1000 3
Perimeter = 2 × 5 = 10 cm 4 √ 256 π = n × 32 π
3 3
x2= 4000 8=n
Sol 5. (a) x=63.25 m2
Volume of a cylinder= Sol 13. (c)
Pi

2
π (R − r2 )h Sol 9. (c) One of the sides of newly formed
Where R = external radius Curved Surface area of cylinder = cuboid = 6 × 2 = 12cm
r = internal radius 2 π rh Other two sides will be 2cm.
h = height or length of the Volume of cylinder = π r2 h Surface area of cuboid =
cylinder According to the question 2(lb+bh+hl)
2 2 = 2(12 × 2 +2 × 2 +2 × 12) = 104
⇒ 7480 = 22 2 π rh = 264 ……(1)
7 × (9 − r ) × 140
⇒ 17 = 81 - r2
2
π r h = 924 ……(2) cm2
⇒ r2 = 64 Dividing eq 2 by eq 1
⇒ r = 8 cm r = 7 cm Sol 14. (a)
Area of Hexagon = 6 × √4 a2
Put this value in either of the 3

Sol 6. (c) equations Here,


For example put r=7 in eq (1)
Volume of a cylinder= 6 × √ a2 =2400 3
3
2 4 √
π (R − r2 )h a2 =1600

www.ssccglpinnacle.com support@ssccglpinnacle.com Ph. 09729327755, 09817390373 542


/
s
se
Days 54-60 Mensuration / े िमित

s
la
_c
ob
bo
ah
sm
a=40 Let the sides be 4 unit and 3 unit. Desired Ratio = (52 − 42 ) :
ur
yo

Perimeter of the hexagonal field = According to the question


e/

(72 − 52 )
t.m
://

6a ⇒ 6 × 40 = 240 metre 2(4 unit + 3 unit) = 56


tp

= 9 : 24 ⇒ 3 : 8
ht

Required cost of fencing = 240 × 1 unit = 4 cm


18.5 = 4440 4 unit = 16 cm Sol 22. (b)
3 unit = 12 cm
Sol 15. (a) Diagonal of Rectangle =
Volume of cone = 1 2 Diameter of circle =
3π r h
Area of its base = π r2 √AB 2 + BC 2
According to the question = √162 + 122 = 20 cm
1 2
3 π r h =924 Radius of the circle = 20 = 10 cm
2
1 Let ABCD be the given
⇒ 3π r2 × 6=924 Area of circle = πr2 = π 102 = 100
⇒ π r2 × 6=924 π trapezium. And AE be its height.
⇒ π r2 =154 cm2 Area of trapezium= 21 × (a+b)h

e
180 = 1 × (20+10)AE
Sol 19. (b) 2
Sol 16. (d) Let the sides of the rectangle are AE = 12 cm
Curved Surface area of cylinder = 7x and 4x. Draw AF ∣ ∣ B C

l
2 π rh Perimeter of rectangle = 2(7x+4x) ⇒ AB=FC = 10 cm and AF = BC
Volume of cylinder = π r2 h = 22x ⇒ DF = DC-FC

2 π rh = 132
2
ac
According to the question
……(1)
π r h = 528 ……(2)
Dividing eq 2 by eq 1
r =8 cm
Perimeter of circle = 2 π r ⇒ 2
× 22
7 × 31.5
According to the question
22x = 2 × 22
⇒ x=9
7 × 31.5
= 20-10 = 10 cm
In Δ ADF
AF = AD

DE=EF = DF
….(AD =BC, given)
2 = 5 cm
Put this value in either of the Smaller side of rectangle = 4x = 4 …..(Perpendicular drawn on the
equations × 9 = 36 cm base of an isosceles triangle )
For example put r=2 in eq (1)
nn
AF 2 = AE 2 + EF 2
2 π rh = 132 ⇒ 2 × 22 7 ×8×h = Sol 20. (d) AF = √AE 2 + EF 2
132 Let the breadth of the rectangle =
⇒ h=2 85 m = √122 + 52
b
= 13
Length of the rectangle = b+20
Sol 17. (c) Perimeter of the rectangle =
Sol 23. (b)
Curved Surface area of cylinder = 2(b+b+20)
Area of Rhombus
2 π rh =4(b+10)
= 21 × d1 × d2
Pi

2 × 22
7 × 7 × h = 440
According to the question
1
4(b+10) × 53 = 21200 = 2 × 12 × 4 = 24
h=10 cm
Volume of cylinder = π r2 h ⇒ b+10 = 100
2 ⇒ b=90 m Sol 24.(d)
⇒ 227 × 7 × 10 = 1540 cm
3

⇒ l = 90+20 = 110 m Let AB = a cm


⇒ AC = a+10
Sol 18. (d) Area of the rectangle = 90 × 110 =
⇒ BC = 2a-2
9900 m2
According to the question
a+a+10+2a-2 = 32
Sol 21. (d)
4a = 24
Area of circle is ∝ r2 .
a = AB = 6 cm
⇒ Area of three circles will be
2 AC = 6+10 = 16 cm
42 , 5 , 72 BC = 2(6)-2 = 10 cm
Clearly AB is the smallest side.

www.ssccglpinnacle.com support@ssccglpinnacle.com Ph. 09729327755, 09817390373 543


/
s
se
Days 54-60 Mensuration / े िमित

s
la
_c
ob
bo
ah
sm
Area of Δ formed on BO = BD = 10 =5
ur

2 2
yo

Sol 25. (d) 2


e/

2√x
hypotenuse(H) = √4 × (a√2) =
3 AC
AO = = 2 = √x
t.m

2
://

%age increase in area = 22+22+


tp

Now,
ht

22×22 = 48.84 √ 3a2


2
100 AB 2 = AO2 + BO2
Area Δ formedof on
⇒ 82 = (√x)2 + 52
perpendicular(A) = √4 × (a) =
3 2
Sol 26.(b)
⇒ x=39
%age change in the area = (-11) + √3a2
(-11) +
(−11)×(−11)
= -20.79%.
4 √x + 10 ⇒ √39 + 10 =7
100 √3a2
A
Here, -ve sign indicates that the ⇒ H = √3a2
4
= 21
2 Sol 35. (a)
area is decreasing.
Let the base is b
Sol 31. (c) We know that area of
Sol 27.(c) Circumference of the circle(C) = parallelogram = h × b
%age change in the area = 2 πr
(−11)×(11)
According to question h=2b
(-11)+11+ = -1.21%. ⇒ r = 2πC

e
100 ⇒ b × 2b = 338
Here, -ve sign indicates that the Area of the sector = 60 × π × r2 b2 = 169
360
area is decreasing. 1 C 2 b=13
6 × π×( 2π )

= c2
Sol 28.(c)

l
24π
Sol 36. (d)
We know that Let the given triangle is ΔABC
d
ac2
a2 = ( 21 ) + ( 22 )
d

Where a = side of the rhombus


2

and d1 , d2 be the diagonals of the


rhombus.
2 2 d 2
Sol 32. (d)
Perimeter of rhombus
=2 √d + d
= 2 √14 + 48
2
1

2
2
2

2
=100
⇒ 10 = ( 12
2 ) + ( 22 )
Here, AD is perpendicular drawn
⇒ d2
2 = √102 − 62 =8 Sol 33. (d)
to the side BC
We know that
nn
⇒ d2 = 16
Volume of cone = 1 π R2 h Let other two sides (AB=AC) = S
Area of the rhombus = 3
According to question
1 Volume of sphere = 4 πr3
2 × d1 × d2 3 6+2S = 16
= 1 × 12 × 16 = 96 Given, R = 2r S=5
2
According to the question BD 26 = 3
1 2 4 3
Sol 29. (c) 3 πR h = 3 πr
AD = √52 − 32 = 4
Since it is an equilateral triangle. ⇒ 1
3 π (2r)2 h = 4
3 πr3
Area of ABC =
Each side will be formed of 4 ⇒ r=h 1 × AD × B C = 1 × 4 × 6 = 12
Pi

2 2
sticks and its length will be 4 ⇒ R:h ⇒ 2r:r
sq.cm
units. =2:1
Area of the triangle = √ a2
3
4 Sol 37. (a)
√3 Sol 34. (c)
= 4 × 4 × 4 = 4 √3 inner surface area of all walls =
2(l+b)h
Sol 30. (b) = 2(7+5)3.5 = 84
Let length of the perpendicular
and base of the right angle Sol 38. (a)
triangle is a cm. Since, the height of the triangle
⇒ Hypotenuse of the triangle = a formed will be half of the
diagonal of the square.
√2
Therefore, Height = √2 = 3 m
3 2
√2

www.ssccglpinnacle.com support@ssccglpinnacle.com Ph. 09729327755, 09817390373 544


/
s
se
Days 54-60 Mensuration / े िमित

s
la
_c
ob
bo
ah
sm
Sol 39. (d) Volume of cubical box = 30 × 30
ur
yo

Surface area of cube = 6 Sol 45. (b) Since the chessboard × 30 = 27000 cm3
e/
t.m
://

× a2
= 1176 ⇒ a = 14 is composed of 64 squares of side Required rise in water level =
tp
ht

Volume of cube = 2cm and the border is of 27000


60×30 = 15 cm
3 3
a = 14 = 2744 cm 3 thickness 2 cm.
Therefore, the length of each side Sol 50.(a)
Sol 40. (a) = 2x8+4 = 20 Perimeter of square = Perimeter
No. of boxes that can be painted So, the perimeter = 4x20 = 80cm of rectangle = 2(16+14) = 60 cm
= Side of square = 60
4 = 15 cm
11.28×10000
2×(30×25+25×12+12×30) = 40 √64 = 8
Sol 46. (b) Side = Perimeter of semicircle = π r +2r
Therefore, Diagonal = 8 √2 m = 22 15
7 × 2 + 15 = 23.58+15 =
Sol41. (c) For a cylinder: 38.58
Radius : Height = Volume Sol 47. (b)
Initial 5 : 5 = 125 Sol 51.(c)

e
Final 11 : 3 = 363 Old : New
Therefore, % Increase in Volume Length 10 : 13
= 363125
−125 × 100 Breadth 5 : 4
= 238 × 100 = 190.4 _________

l
125
Area 50 : 52
ac
Sol 42. (d) Given triangle is a
right angled triangle.
Area of
1 × 8 × 15 = 60
2
triangle

Area covered by the circle =


= Δ OBC
OB=OC=x
BC = 2
52-50 = 2 unit = 100
1 unit = 50
New area = 52 × 50 = 2600 cm2

Sol 52. (c)


180 2 Required number of buckets =
360 × π r = 19.25 x2 + x2 = 22 12×13.5 = 18
Therefore, Area excluding the x2 = 2 9
nn
sectors of circles = 60-19.25 = x= √2
40.75 sq.cm Sol 53. (d)
In Δ OBD
√3 a = 10 3
BD = 1 2 √
Sol 43. (b) Given triangle is a a=20
right angled triangle:
OB = √2
Area = √4 a2 = √4 × 20 × 20 =
3 3


2
Area of triangle = OD = (√2) − 1 = 1 2

1 × 24 × 45 = 540 100 √3
2 Now,
Area of sectors of the circle AO = 2 × OD
Sol 54. (a)
Pi

covering the triangle = = 2×1 = 2 15×15×15


180 × 22 × 10.5 × 10.5 = 173.25 Rise in level = 62×45 = 1.21 cm
360 7 AD = AO+OD = 3
Therefore, Area of triangle Area of Δ ABC = 21 × 3 × 2 = 3
Sol 55. (c)
excluding the portion covered = cm2 66×35×3
Required time = 22 35
× × 35 ×8
=
540-173.25 = 366.75 sq.cm 7 200 200

Sol 48. (a) 9000 sec = 2.5 h


Sol 44. (a) Curved surface area of the
Old : New cylinder = 2 × 22 20 44 Sol 56. (a)
7 × 7 × 100 = 5 11×6×100
Radius 2 : 5 Required Cost = ×
m2 60
Height 5 : 4 Total cost = 44 × 20 = 176 112.50 = 12,375
5
__________________
Volume 20 : 100 Sol 57. (c)
Sol 49. (b)
Required percentage = 10020−20 × Remaining area = 119 × 18-17 × 6
100 = 400% = 2040

www.ssccglpinnacle.com support@ssccglpinnacle.com Ph. 09729327755, 09817390373 545


/
s
se
Days 54-60 Mensuration / े िमित

s
la
_c
ob
bo
ah
sm
Volume of sand = 17 × 6 × 3 = Required % = 1728−125 × 100 = ⇒ n=27
ur

125
yo

306
e/

1282.4%
t.m
://

Required height = 306×100×100×100 Sol 9.(c)


tp
ht

2040×100×100
= 15 cm Sol 3. (c) We know that
1 2 d 2 d 2
Volume of Frustum = 3π (R + a2 = ( 21 ) + ( 22 )
Sol 58. (d) r2 +R.r)h Where a = side of the rhombus
Tota area to be painted = 40 × 30 = 1 × 22 2 2
× ( 5 + 4 +5.4)21 = and d1 , d2 be the diagonals of the
3 7
+ 2(40 × 2.2+30 × 2.2) = 1508 1342 rhombus.
Total cost = 1508 × 25 = 37700 d2 2 2
⇒ 132 = ( 10
2 ) +(2)
Sol 4. (a)
Sol 59. (a) ATQ, Volume of Frustum = 1 2 ⇒ d2
= √132 − 52 =12
3π (R + 2
3√3 ⇒ d2 = 24
2 × a = 108√3
2
r2 +R.r)h
⇒ a = 6√2 Area of the rhombus =
= 1
3 × 22
7 × ( 52 + 22 +5.2)14 = 1
2 × d1 × d2

e
Therefore, Perimeter = 6x 6√2 = 572 1
= 2 × 24 × 10 = 120
36 √2 cm
Sol 5. (a)
Volume of a frustum = 1 2 Sol 10.(a)
Sol 60. (a) Volume = 3π (R +
We know that

l
(12-4)x(12-4)x2 = 8x8x2 = 128 r2 +R.r)h 2 2
d d
cu. cm ac = 1 × 22 × ( 52 + 32 +5.3)21 = a2 = ( 21 ) + ( 22 )
3 7
1078 Where a = side of the rhombus
Sol 61. (d) Given, in a right angle and d1 , d2 be the diagonals of the
triangle Sol 6. (a) rhombus.
Hypotenuse = 25, Base = 24 1 2 d2 2 2
Volume of a frustum = 3π (R + ⇒ 202 = ( 24
2 ) +(2)
Then, Altitude = 7 r2 +R.r)h
Therefore, Area = 1 22 2 2
⇒ d2
2 = √202 − 122 =16
1 × 24 × 7 = 84
= 3 × 7 × ( 5 + 3 +5.3)10.5 = ⇒ d2 = 32
2
539
nn
Area of the rhombus =
1
Sol 62. (a) Longest stick = 2 × d1 × d2
Sol 7. (d) One of the sides of 1
= × 24 × 32 = 384
√162 + 82 + 112 = 21 newly formed cuboid = 5 × 3 = 2

15cm
Practice Question Sol 11.(c)
Other two sides will be 3cm.
Let AB = a cm
Surface area of cuboid =
Sol 1. (d) ⇒ AC = a+9
2(lb+bh+hl)
Inner radius = 27 cm ⇒ BC = 2a-3
= 2(15 × 3 +3 × 3 +3 × 15) = 198
Pi

External radius = 27 + 9 = 36 cm According to the question


cm2
Volume of the roller = a+a+9+2a-3 = 34
4a = 28
π(R2 − r2 )h Sol 8. (a)
2 2 a = AB = 7 cm
= π × ( 36 − 27 ) × 100 = 56700 Volume of bigger sphere =
4 3 AC = 7+9 = 16 cm
π 3 πr
BC = 2(7)-3 = 11 cm
Weight of the roller = 56700 π × 8 = 4
3 × 22
7 × 63 Clearly AB is the smallest side
= 453600 π gram or 453.6 π kg Volume of smaller sphere =
3
= 34 × 22
7 ×2 Sol 12.(d)
Sol 2. (d) Let n be the number of smaller Let AB = a cm
140% = 100140 = 7
5 spheres, then ⇒ AC = a+1
Old : New 4 22 3
n × 34
3 × 7 ×6 =
⇒ ⇒ BC = 2a-3
Radius 5 : 12 According to the question
× 22
7 × 23
Volume 125 : 1728 a+a+1+2a-3 = 34

www.ssccglpinnacle.com support@ssccglpinnacle.com Ph. 09729327755, 09817390373 546


/
s
se
Days 54-60 Mensuration / े िमित

s
la
_c
ob
bo
ah
sm
4a = 36 S19. (c) In the given figure of _________
ur
yo

a = AB = 9 cm rhombus: Volume 125 : 256


e/
t.m
://

AC = 9+1 = 10 cm Required percentage = 256125


−125 ×
tp
ht

BC = 2(9)-3 = 15 cm 100 = 104.8%


Clearly AB is the smallest side
Sol 25. (a) Total Surface area of
Sol 13. (c) joined cube = 2x(lb+bh+lh) =
%age increase in area = 13+13+ 2x(20x4+4x4+4x20)
13×13 = 27.69
100 = 2x176 = 352
Other diagonal is 5 x 2 = 10cm
Sol 14. (d) Therefore, Area = Sol 26. (b) Area of triangle
1 1
%age change in area = 2 × D 1 × D 2 = 2 × 10 × 24 = 120 excluding portion covered by
(−13)×(−13) sq. cm sector =
(-13)+(-13)+ 100 = -24.31
1 × 10 × 24 180 × 22 × 3.5 × 3.5
Here, -ve sign indicates that area 2 − 360 7

e
is decreasing. Sol. 20. (a) After joining 6 cubes = 120-19.25 = 100.75 sq. cm
adjacent to each other.
Sol 15. (b) There are 12 sticks of Then, length = 6x4 = 24 cm Sol 27. (c) No. of spheres =
1 unit length so perimeter of the Breadth = 4cm 6×6×6 = 27
2×2×2

l
triangle = 12 unit Height = 4cm
ac
Right angle triangle with Therefore, Total Surface Area of Sol 28. (d) Other diagonal =
resulting cuboid =2(lb+bh+lh)
perimeter 12 is with dimensions 2 × √52 − 42 = 6
3,4 and 5. =
Therefore, Area = 1 × 8 × 6 = 24
Area of the Δ = 21 × 4 × 3 = 6 2(24x4+4x4+24x4) = 2x208 = 2

416 sq. cm
sq units Sol29. (d) Other diagonal =
Sol 21. (b) 8 cubes will be formed 2 × √262 − 242 = 20
Sol 16. (b)
after cutting the edge of length 32 Therefore, Area =
One of the sides of newly formed
nn
cm into 4 cm identical cubes. 1 × D 1 × D 2 = 480
2
cuboid = 4 × 5 = 20cm
Therefore, Sum of total surface
Other two sides will be 3cm.
area = 8 × 6a2 = 8 × 6 × 42 Sol 30. (b) Area of triangle
Surface area of cuboid =
= 768 cm2 excluding the sectors =
2(lb+bh+hl)
1 × 16 × 30 180 × 22 × 7 × 7
= 2(20 × 5 +5 × 5 +5 × 20) = 450 2 − 360 7
Sol 22. (c) Length of half of other = 240 - 77 = 163 sq.cm
cm2
diagonal =

Sol 17. (b) √6.52 − 62 = √12.5 × 0.5 = 2.5 Sol 31. (d)
Pi

Weight of solid cylinder = π r2 h Therefore, Area of Increase in radius: 2-----5


× Density Rhombus = Increase in height: 2-----3
1 × D 1 × D 2 = 1 × 12 × 5 = 3 0 Increase in volume: 8------125
= 22
7 × 14 × 14 × 35 × 8 = 172480
2 2
Percentage increase =
gm = 172.48 Kg 67 × 100 = 837.5%
Sol 23. (d) No. of spheres = 8
9×9×9 = 729 = 91
Sol 18. (d) 2×2×2 8
Sol 32. (a) Total surface area of
Let the number of spheres casted
Sol 24. (b) the cuboid formed =
be n.
60% = 53 2x(lb+bh+lh)
Therefore, ATQ:
4 22 3 4 22 3 20% = 1 = 2x(40x5+5x5+5x40)
3 × 7 × 5 = n× 3 × 7 × 2 5
= 2x425 = 850 sq. cm
⇒ n= 15 85 ≈ 15 full sphere Old : New
Radius 5 : 8
Sol 33. (b) No. of spheres =
Height 5 : 4 7×7×7 = 343 = 42
2×2×2 8

www.ssccglpinnacle.com support@ssccglpinnacle.com Ph. 09729327755, 09817390373 547


/
s
se
Days 54-60 Mensuration / े िमित

s
la
_c
ob
bo
ah
sm
ur
yo

Sol 34. (c) ATQ, Sol 42. (b) Sol 1. (d)


e/
t.m
://

1 × 154 × h = 924 5000 Let the side of cube = a


Required volume = 3 × 36 ×
tp

3
ht

60
⇒ h = 18 m = 9000m3 Volume of cube = a3
Now,
Radius of the cylinder = a
Sol 35. (b) Total surface area = Sol 43. (d) 2
2x(18x6+6x6+6x18) = 2x252 = Volume = l × b × h Height of the cylinder = a
504 2160 = 20 × 12 × h Volume of the cylinder = π r2 h
H= 2160
20×12 = 9 cm = π ( 2a )2 a = π
4 a3
Sol 36. (d) ATQ: a3 − π4 a3
22 × r 2 × 6 = 231 Required %age = a3
× 100
7 Sol 44. (b) Area = 3 22
a (1− 7×4 )
Therefore, r = 3.5 cm 1 × 20 × (7.5 + 8.6) = 322 = a3
× 100
2 6 × 100 ≈ 21
= 28
Sol 37. (a)
Sol 45. (c) Amount spent for

e
Volume of bigger cube = 100 Sol 2. (c)
dyeing the walls = 230x5x7.5 =
× 100 × 100 = 1000000 cm3 Let the heights of the cylinders
Rs. 8625
Volume of smaller cube = 10 × 10 are h1 and h2 .
× 10 = 1000 cm3 Sol 46. (d) Given, 6 × a2 = 1944 Volume of the first cylinder =

l
Required number of cubes = ⇒ a = 18 πr2 h
1000

Sol 38. (b)


ac
1000000 = 1000

According to question
3+4+5 unit = 24
Therefore, Volume = a3 = 5832

Sol 47. (a) In the given solid


figures. Cuboid has the maximum
no. of vertex i.e. 8.
⇒ x = π ( 2a )2 h1

4y
h2 = πb2
4x
⇒ h1 = πa
Similarly
2

Required ratio = 4x : 4y
1 unit = 2 πa2 πb2
3 unit = 6 2
Sol 48. (b) Required volume = = xb :y a2
4 unit = 8 (90x75x50) - (85x70x45) =
nn
5 unit = 10 337500-267750 Sol 3. (b)
Cleary, given triangle is a right = 69750 20% = 51
angle triangle. And 40% = 2
5
Area of triangle = 21 × 6 × 8 = Sol 49. (c) We know that volume is directly
24cm2 proportional to the product of the
square of the radius and the
Sol 39. (d) height
Required number of soaps = Old : New
Pi

11×0.82×0.63×100×100×100 = 78925
8×4.5×2 Radius 5 : 4
Height 5 : 7
Sol 40. (c) ____________
Lateral surface area = 2(l+b)h Given, AC = 4√5 Volume 125 : 112
= 2(24+16)7.5 = 600 cm2 Decrease in volume = 125-112 =
Let AB = 2a and BC = a
13
Then, AC = 4√5 = a√5 ⇒ a = 4 13 × 100 =
Sol 41. (a) Required %age = 125
Therefore, Area = 4x8 = 32
3+2+4 = 9 unit = 72 10.4%
1 unit = 8
Sol 50. (c) Height of platform =
3 unit = 24 22 7 7
× × ×18 Sol 4. (b)
2 unit = 16
7 2 2
14×18 = 2.75m Let original volume = v , original
4 unit = 32 radius = r and original height = h
SSC CGL TIER II 2018 and new height = H

www.ssccglpinnacle.com support@ssccglpinnacle.com Ph. 09729327755, 09817390373 548


/
s
se
Days 54-60 Mensuration / े िमित

s
la
_c
ob
bo
ah
sm
v= 1 πr2 h Lateral surface area of the Sol 13.(b)
ur

3
yo

cylinder = 2 π rh Area of the base of a cone = π r2


e/

⇒ h= 3v
t.m

πr2
://

⇒ 352 = 2 × 22 7 ×r×7
⇒ 400 π = π r2
tp

According to the question


ht

3v = 1 π (2r)2 H ⇒ r=8 ⇒ r=20


3
⇒H= 9v Volume of the cylinder = π r2 h Slant height (l) of the cone =
4πr2
Required ratio = 3v
πr2
: 9v
4πr2
= 22
7 × 8 × 8 × 7 = 1408 √h2 + r2
=4:3 = √152 + 202 = 25
Sol 9. (a) curved surface area of the cone =
Sol 5. (c)
Total surface area of a hollow π rl
Curved surface area of a cone =
π rl hemisphere = 2 π ( R2 + r2 )+ π ( = π × 20 × 25 = 500 π
⇒ 2310 = 22 R2 − r 2 )
7 × 21 × l
⇒ l =35 cm Here R =7 cm and r = 6 cm Sol 14. (c)
total surface are of the vessel = 2 20, 21 and 29 are triplet pairs,
Height of the cone = √l2 − r2 × π × ( 72 + 62 )+ π × ( 72 − 62 )

e
clearly given triangle is an right
= √352 − 212 =28 cm = π (2 × 85 +13) = 183 π angle triangle.
Now, Sol 10. (a) Area of the triangle = 21
Volume of the cone = 1 πr2 h
3 Volume of the sphere = 34 π r3 × 20 × 21 = 210 cm2
1 22

l
= 3 × 7 × 21 × 21 × 28 Here, Volume of the prism = Base Area
3 3
ac = 12936 cm3 4 4 4 3 4

You might also like